Review of Pediatrics and Neonatology

Download as pdf or txt
Download as pdf or txt
You are on page 1of 609

Review of

Pediatrics & Neonatology


Review of
Pediatrics & Neonatology
Sixth Edition

Taruna Mehra
MBBS (Gold Medalist), MD (Pediatrics), MAMC, New Delhi
Head of Department
Vidya Jeevan Ortho and Pedics Super Speciality Center
New Delhi, India
Consultant Pediatrician
Max Super Speciality Hospital
New Delhi, India

Meenakshi Bothra Gupta


MBBS (Honours & Gold Medalist), Medical College, Kolkata
MD (Pediatrics), AIIMS, New Delhi
Assistant Professor
Department of Pediatrics
Maulana Azad Medical College
New Delhi, India

Apurv Mehra
MBBS, MAMC, New Delhi
MS ORTHO (UCMS), DNB ORTHO DIP. SICOT (Belgium)
Fellowship Computer Navigation Joint Replacement
(From Germany, Australia and Malaysia)
Consultant Orthopedic Surgeon
Computer Navigation Joint Replacement and
Arthroscopy Surgeon, Max Super Speciality Hospital, Delhi
Director, Vidya Jeevan Ortho and Pedics Super Speciality Center, Delhi
Co-Founder MedMiracle-The Learning App for Medicos
Founder Ortho Dhoom Dhadaka-India's Leading Orthopedic Programme for Medicos

JAYPEE BROTHERS MEDICAL PUBLISHERS


The Health Sciences Publisher
New Delhi | London
Jaypee Brothers Medical Publishers (P) Ltd

Headquarters
Jaypee Brothers Medical Publishers (P) Ltd
4838/24, Ansari Road, Daryaganj
New Delhi 110 002, India
Phone: +91-11-43574357
Fax: +91-11-43574314
Email: jaypee@jaypeebrothers.com

Overseas Office
J.P. Medical Ltd
83 Victoria Street, London
SW1H 0HW (UK)
Phone: +44 20 3170 8910
Fax: +44 (0)20 3008 6180
Email: info@jpmedpub.com

Website: www.jaypeebrothers.com
Website: www.jaypeedigital.com
© 2020, Jaypee Brothers Medical Publishers
The views and opinions expressed in this book are solely those of the original contributor(s)/author(s) and do not necessarily
represent those of editor(s) of the book.
All rights reserved. No part of this publication may be reproduced, stored or transmitted in any form or by any means,
electronic, mechanical, photocopying, recording or otherwise, without the prior permission in writing of the publishers.
All brand names and product names used in this book are trade names, service marks, trademarks or registered trademarks
of their respective owners. The publisher is not associated with any product or vendor mentioned in this book.
Medical knowledge and practice change constantly. This book is designed to provide accurate, authoritative information about
the subject matter in question. However, readers are advised to check the most current information available on procedures
included and check information from the manufacturer of each product to be administered, to verify the recommended
dose, formula, method and duration of administration, adverse effects and contraindications. It is the responsibility of the
practitioner to take all appropriate safety precautions. Neither the publisher nor the author(s)/editor(s) assume any liability for
any injury and/or damage to persons or property arising from or related to use of material in this book.
This book is sold on the understanding that the publisher is not engaged in providing professional medical services. If such
advice or services are required, the services of a competent medical professional should be sought.
Every effort has been made where necessary to contact holders of copyright to obtain permission to reproduce copyright
material. If any have been inadvertently overlooked, the publisher will be pleased to make the necessary arrangements at
the first opportunity. The CD/DVD-ROM (if any) provided in the sealed envelope with this book is complimentary and free
of cost. Not meant for sale.
Inquiries for bulk sales may be solicited at: jaypee@jaypeebrothers.com

Review of Pediatrics & Neonatology


First Edition: 2015
Second Edition: 2016
Third Edition: 2017
Fourth Edition: 2018
Fifth Edition: 2019
Sixth Edition: 2020
ISBN 978-93-89776-81-2
Our Mission
Aspiring to Serve Humanity

Dedicated to

—Dr Taruna Mehra & Dr Apurv Mehra

—Dr Meenakshi Bothra Gupta


Preface

“A dream doesn't become reality through magic. It takes sweat, determination and hard work”  —Colin Powell

Self-belief, consistent and sincere efforts and choosing the right study material are the pillars of success in any Postgraduate
Medical Entrance Examination (PGMEE). In this competitive era, getting specialty of choice in one of the the premier institutes of our
country is a dream that every medical student aspires to. But this requires one to complete the formidable task of covering 19 subjects
of medical curriculum in a very limited time span.
Pediatrics, though being a very scoring subject in the final year of MBBS, is overshadowed by the other daunting subjects like
Medicine, Surgery, Gyne-Obs. So, often a medical undergraduate is not able to read his textbooks very well, and do justice to the
subject. In the recent NBE-pattern examinations, 15-20 MCQs out of 300 are usually asked from Pediatrics. These are often
straightforward questions, but, they cannot be solved unless your understanding of the subject is right. You might have noticed that
the current trend of NBE, AIIMS, PGI and other entrance examination questions, not only requires you to have a command on repeat
MCQs, but also a thorough knowledge of those topics with special emphasis on concepts and high yield facts.
Thus, it is very important to strengthen your concepts and know the important facts. Mere rote memorizing will not
suffice. The primary and most important step to achieve this is to choose the right book, with authentic questions and answers with
references from standard textbooks, written by subject specialists.

Highlights of the 6th edition of Review of Pediatrics & Neonatology:


•• Fully colored book: Attractive, reader friendly presentation of the theoretical concepts in the pretext with fully colored, high
resolution images, placed beside the relevant text, to help in concept building.
•• Thoroughly updated pretexts: To provide concise, point-wise overview of all the chapters, thoroughly updated pretexts
with high-yield points, mnemonics and flow diagrams have been provided.
•• Three sections: In order to make your reading more well-organised, the chapters have been re-arranged into 3 different sections:
General Pediatrics, Neonatology & Systemic Pediatrics.
•• Exclusive feature for MCI/FMGE exams: Topics and questions important for MCI/FMGE exams have been distinctively
marked with ‘M' for easy identification and revision.
•• Image-based MCQs: Keeping in mind the current pattern of examinations, we have included over 500 fully colored images
to prepare you for the same. Image-based questions have been presented along with the pretext for ‘integrated approach'.
•• Latest updates: All the latest updates and guidelines in the field of pediatrics, which are potential upcoming questions, have been
highlighted in the respective chapters.
•• Most latest MCQs included: Over 3000 MCQs comprising updated NBE-pattern questions up to January 2020, AIIMS up
to November 2019, PGI & JIPMER up to December 2019 have been included and have been arranged chronologically to
make the students familiar with the current trend.
•• Authentic explanations: Explanations have been provided for the answers, with references from the latest editions of standard
textbooks. Controversial MCQs have been explained in detail, discussing each option and excluding the incorrect ones for
instituting analytical skills in students.
•• Annexures: Important high-yielding topics requiring last-minute revision, like developmental milestones, salient features of
important metabolic disorders, important genetic syndromes, list of ‘Most commons' and many more have been included in
annexures.
•• Online support is available from the author herself at https://www.facebook.com/groups/Pediatricsmadeeasy, to
answer all your queries in Pediatrics, to keep you abreast with the latest updates and help you in revision before
exams as well as for assessing yourself.
•• Utility of this book: This book is useful not only for the students preparing for various PGMEEs, but for all MCQ- based
exams on Pediatrics including MCI/FMGE exams, various DM entrance exams and also for the final-year undergraduate
students for concept-building as well as for quick revision, as adjunct to their text books.
May your hardwork propel you to a speed so fast, that Good Luck has no choice, but to ride with you…
May God bless you with success in all your examinations and may your career grow in leaps and bounds!
Taruna Mehra
Meenakshi Bothra Gupta
Apurv Mehra
Acknowledgments

“We must find time to stop & thank the people, who make a difference in our lives.”  —John F Kennedy

Firstly, we would like to express our eternal gratitude to the Almighty God for His innumerable blessings….! Emotions fail to find
expressions to highlight the roles of our parents and other family members for their blessings, everlasting love, encouragement and
support.
It is my profound privilege to express my respect and regards for my mentors from AIIMS (New Delhi), especially Prof VK
Paul, Prof A Deorari, Prof SK Kabra, Prof M Kabra, Prof A Bagga, Prof S Gulati, Dr R Lodha, Dr V Jain, and Dr R Seth and Prof D K
Gupta, Director, SSPHPGTI, Noida for being a constant source of inspiration, motivation and guidance. Words are not sufficient to
express my feelings for my dear husband, Dr Praveen, for having filled my life with love and happiness. He has also been a constant
source of intellectual inputs & new ideas, without which this book would not have been in its present form. I would also take this
opportunity to convey my gratitude to my dear brothers Shekhar & Samir for always having faith in me. I am also thankful to my dear
friend, Dr Zainab Ahmad, for not only helping me in this project, but also for being a constant source of moral support & valuable
advice. My sincere thanks are also due to my seniors & colleagues at SSPH & PGTI (Noida), including Prof DK Singh, Dr Poonam
Motiani & Dr Divya Jain for their invaluable support, co-operation & help rendered at various times.
We wish to sincerely thank our mentors Prof HP Sachdeva, Prof SK Mittal, Prof P Chaudhary, Dr Urmila Jhamb, Dr Seema
Kapoor, Dr Monika, Dr Rajeshwari, Dr Ramji, and the entire Pediatrics Department of Maulana Azad Medical College, New Delhi
for being our source of inspiration and knowledge. We are also thankful to Dr S Kukreja, Dr S Sehgal, Dr LN Taneja, Dr Shipra,
Department of Pediatrics, Max Superspeciality Hospital (Patparganj), and Dr Ramalingam, Department of Pediatrics, Fortis Hospital.
Special thanks to MedMiracle mentors—Dr Thameem Saif, Dr Gobind Rai Garg, Dr Rebecca James, and Dr Sakshi Arora Hans
for their valuable suggestions and unconditional support.
We would also like to express immense gratitude to Team MedMiracle, Dr Saurabh Taneja, Dr Ravi Sharma, Dr Zainab Vora,
Dr Akhil Monga, Dr Anivita Aggarwal and Dr Dipin Sudhakaran along with a group of Subject Specialists for their stupendous efforts
to help medicos get an extra edge in their preparation for entrance exams.
We are also grateful to our all well-wishers, whose ideas and suggestions have helped us immensely: Dr Vivek Jain, Dr Rajat
Jain, Dr Akhilesh Raj Jhamad, Dr Rajesh Kaushal, Dr Sparsh Gupta, Dr Ranjan Patel, Dr Devesh Mishra, Dr Khushboo Goel, Dr
Shivani Jain, Dr Soumen Manna, Dr Utsav Bansal, Dr B Tiwari, Dr Shubh Vatsya, Dr Praveen Tripathi and Dr Vandana Puri.
A wholehearted thanks to our dear juniors/students for their overwhelming response to the fifth edition of this book, and for
making it one of the bestselling & most appreciated review books available for Pediatrics. We received awesome feedbacks &
suggestions from so many juniors/students all over the country that have helped us immensely in revising this book.
We appreciate the support of Mr Jitendar P Vij (Group Chairman), Mr Ankit Vij (Group President) and Ms Chetna Malhotra
Vohra (Associate Director–Content Strategy) of Jaypee Brothers Medical Publishers in the publication of this book. We are also
thankful to Ms Saima (Manager–Publishing), Ms Ruby Sharma & the whole production team of Jaypee Brothers Medical Publishers
for their untiring efforts.
We take this opportunity to thank, all those who guided us & supported us, at various times and most of all, believed in our
ability to take this project from vision to reality.
Golden Points to Excel in Pgmee

Here Are Answers To Few Questions I Am Often Asked By My Students…


Q. 1. Why didn’t I achieve a rank when I worked so hard…?
My Answer: I would like to share with you, when most of my friends qualified in the first attempt and I didn’t, it was a heartbreak
and quite unbelievable. That is when I realized that our so called failure in entrance exams is not a failure but is the inability to plan
a strategy and identify our areas of lacunae.
I analyzed my lacunas and came to this conclusion:
•• Group all 19 subjects into 4 parts
–– Part 1: First year subjects
–– Part 2: Second Year Subjects
–– Part 3: Final year subjects including PSM
–– Part 4: Short subjects that include skin, Psychiatry, Radio, Anesthesia, ENT, Ophtha, Ortho and Forensic Medicine
•• After assessing last 5 year papers, on the basis of my MCQ knowledge I noted
–– My weak subjects, Strong Subjects and Relatively Strong Subjects
–– In my weak subjects, I made a list of topics to be covered and
–– In my relatively better subjects, I made a list of topics I was weak in.
This is a point that we need to understand and accept that even in our strong subjects there will be areas we are weak in.
A good player of cricket is the one who needs to be a good runner but if his running is weak then he’ll be run out, so he needs
to work on his running. As well at the same time he needs to keep practicing his batting skills too.
Through this I wish to convey that you should keep revising your strengths otherwise soon they become your short comings.

Q. 2. Why do we keep forgetting what we learn…?


My Answer: In your first reading itself, make it a habit to highlight important points to be read in your second reading. This helps
you a lots. In the second reading as it gives you a feel that you have already read this earlier and also saves your time.
I also advice to make 2 schedules:
1. Main schedule for preparation for the whole day.
2. Revision Schedule in which you have 90 minutes each day to revise volatile subjects, for example for me - Pharma was one such
subject.

Q. 3. Why do we lag behind in our schedule …?


My Answer: You can never complete 100% of any subject. Selective Study is the best way to prepare for Medical Entrance
Exams. Read each subject in the order of importance of the topics so that in case you exceed your time allotted to each subject, you
move on to the next subject but don’t miss on vital things. Here our aim is to complete all 19 subjects. But if you till the end don’t
follow this advice of sticking with your schedule, you will end up with a catastrophe of leaving a subject which no one can never
compensate.
Also keep practicing MCQs every day, preferably in the morning hours, at the same time when the exams are conducted, so that your
internal clock is tuned.

Q. 4. Negative Thoughts- What if I study properly and still don’t make it this time then what ...???
My Answer: There is a self belief which tells you to take chances. I agree there will be doubts but you must at the same time realize
that even Rank 1 of any exam, will have doubts but even then the power to conquer the fears is more. So the basic thing you can
do to counter this is - to develop a habit of surrounding yourself with positivity and positive human beings and to my brain, they are
your parents- talk to them, they will always show you the positive aspect.

Q. 5. Sir, when will my time come…?


My Answer: This is a big question and I can just tell you one simple thing. ‘Time never comes, you need to create it.’ God will write
your destiny according to your efforts, and only hard work can beat any talent and I know it surely will.
My batchmates in MAMC were very talented and intelligent but I always had this habit called as hard work, which I gradually
realized is the biggest asset, which if nurtured and fed well, can take you everywhere. Those who work hard can do good not only
with themselves but also to lots of people around. Just as I have learnt from my Grandmother, ‘To live like a king, we need to work
like a slave.’
x Review of Pediatrics & Neonatology

So to summarise:
•• Right approach to examination is the most important step, so start as early as you can.
•• Select the right books with updated pretexts in latest editions.
•• Your preparation decides your fate, analyze and evaluate your current preparation.
•• Accordingly start with a combo of strong and weak subjects. Prepare list of important topics, shortlist topics in weak subjects and
weak topics in strong subjects.
•• Be consistent and keep revising your strong topics too at the same time managing your time well.
•• Practice MCQs daily, preferably from the topics you have read.
•• Plan your day in advance to bring in effectiveness.
•• Prepare a schedule where you also give 90 minutes to revision each day.
•• Stick to your time table and don’t neglect any subject–completing all subjects is more important than trying to complete 100
percent of a subject.
•• Keep talking to your parents, they are the sea of positivity and will always stand by you.
•• Hard work can beat talent, it surely will, so work hard like a slave and then live like a king.
How to Use this Book?

"Reading is important, because if you read, you can learn anything about
everything and everything about anything."  —Tomie dePaola

In order to succeed, it is very important to have the motivation, self-belief and burning desire to do so.. By procuring this book you
have already taken one small step towards getting the PG seat of your choice, the dream of every medical undergraduate. But what
is very important is to formulate a proper strategy to make the best use of it.
It is preferable to begin reading this book during your final year MBBS. Try to read each chapter from this book, after reading the
same from your standard textbook, at least once; Keep underlining/highlighting important points and be actively involved in reading
i.e. try to analyse whatever you are reading and integrate it with your prior knowledge.
If you have already passed your final MBBS, then you can more easily and speedily read this book. This book has been divided
into a total of 24 chapters; Depending on what strategy you have chosen for your PG preparation, if you want to complete each
subject one by one, it should not take more than 7 days to complete reading this book and solving the MCQs provided in it for the
first time; alternative strategy could be to devote around 4 hours per day for next 20 days, to Pediatrics, though the former strategy
would be more preferable.
Make sure that you solve the MCQs of each chapter, shortly after reading the pretext (theoretical part of the same chapter). You
might come across some important extra points while solving the MCQs; keep noting them in the theory part (pretext) of this book,
so that during your subsequent revisions, you need not consult multiple sources for the same subject.
While solving MCQs, in order to sharpen your time management skills you may also earmark a set of say 50 questions and
solve them in a previously calculated time; Best would be, if you can target 30 seconds per MCQ while you practice; because, in
your actual exams, the stress and anxiety might make you spend longer than usual time in solving MCQs. Moreover, some difficult
questions might require spending more time, especially if you have to revisit that MCQ again. While checking the answers to MCQs,
preferable would be to read through all the explanations provided. But, if you are running short of time, you must at least read the
explanations to all the questions answered wrongly by you. Mark all the MCQs answered wrongly by you and try to revisit them
during your revision sessions.
Try to revise this book at least 3 times, keeping at least 3 days, 2 days and 1 day for your 1st, 2nd and 3rd revision sessions,
respectively, for best results. Remember, there is no friend as loyal as a good book!
This is just a suggestion for those of you, who have still not made a plan or strategy for your PG preparation. For the rest of you,
who already have a plan in place, be sincere, work hard and stick to your plan, and you will surely achieve all that you aspire for......

May God Bless you!


Unique Features of this Edition

•• First Pediatrics review book by subject specialists from topmost institutes of India
•• Concise, point-wise, thoroughly revised & updated overview of important topics, provided in pretexts
•• Well organised presentation divided into 3 separate sections of General Pediatrics, Neonatology & Systemic Pediatrics
•• High-yielding points, Mnemonics & latest updates have been highlighted & flow diagrams have been added for easy understanding.
•• Fully coloured book with > 500 images, which are potential Image-based MCQs have been included & presented along with the
pretext for integrated approach
•• Over 3000 MCQs including updated questions from NBE pattern up to Jan 2020, AIIMS up to Nov 2019, JIPMER & PGI up to
Dec 2019 have been covered.
•• MCQs have been arranged chronologically with recent ones highlighted so as to make students familiar with the current trend.
•• Authentic explanations with references from latest edition of standard textbooks
•• Annexures covering important high-yielding topics requiring last-minute revision
•• Online support is available at https://www.facebook.com/groups/Pediatricsmadeeasy.
Contents

Most Recent Questions and Answers with Explanations xxiii


Annexures xxxix

SECTION 1: GENERAL PEDIATRICS

1. Normal and Abnormal Growth ***** 1


Pretext  1; Questions  12; Answers with Explanations 16
2. Development ***** 22
Pretext 22; Questions 28; Answers with Explanations 31
3. Nutrition and Malnutrition ***** 36
Pretext 36; Questions 44; Answers with Explanations 50
4. Fluid and Electrolyte Disturbances *** 57
Pretext 57; Questions 64; Answers with Explanations 67
5. Genetics and Genetic Disorders **** 72
Pretext 72; Questions 84; Answers with Explanations 93
6. Inborn Errors of Metabolism ***** 105
Pretext 105; Questions 120; Answers with Explanations 126
7. Diseases of Immune System **** 134
Pretext 134; Questions 142; Answers with Explanations 145
8. Infectious Diseases ***** 150
Pretext 150; Questions 169; Answers with Explanations 177
9. Immunization *****190
Pretext 190; Questions 199; Answers with Explanations 204
10. Pediatric Basic & Advanced Life Support (PALS) ***210
Pretext 210; Questions 218; Answers with Explanations 219

SECTION 2: NEONATOLOGY

11. Normal Neonate and Neonatal Resuscitation *****220


Pretext 220; Questions 229; Answers with Explanations 233
12. Diseases of Newborn *****239
Pretext 239; Questions 256; Answers with Explanations 265

SECTION 3: SYSTEMIC PEDIATRICS

13. Pediatric Cardiology ****278


Pretext 278; Questions 293; Answers with Explanations 301
14. Pediatric Hematology ***312
Pretext 312; Questions 327; Answers with Explanations 337
15. Tumors of Infancy and Childhood *** 351
Pretext 351; Questions 358; Answers with Explanations 362
xxii Review of Pediatrics & Neonatology

16. Pediatric Gastrointestinology ****369


Pretext 369; Questions 376; Answers with Explanations 380
17. Pediatric Hepatology ***387
Pretext 387; Questions 391; Answers with Explanations 393
18. Pediatric Respiratory Disorders *****396
Pretext 396; Questions 406; Answers with Explanations 411
19. Disorders of Kidney and Urinary Tract *****418
Pretext 418; Questions 431; Answers with Explanations 440
20. Pediatric Endocrinology ****454
Pretext 454; Questions 466; Answers with Explanations 473
21. Pediatric Neurology *****482
Pretext 482; Questions 494; Answers with Explanations 502
22. Behavioral Disorders in Children ****513
Pretext 513; Questions 516; Answers with Explanations 519
23. Musculoskeletal System ****523
Pretext 523; Questions 531; Answers with Explanations 534
24. Pediatric Surgical Disorders ***537
Pretext 537; Questions 544; Answers with Explanations 547

Most Important*****
Very Important****
Important***
Most Recent Questions

GENERAL PEDIATRICS FLUID AND ELECTROLYTE DISTURBANCES


10. Which of the following is true regarding presentation of
DEVELOPMENT dehydration in a child? (PGI May 2019)
1. Milestones developed by a child between 6–9 months of a. Reduced skin turgor—mild dehydration
age is/are: (PGI May 2019) b. Dry mucous membrane—moderate dehydration
a. Can extend arm c. Urine output <0.5 mL/kg/hour—moderate dehydration
b. Can voluntarily drop object d. Fluid deficit of 150 mL/kg—severe dehydration
c. Can hold object with thumb and index finger e. Rapid and deep breathing—severe dehydration
d. Can point something with index finger
e. Can swap object from one hand to other GENETICS AND GENETIC DISORDERS
2. Which of the following is NOT a correct pairing of mile­
11. A child presents with coarse facial features, hypercalcemia
stone and its age of developing? (PGI May 2019)
and an audible murmur suggestive of supravalvular AS.
a. 5 months—holding of objects with both hands Chromosomal analysis reveals deletion of chr 7q11.23.
b. 6 months—transfer of objects What is the diagnosis: (NEET Pattern 2020)
c. 6 months—tripod sitting
a. Williams syndrome b. Hunter’s syndrome
d. 9 months—pulls to standing c. Liddle syndrome d. Bartter syndrome
e. 14 months—pincer grasp
12. True about Fragile X syndrome is: (NEET Pattern 2019)
3. A girl of 6 years age can climb up stairs with alternate feet,
ride a tricycle, can remember name, age but cannot narrate a. Triple nucleotide CAG Sequence mutation
stories. What is her actual developmental age? b. 10% female carriers mentally retarded
 (AIIMS May 2019) c. Males have IQ 20–40
a. 3 years b. 4 years d. Gain of function mutation
c. 5 years d. 6 years
4. Bidextrous grasp is seen at what age? (NEET Pattern 2019) INBORN ERRORS OF METABOLISM
a. 4 months b. 5 months 13. False about Hartnup’s disease:
c. 6 months d. 7 months
a. Defect in neutral amino acid transport
NUTRITION AND MALNUTRITION b. Mental retardation is the common presentation
c. Most children are asymptomatic
5. Which of the following rules out severe protein energy d. Photosensitivity
malnutrition exclusively? (NEET Pattern 2020)
a. BMI > 17 DISEASES OF IMMUNE SYSTEM
b. Normal lean body mass
c. Normal skin fold thickness 14. A 5-year-old boy has recurrent infections, low platelet count
and rash as shown below. What is the likely diagnosis?
d. Normal ECF volume
 (NEET Pattern 2020)
6. Protein in Cow’s milk in comparison to human milk is:
a. Double b. Triple (FMGE June 2019)
c. Quadruple d. Same
7. Chronic malnutrition in a child is best evaluated by:
 (FMGE June 2019)
a. Weight for height b. Weight for age
c. Height for age d. Ponderal index
8. Amount of Protein present in 100 mL of breast milk is:
a. 2.2 g b. 1.1 g (FMGE June 2019)
c. 0.55 g d. 3.3 g
9. Severe acute malnutrition as per WHO criteria:
 (NEET Pattern 2019)
a. Weight for age—2 SD less than median a. Severe combined Immunodeficiency (SCID)
b. Weight for height—2 SD less than median b. Job syndrome
c. Weight for age—3 SD less than median c. Bruton’s agammaglobulinemia
d. Weight for height—3 SD less than median d. Wiskott-Aldrich syndrome
xxiv Review of Pediatrics & Neonatology

15. A child of 7 years presented with fever for 6 days along a. 8–12 weeks b. 14–18 weeks
with conjunctivitis, rash over the palms and soles, red c. 34–37 weeks d. 28–32 weeks
tongue and unilateral cervical lymphadenopathy. What is
22. A 5-year-old child presents with fever and altered
the diagnosis? (NEET Pattern 2020)
sensorium for 3 days. On examination, he has oral ulcers
a. Kawasaki disease b. Scarlet fever & lesions on his palm as shown in the image. What is most
c. Bacterial conjunctivitis likely causative organism? (AIIMS May 2019)
d. HSP
16. A 16-year-old boy had pain in the calf muscles for 2 days.
He gives history of fever, sore throat, cough before 5 days
which lasted for 3 days. There is pain on pressing the calf
muscles. CNS examination reveals no abnormal findings.
Reflexes are normal. CPK levels are 2000 IU. What is the
diagnosis? (AIIMS Nov 2019)
a. DMD
b. Viral myositis
c. Dermatomyositis
d. GBS

INFECTIOUS DISEASES
17. A child presented with the following findings in oral cavity
and skin rash as shown below, along with running nose and
redness of eyes. What is the most common complication of a. Staphylococcus aureus b. Measles virus
this illness? (NEET Pattern 2020) c. Rubella virus d. Picorna virus

IMMUNIZATION
23. A 5-year-old unimmunized child developed Diphtheria.
He has a 3 years old sibling contact, who received last
booster 18 months back. What to do with the contact?
 (NEET Pattern 2020)
a. Two doses of polysaccharide vaccine
b. Two doses of conjugate vaccine
c. Single dose of toxoid vaccine
a. Acute myocarditis b. Acute epididymo-orchitis d. No vaccine needed
c. Acute hepatitis d. Acute otitis media 24. Which vaccine is to be given every year?
18. 6th day disease is: (FMGE June 2019)  (NEET Pattern 2019)
a. Erythema Infectiosum b. Exanthema subitum a. Hepatitis A
c. Erythema marginatum b. Pneumococcal
d. Erythema nodosum c. Influenza
19. Congenital rubella syndrome is characterized by all of the d. Chickenpox
following except: (PGI May 2019)
a. Microcephaly NEONATOLOGY
b. Patent ductus arteriosus
c. Hutchinson teeth
d. Deafness
NORMAL NEONATE AND NEONATAL
e. Mental retardation RESUSCITATION
20. Diphtheria in a child is characterized by all of the following
25. First stool after birth is green due to: (NEET Pattern 2020)
except: (PGI May 2019)
a. Bile salts
a. Incubation period is mostly 2–5 days
b. Bile pigments
b. Pharyngo- tonsillar diphtheria commonly presents as sore
c. Mucus
throat
d. Gut flora
c. Membrane can cover, pharyngeal walls, tonsils, uvula,
soft palate and glottis 26. Which of the following scoring is done to assess respiratory
d. Fever, malaise and tachycardia distress in Neonates?
e. Anterior nares are the most common sites of infection a. CRIB score
21. A lady developed chickenpox during pregnancy. Highest b. APGAR score
risk of transmission to fetus is at which period of gestation? c. Silverman – Anderson score
 (AIIMS May 2019) d. SNAP score
Most Recent Questions xxv

27. Most sensitive sign of effective neonatal resuscitation is: a. PPV with bag and mask
 (AIIMS May 2019) b. Consider CPAP
a. Color change b. Air entry c. SpO2 monitoring
c. Heart rate increases d. None of the above d. Consider ECG monitor
28. Identify the procedure being performed in a baby admitted 31. Where to look for pre-ductal O2 saturation in PDA in a
in NICU as shown in the given images: (AIIMS May 2019) 3 minute born infant? (NEET Pattern 2019)
a. Fetal left upper limb
b. Fetal left lower limb
c. Fetal right upper limb
d. Fetal right lower limb

DISEASES OF NEWBORN
32. A term newborn 2700 gm weight, presented with jaundice
on 5th day of life with serum bilirubin levels of 14 mg%.
What is the appropriate management?
 (NEET Pattern 2020)
a. Phototherapy
b. Exchange transfusion
c. Routine newborn care
d. Stop breast feeding for 2 days
33. Which among the following is not included in the triad of
necrotizing enterocolitis? (NEET Pattern 2019)
a. Thrombocytopenia
b. Metabolic acidosis
c. Hyponatremia
a. Nasogastric feeding d. Hypokalemia
b. Orogastric feeding
34. Neonatal jaundice becomes detectable at serum bilirubin
c. Orogastric suction
exceeding: (FMGE June 2019)
d. Nasopharyngeal suction
29. Criteria for “high risk” infants: (AIIMS Nov 2019) a. 2 mg%
b. 3 mg%
a. Working mother c. 4 mg%
b. Malpresentation
d. 5 mg%
c. 100 Folic acid tablets not taken during pregnancy
d. Pre-eclampsia 35. Which of the following is/are common causes of non-
immune hydrops? (PGI May 2019)
30. All of the following are components of step A except:
 (AIIMS Nov 2019) a. Cytomegalovirus
b. Human papilloma virus
c. Herpes simplex virus
d. Hepatitis B virus
e. Parvovirus B l9
36. A preterm neonate is presenting with respiratory distress.
On clinical examination there is visible marked lower
chest and xiphoid retraction with inspiratory lag and
minimal nasal flaring. Grunting is audible by the help of
stethoscope. Silverman Anderson score is:
 (JIPMER May 2019)
a. 4 b. 5
c. 6 d. 7
37. Complications of hemolytic disease of newborn include all
of the following except: (PGI May 2019)
a. Thrombocytopenia
b. Late anemia of infancy
c. Inspissated bile syndrome
d. Hypoglycemia
e. Cerebral palsy
xxvi Review of Pediatrics & Neonatology

38. The given investigation is used for all except: a. Transposition of great arteries
 (AIIMS Nov 2019) b. Total anomalous pulmonary venous connection
c. Tetralogy of Fallot
d. Ebstein’s anomaly
44. Severity of cyanosis in Fallot’s physiology is best decided
by: (FMGE June 2019)
a. Size of VSD
b. Overriding of aorta
c. Degree of RVH
d. Degree of pulmonary stenosis

PEDIATRIC HEMATOLOGY
45. Vitamin K dependent coagulation factors in children are:
a. To detect apnea b. Neonatal resuscitation  (PGI May 2019)
c. To detect central cyanosis a. Factor 2
d. In labour room for newborn assessment b. Factor 7
39. APGAR score 3 at 1 minute indicates:(NEET Pattern 2019) c. Factor 8
a. Mildly depressed baby d. Factor 9
b. Further resuscitation not needed e. Factor 10
c. Severely depressed baby
d. Normal baby TUMORS OF INFANCY AND CHILDHOOD
SYSTEMIC PEDIATRICS 46. Which of the following malignancy is seen in a male child
with short stature, low IQ, murmur in pulmonary area and
lymphedema? (JIPMER May 2019)
PEDIATRIC CARDIOLOGY
a. Retinoblastoma
40. In fetal circulation, maximum oxygen saturation is seen in: b. Hepatocellular carcinoma
 (NEET Pattern 2020) c. Juvenile myelo-monocytic leukemia
a. SVC b. IVC d. Germ cell tumor
c. Ascending aorta d. Right ventricle 47. A 9-year-old boy presenting with lytic lesion on midshaft
41. All of the following produce pulmonary oligemia except: tibia. On HPE, there are grooved nuclei, giant cells,
 (NEET Pattern 2020) eosinophilic cytoplasm and fibroblastic proliferation. X-ray
a. TGA b. TOF is as shown below. What is the likely diagnosis?
c. Small ASD d. Tricuspid atresia  (AIIMS Nov 2019)
42. Figure of 3 sign is seen in?
a. Mitral dtenosis b. Coarctation of aorta
c. Transposition of great arteries
d. Aortic stenosis
43. A 2-year-old boy presented with cyanosis and clubbing.
The chest X-ray picture of the child is shown alongside.
What is the likely diagnosis? (NEET Pattern 2020)

a. Langerhans cell histiocytosis


b. Giant cell tumor
c. Chondroblastoma
d. Osteoid osteoma
Most Recent Questions xxvii

PEDIATRIC GASTROINTESTINOLOGY 52. An infant is admitted with respiratory distress and


prolonged expiration with rhonchi in chest. CXR shows
48. Assertion: Antibiotics should not be given to children hyperinflation. What is the diagnosis? (FMGE June 2019)
having diarrhea (AIIMS May 2019)
a. Pneumonia
Reason: Diarrhea in children is mostly caused by viruses b. Croup
c. Asthma
a. Both Assertion and Reasoning are independently true/
d. Bronchiolitis
correct and Reason is the correct explanation of the
Assertion 53. Which of these is used in treatment of bronchiolitis?
b. Both Assertion and Reasoning are independently true/  (FMGE June 2019)
correct and Reason is not the correct explanation of the
a. Ribavarin
Assertion
b. Amantadine
c. Assertion is independently a true/correct statement but the
c. Racemic epinephrine
Reason is independently an incorrect/false statement
d. Assertion is independently a false/incorrect statement but d. Corticosteroids
the Reason is independently a true/correct statement 54. An 8-year-old child presents with prolonged cough for
e. Both Assertion and Reason are independently a false/ 3 months, hemoptysis for 1 month and fever for 4 days.
incorrect statement Chest X-ray is shown below. What is the diagnosis?
49. A 3-year-old child with weight of 12 kg is having loose  (AIIMS Nov 2019)
motions. He is thirsty, drinks eagerly, tears are absent and
skin pinch goes back slowly. What is the best treatment
plan for this child? (FMGE June 2019)
a. 1200 mL RL over 12 hours
b. 600 mL RL over 6 hours
c. 900 mL ORS over 4 hours
d. 300 mL ORS per episode of loose stool

PEDIATRIC RESPIRATORY DISORDERS


50. A 2-year-old baby weighing 11 kg had fever, cough, fast
breathing and chest indrawing. His RR was 38/min. Under
what category of IMNCI is he classified for pneumonia and
what is the management? (NEET Pattern 2020)
a. No pneumonia; send him home with advice on follow up
b. Pneumonia; give oral amoxicillin
c. Severe pneumonia; immediate referral to higher centre a. Hydatid cyst
d. Very severe pneumonia; give first dose of antibiotic and b. Lung abscess
refer to higher centre c. Pulmonary tuberculosis
51. A 2-year-child presented with low grade fever and stridor. d. Congenital thoracic malformation
What is the likely diagnosis? (NEET Pattern 2020)

DISORDERS OF KIDNEY AND URINARY TRACT


55. Primary pathology in Potter’s syndrome is:
 (JIPMER May 2019)
a. Pulmonary hypoplasia
b. Renal agenesis
c. Oligohydramnios
d. Single umbilical artery

PEDIATRIC ENDOCRINOLOGY
56. Wide open posterior fontanelle with large tongue with
rough dry skin is seen in: (FMGE June 2019)
a. Acute Laryngotracheobronchitis a. Pellagra
b. Acute Epiglottitis b. Down syndrome
c. Foreign Body aspiration c. Congenital hypothyroidism
d. Acute Bronchiolitis d. Nutritional rickets
xxviii Review of Pediatrics & Neonatology

57. A phenotypically female child presented with ambiguous a. Oxcarbamazepine


genitalia and clitoromegaly. On examination, she was b. Zonisamide
found to have hypertension. Her karyotype picture is c. Lacosamide
shown below. d. Gabapentin
62. The drug of choice for Juvenile myoclonic epilepsy (Janz
syndrome):
a. Oxcarbamazepine
b. Zonisamide
c. Valproic acid
d. Lacosamide
63. All of the following are causes of hypotonia in 2 days old
newborn except: (JIPMER May 2019)
a. Prematurity
b. Anterior horn disease
c. Down syndrome
d. Cerebral palsy
64. Febrile seizure is called as complex when it lasts for:
 (JIPMER May 2019)
a. > 5 minutes
b. > 10 minutes
c. > 15 minutes
d. > 20 minutes
65. Which of the following genes is implicated in the
Most likely enzyme deficiency is: (NEET Pattern 2020)
pathogenesis of juvenile myoclonic epilepsy?
a. 21 hydroxylase  (AIIMS May 2019)
b. 17, 20 lyase
a. FMR-1
c. 11 β hydroxylase
d. 3 β OH steroid dehydrogenase b. Fibrillin-1
c. CHYNN-1
58. Which of the following can cause hypertension in children?
d. GABRA-1
 (PGI May 2019)
66. A baby, 18 months of age, presents with poor feeding,
a. Multiple pituitary hormone deficiency
irritability and low grade fever for 3 days. On examination,
b. Hyperadrenalism
he has a bulging anterior fontanelle. CSF examination
c. Pheochromocytoma
revealed low glucose. What is the most likely diagnosis?
d. Hypothyroidism
 (AIIMS May 2019)
e. Hyperthyroidism
59. A three weeks old baby has ambiguous genitalia, BP- a. Meningitis
46/30 mm Hg, serum potassium- 6 mEq/dL, serum sodium b. Pneumonia
127 mEq/dL, blood glucose- 70 mg%. Patient was started c. Intracranial hemorrhage
on intravenous fluids. Which of the following specific d. Pseudotumor cerebri
treatment options will you add? (AIIMS May 2019) 67. Choose the right option among the keywords, for the
a. Insulin following clinical scenarios: (AIIMS May 2019)
b. Potassium exchange resin Theme: Seizure
c. Hydrocortisone Keywords:
d. Hypertonic saline a. Rectal diazepam
PEDIATRIC NEUROLOGY b. Inhalation phenobarbitone
c. Intramuscular propofol
60. Which epileptic syndrome is associated with left d. Intravenous phenytoin
hemispheric atrophy in child? e. Intravenous midazolam
a. Dravet syndrome f. Oral valproate
b. West syndrome g. Intranasal carbamazepine
c. Lennox-Gastaut syndrome Clinical Scenario 1
d. Dyke-Davidson-Masson syndrome A 10-month-old baby was found convulsing for 45
61. A 15-year-old boy with partial seizures who develops minutes. Senior Resident recommends intravenous
secondary generalization with tonic clonic seizures should lorazepam. Junior Resident could not get intravenous
be given: access. Next step in the management is:
Most Recent Questions xxix

Clinical Scenario 2 c. Due to past varicella infection 10 years back


When Senior Resident comes back, intravenous access d. Diagnosis is made by S. electrophoresis
has been established but Baby is still convulsing. What e. Patient can develop seizures
should be the next step?
68. Choose the correct statements about juvenile myoclonic BEHAVIORAL DISORDERS IN CHILDREN
epilepsy: (AIIMS Nov 2019)
70. True about Asperger syndrome is: (PGI May 2019)
a. Polygenic inheritance a. More common in girls
b. Lamotrigine decreases myoclonus b. Repetitive activity patterns
c. Benzodiazepines can treat myoclonus c. Subnormal intelligence is a consistent feature
d. Valproate should be avoided d. Severe language impairment is characteristic
A. a, b, c are correct
B. a and c are correct PEDIATRIC SURGICAL DISORDERS
C. b and d are correct
71. VACTERL anomalies include: (PGI May 2019)
D. a, b, c, d are correct
a. Ventriculomegaly
69. Which of the following statements are true/false regarding
b. Cardiomegaly
a patient with SSPE: (AIIMS Nov 2019)
c. Tracheal stenosis
a. Myoclonic jerks are usually seen d. Anorectal anomaly
b. EEG shows periodic complexes e. Radial limb anomaly
Answers with Explanations

GENERAL PEDIATRICS
DEVELOPMENT
1. a. Can extend arm; b. Can voluntarily drop object; c. Can hold object with thumb and index finger and e. Can swap object
from one hand to other  Ref: Ghai’s Essential Pediatrics, 9th ed, p 48-49. Ronald S. Illingworth. The development of the infant
and the Young Child. 9th ed, p 118-120
The usual age at which the milestones mentioned in the question are attained is:
Milestone Usual age
a. Can extend arm 6 months: Stretches arms out to be taken;
b. Can voluntarily drop object 6 months: Drops 1 cube when another is given
c. Can hold object with thumb and index finger 9 months: Immature pincer grasp appears
d. Can point something with index finger 15 months: Points to objects in which he is interested
e. Can swap object from one hand to other 7 months: Transfers objects
2. b. 6 months—transfer of objects and e. 14 months—pincer grasp  Ref: Ghai’s Essential Pediatrics, 9th ed, p 43-49; Ronald S.
Illingworth. The Development of the Infant and the young child. 9th ed, p 117-120
The usual age at which the milestones mentioned in the question are attained is:
Milestone Usual age
a. Holding of objects with both hands Bidextrous grasp is usually attained around 20 weeks (5 months)
b. Transfer of objects 7 months
c. Tripod sitting 6 months
d. Pulls to standing 9 months: Child pulls himself to standing position & begins to stand holding onto furniture
e. Pincer grasp Appears at 9 months
3. b. 4 years  Ref: Ghai’s Essential Pediatrics, 9th ed, p 43-49
The usual age at which the milestones mentioned in the question are attained is:
Milestone Usual age
Can climb up stairs with alternate feet 3 years
Ride a tricycle 3 years
Can remember name, age 3 years
Narrate stories 4 years

So, the developmental age of this child is 3 years.


4. b. 5 months  Ref: Ronald S. Illingworth. The Development of the Infant and the Young Child. 9th ed, p 117
Bidextrous grasp appears at 20 weeks (5 months).

NUTRITION AND MALNUTRITION


5. b. Normal lean body mass  Ref: Nelson 20/e p 272
Inadequate energy intake and/or inadequate protein intake increases catabolism of body protein reservoirs (i.e. decrease in lean body
mass) so as to provide substrate for energy and free amino acids required to support normal physiologic function.
Skin fold thickness and BMI (body mass index) may be falsely elevated in edema, even when a child is malnourished.
ECF volume may be normal or preserved, even in case of malnutrition.
So, a normal lean body mass, virtually rules out PEM.
6. b. Triple  Ref: Ghai’s Essential Pediatrics, 9th ed, p 89
Milk source Energy (Kcal/100 mL) Protein (g/100 mL)
Cow 67 3.3
Buffalo 117 4.3
Goat 72 3.7
Human 65 1.1
Answers with Explanations xxxi

7. c. Height for age   Ref: Ghai’s Essential Pediatrics, 9th ed, p 93


‘Stunting’ or decrease in height for age is an indicator of chronic malnutrition.
8. b. 1.1 g  Ref: Ghai’s Essential Pediatrics, 9th ed, p 89
Amount of Protein present in 100 mL of breast milk is 1.1 g.
9. d. Weight for height—3 SD less than median  Ref: Ghai’s Essential Pediatrics, 9th ed, p 97
As per WHO criteria, severe acute malnutrition (SAM), is defined, in children between 6 months to 5 years age, by the presence of
any 1 or more of the following:
•• Weight for height—3 SD less than median, on WHO growth standards
•• Presence of bipedal edema of nutritional origin
•• Mid upper arm circumference < 11.5 cm.

FLUID AND ELECTROLYTE DISTURBANCES


10. a. Reduced skin turgor—mild dehydration; b. Dry mucus membrane—moderate dehydration; c. Urine output <0.5 mL/kg/
hour—moderate dehydration; d. Fluid deficit of 150 mL/kg—severe dehydration and e. Rapid and deep breathing—severe
dehydration  Ref: Nelson 20/e p 1870
The following are the features of varying degrees of dehydration in children:

Minimal or No dehydration MILD to moderate dehydration Severe dehydration


Symptom
(<3% loss of body weight) (3–9% loss of body weight) (>9% loss of body weight)
Mental status Well; alert Normal, fatigued or restless, irritable Apathetic, lethargic, unconscious
Thirst Drinks normally; might refuse liquids Thirsty; eager to drink Drinks poorly; unable to drink
Heart rate Normal Normal to increased Tachycardia, with bradycardia in
most severe cases
Quality of pulses Normal Normal to decreased Weak, thready, or impalpable
Breathing Normal Normal, fast Deep
Eyes Normal Slightly sunken Deeply sunken
Tears Present Decreased Absent
Mouth and tongue Moist Dry Parched
Skinfold Instant recoil Recoil in <2 sec Recoil in >2 sec
Capillary refill Normal Prolonged Prolonged; minimal
Extremities Warm Cool Cold; mottled; cyanotic
Urine output Normal to decreased Decreased Minimal

GENETICS AND GENETIC DISORDERS


11. a. Williams syndrome  Ref: Nelson 20/e p 618
The clinical features of Williams syndrome are: round face with full cheeks and lips, long philtrum, squint, supravalvular aortic
stenosis and other cardiac malformations, varying degrees of intellectual disability and friendly personality. It results from a deletion
in chr 7q11.23.
12. c. Males have IQ 20–40  Ref: Nelson 20/e p 622-623
Discussing about the options one by one:

Options T/F Explanation


A. Triple nucleotide CAG sequence mutation False CGG repeats are seen in Fragile X syndrome

B. 10% Female carriers mentally retarded False Mental impairment with a retarded or borderline IQ occurs in 35% of carriers

C. Males have IQ 20–40 True Developmental delay: mean IQ = 42 in Males; IQ is higher if significant residual
FMRP is produced (e.g. females and mosaic males)

D. Gain of function mutation False The abnormal expansion of CGG repeats leads to hypermethylation and consequent
silencing of FMR1 gene
xxxii Review of Pediatrics & Neonatology

INBORN ERRORS OF METABOLISM


13. b. Mental retardation is the common presentation  Ref: Nelson’s Textbook of Pediatrics. 20/e p 647-648
Hartnup disease:
•• It is a rare inborn error of metabolism with autosomal recessive inheritance
•• Neutral amino acids, including tryptophan, are not transported across the brush border epithelium of the intestine and kidneys
•• This results in deficiency of synthesis of nicotinamide
•• Most cases are asymptomatic
•• In symptomatic cases. a photo-induced pellagra-like syndrome may be seen
•• Mental development is usually normal; Neurologic symptoms are usually reversible
•• Administration of nicotinamide and protection from sunlight improves both cutaneous and neurologic manifestations.

DISEASES OF IMMUNE SYSTEM


14. d. Wiskott-Aldrich syndrome  Ref: Nelson’s Textbook of Pediatrics. 20/e p 2420-2421
Eczematous rash, thrombocytopenia and immunodeficiency (recurrent infections) are features of Wiskott Aldrich syndrome, a X
linked recessive disorder due to mutation of WAS gene mutation on chr Xp11.
15. a. Kawasaki disease  Ref: Nelson’s Textbook of Pediatrics. 20/e p 1209-1211
Fever for >5 days, along with the clinical features mentioned in the question are suggestive of Kawasaki disease, for which the
treatment of choice is IV Immunoglobulin.
16. b. Viral myositis  Ref: Nelson’s Textbook of Pediatrics. 20/e p 1559
A short 3 day history of pain and tenderness in calf muscles, following a viral prodrome, along with elevated CPK levels suggests a
diagnosis of Viral myositis.
The diagnostic criteria for Juvenile Dermatomyositis are the following:
Diagnostic criteria for juvenile dermatomyositis
Classic rash Heliotrope rash of the eyelids Gottron papules
Plus any 3 of the following:
Weakness Symmetric Proximal
Muscle enzyme elevation (≥1) Creatine kinase
Aspartate aminotransferase
Lactate dehydrogenase
Aldolase
Electromyographic changes Short, small polyphasic motor unit potentials
Fibrillations
Positive sharp waves
Insertional irritability
Bizarre, high-frequency repetitive discharges
Muscle biopsy Necrosis
Inflammation

INFECTIOUS DISEASES
17. d. Acute otitis media  Ref: Ghai 9th ed p 209-210
The given picture shows Koplik spots in buccal mucosa and erythematous maculopapular confluent rash all over the body, along
with coryza and conjunctivitis, which point towards a diagnosis of Measles. Acute otitis media is the most common complication
of Measles.
18. b. Exanthema subitum  Ref: Ghai 9th ed p 212
The following are the terminologies used for some diseases seen in children, where there is fever with rash:
Terminology Other names for the disease Etiology
First disease Rubeola/Measles Measles virus
Second disease Scarlet fever Streptococcus pyogenes
Third disease Rubella, German measles, 3-day measles Rubella virus
Fourth disease Staphylococcal Scalded Skin Syndrome, Ritter’s disease Staphylococcus aureus
Fifth disease Erythema infectiosum Parvovirus B19
Sixth disease Exanthem subitum/ Roseola infantum Human Herpes Virus 6B or 7
Answers with Explanations xxxiii

19. c. Hutchinson teeth  Ref: Nelson’s Textbook of Pediatrics. 20/e p 1551


Hutchinson teeth is a part of the ‘Hutchinson triad’ seen in congenital Syphilis. Rest of the above mentioned features can be seen in
Congenital Rubella syndrome.
20. d. Fever, malaise and tachycardia and e. Anterior nares are the most common sites of infection  Ref: Ghai 9th ed p 236-
237; Nelson 20/e p 1346
Discussing about the options one by one:
Options T/F Comments
a. Incubation period is mostly 2–5 days True Respiratory tract is the most common portal of entry
b. P
 haryngo-tonsillar diphtheria commonly presents True Sore throat, dysphagia and muffled voice are frequently present
as sore throat
c. M
 embrane can cover, pharyngeal walls, tonsils, True Membrane can extend to adjacent areas
uvula, soft palate and glottis
d. Fever, malaise and tachycardia is seen in all False Fever is present in 50% cases; Tachycardia is an evidence of cardiac toxicity,
which occurs in 10–25% patients
e. Anterior nares are the most common sites of False Infection of the anterior nares is more common among infants and causes
infection in older children serosanguineous, purulent, erosive rhinitis with membrane formation.

21. b. 14–18 weeks  Ref: Nelson 20/e p 1580-1582


Congenital varicella syndrome occurs in ~0.4% of infants born to women who have varicella during pregnancy before 13 week
of gestation and in ~2% of infants born to women with varicella between 13 and 20 week of gestation. Rarely, cases of congenital
varicella syndrome have been reported in infants of women infected after 20 week of pregnancy.
22. d. Picorna virus  Ref: Ghai 9th ed p 215
The given clinical scenario of fever along with oral ulcers and papulo-vesicular rashes on palms and soles suggest a diagnosis of
‘hand-foot-mouth disease’ (HFMD). This illness is caused by the viruses of enterovirus genus, belonging to family Picornaviridae, the
most common causative organisms being Coxsackie virus A16 and Enterovirus 71.
Rare complications include aseptic meningitis, encephalitis, polio like paralysis, myocarditis and respiratory distress.

IMMUNIZATION
23. d. No vaccine needed  Ref: Nelson 20/e p 1346-1347
Management of contacts of Diphtheria
•• All household contacts and people who have had intimate respiratory or habitual physical contact with a patient are closely
monitored for illness through the 7-day incubation period.
•• Cultures of the nose, throat, and any cutaneous lesions are performed.
•• Antimicrobial prophylaxis is presumed effective and is administered regardless of immunization status, using a single injection
of benzathine penicillin G (600,000 units IM for patients < 6 years age, or 1,200,000 units IM for patients > 6 years of age) or
erythromycin (40–50 mg/ kg/day divided qid PO for 10 days; maximum: 2 g/day).
•• Diphtheria toxoid vaccine, in age-appropriate form, is given to immunized individuals who have not received a booster dose
within 5 years.
•• Children who have not received their 4th dose should be vaccinated.
•• Those who have received fewer than 3 doses of diphtheria toxoid or who have uncertain immunization status are immunized
with an age-appropriate preparation on a primary schedule.
24. c. Influenza  Ref: IAP guidebook on Immunization 2018-2019, p 399
For the 2019-20 influenza season, the recommended vaccines should contain:
•• A/Brisbane/02/2018 (H1N1)
•• A/Kansas/14/2017 (H3N2)
•• B/Colorado/06/2017
•• B/Yamagata/16/88 lineage
Recommended for children with high-risk conditions; for 1st time vaccination in age group 6 months-9 years, 2 doses 1 month apart,
while for 9 years and above: single dose; Annual revaccination with a single dose is recommended.

NEONATOLOGY

NORMAL NEONATE AND NEONATAL RESUSCITATION


25. b. Bile pigments  Ref: Ghai 9th ed p 136
Meconium, the first stools passed by the baby, gets its greenish black colour, mainly due to bile pigments.
xxxiv Review of Pediatrics & Neonatology

26. c. Silverman – Anderson score  Ref: Dorling JS, Field DJ, Manktelow B. Neonatal disease severity scoring systems Archives of
Disease in Childhood - Fetal and Neonatal Edition 2005;90:F11-F16
The CRIB (clinical risk index for babies) score: a tool for assessing initial neonatal risk and comparing performance of neonatal
intensive care units. CRIB II score is a validated measure of initial mortality risk and illness severity within one hour of admission. It
takes into account the birth weight, gestational age, body temperature, base excess and sex of the baby to determine initial mortality
risk.
The Score for Neonatal Acute Physiology (SNAP) developed by Richardson et al., in 1993 for babies of all birth weights and
validated as a predictor of mortality, morbidity, is a physiology-based score that uses 34 routinely available vital signs and laboratory
test results.
Silverman-Anderson score is used to quantify respiratory distress in preterm neonates. It is calculated as shown in the following
table:

27. c. Heart rate increases  Ref: Neonatal Resuscitation Guidelines, 2010


Increase in heart rate is the most sensitive indicator of effective positive pressure ventilation.
28. b. Orogastric feeding  Ref: AIIMS NICU Protocol 2019
The given images show an orogastric feeding tube being inserted and orogastric feeding being administered in a neonate.
29. d. Pre-eclampsia  Ref: Indian Academy of Pediatrics Guidelines
As per IAP,
A ‘high-risk’ infant is a baby, who has a greater than average chance of morbidity or mortality because of conditions or
circumstances superimposed on the normal course of events associated with birth & the adjustment to extrauterine existence.
Factors to define ‘high risk’ infant:
•• Social factors: Maternal age <16 years or >40 years, unmarried, physical stress,
•• Past medical history: Diabetes Mellitus, genetic disorders, hypertension
•• Previous pregnancy: Intrauterine death, neonatal death, IUGR, IEMs, congenital malformations
•• Present Pregnancy: Vaginal bleeding, PROM, multiple gestation, pre-eclampsia, abnormal USG findings
•• Labor & delivery: Obstructed labor, fetal distress, forceps delivery, meconium stained liquor
•• Neonate: Birthweight <2000 g or >4000 g, gestation <37 or >42 wk, SGA, respiratory distress, congenital malformation.
30. b. Consider CPAP  Ref: Neonatal Resuscitation Guidelines, 2010
CPAP (Continuous positive airway pressure) is preferred in babies with laboured breathing or persistent cyanosis. Rest all are the steps
taken, whenever a neonate presents with apnea, gasping or heart rate < 100/minute just after birth.
31. c. Fetal right upper limb  Ref: AIIMS NICU Protocol 2019
As the right brachial artery arises from right subclavian artery, which arises from brachiocephalic trunk (1st branch of arch of aorta),
it shows pre-ductal saturation. Ductus arteriosus usually connects pulmonary artery to aorta (just distal to the origin of left subclavian
artery).
Answers with Explanations xxxv

DISEASES OF NEWBORN
32. c. Routine newborn care  Ref: AIIMS NICU Protocol 2019
The phototherapy cut-off for an otherwise well neonate, on 5th day of life would be ~ 20 mg/dL. SO, as the bilirubin level of this
neonate is less than that, routine newborn care needs to be continued, and nothing needs to be done in addition.
33. d. Hypokalemia  Ref: AIIMS NICU Protocol 2019
The triad of blood findings seen in necrotising enterocolitis include: thrombocytopenia, metabolic acidosis and hyponatremia.
34. d. 5 mg%  Ref: AIIMS NICU Protocol 2019
Clinical jaundice becomes evident in a neonate at serum bilirubin level > 5 mg/dL.
35. a. Cytomegalovirus; c. Herpes simplex virus and e. Parvovirus B l9   Ref: Nelson 20/e p 883-84
This clinical picture of excessive abnormal fluid in 2 or more fetal compartments (skin, pleura, pericardium, placenta, peritoneum,
amniotic fluid), termed hydrops fetalis.
36. d. 7  Ref: Dorling JS, Field DJ, Manktelow B. Neonatal Disease Severity Scoring Systems Archives of Disease in Childhood - Fetal
and Neonatal Edition 2005;90:F11-F16
Silverman scoring for this neonate is done, as shown below:
Marked lower chest retractions → 2
Marked xiphisternal retractions → 2
Inspiratory lag → 1
Minimal nasal flare → 1
Grunting heard by stethoscope → 1
Total score = 7.
37. a. Thrombocytopenia; b. Late anemia of infancy; c. Inspissated bile syndrome; d. Hypoglycemia and e. Cerebral palsy 
Ref: Nelson 20/e p 886
Complications of hemolytic disease of newborn include late anemia of infancy, inspissated bile syndrome, portal vein thrombosis
& kernicterus (leading to extrapyramidal cerebral palsy). Symptomatic hypoglycemia may occur before or during an exchange
transfusion in moderately to severely affected infants.
38. a. To detect apnea  Ref: Nelson’s 20/e p 2211-17
For early detection of neonatal apnea, respiratory impedance monitoring is a better and more sensitive tool than pulse oximetry.
39. c. Severely depressed baby  Ref: Nelson 20/e p 798
Apgar score of 0-3 indicates severely depressed baby.

SYSTEMIC PEDIATRICS

PEDIATRIC CARDIOLOGY
40. b. IVC  Ref: Nelson’s 20/e p 2161-62
In fetal circulation, placenta is the source of O2. Oxygenated blood from placenta is carried by the umbilical vein (spO2 80%), that
joins the portal vein. This blood bypasses hepatic circulation via ductus venosus and reaches IVC, which has a saturation of 70% in
the fetal circulation. The spO2 in right ventricle, aorta and superior vena cava are less than the IVC, in fetal life.
41. a. TGA  Ref: Nelson’s 20/e p 2223-25
In TGA, there is usually pulmonary plethora, unless there is associated pulmonary stenosis.
42. b. Coarctation of aorta  Ref: Nelson’s 20/e p 2161-62
X-ray findings of coarctation of aorta include notching of inferior margin of ribs, figure of 3 appearance & E sign.
43. c. Tetralogy of Fallot  Ref: Nelson’s 20/e p 2211-17
The given chest X-ray shows ‘boot shaped heart’, along with pulmonary oligemia, suggestive of Tetralogy of Fallot.
44. d. Degree of pulmonary stenosis  Ref: Nelson’s 20/e p 2211-17
The severity of cyanosis in tetralogy of Fallot is directly proportional to the severity of pulmonary stenosis. However, the intensity and
severity of the systolic murmur is inversely proportional to the severity of disease.

PEDIATRIC HEMATOLOGY
45. a. Factor 2; b. Factor 7; d. Factor 9 and e. Factor 10  Ref: Nelson 20/e p 342
Vitamin K is a cofactor for γ-glutamyl carboxylase, an enzyme that performs post-translational carboxylation, converting glutamate
residues in proteins to γ-carboxyglutamate (Gla). The Gla residues, by facilitating calcium binding, are necessary for protein
function. The classic Gla-containing proteins involved in blood coagulation that are decreased in vitamin K deficiency are factors II
(prothrombin), VII, IX, and X.
xxxvi Review of Pediatrics & Neonatology

TUMORS OF INFANCY AND CHILDHOOD


46. c. Juvenile myelo-monocytic leukemia  Ref: Nelson Textbook of pediatrics 20/e p 2420
•• Male child with short stature, low IQ, murmur in pulmonary area and lymphedema suggests Noonan syndrome with
pulmonary valve stenosis
•• Hematological abnormality seen in Noonan syndrome is Juvenile Myelomonocytic Leukemia
47. a. Langerhans cell histiocytosis  Ref: Nelson 20/e p 2484-89
The given X-ray shows punched out lytic lesions, histopathology of which shows cells with grooved nuclei and giant cells, which
suggest a diagnosis of Langerhans cell histiocytosis.

PEDIATRIC GASTROINTESTINOLOGY
48. a. Both Assertion and Reasoning are independently true/correct and Reason is the correct explanation of the Assertion 
Ref: Nelson's 20/e p 1854-74
Most common cause of diarrhea in children is Rotavirus. So, antibiotics are usually not recommended in treatment of children with
diarrhea.
49. c. 900 mL ORS over 4 hours  Ref: Nelson 20/e p 1869-71
This child probably has ‘some dehydration’, so he needs treatment plan B, i.e. 75 mL/kg ORS over 4 hours → 75 × 12 = 900 mL
ORS over 4 hours.

PEDIATRIC RESPIRATORY DISORDERS


50. b. Pneumonia; give oral amoxicillin  Ref: Nelson's 20/e p 2088-93
As this child’s age is 2 years, respiratory rate < 40/min is normal for him. But, as chest indrawing is also present, but there are no
danger signs, this child has ‘pneumonia’. So, he needs to be treated with oral antibiotics.
51. a. Acute Laryngotracheobronchitis  Ref: Nelson's 20/e p 2032-34
The given X-ray in a 2-year-child with fever and stridor shows ‘steeple sign’, with narrowing of upper airway, suggestive of acute
laryngotracheobronchitis or croup.
52. d. Bronchiolitis  Ref: Nelson's 20/e p 2044-47
The given clinical features, during infancy, along with hyperinflation on chest X-ray suggests a diagnosis of ‘acute bronchiolitis’.
53. a. Ribavarin  Ref: Nelson's 20/e p 2044-47
Nebulised Ribavirin may be used in immunocompromised babies with bronchiolitis due to RSV infection.
54. c. Pulmonary tuberculosis  Ref: Nelson's 20/e p 1450-53
The prolonged fever, cough and hemoptysis in an 8-year-old child with the above chest X-ray, suggests a possible diagnosis of
pulmonary tuberculosis.

DISORDERS OF KIDNEY AND URINARY TRACT


55. b. Renal agenesis  Ref: Nelson 20/e p 2513, 2554
The primary pathology in Potter’s sequence is bilateral renal agenesis → Oligohydramnios → Flat facies & pulmonary hypoplasia.

PEDIATRIC ENDOCRINOLOGY
56. c. Congenital Hypothyroidism  Ref: Nelson's 20/e p 2669-73
Wide open posterior fontanelle with large tongue with rough dry skin is seen in congenital hypothyroidism.
57. b. 17, 20 lyase  Ref: Nelson's 20/e p 2714
A genotypic male, as shown in the given karyotype (46, XY), who is phenotypically female, suggests undervirilisation; Presence of
undervirilisation suggests decreased production of sex steroids, while presence of hypertension suggests increased production of
mineralocorticoids, both of which are compatible with a diagnosis of 17, 20 lyase (or 17 hydroxylase) deficiency.
58. b. Hyperadrenalism; c. Pheochromocytoma and e. Hyperthyroidism  Ref: Nelson Textbook of Pediatrics 20/e p 2668-69
Hypotension is usually seen in multiple pituitary hormone deficiency. In hypothyroidism, Bradycardia, decreased ventricular filling
and cardiac contractility together lead to low cardiac output and narrow pulse pressure. Elevated diastolic blood pressure may be
present in some adult patients with overt hypothyroidism.
Answers with Explanations xxxvii

59. c. Hydrocortisone  Ref: Nelson's 20/e p 2714


A neonate with ambiguous genitalia, hypotension and hyperkalemia, suggest a diagnosis of congenital adrenal hyperplasia due
to 21 hydroxylase deficiency. In this condition the hyperkalemia and hypotension is due to mineralocorticoid and glucocorticoid
deficiency. So, hydrocortisone can be started, to replace the deficient glucocorticoids and mineralocorticoids.
Potassium exchange resins are used in the long-term management of persistent hyperkalemia, in conditions like chronic kidney
disease.

PEDIATRIC NEUROLOGY
60. d. Dyke-Davidson-Masson syndrome  Ref: Glob J Health Sci. 2013;5(3):195–207
All the above options are epileptic syndromes, among these only Dyke-Davidson-Masson syndrome have characteristic
Childhood Cerebral Hemiatrophy features.
Dyke-Davidson-Masson syndrome:
•• Hemiatrophy or hypoplasia of unilateral cerebral hemisphere resulting from CNS insult in fetal or early childhood period
with compensatory ipsilateral calvarial hypertrophy
•• Insult could be vascular, congenital or acquired ischaemic disease, trauma or inflammation
•• In-utero gestational vascular occlusion involving middle cerebral vascular territory or coarctation of mid-aortic arch may decrease
carotid arterial flow.
61. a. Oxcarbamazepine  Ref: Seizures and Epilepsy in Children by Prathiba Singhi, p 111

Epilepsy syndrome First line AED 2nd line AED


Juvenile Myoclonic Epilepsy VPA PB, CZP
Progressive Myoclonic Epilepsy VPA CZP
Lennox-Gastaut Syndrome VPA, LTG, TPM CZP, LEV
Infantile Spasm ACTH, VGB VPA, CLB
Benign Rolandic Epilepsy CBZ, VPA, LTG PHT, LEV
Severe Myoclonic Epilepsy of Infancy CLB,CZP, VPA LEV
Landau-Kleffner Syndrome VPA, LTG, STEROIDS LEV, TPM
Myoclonic Astatic Epilepsy CZP, ZLB, VPZ LEV, LTG

Carbamazepine or Oxcarbamazepine are the drug of choice for partial seizures with or without secondary generalization.
62. c. Valproic acid  Ref: Seizures and Epilepsy in Children by Prathiba Singhi, p 98
Juvenile Myoclonic Epilepsy (JME):
•• Valproate—Drug of choice and controls seizures in 80% of cases (relapse rate is high on stoping the drug).
•• Lamotrigine preferred in adolescent girls due to risk of Polycystic Ovarian Syndrome with valproate.
•• Azetazolamide—given on long-term help in cases with poor response to Antiepileptic drugs.
63. d. Cerebral Palsy  Ref: Nelson 20/e p 2797
•• All the above causes can present with hypotonia (either central or peripheral), though, hypotonic Cerebral palsy is a very rare
subtype of CP which presents with hypotonia
•• Hypotonia with profound weakness sugests LMN lesion
•• Hypotonia with preserved power (except in acute stage) suggests UMN lesion.
64. c. > 15 minutes  Ref: Nelson 20/e p 2839-49
Complex febrile seizures refer to the febrile seizures, that last for > 15 minutes, or are focal or recur within 24 hours of onset.
65. d. GABRA-1  Ref: Nelson 20/e p 2836-37
Juvenile Myoclonic epilepsy has been linked to mutations in many genes including CACNB4; CLNC2; EJM2, 3, 4, 5, 6, 7, 9;
GABRA1; GABRD; and Myoclonin1/EFHC1.
66. a. Meningitis  Ref: Nelson 20/e p 2830
The given clinical scenario, along with CSF finding of low glucose points towards an underlying diagnosis of meningitis. Meningitis
should be considered in the differential diagnosis, and lumbar puncture should be performed for all infants younger than 6 months of
age who present with fever and seizure, or if the child is ill appearing or at any age if there are clinical signs or symptoms of concern.
67. Clinical Scenario 1: a. Rectal diazepam; Clinical Scenario 2: d. Intravenous phenytoin  Ref: Nelson 20/e p 2839-49
The first drug given to abort seizures in an infant is a short acting benzodiazepine like Lorazepam / Midazolam / Diazepam. Preferred
route of administration is IV, but if IV access is not available, intraosseous, rectal, buccal or nasal route may be used.
xxxviii Review of Pediatrics & Neonatology

68. A. a, b, c are correct  Ref: Nelson 20/e p 2836-41


Juvenile myoclonic epilepsy:
•• It is the most common generalized epilepsy in young adults.
•• It has been linked to mutations in many genes including CACNB4; CLNC2; EJM2, 3, 4, 5, 6, 7, 9; GABRA1; GABRD; and
Myoclonin1/EFHC1
•• It usually starts in early adolescence with myoclonic jerks in the morning, often causing the patient to drop things; generalized
tonic–clonic or clonic–tonic–clonic seizures upon awakening; and juvenile absences.
•• Sleep deprivation, alcohol, photic stimulation, or, certain cognitive activities can act as precipitants.
•• EEG shows generalized 4-5 Hz polyspike–and–slow wave discharges.
•• Drugs useful in treatment include: Valproate, Benzo­diazepines, clonazepam, lamotrigine, and topiramate.
69. Ans: a-T, b-T, c-F, d-F, e-T  Ref: Nelson 20/e p 1545-46

Options T/F Comments


a. Myoclonic jerks are usually seen True Subacute sclerosing panencephalitis (SSPE) is an important cause of
myoclonic jerks in children
b. EEG shows periodic complexes True Early in the course of SSPE, EEG may be normal or show only moderate,
non-specific generalized slowing. The typical EEG picture is characterized by
periodic complexes consisting of bilaterally symmetrical, synchronous, high
voltage bursts of polyphasic, stereotyped delta waves, that repeat at 4–10
second intervals.
c. Due to past varicella infection 10 years back False Past infection with Measles virus has been suspected as an etiological factor
d. Diagnosis is made by s. electrophoresis False Diagnosis is made by serum and CSF anti Measles antibody
e. Patient can develop seizures True Seizures can be seen in a child with SSPE

BEHAVIORAL DISORDERS IN CHILDREN


70. b. Repetitive activity patterns  Ref: Nelson 20/e p 210
Autism spectrum disorder (including Asperger syndrome) is more commonly seen in boys than girls. Although sharing many
characteristics of autism (deficits in social relatedness and restricted range of interests), individuals with Asperger syndrome typically
show normal early language development (syntax and semantics). As they mature, higher-order social and language pragmatic
impairments become prominent features of this disorder.
Affected children have an unusually circumscribed range of interests, which are all-absorbing and interfere with learning of
other skills and with social adaptation. These children may engage in long-winded, verbose monologues about their topics of special
interest, with little regard to the reaction of others. Their inflection pattern (prosody) may be inappropriate to the content of their
conversation, and they might not adjust their rate of speech or vocal volume to the setting.

PEDIATRIC SURGICAL DISORDERS


71. d. Anorectal anomaly and e. Radial limb anomaly  Ref: Nelson’s 20/e p 1783
VACTERL association (also called VATER association) refers to the non-random co-occurrence of birth defects including Vertebral
anomalies, Anal atresia, Cardiac defects, Tracheoesophageal fistula and/or Esophageal atresia, Renal and Radial anomalies and Limb
defects.
Annexures

Annexure 1
 IMPORTANT DEVELOPMENTAL MILESTONES

Age Gross motor Fine motor Language Social


1 month Raises head in prone position, but Hands kept closed; Alerts to sound Regards face of mother
head below plane of body in Palmar grasp reflex present
ventral suspension

2 months Prone: lifts chest off table; No longer clenches fists tightly Vocalises (produces some Smiles socially (after being
Ventral: Holds head in the plane of sound) stroked or talked to)
rest of body

3 months Prone: supports weight on forearms; Holds hands open at rest; Coos (produces long vowel Recognizes mother
In ventral suspension: holds Hand regard present; sounds in musical fashion)
head up weight Holds an object placed in hand

4 months Prone: supports on wrist; Partial Tries to reach for objects; Laughs aloud; orients to Enjoys looking around
weight bearing when made to stand At 5 months, bidextrous grasp voice

6 months Rolls over; Unidextrous or palmar grasp; Babbles, ah-goo, razzes; Mirror play;
Sits with support transfers objects at 7 months lateral orientation to bell Stranger anxiety at 7
Sits without Support (8 months) months

9 months Pivots Uses immature pincer grasp, Says “mama”, “dada” Waves bye bye; understands
Stands with support probes with forefinger indiscriminately “no”; Plays peek a boo

12 months Stands without support; Uses mature pincer grasp; 2-3 words with meaning; Kisses on request; Plays a
Walks with support; Casting; Releases objects Imitates simple sounds; simple ball game
Cruising; Bear walking voluntarily follows simple command
with gesture

15 months Creeps upstairs, walks backwards Scribbles spontaneously; Jargoning; Uses 4-6 words; Indicates wet pants; Asks for
independently builds a tower of 2 blocks follows one step command objects by pointing
without gesture

18 months Runs stiffly; Goes upstairs & Builds a tower of 4 blocks; 10 words with meaning Domestic mimicry;
downstairs holding the railing Turns 2-3 pages at a time; Dry by day
Feeds self with spoon

2 years Walks up and down stairs without Imitates a straight line; Uses pronouns (I, You , Me); Parallel play
help (2 feet/step); Runs well; Builds tower of 7 blocks; follows two step commands;
kicks a ball Turns door knob 2-3 word sentences

3 years Can alternate feet when going up Handedness established; Uses 250 words; uses Group play, shares toys;
steps; pedals tricycle Copies a circle; Dresses & plurals & pass tense; knows full name , age &
undresses except buttons repeats 3 digits gender

4 years Hops; alternate feet going down Copies a rectangle or a cross; Knows colors; says song or Tells ”tall tales”; Goes to toilet
steps also buttons clothing poem from memory; asks alone
questions

5 years Skips; jumps over low obstacles Copies triangle; Asks meaning of words Plays competitive games,
ties shoe laces abides by rules; helps in
household tasks
xl Review of Pediatrics & Neonatology

Annexure 2
 IMPORTANT DEFINITIONS
Terminology Definition / Description
Anticipation Clinical features of a genetic disorder worsens with each successive generation. Seen in trinucleotide repeat
disorders like fragile X syndrome and myotonic dystrophy
Brachycephaly Head shape is shortened from front to back along the sagittal plane
Brachydactyly A condition of having short digits i.e. short fingers
Clinodactyly A medial or lateral curving of fingers; usually refers to incurving of the 5th finger
Development quotient (DQ) Developmental age/actual age * 100
ELBW neonate Any neonate with birth weight < 1000 g
Embryo From day of fertilization (generally 14th day post ovulation) to 8 weeks
Fetal death It is a product of conception that, after separation from mother, does not show any evidence of life
Fetus 9 weeks of gestation to birth
IUGR Babies with clinical features of malnutrition, like: ≥ 3 loose skin folds in buttock region, decreased subcutaneous fat
Large for Date Birth weight > 90th centile of expected, according to gestational age
Live birth A product of conception irrespective of weight or gestational age, that after separation from mother, shows any
evidence of life, like breathing, heart beat or pulsation of umbilical cord
Low Birth weight Any neonate with birth weight < 2500 g, irrespective of gestational age
Macrocephaly Head circumference > 95th percentile for age and sex
Microcephaly Head circumference < -3 SD below the mean for age and sex
Mosaicism Presence of 2 or more different cell lines in the same individual
Neonatal hypothermia Axillary temperature of baby < 36.5ºC
Neonatal period From birth to under 4 weeks (<28 days) age
Nocturnal enuresis Occurrence of involuntary voiding at night after 5 yr age
Obesity Body mass index (BMI) > 95th centile of expected
Plagiocephaly Asymmetric head shape in the sagittal or coronal plane
Post-term neonate A neonate born at a gestational age of ≥ 42 weeks, irrespective of birth weight
Preterm neonate Any neonate born before 37 weeks of gestation irrespective of birth weight
Reduced Penetrance Phenomenon by which some individuals inherit the disease but are phenotypically normal. 50% penetrance means
that 50% of those who carry the gene express the trait
Severe Acute Malnutrition In children, 6–59 months of age, Weight/height or length < -3 Z score, using WHO Growth Charts or Presence of
(SAM) bipedal edema; or Mid- upper arm circumference (MUAC) < 115 mm
Short stature Height < 3rd percentile or - 2 SD below the median height for that age & sex
Small for Date Birth weight < 10th centile of expected, according to gestational age
Stereotypies Stereotyped, rhythmic, repetitive movements or patterns of speech, with lack of variation over time
Still birth A fetal death at a gestational age of 20 weeks or more or weighing > 500 gm
Syndactyly Incomplete separation of fingers. Most commonly seen between 3rd & 4th fingers or 2nd & 3rd toe
Telecanthus Lateral displacement of inner canthi, Inter canthal distance is increased, but interpupillary distance is normal
Term neonate Any neonate born between 37 completed weeks to less than 42 weeks gestation
Tics Sudden, rapid, recurrent, non-rhythmic motor movements or vocalizations
Variable expressivity If a trait is seen in all individuals carrying the mutant gene but is expressed differently among individuals, it is called
variable expressivity, e.g. tuberous sclerosis
VLBW neonate Any neonate with birth weight < 1500 g
Annexures xli

Annexure 3
 IMPORTANT METABOLIC DISORDERS & THEIR SALIENT FEATURES
Disease Enzyme deficient Important features
Alkaptonuria Homogentisic acid oxidase Blackening of urine on standing in children; oochronosis and arthritis in adulthood.
Valvular heart diseases
Andersen disease Branching enzyme Failure to thrive, hypotonia, hepatomegaly, splenomegaly, progressive cirrhosis (death by
5th yr), elevated transaminases
Canavan disease Aspartoacylase Progressive macrocephaly, severe hypotonia in infancy, persistent head lag, delayed
milestones, Feeding difficulties. Later, spasticity, contractures, optic atrophy, seizures
Cori or Forbes Debrancher deficiency Childhood: hepatomegaly, growth retardation, muscle weakness, hypoglycemia,
disease (amylo-1,6-glucosidase) hyperlipidemia, elevated transaminase levels; can progress to liver failure
Fabry disease α-Galactosidase Angiokeratomas in ‘bathing trunk area’; Severe Pain; Decreased sweating; Corneal
(X linked recessive) opacity; Renal, CVS & CNS involvement
Fanconi-Bickel Glucose transporter 2 Failure to thrive, rickets, hepatorenomegaly, proximal renal tubular dysfunction,
syndrome (GLUT-2) impaired glucose & galactose utilization
Farber disease Ceramidase Subcutaneous skin nodules, lymphadenopathy, joint contractures, hoarseness
Galactosemia Galactose-1-phosphate Vomiting, hepatomegaly, cataracts, hypoglycemia, jaundice, aminoaciduria, failure to
uridyl transferase (GALT), thrive
Epimerase, Galactokinase
Gaucher disease Glucocerebrosidase Gaucher cells in bone marrow → pancytopenia → bruising, splenohepatomegaly,
skeletal involvement (bone pain & pathologic fractures); Neurological features may or
may not be present
GM1 β-galactosidase Hepatosplenomegaly, edema, skin eruptions (angiokeratoma).
gangliosidosis Developmental delay; seizures. Cherry-red spot present; Severe neurologic impairment;
decerebrate rigidity. Death by 3-4 yr of age
Hartnup disorder Defective transport of neutral Mostly asymptomatic; photosensitivity, pellagra-like rash; ataxia; Mental development
amino acids normal. Rx: Nicotinic acid
Hawkinsinuria 4-hydroxy phenyl pyruvate Failure to thrive, hepatomegaly, swimming pool odor; metabolic acidosis, ketosis, appear
dioxygenase (4-HPPD) after weaning (~ 6 months)
Hereditary fructose Aldolase B Jaundice, vomiting, Failure to thrive, hepatic failure; symptoms appear on intake of
intolerance fructose-containing substance
Hers disease Liver phosphorylase Hepatomegaly, typically mild hypoglycemia, hyperlipidemia, and ketosis
Homocystinuria Cystathionine β-Synthase Developmental delay; intellectual disability, Seizures;
(Classic), Methylene Ectopia lentis, cataract, retinal detachment;
tetrahydrofolate Reductase, Skeletal abnormalities like Marfan syndrome;
Defects in Methyl-cobalamin Behavioral problems; Thromboembolic episodes
Formation Rx: high doses of vitamin B6
Hurler disease α-L-Iduronidase Hepatosplenomegaly, coarse facies, corneal clouding, large tongue, joint stiffness, short
stature, skeletal dysplasia, copious nasal discharge, Valvular heart disease
Krabbe disease Galactocerebrosidase Irritability, seizures, hypertonia; Optic atrophy; severe mental impairment; Opisthotonos
Lesch-Nyhan Hypoxanthine-Guanine Hyperuricemia, intellectual disability, dystonia, spasticity, dysarthria, compulsive self-
disease Phosphoribosyl Transferase biting (Self-mutilation) cognitive dysfunction
(HPRT)
Maple syrup urine Branched-chain α-ketoacid Poor feeding & vomiting in 1st week of life; lethargy; Hypertonia; Opisthotonos. Seizures &
disease (MSUD) dehydrogenase hypoglycemia; Maple syrup odor of body fluids; Elevated plasma leucine, isoleucine & valine
McArdle disease Myophosphorylase Exercise intolerance, muscle cramps, rhabdomyolysis, myoglobinuria
Metachromatic Arylsulfatase A Mixture of cognitive, UMN & LMN signs; Irritability, genu recurvatum, absent deep tendon
Leukodystrophy reflexes, muscle wasting, hypotonia; nystagmus, myoclonic seizures, optic atrophy &
quadriparesis
Niemann Pick Sphingomyelinase Hepatosplenomegaly, lymphadenopathy, neurodevelopmental regression, spasticity &
disease rigidity
Contd...
xlii Review of Pediatrics & Neonatology
Contd...

Disease Enzyme deficient Important features


Oculocutaneous Tyrosinase Generalised hypopigmentation including that of iris & retina, reduced visual acuity,
Albinism type I nystagmus, squint
Phenylketonuria Phenylalanine hydroxylase or Normal at birth; Intellectual disability; seizures, hyperreflexia, Microcephaly, dental
tetrahydrobiopterin problems; Rx: Low Phenylalanine diet & Tyrosine supplementation
Pompe disease α-glucosidase (acid maltase) Cardiomegaly, hypotonia, hepatomegaly; onset: birth to 6 months
Tarui disease Phosphofructokinase Exercise intolerance, muscle cramps, hemolytic anemia, myoglobinuria
Tay-Sachs disease β-Hexosaminidase A Exaggerated startle response to noise (hyperacusis); Macrocephaly, Seizures; Cherry-red
spots seen; No splenomegaly
Tyrosinemia Type I: Fumaryl-acetoacetate Type I: Presents at 2-6 months with acute hepatic crisis;
hydrolase Boiled cabbage urine odor;
Type II: tyrosine Cirrhosis & hepatocellular carcinoma may occur
aminotransferase Episodic peripheral neuropathy resembling porphyria
Type III: 4-hydroxy-phenyl- Renal involvement: Fanconi-like syndrome, normal anion gap metabolic acidosis &
pyruvate dioxygenase vitamin D–resistant rickets
Von Gierke disease Glucose-6-phosphatase Growth retardation, hepatomegaly, hypoglycemia; elevated blood lactate, cholesterol,
triglyceride, and uric acid levels

Annexure 4
 FETAL DEVELOPMENT 
Time line of important events in fetal development (starting from conception):
Week Developmental events
1st week Fertilization & implantation; beginning of embryonic period; Day 6: embryo begins implantation in the uterus
2nd week Endoderm & ectoderm appear (bilaminar embryo)
3rd week Mesoderm appears (trilaminar embryo); somites begin to form
4th week Neural folds fuse; folding of embryo; arm & leg buds appear; Foregut, midgut & hindgut forms;
Primitive heart tube forms by 22 days & heart begins to beat;
Pharyngeal arches start appearing; Crown-rump length (CRL): 4-5 mm
5th week The 3 main subdivisions of forebrain, midbrain, and hindbrain are evident;
Lens placodes, primitive mouth, hands begin to develop; Mesonephric kidney starts forming
6th week 4 chambered structure of heart develops; Lower limb buds & external ears start forming
Primitive nose, philtrum, primary palate develop; CRL 21–23 mm
7th week Hand shows formation of digits; Ossification of bones start in upper limb, followed by lower limbs;
Eyelids & toes form; CRL 30 mm
8th week Limbs & digits are fully formed & their movement starts; Ovaries & testes distinguishable; Baby begins to hear;
9th week Fetal period begins; crown-rump length 5 cm; weight 9 g
10th week External genitals distinguishable; Fingernails develop; Baby starts urinating at 10-11 weeks;
Intestinal loops that had herniated out of abdominal cavity return into it;
14th week Breathing & swallowing motions appear
16th week Bone marrow begins to form
20th week Lung alveoli begin to secrete surfactant; Fetal weight 460 g; length 19 cm;
24th week Fetal weight 900 g; length 25 cm; Sweat glands start functioning
28th week Eyes opening appears; fetus turns head down; weight 1,000 g
29th week Pupils begin to react to light
>35th week Mature surfactant usually present, in adequate amount
Annexures xliii

Annexure 5
 IMPORTANT SYNDROMES & THEIR SALIENT FEATURES
Syndrome Genetic basis Salient features
Alagille syndrome Chr 20p12 deletion Bile duct paucity with cholestasis; Pulmonary artery stenosis;
butterfly vertebrae; ocular abnormalities
Angelman syndrome Uniparental disomy / Hypotonia, feeding difficulties, prognathism, seizures, ataxia, inappropriate
(maternal copy deleted) imprinting of chr15q11 laughter, poor speech, intellectual disability
Beckwith Wiedemann Imprinting on chr 11p15 Exomphalos, macroglossia, hemihypertrophy, Increased tumor risk;
syndrome
Chédiak-Higashi syndrome Autosomal recessive; LYST gene Defective degranulation of neutrophils, a mild bleeding diathesis, partial
on chr 1q2 albinism, peripheral neuropathy, neutropenia & abnormal function of neutrophils
& NK cells, presence of giant granules
Cri-du-chat syndrome 5p deletion Low-set ears, hypertelorism, characteristic cat cry

Down syndrome Trisomy 21 “I C a PROBLEM Somewhere” Low (depressed) nasal bridge


Incurved 5th finger (clinodactyly) / Intellectual Epicanthic fold / Ears low-set &
disability dysplastic
Congenital heart disease / Congenital Mongoloid slant of eyes/ Mental
Hypothyroidism retardation
Acute Leukemia/ Alzheimer’s disease / Sandle gap/ Simian palmar
Atlantoaxial instability crease
Protruding tongue
Round face
Occiput flat/Open, wide fontanelle
Brushfield spots in iris / Brachycephaly
Edward syndrome Trisomy 18 “ROCKY Mountain” K: Kidney malformations
R: Rocker bottom foot Y: MicrocephalY
O: Overlapping fingers M: Mental retardation
C: Cardiac defects
Ellis van Creveld syndrome EVC, EVC2 gene Chondrodystrophy, polydactyly, ectodermal dysplasia, cong. heart disease
Fragile X syndrome Triple nucleotide repeats Mental retardation, large ears & jaws, macroorchidism in males
Hyper-IgE syndrome Stat3; Autosomal dominant Recurrent skin & sinopulmonary infections, candidiasis, eosinophilia, scoliosis
Kenny-Caffey syndrome TBCE gene (1q 43-44) Episodic symptomatic hypocalcemia, medullary stenosis of long bones, short
stature, delayed closure of fontanel, delayed bone age, eye abnormalities,
hypoparathyroidism
Klinefelter syndrome 47, XXY Phenotypically male; taller stature; hypogonadism, infertility, gynecomastia
Laurence Moon Bardet BBS1-12, MKS1, MKS3, CEP290 Obesity, polydactyly, Intellectual disability, Retinitis Pigmentosa, renal
Biedel syndrome anomalies, anosmia, Cong. Heart disease
Marfan syndrome FBN1 gene (Chr 15q21.1) → “M-A-R-F-A-N-S” Fibrillin deficiency
defective fibrillin-1 Mitral valve prolapse Arachnodactyly (long, tapering fingers)
Aortic Aneurysm Negative Nitroprusside test
Retinal detachment Supero-temporal lens subluxation, Skeletal changes
McCune-Albright syndrome Missense mutation in gene for Precocious Puberty with Polyostotic Fibrous Dysplasia & Abnormal
α-subunit of GS Pigmentation (café au lait spots)
Noonan syndrome PTPN11 gene Seen in both sexes, Short stature, low posterior hairline, shield chest, webbed
neck, pulmonary stenosis, intellectual disability, delayed puberty
Patau syndrome Trisomy 13 “CMC DOCTOR OPD”
C: Cutis aplasia D: Deafness
M: Microphthalmia, Microcephaly O: Ocular hypotelorism
C: Congenital Heart diseases C: Cleft lip & cleft palate
O: Others (Renal abnormalities) T: Trisomy 13
P: Polydactyly O: hOLOprosencephaly
D: Developmental delay R: Rib abnormalities
Contd...
xliv Review of Pediatrics & Neonatology
Contd...

Syndrome Genetic basis Salient features


Pendred syndrome gene for Pendrin (chr 7) Sensorineural deafness and goiter
Pierre Robin syndrome DNA near SOX9 gene Micrognathia, cleft palate, glossoptosis, airway obstruction
Prader-Willi syndrome Uniparental disomy/ genomic Severe hypotonia and feeding difficulties at birth, voracious appetite & obesity,
(paternal copy deleted) imprinting of chr 15q11 short stature, small hands & feet, hypogonadism, intellectual disability
Rubinstein Taybi syndrome CBP mutations Mental retardation; Broad thumbs & toes; Prominent nose;
Congenital heart disease (ASD)
Russell Silver syndrome Chr 7q32 deletion Growth retardation, triangular face; short stature
Soto’s syndrome Chr 5q35 deletion Overgrowth, macrocephaly, prominent forehead, large hands & feet, hypotonia,
clumsiness, mental disabilities

Treacher Collins syndrome Autosomal dominant, TCOF1 Anti-mongoloid slant, coloboma of lower eyelids , Micrognathia, underdeveloped
gene mutation cheek, ear malformations & deafness
Turner syndrome Karyotype 45,XO “See there is A Baby CLOWN”
S: Short stature, Sensorineural hearing loss, Short 4th metacarpal
A: Primary Amenorrhea
B: Absent Barr body, Bicuspid aortic valve (MC heart disease in Turner)
C: Cardiac anomalies, cystic Hygroma
L: Lymphedema, low thyroid
O: Ovaries under developed (streak ovaries)
W: Webbed neck
N: Nipples widely placed
Velo cardio facial or Chr 22q11 deletion CATCH 22 (Conotruncal cardiac abnormalities, abnormal facies, thymic hypoplasia,
DiGeorge syndrome cleft palate, hypocalcemia)
Waardenburg syndrome PAX3 gene mutation White forelock of hairs; heterochromia irides, sensorineural deafness
WAGR syndrome Chr 11p13 deletion Wilms tumor, aniridia, male genital hypoplasia, gonadoblastoma, intellectual
disability, retarded growth
William syndrome Chr 7q11 deletion Round face, long philtrum, upturned nostrils (elfin facies), supravalvular aortic
stenosis, intellectual disability, hypertension, friendly personality, hypercalcemia
Wiskott Aldrich syndrome WASP gene (Chr Xp11) Thrombocytopenia, small & defective platelets; atopic dermatitis,
X-linked recessive immunodeficiency

Annexure 6
 EFFECT OF IMPORTANT MATERNAL DRUGS ON FETUS & NEWBORN
Drug Effect on fetus
Alcohol ‘Fetal alcohol syndrome’: Congenital cardiacQ, CNS, limb anomalies; IUGR; developmental delay; attention deficits; autism
Busulfan Stunted growth; corneal opacities; cleft palate; hypoplasia of ovaries, thyroid & parathyroids
Carbamazepine Spina bifidaQ, possible neurodevelopmental delay
Carbimazole Scalp defects, choanal atresia, esophageal atresia, developmental delay
Chloroquine Deafness
Cocaine Microcephaly, LBW, IUGR, behavioral disturbances
Danazol Virilization
Lithium Ebstein anomalyQ, macrosomia
Misoprostol Arthrogryposis, cranial neuropathies (Möbius syndrome), equinovarus
Phenytoin Congenital anomalies, IUGR, neuroblastoma, bleeding (vit K deficiency)
Quinine Abortion, thrombocytopenia, deafness
Statins IUGR, limb deficiencies, VACTERL
Tetracycline Retarded skeletal growth, pigmentation of teethQ, hypoplasia of enamel, cataract, limb malformations
Thalidomide PhocomeliaQ, deafness, other malformations
Valproate CNS, facial and cardiac anomalies, limb defects, neurologic abnormalities
Vitamin D Supravalvular aortic stenosis, hypercalcemia
Warfarin Fetal bleeding and death, hypoplastic nasal structuresQ
Annexures xlv

Annexure 7
 GENETIC SYNDROMES & ASSOCIATED CANCERS

Disorder Tumor/Cancer
Chromosomal Syndromes
Down syndrome AML M7, ALL, Transient leukemoid reaction
Klinefelter syndrome Breast cancer, Extragonadal germ cell tumors

Noonan syndrome Juvenile myelomonocytic leukemia


Monosomy 5 or 7 Myelodysplastic syndrome
Chromosomal Instability
Xeroderma pigmentosum Basal cell & squamous cell carcinomas; Melanoma
Fanconi anemia Leukemia, MDS, Liver, CNS, GI cancers
Bloom syndrome Leukemia, Lymphoma, Solid tumors

Ataxia-telangiectasia Lymphoma, Leukemia, CNS tumors


Immunodeficiency Syndromes
Wiskott-Aldrich syndrome Lymphoma, leukemia
Bruton’s agammaglobulinemia Lymphoma, leukemia
Others
Neurofibromatosis type 1 Neurofibroma, Optic glioma, Acoustic neuroma, Astrocytoma, Meningioma, Pheochromocytoma
Neurofibromatosis type 2 Bilateral acoustic neuromas, Meningiomas
Tuberous sclerosis Myocardial rhabdomyoma, Fibroangiomatous nevi

Li-Fraumeni syndrome Bone tumors, Soft tissue sarcoma, Breast Cancer


Retinoblastoma Sarcoma
Beckwith syndrome Wilms tumor, Hepatoblastoma, Adrenal carcinoma
von Hippel-Lindau disease Hemangioblastoma of cerebellum & retina, Pheochromocytoma, Renal cancer
MEN 1 (Wermer syndrome) Parathyroid, pancreatic islet, and pituitary tumors
MEN 2A (Sipple syndrome) Medullary carcinoma of thyroid; Pheochromocytoma
MEN 2B Mucosal neuroma, Pheochromocytoma, Medullary thyroid carcinoma, Marfan habitus
Familial adenomatous polyposis  Colorectal & thyroid carcinoma, Duodenal & periampullar carcinomas; Hepatoblastoma
Turcot syndrome Pediatric brain tumors; Colon carcinoma and polyps
Gardner syndrome Adenocarcinoma of colon; Skull and soft tissue tumors
Peutz-Jeghers syndrome Gastrointestinal carcinoma; Ovarian neoplasia
xlvi Review of Pediatrics & Neonatology

Annexure 8
 MICRONUTRIENTS & THEIR DEFICIENCY

Vitamin Function Disease caused by deficiency


Vitamin A Required for synthesis of visual pigments rhodopsin • Nyctalopia (Night blindness), Photophobia, xerophthalmia,
& iodopsin, gene expression, reproduction, embryonic Bitot spots, conjunctivitis, keratomalacia, blindness
development & immune function • Defective tooth enamel
• Impaired resistance to infection, anemia

Biotin Cofactor for carboxylases, important in gluconeogenesis, Periorificial dermatitis, conjunctivitis, alopecia, lethargy,
fatty acid and amino acid metabolism hypotonia, seizures

Folic acid Coenzyme in the metabolism of nucleic & amino acid; Megaloblastic anemia; Ulcers, glossitis;
Protects against atherosclerosis & neural tube defects Periungual hyper pigmentation

Niacin Coenzyme in many redox reactions; component of NAD Hartnup disease, Pellagra (Dermatitis, dementia, diarrhea)

Thiamine Coenzyme in metabolism of carbohydrates & branched- Dry beriberi: Irritability, peripheral neuritis, muscle tenderness,
(Vitamin B1) chain amino acids, nucleic acid synthesis ataxia; Wet beriberi: Tachycardia, edema, cardiomegaly, cardiac
failure

Riboflavin Coenzyme in many redox reactions Glossitis, cheilosis, nasolabial dysbacea, keratitis, blurring of
(vitamin B2) vision, peripheral neuropathy

Pyridoxine Role in transamination, decarboxylation & desulphuration; Neonatal seizures, microcytic anemia, homocystinuria, oxalosis,
(vitamin B6) (Prolonged INH, penicillamine & hydralazine causes it) cystathioninuria

Cobalamin Coenzyme in nucleic acid metabolism; deficiency seen Megaloblastic anemia;


(vitamin B12) in methyl malonic aciduria, pernicious anemia, pure Subacute combined degeneration of spinal cord
vegetarians & fish tape worm infestation

Ascorbic acid Strong reducing agent & antioxidant Scurvy: pseudo paralysis (due to tenderness of muscles), rosary
(vitamin C) Cofactor for reactions requiring reduced copper or iron of ribs (which are sharper compared with rickety rosary), gum
Helps in collagen, teeth, bone formation bleeding, irritability, tachypnea

Vitamin E Antioxidant, antineoplastic effect, reduces incidence of Areflexia, ataxia of trunk & limbs, myopathy; Retinal changes;
Retinopathy of Prematurity (ROP) & Bronchopulmonary Hemolytic anemia, hyperbilirubinemia & IVH in preterm babies
dysplasia (BPD)

Vitamin K Coenzyme for synthesis of clotting factors 2, 7, 9, 10; stimulate VKDB (Vitamin K deficiency bleeding) or HDN (Hemorrhagic
osteocalcin, that helps in calcium metabolism disease of newborn)

Manganese Enzyme cofactor Hypercholesterolemia, weight loss, decreased clotting proteins

Selenium Enzyme cofactor Keshan disease (cardiomyopathy) Kashinbeck disease (endemic


osteoarthritis)

Chromium Potentiates the action of insulin Impaired glucose tolerance, peripheral neuropathy, encephalopathy
Zinc Enzyme cofactor; Constituent of zinc-finger proteins, which Growth retardation, hypogonadism, anorexia, alopecia,
regulate gene transcription acrodermatitis enteropathica
Iron Component of Hb, myoglobin & cytochromes Anemia, decreased alertness, impaired learning
Copper Enzyme cofactor (superoxide dismutase, cytochrome oxidase) Microcytic Anemia, neutropenia, hypopigmentation of hair & skin,
vascular abnormalities
Iodine Component of thyroid hormone Hypothyroidism
Annexures xlvii

Annexure 9
 IMPORTANT “MOST COMMONS” IN PEDIATRICS

Neonatology
• Most common bone to fracture in a newborn is Fracture clavicle
• Most common congenital abnormality in Infant of Diabetic mother (IDM) is Congenital heart disease
• Most common heart disease in infant of Diabetic mother is ventricular septal defect
• Most common neurologic abnormality in Infant of diabetic mother is Neural tube defect
• Most common life-threatening emergency of GIT in newborn period is NEC (Necrotizing Enterocolitis)
• Most common cause of neonatal mortality in India are prematurity & low birth weight
• Most important risk factor for NEC is Prematurity
• Most common location for a cephalhematoma : Parietal
• Most common cause of Neonatal Sepsis in India: Acinetobacter (Latest update)
• Most common cause of Neonatal Sepsis in hospitals in India: Acinetobacter (Latest update)
• Most common cause of Neonatal Sepsis in hospitals across the world: E. coli
• Most common cause of Neonatal Sepsis overall, throughout the world: Gr B streptococci
• Most common cause of Early onset Neonatal Sepsis: Gr B streptococci.

Inborn errors of Metabolism & Genetic Disorders


• Most common lysosomal storage disorder is Gaucher disease
• Most common type of Gaucher disease is type 1
• Most common clinical manifestation of Gaucher disease is splenomegaly
• Most common Glycogen storage disease in children is Von Gierke disease (type I GSD)
• Most common Glycogen storage disease in adolescents is Mc Ardle disease (type V GSD)
• Most common enzyme deficient in Galactosemia is GALT (Galactose 1 phosphate uridyl transferase)
• Most common cause of sepsis in a child with Galactosemia is E. coli
• Most common chromosomal abnormality in children: Down syndrome
• Most common cause of increased nuchal thickness : Down syndrome

Infections & Immune Mediated Disorders


• Most common vasculitis of childhood is Henoch-Schönlein purpura (HSP)
• Most common complications of mumps are meningitis, encephalitis & gonadal involvement.
• Most common cause of congenital infection in neonates is CMV
• Most common finding in Congenital Rubella Syndrome is Nerve deafness
• Most common complication of Measles is Otitis media
• Most common cause of death in Measles is Pneumonia.

Pediatric Cardiology
• Most common congenital heart disease is Ventricular septal defect
• Most common congenital heart disease, affected by Infective endocarditis is Ventricular septal defect
• Most common heart disease in Down syndrome is AV septal defect or Endocardial cushion defect
• Most common type of Atrial Septal Defect is Ostium secondum
• Most common heart disease in Turner syndrome is Bicuspid Aortic valve
• Most common site of coarctation of aorta is juxtaductal
• Most common congenital heart disease causing mortality in 1st week of life is Hypoplastic left heart syndrome
• Most common cause of heart disease in children in developing countries is Rheumatic Heart disease (RHD)
• Most common age group involved in Rheumatic fever is 5–15 years.
• Most common symptom of infective endocarditis is fever
• Most common cause of myocarditis in Infant = Cox sackie virus
• Most common aortic branch involved in Takayasu arteritis: Left subclavian artery
• Most common associated anomaly with coarctation of aorta : Bicuspid valve
• Most common cause of coronary artery aneurysm worldwide : Kawasaki diseases
• Most common cause of cyanosis in a child : Tetralogy of Fallot
xlviii Review of Pediatrics & Neonatology

• Most common cause of cyanosis in newborn: Transposition of great vessels


• Most common cyanotic congenital heart malformation beyond neonatal period : TOF
• Most common site for cardiac myxoma : Left atrium.

Pediatric Hematology & Oncology


•• Most common cause of Beta thalassemia is Splicing mutation
•• Most common bone involved in LCH is skull
•• Most common mode of inheritance of Von Willebrand disease is Autosomal dominant
•• Most common primary intraocular tumor in children is Retinoblastoma
•• Most common presentation of Retinoblastoma is leukocoriaQ (white pupil)
•• Most common pediatric soft tissue sarcoma is Rhabdomyosarcoma
• Most common presenting feature of RMS is a mass that may or may not be painful
•• Most common primary malignant orbital tumor in childhood : Rhabdomyosarcoma
•• Most common benign hepatic tumor during first 6 months of life: Hemangioendothelioma
•• Most common benign orbital tumor in childhood : Dermoid Cyst of Orbit
•• Most common bladder neoplasm in children younger than 10 years: Rhabdomyosarcoma
•• Most common cause of orbital calcifications : Retinoblastoma
•• Most common cause of osteolytic bone metastases in a child : Neuroblastoma
•• Most common cerebellar neoplasm in children : Medulloblastoma
•• Most common congenital solid tumor in the newborn : Sacrococcygeal Teratoma
•• Most common testicular tumor in a young boy : Yolk sac tumor
•• Most common intraocular neoplasm in childhood : Retinoblastoma
•• Most common location for a pilocytic astrocytoma : Cerebellum
•• Most common solid extracranial tumor of childhood : Neuroblastoma.

Pediatric Gastroenterology
•• Most common esophageal disorder in children is Gastroesophageal reflux disease (GERD)
•• Most common bacteria causing diarrhea in children in developing countries is E.coli
•• Most common viral agent causing diarrhea in children is Rotavirus
•• Most common extraintestinal manifestation of celiac disease is refractory iron deficiency anemia
•• Most common cause of portal hypertension in children in India is Extrahepatic portal vein obstruction.

Pediatric Respiratory Disorders


•• Most common congenital laryngeal anomaly is Laryngomalacia
•• Most common cause of stridor in neonate and young infant is Laryngomalacia
•• Most common benign tumor of the lung : Hamartoma
•• Most common cause of Neonatal Pneumonia : Group B streptococcus
•• Most common cause of pulmonary hypoplasia : Diaphragmatic hernia
•• Most common cause of Common cold is Rhinovirus
•• Most common cause of Pharyngitis are Adeno Virus & Streptococcus pyogenes
•• Most common cause of Croup is Parainfluenza virus
•• Most common cause of Acute Epiglottitis is H. influenzae
•• Most common cause of Whooping cough is Bordetella pertussis
•• Most common cause of Bronchiolitis is Respiratory syncytial virus
•• Most common cause of Bacterial pneumonia in children is S. pneumonia > Hemophilus influenzae type b
•• Most common cause of Viral pneumonia in children is Respiratory Syncytial Virus (RSV)
•• Most common cause of Pneumonia in infants with HIV is Pneumocystis jiroveci.

Pediatric Nephrology
•• Most common presenting symptom in children with nephrotic syndrome is edema
•• Most common cause of idiopathic nephrotic syndrome in children is minimal change disease
•• Most common cause of congenital Nephrotic syndrome is NPHS1 mutation (nephrin)
•• Most common gene defect in idiopathic steroid resistant nephrotic syndrome is NPHS2
•• Most common cause of abdominal mass in a newborn is Multicystic dysplastic kidney
Annexures xlix

•• Most common cause of hydronephrosis in a newborn : Pelvi-ureteric junction obstruction


•• Most common cause of urinary obstruction in boys : Posterior urethral valves
•• Most common cause of pyelonephritis in children is Vesicoureteric Reflux (VUR)
•• Most common cause of Renal artery stenosis in India is Takayasu Aortoarteritis
•• Most common type of glomerulonephritis worldwide is IgA nephropathy
•• Most common cause of gross hematuria in children is IgA nephropathy
•• Most common renal cystic disease in childhood is Multicystic dysplastic kidney.

Pediatric Endocrinology
•• Most common genetic cause for multiple pituitary hormone deficiency is Mutation of PROP1
•• Most common cause of hypothyroidism worldwide is Iodine deficiency
•• Most common cause of congenital hypothyroidism is Thyroid dysgenesis
•• Most common cause of congenital hypothyroidism in a child with goitre is Thyroid dyshormonogenesis
•• Most common cause of Acquired hypothyroidism is Hashimoto’s thyroiditis
•• Most common cause of Congenital Adrenal Hyperplasia (CAH) is 21 hydroxylase deficiency
•• Most common type of diabetes in children is type 1 Diabetes Mellitus
•• Most commonly diagnosed Pituitary adenoma during childhood is prolactinoma > corticotropinoma
•• Most common cause of Cushing syndrome is prolonged exogenous administration of glucocorticoids
•• Most common clinical symptom of Cushing syndrome in all age groups is obesity
•• Most common antigen against which antibodies are found in Addisons disease is CYP 21 (21 hydroxylase)
•• Most common cause of adrenal insufficiency is sudden withdrawal of steroids in a patient on long term steroid therapy
•• Most common infectious etiology for adrenal insufficiency in children is meningococcemia
•• Most common cause of short stature in childhood is constitutional delay in growth & puberty (CDGP).

Pediatric Neurology
•• Most common cause of seizure in children < 5 years is Febrile seizures
•• Most common cause of neonatal seizures is Hypoxic–Ischemic Encephalopathy
•• Most common location of meningomyelocele is lumbosacral region
•• Most common congenital anomaly of CNS are Neural tube defects
•• Most common type of cerebral palsy is spastic diplegia
•• Most common cause of megalencephaly is benign familial megalencephaly
•• Most common cause of an elevated CSF opening pressure is an agitated patient
•• Most common cause of subdural fluid collection in children is meningitis
•• Most common cause of a large choroid plexus cyst : Trisomy 18
•• Most common neurobehavioral disorder of childhood is Attention Deficit hyperactivity disorder (ADHD)
•• Most common cause of viral meningoencephalitis in children is Enterovirus
•• Most common sequelae of bacterial meningitis is Sensorineural hearing loss
•• Most common cause of meningitis in Neonates in India is E. coli
•• Most common cause of meningitis in Older children in India is S. pneumoniae > N. meningitidis
•• Most common cause of meningitis in Neonates, all over the world is Gr B Streptococcus
•• Most common cause of meningitis in older children, all over the world is S. pneumonia > N. meningitidis.

Pediatric Surgical Disorders


•• Most common congenital anomaly of esophagus is Esophageal Atresia
•• Most common type of TEF is type C (upper esophagus ends in a blind pouch & TEF to distal esophagus)
•• Most common surgical disorder of the stomach during infancy is Hypertrophic pyloric stenosis
•• Most common surgical cause of non-bilious vomiting in infants is Hypertrophic pyloric stenosis
•• Most common congenital anomaly of GI tract is Meckel’s diverticulum
•• Most common extraintestinal manifestation of celiac disease is refractory iron deficiency anemia
•• Most common abdominal emergency of early childhood is Intussusception
•• Most common cause of neonatal intestinal obstruction is—Meconium plug syndrome
•• Most common cause of intestinal obstruction between 3 months to 6 years of age: Intussusception
•• Most common type of intussusception is Ileocolic
•• Acute appendicitis is the most common acute surgical condition in children
•• Most common type of Congenital Diaphragmatic Hernia is Bochdalek (posterolateral) variety.
l Review of Pediatrics & Neonatology

Annexure 10
 IMPORTANT DIAGNOSTIC CLINICAL FEATURES / SIGNS IN PEDIATRIC DISEASES
Clinical sign Disease
Abnormal kinky hair Menke’s disease
Absent tonsils Bruton’s X-Linked Agammaglobulinemia, severe combined immunodeficiency (SCID)
Antimongoloid slant of eyes Noonan syndrome, Treacher Collin syndrome
Arachnodactyly & Tall stature Marfan syndrome, Homocystinuria
Ash leaf macules & Shagreen patches Tuberous sclerosis
Bitot’s spots Vitamin A deficiency
Blue sclera & recurrent fractures Osteogenesis imperfecta
Bunch of grapes appearance Rhabdomyosarcoma (Botryoid type)
Calf muscle pseudohypertrophy Duchenne muscular dystrophy, Becker muscular dystrophy
Casal’s necklace Pellagra, (Niacin deficiency), Hartnup disorder
Cherry red epiglottis Acute epiglottitis
Cherry Red spot GM1 gangliosidosis, Tay Sachs disease, Niemann Pick disease
Cicatrising skin lesions & limb defects Congenital Varicella syndrome
Clinodactyly, Sandle gap, Simian crease Down syndrome (Trisomy 21)
Coarse facies Mucopolysaccharidoses, GM1 gangliosidosis
Darkening of urine on standing Alkaptonuria
Delayed fall of umbilical cord Leukocyte adhesion defects
Doll like facies Von Gierke disease
Erythema marginatum Rheumatic fever (Major criteria)
Flag sign & Flaky paint dermatosis Severe malnutrition (Esp. Kwashiorkor)
Gower sign Duchenne & Becker muscular dystrophy
Gum hypertrophy Acute Myeloid Leukemia (M4 & M5)
Heliotrope rash & Gottron’s papules Juvenile Dermatomyositis
Hemarthrosis Hemophilia
Hemihypertophy Beckwith Wiedemann syndrome
Hutchinson’s teeth Congenital Syphilis
Hypopigmented hairs Chediak-Higashi syndrome, Griscelli syndrome
Inverted nipples Congenital disorders of glycosylation
Kayser Fleicher ring +/- Sunflower cataract Wilson disease
Koplik’s spots Measles
Lisch nodules Neurofibromatosis
Macro orchidism Fragile X syndrome
Myotonia Myotonic dystrophy
Osler nodes, Janeway lesions Infective endocarditis
Periorificial rash Acrodermatitis enteropathica; zinc deficiency
Pleomorphic rash (papules, vesicles, pustules) Chickenpox (Varicella)
Port wine stain of half of face Sturge Weber syndrome
Pseudoparalysis of limbs Scurvy
Raccoon eyes Neuroblastoma
Retinitis pigmentosa Mitochondrial disorders

Contd...
Annexures li
Contd...
Retrognathia, micrognathia, glossoptosis Pierre Robin sequence
Rocker bottom foot Edward syndrome
Salt & pepper Retinopathy Congenital Rubella syndrome
Slapped cheek appearance Erythema Infectiosum (Parvovirus infection)
Strawberry tongue Scarlet fever, Kawasaki disease
Subluxation of lens Marfan syndrome, Homocystinuria
Tongue fasciculations Spinal muscular atrophy
Tonsillar pseudomembrane & bull neck Diphtheria
Triangular face Russell Silver syndrome
Trident hand Achondroplasia
Washerman’s hands & Potter’s facies Renal agenesis (Potter sequence)
Webbed neck, widely spaced nipples Turner syndrome
Wrist widening, Rachitic rosary Rickets

Annexure 11
 IMPORTANT RADIOLOGICAL SIGNS IN PEDIATRIC DISORDERS
Radiological sign Disease
Anterior beaking of vertebra & bullet metacarpals Mucopolysaccharidosis (Dysostosis multiplex seen)
‘Boot’ shaped’ heart or Cor en sabot Tetralogy of Fallot (Pulmonary oligemia also seen)
‘Box shaped’ heart Ebstein anomaly
‘Champagne glass’ pelvis Achondroplasia
Chondrodysplasia punctata Warfarin embryopathy (Stippled calcification of vertebra)
‘Claw’ sign or ‘coiled spring’ sign Intussusception
Coronary artery aneurysm Kawasaki disease (Seen on coronary angiography)
Cupping, fraying & splaying Rickets
‘Egg on side’ appearance on CXR Transposition of Great Arteries (TGA)
‘Erlenmeyer flask’ deformity Gaucher disease
‘Football’ sign or pneumoperitoneum Intestinal perforation / Stage IIIB Necrotising enterocolitis
Ground glass appearance of lungs Hyaline membrane disease
Obstructive TAPVC (Total anomalous pulmonary venous connection)
‘Hair on end’ appearance Chronic hemolytic anemia (Thalassemia)
Molar tooth sign Joubert syndrome (cerebellar vermis hypoplasia)
Notching of inferior margin of ribs Coarctation of aorta (‘figure of 3’ & ‘E-sign’ also seen)
Pneumatosis intestinalis &/or portal vein gas Necrotising enterocolitis (NEC)
‘Puff of smoke’ appearance Moyamoya disease (seen on cerebral angiography)
Punched out skull lesions Langerhan cell histiocytosis
‘Signet ring’ appearance on CT Chest Bronchiectasis
‘Snowman’ or ‘figure of 8’ appearance Supracardiac TAPVC
Steeple sign Croup or Laryngotracheobronchitis
Thumb sign Acute epiglottitis
Tiger eye appearance on MRI Brain Pantothenate Kinase-Associated Neurodegeneration (PKAN)
Wimberger sign & Trummerfeld zone Scurvy (Pelkan’s spurs, subperiosteal hemorrhages also seen)
lii Review of Pediatrics & Neonatology

Annexure 12
 TREATMENT OF CHOICE IN VARIOUS DISORDERS OF CHILDREN
Name of the disease Treatment of choice
Absence seizures  Ethosuximide (Next choice: Sodium valproate)
Acute diarrhea  WHO ORS & Zinc
Acute Epiglottitis  IV Ceftriaxone (3rd gen Cephalosporin)
Acute Intermittent Porphyria  Hemin
Anaphylaxis Epinephrine
Apnea of prematurity Caffeine citrate
Cholera Azithromycin/Doxycycline (for older child)
Congenital adrenal hyperplasia (21 hydroxylase def) Hydrocortisone +/- Fludrocortisone
Croup (Laryngotracheobronchitis) Dexamethasone +/- Nebulized Epinephrine
Diphtheria Diphtheria antitoxin & Penicillin
Foreign body esophagus Endoscopic visualization & removal
Galactosemia Lactose free diet (Breast milk should also be avoided)
Gaucher disease Enzyme replacement therapy
Hemorrhagic disease of newborn (HDN) Vitamin K (also used for prophylaxis of HDN)
Hereditary fructose intolerance Fructose free diet
Hypothyroidism Oral Thyroxine
Infantile spasms in a child with Tuberous sclerosis Vigabatrin
Methanol or ethylene glycol poisoning Fomepizole
Myoclonic seizures Valproate
Neonatal Jaundice Phototherapy +/- Exchange transfusion
Neonatal seizures Phenobarbitone
Neonatal sepsis IV broad spectrum antibiotics
Nephrotic syndrome (first episode) Prednisolone
Nocturnal enuresis Motivational & Bed alarm system
Nutritional Rickets Vit D3 +/- Calcium
Pertussis Azithromycin
Phenylketonuria Low phenylalanine diet
PSVT (Paroxysmal Supraventricular Tachycardia) IV Adenosine
Respiratory distress syndrome (HMD) CPAP+/- surfactant
Rh Isoimmunisation IV Immunoglobulins (IVIg)
Scorpion sting Prazosin
Scurvy Vitamin C
Severe Combined Immunodeficiency Hematopoietic Stem cell transplantation
SIADH (Syndrome of Inappropriate ADH secretion) Restriction of free water intake
Toxoplasmosis Sulfadiazine
Tricyclic antidepressant overdose/toxicity Sodium Bicarbonate
Type I Diabetes mellitus Insulin
Tyrosinemia Nitisinone (NTBC), Liver transplantation
Von Gierke disease Corn starch diet
West syndrome Adrenocorticotrophic hormone (ACTH)
Wilson disease Zinc, Trientine
Chapter 1
Normal and
Abnormal Growth
 NORMAL GROWTH M

•• Growth refers to increase in physical size of an organ or body


Question 1
•• Anthropometric parameters are used to assess growth like: Weight, Height, Head What is this instrument used for?
circumference, body mass index, skin fold thickness
•• To know whether growth of a child is appropriate for his age or not, we need to compare it
with a standard (expected value).

Devices used for assessment of growth M

Name of parameter Device used


Length InfantometerQ
a. Measure weight
Height StadiometerQ b. Measure height
Head circumference Non-stretchable measuring tape c. Measure length
d. Measure upper segment: lower seg­
Skin fold thickness Harpenden CallipersQ ment ratio

GROWTH CHARTS M

•• Growth standards are norms of growth presented in tabular or graphical manner


•• Initial growth charts in 1977, were developed by NCHS (National Centre for Health Statistics)
•• Followed by CDC growth charts in 2000Q. (Based on formula-fed US children) Mnemonic
•• WHO growth charts are preferred for under-5 children all over the world. It is based
•• WHO growth charts are based on
on breast-fed babies & it first came into use in 2006Q. multicentre growth reference study
(MGRS)Q conducted in six different
WHO growth chart for ‘Height for age’ for boys countries: Brazil, Oman, Norway,
Ghana, US, India and (Mnemonic:
‘BONGUI’).
2 Section 1: General Pediatrics

 NORMAL ANTHROPOMETRIC PARAMETERS


Review of Pediatrics and Neonatology

I. WEIGHT
Increase in weight with age M

Age Weight (kg) Description


Birth w Birth weight
5 months 2w Twice of birth weightQ
High Yield Points
1 year 3w Thrice of birth weightQ
•• Birth weight of an average Indian 2 years 4w Four times the birth weight
baby is 2.8 kgQ 3 years 5w Five times the birth weight
•• A term newborn loses up to 10% 5 years 6w Six times the birth weight
of its birth weight in 3–5 days after 7 years 7w Seven times the birth weight
birth, regained by day 10 of life 10 years 10 w Ten times the birth weight
•• Birth weight doubles by 5 months of
If birth weight is not known, following formula can be used to calculate expected weight of a child
age
at different ages:
•• Birth weight triples by 1 year &
quadruples by 2 years For age group Expected weight (kg)
Infants (age < 1 year) x+9
, where, x is age in months
2
1–6 years 2 x + 8 , where, x is age in years
High Yield Points
7–12 years 7x – 5
, where, x is age in years
•• Maximum increase in height occurs 2
in 1st yearQ followed by pubertyQ
•• Physiological hypertrophy of lym­ II. HEIGHT
phoid tissue is seen between 4 and •• If a child < 2 years old, measure recumbent length.
8 yearsQ •• If ≥ 2 years age & able to stand, measure standing height.
•• Standing height is about 0.7 cm less than recumbent length.
Increase in length or height with age M
High Yield Points
Age Birth 1 year 2 years 4 years 12 years
•• Length increases by 50% in the first Height 50 cm 75 cm 90 cm 100 cm 150 cm
year of lifeQ
Formula for calculating expected height for age group 2–12 years:
•• Height doubles at around 4 yearsQ
Expected Height = 6x + 77 cm, where x is the age in years.
•• Height becomes triple of the height
at birth by 12 years III. UPPER SEGMENT:LOWER SEGMENT RATIO (US:LS) M
•• Upper segment is measured from vertex to symphysis pubis
•• Lower segment is measured from pubic symphysis to heel (sole).
Question 2 Normal upper segment to lower segment ratio in children:
Age Newborn 3 years 7–10 years
Identify the syndrome this child
US : LS 1.7–1.9 : 1Q 1.3 : 1 1 : 1Q
with short stature has?
 SHORT STATURE M

Definition
Height < 3rd percentileQ or - 2 SDQ below the median height for that age & sex.
Types
A B
•• Proportionate: US:LS normal
a. Seckel syndrome •• Disproportionate: US:LS altered
b. Turner syndrome –– Short trunk dwarfism: US:LS ratio decreased
c. Cornelia de lange syndrome
–– Short limb dwarfism: US:LS ratio increased.
d. Rubinstein-Taybi syndrome
Important Causes of Proportionate Short Stature M

Normal variants Intrauterine causes Acquired causes


High Yield Points •• Familial •• Small for gestational age (SGA) •• Under nutrition
•• Constitutional delay •• Congenital infections: TORCH •• Chronic systemic illness, e.g. chronic
•• Most common cause of short stat­ in growth & puberty •• Genetic: kidney disease
ure in childhood is constitutional (CDGP)Q –– Turner syndrome •• Endocrine causes:
delay in growth & puberty (CDGP) –– Down syndrome –– GH deficiency
–– Seckel syndrome
•• In CDGP, the final adult height –– Cushing syndrome
attained is normal Mnemonic –– Precocious or delayed puberty
•• 20% of IUGR babies remain short •• Psychosocial dwarfism
throughout life “T-D-S”
Chapter 1: Normal and Abnormal Growth 3

Section 1: General Pediatrics


Question 3
The growth chart of Rahul is shown from his early childhood till he becomes an adult. He had short stature during childhood. What
could be the underlying cause for the same?

a. Familial short stature b.  Growth hormone deficiency


c. Constitutional delay in growth & puberty d.  Achondroplasia
4 Section 1: General Pediatrics

Important Causes of Disproportionate Short Stature


Review of Pediatrics and Neonatology

M
Question 4 M

Identify the abnormality this child Short trunk dwarfism Short limb dwarfism
is suffering from:
•• Spondyloepiphyseal dysplasia •• RicketsQ
•• MucopolysaccharidosisQ •• Osteogenesis imperfectaQ
•• Mucolipidosis •• HypothyroidismQ
•• Caries spineQ •• AchondroplasiaQ
•• Hemivertebrae (congenital vertebral defect) •• Hypochondroplasia
•• Chondrodysplasia punctata
Mnemonic •• Chondroectodermal dysplasia
A B C “Short Man May Climb High” •• Metaphyseal chondrodysplasia

a. Osteogenesis imperfecta
b. Achondroplasia Comparison of Important Causes of Proportionate Short Stature M
c. Mucopolysaccharidosis
d. Rickets Birth weight & Skeletal Timing of
Cause length maturation puberty Other features Treatment
Familial short Normal Normal Normal Normal growth Not required/
stature velocity Assurance
High Yield Points
Constitutional Normal Delayed Delayed Normal growth Not required/
•• In familial short stature, the final growth delay velocity Assurance
height attained is less than the
Growth Normal weight Delayed Normal Hypoglycemia Subcutaneous
expected height according to age
hormone Slightly reduced Hypothyroidism recombinant
& sex, but it is normal as per the
deficiency length Micropenis GH
child’s target height
•• Mid parental height for: Truncal adiposity
Father’s height + Mother’s
height + 13
•• Boys = cm  TALL STATURE
     2
Father’s height + Mother’s Important causes of tall stature in childhood are:
height – 13
Girls = cm
     2 Fetal overgrowth Nonendocrine causes Endocrine causes
•• Maternal diabetes mellitus •• Familial/constitutional •• Excess GH secretion
•• Cerebral gigantism (Sotos syndrome) •• Exogenous obesity •• McCune-Albright syndrome
•• Weaver syndrome •• Marfan syndrome •• Precocious puberty
•• Beckwith-Wiedemann syndrome •• Fragile X syndrome •• Hyperthyroidism
High Yield Points •• Klinefelter syndrome (XXY)

•• Bone age is equal to chrono­logical


age in familial short stature  HEAD CIRCUMFERENCE (HC) M
•• In constitutional growth delay,
under nutrition hypothyroidism & High Yield Points
GH deficiency, bone age is less than
chronological age •• Total brain volume doubles in 1st year of life
•• Total brain volume at age 1 month is 36% of adult volume, at 1 year is 72%, while it is 83% of adult brain
size by 2 years
•• Rapid increase in head circumference suggests a tumor or hydrocephalus

•• It is measured through the occipital protuberance & supraorbital ridge


•• A non-stretchable measuring tape taken
Mnemonic •• Measurement is repeated thrice & maximum value is taken
Causes of large anterior fontanel in •• Normal Head circumference in a newborn is 33-35 cmQ.
children: “AAP CCHOR”
Rate of increase in head circumference
•• Achondroplasia
•• Apert syndrome
•• Prematurity Time after birth Age Increase in HC (cm)
•• Cleidocranial dysostosis 1st 3 months 1–3 months 2 cm/month
•• Congenital rubella syndrome Next 3 months 4–6 months 1 cm/month
•• Hypophosphatasia
•• Osteogenesis imperfecta Next 6 months 7–12 months 0.5 cm/month
•• Rickets Next 2 years 1–3 years 0.2 cm/month
Chapter 1: Normal and Abnormal Growth 5

Section 1: General Pediatrics


Question 5
Identify this syndrome in which the following are seen:

A B C
a. Down syndrome b. Patau syndrome
c. Edward syndrome d.
Turner syndrome

High Yield Points


•• MicrocephalyQ: Head circumference < -3 SD below the mean for age and sex
•• MacrocephalyQ: Head circumference > 95th percentile for age and sex

Important Causes of Microcephaly


Primary causes Secondary causes M

•• Cornelia de Lange syndrome Congenital infectionsQ: CMV, Rubella,


•• Smith-Lemli-Opitz syndrome Toxoplasmosis
•• Patau syndromeQ Maternal causes:
•• Edward syndromeQ •• Alcohol
•• Familial •• Smoking
•• Rubinstein-Taybi syndrome
•• Phenytoin
•• Cri du chat syndrome
•• Radiation
•• PhenylketonuriaQ
Mnemonic •• Diabetes
“Cannot See PEFR in Child” Infections in infancy: Meningitis, Encephalitis
Malnutrition
Perinatal asphyxia/HIE
Acquired microcephaly:
•• Rett syndromeQ
•• Angelman syndrome
•• Seckel syndrome

Important Causes of Large Head (Macrocephaly)


A. Thickened Cranium
•• Chronic anemia
•• Rickets
•• Osteogenesis imperfecta.

B. Chronic Subdural Collection High Yield Points


•• Can present as chronic subdural hematoma or subdural effusion NewbornQ First 4 weeks after
•• Most common cause of subdural fluid collection in children is meningitisQ. birth
Infancy First year
C. Megalencephaly
Toddler 1–3 years
•• Benign familial megalencephalyQ
Preschool child 3–6 years
•• Lysosomal storage disorders (Tay -Sachs, GM1 gangliosidosis, Mucopolysaccharidosis)
School age child 6–12 years
•• Aminoacidurias (Maple syrup urine disease, Glutaric aciduria type I)
•• Carbohydrate disorder: Galactosemia Adolescence : 10–19 years
•• Neurocutaneous: Neurofibromatosis, Tuberous sclerosis •• EarlyQ 10–13 years
•• Achondroplasia •• MiddleQ 14–16 years
•• Leukodystrophies (Alexander ds, Canavan dsQ, Metachromatic leukodystrophy)
•• LateQ 17–19 years
•• Cerebral gigantism (Sotos syndrome)Q.
6 Section 1: General Pediatrics

D. Hydranencephaly
Review of Pediatrics and Neonatology

Question 6 M

The given picture of a child with


•• Cerebral hemispheres are absent or represented by membranous sacs
recurrent frac­tures is suggestive of: •• Midbrain & brainstem is intact; but there is no cognitive development
•• Transillumination shows absent cerebral hemispheres.

E. Hydrocephalus M

•• Caused by impaired circulation &/or absorption of CSF, rarely by increased production


•• Ventriculoperitoneal shuntQ is the shunt of choice for treatment of hydrocephalus in
children.
a. Ehler Danlos syndrome
b. Osteogenesis imperfecta
c. Bitot’s spots Mnemonic M
d. Alkaptonuria
Causes of Hydrocephalus
Communicating Non-Communicating
“C-A-M-P” “M-A-A-A-D H-I-V”
Question 7 M
•• Choroid plexus papilloma •• Mass lesions
What is the most probable cause of •• AchondroplasiaQ •• Aqueductal stenosisQ
large head in this child? •• Meningeal malignancy •• Arnold Chiari malformationQ
•• Post-hemorrhagic •• Abscess
•• Dandy-Walker malformationQ
•• Hematoma
•• Infections (Toxoplasmosis, Mumps, Neurocysticercosis)
•• Vein of Galen malformation

Question 9
Identify this defect in a baby with large head:
a. Osteogenesis imperfecta
b. Mucopolysaccharidosis
c. Hydrocephalus
d. Cerebral gigantism

Question 8
Identify the abnormality seen in
the MRI given:
a. Encephalocele b. Hydranencephaly
c. Hydrocephalus d. Anencephaly

V. CHEST CIRCUMFERENCE (CC) M

•• It is usually measured at the level of nipples (4th Intercostal space)


•• At birth, HC > CC but difference between CC & HC is usually < 3 cmQ
•• HC = CC by the age of 9 months to 1 year.Q
a. Vein of Galen malformation •• If even after 1 year age, CC < HC it indicates malnutrition.
b. Arnold Chiari malformation
c. Dandy-Walker malformation
d. Choroid plexus papilloma VI. MID ARM CIRCUMFERENCE (MAC) M

•• It is measured in the middle of arm, using a non-stretchable measuring tape


•• Overlapping technique is to be used for measurement
•• Shakir’s tape is used by Health workers to measure MAC
•• Shakir's tape is divided into 3 colored zones:
–– Green zone is for MAC > 12.5 cm → normal nutritional status
–– Yellow zone for MAC 11.5–12.5 cm → borderline malnutrition
–– Red zone is for MAC < 11.5 cm → severe malnutrition.
Chapter 1: Normal and Abnormal Growth 7

Section 1: General Pediatrics


Question 10 Mnemonic
What is this instrument used to measure? For sequence of teeth eruption in
secondary dentition:
“Mama Is In Pain Papa Can Make
Medicine”
•• Molars (first)
a. Body mass index b. Skin fold thickness •• Incisors (Central)
c. Upper segment : lower segment ratio d. Mid arm circumference •• Incisors (Lateral)
•• Premolars (First)
•• Premolars (Second)
VII. AGE INDEPENDENT ANTHROPOMETRIC INDICES •• Canine
•• Molars (Second)
Used to assess nutritional status when child's age is not known. •• Molars (third)

Name of index Formula Normal value In malnutrition L at e s t U p d at e s


Kanawati & Mid arm circumference /Head 0.32 – 0.33 < 0.25 •• Previously mid arm circumference
Mc Laren’s index circumference (cm) was considered to be an age inde­
pendent anthropometric index,
Rao & Singh’s index Weight (kg)/ height2 (cm) × 100 > 0.14 0.12 – 0.14 with a constant value of ~ 16 cm
between 1 and 5 years
Dugdale’s index Weight (kg)/height1.6 (cm × 100) 0.88 – 0.97 < 0.79 •• Now we know that even MAC varies
with age
Quaker arm Mid arm circumference 75 – 85% Malnourished
•• Mid arm circumference for age
circumference measuring expected for a given height < 75% Severely malnourished charts are available for boys and
stick (quac stik)
girls in WHO growth charts
Jeliff’s ratio Head circumference/ Chest <1 in a child Ratio > 1 in a child
circumference >1 year age > 1 year indicates
Malnourished child

DENTITION M

Characteristics Primary dentition Secondary dentition

Also called Milk teeth or temporary teeth Permanent teeth

Begins at 6–7 monthsQ 6 yearsQ

Completes at 3 years 12 years (except for 3rd molar)

Total number of teeth 20 28–32 (depending on number of 3rd molars


erupted)

1st tooth Lower Central incisorsQ First molarsQ

Last tooth Second molars Third molars

Sequence Central incisors (appear First molars (appear first)Q


first)Q Central incisors
Lateral incisors Lateral incisors
First molars First premolars
Canines Second premolars
Second molars (appears Canine
last)Q Second molars Mnemonic
Third molars (appear last)Q Causes of Natal teeth
S: Soto’s syndrome
E: Ellis-van Creveld syndrome
High Yield Point E: Epidermolysis bullosa, lethal
acantholytic variety
Delayed Dentition is considered when there are no teeth by the age of 13 months P: Pierre Robin syndrome
8 Section 1: General Pediatrics
Review of Pediatrics and Neonatology

High Yield Points M n e m o n i c M

•• Advanced dentition is seen in Causes of Delayed Dentition


precocious puberty and hyper­ 'FRIED ChoP'
thyroidism •• Familial
•• Supernumerary teeth (more than •• Rickets
normal number) are seen in: Cleft •• Idiopathic; Incontinentia pigmenti
lip, cleft palate, cleidocranial •• Endocrine causes: Hypothyroidism, hypoparathyroi­dism, hypopituitarism
dysplasia, Gardner syndrome •• Down syndrome
•• Natal teeth refer to teeth already •• Cleidocranial dysplasia
present at time of birth •• Progeria

Question 11 M

What is the diagnosis in this child with delayed dentition?

a. Rickets b. Achondroplasia
c. Cleidocranial dysostosis d. Incontinentia pigmenti

Question 12 M

What is this teeth abnor­mality?

a. Mulberry molars b. Natal teeth


c. Hutchinson’s teeth d. Supernumerary teeth

High Yield Points  BONE DEVELOPMENT


•• Earliest ossification center to develop in a fetus is of calcaneusQ
•• For 1–6 years, Formula for number
•• Other centers seen at birth are distal end of femur, proximal end of tibia, head of humerus,
of carpal centers seen is: Age (in
years) + 1
talus & cuboid.
•• 1st carpal bone to appear in a child
is capitate
•• X-ray wrist and hand is best to assess
 NORMAL PUBERTY
bone age in a child. •• The term adolescence is derived from the latin word adolescere, meaning to grow, to
mature; Adolescent age group is 10–19 yrs age
•• ‘Puberty' is the biologic process in which a child becomes an adult
•• During the growth spurt that occurs in puberty, the increase in height seen in boys is 20–30 cm,
while in girls, it is 16–28 cm.
Chapter 1: Normal and Abnormal Growth 9

Sequence of Changes in Puberty in Females and Males

Section 1: General Pediatrics


M
High Yield Points
Females Males
WHO/UN definitions
ThelarcheQ Testicular enlargementQ •• Adolescent → 10–19 yrsQ
↓ ↓ •• Youth → 15–24 yrs
Pubarche Penile enlargement •• Young people → 10–24 yrs
↓ ↓
Growth spurt
Pubic hair growth


Menarche
Growth spurt change in
voice

Axillary hair

Facial hair

Pubertal status of a child/adolescent is assessed using sexual maturity rating (SMR), or Tanner
stagesQ.

High Yield Points


•• In girls, 1st visible sign of puberty & hallmark of SMR2 is appearance of breast buds or thelarche
•• Thelarche usually occurs between 8–12 yrs of age in girls
•• Menses typically begins 2–2½ yrs later, during SMR 3–4
•• Peak growth (growth spurt) occurs in females in Tanner’s stage 3

Tanner’s Staging in Females


10 Section 1: General Pediatrics
Review of Pediatrics and Neonatology

Question 13 M Question 14


Identify the instrument: What is the SMR staging based on the following findings in a female?

a. Growth assessment tool a. 2 b. 3


b. Urometer c. 4 d. 5
c. Stool assessment tool
d. Orchidometer
High Yield Points
•• In boys, the 1st visible sign of puberty and the hallmark of SMR 2 is testicular enlargement
•• Testicular enlargement begins around 9–10 yrs
•• Testicular volume is assessed using an ‘orchidometer’
•• Peak growth occurs in boys when testis volumes reach approximately 9–10 cm3 during SMR 4

Tanner’s Staging in Males

 GYNECOMASTIA
•• BenignQ subareolar glandular breast enlargement seen in up to 65% adolescent males
•• Typically appears at least 6 months after onset of secondary sex characteristics
•• Peak incidence is during SMR 3 and 4.Q

Difference between True & Pseudo Gynecomastia


•• In true gynecomastia, glandular tissue is presentQ, which is palpable as a fibro-glandular
mass located concentrically beneath the nipple areolar complex.
High Yield Points
•• Pubertal gynecomastia usually When to Evaluate?
regresses within 1–3 yrsQ, as it is •• If patient is Pre-pubertal, has eccentric breast mass with rapid progression
caused by a transient imbalance bet­ •• Any stigmata of chronic liver disease, thyroid or renal disease, hypogonadism.
ween estrogens and androgens •• History of drug use or sexual dysfunction or visual field defects.
Chapter 1: Normal and Abnormal Growth 11

Section 1: General Pediatrics


Answer Keys for Image-Based Questions

Answers Explanations/Identifying features


1. Ans. c. Measure length The instrument shown in the picture is an ‘Infanto­meter’. Infantometer is a horizontal rod with a ruler with
2 vertical boards on 2 ends (one fixed and other moveable), used to measure length of an infant.
2. Ans. d. Rubinstein-Taybi beaked nose (A), highly-arched eyebrows, retrognathia and broad thumbs (B) and great toes which are
syndrome often deviated medially.
3. Ans. c. Constitutional delay in He had short stature during childhood, but the final adult height attained is normal.
growth and puberty
4. Ans. b. Achondroplasia A → Short child with mainly short limbs;
B → Trident hand present
C → Champagne glass pelvis
All these are features of Achondroplasia
It is due to mutation in FGFR3 gene
5. Ans. c. Edward syndrome A.  Shows microcephaly, limb defects & rocker-bottom feet
B. Shows overlapping fingers
C.  Shows rocker-bottom foot (convexity of soles, like a rocking chair)
These features suggest a diagnosis of Edward syndrome or Trisomy 18 (Eighteen) [E-E].
6. Ans. b. Osteogenesis imperfecta Blue sclera in a child with recurrent fractures suggests a diagnosis of Osteogenesis imperfecta.
7. Ans. c. Hydrocephalus Large head in an infant with venous prominences over scalp and presence of ‘setting-sun sign’, suggest
hydrocephalus.
8. Ans. c. Dandy-Walker malfor­­ MRI brain shows dilated 4th ventricle with Cerebellar hypoplasia, suggestive of Dandy-Walker
mation syndrome.
9. Ans. b. Hydranencephaly Both cerebral hemispheres are absent & replaced by fluid filled sacs; Transillumination is positive.
10. Ans. d. Mid arm circum­ference A measuring tape with cm markings (from 6 to 26 cm) with 3 colored zones (red, yellow & green) is a Shakir’s
tape.
11. Ans. c. Cleidocranial dysostosis Large head with bilateral absent clavicles, such that the child can touch both his shoulders with each
other.
12. Ans. c. Hutchinson’s teeth Notched incisors, as seen in this image are known as Hutchinson’s teeth. They are seen in congenital
syphilis.
13. Ans. d. Orchidometer A string with ovoid bodies of different sizes with volume (in mL) written on each; This instrument is an
Orchidometer.
14. Ans. b. 3 Developed breast and areola, but no contour separation, no secondary mound; dark pubic hairs present,
but not curled; These are features of Tanner’s stage 3.
12 Section 1: General Pediatrics
Review of Pediatrics and Neonatology

Questions

 NORMAL GROWTH 11. Fetal growth is maximally affected by:


 (Recent Question 2013)
1.
All are true about anthropometric measurements except: a. Insulin b. Growth hormone
a. Birth weight triples by 1 year (JIPMER May 2018) c. Cortisol d. Thyroxine
b. Skinfold thickness is measured in sub scapular region
12. You order a wrist X-ray for a 2 months old child. Which
c. Weight is measured to the nearest 100 gram
carpal bone are you expecting to be present?
d. Standing height is about 1.7 cm less than recumbent length
2. Which of the following is not true in assessing skin fold a. Capitate b. Scaphoid  (JIPMER 2012)
thickness? (JIPMER May 2018) c. Lunate d. Trapezoid
a. Measured at the triceps level 13. Which of the following X-ray should be advised for age
b. Measured to the nearest of 1 cm determination between 1 and 13 years of age?
c. Tanner's chart contains the normal values  (MAHA PG 11)
d. Indirectly indicates the caloric reserve of the body
a. Shoulder b. Wrists
3. A newborn is double his height at birth by what age? M c. Elbow d. Iliac bones
 (Recent Question 2017)
a. 2½ years b. 3½ years 14. At the end of 1 year of age, the number of carpal bones
c. 4½ years d. 5½ years seen in the skiagram of the hand is: M (COMEDK 08)
4. True for a 2 years old boy is: (PGI Nov 2017) a. Nil b. 1
a. Weight four times that of birth weight c. 2 d. 3
b. Rides tricycle 15. The first epiphyseal centre appears in: M 
c. Climbs up and down the stairs with alternating steps  (Recent Question 08)
d. Both upper and lower canines erupted
e. Ossification centre for radius head appears a. Femur b. Cuboid
c. Ossicles d. Clavicle
5. Early adolescent age is: M (Recent Question 2017)
16. Birth weight doubles by? M  (Recent Question 2014)
a. 8–11 years b. 10–13 years
c. 14–15 years d. 16–19 years a. 3 months b. 5 months
6. True about constitutional growth delay is/are: c. 9 months d. 12 months
a. Normal length and weight at birth (PGI May 2016) 17. Weight of newborn quadruples by: M  (NEET Pattern 2013)
b. Catch up growth at 6–12 months a. 9 months b. 12 months
c. Height velocity assume normal rate at 2–3 years c. 2 years d. 3 years
d. Bone age > chronological age
e. Puberty occurs at normal age 18. The weight of the child at 3 years is usually—times the
7. All are unique features of WHO growth charts 2006 birth weight: M  (COMEDK 11)
except: M  (Recent Question 2016) a. 3 b. 4
a. Breast-fed infant as normative growth model c. 5 d. 7
b. Sample collected from six States in the US 19. Birth length doubles by what age? M 
c. Includes new indicators such as skinfold thickness  (Recent Question 2014)
d. Reiterates children grow similarly across the World a. 1 year b. 2 years
8. Postnatally when is the growth velocity maximum? M  c. 3 years d. 4 years
 (Recent Question 2015) 20. A normal healthy child has a height of 100 cm and weighs
a. In the first year of life b. In the second year of life 16 kg. What is his most likely age?
c. In the seventh year of life d. In adolescence  (Recent Question 2014)
9. Embryonic period is up to: M  (Recent Question 2014) a. 3 years b. 4 years
a. 8 weeks b. 10 weeks c. 5 years d. 6 years
c. 12 weeks d. 6 weeks 21. The normal gain in length in a full term baby, at first
10. Which of the following is/are true regarding development 6 months of life is: (JIPMER 2014)
of child during 6 to 12 years of age: (PGI Nov 2014) a. 6 cm b. 9 cm
a. Weight increases by 1–2 kg/yr c. 15 cm d. 24 cm
b. Head grow at the rate of 2–3 cm/yr 22. Average gain of length in first year is: M
c. Growth occur continuously  (Recent Question 2013)
d. Growth occur in spurts a. 25 cm b. 50 cm
e. Height increases at rate of 6–7 cm/yr c. 75 cm d. 100 cm
Chapter 1: Normal and Abnormal Growth 13

23. The height of a child is double the birth height at the age

Questions
36. Microcephaly is defined as HC: M 
of: M  (NEET Pattern 2013)  (Recent Question 2014)
a. 1 year b. 2 years a. 2SD less than the mean b. 3SD less than the mean
c. 4 years d. 6 years c. <33 cm at birth d. <40 cm at 6 months
24. Increase in height in first year is by: M  (DNB 2011) 37. The rate of increase in HC in the first 3 months: M
a. 40% b. 50%  (DNB Dec 2012)
c. 60% d. 75% a. 2 cm/month b. 1 cm/month
25. Increase in height of the child from 1–3 years: c. 0.5 cm/month d. 0.25 cm/month
a. 10 cm b. 15 cm (DNB Dec 2010) 38. Sagittal sutures overlapping causes: (JIPMER 2012)
c. 20 cm d. 25 cm a. Oxycephaly b. Dolichocephaly
26. Maximum growth spurt in boys occurs at: (DNB June 2010) c. Plagiocephaly d. Cephalocephaly
a. 8–10 years b. 10–11 years 39. In a healthy child, the head and chest circumference equal
c. 12–13 years d. 14–15 years each other around the age of: M  (COMEDK 2012)
a. 3–6 months b. 6–9 months
 HEAD CIRCUMFERENCE c. 9–12 months d. 12–15 months
40. 90% of brain growth is achieved by the: (COMED 2012)
27. Which of the fontanelle is the last to close?
a. 2nd year b. 3rd year
 (NEET pattern Jan 2018)
c. 5th year d. 15th year
a. Posterior fontanelle b. Anterior fontanelle
41. True about head circumference measurement:
c. Mastoid fontanelle d. Sphenoidal fontanelle
a. Measured at supraorbital ridge (PGI Dec 2008)
28. Crouzon syndrome consists of: (PGI May 2018)
b. Measures hydrocephalus/microcephaly
a. Maxillary hypoplasia b. Syndactyly c. Serial measurement is useful
c. Hydrocephalus d. Macrocephaly d. Helps in measurement of neurological development
e. Mandibular prognathism
e. Is an indicator of brain growth
29. Fetal alcohol syndrome comprises of: (DNB June 2018)
a. Microcephaly b. Deafness  DENTITION
c. Short palpebral fissure d. All of the above
30. Turricephaly is: (DNB June 2018) 42. Delayed eruption is usually considered when there are no
teeth by approximately by the age of: M 
a. Tall head b. Narrow head
c. Wide head d. Short head a. 18 months of age (mean +3 SD) (Recent Question 2017)
b. 9 months of age (mean +3 SD)
31. Causes of macrocephaly includes: (PGI Nov 2017) c. 16 months of age (mean +3 SD)
a. Galactosemia b. Weaver syndrome d. 13 months of age (mean +3 SD)
c. Rett syndrome d. Soto’s syndrome 43. Eruption of 1st molar permanent tooth occurs at the age
e. Morquio syndrome of: M  (WB PGMEE 2016)
32. Following feature differentiate hydranencephaly from a. 4–5 years b. 6–7 years
hydrocephalus: (Recent Question 2016) c. 7–8 years d. 8–10 years
a. Hydranencephaly is static while hydrocephalus continue 44. Most common location of supernumerary teeth is:
to increase in size  (Recent Question 2014)
b. Hydranencephaly does not cause increase in head size a. Between incisors b. Between canines
c. Cerebral cortex is deficient/ hypoplastic in c. Between molars d. Between premolars
hydranencephaly
d. All of the above 45. True about dentition are all except:
 (Recent Question 2014)
33. Anterior fontanelle closes at: M 
 (Recent Question 2016) a. Hypothyroidism causes delayed dentition
b. Premolar is not seen in primary dentition
a. 2–3 months b. 4–7 months
c. Third molar is the last to appear in secondary dentition
c. 12–20 months d. 24–30 months
d. Canine is the first in primary dentition
34. Anterior fontanelle is located between which two bones? M
 (Recent Question 2015) 46. Which of the following are the first incisors to erupt in an
infant? (DNB June 2013)
a. Frontal & occipital b. Frontal & parietal
a. Lower central b. Lower lateral
c. Parietal & occipital d. Frontal & zygomatic
c. Upper central d. Upper lateral
35. Premature fusion of coronal, sphenofrontal and fronto­ 47. Primary dentition begins to show teeth eruption by: M
ethmoidal sutures is seen in: (MAHA PGM CET 2015)  (NEET Pattern 2013)
a. Turricephaly b. Dolichocephaly a. 6 weeks b. 12 weeks
c. Plagiocephaly d. Trigoncephaly c. 6 months d. 12 months
14 Section 1: General Pediatrics
Review of Pediatrics and Neonatology

48. Ist permanent teeth to appear: M  (Recent Question 2013) 59. In boys which is the first visible sign of puberty? M
a. Molar b. Premolar  (MAHA PGM CET 2015)
c. Incisor d. Canine a. Testicular enlargement b. Penile growth
49. Milk teeth—Total no. in human being: M  c. Breast hypertrophy d. Dark scrotum
 (Recent Question 2013) 60. True about Tanner stage III: (PGI May 2015)
a. 20 b. 28 a. Penis increases in length
c. 32 d. 24 b. Penis increases in width
50. Delayed dentition is seen in: M  (Recent Question 2013) c. Scanty hair at base of penis
d. Darkening of scrotum
a. Down syndrome b. Congenital hypothyroidism e. More growth occur in boys than girls
c. Rickets d. All of the above
61. What is thelarche? (NEET Pattern 2013-2012)
51. By ___ years all milk teeth are erupted: M
a. Pubertal breast enlargement in boys
 (Recent Question 2013) b. Breast enlargement in pregnancy
a. 1.5 b. 2 c. Hormone related breast enlargement in girls
c. 2.5 d. 3 d. Post hormonal therapy breast enlargement in postmeno­
52. First teeth to erupt in primary dentition is: M  pausal females
 (NEET Pattern 2012) 62. Adolescent age starts at the age of:
a. First molar b. Premolar  (Recent Question 2013)
c. Incisor d. Canine a. 7 years b. 10 years
53. Which of the following is true about eruption of teeth? c. 14 years d. 17 years
 (PGI Dec 2012) 63. Infantile proportion in adult is seen in:
a. Premolar appears first in primary dentition  (Recent Question 2013)
b. Incisors appear first in secondary dentition a. Morquio's disease b. Achondroplasia
c. Third molar is last to develop c. Hypothyroidism d. Malnutrition
d. Hypothyroidism delays dentition 64. Which one of the following is the correct order of events at
e. Canines are last to appear in primary dentition puberty in a girl? (Recent Question 2013)
a. Thelarche-pubarche-menarche-growth spurt
 NORMAL PUBERTY b. Pubarche-thelarche-growth spurt-menarche
c. Menarche-growth spurt-thelarche-pubarche
54. Earliest indication of sexual maturation in females: d. Thelarche-pubarche-growth spurt-menarche
 (FMGE June 2018)
65. Growth spurt occurs: (DNB Dec 2012)
a. Menarche b. Thelarche
c. Pubarche d. Genital maturation a. Just before appearance of axillary hair
b. Just before menarche
55. What is the first visible sign of puberty and the hallmark of
c. After 16 years
SMR2 in boys and at what age does it occur?
 (Recent Question 2017) d. Before thelarche
66. WHO defines adolescent age between: M
a. Testicular enlargement and 9 and ½ years
 (DNB June 2012)
b. Pubic hair and 11 and ½ years
c. Testicular enlargement and 11 and ½ years a. 10–19 years b. 10–14 years
d. Pubic hair and 9 and ½ years c. 10–25 years d. 9–14 years
56. What age range does the Phallic/Oedipal stage of Freud’ 67. Mid adolescence is the age between: (TN PGMEE 2012)
theory correspond to: (Recent Question 2017)
a. 10–13 years b. 14–16 years
a. 0–1 year b. 3–6 years c. 17–19 years d. 14–18 years
c. 2–3 years d. 6–12 years
68. Areola and papilla forming secondary mound in adoles­
57. Which of the following age group, falls under early cent girls is classified under which stage of sexual maturity
adolescence age group: M  rating (SMR)? (JIPMER 2009)
 (Recent Question 2018, MAHA PGM CET 2016) a. SMR stage 2 b. SMR stage 3
a. 8 – <10 years b. 10–13 years c. SMR stage 4 d. SMR stage 5
c. 14–16 years d. 17–19 years 69. A 13 years old boy has bilateral gynecomastia. His height is
58. Earliest sign of puberty in females is? M  148 cm, weight 58 kg, the sexual maturity rating is stage 2.
The gynecomastia is most likely due to:
 (Recent Question 2018, AIIMS Nov 2015)
 (AIIMS Nov 2004)
a. Growth spurt b. Pubarche a. Prolactinoma b. Testicular tumor
c. Thelarche d. Menarche c. Pubertal gynecomastia d. Chronic liver disease
Chapter 1: Normal and Abnormal Growth 15

 SHORT STATURE 77. Short stature are found in all except: (PGI Nov 2011)

Questions
a. Malnutrition b. Emotional deprivation
70. GH deficiency is diagnosed by: M  c. Congenital heart disease d. Klinefelter syndrome
 (Recent Question 2017) e. Hypothyroidism
a. Bone age less than chronological age
78. True about constitutional delay in growth:
b. Bone age more than chronological age
c. Bone age equal to chronological age a. Normal length and weight at birth (PGI May 2011)
d. Ratio of upper to lower segment of body is increased b. Catch up growth at 6–12 months
71. Which of the following statement regarding ‘familial short c. Height velocity assume normal rate at 2–3 years
stature’ is incorrect? (WB PGMEE 2015) d. Bone age < chronological age
a. Puberty is delayed b. Bone age is normal e. Puberty occurs at normal age
c. Growth is slow from birth 79. Bone age is less than chronological age in all except:
d. Growth hormone levels are normal  (WBPGMEE 2010)
72. Which of the following is true about constitutional delay in a. Familial short stature b. Constitutional short stature
growth? (Recent Question 2013) c. Under nutrition d. Hypopituitarism
a. Neonates with constitutional delay show anomalies at birth 80. Growth hormone deficiency is associated with all except:
b. IGF-1 levels are low for chronological age  (DNB June 2010)
c. Bone age is normal
a. Micropenis b. Doll like facies
d. IGF-1 levels are low for bone age
c. IUGR d. Obesity
73. A child’s growth velocity is normal but bone development is
not according to the chronological age. What is diagnosis? 81. Cretinism is: (DNB June 2008)
M (Recent Question 2013) a. Disproportionate dwarfism
a. Genetic b. Dwarfism b. Short stature with long trunk
c. Constitutional delay d. Familial short stature c. Short stature with short trunk
74. Short stature, secondary to growth hormone deficiency is
d. Long stature with long trunk
asso­ciated with: M  82. Most common cause of short stature: M 
 (NBE based NEET 2013; AIIMS Dec 1995)  (TN PGMEE 2008, AIPGMEE 2008)
a. Normal body proportion b. Low birth weight a. Familial
c. Normal epiphyseal development b. Constitutional
d. Height age equal to skeletal age c. Hypothyroidism
75. Upper segment to lower segment ratio in 3 years age child d. Growth hormone deficiency
is: M  (Recent Question 2013) 83. A newborn is noted to have a large head and short limbs.
a. 1.2:1 b. 1.3:1 On further examination, short broad fingers, a small face,
c. 1.4:1 d. 1.6:1 and low-normal length are noted. The trunk appears long
and narrow, to confirm the diagnosis you should:
76. A child is below the third percentile for height. His growth
 (MAHA 2005)
velocity is normal, but chronologic age is more than
skeletal age. The most likely diagnosis is: a. Order an ophthalmologic examination
a. Constitutional delay in growth (AIPGMEE 2011) b. Obtain skeletal radiographs
b. Genetic short stature c. Order chromosome analysis
c. Primordial dwarfism d. Hypopituitarism d. Examine the parents
16 Section 1: General Pediatrics
Review of Pediatrics and Neonatology

Answers with Explanations

 NORMAL GROWTH
1. d. Standing height is about 1.7 cm less than recumbent length  Ref: Training Course on Child Growth Assessment WHO Child
Growth Standards, 2008
•• If a child < 2 years old, measure recumbent length.
•• If ≥≥ 2 years age & able to stand, measure standing height.
•• Standing height is about 0.7 cm less than recumbent length.
2. b. Measured to the nearest of 1 cm  Ref: Training Course on Child Growth Assessment WHO Child Growth Standards, 2008
Tanner skinfold standards (1975), were considerably higher than subsequent reported measurements.
3. c. 4½ years  Ref: Nelson’s 20/e p 84–89
Weight doubles at 5 months while height at 4–4½ years.
4. a. weight four times that of birth weight, d. Both upper and lower canines erupted  Ref: Nelson’s 20/e p 1768

a. True At 1 year weight becomes 3 times & at 2 years, 4 times of birth weight
b. False A child is able to ride a tricycle usually at 3 years age
c. False A child is able to climb up & down stairs with alternating steps at 4 years
d. True Upper & lower canines usually erupt at around 16–20 weeks age
e. False Ossification centre for radius head appears between 3 and 4 years

5. b. 10–13 years Ref: Nelson’s 20/e p 926–29


6. a. Normal length and weight at birth  Ref: Nelson’s 20/e p 2643
In constitutional growth delay, weight and length (height) are normal at birth, but decrease near the end of infancy, parallel the
norm through middle childhood (6–11 years age), and accelerate toward the end of adolescence.
The bone age is less than chronological age, but comparable to the height age. Adult size is normal.
7. b. Sample collected from six States in the US  Ref: Nelson’s 20/e p 84-89, Ghai 8/e p 96-97
WHO growth charts are based on multicentre growth reference study (MGRS)Q conducted in 6 countries all over the world
including: Brazil, Oman, Norway, Ghana, US and India (New Delhi).
8. a. In the first year of life  Ref: Nelson’s 20/e p 84, Ghai 8/e p 96
Postnatally there are 2 periods of accelerated growth: first year of life and growth spurt during puberty.
9. a. 8 weeks  Ref: Nelson’s 20/e p 84, Ghai 8/e p 96
Embryo refers to the product of conception from the day of fertilization (generally 14th day post ovulation) to 8 weeks.
10. d. Growth occur in spurts; e. Height increases at rate of 6–7 cm/year  Ref: Nelson’s 20/e p 86, Ghai 8/e p 97
During 6–11 years (middle childhood): growth averages 3–3.5 kg/year and 6–7 cm/year.
Growth occurs discontinuously, in 3–6 irregularly timed spurts/year.
Head grows only 2 cm in circumference throughout the entire period, reflecting a slowing of brain growth.
11. a. Insulin  Ref: Nelson’s 20/e p 84, Ghai 8/e p 96-97
Insulin and Insulin-like growth factor stimulate fetal growth.
12. a. Capitate  Ref: Nelson’s 20/e p 3241, Ghai 8/e p 35-36
At birth, there is no calcification in the carpal bones.
Although there is great individual variability, approximate ossification times are as follows:

Capitate: 1–3 months Scaphoid: 4–6 years


Hamate: 2–4 months Trapezium: 4–6 years
Triquetrum: 2–3 years Trapezoid: 4–6 years
Lunate: 2–4 years Pisiform: 8–12 years

Chapter 1: Normal and Abnormal Growth 17

Answers with Explanations


Mnemonic
Remember order of ossification of carpal bones
Excluding the pisiform, an easy way to remember the order of ossification is
to start at the capitate then move in a clockwise direction on the dorsal
surface of the right hand

13. b. Wrists  Ref: Nelson’s 20/e p 3241
14. c. 2  Ref: Nelson’s 20/e p 3241
15. a. Femur  Ref: Nelson’s 20/e p 3241
First epiphyseal centre to appear are: Dorsal side of right hand
The calcaneal and talar ossification centers & the distal femoral & proximal tibial epiphyseal ossification centers are found at
the level of the knee.
16. b. 5 months  Ref: Nelson’s 20/e p 3241
17. c. 2 years  Ref: Nelson’s 20/e p 84-89, Ghai 8/e p 13-14
18. c. 5  Ref: Nelson’s 20/e p 84-89, Ghai 8/e p 13-14
19. d. 4 years  Ref: Nelson’s 20/e p 84-89, Ghai 8/e p 13-14
20. b. 4 years  Ref: Nelson’s 20/e p 84-89, Ghai 8/e p 13-14
At 4 years, height becomes 100 cm and weight becomes 2 x 4 + 8 = 16 kg.
21. c. 15 cm  Ref: Nelson’s 20/e p 84-89, Ghai 8/e p 13-14
•• Length at birth = 50 cm
•• Length at 3 months = 60 cm; So, gain in length in 3 months = 60–50 = 10 cm
•• Length at 9 months = 70 cm; So, gain in length in 9 months = 70–50 = 20 cm
•• So, gain in length in 6 months is between 10 and 20 cm, so 15 cm is the best answer.
22. a. 25 cm  Ref: Nelson’s 20/e p 84-89, Ghai 8/e p 13–14
Gain of Length in first year is 75–50 cm = 25 cm.
23. c. 4 years  Ref: Nelson’s 20/e p 84–89, Ghai 8/e p 13–14
24. b. 50%  Ref: Nelson’s 20/e p 84-89, Ghai 8/e p 13–14
In 1st year, From 50 cm, the height increases by 25 cm (i.e. 50%) to 75 cm.
25. c. 20 cm  Ref: Nelson’s 20/e p 84-89, Ghai 8/e p 13-14
Height at 1 year is 75 cm and at 4 years is 100 cm; So increase in height from 1–4 years is 100 – 75 = 25 cm. Height at 1 year is
75 cm and at 2 years is 90 cm; So increase in height from 1–2 years is 90 – 75 = 15 cm.
So increase in height from 1–3 years lies between 15 and 25 cm, so 20 cm is the best answer.
26. d. 14–15 years  Ref: Nelson’s 20/e p 84-89, Ghai 8/e p 13-14
Height velocity curves for males (solid line) and females (dashed line) showing maximum growth velocity in:
•• Boys at 14–15 yrs
•• Girls at 11–13 yrs
18 Section 1: General Pediatrics

 HEAD CIRCUMFERENCE
Review of Pediatrics and Neonatology

27. b. Anterior fontanelle  Ref: Nelson’s 20/e p 2793


Time of closure of different fontanelle:
•• Posterior fontanelle—2 to 3 months after birth
•• Sphenoidal fontanelle—6 months after birth
•• Mastoid fontanelle—6 to 18 months after birth
•• Anterior fontanelle—10–24 months after birth.
28. a. Maxillary hypoplasia; c. Hydrocephalus; e. Mandibular prognathism  Ref: Nelson’s 20/e p 2819
Crouzon syndrome:
•• Autosomal dominant (FGFR2 gene)
•• Craniosynostosis involving coronal suture
•• Wide-set, bulging eyes & beaked nose
•• Maxillary hypoplasia
•• Underdeveloped upper jaw & protruding lower jaw
•• Dental problems, hearing loss & high risk for hydrocephalus
•• It is the most common craniosynostosis syndrome
29. d. All of the above  Ref: Nelson’s 20/e p 2813;896
Fetal alcohol syndrome:
•• Growth retardation
•• Microcephaly, small palpebral fissure
•• Smooth philtrum, thin upper lip
•• Increased risk of hearing loss of both conductive and sensorineural origin.
30. a. Tall head  Ref: Nelson’s 20/e p 2818
Turricephaly is cone-shaped head from premature fusion of the coronal, sphenofrontal & frontoethmoidal sutures.
31. a. Galactosemia, b. Weaver syndrome, d. Soto’s syndrome, e. Morquio syndrome  Ref: Nelson’s Fenichel 6/e p 370
Causes of Megalencephaly:
•• Lysosomal storage disorders: MPS, Gangliosidosis
•• Carbohydrate disorder: Galactosemia
•• Aminoacidurias: MSUD, Glutaric aciduria type I
•• Neurodegenerative ds: Alexander ds, Canavan ds, Metachromatic leukodystrophy
•• Neurocutaneous ds: Neurofibromatosis, Tuberous sclerosis Soto's syndrome, Achondroplasia
•• Benign familial megalencephaly
•• Weaver syndrome is a condition that involves tall stature with or without macrocephaly, a variable degree of intellectual disability
& characteristic facial features.
32. c. Cerebral cortex is deficient/hypoplastic in hydranencephaly  Ref: Nelson’s 20/e p 2817
In Hydranencephaly, cerebral hemispheres are absent or represented by membranous sacs with remnants of frontal, temporal,
or occipital cortex dispersed over the membrane, while the midbrain and brainstem are relatively intact
33. c. 12–20 months  Ref: Nelson’s 20/e p 2792
Anterior fontanelle is a diamond-shaped structure at the junction of the frontal and parietal bones, that is open at birth
and usually measures 2 × 2 cm. The average time of closure is 18 mo, but the fontanelle can close normally as early as
9 months.
34. b. Frontal and parietal  Ref: Nelson’s 20/e p 71-72, Ghai 8/e p 12-13
Chapter 1: Normal and Abnormal Growth 19

35. a. Turricephaly  Ref: Nelson’s 20/e p 71-72, Ghai 8/e p 12-13

Answers with Explanations


Discussing about the options one by one,
a. Turricephaly Cone-shaped head caused by premature fusion of coronal, sphenofrontal and frontoethmoidal sutures
b. Dolichocephaly Due to premature closure of sagittal suture produces a long and narrow skull
Also called scaphocephaly; It is the most common form of craniosynostosis
c. Plagiocephaly Asymmetrical distortion or flattening of one side of the skull
d. Trigoncephaly Caused by premature fusion of metopic suture → Keel-shaped forehead and hypotelorism Increased risk for
associated developmental abnormalities of the forebrain

36. b. 3SD less than the mean  Ref: Nelson’s 20/e p 71-72, Ghai 8/e p 12-13
37. a. 2 cm/month  Ref: Nelson’s 20/e p 71-72, Ghai 8/e p 12-13
Rate of increase in head circumference in 1st 3 months of life is 2 cm/month.
38. b. Dolichocephaly  Ref: Nelson’s 20/e p 71-72, Ghai 8/e p 12-13
39. c. 9-12 months  Ref: Nelson’s 20/e p 71-72, Ghai 8/e p 12-13
40. b. 3rd year  Ref: Nelson’s 20/e p 71-72, Ghai 8/e p 12-13
About 85% of adult head circumference is achieved by age of 2 years and 90% of brain growth is by 3 years of age.
41. a. Measured at supraorbital ridge; b. Measures hydrocephalus/microcephaly; c. Serial measurement is useful;
d. Helps in measurement of neurological development; e. Is an indicator of brain growth  Ref: Nelson’s 20/e p 71-72

 DENTITION

42. d. 13 months of age (mean +3 SD)  Ref: Nelson’s 20/e p 89


43. b. 6–7 years  Ref: Nelson’s 20/e p 1768-1771, Ghai 8/e p 11
Permanent dentition begins at 6 years and is completed by 12 years (except for 3rd molar).
44. a. Between incisors  Ref: Nelson’s 20/e p 1768-1771, Ghai 8/e p 11
Supernumerary teeth most commonly involves maxillary incisor region.
45. d. Canine is the first in primary dentition  Ref: Nelson’s 20/e p 1768-1771, Ghai 8/e p 11
The first tooth to erupt in primary dentition is lower central incisor, while in secondary dentition, it is the first molar.
46. a. Lower central  Ref: Nelson’s 20/e p 1768-1771, Ghai 8/e p 11
47. c. 6 months  Ref: Nelson’s 20/e p 1768-1771, Ghai 8/e p 11
48. a. Molar  Ref: Nelson’s 20/e p 1768-1771, Ghai 8/e p 11
49. a. 20  Ref: Nelson’s 20/e p 1768-1771, Ghai 8/e p 11
Primary dentition comprises of 20 teeth, which begins at 6 months of age and completes by 3 years.
50. d. All of the above  Ref: Nelson’s 20/e p 1768-1771, Ghai 8/e p 11
51. d. 3  Ref: Nelson’s 20/e p 1768-1771, Ghai 8/e p 11
52. c. Incisor  Ref: Nelson’s 20/e p 1768-1771, Ghai 8/e p 11
53. c. Third molar is last to develop, d. Hypothyroidism delays dentition  Ref: Nelson’s 20/e p 1768-1771, Ghai 8/e p 11

 NORMAL PUBERTY
54. b. Thelarche  Ref: Nelson’s 20/e p 926-929
Earliest indication of sexual maturation in females is thelarche, while in males, it is testicular enlargement.
55. a. Testicular enlargement and 9 and ½ years  Ref: Nelson’s 20/e p 926
56. b. 3–6 years  Ref: Nelson’s 20/e p 52
Phallic/Oedipal stage of Freud theory correspond to the preschool age (3–6 years); At the core of Freudian theory is the idea of body-
centered (or, broadly, “sexual”) drives.
57. b. 10–13 years  Ref: Nelson’s 20/e p 926-929, Ghai 8/e p 63-69
Adolescent group Early Middle Late
Age range 10–13 years 14–16 years 17–19 years

20 Section 1: General Pediatrics

58. c. Thelarche  Ref: Nelson’s 20/e p 926-929, Ghai 8/e p 63-69


Review of Pediatrics and Neonatology

59. a. Testicular enlargement  Ref: Nelson’s 20/e p 926-929, Ghai 8/e p 63-69
60. a. Penis increases in length  Ref: Nelson’s 20/e p 926-929, Ghai 8/e p 63-69
For Classification of Sexual Maturity Status in Boys, refer pretext of this chapter;

In Tanner's stage 3 Pubic hair is darker, starting to curl Penis lengthens and testes becomes Larger

Note: Peak increase in height in boys is usually seen in Tanner’s stage 4.
61. c. Hormone related breast enlargement in girls  Ref: Nelson’s 20/e p 926-929, Ghai 8/e p 63-69
62. b. 10 years  Ref: Nelson’s 20/e p 926-929, Ghai 8/e p 63-69
63. b. Achondroplasia  Ref: Nelson’s 20/e p 3371
Normal body proportion
  In adults   In infants
Height = arm span Height (length) > arm span
Upper segment = Lower segment Upper segment > Lower segment

Infantile proportion refers to increased upper segment: Lower segment ratio, as seen in an infant. It is seen in causes of short
limb dwarfism like Achondroplasia and Hypothyroidism.
Reverse infantile body proportion is seen in Eunuchoidism, Marfan syndrome, Homocystinuria and Klinefelter’s syndrome.
64. d. Thelarche-pubarche-growth spurt-menarche  Ref: Nelson’s 20/e p 926-929, Ghai 8/e p 63-69
65. b. Just before menarche  Ref: Nelson’s 20/e p 926-929, Ghai 8/e p 63-69
66. a. 10–19 years  Ref: Nelson’s 20/e p 926-929, Ghai 8/e p 63-69
67. b. 14–16 years  Ref: Nelson’s 20/e p 926-929, Ghai 8/e p 63-69
68. c. SMR stage 4  Ref: Nelson’s 20/e p 926-929, Ghai 8/e p 63-69
69. c. Pubertal gynecomastia  Ref: Nelson’s 20/e p 926-929, Ghai 8/e p 63-69
Pubertal gynecomastia:
•• It is a benign subareolar glandular breast enlargement seen in up to 65% adolescent males.
•• Typically appears at least 6 months after onset of secondary sex characteristics.

 SHORT STATURE
70. a. Bone age less than chronological age  Ref: Nelson’s 20/e p 2643
In GH deficiency bone age is less than chronological age by 2 years
71. a. Puberty is delayed  Ref: Nelson’s 20/e p 2643, Ghai 8/e p 35-37
Normal variants of short stature:

Birth weight Skeletal Timing of


Cause and length maturation puberty Family history Other features
Familial short stature Normal Normal Normal Of short stature Normal growth velocity
Constitutional growth delay Normal Delayed Delayed Of delayed puberty Normal growth velocity

72. b. IGF-1 levels are low for chronological age  Ref: Nelson’s 20/e p 2643, Ghai 8/e p 35-37
In constitutional delay, bone age is less than chronological age and IGF-1 levels are also low for chronological age.
73. c. Constitutional delay  Ref: Nelson’s 20/e p 2643, Ghai 8/e p 35-37
74. a. Normal body proportion  Ref: Nelson’s 20/e p 2643, Ghai 8/e p 35-37
Short stature due to human growth hormone deficiency is characterized by:
•• Normal ratio of upper to lower segment, so normal body proportion
•• Bone age or epiphyseal development is less than chronological age by about 2 years
•• Normal height and weight at birth; Delay in growth is usually observed after the age of one year.
75. b. 1.3:1  Ref: Nelson’s 20/e p 2643, Ghai 8/e p 35–37
Normal upper segment to lower segment ratio in children:
Newborn → 1:7 – 1.9:1  ⇒ 3 years → 1:3 : 1  ⇒ 7 years → 1:1Q.
Chapter 1: Normal and Abnormal Growth 21

76. a. Constitutional delay in growth   Ref: Nelson’s 20/e p 2643, Ghai 8/e p 35-37

Answers with Explanations


The child in this Question has: Height below 3rd percentile, i.e., short stature.

Normal growth velocity, i.e., normal variant of short stature and Skeletal age is less than chronological age.
So, this child has Constitutional delay in growth.
77. d. Klinefelter syndrome  Ref: Nelson’s 20/e p 2643, Ghai 8/e p 35-37
78. a. Normal length and weight at birth; d. Bone age < chronological age  Ref: Nelson’s 20/e p 2640, Ghai 8/e p 35-37
79. a. Familial short stature  Ref: Nelson’s 20/e p 2643, Ghai 8/e p 35-37
80. c. IUGR  Ref: Nelson’s 20/e p 821-829, Ghai 8/e p 155
Growth hormone deficiency is associated with Hypoglycemia, Hypothyroidism, Micropenis, Truncal adiposity.
81. a. Disproportionate dwarfism  Ref: Nelson’s 20/e p 821, Ghai 8/e p 155
In hypothyroidism, patient has disproportionate short stature with short limbs.
82. b. Constitutional  Ref: Nelson’s 20/e p 821, Ghai 8/e p 155
83. b. Obtain skeletal radiographs  Ref: Nelson’s 20/e p 3371-3372
The given description suggests a diagnosis of Achondroplasia; Skeletal X-rays will help in diagnosing it.
Chapter 2
Development
Question 1  NORMAL DEVELOPMENT M

What is the probable age of this What is it? Maturation of functions


baby?
Domains of Gross motor Milestones related to posture & movement
development Fine motor Related to manipulation of objects by hand
Language Related to hearing, understanding & speaking
Personal social Related to interaction with self & others

 RULES OF DEVELOPMENT
a. 2 weeks  b. 6 weeks •• It is a continuous process, beginning in utero
c. 3 months  d. 4 months •• It depends on functional maturation of CNS
•• Sequence of attainment of milestones usually remains same
•• It progresses in cephalocaudal direction
High Yield Points •• Some milestones appear on loss of primitive reflexes
•• Disorganised mass activity is replaced by specific action
•• Development quotient (DQ) =
Developmental age/actual age * 100  IMPORTANT DEVELOPMENTAL MILESTONES

A. GROSS MOTOR
Milestones seen in Prone position

Mnemonic  M
•• A child learns to walk in 1 year
& sits in half of that time, i.e.
6 months Birth-2 weeks 1-2 months 3 months 6 months
•• ‘a’ comes before ‘e’, so crawling High pelvis, knees Raises head & chest off Bears weight on forearm Supports weight on
comes before creeping drawn up under table at 45° extended forearms
abdomen, head
turned to one side
Question 2
When does the milestone shown in Important Gross Motor Milestones in the 1st Year M

this picture appear?


Age Important milestones
1 month •• In ventral suspension: head belowQ the plane of rest of body
•• Pulled to sit – almost complete head lag
2 months •• In ventral suspension, head in the plane of rest of bodyQ
•• Prone position: Lifts chest off table at 45o
3 months •• In ventral suspension, head above the plane of rest of body
•• Prone position: Bears weight on forearms with chin & shoulders off couch
•• Neck holding present; Rounded back when made to sit
a. 6-7 months b. 10-11 months
c. 15 months d. 18 months 4 months Partial weight bearing, when made to stand
5 months Feet to mouth; Complete neck control;
6 months Sitting with support;Q Prone to supine (Rolls over)
Contd...
Chapter 2: Development 23

Contd...

Section 1: General Pediatrics


Question 3
7 months Supine to prone
What is the sequence in which the
8 months Sitting without support; Crawls (Ghai 9th ed)
following miles­tones are attained?
9 months Standing with support
10 months Creeping (Ghai 9th ed)
10-11 months Pivots, Cruising
12 months •• Standing without support A B
•• Walking with support
•• Walks with 1 hand held
•• “Bear walking”

C D
Gross Motor Milestones beyond 1st Year M
a. A → B → C → D
Age Important milestones b. D → C → B → A
c. D → C → A → B
15 months Creeps upstairs, walks without support d. D → B → C → A
18 months Goes upstairs & downstairs holding the railing, runs, pulls a toy
24 months Walks backwards
Kicks a ballQ Mnemonic  M
Goes upstairs & downstairs 2 feet/stepQ
•• Rides a tricycle (3 wheels): 3 years
36 months Riding a tricycleQ
•• Goes upstairs & downstairs
Goes upstairs 1 foot /step & downstairs 2 feet/step 2 feet/step at 2 years
4 years Hops on one foot; Goes upstairs & downstairs with alternating feet
5 years Skipping on both feet
Question 4
B. FINE MOTOR At what age can a child with a DQ
of 50, do the following?
Milestones Attained in the 1st Year M

Age Important Milestones


1 month Hands kept closed
2 months Hands intermittently open
3 months •• Hands kept open
•• Hand regard appears
a. 3 years b. 4 years
•• Holds an object when placed in hand
c. 5 years d. 6 years
4 months Tries to reach for an object but overshoots
5 months Bidextrous grasp; Mouthing
6 months Unidextrous / palmar graspQ; Takes biscuit to mouth
7 months Transfers objects
9 months Immature / assisted pincer graspQ
12 months •• Mature / unassisted pincer grasp;
•• CastingQ

Fine Motor Milestones Beyond 1 Year Age M


Question 5
Age Important Milestones When does this milestone dis­
15 months •• ScribblesQ spontaneously appear?
•• Tower of 2 cubesQ
•• Drinks from cup
18 months •• Turns 2-3 pages at a time
•• Feeds self with spoon
•• Tower of 3 cubes
24 months •• Turns pages of a book singly
•• Turns door knob, unscrews lid
•• Makes tower of 6-7 cubesQ a. 12 weeks b. 15 weeks
•• Imitates horizontal or vertical line puts on shoes & socks c. 20 weeks d. 28 weeks
Contd...
24 Section 1: General Pediatrics

Contd...
Review of Pediatrics and Neonatology

Mnemonic  M
36 months •• Dresses & undresses self except buttons
Number of cubes put in tower by a •• Tower of 9-10 cubes
child is age × 3; so, at 2 yrs, 6 cubes and at •• HandednessQ established
3 years, 9 cubes. •• Copies circle
4 years •• Buttons clothes
•• Copies rectangle / plus sign / cross
•• Catches ball reliably
5 years •• Can tie shoe laces
•• Copies triangle / multiplication sign / tilted cross

Question 6
When does a child learn to make
the following?

a. 9 months b. 12 months Mnemonic  M


c. 15 months d. 18 months Drawing skills of a child
Copies Age How to remember?
Straight line 2 years A line has 2 ends, so at 2 years
Circle 3 years Circle is easier to draw than triangle
Rectangle 4 years A rectangle has 4 vertices, so at 4 years

Triangle 5 years Difficult to draw than circle

Question 7 Diamond 6 years Most difficult out of all

When does the child first attain this


milestone?
C. PERSONAL SOCIAL
Milestones Attained in the 1st Year M

Age Important Milestones


1 month Looks at mother intently when talked to
2 months Social smileQ
3 months Recognizes mother
6 months Smiles at mirror image (‘mirror play’)Q
7 months Stranger anxietyQ
8 months Object permanenceQ
9 months Waves ‘bye’bye’Q
a. 6 months b. 7 months 10 months Plays peek a booQ
c. 9 months d. 12 months 12 months Kisses on request
Plays a simple ball game
Releases objects

Social Milestones Beyond 1 Year Age M

Mnemonic  M Age Important Milestones


15 months Indicates wet pants
At 15 months: 2 Ps:
Asks for objects by pointing
•• Indicates wet pants
18 months Domestic mimicryQ
•• Asks for objects by pointing
Dry by day
At 18 months: 2 Ds
24 months Parallel play
•• Domestic mimicry
Asks for food & drink
•• Dry by day
Contd...
Chapter 2: Development 25

Contd...

Section 1: General Pediatrics


36 months Knows gender and full name,Q Question 8
Joins in play & shares toys
A child can do the follo­ wing
Dry at night
activities. What is his approximate
4 years Questions age?
Goes to toilet alone
5 years Helps in household tasksQ
Distinguishes morning from evening
Compares 2 weights
Can follow 3 step commands

D. LANGUAGE A
Milestones Attained in the 1st Year M

Age Important Milestones


1 month Quietens when a bell is rung
2 months VocalizesQ
3 months Cooing
B
4 months Laughs aloud
a. 2 years b. 3 years
5 months Razzing
c. 4 years d. 5 years
6 months MonosyllablesQ (‘ma’ , ‘ba’)
9 months BisyllablesQ (‘Mama’ , ‘baba’) but without meaning
12 months 2 to 3 words with meaning
Imitates sound of dog, cow, clock

Language Milestones Beyond 1 Year Age M


Mnemonic  M

Age Important Milestones Between the ages of 2 and 5 year, the


number of words in a typical sen-
15 months Jargon speechQ
tence equals the child’s age (2 by age
Follows simple commands 2 yrs, 3 by age 3 yrs, and so on)
18 months Ten words with meaningQ
Identifies one or more parts of body
24 months Simple sentence with 2-3 words; vocabulary of 100 words
36 months Uses plurals & pass tense
Repeats 3 digits
4 years Sings song or tells poem or stories
5 years Asks meaning of words
Names 4 colors; repeats 4 digits

VISION
Age Important Milestones
Birth Regards a dangling object & follows it upto 45 degrees
4 weeks Follows a ring to 90 degreesQ
Fixates on mother when she talks
8-12 weeks Follows objects upto 180 degreesQ
3-4 months Fixates instantaneously on an object shown to him
Binocular vision established
6 months Visual acuity of an adult develops

HEARING
•• 3 to 4 months: Turns head towards source of sound
•• 5 to 6 months: Turns head to one side and then downward, if sound made below the level of ear
•• 7 months: Able to localise sound made above the level of ears
•• 10 months: Directly looks at source of sound diagonally
26 Section 1: General Pediatrics

 ABNORMALITIES OF DEVELOPMENT
Review of Pediatrics and Neonatology

Question 9
What is the sequence in which the Name of abnormality Description
following miles­tones appear?
Developmental delay Performance in 1 or more domains significantly below average
Dissociation Substantial difference in rate of development between 2 or more domains.
e.g. isolated speech delay
Deviancy Developmental milestones occurring out of sequence. e.g. crawling before
A B sitting

Important Causes of Developmental Delay


•• Chromosomal: Trisomies 21, 18, 13, Klinefelter syndrome
C D
•• Genetic: Fragile X syndrome, Prader-Willi syndrome, Rett syndrome
a. A → B → C → D •• Developmental brain abnormality: Hydrocephalus, meningomyelocele, lissencephaly
b. D → C → B → A •• Inborn errors of metabolism: Phenylketonuria, Tay-Sachs disease
c. A → D → B → C
•• Congenital infections: HIV, toxoplasmosis, rubella, CMV, syphilis, herpes simplex
d. C → D → B → A
•• Perinatal: HIE, cerebral palsy, Intraventricular hemorrhage, PVL, fetal alcohol syndrome
•• Postnatal: Trauma (abuse), meningitis, hypothyroidism

Red flag signs for important milestones:

Gross motor Fine motor


Sitting with support 9 monthsQ Pincer grasp 12 monthsQ
Standing with support 12 monthsQ Scribbling 24 months
Walking with support 15 months
Walking without support 18 months
Language Social
Vocalisation 6 months Social smile 6 monthsQ
Babbling 12 monthsQ Waving bye bye 12 months
Single words 16 monthsQ

Screening Tests for Developmental Assessment


•• Phatak's Baroda Screening test
•• Ages & stages questionnaire
•• Revised Denver Development screening test
•• Trivandrum development screening chart

Definitive Tests for Intellectual and Developmental Assessment


Name of the test Age group
Bayley scale for Infant development II 1 month – 3.5 yrs
Wechsler Intelligence scale for children IV 6 yrs – 17 yrs
Stanford Binet Intelligence scale, 5th ed 2 yrs – 85 yrs
Vineland adaptive behaviour scale II Birth to 89 yrs

 MENTAL RETARDATION/INTELLECTUAL DISABILITY (ID)


Sub-average general intelligence, manifesting during early developmental period.

High Yield Points Grades of Mental Handicap


Degree IQ level
Intelligence quotient (IQ)Q is mental
age divided by chromological age Mild ID 51-70
multiplied by 100 Moderate ID 36-50
Severe ID 21-35
Profound ID 0-20
Chapter 2: Development 27

Section 1: General Pediatrics


Answer Keys for Image-Based Questions

Answers Explanations / Identifying features


1. Ans. c. 3 months In Prone position, baby is lying on bed with high pelvis, knees drawn up under abdomen, head turned to
one side; this is seen at birth-2 weeks
2. Ans. b. 10-11 months The given picture shows pivotting, that comes around 10–11 months
3. Ans. c. D → C → A → B A → Walks with support (12 months)
B → Creeps upstairs (15 months)
C → Sits with support (6 months)
D → Feet to mouth (5 months)
So, the correct sequence is: D → C → A → B
4. Ans. d. 6 years The child shown in the picture is riding a tricycle, which is usually attained at 3 years age, in a normal child;
DQ of this child is 50, developmental age = 3 yrs
So, actual age is 6 years;
5. Ans. a. 12 weeks The given image shows ‘hand regard’ that appears around 12 weeks age
6. Ans. d. 18 months This picture shows a tower of 5 cubes
7. Ans. c. 9 months Immature or assisted pincer grasp is shown in the picture, as the child is holding small objects using his
thumb & index finger, but is taking help from other fingers also
8. Ans. d. 5 years A → Buttoning clothes (4 years)
B → Tying shoe laces (5 years)
9. Ans. c. A → D → B → C A → Mirror play (6 months)
B → Plays peek a boo (10 months)
C → Kisses on request (12 months)
D → Waves bye bye (9 months)
So, the sequence in which these milestones appear is: A → D → B → C
28 Section 1: General Pediatrics
Review of Pediatrics and Neonatology

Questions

 GROSS MOTOR 10. A 4-year-child can perform which one of the following?
a. Can hop on single leg for 15 feet (AIIMS Dec 08)
1. A child of 4 years can do all of the following except: M b. Can skip without falling to either side
a. Skip (FMGE June 2018) c. Can stand on one feet for 20 seconds
b. Goes down stairs one foot per step d. Can walk down the stairs with alternating steps holding
c. Tell a story d. Copies triangle on the iron rail
2. When can a child kick a ball? M  (Recent Question 2017)  FINE MOTOR
a. 12 months b. 18 months
c. 24 months d. 36 months 11. A child is able to copy a cross at what age?
3. Arrange the above milestones in ascending order of  (DNB June 2018)
appearance (starting from earliest to later milestones): a. 4 years b. 5 years
 (APPG 2016) c. 2 and half years d. 3 years
12. Child is able to make tower of 5–6 cubes at which age?
 (DNB June 2018)
a. 18 months b. 24 months
c. 21 months d. 30 months
A B 13. Absence of which of the given milestones in a 3-year-
old child should be called ‘delayed’ development?
 (AIIMS May 2018 & AIIMS Nov 2017)
a. Hopping on one leg b. Drawing square
c. Feeding by spoon d. Catching a ball reliably
14. Bidextrous grasp appears at: (NEET PG Jan 2019)
C D
a. 3 months b. 4 months
a. DACB b. DCAB c. 5 months d. 6 months
c. ACDB d. ADCB
15. The milestones of a 3-year-old child are considered
4. Parachute reflex appears at? M  (Recent Question 2016) delayed if he is unable to: (AIIMS Nov 2017)
a. 3 months b. 5 months a. Hop on 1 foot b. Use spoon effectively
c. 7 months d. 9 months c. Copy a square d. Reliably catch a ball
5. Child rolls over by: M  (Recent Question 2016) 16. Child starts making tower of 9 cubes by: M 
a. 3 months b. 6 months  (Recent Question 2016)
c. 7 months d. 8 months a. 18 months b. 24 months
6. Which of the following can be done by 15 months child? c. 30 months d. 36 months
a. Walks alone (PGI Nov 2015) 17. Which of the following milestones is developed in
b. Transfers object hand-to-hand child between 6 to 9 months of age? (PGI May 15)
c. Builds tower of 2 cubes
a. Can point something with index finger
d. Speaks 10–15 words
e. Speaks two-word sentences b. Swaps object from one palm to another
c. Can hold object with thumb and index finger
7. A 2-month-old child is most likely to:
d. Can voluntarily drop object
 (DNB Jun 2012, AIPGMEE 2009)
e. Can extend arm
a. Show a positive parachute protective reflex
b. Hold head steady in a seated position 18. A baby can draw a triangle at the age of: M 
c. Lift head and chest off a flat surface with extended elbows a. 3 years b. 5 years (WBPG 2014)
d. Sustain head level with the body when placed in ventral c. 6 years d. 7 years
suspension
19. A child draws circle at: M (DNB June 11)
8. An 8-week-infant can do all the following except:
a. 12 months b. 24 months
a. Head control  (PGI Dec 11)
c. 30 months d. 36 months
b. Lift its head up to horizontal line in ventral suspension
c. Follows red object up to 180 degrees 20. A child can make a 3 cube tower at the age of: M 
d. Social smile e. Turns head towards sound  (WBPG 2011)
9. A baby can follow an object till 180º, can hold neck, a. 36 months b. 16 months
can sit with support but cannot sit without support. The c. 20 months d. 24 months
age of the baby is: (PGI June 08)
21. Handedness develops by: M  (TN PGMEE 2010)
a. 1 month b. 3 months
c. 5 months d. 6 months a. 12 months b. 18 months
e. 9 months c. 24 months d. 36 months
Chapter 2: Development 29

22. At which of the following ages, a child can remove a 35. All of the following suggest a developmental delay in

Questions
front opening garment? (COMEDK 08) milestones except: (AIIMS May 2013)
a. 24 months b. 36 months a. Absence of pincer grasp at 9 months
c. 48 months d. 60 months b. Not climbing stairs up and down at 2 years
c. Not speaking 2 word phrases at 18 months
23. Children can reach for objects by: (JIPMER 2008)
d. Not able to sit at 9 months
a. 4 months b. 6 months
36. A child sits with support, identifies parts of body and
c. 8 months d. 10 months
builds tower of 4 cubes. Age is: (Recent Question 2013)
24. A 15-month-old child is able to do all the following a. 1–2 yrs b. 2 yrs
except: M  (JIPMER 2007) c. 2–3 yrs d. 4 yrs
a. Crawl upstairs b. Make a tower of 5 cubes 37. Pincer grasp develops at what age? M 
c. Name familiar objects d. Point to familiar things  (Recent Question 2013)
25. A normally developing 10-month-old child should be able a. 5–8 months b. 9–11 months
to do all of the following except: M  (AIPGMEE 06) c. 12–15 months d. 15–18 months
a. Stand alone b. Play peek a boo 38. A child is able to build blocks of 4 cubes. His deve­lop­
c. Pick up a pellet with thumb and index finger mental age is: (Recent Question 2013, DNB Jun 2012)
d. Build a tower of 3–4 cubes a. 12 months b. 15 months
c. 18 months d. 30 months
 SOCIAL MILESTONES
39. Pincer grasp is attained at ...... months:
 (Recent Question 2013)
26. Child knows his sex by what age? M 
a. 4 b. 10
 (Recent Question 2017)
c. 12 d. 18
a. 12 months b. 24 months
c. 36 months d. 48 months 40. At what age a child can eat food without spilling,
identify her gender, say her full name, can dress self with
27. The developmental implication of developmental supervision?
miles­tone uncovers toys after seeing it hidden is: a. 2 yrs b. 3 yrs (AIIMS Nov 2012)
a. Symbolic thought (Recent Question 2017) c. 4 yrs d. 5 yrs
b. Beginning of labelling c. Object permanence 41. A child plays a simple ball game at: M  (DNB Jun 2010)
d. Self-discovery e. Cause and effect
a. 52 weeks b. 36 weeks
28. Range of IQ in mild mental retardation is? c. 12 weeks d. 48 weeks
 (Recent Question 2015) 42. Which one of the following activities cannot be
a. < 50 b. 50–70 performed by a 7 month old infant? (DNB Dec 2010)
c. 70–100 d. >100 a. Pivot b. Cruise
29. Which of the following cannot be done by a 3 years old c. Transfer objects d. Enjoy mirror
child? M  (Recent Question 2015) 43. Handedness gets established at what age? M
 (DNB June 2010)
a. Draw a triangle b. Draw a circle
c. Can arrange 9 cubes a. 12 months b. 24 months
d. Can go up and down the stairs c. 36 months d. 48 months
30. A child can ride a tricycle, copy a circle and knows his 44. A 2-year-old child can do: M  (PGI Dec 2009)
age and sex by the age of: M  (Recent Question 2015) a. Can ride tricycle
a. 30 months b. 42 months b. Climb up and downstairs with one step each time
c. 36 months d. 48 months c. Knows sex and age
d. Handles spoon well e. Can read story with picture
31. A child draws triangle at what age? M 
 (Recent Question 2014, Recent Question 2013) 45. A 40-week-old infant can do all the following except:
a. 3 years b. 4 years a. Waves bye bye (JIPMER 2009)
b. Transfer object from one hand to another
c. 5 years d. 7 years
c. Sits without support
32. An 18 months baby can do? M  (Recent Question 2014) d. Makes a tower of 3–4 cubes
a. Hide and seek game b. Write alphabet 46. An infant can sit with leaning forward on his hands. He
c. Say short sentence d. Run bounces actively when made to stand. He laughs aloud
33. Red flag sign in child development, if not attained? and becomes concerned when the mother moves away.
 (Recent Question 2014) What is his most likely age? (DNB Jun 2009)
a. Vocalization at 2 months b. Walking at 12 months a. 12 weeks b. 16 weeks
c. Single word at 12 months d. Social smile at 6 months c. 22 weeks d. 28 weeks
34. Object permanence in a child is seen after: 47. A female child has recently learned to eat with spoon
 (JIPMER 2014) without spilling, to dress and undress herself with super­
a. 6 months b. 9 months vision and to understand that she is a girl. These skills are
c. 12 months d. 15 months first mastered between the ages of: (AIIMS Nov 2008)
30 Section 1: General Pediatrics

a. 2 and 3 years b. 3 and 4 years


Review of Pediatrics and Neonatology

60. In development of language, a child is able to tell


c. 4 and 5 years d. 5 and 6 years stories by: M  (MAHA PGM CET 2015)
48. A boy can grasp a rattle and recently he is able to
a. 1 year b. 2 years
transfer objects from hand to hand. He should also be
c. 3 years d. 4 years
able to:
a. Babble b. Say ‘mama' or ‘dada' 61. A 4-year-old child cannot do which of the following?
c. Sit with support d. Stand with support  (APPG 2014)
e. Able to walk (PGI Dec 2008) a. Count 10 pennies b. Hop on one foot
49. A boy draws triangle but not diamond shape; what is his c. Copy a bridge
age? M  (APPG 2008) d. Identify the longer of two lines
a. 3 years b. 4 years 62. Vocabulary of a 2-year-old child is: M 
c. 5 years d. 6 years  (Recent Question 2014)
50. Child changes a rattle from one hand to another at the a. 20 words b. 30 words
age of: M  (COMEDK 2007) c. 100 words d. 200 words
a. 3 months b. 6 months 63. A child is able to say sentence of 6 words at: M 
c. 7 months d. 1 year  (Recent Question 2013)
51. A child has started mouthing objects, shows likes and a. 2 years b. 3 years
dislikes, but has not yet developed stranger anxiety. c. 4 years d. 5 years
The age of child is: M  (AIPGMEE 2007)
64. Child starts speaking monosyllables at what age: M 
a. 3 months b. 5 months  (DNB Dec 2012)
c. 7 months d. 9 months
a. 4 months b. 6 months
52. Which of the following appears 1st in child? M c. 8 months d. 10 months
(AIPGMEE 2007) 65. A normal child develops the ability to use 10 words
a. Creeping b. Crawling with meaning at the age of: (COMEDK 2011)
c. Mirror play d. Pincer grasp a. 12 months b. 15 months
53. Child knows his/her sex at the age of: M  c. 18 months d. 24 months
 (TN PGMEE 2006) 66. A child climbs with alternate steps, builds a tower of
a. 1 year b. 3 years 8–9 cubes, tells ‘I’ but not his name and cannot say his
c. 5 years d. 7 years age and sex, the probable age is: (AIIMS May 2011)
54. A 3 months baby can do: (PGI June 2006) a. 36 months b. 24 months
a. Social smile b. Can sit without support c. 30 months d. 48 months
c. Transfer objects from right to left 67. A 3-year-old child is expected to have all except: M 
d. Hold his neck
 (DNB June 2010)
e. Can change position from prone to supine
a. Speak in sentences b. Copy a circle
55. Child copies a circle at the age of? M  c. Hobble 5 steps
 (TN PGMEE 2005)
d. Climbs upstairs and downstairs
a. 1 year b. 2 years
68. A 3-year-old child can do all, EXCEPT: M
c. 3 years d. 4 years
 (AIIMS May 2010)
 LANGUAGE MILESTONES a. Eat with a spoon b. Climb stairs running
c. Walk properly
56. Which of the following milestones is not developed at d. Vocabulary of 20–100 words
1 year? M  (Recent Question 2017) 69. A 6-year-old child has a DQ of 50. Which of the
a. Stand without support b. Plays peek a boo following can the child do?
c. Can speak “mama” and “papa”  (AIIMS May 2007, AIPGMEE 2007)
d. Draws a circle a. Identify colors b. Read a sentence
57. A normal child has dysfluency of speech between: c. Ride a bicycle d. Copy a triangle
 (Recent Question 2017)
70. A 10-month-old child cannot do: (PGI Dec 2005)
a. 2 and 4 years b. 4 and 6 years
a. Change cube from one hand to another
c. 6 and 8 years d. 8 and 10 years
b. Can build a tower of six cubes
58. Behavioral observation audiometry is used as a scree­ c. Can pull to stand
ning test for hearing in age group: d. Can speak a sentence of 4 to 5 words
 (Recent Question 2017) e. Pincer grasp
a. <6 months b. 6–30 months
71. An 18-month-old infant can do all except:
c. 30–60 months d. >60 months
a. Climbing upstairs (PGI Dec 03)
59. In ‘Milestones of Development’ ‘Listening’ refers to b. Can follow mother's activities
which type of development? (MAHA PGM CET 2015) c. Can turn 2–3 pages at a time
a. Motor b. Language d. Can say 2–3 words
c. Adaptive d. Socio-personal e. Can make tower of 8 cubes
Chapter 2: Development 31

Answers with Explanations


Answers with Explanations
 GROSS MOTOR
1. d. Copies triangle  Ref: Nelson’s 20/e p 75
Copying a triangle comes at 5 years.
2. c. 24 months  Ref: Illingworth’s normal and abnormal child
A child can run and kick a ball at 2 years age and can go upstairs and downstairs 2 feet/step.
3. a. DACB  Ref: Nelson’s 20/e p 65-77, Ghai 8/e p 45–49
A shows mouthing that comes at 5 months.
B shows spontaneous scribbling, that appears at 15 months.
C shows Pivotting that appears around 7 months.
D shows rounded back on made to sit, that is seen at 3 months.
So, the correct sequence of the given milestones is D → A → C → B.
4. d. 9 months  Ref: Nelson’s 20/e p 65-77
5. b. 6 months  Ref: Nelson’s 20/e p 66
Once the Asymmetric Tonic Neck reflex (ATNR) disappears, baby learns to roll over from prone to supine and vice versa.
6. a. Walks alone; b. Transfers object hand-to-hand; c. Builds tower of 2 cubes  Ref: Nelson’s 20/e p 66
A child walks without support 13 months onwards, transfers objects at 7 months, builds a tower of 2 cubes at 15 months,
speaks 10 words with meaning at 18 months, while speaks 2 word sentences at 2 years.
7. d. Sustain head level with the body when placed in ventral suspension  Ref: Nelson’s 20/e p 65-77, Ghai 8/e p 45-49
Discussing about the options one by one:
Milestone Age at which attained
a. Show a positive parachute protective reflex 9–10 months
b. Hold head steady in a seated position 4 months
c. Lift head and chest off a flat surface with extended elbows 3 months
d. Sustain head level with the body when placed in ventral suspension 2 months

8. a. Head control; e. Turns head towards sound  Ref: Nelson’s 20/e p 65-77, Ghai 8/e p 45-49
Milestone Age at which attained
a. Head control 3 months
b. Lift its head up to horizontal line in ventral suspension 2 months
c. Follows red object up to 180 degrees 2 months
d. Social smile 2 months

e. Turns head towards sound 4 months

9. d. 6 months  Ref: Nelson’s 20/e p 65-77, Ghai 8/e p 45-49


Following an object up to 180° comes at 2 months, Neck holding comes at 3 months, Sitting with support is attained at 6 months;
So, the age of this baby is 6 months.
10. a. Can hop on single leg for 15 feet; d. Can walk down the stairs with alternating steps holding on the iron rail

Ref: Nelson’s 20/e p 65-77, Ghai 8/e p 45-49
Milestone Age at which attained
a. Can hop on single leg for 15 feet 4 years
b. Can skip without falling to either side 5 years
c. Can stand one feet for 20 seconds 5 years
d. Can walk down the stairs with alternating steps holding on the iron rail 4 years

Note: (d) is a better answer than (c), because child hops on a single leg at 4 years, but hopping on 1 leg for a distance as long as 15
feet, may not appear at 4 years.
32 Section 1: General Pediatrics

 FINE MOTOR
Review of Pediatrics and Neonatology

11. a. 4 years  Ref: Nelson’s 20/e p 75


Copying a cross or rectangle comes at 4 years age.
12. c. 21 months  Ref: Nelson’s 20/e p 75
• Tower of 2 cubes: 15 months •  Makes “bridge” of 3 cubes: 3 years
•• Tower of 4 cubes: 18 months •  Makes “gate” of 5 cubes: 4 years
•• Tower of 6–7 cubes: 24 months
13. c. Feeding by spoon  Ref: Nelson’s 20/e p 75
•• Hopping on one leg, drawing a square & catching a ball reliably appear at 4 years.
•• Feeding by spoon comes by 18 months, so its absence in a 3 year child is considered abnormal.
14. c. 5 months  Ref: Illingworth’s ‘The development of young child & Infant’ p 278
•• 16 weeks: Tries to reach cube, but overshoots and misses.
•• 20 weeks: Able to grasp voluntarily. Bidextrous approach
•• 24 weeks: More mature grasp. Drops one cube when another is given
•• 28 weeks: Unidextrous approach. Bangs cube on table. Transfers. Retains one when another is given.
15. b. Use spoon effectively  Ref: Nelson’s 20/e p 75
Hop on 1 foot, copying a square & catching a ball reliably, comes at 48 m (4 yrs)
A child is able to feed self at 18 months, but handles spoon effectively at 24 months (2 yrs)
16. d. 36 months  Ref: Nelson’s p 65-77, Ghai 8/e p 49-53; Refer Ans 14 below
17. b. Swaps object from one palm to another; c. Can hold object with thumb and index finger  Ref: Nelson’s 20/e p 65-77
Milestone Age at which attained
a. Can point something with index finger 15 months
b. Swaps object from one palm to another 7 months
c. Can hold object with thumb and index finger 9 months
d. Can voluntarily drop object 12 months
e. Can extend arm (helps to undress) 2 years

18. b. 5 years  Ref: Nelson’s 20/e p 65-77, Ghai 8/e p 49-53
19. d. 36 months  Ref: Nelson’s p 65-77, Ghai 8/e p 49-53
20. b. 16 months  Ref: Nelson’s p 65-77, Ghai 8/e p 49-53
Milestones based on Tower of cubes:
Age 15 months 18 months 2 years 3 years
Number of cubes in tower 2 4 6–7 9–10

21. d. 36 months  Ref: Nelson’s 20/e p 65-77, Ghai 8/e p 49-53
Handedness appears at 24 months, but is usually established by or developed by the 3rd year (36 months).
22. c. 48 months  Ref: Nelson’s 20/e p 65-77, Ghai 8/e p 49-53
A child can button and unbutton at 4 years age.
23. a. 4 months  Ref: Nelson’s 20/e p 65-77, Ghai 8/e p 49-53
At 4 months a child tries to reach for an object but overshoots.
24. b. Make a tower of 5 cubes  Ref: Nelson’s 20/e p 65-77, Ghai 8/e p 49-53
25. d. Build a tower of 3–4 cubes  Ref: Nelson’s 20/e p 65-77, Ghai 8/e p 49-53

 SOCIAL MILESTONES
26. c. 36 months  Ref: Nelson’s 20/e p 75
A child knows his sex by 36 months.
27. c. Object permanence  Ref: Nelson’s 20/e p 66
Uncovers toy (after seeing Pretend play with doll (e.g.,
Milestone Stares at own hand
it hidden) gives doll bottle)
Developmental implication Self-discovery Object permanence Symbolic thought

Chapter 2: Development 33

28. b. 50–70  Ref: Nelson’s 20/e p 217, Ghai 8/e p 52-54

Answers with Explanations


Mild intellectual disability (ID) is IQ between 50-70, while severe ID is IQ <50
29. a. Draw a triangle  Ref: Nelson’s 20/e p 65-77, Ghai 8/e p 52-54
30. c. 36 months  Ref: Nelson’s 20/e p 65-77, Ghai 8/e p 52-54
31. c. 5 years  Ref: Nelson’s 20/e p 65-77, Ghai 8/e p 52-54
32. a. Hide and seek game  Ref: Nelson’s 20/e p 65-77, Ghai 8/e p 52-54
Hide and seek game or peek a boo comes at around 10 months age.
33. d. Social smile at 6 months  Ref: Nelson’s 20/e p 65-77, Ghai 8/e p 52-54
For red flag signs for some important milestones refer pretext of this chapter.
34. b. 9 months  Ref: Nelson’s 20/e p 65-77, Ghai 8/e p 52-54
Object permanence is the understanding that objects continue to exist, even when not seen and appears at 9 months.
35. a. Absence of pincer grasp at 9 months  Ref: Nelson’s 20/e p 65-77, Ghai 8/e p 52-54; Refer ans 31 above
36. a. 1–2 yrs  Ref: Nelson’s 20/e p 65-77, Ghai 8/e p 52-54
Sitting with support comes at 6 months, identify 1 or more parts of body at 18 months and builds tower of 4 cubes at 18
months. So the age of the child is between 1 and 2 years.
37. b. 9–11 months  Ref: Nelson’s 20/e p 65-77, Ghai 8/e p 52-54
Immature pincer grasp usually develops by 9 months and mature pincer grasp by 12 months.
38. c. 18 months  Ref: Nelson’s 20/e p 65-77, Ghai 8/e p 52-54; Refer Ans 19 above
39. b. 10  Ref: Nelson’s 20/e p 65-77, Ghai 8/e p 52-54
As 9 months is not given in the options, b. 10 months is the best answer.
40. b. 3 yrs  Ref: Nelson’s 20/e p 65-77, Ghai 8/e p 52-54
Eating food without spilling comes at 2 years, identifying her gender and saying her full name and dressing self with supervision
comes at 3 years.
41. a. 52 weeks  Ref: Nelson’s 20/e p 65-77, Ghai 8/e p 52-54
Playing a simple ball game comes at 1 year or 52 weeks.
42. b. Cruise  Ref: Nelson’s 20/e p 65-77, Ghai 8/e p 52-54
Cruising or walking holding on to furniture comes at around 10–12 months.
43. c. 36 months  Ref: Nelson’s 20/e p 65-77, Ghai 8/e p 52-54
44. b. Climb up and downstairs with one step each time; d. Handles spoon well  Ref: Nelson’s 20/e p 65-77
Discussing about the options one by one.
Milestone Age at which attained
a. Can ride tricycle 3 years
b. Climb up and downstairs with one step each time (means with 2 feet per step) 2 years
c. Knows sex and age 3 years
d. Handles spoon well 2 years
e. Can read story with picture 4–5 years

45. d. Makes a tower of 3–4 cubes  Ref: Nelson’s 20/e p 65-77, Ghai 8/e p 52-54
40 week old infant is around 10 months old; Tower of 3–4 cubes comes by around 18 months age.
46. d. 28 weeks  Ref: Nelson’s 20/e p 65-77, Ghai 8/e p 52-54
•• An infant can sit with leaning forward on his hands (sitting with support) at around 5–6 months
•• He bounces actively when made to stand at 7 months, laughs aloud at 4 months and becomes concerned when the mother
moves away at around 7 months So, his most likely age is 7 months (28 weeks).
47. a. 2 and 3 years  Ref: Nelson’s 20/e p 65-77, Ghai 8/e p 52-54
48. a. Babble; c. Sit with support  Ref: Nelson’s 20/e p 65-77, Ghai 8/e p 52-54
The boy is able to transfer objects. So, his age is around 7 months. At 7 months he can also babble and sit with support.
49. c. 5 years  Ref: Nelson’s 20/e p 65-77, Ghai 8/e p 52-54
50. c. 7 months  Ref: Nelson’s 20/e p 65-77, Ghai 8/e p 52-54
51. b. 5 months  Ref: Nelson’s 20/e p 65-77, Ghai 8/e p 52-54
Stranger anxiety, that develops at around 7 months has not yet appeared. So, the age of the child is 5 months.
34 Section 1: General Pediatrics

52. c. Mirror play  Ref: Nelson’s 20/e p 65-77, Ghai 8/e p 52-54
Review of Pediatrics and Neonatology

Age of appearance of the given milestones is: Creeping: 11 months, Crawling: 10 months, Mirror play: 6 months and Pincer grasp: 9
months; So the 1st to appear out of these is mirror play.
53. b. 3 years  Ref: Nelson’s 20/e p 65-77, Ghai 8/e p 52-54
54. a. Social smile; d. Hold his neck  Ref: Nelson’s 20/e p 65-77, Ghai 8/e p 52–54
Sitting without support appears at 8 months, transfers objects at 7 months and prone to supine at around 6–7 months.
55. c. 3 years  Ref: Nelson’s 20/e p 65-77, Ghai 8/e p 52-54

 LANGUAGE MILESTONES
56. d. Draws a circle.  Ref: Nelson’s 20/e p 67, Ghai 8/e p 52–53
A child is able to draw a circle at around 3 years age.
57. Ans a. 2 and 4 years  Ref: Nelson’s 20/e p 65-77, Ghai 8/e p 52–53
Language development occurs most rapidly between 2 and 5 years of age. The period of rapid language acquisition is also when
developmental dysfluency and stuttering are most likely to emerge

58. a. <6 months   Ref: Nelson’s 20/e p 3078
Behavioral observation Visual reinforcement
Type of audiometry Play audiometry
audiometry audiometry
Used in age group <5 months 6–30 months 30 months to 5 years

59. b. Language  Ref: Nelson’s 20/e p 65-77, Ghai 8/e p 52-53
60. d. 4 years  Ref: Nelson’s 20/e p 65-77, Ghai 8/e p 52-53
Telling a story/poem or singing a song comes at 4 years age.
61. a. Count 10 pennies  Ref: Nelson’s 20/e p 65-77, Ghai 8/e p 52-53
A child is able to count 10 pennies at the age of 5 years.
Important milestones attained at 4 year age:

Gross motor Fine motor Social Language


•• Hops on 1 foot •• Copies bridge from a model •• Plays with several children •• Counts 4 pennies accurately
•• Throws ball overhand •• Imitates“gate” of 5 cubes •• Role-playing •• Tells a story or a poem
•• Climbs well •• Copies cross and square •• Goes to toilet alone •• Sings a song
•• Draws man with 2-4 parts
•• Identifies longer of 2 lines
•• Uses scissors to cut out pictures

62. c. 100 words  Ref: Nelson’s 20/e p 65-77, Ghai 8/e p 52-53
Vocabulary at 18 months is 10-15 words, while at 2 years, it is 50–100 words.
63. d. 5 years  Ref: Nelson’s 20/e p 65-77, Ghai 8/e p 52-53

Mnemonic
Between ages of 2 and 5 years, number of words in a typical sentence equals the child’s age (e.g. 2 by age 2 year, 3 by age 3 year);
So, a child speaks 5 word sentences at 5 years; So, out of the given options 5 years is the best answer

64. b. 6 months  Ref: Nelson’s 20/e p 65-77, Ghai 8/e p 52-53
65. c. 18 months  Ref: Nelson’s 20/e p 65-77, Ghai 8/e p 52-53
66. c. 30 months  Ref: Nelson’s 20/e p 65-77, Ghai 8/e p 52-53
A child climbs with alternate steps at around 30 months, builds a tower 8–9 cubes between 2–3 years, tells ‘I’ but not his name at
2 years and tells his age and sex at 3 years. So, the probable age of this child is a little less than 3 years. Hence, among the given
options, 30 months is the best answer.
67. c. Hobble 5 steps  Ref: Nelson’s 20/e p 65-77, Ghai 8/e p 52-53
68. b. Climb stairs running  Ref: Nelson’s 20/e p 65-77, Ghai 8/e p 52-53
69. a. Identify colors  Ref: Nelson’s 20/e p 65-77, Ghai 8/e p 52-53
DQ = Developmental age x 100/ actual age.
Chapter 2: Development 35

In the given question, a 6 year old child has a DQ of 50; so his developmental age is 3 years.

Answers with Explanations


So, he can do milestones corresponding to 3 years age only; At 3 years, a child can name 2 colors and at 5 years, 4 colors.
70. b. Can build a tower of six cubes; d. Can speak a sentence of 4 to 5 words  Ref: Nelson’s 20/e p 65-77
Building a tower of 6 cubes comes around 2 years age and talking a sentence of 4–5 words comes at 4–5 years. Rest of
the given milestones appear within or at the age of 10 months.
71. e. Can make tower of 8 cubes  Ref: Nelson’s 20/e p 65-77, Ghai 8/e p 52-53
Discussing about the options one by one:

Milestone Time of appearance


a. Climbing upstairs A child can climb upstairs holding the rail at 18 months
b. Can follows mother’s activities This is domestic mimicry, that appears at 18 months
c. Can turn 2–3 pages at a time 18 months
d. Can say 2–3 words At 1 year, a child can speak 2–3 words with meaning
e. Can make tower of 8 cubes Between 2 and 3 years age

Chapter 3
Nutrition and
Malnutrition
 BREAST MILK & BREASTFEEDING M
Mnemonic  M
Complimentary feeding should be •• Exclusive breastfeeding means child should be given only breast milk, not even sips of
‘AFASS’ water, unless medically indicated
•• Acceptable •• Colostrum is beneficial for the baby & should be given
•• Feasible •• Any form of pre-lacteal feeding is strictly contraindicated
•• Affordable
•• Sustainable •• Complimentary feeding refers to semisolid food given in addition to breast milk.
•• Safe
Signs of Good Positioning while Breastfeeding M

•• Body of baby should be well supported


•• Baby should be turned towards the mother
•• Occiput, shoulder & buttocks of baby should be in a straight line
•• Abdomen of baby should touch the abdomen of mother.
High Yield Points
Signs of Good Attachment while Breastfeeding M
•• Breastfeeding should be initi­ated:
–– As soon as possibleQ after birth •• Mouth of baby should be wide open
(best answer)
•• Whole of areola should be in baby's mouth except a small part of upper areola which may
–– Within 1 hour of birth (Ghai 9th
Ed) be visible
•• Exclusive breastfeeding is to be •• Lower lip of baby should be everted
continued for 1st 6 months of lifeQ •• Chin of the baby should touch mother’s breast.
•• At 6 months, complimentaryQ
feeding is started Reflexes Helping in Breastfeeding M

Seen in Name of reflex Description


Mother Milk secretion reflex Baby sucks nipple → Nerves carry impulse to anterior pituitary →
Prolactin released → More breast milk production
Milk ejection reflex Oxytocin stimulates the muscles surrounding the breast to
squeeze out the milk

Baby Rooting reflex Helps to locate & attach to breast while breastfeeding
Suckling reflex Helps in breastfeeding

High Yield Points


Benefits of Breastfeeding for the Baby M

•• Maximum breast milk output


occurs at 5–6 months of lactationQ
1. Breast milk (BM) provides all the nutrients a baby needs during first 6 months of life.
(~730 mL/day) a. Carbohydrates: BM contains more lactoseQ compared to cow's milk (CM)
•• Expressed breast milk can be b. Proteins:
stored for: -- BM contains lesser proteinQ as compared to CM → lesser solute load on kidneys
–– 8-10 hoursQ at room tempe- -- BM contains adequate cysteine, taurine → help in brain development
rature
-- BM is richer in whey proteins (lactalbumin), which are easily digestible as compared
–– 24 hoursQ in a refrigerator
–– 3 months in a deep freezer at to CM, which is richer in casein.
–20°C c. Lipids:
•• To promote exclusive breastfeeding, -- Breast milk is richer in PUFA (polyunsaturated fatty acids)
the Baby Friendly Hospital Ini­ -- It contains adequate amounts of DHAQ (Docosa Hexaenoic Acid) → helps in brain
tiative (BFHI) was launched in 1992 & eye development
-- It is more easily digestible due to presence of lipase in breast milk.
Chapter 3: Nutrition and Malnutrition 37

d. Minerals:

Section 1: General Pediatrics


Question 1
Ca:P ratio in breast milk favors calcium absorption
Fe present in breast milk is more easily absorbable than in cow's milk The given picture indicates which
of the following?
e. Vitamins: Breast milk contains adequate amounts of all vitamins except vitamin D, K
(& B12 in strictly vegan mothers).
2. Breastfed babies have been shown to have higher intelligence quotient (IQ)
3. Breastfed babies have less caries teeth, diabetes, obesity, hypertension
4. Breast milk contains anti-infective substances that protect against infection & NEC
5. Breast milk promotes close bonding between mother and her baby.

Benefits of Breastfeeding for Mother


1. During breastfeeding, oxytocin released promotes involution of uterus
2. Breastfeeding delays ovulation and provides a natural means of contraception
3. It helps to regain & maintain pre-pregnancy body weight
4. Mother who breastfeed have a reduced risk of breast & ovarian cancer
5. Reduces risk of osteoporosis & economic saving to the family.

Differences between Human and Cow’s Milk


Nutrients Human milk (per 100 mL) Cow’s milk (per 100 mL)
a. Good positioning
Protein & calories 1.1 gmQ & 65 kcal 3.5 gmQ & 67 kcal b. Rooting reflex
Casein 30–40%; 80%; Rich in casein, c. Good attachment
High Lactalbumin & whey protein responsible for milk allergy d. Weaning
& colic
Lactose 7.4 gm 4.5 gm
Fat 3.5 gm; Richer in PUFA 4.0 gm
Na & K (mEq/L) 6.5 & 14 25 (more) & 35 (more)
Osmolality (mOsm/L) 290 350 (more) Mnemonic  M
Calcium/Phosphorus ratio >2 <2 Anti-infective Substances Pre­sent in
Breast Milk “Teach for”: PLAB
Vitamin K 1.5 µg 6.0 µg
•• TGFb
Iron 0.29 – 0.45 mg 0.01 – 0.38 mg •• Phagocytic macrophages
Vitamin D 0.5 – 10 I.U. 0.5 – 4.5 I.U. •• PABA (Para amino benzoic acid)
•• Lactoferrin
•• Lysosyme
Variations in Composition of Breast Milk M
•• Antibodies esp IgA
A. Based on the time after birth: •• Anti-staphylococcal factor
•• Bifidus factor
Name of milk Characteristics
Colostrum Golden thick yellow milk secreted during the first 3 days after delivery.
It contains more antibodies (IgA), WBCs & protein. It is ideal first meal of the baby.
Transitional milk Composition is in between that of colostrum & mature milk
Mature milk Thin & watery; contains less proteins
Mnemonic  M
Preterm Breast Milk is Richer in: “SIP
B. Based on gestational age: for Intelligent CNS”
Preterm milk contains more calories, fat, protein, immunoglobulins, iron and sodium •• Sodium
which are needed by preterm baby. •• Iron
•• Protein
C. Based on each feed:
•• Fat
Characteristics Fore milk Hind milk •• Immunoglobulins
•• Calories
What is it? Milk released at beginning of feed Milk released towards the end of feed
Richer in Water, vitamins, minerals, proteins Fat
Satisfies Thirst of baby Hunger of baby
High Yield Points
Contraindications to Breastfeeding M
Colostrum is the ‘first immunization’ of
Related to baby: Galactosemia, Lactose intolerance the baby
38 Section 1: General Pediatrics

Related to mother:
Review of Pediatrics and Neonatology

High Yield Points


Absolute Relative contraindications to breastfeeding
•• Human milk contains 30 times Mother on Condition Comment
more DHA than cow’s milk Chemotherapy Maternal HIV
•• Highest concentration of secretory Contraindicated in developed countries;
or Radiotherapy In developing countries, mixed feeding is contraindicated
IgA in any body fluid are found in co-
lostrum Active Breastfeeding is contraindicated until completion of at least
•• Breast milk contains adequate untreated TB 2 weeks of antitubercular therapy
amounts of all vitamins except vi-
Varicella- Infant should not have direct contact to active lesions; Infant
tamin D, K (& B12 in strictly vegan
zoster should receive immune globulin
mothers)
Herpes simplex Breastfeeding is contraindicated with active herpetic lesions
infection of the breast

 NUTRITION
Recommended Daily Energy Requirement
Body weight Recommended daily energy requirement
≤ 10 kg 100 kcal/kg
10 – 20 kg 1000 kcal + 50 kcal/kg for each kg above 10 kg
Question 2 > 20 kg 1500 kcal + 20 kcal/kg for each kg above 20 kg
The benefits of the tech­
nique
shown below are all of the Recommended Daily Protein Requirement
following except:
Group Age Protein req (g/day)
Infants 0–6 months 1.2 g/kg/day
6–12 months 1.7 g/kg/day
Children 1–3 years 17
4–6 years 20
7–9 years 30
10–12 years 40
Boys 13–15 years 54
Girls 13–15 years 52
Boys 16–17 years 62
a. Protects from neonatal jaundice
b. Protects from hypothermia Girls 16–17 years 56
c. Decreases duration of hospital stay
d. Promotes exclusive breastfeeding  MALNUTRITION
Three cardinal determinants that account for more than 90% of childhood undernutrition are: Low
birth weight, Infections (including diarrhea & pneumonia) and low food intake.

High Yield Points


•• Best indicator of acute malnutrition is wastingQ (decrease in weight for height)
•• Best indicator of chronic malnutrition is stuntingQ (decrease in height for age)
•• Common to both acute & chromic malnutrition is weight for age
•• WHO growth charts are best charts for under 5 children followed throughout the world

Important Classifications of Malnutrition M

A. WHO classification: Based on Weight for height, Height for age & presence of edema.
Moderate malnutrition Severe malnutrition
Weight for height Between 70–79% of expected or < 70% of expected or
SD score between -2 to -3 SD score <-3
(WASTING) (SEVERE WASTING)
Height for age Between 85–89% of expected or < 85% of expected or
SD score between -2 to -3 SD score <-3
(STUNTING) (SEVERE STUNTING)
In presence of symmetrical edema, it is called ‘edematous’ malnutrition.
Chapter 3: Nutrition and Malnutrition 39

B. IAP classification: Based on weight for age & edema.

Section 1: General Pediatrics


Question 3
Grade Weight for age (% of expected)
What is the disease this child is
Normal > 80 suffering from?
Grade I 71 – 80 (Mild malnutrition)
Grade II 61 – 70 (Moderate malnutrition)
Grade III 51 – 60 (Severe malnutrition)
Grade IV <50 (Very severe malnutrition)
Add the alphabet ‘K’ in front of the grade, if symmetric bipedal edema present.
C. Gomez classification: Based on weight for age.
Classification % of expected
Normal >90
Mild malnutrition 75 – 90
Moderate malnutrition 60 – 74
Severe malnutrition <60

D. Wellcome trust classification: Based on weight for age and edema. a. Kwashiorkor
Weight for age Edema Category b. Marasmus
c. Osteogenesis imperfecta
60%–80% of expected Absent Under-nutrition d. Progeria
60%–80% of expected Present Kwashiorkor
< 60% of expected Absent Marasmus
< 60% of expected Present Marasmic Kwashiorkor

Difference between Kwashiorkor and Marasmus M

Clinical finding Kwashiorkor Marasmus


Edema Present Absent
Activity Apathetic, lethargic Active
Question 4
Appetite Poor Good, voracious
What is being measured in this
Liver enlargementQ Present Absent
picture?
Mortality Less than
High in early stage
Kwashiorkor
Recovery Takes long time
Recover early
to recover
Skin & hair changes More commonQ Less common

Age Independent Anthropometric Indices: See chapter 1 pretext

 SEVERE ACUTE MALNUTRITION (SAM) a. Skin fold thickness


b. Body mass index
Definition
c. Mid arm circumference
In children who are 6–59 months of age, severe acute malnutrition is defined as: d. Kanawati index
•• Weight/height or Weight/length < -3 Z score, using the WHO Growth Charts; OR
•• Presence of bipedal edema of nutri­tional origin; OR
•• Mid-upper arm circumference (MUAC) < 115 mm

Changes in immune system in malnutrition:


•• In PEM, Humoral immunity is relatively spared
•• Number & proportion of B-lymphocytes are not altered
•• Levels of all immunoglobulins are normal or increased except IgA. High Yield Points

Indications for Hospitalization M •• Important skin changes seen in


malnutrition are ‘Flaky paint derma­
Children who have medical complications, severe edema (+++), or poor appetite (fail the tosis’ or ‘crazy pavement dermatosis’
appetite test), or present with 1 or more Integrated Management of Childhood Illness (IMCI) •• Important hair changes of malnu­trition
danger signs should be treated as in-patients. are ‘flag sign’ & easy pluckability
40 Section 1: General Pediatrics

Management of Malnutrition
Review of Pediatrics and Neonatology

M
Question 5
Identify this skin lesion in a child •• Triage must be done for all pediatric patients coming to the health facility
with severe malnutri­tion. •• Check ABCD steps: Airway, Breathing, Circulation, Coma, Convulsion, and Dehydration
•• 3 phases: Stabilization Phase, Transition Phase & Rehabilitative Phase

10 Steps for Management of SAM M

Management steps Stabilization Transition Rehabilitation


Day 1–2 Day 3–7 Day 7–14 Week 2–6
 1. Treat/prevent hypoglycemia
 2. Treat/prevent hypothermia
 3. Treat/prevent dehydration
  4.  Correct imbalance of electrolytes
 5. Treat infections
a. Xeroderma   6.  Correct deficiencies of micronutrients No iron With iron
b. Flaky paint dermatosis  7. Start cautious feeding
c. Ichthyosis
d. Flag sign   8. Rebuild wasted tissues (catch-up
growth)
 9. Provide loving care and play
10.  Prepare for follow-up

Management of complications of severe malnutrition


Complication Treatment
Mnemonic  M Hypoglycemia Immediately give a 50 mL bolus of 10% glucoseQ orally or through NG tube
Complications of Malnutrition: Hypothermia Re-warm; Remove wet clothing/bedding
“SHIELDED”
Infections If no other complications give oral amoxicillin 15 mg/kg 8-hourly for 5 days
S: Sugar deficiency or Hypoglyce-
mia (blood glucose < 54 mg/dL)Q If complications present: IV Ampicillin & Gentamicin/ IV Ceftriaxone, if shock present
H: Hypothermia (Rectal temperature Electrolyte •• Supplement potassium at 3–4 mEq/kg/day for at least 2 weeks
< 35.5°C)Q imbalance •• Give food without added salt to avoid sodium overload
I: Infections •• Give 50% magnesium sulphate IM once (0.3 mL/kg, max 2 mL) followed by
EL: Electrolyte imbalance 0.4–0.6 mmol/kg/daily orally
DE: Dehydration
D: Deficiency of micronutrients like Dehydration If no shock: Give ORS 5 mL/kg every 30 minutes for first 2 hours & then 5–10 mL/kg
Potassium,Q Iron every 2 hours for up to 10 hours ; Resomal or ORS prepared in half dilution (1 sachet
mixed in 2L water) with added syrup PotKlor and sugar can be used
If shock present: Give IV Ringer’s lactate in 5% Dextrose 15 mL/kg over 1 hour →
Reassess the child → may repeat
Deficiency of •• Give Vitamin A single dose to all children, repeat after 24 hours & 14 days if clinical
micronutrients features of vitamin A deficiency seen; dose according to age:
Question 6
–– < 6 months: 50 000 IU
What is the abnorma­ lity shown –– 6–12 months or if weight < 8 kg: 100 000 IU
in the hairs of a child with severe –– > 12 months: 200 000 IU
malnutrition? •• Multivitamins (vitamin B complex, A, C, D, E & B12): Twice of RDA
•• Folic acid: 5 mg on day 1, then 1 mg/day
•• Elemental Zinc: 2 mg/kg/day & Copper: 0.3 mg/kg/day
•• Iron:
–– Do not give iron in stabilization phase.
–– Start after 2 days of child being on Catch up diet (rehabilitation phase)
–– Dose 3 mg/kg/day in two divided doses, preferably between meals

Nutritional Management M

•• Start feeding with a ‘Starter’ (F-75) diet (75 kcal & 0.9 g protein/100 mL), 8–12 feeds over
24 hours, in the stabilization phase and gradually build it up.
a. Light dark sign b. Flag sign •• Catch up diet (F-100) contains 100 kcal and 3 g protein per 100 mL
c. Hair sign d. Color sign •• Gradually increase to 150–200 kcal/kg/day & proteins to 4–6 gm/kg/day after 1 wk.
Chapter 3: Nutrition and Malnutrition 41

Ready to use therapeutic food (RUTF)

Section 1: General Pediatrics


Question 7
•• It is an energy dense, vitamin & mineral enriched food made of peanut paste, milk solids, Identify the eye abnormality shown
sugar & oil. in the given picture.
•• It has a smooth, pasty consistency, good taste and is easy for the child to eat & digest.
•• Each 100 g contains 543 kcal.
•• It helps avoid cumbersome preparation of recipes at home, and is a more practical and
easier option.

Failure of Treatment
Primary failure to respond is indicated by:
• Failure to regain appetite by day 4 • Presence of edema on day 10
•• Failure to start losing edema by day 4 • Failure to gain ≥ 5 gm/kg/day by day 10
a. Corneal xerosis b. Bitot’s spots
Secondary failure to respond is indicated by: Failure to gain at least 5 gm/kg/day for c. Night blindness d. Keratomalacia
3 consecutive days during the rehabilitation phase.

Discharge Criteria M

•• Good appetite
•• No edema & consistent weight gain (>5 g/kg/day for 3 consecutive days)
•• Infection/ micronutrient deficiency & immunization taken care of
•• Caregiver motivated & skilled to provide care

Nutritional Recovery Syndrome/Refeeding Syndrome M

Refeeding syndrome involves abnormalities in fluid balance, glucose metabolism, vitamin


High Yield Points
deficiency, hypophosphatemia, hypomagnesemia, and hypokalemia in patients exposed to
enteral or parenteral nutrition after a period of starvation and weight loss. Thiamine may be useful in treating:
Thiamine-responsive megaloblastic
L at e s t U p d at e s anemia (TRMA) syndrome (autosomal
recessive SLC19A2 mutation with dia-
•• ‘POSHAN’ abhiyaan stands for ‘PM’s overarching scheme for holistic nutrition’. betes mellitus & sensorineural hearing
•• It has been launched by Prime Minister in February 2018. loss), Leigh disease & type 1 diabetes
•• The focus is on health and nutrition in first 1000 days of life. mellitus.
•• The goals for this program include reduction in stunting, undernutrition & low birth weight by 2% per
annum and anemia in young children by 3% per annum.

Long-term problems due to malnutrition:


It is associated with shorter adult height, poor lean weight, lower cognition, reduced economic
productivity, lower offspring birth weight & increased risk of diabetes mellitus, hypertension and
dyslipidemias during adulthood.
High Yield Points
Functions of Vitamins & Diseases caused by Deficiency M
WHO classification of Vitamin A
Vitamin Function Disease caused by deficiency deficiency:

Vitamin A Required for synthesis of visual pigments Nyctalopia (Night blindness)Q Indicator Category
rhodopsin & iodopsin, gene expression, Photophobia, xerophthalmia,Q Bitot Night blindness XN
reproduction, embryonic development & spots,Q conjunctivitis, keratomalacia,Q
immune function Conjunctival xerosis X1A
phrynoderma; defective tooth enamel;
Impaired resistance to infection Bitot’s spots X1B
Biotin Cofactor for carboxylases, important in gluco- Periorificial dermatitis,Q conjunctivitis, Corneal xerosis X2
neogenesis, fatty acid and amino acid metabo- alopecia,Q lethargy, hypotonia, seizures Corneal ulceration X3A
lism
Keratomalacia X3B
Folic acid Coenzyme in the metabolism of nucleic & Megaloblastic anemia;Q
amino acid; Reduces homocysteine levels, Ulcers, glossitis, periungual hyper­ Xerophthalmia- XS
protects against atherosclerosis; pigmentation related corneal scars

Contd...
42 Section 1: General Pediatrics

Contd...
Review of Pediatrics and Neonatology

High Yield Points


Vitamin Function Disease caused by deficiency
•• Decreased levels of RBC trans­
ketolase is the best indicator of Niacin Coenzyme in many redox reactions; Hartnup disease,Q PellagraQ (Dermatitis,
thiamine deficiency component of NAD dementia, diarrhea)
•• Niacin deficiency is common in Thiamine Coenzyme in metabolism of Dry beriberi: Irritability, peripheral neuritis,
maize eaters (due to high leucine (Vitamin B1) carbohydrates & branched-chain amino muscle tenderness, ataxia
content in maize) acids, nucleic acid synthesis Wet beriberi: Tachycardia, edema,
•• Photosensitive rash seen in Pellagra
cardiomegaly, cardiac failure
is called Casal’s necklace
Riboflavin Coenzyme in many redox reactions Glossitis,Q cheilosis, nasolabial dyssebacia,
(vitamin B2) keratitis, peripheral neuropathy
Pyridoxine Role in transamination, decarboxylation Neonatal seizures,Q microcytic
(vitamin B6) & desulphuration; (Prolonged INH, anemia, homocystinuriaQ, oxalosis,
Mnemonic  M
penicillamine & hydralazine intake results in cystathioninuriaQ, xanthurenicQ, aciduria
Hypervitaminosis A: “H-A-R-D Puzzle” deficiency)
•• Hepatosplenomegaly, Hair sparse, Cobalamin Coenzyme in nucleic acid metabolism; Megaloblastic anemia;
Hyperostosis (vitamin deficiency seen in methyl malonic Subacute combined degeneration of spinal
•• Anemia, Anorexia B12) aciduria, pernicious anemia, pure cord
•• Really painful bones vegetarians & fish tape worm infestation
•• Dry skin along with Ascorbic Strong reducing agent & antioxidant Scurvy: pseudo paralysisQ (due to
•• Pseudotumor cerebri acid Cofactor for reactions requiring reduced tenderness of muscles), rosary of ribs (which
(vitamin C) copper or iron are sharper compared with rickety rosary),
Helps in collagen, teeth, bone formation gum bleeding, irritability, tachypnea

Vitamin E Antioxidant, antineoplastic effect, reduces Areflexia, ataxia of trunk & limbs, myopathy;
Question 8 incidence of Retinopathy of Prematurity Retinal changes; Hemolytic anemia,
(ROP) & Bronchopulmonary dysplasia (BPD) hyperbilirubinemia & IVH in preterm babies
The following rashes seen in a
previously well 5 month old child Vitamin K Coenzyme for synthesis of clotting VKDB (Vitamin K deficiency bleeding) or
can be treated by administration of: factors 2, 7, 9, 10; stimulate platelet HDN (Hemorrhagic disease of newborn)
activity & osteocalcin, that helps in calcium
metabolism
Pantothenic Component of coenzyme A & acyl carrier Irritability, burning feet syndrome, muscle
Acid protein involved in fatty acid metabolism cramps; isolated deficiency rare in humans

For details about Rickets and Scurvy, refer to chapter on Musculoskeletal disorders in children.

Question 9
This is seen commonly due to deficiency of:

a. Calcium
b. Pyridoxine
c. Niacin
d. Zinc

a. Vitamin B1 b. Vitamin B2 c. Niacin d. Vitamin B12

Question 10
This skin condition can be seen due to deficiency of any of the following EXCEPT:

a. Vitamin A b. Vitamin D c. Vitamin E d. Essential fatty acids


Chapter 3: Nutrition and Malnutrition 43

Section 1: General Pediatrics


Answer Keys for Image-Based Questions

Answers Explanations / Identifying features


1. Ans. c. Good attachment The neonate’s mouth is wide open with most of areola inside baby’s mouth, lower lip everted & chin
touching the breast; These are signs of good attachment while breastfeeding
2. Ans. a. Protects from neonatal The neonate is kept in skin to skin contact with mother, in between the breasts, inside clothing of mother.
jaundice This is Kangaroo Mother Care (KMC)
3. Ans. b. Marasmus Emaciated child with severe malnutrition, without edema, suggests ‘Marasmus’
4. Ans. c. Mid arm circumference Mid arm circumference is being measured in this picture; Shakir’s tape can be used to measure it
5. Ans. b. Flaky paint dermatosis This picture shows irregular, peeling off skin, suggestive of flaky paint dermatosis, a skin change seen in
severe malnutrition
6. Ans. b. Flag sign Alternate bands of dark & light (hypopigmented) hair is called ‘Flag sign’
7. Ans. b. Bitot’s spots White, triangular lesion on the bulbar conjunctiva, is known as ‘Bitot’s spots’
8. Ans. d. Zinc Erythematous rahses around orifices can be seen in Acrodermatitis Enteropathica (AE)
9. Ans b. Vitamin B2 The given picture shows lesions at angles of mouth (angular cheilitis), usually seen due to deficiency of
vitamin B2 (Riboflavin)
10. Ans b. Vitamin D It is ‘Phrynoderma’, a form of follicular hyperkeratosis, due to deficiency of vitamins E, B, A, and essential
fatty acids
44 Section 1: General Pediatrics
Review of Pediatrics and Neonatology

Questions

 BREAST MILK & BREASTFEEDING 11. The following component of human milk is protective
against Giardia lamblia infection? (APPG 2014)
1. Which of the following is a contraindication for breast­ a. Lysozyme b. Lactoferrin
feeding? M  (Recent Question 2016) c. Bifidus factor d. Glycoconjugate
a. Hep A b. Hep B 12. The average whey/casein ratio in breast milk is:
b. CMV d. Active untreated TB  (MAHA PGM CET 2014)
2. Which of the following is true regarding storage of a. 60:40 b. 80:20
expressed breast milk? M  (Recent Question 2017)
c. 20:80 d. 40:60
a. EBM can be stored at room temperature for about 6 hours 13. Which of the following is not a component of Kangaroo
and 24 hours in refrigerator mother care (KMC)? (AIIMS May 2014)
b. EBM can be stored at room temperature for about
24 hours and 6 days in refrigerator a. Skin to skin contact b. Supplementary nutrition
c. EBM can be stored at room temperature for about c. Exclusive breastfeeding d. Early discharge and follow-up
18 hours and 1 month in refrigerator 14. Amount of protein present in 100 mL of breast milk is:
d. EBM can be stored at room temperature for about 6 hours
a. 2.2 g b. 1.1 g  (JIPMER 2013)
and 6 days in refrigerator
c. 0.55 g d. 3.3 g
3. Which one of the following is a bioactive factor of
human milk when compared to formula milk? 15. Compared to breast milk, colostrum is richer in: M

a. Lipoprotein A (APPG 2016) a. Protein b. Fat  (WB PGMEE 2012)


b. Transforming growth factor c. Lactose d. Water
c. Immunoglobulin-M d. Polysaccharides 16. Breast milk at room temperature stored for: M

4. Breastfeeding should begin within … hours after a. 4 hours b. 8 hours  (DNB June 2012)
cesarean delivery: M  (Recent Question 2016) c. 12 hours d. 24 hours
a. 2 b. 4 17. Breastfeeding for a new born to be started: M 
c. 8 d. 24  (DNB Dec 2012)
5. Breast milk storage in a refrigerator is up to: M  a. As soon as possible b. After 6 hours
 (Recent Question 2016) c. After 1 day d. After meconium has passed
a. 4 hours b. 8 hours 18. Breast milk is rich in ___ when compared to cow’s milk: M
c. 12 hours d. 24 hours
a. Lactose b. Protein (JIPMER 2012)
6. Milk is deficient in? (AIIMS Nov 2015) c. Fat d. Vitamin K
a. Iron and Vit C b. Iron and Vit A 19. All are true about breastfeeding except: M
c. Phosphorus and Vit C d. Phosphorus and Vit A
a. It is available at proper temperature  (WB PGMEE 2012)
7. Colostrum is rich in: M (TN PGMEE 2015) b. Breast milk contains antibodies
a. IgA b. IgE c. Exclusive breastfeeding up to 6 months of age
c. IgG d. IgM d. To be started after 4 hours of normal delivery
8. Compared with cow’s milk, mother’s milk has more? M 20. Exclusive breastfeeding is done till: M  (DNB June 2011)
 (Bihar PG 2015)
a. 4 months b. 6 months
a. Lactose b. Vitamin D c. 8 months d. 10 months
c. Proteins d. Fat 21. Fatty acid present in breast milk which is important for
9. Which of the following are true about breast milk? growth & CNS development is:
a. It is a rich source of Iron (Recent Question 2015)  (AIIMS Nov 2011, Nov 2006)
b. It contains more carbohydrates than cow's milk a. Docosahexaenoic acid b. Palmitic acid
c. It contains more proteins than cow's milk c. Linoleic acid d. Linolenic acid
d. It contains less DHA than cow's milk 22. After premature delivery, mother’s milk is low in: 
10. Nipple confusion means? M (APPG 2014)
a. Lactose b. Fat  (MAHA PG 2010)
a. Baby fed with a bottle finding it difficult and confusing to c. Protein d. Sodium
suckle at breast
23. Breast milk is maximum at: (APPG 2010, APPG 2004)
b. Baby not able to suckle with bottle
c. Baby not able to feed with spoon a. 1–2 months b. 3–4 months
d. Baby not able to feed with paladin c. 5–6 months d. 7–8 months
Chapter 3: Nutrition and Malnutrition 45

36. The protective effects of breast milk are known to be

Questions
24. Complimentary feeding starts at: M (WB PGMEE 2010)
associated with: (AIPGMEE 2005)
a. 1 month b. 3 months
c. 6 months d. 12 months a. IgM antibodies b. Lysozyme
25. Calorie requirement per day of a child weighing 15 kg c. Mast cells d. IgA antibodies
would be: (MAHA PG 2010) 37. The current recommendation for breastfeeding is that:
M (AIPGMEE 2004)
a. 1150 kcal b. 1250 kcal
c. 1450 kcal d. 1550 kcal a. Exclusive breastfeeding should be continued till 6 months
26. True about cow’s milk are all except:  of age followed by supplementation with additional foods
 (AIIMS May 2010, May 2007) b. Exclusive breastfeeding should be continued till 4 months
of age followed by supplementation with additional foods
a. Cow's milk contains 80% whey protein not casein
c. Colostrum is the most suitable food for a newborn baby
b. Cow milk has less carbohydrate than mother's milk
but it is best avoided in first 2 days
c. Has more K+ and Na+ than infant formula feeds
d. The baby should be allowed to breastfeed till one year of age
d. Has more protein than breast milk
27. Para-aminobenzoic acid of breast milk prevent the
infection of: (DNB Dec 2010)
 MALNUTRITION
a. Plasmodium vivax b.
Klebsiella pneumoniae 38. Specific sign of Kwashiorkor is: (FMGE June 2018)
c. Giardia d. E. coli
a. Pitting edema b. Weight loss
28. Which of the following is not the correct sign of good c. Flag sign d. Muscle wasting
attachment of a baby to the breast: M  (DNB June 2009)
39. Definition of severe acute malnutrition includes which
a. Baby's mouth wide open of the following? (NEET PG Jan 2019)
b. Lower areola more visible
c. Baby's lower lip everted a. Wt for Ht < – 2 SD b. Wt for Ht < – 3 SD
d. Baby's chin touching the breast c. Wt for age < – 2 SD d. Wt for age < – 3 SD
29. True about breastfeeding: (PGI Nov 2009) 40. Which is the most specific clinical feature for diagnosis
a. Best for both preterm and term for Kwashiorkor? (FMGE Nov 2017)
b. 50% energy from protein a. Fatty liver b. Easy pluckable hair
c. Promote lactobacillus growth in bowel c. Low serum albumin d. Edema
d. Predispose to necrotizing enterocolitis
41. Adaptive starvation with increased metabolism:
e. Decrease allergic disorder
 (FMGE Nov 2017)
30. In breastfed infant less chances of enteric infection is
a. Bulimia b. Kwashiorkor
due to: (PGI Dec 2008)
c. Marasmus d. Cachexia
a. Alkaline pH of stool
42. Proximal muscle weakness & atrophy in a child with
b. Breast milk nutrients, beneficial effect on immune system
PEM is due to which electrolyte disturbance? (JIPMER
c. IgA in breast milk
2017)
d. Bacteroides and Clostridium in gut
e. Sterile nature of breast milk a. Hypokalemia b. Hypomagnesemia
31. Benefits of breast milk are: (PGI June 2008) c. Hypophosphatemia d. Hypocalcemia
43. Proximal muscle weakness atrophy in a child with PEM
a. Better nutrition b. Less infection
due to which electrolyte disturbance: (JIPMER Nov 2017)
c. More allergy d. Less allergy
e. High sodium contents a. Magnesium b. Calcium
32. Breast milk is known to transmit: (MAHA PG 2007) c. Zinc d. Phosphate

a. Tuberculosis b. CMV 44. Which of the following is false about Kwashiorkor?


c. Varicella d. Rubella a. Flaky paint dermatosis (Recent Question 2017)
33. Compared with cow’s milk, mother’s milk has more: M b. Atrophic changes in liver on biopsy
 c. Can pull loose hair easily
d. Serum albumin < 3 gm per 100 mL
a. Lactose b. Vit D  (AIPGMEE 2007)
c. Proteins d. Fats 45. Dose of treatment of vitamin A deficiency in 10 kg
child who is 2 years old is: M  (Recent Question 2017)
34. Hind milk is richer in: M (COMEDK 2007)
a. 5,000 IU b. 100,000 IU
a. Carbohydrate b. Protein
c. 150,000 IU d. 200,000 IU
c. Fat d. Minerals
46. Which of the following would be affected if malnutrition
35. Breast milk is rich in _______ when compared to cow’s
is long-standing? M  (Recent Question 2017)
milk: M  (JIPMER 2007)
a. Lactose b. Protein a. Weight for age b. Height for age
c. Fat d. Vitamin K c. Weight for height d. All of the above
46 Section 1: General Pediatrics

58. According to WHO, severe acute malnutrition in


Review of Pediatrics and Neonatology

47. A 15-month-old child feeding on cow milk with water


with severe wasting and bipedal edema presents with children is defined as: (JIPMER 2014)
poor appetite. Diagnosis? M  (Recent Question 2016) a. Weight for height Z score < –3
a. Kwashiorkor b. Marasmus b. Weight for height Z score < –2
c. Weight for height Z score < –1
c. Both (a) and (b) d. None of the above
d. Weight for height Z score between –2 and –3
48. A 4-year-old child with severe wasting and voracious
59. All are seen in marasmus except:
appe­tite without pedal edema: M (Recent Question 2016)
 (Recent Question 2014)
a. Kwashiorkor b. Marasmus
a. Hepatomegaly b. Muscle wasting
c. Both d. None
c. Voracious appetite d. Weight loss
49. Decrease in weight for height indicates? M 
60. Admission to nutrition rehabilitation center for severe
 (Recent Question 2016)
acute malnourished child between 6-month and 5-year
a. Acute malnutrition b. Chronic malnutrition is all except: (MAHA PGM CET 2014)
c. Both acute and chronic malnutrition
d. Short stature a. Wt/ht < –3 SD b. MUAC < 11.5 cm
c. Wt < 2 SD for age d. Bilateral pedal edema
50. Kwashiorkor is diagnosed in growth retarded children
along with: M  (Recent Question 2016)
61. Fatty liver in Kwashiorkor is due to: (JIPMER 2013)

a. Edema and mental changes a. Decrease in carbohydrates


b. Hypopigmentation and anemia b. Decrease in protein substrates
c. Edema and hypopigmentation c. Abnormal protein synthesis
d. Hepatomegaly and anemia d. Reduced synthesis of lipoprotein carrier
51. In severe malnutrition feeding is started with diet which 62. Waterlow classification of malnutrition in children
has all except: (MAHA PGM CET 2016) takes into account: (Recent Question 2013)

a. Osmolarity less than 350 mOsmol/L a. Weight for height (wasting)


b. Lactose 2–3 g/kg/day b. Height for age (stunting)
c. Calories from protein > 15% c. Weight for height (wasting) and height for age (stunting)
d. Renal solute load urinary < 600 mOsmol/L d. Percent of reference weight for age
52. In a child with malnutrition, the first line of management 63. In protein deficiency, all are seen except: 
would be: (TN PGMEE 2015)  (Recent Question 2013)

a. Prevention of hypothermia a. Flaky paint like skin b. Glossitis


b. Prevention of hypoglycemia c. Nail change d. Cherry like skin
c. Treatment of infection 64. Not seen in Kwashiorkor: (Recent Question 2013)
d. Treatment of dehydration a. Apathy b. Flaky paint dermatosis
53. Monkey facies and Baggy pants refers to: M c. Baggy pant appearance d. Increased transaminase
 (MAHA PGM CET 2015) 65. In order to discharge from hospital, malnourished child
a. Peculiar facies of monkeypox infection should have minimum weight gain of: (NEET Pattern 2013)
b. Peculiar dress of African tribe a. 5 gm/kg/day b. 10 gm/kg/day
c. Look in marasmus because of loss of fat c. 15 gm/kg/day d. 20 gm/kg/day
d. Look in monkey bite
66. Flaky paint appearance of skin is seen in: M 
54. While assessing a child for severe acute malnutrition,  (Recent Question 2013)
all are criteria except: M  (AIIMS Nov 2015)
a. Dermatitis b. Pellagra
a. Mid arm circumference b. Edema c. Marasmus d. Kwashiorkor
c. Weight for height d. Height for age
67. Flag sign is seen in: M (DNB Dec 2012)
55. Select the false statement regarding marasmus:
(APPG 2015) a. Kwashiorkor b. Beriberi
c. Pellagra d. Marasmus
a. There is muscle atrophy and resultant hypotonia
68. Which of the following is a feature of marasmus and not
b. Loss of fat from sucking pads of cheeks is earliest sign
Kwarshiorkar? (JIPMER 2011)
c. Besan panjiri is an energy dense food used in treatment
d. This is non-edematous severe childhood undernutrition a. Voracious appetite b. Fatty change in liver
c. Hypoalbuminemia d. Edema
56. Which of the following is the best indicator of long-
term nutritional status? M  (Bihar PG 2015) 69. In a child with severe PEM which of the following
should not be included in the initial treatment: M 
a. Mid-arm circumference b. Height for age
c. Weight for age d. Weight for height a. Vitamin A b. Iron  (WB PGMEE 2010)
57. Best marker for chronic PEM is: M  c. Zinc d. Folic acid
 (Recent Question 2015) 70. Kwashiorkor, not an essential criteria: (WB PGMEE 2010)
a. Weight for age b. Height for age a. Edema b. Growth retardation
c. Weight for height d. Head circumference c. Hepatomegaly d. Apathy
Chapter 3: Nutrition and Malnutrition 47

82. Acute complication of PEM: (PGI June 2006, PGI Dec 2005)

Questions
71. In Kwashiorkor, the letter ‘K’ is post-fixed to denote: M
 (MAHA 2010) a. Hypothermia b. Hypoglycemia
a. Weight for height b. Skin changes c. Hypokalemia d. Hypermagnesemia
c. Edema d. Muscle wasting e. Eosinophilia
72. Poor indicator to response during treatment of malnu­ 83. False about marasmus are all except: (TN PGMEE 2005)
trition includes: (PGI Nov 2010)
a. Hepatomegaly b. Edema
a. Failure to gain appetite by day 10 c. Good appetite d. Flaky paint appearance
b. Presence of edema on day 10
c. Failure to gain at least 5 g/kg/day by day 4 84. All of the following conditions are observed in
d. Failure to gain at least 5 g/kg/day by day 10 marasmus, except: (AIIMS May 2005)
e. Failure to gain at least 5 g/kg/day by day for 3 consecutive a. Hepatomegaly b. Muscle wasting
days during the rehabilitation phase c. Low insulin levels d. Extreme weakness
73. Best indicator of growth monitoring in children: M  85. Acute malnutrition is assessed by: (PGI June 2005)
 (AIIMS May 2010)
a. Weight for age b. Weight for height
a. Weight b. Mid-arm circumference
c. Rate of increase in height and weight c. Height for age d. Broca's index
d. Head circumference e. Ponderal index
74. With reference to a malnourished child, the following 86. Deficit in weight for height in a 3-year-old child
statements are correct except: (DNB Dec 2009) indicates: (AIIMS Nov 2005)
a. Skin and mucosa are not effective barriers to infection a. Acute malnutrition b. Chronic malnutrition
b. There is impairment of chemotaxis and defective c. Concomitant acute and chronic
candidacidal, bactericidal capacities of polymorphs d. Underweight
c. There is impairment of cell mediated immunity and
87. All of the following are characteristic features of
delayed hypersensitivity
d. There is impairment of humoral response to immunizing Kwashiorkor except: (AIIMS May 2003)
agents and reduced number of B-cells a. High blood osmolarity b. Hypoalbuminemia
75. In PEM hypoglycemia is corrected by: (TN PGMEE 2009) c. Edema d. Fatty liver
a. 5% dextrose b. 10% dextrose 88. Kwashiorkor in Ghanaian language means: 
c. 25% dextrose d. 50% dextrose a. Condition seen in 2nd child (PGI Nov 2000)
76. When can a severely malnourished child be safely b. Condition seen in the displaced child
discharged from the hospital? (DNB June 2009) c. Condition seen in cousin
a. The child attains his height for age d. Condition seen in the stepchild
b. The child reaches his ideal weight for height e. Condition seen in the 3rd child
c. The child loses edema and starts gaining weight
d. The child attains weight for his age
 MICRONUTRIENTS & THEIR DEFICIENCY
77. Mostly death in PEM is due to all except: 
a. Hypothermia b. CCF (DNB June 2008) 89. Recommended daily dose of Iodine in a child is:
c. Worm infestation d. Electrolyte imbalance  (FMGE Dec 2018)
78. The following statement about Gomez classification is a. 120–200 mg b. 90–120 mg
false: (AIPGMEE 2008)
c. 30–60 mg d. 500 mg
a. Based on height retardation 90. Dose of vitamin A for a 18-month-old baby, with
b. Based on 50th centile Boston standards
keratomalacia, weighing 10 kg is: (FMGE Dec 2018)
c. Between 75–89% implies mild malnutrition
d. This classification has prognostic value for hospitalization a. 50,000 IU b. 1,00,000 IU
of children c. 2,00,000 IU d. 5,00,000 IU
79. One of the following is false regarding complications of 91. First clinical sign of vitamin A deficiency: M 
protein-energy malnutrition: (COMEDK 2007)
 (Recent Question 2017)
a. Hyperglycemia b. Hypothermia
a. Poor growth b. Conjunctival xerosis
c. Septic shock d. Electrolyte imbalance
c. Hydrocephalus d. Phrynoderma
80. Common to both acute and chronic malnutrition is: 
 (AIIMS May 2007) 92. Casal necklace is seen in? M (Recent Question 2017)

a. Weight for age b. Weight for height a. Scurvy b. Kwashiorkar


c. Height for age d. BMI c. Pellagra d. Indicanuria
81. Initial protein requirement of a Kwashiorkor child (g/kg/ 93. Daily protein requirement in a 2 years old child is?
day): (DNB Dec 2007)  (Recent Question 2017)
a. 0.5–1.0 b. 1.0–1.5 a. 13 gm b. 25 gm
c. 1.5–2.0 d. 2–3 c. 40 gm d. 60 gm
48 Section 1: General Pediatrics
Review of Pediatrics and Neonatology

94. Which of the following are the qualities of a green light 105. Zinc deficiency leads to all except?
food as per traffic light diet plan for children?  a. Delayed sexual maturation (MAHA PGM CET 2015)
 (Recent Question 2017)
b. Impaired immune function
a. Nutrition dense, but higher in calories and fat c. Skeletal abnormalities d. Excessive appetite
b. Low calories, high fiber, low fat, Nutrition-dense
c. High in calories, sugar and fat 106. Recommended dose of folic acid during pregnancy is:
d. None of the above  (MAHA PGM CET 2015)

95. A rare autosomal recessive disorder caused by mutation a. 200 mcg/day b. 300 mcg/day
in SLC19A2 gene and characterized by megaloblastic c. 400 mcg/day d. 500 mcg/day
anemia, diabetes mellitus and sensorineural hearing 107. A 6-year-old girl on a predominantly maize based diet
loss responds to which of the following nutrient? presents with history of recurrent diarrhea, dermatitis
 (Recent Question 2017) with rashes involving the neck area is seen in: 
a. Vitamin C b. Vitamin B12  (Recent Question 2015)
c. Vitamin B1 d. Folic acid a. Pellagra b. Tyrosinemia
96. All of the following are vitamin B6 dependent syn­ c. Phenylketonuria d. Albinism
dromes, except: (Recent Question 2017)
108. In which stage corneal xerosis is seen? M
a. Pyridoxine dependent seizures  (Recent Question 2015)
b. Xanthurenic aciduria
c. Cystanthioninuria d. Darier disease a. Stage X1 b. Stage X2
c. Stage X3A d. Stage X3B
97. In children having mild vitamin K deficiency, prothrombin
time may be normal and estimation of protein Induced in 109. A child presents with malnutrition, persistent diarrhea.
vitamin K absence (PIVKA) is useful to detect early vitamin Apart from antibiotics what would you like to add in
K deficiency. What is the exact form of PIVKA? your prescription? (WB PGMEE 2015)
a. Undercarboxylated factor VII (Recent Question 2017) a. Zinc, Vit A, Iron b. Zinc, Iron
b. Undercarboxylated factor II c. Zinc, Vit A d. B complex, Zinc
c. Carboxylated factor VII 110. The symptoms of dietary deficiency of niacin (which
d. Carboxylated factor II results in pellagra) will be less severe if the diet has a
98. Cardiac edema and neuropathy is seen in which high content of: M  (MAHA PGM CET 2014)
micronutrient deficiency? (JIPMER May 2016)
a. Tryptophan b. Tyrosine
a. Biotin b. Pyridoxine c. Thymine d. Thiamine
c. Thiamine d. Riboflavin 111. Consumption of which of the following cereal as staple
99. True abut vitamin D deficiency in children is/are:  diet is associated with Pellagra: M  (MAHA PGM CET 2014)
a. RDA for vitamin D is 400 IU (PGI May 2016) a. Rice b. Wheat
b. PTH & 1,25-(OH)2 vitamin D play key roles in the c. Maize d. Bajara
regulation of calcium and phosphate balance
112. Abnormalities of copper metabolism are implicated in
c. Serum calcium is always low
the pathogenesis of all the following except: M 
d. Serum phosphorus is low
e. PTH is normal a. Wilson's disease  (Recent Question 2014)
b. Menke's Kinky-hair syndrome
100. Thiamine deficiency-causes: M  (Recent Question 2016)
c. Indian childhood cirrhosis
a. Pellagra b. Beri-beri d. Keshan disease
c. Keshan's disease d. Rickets
113. For which vitamin, RDA is determined according to
101. Acrodermatitis enteropathica is seen with deficiency of: protein intake: M  (JIPMER 2014)
M (MAHA PGM CET 2016)
a. Zinc b. Iron a. Thiamine b. Niacin
c. Copper d. Vitamin A c. Pyridoxine d. Vitamin B12
102. Stage X3B refers to the following in the classification of 114. All of the following are features of vitamin E deficiency
vitamin A deficiency: (Recent Question 2016) except: (Recent Question 2013)
a. Night blindness b. Corneal xerosis a. Areflexia b. Seizure
c. Conjunctival xerosis d. Keratomalacia c. Myopathy d. Ataxia
103. Vitamin B12 gets absorbed from: (WB PGMEE 2016) 115. Selenium deficiency is seen in: (Recent Question 2013)
a. Stomach b. Duodenum a. Keshan disease b. Wilson disease
c. Ileum  d. Colon c. Acrodermatitis enteropathica

104. The disease which is characterized by three D’s— d. None of above
diarrhea, dermatitis and dementia is due to deficiency 116. ‘Toad skin’ is seen in deficiency of vitamin:
of? (MAHA PGM CET 2015)  (Recent Question 2013)
a. Vitamin A b. Niacin a. A b. B2
c. Folate d. Vitamin C c. D d. Biotin
Chapter 3: Nutrition and Malnutrition 49

123. Which one of the following vitamin deficiencies causes

Questions
117. Diarrhea in a child of 12 months, dose of zinc is: M 
corneal vascularization, poor growth and photophobia:
a. 1 mg/day for 10–14 days (Recent Question 2013)
 (DNB June 2007)
b. 10 mg/ day for 10–14 days
c. 15 mg/ day for 10–14 days a. Riboflavin b. Pyridoxine
d. 20 mg/ day for 10–14 days c. Niacin d. Thiamine
118. The recommended oral dose of vitamin A to be given in 124. Hypervitaminosis of which of the following will cause
a 10 months child with deficiency on each of day 1, 2 bony abnormalities: (PGI Dec 2006)
and 28 is: (COMEDK 2011)
a. Vit A b. Vit D
a. 50,000 IU b. 1,00,000 IU c. Vit C d. Vit E
c. 2,00,000 IU d. 6,00,000 IU e. Vit K
119. Administration of which of the following can prevent 125. Vitamin A deficiency is characterized by: (PGI Dec 2004)
neural defects in newborn? (JIPMER 2011)
a. Bitot spot b. Xerophthalmia
a. Folic acid b. Vit B12 c. Night blindness d. Tranta's spot
c. Pyridoxine d. Vitamin C e. Conjunctival xerosis
120. False about Hartnup’s disease: (JIPMER 2010, 2008) 126. Zinc deficiency causes: (PGI Dec 2003)
a. Defect in neutral amino acid transport a. Sexual infantilism
b. Mental retardation is the common presentation
b. Loss of libido
c. Most children are asymptomatic
c. Poor weight gain
d. Photosensitivity
d. Poor wound healing
121. Deficiency of which element can lead to syndrome of e. Acrodermatitis enteropathica
growth failure, anemia and hypogonadism? 
127. Which of the following congenital malformation is seen
a. Calcium b. Copper (COMEDK 2009) in a child of a mother who is on treatment with oral
c. Zinc d. Magnesium anticoagulants? (AIIMS Nov 2000)
122. Baby fed exclusively on goat’s milk develop: M  a. Craniofacial malformations
 (TN PGMEE 2007)
b. Renal agenesis
a. Megaloblastic anemia b. Microcytic anemia c. Long bone defects
c. Hypochromic anemia d. Normochromic anemia d. Chondrodysplasia punctata
50 Section 1: General Pediatrics
Review of Pediatrics and Neonatology

Answers with Explanations

 BREAST MILK & BREASTFEEDING


1. d. Active untreated TB  Ref: Nelson’s 20/e p 287; Refer pretext for details
2. a. EBM can be stored at room temperature for about 6 hours and 24 hours in refrigerator  Ref: Nelson’s 20/e p 286-290
3. b. Transforming growth factor  Ref: Nelson’s 20/e p 286
•• TGF-β is the most abundant cytokine in human milk; It has 3 isoforms, of which TGF-β2 predominates
•• Milk-borne TGF-β regulates inflammation, wound repair & helps prevent allergic diseases.
4. b. 4  Ref: Nelson’s 20/e p 286-290, Ghai 8/e p 150-161
Breastfeeding should be initiated within 30 mins. of a normal vaginal delivery & 4 hours of cesarean section.
5. d. 24 hours  Ref: Nelson’s 20/e p 286-290, Ghai 8/e p 150-161
Site of storage/temperature Room temperature Refrigerator Freezer at –20° C
Time for which EBM can be stored 8–10 hoursQ 24 hoursQ 3 months

6. a. Iron and Vit C  Ref: Nelson’s 20/e p 286-290, Ghai 8/e p 150-161
About the options:
Iron Iron content (1 mg/L) in cow & breast milk makes it difficult to maintain body iron & meet RDA of infant (8–10 mg)
Breastfed infants absorb iron 2–3 times more efficiently than infants fed cow’s milk
Vit C Infants consuming pasteurized or boiled animal milk are at significant risk of developing Vitamin C deficiency
Phosphorus Cow’s milk is rich in phosphate; So, an infant on a diet rich in cow’s milk has chances of developing hypocalcemia
Vit A Breast milk as well as cow’s milk contains adequate amounts of Vit A to meet the needs of infants

Hence, cow's milk is deficient in Iron & Vitamin C.


Note: Breast milk contains adequate amounts of all vitamins except Vit D, Vit K & Vit B12 (in strictly vegan mothers).
7. a. IgA  Ref: Nelson’s 20/e p 286-290, Ghai 8/e p 150-161
Colostrum contains more antibodies (IgA), WBCs and protein. Also called ‘first immunization’ of the baby.
8. a. Lactose  Ref: Nelson’s 20/e p 286-290, Ghai 8/e p 150-161
9. b. It contains more carbohydrates than cow’s milk  Ref: Nelson’s 20/e p 286-290
10. a. Baby fed with a bottle finding it difficult and confusing to suckle at breast  Ref: DC Dutta Obstetrics 7/e p 451
11. b. Lactoferrin  Ref: Nelson’s 20/e p 286-290, Ghai 8/e p 150-161
Discussing about the options one by one:
Component of breast milk Protects against
a. Lysozyme E. histolytica
b. Lactoferrin G. lamblia, Plasmodium falciparum, T. gondii, E. histolytica & Pneumocystis carinii
c. Bifidus factor Shigella, Salmonella, E. coli
d. Glycoconjugate V. cholerae, E. coli

12. a. 60:40  Ref: Nelson’s 20/e p 286-290, Ghai 8/e p 150-161


Human milk contains two types of proteins 60% is: whey and 40% is casein; helps in quick & easy digestion.
13. b. Supplementary nutrition  Ref: Nelson’s 20/e p 286-290, Ghai 8/e p 150-161
Components of Kangaroo Mother Care (KMC) are:

a. K
 angaroo position: skin-to-skin + b. E
 xclusive breastfeeding ± forti- + c. Early discharge and follow-up
contact fication

14. b. 1.1 g  Ref: Nelson’s 20/e p 286-290, Ghai 8/e p 150-161; Refer pretext of this chapter for details
15. a. Protein  Ref: Nelson’s 20/e p 286-290, Ghai 8/e p 150-161
16. b. 8 hours  Ref: Nelson’s 20/e p 286-290, Ghai 8/e p 150-161
17. a. As soon as possible  Ref: Nelson’s 20/e p 286-290, Ghai 8/e p 150-161
Chapter 3: Nutrition and Malnutrition 51

18. a. Lactose  Ref: Nelson’s 20/e p 286-290, Ghai 8/e p 150-161

Answers with Explanations


19. d. To be started after 4 hours of normal delivery  Ref: Nelson’s 20/e p 286-290, Ghai 8/e p 150-161
20. b. 6 months  Ref: Nelson’s 20/e p 286-290, Ghai 8/e p 150-161
21. a. Docosahexaenoic acid  Ref: Nelson’s 20/e p 286-290, Ghai 8/e p 150-161
Human milk contains 30 times more DHA (10 mg/dl) than cow’s milk, which is a predominant fatty acid in brain and retina.
22. a. Lactose  Ref: Nelson’s 20/e p 286-290; For Mnemonic, refer pretext of this chapter.
23. c. 5–6 months  Ref: Nelson’s 20/e p 286-290, Ghai 8/e p 150-161
Breast milk output is maximum at 5–6 months of lactation ~ 730 mL/day.
24. c. 6 months  Ref: Nelson’s 20/e p 286-290, Ghai 8/e p 150-161
25. b. 1250 kcal  Ref: Nelson’s 20/e p 286-290, Ghai 8/e p 150-161
For children with normal body weights, energy requirement can be calculated as:
For 1st 10 kg: 100 kcal/kg ⇒ For next 10 kg: 50 kcal/kg ⇒ Beyond 20 kg: 20 kcal/kg.
So, for 15 kg, caloric requirement is 10 x 100 + 5 x 50 = 1000 + 250 = 1250 kcal/day.
26. a. Cow’s milk contains 80% whey protein not casein  Ref: Nelson’s 20/e p 286-290, Ghai 8/e p 150-161
27. a. Plasmodium vivax  Ref: Nelson’s 20/e p 286-290, Ghai 8/e p 150-161
28. b. Lower areola more visible  Ref: Nelson’s 20/e p 286-290; Refer pretext of this chapter
29. a. Best for both preterm and term; c. Promote lactobacillus growth in bowel; e. Decrease allergic disorder  Ref: Nelson’s
20/e p 286-290, Ghai 8/e p 150-161
Discussing about the options one by one:
a Composition of breast milk varies at different stages of lactation to suit the needs of the baby
b Energy from different constituents of breast milk is Carbohydrate: 30–40%, from Protein 7–10%; from fat 45–60%
c Bifidus factor supports the growth of lactobacillus in gut thereby preventing growth of harmful bacteria

d and e Breast milk protects against NEC and allergic disorders

30. b. Breast milk nutrients, beneficial effect on immune system; c. IgA in breast milk; e. Sterile nature of breast milk
Ref: Nelson’s 20/e p 286-290; Note: pH of stool in breastfed babies is usually acidic & not alkaline.
31. a. Better nutrition; b. Less infection; d. Less allergy  Ref: Nelson’s 20/e p 286-290, Ghai 8/e p 150-161
32. b. CMV  Ref: Nelson’s 20/e p 286-290, Ghai 8/e p 150-161
Breast milk transmits (with definitive evidence) → HIV & CMV.
33. a. Lactose  Ref: Nelson’s 20/e p 286-290, Ghai 8/e p 150-161
34. c. Fat  Ref: Nelson’s 20/e p 286-290, Ghai 8/e p 150-161; Refer pretext for details;
35. a. Lactose  Ref: Nelson’s 20/e p 286-290, Ghai 8/e p 150-161
36. d. IgA antibodies  Ref: Nelson’s 20/e p 286-290, Ghai 8/e p 150-161
Although antibodies & lysozyme, both are important for protective effects of breast milk, antibodies (IgA) play a major role.
37. a. Exclusive breastfeeding should be continued till 6 months of age followed by supplementation with additional foods
Ref: Nelson’s 20/e p 286-290, Ghai 8/e p 150-161

 MALNUTRITION
38. a. Pitting edema  Ref: Ghai 9/e p 96
Pitting edema is the main sign of Kwashiorkor and it usually starts from feet.
39. b. Wt for Ht < -3 SD  Ref: Ghai 9/e p 97
Wt for Ht < – 3 SD on WHO growth standard is one of the diagnostic criteria for severe acute malnutrition.
40. d. Edema  Ref: Ghai 9/e p 96
Kwashiorkor is primarily a clinical diagnosis.
There is no specific diagnostic test for the condition. Edema is the defining characteristic.
41. c. Marasmus  Ref: Nelson 20/e p 295–306
Marasmus is a chronic, fairly well-adapted form of starvation rather than an acute illness.
42. c. Hypophosphatemia  Ref: Nelson’s 20/e p 368
•• Chronic hypophosphatemia causes proximal muscle weakness & atrophy
•• Hypokalemia in skeletal muscle may cause muscle weakness & cramps
•• Paralysis is a possible complication, generally only at K < 2.5 mEq/L.
52 Section 1: General Pediatrics

43. d. Phosphate  Ref: Nelson 20/e p 368


Review of Pediatrics and Neonatology

Chronic hypophosphatemia causes proximal muscle weakness and atrophy. However, hypokalemia in skeletal muscle may cause
muscle weakness and cramps. Paralysis is a possible complication, generally only at potassium levels <2.5 mEq/L.
44. b. Atrophic changes in liver on biopsy  Ref: Nelson 20/e p 301
In kwashiorkor, hepatomegaly with fatty liver is usually seen.
45. d. 200,000 IU  Ref: Nelson 20/e p 318–320
In children > 1 yr age with weight > 8 kg, therapeutic dose of vit A is 200,000 IU
46. b. Height for age  Ref: Nelson 20/e p 295–306
47. a. Kwashiorkor  Ref: Nelson’s 20/e p 295-306, Ghai 8/e p 96-107; Refer pretext of this chapter for details
48. b. Marasmus  Ref: Nelson’s 20/e p 295-306, Ghai 8/e p 96-107; Refer pretext of this chapter for details
49. a. Acute malnutrition  Ref: Nelson’s 20/e p 295-306, Ghai 8/e p 96-107
50. a. Edema and mental changes  Ref: Nelson’s 20/e p 295-306, Ghai 8/e p 96-107
51. c. Calories from protein > 15%  Ref: IAP Guidelines 2006; Indian Pediatrics, Vol 44;2007
For a child with severe malnutrition, start feeding as soon as possible with a diet, which has:
•• Osmolarity <350 mOsm/L; Lactose < 2–3 g/kg/day •• Adequate bioavailability of micronutrients
•• Appropriate solute load (urinary osmolarity <600 mOsm/L) •• Low viscosity, easy to prepare and socially acceptable
•• Initial percentage of calories from protein of 5% •• Adequate storage, cooking and refrigeration

52. b. Prevention of hypoglycemia  Ref: Nelson’s 20/e p 295-306, Ghai 8/e p 96-107
Management of hypoglycemia, hypothermia & dehydration are the first 3 steps in a child with severe acute malnutrition
53. c. Look in Marasmus because of loss of fat  Ref: Nelson’s 20/e p 295-306, Ghai 8/e p 96-107
54. d. Height for age  Ref: Nelson’s 20/e p 295-306, Ghai 8/e p 96-107
In children 6–59 months of age, severe acute malnutrition (SAM) is defined as: Weight/height or Weight/length < – 3 Z score,
using WHO Growth Charts or presence of bipedal edema of nutritional origin or Mid-upper arm circumference (MUAC) < 115 mm.
55. b. Loss of fat from sucking pads of the cheeks is the earliest sign  Ref: Nelson’s 20/e p 295-306
Buccal pad of fat is usually the last to go in severe malnutrition.
56. b. Height for age  Ref: Nelson’s 20/e p 295-306, Ghai 8/e p 96-107
57. b. Height for age  Ref: Nelson’s 20/e p 295-306, Ghai 8/e p 96-107
58. a. Weight for height Z score < – 3  Ref: Nelson’s 20/e p 295-306, Ghai 8/e p 96-107
59. a. Hepatomegaly  Ref: Nelson’s 20/e p 295-306, Ghai 8/e p 96-107
60. c. Wt < 2 SD for age  Ref: Nelson’s 20/e p 295-306, Ghai 8/e p 96-107
61. d. Reduced synthesis of lipoprotein carrier  Ref: Nelson’s 20/e p 295-306, Ghai 8/e p 96-107
In kwashiorkor there is protein & calorie deficiency, hence the liver is incapable of making apolipoproteins which are needed
for transportation of the lipids; So lipids accumulate in liver causing fatty liver.
62. c. Weight for height (wasting) and height for age (stunting)  Ref: Nelson’s 20/e p 295-306, Ghai 8/e p 96-107

Gomez Criteria Waterlow Criteria


Grade of malnutrition Weight for age (%) Weight for height (%) Height for age (%)
Normal 90–110 90–110 >95
Mild 75–89 80–89 90–94
Moderate 60–74 70–79 85–89
Severe <60 <70 <85

63. b. Glossitis  Ref: Nelson’s 20/e p 295-306, Ghai 8/e p 96-107


Glossitis is a feature of vitamin B complex deficiency.
64. c. Baggy pant appearance; d. Increased transaminase  Ref: Nelson’s 20/e p 295-306
In Kwashiorkor, there is generalized edema, so loose folds of skin (baggy pants appearance) are usually not seen;
Though fatty liver & hepatomegaly are seen in Kwashiorkor, Liver function tests are usually normal.
65. a. 5 gm/kg/day  Ref: Nelson’s 20/e p 295-306, Ghai 8/e p 96-107
66. d. Kwashiorkor  Ref: Nelson’s 20/e p 295-306, Ghai 8/e p 96-107
67. a. Kwashiorkor  Ref: Nelson’s 20/e p 295-306, Ghai 8/e p 96-107
68. a. Voracious appetite  Ref: Nelson’s 20/e p 295-306, Ghai 8/e p 96-107
Chapter 3: Nutrition and Malnutrition 53

69. b. Iron  Ref: Nelson’s 20/e p 295-306, Ghai 8/e p 96-107; Iron should be started in rehabilitation phase

Answers with Explanations


70. c. Hepatomegaly  Ref: Nelson’s 20/e p 295-306, Ghai 8/e p 96-107
Hepatomegaly may be seen in Kwashiorkor, but it is not an essential criteria.
71. c. Edema  Ref: Nelson’s 20/e p 295-306, Ghai 8/e p 96-107
72. b. Presence of edema on day 10; d. Failure to gain at least 5 g/kg/day by day 10; e. Failure to gain at least 5 g/kg/day by
day for 3 consecutive days during the rehabilitation phase;  Refer pretext of this chapter for details
73. c. Rate of increase in height and weight  Ref: Nelson’s 20/e p 295-306, Ghai 8/e p 96-107
Serial measurements of Height & weight (using growth charts) are more important than one time measurement.
74. d. There is impairment of humoral response to immunizing agents and reduced number of B-cells
 Ref: Nelson’s 20/e p 295-306, Ghai 8/e p 96-107
In PEM, Humoral immunity is relatively spared; Number & proportion of B lymphocytes are not altered.
•• Levels of all Immunoglobulins are normal or increased except IgAQ.
75. b. 10% dextrose  Ref: Nelson’s 20/e p 295-306, Ghai 8/e p 96-107
Hypoglycemia is defined as blood glucose < 54 mg/dL in a severely malnourished child. It is treated by immediately giving a
50 mL bolus of 10% glucose orally or through NG tube.
76. c. The child loses edema and starts gaining weight  Ref: Nelson’s 20/e p 295-306; Refer pretext for details
77. c. Worm infestation  Ref: Nelson’s 20/e p 295-306, Ghai 8/e p 96-107

Mnemonic
Mnemonic for indicators of poor prognosis in malnutrition
“SHIELDED SCALD”
S: Sugar deficiency or Hypoglycemia (blood glucose < 54 mg/dL) S Severe anemia
H: Hypothermia (Rectal temperature < 35.5°C) CA: Cardiac failure
I: Infections (Systemic infections) L: Liver failure
EL: Electrolyte imbalance D: Diarrhea
DE: Dehydration or Overhydration
D: Deficiency of micronutrients like potassium, Iron

78. a. Based on height retardation  Ref: Nelson’s 20/e p 295-306, Ghai 8/e p 96-107
79. a. Hyperglycemia  Ref: Nelson’s 20/e p 295-306, Ghai 8/e p 96-107
80. a. Weight for age  Ref: Nelson’s 20/e p 295-306, Ghai 8/e p 96-107
81. b. 1.0–1.5  Ref: Nelson’s 20/e p 295-306, Ghai 8/e p 96-107
Recommended daily energy and protein intake from initial feeds in a child with SAM is 100 kcal/kg & 1-1.5 g/kg.
82. a. Hypothermia; b. Hypoglycemia; c. Hypokalemia  Ref: Nelson’s 20/e p 295-306, Ghai 8/e p 96-107
83. c. Good appetite  Ref: Nelson’s 20/e p 295-306, Ghai 8/e p 96-107
84. a. Hepatomegaly  Ref: Nelson’s 20/e p 295-306, Ghai 8/e p 96-107
85. b. Weight for height  Ref: Nelson’s 20/e p 295-306, Ghai 8/e p 96-107
86. a. Acute malnutrition  Ref: Nelson’s 20/e p 295-306, Ghai 8/e p 96-107
87. a. High blood osmolarity  Ref: Nelson’s 20/e p 295-306, Ghai 8/e p 96-107
88. b. Condition seen in the displaced child  Ref: Nelson’s 20/e p 295-306, Ghai 8/e p 96-107
The name Kwashiorkor, comes from the Ghanian language, meaning disease of the displaced baby when the next one is born.

 MICRONUTRIENTS & THEIR DEFICIENCY


89. b. 90–120 mg  Ref: Nelson’s 20/e p 2663
The recommended dietary allowance of iodine is 30 mg/kg/24 hr for infants, 90–120 mg/24 hr for children, and 150 mg/24 hr for
adolescents and adults.
90. c. 2,00,000 IU  Ref: Nelson’s 20/e p 318–20
91. b. Conjunctival xerosis  Ref: Nelson’s 20/e p 318–20
First clinical sign of vitamin A deficiency is conjunctival xerosis, while 1st symptom is night blindness
92. c. Pellagra  Ref: Nelson’s 20/e p 324–26
54 Section 1: General Pediatrics

93. a. 13 gm  Ref: Nelson 20/e p 271


Review of Pediatrics and Neonatology

Daily protein requirement is: 1.5 g/kg/day for infants (~ 10 g/day), 1.1 g/kg/day for 1–3 years (~ 13 g/day), 0.95 g/kg/day for 4–13
years, (4–8 years → 19 g/day, 9–13 years → 34 g/day) & 0.85 g/kg/day for 14–18 years, (~52 g/day for males & 46 g/day for females)
94. b. Low calories, high fiber, low fat, Nutrition-dense  Ref: Nelson’s 20/e p 314
The “traffic light” diet groups foods into the following categories:

Category Characteristics Recommended intake Examples


Red High in calories, sugar, and fat Reserved for infrequent treats Fatty meats, sugar, sweet beverages, fried foods
Yellow Nutrient-dense, but higher in calories & fat In moderation Lean meats, dairy, starches, grains
Green Low-calorie, high-fiber, low-fat, nutrient-dense Without any limitations Fruits, vegetables

95. c. Vitamin B1  Ref: Nelson’s 20/e p 321


Thiamine-responsive megaloblastic anemia (TRMA) syndrome:
•• It is a rare autosomal recessive disorder characterized by megaloblastic anemia, diabetes mellitus, and sensorineural hearing loss,
responding in varying degrees to thiamine treatment.
•• It occurs because of mutations in the SLC19A2 gene, encoding a thiamine transporter protein, leading to abnormal thiamine
transportation and cellular vitamin deficiency.
Note: Thiamine may improve the outcome in children with Leigh's disease and type 1 diabetes mellitus.
96. d. Darier disease  Ref: Nelson’s 20/e p 326
•• Vitamin B6 dependence syndromes include: pyridoxine-dependent epilepsy, a vitamin B6–responsive anemia, xanthurenic
aciduria, cystathioninuria, and homocystinuria.
•• Pyridoxine-dependent epilepsy involves mutations in ALDH7A1 gene causing deficiency of antiquitin, an enzyme involved in
dehydrogenation of l-alpha-aminoadipic semialdehyde.
97. b. Undercarboxylated factor II  Ref: Nelson’s 20/e p 343
•• In mild vitamin K deficiency, PT is normal, but there are elevated levels of undercarboxylated forms of proteins induced by
vitamin K absence (PIVKA) that are normally carboxylated in the presence of vitamin K.
•• Measurement of undercarboxylated factor II (PIVKA-II) can be used to detect mild vitamin K deficiency.
•• Determination of blood vitamin K level is less useful because of significant variation based on recent dietary intake & levels do
not always reflect tissue stores.
98. c. Thiamine  Ref: Nelson’s 20/e p 321
Thiamine deficiency gives rise to cardiomyopathy and peripheral neuropathy.
99. b. PTH & 1,25-(OH)2 vitamin D play key roles in the regulation of calcium & phosphate balance; d. Serum phosphorus is
low  Ref: Nelson’s 20/e p 337
RDA of vitamin D for infants who are breastfed is 400 IU, while for older children it is 600 IU/day.
In vitamin D deficiency
•• PTH level gets elevated
•• Hypocalcemia is a variable finding as a result of actions of elevated PTH to increase serum calcium
•• Hypophosphatemia is caused by PTH-induced renal losses of phosphate & a decrease in intestinal absorption.
100. b. Beri-beri  Ref: Nelson’s 20/e p 321
Clinical features of Beri-beri (caused by thiamine deficiency):

•• Peripheral neuritis (tingling, burning, paresthesias of toes and feet), decreased deep tendon reflexes, loss of vibration sense,
tenderness and cramping of leg muscles, heart failure and psychological disturbances.
•• Hoarseness or aphonia caused by paralysis of the laryngeal nerve is a characteristic sign.
•• Muscle atrophy and tenderness of nerve trunks, ataxia, loss of coordination and loss of deep sensation.
•• Later signs include increased intracranial pressure, meningismus, and coma.

101. a. Zinc  Ref: Nelson’s 20/e p 344-345, Ghai 8/e p 121-122


Deficiency of zinc leads to acrodermatitis enteropathica, in which rashes shown on skin, especially around the orifices.
102. d. Keratomalacia  Ref: Nelson’s 20/e p 318-320, Ghai 8/e p 111-112; Refer pretext for details
103. c. Ileum  Ref: Nelson’s 20/e p 328, Ghai 8/e p 119
104. b. Niacin  Ref: Nelson’s 20/e p 324, Ghai 8/e p 118
Diarrhea, Dermatitis & Dementia are seen in Pellagra, which is due to Niacin deficiency.
105. d. Excessive appetite  Ref: Nelson’s 20/e p 324, Ghai 8/e p 118
Chapter 3: Nutrition and Malnutrition 55

106. c. 400 mcg/day  Ref: Nelson’s 20/e p 327, Ghai 8/e p 119

Answers with Explanations


All females of childbearing age should take 400 mcg of folic acid/day.
107. a. Pellagra  Ref: Nelson’s 20/e p 324-326, Ghai 8/e p 118-119
Maize based diet predisposes to Niacin deficiency, which gives rise to Pellagra.
108. b. Stage X2  Ref: Nelson’s 20/e p 317-320; Refer pretext of this chapter for details on staging
109. c. Zinc, Vit A  Ref: Nelson’s 20/e p 318, Ghai 8/e p 111

High Yield Points


An episode of diarrhea, of presumed infectious etiology, which starts acutely but lasts for > 14 days
Principles of management are: (i) correction of dehydration, electrolytes & hypoglycemia, (ii) evaluation for infections & their manage-
ment, and (iii) nutritional therapy with supplemental multivitamins & minerals, twice the RDA
Vitamin A (1 lac–2 lac IU) & zinc (10–20 mg/day) are supplemented as both of them enhance the recovery from persistent diarrhea.

110. a. Tryptophan  Ref: Nelson’s 20/e p 325, Ghai 8/e p 118
Tryptophan can be converted to nicotinamide adenine dinucleotide (NAD) 60 mg tryptophan gives rise to 1 mg niacin.
111. c. Maize  Ref: Ghai 8/e p 118-119; Nelson 20/e 324-326
112. d. Keshan disease  Ref: Nelson’s 20/e p 344, Ghai 8/e p 112
Keshan disease is a disorder of selenium metabolism.
Diseases of copper metabolism are Wilson’s disease, Menkes kinky hair syndrome, Indian childhood cirrhosis.
113. c. Pyridoxine  Ref: Nelson’s 20/e p 326

Because of the importance of Vit B6 (Pyridoxine) in amino acid metabolism high protein intake can increase the
requirement for pyridoxine.
114. b. Seizure  Ref: Nelson’s 20/e p 342
Clinical Features of Vitamin E Deficiency:
Loss of deep tendon reflexes (areflexia) is usually the initial finding
Progressive cerebellar disease & posterior column dysfunction
Limb ataxia and truncal ataxia, dysarthria, nystagmus, positive Romberg test, decreased vibratory sensation seen
Some patients have pigmentary retinopathy & visual field constriction that can progress to blindness
Cognition & behavior can also be affected; Myopathy and cardiac arrhythmias are less-common findings, but may be seen
In premature infants, hemolysis & edema may be seen as a result of vitamin E deficiency

115. a. Keshan disease  Ref: Nelson’s 20/e p 344, Ghai 8/e p 112
116. a. A  Ref: Nelson’s 20/e p 318-319
Here, essential fatty acid has not been provided as an option; So, amongst the given options vitamin ‘A' is the best answer. Characteristic
changes as a result of vitamin A deficiency in the epithelia include: a proliferation of basal cells and hyperkeratosis dry, scaly
patches of skin, commonly on the arms, legs, shoulders, and buttocks.
117. d. 20 mg/day for 10–14 days  Ref: Nelson’s 20/e p 344, Ghai 8/e p 122
According to WHO guidelines, dose of Zinc for age group of 2–6 months is 10 mg/day & for > 6 months age, it is
20 mg/day for 14 days.
118. b. 1,00,000 IU  Ref: Nelson’s 20/e p 318-320, Ghai 8/e p 111-112
Dose of oral vitamin A for the treatment of Vitamin A deficiency consists of:
Age < 6 months 6–12 months > 1 year
Dose of vitamin A 50,000 IU 1,00,000 IU 2,00,000 IU

119. a. Folic acid  Ref: Nelson’s 20/e p 327, Ghai 8/e p 119
120. b. Mental retardation is the common presentation  Ref: Nelson’s 20/e p 648-649
Hartnup Disorder  (Refer to pretext of chapter on Inborn errors of metabolism, for details)

What is it? Defect in transport of monoamino-monocarboxylic (neutral) amino acids including tryptophan
Clinical features Most children remain asymptomatic; Major clinical manifestation is cutaneous photosensitivity: skin becomes
rough & red & with greater sun exposure, a pellagra-like, pruritic rash
Intermittent ataxia (wide-based gait) may be seen; Mental development is usually normal
Episodic psychiatric manifestations such as irritability, emotional instability, depression
56 Section 1: General Pediatrics

121. c. Zinc  Ref: Nelson’s 20/e p 327, Ghai 8/e p 111-112


Review of Pediatrics and Neonatology

122. a. Megaloblastic anemia  Ref: Nelson’s 20/e p 327, Ghai 8/e p 119
Goat’s milk is a poor source of folate; Hence, babies fed exclusively on Goat’s milk may develop Megaloblastic Anemia.
123. a. Riboflavin  Ref: Nelson’s 20/e p 322-323
Riboflavin deficiency causes cheilosis, glossitis, keratitis, conjunctivitis, photophobia, corneal vascularization & seborrheic
dermatitis.
124. a. Vit A  Ref: Nelson’s 20/e p 318-320, Ghai 8/e p 111-112
Hypervitaminosis A Hypervitaminosis D
Acute manifestation: Features of raised intracranial tension GIT: Vomiting, abdominal pain, constipation, pancreatitis
(pseudotumor cerebri) like headache, nausea, vomiting, drowsiness, Cardiac: Hypertension, decreased Q-T interval, arrhythmias
bulging fontanelles, papilledema & CN palsies CNS: Lethargy, hypotonia, confusion, hallucinations, coma
Chronic intoxication Kidney: Polyuria, dehydration, hypernatremia, nephrocalcinosis,
Headache vomiting, anorexia, hepatosplenomegaly nephrolithiasis
Dry, itchy desquamating skin and mucosa Metastatic calcification
Hyperostosis of bones (especially middle of shafts)
Though hypercalcemia is seen in hypervitaminosis D, as such, bony abnormalities are not seen.
125. a. Bitot spot; b. Xerophthalmia; c. Night blindness; e. Conjunctival xerosis  Ref: Nelson’s 20/e p 318, Ghai 8/e p 112
Tranta’s spots are whitish dots along the limbus present in bulbar vernal Keratoconjunctivitis (or spring catarrh).
126. a. Sexual infantilism; d. Poor wound healing; e. Acrodermatitis enteropathica  Ref: Nelson’s 20/e p 318
Clinical features of zinc deficiency:
•• Growth retardation, hypogonadism, anemia, diarrhea, hair loss, anorexia, dermatitis
•• Impaired immune function, skeletal abnormalities & poor wound healing
•• Acrodermatitis enteropathica is an autosomal recessive syndrome of severe zinc deficiency with dermatitis of extremities &
around orifices.
127. d. Chondrodysplasia punctata  Ref: Nelson’s 20/e p 814, Ghai 8/e p 181

High Yield Points


Maternal Warfarin therapy leads to Warfarin Embryopathy in the baby, that mainly involves skeletal system
Nasal hypoplasiaQ & epiphyseal & vertebral stippling (chondrodysplasia punctata)Q are most consistent & classical features
Pathophysiological
basis: interference in process of osteocalcin carboxylation in bone formation, which is vitamin K-dependent.
Chapter 4
Fluid and Electrolyte
Disturbances
 FLUID COMPARTMENTS High Yield Points   M
•• TBW is compartmentalized into intracellular (ICF) and extracellular fluid (ECF)
•• Total body water (TBW) constitutes
•• In fetus & newborn, ECF > ICF volume
75% of the body weight at birth
•• Normal postnatal diuresis → immediate decrease in ECF volume and declines to about 60% from the
•• Cellular growth → continued expansion of ICF volume age of 2 year onwards
•• By 1 year age, ICF:ECF volume approaches adult levels •• Premature infants have higher TBW
•• ICF volume is 30-40% of body weight & ECF volume is 20-25% beyond infancy than term infants
•• Extracellular fluid comprises: (i) Plasma (6%), (ii) Interstitial fluid, (iii) Transcellular (2-3%)
fluid (CSF, synovial fluid, digestive juices, intraocular, pleural, pericardial and peritoneal fluids).

Cations and Anions in Plasma & Intracellular Fluid (mEq/L) High Yield Points   M
•• Normal plasma osmolality is
285-295 mOsm/kg
•• Plasma Osmolality = 2 [Na] + [glu-
cose] /18 + [ BUN] /2.8, where, glu-
cose & BUN are in mg/dL.
•• The maximum urine osmolality is
approximately 1,200 mOsm/kg.

Question 1
What is the name of this test to be
done before drawing an arterial
blood gas sample:
Cations and Anions in Important Biological Fluids (mEq/L)
Fluid Na+ K+ Cl–
Gastric juice 60 10 85
Diarrheal* stool 10–90 10–80 10–110
CSF 140 3 120
*Upper end of the range is for cholera stools, lower end for non-cholera.

 MAINTENANCE OF ACID-BASE EQUILIBRIUM


Control of acid-base balance depends on kidneys, lungs, & buffers; normal pH is 7.35-7.45, PCO2 a. Virchow test

is 35-45 mm Hg & HCO 3 is 2-28 mEq/L. b. Water hammer test
c. Allen test
Abnormalities of Acid-Base M d. Trendelenburg test
•• In acidosis there is an increase in hydrogen ion concentration (pH < 7.35)
•• In alkalosis there is a decrease in hydrogen ion concentration (pH > 7.45)
•• During a simple metabolic disorder, there is respiratory compensation High Yield Points   
•• During a primary respiratory process, metabolic compensation is by kidneys Compensation in Metabolic Dis­orders
•• Unlike respiratory compensation, which occurs rapidly, it takes 3–4 days for the kidneys to
Expected
complete appropriate metabolic compensation
Disorder Compensation
Mixed Acid-Base Disorders Metabolic PCO2 = 1.5 × (HCO–3)
acidosis +8±2
•• In mixed acid-base disturbance, where pH is nearly normal (due to 2 opposing processes Metabolic PCO2 increases by
like metabolic acidosis and respiratory alkalosis), the presence of an acid base disturbance is alkalosis 7 for each 10 mEq/L
deduced because of the abnormal CO2 and/or HCO3– levels increase in HCO–3
58 Section 1: General Pediatrics

•• And also if the compensation is appropriate then a simple acid-base disorder is present
Review of Pediatrics and Neonatology

Mnemonic  M •• If the compensation is not appropriate, then a mixed disorder is present


Metabolic acidosis with normal anion •• Compensatory mechanism does not over correct the primary disturbance.
gap: “P-U-R-D-A”
•• Post hypocapnia Important Causes of Metabolic Acidosis M
•• Urinary tract diversions
Most common cause of metabolic acidosis in children: Diarrhea
•• Renal tubular acidosis
Amount of bicarbonate lost, depends on the volume of diarrhea and the bicarbonate concentration
•• Diarrhea
•• Ammonium chloride intake of stool.
Normal anion gap Increased anion gap
Diarrhea Lactic acidosis
Renal tubular acidosis (RTA): Tissue hypoxia: Shock / Hypoxemia / Severe anemia
Mnemonic  M
•• Distal (type I) RTA Liver / Kidney failure
Causes of Increased anion gap meta­ •• Proximal (type II) RTA Malignancy
bolic acidosis “KaLaM TIP •• Hyperkalemic (type IV) RTA Inborn errors of metabolism
•• K: Ketoacidosis, Kidney failure Urinary tract diversions
•• L: Lactic acidosis, Liver failure Medications: NRTI, Metformin, Propofol
Post hypocapnia
•• M: Malignancy, Medications Ketoacidosis: Diabetic ketoacidosis/ Starvation
•• T: Tissue hypoxia Ammonium chloride intake
Poisoning: Ethylene glycol / Methanol /Salicylate
•• I: Inborn errors of metabolism
•• P: Poisoning e.g. Ethylene gly­col
Important Causes of Metabolic Alkalosis
In children, metabolic alkalosis is most commonly secondary to emesis or diuretic use.
Causes of metabolic alkalosis in children can be grouped into:
Chloride-Responsive Chloride-Resistant
(Urinary Cl- <15 mEq/L) (Urinary Cl- >20 mEq/L)
High blood pressure Normal blood pressure
•• Adrenal adenoma or hyperplasia •• Bartter syndrome
•• Gastric losses
•• Glucocor ticoid-remediable •• Gitelman syndrome
•• Emesis
aldosteronism •• Autosomal dominant
•• Nasogastric suction
•• Renovascular disease hypoparathyroidism
•• Diuretics (loop or thiazide)
High Yield Points   M •• Renin-secreting tumor •• EAST syndrome
•• Elevated CO2 (Post-hypercapnia)
•• 17β-Hydroxylase deficiency •• Base administration
••Anion gap = [Na+] –[Cl–]– [HCO3–] •• Raised sweat chloride (Cystic
•• 11β-Hydroxylase deficiency
•• A normal anion gap is 4-11 fibrosis)
•• Cushing syndrome
•• An increased anion gap occurs when
•• Liddle syndrome
there is an increase in unmeasured
anions.
 SODIUM METABOLISM & ITS DISORDERS
Na is the dominant cation of ECF
•• Low intracellular Na is maintained by Na+, K+-ATPase
•• Normal daily sodium requirement is 3 mEq/kg

Regulation of Water and Sodium Balance in the Body

High Yield Points  


Compensation in Primary Respira­tory
Disorders
For every 10 mm increase in pCO2:
Respiratory Respiratory
Disorder
acidosis alkalosis
Acute HCO3– HCO3–
increases decreases
by 1 by 2
Chronic HCO3– HCO3–
increases by decreases
3.5 by 4
Chapter 4: Fluid and Electrolyte Disturbances 59

HYPONATREMIA

Section 1: General Pediatrics


High Yield Points
Definition: Serum sodium < 130 mEq/L.
•• Urine is the most important contri­
Etiology butor to normal water loss
•• Hypovolemic hyponatremia: •• Insensible losses represent 40% of
–– Extra renal losses: GIT (emesis, diarrhea), skin (sweating, burns) total maintenance fluid in infants &
–– Renal losses: Diuretics, Osmotic diuresis, polyuric phase of ATN, Juvenile nephronophthis, 25% in adolescents
ARPKD, Obstructive uropathy, Cerebral salt wasting, Proximal (Type II) RTA
•• Euvolemic: SIADH, Hypothyroidism, Glucocorticoid deficiency, Desmo­pressin, Water intoxication
•• Hypervolemic: CHF, Cirrhosis, Nephrotic syndrome, Renal failure, Hypo­albuminemia
•• Pseudo hyponatremia: Hyperglycemia, Mannitol, Sucrose.
Clinical Manifestations M

•• Cerebral edema is responsible for most of the symptoms of hyponatremia


•• Neurologic symptoms include lethargy, headache, seizures, coma, decreased reflexes
•• Hyponatremia can cause muscle cramps and weakness
•• A patient with chronic hyponatremia may have only subtle neurologic abnormalities with
a serum sodium level of 110 mEq/L, but other patient may have seizures because of an acute
decline in S. sodium level from 140 to 125 mEq/L.

Diagnostic Criteria for Syndrome of Inappropriate Antidiuretic Hormone High Yield Points
Secretion (SIADH) M
•• An overly rapid (>12 mEq/L over
Presence of: Absence of: the first 24 hr) correction of hypo-
•• Urine osmolality >100 mOsm/kg (usually > plasma) •• Renal, adrenal, or thyroid insufficiency natremic dehydration is associated
•• Serum osmolality <280 mOsm/kg and serum sodium •• Heart failure, nephrotic syndrome, or with an increased risk of central pon-
tine myelinolysis
<135 mEq/L cirrhosis
•• Urine sodium >30 mEq/L •• Diuretic ingestion
•• Reversal of “sodium wasting” and correction of •• Dehydration
hyponatremia with water restriction

Management of Hyponatremia
•• In patients with low ECF volume, volume is expanded using isonatremic solutions
•• Patients with coma or seizures should have sodium deficit corrected quickly 3-5 ml/kg of 3%
hypertonic saline to acutely raise S. sodium level by 5 mEq/L
•• SIADH is usually managed by restricting free water
•• In chronic hyponatremia, (to be corrected over 48 hrs)
Sodium deficit (mEq) = (Desired Na+-present Na+) × Weight × 0.6

HYPERNATREMIA
Definition: Hypernatremia is serum sodium >150 mEq/L. M

Etiology
Excessive Sodium Water Deficit Water and Sodium Deficits
•• Improperly mixed formula •• Diabetes Insipidus •• GIT losses–Diarrhea, Emesis
•• Excess sodium bicarbonate –– Nephrogenic / Central •• Cutaneous losses–Burns, Exces­
•• Intravenous hypertonic saline •• Increased insensible losses sive sweating
•• Hyper aldosteronism (Premature infants, Radiant •• Renal losses High Yield Points
Warmers) –– Osmotic diuretics, Diabetes
•• Inadequate intake (Ineffec­ –– Polyuric phase of ATN, CKD Brain hemorrhage is the most deva­
tive breast feeding, adipsia) –– Post- obstructive diuresis. stating consequence of untreated hyper­
natremia.
Clinical Manifestation
Doughy abdomen, Irritability, Lethargy, Fever, Seizures (Intracranial Hemorrhage, Stroke, dural
sinus thrombosis).
Management
•• Goal: Decrease the serum sodium by <12 mEq/L every 24 hrs (0.5 mEq/L/hr)
•• In a child with hypernatremic dehydration, the 1st priority is restoration of intravascular
volume with isotonic fluid (20–30% more than maintenance fluid)
•• In hypernatremic dehydration, time to be taken for correction is decided on basis of initial
sodium concentration:
60 Section 1: General Pediatrics

•• [Na+] 145-157 mEq/L: 24 hr


Review of Pediatrics and Neonatology

•• [Na+] 158-170 mEq/L: 48 hr


•• [Na+] 171-183 mEq/L: 72 hr
•• [Na+] 184-196 mEq/L: 84 hr
•• Acute severe hypernatremia can be corrected more rapidly because idiogenic osmoles have
not had time to accumulate Hemodialysis may be required in some cases.

 POTASSIUM METABOLISM & ITS DISORDERS


•• The intracellular concentration of potassium is 150 mEq/L
•• Plasma concentration of K is 3.5 to 5.5 mEq/L
•• Majority of body potassium is contained in muscle
•• Recommended daily intake of potassium is 1-2 mEq/kg

Factors Influencing Potassium Homeostasis

Intracellular shift

HYPERKALEMIA
Definition: Serum K+ >5.5 mEq/L.
Etiology:
High Yield Points Spurious Laboratory value Increased intake
•• Spurious hyperkalemia or pseu- •• Hemolysis •• Intravenous or oral
dohyperkalemia is very common in •• Tissue ischemia during blood drawing •• Blood transfusions
children because of the difficulties in •• Thrombocytosis / Leukocytosis
obtaining blood specimens. Transcellular shifts Decreased excretion
•• Acidosis •• Renal failure
•• Rhabdomyolysis / Hemolysis •• Primary adrenal disease: e.g. Addison disease,
•• Tumor lysis syndrome 21-Hydroxylase deficiency
•• Drugs: Succinylcholine, Digitalis, •• Hyporeninemic hypoaldosteronism: Urinary tract
β-blockers obstruction, Sickle cell disease
•• Malignant hyperthermia •• Renal tubular disease:
•• Hyperkalemic periodic paralysis •• Pseudohypoaldosteronism I & II, Bartter syndrome, type 2
•• Medications: ACE inhibitors, Angiotensin II blockers,
K-sparing diuretics

Clinical Features: Arrhythmias, muscle weakness (in severe hyperkalemia).


ECG changes due to hyperkalemia
Chapter 4: Fluid and Electrolyte Disturbances 61

Management

Section 1: General Pediatrics


High Yield Points
•• Mild (K+ 5.5–6 mEq/L) → Stop potassium intake and offending drugs
•• Moderate (K+ 6–8 mEq/L) → soda bicarbonate or a glucose insulin infusion •• Hypokalemia makes the heart
especially susceptible to digitalis-
•• Severe – (K+ > 8 mEq/L) → IV calcium gluconate (10%), it reverses the cardiac effects of
induced arrhythmias
hyperkalemia immediately •• TTKG = [K]urine / [K]plasma X (plasma
–– I/V or nebulized salbutamol also rapidly lowers serum K+ osmolality/urine osmolality), where
•• Refractory hyperkalemia may require Hemodialysis [K]urine = urine potassium concentra-
•• Sodium polystyrene sulphonate, potassium binding ion exchange resin, can be used for long tion and [K]plasma = plasma potassium
concentration
term management of hyperkalemia.
•• A TTKG > 4 in the presence of hypo­
kalemia suggests excessive urinary
HYPOKALEMIA losses of potassium.
Definition: Serum K+ <3.5 mEq/L.

Etiology Question 2
•• Increased losses ∙ Decreased stores: Malnutrition In which electrolyte abnormality,
the following ECG finding is seen?
–– Diarrhea ∙ Shift into intracellular compartment: Alkalosis;
–– Renal: RTA, Cystic kidneys
–– Endocrine: Cushing syndrome, Hyperaldosteronism
a. Hyponatremia b. Hypernatremia
Clinical Features c. Hypokalemia d. Hyperkalemia
Muscle weakness, hypotonia, Paralytic ileus, constipation, Polyuria & polydipsia.

ECG Changes
ECG changes due to hypokalemia

Management
•• The deficit of K+ should be corrected over a 24 hr period
•• I/V correction is required when the patient is unable to take orally, Serum K+< 2.5 mEq/L
or there are cardiac rhythm disturbances
•• Oral dose: 2-4 mEq/kg/day & IV dose is 0.5-1.0 mEq/kg, usually given over 1 hr
•• The infused fluid should ideally not contain > 40 mEq/L of potassium.
62 Section 1: General Pediatrics

 MAINTENANCE FLUID IN CHILDREN


Review of Pediatrics and Neonatology

M
Question 3
What is the sodium con­tent of this Indications of Maintenance IV Fluid in Children
IV fluid?
•• Maintenance IV fluids are used in a child who cannot be fed enterally.
•• Along with maintenance fluids, children may require concurrent replacement fluids if they
have continued excessive losses, such as may occur with drainage from a nasogastric (NG)
tube or with high urine output because of nephrogenic diabetes insipidus.
•• If dehydration is present, the patient also needs to receive deficit replacement

Body Weight Method for Calculating Daily Maintenance Fluid Volume


Body Weight Fluid Per Day Maintenance fluid rate
0–10 kg 100 mL/kg 4 mL/kg/hr
10–20 kg 1,000 mL + 50 mL/kg for each 40 mL/hr + 2 mL/kg/hr ×
a. 130 b. 134
kg > 10 kg (wt − 10 kg)
c. 150 d. 154
>20 kg 1,500 mL + 20 mL/kg for each 60 mL/hr + 1 mL/kg/hr ×
kg > 20 kg (wt − 20 kg)

Intravenous Fluids in Neonates


Daily fluid requirements during 1st wk of life (ml/kg/day)

Birth wt Day 1 Day 2 Day 3 Day 4 Day 5 Day 6 ≥ Day 7


<1500 g 80 95 110 120 130 140 150
≥1500 g 60 75 90 105 120 135 150

•• Initial fluid is 10% Dextrose with no electrolytes


•• Maintain glucose infusion rate 4-6 mg/kg/min
•• Na & K should be added to IV fluid after 48 hours
L at e s t U p d at e s
If children need IV fluids for routine
Composition of Intravenous Solutions
maintenance, initially use isotonic
crystalloids that contain sodium in Fluid [Na] [Cl–] [K+]
the range 131–154 mmol/litre. Normal saline (0.9% NaCl) 154Q 154 –
Half-normal saline (0.45% NaCl) 77 77 –
Ringer lactate 130Q 109 4Q

High Yield Points Adjusting Fluid Therapy for Altered Renal Output
•• Hypotonic fluids increase the risk of •• Replacement of insensible fluid losses (25–40% of maintenance) with D5 + NS
hyponatremia; •• Oliguria/anuria: Replace urine output mL/mL with D5 NS ± KCl
•• 0.2 NS is no longer recommended as
•• Polyuria: Replace urine output mL/mL with fluid based on measured urine electrolytes
a standard maintenance fluid
Treatment of dehydration due to diarrhea: Discussed in Pediatric Gastroenterology, chapter 14.

 SHOCK

What is it Shock is an acute syndrome characterized by inability to deliver adequate


oxygen to meet the metabolic demands of vital organs and tissues
Types Hypovolemic, cardiogenic, distributive, obstructive, and septic
Pathophysiology of Septic shock is a unique combination of:
High Yield Points M septic shock •• Hypovolemia from intravascular fluid losses occurs through capillary leak

•• Cardiogenic shock results from myocardial-depressant effects of sepsis
•• D5 + ½ NS + 20 mEq/L KCl is recom- •• Distributive shock results from decreased systemic vascular resistance.
mended in the child who is NPO and
does not have volume depletion or Compensatory Seen in early phases of shock, like increase in heart rate, stroke volume
any other risk factors mechanisms and vascular smooth muscle tone to maintain BP and preserve tissue
perfusion
Chapter 4: Fluid and Electrolyte Disturbances 63

Management of Septic Shock in Children

Section 1: General Pediatrics


High Yield Points   M
•• Hypovolemic shock is the most
common cause of shock in children
worldwide
•• Primary initiating factor in septic
shock is cytokine release

High Yield Points   M


•• Important complication of shock
is Multiple Organ Dys­ function
Syndrome (MODS).
•• MODS is defined as any alteration
of organ function that requires
medical support for mainte­nance;
•• Presence of MODS in patients
with shock substantially increases
probability of death.

L at e s t U p d at e s
What's new in Septic shock manage­ment?
•• Evidence points towards a trend for eliminating protocolized approach, it must be individualized to suit the patient physiology & setting
•• Evidence suggests that crystalloids whether balanced or not, are the most preferred
•• Restricting maintenance fluids after initial fluid resuscitation & use of diuretics for fluid removal termed as ‘de-resuscitation’ is associated with more
ventilator-free days & shorter length of ICU stay
•• Clinical signs like HR & SBP have been found to be poorly predictive of fluid responsiveness.
•• Static variables like CVP & preload estimates from thermo-dilution are also poorly predictive of fluid responsiveness.
•• Among the dynamic variables, respiratory variation in aortic blood flow peak velocity, consistently predicts fluid responsiveness in children
•• At bedside, hemodynamic changes induced during passive leg raising test have been reported to be good predictor of fluid responsiveness

Answer Keys for Image-Based Questions

Answers Explanations / Identifying features


1. Ans. c. Allen test A → Both radial & ulnar artery are occluded;
B → Ulnar artery kept occluded, Radial artery released; These are steps of Allen’s test
2. Ans. c. Hypokalemia Presence of U waves & flattening of T waves are ECG features of Hypokalemia
3. Ans. a. 130 This is a bottle of Ringer Lactate (RL) solution, which contains Na+ 130 mEq/L, Cl– 109 mEq/L & K+ 4 mEq/L
64 Section 1: General Pediatrics
Review of Pediatrics and Neonatology

Questions

 BODY FLUIDS 12. Widened anion gap is not seen in: M (MAHA PGM CET 2014)
a. Acute renal failure b. Diarrhea
1. At what age extracellular fluid is equal to the inter­
cellular fluid? M  (Recent Question 2017)
c. Lactic acidosis d. Diabetic ketoacidosis
a. 14 days b. 4 weeks
c. 2 months d. 3 months
 DISORDERS OF SODIUM & POTASSIUM
2. Total body water at birth is what proportion of body
13. All of the following are used in the treatment of
weight? M  (Recent Question 2017)
hyperkalemia except:
a. 60% b. 65%
a. Calcium gluconate b. Hypertonic saline
c. 70% d. 75%
c. Sodium bicarbonate d. Insulin
3. Which one of these is a 3rd space fluid loss?
14. Hypernatremic dehydration in children leads to irrita­
 (Recent Question 2016) (APPG 2016)
bility, restlessness, weakness, lethargy and fever. What
a. Fluid loss through a stoma
is the most devastating consequence of hyper­natremia
b. Tissue edema
in children? (Recent Question 2017)
c. Urinary losses d. Ryle's tube aspirate
a. Seizures b. Hyperglycemia
 ACID BASE BALANCE c. Brain hemorrhage d. Hypocalcemia
15. Diagnostic criteria for Syndrome of Inappropriate
4. A child with diarrhea has deep & rapid respiration. Most Antidiuretic hormone secretion include all except:
likely diagnosis is: (FMGE June 2018)  (Recent Question 2017)
a. Respiratory acidosis b. Respiratory alkalosis a. Urine Osmolality >100 mOsm/kg (Usually >Plasma)
c. Metabolic acidosis d. Metabolic alkalosis b. Serum Osmolality <280 mOsm/kg and
5. All of the following are causes of normal anion gap serum sodium <135 mEq/L
metabolic acidosis, except: M  (Recent Question 2017) c. Urine Sodium <30 mEq/L
a. Renal Tubular acidosis b. Diarrhea d. Reversal of “Sodium Wasting” and correction of hypona­
c. Acidosis due to Intestinal bacterial overgrowth tremia with water restriction
d. Urinary tract diversions 16. The clinical consequences of hypokalemia in skeletal
6. High osmolality with high anion gap is seen in: muscle include muscle weakness and cramps. What
a. Diarrhea (PGI May 2016) is the level of Serum Potassium at which Paralysis Is a
b. Distal renal tubular acidosis possible complication of hypokalemia?
c. Ammonium chloride intake a. Serum Potassium at 2.0 mEq/L (Recent Question 2017)
d. Urinary tract diversion b. Serum Potassium at 2.5 mEq/L
e. Methanol poisoning c. Serum Potassium at 3.0 mEq/L
7. Metabolic compensation in a child with respiratory d. Serum Potassium at 3.5 mEq/L
acidosis takes how much time? (Recent Question 2016) 17. Tall T wave is seen in? M  (Recent Question 2016)
a. < 1 day b. 1–2 days a. Hyperkalemia b. Hypokalemia
c. 3–4 days d. More than 7 days c. Hypercalcemia d. Hypocalcemia
8. A Normal-anion-gap metabolic acidosis occurs in 18. Sine wave in ECG is seen in? (WB PGMEE 2016)
patients with? M  (MAHA PGM CET 2015)
a. Hyperkalemia b. Hypokalemia
a. Diabetic ketoacidosis b. Methyl alcohol poisoning
c. Hypercalcemia d. Hypocalcemia
c. Diarrhea d. Acute kidney injury
19. Immediate treatment for hyperkalemia + bradycardia is?
9. High anion gap acidosis is seen in all except: M   (WB PGMEE 2016)
 (JIPMER 2014)
a. Sodium bicarbonate b. Ca gluconate
a. Diarrhea b. Salicylate poisoning c. Salbutamol  d. Kayexalate
c. Acute renal failure d. Lactic acidosis
20. Which of the following inherited disorders is a Potassium
10. An 11-year-child with failure to thrive, metabolic Channelopathy? (COMEDK 2016)
acidosis with increased anion gap. Not true regarding
the differential diagnosis: (Dec 2014)
a. Hyperkalemic periodic paralysis
b. Hypokalemic periodic paralysis
a. Insulin dependent diabetes mellitus
c. Beckers's disease
b. Chronic renal failure
d. Anderson Tawil syndrome
c. Renal tubular acidosis d. Inborn errors of metabolism
11. Respiratory acidosis is characterized by primary: M  21. Requirement of potassium in child is: M 
 (MAHA PGM CET 2014)  (Recent Question 2013, AIPMEE 2006)
a. Deficit of carbonic acid b. Excess of carbonic acid a. 1–2 mEq/kg b. 4–7 mEq/kg
c. Deficit of bicarbonate d. Excess of bicarbonate c. 10–12 mEq/kg d. 13–14 mEq/kg
Chapter 4: Fluid and Electrolyte Disturbances 65

22. A 10-day-old neonate is posted for pyloric stenosis in 34. Molar ratio of Sodium and Glucose in WHO ORS is:

Questions
surgery. The investigation report shows a serum calcium a. 1 : 1 b. 1 : 2  (JIPMER 2014)
level of 6 mg/dL. What information would you like to c. 1 : 4 d. 2 : 1
know before you supplement calcium to this neonate? 35. Sodium content of ReSoMal is:
 (AIIMS Nov 2004)  (Recent Question 2016, 2013)
a. Blood glucose b. Serum protein a. 90 mmoL/L b. 60 mmoL/L
c. Serum bilirubin d. Oxygen saturation c. 45 mmoL/L d. 30 mmoL/L
23. Hypernatremic dehydration is characterized by: 36. Sodium content in mmol/L in WHO oral rehydration
a. S. Sodium > 150 mmol/L (PGI Dec 2003) solution is: M  (COMEDK 2011)
b. Signs of dehydration are minimal a. 20 b. 80
c. ECF volume increased c. 90 d. 111
d. Rapid correction is required 37. Amount of ORS given in a 4 kg child with diarrhea with
e. Shift of water from ECF to ICF some dehydration: M  (WBPG 2011)
24. A breastfed baby presents with hypernatremia (S. Sodium a. 200 mL b. 300 mL
>170 mEq/L). His urine sodium is 70 mEq/L. Which of the c. 400 mL d. 800 mL
following is the most likely cause? (AIIMS Nov 2000)
38. What is the amount of trisodium citrate in WHO ORS? M
a. Diabetes insipidus b. Acute tubular necrosis
a. 1.5 gm b. 2.5 gm  (MAHA PG 11)
c. Severe dehydration d. Excessive intake of sodium
c. 2.9 gm d. 3.9 gm
25. In a child who has diarrhea and vomiting with inadequate 39. What is the correct composition (per liter) of WHO
water intake, which of the following is seen: reduced osmolarity ORS?  (DNB June 2011)
 (AIIMS June 2000)
a. NaCl 2.29 g; KCl 1.1 g; Glucose 13.1 g; Citrate 2.5 g
a. Intracellular dehydration with hypernatremia b. NaCl 2.49 g; KCl 1.3 g; Glucose 13.3 g; Citrate 2.7 g
b. Intracellular dehydration with hyponatremia c. NaCl 2.69 g; KCl 1.5 g; Glucose 13.5 g; Citrate 2.9 g
c. Extracellular dehydration with hyponatremia d. NaCl 2.89 g; KCl 1.7 g; Glucose 13.7 g; Citrate 3.1 g
d. Extracellular dehydration with hypernatremia
40. Concentration of K+ in ORS is _____ mEq/L M
a. 20 b. 40  (WBPG 2010)
 ORAL REHYDRATION (ORS) c. 90 d. 10
26. Concentration of K+ (mEq/L) in most commonly used  INTRAVENOUS FLUID ADMINISTRATION
ORS is: M  (Recent Question 2017)
a. 10 b. 20 41. Na content (mEq/L) in Normal saline is: M
c. 65 d. 75 a. 111 b. 130 (JIPMER May 2016)
27. The predominant buffering system in the blood plasma c. 135 d. 154
is: (APPG 2016) 42. Through a peripheral line, max % of dextrose that can be
a. Hemoglobin b. Plasma proteins given is: (JIPMER May 2016)
c. K2HPO4 & KH2PO4 d. NaHCO3 & H2CO3 a. 10% b. 12.5%
28. The total osmolality of low-osmolality WHO ORS is? c. 15% d. 25%
 (MAHA PGM CET 2015) 43. Green IV cannula is: M  (Recent Question 2016)
a. 200 mOsm/L b. 245 mOsm/L a. 16G b. 18G
c. 305 mOsm/L d. 311 mOsm/L c. 20G d. 22 G
29. Initial Fluid of choice for diarrhea in an infant? M  44. In case of massive bleeding during surgery which sized
 (AIIMS May 2015) cannula is used? M  (AIIMS Nov 2015)
a. ORS b. Salt water a. 16G b. 20G
c. Sugar water d. Dextrose c. 22G d. 24G
30. A child has serum osmolality 270 and urine osmolality 45. In children with burns maintenance intravenous fluid
1200. What is the most probable diagnosis? normally given is: (MAHA PGM CET 2016)
a. Water deprivation  (Recent Question 2015) a. Ringer's lactate b. 5% Dextrose
b. SIADH c. Normal saline d. Dextrose saline
c. Central diabetes insipidus
46. Maximum concentration of dextrose that can be given
d. Nephrogenic diabetes insipidus
through peripheral vascular line in neonate:
31. Dosage of intravenous fluid for 2 months old child in dia­  (Recent Question 2016)
rrhea with severe dehydration: (Recent Question 2015)
a. 5% b. 10%
a. 100 mL/kg in 6 hours b. 50 mL/kg in 6 hours c. 12.5% d. 25%
c. 75 mL/kg in 6 hours d. 80 mL/kg in 6 hours
47. Parklands formula used for Burn’s resuscitation is:
32. Low osmolar ORS composition: M  (WBPG 2014)  (MAHA PGM CET 2014)
a. Na 90 + 311 mOsm/L b. Na 75 + 245 mOsm/L a. 4 mL/kg/% TBSA b. 5 mL/kg/%TBSA
c. Na 60 + 245 mOsm/L d. Na 60 + 240 mOsm/L c. 6 mL/kg/%TBSA d. 8 mL/kg/% TBSA
33. A 5-year-child comes with no dehydration. Plan A was 48. A 6-year-old drowsy child came in emergency with
proposed for treatment of dehydration. How much ORS history of vomiting, loose motion for 3 days. On exami­
should be given per loose stool?  (Recent Question 2014) nation he had very sunken eyes, hypothermia, skin on
a. 0–50 mL b. 50–100 mL pinching takes time to revert; the diagnosis is:
c. 100–200 mL d. 200–300 mL  (Recent Question 2013, AIIMS June 2002)
66 Section 1: General Pediatrics

a. No dehydration b. Mild dehydration a. 360 mOsm/kg b. 306 mOsm/kg


Review of Pediatrics and Neonatology

c. Some dehydration d. Severe dehydration c. 312 mOsm/kg d. 318 mOsm/kg


49. What percentage of weight loss in children constitutes 61. A 5-year-old boy passed 18 loose stools in last 24 hours
severe dehydration? (JIPMER 2013, 2009) and vomited twice in last 4 hours he is irritable but
a. >4% loss of body weight b. >5% loss of body weight drinking fluids. The optimal therapy for this child is:
c. 5% loss of body weight d. 7% loss of body weight a. Intravenous fluids  (AIPGMEE 2003)
50. A child with acute diarrhea with signs of dehydration, b. Oral rehydration therapy
Na+ 125, K+3, HCO3– 16, pH 7.23; IV Fluid to be given is: c. Intravenous fluid initially for 4 hours followed by oral fluids
d. Plain water
a. Normal saline  (TN PGMEE 2013, AIIMS May 2005)
62. Kallu, a 2-year-old child weighing 6.7 kg presents in the
b. 3% saline casualty with h/o vomiting and diarrhea for last 2 days.
c. N/3 saline + 10% dextrose On examination, skin pinch over the anterior abdominal
d. N/3 saline + 5% dextrose wall goes quickly to its original position. Interpretation
51. A 2-year-old child presents with loose stools. On of skin pinch test in this child will be: (AIPGMEE 2002)
examination she has signs of severe dehydration. If her a. No dehydration b. Some dehydration
weight is 10 kg, what would be the volume of fluid to be c. Severe dehydration
given in the first 4 hours? (JIPMER 2012) d. Skin pinch cannot be evaluated in this child
a. 1000 mL b. 100 mL
c. 200 mL d. 500 mL  SHOCK
52. Na+ content of RL in mmol/L: M  (WBPG 2012)
a. 120 b. 125 63. A 5-year-old child presented with continuous fever and
c. 130 d. 135 features of sepsis with a BP of 90/60 mm Hg, Pulse rate
144/min and Respiratory rate of 30/min. What is the
53. Rehydration therapy in a 2-year-old child with diarrhea
initial fluid of choice for management?
with some dehydration: (AIPGMEE 2012)  (AIIMS May 2016)
a. 30 mL/kg in 1 hour – 70 mL in 5 hours a. 10 mL/kg of hydroxyethyl starch
b. 30 mL/kg in 30 mm – 70 mL/kg in 2½ hours b. 10 mL/kg of 10% dextrose
c. 20 mL/kg in 30 mm – 50 mL/kg in 2½ hours c. 20 mL/kg of 0.9% normal saline
d. 75 mL/kg in 4 hours d. 20 mL/kg of 0.45% normal saline
54. Daily maintenance fluid for a child weighing 10 kg is: M 64. Following pathogenetic mechanisms operate in septic
 (JIPMER 2011, 2010, 2009) shock except? (MAHA PGM CET 2015)
a. 1000 mL/day b. 800 mL/day a. Increased peripheral vascular resistance
c. 500 mL/day d. 1200 mL/day b. Venoconstriction
55. Daily water requirement in child weighing 30 kg, height c. Direct toxic endothelial injury
123 cm and BSA of 1 m2 is: (DNB June 2011) d. Activation of complement
a. 1300 mL b. 1700 mL 65. Septic Shock is due to:  (Recent Question 2015)
c. 2000 mL d. 2500 mL a. Vasodilatation b. Decreased cardiac output
56. When severe dehydration in a neonate occurs, amount c. Endothelial damage d. All of the above
of fluid replacement in 1st hour: (DNB Dec 2010) 66. Rehydration therapy in a 2-year-old severely dehydrated
a. 20–40 mL/kg b. 5–10 mL/kg child is: (Recent Question 2014)
c. 10–15 mL/kg d. 15–20 mL/kg a. 30 mL/kg in 1 hour, 70 mL in 5 hours
57. Parkland formula is: (JIPMER 2010) b. 30 mL/kg in 30 minutes, 70 mL/kg in 2½ hours
a. Percentage of burn area × body wt × 4 c. 20 mL/kg in 30 minutes, 80 mL/kg in 2½ hours
b. Percentage of burn area × body wt/4 d. 75 mL/kg in 4 hours
c. Percentage of burn area × body surface area × 4 67. A child presents with hypovolemic shock, anuria and
d. Percentage of burn area × body surface area/4 respiratory acidosis, treatment is: (WBPG 2010)
58. In malnourished children the following parameters are a. RL b. Dopamine
reliable indicators of dehydration except: c. Dobutamine d. Inj. Sodium Bicarbonate
 (COMEDK 2009, 2006) 68. An alert 6-month-old child is brought with vomiting
and diarrhea. RR-45/min, HR-130/min, SBP-85 mm Hg.
a. Skin turgor b. Dry buccal mucosa Capillary refilling time is 4 seconds, Diagnosis is 
c. Oliguria d. Thirst  (AIIMS May 2010)
59. In a dehydrated child with hypertrophic pyloric stenosis a. Early compensated hypovolemic shock
the fluid of choice is: M  (WBPG 2007) b. Early decompensated hypovolemic shock
a. Normal saline b. 5% dextrose c. Late compensated hypovolemic shock
c. Ringers lactate d. Darrows solution d. Late decompensated shock due to SVT
60. Which of the following would be plasma osmolality 69. The most sensitive indicator of depletion of intravascular
of child with plasma Na+ 125 mEq/L, glucose of 108 volume in infant is: M  (AIIMS May 2010, AIPGMEE 2009)
mg/dL, and blood urea Nitrogen (BUN) of 140 mg/dL? a. Cardiac output b. Stroke volume
 (AIIMS May 2005, Nov 2003) c. Heart rate d. Blood pressure
Chapter 4: Fluid and Electrolyte Disturbances 67

Answers with Explanations


Answers with Explanations

 BODY FLUIDS
1. b. 4 weeks  Ref: Avery’s 8/e p 373

High Yield Points


•• In fetus & newborn, ECF volume is larger than ICF volume.
•• Normal postnatal diuresis causes an immediate decrease in the
ECF volume.
•• This is followed by continued expansion of ICF volume, which
results from cellular growth.
•• By 1 year of age, the ratio of the ICF to ECF volume approaches
adult levels.

2. d. 75%  Ref: Nelson’s 20/e p 346, Ghai 8/e p 70

High Yield Points


•• Total body water (TBW) constitutes 75% of the body weight at birth
•• During the 1st year of life, TBW decreases to 60% of body weight & remains at this level until puberty
•• By the end of puberty TBW in males remains at 60%, but TBW in females decreases to 50% of body weight

3. b. Tissue edema  Ref: Harrison’s 19/e p 298, 462 “Third spacing” Refers to fluid loss into interstitial spaces, caused by increases
in vascular permeability and/or a reduction in oncotic pressure (hypoalbuminemia) ”

•• This occurs in sepsis syndrome, burns, pancreatitis, nutritional hypoalbuminemia, and peritonitis
•• Third-space losses are isotonic & can cause significant volume depletion, leading to ↑ADH production and water retention, which can cause
hyponatremia if the patient receives hypotonic fluid.

 ACID BASE BALANCE


4. c. Metabolic acidosis  Ref: Nelson’s 20/e p 369–83
5. c. Acidosis due to Intestinal bacterial overgrowth  Ref: Nelson’s 20/e p 375
In patients with intestinal bacterial overgrowth, excessive bacterial metabolism of glucose into d-lactic acid can cause a lactic acidosis;
so it is a high anion gap metabolic acidosis;
6. e. Methanol poisoning  Ref: Nelson’s 20/e p 375-376
In Methanol poisoning, there is high osmolality with high anion gap; In all others given in the options, anion gap is usually normal;
7. c. 3-4 days  Ref: Nelson’s 20/e p 369-383, Ghai 8/e p 85-86
Unlike respiratory compensation, which occurs rapidly, it takes 3–4 days for the kidneys to complete appropriate metabolic
compensation.
8. c. Diarrhea  Ref: Nelson’s 20/e p 369-383, Ghai 8/e p 85-86
Diarrhea and Renal tubular acidosis (RTA) are two important causes of normal anion gap metabolic acidosis.
9. a. Diarrhea  Ref: Nelson’s 20/e p 369-383, Ghai 8/e p 85-86
10. c. Renal tubular acidosis  Ref: Nelson’s 20/e p 369-383, Ghai 8/e p 85-86
11. b. Excess of carbonic acid  Ref: Nelson’s 20/e p 369-383, Ghai 8/e p 85-86
Respiratory acidosis is caused by CO2 retention → excess carbonic acid.
12. b. Diarrhea  Ref: Nelson’s 20/e p 369-383, Ghai 8/e p 85-86
68 Section 1: General Pediatrics

 DISORDERS OF SODIUM & POTASSIUM


Review of Pediatrics and Neonatology

13. b. Hypertonic saline  Ref: Nelson 20/e p 357-362


Drugs used in the treatment of acute hyperkalemia include: Calcium gluconate, Sodium bicarbonate, Insulin infusions & salbutamol
nebulisation
14. c. Brain hemorrhage  Ref: Nelson’s 20/e p 352
•• Brain hemorrhage is the most devastating consequence of untreated hypernatremia.
•• Seizures & coma are possible sequelae of hemorrhage, although seizures are more common during correction of hypernatremia.
Mechanism of Intracranial hemorrhage in hypernatremia:
Due to hypernatremia, extracellular osmolality increases

Water moves out of brain cells, leading to a decrease in brain volume

Tearing of intracerebral veins and bridging blood vessels as the brain moves away from skull and meninges

Subarachnoid, subdural, and parenchymal hemorrhages
15. c. Urine Sodium <30 mEq/L  Ref: Nelson’s 20/e p 354-355; Refer pretext of this chapter for details
In SIADH, Urine sodium should be more than 30 mEq/L
16. a. Serum Potassium at 2.0 mEq/L  Ref: Nelson’s 20/e p 361
Paralysis is a possible complication, generally only at potassium levels <2.5 mEq/L. It usually starts in legs and moves to arms;
17. a. Hyperkalemia  Ref: Nelson’s 20/e p 357-362, Ghai 8/e p 78
18. a. Hyperkalemia  Ref: Nelson’s 20/e p 357-360
Hyperkalemia produces a sequence of changes in ECG:
•• 1st ECG finding: Narrowing and peaking (tenting) of the T wavesQ
•• Further elevation of K+ leads to AV conduction disturbances, ↓ P-wave amplitude & widening of QRS interval
•• Severe hyperkalemia eventually causes with a slow sinusoidal “sine-wave” pattern followed by asystole.


Peaked T waves Sine wave
19. b. Ca gluconate  Ref: Nelson’s 20/e p 357, Ghai 8/e p 80-81
IV calcium gluconate (10%) is used in treatment of Severe hyperkalemia (K+ > 8 mEq/L), to reverse the cardiac effects of
hyperkalemia immediately.
20. d. Anderson Tawil syndrome  Ref: Harrison’s 19/e p 462
Channelopathies are progressive syndromic cluster of symptoms not localizing to single lesion;
Important Channelopathies: (All of these are autosomal dominant & show episodic weakness)
Calcium Channel Sodium Channel Potassium Channel
Feature Hypokalemic PP Hypokalemic PP Paramyotonia Congenita Andersen-Tawil syndrome
Age of onset Adolescence Early childhood Early childhood Early childhood
Myotonia No Yes Yes No
Frequency of Daily to yearly May be 2-3/day With cold, usually rare Daily to yearly
attacks of weakness

21. a. 1–2 mEq/kg  Ref: Nelson’s 20/e p 357-360, Ghai 8/e p 76-78
•• Daily requirement of potassium in a child is 1–2 mEq/kg/day and sodium is 3 mEq/kg/day.
22. b. Serum protein  Ref: Nelson’s 20/e p 357, Ghai 8/e p 80-81
In hypoalbuminemia, the total calcium is not an adequate surrogate for the ionized Ca2+ concentration.
Each 1 gm/dL of albumin in the serum binds about 0.8 mg/dL of calcium.
23. a. S. Sodium > 150 mmol/L and b. Signs of dehydration are minimal  Ref: Nelson’s 20/e p 357, Ghai 8/e p 75-76
Discussing about the options one by one
Chapter 4: Fluid and Electrolyte Disturbances 69

Answers with Explanations


a. True It is the definition of hypernatremia
b. True In hypernatremic dehydration, shift of water from intracellular space to extracellular space → blood pressure and
urine output maintained → minimal signs of dehydration
c. False ECF volume usually remains normal
d. False Rapid correction of hypernatremia is harmful; The goal is to decrease the serum sodium by < 12 mEq/L every 24
hours, a rate of 0.5 mEq/L/hour
e. False Water shifts from ICF to ECF in hypernatremic dehydration

24. d. Excessive intake of sodium Ref: Indian Pediatrics 2002; 39:193-196; Nelson’s 20/e p 346-349;
This is a controversial question. Discussing about the options one by one,
a. Diabetes insipidus → serum sodium is usually high and urinary sodium low
b. Acute tubular necrosis → It is a cause of intrinsic ARF in which though urinary sodium is high with a low urine osmolality but
Serum Sodium is usually low
c. Severe Dehydration → It represents a prerenal state with a low urine sodium
d. Excessive intake of sodium can lead to high serum sodium in blood and urine
This case presents with hypernatremia with high urinary sodium (Normal urinary Na <20 meq/L)
High urinary sodium in association with hypernatremia points to sodium load being the etiological agent
There have been multiple case reports of high breast milk sodium being the underlying cause responsible for hypernatremia in
newborn; Important causes of elevated breast milk sodium are: Mastitis, Adrenal insufficiency & cystic fibrosis
25. a. Intracellular dehydration with hypernatremia Ref: Nelson’s 20/e p 350-351, Ghai 8/e p 291-297
•• Diarrhea results in depletion of both sodium and water; Because diarrhea is hypotonic, with typical sodium concentration of 35-
65 meq/L, water losses exceeds sodium losses, potentially leading to hypernatremia
•• Most children with gastroenteritis do not have hypernatremia because they drink enough hypotonic fluid to compensate for stool
water losses. But in this case, as the baby has inadequate water and fluid intake, he is at risk of developing hypernatremic
dehydration.

 ORAL REHYDRATION (ORS)


26. b. 20  Ref: Nelson’s 20/e p 1869-1872, Ghai 8/e p 294-295;
27. d. NaHCO3 & H2CO3  Ref: Nelson’s 20/e p 369-370
The bicarbonate buffer system is very effective as a result of the high concentration of bicarbonate in the body (24 mEq/L) and the fact
that it is an open system, so lungs increase CO2 excretion when required
28. b. 245 mOsm/L  Ref: Nelson’s 20/e p 1869-1872, Ghai 8/e p 294-295
29. a. ORS  Ref: Nelson’s 20/e p 1869-1872, Ghai 8/e p 294-295
•• Oral rehydration solution (ORS) containing appropriate quantities of sodium and glucose, promote optimum absorption of fluid
from the intestine
•• Other clear liquids, such as rice water, fruit juice, and sports drinks, are inappropriate for rehydration of young children with
significant stool loss.
30. b. SIADH  Ref: Nelson’s 20/e p 1869-1872, Ghai 8/e p 294-295; For diagnostic criteria of SIADH, refer pretext
Discussing about the options one by one:
a. Water deprivation Both serum osmolality and urine osmolality increase
b. SIADH Serum osmolality decrease and urine osmolality increases
c. Central diabetes insipidus Serum osmolality increase and urine osmolality decreases in diabetes insipidus (both
central and nephrogenic)
d. Nephrogenic diabetes insipidus

31. a. 100 mL/kg in 6 hours  Ref: Nelson’s 20/e p 1869-1872, Ghai 8/e p 294-295
Treatment of severe dehydration in children:
Age First give Then give Total treatment
< 12 months 30 mL/kg in 1 hour 70 mL/kg in 5 hours 100 mL/kg in 6 hours
1–5 years 30 mL/kg in 30 minutes 70 mL/kg in 2½ hours 100 mL/kg in 3 hours

32. b. Na 75 + 245 mOsm/L  Ref: Nelson’s 20/e p 1869-1872, Ghai 8/e p 294; see ans 26 below
33. c. 100–200 mL  Ref: Nelson’s 20/e p 1869-1872, Ghai 8/e p 294
Plan A: For Patients without physical signs of dehydration, amount of ORS to be given after each loose stool:
70 Section 1: General Pediatrics
Review of Pediatrics and Neonatology

Age < 24 months 2–10 years > 10 years


Amount of ORS 50–100 mL 100–200 mL As much as wants
34. a. 1 : 1  Ref: Approach to Practical Pediatrics 2nd ed By Narang Manish/ pg 24
Molarity of sodium and glucose in ORS should be 1:1, which helps in the efficient transport of sodium and water.
35. c. 45 mmol/Liter  Ref: Nelson’s 20/e p 1869-1872, Ghai 8/e p 294
ReSoMAL (Rehydration solution for severely Malnourished child):
As severely malnourished children are deficient in potassium and have relatively high levels of sodium, ReSoMal has a
lower sodium concentration and higher potassium concentration to meet the electrolyte needs of these children.
ReSoMal Low Osmolarity New Composition of WHO ORS (Low osmolarity)
Component (mEq/L) WHO ORS (mEq/L)
Glucose 125 75 Name of the constituent Amount in gm
Sodium 45 75 Sodium chloride 2.6
Potassium 40 20
Glucose, anhydrous 13.5
Chloride 70 65
Citrate 7 10 Potassium chloride 1.5
Magnesium 3 Trisodium citrate dihydrate 2.9
Zinc 0.3
Copper 0.045
Osmolarity 300 245

36. b. 80  Ref: Nelson’s 20/e p 1869-1872, Ghai 8/e p 294


WHO ORS contains 75 mmol/L sodium; 80 is the closest option to it; So it is the best answer amongst the given options.
37. a. 300 mL  Ref: Ghai 8/e p 294 75 mL/kg ORS is to be given in some dehydration, i.e. 75 × 4 = 300 mL in this case.
38. c. 2.9 g  Ref: Nelson’s 20/e p 1869-1872, Ghai 8/e p 294; Refer Ans 32;
39. c. NaCl 2.69 g; KCl 1.5 g; Glucose 13.5 g; Citrate 2.9 g  Ref: Nelson’s 20/e p 1869-1872, Ghai 8/e p 294-295
40. a. 20  Ref: Nelson’s 20/e p 1869-1872, Ghai 8/e p 295

 INTRAVENOUS FLUID ADMINISTRATION


41. d. 154  Ref: Nelson’s 20/e p 387, Ghai 8/e p 572; Refer pretext for details;
42. b. 12.5%  Ref: Nelson’s 20/e p 574, AIIMS NICU Protocols 2014
43. b. 18G  Ref: AIIMS Protocol; Green IV cannula is 18G in size; Refer to the list in Ans 42;
44. a. 16G  Ref: AIIMS Protocol
During massive bleeding wide bore cannula is to be used. Among the given options, 16G is the widest and so the best answer.
Size of IV cannula 14G 16G 18G 20G 22G 24G 26G
Color coding Orange Gray Green Pink Blue Yellow Violet

45. a. Ringer’s lactate  Ref: Nelson’s 20/e p 572


Ringer lactate is usually used for fluid resuscitation in a child with burns.
46. c. 12.5% Ref: Nelson’s 20/e p 574, AIIMS NICU Protocols 2014\
It is recommended to avoid using > 12.5% dextrose infusion through a peripheral vein due to risk of thrombophlebitis.
47. a. 4 mL/kg/% TBSA Ref: Nelson’s 20/e p 572, Ghai 8/e p 294-295
•• For most children with burns, the Parkland formula is an appropriate starting guideline for fluid resuscitation.
•• According to it, 4 mL lactated Ringer solution/kg/% BSA burned is used
•• Half of the fluid is given over 1st 8 hr, calculated from the time of onset of injury; remaining over next 16 hours.
48. d. Severe dehydration  Ref: Nelson’s 20/e p 574, Ghai 8/e p 293-297
Parameters No dehydration Some dehydration Severe dehydration
Look at Sensorium Well alert “Restless, irritable” “Iethargic”, floppy
Eyes Normal Sunken Very sunken and dry
Tears Present Absent Absent
Mouth & Tongue Moist Dry Very dry
Thirst Drinks normally, not thirsty “Thirsty, drinks eagerly” “Drinks poorly/ not able to drink”
Feel Skin pinch Goes back quickly “Goes back slowly” “Goes back very slowly”
Decide No signs of dehydration If the patient has 2 or more signs, including at least 1 “sign”
In this case, eyes are very sunken & skin “goes back very slowly” in a lethargic (drowsy) child, so it is a case of severe dehydration.
49. d. 7% loss of body weight Ref: Nelson's 20/e p 1869-1872, Ghai 8/e p 572
Severe dehydration is defined as weight loss of >10% of body in an infant; >6% in an older child.
Chapter 4: Fluid and Electrolyte Disturbances 71

50. a. Normal saline Ref: Nelson's 20/e p 1869-1872

Answers with Explanations


In the given scenario, the child has dehydration with low sodium, along with that metabolic acidosis, which is most probably
secondary to hypovolemia; So, the fluid of choice in this case is an isotonic fluid to correct dehydration metabolic acidosis; So normal
saline is the best answer.
51. a. 1000 mL Ref: Nelson's 20/e p 387, Ghai 8/e p 294-295
In a 2-year-old child with severe dehydration, fluid given over 1st 30 minutes is 30 mL/kg over next 2 and 1/2 hour is 70 mL/kg; So
fluid given over 3 hours is 100 mL/kg. In this case, as weight of child is 10 kg, amount of fluid to be given is 10 x 100 = 1000 mL.
52. c. 130 Ref: Nelson's 20/e p 387, Ghai 8/e p 572; Refer pretext of this chapter for details;
53. d. 75 mL/kg in 4 hours  Ref: Nelson's 20/e p 387, Ghai 8/e p 572
54. a. 1000 mL/day  Ref: Nelson's 20/e p 387, Ghai 8/e p 572; Refer pretext of this chapter for details;
55. b. 1700 mL  Ref: Nelson's 20/e p 386-387, Ghai 8/e p 572
Fluid requirement in a child in this child weighing 30 kg is: For first 10 kg (1–10 kg) 100 x 10 = 1000 mL
For next 10 kg (11–20 kg) 50 x 10 = 500 mL; For next 10 kg (21–30 kg) 20 x 10 = 200 mL; So, total requirement = 1700 mL.
56. a. 20–40 mL/kg  Ref: Nelson’s 20/e p 386, Ghai 8/e p 572
In a child < 1 year old, with severe dehydration, Fluid given over 1st hour is 30 mL/kg.
57. a. Percentage of burn area × body wt × 4  Ref: Nelson’s 20/e p, Ghai 8/e p
58. a. Skin turgor  Ref: Nelson’s 20/e p 386, Ghai 8/e p 572
Skin turgor is not a reliable parameter for assessing dehydration in a child with malnutrition, as it may appear to be decreased
due to decreased subcutaneous fat.
59. a. Normal saline  Ref: Nelson’s 20/e p 1798-1799
•• Surgical patients with pyloric stenosis can develop hypochloremic, hypokalemic, or metabolic alkalosis
•• Hydrogen ion re-absorption occurs with an accompanied potassium ion excretion.
•• The resulting hypokalemia leads to the excretion of hydrogen ions in the face of alkalosis, a paradoxic aciduria
•• Treatment includes replacement of volume deficit with isotonic saline & potassium once adequate urine output is ensured.
60. b. 306 mOsm/kg  Ref: Nelson’s 20/e p 1869, Ghai 8/e p 294
Formula for calculating Plasma Osmolality = 2 x (Na+) + glucose/ 18 + BUN/2.8.
So, in this child, plasma osmolality = 2 x 125 + 108/18 + 140/2.8 = 306 mOsm/L.
61. b. Oral rehydration therapy  Ref: Nelson’s 20/e p 1869, Ghai 8/e p 294
The child is irritable but drinking fluids. He thus falls in category of some dehydration and needs to be managed with Plan B.
Treatment plan B is oral Rehydration therapy and hence the answer here.
62. d. Skin pinch cannot be evaluated in this child Ref: Nelson's 20/e p 1869, Ghai 8/e p 294
The given child in question has a weight of 6.7 kg at 2 years, against expected of 12 kg (2 x 2 + 8 =12).
So this child is malnourished; For assessment of dehydration in a malnourished child, skin pinch is less useful.

 SHOCK
63. c. 20 mL/kg of 0.9% Normal Saline Ref: Nelson's 20/e p 517; Ghai 8/e p718; Refer pretext for details;
64. a. Increased peripheral vascular resistance Ref: Nelson's 20/e p 518, Ghai 8/e p 718
In Septic shock in children, there is decreased systemic vascular resistance, due to peripheral vasodilatation;
65. d. All of the above Ref: Robbins 9/e p 131-32; Refer pretext of this chapter for details
66. b. 30 mL/kg in 30 minutes, 70 mL/kg in 2½ hours  Ref: Ghai 8/e p 715-718

High Yield Points


•• Intraosseous (IO) cannulation is recommended for patients for whom IV access proves difficult or unattainable
•• If venous access is not available within 1 minute in a child with cardiopulmonary arrest, an IO needle should be placed in anterior proximal
tibia
67. a. RL  Ref: Nelson's 20/e p 517, Ghai 8/e p 715-718
Initial resuscitation of any child with shock consists of Pushing IV fluid boluses (RL or NS) of 20 mL/kg up to 60 mL/kg
until perfusion improves or unless rales or hepatomegaly develop; Next step is use of Inotropes like Dopamine.
68. a. Early compensated hypovolemic shock  Ref: Nelson's 20/e p 523-528, Ghai 8/e p 715-718
This is a case of hypovolemia as h/o vomiting and diarrhea is present. In this case the infant has poor peripheral perfusion (shock) as
evident from a prolonged capillary refill time, but BP is normal, so it is a compensated shock.
Hypotension is defined as a systemic blood pressure less than the 5th percentile of normal for age, as given below:
•• < 60 mm Hg in term neonates (0–28 days) •• < 70 mm Hg + (2 x age in years) in children 1–10 years
•• < 70 mm Hg in infants (1–12 months) •• < 90 mm Hg for >10 years of age.

69. c. Heart rate  Ref: Nelson’s 20/e p 523-525, Ghai 8/e p 715-718
In infants with shock, compensatory increases in cardiac output are achieved primarily by “tachycardia” rather than by increase in
stroke volume. Heart rates of 190–210/min are common in infants with compensated shock.
Chapter 5
Genetics and
Genetic Disorders
Question 1  NORMAL HUMAN GENOME M

What does the given Karyo­


type •• Humans contain 46 chromosomes comprising 22 pairs of autosomes numbered
show? 1 to 22 & a pair of sex chromosomes (2 X chromosomes in females and a X and Y in males)
•• Chromosomal numbers are assigned in descending order of lengths of chromosomes
•• Each chromosome has a short arm “p” & long arm “q” joined at centromere.

Classification of Chromosomes Depending on Location of Centromere

a. Normal male
b. Normal female Important Terminologies
c. Down syndrome
d. Turner syndrome •• Codominance—When both alleles of a gene pair are fully expressed in a heterozygote, e.g.
blood group
•• Reduced penetranceQ—Some individuals inherit the disease but are phenotypically
normal. 50% penetrance means that 50% of those who carry the gene express the trait
•• Variable expressivityQ—If a trait is seen in all individuals carrying the mutant gene but is
expressed differently among individuals, e.g. tuberous sclerosis
•• Pleiotropism—When single gene affects multiple traits.
•• Epistasis—>1 gene affecting a single phenotype is epistasis
•• Chimerism—Two cell lines derived from two different zygotes exist
High Yield Points •• Mosaicism: Presence of 2 or more different cell lines in the same individual
•• Overall, missense or nonsense TYPES OF MUTATIONS M
muta­tions are the most common
•• Gain-of-function mutations typi­­cally Class Group Type
cause dominantly inherited diseases Substitution Synonymous Silent
•• In Haploinsufficiency, maintenance
Non-synonymous Missense/Nonsense
of a normal phenotype requires pro-
teins produced by both copies of a Deletion or Insertion Multiple of 3 (codon) 1 codon altered
gene Not multiple of 3 Frameshift

PEDIGREE ANALYSIS M

What is a Pedigree Chart?


Diagram of a family history which illustrates relationships among family members; it shows which
family members are affected with specific medical conditions.

How Many Generations should be Examined?


Information for a three generation pedigree should be obtained
Important Terminologies:
•• Proband—Affected individual who brings the family to notice
•• Consultand—Person who comes for genetic counseling
•• First degree relatives—Individuals who share half of their genetic material with the proband-
brothers, sisters, children, parents.
Chapter 5: Genetics and Genetic Disorders 73

Important Symbols Used

Section 1: General Pediatrics


Different lines in a pedigree chart M

How is Adoption Depicted on a Pedigree Chart?

Question 2
Identify the mode of inheritance in
the given pedigree:

Systematic Approach to Pedigree Analysis M

Pattern Inheritance
All or most children of a mother are affected Mitochondrial
If a least one of the parents always have the disorder Dominant a. Autosomal dominant
If neither parent has the disorder because they are heterozygous Recessive b. Autosomal recessive
c. X-linked dominant
If both males and females are affected, with almost equal frequency Autosomal
d. X-linked recessive
Father to son transmission of trait does not occur X linked
More males affected; Affected sons usually born to unaffected mothers X linked recessive
More females affected; Affected sons must have an affected mother X linked dominant
It is passed from father to all sons Y linked dominant
High Yield Points
CLASSIFICATION OF GENETIC DISORDERS M Indications for Genetic Counseling
•• Advanced maternal (> 35) or
Single gene disorders Disorders in which single genes are altered, since they follow Mendelian paternal (> 50) age
(Mendelian disorders) mode of inheritance. E.g., Hemophilia •• Consanguinity
•• Previous history of a child with birth
Chromosomal disorders Entire chromosome or segments of them are missing, duplicated or altered. defects or a genetic disorder
Can be disorders of chromosomal number or structure, e.g. Down syndrome •• Personal or family history suggestive
of a genetic disorder
Non-Mendelian disorders • Trinucleotide repeats   •  Genomic imprinting •• High-risk ethnic groups; known
•• Mitochondrial inheritance •  Gonadal mosaicism carriers of genetic alterations
•• Ultrasound or prenatal testing
Multifactorial disorders Results from a combination of multiple genetic and environmental causes.
suggesting a genetic disorder.
E.g: cleft palate/cleft lip, neural tube defects, diabetes, asthma, hypertension
74 Section 1: General Pediatrics

 SINGLE GENE DISORDERS (MENDELIAN DISORDERS)


Review of Pediatrics and Neonatology

Mnemonic M

“Vishwa Hindu Parishad is DOMINANT” AUTOSOMAL DOMINANT INHERITANCE M


for Autosomal Dominant diseases
V Von Willebrand disease/Von Hippel Features
Lindau disease
H Hypercholesterolemia, Huntington •• Affected individuals present in every
disease, Hereditary spherocytosis
P Pseudo hypoparathyroidism  generation
D Dystrophia myotonica  •• Any child of an affected parent has a
O Osteogenesis imperfecta 50% risk of inheriting the disorder
M Marfans syndrome 
I Intermittent porphyria 
•• Phenotypically normal parents do not
N Neurofibromatosis  transmit disease to the offspring
A Achondroplasia, ADPKD •• Males and females equally affected
N Noonan syndrome 
T Tuberous sclerosis

Pedigree showing Autosomal dominant inheritance

Question 3 IMPORTANT AUTOSOMAL DOMINANT DISORDERS


What is the disease in which these
hand abnormalities are seen? Marfan Syndrome
Autosomal dominant disorder caused by mutation in fibrillin-1 (FBN1) gene on long arm of
chromosome 15 (15q21)
Important Clinical Features of Marfan Syndrome
B
Skeletal CVS Eyes
•• Overgrowth of long bones •• Mitral valve prolapse → heart •• Dislocation of ocular lens
•• Reduced US : LS ratio failure, pulmonary hyperten- (ectopia lentis) in 60-70%
•• Arm span > height sion & death in infancy •• Early and severe myopia
A C •• Pectus carinatum/excavatum •• Arrhythmias •• Flat cornea,
a. Down syndrome •• Thoracolumbar scoliosis •• Aortic aneurysm, dissection •• Retinal detachment
b. Osteogenesis imperfecta •• Arachnodactyly and rupture at the level of •• Early cataracts or glaucoma
c. Turner syndrome •• Pes planus sinuses of Valsalva
d. Marfan syndrome

High Yield Points   


•• Mitral insufficiency is the leading cause of morbidity and mortality in children with Marfan syndrome
•• Aortic aneurysm, dissection & rupture are the most life-threatening manifestations of Marfan
Mnemonic M
syndrome
“M-A-R-F-A-N-S” for Marfan syndrome:
•• Mitral valve prolapseQ
Neurofibromatosis (NF) & Tuberous Sclerosis (Refer to chapter on Pediatric Neurology)
•• Aortic AneurysmQ/Arm span ↑
•• Reduced US:LS ratio Achondroplasia & Osteogenesis Imperfecta (Refer to chapter on Musculoskeletal disorders)
•• Fibrillin deficiencyQ
•• Arachnodactyly (long fingers)Q Noonan syndrome
•• Negative Nitroprusside test •• Most commonly caused by mutation in PTPN11 gene (autosomal dominant)
•• Supero-temporal subluxation of lens •• Similarities with Turner syndrome: short stature, webbed neck, cubitus valgus.
(ectopia lentis)Q/Skeletal changes
Differences between Turner Syndrome & Noonan Syndrome

L at e s t U p d at e s Turner syndrome Noonan syndrome


•• Noonan syndrome is one of the most Karyotype 45, XO Karyotype normal
common genetic diseases associated
with congenital heart defect, being Infertile Fertile but delayed puberty
second only to Down syndrome. Seen only in females May be seen in males or females
•• Incidence of congenital heart disease
is 30% in those with webbed neck Intelligence normal Intellectual disability present
and 9% of those with normal neck. Noonan syndrome
Chapter 5: Genetics and Genetic Disorders 75

AUTOSOMAL RECESSIVE (AR) INHERITANCE

Section 1: General Pediatrics


M
M n e m o n i c M

•• Males and females are equally affected Important autosomal recessive dis-
eases: “ABCDEFGHI”
•• Parents of an affected child are asymptomatic
A: AlbinismQ, AlkaptonuriaQ, Ataxia
hetero­zygous carriers of the gene TelangiectasiaQ
•• If both parents are heterozygotes: 25% of children B: Beta (thalassemiaQ, Sickle cell anemiaQ)
will have normal genotype, 50% will be hetero­ C: Cystic fibrosisQ, Congenital adrenal
hyperplasia
zygotes and 25% will have the risk of homozygous D: Deafness
state E: Emphysema (α1-antitrypsin deficiency)
•• If one parent is heterozygote and another F: Friedrich’s AtaxiaQ
G: Gaucher diseaseQ, Galactosemia
parent is affected (homozygous): probability of H: Homocystinuria, HemochromatosisQ
disease increases to 50 % to each child (Pseudo Autosomal recessive
I: Inborn errors of metabolismQ
dominance).

X-LINKED RECESSIVE INHERITANCE M

•• Males are more commonly and


more severely affected than
females
•• Female carriers have a 25% risk
for having an affected son, a
25% risk for a carrier daughter, M n e m o n i c M

and a 50% chance of having a X linked recessive diseases: “Girls Do


normal child Care For CHAWAL”
G: G6PD deficiency
•• Affected males have carrier
D: Duchenne muscular dystrophy,
daughters and unaffected Diabetes insipidus
sons because they pass their C: Color blindness
X linked recessive F: Fragile-X syndrome, Fabry disease
X chromosome to all of
C: Chronic granulomatous disease
their daughters and their Y
H: Hemophilia A and B, Hunter disease
chromosome to all of their sons A: Agammaglobulinemia
•• Male-to-male transmission W: Wiskott-Aldrich syndrome
A: Albinism
excludes X-linkage.
L: Lesch-Nyhan syndr, Lowe syndr

X-LINKED RECESSIVE INHERITANCE


When mother is carrier, father normal: When father is affected, mother is normal:

Question 4
Identify the disease this newborn is
suffering from:

a. Rickets
b. Down syndrome
c. Osteogenesis imperfecta
d. Achondroplasia
76 Section 1: General Pediatrics

X-LINKED DOMINANT INHERITANCE


Review of Pediatrics and Neonatology

M n e m o n i c M

X-linked dominant disorders: •• All daughters, but no sons of an affected


“Red Ribbon for All Children” man have the condition: (Criss cross inheri­
Red- X-linked hypophosphatemic tance)
RicketsQ •• Both male & female offspring of affected females
Ribbon- Rett syndromeQ have 50% risk of inheriting it
For -Fragile X syndromeQ
•• Heterozygous females and hemizygous
All- Alport syndromeQ
Children -Charcot–Marie–Tooth
males are affected
disease •• Majority of alive patients are heterozygous
females
Pedigree with X-linked dominant
inheritance
High Yield Points
•• All Ehlers-Danlos syndrome types are
 NON-MENDELIAN DISORDERS
AD except type V, VI, VII A. TRINUCLEOTIDE REPEAT DISORDERS
•• All DNA repair abnormalities/
chromosome instability syndrome Important Features
are AR except HNPCC (AD).
•• All Glycogen storage diseases are •• Due to an increase in number of nucleotide repeats above a certain threshold
AR, except, GSD IX—phosphorylase •• Length of the nucleotide repeat often correlates with the severity of the disease
kinase deficiency (XR). •• When repeat length increases from one generation to the next, disease manifestations
•• All urea cycle disorders are AR may worsen or be observed at an earlier age: AnticipationQ
except ornithine transcarbamylase •• Repeat number may also vary in a tissue-specific manner; E.g. in myotonic dystrophy, the
deficiency (XR). CTG repeat may be tenfold greater in muscle tissue than in lymphocytes.
•• All lysosomal storage disorders are
AR except Fabry’s disease (XR)
Selected Trinucleotide Repeat Disorders
•• All Mucopolysaccharidoses are AR
except Hunter’s disease (XR). Disease Repeat Inheritance Gene Product
•• All osteogenesis imperfecta are AD
except type II, III & VII. Fragile X-syndrome (FRAXA) CGG XR FMR-1 proteinQ
Dystrophia myotonica (DM) CTG AD Myotonin protein kinase
Huntington’s disease (HD) CAGQ AD Huntingtin
High Yield Points Spinocerebellar ataxia type 1 CAG AD Ataxin 1
All porphyrias are AD except Friedreich’s ataxia (FRDA1) GAAQ AR Frataxin
congenital erythropoietic porphyria
& 5 ALA dehydratase deficiency
porphyria (AR)
Fragile X Syndrome M

Caused by A trinucleotide repeat mutation in familial mental retardation-1 (FMR1) gene


Epidemiology Second most common genetic cause of mental retardation after Down syndrome
Genetic basis •• Normal population: 29–55 CGG repeats in the FMR1 gene
•• Carrier males and females: carry pre-mutationsQ 55–200 CGG repeats that can expand
Question 5 during oogenesis
What is the diagnosis in this child •• Full mutation (fragile X syndrome): 4000 repeats
with intellectual disability? Clinical •• Long face with a large mandible, large ears, large testiclesQ (macro-orchidism)
features •• Most distinctive feature is post-pubertal males macro-orchidismQ
•• Hyperextensible joints, a high arched palate, and mitral valve prolapse
Anticipation Clinical features worsen with each successive generation,Q as the number of repeats
increases with each generations (Sherman’s paradox)

L at e s t U p d at e s
•• Inhibitors of mGluR (overex­pressed in fragile X) are under­going clinical trials.
•• Minocycline (lowers MMP9) results in improvements in anxiety, mood & clinical Global Impression
scale in Fragile X syndrome.

a. Down syndrome B. MITOCHONDRIAL INHERITANCE


b. Fragile X syndrome
•• Since mitochondrial DNA is present in cytoplasm, it is exclusively derived from maternal
c. Noonan syndrome
d. Prader Willi syndrome side (ovum), as only head of sperm (from father), contributes in zygote formation.
•• All offsprings receive their mitochondrial DNA from their mothers only
Chapter 5: Genetics and Genetic Disorders 77

•• At the cellular level, the proportion of mutated mitochondrial DNA (mutation load) determines

Section 1: General Pediatrics


phenotypic expression called threshold of expression
High Yield Points   M
•• Both male and female children born to a affected mother will inherit the disease. Mitochondrial Diseases
•• Will not be transmitted from an
Examples of Mitochondrial Disorders (Mnemonic: K-L-M-N-O-P) affected father to his children
•• All affected daughters will transmit
Disease Phenotype the disease
KSS Kearn-Sayre syndrome •• Sons will be affected but will not
External ophthalmoplegia, heart block, retinal pigmentation, ataxia transmit the diseaseQ
•• Heteroplasmy: presence of more
LHON Leber hereditary optic neuropathy than 1 type of mitochondrial DNA
Bilateral subacute or acute painless optic atrophy within a cell.
MELAS Mitochondrial encephalomyopathy, lactic acidosis, and stroke like episodes; may
manifest only as diabetes
MERRF Myoclonic epilepsy, ragged red fibers in muscle, ataxia, increased CSF protein,
sensorineural deafness, dementia
NARP Neuropathy, Ataxia, Retinitis Pigmentosa, Leigh disease
Loss of central vision leading to blindness in young adult life
CPEO Chronic progressive external Ophthalmoplegia
Bilateral ptosis, ophthalmoplegia & proximal muscle weakness
Pearson Pancreatic insufficiency, pancytopenia, lactic acidosis
syndrome

C. GENOMIC IMPRINTING
•• Occurs when the phenotypic expression of a gene depends on the parent of originQ Question 6
•• Classic examples: Prader-Willi syndromeQ and Angelman syndrome,Q Beckwith- Identify the mode of inheritance in
Wiedemann syndrome,Q Russell-Silver syndrome.Q the pedigree shown below:

D. UNIPARENTAL DISOMY M

•• Occurs when both chromosomes of a pair have been inherited from a single parent
•• 2 types:
–– Uniparental isodisomy: Both chromosomes or chromosomal regions are identical
–– Uniparental heterodisomy means that the 2 chromosomes are different members of a
a. Autosomal recessive
pair, both of which were still inherited from 1 parent. b. X-linked recessive
c. X-linked dominant
Mechanisms in Prader-Willi & Angelman SyndromeQ d. Mitochondrial inheritance
Deletions Deletion of Paternal genesQ after maternal gene silencing-Prader-Willi syndrome
(70% cases) Deletion of Maternal genesQ after paternal gene silencing-AngelMan syndrome
Uniparental disomy Uniparental disomy of paternalQ chromosome 15-Angelman syndrome
(20-25 % cases) Uniparental disomy of maternalQ chromosome 15- Prader-Willi syndrome
Defective imprinting Defective imprinting of paternal chromosome in Prader Willi syndrome
(1-4%) Defective imprinting of maternal chromosome in Angelman syndrome

 CHROMOSOMAL DISORDERS Question 7


The structural abnormality of
Disorders of Chromosome Structure M chromosome in which one arm
is lost and remaining arm is
Translocations: Involve breakage & rejoining of chromosomes; can be balanced or unbalanced;
duplicated, as shown below is
Two main types of translocations:
called?
•• Robertsonian translocations:  long (q) arms of two  acrocentric  chromosomes
(13, 14, 15, 21 and 22) fuse at the centromere; They are considered to be balanced
because missing genetic material from short (p) arms is redundant
•• Reciprocal translocations: 2 different chromosomes have exchanged fragments
with each other.
Inversions Breakage & reinsertion of a piece of a chromosome upside down at same location
Ring Occur when the ends of a chromosome join together to form a ring as the result of a
chromosomes loss of genetic material from both ends of the chromosome. a. Ring chromosome
b. Isochromosome
Deletions Loss or absence of a piece of a chromosome, resulting in monosomy of a region
c. Translocation
Duplications Presence of an extra piece of a chromosome, resulting in trisomy of a region d. Mutation
78 Section 1: General Pediatrics

Micro-deletion Syndromes
Review of Pediatrics and Neonatology

Mnemonic M

•• Deletion of Paternal gene → Prader Entity Chr Features


Willi syndrome
CATCH 22 22q11 Cardiac anomaly, Anomalous face, Thymus hypoplasia/aplasia, Cleft
•• Deletion of Maternal gene → Angel
palate, and Hypocalcaemia <Mnemonic>
Man syndrome
Previously called Di-George syndrome
or
VelocardiofacialQ syndrome
Williams 7q11 •• Autosomal dominant condition; elastin (ELN) defect
syndrome •• Distinct facial features: upturned nose, long philtrum,Q slightly
wide-spaced eyes, periorbital fullness, and full lips
•• Very pleasant, outgoing, and friendly personality, very talk-
ative, learning disabilities, exceptional ear for music
•• Increased risk for cardiovascular diseaseQ (supravalvular pul­
monic stenosis, hypertension), hypercalcemia
Wolf-Hirschhorn 4p16.3 Dysmorphic facial appearance (“Greek warrior helmet
syndrome appearance of the nose)Q, growth deficiency, MR
Cri-du-chat synd 5p15.2 “Cat-like” cry, microcephaly, growth deficiency, MR

Question 8 Disorders of Chromosome Number M


Identify the mode of inheritance in
the given pedigree: Euploidy Chromosomes are an exact multiple of 23 (46, 69, 92 in humans)
Aneuploidy •• If a cell’s chromosomes are not an exact multiple of 23(2n+1; 2n-1)
•• Most common and clinically significant type of human chromosome abnormality
•• Cause of aneuploidy: NondisjunctionQ during meiosis and Anaphase lag
•• Most common type: Trisomy

 DOWN SYNDROME M

•• Most common chromosomal abnormalityQ


a. Autosomal dominant •• Incidence: 1:800-1:1000 live births
b. Autosomal recessive •• Genetics:
c. X-linked dominant –– 95% Non-disjunctionQ
d. X-linked recessive –– 3% TranslocationQ
–– 1–2% MosaicismQ.

Risk of Down Syndrome Increases with Increase in Maternal Age


Maternal age Incidence of Down syndrome
20 years 1 in 1500

30 years 1 in 900

40 years 1 in 100
Question 9 45 years 1 in 30
The following clinical features are
seen in which disease?
Mnemonic M

Important clinical features of Down Syndrome


I C a PROBLEM Somewhere
•• Incurved 5th finger (clinodactylyQ/Intellectual disability High Yield Points   M
•• Congenital heart diseaseQ
•• Acute Leukemia/Alzheimer’s diseaseQ/Atlantoaxial instabilityQ/ •• Most common cardiac defect in Down
•• Protruding tongue syndrome is Endocardial cushion
•• Round face defect or Atrioventricular septal defect
•• Occiput flat/Open, wide fontanelle Q •• Most common cause of Intestinal
•• Brushfield spots in irisQ/Brachycephaly obstruction in Down syndrome is
a. Turner syndrome •• Low (depressed) nasal bridge Duodenal atresia
b. Fragile X-syndrome •• Epicanthic foldQ/Ears low-set & dysplastic •• Trisomy 16: Most common trisomy
c. Achondroplasia •• Mongoloid slantQ/Mental retardation leading to miscarriage; Incompatible
d. Down syndrome •• Sandle gapQ/Simian palmar creaseQ with life
Chapter 5: Genetics and Genetic Disorders 79

Methods for Antenatal Screening for Down Syndrome

Section 1: General Pediatrics


M
High Yield Points
In 1st Trimester •• A form of fetal programming where
2 genes are inherited but 1 is turned
Screening method Detection rate
off through environmentally induced
Maternal age + Biochemical markers (b-hCG, PAPPA)Q 70% modification is called ‘epi­genetc
imprinting’
Maternal age + Radiology (USG- NT>3 mm) 80%-83% •• CG methylation is the most common
Combined test: Mat. age + Radiology (USG-NT>3 mm) 82%-87% epigenetic change in humans.
+ Biochemical markers (b-hCG, PAPPA)

*PAPPA is Pregnancy Associated Plasma Protein A.

In 2nd Trimester
Screening method Detection rate
Triple ScreenQ (b-hCG + AFP + U estriol)Q 67%
Quadruple screen (Triple screen + Inhibin A) 77%

Best screening test: for Down syndrome is Integrated test, that consists of:
Mat. age + T1- (NT>3 mm + PAPPA) + T2- (TS + Inhibin A) → Detection rate: 94-96%Q.

Confirmatory Methods for Antenatal Diagnosis of Down Syndrome M


L at e s t U p d at e s
•• Chorionic villi sampling (11-13 weeks)Q
→ Karyotype from fetal genetic material
•• Amniocentesis (14-16 weeks)Q. Circulating fetal DNA in maternal
blood can be used for antenatal
Recurrence risk of Down syndrome, when a couple already has a child with Down syndrome diagnosis of genetic disorders like
Down syndrome, non-invasively
depends on the karyotype of the affected child & his parents:

Karyotype of Karyotype of parents


affected child Father Mother Recurrence risk
Trisomy 21 N N ~1%
N N ~1%
Translocation 21q21q
Either parent carrierQ 100%Q
N N ~1%
Translocation
carrier N 1-3%
other chr with chr 21
N carrier 10-15%

Other Chromosomal Trisomies and their Clinical Findings

Mnemonic
Important Features of Trisomy 13 (Patau Syndrome)
“CMC DOCTOR OPD”
C: Cutis aplasia D: Deafness O: Others (Renal abnormalities)
M: Microphthalmia, Microcephaly O: Ocular hypotelorism P: Polydactyly
C: Congenital Heart diseases C: Cleft lip & cleft palate D: Developmental delay
T: Trisomy 13
O: hOLOprosencephaly
R: Rib abnormalities

Mnemonic M

Important features of Trisomy 18: “ROCKY Mountain”


R: Rocker bottom foot C: Cardiac defects Y: MicrocephalY
O: Overlapping fingers K: Kidney malformations M: Mental retardation Mnemonic M

•• Trisomy 13: Patau syndrome (Thir-


TURNER SYNDROME M
teen – PaTau)
•• Karyotype: 45, XO,Q Always seen in FEMALESQ •• Trisomy 18: Edward syndrome (Eigh-
•• Short stature, Webbed neck, Cubitus valgus, Short 4th metacarpal teen – Edward)
80 Section 1: General Pediatrics

•• Intelligence remains normal (No Intellectual disability)Q


Review of Pediatrics and Neonatology

Mnemonic M
•• Females with Turner syndrome are infertileQ (streak ovaries & rudimentary uterus)
Important features of Klinefelter •• Most common heart disease in Turner syndrome: Bicuspid aortic valveQ (50%)
Syndrome: “MALE KING” > Coarctation of Aorta (30%).
•• Males only
•• Atrophic testis
•• Long legs Question 10 M
•• Elevated FSH & LH
The following clinical features are seen in which
•• Karyotype: 47, XXY
•• Infertility disease?
•• No secondary sex characteristics
•• Gynecomastia

Mnemonic M

Features of Turner Syn­drome: “See


there is A Baby CLOWN”
S: Short stature, Sensorineural hearing
A B C
loss, Short 4th metacarpal
A: Amenorrhea primary
B: Barr body absent
C: Cardiac anomalies
L: Lymphoedema, Low thyroid
O: Ovaries under developed (streak
D E ovaries)
a. Noonan syndrome b. Turner syndrome W: Webbed neck
N: Nipples widely placed
c. Achondroplasia d. Down syndrome   
Some distinctive features seen in various syndromes:
Distinct feature Name of disorder
Broad thumbs and great toes Rubinstein—Taybi syndrome
Absent clavicle Cleidocranial dysostosis
Heterochromic iridis Waardenburg syndrome
Mitten hands Apert syndrome
Inverted nipples Congenital disorder of glycosylation
Webbing of neck Turner and Noonan syndrome
Eversion of the lateral third of lower eyelid Kabuki makeup syndrome
Alopecia, skin rash Biotinidase (multiple carboxylase) deficiency
Pale, brittle, steely hair Menkes kinky hair disease

Important Syndromes with Distinctive Facial Features

William syndrome: Periorbital fullness, Fragile X syndrome: Long face, large ears,
upturned nose (elfin facies), long philtrum, and prominent jaw.
full lips, prominent cheeks
Contd...
Chapter 5: Genetics and Genetic Disorders 81

Contd...

Section 1: General Pediatrics


Question 11
The following karyotype is seen in
which of the following syndromes?

a. Bloom syndrome
Pierre Robin syndrome (sequence): Treacher Collin syndrome: Anti- b. Fragile X-syndrome
Micrognathia, Retrognathia, Glossoptosis. mongoloid slant, coloboma of lower eyelids, c. Angelman syndrome
underdeveloped cheeks, malformed ears. d. Cri du chat syndrome

Seckel syndrome: Small head, narrow bird- Noonan syndrome: Hypertelorism, down­
like face, beak-like nose, micrognathia. slanting palpebral fissures, epicanthic folds,
micrognathia, low-set ears, webbed neck
Question 12
Karyotype of a patient shows the
following. What is the clinical
abnormality that is expected?

Goldenhar syndrome: Limbal dermoids, Kabuki syndrome: Arched eyebrows, long a. Gynecomastia with long thin limbs
preauricular skin tags, ear malformations, palpebral fissures everted lower lids. b. Short stature with polydactyly
hemifacial microsomia c. Webbed neck with widely spaced
nipples
d. Rocker bottom feet

Crouzon syndrome: Exophthalmos, hyper­ Russell-Silver syndrome: Triangular face,


telorism, brachycephaly & facial hypoplasia prominent forehead, narrow chin, small jaw
& downturned corners of the mouth
82 Section 1: General Pediatrics

 IMPORTANT INVESTIGATIONS FOR GENETIC DISEASES


Review of Pediatrics and Neonatology

Karyotype: Orderly arrangement of chromosomes from chr 1 to 22 (according to length) followed by sex chromosome; detects
chromosomal abnormalities

Multiplex Ligation-dependent probe Amplification (MLPA): Can be used to detect sub-microscopic chromosomal abnormalities in
a targeted manner

Fluorescent in situ hybridization (FISH): A molecular cytogenetic DNA microarray: Used to identify thousands of different gene
technique that uses fluorescent probes to test for the presence or expressions at the same time
absence of specific chromosome regions
Chapter 5: Genetics and Genetic Disorders 83

Section 1: General Pediatrics


Answer Keys for Image-Based Questions

Answers Explanations/Identifying features


1. Ans. b. Normal female This is a normal karyotype with 46 chromosomes, including 2 copies of X chromosome with no Y, as seen in
a normal female
2. Ans. a. Autosomal dominant Affected individuals present in every generation & both males & females affected; So, it is autosomal
dominant inheritance
3. Ans. d. Marfan syndrome A → Arachnodactyly
B → Steinberg sign (thumb tip extends from palm of hand, in a closed fist)
C → Walker-Murdoch sign (thumb & 5th finger overlap each other, when opposite wrist is gripped)
4. Ans. d. Achondroplasia This newborn has short limbs, large head & trident hand, suggestive of Achondroplasia
5. Ans. b. Fragile X syndrome The child has elongated face with large ears suggestive of Fragile X syndrome
6. Ans. d. Mitochondrial All children (boys & girls) of affected mothers are affected
inheritance
7. Ans. b. Isochromosome It is an isochromosome, formed when one arm of a chromosome is lost and remaining arm is duplicated
8. Ans. b. Autosomal recessive Both the parents of the affected children are carriers; 1 parent of each of the parents are also carriers; So it is
autosomal recessive inheritance;
9. Ans. d. Down syndrome A → Mongoloid slant, epicanthic fold;
B → Single transverse palmar (simian) crease
C → Sandle gap;
All these are seen in Down syndrome
10. Ans. b. Turner syndrome A → Webbed neck
B → Low posterior hairline
C → Shield shaped chest with widely spaced nipples
D & E → Lymphedema
11. Ans. d. Cri du chat syndrome There is loss of genetic material from the short arm of chromosome 5 (Chr 5p- deletion), as seen in Cri du chat
syndrome
12. Ans. a. Gynecomastia with The given karyotype shows 2 copies of X chromosome, along with 1 Y chromosome;
long thin limbs This 47,XXY is seen in Klinefelter syndrome, in which Gynecomastia & tall stature are seen
84 Section 1: General Pediatrics
Review of Pediatrics and Neonatology

Questions

 IMPORTANT TERMINOLOGIES 11. In a family, mother is normal while father has a genetic
disease. All their daughters are carriers and sons are
1. What is proband in pedigree? M  (Recent Question 2017) normal, what is the pattern of inheritance of disease?
a. Male child of diseased b. Female child of diseased  (Recent Question 2016)
c. Diseased individual d. Pregnant lady a. X-linked recessive b. X-linked dominant
2. A form of fetal programming where two genes are c. AD d. AR
inherited but one is turned off through environmentally 12. Which of the following is inherited as autosomal
induced modification is called as: recessive? M  (Recent Question 2015)
 (Recent Question 2017)
a. Achondroplasia b. Tuberous sclerosis
a. Epigenetic imprinting b. Polymorphism c. Hemochromatosis d. Osteogenesis imperfecta
c. Gene modification d. Gene heterogeneity 13. RET proto oncogene is located on which chromosome:
3. Children heterozygous for having _____ {this is a  (Recent Question 2015)
genetic phenomenon) for KCNE2 gene (Potassium
a. 9 b. 10
Channel voltage gated) have higher chance of having
life threatening arrhythmias after administration of c. 11 d. 12
Erythromycin? (Recent Question 2017) 14. Which of the following is autosomal recessive inherited
cancer syndrome? (Recent Question 2015)
a. Gene Polymorphism b. Anticipation
c. Ontogeny d. Phylogeny a. Ataxia telangiectasia b. Cowden syndrome
4. Different mutations in the same genetic locus causing c. Retinoblastoma d. HNPCC
similar or identical phenotype is: 15. Name the proposed mechanism where one gene (among
 (MAHA PGM CET 2016) two inherited genes) is turned off through environmentally
a. Phenotypic heterogeneity induced modifications leading to fetal programming?
b. Allelic heterogeneity a. The one-two selection mechanism (APPG 2016)
c. Locus heterogeneity b. Fetal programming service
d. Non-allelic heterogeneity c. Cytogenetic manipulation
5. The structural abnormality of chromosome in which one d. Epigenetic imprinting
arm is lost and remaining arm is duplicated is called: M 16. Inheritance of familial hypercholesterolemia:
 (MAHA PGM CET 2016)  (Recent Question 2015)
a. Ring chromosome b. Isochromosome a. Autosomal dominant b. Autosomal recessive
c. Translocation d. Mutation c. X-linked dominant d. X-linked recessive
6. Regarding ‘Davidson body', all are correct except: M  17. Inheritance of congenital adrenal hyperplasia:
 (MAHA PGM CET 2016)  (Recent Question 2015)
a. Present in males a. Autosomal dominant b. Autosomal recessive
b. Present in neutrophils c. X-linked dominant d. X-linked recessive
c. Present in 4 – 6 % of cells 18. Which of the following is a Polygenic disorder:
d. Drumstick appearance  (Recent Question 2015)
7. Frameshift mutation occurs due to: M 
a. Cystic fibrosis b. Phenylketonuria
 (Recent Question 2016)
c. Multiple sclerosis d. Hypertension
a. Transition b. Transversion
19. % of offsprings affected by colour blindness, of a healthy
c. Insertion d. Point mutation
male and heterozygous female: (Recent Question 2015)
8. % of individual who inherited the gene and will express
a. None b. 25%
the trait is known as? M  (Recent Question 2016)
c. 50% d. 100%
a. Penetrance b. Inheritance
20. Inheritance of Alport syndrome: (Recent Question 2015)
c. Co-dominance d. Pleiotropism
9. Inheritance of Beckers muscular dystrophy is: M  a. Autosomal recessive b. X-linked dominant
 (Recent Question 2016) c. X-linked recessive d. Mitochondrial inheritance
a. X-linked recessive b. X-linked dominant 21. Trisomy 13 is: M  (Recent Question 2015)
c. Autosomal recessive d. Autosomal dominant a. Patau syndrome b. Edward syndrome
10. Which of the following is an autosomal dominant c. Down syndrome d. Di George syndrome
metabolic disorder? M  (Recent Question 2016) 22. Pedigree Chart is: (Recent Question 2013)
a. Hereditary hypercholesterolemia a. Used for growth monitoring.
b. Tay sachs disease b. Used to assess side effect during chemotherapy
c. Gaucher's disease c. Used to see genetic transmission.
d. Tyrosinemia d. Used to assess developmental delay in infant
Chapter 5: Genetics and Genetic Disorders 85

23. Increased distance between medial canthus is:

Questions
32. Discrete nodule in the iris called as lisch nodule
 (TN PGMEE 2008) representing melanocytic hamartomas of the iris seen
a. Telecanthus b. Epicanthus most commonly in: (Recent Question 2017)
c. Epiphora d. Strabismus
a. Tuberous sclerosis b. Neurofibromatosis
24. Differential expression of same gene depending on c. Incontinentia Pigmenti d. I cell disease
parent of origin is referred to as: 33. Lisch Nodule is seen in: M  (Recent Question 2016)
 (AIPGMEE 2006, AIPGMEE 2005)
a. Neurofibromatosis type 1
a. Genomic imprinting b. Mosaicism b. Neurofibromatosis type 2
c. Anticipation d. Non-penetrance c. Tuberous sclerosis
25. Single gene defect causing multiple unrelated problems: d. MEN
 (AIIMS Nov 2006)
34. Hypophosphaemic Vit D Resistant Rickets is?
a. Pleiotropism b. Pseudodominance  (Recent Question 2016)
c. Penetrance d. Anticipation
a. AD b. AR
26. Atavism means child resembles with his: M  c. XD d. XR
 (AIIMS Nov 2002)
35. Which of the following is X-linked recessive?
a. Father b. Siblings  (Recent Question 2016)
c. Grand parents d. Neighbour
a. Duchenne muscular dystrophy
 MENDELIAN DISORDERS b. Hypophosphatemic rickets
c. Marfan's syndrome d. Downs syndrome
27. Abnormality involving which chromosome is seen in 36. Abnormal color vision is 20 times more common in
Hunter disease? (FMGE Dec 2018) men than women because most cases are caused by an
abnormal: (MAHA PGM CET 2016)
a. X chromosome b. Y chromosome
c. Chromosome 21 d. Chromosome 22 a. Dominant gene on Y chromosome
28. Find Correct match about inheritance: (PGI Nov 2017) b. Recessive gene on Y chromosome
c. Dominant gene on X chromosome
a. Hypophosphatemic rickets- X linked recessive
d. Recessive gene on X chromosome
b. Duchenne muscular dystrophy- X linked recessive
c. Sickle cell: AR 37. Which of the following diseases/disorders is NOT
d. Osteogenesis imperfecta-1 :AR transmitted as a recessive sex linked trait?
e. Achondroplasia : AR  (MAHA PGM CET 2016)
29. Alpha1 antitrypsin gene is on which chromosome? a. Hemophilia b. Cystic fibrosis
 (JIPMER Nov 2017) c. G6PD deficiency d. Retinitis pigmentosa
a. Chr 11 b. Chr 12 38. Which of the following has autosomal dominant
c. Chr 13 d. Chr 14 inheritance? M  (Recent Question 2016)
30. Study the following carefully: a. Hereditary spherocytosis
b. Thalassemia
c. Sickle cell disease d. G6PD deficiency
39. Which one of the following conditions is not inherited in
autosomal dominant pattern? M  (Recent Question 2016)
a. Neurofibromatosis b. Friedrich's ataxia
c. Marfan's syndrome d. Hereditary spherocytosis
40. All of the following conditions have autosomal dominant
inheritance except:
 (Recent Question 2016, AlPGMEE 2007)
Read the pedigree. Inheritance pattern of the disease in the
family is: (JIPMER Nov 17) a. Fabry disease b. Marfan's syndrome
a. Autosomal recessive b. Autosomal dominant c. Osteogenesis imperfecta d. Ehlers-Danlos syndrome
c. X-linked dominant d. X-Linked recessive 41. True about autosomal dominant type of inheritance:
31. Which of the following is the most likely inheritance pattern  (PGI May 2015)
in the pedigree given below? M  (JIPMER May 2017) a. 25% affected and 50% carrier if one parent affected
b. 50% affected and 75% carrier if both parent affected
c. 75% affected if both parent affected
d. 50% affected if one parent affected
e. All carrier irrespective of either one parent affected or
both parent affected
42. All the following are autosomal dominant except:
a. Cronkhite Canada syndrome (Recent Question 2015)
b. Bannayan Ruvalcaba Riley syndrome
a. Autosomal dominant b. Mitochondrial c. Peutz Jeghers syndrome
c. Autosomal recessive d. X-linked dominant d. Gardner's syndrome
86 Section 1: General Pediatrics

56. Jeune syndrome is transmitted as: (JIPMER 2011)


Review of Pediatrics and Neonatology

43. Waardenburg syndrome is due to mutation of:


 (Recent Question 2015) a. Autosomal recessive b. Autosomal dominant
a. PAX2 gene b. PAX3 gene c. X-linked Dominant d. X-linked Recessive
c. PAX6 gene d. PAX9 gene 57. The following is not a feature of Pierre-Robin syndrome:
44. The law “Relative frequencies of each gene allele tends  (DNB Dec 2011)
to remain constant from generation to generation”, was a. Hearing defect b. Coloboma Iris
given by: (Recent Question 2015) c. Respiratory distress d. Mandibular hypoplasia
a. Henry Sigerist b. Hardy Weinberg 58. Which of the following is autosomal dominant type of
c. Doug Engelberg d. Johanna Frank genetic disorder? M  (DNB June 2010)
45. ABO blood group is an example of? M  a. Colour blindness b. Hemophilia
 (AIIMS May 2015) c. Phenylketonuria d. Tuberous sclerosis
a. Co-dominance b. AD 59. A particular genetic disorder appears in three consecutive
c. AR d. Mitochondrial inheritance generations of a family without any sex predilection.
46. The population frequency for phenylketonuria is 1 in It was also noticed that phenotypically normal family
10,000. What is the carrier frequency for the disease? members were having healthy offspring, What is the
 (Recent Question 2015) pattern of inheritance of this disorder? (DNB Dec 2010)
a. 1/50 b. 1/100 a. Autosomal recessive b. Autosomal dominant
c. 1/500 d. 1/1000 c. Mitochondrial inheritance
47. What is the risk of an affected individual transmitting d. Uniparental disomy
an autosomal dominant disease to his/her offspring? M
60. The following are autosomal dominant disorders
 (MAHA PGM CET 2015)
except: M  (COMEDK 07)
a. 50% b. 100%
c. 25% d. 60% a. Myotonic dystrophy b. Von Willebrand disease
48. What is the mode of inheritance of Hereditary c. Hemochromatosis d. Marfan's syndrome
spherocytosis? (Recent Question 2015) 61. Pedigree analysis—Analyze the following pedigree and
a. Autosomal dominant b. Autosomal recessive give the mode of inheritance: M  (AIPGMEE 2007)
c. X-linked recessive d. X-linked dominant
49. Testicular feminization syndrome is:
 (Recent Question 2015)
a. XX b. XY
c. XYY d. XXY
50. Which is X-linked dominant condition?
 (Recent Question 2015)
a. Hypophosphatemic Rickets
b. Hemophilia a. Autosomal recessive
c. Gaucher disease d. Cystic fibrosis b. Autosomal dominant
c. Mitochondrial inheritance
51. The inheritance of hemophilia is: M  (TN PGMEE 2014) d. X-linked dominant
a. Autosomal dominant b. Autosomal recessive 62. In an autosomal Recessive (AR) disorder, one parent
c. X-linked dominant d. X-linked recessive is normal and the other is carrier and the child is also
52. X-linked dominant disease is: (TN PGMEE 2014) affected. What is the reason? (AIPGMEE 2007)
a. Fragile X syndrome b. Vit D resistant rickets a. Germline mosaicism b. Genomic imprinting
c. Lesch-Nyhan syndrome d. Adrenoleukodystrophy c. Penetration d. Uniparental disomy
53. True statement about inheritance of an X-linked 63. An affected male infant born to normal parents could be
recessive trait is: (Recent Question 2014)
an example of all of the following except:
a. 50% of boys of carrier mother are affected
a. An autosomal dominant disorder (AIPGMEE 2006)
b. 50% of girls of diseased father are carrier
b. An autosomal recessive disorder
c. Father transmits disease to the son
c. A polygenic disorder
d. Mother transmits the disease to the daughter
d. A vertically transmitted disorder
54. 10-year-old African-American boy with abnormal
shaped RBC, mild splenomegaly and recurrent infection. 64. For a normal husband and wife the first child was
What should be the probable diagnosis: diagnosed to have cystic fibrosis. What is the percentage
 (AIIMS Nov 2013) of chances for the second child be affected? M 
a. Single nucleotide mutation a. 25 b. 50 (PGI June 06)
b. Trinucleotide repeat mutation c. 0 d. 75
c. Base excision repair d. DNA breakage e. 100
55. A 4-year-baby is having large face, large jaw, large ear 65. Which of the following diseases is an autosomal
and macroorchidism. Diagnosis? M (DNB June 2011) dominant disorder: M  (AIIMS May 2005)
a. Mc Cune Albright syndrome a. Hemochromatosis
b. Down's syndrome b. Phenylketonuria
c. Cri-du chat syndrome c. Maturity onset diabetes of the young
d. Fragile X syndrome d. G6PD deficiency
Chapter 5: Genetics and Genetic Disorders 87

66. Single gene autosomal recessive disease is:

Questions
77. In Down syndrome, which of the following is decreased?
 (PGI June 2005, PGI June 1999)
 (JIPMER Nov 2016)
a. Wilson's disease b. Tuberous sclerosis a. Nuchal thickness b. Beta HCG
c. Huntington's disease d. Schizophrenia c. Inhibin d. Alfa fetoprotein
67. The chances of having an unaffected baby, when both
78. Triple test includes all the following except: M 
parents have achondroplasia are:
 (JIPMER May 2016)
 (AIPGMEE 05, AIIMS May 2004)
a. AFP b. HCG
a. 0% b. 25%
c. Inhibin d. Estradiol
c. 50% d. 100%
79. Definitive diagnosis of Down syndrome is by:
68. All of the following are chromosomal breakage
 (Recent Question 2016)
syndromes except: (COMEDK 04)
a. Karyotyping b. Triple test
a. Fanconi's anemia b. Ehler-Danlos syndrome
c. Fetal ultrasonography d. All of above
c. Bloom syndrome d. Ataxia telangiectasia
80. Risk of having a child with Down syndrome: M 
69. A is hesitant about having children because her two
 (Recent Question 2016)
sisters had sons who had died form kinky hair disease.
Her mother's brother also died of the same condition. a. Is unaffected by maternal age
Which of the following is the possible mode of b. Decreases with increase in maternal age
inheritance in her family? (AIPGMEE 04) c. Increases with increase in paternal age
d. Increases with increase in maternal age
a. X-linked recessive b. X-Iinked dominant
c. Autosomal recessive d. Autosomal dominant 81. Most common heart lesion in down syndrome: M 
 (Recent Question 2016)
70. An albino girl gets married to a normal boy. What are
the chances of their having an affected child and what a. Endocardial cushion defect
are the chances of their children being carriers? b. ASD with ostium secundum
 (AlPGMEE 03) c. VSD
a. None affected, all carriers d. Coarctation of aorta
b. All normal 82. Nuchal thickness more than __________ mm at 11-14
c. 50% carriers weeks of pregnancy is suggestive of Down's syndrome:
d. 50% affected, 50% carriers  (MAHA PGM CET 2016)
71. Mutation leading to sickle cell anemia: (PGI June 2001) a. 2 b. 3
a. Crossover mutation b. Frameshift c. 4 d. 5
c. Deletion d. Non-disjunction 83. Which of the following syndromes is due to non-
e. Point mutation disjunction? M  (Recent Question 2016)
72. A parent is homozygous and a parent heterozygous for a. Edward b. Down
an autosomal recessive gene. What will be the outcome: c. Patau d. Klinefelter
 (AIIMS May 94) 84. In Downs syndrome there is: (Recent Question 2016)
a. 75% children affected a. Translocation b. Mutations
b. No child affected, but all are carriers c. Paternal nondisjunction d. Maternal nondisjunction
c. 50% children affected, rest are carriers 85. All the following are features of Down syndrome except:
d. 25% children affected, rest are carriers  (Recent Question 2016)
a. Brushfield's spots in iris b. Simian crease
 DOWN SYNDROME c. Mental retardation d. Hypertonia
86. Among the following, the most sensitive USG finding of
73. Trisomy seen in Down syndrome is: (FMGE Nov 2017)
Trisomy 21 is: (Recent Question 2015)
a. 21 b. 22
a. Absent nasal bone b. Thickened nuchal fold
c. 18 d. 13 c. Short femur d. Echogenic bowel
74. MC congenital heart defect in Down’s syndrome: 87. In Down syndrome the shape of the head is:
a. Patent ductus arteriosus (FMGE June 2018)  (Recent Question 2015)
b. Atrial septal defect a. Oxycephalic b. Scaphocephalic
c. Atrioventricular septal defect c. Brachicephalic d. Plagiocephalic
d. Ventricular septal defect 88. In Down syndrome, all are seen except:
75. Most common cause of Down syndrome is: M   (Recent Question 2015)
 (Recent Question 2017) a. t (14;21) b. Trisomy 21
a. Meiotic non-disjunction b. Robertsonian translocation c. t (11;14) d. t (15;21)
c. Mosaicism d. Partial trisomy 89. All the following are biochemical serum markers in
76. In the first trimester screening for Down syndrome second trimester screening of Down's syndrome except:
which of the following test is to be done? M   (APPG 2014)
 (AIIMS Nov 2016) a. PAPPA b. Beta HCG
a. Beta HCG, PAPPA b. MSAFP, PAPPA c. Inhibin d. Unconjugated oestriol
c. Inhibin, PAPPA 90. Prenatal diagnosis of Down syndrome is by:
d. Unconjugated Estriol, PAPPAs  (Recent Question 2014)
88 Section 1: General Pediatrics

a. Karyotyping b. Triple test 100. Transient Myeloproliferative disorders occur with increased
Review of Pediatrics and Neonatology

c. Fetal ultrasonography d. All of the above incidence in children with: M  (WB PGMEE 2008)
91. Most common CVS lesion in Down syndrome is: a. Down syndrome b. Turner's syndrome
 (Recent Question 2013) c. Ataxia telangiectasia d. Neurofibromatosis
a. VSD b. Endocardial cushion defect 101. A Down syndrome patient is posted for surgery, the
c. TOF d. COA necessary preoperative investigation to be done is:
92. Simian crease is not seen in: (Recent Question 2013)  (AIPGMEE 2008)
a. Down syndrome b. Trisomy 13 a. Echocardiography b. CT brain
c. Turner syndrome d. Noonan syndrome c. X-ray cervical spine d. USG abdomen
93. Most common presentation of Down syndrome is: 102. All of the following are features of Down syndrome
except: (AIIMS Nov 2007)
 (Recent Question 2013)
a. Increased PAPPA
a. Cognitive impairment b. Delayed dentition b. Increased free beta HCG levels
c. Recurrent chest infection d. Constipation c. Absent nasal bone
94. Thirteen pair of Ribs can be seen in all of the following d. Abnormal ductus venosus flow velocity
except: (Recent Question 2013) 103. True about Down syndrome are all except: M 
a. Down syndrome b. Holt-Oram syndrome  (TN PGMEE 2007)
c. Turner syndrome d. Incontinentia pigmenti a. Mental retardation b. Hypotonia
95. Identify the syndrome in a male child with the following c. Coarctation of aorta is the most common cardiac lesion
karyotype: (Recent Question 2013) d. Epicanthal folds
104. All of the following may occur in Down's syndrome
except: M  (AIPGMEE 2006)
a. Hypothyroidism b. Undescended testis
c. Ventricular septal defect d. Brushfield's spots
105. In Down syndrome, which of the following are seen? M
 (PGI Dec 2006)
a. Sandal gap b. Antimongoloid slant
c. Clinodactyly d. Hypotonia
e. Hypertonia
106. Down syndrome predisposes to which cancer:
 (MAHA PGM CET 05)
a. AML b. CML
c. ALL d. CLL
a. Turner syndrome b. Edward syndrome
c. Down syndrome d. Patau syndrome 107. Triple test for diagnosis of Down syndrome includes all
of the following except: M  (AIIMS May 2003)
96. One of the parents who have a child with Down
syndrome, has a balanced translocation between a. HCG b. Alfa Fetoprotein
chromosome 15 and 21. What advice will you provide c. Serum Prolactin level d. Serum estriol level
to the couple to prevent a child being born with Down 108. Transient myeloproliferative disorder of newborn is seen
syndrome? (AIIMS Nov 2012) in association with: (AIPGMEE 03)
a. Prenatal diagnosis and advice abortion a. Turner syndrome b. Down syndrome
b. Artificial insemination with donor's sperm c. Neurofibromatosis d. Ataxia telangiectasia
c. Adoption 109. Infants with Down syndrome have all except:
d. Does not matter  (PGI June 03)
97. Down syndrome in advanced maternal age is most a. VSD b. Duodenal atresia
commonly due to: (TN PGMEE 2010) c. Leukemia d. Normal intelligence
a. Non disjunction b. Translocation e. Delayed skeletal maturation
c. Mosaicism d. Deletion 110. Down syndrome is associated with: (PGI Dec 2002)
98. Down syndrome most commonly occurs due to: a. Congenital heart disease
 (AIPGMEE 2010) b. ALL
a. Reciprocal translocation c. Early onset Alzheimer's disease
b. Non-disjunction in maternal meiosis d. CNS tumors e. Infection
c. Translocation defect 111. Commonest cause of intestinal obstruction in Down
d. Non-disjunction in paternal meiosis syndrome is: (AIIMS June 2000)
99. A Down syndrome child is mentally retarded. All a. Colonic atresia b. Intestinal atresia
cytogenetic abnormalities may occur except? c. Duodenal atresia d. Esophageal atresia
 (AIIMS Nov 2010) 112. All are seen in Down syndrome except: M 
a. Deleted chromosome 21  (AIIMS May 2000)
b. Trisomy 21 a. Simian crease b. Clinodactyly
c. Robertsonian translocation c. Mother's age >35 years
d. Mosaic d Respiratory tract infection uncommon
Chapter 5: Genetics and Genetic Disorders 89

113. True about Down's syndrome: (PGI Dec 2000)

Questions
a. Hypertonia b. Simian crease
c. Lymphedema d. Mongoloid slant
e. Anti-mongoloid slant
114. A 35-years-old lady has chromosomal translocation
21/21. The risk of Down syndrome in the child is:
 (AIIMS June 99)
a. 100% b. 0%
c. 10% d. 50%
115. Increased nuchal fold thickness is a feature of:
 (AIPGMEE 99)
a. Paul-Bunnell syndrome
b. DiGeorge syndrome
a. Turner syndrome b. Edward syndrome
c. Down syndrome
c. Down syndrome d. Patau syndrome
d. Cri-du chat syndrome
125. In Turner syndrome, karyotyping shows: M 
 TURNER SYNDROME  (Recent Question 2013)
a. XXY b. XO
116. Which of the following is true about 45,XO? c. XXX d. Trisomy 21
 (AIIMS Nov 2018) 126. A male has anti-mongoloid slant, pulmonary stenosis,
cryptorchidism and mental retardation. Most likely he is
a. Webbed neck has 10 times more risk of congenital heart
having: (WB PGMEE 2010)
disease
a. Turner syndrome b. Noonan syndrome
b. If webbed neck is present, coarctation of aorta is absent
c. Down syndrome d. Often syndrome
c. Male Turner (Noonan) has higher risk of cardiac diseases
127. Which one of the following is NOT a feature of Turner
d. Webbing of fingers and toes is associated with visceral
syndrome? M  (DNB June 2009)
anomalies
a. Short stature b. Mental retardation
117. In Turner syndrome all are seen except? M 
c. Coarctation of aorta d. Lymphedema
 (WB PGMEE 2016)
128. Turner syndrome is maximally associated with:
a. Mental retardation b. 45, XO  (AIIMS May 2008)
c. Webbed neck d. Short stature
a. Horseshoe kidney b. Coarctation of aorta
118. Correct about Turner syndrome is? M  c. VSD d. ASD
 (Recent Question 2016) 129. All of the following may occur in Noonan syndrome
a. Tall stature b. Karyotype 45,XY except: (AIPGMEE 2008)
c. Testis present d. High FSH & LH a. Hypertrophic cardiomyopathy
119. Turner syndrome is associated with which of the b. Cryptorchidism
following cardiac abnormality? (Recent Question 2015) c. Infertility in females
a. Atrial septal defect d. Autosomal dominant transmission
b. Ventricular septal defect 130. Webbing of neck, increased carrying angle, low posterior
c. Patent ductus arteriosus hair line and short fourth metacarpal are characteristics
d. Coarctation of aorta of: (AIPGMEE 2004)
120. A female presents with karyotype 45, XO and absent a. Klinefelter syndrome b. Turner syndrome
gonads. What is your diagnosis? (Recent Question 2015) c. Cri du chat syndrome d. Noonan syndrome
a. Klinefelter's syndrome 131. A patient with short stature, sexual infantilism and
congenital anomalies with chromosomal abnormalities
b. Androgen insensitivity syndrome
‘XO'. Diagnosis is: (PGI June 2004)
c. Turner Syndrome
d. Kallman syndrome a. Turner syndrome b. Klinefelter syndrome
c. Testicular feminization syndrome
121. Cystic hygroma is found in: M  (MAHA PGM CET 2014) d. Gonadal agenesis e. Gonadal dysgenesis
a. Klinefelter's syndromeb. Down's syndrome 132. The following statements regarding Turner syndrome are
c. Edward's syndromed. Turner's syndrome true except: (AIIMS May 2003)
122. Streak ovaries are seen in which one of the following a. Occurrence of Turner syndrome is influenced by maternal
syndrome? M  (MAHA PGM CET 2014) age
a. Triple X b. Klinefelter's b. Most patients have primary amenorrhea
c. Turner d. Swyer's c. Most patients have short stature
123. Edema of hands and feet in infants is characteristic of: M d. Edema of feet and hand is an important feature during infancy
 (Recent Question 2014) 133. A nineteen-year-old female with short stature, widely
a. Klinefelter's syndrome b. Noonan syndrome spaced nipples and primary amenorrhoea most likely
c. Turner syndrome d. Fragile-X syndrome has karyotype of: (AIPGMEE 2003)
124. Identify the syndrome in following given karyotype? M a. 47, XX + 18 b. 46, XXXY
 (Recent Question 2013) c. 47, XXY d. 45, XO
90 Section 1: General Pediatrics

134. True about Turner syndrome: (PGI June 2002)


Review of Pediatrics and Neonatology

145. What is the commonest aneuploidy seen in neonates born


a. Primary amenorrhea b. 45, XO with single umbilical artery? (Recent Question 2017)
c. Short stature d. Puberty usually late a. Trisomy 18 b. Trisomy 13
e. Streak ovaries c. Trisomy 21 d. Trisomy 15
135. A child presents with anti-mongoloid slant, pulmonary 146. Maternal disomy is seen in which of the following?
stenosis, short stature and undescended testis. The likely  (JIPMER May 2016)
diagnosis is: (AIIMS Dec 2001, AIPGMEE 97)
a. Angelman syndrome b. Prader Willi syndrome
a. Hypoparathyroidism b. Noonan syndrome c. Laurence Moon syndrome d. Fragile X syndrome
c. Klinefelter syndrome d. XYY sex chromosomes 147. Cri-du-chat syndrome is due to deletion involving which
136. In Turner syndrome which of the following is not seen: M of the following chromosomes? (Recent Question 2016)
 (AIIMS June 2000) a. 5p b. 5q
a. Short stature b. Widely spaced nipple c. 15p d. 15q
c. Webbed neck d. Mental retardation 148. Barr body is absent in female having: M 
 (Recent Question 2016)
 OTHER IMPORTANT GENETIC DISORDERS a. 46, XX genome b. 45, XO genome
c. 47, XXX d. All of above
137. A child with retarded growth came with triangular face, 149. Karyotype of a patient shows the following, what is the
micrognathia, Tripod skull and fifth finger clinodactyly: clinical abnormality that is expected? (AIIMS May 2015)
 (JIPMER 2017)
a. Russell Silver syndrome
b. Beckwith–Wiedemann syndrome
c. Angelman syndrome d. Prader–Willi syndrome
138. Hypoglycemia, jaundice, microphallus with absent
septum pellucidum is characteristic of which syndrome?
 (JIPMER 2017)
a. De Morsier syndrome b. Silver–Russell syndrome
c. Kallmann syndrome d. Kearns–Sayre syndrome
139. A child presents with congenital cyanotic heart disease
and polydactyly. Likely diagnosis is: (JIPMER May 2017) a. Gynecomastia with long thin limbs
a. Ellis-van Creveld syndrome b. Short stature with polydactyly
b. Rubinstein- Taybi syndrome c. Webbed neck with widely spaced nipples
c. McKusick – Kaufman syndrome d. Rocker bottom feet
d. Edward syndrome 150. True statement regarding chromosomes: (JIPMER 2015)
140. Which of the following is true about Treacher Collins a. In females both X chromosomes are activated
syndrome: (Recent Question 2017) b. Klinefelter syndrome results due to an extra Y chromosome
a. Autosomal recessive in males
b. First and second arch affected c. Germinal cells contain 23 chromosomes
c. Dental structure preserved d. Turner syndrome results due to an extra X chromosome in
d. Hyperplastic mandible females
141. Which of the following about Pierre Robins syndrome is 151. Cri du chat syndrome is? (DNB 2015 pattern)
false? (Recent Question 2017) a. 22q- b. 5q-
a. Abdominal muscles are absent c. 22p- d. 5p-
b. Floor of mouth is shortened 152. Klinefelter syndrome is diagnosed by:
c. Prone position is relieving  (Recent Question 2015)
d. Achieves normal growth in 5 years of life a. Karyotyping b. USG abdomen
142. Which of the following feature is not seen in Prader c. Triple test d. Echocardiography
Willi syndrome? (Recent Question 2017) 153. What is the most likely diagnosis of the picture shown
a. Obesity b. Mental retardation below? (APPG 2015)
c. Ataxia d. Hypotonia
143. Which of the following is seen in Noonan syndrome
with pulmonic stenosis? (JIPMER Nov 2016)
a. Chromosomal aberration
b. Coarctation of aorta
c. Pulmonary lymphangiectasia
d. Abnormal intelligence
144. Triangular facies is indicative of?
 (Recent Question 2017)
a. Russell Silver syndrome b. Mobius syndrome a. Down's syndrome b. Pierre-Robin syndrome
c. Soto syndrome d. Nephrotic syndrome c. Noonan's syndrome d. Williams syndrome
Chapter 5: Genetics and Genetic Disorders 91

166. Patau syndrome feature include all except:

Questions
154. In the genetic disease, xeroderma pigmentosum, the
 (Recent Question 2013)
cells fail to repair damaged DNA, due to the defect in:
 (MAHA PGM CET 2015) a. Cleft lip b. Hypotelorism
c. Holoprosencephaly d. Rocker bottom foot
a. Double stranded break repair
b. Base excision repair 167. Trisomy 13 is seen in: (Recent Question 2012)
c. Mismatch repair d. Nucleotide excision repair a. Edward syndrome b. Patau syndrome
155. Not seen in William syndrome? (Recent Question 2015) c. Down syndrome d. Turner syndrome
a. Elfin facies b. Subvalvular aortic stenosis 168. About Trisomy 13, Which the following are true statements?
c. Hypercalcemia d. Hypertension  (AIIMS Nov 2012)
156. The disorder in Patau syndrome involves which of the a. Bilateral microphthalmia b. Neurofibroma
following? (Recent Question 2015) c. Rocker bottom feet d. Dermoid cyst
a. Chromosome 21 b. Chromosome 18 169. Edward syndrome is: (MAHA 11)
c. Chromosome 13 d. Chromosome 16 a. Trisomy 13 b. Trisomy 18
157. F body is?  (Recent Question 2015) c. Trisomy 21 d. Trisomy 20
a. Y chromatin b. X chromatin 170. Which one of the following is a distinguishing feature of
c. Chromosome 1 d. Chromosome 21 Edward's syndrome? (DNB Dec 2011)
158. Identify the disease shown in the karyotype: a. Hypotonia b. Hypotelorism
 (AIIMS Nov 2014) c. Holoprosencephaly d. Rocker bottom feet
171. An infant with cleft lip, cleft palate, polydactyly, micro-
cephaly with holoprosencephaly, ectodermal scalp defect
is suffering from: (AIIMS May 2008)
a. Trisomy 21 b. Trisomy 18
c. Trisomy 13 d. Turner syndrome
172. Cat eye syndrome is: (AIPGMEE 2007)
a. Partial trisomy 18 b. Partial trisomy 13
c. Partial trisomy 21 d. Partial trisomy 22
173. Not a feature of Edward syndrome: (TN PGMEE 2006)
a. Long neck b. Low set ears
c. Rocker bottom foot d. Renal malformations
174. A child with a small head, minor anomalies of the face
a. Cri-du-chat syndrome b. Bloom syndrome
including a thin upper lip, growth delay, and develop-
c. Angelman syndrome d. Fragile X syndrome
mental disability can have all of the following, except:
159. Chromosome 22 deletion syndrome is: M   (AIPGMEE 06)
 (Recent Question 2014)
a. A chromosomal syndrome b. A teratogenic syndrome
a. Down syndrome b. Di George syndrome c. A Mendelian syndrome d. A polygenic syndrome
c. Turner syndrome d. Klinefelter syndrome
175. Which of the following is an example of disorders of sex
160. Patau's syndrome is due to: M  (Recent Question 2014) chromosomes? (AIIMS May 06)
a. Trisomy 21 b. Trisomy 18
a. Marfan's syndrome
c. Trisomy 15 d. Trisomy l3
b. Testicular feminization syndrome
161. Klinefelter's syndrome is associated with:
c. Klinefelter syndrome d. Down syndrome
 (Recent Question 2014)
a. 47, XYY b. 45, XO 176. Males who are sexually underdeveloped with rudimentary
c. 46, XY d. 47, XXY testes and prostate glands, sparse pubic and facial hairs,
162. Absence of clavicle is a feature of: long arms and legs and large hands and feet are likely to
a. Cleidocranial dysplasia (MAHA PGM CET 2014) have the chromosome complement:
b. Multiple epiphyseal dysplasia  (AIIMS May 2015, AIPGMEE 04)
c. Fibrous dysplasia a. 45XXY b. 46XY
d. Cranio-Metaphyseal dysplasia c. 47XXY d. 46X
163. Common features of Marfan syndrome & Homocystinuria 177. A baby presenting with multiple deformities, cleft lip,
are: (PGI Nov 2014) cleft palate, microcephaly, small eyes, scalp defect and
a. Arm span > Height b. Ectopia lentis polydactyly, seen in which syndrome: (AIIMS Nov 03)
c. Hypermobility of joint d. Mental retardation a. Trisomy 13 b. Trisomy 18
e. Arachnodactyly c. Trisomy 21 d. Monosomy 2
164. In Treacher Collins all are seen except:
178. Common ocular manifestation in Trisomy 13 is:
 (WB PGMEE 2014)
 (AIIMS 03)
a. Coloboma of inf. eyelid b. Mandibular angle obtuse a. Capillary hemangioma b. Bilateral microphthalmos
c. Low intelligence d. Cleft palate c. Neurofibroma d. Dermoid cyst
165. The karyotype in testicular feminising syndrome is?
179. Most common chromosomal syndrome is: M 
 (Recent Question 2013, 2012)
 (AIIMS Dec 94)
a. XX b. XY
a. Fragile X-syndrome b. Trisomy 17
c. XXY d. XXXY
c. Trisomy 21 d. Trisomy 13
92 Section 1: General Pediatrics

 TRINUCLEOTIDE REPEATS  DIAGNOSTIC TOOLS


Review of Pediatrics and Neonatology

180. Which of the following is a trinucleotide repeat disorder? 193. Cordocentesis is done in which time of pregnancy
 (Recent Question 2015)  (JIPMER Nov 2017)
a. Inflammatory Myopathy b. Myotonia dystrophica a. 18-20 weeks b. 14-16 weeks
c. Duchenne muscular dystrophy c. 12-14 weeks d. 8-10 weeks
d. Congenital muscular dystrophy 194. What is the diagnostic test you would do for a P2L2
181. Anticipation phenomenon is seen in pregnant female at 18 weeks of gestation having a
history of 1 baby with thalassemia? (AIIMS Nov 2015)
a. Duchenne muscular dystrophy (Recent Question 2015)
b. Becker muscular dystrophy a. CVS b. NIPT
c. Limb girdle dystrophy d. Myotonic dystrophy c. Amniocentesis d. Cordocentesis
182. Which is the most common genetic cause of mental 195. For karyotyping, the dividing cells are arrested by the
retardation: (Recent Question 2014) addition of colchicines in the following mitotic phase
 (Recent Question 2015)
a. Tuberous sclerosis b. Fragile-X-syndrome
a. Prophase b. Metaphase
c. Cri-du-chat syndrome d. Angelman's syndrome
c. Anaphase d. Telophase
183. Fragile X syndrome is associated with all except:
196. Submicroscopic deletions of any size can be detected
 (Recent Question 2013, WB PGMEE 2008, 2009)
by: (Recent Question 2015)
a. Large nose b. Large face a. Multiplex ligation-Dependent probe amplification (MLPA)
c. Large ears d. Large testis b. Southern blotting
c. Cytogenomic array technology
 MITOCHONDRIAL INHERITANCE d. Chromosome painting
197. Known gene loci can be diagnosed by:
184. All are mitochondrial disorders except: a. FISH (AIIMS Nov 2013)
a. Pearson Syndrome (MAHA PGM CET 2016) b. Comparative gene hybridization
b. MERRF (Myoclonic Epilepsy Ragged Red Fibre) c. PCR d. Chromosomal painting
c. MELAS (Mitochondrial Encephalopathy Lactic Acidosis 198. Karyotyping is useful in diagnosis of: M  (AIPGMEE 09)
and Stroke-like episode) a. Autosomal recessive disorders
d. Fragile X Syndrome b. X-linked recessive disorders
185. Type of inheritance in MELAS: (Recent Question 2014) c. Chromosomal abnormalities
a. AD b. AR d. Biochemical abnormalities
c. Mitochondrial d. X-linked 199. Karyotyping is done in which phase of cell cycle:
186. NARP is a: (AIPGMEE 2011, AIIMS Nov 2009)  (Recent Question 2016)
a. Lipid storage disorder b. Glycogen storage disorder a. Anaphase b. Metaphase
c. Mitochondrial disorder d. Lysosomal storage disorder c. Telophase d. S phase
200. Microarray is: (Recent Question 2016)
 GENOMIC IMPRINTING & UNIPARENTAL DISOMY a. Study of multiple genes b. Study of diseases
c. Study of organisms d. Study of blood group
187. Genomic imprinting is seen in: (Recent Question 2016)
201. Which of the following procedures is routine technique
a. Prader Willi syndrome b. Marfan syndrome for karyotyping using light microscopy:
c. Down syndrome d. Osteogenesis imperfect  (DNB Dec 10, AIPGMEE 03)
188. Which of the following hormones are raised in Prader- a. C-banding b. G-banding
Willi syndrome? (Bihar PG 2015/ AIPGMEE 12) c. Q-banding d. Brd V-staining
a. Growth Hormone (GH) b. Leuteinizing Hormone (LH)
c. Follicle Stimulating Hormone  GENE THERAPY
d. Ghrelin
189. The maternal uniparental disomy for chromosome 202. Gene therapy is most commonly targeted against:
15 results in: (MAHA PGM CET 2014) a. SCID b. cancer(WB PGMEE 2016)
c. cystic fibrosis  d. Leukemia
a. Angelman Syndrome b. Prader – Willi Syndrome
c. Pallister – Killian Syndrome 203. Which of the following disease can NOT be treated by
d. Hypomelanosis of ITO Gene Therapy: (MAHA PGM CET 2014)
190. Angelman syndrome is due to: (Recent Question 2014) a. Adenosine deaminase deficiency
a. Digenic inheritance b. Inversion b. Leukemia
c. Uniparental disomy d. Mitochondrial disorder c. Cystic Fibrosis d. Thalassemia
191. All of the following are associated with uniparental 204. Following are potential complications of gene therapy
disomy except: (AIIMS Nov 2013) except: (MAHA PGM CET 2014)
a. Prader willi syndrome b. Bloom syndrome a. Genotoxicity b. Genome integration
c. Angelman syndrome d. Russell silver syndrome c. Gene silencing d. Immunotoxicity
205. Which of the following techniques is used in gene therapy?
192. Maternal disomy of chromosome 15 is seen in:
 (AIIMS Nov 10) a. Electroporation (PGI 01)
b. Electrofocusing
a. Prader- Willi syndrome b. Klinefelter's syndrome c. Selectively targeted recombination
c. Angelman syndrome d. Turner's syndrome d. Intranuclear injection
Chapter 5: Genetics and Genetic Disorders 93

Answers with Explanations


Answers with Explanations

 IMPORTANT TERMINOLOGIES
1. c. Diseased individual  Refer pretext for details Ref: Emery Medical Genetics 14/e p 109
2. a. Epigenetic imprinting  Ref: Nelson's 20/e p 602
3. a. Gene Polymorphism  Ref: Nelson's 20/e p 2000
4. b. Allelic heterogeneity  Ref: Genomics and Clinical Medicine pg 69
•• “Allelic Heterogeneity” refers to the fact that different mutations in the same genetic locus can cause an identical or similar phenotype.
For example, many different mutations of the β-globin locus can cause β-thalassemia

•• “Phenotypic Heterogeneity” occurs when more than one phenotype is caused by alletic mutations, e.g. Identical mutations in the
FGFR2 gene can result in very distinct phenotypes: Crouzon syndrome, or Pfeiffer syndrome
•• “Locus or Nonallelic Heterogeneity”: Refers to the situation in which a similar disease phenotype results form mutations at different
genetic loci. For example, osteogenesis imperfecta can arise from mutations in two different procollagen genes (COL1A1 or COL 1A2) that
are located on different chromosomes
•• “Variable Expressivity” and “Incomplete Penetrance”: The same genetic mutation causes a phenotypic spectrum illustrating the
phenomenon of variable expressivity
5. b. Isochromosome  Ref: Nelson's 20/e p 590, Ghai 8/e p 637
Iso-chromosome results when 1 arm of a chromosome is lost and the remaining arm is duplicated or when the axis of division occurs
perpendicular to the normal axis of division.
6. a. Present in males  Ref: Concept of Genetics/Pg 208
Davidson bodies are small nuclear buds of chromatin shaped like a drumstick found in neutrophils of females, but absent in males.
7. c. Insertion  Ref: Nelson's 20/e p 589-591, Ghai 8/e p 635-636
A frameshift mutation is caused by a deletion or insertion in a DNA sequence that shifts the way the sequence is read.
8. a. Penetrance  Ref: Nelson's 20/e p 590, Ghai 8/e p 640
Penetrance refers to the proportion of people with a particular genetic change (e.g. a mutation) who exhibit signs and symptoms of that
genetic disorder; 50% penetrance means that 50% of those who carry the gene express the trait.
9. a. X-linked recessive  Ref: Nelson's 20/e p595-600, Ghai 8/e p 642
Both Duchenne's & Becker's muscular dystrophy have x linked recessive inheritance.
10. a. Hereditary hypercholesterolemia  Ref: Nelson's 20/e p 595, Ghai 8/e p 640; Refer to pretext for mnemonic
11. a. X-linked recessive  Ref: Nelson's 20/e p 595, Ghai 8/e p 640
•• In a X-linked recessive disease; affected male transmits the disorder to all his daughters (carriers)
•• Sons get their X chromosome from their mother & not from their father
•• Hence sons of a diseased father are unaffected in X-linked diseases
In an X-linked dominant disorder all the daughters of an affected father would be diseased and not carriers.
12. c. Hemochromatosis  Ref: Nelson's 20/e p 595, Ghai 8/e p 640; Refer to pretext of this chapter for mnemonic
13. b. 10  Ref: Nelson's 20/e p 595
Important genes and their chromosomal location:

Gene (Associated diseases) Chromosomal location Gene (Associated diseases) Chromosomal location
Rb (Retinoblastoma) 13q14.3 NF2 (Neurofibromatosis 2) 22p12
p53 (Li-Fraumeni syndrome, Sarcomas) 17q13.1 WT1 (Wilm's tumor) 11p13
APC (Carcinoma Colon) 5q21 RET (MEN2 syndrome) 10q
NF1 (Neurofibromatosis 1) 17q11

14. a. Ataxia telangiectasia  Ref: Nelson's 20/e p 595, Ghai 8/e p 640
Cowden syndrome, Retinoblastoma and HNPCC have autosomal dominant inheritance
15. d. Epigenetic imprinting  Ref: Nelson's 20/e p62, Refer to pretext of this chapter for details
16. a. Autosomal dominant  Ref: Nelson's 20/e p 595, Ghai 8/e p 641
17. b. Autosomal recessive  Ref: Nelson's 20/e p 595, Ghai 8/e p 641
Congenital adrenal hyperplasia (CAH) are autosomal recessive diseases resulting from mutations of genes for enzymes mediating
the biochemical steps of production of cortisol from cholesterol by the adrenal glands.
94 Section 1: General Pediatrics

18. d. Hypertension  Ref: Nelson's 20/e p 650, Ghai 8/e p 643


Review of Pediatrics and Neonatology

Polygenic disorders:
•• Disorders caused by interactions between variant forms of genes and environmental factors.
•• Occur when many polymorphisms, each with a modest effect and low penetrance, are co-inherited
•• E.g. HypertensionQ, Diabetes mellitus, Coronary Artery disease, Septal Defects,Q Cleft lip
19. b. 25%  Ref: Nelson's 20/e p 642, Ghai 8/e p 641
•• Color blindness is an X-linked recessive disorder.
•• In X-linked disorders, if father is normal and mother is a carrier, 50% of children will inherit defective X chromosome.

•• Out of these, the daughters (having 1 normal and 1 defective X chromosome) will act as carriers
•• While, all sons, inheriting the defective X chromosome will be affected by colour blindness. Hence, 25% of children will be affected.
20. b. X-linked dominant  Ref: Nelson's 20/e p 595, Ghai 8/e p 641; Refer pretext for details
21. a. Patau syndrome  Ref: Nelson's 20/e p 590, Ghai 8/e p 641; Refer to pretext for mnemonic
22. c. Used to see genetic transmission  Ref: Nelson's 20/e p 590-600, Ghai 8/e p 644; Refer to pretext for details
23. a. Telecanthus  Ref: Smith's Congenital malformation
Telecanthus is derived from the Greek word “tele” meaning far, and the Latin word canthus, meaning corner of the eye
Telecanthus refers to increased distance between medial canthi of eyes, while inter-pupillary distance is normal.
Note: In hypertelorism, the inter-pupillary distance is increased.
24. a. Genomic imprinting  Ref: Nelson's 20/e p 600
25. a. Pleiotropism  Ref: Nelson's 20/e p 595; Pleiotropism refers to single gene defect causing multiple traits.
26. c. Grand parents  Ref: Nelson's 20/e p 635-637
Atavism means the appearance of a characteristic is presumed to have been present in some remote ancestor due to chance
recombination of genes or environmental conditions favourable to their expression in the embryo.

 MENDELIAN DISORDERS
27. a. X chromosome  Ref: Ghai 9/e p 640 9/e p 640
All types of mucopolysaccharidoses have autosomal recessive inheritance except Hunter disease, which is X linked recessive.
28. b. Duchenne muscular dystrophy-X linked recessive & (c) Sickle cell: AR  Ref: Nelson 20/e p338
a. False; In X linked Hypophosphatemic rickets, female carriers are affected, so it is an X-linked dominant disorder.
b. True; Duchenne muscular dystrophy has X linked recessive
c. True; Sickle cell has Autosomal recessive inheritance
d. False; All osteogenesis imperfecta are AD except type II, III & VII
e. False; Achondroplasia has autosomal dominant inheritance
29. d. Chr 14  Ref: Nelson 20/e p 1941, 2053
α1-Antitrypsin deficiency
• It is caused by mutation in SERPINA1 gene (chr 14q), which has autosomal recessive inheritance.
• α1-Antitrypsin, a protease inhibitor synthesized by liver, protects lung tissue from destruction by neutrophil elastase;
•• Most common allele of protease inhibitor (Pi) system is M & the normal phenotype is PiMM.
30. d. X-linked recessive  Ref: Nelson's 20/e p 593-598
Only males are manifesting disease and females are acting as carrier; so it is X-linked recessive inheritance.
31. d. X-linked dominant  Ref: Nelson's 20/e p 593, Ghai 8/e p 641
It is an X-linked dominant pattern of inheritance, all the female offsprings of a diseased male (XY) receive abnormal
X chromosome & express the disease, whereas none of the male offsprings of diseased father have the disease.
32. b. Neurofibromatosis  Ref: Nelson's 20/e p 2874-2876, Ghai 8/e p 586-588
33. a. Neurofibromatosis type 1  Ref: Nelson's 20/e p 2874-2876; Refer to pretext of chapter 18 for details
34. c. XD  Ref: Nelson's 20/e p 598, Ghai 8/e p 643-644; Refer to Ans 19
35. a. Duchenne muscular dystrophy  Ref: Nelson's 20/e p 2976-2978, Ghai 8/e p 643
Chapter 5: Genetics and Genetic Disorders 95

36. d. Recessive gene on X chromosome  Ref: H20 197, Nelson's 20/e p, Ghai 8/e p 643

Answers with Explanations


•• Red and green cone pigments are encoded on the X chromosome, and the blue cone pigment on chromosome 7.
•• Mutations of the red and green pigments cause congenital X-linked color blindness in 8% of males
•• Because color blindness is almost exclusively X-linked, it is worth screening only male children.
37. b. Cystic fibrosis  Ref: Nelson's 20/e p 593, Ghai 8/e p 641; Refer to Ans 9
38. a. Hereditary spherocytosis  Ref: Nelson's 20/e p 593, Ghai 8/e p 640
39. b. Friedrich's ataxia  Ref: Nelson's 20/e p 595, Ghai 8/e p 641; Refer to Ans 9
40. a. Fabry disease  Ref: Nelson's 20/e p 595, Ghai 8/e p 641; Refer to pretext of this chapter for details
41. c. 75% affected if both parent affected and d. 50% affected if one parent affected  Ref: Nelson's 20/e p 595;
In an autosomal dominant disorder,
If one parent is affected: If both parents are affected:

So, 50% children are affected, if only one parent is affected 75% affected if both parents affected

42. a. Cronkhite Canada syndrome  Ref: Nelson's 20/e p 1901-1902


Discussing about options one by one,
a. Cronkhite Canada syndrome is a non-inherited condition occurring in old age with features of loss of taste, intestinal polyps,
hair loss and nail growth problem
b. Bannayan-Riley-Ruvalcaba syndrome is an autosomal dominant condition characterized by a large head size (macrocephaly),
multiple noncancerous tumors and hamartomas, and dark freckles on penis in males
c. Peutz-Jeghers syndrome (PJS) is an autosomal dominant inherited disorder characterized by intestinal hamartomatous
polyps in association with a distinct pattern of skin and mucosal macular melanin deposition
d. Gardner syndrome, also known as familial colorectal polyposis, is an autosomal dominant form of polyposis characterized by
multiple colonic polyps along with osteomas, thyroid cancer, epidermoid cysts, fibromas.
43. b. PAX3 gene  Ref: Nelson's 20/e p 644
Waardenburg syndrome (WS):

•• Autosomal dominant inherited condition due to mutations in EDN3, EDNRB, MITF, PAX3, SNAI2 or SOX10 genes

44. b. Hardy Weinberg  Ref: Short Textbook of Genetics pg 90, Nelson's 20/e p 598
Hardy-weinberg's law

•• It describes a theoretical situation in which a population is undergoing no evolutionary change


•• It explains that if the evolutionary forces are absent, the population is large and its individuals have random mating
•• Thus each parent produces equal number of gametes
•• Such gametes combine at random and the gene frequency remains constant. Finally the genetic equilibrium of the genes is maintained
and the variability present in the population is preserved.

45. a. Co-dominance  Ref: Nelson's 20/e p, Ghai 8/e p 641


Codominance—When both alleles of a gene pair are fully expressed in a heterozygote. e.g. blood group antigens.
46. a. 1/50  Ref: Nelson's 20/e p 598
It is based on Hardy Weinberg principle: Simplifying it,
Carrier frequency for an autosomal recessive disorder = twice the square root of disease incidence
So 2 X square root of 1/10,000 gives 1/50 as the answer.
47. a. 50%  Ref: Nelson's 20/e p 593, Ghai 8/e p 641
96 Section 1: General Pediatrics

48. a. Autosomal dominant  Ref: Nelson's 20/e p 593, Ghai 8/e p 641
Review of Pediatrics and Neonatology

49. b. XY  Ref: Nelson's 20/e p 2757–2759


Testicular feminization syndrome
•• It is also called the complete androgen insensitivity syndrome (AIS);
•• XY fetuses with this disorder are insensitive (unresponsive) to androgens; So they are born looking externally like normal girls; Internally,
there is a short blind-pouch vagina & no uterus, fallopian tubes or ovaries
•• There are testes in abdomen or inguinal canal; AIS is the most common cause of male DSD.
50. a. Hypophosphatemic rickets  Ref: Nelson's 20/e p 591, Ghai 8/e p 642
51. d. X-linked recessive  Ref: Nelson's 20/e p 598, Ghai 8/e p 642
52. b. Vit D resistant rickets  Ref: Nelson's 20/e p 598, Ghai 8/e p 642
53. a. 50% of boys of carrier mother are affected  Ref: Nelson's 20/e p 598, Ghai 8/e p 642
For a X-linked recessive disease:
If mother is a carrier: If father is diseased:

So, 50% of boys of carrier mothers are affected All daughters are carriers
Mothers transmit defective gene to 50% of the sons (diseased) as well Fathers transmit defective gene to daughters
as daughters (carriers)

54. a. Single nucleotide mutation  Ref: Nelson's 20/e p 392-393


Mild splenomegaly and recurrent infection in a African American boy with abnormal shaped RBC suggests Sickle cell anemia. The
disease is caused by Single Nucleotide Mutation on β chain 6th position.
55. d. Fragile X syndrome  Ref: Nelson's 20/e p 2741-2742, 622-623; Refer to pretext of this chapter for details
56. a. Autosomal recessive  Ref: Oski pg 2648
Jeune syndrome
•• It is a rare autosomal recessive genetic disorder that affects the cartilages and bones
•• It mainly affects the rib cage (bell-shaped thorax), pelvis, arms and legs
•• Thoracic insufficiency is the most important feature of this syndrome.
57. b. Coloboma Iris  Ref: Nelson's 20/e p 1773; J Laryngol Otol 1996 Sep;110(9):830-5;
Pierre Robin syndrome

Components •• It consists of micrognathia usually accompanied by a high arched or cleft palate


•• Tongue is usually of normal size, but the floor of the mouth is foreshortened (mandibular hypoplasia)
•• The air passages can become obstructed, particularly on inspiration (respiratory distress)

Associations •• 30-50% of these patients have Stickler syndrome, an autosomal dominant condition with prominent joints,
arthritis, hypotonia, hypermobile joints, mitral valve prolapse, hearing loss, and ocular problems
•• Other syndromes associated include 22q11.2 deletion syndrome (Velocardio facial syndrome)
•• Pierre Robin patients have higher risk of conductive hearing loss.

58. d. Tuberous sclerosis  Ref: Nelson's 20/e p 593, Ghai 8/e p 640
59. b. Autosomal dominant  Ref: Nelson's 20/e p 593, Ghai 8/e p 640
Features of Autosomal Dominant Inheritance:
•• Affected individuals present in every generation and Males and females are equally affected
•• Any child of an affected parent has a 50% risk of inheriting the disorder
•• Phenotypically normal family members do not transmit the condition to the offspring
60. c. Hemochromatosis  Ref: Nelson's 20/e p 594, Ghai 8/e p 641; Hemochromatosis is autosomal recessive.
61. c. Mitochondrial inheritance  Ref: Nelson's 20/e p 600, Ghai 8/e p 644
This disease is manifesting in both males and females and all children of a diseased female have disease;
Both these are characteristics of mitochondrial diseases.
Chapter 5: Genetics and Genetic Disorders 97

62. d. Uniparental disomy  Ref: Nelson's 20/e p 641, Ghai 8/e p 644

Answers with Explanations


•• If one parent is normal for an autosomal recessive disorder and the other is carrier. No child should be affected.
•• But there is exception to this rule, i.e. uniparental disomy, when a person receives 2 copies of a chromosome, from one
parent and no copies from the other parent.
63. a. An Autosomal dominant disorder  Ref: Nelson's 20/e p 594, Ghai 8/e p 641
In autosomal dominant disorder, parents of an affected child cannot be normal, as, even if a parent has 1 copy of defective gene,
parent will be diseased.
64. a. 25  Ref: Nelson's 20/e p 593-598, Ghai 8/e p 641
•• Cystic fibrosis is an autosomal recessive disorder
•• If husband and wife are clinically normal and the child has developed
the disease that means both husband and wife are carriers of cystic
fibrosis
•• If both father & mother are carriers of an autosomal recessive disorder,
each child has 25% chances (1 in 4) of being affected by the disease

65. c. Maturity onset diabetes of the young  Ref: Nelson's 20/e p 2761-2763
Maturity-onset diabetes of the young (MODY):

•• Now referred to as monogenic diabetes; it is autosomal dominant and has strong family history
•• MODY results from mutations in genes for pancreatic β-cell or liver glucokinase or hepatocyte nuclear factor
(1 , 4 , or 1β); there is no autoimmune destruction of β cells and no HLA association
•• It may require insulin treatment or can be treated with sulfonylureas with varying degrees of success.

66. a. Wilson's disease  Ref: Nelson's 20/e p 593


67. b. 25%  Ref: Nelson's 20/e p 593, Ghai 8/e p 640

•• Achondroplasia is an autosomal dominant disorder:


•• If both parents have Achondroplasia, chances of a child being
affected with Achondroplasia are 75%

68. b. Ehler-Danlos syndrome  Ref: Nelson's 20/e p 593, Ghai 8/e p 640
Chromosome instability syndromes (previously called chromosome breakage syndromes)

•• Disorders resulting from specific defects in DNA repair, cell cycle control and apoptosis with autosomal recessive inheritance →
chromosomal instability → increased risk of developing neoplasms
•• E.g. Fanconi anemia, ataxia telangiectasia, Nijmegen syndrome, ICF (immunodeficiency, centromere instability, and facial anomalies)
syndrome, Roberts syndrome, Werner syndrome and Bloom syndrome.
69. a. X-linked recessive  Ref: Nelson's 20/e p 593-595, Ghai 8/e p 641
The clues in this Question are: Only males are manifesting disease and Females are acting as carriers (sister's sons had suffered
from the disease); So, this disease has X-linked recessive inheritance
70. a. None affected, all carriers  Ref: Nelson's 20/e p 593-595, Ghai 8/e p 640
•• Albinism is autosomal recessive; Female here is affected (2 defective genes), while male is normal (both genes normal).
•• So each offspring (son/daughter) will inherit one defective gene only, from the mother.
•• So all children will be carriers of albinism and none will be affected.

71. e. Point mutation  Ref: Nelson's 20/e p 593, Ghai 8/e p 640
72. c. 50% children affected, rest are carriers  Ref: Nelson's 20/e p 593, Ghai 8/e p 640
98 Section 1: General Pediatrics
Review of Pediatrics and Neonatology

As it is autosomal recessive, 50% affected and 50% carrier

 DOWN SYNDROME
73. a. 21  Ref: Ghai 9/e p 635
In Down syndrome, Trisomy 21 is usually due to meiotic non-disjunction.
74. c. Atrioventricular septal defect  Ref: Ghai 9/e p 636
Atrioventricular septal defect or Endocardial cushion defect is the most common congenital heart disease in Down syndrome.
75. a. Meiotic non-disjunction  Ref: Nelson 20/e p 611 Refer Ans below
76. a. Beta HCG, PAPPA  Ref: Nelson's 20/e p 610-615; Refer to pretext of this chapter for details
77. d. Alfa fetoprotein  Ref: Nelson's 20/e p 610-615; Refer to pretext of this chapter for details
78. c. Inhibin  Ref: Nelson's 20/e p 610-614; Refer to pretext of this chapter for details
79. a. Karyotyping  Ref: Nelson's 20/e p 610-614; Refer to pretext of this chapter for details
80. d. Increases with increase in maternal age  Ref: Nelson's 20/e p 610-614; Refer to pretext
81. a. Endocardial cushion defect  Ref: Nelson's 20/e p 610-614; Refer to pretext of this chapter for details
82. b. 3  Ref: Nelson's 20/e p 816
Nuchal pad thickening is > 3 mm at 11-14 wk or > 6 mm at 15-20 wk
Important causes: Cystic hygroma, Trisomy 21, Trisomy 18, Turner syndrome (45, XO), Normal (~25%).
83. b. Down  Ref: Nelson's 20/e p 610-615, Ghai 8/e p 637-639
95% cases of Down Syndrome are due to Non-disjunction
84. d. Maternal nondisjunction  Ref: Nelson's 20/e p 610-615, Ghai 8/e p 637-639
•• In trisomy 21, the additional number 21 chromosome is derived from mother in 95% of cases
•• It is due to non-dysjunction of 21st chromosome at the time of meiosis
•• In non-dysjunction, chromosomal pair fails to separate in 1st meiotic division during gametogenesis.
85. d. Hypertonia  Ref: Nelson's 20/e p 610-615, Ghai 8/e p 637-639; Refer to pretext for mnemonic
Hypotonia is a feature of Down syndrome and not hypertonia
86. b. Thickened nuchal fold  Ref: Textbook of Perinatal Medicine, 2nd Edition, edited by Asim Kurjak, Frank A. A. Chervenak, pg
835, Nelson's 20/e p 610-615;
Prenatal USG markers of Down syndrome include thickness of nuchal fold (75% sensitive), absence of nasal bone (58%
sensitive), cardiac abnormalities, duodenal atresia, shortened femur, shortened humerus, renal pyelectasis, a hyperechogenic
bowel, and a choroid plexus cyst.
87. c. Brachicephalic  Ref: Nelson's 20/e p 610-615, Ghai 8/e p 637-639
88. c. t (11 ; 14)  Ref: Nelson's 20/e p 610-615, Ghai 8/e p 637-639
3 copies of chr 21 should be present to cause Down syndrome
89. a. PAPPA  Ref: Nelson's 20/e p 610-615, Ghai 8/e p 637-639; Refer to pretext of this chapter for details
PAPPA is used for screening for Down syndrome in 1st trimester
90. d. All of above  Ref: Nelson's 20/e p 610-615, Ghai 8/e p 637-639
91. b. Endocardial cushion defect  Ref: Nelson's 20/e p 610-615, Ghai 8/e p 637-639
Most common cardiac defect in Down syndrome is Endocardial cushion defect or Atrioventricular septal defect
92. d. Noonan syn drome  Ref: Common Malformations, p 386, Nelson's 20/e p 610-615;
Important information about Simian crease:
•• The term “simian crease” is not used much anymore, since it tends to have a negative meaning (the word “simian” refers to a
monkey or ape); The crease is usually just referred to as a single transverse palmar crease.
•• A simian crease is a single line that runs across the palm of the hand.
Conditions in which similar crease is seen:
Chapter 5: Genetics and Genetic Disorders 99

Answers with Explanations


•• Down syndrome •• Fetal alcohol syndrome •• Rubella syndrome •• Pseudohypoparathyroidism
•• Aarskog syndrome •• Trisomy 13 •• Turner syndrome •• Gonadal dysgenesis
•• Cohen syndrome •• May be normally seen •• Klinefelter syndrome •• Cri du chat syndrome

93. a. Cognitive impairment  Ref: Nelson's 20/e p 610-615, Ghai 8/e p 637-639
In Down syndrome, developmental delay is universal and so is the most common finding and cognitive impairment does
not uniformly affect all areas of development.
94. a. Down syndrome  Ref: Pediatric Surgical Diagnosis pg 10, Ref: Nelson's 20/e p 610-615, Ghai 8/e p 637-639
Thirteen pairs of ribs may be seen in:
•• Aarskog syndrome •• Turner syndrome •• Holt-Oram syndrome
•• Alagille syndrome •• Incontinentia pigmenti •• Fetal akinesia deformation sequence
•• Idiopathic
95. c. Down syndrome  Ref: Nelson's 20/e p 610-615, Ghai 8/e p 637-639;
The given karyotype shows trisomy 21 (3 chromosomes on chr no 21) and so is suggestive of Down syndrome.
96. a. Prenatal diagnosis and advice abortion  Ref: Nelson's 20/e p 610-615;
For details about Recurrence risk of Down syndrome, refer to pretext of this chapter; Taking into view the chances of baby
affected with Downs syndrome, prenatal diagnosis is advocated and abortion advised, if fetus is affected.
97. a. Non disjunction  Ref: Nelson's 20/e p 610-615, Ghai 8/e p 637-639;
98. b. Non-disjunction in maternal meiosis  Ref: Nelson's 20/e p 610-615;
99. a. Deleted chromosome 21  Ref: Nelson's 20/e p 610-615, Ghai 8/e p 637-639;
An extra copy of chr 21 is required to cause trisomy 21 (Down syndrome), so deletion of chr 21 will not cause it.
100. a. Down syndrome  Ref: Nelson's 20/e p 609
Transient Myeloproliferative disorders in Down syndrome
•• 10% of children with Down syndrome may develop a transient leukemia or myeloproliferative syndrome with high leukocyte counts,
blast cells in peripheral blood, anemia, thrombocytopenia and hepatosplenomegaly
•• 20–30% of these will develop typical leukemia (often acute megakaryocytic leukemia) by 3 yr of life
•• GATA1 mutations (a transcription factor that controls megakaryopoiesis) are present in blasts from patients with Down syndrome who
have transient myeloproliferative disease and also in those with leukemia.
101. a. Echocardiography > c. X-ray cervical spine  Ref: Nelson's 20/e p 610-615, Ghai 8/e p 637-639
Congenital heart diseases are more common in Down syndrome (in 50%), than atlantoaxial instability (20%); so a preoperative
echocardiogram is warranted, so that management of child can be planned accordingly.
102. a. Increased PAPPA  Ref: Nelson's 20/e p 610-615, Ghai 8/e p 637-639
Decreased PAPPA (pregnancy associated plasma protein A) level is used in 1st trimester screening for Down syndrome.
Mnemonic

Levels of all biochemical markers decrease in Down syndrome except ‘HI' that increases:

So level of HCG and Inhibin increase, while, Level of a-fetoprotein, unconjugated estriol & PAPPA decrease
103. c. Coarctation of aorta is the most common cardiac lesion  Ref: Nelson's 20/e p 610-615, Ghai 8/e p 637-639
104. b. Undescended testis   Ref: Nelson's 20/e p 610-615, Ghai 8/e p 637-639
105. a. Sandle gap, c. Clinodactyly, d. Hypotonia  Ref: Nelson's 20/e p 610-615, Ghai 8/e p 637-639;
106. a. AML & c. ALL  Ref: Nelson's 20/e p 2443

•• Acute leukemia occurs 15-20 times more frequently in children with Down syndrome than in general population.
•• The ratio of ALL to AML in patients with Down syndrome is the same as that in the general population.
•• The exception is during the 1st 3 yr of life, when AML is more common.

107. c. Serum Prolactin level  Ref: Nelson's 20/e p 610-615, Ghai 8/e p 637-639
108. b. Down syndrome  Ref: Nelson's 20/e p 609-615
109. d. Normal intelligence  Ref: Nelson's 20/e p 610-615, Ghai 8/e p 637-639
110. a. Congenital heart disease, b. ALL, c. Early onset Alzheimer's disease, e. Infection  Ref: Nelson's 20/e p613
Frequent infections (sinusitis, nasopharyngitis, pneumonia) are seen in Down Syndrome
111. c. Duodenal atresia  Ref: Nelson's 20/e p 610-615, Ghai 8/e p 637-639
•• Most common cause of Intestinal obstruction in Down's syndrome is duodenal atresia.
112. d. Respiratory tract infection uncommon  Ref: Nelson's 20/e p 613
113. b. Simian crease and d. Mongoloid slant  Ref: Nelson's 20/e p 610-615, Ghai 8/e p 637-639
Upward sloping palpebral fissure is known as ‘mongoloid slant'.
100 Section 1: General Pediatrics

114. a. 100%  Ref: Nelson's 20/e p 610-615, Ghai 8/e p 637-639; Refer to pretext of this chapter for details
Review of Pediatrics and Neonatology

115. c. Down syndrome  Ref: Nelson's 20/e p 610-615, Ghai 8/e p 637-639

 TURNER SYNDROME
116. c. Male Turner (Noonan) has higher risk of cardiac diseases  Ref: Edward B Clark. Neck web and congenital heart defects: A
pathogenic association in 45 X-O Turner syndrome? Teratology 29(3):355-61
Discussing about options one by one,
a. False; Incidence of congenital heart disease has been found to be 30% in those with webbed neck and 9% of those with normal
neck (p value < 0.0005).
b. False
c. True; Cardiac disease is seen in almost 80% of individuals with Noonan syndrome; Noonan syndrome is one of the most common
genetic diseases associated with congenital heart defect, being second for frequency only to Down syndrome.
d. False; No such association seen till now.
117. a. Mental retardation  Ref: Nelson's 20/e p 2744-2746; Refer to pretext for Mnemonic on Turner Syndrome
118. d. High FSH & LH  Ref: Nelson's 20/e p 2744-2746, Ghai 8/e p 640-641
In Turner syndrome, the gonads (ovaries & uterus) are underdeveloped, but FSH & LH levels are increased.
119. d. Coarctation of aorta  Ref: Nelson's 20/e p 2744-2746, Ghai 8/e p 640-641
120. c. Turner Syndrome  Ref: Nelson's 20/e p 2744-2746, Ghai 8/e p 640-641
121. d. Turner's syndrome  Ref: Nelson's 20/e p 2744-2746, Ghai 8/e p 640-641
122. c. Turner  Ref: Nelson's 20/e p 2744-2746, Ghai 8/e p 640-641
123. c. Turner syndrome  Ref: Nelson's 20/e p 2744-2746, Ghai 8/e p 640-641
124. a. Turner syndrome  Ref: Nelson's 20/e p 2744-2746, Ghai 8/e p 640-641
The given karyotype shows single X chromosome and no Y chromosome and a total of 45 chromosomes which suggests 45, XO i.e
Turner syndrome.
125. b. XO  Ref: Nelson's 20/e p 2744-2746, Ghai 8/e p 640-641
126. b. Noonan syndrome  Ref: Nelson's 20/e p 2744-2746, Ghai 8/e p 640-641
127. b. Mental retardation  Ref: Nelson's 20/e p 2744-2746, Ghai 8/e p 640-641
128. b. Coarctation of aorta  Ref: Nelson's 20/e p 2744-2746, Ghai 8/e p 640-641
•• Most common heart disease in Turner syndrome is Bicuspid aortic valve, followed by coarctation of aorta
•• Out of the given options, best answer is coarctation of aorta.
129. c. Infertility in females  Ref: Nelson's 20/e p 2744-2746, Ghai 8/e p 640-641
For differences between Turner syndrome & Noonan syndrome, refer to pretext of this chapter
130. b. Turner syndrome  Ref: Nelson's 20/e p 2744-2746, Ghai 8/e p 640-641
131. a. Turner's syndrome  Ref: Nelson's 20/e p 2744-2746, Ghai 8/e p 640-641
132. a. Occurrence of Turner syndrome is influenced by maternal age  Ref: Nelson's 20/e p 2744-2746
Turner syndrome is not influenced by maternal age, rather, risk of Down syndrome increases with increase in maternal age.
133. d. 45, XO  Ref: Nelson's 20/e p 2744-2746, Ghai 8/e p 640-641
134. a. Primary amenorrhea, b. 45, XO, c. Short stature, d. Puberty usually late, e. Streak ovaries  Ref: Nelson's 20/e p 2744-
2746, Ghai 8/e p 640-641
135. b. Noonan syndrome  Ref: Nelson's 20/e p 2744-2746; Refer to pretext of this chapter for details
136. d. Mental retardation  Ref: Nelson's 20/e p 2744-2746, Ghai 8/e p 640-641

 OTHER IMPORTANT GENETIC DISORDERS


137. a. Russell Silver syndrome  Ref: Nelson 20/e p 3136
Features of Russell Silver syndrome:
•• Short stature
•• Craniofacial & body asymmetry
•• Low birth weight
•• Microcephaly
•• Triangular facies
•• Fifth finger clinodactyly
Chapter 5: Genetics and Genetic Disorders 101

138. a. De Morsier syndrome  Ref: Nelson 20/e p 784

Answers with Explanations


•• Also called septo-optic dysplasia
•• Underdevelopment of the optic nerve, pituitary gland dysfunction, and absence of the septum pellucidum
•• Two of these features need to be present for a clinical diagnosis—only 30% of patients have all three.
•• De Mosier first recognized the relation of a rudimentary or absent septum pellucidum with hypoplasia of the optic nerves and
chiasm in 1956.
•• Intracranial malformations associated with septo-optic dysplasia include agenesis of septum pellucidum, schizencephaly, and
lobar holoprosencephaly.
139. a. Ellis-van Creveld syndrome  Ref: Nelson 20/e p 619, 907, 2165, 3376
Discussing about the options, one by one,
a. Ellis-van Creveld syndrome Also called ‘chondroectodermal dysplasia'
Chondrodystrophy, polydactyly, ectodermal dysplasia, CHD (Single atrium, VSD) seen
b. Rubinstein- Taybi syndrome Microcephaly, ptosis, beaked nose with low-lying philtrum, broad thumbs and large
toes, intellectual disability , VSD
c. McKusick – Kaufman also called Cartilage-hair hypoplasia; Skeletal defects involving legs & rib cage; Immunodeficiency, celiac
syndrome disease & Hirschsprung disease may be associated
d. Edward syndrome VSD, ASD, PDA, coarctation of aorta, bicuspid aortic or pulmonary valve

140. b. First and second arch affected  Ref: Nelson 20/e p 1773
Treacher Collins syndrome is a congenital disorder involving structures developing from the first and second branchial arches.
141. a. Abdominal muscles are absent  Ref: Nelson 20/e p 1773
Pierre Robin syndrome
•• It consists of micrognathia usually accompanied by a high arched or cleft palate.
•• The tongue is usually of normal size, but the floor of the mouth is foreshortened.
•• The infant should be maintained in a prone position to relieve respiratory obstruction.
•• In some children, the jaw may achieve normal growth by 4-6 years of age
142. c. Ataxia  Refer answer 182 for details.
143. d. Abnormal intelligence
144. a. Russel Silver syndrome  Ref: Nelson's 20/e p 625, 2875
In Russell silver syndrome, short stature, triangular face (small jaw & pointed chin), & limb anomalies are seen;
It is an example of genomic imprinting disorder involving chromosome 11; Triangular facies is also seen in Alagille syndrome,
145. Ans. a. Trisomy 18  Ref: Nelson's 20/e p 890
Single umbilical artery

•• It is seen in 5-10/1,000 births & 35-70/1,000 in twin births (more common in twins).
•• 30% of these infants have congenital abnormalities; Trisomy 18 is one of the more frequent abnormalities.
•• Though it is associated with increased risk for occult renal anomaly, if no other anomalies are present, need for renal USG is controversial

146. b. Prader Willi syndrome  Ref: Nelson's 20/e p 624-626; Refer to pretext for details
147. a. 5p  Ref: Nelson's 20/e p 618-614, Ghai 8/e p 637-640
Cri-du-chat syndrome occurs due to deletion on chr 5p15.2
“Cat-like” cry, microcephaly, growth deficiency, intellectual disability and congenital heart disease are seen.
148. b. 45 XO genome  Ref: Nelson's 20/e p 618, Ghai 8/e p 637-640
Number of Barr bodies visible at interphase is always one less than the total number of X chromosomes;
So in 45 XO, where a single X chromosome is present, no. of Barr bodies is 1-1 = 0.
149. a. Gynecomastia with long thin limbs  Ref: Nelson's 20/e p 618, Ghai 8/e p 640
Given Karyotype is 47 XXY → Kilnefelters syndrome with Gynecomastia with long thin limbs as expected findings.
150. c. Germinal cells contain 23 chromosomes  Ref: Nelson's 20/e p 618, Ghai 8/e p 640
Discussing about the options one by one,
a. False: Only one of the X chromosomes is genetically active in female, the other X of either maternal or paternal origin undergoes
heteropyknosis and is rendered inactive called Barr body
b. False: In Klinefelter syndrome, there are 2 or more X chromosomes and 1 or more Y chromosomes (extra Y chromosome in a male
would mean XY + Y = XYY)
c. True: Germs cells have haploid set of 23 chromosomes
d. False: Turners is due to missing X and not due to extra X chromosome.
151. d. 5p-  Ref: Nelson's 20/e p 618, Ghai 8/e p 640
102 Section 1: General Pediatrics

152. a. Karyotyping  Ref: Nelson's 20/e p 618, Ghai 8/e p 637


Review of Pediatrics and Neonatology

153. b. Pierre-Robin syndrome  Ref: Nelson's 20/e p 618, Ghai 8/e p 637
This child has a small chin (micrognathia) and retrognathia which is suggestive of Pierre Robin syndrome.
154. d. Nucleotide excision repair  Ref: Nelson's 20/e p 620
Patients with xeroderma pigmentosum have defects in DNA damage recognition or in nucleotide excision repair pathway. Exposed
skin is dry and pigmented and is susceptible to the mutagenic effects of ultraviolet irradiation.
155. b. Subvalvular aortic stenosis  Ref: Nelson's 20/e p 618-619
Williams syndrome usually presents with Supravalvular aortic stenosis, hypercalcemia and hypertension;
Face of a child with William's syndrome shows: ‘elfin facies' with small upturned nose, long philtrum, wide mouth, full cheeks,
full lips, small chin, and puffiness around the eyes.
156. c. Chromosome 13  Ref: Nelson's 20/e p 618-619; Refer to pretext for mnemonic
157. a. Y chromatin  Ref: Nelson's 20/e p 618-619, Ref: Emery Genetics
Quinacrine, a fluorescence dye, binds strongly to Y chromosome forming a bright fluorescent spot (F body). This is clearly visible in
stained interphase cells from various tissues from human male and in mature spermatozoa
158. a. Cri-du-chat syndrome  Ref: Nelson's 20/e p 618-619; Longman 11/e p 21;
In the karyotype, there is deletion of chromosome 5, which leads to Cri-du-chat syndrome.
159. b. Di George syndrome  Ref: Nelson's 20/e p 618-619
160. d. Trisomy 13  Ref: Nelson's 20/e p 618-619
161. d. 47, XXY  Ref: Nelson's 20/e p 618-619
162. a. Cleido cranial dysplasia Ref: Nelson's 20/e p 618-619
Cleidocranial dysostosis, also called cleidocranial dysplasia or mutational dysostosis, is a hereditary congenital disorder, where
there is delayed ossification of midline structures; Absent clavicles and delayed dentition.
163. a. Arm spam > Height, b. Ectopia lentis, c. Hypermobility of joint and e. Arachnodactyly  Ref: Nelson's 20/e p 618–619,
644-645; For details about Homocystinuria, refer to pretext of Inborn errors of metabolism;
Lens dislocation in Marfan syndrome is Supero-lateral (Remember: “M-S-L”), while in Homocystinuria it is Infero-nasal.
In homocystinuria, skeletal abnormalities resembling those of Marfan syndrome seen like tall stature, arachnodactyly, scoliosis,
pectus excavatum or carinatum, genu valgum, pes cavus, high-arched palate, & crowding of the teeth are seen and progressive
intellectual disability is common, but normal intelligence has been reported; In Marfan syndrome, intelligence is usually normal
164. c. Low intelligence  Ref: Nelson's 20/e p 618-619
Treacher Collins syndrome
Also called Mandibulofacial dysostosis
Inheritance Autosomal dominant syndrome that primarily affects the face
Identifying •• Down-slanting eyes (anti-mongoloid slant)
features •• Drooping part of the lateral lower eyelids & coloboma of lower eyelids
•• Micrognathia (small jaw) with underdeveloped & sunken cheek bones
•• Malformed or absent ears &/or deafness
•• Cleft palate; Teeth may be hypoplastic, or displaced
•• Intelligence is usually normal

Associated with Breathing and feeding problems


Treatment Surgery to restore normal structure of the face can be performed, which may include repair of cleft palate, zygomatic
and orbit reconstruction, reconstruction of lower eyelid and external ear
165. b. XY  Ref: Nelson's 20/e p 618-619
166. d. Rocker bottom foot  Ref: Nelson's 20/e p 618-619
Rocker bottom foot is a feature of Trisomy 18, not trisomy13
167. b. Patau syndrome  Ref: Nelson's 20/e p 618-619
168. a. Bilateral microphthalmia  Ref: Nelson's 20/e p 618-619; Refer to pretext for mnemonic
169. b. Trisomy 18  Ref: Nelson's 20/e p 618-619
170. d. Rocker bottom feet  Ref: Nelson's 20/e p 618-619
171. c. Trisomy 13  Ref: Nelson's 20/e p 618-619
172. d. Partial trisomy 22  Ref: Nelson's 20/e p 2165; Harrison's 19/e p 83e6
Chapter 5: Genetics and Genetic Disorders 103

Cat eye syndrome: It is due to trisomy or tetrasomy of short arm & small part of long arm of chromosome 22. Features seen in cat

Answers with Explanations


eye syndrome include:
•• Ocular coloboma of iris, choroid or optic nerve •• Renal malformation—U/L renal agenesis
•• Preauricular skin tags or pits •• Antimongoloid slant of eye
•• Congenital heart disease •• Microphthalmia
173. a. Long neck  Ref: Nelson's 20/e p 618-619; Refer to pretext for details
174. d. A polygenic syndrome  Ref: Nelson's 20/e p 618-619
The given child has microcephaly, thin upper lip & growth & developmental delay. These features can be seen in:
a. A chromosomal syndrome like Tetrasomy 12p (Pallister-Killian syndrome)
b. A teratogenic syndrome like fetal alcohol syndrome
c. A mendelian syndrome like Rubinstein-Taybi syndrome
Hence, a polygenic syndrome is least likely in the given scenario
In a polygenic disease, multiple genes contribute to the disease like hypertension & diabetes
175. c. Klinefelter syndrome Ref: Nelson's 20/e p 618-619
Klinefelter syndrome is 47, XXY; In none of the other 3 disorders, abnormality of X or Y chromosomes is seen
176. c. 47 XXY  Ref: Nelson's 20/e p 618-619
This is clearly a case of Klinefelter syndrome:
•• The male has a normal external genitalia but there is microorchidism and decreased testosterone level.
•• The male in Kleinfelter syndrome is tall with long arms and legs.
177. a. Trisomy 13  Ref: Nelson's 20/e p 618-619; Refer Ans. 140 above
178. b. Bilateral microphthalmos  Ref: Nelson's 20/e p 618-619; Refer Ans. 140 above
179. c. Trisomy 21  Ref: Nelson's 20/e p 618-619
•• Down syndrome (Trisomy 21) is the most common of the chromosomal disorder.
•• Trisomy 16 is the most common trisomy overall, however most do not survive and die in utero
Hence, among the live births, or among children, most common chromosomal abnormality is Trisomy 21

 TRINUCLEOTIDE REPEATS
180. b. Myotonia dystrophica  Ref: Nelson's 20/e p 600-602
181. d. Myotonic dystrophy  Ref: Nelson's 20/e p 600-602
Anticipation—Clinical features of a genetic disorder worsens with each successive generation. Seen in trinucleotide repeat disorders
like fragile X syndrome and myotonic dystrophy.
182. b. Fragile-X-syndrome  Ref: Nelson's 20/e p 600-602
183. a. Large nose  Ref: Nelson's 20/e p 600-602

 MITOCHONDRIAL INHERITANCE
184. d. Fragile X Syndrome  Ref: Nelson's 20/e p 600-601, Ghai 8/e p 644; Refer to pretext for details
Mitochondrial Disorders, include Leber Hereditary Optic Neuropathy, MELAS (Mitochondrial encephalomyopathy, lactic acidosis,
and stroke like episodes), MERRF (Myoclonic epilepsy, ragged red fibers) in muscle, & Pearson syndrome
185. c. Mitochondrial  Ref: Nelson's 20/e p 600, Ghai 8/e p 644
186. c. Mitochondrial disorder  Ref: Nelson's 20/e p 600, Ghai 8/e p 644

 GENOMIC IMPRINTING AND UNIPARENTAL DISOMY


187. a. Prader-Willi syndrome  Ref: Nelson's 20/e p 624-626, Ghai 8/e p 637
•• Genomic imprinting occurs when the phenotypic expression of a gene depends on the parent of origin
•• Classic examples: Prader-Willi syndromeQ and Angelman syndromeQ, Beckwith-Wiedemann syndromeQ, Russell- Silver
syndromeQ, pseudohypoparathyroidism and neonatal diabetes.
188. d. Ghrelin  Ref: Nelson's 20/e p 624-626, Ghai 8/e p 637
Prader-Willi syndrome:
•• It is caused by lack of expression of genes on the paternally inherited chromosome 15q11.2-q13
•• In infancy it is characterized by hypotonia with poor suck resulting in failure to thrive
•• As the child ages, developmental delay, cognitive disability, behavior problems and obesity become evident
•• Hypothalamic dysfunction may lead to hyperphagia, temperature instability, high pain threshold, sleep disorders
•• Endocrine complications include GH deficiency, adrenal insufficiency, hypogonadism, hypothyroidism, type 2 diabetes
•• Circulating total ghrelin levels are elevated in older children, which may contribute to hyperphagia & increased weight gain
104 Section 1: General Pediatrics

189. b. Prader-Willi Syndrome  Ref: Nelson's 20/e p 624-626, Ghai 8/e p 637
Review of Pediatrics and Neonatology


Uniparental disomy of maternal chr 15 as well as Paternal chr 15 deletion give rise to Prader-Willi syndrome.
190. c. Uniparental disomy  Ref: Nelson's 20/e p 624-626, Ghai 8/e p 637
191. b. Bloom syndrome  Ref: Nelson's 20/e p 624-626, Ghai 8/e p 637
192. a. Prader-Willi syndrome  Ref: Nelson's 20/e p 624-626, Ghai 8/e p 637

 DIAGNOSTIC TOOLS
193. a. 18-20 weeks  Ref: Ghai Pediatrics 8/341-344; ReferAns below
194. d. Cordocentesis  Ref: Multiple sources including Ghai Pediatrics 8/341-344; Thalassemias p 160-161
•• Thalassemia major is caused by numerous mutations, prenatal diagnosis is difficult
•• Prior identification of the familial mutation is required to offer prenatal diagnosis
In the given scenario, the lady has already 1 child with thalassemia, so both she & her husband are carriers; As she has presented late
in pregnancy (18 weeks), CVS or Amniocentesis cannot be done. Hence, Cordocentesis is the investigation of choice.
NIPT (Noninvasive prenatal testing) is an upcoming diagnostic modality which is not readily available at most of the centres currently.
Invasive methods of prenatal diagnosis for genetic disorders:
Chorionic Villus •• Done under ultrasound guidance between 10-12 weeks of gestation
Sampling (CVS) •• Disadvantage: Increased risk of abortions & fetal limb defects
Amniocentesis Fetal DNA analysis done by extracting DNA from amniotic fluid, done between 15-17 wk gestation;
Complications: Pregnancy loss, Amniotic fluid leakage, vaginal bleeding
Cordocentesis or Per- •• It examines blood from fetus to detect fetal abnormalities & is done after 17 weeks gestation
cutaneous umbilical •• Especially useful for prenatal diagnosis in couples who are referred late, as often happens in India.
cord blood sampling •• Fetal blood can be analyzed directly by HPLC after ensuring that there is no maternal contamination
(PUBS) •• When the percentage of HbA (adult Hb) in the fetal sample is <1%, mutation analysis must be done to
differentiate a heterozygous & homozygous β-thalassemia fetus

195. b. Metaphase  Ref: Nelson's 20/e p 605-606, Ghai 8/e p 645


Chromosomes are examined after arresting dividing cells in metaphase with mitotic spindle inhibitors (e.g., N-diacetyl-
N-methylcolchicine), followed by staining.
196. a. Multiplex ligation-Dependent probe amplification  Ref: Nelson's 20/e p 605, Ghai 8/e p 6045
Multiplex Ligation-Dependent Probe Amplification (MLPA): It is a technique which mixes DNA hybridization, ligation, & PCR
amplification to detect deletions & duplications of any size, including anomalies that are too large to be detected by PCR and too
small to be identified by FISH.
197. a. FISH  Ref: Nelson's 20/e p 605, Ghai 8/e p 645
FISH involves DNA probes that recognize sequences specific to particular chromosomal regions (diagnosis of known loci)
198. c. Chromosomal abnormalities  Ref: Nelson's 20/e p 605, Ghai 8/e p 645
199. b. Metaphase  Ref: Nelson's 20/e p 606, Ghai 8/e p 645
200. a. Study of multiple genes  Ref: Nelson's 20/e p 606, Ghai 8/e p 645
201. b. G-banding  Ref: Nelson's 20/e p 606, Ghai 8/e p 645
G-banding which includes staining with Giemsa is the most common technique used for karyotyping.

 GENE THERAPY

202. a. SCID  Ref: Nelson's 20/e p 584-586


FDA-approved gene therapy is used for ADA-SCID, Leber's congenital amaurosis, multiple myeloma & Adrenoleukodystrophy;
203. b. Leukemia  Ref: Nelson's 20/e p 584-586
Gene therapy has been found to be potentially useful in disorders like SCID (due to Adenosine deaminase deficiency), Cystic fibrosis,
Thalassemia, not in leukemia (till now), where chemotherapy is required.
204. b. Genome integration  Ref: Nelson's 20/e p 584-586
Problems with gene therapy especially with viral vectors include toxicity, immune and inflammatory responses.
205. a. Electroporation, d. Intranuclear injection  Ref: Nelson's 20/e p 584-586
Gene therapy: An experimental technique that uses genes to treat or prevent disease. Delivery of DNA into cells, can be done using
Recombinant viruses or ‘viral vectors' or naked DNA or DNA complexes
Methods used include: Injection, Electroporation, ‘gene-gun', sonoporation, magnetofection, & use of oligonucleotides,
lipoplexes or nanoparticles.
Chapter 6
Inborn Errors of
Metabolism
When to Suspect a Metabolic Disorder in Neonates Infants?
Question 1 M
•• Rapidly progressive encephalopathy ± seizures In which of the following diseases,
•• Sepsis like presentation with –ve sepsis screen, the eye finding shown below is
•• Persistent or recurrent vomiting, peculiar body/ urine odor seen?
•• Parental consanguinity, significant family history
•• After an initial normalcy metabolic acidosis, ketosis, hypoglycemia.

When to Suspect a Metabolic Disorder in Older Children?


•• Unexplained acute illness in a previously well child with episodic presentation
•• Recurrent coma/stroke/ataxia/cramps worsening with febrile illnesses
•• Multisystem involvement with family history of similar illness/sib deaths.
a. Gaucher disease
Important Clinical Pointers for Inborn Errors of Metabolism M
b. Von Gierke disease
Clinical finding Disorder c. Galactosemia
d. GM1 gangliosidosis
Coarse facies Mucopolysaccharidosis, GM1 Gangliosidosis
Cataract Galactosemia,Q Wilson diseaseQ
Retinitis pigmentosa Mitochondrial disorders Question 2
Cherry red spot GM1 GangliosidosisQ, Tay Sachs dsQ, Niemann Pick ds Which of the following meta­bolic
Eczema/alopecia Biotinidase deficiency, Multiple carboxylase deficiency disorders is associated with this
Abnormal kinky hair Menke diseaseQ finding seen on fun­doscopic exa­
Decreased pigmentation Phenylketonuria, Albinism mination of eyes?

 IMPORTANT DISORDERS OF CARBOHYDRATE METABOLISM


•• Glycogen storage diseases
•• Galactosemia
•• Hereditary Fructose intolerance.

A. GLYCOGEN STORAGE DISEASES (GSD)


a. Galactosemia
b. Mitochondrial disorders
c. Albinism
d. GM1 Gangliosidosis

Mnemonic
Muscle Glycogenoses 2 + 5 = 7
Type Disease Enz def.
II Pompe ds Alpha 1, 4
glucosidase
V Mc Ardle ds Muscle
phosphorylase
Von Gierke Disease (Type I Glycogenosis) M
VII Tarui ds Phosphofruco­
It is an autosomal recessive disorder due to deficient activity of glucose-6-phosphatase in Liver, kinase
Kidney, Intestinal mucosa.
106 Section 1: General Pediatrics

Types
Review of Pediatrics and Neonatology

Mnemonic
•• Type Ia: glucose-6-phosphatase is defective
Liver glycogenoses: “Violet Colour
•• Type Ib: Translocase is defective
Accha Hai”
Note: Translocase transports glucose-6-phosphate across microsomal membrane; glucose-6-
Type Disease Enz def phosphatase acts inside the microsome.
I Von Gierke Glucose 6
ds phosphatase Clinical Features
•• Hypoglycemia: usually presents with early morning lethargy ± seizures
III Cori Debranching
disease enz •• Hepatomegaly & renomegaly with Protuberant abdomen but normal spleen size
IV Anderson Branching enz
•• Doll like face with Short stature with thin extremities
ds Pathogenesis
VI Her’s Liver
disease phosphorylase

Question 3
A 6-month-old male baby (photo­
graph shown below) pre­sents with
recurrent hypo­glycemic attacks
and hepato­megaly. Which disease
is he likely to be suffering from?

Biochemical Findings in Von Gierke Disease


a. Pompe disease • Hypoglycemia • Hyperuricemia
b. Von Gierke disease • Hyperlipidemia • Lactic acidosis
c. Her’s disease
d. Mc Ardle disease Definite Diagnosis: Liver biopsy, gene based mutational analysis.
Treatment
•• Frequent high carbohydrate feeds like cornstarch including night time feeds
High Yield Points   M •• Control of hyperuricemia, hypercholesterolemia
•• Most common Glycogen storage
•• Liver transplantation.
disease in children is Von Gierke
disease (type I GSD)
Question 4
•• Most common Glycogen storage The clinical photographs, Chest X-ray and ECG findings of a 9-month-old baby are shown
disease in adolescents and adults is below. Which enzyme is deficient in this child?
Mc Ardle disease (type V GSD)
D

High Yield Points


A C
Fanconi Bickel syndrome is type XI GSD
due to defect in Glucose transporter–2 a. Glucose 6 phosphatase b. Muscle phosphorylase
(Glut 2). c. Acid maltase d. Liver phosphorylase
Chapter 6: Inborn Errors of Metabolism 107

Pompe Disease (GSD Type II)

Section 1: General Pediatrics


M
High Yield Points   M
Enzyme deficient: Alpha 1,4 glucosidase or Acid maltase Diseases for which Enzyme Replace-
ment Therapy (ERT) is available are:
Clinical features: • Floppy infant, coarse facies     •  Hypotonia •• Gaucher diseaseQ
• Cardiomegaly, Heart failure      •  Hepatomegaly •• Pompe disease (GSD type II)Q
•• Hurler syndrome (MPS type l)Q
Diagnosis •• Enzyme assay in leukocytes: Deficient Acid maltase
•• Maroteaux Lamy disease (Type VI
•• Chest X-ray: Cardiomegaly MPS)
•• ECG: Tall QRS complexes and short PR interval •• Fabry disease
Treatment •• Enzyme replacement therapy and supportive measures

Prognosis •• In the infantile form, death occurs before 2 years of age

B. GALACTOSEMIA M

Caused By
Deficiency of any of these 3 enzymes:
•• GALT (Galactose 1 PO4 uridyl transferase)Q (most common)
•• Galactokinase (may present with cataract alone)
Mnemonic
•• Epimerase. Important Metabolic diseases in
which hypoglycemia is seen:
Mode of Inheritance “Galat FAHMI”
Glycogen storage diseases
Autosomal recessive Fatty acid oxidation defects
gAlactosemia
Pathophysiology Hereditary fructose intolerance
Maple syrup urine disease
GALT def. → Galactose-1-P accumulates → Injury to kidney, liver and brain Mitochondrial diseases
Galactokinase def. → accumulation of galactose & galactitol
The Duarte variant, a single amino acid substitution, has 50% of normal RBC enzyme activity,
but usually no clinical significance.
This variant is the most common, with a carrier frequency of 12% in the general population.

Clinical Features
Usually appear after intake of milk (lactose)
•• Jaundice, Hepatomegaly, diarrhea/Vomiting, Failure to thrive. Question 5 M
•• Seizures +/- Intellectual disability, Cataract.
In which of the following meta­bolic
•• Megalencephaly causing large head may be seen. disorders, the following finding is
not seen?
Diagnosis
•• Reducing substance in urine while the patient is on human milk, cow's milk, or any other
formula containing lactose (by Clinitest, dipstix or Benedict’s test)
•• Direct enzyme assay using erythrocytes establishes the diagnosis
•• Carrier testing and prenatal diagnosis can be performed by direct enzyme analysis or DNA
testing of amniocytes or chorionic villi.

Treatment
a. Galactosemia
Eliminate Milk & dairy products containing lactose, from diet. b. Diabetes
c. Phenylketonuria
High Yield Points   M d. Wilson disease
•• Breastfeeding is contraindicated in Galacto­semia
•• Most common cause of sepsis in a child with Galactosemia is E. coli
•• Cataracts are usually the sole manifestation of galactokinase deficiency.

C. HEREDITARY FRUCTOSE INTOLERANCE M

Enzyme deficient: Aldolase B


Pathogenesis: Ingestion of fructose (sugar) leads to accumulation of fructose-1-phosphate
which causes severe toxic symptoms, when exposed to fructose.
108 Section 1: General Pediatrics

Clinical Features
Review of Pediatrics and Neonatology

Question 6
A 3-year-child presents with fail­ •• Seen on ingestion of fructose or sucrose (table sugar, fruit, juice, or syrups):
ure to thrive & seizures. On exami­ –– Hypoglycemia/Jaundice/Vomiting
nation, he shows the following. –– Fructosuria (this explains the presence of reducing substance in urine)
Which of the following diagnoses •• If the intake of fructose persists, hypoglycemic episodes recur, and liver and kidney
is likely? failure progress, eventually leading to death
•• Symptoms may occur early in life, if foods or formulas containing sugars are introduced
•• Child develops an aversion to sweet food.

Diagnosis
•• Reducing substance in urine +ve during an episode (Benedict test +ve)
•• IV fructose tolerance test: rapid fall of blood glucose, & subsequent increase in uric acid
•• Definitive diagnosis is made by assay of aldolase B enzyme activity in the liver
a. GSD type 1 •• Gene-based diagnosis is possible.
b. Galactosemia
Treatment
c. Congenital disorder of Glycosylation
d. Tuberous sclerosis •• Complete elimination of all sources of sucrose, fructose, and sorbitol from the diet
•• Intellectual development is usually unimpaired, if diet restricted.

 IMPORTANT DISORDERS OF AMINO ACID METABOLISM


Question 7
A 6-month-baby presented with Disorders of amino acid meta­bolism include:
recurrent sei­zures and develop­ •• Multiple Carboxylase defici­encyQ
mental delay. On exami­nation •• Maple Syrup Urine disease (MSUD)Q
there was alopecia and scaly •• PhenylketonuriaQ
skin rashes. Investi­gations re­ •• AlkaptonuriaQ
vealed metabolic acidosis, el­ •• Urea cycle defectsQ
evated lactates & keto­nuria. What •• Hartnup disorderQ
could be the under­lying enzyme
deficiency? A. ORGANIC ACIDEMIAS
Important organic acidemias are:
•• Methylmalonic acidemia
•• Propionic acidemia
•• Isoavaleric acidemia
•• Maple syrup urine disease
•• Glutaric acidemia
•• Multiple carboxylase deficiency
Salient Features
a. Phenylalanine hydroxylase •• Age at onset and clinical features: Variable
b. Epimerase •• Lethargy, poor feeding, vomiting, seizures, developmental delay, specific odors
c. Multiple carboxylase •• Metabolic acidosis, lactic acidosis, ketosis, hyperammonemia, hypoglycemia
d. Glucose-6-phosphatase •• Neutropenia: Methylmalonic / Isovaleric / Propionic acidemia.

An Approach to IEM with Acidosis


Chapter 6: Inborn Errors of Metabolism 109

MULTIPLE CARBOXYLASE DEFICIENCY

Section 1: General Pediatrics


M
High Yield Points   M
Inheritance Autosomal recessive disorder
Organic acidemias with charac­teristic
Clinical features •• Breathing difficulty, Feeding problems, hypotonia, seizure, dev. delay odor
•• Generalized erythematous rash with exfoliation & alopeciaQ; failure to thrive IEM URINE ODOR
•• Tomcat urine odorQ
Glutaric acidemia Sweaty feet
Lab findings Metabolic acidosis, ketosis, hyperammonemia
Diagnosis Confirmed by enzyme assay in lymphocytes MSUD Maple syrup
Treatment 10 mg/day biotin; early diagnosis & Rx prevents irreversible neurologic damage Tyrosinemia Boiled cabbage
Multiple carbox- Tomcat
Biotin is a co-factor for carboxylase enzymes: ylase deficiency
•• Acetyl CoA carboxylase
•• Pyruvate carboxylase Phenylketonuria Mousy or musty
•• Propionyl CoA carboxylase
•• Methylcrotonyl CoA carboxylase
•• Multiple carboxylase

MAPLE SYRUP URINE DISEASE (MSUD) M


High Yield Points
It is an autosomal recessive disorder of branched chain amino acid, i.e. valine, leucine &
Isoleucine, due to defect in branched chain α-keto acid dehydrogenase. Thiamine pyrophosphate (vitamin B1)
acts as a cofactor for branched chain
Clinical Features α-keto acid dehydrogenase.

•• Usually normal at birth but develop poor feeding, vomiting & lethargy
•• Physical examination reveals hypertonicity and muscular rigidity with opisthotonos
•• Periods of hypertonicity may alternate with bouts of flaccidity
•• Convulsion occurs in most infants, and hypoglycemia is common.
Diagnosis
•• Increased leucine, isoleucine, valine in blood & urine, seen on HPLC / electrophoresis
•• DNPH test: Yellow colorQ & Ferric Chloride test: Blue color.Q

B. UREA CYCLE DEFECTS


Ammonia is mostly derived from catabolism of amino acid & is converted to urea by urea cycle

High Yield Points


Enzymes involved in Urea cycle
Mitochondrial:
•• Carbamyl phosphate synthetase 1
•• Ornithine transcarbamylase 2
•• Ornithine 5-aminotransferase 6
•• N-acetylglutamate synthetase 7
•• Citrin 8
Cytosolic:
•• Argininosuccinic acid synthetase 3
•• Argininosuccinic acid lyase 4
•• Arginase 5
110 Section 1: General Pediatrics

Approach to a Child with Hyperammonemia


Review of Pediatrics and Neonatology

High Yield Points   M


Urea cycle defects cause:
•• Hyperammonemia
•• Encephalopathy
•• Vomiting

High Yield Points


HHH syndrome
•• It is Hyperammonemia-Hypero-
rnithinemia Homocitrullinemia
syndrome
•• There is a defect in the transport
system of ornithine from the cytosol
into the mitochondria.

Salient features of various Urea Cycle Disorders

Defective enzyme Laboratory findings (during acute attack)

Carbamyl Phosphate Markedly increased glutamine & alanine with relatively low levels of citrulline
Synthetase (CPS) or (differentiates from Transient hyperammonemia of newborn) and arginine
N-Acetylglutamate Urinary orotic acid is usually low or may be absent
(NAG) Synthetase

Ornithine Elevated plasma glutamine & alanine with low citrulline, arginine & BUN
Transcarbamylase Marked increase in urinary orotic acid (diff. from CPS deficiency)

Argininosuccinate Elevated plasma citrulline to 50-100 times normal (diff. from OTC deficiency)
Synthetase (AS) Urinary excretion of orotic acid is moderately increased; crystalluria due to
(Citrullinemia) precipitation of orotates may also occur

Argininosuccinate Marked increase in plasma argininosuccinic acid, citrulline (less than in


Lyase (AL) citrullinemia) glutamine, alanine & transaminases
Argininosuccinic acid is found in large amounts in urine & CSF

Arginase Elevated arginine in plasma & CSF; Increased urine orotic acid
(Hyperargininemia) Guanidino compounds (α-keto-guanidinovaleric acid, argininic acid) are
markedly increased in urine

  Hyperammonemia is seen in all urea cycle defects except Arginase deficiency where
High Yield Points ammonia levels may be normal or mildly elevated.
Children born to mothers with PKU
have: C. DISORDERS OF PHENYLALANINE METABOLISM
•• Microcephaly
•• Mental retardation Important diseases of Phenylalanine metabolism:
•• Growth retardation •• Phenylketonuria
•• Congenital heart disease •• Albinism
Chapter 6: Inborn Errors of Metabolism 111

•• Tyrosinemia

Section 1: General Pediatrics


•• Alkaptonuria
Question 8
This fair-skinned child may be suf­
fering with which of the following
diseases?

a. Von Gierke disease


b. Alkaptonuria
c. Albinism
d. Tyrosinemia

PHENYLKETONURIA (PKU)
Autosomal recessive condition due to deficiency of phenylalanine hydroxylase.
Question 9
This child presented with hypopig­
Pathophysiology mentation & the plasma HPLC
showed reduced Tyrosine levels.
Identify the underlying disease.

a. Tyrosinemia
b. Phenylketonuria
c. Alkaptonuria
Clinical Manifestations of Phenylketonuria d. Galactosemia
•• Normal at birth; Later, intellectual disability
•• Blond hair, blue iris, fair skin; musty or mousy body odour; Vomiting
•• Neurological signs — Irritability, tremors, convulsions, hypertonia, exaggerated reflexes
•• Dental Enamel hypoplasia, growth retardation, Microcephaly

Diagnosis
•• FeCl3 test with urine gives green colour (detects phenylalanine in urine) High Yield Points
•• Elevated blood Phenylalanine level (Guthrie’s test: detects phenylalanine in serum).
Important screening tests for meta­
bolic disorders
Treatment •• TMS (Tandem Mass Spec­trometry)
•• Low phenylalanine diet should be started as soon as the diagnosis is made of dried blood spots
•• Tyrosine becomes an essential A.A. in this disorder, and its adequate intake must be ensured. •• GCMS-(Gas Chromatography Mass
Spectroscopy)- of urine

L at e s t U p d at e s
•• In infants with PKU, excess phe­nylalanine metabolites like phenyl­pyruvate & phenylacetate accu­mulate in High Yield Points
brain. But these metabolites have no role in pathogenesis of CNS damage in PKU;
Acquired hypertyrosinemia may
•• CNS damage in PKU is caused by elevated phenylalanine in brain tissue, which causes inhibition of occur in liver failure, scurvy (vitamin C is
cerebral uptake of other neutral amino acids such as tyrosine & tryptophan. cofactor for 4-HPPD) & hyperthyroidism.
112 Section 1: General Pediatrics

ALKAPTONURIA
Review of Pediatrics and Neonatology

•• Autosomal recessive disorder due to deficiency of enzyme homogentisic acid oxidase.


•• Clinical features:
–– Oochronosis (dark spot on sclera/ ear cartilage); Arthritis
–– Darkening of urine on standing (due to oxidation of Homogentisic Acid)
–– High incidence of heart ds (mitral/ aortic valvulitis/ calcification).
Question 10 M

An 8-year-old child presents with TYROSINEMIA


black spots on sclera and ear. His
Tyrosinemia type I
urine sample was collected for ex­
•• It is a severe disease of liver, kidney & peripheral nerves
amination (A). After 30 minutes of
•• Caused by deficiency of fumaryl acetoacetate hydrolase (FAH).
collection, the appearance of urine
•• Increased succinylacetone in serum & urine, & increased blood alpha-fetoprotein; boiled
sample had changed to (B).
cabbage urine odor present;
What is the most probable enzyme •• Decreased hepatic coagulation factors & increased transaminases;
deficiency, child is suffering from? •• Treatment of choice is Nitisinone, which inhibits tyrosine degradation at 4-HPPD
•• Long standing complications are cirrhosis & hepatocellular carcinoma.
Tyrosinemia type II
•• It is an autosomal recessive disorder caused by deficiency of tyrosine aminotransferase
•• Palmar/plantar hyperkeratosis, corneal ulcers & intellectual disability seen.
Tyrosinemia type III
•• It is due to primary deficiency of 4-hydroxy phenyl pyruvate dioxygenase [4-HPPD]
A B •• Developmental delay, seizures, intermittent ataxia, and self-destructive behavior are seen.

a. Phenylalanine hydroxylase D. HARTNUP DISORDER


b. Epimerase
c. Multiple carboxylase Basic Defect
d. Homogentisic acid oxidase
Defect in transport of monoamino-monocarboxylic amino acids (neutral amino acids)Q by
the intestinal mucosa and renal tubules; Autosomal recessive inheritance.
Question 11 M
Clinical Features M
In which of the following diseases,
the skin rash shown in this picture •• Most children with Hartnup defect remain asymptomatic
is usually seen? •• In symptomatic patients cutaneous photosensitivity and pellagra-like rash on sun exposure
•• Some patients may have intermittent ataxia manifested as an unsteady, wide-based gait.

Diagnosis
•• Aminoaciduria, which is restricted to neutral amino acids (alanine, serine, threonine, valine,
leucine, isoleucine, phenylalanine, tyrosine, tryptophan, and histidine)
•• Urinary excretion of proline, hydroxyproline, and arginine remains normal.

Treatment M
a. Multiple Carboxylase deficiency
b. Biotinidase deficiency Nicotinic acid or nicotinamide (50–300 mg/24 hr) and a high-protein diet.
c. Alkaptonuria
d. Hartnup disorder
E. HOMOCYSTINURIA
Basic Defect
•• Classic Homocystinuria is caused by Cystathionine β-Synthase deficiency
•• Homocystinuria can also be caused by deficiency of Methylene tetrahydrofolate Reductase
& defects in Methylcobalamin formation.

Clinical Features of Homocystinuria


•• Normal at birth; develop failure to thrive & developmental delay.
•• Ectopia lentis (infero nasal subluxation) usually after 3 yr age;
•• Progressive intellectual disability is common; higher IQ in vit B6–responsive patients.
•• Psychiatric & behavioral disorders seen in > 50% of patients. Convulsions in 20%;
•• Skeletal abnormalities resembling Marfan syndrome seen like tall stature, arachnodactyly,
scoliosis, pectus excavatum, genu valgum, pes cavus & high-arched palate seen
•• Generalized osteoporosis, especially of spine (main X-ray finding).
Chapter 6: Inborn Errors of Metabolism 113

Skeletal and Ocular Abnormalities in Homocystinuria

Section 1: General Pediatrics


Complications
•• Thromboembolic episodes involving both large & small vessels, especially of brain.
•• Spontaneous pneumothorax & acute pancreatitis are rare complications.

Diagnosis of Homocystinuria
•• Elevations of methionine & homocysteine in body fluids; Cystine is low in plasma.
•• Confirmatory diagnosis: by enzyme assay in liver biopsy / fibroblasts, or DNA analysis.

Treatment of Homocystinuria M

•• High doses of vitamin B6 (200–1,000 mg/24 hr) & folic acid causes dramatic improvement;
•• Restriction of methionine intake along with cysteine supplementation is recommended
for patients unresponsive to vitamin B6.
High Yield Points
 LYSOSOMAL STORAGE DISEASES (LSD) •• All lysosomal storage diseases
•• These are metabolic disorders caused by mutations in genes encoding a single lysosomal are autosomal recessive, except
enzyme, resulting in intracellular accumulation of undegraded substrates Fabry’s and Hunter’s disease, which
are both X-linked recessive
•• Most LSDs are inherited in autosomal recessive manner.

Question 12 M
Important Lysosomal Storage Disorders
A 3-year-old child presented with
Disease Enzyme deficient Cherry red spot Visceromegaly Skeletal lesions anemia and throm­bocytopenia. On
GM1 Beta galactosidase + + + examina­tion, there was massive
Gangliosidosis splenomegaly. A bone marrow
aspiration revealed the following
Gaucher disease GlucocerebrosidaseQ _ + +
cells. What is the diagnosis?
Niemann Pick ds SphingomyelinaseQ + + _

Tay Sachs ds Hexosaminidase AQ + _ _

A. GAUCHER DISEASE M

Basic Defect: defective activity of acid β-glucosidase (Glucocerebrosidase)Q

Pathogenesis
a. Mucopolysaccharidosis
•• Accumulation of ‘Gaucher cells' in spleen, liver, bone marrow & bone b. Gaucher disease
•• Gaucher cells are Lipid Laden Macrophages with “wrinkled tissue paper” appearance of c. Acute Leukemia
cytoplasm, containing glucocerebroside. d. Thalassemia
114 Section 1: General Pediatrics

Clinical Manifestations
Review of Pediatrics and Neonatology

•• Disease severity can range from lethal to asymptomatic; can present at any age;
•• Usual clinical features are:
–– Massive SplenomegalyQ ± hypersplenism; hepatomegaly
–– Bone involvement → Pathological fractures
–– Bone marrow involvement → Pancytopenia → easy fatiguability & bleeding
–– Neurological involvement (seen in types 2 and 3).

Diagnosis
•• Deficient glucocerebrosidase enzyme activity in leukocytes/ fibroblasts
•• Gaucher cells in bone marrow/liver biopsy showing wrinkled paper appearance of
cytoplasm (light microscopy) & lipid bilayers in lysosomes (Electron microscopy)
•• X-ray long bones: ‘Erlenmeyer flask deformity’.

Treatment
•• Enzyme Replacement therapy / Stem Cell transplantation.

High Yield Points   M


•• Gaucher disease is the most common lysosomal storage disorderQ.
•• In type 1 Gaucher disese, neuro­logical involvement is not seen.
•• Most common clinical manifestation of Gaucher disease is splenomegalyQ
•• Substance accumulated in cells in Gaucher disease is glucocerebrosideQ
Question 13 M •• Most common type of Gaucher disease is type 1.
In which of the following metabolic
disorders, this skin finding is char­ B. FABRY DISEASE
acteristically seen? •• An X-linked disorder due to deficiency of α-galactosidase A
•• Affected males have angiokeratomas, hypohidrosis, corneal opacities & acropares­
thesias
•• Angiokeratomas do not blanch with pressure & are most dense between the umbilicus and
knees, in the “bathing trunk area”
•• Pain is the most debilitating symptom in childhood and adolescence.
•• Vascular diseases of kidney, brain & heart develop

C. MUCOPOLYSACCHARIDOSES

a. Fabry disease b. Farber disease Type Name Enzyme deficient Clinical features
c. Galactosemia d. Albinism I Hurler/ α-L-iduronidaseQ Coarse face + Corneal Clouding + Dysostosis multiplex
Scheie + Intellectual disability + Hepatosplenomegaly
II Hunter Iduronate sulfate Same as above but NO corneal clouding
sulfataseQ
Question 14
III San Fillipo Heparan-S- Only Mental Retardation present
What is the disease this child, who sulfamidase
also has hepatosplenomegaly and
IV Morquio N-ac-galactosamine- Bony abnormalities severe and corneal clouding may be
intellectual disability is suffe­ring
6-sulfate sulfatase present
from?
VI Maroteaux- Arylsulfatase B Same as Morquio + Coarse facies + Visceromegaly
Lamy
VII Sly β-glucuronidase HSM + Bony abnormalities

Hurler Disease
Caused by: deficiency of α-L-iduronidase M

Age of Presentation
•• Appears normal at birth, but inguinal hernias are often present
•• Diagnosis is usually made between 6–24 months of age.

a. Gaucher disease Clinical Features M


b. Phenylketonuria •• A broad clinical spectrum seen from more severe Hurler disease to mild Scheie disease
c. Mucopolysaccharidosis
•• Coarse facies, corneal clouding, large tongue, copious nasal dischargeQ
d. Krabbe’s disease
•• Hepatosplenomegaly, Joint stiffness, short stature and skeletal dysplasia.
Chapter 6: Inborn Errors of Metabolism 115

Complications

Section 1: General Pediatrics


Question 15
•• Most patients have recurrent upper respiratory & ear infections, noisy breathing In which of the following diseases,
•• Valvular heart disease with incompetence esp of mitral and aortic valves. these X-ray findings are seen?
•• Obstructive airway disease, notably during sleep, may necessitate tracheotstomy
•• Obstructive airway ds, respiratory infection & cardiac complications are the common causes
of death.

Prognosis
It is a severe, progressive disorder with multi-organ involvement & death, usually by 10 yr of age.

D. NIEMANN PICK DISEASE (NPD)

Basic defect NPD types A and B result from deficient activity of acid sphingomyeli­nase,
a lysosomal enzyme encoded by a gene on chromosome 11
Types Type A: Rapidly progressive neurodegenerative disorder → death by 2-3 a. Gaucher disease
b. Fabry disease
yr of age.
c. Morquio disease
Type B: disease is a nonneuronopathic form observed in children and adults. d. Niemann Pick disease
Type C: disease is a neuronopathic form that results from defective
cholesterol transport
Pathophysiology Deficiency of acid sphingomyelinase

Pathologic accumulation of sphingomyelin,
in monocyte–macrophage system & CNS

Progressive lung disease in type B & neurodegenerative course seen in type A
Genetic basis All subtypes are inherited as autosomal recessive traits
Clinical features Type A NPD: Normal at birth. Hepatosplenomegaly, lymphadenopathy
and psychomotor retardation evident by 6 months of age, followed by
regression & death by 3 years.
Type B NPD: May present as asymptomatic hepatosplenomegaly.
IQ is usually normal; some patients have cherry red maculae
Pulmonary involvement → diffuse reticular or finely nodular infiltration
on CXR seen
In severely affected patients, cirrhosis, portal hypertension & ascites may
be seen.
Type C NPD: Often present with prolonged neonatal jaundice & neuro­
degene­rative course.
Diagnosis Presence of NPD cells in bone marrow aspirates supports diagnosis of type
B & C NPD.
Demonstration of deficient acid sphingomyelinase activity level in
peripheral leukocytes/cultured fibroblasts.
Prenatal diagnosis by measurement of acid sphingomyelinase activity in
cultured amniocytes or chorionic villi; or molecular analysis.
Diagnosis of type C can be supported by the demonstration of filipin stain
positivity in cultured fibroblasts.

 LEUKODYSTROPHY
Leukodystrophy refers to progressive degeneration of white matter of brain due to imperfect
growth or development of the myelin sheath.

Important Types of Leukodystrophy:


High Yield Points
•• Metachromatic leukodystrophy •• Krabbe’s disease
•• All Mucopolysaccharidoses are
•• Adrenoleukodystrophy •• Pelizaeus-Merzbacher disease autosomal recessive
•• Except Hunter’s disease which is
•• Canavan’s disease •• Alexander disease inherited as X-linked recessive.
116 Section 1: General Pediatrics

A. ALEXANDER DISEASE
Review of Pediatrics and Neonatology

High Yield Points


•• Caused by mutations in glial fibrillary acidic protein (GFAP) gene, on chromosome 17q21
•• Most specific & important labora­ •• Deposition of eosinophilic hyaline bodies called Rosenthal fibers in astrocytes
tory finding in Adrenoleukodys- •• In the classic infantile form, degeneration of white matter is most prominent frontally.
trophy (ALD) is the demonstration
•• Affected children develop progressive loss of intellect, spasticity, and unresponsive seizures
of abnormally high levels of VLCFA
causing death by 5 years of age.
(very long chain fatty acids) in plasma
•• 85% of patients with the childhood
form of ALD have elevated levels of
B. CANAVAN’S DISEASE
ACTH •• Autosomal recessive disorder caused due to deficiency of the enzyme N-Aspertoacylase
•• This leads to accumulation of N-Acetyl aspartic acid in brain and urine
•• It is characterized by the clinical triad of hypotonia, head lag, macrocephaly.Q

C. ADRENOLEUKODYSTROPHY
•• X-Iinked recessive disorder caused due to deficiency of Acyl-CoA synthetase
•• Ataxia, spasticity, cortical blindness starting at 5–10 yr age
•• Marocephaly is not a key feature.

D. METACHROMATIC LEUKODYSTROPHY
What is it? A disorder of myelin metabolism due to deficient arylsulfatase A
Genetic basis Autosomal recessive inheritance; ARSA gene (chr 22q)
Called so because Cresyl violet applied to tissue specimens produces metachro­matic
staining of sulfatide granules.
Pathophysiology Accumulation of cerebroside sulfate within myelin sheath of nervous
system cause myelin breakdown & destruction of oligodendroglia
Clinical features •• Insidious onset of gait disturbances (1–2 yr age), hypotonia, absent
reflexes, dysarthric speech & intellectual disability
Question 16 •• Visual fixation is diminished, nystagmus & optic atrophy present
A 4-month-old child presents with •• Decorticate postures & impaired swallowing by 1 yr of onset
excessive irritability & crying, un­ Investigations Slowing of peripheral nerve conduction velocities & changes in
explained hyperpyrexia, vomi­ting, VEPs & auditory brainstem responses CT and MRI brain indicate diffuse
difficulty feeding for last 15 days; symmetric attenuation of cerebellar & cerebral white matter.
On admission he has rigidity & vis­
ual inattentiveness; CT scan brain E. KRABBE DISEASE (GLOBOID CELL LEUKODYSTROPHY)
shows the following finding. What
is the probable diagnosis? What is it? •• It is a neurodegenerative disorder with severe myelin loss & presence
of globoid bodies in white matter
Basic defect •• Deficiency of lysosomal enzyme galactocerebroside β-galactosidase
•• It is a disorder of myelin destruction & not abnormal formation
Genetic basis •• Autosomal recessive; gene for KD (GALC) is located on chr 14q24
Clinical features •• Symptoms appear in 1st few months of life & include excessive
irritability & crying, unexplained episodes of hyperpyrexia,
vomiting, difficulty feeding
•• Hypertonia with rigidity, opisthotonos & visual inattentiveness (due to
optic atrophy) seen
•• In later stages, blindness, deafness, absent deep-tendon reflexes &
a. Alexander disease decerebrate rigidity. Most patients die by 2 yr of age
b. Krabbe disease
CT scan •• Bilateral symmetrical basal ganglia hyperdensities
c. Metachromatic Leukodystrophy
d. Adrenoleukodystrophy
 WILSON DISEASE M

High Yield Points Also called Hepatolenticular degeneration

•• Kayser-Fleischer rings is due to Basic defect It is an autosomal recessive disorder caused by a mutation of ATP7B gene on
deposit of copper in Descemet chromosome 13q14
membrane layer of cornea Pathogenesis Defective ATP7B results in decreased biliary copper excretion & diffuse accumulation
•• Family members of patients with of copper in the cytosol of hepatocytes.
proven cases requires screening for
presymptomatic Wilson disease. Contd...
Chapter 6: Inborn Errors of Metabolism 117

Contd...

Section 1: General Pediatrics


High Yield Points
Also called Hepatolenticular degeneration
•• An important screening test for
Basic defect It is an autosomal recessive disorder caused by a mutation of ATP7B gene on Wilson disease is decreased serum
chromosome 13q14 ceruloplasminQ level.
•• Ceruloplasmin may also be low
Pathogenesis Defective ATP7B results in decreased biliary copper excretion & diffuse accumulation in other hypoproteinemic states;
of copper in the cytosol of hepatocytes. elevated in acute inflammation
•• Most specific screening test for
Clinical features Hepatic involvement: Asymptomatic hepatomegaly (± splenomegaly), subacute or Wilson disease is UrineQ 24 hour
chronic hepatitis, acute hepatic failure, cirrhosis, portal hypertension, ascites, edema, copper
variceal bleeding •• Gold standard test for Wilson dis­
Hematological: Coombs-negative Hemolytic anemia, coagulation defects ease is Liver BiopsyQ & estimation of
hepatic copper content.
Neurological: Intention tremor, dysarthria, rigid dystonia, choreiform movements, lack
of motor coordination, deterioration in school performance, behavioral changes.
Eye: Kayser-Fleischer ring & Sunflower cataract
Question 17 M

Endocrine: Delayed puberty, amenorrhea A 10-year-child presents with


chronic liver disease, dystonia,
Psychiatric: Depression, personality changes, anxiety, psychosis.
frequent falls and pigmentation
Renal: Fanconi syndrome, progressive renal failure in eye as shown below. His serum
Unusual manifestations: Arthritis, pancreatitis, nephrolithiasis, cardiomyopathy, ceruloplasmin is low. What is the
and endocrinopathies (hypoparathyroidism). most probable diagnosis?

Diagnosis •• Most patients with Wilson disease have decreased ceruloplasmin levels
(<20 mg/dL)
•• Serum “free” copper level may be elevated in early Wilson disease (>1.6 µmol/L)
•• Urinary copper excretion is increased to >100 µg/day and often up to 1,000 µg/day
•• Hepatic copper content usually exceeds 250 µg/g dry weight
•• Kayser-Fleischer rings present on slit lamp examination of eye.

Treatment Restrict dietary copper intake to<1 mg/day.


a. Krabbe disease
Foods such as liver, shellfish, nuts, and chocolate should be avoided. b. Gaucher disease
Copper-chelating agents like d-penicillamine, Zinc, Trientine (triethylene tetramine c. Wilson disease
dihydrochloride), Ammonium tetrathiomolybdate d. Alexander disease

 MENKES DISEASE
•• It is caused by mutations in the gene encoding Cu2+ transporting ATPase (ATP7A)
•• Copper level in liver and brain are low in contrast to an increase in enterocytes & fibroblasts
•• Progressive cerebral degeneration (seizures), feeding difficulties, failure to thrive, hypothermia,
apnea, hair abnormalities (kinky hair), hypopigmentation, bone changes and cutis laxa seen
•• Patients with the classic form of Menkes disease usually die before 3 years of age.

Question 18 Question 19
A child presented with anemia, kinky hair, intellectual disability and seizures. The pictures of An infant presents with failure
his hairs are shown below. Which of the following is the most likely diagnosis? to thrive and abdominal disten­
sion. His X-ray chest & abdomen
showed the following. Diagnosis?

a. Menkes disease b. Down syndrome


c. Iron deficiency anemia d. Lesch-Nyhan syndrome

 WOLMAN’S DISEASE
•• It is a rare autosomal recessive disease caused by mutations in the LIPA gene, which
encodes for an enzyme called lysosomal acid lipase, found in the lysosomes a. Wolman disease
•• Accumulation of triglycerides & cholesteryl esters within cells of affected individuals b. Gaucher disease
•• Hepatosplenomegaly, Jaundice, Vomiting, Diarrhea in infancy c. Organic acidemia
•• Characteristic adrenal gland calcificationQ seen on X-ray. d. Tyrosinemia
118 Section 1: General Pediatrics

 DISORDERS OF PURINE & PYRIMIDINE METABOLISM


Review of Pediatrics and Neonatology

Important disorders in this category are:


•• Abnormal purine catabolism: AMP deaminase deficiency, adenosine deaminase (ADA)
deficiency, Hereditary xanthinuria
•• Abnormal purine salvage: HGPRT deficiency, adenine phosphoribosyltransferase (APRT)
deficiency.
•• Abnormal pyrimidine synthesis: Hereditary orotic aciduria
•• Disorders of pyrimidine catabolism: Pyrimidine 5'-nucleotidase deficiency,
•• Abnormal pyrimidine salvage: Thymidine kinase 2 deficiency.

LESCH NYHAN DISEASE M

Basic defect Disorder of purine metabolism that results from hypoxanthine-guanine phospho­
ribosyl transferase (HGPRT) deficiency

Genetics X-linked inheritance

Clinical features Asymptomatic at birth; Developmental delay, intellectual disability & neurologic
signs including dystonia, spasticity & dysarthria; Self-injury is an important feature;

Diagnosis Serum levels of uric acid exceed 4–5 mg & urine uric acid : creatinine ratio is ≥3–4 : 1;
Definitive diagnosis requires an analysis of the HPRT enzyme.

Treatment For hyperuricemia: For high fluid intake, alkalinization & allopurinol.
A low purine diet and reduced fructose intake are desirable;

Prognosis Rarely survive 3rd decade because of renal or respiratory compromise

HEREDITARY OROTIC ACIDURIA

Due to uridine monophosphate synthase deficiency. Presenting features:


•• Developmental delay, failure to thrive, cardiac disease, strabismus
•• Crystalluria →±ureteric obstruction; Renal function usually normal.
•• Macrocytic hypochromic megaloblastic anemia, unresponsive to iron, folic acid & vitamin
B12 & may develop leukopenia.
Chapter 6: Inborn Errors of Metabolism 119

Section 1: General Pediatrics


Answer Keys for Image-Based Questions

Answers Explanations / Identifying features


1. Ans. d. GM1 gangliosidosis This is a ‘cherry red spot’, a dark red colored spot in the light background of surrounding retina
2. Ans. b. Mitochondrial disorders Bone spicule-shaped pigment deposits are present in the periphery along with retinal atrophy, while
the macula is preserved
3. Ans. b. Von Gierke disease Doll like facies with hepatomegaly & recurrent hypoglycemia suggest a diagnosis of Von Gierke
disease
4. Ans. c. Acid maltase A. shows a floppy child with frog leg posture
B. shows head lag (hypotonia)
C. CXR with Cardiomegaly
D. ECG showing tall QRS with short PR interval
5. Ans. c. Phenylketonuria Bilateral cataract is present in this child
6. Ans c. Congenital disorder of The given picture shows inverted nipples, that can be seen in Congenital disorder of Glycosylation
Glycosylation
7. Ans. c. Multiple carboxylase Alopecia & skin rashes with CNS manifestations. Multiple carboxylase or Biotinidase deficiency
8. Ans c. Albinism This child with fair skin, hypopigmented hairs & iris is probably suffering from albinism
9. Ans. b. Phenylketonuria Hypopigmentation along with low tyrosine levels in blood suggest a diagnosis of Phenylketonuria
(PKU)
10. Ans. d. Homogentisic acid Ochronosis & darkening of urine on standing, suggest a diagnosis of Alkaptonuria
oxidase
11. Ans. d. Hartnup disorder Characteristic skin lesions on the neck, known as ‘Casal necklace’, seen in Pellagra (Niacin deficiency)
& Hartnup disorder
12. Ans. b. Gaucher disease Pancytopenia with massive splenomegaly in a child with bone marrow showing ‘Gaucher cells’
with ‘crumpled tissue paper’ appearance of cytoplasm are seen in Gaucher disease
13. Ans. a. Fabry disease The skin lesions shown in the given picture are angiokeratomas, that are characteristically seen in
Fabry disease, but can also be seen in Fucosidosis & β mannosidosis.
14. Ans. c. Mucopolysaccharidosis Hepatosplenomegaly with ID, coarse facies & corneal opacity are suggestive of MPS type I or
Hurler disease
15. Ans. c. Morquio disease Peg-shaped metacarpals & rounded vertebral bodies with “anterior beaking” is seen in Morquio
disease
16. Ans. b. Krabbe disease Bilateral basal ganglia calcification is seen on CT brain in Krabbe disease
17. Ans. c. Wilson disease The picture shows a pigmented ring on the rim of cornea, known as Kayser Fleischer (KF) ring
18. Ans. a. Menkes disease Hair changes including trichorrhexis nodosa & pili torti are seen in Menke disease
19. Ans a. Wolman disease This X-ray shows bilateral adrenal calcification that suggests a diagnosis of Wolman disease
120 Section 1: General Pediatrics
Review of Pediatrics and Neonatology

Questions
 GLYCOGEN STORAGE DISORDERS 10. A child presents with hepatomegaly and hypoglycemia.
There is no improvement in blood sugar even after
1. Metabolic abnormalities in which jaundice is seen are administration of epinephrine. What is the likely
all of the following except? (NEET PG Jan 2019) diagnosis? (AIPGMEE 10)
a. Von Gierke disease b. Galactosemia a. Von Gierke's disease b. Anderson's disease
c. Tyrosinemia c. Pompe's disease d. McArdle's disease
d. Hereditary fructose intolerance 11. Which glycogen storage disease does not affect muscles?
2. Hyperphenylalaninemia is due to defect in the enzyme: a. Type 1 b. Type 2 (APPG 08)
a. Phenylalanine hydroxylase: (PGI Jan 2017) c. Type 3 d. Type 4
b. Tyrosinase 12. A child presents with massive hepatomegaly and
c. Homogentisic acid oxidase hypoglycemia. There is no improvement in blood
d. Ornithine transcarbamylase glucose on administration of glucagon. The probable
e. Phenylalanine oxidase diagnosis is: (AIPGMEE 2008)
3. A child who was normal at birth, develops chronic
a. Von Gierke disease b. McArdle disease
liver failure and muscle weakness at 3 months of age.
c. Coris disease d. Forbe's disease
On investigation, Serum Glucose is low, along with
ketoacidosis and decreased pH. ALT and AST are 13. All are liver glycogenosis except: M  (AIPGMEE 2007)
raised. Blood lactate and uric acid levels are normal. a. Von Gierke disease b. Her's disease
Intravenous glucagon given after meals raises the blood c. Type III glycogenosis d. Pompe's disease
glucose levels but does not raise glucose when given 14. An infant has hepatosplenomegaly, hypoglycemia, hyper­
after an overnight fast. Liver biopsy shows increased lipidemia, acidosis and normal structured glycogen depo­
glycogen in liver. Which is the enzyme likely to be sition in liver. What is the diagnosis? (PGI June 2001)
defective in this child? (AIIMS May 2016) a. Her's disease b. Von Gierke's disease
a. Glucose-6-phosphatase b. Muscle Phosphorylase c. Cori's disease d. Anderson's disease
c. Branching enzyme d. Debranching enzyme e. Pompe's disease
4. Which disorder of carbohydrate metabolism typically 15. Glycogen storage diseases includes: (PGI Dec 2001)
has cardiac involvement? M  (APPG 2016) a. Von Gierke's disease b. Fabry's disease
a. Glycogen Storage Disease Type I (Von Gierke disease) c. McArdle's disease d. Fragile X syndrome
b. Glycogen Storage Disease Type II (Pompe disease) e. Krabbe disease
c. Hereditary fructose intolerance
d. Galactosemia  OTHER DISORDERS OF CARBOHYDRATE
5. All of the following diseases/syndromes are Glycogen METABOLISM
storage disorders except? (COMEDK 2016)
a. Andersen's disease b. Her's disease
16. An 8 days old neonate presents with hypoglycemia
c. Scheie's disease d. Tarui's disease
(Glucose–17 mg%) and jaundice is diagnosed to have
6. Hypoglycemia, hepatomegaly, growth retardation, metabolic disorder. Most likely diagnosis: (JIPMER 2017)
muscle weakness and accumulation of limit dextrins is
caused by which glycogen storage disease? a. Hypothyroidism b. Galactosemia
 (AIIMS May 2015) c. Glycogen storage disease d. Fatty acid oxidation defect
17. In children with classical galactosemia all are true
a. Cori's disease b. Von Gierke's disease
except: (JIPMER May 2018)
c. Anderson disease d. Pompe's disease
a. E. coli neonatal sepsis is common
7. Coarse facies, hepatosplenomegaly and tall QRS on
b. Elimination of galactose in diet will not reverse cataract
ECG are characteristic feature of:
c. Galactose converts to galactitol which is toxic to brain
 (AIIMS May 2015, AIIMS Nov 2001)
and liver
a.Glycogen storage disease type II d. Duarte variant of galactosemia were asymptomatic
b. Hurler's disease 18. A 5-month old baby presents to emergency with
c. Hunter's disease d. Hemochromatosis history of vomiting, irritability & jaundice. The baby
8. An Infant presents with hepatomegaly, hypoglycemia, was previously healthy. History of introducing fruit
hyperlipidemia and acidosis. Most probable underlying juices into the diet one week back. USG revealed
diagnosis is: M  (AIIMS May 2015, Recent question 2013) hepatomegaly and liver function test were abnormal.
a. Von Gierke's b. Con's disease The infants most likely has deficiency of which of the
c. Pompe's disease d. All of the above following enzymes? (JIPMER May 2016)
9. Hers disease is due to deficiency of: (TN PGMEE 2012) a. Fructokinase b. Aldolase B
a. Liver phosphorylase b. Muscle phosphorylase c. Galactose- 1-phosphate uridyl transferase
d. Alpha glucosidase
c. Branching enzyme d. RBC phosphofructokinase
Chapter 6: Inborn Errors of Metabolism 121

 DISORDERS OF AMINO ACID METABOLISM

Questions
19. This young patient came with wing beating tremor and
liver cell failure. His eye shows the finding shown in the
given picture. What is the diagnosis? M  (APPG 2015) 28. Mental retardation is seen in: (FMGE Dec 2018)
a. Phenylketonuria b. Alkaptonuria
c. Gaucher disease d. Von Gierke disease
29. Ectopia lentis in a child is seen in which of the following
diseases? (FMGE Dec 2018)
a. Sarcoidosis b. Homocystinuria
c. Alkaptonuria d. Urea cycle defects
30. Tom cat urine odor is seen in? M  (JIPMER Nov 2017)
a. Multiple carboxylase deficiency
b. Hawkinsinuria 
a. Primary biliary cirrhosis b. Galactosemia c. Tyrosinemia d. Phenylketonuria
c. Paralysis agitans 31. Dried blood spot test in neonates is used to check for:
d. Hepatolenticular degeneration a. Inborn error of metabolism (AIIMS Nov 2017)
20. Lactose intolerance in a suspected patient is diagnosed b. Blood group
by giving an oral load of lactose and measuring c. Total cell count d. Creatinine and bilirubin
concentration of: (Recent question 2015) 32. Mutation seen in Hartnup’s disease: (JIPMER Nov 2016)
a. CO2 b. H2 a. SLC6A 18 b. SLC6A 19
c. One/two carbon compounds c. SLC6A 17 d. SLC6A 16
d. N2 33. A 5-year-old male child with developmental delay
21. A 6-month-child presents with episodes of vomiting was brought with squint & bilateral lens subluxation.
after ingesting fruit juice. Which of following enzyme After 4 years, he dies of massive stroke. Autopsy
deficiency is likely? M  (AIIMS Nov 2014) reveals middle cerebral artery thrombosis and old renal
infracts. Disease course could have been modified by
a. Aldolase B b. Fructokinase supplement of: (JIPMER May 2016)
c. Glucose 6-phosphatase d. Hexokinase
a. Thiamine b. Pyridoxine
22. All are seen in lactose intolerance except: c. Tyrosine d. Methionine
a.Benedict test positive in urine (Recent question 2014) 34. A 7-day-old infant born with insignificant antenatal and
b. Alkaline urine neonatal history presents with poor feeding, vomiting,
c. Acidic stool d. Lactase enzyme deficiency lethargy, rapidly progressive coma, metabolic acidosis,
23. Second degree consanguineous marriage, baby with a peculiar odour of his urine, hypertonicity with severe
diarrhea, perianal diaper area redness suggests ? opisthotonus which alternate with bouts of flaccidity.
 (Recent question 2013) He was initially empirically diagnosed as having CNS
a. Lactose intolerance b. Shigella diarrhea infection and sepsis but was found to have hypoglycemia
c. Salmonella d. Fungal infection and the correction of hypoglycemia did not improve
24. A child with low blood glucose is not able to do glyco­ clinical condition. What is the most likely differential
geno­lysis or gluconeogenesis. Which of the following for this clinical presentation? (Recent Question 2017)
enzyme is missing in the child? (AIIMS Nov 2012) a. Phenylketonuria b. Tyrosinemia
a. Fructokinase b. Glucokinase c. Maple syrup urine disease
c. Glucose 6-phosphatase d. Transketolase d. Isovaleric acidemia
25. A 3-day-child vomits everything he feeds, has a 35. Nitisinone is used in the treatment of: (JIPMER Nov 2016)
distended abdomen and diarrhea. The urine is positive a. Phenylketonuria b. Alkaptonuria
for Benedict's test for reducing substance. The substance c. Homocystinuria d. Tyrosinemia
in urine is: (AIIMS Nov 2010) 36. In Hartnup disorder, metabolism of which of the follo­
a. Sucrose b. Glucose wing amino acids is affected? M  (JIPMER May 2016)
c. Galactose d. Fructose a. Tyrosine b. Tryptophan
26. True regarding galactosemia: (PGI Dec 2001) c. Phenylalanine d. Homocysteine
37. A child with subluxation of lens and mental retardation
a. Mental retardation occurs
died. Which of the following would have helped in his
b. Absent disaccharidase in intestine
treatment? (JIPMER May 2016)
c. Defect in epimerase
d. Defect in galactose 1 PO4 uridyl transferase a. Biotin b. Pyridoxine
e. Reducing substance positive in urine c. Zinc d. Riboflavin
27. Child of Vasanthi was weaned from breast milk on the 5th 38. Boiled cabbage urine odor is seen in:
day and was given sugarcane juice. The child developed  (Recent question 2016)
hypoglycemia and hepatomegaly. Biochemical a. Tyrosinemia b. Isovaleric aciduria
examination showed hypophosphatemia and reducing c. Maple syrup urine disease
substances in urine. The child is probably suffering from d. Phenylketonuria
which of the following enzyme deficiencies? 39. Branched chain keto acids are excreted in urine in:
 (AIIMS Nov 2000) a. Maple syrup urine disease (MAHA PGM CET 2016)
a. Fructokinase b. Aldolase B b. Hartnup disease
c. Glucose 6-phosphatase d. Beta-galactosidase c. Albinism d. Alkaptonuria
122 Section 1: General Pediatrics

53. A normal born child presents with mental retardation,


Review of Pediatrics and Neonatology

40. Most sensitive test to identify metabolic disease in a


newborn: (Recent question 2016) blonde hair and convulsions at 1 year of age. Most
a. Tandem mass spectrometry probable diagnosis is: (Recent question 2014)
b. Arterial blood gas a. Albuminuria b. Phenylketonuria
c. Complete hemogram d. Liver function test c. Gaucher's disease d. Tyrosinemia
41. Which of the following is a branched chain keto acid 54. A child presents with pellagra like dermatitis and
disorder? (Recent question 2016) aminoaciduria. Two siblings have the condition and it is
a. Propionic acidemia b. Methylmalonic acidemia absent in two siblings while it is absent in both parents.
c. Maple syrup urine disease Which of the following condition is most likely?
d. Homocystinuria  (AIIMS Nov 2014)
42. In Hartnup disorder what is seen? a. Hartnup's disease b. Alkaptonuria
 (Recent question 2016) c. Phenylketonuria d. Von Gierke's disease
a. Pellagra like rash b. Burn feet rash 55. “Sweaty feet” odor to urine occurs in:
c. Alopecia d. Vitiligo  (MAHA PGM CET 2014)
43. A child has microcephaly, blue eyes, fair skin, and a. Homocystinuria b. Isovaleric acidemia
mental retardation; ferric chloride test is positive. What c. Phenylketonuria d. Alkaptonuria
is the likely diagnosis?
56. Phenylketonuria–true statement: (JIPMER 2014)
 (Recent question 2016, AIPGMEE 07)
a. Phenylketonuria (PKU) b. Homocystinuria a. Urine phenylalanine metabolites are increased
c. Tyrosinosis d. Alkaptonuria b. Deficiency of tyrosine hydroxylase
44. Enzyme deficient in phenylketonuria: M  c. Serum tyrosine levels are increased
 (Recent question 2016; AIPGMEE 99) d. Mental retardation does not occur if patient is advised to
a. Tyrosinase b. Phenylalanine hydroxylase have phenyl alanine free diet
c. Tyrosine transaminase d. Homogentisic oxidase 57. The enzyme deficient in phenylketonuria:
45. Oochronosis is found in: M  (Recent question 2016) a. Phenylalanine hydroxylase (Recent question 2014)
a. Alkaptonuria b. Tyrosinemia b. Tyrosinase
c. Phenylketonuria d. Homocystinuria c. Aminotransferase d. Homogentisic acid oxidase
46. Mousy odor of urine is seen in: (Recent question 2015) 58. Maple syrup urine disease is due to defective
a. Isovaleric acidemia b. Phenylketonuria decarboxylation of: (TN PGMEE 2013)
c. MSUD d. Tyrosinemia a. Branched chain amino acids
47. A 7-month-old infant presents with history of vomiting b. Unbranched chain amino acids
& failure to thrive. Patient improved with IV glucose. c. Sulfur containing amino acids
After one month, returns with same complaints. On d. Hydroxyl amino acids
evaluation, found to have high glutamine and uracil. 59. If the tyrosinase gene is defective, it results in:
Which is the likely enzyme defect? (AIIMS May 2015)   (TN PGMEE 2013)
a. CPS1
a. Lack of pigmentation b. Maple syrup urine disease
b. Ornithine transcarbamylase 
c. Phenylketonuria d. Kidney stones
c. Arginase d. Arginosuccinase lyase
60. Which of the following is false about alkaptonuria?
48. An infant presents with history of seizures and skin
rashes. Investigations show metabolic acidosis increased a. Genitourinary system not involved (Jharkhand PG 2013)
blood ketone levels and normal NH3. This child is likely b. Homogentisic acid oxidase deficiency
to be suffering from: c. Black urine
 (Recent question 2015, AIPGMEE 2002) d. Calcification in vertebral bodies
a. Propionic aciduria b. Urea cyclic disorder 61. Not a feature of Hartnup's disease: (Recent question 2013)
c. Phenylketonuria a. Pellagroid skin lesion b. Cerebellar ataxia
d. Multiple carboxylase deficiency c. Mental retardation d. Psychological disturbances
49. HHH syndrome is caused by defect in: 62. Guthrie test can be used for the diagnosis of:
a. Ornithine permease (MAHA PGM CET 2015)  (DNB June 2012)
b. Ornithine transcarbamyolase a. Tyrosinemia b. Galactosemia
c. Arginase d. Argininosuccinase lyase c. Alkaptonuria d. Phenylketonuria
50. Treatment of Alkaptonuria: M  (Recent question 2015)
63. Mental retardation is not a feature in this aminoaciduria:
a. Vitamin A b. Vitamin C  (TN PGMEE 2012)
c. Vitamin D d. Vitamin K
a. Phenylketonuria b. Homocystinuria
51. Intervertebral disc calcification is seen in which of the
c. Albinism d. Maple syrup urine disease
following? (Recent question 2015)
a. Rickets b. Alkaptonuria 64. Ferric chloride test is used to diagnose: (TN PGMEE 2012)
c. Scurvy d. Gaucher disease a. Phenylketonuria b. Alkaptonuria
52. Maple syrup urine disease is due to defect in the c. Hartnup disease d. Cystinuria
metabolism of: (Recent question 2015; AIPGMEE 2004) 65. Massive aminoaciduria without a corresponding
a. Branched chain amino acids increase in plasma amino acid level is characteristic of
b. Essential fatty acids which one of the following diseases?
c. Essential amino acids d. Glucogenic amino acids  (DNB Dec 2011; UPSC 2008)
Chapter 6: Inborn Errors of Metabolism 123

a. Homocystinuria b. Hartnup disease 77. A 5-day-old child presents with intractable seizures. He

Questions
c. Tyrosinemia d. Maple syrup urine disease had rashes all over the body. Blood examination showed
66. The enzyme that is deficient in homocysteinuria: hyperammonemia and lactic acidosis. The probable
 (TN PGMEE 2011) diagnosis is: (AIIMS May 2000)
a. Tyrosinase b. Sphingomyelinase a. Organic acidemia
c. Cystathionine synthase d. Acid lipase b. Mitochondrial encephalopathy with lactic aciduria
67. Which one of the following is not a feature of c. Phenylketonuria
Phenylketonuria? (DNB Dec 2011) d. Urea cycle enzyme deficiency
a. Severe mental retardation 78. Mousy odor urine is seen in: (PGI Dec 97)
b. Reduced tendon reflexes
a. Maple syrup urine ds b. Phenylketonuria
c. Enamel hypoplasia
c. Isovaleric acidemia d. Cystinuria
d. Vomiting in early morning
e. Homocystinuria
68. Not a correct match: (TN PGMEE 2011)
a. Alkaptonuria-homogentisate oxidase
b. Albinism-tyrosinase
 LYSOSOMAL STORAGE DISEASES
c. Tyrosinemia-fumaryl acetoacetate hydrolase
d. Phenylketonuria-aryl sulphatase A 79. A 5-year-old child, with developmental delay, pre­
69. False about hartnup's disease: (JIPMER 2010, 2008)
sented with nasal obstruction and a large head with
hearing defect. There was also an associated history of
a. Defect in neutral amino acid transport
sleep apnea. On examination hepatosplenomegaly was
b. Mental retardation is the common presentation
present. ECHO was suggestive of cardiac valve fibrosis.
c. Most children are asymptomatic
What is the most likely diagnosis? (AIIMS Nov 2018)
d. Photosensitivity
70. In phenylketonuria, the treatment of choice is: a. Hunter syndrome b. Hurler syndrome
c. Tay-Sachs disease d. Fragile X syndrome
a. Limit intake of substrate for the enzyme
b. Provide the deficient amino acid 80. A 4-month-boy presents with failure to thrive. He had
c. Correct the enzyme defect several upper respiratory tract infections during the
d. Symptomatic management (AIIMS Nov 10) last two months. On examination, hepatosplenomegaly
71. PKU is a congenital amino acid metabolic disorder. In one and mild hypotonia are noted. Blood tests reveal
of the following rare variants of PKU, dihydrobiopterin pancytopenia & transaminitis. Chest X-ray shows
synthesis is affected. The enzyme deficient is: reticulonodular pattern and calcified nodules. Biopsy
a. Histidine decarboxylase (PGI June 08) of liver shows foamy histiocytes. What is the expected
b. Phenylalanine hydroxylase ophthalmological finding in this patient?
c. Dihydropteridine reductase a. Cherry red macula (JIPMER May 2016)
d. Tyrosine deficiency b. Cataracts
72. Treatment of multiple carboxylase deficiency: M  c. Kayser-Fleisher ring d. Foveal hypoplasia
a. Biotin b. Pyridoxine 81. Niemann-Pick disease is due to deficiency of one of the
c. Thiamine d. Folic acid (AIPGMEE 07) following enzymes–which one? (APPG 2016)
73. Darkening of urine on standing is associated with: a. Arylsulfatase b. Acid Sphingomyelinase
 (AIPGMEE 2007) c. Hexosaminidase A d. Ceramidase
a. Alkaptonuria b. Cystinuria 82. Hunter's syndrome is due to deficiency of:
c. Fabry's disease d. Tyrosinemia  (Recent question 2016)
74. Deficiency of homogentisate oxidase leads to: a. Iduronate sulfatase b. Hexosaminidase
 (TN PGMEE 2006) c. Glucocerebrosidase d. L-Iduronidase
a. Homocystinuria b. Alkaptonuria 83. Gaucher disease is produced due to defect in the
c. Maple syrup urine disease enzyme: M  (MAHA PGM CET 2016)
d. Phenylketonuria a. β-glucosidase b. Glucokinase
75. Deficiency of enzyme alpha-keto acid decarboxylase c. Sphingomyelinase
leading to a block in the metabolism of branch chain d. Glucose-6-phosphate dehydrogenase
amino acids is observed in: (AIIMS May 05)
84. Wrinkled paper appearance of macrophage in bone
a. Maple syrup urine disease marrow aspirate is seen in: (Recent question 2016)
b. Hartnup's disease
c. Alkaptonuria d. Phenylketonuria a. Niemann Pick disease b. Gaucher disease
c. Hemophagocytic lymphohistiocytosis
76. True statement regarding a 3-week-old child with
d. Langerhan cell histiocytosis
phenylketonuria are all, except: (AIPGMEE 2000)
85. Deposition of cells with crumpled tissue paper
a. Provocative protein meal tests help in the diagnosis
appearance of cytoplasm is seen in: M 
b. Tyrosine becomes an essential amino acid in diet
c. Serum phenylalanine is increased and urinary phenyl  (Recent question 2016)
pyruvate level is elevated a. Gaucher disease b. Niemann Pick disease
d. Phenylalanine should be completely stopped in diet c. Tay Sach disease d. Mucopolysaccharidosis
124 Section 1: General Pediatrics
Review of Pediatrics and Neonatology

86. A male child with coarse facies, macroglossia, thick lips 97. Globoid cells is a diagnostic feature of:
with hepatosplenomegaly presents with copious mucus a. Krabbe's disease (Recent Question 2016)
discharge from nose. Probable underlying diagnosis is: b. Progressive multifocal leukoencephalopathy
a. Hurler disease (AIIMS May 2015) c. Tay Sach's disease
b. Beckwith-Wiedemann syndrome d. Metachromatic leukodystrophy
c. Proteus syndrome d. Hypothyroidism
98. Male child with hyper intensities on T2 weighted MRI in
87. Crumpled tissue paper appearance of cytoplasm in bilateral frontal lobe. Most probable diagnosis is:
bone marrow examination is seen due to accumulation
of which of the following inside the cell? a. Canavan disease (WB PGMEE 2016)
b. Krabbe disease
a. Cerebrosides (AIIMS May 2015)
b. Ganglioside accumulation c. X-linked adrenoleukodystrophy 
c. Sphingomyelin d. Alexander disease
d. Glucocerebroside accumulation 99. Metachromatic leukodystrophy is due to deficiency of:
a. β galactosidase (Recent question 2016)
88. A one-year-old boy presented with hepatosplenomegaly
and delayed milestones. The liver biopsy and bone mar­ b. Arylsulfatase A
row biopsy revealed presence of histiocytes with PAS- c. Glycogen phosphorylase
positive Diastase resistant material in the cytoplasm. d. Medium chain acyl-coA synthetase
Electron microscopic examination of these histiocytes is 100. White matter is involved in which of the following
most likely to reveal the presence of: diseases? (Recent question 2015)
 (Recent question 2014) a. Canavan disease b. Gaucher disease
a. Birbeck granules in the cytoplasm c. GM1 Gangliosidosis d. Mucopolysaccharidosis
b. Myelin figures in the cytoplasm
c. Parallel rays of tubular structures in lysosomes 101. An infant presents with irritability, increased tone of
d. Electron dense deposit in the mitochondria extremities and recurrent seizures. Tissue examination
89. Enzyme replacement therapy is not available for which reveals globoid cells in parenchyma around blood
of the following? M  (AIIMS May 2014) vessels. What is the most probable diagnosis?
a. Gaucher's disease b. Pompe disease  (JIPMER 2014)
c. Sanfilippo disease d. Fabry's disease a. Tay Sach's disease b. Krabbe disease
90. Enzyme deficiency in Hunter disease is: c. Adrenoleukodystrophy d. Canavan's disease
 (WB PG 2014/ TN PGMEE 2013) 102. Hyperdense basal ganglia and hypodensity of the white
a. L-iduronidase b. Iduronate sulfatase matter on CT scan is diagnostic of:
c. Heparan sulfamidase d. Hyaluronidase  (MAHA PGM CET 2014)
91. Tay-Sachs disease is due to deficiency of: M  a. Alexander's disease b. Krabbe's disease
 (TN PGMEE 2012) c. Canavan's disease
a. Beta-galactosidase b. Hexosaminidase d. Metachromatic leukodystrophy
c. Sphingomyelinase d. Glucocerebrosidase
103. Macrocephaly is seen in which of the following
92. The following is an X-linked mucopolysaccharidoses:
syndromes? (AIIMS May 2008)
 (TN PGMEE 2011)
a. Metachromatic leukodystrophy
a. Hurler's syndrome b. Hunter's syndrome
c. Scheie's syndrome d. Sanfilippo's syndrome b. Adrenoleukodystrophy
93. True about Gaucher's disease: (PGI Dec 2010) c. Canavan's disease d. Krabbe's disease
a. PAS positive cells b. Oil Red O positive cells 104. Deep white matter lesion with bilateral deep bright
c. Deficiency of acid sphingomyelinase thalamic appearance on CT head is suggestive of:
d. Deficiency of glucocerebrosidase  (AIIMS Nov 2007)
e. Gaucher cells are present a. Alexander disease
94. Only males are affected in: (AIIMS May 08) b. Canavan's disease
a. Scheie's syndrome b. Hunter's syndrome c. Krabbe's disease
c. Hurler's syndrome d. Gaucher's disease d. Metachromatic leukodystrophy
95. For which of the following diseases enzyme replacement 105. All are seen in metachromatic leukodystrophy except:
therapy is available? M   (TN PGMEE 2003)
 (AIPGMEE 04, AIIMS May 06, Nov 03)
a. Mental retardation b. Optic atrophy
a. Albinism b. Niemann-Pick disease
c. Decerebrate posture d. Exaggerated tendon reflexes
c. Metachromatic leukodystrophy
d. Gaucher's disease
 WILSON DISEASE
 LEUKODYSTROPHY
106. K-F ring is seen in: M 
96. Lorenzo oil used in treatment of: (JIPMER May 2017)  (Recent question 2016/ MAHA PGM CET 2015)
a. Gaucher disease b. Adrenoleukodystrophy a. Wilson disease b. Von Gierke disease
c. Fabry's disease d. Hurles disease c. Phenylketonuria d. Diabetes
Chapter 6: Inborn Errors of Metabolism 125

 OTHER METABOLIC DISORDERS

Questions
107. In Wilson disease, which of the following is seen?
 (WB PGMEE 2016)
115. Menke’s disease is due to defect in the metabolism of:
a. Low ceruloplasmin, high urine copper
 (FMGE Dec 2018)
b. Low ceruloplasmin, low urine copper
c. High ceruloplasmin, high urine copper a. Zinc b. Copper
d. High ceruloplasmin, low urine copper c. Selenium d. Iron
108. Wilson's disease is caused by defect in: M  116. Metabolic disease in which intestinal obstruction is
 (MAHA PGM CET 2015) seen? (NEET PG Jan 2019)
a. ATP 7A mutation b. ATP 7B mutation a. Hereditary fructose intolerance
c. Ceruloplasmin d. ATP 7C mutation b. Phenylketonuria
c. Hurler disease d. Cystic fibrosis
109. About Wilson disease, all are true except:
117. A child presented with anemia, kinky hair, intellectual
 (Recent question 2013)
disability and seizures. Which of the following is the
a. Autosomal recessive b. KF ring
most likely diagnosis? (AIIMS Nov 2017)
c. Raised copper level d. Raised ceruloplasmin level
a. Menkes disease b. Down syndrome
110. Which of the following is true for Wilson disease?
c. Iron deficiency anemia d. Lesch Nyhan syndrome
 (AIIMS Nov 2012)
118. All are metabolic causes of liver disease except:
a. High ceruloplasmin
 (PGI May 2018)
b. Low serum ceruloplasmin and high urinary copper
c. Low serum copper a. Histiocytosis b. Hemochromatosis
d. Low urinary copper c. Gaucher disease d. Wilson disease
111. The gene for Wilson's disease is on: e. Galactosemia
a. Long arm of Chromosome 13 (MAHA PGM CET 2010) 119. Dried blood spot test in neonates is used in testing for:
b. Long arm of Chromosome 6  (AIIMS Nov 2017)
c. Short arm of Chromosome 13 a. Inborn error of metabolism
d. Short arm of Chromosome 6 b. Blood group
112. Congenital Wilson's disease is characterized by: c. Total cell count d. Creatinine and bilirubin
a. KF ring is present at birth (DNB Dec 2010) 120. Patient with growth retardation, delayed milestones with
b. May present as acute hepatitis kinky hair. What is the diagnosis? (AIIMS Nov 2017)
c. Decreased urinary copper excretion a. Menke's disease b. Trisomy 21
d. Decreased hepatic copper concentration c. Lesch Nyhan syndrome d. Wilson disease
113. True about Wilson's disease: (AIPGMEE 2010) 121. Hyperuricemia is caused in newborn due to defect in:
a. Increase in urinary copper and increased serum  (PGI Jan 2017)
ceruloplasmin and copper a. Glucose 6 phosphate dehydrogenase
b. Increased serum ceruloplasmin levels with increased b. Glucose 6 phosphatase
urinary copper c. Xanthine oxidase
c. Elevated hepatic copper level and increased serum d. Hypoxanthine guanine ribosyltransferase
ceruloplasmin levels e. Ornithine transcarbamylase
d. Increased in urinary copper and decreased serum 122. A child who had derangement of milestones and was
ceruloplasmin hyperactive later on developed self mutilating behavior
114. A 12-year-old girl with the mood and emotional and hair pulling behavior. The child died a few years
liability has a golden brown discoloration in descemet later. Which of the following enzyme is most likely
membrane. Most likely diagnosis is: (AIPGMEE 2004) deficient in the child? (AIIMS Nov 2016)
a. Fabry's disease b. Wilson's disease a. HGPRT b. Phenylalanine hydroxylase
c. Glycogen storage disease d. Acute rheumatic fever c. Adenosine deaminase d. Hexosaminidase A
126 Section 1: General Pediatrics
Review of Pediatrics and Neonatology

Answers with Explanations

 GLYCOGEN STORAGE DISORDERS


1. a. Von Gierke disease  Ref: Nelson 20/e p 716
In Von Gierke disease, hepatic carbohydrate metabolism is affected, which is responsible for plasma glucose homeostasis. So, fasting
hypoglycemia and hepatomegaly is seen. Despite, marked hepatomegaly, the liver transaminase levels are usually normal or only
slightly elevated. Jaundice is usually not seen.
2. a. Phenylalanine hydroxylase  Ref: Nelson's 20/e p 636-638
Deficiency of the enzyme phenylalanine hydroxylase or of its cofactor tetrahydrobiopterin (BH4) causes accumulation of phenylalanine
in body fluids and in the brain.
3. d. Debranching enzyme  Ref: Nelson's 20/e p 717-720
The given clinical scenario points towards an underlying glycogen storage disease, with both liver and muscle involvement;
During childhood, type III GSD (Debrancher deficiency) may be indistinguishable from type I GSD (Glucose-6-Phosphatase
deficiency), because hepatomegaly, hypoglycemia, hyperlipidemia, & growth retardation are seen in both.
Difference between Type I and Type III GSD:
Type I GSD Type III GSD
Kidney enlargement Kidneys are enlarged, but spleen and Splenomegaly may be present, but the kidneys are not
heart are normal enlarged.
Muscle involvement Not involved Skeletal and cardiac muscles may be involved
Liver transaminases Usually normal or only slightly elevated Elevation of liver transaminase levels and fasting
ketosis are prominent
Blood lactate and uric acid Elevated Usually normal
Creatine kinase level Normal May be elevated
Effect of Glucagon Little or no rise in blood glucose level, Glucagon given 2 hour after a carbohydrate
administration but the lactate level rises significantly on meal provokes a normal increase in blood glucose;
giving Epinephrine or Glucagon after an overnight fast, glucagon causes no change in
blood glucose level.
Liver biopsy finding Distention of hepatocytes by glycogen The fibrosis and the paucity of fat distinguish type III
and fat with little associated fibrosis. glycogenosis
In Type IV Glycogen Storage Disease (Branching Enzyme Deficiency), hypoglycemia is uncommon, and is only seen when there is
overt liver cirrhosis.
4. b. Glycogen Storage Disease Type II (Pompe disease)  Ref: Nelson's 20/e p 715-725, Ghai 8/e p 655-657
5. c. Scheie's disease  Ref: Nelson's 20/e p 715-725, Ghai 8/e p 655-657
Hurler/Scheie is a Mucopolysaccharidosis and not a Glycogen storage disease.
6. a. Cori's disease  Ref: Nelson's 20/e p 715-725, Ghai 8/e p 655-657
Important Glycogen Storage Diseases:
Type Name Enzyme deficient Characteristics
Type I Von Gierke's disease Glucose-6-phosphatase Liver and renal tubule cells loaded with glycogen. Hypo- glycemia,
lactic acidemia, ketosis, hyperlipidemia.
Type II Pompe's disease Lysosomal α-1 → 4 and 1 → Fatal, accumulation of glycogen in lysosomes, heart failure; muscle
6-glucosidase (acid maltase) involvement seen
Type III Limit dextrinosis, or Debranching enzyme Accumulation of a characteristic branched polysaccharide
Cori's disease (resembling ‘Limit dextrin')
Type IV Amylopectinosis, Branching enzyme Accumulation of a polysaccharide having few branch points. Death
Andersen's disease due to cardiac or liver failure in 1st year of life.
Type V McArdle's disease Muscle phosphorylase Diminished exercise tolerance; Little or no lactate in blood after
exercise; exercise-induced myoglobinuria
secondary to rhabdomyolysis seen
Type VI Hers disease Liver phosphorylaseQ High glycogen content in liver, tendency towards hypoglycemia;
Ketosis seen;
Type VII Tarui's disease Phosphofructokinase in As for type V + hemolytic anemia.
muscle and erythrocytes
Chapter 6: Inborn Errors of Metabolism 127

7. a. Glycogen storage disease type II  Ref: Nelson's 20/e p 715-725, Ghai 8/e p 655-657

Answers with Explanations


The given features along with ECG changes is suggestive of Pompe disease (Type II GSD).
8. a. Von Gierke's  Ref: Nelson's 20/e p 715-725, Ghai 8/e p 655-657
Presence of hepatomegaly, hypoglycemia, hyperlipidemia & acidosis, point towards Von Gierke disease.
9. a. Liver phosphorylase  Ref: Nelson's 20/e p 715-725, Ghai 8/e p 655-657
10. a. Von Gierke's disease  Ref: Nelson's 20/e p 719, Ghai 8/e p 655-657
•• Epinephrine increases blood glucose level by stimulating glycogenolysis and gluconeogenesis, mediated by the enzyme
glucose 6 phosphatase, that converts Glucose-6-phosphate to Glucose
•• Von Gierke disease is due to deficiency of Glucose 6 phosphatase enzyme; Hence, epinephrine is not able to increase
blood glucose in a child with Von Gierke disease, but lactate levels increase.
11. a. Type 1  Ref: Nelson's 20/e p 715-725, Ghai 8/e p 655-657
Type 2 Pompe disease predominantly involves muscles, while type I GSD involves liver.
Although type 2 and type 3 GSD are predominantly liver glycogenoses, skeletal muscle involvement may also be seen.
12. a. Von Gierke disease  Ref: Nelson's 20/e p 715-725, Ghai 8/e p 655-657
13. d. Pompe's disease  Ref: Nelson's 20/e p 715-725, Ghai 8/e p 655-657
14. b. Von Gierke's disease  Ref: Nelson's 20/e p 715-725, Ghai 8/e p 655-657
Hepatorenomegaly, hypoglycemia, hyperlipidemia, acidosis and normal structured glycogen deposition in liver is suggestive of
Von Gierke's disease; Normal structured glycogen excludes options c. Cori's and d. Anderson's disease.
15. a. Von Gierke's disease, c. McArdle's disease  Ref: Nelson's 20/e p 715-725, Ghai 8/e p 655-657

 OTHER DISORDERS OF CARBOHYDRATE METABOLISM


16. b. Galactosemia  Ref: Nelson 20/e p 726
Hypoglycemia and jaundice in a neonate, after initiation of breastfeeding, suggests a diagnosis of Galactosemia.
17. b. Elimination of galactose in diet will not reverse cataract  Ref: Nelson 20/e p 726
E. coli sepsis is common & the onset of sepsis often precedes the diagnosis of galactosemia
Elimination of galactose reverses growth failure, cataract, renal & hepatic dysfunction
GALT deficiency → Galactose-1-P accumulates → Injury to kidney, liver & brain
Galactokinase deficiency → accumulation of galactose &galactitol
The Duarte variant, a single amino acid substitution, has 50% of normal RBC enzyme activity, but usually no clinical significance.
This variant is the most common, with a carrier frequency of 12% in the general population
18. b. Aldolase B  Ref: Nelson 20/e p 720–28
The given history of symptoms, on introduction of fruit juices, including hepatic dysfunction, points to a diagnosis of Hereditary
fructose intolerance, which is due to deficiency of Aldolase B.
19. d. Hepatolenticular degeneration  Ref: Nelson's 20/e p 1939, Ghai 8/e p 320-321
The given picture shows Kayser Fleischer (KF) ring; KF ring along with a history of liver dysfunction points towards a diagnosis
of Wilson disease or Hepatolenticular degeneration.
20. b. H2  Ref: Nelson's 20/e p 2870, Ghai 8/e p 297
Breath hydrogen tests can be used to determine a specific carbohydrate malabsorption.
It can be used to diagnose bacterial overgrowth or lactose/fructose malabsorption.
21. a. Aldolase B  Ref: Nelson's 20/e p 720-728, Ghai 8/e p 655-657
In the given scenario, the child becomes symptomatic after ingesting fruit juice, which is often sweet, containing sucrose and hence,
glucose and fructose. So, he is suffering from Hereditary Fructose intolerance.
22. b. Alkaline urine  Ref: Nelson's 20/e p 2870, Ghai 8/e p 297
Acidic stool (pH < 5.5), Benedict's test +ve and Lactase enzyme deficiency are all features of Lactose intolerance.
23. a. Lactose intolerance  Ref: Lactose intolerance by Shinjini Bhatnagar. BMJ. 2007 Jun 30; 334(7608): 1331–1332.
Lactose intolerance
•• It is due to lactase deficiencyQ, which can be primary, secondary, congenital, or developmental
•• It should be suspected in people with abdominal symptoms after ingestion of milkQ and milk products
•• It may lead to diarrhea, dehydration and growth failure; Perianal excoriationsQ due to acidic stools are common.
24. c. Glucose 6 phosphatase  Ref: Nelson's 20/e p 715, Ghai 8/e p 655
Glycogenolysis or gluconeogenesis is mediated by glucose 6 phosphatase which is absent in Von Gierke disease.
128 Section 1: General Pediatrics

25. c. Galactose  Ref: Nelson's 20/e p 726, Ghai 8/e p 655


Review of Pediatrics and Neonatology

The given clinical features of vomiting, abdominal distension, diarrhea and presence of reducing substance in urine, in a 3-day-old
neonate, who is probably on exclusive breast feeding, suggest an underlying diagnosis of Galactosemia.
26. a. Mental retardation occurs, c. Defect in epimerase, d. Defect in galactose 1 PO4 uridyl transferase, e. Reducing substance
positive in urine  Ref: Nelson's 20/e p 726-727, Ghai 8/e p 655-657
Galactosemia is caused by deficiency of: GALT (Galactose 1 PO4 uridyl transferase) [Most common], Galactokinase or Epimerase.
Mental retardation can occur if the intake of milk or milk products is continued, in a child with Galactosemia.
27. b. Aldolase B  Ref: Nelson's 20/e p 726, Ghai 8/e p 655-657
In this question as the symptoms have developed following administration of sugarcane juice, so this suggests hereditary fructose
intolerance in which there is deficiency of the enzyme aldolase B.

 DISORDERS OF AMINO ACID METABOLISM


28. a. Phenylketonuria  Ref: Nelson 20/e p 636
29. b. Homocystinuria  Ref: Nelson 20/e p 326
Ectopia lentis is one of the features of Homocystinuria, where the direction of lens subluxation is infero-medial, unlike supero-lateral
subluxation in Marfan syndrome.
30. a. Multiple carboxylase deficiency  Ref: Nelson 20/e p 651–52
31. a. Inborn error of metabolism
Tandem mass spectrometry (TMS) on dried blood spot can be done for screening for metabolic disorders
32. b. SLC6A 19  Ref: Nelson 20/e p 3160
Hartnup's disease is due to mutation of SLC6A19 gene encoding the neutral amino acid transporter, resulting in deficiency of
nicotinamide synthesis
33. b. Pyridoxine  Ref: Nelson 20/e p 326
The given history of developmental delay, lens subluxation & stroke (thrombosis) points to a diagnosis of homocystinuria. Pyridoxine
(Vit B6) is used in the treatment of homocystinuria, cystathioninuria, pyridoxine-dependent epilepsy, a vitamin B6–responsive
anemia & xanthurenic aciduria.
34. c. Maple syrup urine disease  Ref: Nelson's 20/e p 650
Important clinical features of MSUD:
•• Infants are normal at birth; develop poor feeding and vomiting in 1st wk of life followed by lethargy and coma.
•• Physical examination reveals hypertonicity and muscular rigidity with severe opisthotonos.
•• Periods of hypertonicity may alternate with flaccidity, manifested as repetitive movements of extremities
•• Seizures occur in most infants, & hypoglycemia is common (correction of blood glucose does not improve clinical condition).
35. d. Tyrosinemia  Ref: Nelson's 20/e p 640-643
Nitisinone is the treatment of choice is for Tyrosinemia type I; Refer pretext of this chapter for details;
36. b. Tryptophan  Ref: Nelson's 20/e p 648-649
In Hartnup disorder, there is a defect in transport of monoamino-monocarboxylic amino acids (neutral amino acids), including
tryptophan, by intestinal mucosa & renal tubules due to mutation of SLC6A19 gene located on chr 5p15
37. b. Pyridoxine  Ref: Nelson's 20/e p 644-646
Subluxation of lens and mental retardation in a child who died suggests underlying homocystinuria;
Pyridoxine (vitamin B6) is useful in its treatment; refer pretext of this chapter for details.
38. a. Tyrosinemia  Ref: Nelson's 20/e p 640-643; refer pretext of this chapter for details.
39. a. Maple syrup urine disease  Ref: Nelson's 20/e p 649-656, Ghai 8/e p 652-655
Maple syrup urine disease is an autosomal recessive disorder of branched chain amino acid, i.e. valine, leucine and Isoleucine
40. a. Tandem mass spectrometry  Ref: Nelson's 20/e p 649-657
Tandem mass spectrometry (TMS) is the latest technical advance which requires a few drops of blood to be placed on a filter
paper and mailed to a central laboratory for assay to diagnose a large number of genetic conditions.
As it is a very sensitive test, it is used as one of the screening tests for suspected metabolic diseases.
41. c. Maple syrup urine disease  Ref: Nelson's 20/e p 649-657, Ghai 8/e p 652-655
42. a. Pellagra like rash  Ref: Nelson's 20/e p 648-649; refer pretext of this chapter for details.
43. a. Phenylketonuria (PKU)  Ref: Nelson's 20/e p 636-639, Ghai 8/e p 652-655
Microcephaly, blue eyes, fair skin and mental retardation along with positive ferric chloride test (which detects phenylalanine in
urine) is suggestive of Phenylketonuria caused by deficiency of phenylalanine hydroxylase.
44. b. Phenylalanine hydroxylase  Ref: Nelson's 20/e p 636, Ghai 8/e p 652-655
Chapter 6: Inborn Errors of Metabolism 129

45. a. Alkaptonuria  Ref: Nelson's 20/e p 636, Ghai 8/e p 652

Answers with Explanations


Ochronosis (dark spot on sclera/ear cartilage) is seen in Alkaptonuria which is an autosomal recessive disorder due to deficiency
of enzyme homogentisic acid oxidase.
46. b. Phenylketonuria  Ref: Nelson's 20/e p 636-639, Ghai 8/e p 652
47. b. Ornithine transcarbamylase  Ref: Nelson Textbook of Pediatrics, 19/e p447-450; Nelson's 20/e p 669-674
Discussing about the findings of each of the above enzyme deficiencies, one by one.

a. CPS1 •• Hyperammonemia with markedly increased glutamine and alanine


•• Urinary orotic acid is usually low or may be absent

b. Ornithine •• Hyperammonemia with elevated plasma glutamine and alanine


transcar- bamylase •• Marked increase in urinary excretion of orotic acid* (diff. from CPS deficiency)

c. Arginase •• Marked elevations of arginine in plasma and CSF


•• Urinary excretion of orotic acid, arginine, lysine, cystine, & ornithine are increased

d. •• Increase in plasma argininosuccinic acid, citrulline glutamine and alanine


Arginosuccinase lyase •• Argininosuccinic acid is found in large amounts in urine and spinal fluid

* Note: Orotic acid, (elevated in OTC deficiency) converts to Uracil via Pyrimidine pathway.
48. d. Multiple carboxylase deficiency  Ref: Nelson's 20/e p 651-652, Ghai 8/e p 652-655
Discussing each option one by one:

Option a. Skin rashes are not seen in Propionic aciduria


Option c. Normal NH3 level excludes urea cycle defect
Option b. Absence of hypopigmentation, presence of metabolic acidosis & increased blood ketones excludes PKU
Option d. Skin rashes with presence of ketones and metabolic acidosis are all features of Multiple carboxylase deficiency

49. a. Ornithine Permease  Ref: Nelson's 20/e p 651


Hyperammonemia-Hyperornithinemia-Homocitrullinemia (HHH) Syndrome:
In this rare autosomal recessive disorder, defect is in transport system of ornithine from cytosol into mitochondria, which causes an
accumulation of ornithine in the cytosol and a deficiency of ornithine inside mitochondria.
50. b. Vitamin C  Ref: Nelson's 20/e p 651
Some patients with Alkaptonuria benefit from high-dose vitamin C; In Alkaptonuria, Vitamin C has been shown to decrease the
build-up of brown pigment in cartilage & may slow the development of arthritis.
51. b. Alkaptonuria  Ref: Nelson's 20/e p 636, Ghai 8/e p 652-655
52. a. Branched chain amino acids  Ref: Nelson's 20/e p 636, Ghai 8/e p 652-655
53. b. Phenylketonuria  Ref: Nelson's 20/e p 636, Ghai 8/e p 652-655
Hypopigmentation and CNS involvement with seizures and mental retardation suggest Phenylketonuria.
54. a. Hartnup's disease  Ref: Nelson's 20/e p 636, Ghai 8/e p 652-655
55. b. Isovaleric acidemia  Ref: Nelson's 20/e p 636, Ghai 8/e p 652-655
56. a. Urine phenylalanine metabolites are increased  Ref: Nelson's 20/e p 636-639, Ghai 8/e p 655
Discussing about the options, one by one,

Option a. True Phenylalanine metabolites like phenylacetate, phenylpyruvate increase in blood and urine
Option b. False Phenylalanine hydroxylase is the enzyme deficient in this condition
Option c. False Serum tyrosine levels are decreased, as tyrosine is not getting synthesized
Option d. False If PKU is detected by neonatal screening and low phenylalanine diet is started immediately and continued life long
prognosis is better but, executive dysfunction may occur in spite of diet treatment

57. a. Phenylalamine hydroxylase  Ref: Nelson's 20/e p 636-642, Ghai 8/e p 653-655
58. a. Branched chain amino acids  Ref: Nelson's 20/e p 636-642, Ghai 8/e p 653-655
59. a. Lack of pigmentation  Ref: Nelson's 20/e p 636-642, Ghai 8/e p 653-655
Defect in Tyrosinase leads to Albinism, which presents with hypopigmentation.
60. a. Genitourinary system not involved  Ref: Nelson's 20/e p 636-642, Ghai 8/e p 653-655
Genitourinary system is also involved in Alkaptonuria, as this enzyme is expressed only in liver and kidneys; darkening of urine
on standing is an important feature of Alkaptonuria.
130 Section 1: General Pediatrics

61. c. Mental retardation  Ref: Nelson's 20/e p 636-642, Ghai 8/e p 653-655
Review of Pediatrics and Neonatology

Clinical features of Hartnup disease:


•• Most children with Hartnup defect remain asymptomatic
•• Major clinical manifestation in symptomatic patient is cutaneous photosensitivity: pellagra-like rash on sun exposure
•• Some patients may have intermittent ataxia & Psychological disturbances, but mental development is usually normal.
62. d. Phenylketonuria  Ref: Nelson's 20/e p 636-642, Ghai 8/e p 653-655
The bacterial inhibition assay of Guthrie is used for Phenylketonuria.
63. c. Albinism  Ref: Nelson's 20/e p 636-642, Ghai 8/e p 653-655
Intellectual disability is common in Phenylketonuria, Homocystinuria and MSUD.
64. a. Pheylketonuria  Ref: Nelson's 20/e p 636-642, Ghai 8/e p 653-655
Ferric chloride test is used to determine the presence or absence of phenols in a given sample.
FeCl3 test with urine gives green colour in Phenylketonuria (detects phenylalanine in urine).
65. b. Hartnup disease  Ref: Nelson's 20/e p 636-642, Ghai 8/e p 653-655
Laboratory findings in Hartnup disorder:
Aminoaciduria, involving neutral amino acids (alanine, serine, threonine, valine, leucine, isoleucine, phenylalanine, tyrosine,
tryptophan, histidine); Plasma concentrations of neutral amino acids are usually normal.
This seemingly unexpected finding occurs because these amino acids are absorbed as dipeptides and the transport system for small
peptides is intact in Hartnup disorder.
66. c. Cystathionine synthase  Ref: Nelson's 20/e p 636-642, Ghai 8/e p 653-655; Refer pretext of this chapter
67. b. Reduced tendon reflexes  Ref: Nelson's 20/e p 636-642, Ghai 8/e p 653-655
Clinical Manifestations of Phenylketonuria include:
•• Blonde hair, blue iris, fair skin, musty or mousy body odor, Vomiting, enamel hypoplasia, growth retardation. Mental
retardation — Not present at birth, develops gradually; > 90% of untreated patients have low IQ
•• Neurological signs — Irritability, tremors, convulsions, hypertonia, exaggerated reflexes and microcephaly
68. d. Phenylketonuria-aryl sulphatase A  Ref: Nelson's 20/e p 636-642, Ghai 8/e p 653-655
69. b. Mental retardation is the common presentation  Ref: Nelson's 20/e p 636-642, Ghai 8/e p 653-655
70. a. Limit intake of substrate for the enzyme  Ref: Nelson's 20/e p 636-642, Ghai 8/e p 653-655
Low phenylalanine diet is the treatment of choice for phenylketonuria.
71. c. Dihydropteridine reductase  Ref: Nelson's 20/e p 636-642, Ghai 8/e p 653-655
•• In 1-3% of infants with hyperphenylalaninemia, the defect resides in 1 of the enzymes necessary for production or recycling
of the cofactor BH4;
•• If these infants are misdiagnosed as having PKU, they may deteriorate neurologically despite adequate control of plasma
phenylalanine
•• BH4 is synthesized from guanosine triphosphate (GTP) by following enzymes: pterin-carbinolamine dehydratase,
dihydrobiopterin reductase, GTP cyclohydrolase, 6-pyruvoyl tetrahydropterin synthase.
72. a. Biotin  Ref: Nelson's 20/e p 636-642, Ghai 8/e p 653-655
73. a. Alkaptonuria  Ref: Nelson's 20/e p 636-642, Ghai 8/e p 653-655
Darkening of urine on standing is associated with Alkaptonuria, due to Homogentisic acid oxidase deficiency.
74. b. Alkaptonuria  Ref: Nelson's 20/e p 636-642, Ghai 8/e p 653-655
75. a. Maple syrup urine disease  Ref: Nelson's 20/e p 636-642, Ghai 8/e p 653-655
76. d. Phenylalanine should be completely stopped in diet  Ref: Nelson's 20/e p 636-642, Ghai 8/e p 653-655
In Phenylketonuria, low phenylalanine diet is used; small amount of phenylalanine should be added to diet.
•• Serum and urine phenylalanine and its metabolites are increased; Tyrosine becomes an essential amino acid
•• Blood for screening should be obtained after 24–48 hrs (Protein feeding) to prevent false –ve result.
77. a. Organic acidemia  Ref: Nelson's 20/e p 636-642, Ghai 8/e p 653-655
Presence of hyperammonemia along with lactic acidosis in a child with neurological and cutaneous manifestations suggests
a diagnosis of Organic acidemia.
78. a. Maple syrup urine ds  Ref: Nelson's 20/e p 636-642, Ghai 8/e p 653-655

 LYSOSOMAL STORAGE DISEASES


79. b. Hurler syndrome  Ref: Nelson 20/e p 738-739
Hurler disease (MPS-IH)
•• Severe, progressive disorder with multiple organ & tissue involvement that results in premature death, usually by 10 yr of age.
•• An infant with Hurler syndrome appears normal at birth, but inguinal hernia may be present.
Chapter 6: Inborn Errors of Metabolism 131

•• Diagnosis is usually made between 6 and 24 months with evidence of hepatosplenomegaly, coarse facial features, corneal

Answers with Explanations


clouding, large tongue, prominent forehead, joint stiffness, short stature, and skeletal dysplasia.
•• Acute cardiomyopathy, valvular heart disease with incompetence of mitral & aortic valves & narrowing of coronary arteries.
•• Most patients have recurrent upper respiratory tract and ear infections, noisy breathing, persistent copious nasal discharge &
obstructive airway disease.
•• Obstructive airway disease, respiratory infection, and cardiac complications are the common causes of death.
•• Most children with Hurler syndrome acquire only limited language skills because of developmental delay, combined conductive
and neurosensory hearing loss, and an enlarged tongue.
•• Progressive ventricular enlargement with increased intracranial pressure caused by communicating hydrocephalus also occurs.
•• Corneal clouding, glaucoma, and retinal degeneration are common.
•• Radiographs show a characteristic skeletal dysplasia known as dysostosis multiplex.
Patients with severe Hunter disease have features similar to those of Hurler disease except for the lack of corneal clouding & slower
progression of CNS deterioration.
The most common subtype is MPS-III, followed by MPS-I and MPS-II.
80. a. Cherry red macula  Ref: Nelson 20/e p 710-712
The given history of failure to thrive, recurrent respiratory infections with the given X-ray findings and foamy histiocytes on liver
biopsy, point to a diagnosis of Niemann-Pick disease, in which cherry red spot is seen.
81. b. Acid Sphingomyelinase  Ref: Nelson's 20/e p 708-712, Ghai 8/e p 659-664
82. a. Iduronate sulfatase  Ref: Nelson's 20/e p 708-712, Ghai 8/e p 659-664; Refer Ans 74 below;
83. a. β-glucosidase  Ref: Nelson's 20/e p 708-712, Ghai 8/e p 659-664
Gaucher disease is the most common lysosomal storage disorder with a defect of lysosomal enzyme acid β glucosidase.
84. b. Gaucher disease  Ref: Nelson's 20/e p 708-712, Ghai 8/e p 659-664
•• Gaucher cells are Lipid Laden Macrophages have a characteristic histologic appearance of “wrinkled tissue paper”
•• Membrane-bound inclusions filled with glucocerebroside are seen with electron microscopy.
85. a. Gaucher disease  Ref: Nelson's 20/e p 708-712, Ghai 8/e p 659-664
86. a. Hurler disease  Ref: Nelson's text book of Pediatrics, 20th ed/p 747-743, Ghai 8/e p 659-661
Hurler disease (deficiency of L-iduronidase)
•• Diagnosis is usually made between 6–24 months of age with hepatosplenomegaly, coarse facial features, corneal clouding, large
tongue, prominent forehead, joint stiffness, short stature and skeletal dysplasia
•• Most patients have recurrent upper respiratory tract infections, noisy breathing and copious nasal discharge
•• Valvular heart disease, obstructive airway disease, respiratory infection, and cardiac complications are common.

87. d. Glucocerebroside accumulation  Ref: Nelson's 20/e p 708-710, Ghai 8/e p 661-662
88. c. Parallel rays of tubular structures in lysosomes  Ref: Nelson's 20/e p 708, Ghai 8/e p 657-662
Hepatosplenomegaly with delayed development and given biopsy findings are suggestive of Gaucher disease.
In Gaucher disease electron microscopy reveals lipid material stored as bilayer in lysosomes.
89. c. Sanfilippo disease  Ref: Nelson's 20/e p 584, Ghai 8/e p 661; Refer to pretext of this chapter for details
90. a. L-iduronidase  Ref: Nelson's 20/e p 737-743, Ghai 8/e p 661; Refer to pretext of this chapter for details
91. b. Hexosaminidase  Ref: Nelson's 20/e p 737-743, Ghai 8/e p 661
Tay-Sachs disease is due to Hexosaminidase A deficiency.
92. b. Hunter's syndrome  Ref: Nelson's 20/e p 737-743, Ghai 8/e p 661
All mucopolysaccharidoses are autosomal recessive except Hunter's disease which is inherited as X-linked recessive.
93. a. PAS positive cells, d. Deficiency of glucocerebrosidase, e. Gaucher cells are present  Ref: Nelson's 20/e p 737- 743
94. b. Hunter's syndrome  Ref: Nelson's 20/e p 737-743, Ghai 8/e p 661
As Hunter's disease is a X linked recessive, males are mostly affected by it.
95. d. Gaucher's disease  Ref: Nelson's 20/e p 737-743, Ghai 8/e p 661

 LEUKODYSTROPHY
96. b. Adrenoleukodystrophy  Ref: Nelson 20/e p 2709-2710
Therapeutic approaches to ALD include administration of glycerol trioleate and glycerol trierucate (Lorenzo's oil), bone marrow
transplantation, and lovastatin.
97. a. Krabbe's disease  Ref: Nelson's 20/e p 2913
Krabbe disease (KD) is also called Globoid Cell Leukodystrophy; Refer pretext of this chapter for details
98. d. Alexander disease  Ref: Nelson's 20/e p 713-714;Radiopaedia.org; Caffey's 12e/ p337
132 Section 1: General Pediatrics
Review of Pediatrics and Neonatology

High Yield Points

Name of disease MRI Brain finding


Alexander disease T2 hyperintensities in frontal white matter; shows contrast enhancement;
Canavan disease Macrocephaly; bilateral symmetric T2 white matter hyperintensity diffusely throughout cerebral white matter; does
not enhance at CT & MRI; Large N acetyl aspartate (NAA) peak on MRS

Adrenoleukodystrophy Symmetrical cerebral white matter signal change involving posterior (occipitoparietal) periven­tricular white matter

Krabbe's disease CT: Hyperdense areas symmetrically involving thalami, caudate nuclei (basal ganglia), cerebellum, posterior limbs of
internal capsule and brainstem.
MRI: 2 patterns; (i) A patchy hyperintense periventricular signal with thalamic involvement on T2-weighted images,
consistent with hypomyelination, (ii) patchy low signal on T2-weighted images in a similar distribution to the
hyperdense regions seen on CT, which is suspected to represent a paramagnetic effect from calcium deposition in
the region.

99. b. Arylsulfatase A  Ref: Nelson's 20/e p 714


Metachromatic leukodystrophy is due to deficiency of Arylsulfatase A.
100. a. Canavan disease  Ref: Nelson's 20/e p 677-714
101. b. Krabbe disease  Ref: Nelson's 20/e p 714
102. b. Krabbe's disease  Ref: Nelson's 20/e p 714
103. c. Canavan's disease  Ref: Nelson's 20/e p 677, 713-714
In Canavan disease, there is Hypotonia, head lag & macrocephaly.
104. c. Krabbe's disease  Ref: Nelson's 20/e p 714
1
05. d. Exaggerated tendon reflexes  Ref: Ghai 8/e p 684-662; Refer to pretext for details about MLD;
In metachromatic leukodystrophy (MLD), a disorder of myelin metabolism, there is hypotonia and absent reflexes.

 WILSON DISEASE
106. a. Wilson disease  Ref: Nelson's 20/e p 1939-1940, Ghai 8/e p 320-321; Refer to pretext ch 18 for details;
Wilson disease (hepatolenticular degeneration) is an autosomal recessive disorder characterized by degenerative changes in
the brain, liver disease, and Kayser-Fleischer rings in the cornea.
107. a. Low ceruloplasmin, high urine copper  Ref: Nelson's 20/e p 1939-1940, Ghai 8/e p 320-321
High Yield Points
•• Best screening test for Wilson disease is serum ceruloplasmin (decreased in Wilson disease)
•• Serum copper level may be elevated in early Wilson disease
•• Urinary copper excretion (Normal <40 μg/day) is increased to >100 μg/day

108. b. ATP 7B mutation  Ref: Nelson's 20/e p 1939-1940, Ghai 8/e p 320-321
The abnormal gene for Wilson disease is ATP7B on long arm of chromosome 13 (13q14.3).
109. d. Raised ceruloplasmin level  Ref: Nelson's 20/e p 1939-1940, Ghai 8/e p 320-321
110. b. Low serum ceruloplasmin and high urinary copper  Ref: Nelson's 20/e p 1939-1940, Ghai 8/e p 320-321
111. a. Long arm of Chromosome 13  Ref: Nelson's 20/e p 1939-1940, Ghai 8/e p 320-321
112. b. May present as acute hepatitis  Ref: Nelson's 20/e p 1939-1940, Ghai 8/e p 320-321
Kayser-Fleischer rings are absent in young patients with hepatic Wilson disease up to 50% of the time but are present in 95% of
patients with neurologic symptoms.
113. d. Increased in urinary copper and decreased serum ceruloplasmin  Ref: Nelson's 20/e p 1939-1940
114. b. Wilson's disease  Ref: Nelson's 20/e p 1939-1940, Ghai 8/e p 320-321
KF ring in Wilson disease is due to deposition of Copper in the Descemet membrane layer of cornea.

 OTHER METABOLIC DISORDERS


115. b. Copper  Ref: Nelson 20/e p 344
In Menke's disease, the defective protein is a copper-binding ATPase (ATP7A), responsible for distribution & metabolism of copper
in tissues.
Chapter 6: Inborn Errors of Metabolism 133

116. d. Cystic fibrosis  Ref: Nelson 20/e p 2098

Answers with Explanations


In cystic fibrosis, neonates may present with meconium ileus, meconium plug & meconium peritonitis, while infants may present
with distal intestinal obstruction syndrome.
117. a. Menkes disease  Ref: Nelson 20/e p 1849
Menke disease is a X-linked recessive multisystem disorder that mainly involves CNS & usually presents in early infancy. Scalp hair
become hypopigmented, sparse, short, and brittle. Hair microscopy reveals pili torti (twisted hair) & trichorrhexis nodosa (fractures
of the hair shaft at regular intervals).
118. a. Histiocytosis Ref: Ghai 9/e p 648
Metabolic causes of Liver disease:
•• Carbohydrate disorders: Galactosemia, HFI, GSD
•• Amino acid disorders: Tyrosinemia, Urea cycle defects, MSUD
•• Lipid disorders: Gaucher disease, Niemann-Pick disease, Wolman disease
•• Others: Zellweger syndrome, Wilson disease, alpha1 AT deficiency, CF, Hemochromatosis
Langerhans cell histiocytosis refers to reactive proliferation of dendritic cells
119. a. Inborn error of metabolism  Ref: Ghai 9/e p 645
Tandem Mass spectrometry (TMS) done on dried blood spots is one of the screening tests for inborn errors of metabolism.
120. a. Menke's disease  Ref: Nelson 20/e p 1849
Menke's disease
•• It is caused by mutations in the gene encoding Cu2+ transporting ATPase (ATP7A)
•• Copper level in liver & brain are low in contrast to an increase in enterocytes & fibroblasts.
•• Progressive cerebral degeneration (seizures), feeding difficulties, failure to thrive, hypothermia, apnea, hair abnormalities (kinky
hair), hypopigmentation, bone changes & cutis laxa seen;
•• Patients with the classic form of Menke's disease usually die before 3 yr age.
121. b. Glucose 6 phosphatase, d. Hypoxanthine guanine ribosyl transferase  Ref: Nelson's 20/e p 744-745;
Plasma uric acid is elevated in Von Gierke disease & Lesch Nyhan-disease while it is very low in xanthine oxidase deficiency.
122. a. HGPRT  Ref: Nelson's 20/e p 745-748; Refer pretext of this chapter for details;
Delayed development, hyperactivity and self-mutilating behavior in a child who died later is suggestive of Lesch- Nyhan disease
due to HGPRT deficiency.
Chapter 7
Diseases of
Immune System
 PRIMARY IMMUNODEFICIENCY DISEASES
Question 1
A 1 month old baby presents with Definition: A group of disorders characterized by an impaired ability to produce normal immune
the following picture. He had a his- response. Most of these disorders are caused by mutation in gene involved in development &
tory of neonatal sepsis requiring function of immune organ, cells & molecules.
hospitalization. His Hb was 15 g/dl, A. DEFECTS IN INNATE IMMUNITY: LEUKOCYTE FUNCTION DEFECTS
Total leucocyte count was 18,000
Disease Defect
& Platelet count 2.8 lac/mm.3 What
is the most probable diagnosis? Leukocyte adhesion defi­ Defective WBC adhesion due to mutation in β chain of CD11/CD18
ciency 1 (LAD 1) integrinsQ
Leukocyte adhesion defi­ Due to mutations in fucosyl transferase required for synthesis of
ciency 2 (LAD 2) sialylated oligosaccharide (receptor for selectins)
Chediak – Higashi syndrome Mutations of protein involved in lysosomal membrane trafficQ
CGD Decreased oxidative burstQ due to defective phagocyte oxidase
Myeloperoxidase deficiency Decreased microbial killing because of defective MPO-H2O2 systemQ

High Yield Points


a. Chronic granulomatous disease •• Delayed separation of umbilical cord stump is seen in LAD type 1.
b. Chediak Higashi syndrome •• Bombay blood group is associ­ated with LAD type 2.
c. Leukocyte adhesion defect
d. Myeloperoxidase deficiency CHÉDIAK-HIGASHI SYNDROME
Basic defect Inherited defect in Phago-lysosome function
Autosomal recessive defects in lysosomal transport protein LYST,Q encoded by
Genetic basis
gene CHS1 at 1q42
•• Defective degranulation, impaired chemotaxis of WBCs;
Pathophysiology
•• Delayed microbial killing; NK cell function is also impaired
•• Defective fusion of phagosomes & lysosomes (susceptibility to infections)
•• Abnormalities in melanocytesQ (leading to albinism)
Clinical features
•• Abnormalities in cells of nervous system (associated with nerve defects)
•• Abnormalities in plateletsQ (causing bleeding disorders).
Question 2 •• Neutropenia (most common)Q; Leukocytes contain giant granules,Q
Diagnosis
All of the following syndromes (characteristic) which result from aberrant phagolysosome fusion
can present with recurrent infec- •• High dose Vitamin C is useful in stable phase; the only curative therapy to prevent
Treatment
tions, anemia & hypopigmentation accelerated phase is HSCT.
except?
CHRONIC GRANULOMATOUS DISEASE (CGD)
Basic defect Inherited defect in Microbicidal activity due to decreased oxidative burst
Genetics Defect in genes encoding components of phagocyte oxidaseQ
•• X Linked: Defect in membrane-bound components (gp91phox)Q
•• Autosomal recessive: Defect in cytoplasmic component (p47phox & p67phox)Q
Pathophysiology •• Defective phagocytic killing with inability to kill catalase positive organisms
•• Abscess formation & characteristic granulomas at sites of macrophage
accumulation beneath the skin, liver, lungs, spleen, and lymph nodes
a. Chediak-Higashi syndrome Clinical features •• Recurrent pneumonia, lymphadenitis, subcutaneous abscesses, osteomyelitis,
b. Griscelli syndrome granulomes at multiple sites, with family history of recurrent infections
c. Hermansky Pudlak syndrome type II •• Any infection with an unusual catalase-positive organism requires evaluation
d. Chronic granulomatous disease •• Blood stream infection caused by Salmonella, Burkholderia cepacia, or Candida
Contd...
Chapter 7: Diseases of Immune System 135

Contd...

Section 1: General Pediatrics


High Yield Points   M
Diagnosis • Nitroblue-tetrazolium (NBT) test (qualitative test)
– Negative in CGD patients & positive in normal individuals •• Most widely used screening test for
– Depends upon direct reduction of NBT by superoxide free radical CGD is Nitroblue Tetrazolium test.
•• Dihydrorhodamine (DHR) test: It is confirmatory test for (CGD) •• Confirmatory test for CGD is
•• Cytochrome C reduction assay: Amount of superoxide a patient’s phagocytes Dihy­drorhodamine test.
can produceQ

COMPLEMENT DEFICIENCIES AND ASSOCIATED DISEASES M


Question 3
Pathway Components Associated Diseases A 2 years old child presented with
Clq, Clr, Cls, C4,C2 Immune-complex synd, pyogenic infections recurrent abscesses at multiple
Classic Pathway  sites. Some special staining test
C1 esterase Inhibitor Hereditary Angioneurotic edemaQ was performed on which his blood
showed the findings shown in the
C3 Immune-complex syndromes, pyogenic infections
given picture A, while his sister’s
Alternative D, I Pyogenic infections blood showed the findings seen in
Pathway & C3  figure B. What is the disease that
Properdin Neisseria infectionsQ
this child is probably suffering
H Hemolytic uremic syndromeQ from?

Membrane Attack Complex  C5, C6, C7, C8, C9 Recurrent Neisseria infections, immune-complex ds

 B. DEFECTS IN ADAPTIVE IMMUNE SYSTEM


PREDOMINANT B CELL (HUMORAL) DEFECTS

Disease Primary defect Secondary defect Infections / Features A B

Bruton’s X-Linked Bruton tyrosine Failure of pre B-cells to H. influenzae, S. aureus a. Hyper IgE syndrome
Agammaglobulinemia M kinase (Btk); Xq21.22 mature S. pneumoniae b. Hyper IgM syndrome
gene mutationsQ Plasma cells absentQ c. Chronic Granulomatous disease
Underdeveloped Germinal
d. Chediak Higashi syndrome
T-cells normal centers of L. nodes & Tonsils

Hyper-IgM Syndrome X-linked: mutations in Defect in Ig class Recurrent pyogenic


CD40L (Xq26)Q switchingQ and affinity infectionsQ
Autosomal recessive: maturation Pneumocystis jiroveciQ
Mutations in CD40 & Inability to produce
cytidine deaminase IgG, IgA, and IgE
Isolated IgA Deficiency Impaired Deficient IgA Respiratory infections
differentiation of B Concomitant defect in AllergyQ
cells to IgA-producing IgG2 and IgG4Q Autoimmune: SLE & RAQ
plasma cells
Common Variable Defect in receptor for Defect in survival and Recurrent sinopulmonary &
Immunodeficiency BAFFQ differentiation of B cells pyogenic inf,
Herpes or Enteroviral
Defect in ICOSQ Defect in T-cell activation
meningoencephalitis
(inducible co- & interactions b/w T &
BcellsQ G. lamblia diarrheaQ
stimulator)
hyperplasticQ lymphoid
tissue

How to differentiate Bruton’s Agammaglobulinemia from CVID?


Bruton’s agammaglobulinemia Common variable immunodeficiency (CVID) High Yield Points   M
More common in males No sex predilection
•• Tonsils are absent in X linked agam-
Usually presents at 6 to 18 months of age Presents later in childhood maglobulinemia
Absence of tonsils & adenoids Tonsils & adenoids visible/palpable
Lymph nodes not palpable Lymph nodes may be palpable
B cells (CD19) are absent in peripheral blood Panhypogammaglobulinemia with normal B cell number
136 Section 1: General Pediatrics

COMBINED T AND B CELL DEFECTS


Review of Pediatrics and Neonatology

High Yield Points  


•• Nitroblue tetrazolium (NBT) test is Disease Primary defect Secondary defect Infections / Features
used for Phagocytes
•• Commonest fungal infection in Severe Combined X-linked:γ-chain (γc) Defect in signaling of IL-2, •• CandidaQ
neutropenia is CandidaQ Immunodeficiency M of cytokine receptor 4, 7, 9, 11, 15 &21 •• PneumocystisQ
mutations •• Pseudomonas
Autosomal recessive: Accumulation of •• CMV, Varicella
Question 4 adenosine deaminase deoxyadenosine-toxic to •• Thymus is smallQ
(ADA) deficiencyQ rapidly dividing WBCs
In which of the following immuno-
deficiencies, this chest X-ray find- Ataxia Telangiectasia Autosomal- •• Combination of T & B •• Ataxia
ing is usually seen? recessive mutation cell defects •• Vascular
in ATM geneQ •• Defective isotype telangiectasesQ
(ataxia telangiectasia switching •• Neurologic deficitsQ
mutated) on chr 11 •• Low IgA & IgG-2 •• TumorsQ

Hyper IgE syndrome Autosomal •• Elevated serum IgE •• Recurrent skin abs-
dominant:Qmutation levels cessesQ
in STAT3 gene: •• Defective TH17 effector •• Lung infections &
a. Wiskott-Aldrich syndrome
Defective signaling responsesQ pneumatocelesQ
b. DiGeorge syndrome
pathways •• Dysmorphism
c. Hyper IgE syndrome
d. Chediak Higashi syndrome •• Hyperextensibility

X-linked Mutations in SLAM- Activation of NK, T & B Epstein-Barr virus (EBV)


Lymphoproliferative associated protein cells, including Signalling
disease (SAP)Q Lymphocyte Activation
Molecule (SLAM)
Mnemonic M
Wiskott-Aldrich X-linked: Mutations •• CD8+Tcell def ThrombocytopeniaQ
Important features of Wiskott Aldrich
syndrome → ‘RET’: Syndrome in Wiskott-Aldrich •• IgM-lowQ EczemaQ
•• Recurrent infections syndrome protein •• IgA & IgE-↑edQ Recurrent infectionQ
•• Eczema (WASP), Xp11.23Q •• IgG-normalQ
•• Thrombocytopenia

Question 5
A child with recurrent episodes of sinusitis & progressive cerebellar ataxia presents with the
eye lesion shown in the picture. What could be the diagnosis?
Mnemonic
Immunoglobulins in Wiskott Aldrich
syndrome:
•• Increased → IgE, IgA
•• Low → IgM

a. Hyper IgE syndrome


b. Wiskott Aldrich syndrome
c. Ataxia telangiectasia
d. Severe combined immunodeficiency

SEVERE COMBINED IMMUNODEFICIENCY (SCID)


It is one of the most severe immunodeficiency disorders, characterized by absence of all
adaptive immune function &, in some, a lack of B cells & natural killer (NK) cells also.

Underlying Defects
Due to defects in genes for Adenosine deaminase (ADA), JAK3, CD45, IL-7Ra, RAG1 or RAG2.
Chapter 7: Diseases of Immune System 137

Lymphoid Organ Involvement in SCID

Section 1: General Pediatrics


High Yield Points  
•• Thymus: very small containing no thymocytes or Hassall corpuscles. Thymic epithelium
•• In SCID, due to combined deficiency
appears histologically normal. of B & T cells, so Autoimmune ds,
•• Spleen: Both the follicular & paracortical areas are depleted of lymphocytes. Granuloma formation & Graft
•• Lymph nodes, tonsils, adenoids & Peyer patches are absent or extremely underdeveloped. rejection are rarely seen.
•• The commonest Primary immuno­
Clinical Manifestations deficiency is Isolated IgA immu­no­
deficiency
•• Present within the 1st few months of life with recurrent or persistent diarrhea, pneumonia,
otitis media, sepsis, cutaneous infections & wasting
•• Persistent infections with opportunistic organisms including Candida albicans, Pneumo-
cystis jiroveci, Rotavirus vaccine virus, CMV, EBV, varicella- zoster virus, measles virus or High Yield Points  
BCG. •• X-linked SCID (X-SCID) is the most
•• Affected infants also lack the ability to reject foreign tissue & are therefore at risk for common form of SCID in United
severe graft-versus host disease (GVHD) from T lymphocytes in non-irradiated blood States
•• Second most common form of SCID
products or from maternal immunocompetent T cells that crossed the placenta.
is due to enzyme ADA deficiency

Laboratory Findings
•• T-cell receptor recombination excision circles are absent or extremely low.
•• Absent lymphocyte proliferative responses to mitogens and antigens in vitro.
•• Patients with ADA deficiency have the lowest absolute lymphocyte counts (<500/mm3). Question 6 M
•• T-cells are extremely low or absent.
What is the probable genetic de-
•• Serum immunoglobulin (Ig) concentrations are low or absent, and no antibodies are fect in this child with recurrent in-
formed after immunizations. fections, tetany, oral candidiasis &
a heart murmur?
Treatment
HLA-identical or T-cell–depleted haploidentical (half-matched) parental hematopoietic stem cell
transplantation or gene therapy.

Prognosis
Death usually occurs during the 1st yr of life & almost invariably before 2 yr of age.
If diagnosed within the 1st 3.5 mo of life, >92% of cases can be treated successfully with stem cell a. 21q deletion b. 21p deletion
transplantation. c. 22q deletion d. 22p deletion

PREDOMINANT T CELL DEFECT

Disease Primary defect Secondary defect Infections / Features Mnemonic M

DiGeorge Syndrome Familial cases: •• T-cell defect •• Fungal & viral inf Features of DiGeorge syndrome:
(Thymic Hypoplasia)Q •• TetanyQ •• Paracortical areas of “CATCH-22” : (22q del)
failure of development
of 3rd& 4th pharyngeal •• Congenital defects L. nodes depleted C - Conotruncal abnormalities
pouchesQ of heart & great •• Periarteriolar sheaths A - Abnormal facies
Non-familial cases: vessels of spleen T - Thymic absence, T-cell defect
22q11 delQ syndr (50%) •• Mucocutaneous C - Cleft palate
Candidiasis H - Hypoparathyroidism with hypocal-
cemia
Bare lymphocyte Defect in class II MHC Abnormal development
syndrome gene expression of CD4+ T cells

IMMUNE REGULATORY DEFECTS

Innate immunity •• Autoinflammatory syndromes (outside the scope of this chapter)


•• Severe colitis

Adaptive immunity •• Hemophagocytic lymphohistiocytosis (HLH)


•• Autoimmune lymphoproliferation syndrome (ALPS)Q
•• Autoimmunity and inflammatory diseases (IPEX, APECED)
138 Section 1: General Pediatrics

HEMOPHAGOCYTIC LYMPHOHISTIOCYTOSIS (HLH)


Review of Pediatrics and Neonatology

Question 7
Identify the abnormality seen in the Characte­- •• Activation of CD8+ T lymphocytes and macrophages leading to organ damage
bone marrow aspirate of this child rized by •• Impaired T and NK lymphocyte cytotoxicity
with fever, hepatos­plenomegaly & Triggered Viruses (Most commonly: EBV)Q
pancyto­penia: by
Clinical •• Fever • Hepatosplenomegaly
features •• Edema • Neurologic diseases
Lab •• Blood cytopenia • Increased liver enzymes
findings •• Hypofibrinogenemia • High triglyceride levels
•• Elevated markers of T cell activation
•• Hemophagocytic features in the bone marrow or cerebrospinal fluid
•• Functional assays of post-activation cytotoxic granule exocytosis (CD107 fluorescence
at cell membrane) suggest genetically determined HLH
a. Hemophagocytosis
Subsets of Familial HLH (A.R) HLH with partial X-linked proliferative syndrome
b. LD body
HLH albinism following EBV infection
c. Malaria parasite
d. Myeloblasts •• Perforin def.Q •• Chediak-Higashi •• SH2DIA gene
•• Munc13-4 syndromeQ •• Low 2B4-mediated NK cell cytotoxicity
deficiency •• Griscelli syndrQ •• Impaired differentiation of NK T cells
•• Syntaxin 11 •• Hermansky Pudlak •• Defective antigen-induced T cell death
deficiency syndrome type II. •• Defective T cell helper activity

SECONDARY (ACQUIRED) IMMUNODEFICIENCIES

Cause Mechanism
Human immunodeficiency virus (HIV) infectionQ Depletion of CD4 + helper T cells

Mnemonic Irradiation & chemotherapyQ for cancer Decreased bone marrow precursorsQ for all WBCs
•• IPEX is X-linked immune dysfunc- Involvement of bone marrow by cancers Reduced site of leukocyte development
tion enteropathy polyendocri- (Metastases, Leukemias)
nopathy
•• APECED is Autoimmune, Polyendo- Protein- calorie malnutrition Lymphocyte maturation & function affected
crinopathy, Candidiasis, Ectodermal
Removal of spleen Decreased phagocytosis of microbes
Dystrophy

 VASCULITIS
Definition: Vasculitis is the inflammation of vessel wall.

Mnemonic M

•• Large road → GT road


–– Giant cell arteritis
–– Takayasu arteritis
•• Milk is Poor in vitamin K → Medium
vessel vasculitis are Kawasaki dis-
ease and Polyarteritis nodosa

ANCA (Anti-neutrophilic cytoplasmic Ab)


Mnemonic M
•• c-ANCA is against Proteinase 3, seen in Wegner’s Granulomatosis (90-95%)
Small vessel vasculitis: •• p-ANCA is against Myeloperoxidase
For “My Small hat will give big care”
•• Microscopic polyangitis
p-ANCA can be seen due to following causes:
•• SLE Common Other causes
•• Henoch Schonlein Purpura
•• Wegener’s Granulomatosus •• Microscopic PAN (microscopic polyangitis)Q •• Drugs (hydralazine, propythiouracil)
•• Good pasture syndrome •• Churg-Strauss syndromeQ •• Crescentic GlomerulonephritisQ
•• Behet syndrome •• Good pasteur’s syndromeQ
•• Churg-strauss syndrome •• Ulcerative colitis, primary sclerosing cholangitis
Chapter 7: Diseases of Immune System 139

TAKAYASU AORTOARTERITIS

Section 1: General Pediatrics


•• It is the most common cause of renovascular hypertension in India.
•• It is a segmental inflammatory panarteritis resulting in stenosis & aneurysms of aorta & its
major branches causing weak arterial pulses
•• Diagnosis of Takayasu arteritis is confirmed by Angiography
•• Treatment involves angioplasty & use of prednisolone, methotrexate & antihypertensives

KAWASAKI DISEASE M

•• Kawasaki disease is not seen in adults


•• Acute febrile illness of infancy & childhood (80% patients are < 4 years)
•• Diagnostic criteria: Fever for at least 5 days with any 4 out of the 5 “C-R-E-A-M” features
•• Complications: Coronary artery aneurysm → Acute Myocardial InfarctionQ
•• Treatment: IV Immunoglobulins, Steroids, Aspirin

Question 8
Identify the disease this child with
fever for 6 days is suffering from? Question 9 M

Choose the TRUE statement re-


Mnemonic M garding the disease depic­
ted in
Clinical features of Kawasaki disease: CREAM: the images shown:
•• C-Conjunctivitis (non-exudative); non purulent
conjuncivits
•• R-Rash (Polymorphous non-vesicular)
a. Scarlet fever •• E-Edema (or erythema of hands or feet)
A B
b. Henoch Schonlein purpura •• A-Adenopathy (cervical, often unilateral and non
c. Kawasaki disease suppurative) a. IV immunoglobulin is used in
d. Takayasu disease   •• M-Mucosal involvement (strawberry tongue Q) treatment
b. Bilateral purulent conjunctivitis
c. Lymphadenopathy is rare
High Yield Points   d. Aspirin is contraindicated
•• Coronary art aneurysm is seen in 25% cases of Kawasaki disease
•• There is no diagnostic test for Kawasaki disease
•• Lymph node Bx is not useful in diagnosis of Kawasaki disease
•• Platelet count rapidly increases by the 2nd–3rd week of illness
•• An elevated ESR and/or C-reactive protein value is universally present in the acute phase of illness.

HENOCH-SCHÖNLEIN PURPURA (HSP) M Question 10 M

Henoch-Schönlein purpura (HSP) is the most common vasculitis of childhood. A 5-year-old child presented with
abdominal pain, arthralgia & ele­
Definition •• Characterized by leukocytoclastic vasculitis & IgA deposition in small vessels of vated, red skin rashes, as shown
skin, joints, gastrointestinal tract, and kidney in the picture below:
Epidemiology •• It affects males more than females; more common in winter & spring
•• 90% of HSP cases occur in children, between 3-10 yr age
Pathogenesis •• Infectious triggers suspected; it is mediated by IgA complexes
Clinical •• Palpable purpura, usually symmetric, mainly on lower extremities & buttocks
manifestations •• Oligoarticular arthritis and arthralgias occur in 75% of children with HSP
•• GI manifestations (in 80%) include abdominal pain, diarrhea & melena
•• Renal involvement (in 50%): microscopic hematuria, proteinuria, hypertension,
nephritis, nephrotic syndrome, renal failure
•• Neurologic manifestations include cerebral hemorrhage, seizures, headaches
Criteria By American College of Rheumatology Pediatric Rheumat European society
2 of the following must be present: Palpable purpura (in absence of or
•• Palpable purpura thrombocytopenia) & 1 or more of: What is the most probable
•• Age at onset ≤20 yr •• Abdominal pain diagnosis?
•• Bowel angina (postprandial abdominal •• Arthritis or arthralgia a. Kawasaki disease
pain, bloody diarrhea) •• Biopsy showing IgA deposition b. Henoch-Schönlein purpura
•• Biopsy demonstrating intramural gran- •• Renal involvement (proteinuria, c. Juvenile Idiopathic arthritis
ulocytes in arterioles or venules hema­turia or red cell casts) d. Behcet syndrome

Contd...
140 Section 1: General Pediatrics

Contd...
Review of Pediatrics and Neonatology

High Yield Points   M


Diagnosis •• Leukocytosis, thrombocytosis, mild anemia, normal platelet count
•• Elevations of ESR, C-reactive protein (CRP) & serum IgA
•• The diagnosis of HSP is a clinical •• Occult blood is frequently found in stool specimens
one & no lab finding is diagnostic •• Serum albumin levels may be low due to renal or intestinal protein loss
•• Thrombocytopenia is not seen in
•• Biopsy of involved tissue shows IgA deposition in walls of small vessels
HSP
•• Hallmark of HSP is palpable pur- Treatment •• Treatment for mild and self-limited HSP is supportive
pura •• Prednisone (1 mg/kg/day for 1-2 wk) reduces abdominal and joint pain but does not
alter overall prognosis nor prevent renal disease
Prognosis •• Most children have a self-limited course lasting around 4 wk (prognosis good)
•• 15-60% have recurrences, within 4-6 months of diagnosis
•• Chronic renal disease develops in 1-2% of children with HSP

CHURG-STRAUSS SYNDROME (EOSINOPHILIC GRANULOMATOSIS


WITH POLYANGIITIS)
•• Small-vessel necrotizing vasculitis associated with asthma, allergic rhinitisQ
•• Lung infiltrates, peripheral hypereosinophilia, and extravascular necrotizing granulomataQ;
P-ANCAQ present in less than 50% of patients
•• Palpable purpura, GI bleeding & renal disease (primarily FSGS) seen

WEGENER’S GRANULOMATOSIS
It is now called Granulomatosis with Polyangiitis & is characterized by a triad of:
•• Acute necrotizing granulomas of respiratory tract
•• Necrotizing or granulomatous vasculitisQ affecting vessels in lungs
•• Focal necrotizing, often crescentic, glomerulonephritisQ

BEHÇET DISEASE
•• Clinical triad of recurrent oral aphthous ulcers, genital ulcers, and uveitisQ
•• Sin qua non for diagnosis—oral ulceration (hallmark)Q
•• Microscopic findings are nonspecific-neutrophils, are seen infiltrating vessels walls
•• HLA-B51Q associated; Steroids or TNF-antagonist therapies are effective.

Question 11 JUVENILE DERMATOMYOSITIS (JDM)


Identify the disease in this child
with violet colored rashes around
Clinical Features
the eyes & on hands? Rashes Photosensitive erythematous rash:
•• Known as “shawl sign” if it involves chest & neck
•• Characteristic heliotrope rash: a blue-violet discoloration of eyelids
•• Facial erythema crossing the nasolabial folds is also common
Gottron Pink or pale, shiny, thickened or atrophic plaques over proximal & distal interphalangeal
papules joints & occasionally on knees, elbows, small joints of the toes, & ankles

A B Nails Evidence of small vessel inflammation is often visible in nail folds and gums as individual
capillary loops that are thickened & tortuous
a. Juvenile dermatomyositis Weakness •• Insidious onset, typically symmetric, affecting proximal muscles such as the neck flexors,
b. Systemic lupus erythematosus shoulder girdle, and hip flexors; Muscle tenderness & Gower sign seen
c. Kawasaki disease
•• Difficulty climbing stairs, combing hair, and getting out of bed
d. Henoch-Schönlein purpura
•• Esophageal & respiratory muscles affected, resulting in aspiration or respiratory failure

Diagnostic Criteria for JDM


Classic rash: Heliotrope rash on the eyelids & Gottron papules
Plus 3 of following:
•• Weakness: Symmetric & Proximal
•• Muscle enzyme Elevation: Creatine kinase, SGOT, LDH or Aldolase
Chapter 7: Diseases of Immune System 141

•• Electromyographic changes: Short, small polyphasic motor unit potentials, Fibrillations

Section 1: General Pediatrics


High Yield Points  
•• Muscle biopsy: Necrosis & Inflammation
•• Anti SSA (Ro) Ab is associated with
Investigation Findings in JDM neonatal lupus syndrome (congenital
heart block)
Investigation Findings
Blood •• Elevated enzymes (creatine kinase, aldolase, SGOT & LDH)
investigations •• Alanine aminotransferase is most commonly elevated on initial presentation in
JDM whereas creatine kinase level may be normal initially
•• ESR is often normal & Rheumatoid factor negative
•• Antinuclear antibody is present in >80% of children with JDM
•• Myositis-specific Ab such as anti–Jo-1, anti–Mi-2, anti-p155/140, anti-NXP2 seen

Imaging (MRI) Identifies active sites of disease & increases sensitivity of muscle biopsy

Muscle biopsy Shows evidence of disease activity and chronicity

Electromyography Shows signs of myopathy (increased insertional activity & fibrillations) & muscle fiber
(EMG) necrosis (decreased action potential amplitude & duration)

NCV Typically normal unless severe muscle necrosis and atrophy are present

Muscle biopsy •• Indicated when diagnosis is in doubt or for grading severity


•• Shows focal necrosis, phagocytosis of muscle fibers, fiber regeneration, endo­
mysial proliferation, inflammatory cell infiltrates & tubuloreticular inclusions

Answer Keys for Image-Based Questions

Answers Explanations / Identifying features


1. Ans. c. Leukocyte adhesion This 1 month old child still has the umbilical cord stump in situ, as shown in the picture. So there is delayed
defect fall of umbilical cord.
2. Ans. d. Chronic granulomatous Child in the given picture has hypopigmented hairs & there is a history of recurrent infections & anemia,
disease suggesting underlying pancytopenia, which can be seen in HLH (Hemophagocytic Lymphohistiocytosis)
3. Ans. c. Chronic granulomatous A shows no blue granules, as seen in CGD, while, B shows Macrophages with blue black granules, as seen in
disease normal individuals, as they are able to reduce the colourless NBT dye
4. Ans. c. Hyper IgE syndrome Chest X-ray shows pneumonia with multiple pneumatoceles, suggestive of Staphylococcal pneumonia
5. Ans. c. Ataxia telangiectasia Ocular telangiectasia in a child with ataxia and history suggestive of immunodeficiency
6. Ans. c. 22q deletion This child has a long face with broad nasal bridge, narrow palpebral fissures & micrognathia.
7. Ans. a. Hemophagocytosis The bone marrow aspirate shows erythrophagocytosis by a macrophage, which is a feature of
Hemophagocytic Lymphohistiocytosis (HLH)
8. Ans. c. Kawasaki disease Fever for > 5 days with conjunctivitis, rash, strawberry tongue & cervical lymphadenopathy suggest a
diagnosis of Kawasaki disease
9. Ans. a. IV immunoglobulin is A → shows mucositis (strawberry tongue)
used in treatment B → Angiography showing Coronary artery aneurysm; Both of these findings suggest a diagnosis of
Kawasaki disease; IV immunoglobulin is used in treatment of Kawasaki disease
10. Ans. b. Henoch-Schönlein Abdominal pain, arthralgia & palpable purpura are seen in Henoch Schonlein purpura
purpura
11. Ans. a. Juvenile dermatomyositis A → Heliotrope rash on eyelids
B → Gottron’s papules     }
Both these features are seen in Juvenile Dermatomyositis
142 Section 1: General Pediatrics
Review of Pediatrics and Neonatology

Questions

 PRIMARY IMMUNODEFICIENCY DISEASES 7. “Hereditary angioneurotic edema” is due to deficiency


of: (MAHA PGM CET 2016)
1. What does ‘A’ depict in the given image of serum a. C1 inhibitor b. S protein
immunoglobulin levels in a pediatric patient? c. C3b inactivator d. C2 kinins
 (AIIMS May 2018)
8. A child having delayed separation of umbilical cord
with leukocytosis with Down syndrome and recurrent
infections. Diagnosis: M  (Recent Question 2013)
a. Leucocyte adhesion deficiency
b. Neonatal sepsis
c. Histiocytosis-X
d. All of above
9. Both B and T cell defect is present in: M  (PGI May 12)
a. SCID
b. Common Variable immunodeficiency
c. Wiskott-Aldrich syndrome
d. X-linked Agammaglobulinemia
e. Chronic mucocutaneous candidiasis
10. Chediak Higashi syndrome is characterized by the
following except: (MAHA 2011)
a. Neutrophilia b. Defective degranulation
c. Delayed microbial killing d. Giant granules
11. All are true regarding SCID except: (JIPMER 2011)
a. IgM b. IgG a. Due to deficiency of adenosine deaminase
c. Maternal and Fetal IgM d. Maternal and Fetal IgG b. Toxicity of leucocytes due to accumulation of cAMP
2. Combined T & B cell immunodeficiency includes: c. Both cellular and humoral immunity affected
d. Bone marrow transplantation is the treatment
a. DiGeorge syndrome (PGI Nov 2017)
b. Chronic granulomatous disease 12. All of the following are true except:
c. Adenosine deaminase deficiency  (AIIMS Nov 2010, May 2009)
d. Wiskott-Aldrich syndrome a. NADPH oxidase acts via superoxide ions
e. Ataxia telangiectasia b. Chediak-Higashi syndrome is due to defective phago­
3. True about SCID in children is: (PGI Jan 2017) lysosome formation
c. In Brutons agammaglobulinemia opsonisation is not
a. It is due to adenosine deaminase deficiency
affected
b. Humoral immunity is predominantly affected d. Myeloperoxidase action is mainly due to OCl-
c. Child should receive double dose of all vaccines
13. In genetic deficiency of MPO, the increased susceptibility
d. Patients with SCID have very small thymuses
to infection is due to:
e. Tonsils & adenoids are absent or underdeveloped
 (DNB June 2009, MAHA 2005)
4. Children with Chediak Higashi syndrome have light
a. Defective production of Prostaglandins
skin, silvery hair, solar sensitivity, photophobia, frequent
b. Defective rolling of' neutrophils
infections, neuropathy, prolonged bleeding times with
c. Inability to produce hydroxyl halide radicals
normal platelet count. What is the treatment which d. Inability to produce hydrogen peroxide
improves the clinical status of some children in the stable
14. Nitroblue tetrazolium test is used for? M 
phase? (Recent Question 2017)
 (AIIMS Nov 2008)
a. Vitamin B12 b. Vitamin B1
a. Phagocytes b. Complement
c. Vitamin C d. Vitamin E
c. T cell d. B cell
5. A 3-year-old child presents with recurrent pneumonia,
15. Lack of leucocyte adhesion molecules (LAM) is
eczema & thrombocytopenia. Which protein synthesis
associated with: (AIPGMEE 2008, 1998)
is abnormal in this child? (JIPMER Nov 2016)
a. Delayed fall of umbilical cord
a. Wasp b. Hamartin b. Normal chemotaxis
c. Adenosine deaminase d. HLA 1a c. Complement opsonization
6. All of the following are not seen in SCID except? d. Neutropenia
 (NEET Pattern 2016)
16. About Wiskott Aldrich not true is? (AIIMS May 2007)
a. Autoimmune ds b. Granuloma formation
a. Raised IgE b. Raised IgM
c. GVHD d. Graft rejection
c. Reduced IgA d. CD4 and CD8 defect
Chapter 7: Diseases of Immune System 143

17. Adenosine deaminase deficiency is seen in which of the 26. True about Henoch-Schönlein pupura: (PGI June 09)

Questions
following: (AIPGMEE 2005, 2001) a. Palpable purpura b. Thrombocytopenia
a. Common variable immunodeficiency c. C-ANCA positive d. Kidney commonly affected
b. Severe combined immunodeficiency e. Thrombocytosis
c. Chronic granulomatous disease 27. Henoch-Schönlein purpura is characterized by the depo­
d. Nezelof syndrome sition of the following immunoglobulin around the vessels: M
18. The commonest primary immunodeficiency is: M  a. IgM b. IgG (AIIMS Nov 05)
a. Common variable immunodeficiency (AIPGMEE 2004) c. IgA d. lgE
b. Isolated IgA immunodeficiency
c. Wiskott-Aldrich syndrome  KAWASAKI DISEASE
d. AIDS 28. Which of the following is not seen in adults? M 
19. Which one of the following statements is correct a. Kawasaki disease b. HSP (AIIMS Nov 2014)
regarding chronic granulomatous disease: c. Susac syndrome d. Takayasu arteritis
a. It is an autosomal dominant disease (AIIMS Nov 2004) 29. Most common vasculitis in children: M  (WBPG 2012)
b. It is characterized by abnormal bacterial phagocytosis
a. Kawasaki disease b. Takayasu disease
c. Recurrent streptococcal infections are usual in this disease
d. Nitroblue tetrazolium test is useful for screening c. HSP d. Microscopic PAN
20. Which is found in DiGeorge's syndrome: 30. All the following are indicated in the long-term therapy
of children with coronary abnormalities in Kawasaki
a. Tetany b. Eczema
disease except: (JIPMER 2009)
c. Mucocutaneous candidiasis (PGI June 2001)
d. Absent B and T cells e. Total absence of T cells a. Intravenous immunoglobulins
b. Aspirin
 HENOCH-SCHÖNLEIN PURPURA c. Clopidogrel d. Warfarin
31. Kawasaki disease is associated with all except: M 
21. A 6-year-old child presented with abdominal pain and a. Erythema (AIPGMEE 96, AIIMS May 2006)
red, elevated skin rashes, mainly involving the lower b. Posterior cervical lymphadenopathy
limbs, as shown in the given image: c. Thrombocytopenia d. Conjunctivitis
32. Kawasaki disease is associated with all except: M 
 (AIIMS Nov 1998)
a. Thrombocytopenia b. Cervical lymphadenopathy
c. Red eye d. Rash

 OTHER CONNECTIVE TISSUE DISEASES

33. Small vessel vasculitis includes: (PGI Nov 2017)


a. Anti GBM antibody disease
b. Takayasu disease c. Kawasaki disease
d. Systemic Lupus erythematosus
e. IgA vasculitis
What is the probable diagnosis? (NEET PG Jan 2019) 34. All are features of DiGerorge syndrome except: M 
a. Kawasaki disease  (JIPMER May 2017)
b. Henoch Schonlein purpura a. Recurrent infection b. Thymic hyperplasia
c. Varicella infection c. Impaired T cell function d. Hypocalcemic seizures
d. Meningococcemia 35. Most sensitive antibody for SLE: M 
22. All are seen in Henoch Schönlein purpura except: M   (Recent question 2016)
 (Recent Question 2017, 2016) a. Anti-nuclear b. Anti-ds DNA
a. Thrombocytopenia b. Glomerulonephritis c. Anti-Smith d. Anti-histone
c. Arthralgia d. Abdominal pain 36. Joint erosions are typically absent in: M 
23. Henoch-Schönlein purpura is characterized by the
a. Rheumatoid arthritis (Recent Question 2016)
following except? M  (APPG 2015)
b. Systemic lupus erythematosus
a. Glomerulonephritis b. Hematochezia c. Psoriatic arthritis d. Osteoarthritis
c. Thrombocytopenia d. Palpable purpura
37. In which of the following c-ANCA is seen?
24. Frequency of renal involvement in Henoch Schönlein
a. Wegener's granulomatosis (Recent Question 2016)
purpura is? (Recent Question 2015)
b. Polyarteritis nodosa
a. 1-5% b. 5-10% c. Churg-Strauss syndrome d. Bechet's syndrome
c. 10-50% d. 50-70%
38. A 9-year-girl presents with difficulty in combing,
25. About Henoch-Schönlein purpura, all true except: M  climbing stairs and erythematous lesion at MCP joint.
a. Thrombocytopenia b. Hematuria (WBPG 2010) Gower sign is positive. What is the next step for
c. Palpable purpura d. Intusussception diagnosis? (AIIMS Nov 2015)
144 Section 1: General Pediatrics

51. Which is associated with vasculitis of medium size


Review of Pediatrics and Neonatology

a. CK leve b. Rheumatoid factor


c. Electromyography d. Nerve conduction studies vessels: (Recent Question 2014)
39. Most specific autoantibodies found in Systemic Lupus a. Temporal arteritis
Erythematosus are? (MAHA PGM CET 2015) b. Wegener's granulomatosis
a. Anti-RO b. Anti-Sm c. Classic PAN
c. Anti-RNP d. Anti dsDNA d. Tuberous sclerosis
40. Gottron's papules are found on? 52. In Wegener's granulomatosis cytoplasmic anti
 (Recent Question 2015) neutrophilic antibodies are directed against:
 (Recent Question 2014)
a. Face b. Trunk
c. Spine d. Knee a. Proteinase 1 b. Proteinase 2
41. Class IV lupus nephritis is: (Recent Question 2015) c. Proteinase 3 d. Proteinase 4
a. Mesangial lupus nephritis 53. Characteristic feature of Goodpasture's syndrome?
b. Proliferative lupus nephritis  (APPG 2014)
c. Membranous lupus nephritis a. Lumpy-bumpy deposits on immunofluorescence
d. Diffuse lupus nephritis b. Serum antibodies against alpha 3 NC 1 domain of collagen–
42. Commonest renal lesion in SLE is: IV
 (Recent Question 2015) c. Serum antibodies against alpha 1 NC 1 domain of
a. Focal proliferative b. Diffuse proliferative collagen III
c. Membranous d. Minimal change disease d. Anti DNAse antibodies positive
43. Antibody seen in neonatal lupus with congenital heart 54. ANCA positive entities are? (PGI May 2014)
block: (Recent Question 2017, 2015) a. Polyarteritis nodosa b. Wegener's granulomatosis
a. Anti-ribosomal b. Anti-Ro (SS-A) c. Microscopic polyangitis d. Giant cell arteritis
c. Anti-neuronal d. Anti-histone e. Churg-Strauss syndrome
44. Anti-histone antibodies are specific for: 55. In Wegener's glomerulonephritis characteristic feature
 (Recent question 2015) seen is: (AIIMS Nov 10, Nov 09)
a. CNS lupus b. Drug induced lupus a. Granuloma in the vessel wall
c. Neonatal lupus d. Cutaneous lupus b. Focal necrotizing glomerulonephritis
45. False statement regarding musculoskeletal manifes­ c. Nodular glomerulosclerosis
tations of SLE: (Recent Question 2015) d. Interstitial granuloma
a. Intermittent polyarthritis 56. ANCA positive is: (AIIMS Nov 09)
b. Commonly involves hands, wrist, knees
a. Goodpasture syndrome b. Wegener's granulomatosis
c. Erosions seen on X-rays
c. Sjogren syndrome d. Relapsing polychondritis
d. Ischemic necrosis of bone can occur, when the patient is
on glucocorticoids 57. ANCA positive vasculitis are: (PGI June 09)
46. Silk Road disease is: (Recent Question 2015) a. Churg-Strauss syndrome b. Polyarteritis nodosa
a. Behcet's syndrome b. Giant cell arteritis c. Wegener granulomatosis d. Takayasu arteritis
c. Henoch-Schonlein purpura e. Henoch-Schönlein purpura
d. Wegener's granulomatosis 58. All of the following condition(s) are associated with
47. Which is not a characteristic of Wegener's granulo­ raised ANCA except:  (DNB June 09)
matosis: (Recent Question 2015) a. Wegener's granulomatosis
a. Granuloma in vessel wall b. Polyarteritis nodosa
b. Focal necrotising glomerulonephritis c. Microscopic polyangiitis
c. Positive for cANCA d. Involves large vessels d. Churg-Strauss syndrome
48. 13-year-old female child presented with recurrent 59. Large vessel vasculitis is/are:  (PGI Dec 07)
sinusitis fever, arthralgia, respiratory distress, Hematuria a. Takayasu disease b. Temporal arteritis
and hypertension. Renal biopsy showed necrotizing c. PAN d. Giant cell arteritis
granuloma. The anti proteinase-3 ANCA was positive. e. Wegener granulomatosis
The most likely diagnosis is: (MAHA PGM CET 2014) 60. Wegener's granulomatosis involve:
a. Polyarteritis nodosa b. Wegener's granulomatosis  (PGI Dec 2006; PGI Dec 2003)
c. Microscopic polyangiitis d. Churg-Strauss syndrome a. Lung b. Liver
49. Acute febrile illness with vasculitis having predilection c. Kidney d. Upper respiratory tract
for the coronary arteries is seen in: M 
e. Heart
 (MAHA PGM CET 2014)
61. Lung is characteristically involved in: (PGI June 2001)
a. Kawasaki disease b. Adenovirus infection
c. Diphtheria d. Measles a. Churg-Strauss syndrome b. HSP
50. ANCA positive vasculitis: M  (Recent question 2014)
c. PAN d. ITP
62. Small vessel vasculitis is/are: (PGI Dec 2000)
a. Henoch-Schönlein purpura
b. Behcet's syndrome a. Classical PAN b. Wegener's granulomatosis
c. Wegener's granulomatosis c. Giant cell arteritis d. HSP
d. None e. Churg-Strauss syndrome
Chapter 7: Diseases of Immune System 145

Answers with Explanations


Answers with Explanations
 PRIMARY IMMUNODEFICIENCY DISEASES
1. d. Maternal and Fetal IgG  Ref: Nelson’s 20/e p 1011-1012

• A small amount of IgM is present from 20th week onwards


• Rapid synthesis of IgG starts within 1st few weeks of postnatal life
• Final adult values of all subclasses of IgG is usually reached between 8-10 years age
• Last class of Ig to reach adult serum levels is IgA
2. c. Adenosine deaminase deficiency, d. Wiskott-Aldrich syndrome & e. Ataxia telangiectasia
3. a. It is due to adenosine deaminase deficiency, d. Patients with SCID have very small thymuses; e. Tonsils & adenoids are
absent or underdeveloped  Ref: Nelson's 20/e p 1022-1024; Refer pretext of this chapter for details
SCID is a characteristic example of combined antibody and cellular defects; Live vaccines are contraindicated in SCID
4. c. Vitamin C  Ref: Nelson's 20/e p 1045
High-dose ascorbic acid or Vitamin C (200 mg/day for infants, 2,000 mg/day for adults) may improve clinical status of some children
in stable phase of Chediak Higashi disease
5. a. Wasp  Ref: Nelson's 20/e p 1027
Recurrent infections, eczema & thrombocytopenia in a child suggest a diagnosis of Wiskott Aldrich syndrome; In this
X-linked recessive syndrome, WASP gene & its product are defective;
6. c. GVHD  Ref: Nelson's 20/e p 1022-1024, Ghai 8/e p 185; Refer to pretext for details;
SCID (Severe Combined Immunodeficiency)
•• It is caused by diverse genetic mutations that lead to absence of all adaptive immune function
•• Combined deficiency of B & T cells seen; so Autoimmune ds, Granuloma formation & Graft rejection are absent
•• While, GVHD which is due to immune-competent cells attacking a immunosuppressed host can still be seen

7.
a. C1 inhibitor  Ref: Nelson's 20/e p 1058, Ghai 8/e p 187
Hereditary angioneurotic edema occurs due to deficient or defective C1 inhibitor (autosomal dominant).
8.
a. Leucocyte adhesion deficiency  Ref: Nelson's 20/e p999-1001, Ghai 8/e p 187
Characteristic Clinical Patterns in important Primary Immunodeficiencies:
Diagnosis Features
In newborns and young infants (0 to 6 months)
DiGeorge anomaly HypocalcemiaQ, heart disease, unusual facies
Leukocyte adhesion defect Delayed umbilical cord detachment, leukocytosis, recurrent infections
Severe combined immunodeficiency Diarrhea, pneumonia, thrush, failure to thrive
Wiskott-Aldrich syndrome Bloody stools, draining ears, eczema
Hyper IgM syndrome Mouth ulcers, neutropenia, recurrent infections
Contd...
146 Section 1: General Pediatrics

Contd...
Review of Pediatrics and Neonatology

In infancy and young children (6 months to 5 years)


Hyper-IgE syndrome Recurrent cutaneous and/or systemic staphylococcal abscesses, coarse facies
Chronic mucocutaneous candidiasis Persistent thrush, nail dystrophy, endocrinopathies
Chédiak-Higashi syndrome Oculocutaneous albinism, recurrent infection
Chronic granulomatous disease Lymphadenopathy, dermatitis, pneumonia, osteomyelitis
In older children (older than 5 years) and adults
X-linked agammaglobulinemia Progressive dermatomyositis with chronic enterovirus encephalitis
Ataxia-telangiectasia Sinopulmonary infections, neurologic deterioration, telangiectasia
C6, C7, or C8 deficiency Recurrent neisserial meningitis
Common variable immunodeficiency Sinopulmonary infections, malabsorption, splenomegaly, autoimmunity

9.
a. SCID, b. Common Variable immunodeficiency  Ref: Ghai 8/e p 185
Common variable Immunodeficiency is predominantly a B cell defect, but it also shows defect in T cell activation.
10. a. Neutrophilia  Ref: Ghai 8/e p 187
Main leukocyte abnormalities in Chediak-Higashi syndrome are:
•• Neutropenia (most common); Leukocytes contain giant granules,Q (characteristic) due to aberrant phagolysosome fusionQ
•• Defective degranulation, Impaired chemotaxis, Delayed microbial killing; NK cell function is also impaired

11. b. Toxicity of leucocytes due to accumulation of cAMP  Ref: Nelson's 20/e p 1022-1024; Refer pretext;
In SCID, deficiency of ADA leads to accumulation of deoxyadenosine and its derivatives (e.g., deoxy-ATP) (not AMP), which
are toxic to rapidly dividing immature lymphocytes, especially those of the T-cell lineage.
12. c. In Brutons agammaglobulinemia opsonization is not affected  Ref: Nelson's 20/e p 1012-1016, Ghai 8/e p 186
IgM is important for opsonization, so in Brutons agammaglobulinemia opsonisation is affected.
13. c. Inability to produce hydroxyl halide radicals  Ref: Nelson's 20/e p 1027, Ghai 8/e p 187
In MPO deficiency there is decreased microbial killing because of defective MPO—H2O2 system, so there is an inability to
produce hydroxyl halide radicals.
14. a. Phagocytes  Ref: Nelson's 20/e p 1027, Ghai 8/e p 187
Nitroblue-tetrazolium (NBT) test tells whether or not NADPH oxidase enzymes are present in phagocytes.
15. a. Delayed fall of umbilical cord Ref: Nelson's 20/e p 1027, Ghai 8/e p 187
Leukocyte adhesion deficiency (LAD)

Salient features LAD type 1 LAD type 2


Inheritance Autosomal recessive Autosomal recessive

Basic defect Defect in CD 18 moleculeQrequired for biosynthesis of Absence of Sialyl-Lewis X, the fucose-containing
B2 chain shared by LFA-1 & Mac-1 integrins ligand for P & E-selection, due to defect in fucosyl-
transferase responsible for binding fucose moieties
to protein backbones

Clinical features Recurrent bacterial infections, delayed separation of Recurrent bacterial infections, Bombay blood
umbilical cord stumpQ groupQ and mental retardation

16. c. Reduced IgA  Ref: Nelson's 20/e p 1027, Ghai 8/e p 186; Refer pretext of this chapter for details;
Remember IgE & IgA are ElevAted in Wiskott Aldrich syndrome.
17. b. Severe combined immunodeficiency  Ref: Nelson's 20/e p 1022, Ghai 8/e p 187
18. b. Isolated IgA immunodeficiency  Ref: Nelson's 20/e p 1015, Ghai 8/e p 187
Isolated IgA Deficiency:

•• Isolated IgA deficiency is the commonest immunodeficiency (1/600)
•• Affected individuals have extremely low levels of both serum and secretory IgA
•• It may be familial, or acquired in association with toxoplasmosis, measles, or some other viral infection

18. d. Nitroblue tetrazolium test is useful for screening  Ref: Nelson's 20/e p 1027, Ghai 8/e p 186;
20. a. Tetany, c. Mucocutaneous candidiasis, e. Total absence of T cells.  Ref: Nelson's 20/e p 1019-1020
Chapter 7: Diseases of Immune System 147

Answers with Explanations


Disease Primary defect Secondary defect
DiGeorge Syndrome Familial cases: •• T-cell defect
(Thymic Hypoplasia)Q failure of development of 3rd & 4th pharyn- •• TetanyQ
geal pouchesQ •• Congenital defects of heart and great vessels
Non-familial cases: 22q11 deletion •• Mucocutaneous Candidiasis

 HENOCH-SCHÖNLEIN PURPURA
21. b. Henoch Schonlein purpura  Ref: Nelson’s 20/e p 1216-1217
Abdominal pain and palpable purpura, mainly involving the lower limbs, suggests a diagnosis of Henoch Schonlein purpura.
22. a. Thrombocytopenia  Ref: Nelson's 20/e p 1216-1217, Ghai 8/e p 632-633
Palpable purpura without thrombocytopenia is seen in Henoch Schönlein purpura (HSP).
23. c. Thrombocytopenia  Ref: Nelson's 20/e p 1216-1217, Ghai 8/e p 632-633
Kidney involvement (glomerulonephritis) & bowel involvement (blood in stools, abdominal pain) can be seen in HSP.
24. c. 10-50% Ref: Nelson's 20/e p 1216-1217, Ghai 8/e p 632-633
Renal involvement in HSP:
•• It occurs in up to 50% of children with HSP
•• Manifests as microscopic hematuria, proteinuria, hypertension, frank nephritis, nephrotic syndrome, and acute or chronic renal failure;
Progression to end-stage renal disease is uncommon in children (1-2%)
25. a. Thrombocytopenia  Ref: Nelson's 20/e p 1216-1217, Ghai 8/e p 632-633
26. a. Palpable purpura, d. Kidney commonly affected, e. Thrombocytosis Ref: Nelson's 20/e p, p 1216-1217
27. c. IgA Ref: Nelson's 20/e p 1216-1217, Ghai 8/e p 632-633
Biopsy finding in HSP is deposition of IgA, specifically IgA1 in the affected tissue, by immunofluorescence, accompanied to a
lesser extent by deposition of C3, fibrin & IgM.

 KAWASAKI DISEASE
28. a. Kawasaki disease Ref: Nelson's 20/e p 1216-1217, Ghai 8/e p 632-633
29. c. HSP Ref: Nelson's 20/e p 1216-1217, Ghai 8/e p 632-633
Henoch-Schönlein purpura (HSP) is the most common vasculitis of childhood.
30. a. Intravenous immunoglobulins Ref: Nelson's 20/e p 1216-1217, Ghai 8/e p 632-633
Treatment of Kawasaki Disease

Acute stage •• Intravenous immunoglobulin 2 g/kg over 10-12 hr and


•• Aspirin 80-100 mg/kg/day divided every 6 hr orally until patient is afebrile for at least 48 hr

Convalescent stage •• Aspirin 3-5 mg/kg once daily until 6-8 wk after illness onset if normal coronary findings throughout

Long-term therapy for •• Aspirin 3-5 mg/kg once daily orally or Clopidogrel 1 mg/kg/day (maximum: 75 mg/day)
patients with coronary •• Warfarin or low-molecular-weight heparin added for patients at particularly high risk of thrombosis
abnormalities

Acute coronary throm- Prompt fibrinolytic therapy with tissue plasminogen activator or other thrombolytic agent
bosis

31. c. Thrombocytopenia Ref: Nelson's 20/e p 1216-1217, Ghai 8/e p 632-633

Mnemonic
Diagnostic Criteria for Kawasaki Disease
Fever persisting at least 5 days with presence of at least 4 features out of 5 (CREAM):
Conjunctivitis: Rash, Edema & erythema of palms & soles, Adenopathy and Mucosal involvement: Strawberry tongue

32. a. Thrombocytopenia  Ref: Nelson's 20/e p 1216-1217, Ghai 8/e p 632-633

 OTHER CONNECTIVE TISSUE DISEASES


33. a. Anti GBM antibody disease, d. Systemic lupus erythematosus & e. IgA vasculitis
Note: Anti GBM antibody disease (Goodpasture disease) & IgA vasculitis (Henoch-Schonlein purpura) are examples of small vessel
vasculitis
148 Section 1: General Pediatrics

34. b. Thymic hyperplasia  Ref: Nelson's 20/e p 1019-1020


Review of Pediatrics and Neonatology

Thymic hypoplasia is seen in DiGeorge syndrome.



35. a. Anti-nuclear  Ref: Nelson's 20/e p 1176-1180, Ghai 8/e p 628
Autoantibodies in Systemic Lupus Erythematosus (SLE)Q
Antibody Antigen Recognized Clinical Utility
Antinuclear antibodies Multiple nuclear Best screening test/Most sensitiveQ

Anti-dsDNA DNA (double-stranded) SpecificQ, correlate with disease activity, nephritis, vasculitis

Anti-Sm U1 RNA Most Specific for SLEQ

Anti-RNP U1 RNA Not specific for SLE

Anti-Ro/La (SS-A) hY RNA Sicca syndrome, neonatal lupus with congenital heart blockQ

Antihistone Histones Drug-induced lupusQ



36. b. Systemic lupus erythematosus Ref: Nelson's 20/e p 1176-1180, Ghai 8/e p 628
Arthritis seen in SLE is Non-erosive arthritis (without deformity).
37. a. Wegener's granulomatosis Ref: Nelson's 20/e p 1176-1180, Ghai 8/e p 628
C-ANCA P-ANCA (against Myeloperoxidase)
Wegener's GranulomatosisQ •• Microscopic PAN (microscopic polyangitis) •• Crescentic GlomerulonephritisQ
(90-95%) •• Churg-Strauss syndromeQ •• Goodpasteur's syndromeQ
•• Drugs (hydralazine, propythiouracil) •• Ulcerative colitis
•• Primary sclerosing cholangitisQ

38. c. Electromyography Ref: Nelson's 20/e 1182, Ghai 8/e p 629; Refer pretext of this chapter;
This girl is probably suffering from Juvenile Dermatomyositis, as she has proximal muscle weakness & skin lesions suggestive of
Gottron's papules. Discussing about the options one by one,
a. CK level Creatine kinase level is usually elevated, but may be normal on initial presentation
b. Rheumatoid factor It is negative in JDM
c. Electromyography Shows signs of myopathy as well as muscle fiber necrosis

d. Nerve conduction studies Typically normal unless severe muscle necrosis and atrophy are present

39. b. Anti-Sm Ref: Nelson's 20/e p 1176-1180, Ghai 8/e p 628; Refer Ans 30 above
40. d. Knee Ref: Nelson's 20/e p, Ghai 8/e p
Classic Gottron papules are bright pink or pale, shiny, thickened or atrophic plaques over the proximal interphalangeal joints
& distal interphalangeal joints and occasionally on the knees, elbows, small joints of the toes, and ankle malleoli.
41. d. Diffuse lupus nephritis Ref: Nelson's 20/e p 1176-1180, Ghai 8/e p 628
Classification of Lupus Nephritis (International Society of Nephrology)
Class I: Minimal Mesangial Lupus Nephritis (least common) Class IV: Diffuse Lupus Nephritis (most common)
Class II: Mesangial Proliferative Lupus Nephritis Class V: Membranous Lupus NephritisQ
Class III: Focal Lupus NephritisQ Class VI: Advanced Sclerotic Lupus Nephritis

42. b. Diffuse proliferative Ref: Nelson's 20/e p 1176-1180, Ghai 8/e p 628
Class I is the Least common & class IV is the most common pattern of lupus nephritis.
43. b. Anti-Ro (SS-A) Ref: Nelson's 20/e p 1176-1180, Ghai 8/e p 628
Anti-Ro/La (SS-A against hY RNA is seen in Sicca syndrome, neonatal lupus with congenital heart block.
44. b. Drug induced lupus Ref: Nelson's 20/e p 1176-1180, Ghai 8/e p 628
Anti histone antibodies are directed against Histones & are seen in Drug-induced lupus.
45. c. Erosions seen on X-rays Ref: Nelson's 20/e p 1176-1180, Ghai 8/e p 628
46. a. Behcet's syndrome Ref: Nelson's 20/e p 1190-1191, Ghai 8/e p 631-635
•• Behçet disease is characterized by recurrent oral ulcerations, uveitis & skin abnormalities
•• It has a high prevalence in countries along the Silk Road, extending from Japan to the eastern Mediterranean.

47. d. Involves large vessels  Ref: Nelson's 20/e p 1221-1223, Ghai 8/e p 631-635
Wegener's granulomatosis involves small vessels
Chapter 7: Diseases of Immune System 149

48. b. Wegener's Granulomatosis  Ref: Nelson's 20/e p 1221-1223, Ghai 8/e p 631-635

Answers with Explanations


Respiratory & Renal involvement along with positive cANCA (Anti Proteinase 3) indicates Wegener's Granulomatosis
49. a. Kawasaki disease  Ref: Nelson's 20/e p 1221-1223, Ghai 8/e p 631-635
Coronary artery aneurysm is an important complication of Kawasaki disease
50. c. Wegener's granulomatosis Ref: Nelson's 20/e p 1221-1223, Ghai 8/e p 631-635
51. c. Classic PAN Ref: Nelson's 20/e p 1221-1223, Ghai 8/e p 631-635
Mnemonic: ‘Milk is Poor in vit K': Medium size vessel vasculitis are Polyarteritis nodosa & Kawasaki disease
52. c. Proteinase 3 Ref: Nelson's 20/e p 1221-1223, Ghai 8/e p 631-635
53. b. Serum antibodies against alpha 3 NC1 domain of collagen – IV  Ref: Nelson's 20/e p 1221-1223
•• In Goodpasture syndrome, type 1 RPGN & type II hypersensitivity reaction are seen.
•• It is characterized by serum antibodies against alpha 3 NC1 domain of collagen – IV.
54. b. Wegener's granulomatosis, c. Microscopic polyangiitis, e. Churg-Strauss syndrome  Ref: Nelson's 20/e p 1221-1223
55. b. Focal necrotizing glomerulonephritis Ref: Nelson's 20/e p 1221-1223, Ghai 8/e p 631-635
Note: Granuloma in vessel wall is seen in Wegener's Granulomatosis, but presence of granuloma alone is not characteristic of
Wegener's; It should be granulomatous necrotizing vasculitis.
56. b. Wegener's granulomatosis Ref: Nelson's 20/e p 1221-1223, Ghai 8/e p 631-635
57. a. Churg-Strauss syndrome, c. Wegener granulomatosis  Ref: Nelson's 20/e p 1221-1223, Ghai 8/e p 631-635
58. b. Polyarteritis Nodosa  Ref: Nelson's 20/e p 1221-1223, Ghai 8/e p 631-635
59. a. Takayasu disease, b. Temporal arteritis, d. Giant cell arteritis  Ref: Nelson's 20/e p 1221-1223
60. a. Lung, c. Kidney, d. Upper respiratory tract  Ref: Nelson's 20/e p 1221-1223, Ghai 8/e p 631-635
61. a. Churg-Strauss syndrome  Ref: Nelson's 20/e p 1221-1223, Ghai 8/e p 631-635
Churg Strauss syndrome: It is a small vessel necrotizing granulomatous vasculitis associated with a history of refractory asthma &
peripheral eosinophilia.
62. b. Wegener's granulomatosis, d. HSP, e. Churg-Strauss syndrome  Ref: Nelson's 20/e p 1221-1223, Ghai 8/e p 631-635
Chapter 8
Infectious Diseases
 A. IMPORTANT VIRAL DISEASES
Mnemonic M

Days on which rash appear in differ- 1. RUBELLA (GERMAN MEASLES OR 3-DAY MEASLES) M
ent diseases:
“Very Sick People Must Take Daily
Mild, exanthematous disease of infants and children.
Exercise” Causative agent: Rubella virus, a member of the family Togaviridae, ss RNA virus
Day 1: V—Varicella Pathogenesis: Following infection, the virus replicates in the respiratory epithelium, then
Day 2: S—Scarlet fever spreads to regional lymph nodes
Day 3: P—Pox (Smallpox) Incubation period: 14–21 days
Day 4: M—Measles Period of communicability: 5 days before to 6 days after appearance of rash.
Day 5: T—Typhus
Clinical Manifestations
Day 6: D—Dengue
Day 7: E—Enteric fever
•• Prodrome of low grade fever, sore throat, red eyes, headache & lymphadenopathy
•• Rash begins on face & neck & spreads centrifugally to involve body & extremities
•• Forchheimer spotsQ (tiny, rose colored lesions) on soft palate may be present.

Congenital Rubella
Frequency of congenital infection & risk of congenital defects after maternal Rubella
with a rash is related to the gestational age at the time of infection

Gestational age Risk of congenital defects Frequency of congenital infections


High Yield Points
Before 11 weeks 90% 70–80%
•• Fetuses infected with Rubella after 16 11–12 weeks 33% 70–80%
weeks suffer no major abnormality 13–14 weeks 11% 30–54%
15–16 weeks 24% 10–25%
Clinical Features:
•• Hearing loss; Heart defects IUGR (Intrauterine growth restriction)
•• Meningoencephalitis, Microcephaly • Brain Calcification
•• Salt & pepper retinopathy, Glaucoma • Interstitial pneumonitis
•• Hepatosplenomegaly • Progressive Rubella panencephalitis.
•• Thrombocytopenia (blue berry muffin lesions)
Diagnosis of Congenital Rubella M
Question 1
Presence of IgM rubella antibodies in the infant shortly after birth or persistence of IgG
A mother had fever and rash in antibodies for > 6 months by which time maternally derived antibodies would have
the antenatal period. The newborn disappeared, indicate congenital infection.
delivered had PDA and deafness.
Ophthalmoscopy revealed the The hallmark of fetal Rubella infection is chronic infection that persists throughout fetal life,
follo­wing appearance: with shedding of virus up to 1 year of age.
Late-onset manifestations of CRS are:
•• Progressive rubella panencephalitis (PRP)
•• Diabetes mellitus (20%)
•• Thyroid dysfunction (5%)
•• Glaucoma & visual abnormalities

High Yield Points


a. Congenital syphilis
b. Congenital toxoplasmosis •• Most common finding in Conge­nital Rubella Syndrome is Nerve deafness.
c. Congenital tuberculosis •• Triad of congenital Rubella syn­drome includes deafness, cataracts, and congenital heart disease.
d. Congenital rubella •• Most common cardiac lesion in congenital rubella syndrome is PDAQ > pulmonary stenosis > VSD > ASD
Chapter 8: Infectious Diseases 151

2. MEASLES

Section 1: General Pediatrics


M
Question 2
Causative agent: SS RNA virus in the family Paramyxoviridae & genus Morbilli-virus In which disease a generalized
Incubation period: 8–12 days papular rash, fever, conjunctivitis
& lesion on buccal mucosa shown
Period of communicability: 3 days before to up to 4–6 days after the onset of rash. below are seen?
Pathology: Fusion of infected cells result in multinucleated giant cells, Warthin–Finkeldey
giant cells (pathognomic for measles).
Clinical Manifestations
•• Prodromal phase begins with mild fever followed by conjunctivitis, coryza & cough
•• Koplik spots appear as discrete red lesions with bluish white spots in the centre, on the
inner aspects of cheeks at the level of 1st & 2nd molar
•• Rash begins on forehead behind the ears & spreads downward to torso & extremities
•• Measles rash fades in about 7 days in the same progression as it evolved
a. Mumps b. Measles
c. Chickenpox d. Rubella
High Yield Points
•• Koplik spots are pathognomic sign of measles
•• Koplik spots appear 1–4 days prior to onset of rash in measles
•• MC cause of death in Measles is Pneumonia.
•• May occur in conjunctival folds and in the vaginal mucosa. Koplik spots have been reported in 50–70%
of measles cases

Complications
•• Giant cell pneumonia caused directly by viral infection or superimposed bacterial infection
(most commonly by Streptococcus pneumoniae, H. influenzae, Staph aureus).
•• Acute Otitis media, Croup, tracheitis, bronchiolitis
•• Flaring up of Mycobacterium tuberculosis infection
•• Encephalitis, febrile seizures, subacute Sclerosing Panencephalitis

Treatment: Supportive

L at e s t U p d at e s
•• Initial targets for measles virus are alveolar macrophages, dendritic cells & lymphocytes
•• Receptor used is signaling lymphocyte activating molecule (CD150)
•• The mechanism of infection of respiratory tissues is attachment to the PVRL4 receptor (Nectin4) that is
expressed on cells in the trachea, oral mucosa, nasopharynx, and lungs.
•• These 2 receptors, CD150 and PVRL4, account for the lymphotropic & epitheliotropic nature of natural
measles virus infection.

Subacute Sclerosing Panencephalitis

•• Rare complication of measles; Begins 7–13 years after primary measles infection.Q
•• Results from a persistent infection with an altered measles virus that is harbored
intracellularly in the CNS for several years.
Clinical Features
•• 1st stage: Subtle changes in behavior or school performances appear (EEG normal)
•• 2nd stage: Massive myoclonus but consciousness is maintained Question 3
•• 3rd stage: Choreoathetosis, dystonia, lead pipe rigidity & altered sensorium Infection by which virus gives rise
•• 4th stage: Loss of critical centers that support breathing, heart rate & BP to the following skin lesions?

Diagnosis
Compatible clinical course and at least 1 of the following: (i) Measles antibody in CSF or
(ii) EEG findings (in myoclonic phase, burst suppression episodes).

3. CHICKENPOX (VARICELLA) M

Causative Agent
Varicella zoster virus (VZV), a DNA virus of the herpes virus family. a. Herpes virus
b. Measles virus
Mode of Infection c. Varicella zoster virus
Airborne or through direct contact with skin. d. Parvovirus
152 Section 1: General Pediatrics

Period of Infectivity
Review of Pediatrics and Neonatology

Question 4
The mother of this newborn From 24–48 hours before the rash until all the vesicles are crusted.
had fever with rash at 18 weeks
of gestation. What is this baby Secondary Attack Rate
probably suffering from? 80%, so this disease is highly contagious.

Clinical Features
•• Rash appears 24–48 hrs after prodromal symptoms rapidly spreads to face & extremities.
•• Evolves into papules, clear fluid filled vesicles and then crusted vesicles.
•• Characteristic: Simultaneous presence of skin lesions in varying stages of evolution.

Complications
•• Secondary bacterial infections, Purpura fulminans (due to Ab against protein C)
a. Congenital varicella •• Meningoencephalitis, cerebellar ataxia, transverse myelitis, Optic neuritis
b. Congenital rubella •• Stroke (due to CNS vasculitis), Reye syndrome.
c. Congenital CMV
d. Congenital toxoplasmosis Congenital Varicella Syndrome
•• Seen in 0.4% of babies infected in 1st trimester & 2% in 2nd trimester.
•• Characterized by cicatricial skin scarring in a zoster-like distribution; limb hypoplasia;
low birthweight, CNS abnormalities (microcephaly, seizures & mental retardation), eye
(chorioretinitis, microphthalmia, & cataracts), renal system hydronephrosis), and autonomic
system (neurogenic bladder, swallowing dysfunction).
•• Viral DNA may be detected in tissue samples by PCR & VZV-specific IgM is detectable in
cord blood sample in some infants.

Herpes Zoster
•• Characterized by a mild vesicular rash with dermatomal distributionQ
•• Unlike adults, pain is less and post herpetic neuralgia unusual.
Diagnosis: Tzank smear (showing multinucleated cells) & serology can be used.
Treatment: Symptomatic; IV acyclovir (10 mg/kg × 8 hourly × 14–21 days) is given to patients
with complicated varicella and high-risk patients (neonates, immunocompromised children).

 FEVER WITH RASH IN CHILDREN


Question 5
Identify the disease in which this High Yield Points
rash is seen:
Disease Imp feature
Infectious mononucleosis Ampicillin increases rash
Measles Koplik’s spots
Roseola infantum Rash appears when fever gone
Erythema infectiosum 'Slapped face' appearance
Typhoid Evanescent rash
a. Chickenpox b. Herpes simplex Endemic/epidemic typhus Rash appears on trunk & extends to extremities
c. Herpes zoster d. Measles Rocky mountain spotted fever Rash appears on extremities and extends to trunk

A. Diseases with Maculopapular Lesions M

Incub.
Disease period Typical distribution Clinical features
Measles (Rubeola) 10–14 days Begins at hairline and descends Prodrome f/b rash that des­
downward, becomes confluent. quamates after 7–10 days
Rubella (German 14–21 days Begins on face, generalized by Post auricular, suboccipital
measles) 2nd day, fades within 3 days lym­phadenopathy

Contd...
Chapter 8: Infectious Diseases 153

Contd...

Section 1: General Pediatrics


Incub.
Disease period Typical distribution Clinical features
Roseola (exanthema 10–14 days Non-pruritic, morbilli form rash Prodrome of high fever in
subitum, 6th disease) appears when fever subsides otherwise well child

Erythema infectiosum 7–14 days Red flushed cheeks with May have an aplastic crisis
(fifth disease) reticular, rash on extremities

Scarlet fever (group A 2–4 days Sandpaper texture of rash that Pharyngitis, strawberry
streptococci) appears 1st on flexor surfaces tongue, palatal petechiae

B. Diseases with Vesicular/Pustular Lesions M

Disease Typical distribution Other clinical features


Varicella (chickenpox) Macules, vesicles, umbilicated pustules; Contagious until crusting of
(pleomorphic) involves mucosa lesions appear
Herpes Zoster Erythema f/b papules & pustules in 72 History of varicella present;
(shingles) hours; Crusts in 10–12 days Dermatome distribution
Hand, foot, mouth Palmar & plantar pustules have typical Fever, myalgia, herpangina,
disease (Cox sackie elliptical appearance rhinorrhea, diarrhea
virus A 16)
Impetigo Honey colored crusted lesions, mainly Post streptococcal glomerulo-
on face & exposed areas nephritis is a complication

4. MUMPS M

What is it? Mumps is an acute self-limited infection characterized by fever, bilateral or


Question 6
unilateral parotid swelling and tenderness. Identify the disease this child is
suffering from:
Causative Agent Mumps virus is an RNA virus in family Paramyxoviridae & genus Rubulavirus.
Mode of Spread Mumps is spread from person to person by respiratory droplets.
Period of From 7 days before to 7 days after onset of parotid swelling.
Communicability Incubation period for mumps ranges from 12–25 days
Pathogenesis Following infection, initial viral replication in epithelium of upper respiratory tract.

Infection spreads to the adjacent lymph nodes & viremia occurs

Virus spreads to salivary glands, central nervous system (CNS), pancreas, testes
Clinical Features Prodromal period (1–2 days), consists of fever, headache, vomiting, & achiness. a. Mumps b. Measles
c. Rubella d. Syphilis
Parotitis which may be unilateral initially but becomes bilateral in 70%.
Parotid gland is tender & is accompanied by ear pain on the ipsilateral side.
In adolescent males, orchitis is 2nd only to parotitis as a common finding
Complications Most common complications of mumps are meningitis, & gonadal involvement.
Uncommon complications include conjunctivitis, optic neuritis, pneumonia,
nephritis, pancreatitis, and thrombocytopenia.

5. CYTOMEGALOVIRUS (CMV)
Causative Agent: Member of Herpes virus family.
Mode of Spread: Requires very close or intimate contact because it is very labile (via saliva,
breast milk, cervical and vaginal secretions, urine, semen, tears, blood products).

Clinical Features
•• In immunocompetent adults: Characterized by a mononucleosis–like syndrome
•• In immunosuppressed: CMV pneumonitis, retinitis & gastrointestinal disease are common.
154 Section 1: General Pediatrics

 CONGENITAL CMV INFECTION


Review of Pediatrics and Neonatology

•• 90% infected infants have subclinical, chronic infection


•• 5% have mild involvement; 5% have severe CMV inclusion disease
Clinical Signs and Symptoms: IUGR, prematurity, hepatosplenomegaly, micro­cephaly,
thrombocytopenia, intracranial calcifications, chorioretinitis, sensorineural hearing loss.

Diagnosis
Question 7 •• Virus isolation from urine, saliva, bronchoalveolar washings, breast milk
A 10-year-old child had fever •• The diagnosis of congenital CMV infection is best confirmed by isolation of the virus from
for 5 days, along with which he urine
developed multiple fluid filled •• Seroconversion or simultaneous detection of IgM & IgG antibodies with low avidity.
lesions on the lips as shown below.
What is the probable underlying Treatment
etiology for the skin lesions? •• Immunocompromised: Ganciclovir with immunoglobulin for life-threatening CMV
infections. Oral valganciclovir and foscarnet are alternative agents
•• Congenital infection: Ganciclovir X 6 weeks prevents hearing deterioration
•• CMV is the most common cause of congenital infection in neonates.

High Yield Points


•• Risk for fetal infection is greatest with maternal primary CMV infection (30%) and much less likely with
recurrent infection (<1%).
a. Cytomegalovirus
•• Perinatal transmission is common via genital tract secretions at deli­very (40%) & breast milk (6–12%).
b. Herpes simplex virus
c. HIV •• CMV is the most common cause of congenital infection in neonates.
d. Syphilis

6. HERPES SIMPLEX VIRUS (HSV) M

2 Types: HSV1 usually causes oral & HSV2 genital infections.


High Yield Points
Mode of Transmission: Direct contact between mucocutaneous sufaces.
•• Most common site of Herpes
labialis is vermilion border of lip Clinical Manifestation
•• Herpes encephalitis is mostly caused
by HSV-I, except in the neonatal The hallmark of HSV infection is skin vesicles and shallow ulcers, which present as: Herpes
period, when HSVII is an important gingivostomatitis, Herpes labialis, HSV paronychia, genital ulcers, keratoconjunctivitis
cause •• CNS infections: HSV encephalitis (caused by HSV 1, generally involving the frontal and/or
temporal cortex, limbic system)
•• Immunocompromised persons: Mucositis (most common), esophagitis, tracheobronchitis,
pneumonitis.
Diagnosis: Isolation of virus or viral DNA detection by PCR.
Treatment: Drug of choice for Herpes infection is AcyclovirQ.

7. POLIOMYELITIS M

Causative Agent
•• Polioviruses are non-enveloped, RNA viruses of Picornaviridae family
•• Consist of 3 antigenically distinct serotypes: types 1, 2, and 3.
Pathophysiology and Clinical Features
•• Polioviruses spread from GIT to CNS, where they cause aseptic meningitis & poliomyelitis
•• 90–95% of infections are inapparent but induce protective immunity
•• Clinically apparent but non-paralytic influenza-like illness occurs in 5% of all infections,
known as abortive poliomyelitis
High Yield Points •• Paralytic polio occurs in 1 in 1,000 infections in infants to 1 in 100 infections in adolescents.
•• Vaccine strains of polioviruses do not
Diagnosis
replicate in the CNS
•• Occasionally the vaccine strains de­ •• WHO recommends that diagnosis of poliomyelitis be confirmed by isolation and
velop into a neurovirulent phenotype identification of wild or vaccine type poliovirus in the stool
and cause vaccine-associated para- •• In suspected cases of acute flaccid paralysis, 2 stool specimens should be collected 24–48
lytic poliomyelitis (VAPP) hr apart, as soon as possible after the diagnosis of poliomyelitis is suspected.
Chapter 8: Infectious Diseases 155

Treatment

Section 1: General Pediatrics


•• There is no specific antiviral treatment for poliomyelitis
•• Supportive & aimed at limiting progression of disease & preventing skeletal deformities.

High Yield Points


•• Humans are the only known reservoir for the polioviruses
•• Polio spreads by feco-oral route
•• Poliovirus has been isolated from feces for 2 weeks before paralysis to several weeks after the onset
of symptoms

8. HUMAN IMMUNODEFICIENCY VIRUS (HIV)


Causative Agent High Yield Points
HIV-1 & HIV-2 are members of Retroviridae family and belong to Lentivirus genus. •• Perinatal treatment of HIV-infected
pregnant women with antiretroviral
Transmission of Infection drugs has decreased the rate of fetal
•• Majority of childhood HIV are result of vertical transmission infection to <2%
•• The most important factor that
•• Vertical transmission of HIV can occur before (intrauterine), during (intrapartum), or after
influences maternal to child HIV
delivery (through breastfeeding) transmission is the level of maternal
•• Risk factors influencing rate of vertical transmission: maternal viral load at delivery, preterm viremia
delivery (<34 wk gestation), and low maternal antenatal CD4 count.

High Yield Points


•• Majority of in utero transmissions of HIV occur in late gestation
•• 20–30% of infected newborns are infected in utero
•• Breastfeeding is the least-common route of vertical transmission in industrialized nations
•• Breastfeeding is responsible for as much as 40% of perinatal infections in resource-limited countries.

Clinical Manifestations
•• In most infants, physical examination at birth is normal
•• Initial symptoms may be subtle, such as lymphadenopathy and hepatosplenomegaly, or
nonspecific, such as failure to thrive, chronic or recurrent diarrhea, respiratory symptoms, or
oral thrush and may be distinguishable only by their persistence.
WHO clinical staging of HIV in children:
High Yield Points
Clinical Stage 1, Asymptomatic Clinical Stage 4, Severe symptoms
Clinical manifestations found more
Asymptomatic Unexplained severe wasting, stunting or severe
commonly in children than adults
Persistent generalized lymphadenopathy malnutrition not responding to standard Rx
with HIV infection include :
Pneumocystis pneumonia
• Recurrent bacterial infections
Recurrent severe bacterial infections •• Chronic parotid swelling
Clinical Stage 2, Mild symptoms
Chronic herpes simplex infection •• Lymphocytic interstitial
Unexplained persistent hepatosplenomegaly
Esophageal candidiasis (or candidiasis of pneumonitis (LIP), and
Extensive wart Molluscum contagiosum trachea, bronchi or lungs) •• Early onset of progressive neuro­
Recurrent oral ulcerations Extrapulmonary tuberculosis logic deterioration.
Unexplained persistent parotid enlargement Kaposi’s sarcoma
Herpes zoster CMV retinitis
Recurrent or chronic upper respiratory infections CNS toxoplasmosis (after 1 month of life)
Extrapulmonary cryptococcosis
Clinical Stage 3, Advanced symptoms
HIV encephalopathy
Unexplained moderate malnutrition or wasting Disseminated mycosis (coccidiomycosis or
not adequately responding to standard therapy histoplasmosis)
Unexplained persistent diarrhea/Persistent fever Disseminated non-TB Mycobacterial infection
Persistent oral candidiasis (after 1st 8 wk of age) Chronic cryptosporidiosis/isosporiasis
Oral hairy leukoplakia Cerebral or B-cell non-Hodgkin’s lymphoma
Lymph node tuberculosis/Pulmonary TB Progressive multifocal leukoencephalopathy
Severe recurrent bacterial pneumonia Symptomatic HIV-associated nephropathy or
Symptomatic lymphoid interstitial pneumonitis cardiomyopathy
156 Section 1: General Pediatrics

Diagnosis
Review of Pediatrics and Neonatology

M
High Yield Points
IRIS—Immune reconstitution infla- • All infants born to HIV-infected mothers test antibody-positive at birth (& for 6–12
mmatory syndrome months, there after) because of passive transfer of maternal HIV antibody
•• Exacerbation in systemic or • Presence of IgA or IgM anti-HIV in the infant's circulation can indicate HIV infection,
respiratory symptoms, signs and because these Ig do not cross the placenta; but they are insensitive & nonspecific
lab or radiography manifestations of • In any child > 18 mo of age, demonstration of IgG antibody to HIV by ELISA & Western
TB after initiation of ART blot test establishes HIV infection
•• Usually occurs 1–3 months after • Breastfed infants should have antibody testing performed 12 wk following cessation of
initiation of ART
•• More common in patients with
breastfeeding
advanced immunosuppression and • HIV DNA or RNA PCR or HIV culture, are useful in young infants, allowing a definitive
extrapulmonary  TB diagnosis in most infected infants by 1–6 mo of age.
•• Symptomatic treatment and
glucocorticoids in severe cases
Treatment
•• The currently available therapy does not eradicate the virus and cure the patient.
•• It suppresses the virus for extended periods of time & changes the course of the disease
•• Anti Retroviral Drugs Include:
–– Nucleoside (or nucleotide) reverse transcriptase inhibitors (NRTIs), e.g. stavudine,
zidovudine, didanosine, lamivudine
–– Non-nucleoside reverse transcriptase inhibitors (NNRTIs), e.g. nevirapine, efavirenz,
etravirine, rilpivirine
–– Protease inhibitors, e.g. Indinavir, Ritonavir
–– Fusion inhibitor, e.g. enfuvirtide
–– Integrase inhibitors, e.g. Dalutegravir, Elvitegravir.

High Yield Points


HAART (Highly Active Anti- Retroviral Therapy) consists of Combinations of 3 drugs, a thymidine analog NRTI
(aba­cavir or ZDV), a non-thymidine analog NRTI (lamivudine) to suppress replication in both active and resting
cells and a protease inhibitor (atazanavir or lopinavir/ritonavir) or an NNRTI (efavirenz) produce prolonged viral
suppression.
High Yield Points
WHO Recommendation Regarding L at e s t U p d at e s
Breastfeeding in HIV-infected mother:
•• Where replacement feeding is WHO 2016 recommendations for HIV treatment: M
readily available & safe, breast- •• ART should be initiated in all children living with HIV, regardless of WHO clinical stage or CD4 count;
feeding should be avoided •• As a priority, ART should be initiated in all children ≤2 yrs of age or children <5 yrs age with WHO
•• In low-resource countries where clinical stage 3 or 4 or CD4 count ≤750 cells/mm3 or CD4 percentage <25%, and children ≥5 yrs
other diseases (diarrhea, pneumonia, with WHO HIV clinical stage 3 or 4 disease or CD4 count ≤350 cells/mm³
malnutrition) substantially contrib­
ute to a high infant morta-lity rate,
the benefit of breastfeeding out- Recommended 1st line regimes for HIV treatment (Latest)
weighs the risk for HIV transmis-
sion Age group Preferred 1st line ART regimen
Infant < 2 weeks Zidovudine + Lamivudine + Nevirapine

Children < 3 yrs Abacavir (or Zidovudine) + Lamivudine + Lopinavir/Ritonavir

Children 3–10 yrs Abacavir + Lamivudine + Efavirenz

Adolescents Tenofovir + Lamivudine (or Emtricitabine) + Efavirenz

Cotrimoxazole Prophylaxis
High Yield Points •• Recommended for all infants, children & adolescents with HIV, irrespective of clinical
ART should be initiated urgently in all & immune status.
pregnant and breastfeeding women, •• Priority should be given to all children < 5 years old, regardless of CD4 cell count
even if they are identified late in preg­ or clinical stage, & older children with severe or advanced HIV clinical disease (WHO
nancy or postpartum, because the most clinical stage 3 or 4) and/or those with a CD4 count ≤350 cells/mm3.
effective way to prevent mother-to-
child HIV transmission is to reduce
•• Contrimoxazole prophylaxis is recommended for HIV-exposed infants 4 to 6 weeks of
maternal viral load age and should be continued until HIV infection has been excluded by an age-appropriate
HIV test.
Chapter 8: Infectious Diseases 157

Guidelines for PPTCT

Section 1: General Pediatrics


High Yield Points
Use of ART Prophylaxis with Zidovudine (AZT)
•• All HIV infected pregnant women should receive life-long ART should be given instead of Nevirapine to
babies in mothers with HIV-2
•• Postpartum ART to mother & ARV Prophylaxis to child to improve HIV free child survival
•• HIV exposed infants should be followed-up & breastfed infants should receive NVP for 6 wk if the
mother is receiving cART, or NVP for the duration of breastfeeding if the mother is not on therapy.
Intrapartum care
•• Cesarean section is not recommended for prevention of mother-to-child-transmission & it is to be
done only if there is an Obstetric indication for the same.
•• Mother-to-child -transmission risk is increased by prolonged rupture of membranes, repeated
P/V examinations, assisted instrumental delivery (vacuum or forceps), invasive fetal monitoring
procedures, episiotomy and prematurity.
Feeding of HIV exposed Infant
•• Exclusive breastfeeding should be done for at least 6 months
•• Only in situations where breastfeeding cannot be done (maternal death, severe maternal illness) or
individual mother’s choice (at her own risk), then exclusive replacement feeding may be considered
•• Exclusive replacement feeding should be done only when AFASS criteria is fulfilled: A – Affordable, F –
Feasible, A – Acceptable, S – Sustainable, S – Safe.

L at e s t U p d at e s
WHO 2016 recommendations for prophylaxis to infant:

High risk Infants born to → Should receive dual prophylaxis with AZT (twice daily) & NVP (once Infant Nevirapine Prophylaxis
mothers with HIV daily) for 1st 6 weeks of life, whether they are breastfed or formula fed NVP daily dose
Breastfed, high risk → Should continue infant prophylaxis for an additional 6 weeks (total Birth weight (in mg)
12 weeks) using either AZT & NVP or NVP alone. < 2000 gm 2 mg/kg once daily
Breastfed, low risk → Should receive 6 weeks of infant prophylaxis with daily NVP. 2000–2500 gm 10 mg once daily
Not breastfed → Should be given 4–6 weeks of prophylaxis with daily NVP (or AZT) > 2500 gm 15 mg once daily

High-risk infants (for HIV infection) are defined as those born to women with HIV infection:
•• Who have received < 4 weeks of ART at the time of delivery, or
•• With viral load >1000 copies/mL in 4 weeks before delivery, or
•• Detected (for the 1st time) during pregnancy or breastfeeding

9. INFECTIOUS MONONUCLEOSIS M

What is it? Caused by Epstein-Barr virus (EBV); It derives its name from the mononuclear
lymphocytosis with atypical-appearing lymphocytes seen
Pathophysiology Atypical lymphocytes seen here are CD8 T cells, which exhibit both suppressor &
cytotoxic functions that develop in response to infected B cells
Many features result from cytokine release from host immune response
Incubation period 30–50 days in adolescents & shorter in children; Prodromal period 1–2  weeks
Clinical features Majority of cases in infants & young children are clinically silent
Malaise, fever, headache, sore throat, nausea, abdominal pain seen
Mild Splenic enlargement (50%) left upper abdominal discomfort & tenderness
Generalized lymphadenopathy (90% cases)
Hepatomegaly (10%); Jaundice uncommon, but transaminitis common
Rashes (in 3–15%): maculopapular
Laboratory Relative & absolute increase in CD8 cells results in a reversal of CD4/CD8 ratio
findings Acute phase: rapid IgM and IgG antibody responses to VCA in all cases
IgM response to VCA is transient but can be detected for 4 wk 3 months
Monospot test is used for screening; Paul Bunnel test is positive
158 Section 1: General Pediatrics
Review of Pediatrics and Neonatology

High Yield Points


High Yield Points
HHV-6B is responsible for the majority
of cases of roseola infantum •• Most common clinical feature of infectious mononucleosis is generalized lymphadenopathy.
•• Most common site of lymphadenopathy in EBV infection is cervical.
•• Epitrochlear lymphadenopathy is sugg­estive of Infectious mononucleosis.
•• Patients with infectious mono­nucle­osis treated with ampicillin or amoxicillin may experience immune
mediated, self limiting “ampicillin rash,” which may occur with other β-lactam antibiotics.

10. ROSEOLA INFANTUM (EXANTHEM SUBITUM, OR SIXTH DISEASE)


•• It is an acute, self-limited disease of infancy & early childhood, caused by HHV 6 or 7
•• It is characterized by abrupt onset of high fever, which usually resolves acutely after
72 hr, coincident with appearance of a faint pink or rose-colored, non-pruritic, 2–3 mm
morbilliform rash on the trunk
High Yield Points
•• The rash is evanescent & spreads from trunk to face & extremities
•• High fever is the most consistent •• Nagayama spots (erythematous papules on the mucosa of soft palate & base of uvula) may
finding associated with primary be present on the 4th day in 2/3 of patients.
HHV-6B infection •• Complication of Roseola infan­tum include seizures, irrita­bility, otitis, rhinorrhea, GI
complaints & encephalopathy

11. JAPANESE ENCEPHALITIS (JE)


Etiology
JE virus is a positive-sense, ssRNA virus of the family Flaviviridae.
Clinical Features
•• 4 stages: prodromal (2–3 days), acute (3–4 days), subacute (7–10 days) & convalescence (4–7
wk)
High Yield Points •• Abrupt onset fever, headache, respiratory symptoms, abdominal pain & vomiting
•• Seizures are seen in 10–24% of children; psychotic episodes also occur;
•• Culex tritaeniorhynchus, a night- •• Rapidly changing CNS signs (e.g. hyperreflexia followed by hyporeflexia).
biting mosquito is the principal •• Sensorium may vary from confusion, disorientation, delirium to coma.
vector of JE
Laboratory Findings
•• Pigs serve as an amplifying host in JE
•• Children < 15 yr of age are principally •• CSF: Mild pleocytosis (100–1,000 WBCs/μL), initially polymorphonuclear, later lymphocytic.
affected by JE •• Albuminuria is common.
•• Case: Infection ratio for JE virus is 1 : 25 •• Acute-phase serum shows virus-specific IgM or ≥ 4-fold rise in IgG titers.
to 1 : 1,000. •• The virus can also be identified by polymerase chain reaction (PCR).
•• Incubation period of JE is : 4–14 days Treatment: No specific treatment for JE; only supportive care.
Prognosis:
•• Fatality rates for JE are 24–42% (highest in children 5-9 yr & adults > 65 yr age).
•• Fatal cases usually progress rapidly to coma, and the patient dies within 10 days
•• Frequency of sequelae is 5–70% (most common in patients < 10 yr age)
•• Common sequelae are mental deterioration, emotional instability, personality changes, motor
Question 8
abnormalities, & speech disturbances.
A 4-year old child presented with
low-grade fever, headache & mild Prevention
upper respiratory tract infection. JE vaccine (refer pretext of ch 10); Personal measures to reduce mosquito bites.
He also developed an erythema-
tous rash on cheeks. What is your 12. PARVOVIRUS INFECTIONS IN CHILDREN
diagnosis? Parvovirus B19 causes Erythema infectiosum, Arthropathy, Myocarditis, transient aplastic crisis &
variety of skin eruptions like “gloves-and-socks” syndrome.

Erythema infectiosum (fifth disease)


•• A benign, self-limited exanthematous illness of childhood, caused by Parvovirus B19
•• Incubation period for erythema infectiosum is 4–28 days (average: 16–17 days)
•• Prodromal phase; low-grade fever, headache & upper respiratory tract infection
•• Hallmark is the characteristic rash (“slapped-cheek” appearance).

a. Scarlet fever
13. DENGUE FEVER
b. Systemic lupus erythematosus
c. Kawasaki disease About Dengue virus: M
d. Erythema infectiosum •• Dengue virus (DV) – ss RNA virus - Flaviridae family
•• 4 serotypes, DENV-1, DENV-2, DENV-3 & DENV-4
Chapter 8: Infectious Diseases 159

•• Aedes aegypti – main vector in most urban areas

Section 1: General Pediatrics


High Yield Points
•• DENV2 is the most virulent, follo­wed by DENV4
•• Infection with one dengue serotype confers lifelong immunity to that serotype Transient Aplastic Crisis is caused by
Pathophysiology Parvovirus B19 infection leads to a
sudden fall in serum hemoglobin in in­
dividuals with chronic hemolytic con­
ditions like sickle cell disease, thalas-
semia & hereditary spherocytosis

L at e s t U p d at e s

Dengue Fever Classification: M

New WHO classification


Dengue without warning signs
Dengue with warning signs
Severe dengue
Old WHO classification
Dengue Fever (DF)
Dengue Hemorrhagic Fever (DHF)
{hemorrhagic manifestations,
thrombocytopenia < 1 lac, e/o
plasma leakage}
Grade I, II, III & IV

Course of Illness

High Yield Points


Parvovirus B19 & Human Boca­ virus
(HBoV) are the only 2 parvo­ viruses
known to be patho­genic in humans.

High Yield Points


•• Serial Hematocrit & platelet count
monitoring to look for hemoconcen-
tration & thrombocytopenia, are im­
portant in Dengue fever
•• NS1 Antigen is used to diagnose
dengue in 1st 3 days of illness
•• Dengue IgM levels begin to rise 3–5
days after onset
160 Section 1: General Pediatrics

Course of dengue illness:


Review of Pediatrics and Neonatology

Febrile phase Critical phase Recovery phase


Onset 2–7 days After D3/D4 (lasts 24–48 hours) After D6/D7
C/F Fever, headache, Falling WBC & Platelets ECF lost d/t capillary leakage
retrobulbar pain, Plasma leak returns to circulatory system
myalgia & arthralgia Rising HCT – 3rd spacing
Complications Febrile seizure, Shock, organ dysfunction Pulmonary edema d/t fluid
dehydration Acidosis, DIC, Severe bleeding overload if fluid replacement
not optimized

L at e s t U p d at e s
WHO Dengue management guide­lines:
•• Dengue fever (with or without warning signs): Encouragement for oral fluids; If not tolerated, start IV
fluid therapy (NS/RL) at maintenance rates;
•• Dengue shock syndrome: Start IV crystalloid (RL or NS) @ 10 ml/kg & gradually decrease to 7 mL/kg/hr
→ 5 mL/kg/hr while monitoring hematocrit & hemodynamic status.

14. SWINE FLU


•• Caused by H1N1 strain of Influenza virus (incubation period is 1–4 days)
•• Clinical features: Abrupt onset of high grade fever, cough, running nose, sore throat,
headache, myalgia, abdominal pain, vomiting, diarrhea & acute myositis
•• Candidates at high risk: Children with asthma, obesity & chronic immune suppression
•• When to investigate with H1N1 RT PCR? A child with flu like illness:
1. Who is in contact with a person with known H1N1 flu illness
2. Who has traveled in last 7 days to a country with H1N1 infection or is in contact with a
person who has traveled to such a country in the last 7 days.
3. Who has severe illness with red flag signs - somnolence, high and persistent fever, inability
to feed well, convulsion, shortness of breath, difficulty in breathing, etc.
4. Who has an existing immune-compromised medical condition like HIV, Cardiac/
Pulmonary/Metabolic disease.
Treatment
Most patients with H1N1 virus infection have a mild illness & can be managed at home.
Oseltamivir is indicated for treatment of patients ≥ 1 year age at doses of:
Body weight ≤15 kg 15–23 kg 24–40 kg > 40 kg
Dose (twice daily for 5 days) 30 mg 45 mg 60 mg 75 mg

15. EBOLA VIRUS


•• Belongs to family Filoviridae
•• Incubation period: 2–21 days (mean: 11 days).
•• Clinical features: Sudden onset of fever, fatigue, weakness, myalgias, headache, sore throat,
L at e s t U p d at e s
followed by GI involvement including abdominal pain, vomiting & diarrhea. Serious later
Ebola transmission occurs through phase shows vascular leakage, multiorgan failure & death
direct skin-to-skin contact or exposure •• Hemorrhage is seen in 50% cases of Ebola infection
to infected bodily fluids and tissues; •• Those who survive Ebola infection improve in 6–11 days
there is no evidence of such transmission
by aerosol or respiratory droplets •• Diagnosis: By PCR &/or IgM ELISA.

 B. IMPORTANT BACTERIAL INFECTIONS


1. TUBERCULOSIS

High Yield Points


•• TB is most common in young adults & children < 5 yr age.
•• Pulmonary TB remains the most common form of TB in children
•• Children suffer more from extrapulmonary forms of TB than all other age groups.
•• Commonest type of extrapulmonary TB is Lymph node TB.
Chapter 8: Infectious Diseases 161

Causative Agent

Section 1: General Pediatrics


Question 9
Mycobacterium tuberculosis (MTB) complex comprising M. tuberculosis in most cases, M. bovis in
few cases and atypical mycobacterium in immunodeficient children. A 4-year-old child presented
Mode of Infection: Droplets of infected secretions. with cough persisting for
1 month & low grade fever. There
Incubation Period: 4–8 weeks.
was a history of contact with TB.
Classification What is the Chest X-ray of the child
given below suggestive of?
Intrathoracic tuberculosis Extrathoracic tuberculosis
Primary complex TB of Lymph nodes
Primary progressive disease (PPD) CNS disease–TB Meningitis
Miliary tuberculosis TB of abdomen/Renal TB/Genital TB
Pleural effusion Bone TB/Skin TB
Renal TB/Genital tract TB

Diagnosis M a. Pleural effusion


b. Miliary TB
•• Clinical features & abnormal chest radiograph c. Cavitation
•• Demonstration of M. tuberculosis by Ziehl–Neelsen (ZN) staining, culture, PCR or newer d. Primary TB
methods like Genexpert or line probe assay (LPA)
•• Demonstration of host’s response to M. tuberculosis by Mantoux test.

Points to Remember while Diagnosing TB in Children M

•• All efforts should be made to demonstrate bacteriological evidence for TB from sputum, Gastric
lavage, Induced sputum, broncho-alveolar lavage or any other sample.
•• A positive Tuberculin skin test/Mantoux test is defined as an induration of > 10 mm, measured
48–72 hours after Intradermal injection with Tuberculin 2 TU.
•• In HIV cases the cut off is reduced to > 5 mm induration.
•• There is no role for serology (IgM, IgG, IgA against MTB antigens), BCG test & IGRAs for diagnosis
of TB.

Treatment

L at e s t U p d at e s
High Yield Points
Important recent changes, as per RNTCP 2016: M
•• Daily treatment is to be given now for all pediatric TB patients •• Best specimen: for demonstration
•• Only Pyrazinamide needs to be stopped in the continuation phase of M. tuberculosis in young chil-
dren, early morning gastric aspirate
Type of TB case Intensive phase Continuation phase 3 consecutive specimens are recom-
New (2) HRZE (4) HRE mended

Previously treated (2) HRZES + (1) HRZE (5) HRE

Prefix to the drugs stands for number of months


The CP in both new and previously treated cases may be extended by 12–24 weeks High Yield Points
in certain forms of TB like CNS TB, Skeletal TB, Disseminated TB, etc. based on clinical
•• Indication of Steroids in TB: Given
decision of the treating physician.
as 1 mg/kg/day and tapered gradu­
ally over 6–8 weeks.
Daily doses of:
–– Tuberculous meningitis
•• Rifampicin → 15 mg/kg (max 600 mg/day) –– Pericarditis and pericardial
•• Isoniazid → 10 mg/kg (max 300 mg/day) effusion
•• Ethambutol → 20 mg/kg (max 1500 mg/day) –– Massive pleural effusions.
•• Pyrazinamide → 30–35 mg/kg (max 2000 mg/day)
•• Streptomycin → 15 mg/kg (max 1g/day)

TB and HIV High Yield Points


•• TB in HIV positive children is 30 times more common than HIV negative children
•• Vaccination with BCG appears to
•• TB in HIV is more severe progressive and likely to occur in extrapulmonary sites
decrease the risk of tuberculosis in
•• In late stages of HIV infection, diffuse interstitial or miliary infiltrates, intrathoracic
exposed infants
lymphadenopathy is more common, but there is little or no cavitation •• Mother with TB can continue to
•• Nonspecific respiratory symptoms, fever and weight loss are the most common complaints breastfeed the baby
•• Rates of drug resistant TB are higher.
162 Section 1: General Pediatrics

Pregnant Mother with TB


Review of Pediatrics and Neonatology

Question 10
What is the name of this investi- •• Prophylactic INH dose: is recommended for newborns born to mother with tuberculosis
gation done to diagnose infection after ruling out congenital tuberculosis
with tuberculosis? •• Mother and child separation should occur only if
–– Mother is ill enough to require hospitalization
–– She has been or is expected to become non-adherent to her treatment
–– She is infected with a drug resistant strain of M. tuberculosis

Congenital TB
•• Occurs from a lesion in placenta through the umbilical vein
a. IGRA b. Mantoux test •• In congential TB, the tubercle bacilli first reach the fetal liver, where a primary focus
c. AFB staining d. Kveim test with periportal lymph node involvement may occur M
•• Most common route of infection for the neonate: Postnatal airborne transmission from
an adult with infectious primary TB
•• Symptomatic at 2–3 week with respiratory distress, fever, hepato­splenomegaly, failure to
thrive.

2. DIPHTHERIA
Diphtheria is an acute toxic infection caused by Corynebacterium diphtheriae.

Pathogenesis
•• Within 1st few days of respiratory tract infection, a dense necrotic coagulum of organisms,
Question 11 epithelial cells, fibrin, WBCs & RBCs forms, advances, & becomes a gray-brown, leather-like
adherent pseudomembrane (Diphthera is Greek for leather)
A 5-year-old unimmunized child
•• Paralysis of palate & hypopharynx is an early local effect of diphtheritic toxin
presented with sore throat &
•• Toxin absorption can lead to systemic manifestations: kidney tubule necrosis, thrombocyto-
swelling of neck. On examination
of throat, the following was penia, cardiomyopathy, and/or demyelination of nerves.
seen. What is the most probable
Incubation Period: 2–4 Days
causative organism?
Clinical Features
•• In tonsillar and pharyngeal diphtheria, sore throatQ is the universal early symptom
•• Fever (only in 50%), dysphagia, hoarseness, malaise, or headache may be seen
•• Shallow ulceration of the external nares & upper lip is characteristic
•• Mild pharyngeal injection is followed by tonsillar membrane formation, which can extend
to uvula, soft palate, posterior oropharynx, or glottic areas
•• Underlying soft-tissue edema & lymphadenopathy can cause a ‘bull-neck’ appearance M
•• Hoarseness, stridor, & croupy cough can occur due to infection of larynx/trachea/bronchi.

Diagnosis
a. Streptococcus pyogenes
b. Mycobacterium tuberculosis •• The characteristic adherent (difficult to remove) membrane, extension beyond the
c. Staphylococcus aureus faucial area, dysphagia, and relative lack of fever help to differentiate diphtheria from
d. Corynebacterium diphtheriae exudative pharyngitis caused by Streptococcus pyogenes or EBV
•• Sample—a portion of membrane along with underlying exudate
•• Direct smear using Gram stain, Albert stain or specific fluorescent antibody M
•• Culture isolates of coryneform organisms should be identified to the species level, &
toxigenicity and antimicrobial susceptibility tests should be performed.

Complications
Cardiac:
•• Toxic cardiomyopathy occurs in 10–25% patients & is responsible for 50–60% of deathsQ
High Yield Points •• First evidence of cardiac toxicity is tachycardiaQ disproportionate to fever, during 2nd–
•• Most common primary focus of 3rd wk of illness; Prolonged P-R interval & changes in ST-T wave seen on ECG.
infection in Diphtheria is tonsils or
pharynx (94%) Neurologic
•• Nose and larynx are the next 2 most •• Acutely (1st 2–3 wk) hypesthesia and local paralysis of the soft palate occur
common sites of Diphtheria primary •• Weakness of the posterior pharyngeal, laryngeal, and facial nerves may follow, causing a
focus
nasal quality in the voice, difficulty in swallowing, and risk for aspiration M
Chapter 8: Infectious Diseases 163

•• Cranial neuropathies characteristically occur in the 5th week, leading to oculomotor &

Section 1: General Pediatrics


ciliary paralysis, which can cause strabismus or blurred vision
Question 12
•• Symmetric polyneuropathy has its onset 10 days to 3 mo after oropharyngeal infection and A 5-year-old unimmunized child
causes distal muscle weakness with diminished deep tendon reflexes presented with high fever, sore
•• Distal muscle weakness in the extremities with proximal progression is more common. throat, dysphagia & voice change.
Throat swab from a whitish lesion
Treatment showed the following diagnosis.

•• Specific antitoxin (given as soon as possible) is the mainstay of therapy M


•• The role of antimicrobial therapy is to halt toxin production, treat localized infection, and
prevent transmission of the organism to contacts
•• C. diphtheriae is susceptible to penicillins, erythromycin, clindamycin, rifampicin, &
tetracycline. Antibiotic therapy is not a substitute for antitoxin therapy
•• Asymptomatic case contacts should receive antimicrobial prophylaxis regardless of
immunization status, using benzathine penicillin G or erythromycin.
a. Diphtheria
b. Mumps
3. PERTUSSIS c. Pertussis
d. Streptococcal pharyngitis
Etiology
•• Bordetella pertussis is the usual cause; occasionally, Bordetella parapertussis
•• Bordetella are small, fastidious, Gram –ve coccobacilli that colonize only ciliated epithelium.

Clinical Manifestations
High Yield Points
Pertussis is divided into catarrhal, paroxysmal, and convalescent stages.
1. Catarrhal stage (1–2 wk): begins insidiously with congestion, rhinorrhea, low-grade fever, •• Pertussis toxinQ (PT) is the major
sneezing, lacrimation & conjunctival suffusion virulence protein, that causes
2. Paroxysmal stage (2–6 wk): dry, intermittent, irritative hack with paroxysms of uninterrupted histamine sensitivity, insulin
secretion & leukocyte dysfunction
cough on a single exhalation, eyes bulging & face purple, & a loud whoop follows as inspired
•• Pertactin is important for
air traverses the still partially closed airway. Posttussive emesis is common
attachment to ciliated respiratory
3. Convalescent stage (6 wk): the number, severity, and duration of episodes diminish. epithelial cells.
•• Tracheal cytotoxin & adenylate
Investigations M cyclase, cause local damage.
•• Leukocytosis (15,000–100,000 cells/μL) with absolute lymphocytosis •• Incubation period: 3–12 days
•• Confirmation by culture, PCR or serology.
Diagnosis of Pertussis should be suspected in:
•• Any individual who has a pure or predominant complaint of cough, especially if the
following features are absent: fever, malaise or myal­gia, exanthem or enanthem, sore throat,
hoarseness, tachypnea.
•• Infants <3 mo of age with gagging, gasping, apnea, cyanosis, or an ap­parent life-threatening
event.

Treatment
•• Macrolides are preferred agents: Erythromycin or Azithromycin
•• A 7–10-fold relative risk for infantile hypertrophic pyloric stenosis has been reported in
neonates treated with orally administered erythromycin. Azithromycin is preferred.

Complications
Apnea, otitis media, pneumonia, & physical sequelae of forceful coughing.

High Yield Points


•• Pertussis is extremely contagious, with attack rates as high as 100% in susceptible individuals
•• Dry paroxysmal cough is the hallmark of pertussis

4. TETANUS
Tetanus is an acute, spastic paralytic illness historically called lockjaw, caused by neurotoxin
produced by Clostridium tetani, a motile, Gram-positive, spore-forming obligate anaerobe.
164 Section 1: General Pediatrics

Mode of Infection
Review of Pediatrics and Neonatology

High Yield Points


•• Neonatal: occurs in infants whose mothers are not immunized
The typical setting for tetanus is an
•• Non-neonatal cases: are associated with a traumatic injury, often a penetrating wound
unimmunized patient (and/or mother)
who was injured or born within the
inflicted by a dirty object or unsterile injection.
preceding 2 wk, who presents with Incubation Period: Typically it is 2–14 days; may be months.
trismus, other rigid muscles, and a
clear sensorium Pathogenesis
Spores introduced in an infected injury site germinate, multiply & produce tetanus toxin

Question 13 Toxin binds at the neuromuscular junction & enters motor nerve by endocytosis &
A newborn whose mother received reaches spinal inhibitory interneurons
no antenatal care, developed ↓
progressive difficulty in sucking Prevents release of the neurotransmitters glycine and γ-aminobutyric acid (GABA)
and swallowing & incessant crying
beginning from day 7 of life. He ↓
then developed stiffness of limbs Blocks the normal inhibition of antagonistic muscles
& a posture shown in the picture

below. What is your diagnosis?
Affected muscles sustain maximal contraction and cannot relax

Clinical Features of Tetanus


Most common presenting symp­tom in tetanus is trismus (masseter muscle spasm, or lockjaw).

Generalized tetanus Neonatal tetanus

a. Diphtheria •• Most common presenting symptom is trismus •• Manifests within 3–12 days of birth
b. Neonatal tetanus or lockjaw, due to masseter muscle spasm •• Progressive difficulty in sucking and swallowing
c. Congenital syphilis •• Headache, restlessness, and irritability •• Irritability & excessive crying
d. Congenital toxoplasmosis •• Stiffness, difficulty chewing, dysphagia, neck •• Paralysis, stiffness and rigidity, spasms, with or
muscle spasm & risus sardonicus without opisthotonus
•• Opisthotonus (arched posture)

Diagnosis
•• Results of routine laboratory studies are usually normal except peripheral leukocytosis
•• C. tetani on Gram stain of wound material seen in 30% of cases.

Treatment
High Yield Points
•• Includes eradication of C. tetani & anaerobic wound environment, neutralization of
The most common form of tetanus is tetanus toxin, control of seizures and respiration, & prevention of recurrences
neonatal (or umbilical) tetanus. •• Surgical wound debridement to remove anaerobic growth conditions
•• Tetanus Tg should be given as soon as possible so as to neutralize toxin
•• Penicillin G remains the antibiotic of choice
•• Metronidazole, Erythromycin or tetracycline for penicillin-allergic patients
•• All patients with generalized tetanus need muscle relaxants (Diazepam).

5. CONGENITAL SYPHILIS
Caused
Treponema pallidum, a delicate, tightly spiraled, motile spirochete

Mode of Infection
Results from transplacental transmission or during birth by contact with infectious lesions.

Clinical Features
•• Transmission in pregnancy results in early fetal loss, preterm or LBW infants, stillbirths or
neonatal deaths
Chapter 8: Infectious Diseases 165

•• Among symptomatic infants, manifestations have been divided into early signs (during 1st

Section 1: General Pediatrics


2 yr) & late signs (appear during 1st 2 decades).
Question 14
i. Early Signs of Congenital Syphilis M What is this teeth abnormality seen
in congenital syphilis known as?
-- Hepatosplenomegaly, jaundice, and elevated liver enzymes are common
-- Lymphadenopathy tends to be diffuse and resolve spontaneously
-- Coombs-negative hemolytic anemia is characteristic
-- Thrombocytopenia is often associated with platelet trapping in an enlarged spleen
-- Painful osteochondritis & periostitis resulting in irritability and refusal to move the
involved extremity (pseudoparalysis)
a. Mc Leod teeth
-- Mucocutaneous erythematous maculopapular or vesiculobullous rash followed by b. Hutchinson’s teeth
desquamation involving hands & feet. c. Koplik teeth
ii. Late manifestations: Result from chronic granulomatous inflammation of bone & d. Snuffles
CNS & include: M

Symptom/Sign Descriptions/Comments High Yield Points


Olympian brow Bony prominence of the forehead caused by persistent or recurrent periostitis
Mucous patches, persistent rhini­ tis
Higoumenaki’s sign Unilateral or bilateral thickening of the sternoclavicular third of the clavicle (snuffles), and condylomatous lesions
Saber shins Anterior bowing of the midportion of the tibia are highly characteristic features
of mucous membrane involvement
Hutchinson teeth Peg-shaped upper central incisors with a notch along the biting surface
in congenital syphilis, containing
Mulberry molars Abnormal 1st lower molars with excessive number of cusps abundant spirochetes
Saddle nose Depression of the nasal root, a result of destruction of bone & cartilage
Rhagades Linear scars in a spoke-like pattern from mucocutaneous fissures of mouth, genitalia
Hutchinson triadQ Hutchinson teeth, interstitial keratitis, and 8th nerve deafness
Clutton joint Unilateral or bilateral painless joint swelling (usually involving knees)

Diagnosis
•• CSF analysis for VDRL, cell count, and protein
•• Blood & CBC and platelet count, X-ray, bones, liver function, eye examination.

Treatment
•• Parenteral penicillin G is the only documented effective treatment for congenital syphilis,
syphilis during pregnancy, and neurosyphilis.

Prevention
Routine prenatal screening for syphilis remains the most important factor. High Yield Points
When to suspect congenital syphilis? •• Untreated syphilis during pregnan­
•• Untreated early or latent syphilis in the mother cy has a vertical transmission rate
•• An untreated mother who has contact with a known syphilitic during pregnancy approaching 100%
•• Mother treated for syphilis during pregnancy with a drug other than penicillin •• Fetal or perinatal death occurs in
•• Mother coinfected with HIV. 40% of infants born to mothers with
untreated syphilis

6. IMPORTANT STAPHYLOCOCCAL DISEASES


Toxin-mediated diseases caused by Staphylococcus aureus include scalded skin syndrome, toxic
shock syndrome (TSS), food poisoning & staphylococcal scarlet fever.

Staphylococcal Scalded Skin Syndrome (Ritter Disease)


•• It occurs predominantly in infants & children < 5 yr of age
•• Onset of the rash may be preceded by malaise, fever, irritability, and exquisite tender­
ness of the skin
•• Scarlatiniform erythema develops esp. in flexural & periorificial areas
•• The conjunctivae are inflamed and occasionally become purulent
•• Patients may have pharyngitis & superficial erosions of the lips, but intraoral mucosal
surfaces are spared.
166 Section 1: General Pediatrics

Staphylococcal Toxic Shock Syndrome Diagnostic Criteria Includes


Review of Pediatrics and Neonatology

Mnemonic M

For major criteria of Staphylococcal Major Criteria (All Required) Minor Criteria (Any 3 or More)
Toxic shock syndrome Hypotension Mucous membrane inflammation (pharyngeal or conjunctival
Mnemonic: “H-A-R jagah Acute fever; temperature >38.8°C hyperemia, strawberry tongue, vomiting, diarrhea)
Staphylococcus hai” Rash (erythroderma with desquamation) Liver abnormalities (↑ bilirubin or transaminases)
•• Hypotension Renal abnormalities (↑ urea or creatinine)
•• Acute fever Exclusionary Criteria
Muscle abnormalities (myalgia or ↑ CPK level)
•• Rash Absence of another explanation
CNS abnormalities (alteration in consciousness)
Negative blood cultures
Thrombocytopenia (100,000/mm3 or less)

High Yield Points


7. IMPORTANT STREPTOCOCCAL DISEASES
•• Scarlet fever is an important diffe­
rential diagnosis of Kawasaki disease Scarlet Fever M

•• It is caused by pyrogenic exotoxin producing Group A Streptococcus (GAS)


•• Erythematous papular rash usually appears within 24–48 hr after onset of symptoms

Question 15 •• Examination of the pharynx shows pharyngitis & Strawberry tongue is seen
•• Skin has a goose-pimple appearance and feels rough
A 3-year-old child presented with
fever & rashes. Oral examination •• Desquamation of palms & soles seen
showed the following diagnosis? •• Kawasaki disease is a close differential diagnosis (Fever + 4 out of 5 criteria required
for its diagnosis)
•• Identification of GAS in the pharynx confirms the diagnosis.

Erysipelas
•• It is a Gr A Streptococcus infection involving deeper layers of skin & underlying
a. Kawasaki disease connective tissue
b. Scarlet fever •• Skin in affected area is swollen, red & tender, Superficial blebs may be present
c. Either of the above
•• Most characteristic finding is a sharply defined, slightly elevated border; abrupt onset
d. None of the above
with high fever seen
•• Reddish streaks of lymphangitis may project out from the margins of the lesion.

 C. IMPORTANT PARASITIC INFECTIONS

TOXOPLASMOSIS
Causative agent: Toxoplasma gondii, an obligate intracellular protozoan
Mode of infection: Acquired perorally (from ingesting food that contains oocysts), transplacentally,
vaginal delivery or rarely parenterally.

Clinical Manifestations
•• Acquired Toxoplasmosis: Generally asymptomatic in immunocompetent children
•• Congenital Toxoplasmosis: Clinical features range from hydrops fetalis to SGA, prematurity,
peripheral retinal scars, persistent jaundice, mild thrombocytopenia
•• Systemic signs: Lymphadenopathy, hepatosplenomegaly, pneumonitis, nephrotic syndrome,
myocarditis
•• Skin: Rashes, petechiae, ecchymosis secondary to thrombocytopenia
•• Endocrine abnormalities: Hypothalamic or pituitary involvement or end organ involvement
•• CNS: Hydrocephalus, spinal or bulbar involvement, microcephaly, hearing loss
•• Eyes: Chorioretinitis, retinal detachment, keratic precipitates, glaucoma.

Diagnosis
Blood/ Body fluid culture, Lymph node histology, cysts in placenta/Fetal/newborn tissues; CSF/
amniotic fluid PCR & Serological tests. Sabin Feldman dye test can be used.

Treatment
In both Congenital & acquired, pyrimethamine, sulphadiazine & leukovorin used.
Chapter 8: Infectious Diseases 167

CONGENITAL TOXOPLASMOSIS

Section 1: General Pediatrics


•• It usually occurs when a woman acquires primary infection while pregnant.
•• Risk of fetal infection increases with each trimester of pregnancy being 15%, 25% &
60% in 1st, 2nd and 3rd trimester respectively.
•• But severity of fetal infection is much greater in early pregnancy.
•• Characteristic triad of congenital toxoplasmosis is chorioretinitis, hydrocephalus, and
cerebral calcification
•• Most common manifestation of acquired toxoplasmosis is cervical lymphadenopathy.

MALARIA IN CHILDREN
Differences between severe malaria in adults & children: High Yield Points
•• Once the patient can tolerate oral
Signs/symptoms Adults Children
therapy or after at least 24 hours of
Hypoglycemia & Seizures Less common More common parenteral therapy:
Duration of antecedent illness 5–7 days 1–2 days •• Patients should get full course of oral
ACT.
Pulmonary edema Common Rare •• ACT containing mefloquine should
Jaundice/Renal failure Common Rare be avoided in cerebral malaria due to
neuropsychiatric complications.
Bleeding/clotting disturbances Up to 10% Rare
Neurological sequelae < 5% > 10%

Diagnostic criteria for severe malaria: 1 or more of:

•• Impaired consciousness/coma •• Metabolic acidosis


•• Repeated generalized convulsions •• Circulatory collapse/shock
•• Renal failure (S. Creatinine >3 mg/dL) •• Abnormal bleeding & DIC
•• Jaundice (S. Bilirubin >3 mg/dL) •• Hemoglobinuria
•• Severe anemia (Hb <5 g/dL) •• Hyperpyrexia (Temperature >106°F)
•• Pulmonary edema/ARDS •• Hyperparasitemia (>5% parasitized RBCs)
•• Hypoglycemia (Pl. Glucose <40 mg/dL)

Treatment of malaria in children:


Type of Malaria Treatment Regimens
Chloroquine Chloroquine 10 mg base per kg immediately, followed by 10 mg/kg at 24 hr and High Yield Points
sensitive P. vivax 5 mg/kg at 48 hr
Chloroquine Any one of the following Artemisinin-based Combination Therapy (ACT) should •• Primaquine should be given to all
resistant P. vivax be used: children with Malaria in a dose of
0.25 mg/kg body weight daily for
Uncomplicated Artemether + Lumefantrine or Artesunate + Mefloquine or Artesunate +
Amodiaquine or 14 days, for vivax malaria & mixed
P. falciparum infections, to prevent relapse of vivax
Artesunate + sulfadoxine-primethamine or Dihydroartemisinin-piperaquine
malaria.
Complicated or 1st line: Artesunate: 2.4 mg/kg body weight IV or IM given on admission (time = 0), •• For falciparum malaria, a single
severe vivax or then at 12 and 24 hours, then once a day dose of Primaquine (0.75 mg/kg
falciparum body weight) is recommended, to
2nd line: Quinine: 20 mg quinine salt/kg body weight on admission (IV infusion in
malaria reduce the transmissibility of treated
5% dextrose/dextrose saline over a period of 4 hours) followed by maintenance dose
of 10 mg/kg body weight 8 hourly; falciparum malaria.
With Artesunate or Quinine, add:
For child < 8 years age: Clindamycin or Atovaquone + Proguanil
For child > 8 years age: Doxycycline/Tetracycline or Atovaquone + Proguanil
Supportive care is also very important & includes IV fluids & blood components
168 Section 1: General Pediatrics
Review of Pediatrics and Neonatology

Answer Keys for Image-Based Questions

Answers Explanations / Identifying features


1. Ans. d. Congenital rubella Ophthalmoscopic picture of fundus shows heterogeneous opacities in the retina suggestive of ‘Salt &
Pepper Retinopathy’, along with history of fever and rash in mother, with PDA & deafness in newborn
point to a diagnosis of Congenital Rubella
2. Ans. b. Measles A red lesion with a whitish spot on buccal mucosa, suggestive of Koplik’s spot, along with a papular rash,
fever & conjunctivitis, suggest a diagnosis of Measles
3. Ans. c. Varicella zoster virus Simultaneous presence of various types of skin lesions including macules, papules & vesicles are suggestive
of ‘Chicken Pox’, caused by Varicella zoster virus
4. Ans. a. Congenital varicella Cicatricial skin scarring in a zoster-like distribution & limb hypoplasia in this newborn points to a
diagnosis of Congenital Varicella syndrome
5. Ans. c. Herpes zoster Erythematous vesicles confined to a dermatomal distribution is ‘Herpes Zoster’
6. Ans. a. Mumps Swelling involving the parotid region, in a child, is suggestive of Mumps
7. Ans. b. Herpes simplex virus Multiple fluid filled lesions on the lips, as shown, in a child who had fever, suggests Herpes labialis caused
by Herpes simplex virus (HSV)
8. Ans. d. Erythema infectiosum Slapped cheek appearance is seen in Erythema infectiosum
9. Ans. d. Primary TB Chest X-ray showing lung parenchymal lesion with enlarged hilar lymph nodes, in a child with persistent
cough, fever & history of contact with TB points towards ‘Primary TB’
10. Ans. b. Mantoux test This is a ‘Mantoux test’ or ‘Tuberculin test’, being administered intradermally
11. Ans. d. Corynebacterium A membrane is seen over the tonsil in this child with swelling of neck. Demonstration of bacilli on Albert
diphtheriae staining, in the given scenario is suggestive of Diphtheria
12. Ans. a. Diphtheria On Albert staining, C. diphtheriae appears green colored rod shaped bacteria with bluish black
metachromatic granules at the poles
13. Ans. b. Neonatal tetanus Opisthotonic posturing seen in this newborn, with difficulty in sucking & swallowing, whose mother received no
tetanus toxoids, is suggestive of Neonatal Tetanus
14. Ans. b. Hutchinson’s teeth Smaller & more widely spaced teeth with notches on their biting surfaces are ‘Hutchinson’s teeth’ seen in
Congenital Syphilis
15. Ans. c. Either of the above The given picture shows ‘strawberry tongue’, which can be seen in either Kawasaki disease or Scarlet fever
Chapter 8: Infectious Diseases 169

Questions
Questions

 RUBELLA c. Most common congenital defects are deafness, cardiac


malformation and cataract.
1. All of the following comprise classical triad for con­geni­ d. Infection after 16 weeks of gestation results in major
tal rubella syndrome, except: (DNB June 2018) congenital defects.
a. Sensory neural deafness b. Cataract 12. The following age group is most severely affected by
c. Patent ductus arteriosus d. Micrognathia rubella infection: (JIPMER 2002)
2. True regarding congenital rubella syndrome are all a. Females aged 25–35 year
except: (JIPMER May 2018) b. Young girls
a. Infants of congenital rubella syndrome can shed virus in c. Adolescent girls d. Unborn child
respiratory secretions upto 1 year of age 13. Which of the following is not a common manifestation
b. Risk of developing diabetes mellitus is considerably of congenital rubella? (AIPGMEE 2002)
higher in congenital rubella syndrome a. Deafness b. PDA
c. Cataract in CRS is always bilateral c. Aortic stenosis d. Mental retardation
d. Risk of congenital defects is high when fetus is infected
during the first trimester  VARICELLA
3. Which of the following vaccines are contraindicated in
pregnancy? M  (Recent Question 2017) 14. All of the following are true about Chickenpox except: M
a. Rabies vaccine b. Hepatitis B vaccine  (FMGE pattern 2017)
c. Rubella vaccine d. Diphtheria vaccine a. Incubation period can range from 10-21 days.
4. MC manifestation of congenital rubella syndrome is: M b. Subclinical varicella is rare
 (JIPMER Nov 2016) c. Simultaneous presence of lesions in various stages of
a. Cataract b. Deafness evolution is characteristic
c. Patent ductus arteriosus d. Chorioretinitis d. Umbilicated vesicles are the earliest lesions to appear
5. Congenital rubella syndrome is associated with: M  15. A neonate presented with cicatrising skin lesions all
 (Recent Question 2015) over the body with hypoplasia of all limbs. A MRI
a. Cataract b. Sensory neural hearing loss of the brain revealed diffuse cerebral atrophy. An
c. Congenital heart disease d. All of above ophthalmologic evaluation reveals chorioretinitis.
6. Forcheimer spot is seen in: (Recent Question 2015) Which of these tests is most likely to show a positive
a. Measles b. Mumps result in this patient? (AIIMS Nov 2016)
c. Rubella d. Enterovirus a. Anti HCMV antibody b. Anti toxoplasma antibody
c. Anti VZV antibody d. Anti rubella antibody
7. Rubella causes all except: (Recent Question 2014)
a. Microcephaly b. VSD 16. Following are the complications of chickenpox except:
c. ASD c. Pulmonary stenosis  (Recent Question 2014)
8. Trans-placental spread is least associated with: a. Meningitis b. Pneumonia
 (AIPGMEE 2011) c. Enteritis d. Reye's syndrome
a. HBV b. Rubella 17. A 5-year-old child develops fever with rash on the first
c. HSV d. HIV day of fever. Rash disappears after few days & the child
develops ataxia. Most probable diagnosis is:
9. All of the following statements about congenital rubella
 (Recent Question 2013)
are true except: (AIIMS May 2011, Nov 2008)
a. Measles b. Fifth disease
a. IgG persists for more than 6 months c. Chicken pox
b. IgM antibody present at birth d. Rocky mountain spotted fever
c. Most common anomalies are hearing and heart defects
18. Which of the following intrauterine infections is
d. Increased risk of congenital malformation if infection associated with limb reduction defects and scarring of
occur after 16 weeks. skin? (AI 2009, AIIMS May 2001)
10. Congenital heart lesion(s) in rubella are: (PGI Nov 2009)
a. Varicella virus b. Herpes virus
a. VSD b. ASD c. Rubella d. Parvo virus
c. PDA d. Coarctation of aorta 19. A girl child is suffering from varicella. And child's aunt
e. Pulmonary stenosis is pregnant. When is it earliest that the child can meet
11. All of the following statements are true about congenital her aunt? (PGI Dec 2006)
rubella except: (AIPGMEE 2005) a. When the lesions have crusted
a. It is diagnosed when the infant has IgM antibodies at birth. b. Immediately
b. It is diagnosed when IgG antibodies present for more than c. Anytime as the child is aunt's favorite
6 months. d. After the delivery of the baby
170 Section 1: General Pediatrics

20. True about chickenpox:  (PGI June 2001) 31. True about measles: (PGI Dec 2005)
Review of Pediatrics and Neonatology

a. Incubation period 2–3 days a. Rash appear first on leg


b. Scabs are infective b. Koplik spots are seen in retina
c. Centrifugal rash c. Long term complications follow in form of SSPE
d. Incubation period is 2–3 days
d. Rash appears on first day
e. Caused by RNA virus
e. Rash can occur in axilla
32. True about measles: (PGI June 2004)
21. Most common complication of chickenpox in children:
a. Koplik spots appear in prodromal stage
a. Encephalitis b. Secondary bacterial infection b. Fever stops after the onset of rash
c. Pneumonia d. Otitis media c. Vaccine given at 9 months
e. Orchitis d. It is not diagnosed when coryza and rhinitis is absent.
e. It is not infective before the rash appears
 MEASLES 33. SSPE (Subacute Sclerosing Panencephalitis) is associated
with: (PGI June 2001)
22. Which statement regarding measles is false? a. Mumps b. Chickenpox
 (JIPMER May 2018) c. Herpes d. Measles
a. Koplik's spots cannot be seen in conjunctival or vaginal e. Rubella
mucosa
b. Vitamin A administration is part of management of  MUMPS
clinical measles
c. CD 150 and PVRL4 are receptors for measles virus 34. Parotitis and orchitis are common manifestations of: M 
d. Pneumonia is the common cause of death in children
a. Diphtheria b. Measles (APPG 2015)
with measles
c. Rubella d. Mumps
23. Subacute sclerosing panencephalitis is a late neuro­
35. Which of the following is the most common cause of
logical complication associated with infection due to: M
meningoencephalitis in children? (Bihar PG 2015)
 (MAHA PGM CET 2016)
a. Mumps b. Arbovirus
a. Measles virus b. Mumps virus
c. HSV d. Enterovirus
c. Rubella virus d. Parainfluenza virus
24. Most common complication of measles is: M  36. Which of the following is true of mumps?
 (Recent Question 2016)
 (Recent Question 2014)
a. Salivary gland involvement is limited to the parotids
a. Orchitis b. Pneumonia
b. The patient is not infectious prior to clinical parotid
c. Otitis media d. Seizures
enlargement.
25. Warthin Finkeldey giant cells are seen in: c. Meningoencephalitis can precede parotitis
 (WB PGMEE 2016) d. Mumps orchitis frequently leads to infertility
a. Mumps b. Rubella 37. Most common viral cause of acquired aqueductal
c. Measles d. Polio stenosis is:  (DNB Pattern 2013)
26. Chronic carrier state is seen in all, except: a. Rubella b. Mumps
 (Bihar PG 2015) c. Toxoplasma d. Enterovirus
a. Measles b. Diphtheria 38. Incubation period of mumps is: (MAHA PG 2010)
c. Typhoid d. Gonorrhoea a. 5–7 days b. 8–15 days
27. Incubation period of measles is: M  c. 14–24 days d. 24–30 days
 (Recent Question 2015) 39. Most common complication of mumps in children:
a. 1-4 days b. 5-7 days a. Orchitis (JIPMER 2010)
c. 10-14 days d. 15-20 days b. Aseptic meningitis
28. A child presents with history of high fever for 4 days c. Myocarditis
along with seizures. A rash develops on trunk on the d. Pancreatitis
day after the fever subsides. What is the most probable 40. All of the following are features of mumps, except:
diagnosis? M  (Recent Question 2015)  (Delhi PG Feb. 09)
a. Roseola infantum b. Measles a. Caused by paramyxovirus
c. Rubella d. Varicella b. Aseptic meningitis is a complication in children
29. Subacute sclerosing panencephalitis is a complication c. Orchitis is a complication in adults
of: M  (Recent Question 2015) d. Incubation period is less than 2 weeks
41. Most common complication of mumps in children: M 
a. Mumps b. Rubella
 (JIPMER 2007)
c. Cytomegalovirus d. Measles
a. Orchitis b. Aseptic meningitis
30. Which is not a complication of measles? c. Myocarditis d. Pancreatitis
 (Recent Question 2012) 42. Commonest complication of mumps is: (AIPGMEE 2000)
a. Cataract b. Encephalitis a. Orchitis and oophritis b. Encephalitis
c. Pancreatitis d. Meningitis c. Pneumonia d. Myocarditis
Chapter 8: Infectious Diseases 171

 HIV 51. Parotid enlargement in a HIV infected child is char­

Questions
acterized in which stage of AIDS, according to WHO: M
43. Preferred diagnostic test used for HIV detection in a. Stage 1 b. Stage 2 (JIPMER 2014)
neonate born by caesarean section whose mother is HIV c. Stage 3 d. Stage 4
positive: (PGI Nov 2017) 52. Most common symptom of AIDS in infant is: M 
a. HIV DNA PCR b. ELISA a. GI infection  (Recent Question 2013)
c. HIV culture d. HIV P24 antigen assay b. Persistent cough
e. HIV RNA PCR c. Rashes d. Lymphadenopathy
44. Best method to diagnose HIV in an infant? 53. According to WHO the major sign of AIDS in children in
 (FMGE June 2018) stage I is the following: (Recent Question 2013)
a. Western blot b. ELISA a. Generalised lymphadenopathy
c. PCR d. All b. Not gaining weight
45. Appropriate therapy with antiretroviral agents c. Oral candidiasis d. All of above
results in a syndrome characterized by an increased
54. Indications to start ART in children under NACO all
inflammatory response from the recovered immune
except: (Delhi PG Mar 09)
system to subclinical opportunistic infection. What is
this syndrome called as? (Recent Question 2017) a. <11 months infant if CD4 Count <1500 cells/mm3
b. 12–35 months if CD4 Count <500 cells/mm3
a. HIV related hyper- reactivity syndrome
b. AIDS related inflammatory syndrome c. 36–59 months if CD4 Count <350 cells/mm3
c. Immune reconstitution inflammatory syndrome d. >5 years if CD4 Count <200 cells/mm3
d. CD4 responsive inflammatory syndrome 55. Prophylaxis with Cotrimoxazole is recommended in the
46. Which of the following is FALSE regarding antiretroviral following situation except:  (Delhi PG Mar. 09)
regime for treating infants born to HIV Infected women? a. All symptomatic HIV infected children > 5 years of age
 (Recent Question 2017) irrespective of CD4
a. Daily Nevirapine prophylaxis at birth and continued for b. All HIV exposed infants till HIV infection can be ruled out
6 weeks if mother received ART regularly in antenatal c. All HIV infected infants less than 1 year age irrespective of
period symptoms or CD4 counts
b. Daily Nevirapine prophylaxis at birth and continued for d. As secondary prophylaxis after initial treatment for
12 weeks if mother presented directly in labor ,without pneumocystis carini pneumonia
evidence of adequate treatment of ART (alteast 24 weeks) 56. True statement(s) regarding feeding of HIV-infected
or did not receive any ART earlier child is/are: (PGI Dec 08)
c. Daily Nevirapine prophylaxis at birth and continued for a. Breastfeeding for 4–6 months then start weaning
12 weeks if mother received ART regularly in antenatal b. Breastfeeding for 1 year then start weaning
period c. Exclusively top feeding
d. Dose of Nevirapine for treating infants born to HIV d. Breastfeeding for 6 months and then rapid weaning
infected women is 2 mg/kg once daily in babies with e. Mixed feeding should be done
birth weight <2000 g
57. Perinatal prevention of mother to child, which of the
47. As per WHO classification of immunosuppression, how
following steps are useful? (PGI Dec 08)
much is the CD4 level in HIV/AIDS children with severe
stage in age group of 36 – 59 months: a. Cleaning mother's vagina with antiseptic lotion
 (MAHA PGM CET 2016) b. Elective Caesarian section
c. Avoid breast feeding d. ART prophylaxis
a. < 35% b. < 25%
c. < 20% d. < 15% 58. Drugs used to prevent HIV from mother to child:
48. Which of following is NOT an AIDS defining illness? a. Zidovudine to mother b. Nevirapine to mother
 (PGI May 2016) c. Nevirapine to baby up to 6 weeks
a. Anal cancer b. Hodgkins lymphoma d. Zidovudine to baby (PGI Dec 08)
c. BurKitt's lymphoma d. Kaposi sarcoma 59. Unexplained persistent parotid enlargement in
e. Toxoplasmosis of brain a malnourished child is due to (TN PGMEE 2008)
49. If HIV infected mother has received only AZT during a. HIV b. Leukemia
antenatal period, the antiretroviral drug indicated for c. Infectious mononucleosis
breastfeeding infant is? (MAHA PGM CET 2015) d. Mumps
a. Nevirapine b. AZT 60. Vaccines contraindicated in HIV positive child: M 
c. 3TC d. d4T a. OPV b. MMR (PGI Dec 08)
50. True about HIV infection in infants: (PGI Nov 2014) c. Rabies d. Influenza
a. Mainly occur through horizontal transmission e. Hepatitis
b. HIV DNA PCR positive at 3 months confirms diagnosis 61. For the prevention of parent to child transmission of HIV,
c. Positive antibody test for IgG antibody at 3 month the NACO's recommendation is to give: (UPSC 07)
confirms infected infant a. Nevirapine 200 mg in active labour to mother
d. Passive transfer of maternal antibody generally persists b. Nevirapine 200 mg four hours after rupture to membranes,
for 12 months to mother
172 Section 1: General Pediatrics

c. Nevirapine 200 mg in active about to mother and syrup


Review of Pediatrics and Neonatology

72. The common name for roseola infantum caused by


nevirapine 2 mg/kg body weight to newborn with 72 human herpes virus 6A and 6B is:
hours of delivery  (Recent Question 2017)
d. Syrup Nevirapine 2 mg/kg body weight to newborn within
a. Exanthema subitum b. Fifth disease
72 hours of birth c. Herpes fever d. None of the above
62. Which drug is given to the baby to prevent HIV 73. Papular-purpuric ‘Gloves and socks' syndrome is
transmission from mother to child? (AIIMS Nov 06) caused by (Recent Question 2017)
a. Nevirapine b. Lamivudine a. EB virus b. West Nile virus
c. Stavudine d. Abacavir c. Parvo virus B19 d. KFD virus
63. Which positive test does not necessarily indicate HIV 74. Palivizumab is a humanized monoclonal antibody. For
infection in a newborn? (Karnataka 04) which of the following conditions has it been approved
a. ELISA for HIV IgG antibody for? (AIIMS Nov 2016)
b. p24 antigen a. Coxsackie virus b. Avian influenzae
c. Virus culture c. Parainfluenzae d. Respiratory syncytial virus
d. ELISA for HIV IgA antibody 75. A neonate, born to a mother infected with CMV,
64. All of the following methods are used for the diagnosis on routine examination at birth was found to have
of HIV infection in a 2 months old child, except: hepatomegaly. Rest of the examination was essentially
 (AIIMS May 03) unremarkable. On investigations, Anti HCMV antibodies
a. DNA-PCR b. Viral culture were found to be positive in the baby. What sequelae in
later life is the child at risk of? (AIIMS Nov 2016)
c. HIV ELISA d. p24 antigen assay
a. Renal failure b. Sensorineural hearing loss
c. Hepatic fibrosis d. Mental retardation
 OTHER VIRAL INFECTIONS 76. True for polio virus is: (PGI May 2016)
a. VDPV is usually due to type 3
65. Prophylactic dose of Oseltamivir for infant is:
b. Wild polio virus mostly is type 1
 (FMGE Dec 2018)
c. VAPP is usually due to type 3
a. 1 mg/kg/day b. 3 mg/kg/day d. Bivalent Vaccine does not include type 2
c. 5 mg/kg/day d. 7 mg/kg/day e. Wild polio type 2 is eradicated since 1999
66. Hand foot mouth disease is usually caused by: 77. Most common cause of neonatal diarrhea worldwide is:
 (FMGE Nov 2017)  (Recent Question 2016)
a. Coxsackie A b. Adenovirus a. Rotavirus b. Adenovirus
c. Rotavirus d. Hepatitis B virus c. Norwalk virus d. Cholera
67. A child presented with Microcephaly, Hepatomegaly 78. Which of the following statement is true for Ebola virus?
and periventricular calcification. Best site from where  (PGI Nov 2015)
specimen should be obtained for diagnosis of CMV by a. It is member of filoviridae
PCR? (AIIMS Nov 2017) b. Incubation period is 2 –12 weeks
a. Urine b. Blood c. Transmitted by direct contact
c. Liver biopsy d. CSF d. Human to human transmission is by air and food
68. Nagayama spot are seen in: (JIPMER Nov 2016)
e. In severe cases lead to renal and liver impairment
79. True regarding JE: (PGI Nov 2015)
a. Measles b. Roseola infantum
c. Mumps d. Rubella a. First case was reported in 2005
b. Transmitted by Culex tritaeniorhynchus
69. All are caused by Parvo virus infection except:
c. Permanent neurologic or psychiatric sequelae
a. Erythema infectiosum (JIPMER Nov 2016) d. Major outbreaks of JE occur every 20 years
b. Gianotti crosti syndrome e. Vaccine available in India
c. Aplastic crisis 80. Amplifying host of Japanese encephalitis is? M 
d. Papular- purpuric gloves and socks syndrome (PPGSS)  (Recent Question 2015)
70. Most common vector of Japanese encephalitis is: M  a. Human b. Pig
 (FMGE pattern 2017) c. Cat d. Dog
a. Anopheles b. Culex 81. Roseola infantum is also known as?
c. Ixodes d. Aedes  (Recent Question 2015)
71. Which of the following is true regarding congenital a. Erythema subitum b. Exanthema subitum
CMV infection? (AIIMS May 2017) c. Erythema nodosum d. Exanthem nodosum
a. Diagnosed only by persistent presence of IgM antibody 82. 24 hours after fever, maculopapular rash and erythema
after 6 months appears on the face of infant. This organism can also
b. It is the most common cause of non-syndromic sensori­ cause: (JIPMER 2014)
neural hearing loss a. ALL b. CML
c. All babies born are symptomatic c. Pure red cell aplasia d. Hairy cell leukemia
d. Mothers of developing countries who transmit the virus 83. Most common cause of infectious croup is: M 
are usually symptomatic  (Recent Question 2013)
Chapter 8: Infectious Diseases 173

a. Adenovirus b. Parainfluenza type I 94. An 8-years-old boy presented with fever and bilateral

Questions
c. RSV d. Influenza cervical lymphadenopathy with prior history of sore
84. A newborn presents with petechiae skin lesions, hema­ throat. There was no hepatomegaly. The peripheral
turia, & platelet count is 22,000/L. Most likely caused blood smear shows > 20% lympho-plasmacytoid cells.
by: (UP 08) The most likely diagnosis is: (AIPGMEE 2002)
a. Congenital CMV infection a. Influenza b. Tuberculosis
b. Congenital rubella infection c. Infectious mononucleosis
c. Premature infants d. Acute lymphoblastic leukemia
d. Both CMV or rubella infection 95. Acute flaccid paralysis is reported in a child aged:
85. Symptomatic neonatal CNS involvement is most a. 0–3 years b. 0–5 years (AI 02)
commonly seen in which group of congenital intrauterine c. 0–15 years d. 0–5 years
infection? (AIPGMEE 08) 96. An 8 years old female child following URTI developed
a. CMV and toxoplasmosis maculopapular rash on the jaw spread onto the trunk
b. Rubella and toxoplasmosis which cleared on the 3rd day without desquamation and
c. Rubella and HSV tender post auricular and suboccipital lymphadenopathy.
d. CMV and syphilis The diagnosis is: (AIIMS May 2001)
86. True about polio: (AIIMS May 08, Nov 07) a. Kawasaki disease b. Erythema infectiosum
c. Rubella d. Measles
a. Paralytic polio is most common
97. A patient who had fever and coryza for last 3 days,
b. Spastic paralysis
developed maculopapular erythematous rash, which
c. IM injections and increased muscular activity lead to
lasted for 48 hours and disappeared without leaving
increased paralysis behind pigmentation. This is most commonly due to:
d. Polio drop given only in < 3 year  (AIIMS June 2000)
87. True regarding infectious mononucleosis are all except:
a. Measles b. Typhoid
 (WBPG 2007) c. Roseola infantum d. Fifth disease
a. Drug induced rash b. Agranulocytosis 98. A child with fever and sore throat developed acute
c. Pharyngeal oedema d. Fatigue cervical lymphadenopathy. Most likely investigation to
88. Which of the following does not establish a diagnosis of be done is: (AIIMS June 2000)
congenital CMV infection in a neonate? a. Open biopsy of node b. Radical neck dissection
 (AIIMS May 06) c. Neck X-ray d. Complete hemogram
a. Urine culture of CMV 99. Most common manifestation of HPV infection in
b. IgG CMV antibodies in blood children: (PGI 2000)
c. Intra-nuclear inclusion bodies in hepatocytes a. Single papilloma b. Multiple papillomatosis
d. CMV viral DNA in blood by polymerase chain reaction c. Osteoma d. Sarcoma
89. Risk of congenital defects in a baby is maximum, if the e. Genital wart
gestational age at the time of maternal rubella infection 100. In neonatal herpes, true is: (PGI June 2000)
is: (Recent Question 2014) a. Caused by HSV-II
a. < 11 weeks b. 11-12 weeks b. Hepatosplenomegaly is diagnostic
c. 13-14 weeks d. 15-16 weeks c. Spontaneous recovery occurs
90. Dengue shock syndrome is characterized by the d. If not treated, disseminates
following except: (AIIMS May 05)
a. Hepatomegaly b. Pleural effusion
 TUBERCULOSIS
c. Thrombocytopenia d. Decreased hemoglobin
101. Most common route of TB infection in a neonate born
91. Which of the following is true about roseola infantum?
by caesarean section: (PGI Nov 2017)
 (PGI Dec 2005, PGI June 2003)
a. Hematogenously spread b. Through skin contact
a. Defervescence following the rash
c. Inhalational of aerosol d. Transplacental
b. Caused by HHV 6 and 7 e. By aspiration of infected amniotic fluid
c. Slapped cheek appearance is seen
102. A 3 month old, asymptomatic infant with, H/o TB
d. Otitis media is common complication exposure, has taken 3 months of chemoprophylaxis,
e. Rash appear first on face and neck what is to be done next? (Recent Question 2016)
92. Which of the following is true about erythema infec­
a. Test sputum and then decide
tiosum? (PGI Dec 2004) b. Continue for 3 more months
a. Slapped cheek appearance seen c. Tuberculin test then decide
b. Caused by parvovirus d. Immunise with BCG & stop prophylaxis
c. Defervescence before rash 103. Which of the following is incorrect regarding miliary
d. Rash initially appears on trunk TB? (Recent Question 2016)
e. Known as ‘fifth disease' a. Usually associated with meningeal TB
93. Kenny packs were used in the treatment of: (AIIMS 03) b. In children it is most commonly due to reactivation
a. Poliomyelitis b. Muscular dystrophy c. Mantoux test may yield a false negative result
c. Polyneuropathies d. Nerve injury d. There is hematogenous dissemination
174 Section 1: General Pediatrics

117. Bull-neck is seen in severe cases of which of the


Review of Pediatrics and Neonatology

104. An asymptomatic newborn whose mother had smear


positive TB received INH for 3 months. What should be following? (AIIMS May 2014)
done next? (Recent Question 2015) a. Diphtheria b. Tubercular lymphadenitis
a. Continue INH for further 3 months c. Mumps d. Goitre
b. Discontinue as the child is asymptomatic 118. All of the following statements are true about diphtheria
c. Do a chest X Ray except: (WBPG 2009)
d. BCG vaccination should not be given a. Dose of antitoxin depends on virulence of organism
b. Myocarditis is a dreaded complication
105. Which of the following statements is true about c. Death is due to circulatory failure
childhood TB? (Recent Question 2014)
d. Pseudomembranes may be found on the vulva/perianal
a. 5% prevalence membrane
b. Common in 5-8 years age
c. Diagnosis by sputum is difficult  CONGENITAL SYPHILIS
d. Highly contagious
106. Breastfeeding should be avoided by a mother on ATT for
how many weeks? (JIPMER 2014) 119. All of the following are manifestations of congenital
syphilis except? (FMGE Dec 2018)
a. 1 b. 2
c. 3 d. 4 a. Gumma b. Hutchinson's teeth
107. A child presents with tuberculosis. Steroids are not c. Olympian brow d. Interstitial keratitis
indicated in: (JIPMER 2014) 120. Hutchinson's triad in congenital syphilis include: M 
a. Tuberculoma b. Endobronchial tuberculosis a. Interstitial keratitis, eight nerve involvement, Hutchinson's
c. Massive pleural effusion teeth (AIIMS Nov 2017)
d. Progressive primary pulmonary disease b. Interstitial keratitis, mulberry teeth, periostitis
108. True about childhood tuberculosis is: (DNB Pattern 2014) c. Periostitis, mulberry teeth, eight nerve involvement
a. 5% prevalence b. Common in 5–8 years age d. Hutchinson's teeth, eighth nerve involvement, periostitis
c. Diagnosis by sputum is easily done 121. All of the following are true regarding congenital
d. Highly contagious syphilis except: (Recent Question 2017)
109. Most common age for pediatric TB is: (DNB Pattern 2014) a. Rhinitis is an early sign
a. <1 year b. 1–4 years b. Notched incisors
c. 5–8 years d. 8–14 years c. Conductive hearing loss
110. Most common organ involved by congenital TB is: M  d. Linear scars at the angles of mouth
 (Recent Question 2013)
122. Distinctive triad of interstitial keratitis, Hutchinson
a. Liver b. Pancreas teeth and eighth nerve deafness is manifestation of
c. Kidney d. Lung  (MAHA PGM CET 2016)
111. TB in children, most common is: (Recent Question 2012) a. Primary syphilis b. Secondary syphilis
a. Abscess b. Consolidation c. Tertiary syphilis d. Congenital syphilis
c. Hilar adenopathy d. CNS tuberculosis
123. In Hutchnson triad, all of the following are seen except: M
112. In which of the following tubercular infection, steroid is
a. Interstitial keratitis b. Cataract (JIPMER 2013)
used? (JIPMER 2010)
c. Deafness d. Notched incisors
a. TB Lymphadenitis b. TB Osteomyelitis
124. Thrombocytopenia, macerated skin lesions, rash and
c. Endobronchial TB d. Neurotuberculosis
periostitis in a newborn are seen in: (COMED 09)
113. First line ATT in children: (PGI June 2005)
a. Erythroblastosis fetalis b. Cytomegalovirus infection
a. Streptomycin b. Pyrazinamide c. Syphilis d. HIV infection
c. Ethionamide d. Ethambutol
125. Premature baby of 34 weeks was delivered. Baby
e. Ofloxacin
developed bullous lesion on the skin and X-ray shows
114. Management of a newborn when mother has active periostitis. What should be the next investigation?
Tuberculosis and is not taking ATT: (AIPGMEE 2000)
a. VDRL for mother & baby (AIIMS Nov 11, AI 07)
a. BCG + Rifampicin + INH + Breastfeeding b. ELISA for HIV c. PCR for TB
b. BCG + Isolation of baby d. Hepatitis surface antigen for mother
c. BCG + INH for 6 week + Breastfeeding
126. Hutchnson's triad is seen in: (Manipal 06)
d. BCG + INH + withhold Breastfeeding
a. Primary syphilis b. Congenital syphilis
c. Secondary syphilis d. Tertiary syphilis
 DIPHTHERIA 127. Congenital syphilis can be best diagnosed by: (AI 01)
a. IgM FTA–ABS b. IgG FTA–ABS
115. Late complication of diphtheria is: (DNB June 2018)
c. VDRL d. TPI
a. Airway obstruction b. Encephalitis
c. Pneumonia d. Neuropathy  ENTERIC FEVER
116. The special stain used to identify C. diphtheriae in a
throat swab is? M  (MAHA PGM CET 2015) 128. Rose spot is associated with (MAHA PGM CET 2016)
a. Albert's stain b. Giemsa stain a. Typhoid fever b. Malaria
c. Gram's stain d. India ink c. Scarlet fever d. Wilson disease
Chapter 8: Infectious Diseases 175

129. Child with rash- wrong is:  (Recent Question 2013) 141. A 6-year-old girl presents with fever for the past 5 days,

Questions
a. Typhus – day 5 b. Varicella – day 1 generalized erythematous rash, strawberry tongue and
c. Typhoid – day 5 d. Measles – day 4 cervical lymphadenopathy. The most likely diagnosis is:
130. 10-years-old child with 10 days continuous fever with  (AIIMS May 2014)
soft, enlarged spleen diagnosis is: (JIPMER 2002) a. Kimura disease b. Kawasaki disease
a. Enteric fever b. Malarial c. Scarlet fever d. Rosie-Dorfman syndrome
c. Hodgkins disease d. Meningitis 142. The diagnosis of a child with small abrasion over face
developing fever with chills, induration and fiery red
 PERTUSSIS swelling over the face and extending to the ear: 
 (Recent Question 2013)

131. Drug of choice for pertussis is? M  (JIPMER May 2016) a. Erysipelas b. Carbuncle
c. Cellulitis d. Anthrax
a. Penicillin b. Ceftriaxone
143. Most common organism responsible for infective endo­
c. Erythromycin d. Azithromycin carditis in children: (Recent Question 2012)
132. A child presents with complaints of cough. Characteristic a. β-hemolytic streptococci b. a-hemolytic streptococci
inspiratory whoop present. He is not immunised. Sample c. Staphylococcus aureus d. Group D. streptococci
for investigation is: (AIIMS Nov 2009) 144. In a 3-year-old child, most common cause of pyomenin­
a. Nasopharyngeal swab b. Tracheal aspiration gitis: (Recent Question 2012)
c. Cough plate culture d. Sputum culture a. H. influenza b. Staph aureus
133. Leucocytosis is seen in which of the following childhood c. Strepto pneumoniae d. E. coli
infections? 145. Most common organism in sickle cell anemia causing
a. Tuberculosis b. Typhoid osteomyelitis is: M  (WBPG 2010)
c. Diphtheria d. Pertussis a. Salmonella b. Streptococci
134. Incubation period of pertussis is: (JIPMER 2006) c. E. coli d. Staphylococci
a. One week b. Two weeks 146. Neonatal meningitis is caused by all except:
c. Three weeks d. Four weeks a. N. meningitis b. E. coli (WBPG 2010)
c. Listeria d. Group B streptococci
 OTHER BACTERIAL INFECTIONS 147. A 6-months-old baby was brought with complaints of
difficulty in feeding. The child was found to be hypotonic
135. Criteria for diagnosing SIRS all seen except: with a weak gag. The child is on breast milk and mother
 (JIPMER 2017) also gives honey to the child during periods of excessive
a. Temp > 38.5 degree C or < 36 degree C crying. The causative agent is: (DNB June 2010)
b. RR > 2 SD above normal for age a. Gram positive aerobic coccus
c. Children < 1 year old with persistent bradycardia over b. Gram positive anaerobic spore bearing bacillus
0.5 hour c. Toxin produced by gram positive anaerobic bacillus
d. > 50% immature neutrophils d. Echovirus
136. Symptoms suggestive of Scarlet fever are all except:- 148. Most common cause of acute otitis media in children is: M
 (JIPMER May 2018)  (JIPMER 2010)
a. Gooseflesh b. Nagayama spots a. Strep pneumoniae b. Strep pyogenes
c. Desquamation of palms d. Strawberry tongue c. Staph aureus d. Pseudomonas
149. A 3-months-old male infant developed otitis media for
137. All of the following are major criteria for staphylococcal
which he was given a course of Cotrimoxazole. A few
toxic shock syndrome, except: (Recent Question 2017) days later, he developed extensive peeling of the skin;
a. Acute fever (Temperature >38.8 deg C (101.8 deg F) there were no mucosal lesions and the baby was not
b. Central nervous system abnormalities toxic. The most likely diagnosis is: (AIIMS May 2004)
c. Hypotension (Orthostatic. shock, blood pressure below a. Toxic epidermal necrolysis
age-appropriate norms) b. Staphylococcal scalded skin syndrome
d. Rash c. Steven Johnson syndrome d. Infantile pemphigus
138. The most common pathogenic organism causing pyo­genic 150. True about tetanus: (PGI June 01)
liver abscess In children is: (Recent Question 2017) a. Tetanus bacilli can spread through blood
a. Staphylococcus aureus b. H Influenzae b. Spread along the nerves c. Requires oxygen
c. Entamoeba Hlstolytic d. Klebsiella d. Incubation period is variable
139. Treatment of choice for pneumonia due to Mycoplasma e. Causes deep wound invasion
is? (WB PGMEE 2016)
 CONGENITAL TOXOPLASMOSIS
a. Azithromycin b. Doxycycline
c. Amoxicillin  d. Ciprofloxacin
151. Risk of transplacental transmission of toxoplasma gondii
140. Most common cause of neonatal meningitis is: is maximum when mother is infected:
 (Recent Question 2013)  (MAHA PGM CET 2016)
a. E. coli b. Listeria a. Prior to conception b. During first trimester
c. Staphylococcus aureus d. Pseudomonas c. During second trimester d. During third trimester
176 Section 1: General Pediatrics

152. The most common manifestation of congenital toxopla­


Review of Pediatrics and Neonatology

159. A child from West Bengal presents with fever &


smosis: M  (Recent Question 2014) unconsciousness for 1 day and pallor with no focal
a. Hydrocephalus b. Chorioretinitis neurodeficit. What is the most probable diagnosis?
c. Hepatosplenomegaly d. Thrombocytopenia  (WB PGMEE 2016)
153. Diagnosis of toxoplasmosis in newborn is done by: a. Cerebral malaria b. Viral encephalitis
a. IgG antibody against toxoplasma  (DNB Pattern 2013) c. Pyomeningitis d. Tubercular meningitis
b. IgM antibodies to toxoplasma
c. IgA antibodies to toxoplasma 160. The following is/are helpful in a child with severe
falciparum malaria with high parasitemia:
d. IgE antibodies to toxoplasma
 (APPG 2014)
154. A pregnant lady had no complaints but mild cervical
a. IV Dexamethasone b. Exchange transfusion
lymphadenopathy in first trimester. She was prescribed
c. Hyperbaric oxygen d. All the above
Spiramycin but she was non-compliant. Baby was born
161. In a rural clinic, a 3-year-old girl child is brought by her
with hydrocephalus and intracerebral calcification. mother and is emaciated. Her hemoglobin was 5 g/dL.
Which of these is likely cause? (AIIMS May 2011) The girl also has edema over her knees and ankles with
a. Rubella b. Toxoplasmosis discrete rash on her knees, ankles and elbows. The most
c. CM d. Herpes likely worm infestation causing these manifestations is:
155. Congenital toxoplasmosis–False is:  (AIIMS May 2010)  (AIIMS May 2014)
a. Diagnosed by detection of IgM in cord blood a. Hookworm b. Roundworm
b. IgA is more sensitive than IgM for detection c. Whipworm d. Pinworm
c. Dye test is gold standard for lgG 162. Treatment for the child with frequent tapeworm
d. Avidity testing must be done to differentiate IgA & IgM infestation: (PGI June 09)
a. Albendazole b. Niclosamide
c. Praziquantel d. Mebendazole
 OTHER PARASITIC INFECTIONS
e. Ivermectin
163. Resistant plasmodium falciparum malaria in the pediatric
156. Which of the following causes mesenteric lymphadenitis?
age group should be treated with: (APPG 2008)
a. Adenovirus  (Recent Question 2017)
b. Campylobacter a. Chloroquine b. Tetracycline
c. Yersinia pseudotuberculosis c. Clindamycin d. Doxycycline
d. All of the above 164. Which microorganism is responsible for classical pres­
entation of hydrocephalus, chorioretinitis, intracerebral
157. Sabin Feldman dye test is used for: M calcification? (APPG 2008)
a. Toxoplasma b. Plasmodium a. Toxoplasmosis b. Rubella
c. Syphilis d. Herpes c. Measles d. CMV
158. What is the most common complication of malaria in
children? (Recent Question 2017)  FUNGAL INFECTIONS
a. Cerebral malaria 165. The most common fungal infection in the neonates
b. Respiratory distress transmitted by caregiver's hands is: (AIIMS May 2014)
c. Severe malarial anemia a. Candida albicans b. Candida glabrata
d. Hypogylcemia c. Candida tropicalis d. Candida parapsilosis
Chapter 8: Infectious Diseases 177

Answers with Explanations


Answers with Explanations
 IMPORTANT VIRAL DISEASES RUBELLA
1. d. Micrognathia  Ref: Nelson’s 20/e p 1551
2. c. Cataract in CRS is always bilateral  Ref: Nelson’s 20/e p 1551
Nerve deafness is the single most common finding among infants with CRS
Salt-and-pepper retinopathy is the most common ocular abnormality but has little early effect on vision
Unilateral or bilateral cataracts are the most serious eye finding, occurring in a third of infants
Cardiac abnormalities occur in 50% of children infected during 1st 8 wk of gestation. Patent ductus arteriosus is the most common
CHD;
The hallmark of fetal Rubella infection is chronic infection that persists throughout fetal life, with shedding of virus up to 1 year of
age.
Late-onset manifestations of CRS are:
•• Progressive rubella panencephalitis (PRP)
•• Diabetes mellitus (20%)
•• Thyroid dysfunction (5%)
•• Glaucoma & visual abnormalities
3. c. Rubella vaccine  Ref: Nelson's 20/e p 1550-1551
4. b. Deafness  Ref: Nelson's 20/e p 1550-1551; Refer Ans 3 below
5. d. All of above  Ref: Nelson's 20/e p 1548-1551

High Yielding Points about Congenital Rubella Syndrome (CRS)


•• Nerve deafness is the single most common finding among infants with congenital rubella syndrome (CRS)
•• Retinal findings described as salt & pepper retinopathy are the most common ocular abnormality
•• Unilateral or bilateral cataracts are the most serious eye finding in CRS; Microcephaly is an important feature
•• Cardiac abnormalities occur in 50% of children infected during the 1st 8 wk of gestation & include PDA > PS > VSD

6. c. Rubella  Ref: Nelson's 20/e p 1548-1551
About the time of onset of rash, oropharynx may reveal tiny, rose-colored lesions (Forchheimer spots) in Rubella.
7. c. ASD  Ref: Nelson's 20/e p 1548-1551
8. c. HSV  Ref: Nelson's 20/e p 1548-1551
For HSV infection most common mode of transmission is direct contact between mucocutaneous surfaces;
Source of virus in neonatal infections is typically the mother, & transmission generally occurs during delivery.
9. d. Increased risk of congenital malformation if infection occur after 16 weeks.  Ref: Nelson's 20/e p 1548-1551
•• Maternal Rubella infection during the 1st 8 wk of gestation results in the most severe & widespread defects
•• Risk for congenital defects is 90% for maternal infection before 11 wk of gestation, 33% at 11-12 wk, 11% at 13-14 wk, and 24%
at 15-16 wk; Defects occurring after 16 wk of gestation are uncommon, even if fetal infection occurs.
10. a. VSD, c. PDA and e. Pulmonary stenosis  Ref: Nelson's 20/e p 1548-1551
11. d. Infection after 16 weeks of gestation results in major congenital defects  Ref: Nelson's 20/e p 1548-1551
12. d. Unborn child  Ref: Nelson's 20/e p 1548-1551
•• The pathologic findings for Congenital Rubella syndrome are often severe and may involve nearly every organ system
•• Postnatal infection with rubella is a mild disease not easily discernible from other viral infections, especially in children
•• Complications following postnatal infection with rubella are infrequent and generally not life-threatening.
13. c. Aortic stenosis  Ref: Nelson's 20/e p 1548-1551

 VARICELLA
14. d. Umbilicated vesicles are the earliest lesions to appear  Ref: Nelson 20/e p1580-1581
•• Initial exanthem in Chicken pox consists of pruritic erythematous macules that evolve through papular stage to form clear, fluid-
filled vesicles. Clouding & umbilication of the lesions begin in 24-48 hr.
•• Simultaneous presence of lesions in various stages of evolution is characteristic of varicella.
•• Distribution of rash is predominantly centripetal with greatest concentration on trunk.
15. c. Anti VZV antibody  Ref: Nelson 20/e p 1581-1583
Cicatrising skin lesions, hypoplasia of all limbs and chorioretinitis are all features of Congenital Varicella syndrome. So, Anti VZV
antibody would be positive in this baby
178 Section 1: General Pediatrics

16. c. Enteritis  Ref: Nelson's 20/e p 1579-1585, Ghai 8/e p 214-215


Review of Pediatrics and Neonatology

Complications of Varicella infection:


•• Hematologic: Mild thrombocytopenia (Petechiae. Purpura) •• CVS: myocarditis, pericarditis
•• Neurologic: Acute cerebellar ataxia, Meningoencephalitis •• GI: pancreatitis, gastrointestinal bleeding
•• Respiratory: Pneumonia, •• Eye: Acute retinal necrosis
•• Renal: Nephritis, nephrotic syndrome, hemolytic uremic •• Metabolic: Reye syndrome
syndrome, hematuria •• Others: arthritis, orchitis, secondary bacterial infections

17. c. Chickenpox  Ref: Nelson's 20/e p 1579-1585, Ghai 8/e p 214-215
Rash appearing on 1st day of fever suggests Chickenpox; Ataxia can occur as a complication of varicella infection.
18. a. Varicella virus  Ref: Nelson's 20/e p 1579-1585, Ghai 8/e p 214-215
19. a. When the lesions have crusted  Ref: Nelson's 20/e p 1579-1585, Ghai 8/e p 214-215, CDC website
•• The period of communicability of varicella begins 1–2 days before the onset of rash and ends when all lesions are
crusted, typically 4–7 days after onset of rash in immunocompetent people, longer in immunocompromised people
•• People with varicella should be isolated until the lesions crust over.
20. d. Rash appears on first day, & e. Rash can occur in axilla  Ref: Nelson's 20/e p 1579-1585, Ghai 8/e p 214-215
21. b. Secondary bacterial infection  Ref: Nelson's 20/e p 1579-1585, Ghai 8/e p 214-215
Secondary bacterial infections of skin, by group A Streptococcus & S. aureus, may occur in children with varicella.

 MEASLES
22. a. Koplik's spots cannot be seen in conjunctival or vaginal mucosa  Ref: Nelson’s 20/e p 1544:
Measles
•• Prodromal phase:fever, conjunctivitis, coryza
•• Koplik spots on inner cheeks at level of 1st& 2nd molars
•• Rash begins behind the ears & spreads downwards
•• Rash fades in 7 days in the same progression as it evolved
Koplik spots
•• Are the pathognomonic sign of measles
•• Appear 1-4 days prior to the onset of the rash
•• First appears as discrete red lesions with bluish white spots in the center on the inner aspects of the cheeks at the level of premolars.
•• May spread to involve lips, hard palate, and gingiva
•• May occur in conjunctival folds and in the vaginal mucosa.
•• Koplik spots have been reported in 50-70% of measles cases
•• Initial targets for measles virus are alveolar macrophages, dendritic cells & lymphocytes
•• Receptor used is signaling lymphocyte activating molecule (CD150)
•• The mechanism of infection of respiratory tissues is attachment to the PVRL4 receptor (Nectin4) that is expressed on cells in the
trachea, oral mucosa, nasopharynx, and lungs.
•• These 2 receptors, CD150 and PVRL4, account for the lymphotropic&epitheliotropic nature of natural measles virus infection.
23. a. Measles virus  Ref: Nelson's 20/e p 1542-1547, Ghai 8/e p 213-214
Subacute sclerosing panencephalitis (SSPE) is a chronic complication of measles with a delayed onset & fatal outcome.
24. c. Otitis media  Ref: Nelson's 20/e p 1542-1547, Ghai 8/e p 213-214
Acute otitis media is the most common complication of measles.
25. c. Measles  Ref: Nelson's 20/e p 1542-1547, Ghai 8/e p 213-214
Fusion of infected cells results in multinucleated giant cells, the Warthin-Finkeldey giant cells that are pathognomonic for measles,
with up to 100 nuclei and intracytoplasmic and intranuclear inclusions.
26. a. Measles   Ref: Nelson's 20/e p 1542-1547, Ghai 8/e p 213-214
90% of exposed susceptible individuals experience measles
27. c. 10-14 days  Ref: Nelson's 20/e p 1542-1547, Ghai 8/e p 213-214
Incubation period of Measles is 8-12 days.
28. a. Roseola infantum  Ref: Nelson's 20/e p 1594-1597; Refer pretext of this chapter for details;
History of high fever & rash appearing as the fever subsides is suggestive of Roseola infantum;
29. d. Measles  Ref: Nelson's 20/e p 1542-1547, Ghai 8/e p 213-214
30. a. Cataract  Ref: Nelson's 20/e p 1542-1547, Ghai 8/e p 213-214
Acute otitis media, Croup, tracheitis, bronchiolitis, sinusitis, mastoiditis, encephalitis & meningitis are common complications in
infants & toddlers with measles; Pneumonia is the most common cause of death in measles.
31. c. Long term complications follow in form of SSPE and e. Caused by RNA virus  Ref: Nelson's 20/e p 1542-1547
Measles virus is a single-stranded, lipid-enveloped RNA virus in the family Paramyxoviridae and genus Morbillivirus.
32. a. Koplik spots appear in prodromal stage, b. Fever stops after the onset of rash & c. Vaccine given at 9 months
Ref: Nelson's 20/e p 1542-1547, Ghai 8/e p 213-214
Chapter 8: Infectious Diseases 179

Measles consists of 4 phases: incubation period, prodromal illness, exanthematous phase, and recovery.

Answers with Explanations


•• Incubation period usually lasts for 8-12 days.
•• Prodromal phase: mild fever followed by conjunctivitis, coryza, fever & Koplik spots (in 50- 70%).
•• Exanthematous phase: Rash begins on forehead, behind the ears, as a red maculopapular eruption.
•• Recovery: With onset of rash, symptoms begin to subside; rash fades over 7 days leaving a fine desquamation.
33. d. Measles  Ref: Nelson's 20/e p 1542-1547, Ghai 8/e p 213-214

 MUMPS
34. d. Mumps  Ref: Nelsons' 20/e p 1552-1553, Ghai 8/e p 217-218
Mumps is characterized by fever, bilateral or unilateral parotid swelling and tenderness, and the frequent occurrence of
meningoencephalitis and orchitis.
35. d. Enterovirus  Ref: Nelsons' 20/e p 2946
Enterovirus is the most common cause of meningoencephalitis in children.
36. c. Meningoencephalitis can precede parotitis  Ref: Nelsons' 20/e p 1552-1553, Ghai 8/e p 217-218
a. False; Mumps virus may involve salivary glands (parotid ± submandibular), CNS, pancreas, testes
b. False; Virus appears in saliva from up to 7 days before to 7 days after onset of parotid swelling
c. True; Meningoencephalitis may occur before, along with, or following the parotitis

d. False; Mumps orchitis can lead to atrophy of testes, but Sterility is rare even with bilateral involvement

37. b. Mumps  Ref: Nelsons' 20/e p 1552-1553, Ghai 8/e p 217-218


CNS complications of Mumps include Meningoencephalitis, transverse myelitis, aqueductal stenosis & facial palsy.
38. c. 14–24 days  Ref: Nelsons' 20/e p 1552-1553, Ghai 8/e p 217-218
The incubation period for mumps ranges from 12-25 days but is usually 16-18 days.
39. b. Aseptic meningitis  Ref: Nelsons' 20/e p 1552-1553, Ghai 8/e p 217-218
Common complications of Mumps:
•• Most common complications of mumps are meningitis, with or without encephalitis, and gonadal involvement
•• In adolescent and adult males, orchitis is second only to parotitis as a common finding in mumps.
•• Orchitis is extremely rare in prepubescent boys, but occurs in 30-40% of males after puberty.
40. d. Incubation period is less than 2 weeks  Ref: Nelsons' 20/e p 1552-1553, Ghai 8/e p 217-218
Mumps virus is a single stranded RNA virus in the family Paramyxoviridae and the genus Rubulavirus.
41. b. Aseptic meningitis  Ref: Nelsons' 20/e p 1552-1553, Ghai 8/e p 217-218
42. a. Orchitis and oophritis  Ref: Nelsons' 20/e p 1552-1553, Ghai 8/e p 217-218

 HIV
43. a. HIV DNA PCR, c. HIV culture, e. HIV RNA PCR
Diagnosis of HIV in infants & children
•• Presence of IgA or IgM anti-HIV in the infant's circulation can indicate HIV infection
•• In any child > 18 months age, demonstration of IgG antibody to HIV by ELISA & Western blot test establishes HIV infection
•• Breastfed infants should have antibody testing performed 12 wk following cessation of breastfeeding
•• HIV DNA or RNA PCR or HIV culture, are useful in young infants, to reach to a definitive diagnosis
44. c. PCR  Ref: Update on antiretroviral regimens for treating and preventing HIV infection and update on early infant
diagnosis of HIV July 2018 policy brief HIV treatment–interim guidance
•• WHO recommends that HIV virological testing (HIV DNA or RNA PCR or HIV culture) be used to diagnose HIV infection among
infants and children younger than 18 months
•• In any child > 18 mo of age, demonstration of IgG antibody to HIV by ELISA & Western blot test establishes HIV infection
45. c. Immune reconstitution inflammatory syndrome  Ref: Nelsons' 20/e p 1650
Immune reconstitution inflammatory syndrome (IRIS)
•• Characterized by increased inflammatory response subclinical opportunistic infections
•• It is more common in patients with progressive disease & severe CD4 + T-lymphocyte depletion.
•• There is fever & worsening of clinical manifestations of opportunistic infection or new manifestations (e.g., enlargement of lymph
nodes, pulmonary infiltrates), typically within the 1st few weeks after initiation of antiretroviral therapy.
46. c. Daily Nevirapine prophylaxis at birth and continued for 12 weeks if mother received ART regularly in antenatal
period  Ref: Updated PPTCT guidelines by NACO, Dec 2013; Refer pretext of this chapter for details;
Nevirapine prophylaxis is to be given at birth & continued for 6 weeks (not 12 weeks) if mother received ART regularly in
antenatal period
47. d. < 15%  Ref: Nelson's 20/e p 1645-1665, Ghai 8/e p 229-237, CDC guidelines 2014;
180 Section 1: General Pediatrics
Review of Pediatrics and Neonatology

Severity of immune suppression based on CD4 levels children


HIV-associated Age-related CD4 + cell values
immunodeficiency ≤ 11 month 12–35 month 36–59 month ≥ 5 year
Mild 30–35% 25–30% 20–25% 350–499 cells/mm3
Advanced 25–30% 20–25% 15–20% 200–349 cells/mm3
Severe < 25% or < 1500 cell/ < 20% of < 750 cells/ < 15% or < 350 cells/ < 200 cells/mm3 or
mm3 mm3 mm3 < 15%

Note: The answer given in the Maharashtra PG CET 2016 key was <15%, as per the above grading, which was previously used; Refer
pretext of this chapter for latest WHO 2016 recommendations on treatment of HIV
48. a. Anal cancer b. Hodgkins lymphoma  Ref: CDC website
Important AIDS defining conditions according to CDC are:

•• Candidiasis of bronchi, trachea, lungs or esophagus •• Histoplasmosis, disseminated or extrapulmonary


•• Coccidioidomycosis, Cryptococcosis, Cryptosporidiosis or •• Kaposi sarcoma/Burkitt lymphoma/Primary CNS Lymphoma
Isosporiasis •• Lymphoid interstitial pneumonia or Pneumocystis jirovecii pneu­monia
•• CMV disease (other than liver, spleen, nodes), CMV retinitis •• Mycobacterium avium complex or Mycobacterium kansasii (extra­
(with loss of vision) pulmonary)
•• HIV related Encephalopathy/Progressive multifocal •• Mycobacterium tuberculosis of any site
leukoencephalopathy/CNS Toxoplasmosis
•• HSV: chronic ulcers, bronchitis, pneumonitis, or eso­phagitis
(onset at age >1 month)
49. a. Nevirapine  Ref: Nelson's 20/e p 1645-1665, Ghai 8/e p 229-237; Refer pretext of this chapter for details;
50. b. HIV DNA PCR positive at 3 month confirms diagnosis and d. Passive transfer of maternal body generally persists for
12 months  Ref: Nelson's 20/e p 1645-1665, Ghai 8/e p 229-237; Refer pretext for details;
51. b. Stage 2  Ref: Nelson's 20/e p 1645-1665; WHO 2016 ART guidelines;
In WHO Clinical Stage 1, the patient is asymptomatic or has persistent generalised lymphadenopathy
Unexplained persistent parotid enlargement is included in WHO Clinical Stage 2; Refer pretext of this chapter for details;
52. d. Lymphadenopathy  Ref: Nelson's 20/e p 1645-1665, Ghai 8/e p 229-237
•• In most infants with HIV infection, physical examination at birth is normal.
•• Initial symptoms may be subtle, such as lymphadenopathy and hepatosplenomegaly, or nonspecific, such as failure to thrive,
chronic or recurrent diarrhea, respiratory symptoms, or oral thrush, distinguishable only by their persistence.
53. a. Generalised lymphadenopathy  Ref: Nelson's 20/e p 1645-1665, Ghai 8/e p 229-237
54. None  Ref: NACO guidelines for ART in children, 2013; WHO 2016 guidelines for ART;
NACO 2013 recommendations for ART in infants & children:
Age Clinical stage Immunological
< 24 months Treat all
> 24 months WHO stage 3 & 4 Treat all
WHO stage 1 & 2 Treat when CD4 count at 2-5 years is < 750/mm3 & at > 5 years: < 350/mm3

As per Latest WHO recommendations (2016), ART should be initiated in all children living with HIV, regardless of WHO
clinical stage or at any CD4 cell count;
55. None;  Ref: WHO 2016 guidelines for ART;

Cotrimoxazole prophylaxis is recommended for infants, children & adolescents with HIV, irrespective of clinical and immune
conditions. Dose: 150 mg/m2/day of TMP & 750 mg/m2/day of SMZ (maximum: 320/1,600 mg), 3 days/wk.
Benefits of Cotrimoxazole prophylaxis are prevention of some AIDS-associated opportunistic diseases (Pneumocystis jirovecii

pneumonia & toxoplasmosis), reduction of HIV-associated mortality in people with low CD4 cell counts, prevention of
malaria & severe bacterial infections in children with HIV
56. Ans None;  Ref: WHO guidelines on HIV & Infant feeding 2016; Refer pretext of this chapter for details;
57. a. Cleaning mother's vagina with antiseptic lotion d. ART prophylaxis  Ref: Nelson's 20/e p 1645-1665, Ghai 8/e p 229-
237; Latest Updated guidelines for PPTCT by NACO, December 2013.
This has become a controversial question now. As per previous PPTCT guidelines, Elective caesarean section helped in preventing
perinatal transmission of HIV. But as per the latest recommendations, Caesarean sections in HIV positive pregnant women should be
performed for obstetric indications only;
58. c. Nevirapine to baby up to 6 weeks; d. Zidovudine to baby  Ref: Nelson's 20/e p 1645-1665
Chapter 8: Infectious Diseases 181


ART in antenatal period:

Answers with Explanations


•• Start ART as soon as possible & continue throughout pregnancy, delivery, breastfeeding period and thereafter lifelong
•• Even if the pregnant women presents very late in pregnancy, ART should be initiated promptly
•• Recommended 1st-line regimen in pregnancy is Tenofovir (300 mg) + Lamuvidine (300 mg) + Efavirenz (600 mg)
59. a. HIV  Ref: Nelson's 20/e p 1645-1665, Ghai 8/e p 229-237
60. a. OPV  Ref: IAP guidebook on Immunisation 2013-2014, p 365-367

IAP, WHO, AAP, ACIP, and CDC recommend:
•• All the live vaccines in asymptomatic HIV1 infected children except BCG and OPV
•• However, in a symptomatic child all live vaccines are forbidden
•• Yellow fever vaccine is contraindicated in both symptomatic and asymptomatic.
61. d. Syrup Nevirapine 2 mg/kg body weight to newborn within 72 hours of birth  Ref: Nelson's 20/e p 1645-1665, Ghai 8/e p
229-237; WHO guidelines 2014; Refer pretext for details;
62. a. Nevirapine  Ref: Nelson's 20/e p 1645-1665, Ghai 8/e p 229-237
63. a. ELISA for HIV IgG antibody  Ref: Nelson's 20/e p 1645-1665, Ghai 8/e p 229-237
64. c. HIV ELISA  Ref: Nelson's 20/e p 1645-1665, Ghai 8/e p 229-237

 OTHER VIRAL INFECTIONS


65. b. 3 mg/kg/day  Ref: CDC recommendations
If a child is younger than 3 months old, use of oseltamivir for
chemoprophylaxis is not recommended unless situation is judged critical
due to limited data in this age group.
If child is 3 months or older and younder than 1 year old1
Chemoprophylaxis •• 3 mg/kg/dose once daily2
for H1N1 infection If 1 year or older, dose varies by child’s weight:
•• 15 kg or less, the dose is 30 mg once a day
•• >15 to 23 kg, the dose is 45 mg once a day
•• > 23 to 40 kg, the dose is 60 mg once a day
•• > 40 kg, the dose is 75 mg once a day

66. a. Coxsackie A  Ref: Ghai 9/e p 215


67. a. Urine  Oski's Pediatrics 4/e p 514
The diagnosis of congenital CMV infection is best confirmed by isolation of the virus from urine
PCR assays of dried blood spots from infants have not been shown to have sufficient sensitivity for the identification of most infants
with congenital CMV infection
Saliva PCR assays are currently being assessed as a useful screening method for congenital CMV infection.
68. b. Roseola infantum  Ref: Nelson 20/e p 1594-1596
Ulcers at uvulopalatoglossal junction (Nagayama spots) are commonly reported in infants with roseola infantum. High fever is the
most consistent finding associated with primary HHV-6B infection.
69. b. Gianotti crosti syndrome  Ref: Nelson 20/e p 1570, 1588
Diseases caused by Parvo virus B19 infection include: Erythema infectiosum (most common), Transient arrest of erythropoiesis &
papular purpuric “gloves-and-socks” syndrome (PPGSS)
Gianotti-Crosti syndrome, a symmetric, atopic dermatitis like rash on cheeks with multiple erythematous papules is associated with
EBV infection.
70. b. Culex  Ref: Nelson 20/e p 1627
Culex tritaeniorhynchus summarosus, a nightbiting mosquito that feeds preferentially on large domestic animals & birds but only
infrequently on humans, is the principal vector of JE in northern Asia.
71. b. It is the most common cause of non-syndromic sensorineural hearing loss  Ref: Nelson 20/e p 1592-1593, 3074
•• Diagnosis of congenital CMV infections requires the recovery of replicating virus and/or viral nucleic acids within 1st 3 wk of life;
IgM assays can also be used, but they have limited sensitivity.
•• Congenital infection with CMV presents with symptoms in only 10% of infected newborns;
•• Hearing loss is the most common long-term sequela associated with congenital CMV infection;
•• Most common infectious cause of congenital SNHL is CMV;
•• In developing countries, most of the adults are already seropositive for CMV
72. a. Exanthema subitum  Ref: Nelson's 20/e p 1594-1597; Refer pretext of this chapter for details;
73. c. Parvo virus B19  Ref: Nelson's 20/e p 1570-1571
182 Section 1: General Pediatrics

Papular purpuric “gloves-and-socks” syndrome (PPGSS) is a self-limited condition associated with Parvovirus B19 infection
Review of Pediatrics and Neonatology

& is characterized by fever, pruritus, painful edema & erythema localized to distal extremities in a distinct “gloves-and-socks”
distribution, followed by acral petechiae & oral lesions.
74. d. Respiratory syncytial virus  Ref: Nelson 20/e p 1608
Drug Use in RSV infection
Palivi- It is a monoclonal antibody licensed for prophylaxis in high-risk infants during the RSV season, and does prevent
zumab about half of the expected hospitalizations in that population.
Ribavirin It is an antiviral agent delivered as an aerosol that showed a modest beneficial effect on the course of RSV pneumonia,
with some reduction in the duration of both mechanical ventilation and hospitalization. However, latest studies have failed
to document a clear beneficial effect of ribavirin, and therefore this drug is no longer used for routine therapy of RSV
disease.

75. b. Sensorineural hearing loss  Ref: Nelson 20/e p 1592-1594
Hearing loss is the most common long-term sequela of congenital CMV infection.
76. b. Wild polio virus mostly is type 1, d. Bivalent does not include type 2, e. Wild polio type 2 is eradicated since 1999 
Ref: IAP guidebook on Immunisation 2013-14; Refer pretext ch 10 for details of VAPP & VDPV;
a. False Since 2006, majority of cVDPV cases are due to type 2.
b. True Type 2 poliovirus was declared eradicated in September 2015, with the last virus detected in India in 1999. Type 3 wild
poliovirus has not been detected anywhere in the world since November 2012.
c. False Type 3 has been more often associated with recipient VAPP, type 2 has been more often observed in immunodeficient
individuals and cases with contact VAPP, and type 1 has been less often associated with VAPP cases
d. True Bivalent OPVs mostly contain type 1 & type 3 strains
e. True See option b explanation above;

77. a. Rotavirus  Ref: Nelson's 20/e p 1616
In early childhood, the single most important cause of severe dehydrating diarrhea is rotavirus infection.
78. a. It is member of filoviridae, c. Transmitted by direct contact & e. In severe cases lead to renal and liver impairment
Ref: Nelson's 20/e p 1634-1635; Harrison's 19/e p 1323
a. True Family Filoviridae includes 3 genera: Cuevavirus, Ebolavirus & Marburg virus
b. False Incubation period of Ebola virus disease is 2-21 days
c. True Natural human-to-human transmission takes place through direct person-to-person (usually skin-to-skin) contact or
exposure to infected bodily fluids & tissues; there is no evidence of such transmission by aerosol or respiratory droplets.
d. False
e. True Multi organ failure is seen in severe cases;

79. b. Transmitted by Culex tritaeniorhynchus, c. Permanent neurologic or psychiatric sequelae & e. Vaccine available in
India;  Ref: Nelson's 20/e p1627-1628; Refer pretext of this chapter for details;
a. False Epidemics of Japanese encephalitis were reported in Japan from the late 1800s
b. True Culex tritaeniorhynchus, a night-biting mosquito transmits JE in Asia
c. True Sequelae are seen in 5-70% of JE cases & include mental deterioration, severe emotional instability, personality
changes, motor abnormalities, and speech disturbances.
d. False Over the past decade, there has been a pattern of steadily enlarging recurrent seasonal outbreaks in Vietnam, Thailand,
Nepal & India
e. True Both live attenuated & killed JE vaccines are available in India & JE vaccine is given as a part of our national immuniza-
tion schedule in selected districts of India where it is endemic;

80. b. Pig  Ref: Nelson's 20/e p 1627, Ghai 8/e p 201
Pigs serve as an amplifying host in Japanese encephalitis.
81. b. Exanthema subitum  Ref: Nelson's 20/e p 1596, Ghai 8/e p 201; Refer pretext for details;
82. c. Pure red cell aplasia  Ref: Nelson's 20/e p 1568-1571
The described lesion is slapped cheek rash seen in Erythema infectiosum caused by Parvovirus B19. Parvovirus B19 also causes Pure
red cell aplasia.
83. b. Parainfluenza type I  Ref: Ghai 8/e p 368-376; Refer pretext of chapter on Respiratory disorders for details;
84. d. Both CMV or Rubella infection  Ref: Nelson's 20/e p 1548-1551
Both congenital CMV & rubella infections can cause thrombocytopenia & Purpura due to thrombocytopenia.
Chapter 8: Infectious Diseases 183

85. c. Rubella and HSV  Ref: Nelson's 20/e p 1548-1551

Answers with Explanations


Symptomatic CNS infections in neonatal period are most commonly seen in HSV (75–100%) f/b Rubella (30–60%);
CMV, toxoplasma, and syphillis become symptomatic in less than 10%–15% of cases.
Characteristic clinical features of Intrauterine Infections

Infection IUGR HSM Bleeding Eye signs CNS features Others


Conge­ 60–70% 2/3rd of “Blue berry “Cottage cheese with Microcephaly, periven- Anemia, sensorineural
nital CMV patients muffin Ketchup” chorioretinitis, tricular calcification, deafness, pneumonia
disease spots” sparing of macula, optic mental retardation
atrophy
Conge­ 60% 60–70% 50–60% Cataracts, glaucoma, Meningoencephalitis, Cardiac defects like
nital corneal opacity, salt & deafness, mental PDA, PS, VSD; bone
Rubella pepper chorioretinitis retardation lesions
Conge­ 20–30% 40–50% 30–40% Chorioretinitis with macu- Hydrocephalus, Strabismus, nystagmus,
nital Toxo- lar involvement, optic microcephaly, Diffuse visual impairment
plasmosis atrophy, cataract, uveitis cerebral calcification
Conge­ Uncom- 30–40% Uncommon Interstitial Keratitis, Cho- Meningoencephalitis Rashes, snuffles,
nital mon rioretinitis periosteitis, chondritis,
syphilis deafness
Conge­ LBW Uncommon Uncommon chorioretinitis, microph- microcephaly, cortical cicatricial skin scarring in
nital common thalmia, and cataracts atrophy, seizures, ID, a zoster-like distribution;
Varicella neurogenic bladder, limb hypoplasia; hydrone-
swallowing dysfunction phrosis
Herpes None 50% Uncommon Keratitis, Conjunctivitis, Meningoencephalitis, Vesicles, Pneumonia,
simplex Chorioretinitis, Cataracts microcephaly, Intracra- Hepatitis, mucocutaneous
nial Calcification lesions

86. c. IM injections and increased muscular activity lead to increased paralysis  Ref: Nelson's 20/e p 1554-1560
a. False 90-95% of polio infections are asymptomatic
b. False Acute flaccid paralysis is seen in Polio & not spastic paralysis
c. True Risk of paralytic polio is increased by Tonsillectomy, Strenous physical exercise, Tooth extraction, Cortisone
administration, Adenoidectomy, Injection (intramuscular)
d. False OPV drops are given to all children less than 5 yr age in pulse polio immunisation

87. b. Agranulocytosis  Ref: Nelson's 20/e p 1586-1588; Refer pretext for details;
88. b. IgG CMV antibodies in blood  Ref: Nelson's 20/e p 1590-1593
Diagnosis of Congenital CMV infection
Virus isolation Serology PCR for viral genome (DNA)
Definitive diagnostic method Not very useful Urine or Blood may be used;
Specimens that can be used are urine, IgM test can be done but lacks sensitivity Quantitative estimation possible
saliva, blood, Bronchoalveolar washing & and specificity
Amniotic fluid IgG tests are not diagnostic

About Option C: CMV infected cells contain large intranuclear and smaller intracytoplasmic inclusions which are pathognomic.
89. a. < 11 weeks  Ref: Nelson's 20/e p 1548-1551
Risk of congenital defects in a baby is maximum, if mother acquires Rubella in the first trimester (before 11 weeks).
90. d. Decreased hemoglobin  Ref: Nelson's 20/e p 1629; Refer pretext for details;
In Dengue hemorrhagic fever, there is hemoconcentration, due to increased vascular permeability, so the hematocrit & hemoglobin
level increases & not decreases;
91. b. Caused by HHV 6 and 7 & d. Otitis media is common complication   Ref: Nelson's 20/e p 1595-1596
92. a. Slapped cheek appearance seen, b. Caused by parvovirus and e. Known as ‘fifth disease'  Ref: Nelson's 20/e p 1568-
1571, Ghai 8/e p 216; Refer pretext of this chapter for details about Erythema Infectiosum;
93. a. Poliomyelitis  Ref: Nelson's 20/e p 1554
There is no specific treatment for Poliomyelitis; Management is primary symptomatic and supportive & includes:
•• Bed rest; as physical activity & trauma increase risk of paralysis & optimum position of limbs—to prevent deformity
•• Relief of pain & muscle spasm: Hot moist packs applied to muscles (sister Kenny's treatment); Physiotherapy;
•• Rehabilitation: Physical, emotional and psychological.
94. c. Infectious mononucleosis  Ref: Nelson's 20/e p 1586; Refer pretext of this chapter for details;
184 Section 1: General Pediatrics

95. c. 0–15 years  Ref: Nelson's 20/e p 1554


Review of Pediatrics and Neonatology

For AFP Surveillance, every case of acute flaccid paralysis in any child under 15 years is to be reported promptly.
96. c. Rubella  Ref: Nelson's 20/e p 1548
97. c. Roseola infantum  Ref: Nelson's 20/e p 1568; Refer pretext of this chapter for details;
98. d. Complete hemogram  Ref: Nelson's 20/e p 1560
•• Cervical lymphadenopathy with signs of infection such as fever and sore throat points towards any infectious etiology
•• In this case the most likely investigation to be done should be complete hemogram.
99. b. Multiple papillomatosis  Ref: Nelson's 20/e p 1619
Juvenile laryngeal papillomatosis (also referred to as respiratory papillomatosis):
•• Due to HPV infection acquired by infants during passage through an infected birth canal
•• Median age at diagnosis of recurrent laryngeal papillomatosis is 3 yr; presents with hoarseness, altered cry ± stridor.
100. a. Caused by HSV-II and d. If not treated, disseminates  Ref: Nelson's 20/e p 1572-1575
Neonatal Herpes
•• It is commonly caused by HSV-2 (70% cases are caused by HSV-2 and 30% caused by HSV-1)
•• Neonatal infections usually present on 6th day postpartum can present as: Localized skin, eye and mouth infection, CNS infection, or
disseminated infection; vesicular ulcerative skin lesions occur in only 30–43% children;
•• Pathogenesis of HSV infection in newborns is complicated by their relative immunologic immaturity.
•• With prompt antiviral therapy, virus replication may be restricted to site of inoculation (skin, eye, or mouth).

 IMPORTANT BACTERIAL INFECTIONS TUBERCULOSIS


101. c. Inhalational of aerosol  Ref: Nelson’s 20/e p 1454
Congenital TB
•• Congenital transmission usually occurs from a lesion in the placenta through the umbilical vein, when tubercle bacilli infect the
fetal liver, where a primary focus with periportal lymph node involvement can occur.
•• Congenital tuberculosis can also be caused by aspiration or ingestion of infected amniotic fluid.
However, the most common route of infection for the neonate is postnatal airborne transmission from an adult with infectious
pulmonary tuberculosis.
102. b. Continue for 3 more months  Ref: National Guidelines on Pediatric Tuberculosis, 2012; Refer pretext;
A child born to mother who was diagnosed to have TB in pregnancy should receive prophylaxis for 6 months, provided
congenital TB has been ruled out; BCG vaccination can be given at birth even with INH chemoprophylaxis.
103. b. In children it is most commonly due to reactivation  Ref: Nelson's 20/e p 1450-1453; Harrison's 19/e p 1103-1113
a. True In >50% cases of Tubercular meningitis, evidence of old pulmonary lesions or a miliary pattern is found on chest X ray
b. False Miliary tuberculosis usually complicates primary infection (within 2-6 mo of initial infection) in infants & children; while,
in adolescents & older adults, it usually results from the breakdown of a previously healed primary pulmonary lesion.
c. True In severe cases of TB like military TB, Mantoux test may be falsely negative
d. True Miliary TB is usually due to lymphohematogenous dissemination of tubercle bacilli

104. a. Continue INH for further 3 months  Ref: Ghai 8/e p 250-258; Indian Pediatr 2013;50:301-306
105. c. Diagnosis by sputum is difficult > b. Common in 5-8 years age Ref: Nelson's 20/e p 1445-1460
Option a. False Prevalence of childhood TB is 10-20% of all TB cases
Option b. True Most common in children less than 5 years age; but it is also common in 5-14 years age group
Option c. True Children with TB are usually sputum negative; Moreover, collecting sputum sample is difficult in children
Option d. False Childhood TB is usually a paucibacillary disease, so it is rarely contagious

106. c. 3  Ref: Nelson's 20/e p 1445-1460, Ghai 8/e p 250-258
•• Once the mother and child are taking adequate therapy, it is usually safe for the mother to breastfeed, as the medications, although
found in milk, are present in low concentrations
•• AAP recommends continued feeding with expressed milk in mothers with pulmonary TB who are contagious, untreated or
treated (< 3 wk) with isolation.
•• WHO recommends breastfeeding under all circumstances, however, close contact with the baby should be reduced
107. d. Progressive primary pulmonary disease  Ref: Nelson's 20/e p 1445-1460, Ghai 8/e p 250-258
Indications of Corticosteroids in TB:
•• Tuberculous meningitis: reduces vasculitis, inflammation & ICP •• Tuberculous massive pleural effusion
•• Endobronchial tuberculosis •• Miliary tuberculosis
•• Tuberculous pericardial effusion

108. b. Common in 5–8 years age  Ref: Nelson's 20/e p 1445-1460, Ghai 8/e p 250-258
Chapter 8: Infectious Diseases 185

109. b. 1–4 years  Ref: Nelson's 20/e p 1445-1460, Ghai 8/e p 250-258

Answers with Explanations


110. a. Liver  Ref: Nelson's 20/e p 1445-1460, Ghai 8/e p 250-258
111. c. Hilar adenopathy  Ref: Nelson's 20/e p 1445-1460, Ghai 8/e p 250-258
112. c. Endobronchial TB  Ref: Nelson's 20/e p 1445-1460, Ghai 8/e p 250-258
113. a. Streptomycin, b. Pyrazinamide and d. Ethambutol  Ref: Nelson's 20/e p 1445-1460, Ghai 8/e p 250-258
114. d. BCG + INH + withhold Breastfeeding  Ref: Nelson's 20/e p 1445-1460, Ghai 8/e p 250-258
Breastfeeding is withheld only till the mother is infective (usually for around 3 weeks after starting ATT).

 DIPHTHERIA
115. d. Neuropathy  Ref: Nelson’s 20/e p 1347
Cranial neuropathies
Characteristically occur in the 5th wk, leading to oculomotor and ciliary paralysis, which can cause strabismus, blurred vision, or
difficulty with accommodation.
116. a. Albert's stain  Ref: Nelson's 20/e p 1345-1348, Ghai 8/e p 241-242
117. a. Diphtheria  Ref: Nelson's 20/e p 1345-1348, Ghai 8/e p 241-242
118. a. Dose of antitoxin depends upon the virulence of the organism.   Ref: Nelson's 20/e p 1345-1348
Diphtheria Antitoxin is administered as a single empirical dose of 20,000- 100,000 units based on the degree of toxicity, site and size
of the membrane, and duration of illness.

 CONGENITAL SYPHILIS
119. a. Gumma  Ref: Nelson’s 20/e p 1470-1476
Hutchinson's teeth, Olympian brow & Interstitial keratitis are features of congenital syphilis.
120. a. Interstitial keratitis, eight nerve involvement, Hutchinson's teeth
121. c. Conductive hearing loss  Ref: Nelson 20/e p 1472
Sensorineural hearing loss (due to 8th CN involvement) is seen in Syphilis;
122. d. Congenital syphilis  Ref: Nelson's 20/e p1470-1477, Ghai 8/e p273
123. b. Cataract  Ref: Nelson's 20/e p1470-1477, Ghai 8/e p273
124. c. Syphilis  Ref: Nelson's 20/e p1470-1477, Ghai 8/e p273
All the given features are early manifestations of Congenital Syphilis; Refer pretext of this chapter for details;
125. a. VDRL for mother and baby  Ref: Nelson's 20/e p1470-1477, Ghai 8/e p273
Bullous lesion and periostitis suggest the diagnosis of congenital syphilis. So, VDRL is done for mother & baby;
126. b. Congenital syphilis  Ref: Nelson's 20/e p1470-1477, Ghai 8/e p273
Hutchinson‘s triad seen in Congenital syphilis consists of: Interstitial keratitis, Hutchinson teeth & Eight nerve deafness.
127. a. IgM FTA–ABS  Ref: Nelson's 20/e p1470-1477, Ghai 8/e p273
As IgM does not cross placenta, its presence in neonatal serum confirms congenital syphilis & helps differentiating it from seropositivity
due to passively transferred maternal antibodies; Tests to detect IgM are FTA-ABS & Syphilis Capita M test.

 ENTERIC FEVER
128. a. Typhoid fever  Ref: Nelson's 20/e p 1390
Skin rash in Enteric fever:
•• In 25% of cases, a macular or maculopapular rash (rose spots) may be visible around the 7th-10th day of the illness
•• Lesions may appear in crops of 10-15 on the lower chest and abdomen and last 2-3 days
•• These lesions may be difficult to see in dark-skinned children.
129. c. Typhoid – day 5  Ref: Nelson's 20/e p 1390
130. a. Enteric fever  Ref: Nelson's 20/e p 1390

 PERTUSSIS
31. d. Azithromycin  Ref: Nelson's 20/e p 1379-1380
1
Macrolides are preferred agents; Due to increased risk of infantile hypertrophic pyloric stenosis in neonates treated with oral
erythromycin, Azithromycin is the preferred agent in all age groups.
132. a. Nasopharyngeal swab  Ref: Nelson's 20/e p 1377-1381, Ghai 8/e p 242-243
For diagnosis of Whooping cough isolation of B. pertussis in culture remains the gold standard
•• The specimen is obtained with deep nasopharyngeal aspiration or a swab
•• Cough plates and postnasal swabs are unsatisfactory because of overgrowth by commensal bacteria
•• DNA detection by PCR is replacing culture because of increased sensitivity & quicker results.
186 Section 1: General Pediatrics

133. d. Pertussis  Ref: Nelson's 20/e p 1377-1381, Ghai 8/e p 242-243


Review of Pediatrics and Neonatology

134. a. One week  Ref: Nelson's 20/e p 1377-1381, Ghai 8/e p 242-243
Incubation period for Pertussis ranges from 3-12 days.

 OTHER BACTERIAL INFECTIONS


135. > 50% immature neutrophils  Ref: Nelson’s 20/e p 522
It is an inflammatory cascade that is initiated by the host response to an infectious or non-infectious trigger.
2 of 4 criteria, 1 of which must be abnormal temp. or abnormal WBC count:
1. Core temperature >38.5°C (101.3°F) or <36°C (96.8°F) (rectal, bladder, oral, or central catheter)
2. Tachycardia:
•• Mean heart rate >2 SD above normal for age in absence of external stimuli, chronic drugs or painful stimuli or
•• Unexplained persistent elevation over 0.5-4 hr or
•• In children <1-year-old, persistent bradycardia over 0.5 hr (mean heart rate <10th percentile for age in absence of vagal
stimuli, β-blockers, or cong. heart disease)
3. Respiratory rate >2 SD above normal for age or acute need for mechanical ventilation not related to neuromuscular disease or
general anesthesia
4. Leukocyte count elevated or depressed for age (not secondary to chemotherapy) or >10% immature neutrophils.
136. b. Nagayama spots  Ref: Nelson’s 20/e p 1330
Scarlet fever
•• Caused by Group A Streptococcus
•• Pharyngitis & Strawberry tongue is seen
•• Erythematous rash appears 24-48 hr after onset
•• Skin has a goose-pimple appearance and feels rough
•• Desquamation of palms & soles seen
•• Identification of GAS in the pharynx confirms the diagnosis
Roseola infantum
•• Also known as exanthemsubitum and sixth disease
•• Caused by human herpesvirus 6 or 7 (MC in 6 months to 4 years age)
•• Begins with a sudden high fever (101-105°F) that lasts for 2-5 days
•• After an abrupt loss of fever, the characteristic generalised pink maculopapular rash appears
•• Nagayama spots (erythematous papules on the mucosa of soft palate & base of uvula) may be present on the 4th day in 2/3 of
patients
137. b. Central nervous system abnormalities  Ref: Nelson's 20/e p 1320-1321; Refer pretext of this chapter for details;
Mnemonic for Major Diagnostic Criteria of Staphylococcal Toxic Shock Syndrome:
“H-A-R jagah Staphylococcus hai” → Hypotension, acute fever and rash
138. a. Staphylococcus aureus  Ref: Nelson's 20/e p 1953
In children with pyogenic liver abscesses, the most common pathogenic organisms include Staphylococcus aureus, Streptococcus spp.
Escherichia coli, Klebsiella pneumoniae, Salmonella & anaerobic organisms
Entamoeba histolytica or Toxocara canis–associated liver abscesses have also been reported in developing countries.
139. a. Azithromycin  Ref: Nelson's 20/e p 1489
M. pneumoniae is sensitive to macrolides (erythromycin, clarithromycin, azithromycin), tetracyclines, and quinolones
Recommended treatment is clarithromycin (15 mg/kg/day divided into 2 doses PO for 10 days) or azithromycin (10 mg/kg once PO
on day 1 and 5 mg/kg once daily PO on days 2-5).
140. a. E. coli  Ref: Nelson's 20/e p 2940-2944, Ghai 8/e p 563-565, CDC website
Most common cause of acute Bacterial Meningitis in:
INDIA WORLD
Age group Organism Age group Organism
Neonates E. coli Neonates Gr B Streptococcus, E. coli, Listeria
1 month -2 year H. influenzae Infants & children S. pneumoniae, N. meningitidis, H. influenzae
Older children S. pneumoniae, N. meningitidis Adolescents S. pneumoniae, N. meningitidis

141. c. Scarlet fever  Ref: Nelson's 20/e p 1328; Refer pretext of this chapter & chapter 8 for details;
Fever for at least 5 days, generalized erythematous rash, strawberry tongue and cervical lymphadenopathy are compatible
with a diagnosis of Scarlet fever; Kawasaki disease is a close differential diagnosis (Fever + at least 4 out of 5 criteria required for
its diagnosis, here aonly 3 criteria present apart from fever)
142. c. Cellulitis  Ref: Nelson's 20/e p 1327-1331; Refer pretext of this chapter for details;
Chapter 8: Infectious Diseases 187

Discussing about the options one by one,

Answers with Explanations


a. Erysipelas: It involves deeper layers of skin & connective tissue with swelling, redness & tenderness, sharply defined border;
abrupt onset with high fever seen
b. Carbuncles refer to multiple coalesced boils, usually caused by Staphylococcal infection
c. Cellulitis: Refers to a non-necrotizing inflammation of skin & subcutaneous tissues, that usually follows a breach in the skin;
Erythema, Pain, Swelling & Warmth of affected part is seen (hence, it is the answer)
d. Anthrax: Cutaneous anthrax presents as a boil-like skin lesion that eventually forms an ulcer with a black centre (eschar);
Cutaneous anthrax is the most common & least dangerous form of Anthrax;
143. c. Staphylococcus aureus  Ref: Nelson's 20/e p 2264
Viridans-type streptococci (a-hemolytic streptococci) and Staphylococcus aureus remain the leading causative agents for endocarditis
in pediatric patients; Staphylococcal endocarditis is more common in patients with no underlying heart disease, Strepto viridans is
more common after dental procedures & Enterococcus is more common after bowel or genitourinary manipulation.
144. c. Strepto pneumoniae  Ref: Nelson's 20/e p 2940-2944, Ghai 8/e p 563-565
145. a. Salmonella  Ref: Nelson 20/e p 3322; Harrison's latest 19th ed pg 634; CDC website
Nelson 20th ed says: “Salmonella & Staph aureus are the 2 most common causes of osteomyelitis in children with sickle cell
anemia”; So it does not help us with the answer to this question;
Harrison Latest 19th ed says: “Unusual susceptibility to osteomyelitis, which may be caused by organisms, such as Salmonella, is
seen in sickle cell disease”.
Wintrobes Hematology also says: “Although >80% of hematogenous osteomyelitis in the general population is caused by
Staphylococcus, most cases of osteomyelitis occurring in individuals with sickle cell anemia are caused by Salmonella”

Microorganisms in Osteomyelitis & their Clinical Associations:

Most common clinical association Microorganism


Frequent microoganism in any type of osteomyelitis Staphylococcus aureus
Foreign body-associated infection Coagulase-negative staphylococci, atypical Mycobacteria, fungi
Common in nosocomial infections Enterobacteriaceae, Pseudomonas aeruginosa, Candida spp.
Sickle cell disease Salmonella spp., Staphylococcus aureus, or Streptococcus pneumoniae
Exposure to kittens / cats Bartonella henselae
Human or animal bites Pasteurella multocida or Eikenella corrodens
Immunocompromised patients Aspergillus spp., Candida albicans, or Mycobacteria spp.

146. a. N.meningitidis  Ref: Nelson's 20/e p 2940-2944, Ghai 8/e p 563-565; CDC website
147. c. Toxin produced by gram positive anaerobic bacillus  Ref: Nelson's 20/e p 1428-1431
•• Food poisoning due to ingestion of honey is caused by Clostridium botulinum
•• Clostridium botulinum is a gram positive spore forming anaerobic bacilli, which acts through botulinum toxin.
148. a. Strep pneumoniae  Ref: Nelson's 20/e p 3087
•• 3 pathogens predominate in Acute Otitis Media: Strepto pneumoniae, non-typeable H. influenzae, & Moraxella catarrhalis
•• With the use of conjugate pneumococcal vaccine, nontypeable H. influenzae initially overtook S. pneumoniae as the most
common pathogen, being found in 40-50% of cases;
•• However, over time, S. pneumoniae serotypes not covered in the conjugate vaccine have emerged, with S. pneumoniae again
overtaking non-typeable H. influenzae as the most common pathogen in many studies.
149. b. Staphylococcal scalded skin syndrome  Ref: Nelson's 20/e p 1315
Rashes are seen in all these diseases; However, in Toxic epidermal necrolysis, Steven Johnson syndrome & Infantile pemphigus,
intraoral mucosal involvement is usually seen, but in Staphylococcal scalded skin syndrome, intraoral mucosal surfaces are
spared (as in this case).
150. b. Spreads along the nerves, d. Incubation period is variable.  Ref: Nelson's 20/e p 1432-1434
Option a False Tetanus bacillus is not an invasive organism
Option b True Tetanus toxin binds at neuromuscular junction & enters motor nerve by endocytosis, after which it under-
goes retrograde axonal transport to cytoplasm of neuron
Option c False Clostridium tetani, is a motile, Gram-positive, spore-forming obligate anaerobe, so does not require O2
Option d True The incubation period of tetanus is typically is 2-14 days but may be as long as months after the injury.
Option e False Because C. tetani is not an invasive organism, its toxin-producing vegetative cells remain where introduced
into the wound

188 Section 1: General Pediatrics

 CONGENITAL TOXOPLASMOSIS
Review of Pediatrics and Neonatology

151. d. During third trimester  Ref: Nelson's 20/e p 1723-1725, Ghai 8/e p 272
Risk of fetal infection in acute maternal toxoplasmosis increases with each trimester of pregnancy being 15%, 25% and 60%
in 1st, 2nd and 3rd trimesters respectively; But severity of fetal infection is much greater in early pregnancy.
152. b. Chorioretinitis  Ref: Nelson's 20/e p 1726
Characteristic triad of Congenital Toxoplasmosis is chorioretinitis (most common), hydrocephalus, & cerebral calcifications.
153. b. IgM antibodies to toxoplasma and c. IgA antibodies to toxoplasma  Ref: Nelson's 20/e p 1723-1725
154. b. Toxoplasmosis  Ref: Nelson's 20/e p 1723-1725;
Presence of cervical Lymphadenopathy, treatment (which was not done properly) with Spiramycin and neonatal manifestation
in the form of hydrocephalus and intracerebral calcification, all point to diagnosis of congenital toxoplasmosis.
155. d. Avidity testing must be done to differentiate between IgA and IgM  Ref: Nelson's 20/e p 1723
Serological tests in Congenital Toxoplasmosis:

Assay Characteristics
IgG Appears in 1–2 weeks, peaks at 1–2 months & persists throughout life; Transplacental IgG disappears by 6–12 m age;
But presence of IgG in the serum does not confirm the diagnosis because IgG can easily cross the placenta
IgA It rises rapidly & usually disappears by 7 months;
IgA has greater sensitivity for neonates compared to IgM assays; So IgA ELISA is better than IgM
IgM IgM appears within 2 weeks, peaks at 1 month & disappears by 6–9 months
Because IgM does not cross the placenta, it is the test of choice for determining congenital infection
Avidity It measures the strength of antigen antibody reaction, which increases with duration of infection
Test High avidity requires at least 3 months to develop, thus helps to rule out acute infection;
High avidity test → Infection began > 16 weeks earlier; Low avidity test → Indicates recent infection

 OTHER PARASITIC INFECTIONS


156. d. All of the above  Ref: Nelson 20/e p 1405-6, 1611
•• Acute appendicitis, mesenteric lymphadenitis & ileocolitis have been reported in patients who have had appendectomies during
C. jejuni infection.
•• Yersinia pseudotuberculosis is most often associated with mesenteric lymphadenitis.
•• Human Adenovirus may also cause mesenteric adenitis.
157. a. Toxoplasma  Ref: Nelson 20/e p 1728
The Sabin-Feldman dye test is sensitive & specific for Toxoplasma; It measures primarily IgG antibodies;
158. c. Severe malarial anemia   Ref: Nelson's 20/e p 1712-1719
Most common serious complication of malaria is severe anemiaQ
159. a. Cerebral malaria  Ref: Nelson's 20/e p 1712-1719, Ghai 8/e p 260-266
Malaria due to Plasmodium falciparum:
•• Acute onset of fever, anemia with altered sensorium in an endemic area suggests cerebral malaria
•• It is the most severe form of malaria and is associated with higher density parasitemia and a number of complications
•• Serious complications that appear unique to P. falciparum include cerebral malaria, acute renal failure, respiratory distress
from metabolic acidosis, algid malaria and bleeding diatheses.
Cerebral malaria:
•• Defined as presence of coma in a child with P. falciparum parasitemia & absence of other reasons for coma
•• Cerebral malaria is most common in children in areas of midlevel transmission
•• It has a fatality rate of 15-20% and is associated with long-term cognitive impairment in children
•• Repeated seizures & hypoglycemia are frequent in children with cerebral malaria

160. b. Exchange transfusion  Ref: Nelson's 20/e p 1718
Supportive treatment for severe Malaria includes exchange transfusion in P. falciparum malaria with parasitemia greater than 10%
with evidence of severe complications (e.g., severe malarial anemia, cerebral malaria).
Nelson also says:
“Exchange transfusion is thought to be useful in severe malaria with high-level parasitemia, but no randomized clinical trial has ever
been conducted to assess its utility, & some groups, including CDC no longer advocate its use for severe malaria. Corticosteroids are
not recommended for cerebral malaria.”
161. a. Hookworm  Ref: Nelson 20/e p 1734-1735
Hookworm infestation:
Chapter 8: Infectious Diseases 189

•• Leads to intestinal blood loss → iron deficiency anemia & malnutrition → growth retardation

Answers with Explanations


•• Dermatitis referred to as ground itch when they penetrate human skin, manifests as vesiculation & edema
•• Infection with a zoonotic hookworm, especially A. braziliense → cutaneous tracts of cutaneous larva migrans
•• Cough occurs when larvae migrate through lungs ~ 1 week after exposure; Eosinophilia is often seen
162. b. Niclosamide, c. Praziquantel and d. Mebendazole  Ref: Nelson's 20/e p 1749, Ghai 8/e p 275
Infecting organism Drug of choice Alternative
Taenia saginata (Beef tapeworm) Praziquantel or Niclosamide Mebendazole
Taenia solium (Pork tapeworm) Praziquantel or Niclosamide
Cysticercosis (Pork tapeworm larval stage) Albendazole Praziquantel

163. c. Clindamycin  Ref: Nelson's 20/e p 1712-1719, Ghai 8/e p 260-266
164. a. Toxoplasmosis  Ref: Nelson's 20/e p 1593, Ghai 8/e p 272
•• Intracerebral calcification can be seen in both congenital Toxoplasmosis & CMV infection
•• Hydrocephalus is seen in Toxoplasmosis while microcephaly in CMV infection.

 FUNGAL INFECTIONS
165. d. Candida parapsilosis  Ref: Study Nelson 20/e p 1515-1517, Ghai 8/e 259-260
Candida parapsilosis

•• It is ubiquitous in nature and is found as a commensal on the human skin


•• It is most frequently isolated from hands (subungual space) and the gastrointestinal tract
•• It is the most common fungal infection in the neonates transmitted by the hands of caregivers
•• Neonatal risk factors for invasive C. parapsilosis infections are: Birth weight <1500 g, Prematurity, Prior colonization, Parenteral nutrition,
Intravascular catheters

Chapter 9
Immunization
High Yield Points  TYPES OF VACCINES M

•• Vaccine immunogenicity: Ability of a Live Killed Toxoid Subunit


vaccine to induce anti­bodies.
•• Vaccine efficacy: Ability of the BCG IPV (Inactivated Polio Diphtheria Pneumococcal
vaccine to protect an individual. OPV vaccine) Tetanus HPV vaccine
•• Vaccine effectiveness: Ability of the Measles/MMR Pertussis Vi Typhoid
vaccine to protect the community Yellow fever Inactivated Rabies H. influenzae type b
and is a sum of the vaccine efficacy Live JE Inactivated JE
and herd effect
Live Hep A
Varicella

Combination Vaccines

High Yield Points Merits Concerns


•• All parenteral vaccines except •• Fewer injections •• Stability of product
BCG (& sometimes Rabies) are given •• Reduced burden on cold chain •• Immune interference b/w various antigens
by either intramuscular (IM) or
•• Reduced requirement of needles, syringes causing suboptimal immunogenicity
subcutaneous (SC) route.
•• SC route is recommended •• Easier Record keeping •• Costs
for measles, MMR, varicella,
meningococcal polysaccharide, JE,
Yellow fever vaccines;  STORAGE OF VACCINES
•• Either SC or IM route may be used
for pneumococcal polysacc­haride
vaccines & IPV

High Yield Points


•• Light sensitive vaccines are- BCG,
DTaP, Measles, MMR, HPV, Rotavirus
•• Vaccines without preservatives-
BCG, Measles
Chapter 9: Immunization 191

Heat and Freeze Sensitivity of Commonly Used Vaccines

Section 1: General Pediatrics


High Yield Points
Vaccine Monitoring Tools
•• Vaccine vial monitor (VVM)-Label
with heat sensitive material on
vaccine vial, to register cumulative
heat exposure over time
•• Dial thermometer–for tempera­
ture documentation, especially
opening & closing temperature
•• Freeze Watch Indicators

High Yield Points


Influenza vaccineQ & yellow feverQ
vaccine are contraindicated in those with
egg allergy
Graphic depiction of heat and freeze sensitivity of commonly used vaccines

 LATEST IMMUNIZATION SCHEDULES FOLLOWED IN INDIA

Latest National Immunization Schedule IAP Latest Vaccination Guidelines


Age Vaccine Age Vaccine
At birth BCG, bOPV-0, Hep B-0 At birth BCG, bOPV-0 , Hep B1

6 weeks bOPV-1, Pentavalent-1, Rotavirus-1*, fIPV-1, PCV-1* 6 weeks DPT-1, IPV-1, Hep-B2, HiB-1, PCV 1, Rotavirus 1

10 weeks bOPV-2, Pentavalent-2, Rotavirus-2* 10 weeks DPT-2, IPV-2, HiB-2, PCV-2, Rotavirus-2

14 weeks bOPV-3, Pentavalent-3, Rotavirus-3*, fIPV-2, PCV-2* 14 weeks DPT-3, IPV-3, HiB-3, PCV-3, Rotavirus-3

9 months MR-1*, JE-1*, PCV-3* 6 months bOPV-1, Hep B3

16–24 months DPT-B1, bOPV-B, JE-2*, MR-2* 9 months bOPV2, MMR-1, Typhoid conjugate, JE-1

5–6 years DPT-B2 12 months IPV-B1, Hib-B1, PCV-B1, Hep A 1

11–13 years HPV-1*, HPV-2* 15 months DTP-B1, MMR-2, Varicella 1, JE-2

* In selected districts only 18 months Hep A 2

2 years Typhoid conjugate-B

4 – 6 years DPT B2, bOPV-3, MMR-3, Varicella-2

11–12 years Tdap, HPV-1, HPV-2

Time Limits for Using Vaccines after Reconstitution


•• Varicella: 30 min (and protect from light) •• Measles/MMR: 4 to 6 hours
•• MMRV: 30 min (and protect from light) •• Meningococcal polysaccharide 30 min
•• Yellow fever: 1 hour •• DTaP/Hib combination: 30 min

General Principles of Vaccination M

•• Any number of vaccines live/inactivated may be given on the same day maintaining a gap of
5 cm between different vaccines (exception BCG and Measles / MMR)
•• Inactivated vaccines can be given at any time in relation to any other live/ inactivated
vaccines
•• If missed on a single day, gap between any 2 live vaccines should be at least 4 weeks.
192 Section 1: General Pediatrics

 ADVERSE EVENTS FOLLOWING IMMUNIZATION (AEFI)


Review of Pediatrics and Neonatology

L at e s t U p d at e s

Mission Indradhanush M Type of AEFI Description


It was launched by Ministry of Health
and Family Welfare Government of Vaccine reaction Event caused by the vaccine, e.g., VAPP following OPV; or precipitated by the
India on 25 December, 2014, to vaccine when given correctly, e.g. febrile seizure following vaccination.
ensure that all children & pregnant Programme error Event caused by an error in vaccine preparation, handling, or administration,
women are fully immunized with 7 e.g. deaths following measles vaccination due to toxic shock syndrome resulting
vaccine preventable diseases. from improper reconstitution and storage of measles vaccine
The mission indradhanush, depic­
ting seven colours of the rainbow, Injection reaction Event from anxiety about, or pain from, the injection itself rather than the
targets to immunize all children against vaccine. Examples include syncope due to pain of vaccination, injection site
7 vaccine preventable diseases, namely: abscesses, sciatic nerve damage due to gluteal injection.
•• Diphtheria
Coincidental Event that happens after immunization but not caused by the vaccine—a
•• Pertussis (Whooping Cough)
chance association, e.g. association between immunization & Sudden Infant
•• Tetanus
•• Tuberculosis Death Syndrome (SIDS), as the incidence of SIDS peaks around the age when infant
•• Polio immunizations are delivered.
•• Hepatitis B Unknown The cause of the event cannot be determined.
•• Measles
In addition to this, vaccines for
Japanese Encephalitis (JE) and Hae- Important Side Effects/Contraindications of Vaccines M
mophilus influenzae type B (Hib) are
Vaccines Side effects Contraindications
also being provided in selected states
DPT Incessant cry, hyperpyrexia, seizures, Progressive neurological disorders,
encephalopathy encephalopathy within 7 days, H/o
anaphy­laxis
BCG Axillary lymphadenitis Immunocompromised
L at e s t U p d at e s
OPV VAPP (Vaccine associated Paralytic Polio), Immunocompromised
‘Intensified Mission Indradhanush’: VDPV (Vaccine derived Paralytic Polio)
Launched in Oct 2017; Week-long
immunization drives from 7th of each Rotavirus Intussusception, irritability, diarrhea, Acute gastroenteritis, H/O Intussusception,
month. vomiting SCID
Inactivated Fever, headache, fatigue, Guillain-Barré Egg allergy, GBS
Influenza syndrome
Measles Toxic Shock Syndrome Immunocompromised
Question 1
What is the name of this ins­trument  IMPORTANT DETAILS ABOUT INDIVIDUAL VACCINES
used in vaccination?
BCG VACCINE M

Strain •• It is derived from the bovine tuberculosis strain, first developed in 1921.
a. Mantoux syringe •• The 2 common strains in use are Copenhagen (Danish 1331) & Pasteur
b. BCG syringe
Efficacy •• BCG induces cell-mediated immunity; Protective efficacy is 75–86% for
c. Tuberculin syringe
prevention of miliary and meningeal TB & 50% for pulmonary tuberculosis
d. DOT syringe
Composition •• The vaccine contains 0.1–0.4 million live viable bacilli per dose
•• It is supplied as a lyophilized (freeze-dried) preparation in vacuum sealed, multi-
dose, dark colored ampoules with normal saline as diluentQ

Storage •• The vaccine is light sensitive & In lyophilized form, it can be stored at 2 to 8oC for
up to 12 months without losing its potency

Dosage •• Recommended dose is 0.1 ml, irrespective of age & weight of the baby
•• It should be given intradermal, using a tuberculin syringe & 26G needle
•• Should be given at birth (for institutional deliveries) or at 6 weeks
•• Catch up vaccination with BCG is recommended till age of 1 year according to our
national immunisation schedule & 5 years according to IAP

Changes following •• No visible change at injected site for several days


BCG injection •• After 2–3 weeks, a papule develops, which
•• By 5–6 weeks, papule increases to a size of 4–8 mm
•• This papule heals with ulceration & results in a scar after 6–12 weeks

Contd...
Chapter 9: Immunization 193

Contd...

Section 1: General Pediatrics


Question 2
Adverse •• BCG adenitis: Associated with subcutaneous administration of BCG
How much time does it take for the
effects •• Disseminated BCG infection may occur in children with cellular immunodefi­
finding shown below, after BCG
ciency
vaccination?
•• Osteitis is a rare complication of BCG vaccination that appears to be related to
certain strains of the vaccine that are no longer in wide use
•• Systemic complaints such as fever, convulsions, and irritability are rare

Contraindication •• Should be avoided in immunocompromised, especially cellular immunodefi­


ciency

Special •• BCG may be given with other vaccines on the same day except with MMR vaccine
precautions where a gap of 4 weeks between the two vaccines is recommended a. 1-2 days b. 1-2 weeks
•• Routine tuberculin testing prior to catch up vaccination is not necessary c. 2-4 weeks d. 6-12 weeks
•• BCG may be repeated once in children < 5 years old in the absence of a reaction/
scar presuming that BCG has not been taken up

High Yield Points


POLIO VACCINES M

•• The last polio case was reported in


•• 2 main types:
India in 2011
–– Oral polio vaccine (OPV): A live vaccine; Also called Sabin •• India was certified polio-free on 27
–– Injectable polio vaccine (IPV): A killed vaccine; Also called Salk March 2014
•• OPV can be Monovalent (mOPVs), bivalent (bOPV) or trivalent •• In April 2016, switch from trivalent to
•• Monovalent & bivalent polio vaccines are 2.5–3 times more efficacious than trivalent bivalent vaccine was done.
OPV
•• IAP Recommended schedule: Birth dose of OPV, three primary doses of IPV at 6, 10 and
14 weeks, followed by two doses of OPV at 6 and 9 months, another dose (booster) of IPV at
15–18 months, and OPV at 5 years High Yield Points
•• Birth dose of OPV usually does not lead to VAPP
•• OPV in place of IPV, if IPV is unfeasible, minimum 3 doses •• Wild poliovirus is defined as
•• Additional doses of OPV with Pulse polio Immunization. polioviruses with no genetic
evidence of derivation from any
Vaccine Associated Paralytic Poliomyelitis (VAPP) vaccine strain, having capability of
person-to-person transmission.
•• VAPP is defined as those cases of AFP which have residual weakness 60 days after the •• Type 2 poliovirus was declared
eradicated in September 2015.
onset of paralysis and from whose stool samples, vaccine-related poliovirus but no wild
•• Type 3 wild poliovirus has not been
poliovirus is isolated. detected anywhere in the world
•• VAPP occurs due to loss of attenuating mutations and reversion to neurovirulence during since November 2012.
replication of the vaccine virus in the gut. •• Type 3 has been more often
•• VAPP may occur in the vaccine recipient (‘recipient VAPP’, occurring within 4–40 days of associated with recipient VAPP, type
2 has been more often observed in
receiving OPV) or contact of the vaccine recipient (‘contact VAPP’).
immunodeficient individuals & cases
•• In developed nations the risk of VAPP is higher with the first dose that “takes”, with P3 virus with contact VAPP.
and in patients with B cell immunodeficiency.

Vaccine Derived Polio Viruses (VDPVs)


VDPV arises due to mutation & recombination in the human gut Question 3
VDPV is 1–15% divergent from the parent vaccine virus. An OPV vial shows this symbol on
3 categories of VDPV are: it. What does it indicate?
•• circulating VDPV (cVDPV)—VDPV with evidence of virus circulation in the population
causing two or more paralytic cases,
•• iVDPV—VDPV in the immunodeficient person, and
•• VDPV of ambiguous origin (aVDPV)—VDPV isolated from environmental sources.

L at e s t U p d at e s
According to Polio Eradication & Endgame Strategic Plan 2013- 2018 (by WHO): a. The vaccine has reached its expiry
•• Phased removal of all oral polio vaccines (OPVs) to eliminate the risks of VAPP & cVDPV. date
•• Removal of all OPVs begins with a switch from trivalent OPV (tOPV) to bivalent OPV (bOPV), removing b. The vaccine has not been stored
type 2 component (OPV2) from immunization programmes. properly
•• Over 90% of cVDPV cases & 40% of VAPP cases, are due to the type 2 component of tOPV. c. The vaccine vial is empty
•• Type 2 component of tOPV also interferes with immune response to poliovirus types 1 & 3. d. Use the vaccine if expiry date not
•• After all wild polioviruses have been fully eradicated, then all OPVs will be withdrawn. reached
194 Section 1: General Pediatrics

HEPATITIS B VACCINE
Review of Pediatrics and Neonatology

•• Currently available vaccine containing HBsAg is produced by recombinant technology in


yeast & adjuvanted with aluminum salts and preserved with thiomersal
•• An anti-HBs concentration of 10 mIU/ml is considered protective
•• The primary 3-dose vaccine series induces protective antibody concentrations in > 95%
•• The classical schedule is 0, 1 and 6 months.
•• Can be given at Birth, 6 weeks, 10 weeks, 14 weeks, along with other vaccines.

Management of an infant born to hepatitis B positive mother M

•• The baby should be given hepatitis B Immunoglobulin (HBIG) along with hepatitis B
vaccine within 12 hours of birth, using 2 separate syringes & separate sites
•• The dose of HBIG is 0.5 ml intramuscular, which may be given up to 7 days of birth
•• Two more doses of hepatitis B vaccine at 1 month/6 weeks and 6 months are needed.

L at e s t U p d at e s DTP VACCINE
Pertussis vaccine may induce some •• Popularly known as triple antigen
cross immunity against Bordetella •• DTwP is composed of tetanus (5 to 25 Lf per dose) & diphtheria toxoids (20 to 30
parapertussis Lf) & whole cell pertussis bacilli adsorbed on insoluble aluminium salts which act as
adjuvants.
•• 3 primary doses at 6, 10 & 14 weeks & 2 boosters at 15–18 months & 5 years
•• Catch up below 7 years: DTwP/DTaP at 0, 1 and 6 months
•• Catch up above 7 years: Tdap, Td, and Td at 0, 1 and 6 months.

High Yield Points


•• The effectiveness against diph­theria/tetanus of three doses of vaccine is > 95%
•• Most adverse effects are due to the pertussis component
•• Serious adverse effects of DTP vaccine include persistent crying (most common), fever > 40.5°C,
High Yield Points hypotonic hyporesponsive episodes (HHE), seizures & encephalopathy

Catch-up Hib vaccination is recommen­


ded till 5 years of age. HAEMOPHILUS INFLUENZAE TYPE B (HIB) VACCINE
•• 6–12 months; 2 primary doses
4 weeks apart and 1 booster; •• All Hib vaccines are conjugated vaccines where the Hib capsular polysaccharide
•• 12–15 months: 1 primary dose and (polyribosylribitol phosphate or PRP) is conjugated with a protein carrier
1 booster; •• Currently available vaccines use CRM197 mutant C. diphtheria toxin protein, N. meningitidis
•• Above 15 months: Single dose. protein outer membrane protein complex & tetanus toxoid as carrier
•• Schedule: Primary series includes Hib conjugate vaccine at ages 6, 10, 14 weeks with a
booster at age 12 through 18 months.

High Yield Points PNEUMOCOCCAL VACCINES

Pneumococcal polysaccharide vaccine Pneumococcal Conjugate Vaccine Pneumococcal Polysaccharide vaccine


is not given at less than 2 years age, (PCV13) (PPSV-23)
as it is unable to evoke an immune
response in them Conjugated to diphtheria toxoid cross-reactive T cell independent vaccine - poorly
material 197 (CRM197) protein, protein D of non immunogenic below the age of 2 yrs, low immune
capsulated Hib, DT & TT memory

•• PCV covers 4,6B, 9V, 14, 18C, 19F & 23F •• No reduction in nasopharyngeal carriage
•• Currently 13 valent vaccine available •• Does not provide herd immunity
•• Robust immune response, immune memory •• 70% efficacy against invasive disease

Vaccination Schedule
•• 4 dose schedule - ages 6, 10, 14 weeks & a booster at 12-15 months
•• Routine use of PCV13 not recommended for healthy children aged >5 years
•• Minimum age for 1st dose is 6 weeks & minimum interval b/w 2 doses – 4 to 8 weeks.
Chapter 9: Immunization 195

MEASLES & MMR VACCINE

Section 1: General Pediatrics


High Yield Points
•• Live attenuated measles vaccine contains Edmonston-Zagreb strain, with distilled water
used as a diluent •• Efficacy of Measles vaccine is
•• Adverse reactions: local pain, tenderness, mild measles like illness 7–12 days after 60–80% when given at the age of
9 months
vaccination in 2–5% of the vaccines. ThrombocytopenicQ purpura may occur.
•• In HIV infected infants superior
seroconversion rates are seen at 6
High Yield Points months as compared to 9 months due
to progressive immunodeficiency with
Measles vaccine can be admini­stered to infants aged 6–11 months during outbreaks; But these children age.
should be revaccinated with 2 additional doses of measles containing vaccines.

Immunization Schedule
•• Measles vaccine: Minimum age: 9 months or 270 completed days L at e s t U p d at e s
•• Catch-up vaccination beyond 12 months should be MMR •• As per IAP 2016, 3 doses of MMR at
•• Measles, mumps, and rubella (MMR) vaccine: Minimum age: 12 months 9 months, 15 months & between
•• MMR: First dose at age 12-18 months & second dose at age 4-6 years. 4-6 years are recommended.
•• No stand alone measles dose at
INFLUENZA VACCINE 9 months.
Inactivated Influenza Vaccines Live Attenuated Influenza Vaccines
3 types: whole virus, split product, subunit •• Given as nasal spray
surface–antigen •• Used in Healthy non pregnant individuals 2-49 yrs
•• Trivalent - 2 influenza A (H1N1 & H3N2) + 1 age
influenza B strains •• Not available in India
•• Monovalent -novel H1N1 2009 strain

Indications L at e s t U p d at e s
•• India is committed to the goal of
•• In all children with risk factors
measles elimination & control of
•• Wherein the vaccine is desired/requested by parents.
rubella by 2020.
•• To achieve this goal, Measles Rubella
When to Administer? (MR) campaign is being launched in
In tropical countries like India, illness occurs all year round (peaks -June to September) so vaccine the country covering all children of
9 months to <15 years age group.
be given as soon as the new vaccine is released in the market (IAP).
•• In Routine Immunization, MR vaccine
will replace measles vaccine and will
ROTAVIRUS VACCINATION be given as two doses schedule.
•• 2 types of oral live vaccines:
–– Human monovalent (G1P[8] strain) (Rotarix) and
–– Human Bovine pentavalent (G1, G2, G3, G4 and P1A [8]) (RotaTeq)
–– Indian Neonatal Rotavirus live vaccine 116E
•• 1st dose of RV1 or RV5 - between 6 - 15 weeks age
•• Last Dose -maximum age -6 months for monovalent & 8 month age for pentavalent. L at e s t U p d at e s

Practical Considerations with Rotavirus Vaccine: 116E strain Rotavirus Vaccine, origi-
nally isolated from an asymptomatic
•• Both vaccines effectively prevent severe (but not mild) gastroenteritis. rotavirus-infected neonate in AIIMS is a
•• In developing countries, efficacy rates are between 50 and 80%. Live, attenuated, oral vaccine, recently
developed in India
•• Rotavirus vaccines have been associated with an increased risk of intussusceptions
•• Vaccination should be avoided in those with history of hypersensitivity to any of the
vaccine components or previous vaccine dose & in babies with SCID
•• H/O intussusception in past is an absolute contraindication for rotavirus vaccine
•• Latex rubber is contained in the RV1 oral applicator, so infants with a severe (anaphylactic)
allergy to latex should not receive RV1 vaccine.
•• Vaccination should be postponed in infants with acute gastroenteritis.

HEPATITIS A VACCINE
•• Both live & killed Hepatitis A vaccines are available;
•• Inactivated HepA vaccine: Start the 2-dose HepA vaccine series for children aged 12–23
months; separate the 2 doses by 6 to 18 months.
196 Section 1: General Pediatrics

•• Live attenuated Hepatitis A vaccine: Single dose at 12 to 23 months of age


Review of Pediatrics and Neonatology

•• Recommended for: All healthy children with special emphasis in risk groups like patients
with chronic liver disease, Carriers of Hep B & Hep C, Immunodeficiency, household contacts
of patients with acute Hep A virus infection within 10 days of onset of illness in index case.

VARICELLA VACCINATION
High Yield Points
•• Live attenuated vaccine from the Oka strain
For post-exposure prophylaxis Varicella •• Dosage: 0.5 ml subcutaneous, 2 doses, 1st -15 month, 2nd – 4-6 yrs age
vaccine must be given-preferably within •• For children aged ≥ 13 yrs- 2 doses 4-8 weeks apart
3 days of exposure, potentially up to •• IAP recommends it for all healthy children with no prior history of varicella with special
5 days of exposure. emphasis on high risk groups.

JAPANESE ENCEPHALITIS (JE) VACCINE


Strain used in vaccine: SA 14-14-2Q
Both live attenuated & inactivated vaccines available
It is not recommended for routine use, but only for individuals living in endemic areas.

Dosage of Japanese Encephalitis Vaccine


•• Live attenuated, cell culture-derived: Two dose schedule, first dose at 9 months along
with measles vaccine and second at 16 to 18 months along with DTP booster
•• Inactivated cell culture-derived (JEEV or JENVAC):
2 doses IM on days 0 & 28.

TYPHOID VACCINES
L at e s t U p d at e s
The new generation current typhoid fever vaccines include oral live attenuated Ty21a vaccine,
Parenteral Vi-polysaccharide and Vi-Polysaccharide Conjugate vaccines.
Recommended age for JE vaccina-
tion raised to 18 years in endemic Vi-capsular Polysaccharide (Vi-PS) Vaccine
regions.
•• As it is a polysaccharide vaccine, it is not immunogenic in children < 2 yrs age & has no
immune memory; Efficacy is only 50–60%.
•• The vaccine does not interfere with the interpretation of the Widal test.
•• Single dose in children ≥ 2 years with revaccination every 3 years.

Vi-Capsular Polysaccharide Conjugate Vaccines


•• Vi-PS Conjugate vaccine is conjugated with either Pseudomonas aeruginosa exotoxin
A or tetanus toxoid; overall efficacy is around 89%
•• Vaccination schedule for Typhoid conjugate vaccines is a single dose at 9–12 months through
23 months followed by a booster at 2 years of age.

L at e s t U p d at e s
Typhoid conjugate vaccine (TCV): A single dose of TCV at 9-12 months of age, with an interval of at least
4 weeks should be maintained between TCV & MMR vaccine; Booster dose at 2 years of age.

HUMAN PAPILLOMA VIRUS (HPV) VACCINE


•• 2 recombinant DNA vaccines quadrivalent & bivalent vaccines available.
High Yield Points •• Quadrivalent vaccine (HPV4 or Gardasil) is a mixture of L1 proteins of HPV serotypes
16, 18, 6 & 11 while bivalent vaccine (HPV2 or Cervarix) contains HPV 16 & 18
•• Minimum age for HPV vaccination •• 99–100% efficacy is seen against vaccine type related genital warts, vaginal intraepithelial
is 9 years neoplasia (VaIN), vulvar intraepithelial neoplasia (VIN) & CIN- 2/3
•• Catch-up HPV vaccination can be •• Either HPV4 (0, 2, 6 months) or HPV2 (0, 1, 6 months) is recommended in a 3-dose
done in females (either HPV2 or series for females aged 11 or 12 years (before their sexual debut)
HPV4) at age 13 through 45 years if •• HPV vaccines do not protect against the serotype with which infection has already occurred
not previously vaccinated. before vaccination.
•• Higher immune response to HPV vaccine is seen in preadolescents through 9–13 years
as compared to adolescents and young adults.

 PROTECTION AGAINST RABIES


For Pre-exposure Prophylaxis of Rabies:
3 doses of Rabies vaccine are given intramuscularly in deltoid/ antero­lateral thigh on days 0, 7
& 28.
Chapter 9: Immunization 197

A. Care of Animal Bite Wounds

Section 1: General Pediatrics


M

•• 1st step is thorough cleansing of wound with soap & flushing under running water for
10 minutes followed by irrigation with 70% alcohol or povidone iodine
•• Antimicrobials and tetanus toxoid should be given if indicated
•• Rabies immunoglobulin (RIG) should be infiltrated in & around category 3 bites
•• Any suturing of wound should be avoided.

B. Post-exposure Prophylaxis of Rabies


Indication:
•• It is a medical urgency & is indicated following a significant contact with any warm-blooded
animal like dogs, cats, cows, sheep, goats, pigs, horses, & monkeys High Yield Points
•• In case of bites by pet animals, PEP may be deferred only if the pet at the origin of exposure Rabies immunoglobulin (RIG) along
is > 1 year old & has received at least 2 doses of a potent vaccine; with rabies vaccines are recommended in
•• If vaccination is deferred, the pet should be observed for 10 days; if the dog shows any sign all category III bites
of illness during this, the patient should receive full rabies post-exposure prophylaxis.
Categories of rabies exposure and recommended postexposure prophylaxis

Type of Recommended post-


Category Type of contact
exposure exposure prophylaxis (PEP)
I Touching animals or Licks on intact skin None None, if reliable history available
II Nibbling of uncovered skin, Minor Minor Wound management +
scratches or abrasions without bleeding. Anti-rabies vaccine
III Single or multiple transdermal bites Severe Wound management + Rabies
or scratches, licks on broken skin, immunoglobulin + Anti-rabies
Contamination of mucous membrane vaccine
L at e s t U p d at e s
with saliva (i.e. licks)
Dengue vaccine or Dengvaxia
Schedule: 0, 3, 7, 14, and 30 with day ‘0’ being the day of commencement of vaccination; •• It is a tetravalent live attenuated
A sixth dose on day 90 is optional and may be offered to immunosuppressed chimeric vaccine with yellow fever
virus as the backbone for use in
C. About Rabies Vaccine individuals 9-45 years old.
•• 3 dose series at 0, 6 & 12-month.
•• The currently available vaccines are the Cell culture vaccines & include Purified Chick •• The vaccine is licensed in the Mexico,
Embryo Cell Vaccine (PCECV), Human Diploid Cell Vaccine (HDCV), Purified Vero Cell Philippines, Brazil, El Salvador and
Vaccine (PVRV) & Purified Duck Embryo Vaccine (PDEV). Paraguay
•• These vaccines induce protective antibodies in > 99% of vaccines following pre-/ post- •• Vaccine efficacy against sympto­
exposure prophylaxis. matic virologically confirmed
dengue is 55-60%
•• The dengue vaccine has finished
D. About Rabies immunoglobulin (RIG) phase II trials in India.

What is it? Specific anti-rabies antibodies that provide passive protection

Indication RIG is indicated in all cases of category 3 wounds

Dosage: Human rabies immunoglobulin (20 U/kg body weight, maximum dose 1500 IU);
Equine rabies immunoglobulin (40 U/kg, maximum dose 3000 IU)
L at e s t U p d at e s
Preference HRIG is preferred, but if not available/unaffordable ERIG may be used
Vaccines recommended for adoles­
How to •• RIG should be infiltrated thoroughly into and around the wound cents include:
administer? •• The remaining part if any is to be injected IM into the deltoid region or anterolateral Tdap, Td, TT, HPV (females) & in special
aspect of thigh away from the site of vaccine administration circumstances: Influenza, Japanese
•• Most of the new ERIG preparations are potent, safe, highly purified & less expensive encephalitis, Pneumococcal poly­
•• As per latest recommendations from WHO, skin testing prior to ERIG administration is saccharide & Rabies vaccines.
not recommended as skin tests do not accurately predict anaphylaxis risk

High Yield Points


Children who have received pre­viously full rabies postexposure pro­phylaxis should be given only two
booster doses, intramus­cularly on days 0 and 3, irrespective of the duration of previous vaccination. In these
situations treat­ment with Rabies Ig is not necessary.
198 Section 1: General Pediatrics

 VACCINATION IN SPECIAL SITUATIONS


Review of Pediatrics and Neonatology

High Yield Points


•• Meningococcal vaccine protects
Preterm and low •• Should receive all routine recommended vaccines at same chronological age
against A, C, Y, W-135 strains
birth weight babies: with few exceptions; No reduction in vaccine doses required
•• Vaccine used for Cholera is WC-rBS
vaccine (Dukoral oral vaccine) Babies of HIV •• Immunization should take place as early in life as possible
•• Rabies vaccine can be given during infected mothers: •• IAP, WHO & CDC recommend all the live vaccines in asymptomatic HIV1
pregnancy infected children except BCG & OPV
•• Yellow fever vaccine is a live •• However, in a symptomatic child all live vaccines are forbidden, but measles/
attenuated vaccine derived from 17D MMR/varicella vaccines may be considered
strain •• Yellow fever vaccine is contraindicated in both symptomatic and asymptomatic

Child on steroids •• Topical, local injections or aerosol: no contraindication to Live vaccines


•• HIGH dose: >= 2 mg/kg/day of prednisolone or equivalent to a total of
20 mg/day for children more than 14 days—wait for 1 month

After IV •• Antibody response to Live virus vaccine diminishes but no inhibition of


Immunoglobulin responses to killed vaccines or toxoids; Hence live vaccines given 14 days before
administration or 3 months after IVIg

Immunodeficiency •• Severe immunodeficiency: all live vaccines contraindicated


•• Mild/moderate: given if benefits outweigh risks

Answer Keys for Image-Based Questions

Answers Explanations / Identifying features


1. Ans. c. Tuberculin syringe This is a 1 ml syringe, used for intradermal Mantoux test & BCG vaccination (both contain tuberculin); So, it
is called ‘Tuberculin syringe’
2. Ans. d. 6-12 weeks A scar in the left upper arm, at the site of BCG vaccination, takes 6-12 weeks to form
3. Ans. d. Use the vaccine if This is a vaccine vial monitor (VVM) showing white inner square & blue outer circle; VVM provides an
expiry date not reached indication of the cumulative heat to which the vial has been exposed
Chapter 9: Immunization 199

Questions
Questions
 GENERAL PRINCIPLES 11. Which vaccines are recommended at birth as per the
universal immunization programme? M 
1. Vaccines that must be given before age of 2 years  (Recent Question 2015)
according to IAP: (PGI Nov 2017) a. BCG b. BCG & Hepatitis B
a. Varicella b. MMR c. BCG & DPT d. DT & OPV
c. Conjugated typhoid 12. Which vaccine cannot be given to a child < 2 years
d. Conjugated pneumococcal age? M  (Recent Question 2015)
e. HPV a. BCG b. Pneumococcal conjugate
2. A child presented at 18 months of age who has never c. Pneumococcal polysaccharide
been vaccinated before. Which vaccines will you d. Haemophilus influenzae
administer?  (AIIMS Nov 2017) 13. Active immunity is not acquired by:
a. DPT Booster and OPV b. Pentavalent vaccine  (Recent Question 2015)
c. BCG and OPV a. Infection b. Vaccination
d. Pentavalent, BCG, MMR, OPV c. Immunoglobulin transfer d. Subclinical infection
3. Which vaccine can not be used in adults? M  14. Which of the following vaccines is given subcuta­
 (FMGE pattern 2017) neously? (Recent Question 2015)
a. Hep A b. MMR a. BCG b. DPT
c. DPT d. Pneumococcal c. Hep B d. Measles
4. An eight-month-old child had history of unusual crying, 15. According to latest guidelines of vaccination, which of
altered sensorium & convulsions following previous the following is applicable at the age of 5 years?
vaccination after BCG, DPT & OPV (first dose) &  (AIIMS November 2013)
Hepatitis B. Now parents have brought child for next
a. DT booster + Vitamin A b. DT
doses of vaccination. Which vaccine is contraindicated
c. DPT + OPV d. DPT + Vitamin A
in this situation? (Recent Question 2017)
16. Killed vaccine is: M  (Recent Question 2013)
a. Measles b. DPT
c. Hepatitis B d. DT a. Hepatitis A b. Measles
5. An event that is caused by an error in vaccine c. OPV d. BCG
preparation, handling, or administration is called as: 17. Which of the following is called first immunization of
 (Recent Question 2017) the baby? (Recent Question 2013)
a. Coincidental event b. Injection reaction a. Colostrum
c. Vaccine reaction d. Programme error b. Handing over the baby to mother
6. Vaccines that must be given before age of two years c. OPV
according to IAP are: (PGI Nov 2016) d. DPT + BCG
a. Varicella b. MMR 18. Not given in adolescent: M ( NEET Pattern 2013)
c. Conjugated typhoid d. Pneumococcal a. MMR vaccine b. Tetanus vaccine
e. Hep B c. Rotavirus vaccine d. HPV vaccine
7. ‘Mission Indradhanush' includes all of the vaccines 19. Immunoglobulin is not used in prophylaxis of:
except: M  (Recent Question 2016)  (WBPG 2012)
a. BCG b. Japanese Encephalitis a. Hepatitis A b. Varicella
c. Typhoid d. Measles c. Influenza d. Measles
8. All of the following diseases are covered under recently 20. Mass vaccination is ineffective in:
launched “MISSION INDRADHANUSH” program  (Recent Question 2012)
except: (APPG 2016) a. Measles b. Poliomyelitis
a. Japanese encephalitis b. Hepatitis B c. Tetanus d. None of the above
c. Whooping cough d. Diphtheria 21. False about vaccines: (AIIMS May 2011)
9. Childhood vaccines are administered by all of the a. Thiomersal is used as preservative in DPT vaccine
following routes except: M  (MAHA PGM CET 2016) b. Kanamycin is used as preservative in measles vaccine
a. Inhalation b. Intravenous c. Neomycin is used as preservative in BCG vaccine
c. Intradermal d. Percutaneous d. Magnesium chloride used to stabilize OPV
10. All of the following vaccines are live vaccines except: M 22. All are live vaccines except:  (PGI May 2011)
 (MAHA PGM CET 2016) a. Japanese encephalitis b. Rabies
a. BCG b. OPV c. Poliomyelitis d. Typhoid
c. DPT d. Measles e. Measles
200 Section 1: General Pediatrics

23. Which disease is prevented by giving booster dose to a 37. All of the following are killed vaccines except:
Review of Pediatrics and Neonatology

5-6 years old child?  (DNB December 2011)  (AIIMS Dec 1997)
a. Measles b. BCG a. Salk Polio b. Japanese encephalitis
c. DT d. DPT c. Rabies d. Yellow fever
24. Killed vaccine among the following is: M  (JIPMER 2011)
 BCG VACCINE
a. Oral Polio Vaccine b. Hepatitis B
c. Yellow fever d. Japanese encephalitis 38. BCG vaccine gives protection against:
25. Vaccine with maximum efficacy (AIIMS May 2010)
a. CNS TB and pulmonary TB (AIIMS Nov 2018)
a. OPV b. Measles b. CNS TB and disseminated TB
c. BCG d. TT c. Pulmonary TB
26. The guidelines for immunization against infectious d. Pulmonary TB and skeletal TB
disease includes all of the following except: 39. Osteitis may be seen as a side effect of:
 (UP PG 2009) a. Hepatitis B vaccine b. BCG (WB PGMEE 2014)
a. Live vaccines are contraindicated in pregnant women c. Measles vaccine d. IPV
b. Live vaccines should not be administered within 3 months 40. Which is true about BCG?
of giving human immunoglobulin  (AIIMS May 2011, AIIMS May 2005)
c. Inactivated vaccines should not be given if there has been a. Distilled water is used as diluent
significant reaction to previous dose b. Site for injection is cleaned with spirit
d. Hay fever, sickle cell anemia and tuberculosis are c. Mantoux test positive in 6 weeks
contraindications for vaccination d. WHO recommends Danish 1331 for vaccine production
27. Most heat sensitive vaccine is: M  (AIIMS Nov 2008) 41. Strain used for BCG vaccine: (AIPGMEE 07)
a. BCG b. Polio a. Edmonston Zagreb strain b. Oka strain
c. Measles d. DPT c. Danish 1331 d. RA 27/3 strain
28. Which is not given at the time of birth? M 
 (DPG 2008, DPG 2007)  POLIO VACCINE
a. OPV b. BCG
c. Hepatitis B d. Hib vaccine 42. Vaccine associated paralytic poliomyelitis is defined as
29. Active and passive immunity should be given together in those cases of acute flaccid paralysis:
all except: (AIPMGE 2006, AIIMS Nov 01)  (Recent Question 2017)
a. Tetanus b. Rabies a. Who have residual weakness 6 months after the onset of
c. Measles d. Hepatitis B paralysis and from whose stool samples, vaccine related
30. Antisera is obtained from: (DPG 2005) poliovirus but no wild virus is isolated.
b. Who have residual weakness 60 days after the onset of
a. Guinea pig b. Rabbit paralysis and from whose stool samples, vaccine related
c. Rat d. Horse poliovirus but no wild virus is isolated.
31. Which of the following is a live vaccine? M  c. Who have residual weakness 6 days after the onset of
 (DNB Dec 2004) paralysis and from whose stool samples, vaccine related
a. Salk polio vaccine b. Sabin polio vaccine poliovirus but no wild virus is isolated.
c. Hepatitis B vaccine d. Rabies vaccine d. Who have residual weakness 60 days after the onset of
32. Which one of the following is a conjugated vaccine: M  paralysis and from whose stool samples, wild polio virus
 (UPSC 04)
is isolated.
43. Salk vaccine is a: M  (Recent Question 2016)
a. Hepatitis B b. Rubella
a. Live vaccine b. Live attenuated vaccine
c. Haemophilus influenzae d. Pertussis
c. Killed vaccine d. Toxoid
33. Live vaccines are all except: (PGI Dec 2001)
44. Which of the following serotypes of Polio virus is most
a. BCG b. Measles commonly associated with vaccine associated paralytic
c. Polio d. Rabies poliomyelitis? (MAHA PGM CET 2015)
e. Typhoid a. Serotype 1 b. Serotype 2
34. Live vaccine are: (PGI June 01) c. Serotype 3 d. Serotype 1 and 2
a. BCG b. Oral polio Vaccine 45. A PHC medical officer wants to give pulse polio
c. Chickenpox vaccine d. MMR immunization. The following finding in the Vaccine Vial
e. DPT Monitor (VVM) indicates that the vaccine should not be
35. Vaccine (s) not to be frozen is/are (PGI June 01) used: (Recent Question 2014)
a. BCG b. OPV a. Inner square is white, outer circle is blue
c. DPT d. Measles b. Inner square is blue but lighter than outer circle
e. TT c. Inner square matches the color of the outer circle
36. Administration of which vaccine can result in paralysis d. Inner square is lighter than outer circle
in children? (DNB 2000) 46. OPV Bivalent vaccine contain: (Recent Question 2014)
a. Measles vaccine b. Sabin polio vaccine a. P1 & P2 b. P1 & P3
c. DT vaccine d. DPT vaccine c. P2 & P3 d. P1, P2 & P3
Chapter 9: Immunization 201

47. Which of the following vaccines is most sensitive to  HIB & PNEUMOCOCCAL VACCINE

Questions
heat? (MAHA PGM CET 2014)
a. Oral Polio vaccine b. BCG vaccine 58. All of the following vaccines can prevent pneumonia,
c. DPT vaccine d. Measles vaccine except: (DNB June 2018)
48. Zero dose of polio vaccine is given: (DNB 2007) a. Measles b. Rubella
a. Before giving DPT b. At birth c. Pneumococcal d. H. influenzae
c. When child is having diarrhea 59. All are true regarding Hib vaccine except:
d. When child is having polio  (JIPMER 2017)
49. The efficiency of cold chain system for oral polio a. Protects against non capsulated H. influenza disease
vaccine as monitored by Vaccine Vial Monitor (VVM) b. Catch up vaccination before 5 years
c. Can be given in less than 6 months
depends on: (AIPGME 2004)
d. Given 2 weeks before splenectomy
a. Change in the colour of vaccine 60. A child who has not been vaccinated with
b. Temperature indicator of the system H. influenzae comes to the OPD at 14 months. How
c. Viral potency test many doses, including booster would you like to give to
d. Change in colour of monitor this child? (Recent Question 2015)
a. 4 doses b. 3 doses
 DPT & TT VACCINE c. 2 doses d. Only one dose

50. Which of the following is true regarding whole cell 61. A child unimmunized previously with Hib vaccine,
pertussis vaccine? (JIPMER 2017)
received 1st dose of Hib vaccine at 16 months, the next
appropriate step is: (WBPG 2010)
a. Maternal antibody provides protection against pertussis
a. Give 3 doses of Hib at 2 months interval
b. Effectiveness against pertussis is more than diphtheria/
b. No further doses needed
tetanus
c. Give one booster dose
c. Progressive neurological deficit is an absolute contra­
d. Give two booster doses at 4 weeks interval
indication
d. Not recommended for children more than 7 years of age
 MEASLES & MMR VACCINE
51. Whole cell pertussis vaccine used to prevent infection
can protect against? (JIPMER Nov 2017) 62. True regarding MMR vaccine is: (PGI May 2016)
a. Bortedella parapertussis only a. May lead to local reaction in approx. 10% of vaccinated
b. Mild forms of Bordetella pertussis only population
c. Both Bordetella pertussis & parapertussis b. It is a live vaccine
d. Only against severe Pertussis c. It can be given at age 9 months
52. Pertussis vaccine side effect: (Recent Question 2016) d. MMR can be given to immunocompromised but not to
a. Local pain b. Excessive cry pregnant females
c. Fever d. All of the above e. Jeryl Lynn strain has the lowest incidence of post vaccine
53. All of the following statements are true about DPT aseptic meningitis
vaccine except: (Recent Question 2015) 63. In measles outbreak, measles vaccine can be given
a. It should be stored in deep freezer within: (Recent Question 2013)
b. Exposure to direct sunlight when in use should be avoided a. 2-3 months b. 3-5 months
c. Stored stocks are needed for the months at PHC level c. 2-7 months d. 6-9 months
d. Half used vials should not be put back into the cold chain 64. Name of mumps vaccine is:
after the session  (Recent Question 2012, 2013)
54. Which one of the following doses in Loffler … Diphtheria a. Jeryll Lynn b. Edmonston
Toxoid is incorporated in DPT? (Recent Question 2014) c. Danish 1331 d. OKA
a. 5 b. 15 65. Which type of vaccine is MMR? (DNB December 2010)
c. 25 d. 35 a. Live attenuated b. Killed
c. Toxoid d. Subunit
55. Protective levels of Tetanus anti-toxin is:
 (Recent Question 2012)  RABIES IMMUNISATION
a. >0.01 IU/mL b. >0.5 IU/mL
c. >1.0 IU/mL d. >5 IU/mL 66. Rabies pre-exposure prophylaxis includes how many
56. A full course of immunization with 3 doses of tetanus doses of vaccine? M  (FMGE pattern 2017)
toxoid, confers immunity for how many years? a. 1 b. 2
a. 5 b. 10 (DNB 2008) c. 3 d. 4
c. 15 d. 20 67. Dose of Human Rabies Immunoglobulin is:
57. Adjuvant used in DPT is:  (DNB Dec 2005)  (Recent Question 2015)
a. Silica b. Magnesium a. 10 IU/kg b. 20 IU/kg
c. Manganese d. Aluminium c. 30 IU/kg d. 40 IU/kg
202 Section 1: General Pediatrics

79. Which of the following Human papilloma virus subtypes


Review of Pediatrics and Neonatology

68. For the prevention of human rabies, immediate flushing


are not covered by Quadrivalent Anti-cervical cancer
and washing the wound(s) in animal bite cases, with
vaccine? (PGI May 2013)
plenty of soap and water, under running tap should be
a. Type 6 b. Type 7
carried out for how much time? (MAHA PGM CET 2015)
c. Type 11 d. Type 16
a. 2 minutes b. 1 minute e. Type 18
c. 15 minutes d. 5 minutes 80. True regarding Cervical cancer vaccine is/are:
69. Rabies vaccine for pre- exposure prophylaxis is given a. Bivalent and quadrivalent (PGI November 2013)
at? M  (DNB June 2010) b. Given to married women in 20-45 years age group
a. 0, 3, 7 days b. 0, 3, 7, 14 days c. Protects against subtypes 16, 18
d. Two doses given
c. 0, 3, 7, 14, 30 days d. 0, 7, 28 days
e. Gives 100% protection against cervical cancer
81. Typhoid Vi polysaccharide vaccine is usually adminis­
 OTHER VACCINES tered in children above the age of: M  (COMEDK 2011)
a. 6 months b. 1 year
70. What is the minimum age of vaccination of human c. 2 years d. 1 year 6 months
papilloma virus vaccine? (Recent Question 2017) 82. A child is hepatitis B immunized, the marker for
a. 11 years b. 9 years seroconversion is? (UP 2011)
c. 15 years d. 13 years a. HBs Ag b. Anti HBc IgM
71. About Hepatitis A vaccine schedule, true is: c. Anti HBs Ag d. Anti HBc IgG
 (Recent Question 2016) 83. HPV vaccine is:  (AIIMS November 2009)
a. Recommended at age of 12 months a. Monovalent b. Bivalent
b. 2 dose of killed vaccine 6 months apart c. Quadrivalent d. Bivalent & quadrivalent
c. 1 dose of live vaccine 84. Incorrect matches are: (PGI June 04)
d. All are true a. Measles: Jeryl-Lynn strain b. Chickenpox: OKA-strain
72. Rotavirus vaccine- contraindication is: c. Oral polio: Sabin
 (Recent Question 2016) d. Rubella: Edmonton Zagreb strain
a. SCID b. Intussusception
 VACCINATION IN SPECIAL SITUATIONS
c. Severe allergic reaction d. All of the above
73. Which among the following is vaccine for cholera? 85. A child presented at 18 months of age who has never
 (Recent Question 2015, Recent Question 2014) been vaccinated before. Which vaccines will you
a. PPV 23 b. Dukoral administer? (AIIMS Nov 2017)
c. Ty 21 a vaccine d. JE-MB Nakayama a. DPT Booster and OPV b. Pentavalent vaccine
74. Which of the following statement is not true about the c. BCG and OPV d. Pentavalent, MMR, OPV
following vaccines? (WB PGMEE 2015) 86. An un-immunized 13 months old child comes to you in
a. Rotavirus vaccine is a live vaccine OPD, according to the latest immunization schedule,
b. Vi capsular polysaccharide Typhoid vaccine can be what vaccines will you advise? (AIIMS May 2018)
administered in children at 18 months of age a. OPV 3 doses, 1 IPV, 3 Pentavalent
c. The Hib vaccine which are routinely given these days are b. BCG, OPV 3 doses, 3 IPV, 3 Pentavalent and 1 measles
conjugated vaccine c. OPV 3 doses, 1 IPV, 3 Pentavalent and 2 measles
d. IPV vaccine can be given at 6 weeks of life d. OPV 3 doses, 3 IPV, 3 DPT, 3 Hep- B
75. Which one of the following vaccines is contraindi- 87. A 11-month-old child came for vaccination. He had
cated in children with egg allergy? M  (APPG 2015) received Polio vaccine & BCG at birth. What vaccines
a. MMR b. Yellow Fever should be given to him now? (Recent Question 2016)
c. DPT d. BCG a. BCG, OPV, Hep B
76. Which of the following vaccines has maximum efficacy b. Measles, DPT, OPV, Hib, Hep B
after a single dose? (Bihar PG 2015) c. DPT, OPV, Hib, Hep B d. DPT, OPV, Hep B
a. Tetanus Toxoid b. DPT 88. All of the following are contraindicated during preg­
c. Measles d. Typhoid nancy except: (MAHA PGM CET 2015)
a. Measles vaccine b. Mumps vaccine
77. Which of the following vaccines can result in
c. HPV vaccine d. Rabies vaccine
Thrombocytopenia? (AIIMS MAY 2014)
89. An 18-month-old child, who has received one dose of
a. MMR vaccine b. Typhoid vaccine
DTP and OPV at 2 months of age. What will be your
c. Influenza vaccine d. HIB vaccine
next immunization plan? (Recent Question 2015)
78. Which vaccine is contraindicated in pregnancy? M 
a. Restart immunization schedule as per age
 (Recent Question 2014)
b. Measles, BCG, booster dose of DPT and OPV
a. Rubella b. Diphtheria c. Measles, booster dose of DPT and OPV
c. Tetanus d. Hepatitis B d. BCG, 2nd dose of DPT and OPV
Chapter 9: Immunization 203

90. A child is receiving corticosteroids for medical therapy. 91. A 9-month-old un-immunized child was brought to the

Questions
The following is not a contraindication in this child for dispensary, which vaccination should be given to this
receiving a live viral vaccine: baby at first visit? (DPG 2007)
 (Recent Question 2013) a. OPV + BCG
a. When child is receiving 2 mg/kg of prednisolone for at b. OPV + DPT + Measles
least 2 weeks at present c. OPV + BCG + DPT + Measles
b. When child has received 2 mg/kg of prednisolone for at d. OPV + DPT + BCG
least 2 weeks in the past one month 92. Which vaccine(s) is/are not contraindicated in
c. Child receiving 4 mg/kg of prednisolone for 4 weeks, prior Pregnancy? (PGI May 2012)
to 2 weeks a. Rubella b. Varicella
d. Chronic asthmatic child receiving low dose inhaled c. Hepatitis B d. Measles
steroid for 10 months e. Rabies
204 Section 1: General Pediatrics
Review of Pediatrics and Neonatology

Answers with Explanations


 GENERAL PRINCIPLES
1. a. Varicella, b. MMR, c. Conjugated Typhoid, d. Conjugated Pneumococcal  Ref: IAP Guidebook on Immunization 2013–14
2. d. Pentavalent, BCG, MMR, OPV  Ref: IAP Guidebook on Immunization 2013–14
3. c. DPT  Ref: IAP Guidebook on Immunization 2013–14, p 387
Tdap vaccine (single dose) is recommended in adolescents / adults & not DTP
Tdap contains lesser amount of Diphtheria & Pertussis antigens, compared to DTP
4. b. DPT  Ref: IAP Guidebook on immunization 2013-2014, p 141-142
Absolute contraindications to any pertussis vaccination (including DTwP vaccine) are: history of anaphylaxis or development
of encephalopathy within 7 days following previous DTwP vaccination.
Events such as persistent inconsolable crying of > 3 hours duration/ hyperpyrexia (fever > 40.5°C)/ hypotonic hyporesponsive
episodes (HHE) within 48 hours of DTwP administration and seizures with or without fever within 72 hours of administration of
DTwP are considered as precautions but not contraindications to future doses of DTwP because these events generally do not recur
with the next dose and they have not been proven to cause permanent sequelae.
Progressive/evolving neurological illnesses is a relative contraindication to first dose of DTwP immunization.
5. d. Programme error  Ref: IAP Guidebook on immunization 2013-14/ p 89; For details, refer pretext of this chapter
Event caused by an error in vaccine preparation, handling, or administration is called a ‘Programme error', e.g. deaths
following measles vaccination due to toxic shock syndrome resulting from improper reconstitution & storage of measles vaccine.
6. a. Varicella, b. MMR, c. Conjugated typhoid, d. Pneumococcal, e. Hep B
Discussing about the options,
Vaccine Recommended age of administration
a. Varicella Varicella vaccines are licensed for age 12 months and above.
b. MMR 1st dose is recommended at 15 months age
c. Conjugated typhoid Can be given between 9–12 months
d. Pneumococcal Conjugate vaccine can be given starting at 6 weeks age, but polysaccharide vaccines are given only beyond 2
year age
e. Hep B Can be given even at birth

7. c. Typhoid  Ref: Nelson's 20/e p 1246, 1292-1293, Ghai 8/e p 197-198
Mission Indradhanush
Includes vaccination against 7 life-threatening diseases (diphtheria, whooping cough, tetanus, polio, tuberculosis, measles
and hepatitis B) in the entire country
8. a. Japanese encephalitis  Ref: Nelson's 20/e p 1627-1628, Ghai 8/e p 201
Japanese encephalitis is not included among the seven life-threatening diseases of Indradhanush (with seven colours); It is
one of the additional vaccine given in selected districts. Rest 3 vaccines are given throughout the country under this programme;
So, among the given options, Japanese encephalitis is the best answer to this question.
9. b. Intravenous  Ref: Ghai 8/e p 193
None of the childhood vaccines are given intravenously.
10. c. DPT  Ref: Ghai 8/e p 193
DTwP is composed of tetanus and diphtheria toxoids as well as killed whole cell pertussis bacilli adsorbed on insoluble
aluminium salts which act as adjuvants.
11. b. BCG & Hepatitis B  Ref: Nelson's 20/e p 3212, Ghai 8/e p 190-191
Vaccines recommended at birth as per the universal immunization programme are: BCG, OPV zero dose & Hepatitis B vaccine.
12. c. Pneumococcal polysaccharide  Ref: IAP guidebook of immunization 2013-2014; Refer pretext for details
13. c. Immunoglobulin transfer  Ref: Nelson's 20/e p 1242-1259, Ghai 8/e p 188-189
Immunoglobulins give passive immunity & not active immunity.
14. d. Measles  Refer pretext of this chapter for details;
15. d. DPT + Vitamin A  Ref: Nelson's 20/e p 3212, Ghai 8/e p 193-194
DPT booster & Vitamin A supplementation are recommended at 5 years age.
16. a. Hepatitis A  Ref: Nelson's 20/e p 1945-1949, Ghai 8/e p 195-197
17. a. Colostrum  Ref: Ghai 8/e p 192
Colostrum is rich in immunoglobulins & other substances that protect the baby against infections; So, Colostrum is called the ‘First
immunization' of the baby.
Chapter 9: Immunization 205

18. c. Rotavirus vaccine  Ref: Nelson's 20/e p 1618

Answers with Explanations


High Yield Points
•• The maximum age for the first dose of Rotavirus vaccine is 14 weeks, 6 days, & final dose is 8 months
•• Vaccination should not be initiated for infants aged 15 weeks or older

19. c. Influenza  Ref: IAP Guidebook on immunization 2013-2014, p 378, Nelson's 20/e p 1603, Ghai 8/e p 201-202
Following immunoglobulins are used for secondary prophylaxis: Tetanus, Hep A, Hep B, Rabies & Varicella Ig.
20. c. Tetanus  Ref: IAP 49, Nelson's 20/e p 1432-1434, 1546-1547, Ghai 8/e p 145-150
‘Herd effect'
•• It is defined as “reduction of infection or disease in unimmunized segment as a result of immunizing a proportion of
population”
•• Herd effect is due to reduced carriage of the causative microorganism by the vaccinated cohort and thus is seen only with
vaccines against those diseases where humans are the only source
•• An effective vaccine is a prerequisite for good herd effect; e.g. conjugated pneumococcal vaccine
•• Tetanus and BCG vaccines have no herd effect So, mass vaccination is ineffective
21. c. Neomycin is used as preservative in BCG vaccine & b. Kanamycin is used as preservative in measles vaccine 
Ref: Nelson's 20/e p 3212-3214, Ghai 8/e p 190-191
Additional substances present in different vaccines & their function:
Vaccine Additional substance (function)
DPT ThiomersalQ (Preservative); Aluminium hydroxideQ (adjuvant)
Measles No Preservative
BCG No preservativeQ
OPV Magnesium chloride (Thermostabilizer)Q

22. b. Rabies  Ref: IAP 260, Nelson's 20/e p, Ghai 8/e p 202-203
•• For Japanese Encephalitis, Polio & Typhoid, both live & killed vaccines are available
•• New generation typhoid vaccines include oral live attenuated Ty21a vaccine, Parenteral Vi-polysaccharide & Vi Polysaccharide
Conjugate vaccines; For Rabies, only inactivated vaccines are used.
23. d. DPT  Ref: Ghai 8/e p 193
24. b. Hepatitis B  Ref: Nelson's 20/e p 1255, Ghai 8/e p 196
•• Hepatitis B is an inactivated (killed) vaccine, while OPV & Yellow fever are live vaccines
•• For Japanese encephalitis, both live & killed vaccines are available.
25. b. Measles  Ref: Nelson's 20/e p 1546, Ghai 8/e p 145; IAP guidebook of Immunisation 2013-14, p 222-223
Seroconversion rates after measles vaccination are around 60% at the age of 6 months, 80–85% at the age of 9 months and beyond
95% at the age of 12–15 months.
26. d. Hay fever, sickle cell anemia and tuberculosis are contraindications for vaccination  Ref: Nelson's 20/e p 1564; IAP
guidebook of Immunisation 2013-14, p 70
True contraindications to vaccination are:
•• Severe allergic reaction to a vaccine component or following a prior dose of a vaccine;
•• Encephalopathy occurring within 7 days of pertussis vaccination;
•• Severe combined immunodeficiency (SCID) as a contraindication to rotavirus vaccine
27. b. Polio  Ref: Ghai 8/e p 191-192
28. d. Hib vaccine  Ref: Ghai 8/e p 192
29. c. Measles  Ref: Nelson's 20/e p 1546, Ghai 8/e p 145-150
30. d. Horse  Ref: Ghai 8/e p 175
Antisera are usually prepared in horses; they provide passive immunization; Antisera are used against diseases like Diphtheria,
Tetanus, Rabies
31. b. Sabin polio vaccine  Ref: Nelson's 20/e p 1560, Ghai 8/e p 192
32. c. Haemophilus influenzae  Ref: Nelson's 20/e p 1546, Ghai 8/e p 145-150
33. d. Rabies  Ref: Nelson's 20/e p 1546, Ghai 8/e p 145-150
Note: Both live & killed typhoid vaccines are available
34. a. BCG, b. Oral polio Vaccine, c. Chickenpox vaccine, d. MMR  Ref: Nelson's 20/e p 1546, Ghai 8/e p 145-150
35. c. DPT, e. TT  Ref: Nelson's 20/e p 1546, Ghai 8/e p 145-150; Refer pretext of this chapter for details;
36. b. Sabin polio vaccine  Ref: Nelson's 20/e p 1546, Ghai 8/e p 145-150
37. d. Yellow fever  Ref: Nelson's 20/e p 1546, Ghai 8/e p 145-150;
206 Section 1: General Pediatrics

 BCG VACCINE
Review of Pediatrics and Neonatology

38. b. CNSTB & and disseminated TB  Ref: Nelson 20/e p1460
BCG is 50% effective in preventing pulmonary tuberculosis in adults and children. The protective effect for disseminated and
meningeal tuberculosis appears to be slightly higher, with BCG preventing 50–80% of cases.
39. b. BCG  Ref: IAP Guidebook on Immunization 2013-14; pg 113-117, Nelson's 20/e p 3212, Ghai, 8/e p 190-191
Side effects of BCG vaccine
•• BCG adenitis: Associated with subcutaneous administration of BCG
•• Disseminated BCG infection is extremely unusual but may occur in children with cellular immunodeficiency
•• Osteitis is a rare complication of BCG vaccination that appears to be related to certain strains of the vaccine
•• Systemic complaints such as fever, convulsions, loss of appetite, & irritability are after BCG

40. d. WHO recommends Danish 1331 for vaccine production Ref: IAP Guidebook on Immunization 2013-14; pg 113-117,
Nelson's 20/e p 3212, Ghai, 8/e p 190-191
Discussing about the options one by one,
Option a False; Sterile normal saline may be used as diluent for BCG
Option b False; For BCG Vaccination, the selected site may be swabbed clean using sterile saline and local antiseptics should be
avoided
Option c False; Of BCG vaccine recipients whose initial tuberculin skin test is negative, 10%–25% will have a positive tuberculin skin
test if they are retested within 1–4 weeks—the so-called “booster phenomenon”
Option d True; The two common strains of for use in BCG vaccine are Copenhagen (Danish 1331) and Pasteur
41. c. Danish 1331 Ref: IAP Guidebook on Immunization 2013-14; pg 113-117, Nelson's 20/e p 3212, Ghai, 8/e p 190-191

 POLIO VACCINE
42. b. Who have residual weakness 60 days after the onset of paralysis and from whose stool samples, vaccine related
poliovirus but no wild virus is isolated  Ref: IAP Guidebook on Immunization 2013-14; p 119
Vaccine associated paralytic poliomyelitis (VAPP) is defined as those cases of AFP which have residual weakness 60 days after the
onset of paralysis and from whose stool samples, vaccine-related poliovirus but no wild poliovirus is isolated.
43. c. Killed vaccine  Ref: IAP Guidebook on Immunization 2013-14; pg 113-117, Nelson's 20/e p 3212, Ghai, 8/e p 190-
191
Polio vaccines were developed in the USA during 1950, first the inactivated polio vaccine (IPV) by Jonas Salk and later the live oral
polio vaccine (OPV) by Albert Sabin. Remember: ‘Salk' is a ‘killed' vaccine.
44. c. Serotype 3  Ref: IAP 119, Nelson's 20/e p 1246, Ghai 8/e p 192;
45. c. Inner square matches the color of the outer circle  Ref: Nelson's 20/e p 1561, Ghai 8/e p 190-194
Interpretation of the color change of VVM is as follows:
•• Inner square is white, or lighter than outer circle: If the expiry date has not passed, vaccine can be used
•• Inner square matches color of outer circle or is darker than outer circle: Vaccine should be discarded, regardless of the expiry
date.
46. b. P1 & P3  Ref: Nelson's 20/e p 1561, 3212, Ghai 8/e p 190-194
•• Monovalent OPVs (mOPVs) & bivalent OPV (bOPV) are 2.5–3 times more efficacious than trivalent OPV
•• In bivalent OPV, the type 2 component is removed

47. a. Oral Polio vaccine  Ref: Nelson's 20/e p 1561, Ghai 8/e p 190-194
Of the given vaccines, OPV is the most heat sensitive. Refer pretext of this chapter for details
48. b. At birth  Ref: Nelson's 20/e p 1561, Ghai 8/e p 190-194
OPV zero dose: At birth or as early as possible within the first 15 days.
49. d. Change in colour of monitor  Ref: Nelson's 20/e p 1561, Ghai 8/e p 190-194
Vaccine Vial Monitor (VVM)
•• It is a time and temperature sensitive colored label that provides an indication of cumulative heat to which the vial has
been exposed
•• Whether the OPV vial can be used or not depends upon the irreversible colour changes in vaccine vial monitor.

 DPT & TT VACCINE


50. c. Progressive neurological deficit is an absolute contra­indication  Ref: Nelson 20/e p 1381
Pregnant women should be given Tdap during every pregnancy to provide passive antibody protection to the infant until administration
of DTaP. Optimal timing of maternal Tdap is 26 through 37 wk of gestation but Tdap can be given at any time during pregnancy.
Progressive neurological deficit and anaplnglasis to previous doses is a contraindication for pertussis vaccination
51. c. Both Bordetella pertussis & parapertussis  Ref: Clin Infect Dis. 2017 Jun 15;64(12):1795-1797
Chapter 9: Immunization 207

Pertussis vaccine may induce cross-immunity & has been found to have 66%-82% effectiveness against parapertussis.

Answers with Explanations


Controversial; Some sources say: As measles can depress cell mediated immunity & as BCG induces cell mediated immunity BCG
should not be given along with measles or MMR; while some do not comment about it; So, there is not much recent evidence to
substantiate the same.
52. d. All of the above  Ref: IAP Guidebook on Immunization 2013-14; pg 141
Adverse effects of Pertussis vaccination are:
Minor: like pain, swelling and redness at the local site, fever, fussiness, anorexia and vomiting
Serious adverse effects are persistent crying (most common), fever > 40.5°C, hypotonic hyporesponsive episodes , seizures &
encephalopathy.
53. a. It should be stored in deep freezer  Ref: Nelson's 20/e p 1381, Ghai 8/e p 193-194
•• Vaccines susceptible to damage by freezing include mainly all aluminium adjuvanted vaccines (DTwP, DTaP, TT, DT, Td,
TT, hepatitis B, combination vaccines, hepatitis A, HPV, PCV 7) but also other vaccines including IPV, PPV 23, inactivated
influenza vaccines, meningococcal vaccines, rotavirus vaccines, typhoid vaccines, & Hib and vaccines
•• Vaccines that can be frozen without harm include OPV (vial must not be frozen and thawed repeatedly), and lyophilized
measles, MMR, BCG vaccines, live attenuated influenza vaccine, certain brands of varicella and MMRV.
54. c. 25  Ref: Nelson's 20/e p 1381, Ghai 8/e p 193-194
Diphtheria toxoid 20–30 Lf & tetanus toxoid 5–25 Lf in DPT vaccine.
55. a. >0.01 IU/mL  Ref: Nelson's 20/e p 1381, Ghai 8/e p 193-194
Protective titer for diphtheria > 0.1 IU/ml and for tetanus > 0.01 IU/ml.
56. b. 10  Ref: IAP 157, Nelson's 20/e p 1255, Ghai 8/e p 196-197
In individuals who have completed primary & booster vaccination with DTP, TT boosters every 10 yrs provide sufficient protection.
57. d. Aluminium  Ref: Nelson's 20/e p 1381-1382, Ghai 8/e p 193

 HIB & PNEUMOCOCCAL VACCINE


58. b. Rubella  Ref: Nelson 20/e p 1545
Pneumonia is the most common cause of death in measles. It may manifest as giant cell pneumonia caused directly by the viral
infection or as superimposed bacterial infection. The most common bacterial pathogens are Streptococcus pneumoniae, Haemophilus
influenzae, and Staphylococcus aureus. So, vaccination against Measles, Pneumococcus & H.influenzae would give some protection
against pneumonia.
59. a. Protects against non capsulated H. influenza disease  Ref: Nelson 20/e p 1375, 2332
Hib conjugate vaccine
It protects against encapsulated H. influenzae type B strain only.
Routine vaccination:
•• Minimum age: 6 weeks
•• Primary series includes Hib conjugate vaccine at ages 6, 10, 14 weeks with a booster at age 12 through 18 months.
Catch-up vaccination:
•• Catch-up is recommended till 5 years of age.
•• 6–12 months; 2 primary doses 4 weeks apart and 1 booster;
•• 12–15 months: 1 primary dose and 1 booster;
•• Above 15 months: single dose.
•• If the first dose was administered at age 7 through 11 months, administer the second dose at least 4 weeks later and a final dose at
age 12–18 months at least 8 weeks after the second dose.
Vaccines for encapsulated organisms, such as pneumococcus, meningococcus, and H. influenzae type b, should be administered
at least 14 days before splenectomy, and prophylactic oral penicillin VK prescribed indefinitely.
60. c. 2 doses  Ref: Nelson's 20/e p 1948-1949, Ghai 8/e p 196-197
Haemophilus influenzae type b (Hib) conjugate vaccine schedule:
Routine vaccination Catch-up vaccination
•• Minimum age: 6 weeks •• Catch-up is recommended till 5 years of age.
•• Primary series includes Hib conjugate vaccine at ages 6, 10, 14 •• 6–12 months; 2 primary doses 4 weeks apart & 1 booster;
weeks with a booster at age 12 through 18 months. •• 12–15 months: 1 primary dose and 1 booster;
•• Above 15 months: Single dose

61. b. No further doses needed  Ref: Nelson's 20/e p 3450, 1642-1644, Ghai 8/e p 202-203

 MEASLES & MMR VACCINE


62. b. It is a live vaccine, d. MMR can be given to immunocompromised but not to pregnant females & e. Jeryl Lynn strain
has the lowest incidence of post vaccine aseptic meningitis  Ref: Nelson's 20/e p 1546-1549; Miller et al. Am J Epidemiol
(2007) 165 (6): 704-709.
208 Section 1: General Pediatrics

Discussing about the options one by one,


Review of Pediatrics and Neonatology

a. False; Adverse events from MMR vaccine include fever (usually 6-12 days following vaccination), rash in 5% of vaccinated
persons, and, rarely, transient thrombocytopenia
b True; MMR is a live vaccine
c. False; 2 doses of MMR vaccine are recommended; 1st at the age of 12–15 months & 2nd at school entry (4–6 years) or at any time
8 weeks after the first dose.
d. True; Live vaccines should not be administered to pregnant women or to immunodeficient patients. However, patients with HIV
who are not severely immunocompromised can be given MMR vaccine.
e. True; Multiple studies showed that aseptic meningitis was associated with all mumps vaccine strains except the Jeryl Lynn strain.
63. d. 6-9 months  Ref: IAP 224, Nelson's 20/e p 1381, Ghai 8/e p 193-194
In case of an outbreak, the measles vaccine can be given to infants as young as 6 completed months of age.
64. a. Jeryll Lynn  Ref: IAP 228, Nelson's 20/e p 1381, Ghai 8/e p 193-194
Mumps vaccine virus strains include Leningrad-Zagreb, Leningrad-3, Jeryl Lynn strains and are grown in chick embryo/human
diploid cell cultures.
65. a. Live attenuated  Ref: IAP 33, Nelson's 20/e p 1381, Ghai 8/e p 193-194

Commonly used live attenuated vaccines include BCG, oral polio, measles, MMR and chicken pox vaccines

 RABIES IMMUNISATION
66. c. 3  Ref: Nelson's 20/e p 3450
67. b. 20 IU/kg  Ref: Nelson's 20/e p 3450, 1642-1644, Ghai 8/e p 202-203
•• Human rabies immunoglobulin (HRIG— dose is 20 U/kg body weight, maximum dose 1500 IU); and
•• Equine rabies immunoglobulin (ERIG—dose is 40 U/kg, maximum dose 3000 IU)
68. c. 15 minutes  Ref: Nelson's 20/e p 3450, 1642-1644, Ghai 8/e p 202-203; Refer pretext for details
In case of animal bite wounds, the 1st step is thorough cleansing of the wound with soap & flushing under running water for 10
minutes; as 15 minutes is the option closest to 10 minutes, that is the best answer.
69. d. 0, 7, 28 days  Ref: Nelson's 20/e p 3450, 1642-1644, Ghai 8/e p 202-203
For pre-exposure prophylaxis of Rabies, 3 doses of Rabies vaccine are given intramuscularly on days 0, 7 & 28

 OTHER VACCINES
70. b. 9 years  Ref: IAP Guidebook on Immunization 2013-14; pg 279
Minimum age for HPV vaccine administration is 9 years; Refer pretext of this chapter for details
71. d. All are true  Ref: IAP guidebook of Immunisation 2013-14, p254-255
As per IAP, Two doses of both killed and live hepatitis A vaccines were recommended in 2013-14;
IAP ACVIP recommends initiating hepatitis A vaccine at the age of 12-23 months & separate the 2 doses by 6 to 18 months
72. d. All of the above  Ref: IAP ACVIP recommendations, 2016
2 doses of inactivated vaccine at least 6 months apart or only a single dose of live attenuated vaccine needs to be given.
73. b. Dukoral  Ref: IAP guidebook of Immunisation 2013-14, p 344, Nelson's 20/e p 140
•• The WC-rBS vaccine available internationally as Dukoral oral vaccine is widely used against Cholera
•• It is a vaccine comprising of killed V. cholerae O1 with recombinant b subunit of cholera toxoid.
74. b. Vi capsular polysaccharide Typhoid vaccine can be administered in children at 18 months of age  Ref: IAP 265;
IAP ACVIP recommends the administration of the currently available Vi polysaccharide Typhoid vaccine 0.5 ml IM every three
years beginning at the age of 2 years.
75. b. Yellow Fever  Ref: Nelson's 20/e p 1633
Immediate severe hypersensitivity or anaphylactic reactions is seen with Yellow fever vaccine, most commonly in people with
allergies to eggs or gelatine.
76. c. Measles  Ref: Park 22nd ed/p 136-139, Nelson's 20/e p 1546, Ghai 8/e p 145
Efficacy with single dose of:
•• Rubella: 95% (maximum) • OPV: 65-80%
•• Measles: 85% • BCG: 50% (maximum)
• Tetanus toxoid: 70%, So, among the given options, Measles vaccine is the best answer.
77. a. MMR vaccine  Ref: IAP 223, Ghai 8/e p 145
With Measles vaccine, Thrombocytopenic purpura may occur at a frequency of 1/30,000 vaccinees.
One of the components of MMR is the Measles vaccine; So MMR vaccine can cause thrombocytopenia.
78. a. Rubella   Ref: Nelson's 20/e p 1551, Ghai 8/e p 195-196
Rubella infection during pregnancy can lead to severe consequences in the fetus.
79. b. Type 7  Ref: Nelson's 20/e p, Ghai 8/e p 200-201
Quadrivalent HPV vaccine contains serotypes 16, 18, 6 & 11 with aluminium containing adjuvant.
Chapter 9: Immunization 209

80. a. Bivalent and quadrivalent, c. Protects against subtypes 16, 18  Ref: Nelson's 20/e p 1246

Answers with Explanations


About Cervical cancer vaccine or HPV vaccine:
•• Either HPV4 (0, 2, 6 months) or HPV2 (0, 1, 6 months) is recommended for females aged 11 or 12 years
•• The vaccine series can be started beginning at age 9 years & can be given upto 45 years if not previously vaccinated
•• 3 doses of quadrivalent HPV vaccine at 0, 2 & 6 months have shown 99% efficacy against types 16, 18 related CIN- 2/3 99–100% efficacy was
seen against vaccine type related genital warts, vaginal & vulvar intraepithelial neoplasia

81. c. 2 years  Ref: Ghai 8/e p 175
82. c. Anti HBs Ag  Ref: Nelson's 20/e p 1948-1949, Ghai 8/e p 196-197
An anti-HBs concentration of 10 mIU/ml measured 1–3 months after administration of the last dose of the primary vaccination
series is considered a reliable correlate of protection against infection.
83. d. Bivalent & quadrivalent  Ref: Nelson's 20/e p 1621-1622, Ghai 8/e p 1246
84. a. Measles: Jeryl-Lynn strain and d. Rubella: Edmonton Zagreb strain  Ref: Nelson's 20/e p 1546;
For Measles vaccine: Edmonston Zagreb strain is used; For Rubella vaccine: RA 27/3 vaccine strain is used.

 VACCINATION IN SPECIAL SITUATIONS


85. d. Pentavalent, MMR, OPV  Ref: Ghai 9/e p 187
A measles containing vaccine should be given in an unimmunized child beyond 9 months age. OPV should be given at less than 5
years age.
86. c. OPV 3 doses, 1 IPV, 3 Pentavalent and 2 measles  Ref: Ghai 9/e pg 87
Catch up vaccination with BCG is not recommended beyond 1 year age as per the national immunisation schedule.
87. b. Measles, DPT, OPV, Hib, Hep B  Ref: Nelson's 20/e p 1546-1548, Ghai 8/e p 195, IAP 115
All the above vaccines should be given for catch up immunization now.
88. d. Rabies vaccine  Ref: IAP 334-335, Ghai 8/e p 202
Because rabies is a lethal disease there are no contraindications for post-exposure prophylaxis including infants, pregnant & lactating
women.
89. b. Measles, BCG, booster dose of DPT and OPV  Ref: Nelson's 20/e p 1546-1548, Ghai 8/e p 195, IAP 115
For catch up immunisation, vaccines to be given on the first visit: 1 dose of all the vaccines that the child should have received
till now, but did not receive, are to be given including Measles, OPV & DPT.
BCG may be given with other live vaccines on the same day.
90. d. Chronic asthmatic child receiving low dose inhaled steroid for 10 months  Ref: Ghai 8/e p 142
Low dose steroids are not a contraindication to live vaccines.
91. c. OPV + BCG + DPT + Measles  Ref: IAP 115, 383, Ghai 8/e p 145
Recommendations for Catch up immunisation
•• Vaccination catch up regimens should preferably be individualized
•• Any number of vaccines live/inactivated may be given on the same day either singly or as combination vaccines maintaining a gap of 5 cm
between different vaccines
•• Inactivated vaccines can be given at any time in relation to any other live/ inactivated vaccines
•• If not given on the same day, a gap of 4 weeks should be maintained between two live injectable vaccines

92. c. Hepatitis B, e. Rabies  Ref: Nelson's 20/e p 1246, Ghai 8/e p 175
If the mother is known to be HBsAg negative, hepatitis B vaccine can be given in the 0–6 weeks–6 months schedule; Rabies
vaccine can also be given during pregnancy.
Chapter 10
Pediatric Basic &
Advanced Life Support (PALS)
Based on 2015 American Heart Association Guidelines Update for Cardiopulmonary Resus­
Question 1 M
citation and Emergency Cardiovascular Care
What is being done in this infant?
BLS (Basic life support) is presented as a series of sequential events with the assumption that there
is only one responder, but PALS usually takes place in an environment where many rescuers are
rapidly mobilized & multiple actions are performed simultaneously.
•• Infant BLS guidelines apply to infants < 1 year of age.
•• Child BLS guidelines apply to children from 1 year of age until puberty.
•• Adult BLS guidelines apply at and beyond puberty.

L at e s t U p d at e s
a. Conventional CPR Changes in pediatric BLS (2015)
b. Compression only CPR •• Reaffirming the C-A-B sequence as the preferred sequence for pediatric CPR
c. Airway is being secured •• New algorithms for 1-rescuer and multiple-rescuer pediatric CPR
d. Cervical spine is being stabilized •• Establishing an upper limit of 6 cm for chest compression depth in an adolescent
•• Mirroring the adult BLS recommended chest compression rate of 100 to 120/min
•• Strongly reaffirming that both compressions & ventilation are needed for pediatric BLS

Question 2 M
 IMPORTANT POINTS ABOUT PEDIATRIC RESUSCITATION
What is being shown in the picture
below? •• Chest compressions should be immediately started by one rescuer
•• Second rescuer should prepare to start ventilations with a bag and mask.
•• Other rescuers should obtain a monitor/defibrillator, establish vascular access, and calculate
and prepare the anticipated medications
•• In infants & children, initiate CPR with chest compressions rather than rescue breaths, i.e.
C-A-B (compressions, airway, breathing) rather than A-B-C. M
•• CPR should begin with 30 compressions (by a single rescuer) or 15 compressions (for
resuscitation by 2 health care professionals) rather than with 2 ventilations
•• Compression-to-ventilation ratio recommended in newborns is 3:1, while in an infant,
with two rescuers is 15:2
a. W-C clamp technique
•• It is reasonable to resuscitate newborns with a primary cardiac etiology of arrest,
b. E-C clamp technique
c. B-C clamp technique according to infant guidelines, with emphasis on chest compressions
d. W-E clamp technique
 CHEST COMPRESSIONS
•• Use the adult BLS recommended chest compression rate of 100–120/min for infants &
L at e s t U p d at e s children
•• Chest compressions should depress at least 1/3 of anterior-posterior diameter of the chest
Compression only CPR
i.e. approx 1.5 inches (4 cm) in infants to 2 inches (5 cm) in children.
•• It is an alternative for lay rescuer CPR
in adults. •• For adolescents (after puberty), the recommended depth is same as the recommended adult
•• Conventional CPR should be provided compression depth of at least 5 cm, but no more than 6 cm.
for pediatric cardiac arrests.
•• Ventilation is necessary in CPR Components of High-Quality CPR
in children because MC causes of
•• Ensuring chest compressions of adequate rate
arrest in children are asphyxial.
•• Ensuring chest compressions of adequate depth
•• If rescuers are unwilling/unable to
deliver breaths, compression-only •• Allowing full chest recoil between compressions
CPR for infants & children may be •• Minimizing interruptions in chest compressions
done. •• Avoiding excessive ventilation
Chapter 10: Pediatric Basic & Advanced Life Support (PALS) 211

BLS healthcare provide pediatric cardiac arrest Algorithm for 2 or more rescuers—2015 update.

Section 1: General Pediatrics


 VASCULAR ACCESS M

•• Intraosseous (IO) access is a rapid, safe, effective, and acceptable route for vascular access
in children and it is useful as the initial vascular access in cases of cardiac arrest Question 3 M
•• IO cannulation is recommended for patients for whom IV access proves difficult or unattainable,
What is being shown in the picture
even in older children.
below?
•• Vascular access (IO or IV) is the preferred method for drug delivery during CPR, but if it is not
possible, some lipid soluble drugs, can be administered intratracheally.

High Yield Points   M


If venous access is not available within 1 min in a child with cardiopulmonary arrest, an IO needle
should be placed in the anterior proximal tibia (avoid traversing epiphyseal plate).

Predictors of poor prognosis for infants and children with cardiac arrest
Out of hospital (OHCA) In hospital cardiac arrest (IHCA)
a. Central venous line
Age less than 1 year Age greater than 1 year b. Central arterial line
Longer durations of cardiac arrest Longer durations of cardiac arrest c. Peripheral venous access
Presentation with a non-shockable rhythm d. Intraosseous access
212 Section 1: General Pediatrics

 ENDOTRACHEAL INTUBATION
Review of Pediatrics and Neonatology

Mnemonic M

Drugs that can administered via an Size of Endotracheal Tube (ET) M

endotracheal tube are “LEANS”:


If an uncuffed endotracheal tube is used for emergency intubation:
Lidocaine, Epinephrine, Atropine,
Naloxone, Surfactant •• For infants up to one year of age → 3.5-mm ID tube
•• For patients between 1-2 years age → 4.0-mm ID tube.
•• After 2 years age, Uncuffed ET size (mm) = 4+(age/4)
Size of cuffed ET is taken to be 0.5 mm less than that of uncuffed ET

Mnemonic M High Yield Points  


If an intubated patient's condition dete- •• Exhaled CO2 detection (capnography) is recommended as confirmation of tracheal tube position for
riorates, consider “DOPE”: neonates, infants, and children with a perfusing cardiac rhythm in all settings
•• Displacement of the tube •• A color change or the presence of a capnography waveform confirms tube position in the airway but does
•• Obstruction of the tube not rule out right mainstem bronchus intubation.
•• Pneumothorax •• During cardiac arrest, if exhaled CO2 is not detected, confirm tube position with direct laryngoscopy
•• Equipment failure because the absence of CO2 may reflect very low pulmonary blood flow rather than tube
misplacement.

L at e s t U p d at e s
PALS 2015 Recommendations
Points Latest recommendations
Fluid resuscitation In specific settings, when treating pediatric patients with febrile illnesses,
the use of restrictive volumes of isotonic crystalloid leads to improved
survival.

Atropine •• Routine use of atropine is no longer recommended before intubation of


critically ill infants & children.
•• Can be used in specific emergency intubations when there is higher risk of
bradycardia.
•• A dose of 0.02 mg/kg of atropine with no minimum dose may be used

Monitoring •• ETCO2 monitoring may be considered to evaluate the quality of chest


compressions.
•• If invasive hemodynamic monitoring is in place, rescuers may use blood
pressure to guide CPR quality.

Defibrillation Use an initial dose of 2 to 4 J/kg. For refractory VF, increase the dose to
4 J/kg. Dose may be increased to maximum up to10 J/kg or the adult
maximum dose

For Shock-Refractory Either amiodarone or lidocaine may be used (Earlier the recommendation was
High Yield Points   M VF and pVT to use Amiodarone)
Hypotension is defined as a systolic
blood pressure:
•• <60 mm Hg in term neonates  POSTCARDIAC ARREST CARE
•• <70 mm Hg in infants
•• <70 mm Hg + (2 × age in years) in Temperature
children 1 to 10 years
•• <90 mm Hg in children >10 years •• For infants/children remaining comatose after cardiac arrest, it is reasonable either to maintain
of age 5 days of continuous normothermia (36˚C to 37.5˚C) or to maintain 2 days of initial
hypothermia (32˚C to 34˚C) followed by 3 days of normothermia.
•• Fever (temperature 38˚C or higher) should be aggressively treated after ROSC (Return of
spontaneous circulation)

Normoxemia
•• Oxyhemoglobin saturation should be targeted to less than 100%, but 94% or greater.
•• The goal of such an approach is to achieve normoxemia & PaO2 be kept between 60–100 mm Hg.
•• Also severe hypercapnia or hypocapnia should also be avoided.
Chapter 10: Pediatric Basic & Advanced Life Support (PALS) 213

Postcardiac arrest fluids and inotropes

Section 1: General Pediatrics


High Yield Points  
Parenteral fluids and/or inotropes or vasoactive drugs be used to maintain a systolic blood pressure
Alert, Verbal, Pain, Unresponsive
>5th percentile for age.
(AVPU) Pediatric Response Scale is
used to determine both a child’s level
Use of EEG for Prognosis of consciousness and cerebral cortex
EEGs performed within the first 7 days after pediatric cardiac arrest may be considered in function. Unlike the GCS (see later),
prognosticating neurologic outcome at the time of hospital discharge. the AVPU scale is not developmentally
dependent
Pediatric Cardiac Arrest Algorithm—2015 Update

Mnemonic M

Reversible causes of pediatric cardiac


arrest include: 6Hs & 4Ts
•• Hypovolemia
•• Hypoxia
•• Hydrogen ion (acidosis)
•• Hypoglycemia
•• Hypo-/hyperkalemia
•• Hypothermia
•• Tension pneumothorax
•• Tamponade, cardiac
•• Toxins
•• Thrombosis, pulmonary
•• Thrombosis, coronary

Question 4 M

Identify this device used in resus-


citation:

a. Pulse oximeter
b. NIBP monitor
c. Defibrillator
d. ETCO2 monitor
214 Section 1: General Pediatrics

 DEFIBRILLATION
Review of Pediatrics and Neonatology

Question 5
What is the indication for the follo­ •• An appropriate paddle or self-adhesive pad size is “Adult” size (8–10 cm) for children
wing maneuvers being done in >10 kg and “Infant” size for infants <10 kg
this infant? •• Place manual paddles over the right side of upper chest and apex of heart (to the left of
nipple over left lower ribs), so the heart is between the two paddles.
•• Shock given of initial dose of 2 to 4 J/kg.

 AIRWAY OBSTRUCTION

Epidemiology
•• Children <5 years old are particularly susceptible to foreign-body aspiration and choking.
•• Liquids are the MC cause of choking in infants, whereas small objects and food are MC in
toddlers & older children.

Diagnosis
Any child in the proper setting with sudden onset of choking, stridor, or wheezing has foreign
body aspiration until proven otherwise.

Treatment
a. Cardiac arrest
b. Foreign body aspiration •• Airway obstruction is treated with a sequential approach, starting with the head-tilt/chin-lift
c. Ventricular tachycardia
maneuver to open and support the airway, followed by inspection for a foreign body, and
d. Hypothermia
finger-sweep clearance or suctioning if one is visualized
•• In an infant <1 years old, a combination of 5 back blows and 5 chest thrusts is adminis­
tered.
•• After each cycle of back blows & chest thrusts, child's mouth should be visually inspected for
presence of foreign body.
Question 6 M
•• If identified within finger's reach, it should be removed with a gentle finger sweep
What is being shown in picture •• For a conscious child >1 years old, providers should give a series of 5 abdominal thrusts
below?
(Heimlich maneuver) with the child standing or sitting.

 ARRHYTHMIAS IN CHILDREN
ECG Findings of Important Tachyarrhythmias
Heart rate P wave QRS duration Regularity

Sinus < 230/min Always present, Normal Rate varies with


tachycardia normal axis respiration

Atrial 180–322/min Present but abnor- Normal/↑ed Usually regular but


tachycardia mal morphology (with aberration) ventricular response may
a. Head tilt-jaw lift and axis be variable because of
b. Chest thrust Wenckebach conduction
c. Heimlich maneuver
d. Parachute reflex Atrial 120–180/min Fibrillatory waves Normal/↑ed Irregularly irregular
fibrillation (with aberration) (no 2 R-R intervals alike)

Atrial flutter Atrial: 250–400/min Saw-toothed Normal/↑ed Regular ventricular


Ventricular flutter waves (with aberration) response (e.g., 2:1,
100–320/min 3:1, 3:2, and so on)

Junctional 120–280/min Atrioventricular Normal/↑ed Regular (except with


tachycardia dissociation (with aberration) capture beats)

Ventricular 120–300/min AV dissociation with Prolonged for Regular (except with


tachycardia capture beats & age capture beats)
fusion beats
Chapter 10: Pediatric Basic & Advanced Life Support (PALS) 215

 SUPRAVENTRICULAR TACHYCARDIA

Section 1: General Pediatrics


High Yield Points   M
Classification •• Supraventricular tachycardia is the
Three major categories: most frequent form of symp­tomatic
tachyarrhythmia in children.
•• Re-entrant tachycardias using an •• Most common mechanism of SVT
accessory pathway, is re-entry
•• Re-entrant tachycardias without an
accessory pathway, and
•• Ectopic or automatic tachycardias (least
common).

Clinical Features
Palpitation is the main symptom in older children & adolescents.

Treatment
•• For stable SVT, Vagal stimulation by placing of the face in ice water (in older children) may
abort the attack High Yield Points  
•• To terminate the attack, older children may be taught vagal maneuvers such as the Valsalva Sinus Arrhythmias
maneuver, straining, breath holding •• Sinus bradycardia: A sinus rate
•• Drug of choice for acute termination of SVT in children is adenosineQ < 90/min in neonates & < 60/min in
•• Adenosine is usually given via a IV line in the right upper arm (as it is closer to heart) and as older children's
a rapid push, followed by a flush (as it is short acting) •• Sinus tachycardia is defined as a
•• Verapamil is the second drug of choice, except in infants, where it is contraindicated. heart rate upper limit of normal for
the patient’s age
 ATRIAL FLUTTER
•• Mechanism: Reentrant rhythm ori­ginating
in right atrium circling tricuspid valve
annulus
•• Common in children with enlarged right
atrium (Conge­nital MS, Ebsteins anomaly).
•• DC cardioversion is done after anticoagu-
lation if thrombus is present.

 ATRIAL FIBRILLATION (AF)


•• AF is less common in children; seen in
structural heart disease (e.g. Mitral stenosis
ASD, DCM) & thyro­toxicosis
•• Most serious consequence is thrombus
formation espe­ cially in left atrial
appendage and consequent emboli­zation.

 VENTRICULAR TACHYCARDIA (VT) High Yield Points  


•• 3 consecutive ventricular premature beats; Long QT Syndromes (LQTS)
usual rate 160–240 beats/min •• Long QT syndromes are genetic
•• Usual mechanism in re-entry but abnormalities of ventricular repo-
abnormally triggered rhythms occur larization
•• T during cardiac arrest—drug of choice •• Associated with sudden death.
is Amio­darone. •• QTc (Heart rate—corrected QT
interval) of > 0.47 seconds is highly
indicative, where as QT interval of
> 0.44 sec is suggestive of LQTS
•• Treatment is beta blocking agents
or pacemaker therapy
216 Section 1: General Pediatrics

 VENTRICULAR FIBRILLATION (VF)


Review of Pediatrics and Neonatology

High Yield Points  


Defibrillation is the definitive treat-
•• VF is defined as turbulent, dis­
ment for VF with an overall survival rate organized, electrical acti­ vity of
of 17% to 20%. the heart pro­ducing ECG deflections
that continuously change in shape,
magnitude and direction
Question 7 M •• It is the most immediately dan­
Identify the arrhythmia in the given gerous arrhythmia and produces
ECG: cardiac arrest
•• For VF, external cardiac massage with artificial ventilation and DC defibrillation
•• If defibrillation is ineffective or fibrillation recurs, amiodarone or lidocaine may be given IV
and defibrillation repeated.
a. Atrial fibrillation WPW SYNDROME
b. Atrial flutter
c. Ventricular tachycardia •• Occurs due to abnormal band of specialized conductible atrial tissue that acts as an
d. Sinus tachycardia accessory pathway bypassing the junctional tissue (Bundle of Kent).

ECG Findings
High Yield Points •• Short PR interval, slurred upstroke of QRS complex (delta wave) & wide QRS complex

•• Most commonly associated with Ebstein’s anomaly.
•• Torsade de pointes is a form of VT
in which QRS morphology twists Treatment
around the baseline and may occur •• Long term beta blockers used
spontaneously in the setting of •• Digoxin and calcium channel blockers are contraindicated.
hypokalemia or hypomagnesemia
or after any drug or condition that  BRADYARRHYTHMIAS
prolongs the QT interval
AV BLOCKS
Types
•• First degree—prolongation of PR interval beyond what is normal for age
•• Second degree—Mobitz type 1 (Wenckebach phenomenon)—AV conduction time (PR
interval) progressively lengthens, with RR interval shortening, before the blocked beat
•• Mobitz type 2—Intermittently non-conducted atrial beats not preceded by lengthening
AV conduction. It is almost always due to organic disease involving the infranodal conduction
system
•• Third degree (Complete AV block)—Transmission of atrial impulses through AV node is
completely blocked & a slow, regular ventricular rate, usually < 45 beats/min seen
•• Bifascicular block—Bifascicular block is present when two of these—Right bundle, left
anterior and left posterior fascicle—are involved
•• Trifascicular block—It is defined as right bundle branch block with alternating left hemiblock,
High Yield Points   alternating right and left bundle branch block or bifascicular block with prolonged infranodal
conduction
Congenital Complete Heart Block
•• It is most commonly due to maternal
Treatment
SLE or Sjogren syndrome
•• Maternally derived IgG antibodies •• No treatment is necessary for 1st & 2nd degree AV blocks, other than Rx of primary cause
•• Anti SSA Ro or Anti SSB La •• In third degree AV block, symptomatic children with Stokes Adams syndrome, insertion of
anti­bodies artificial pacemaker is imperative.
•• Can present in utero and lead on to
hydrops fetalis
Question 8
Identify the abnormality in the ECG strip shown:

High Yield Points  


•• Left bundle branch block—(but not
right) is associated with a higher
risk of development of overt cardiac a. Wenckebach phenomenon b. WPW syndrome
disease and cardiac mortality. c. Complete heart block d. First degree heart block
Chapter 10: Pediatric Basic & Advanced Life Support (PALS) 217

Section 1: General Pediatrics


Answer Keys for Image-Based Questions

Answers Explanations/Identifying features


1. Ans. a. Conventional CPR Conventional CPR includes chest compressions & rescue breaths
2. Ans. b. E-C clamp technique ‘E-C clamp’ technique is used to fix the mask on face while doing bag & mask ventilation. Three fingers
of one hand lift the jaw (they form the “E”) while the thumb and index finger hold the mask to the face
(making a “C”).
3. Ans. d. Intraosseous access Intraosseous (IO) needles are special rigid, large-bore needles that resemble those used for bone marrow
aspiration. It is usually put in the anterior aspect of Tibia
4. Ans. c. Defibrillator For children with pulseless VT or VF, emergency defibrillation is indicated
5. Ans. b. Foreign body Back blows and chest thrusts are being done to relieve foreign body obstruction in this infant
aspiration
6. Ans. c. Heimlich maneuver For a conscious child >1 year old, with foreign body aspiration, Heimlich maneuver should be used
7. Ans. b. Atrial flutter Saw tooth waves on ECG are suggestive of atrial flutter
8. Ans. a. Wenckebach Progressive lengthening of PR interval, followed by a blocked beat (P wave not followed by QRS) is seen
phenomenon in Wenckebach phenomenon
218 Section 1: General Pediatrics
Review of Pediatrics and Neonatology

Questions

1. In a child with cardiopulmonary arrest, intraosseous 7. A 6-months-old child having severe dehydration comes
access should be attempted, if venous access is not to the casualty with weak pulse and unrecordable B.P.
available within: (Recent Question 2018) Repeated attempt in gaining IV access has failed. The
next best step is: (AIIMS May 2001)
a. 1 minute b. 2 minutes
c. 3 minutes d. 5 minutes a. Try again
2. An infant brought to emergency, unresponsive, brady- b. Jugular Vein Catheterization
cardia, shallow breathing. Next step of management: c. Intraosseous IV fluids
 (JIPMER May 2017) d. Venesection
a. IV epinephrine b. IV atropine 8. Torsades de pointes can be seen in: (PGI Nov 2015)
c. Stabilize the airway and assist breathing a. Hyponatremia
d. Cardiopulmonary resuscitation b. Jervell and Lange-Nielsen syndrome
3. How are chest compressions given in a newborn? c. Hypomagnesemia
 (Recent Question 2017) d. Hypokalemia
a. Using palm on the lower third of sternum e. Hypocalcemia
b. Using two fingers on the middle third of sternum 9. What is the drug of choice to control supraventricular
c. Using the two thumbs on the lower third of sternum
tachycardia? (Bihar PG 2015)
d. Using three fingers on the lower third of sternum
a. Adenosine
4. According to PALS 2010, all are components of initial
b. Propranolol
impression of child EXCEPT: (JIPMER 2014)
c. Verapamil
a. Consciousness b. Color d. Digoxin
c. Airway d. Breathing
5. Intraosseous line is placed in which bone in children? 10. An 8-month-old female child presented to emergency
 (Recent Question 2014) with a heart rate of 220/minute and features of congestive
a. Femur b. Tibia heart failure. Her heart rate comes down to normal after
c. ASIS d. Iliac crest administering intravenous Adenosine. What is the most
6. In Pediatric advanced life support, intraosseous access is likely diagnosis? (UPSC-I 10)
used for drug/fluid administration for pediatric age of? a. Atrial fibrillation
 (AIIMS Nov 2002) b. Atrial flutter
a. < 1 year age b. < 5 years age c. Paroxysmal supraventricular tachycardia
c. < 6 years age d. Any age d. Ventricular tachycardia
Chapter 10: Pediatric Basic & Advanced Life Support (PALS) 219

Answers with Explanations


Answers with Explanations
1. a. 1 minute  Ref: Nelson’s 20/e p 523-528, Ghai 8/e p 715-718
If venous access is not available within 1 minute in a child with cardiopulmonary arrest, an IO needle should be placed in anterior
proximal tibia
2. d. Cardiopulmonary resuscitation
The sequence of resuscitation in infants & children is C-A-B (compressions, airway, breathing) rather than A-B-C i.e initiate CPR with
chest compressions, rather than with rescue breaths
3. c. Using the two thumbs on the lower third of sternum
The two thumb encircling technique is preferred for chest compressions in newborn
4. c. Airway  Ref: Nelson’s 20/e p 489, Ghai 8/e p 718
PALS Systematic Approach Summary
Initial Impression Your first quick (in a few seconds) “from the doorway” observation; It includes

Consciousness Level of consciousness (e.g. unresponsive, irritable, alert)

Breathing Increased work of breathing, decreased respiratory effort, or abnormal sounds heard

Color Abnormal skin color, such as cyanosis, pallor, or mottling

The purpose is to quickly identify a life-threatening problem.



5. b. Tibia  Ref: Nelson’s 20/e p 502, Ghai 8/e p 715-718; Refer Ans 1
6. d. Any age  Ref: Nelson’s 20/e p 523
7. c. Intraosseous IV fluids  Ref: Nelson’s 20/e p 523-528, Ghai 8/e p 715-718
8. b. Jervell and Lange-Nielsen syndrome, c. Hypomagnesemia, d. Hypokalemia, e. Hypocalcemia
It is a distinctive form of polymorphic ventricular tachycardia (VT) characterized by a gradual change in the amplitude and
twisting of QRS complexes around the isoelectric line.
Important Causes of Torsades De Pointes
Cogenital long QT syndrome
•• Romano-Ward syndrome
•• Jervell and Lange-Nielsen syndrome
Acquired disorders:
•• Electrolyte disorders: Hypokalemia,
Hypomagnesemia, Hypocalcemia
•• Bradyarrhythmias
•• Structural heart disease
Ischemic cardiomyopathy Drugs causing Torsades De Pointes
Idiopathic dilated cardiomyopathy •• Antiarrhythmics: Quinidine, procainamide, amiodarone
•• Neurologic e.g. Subarachnoid hemorrhage •• Psychotropic medications: TCA, Haloperidol, Risperidone
•• Drugs •• Antimicrobials: Macrolides, Cotrimoxazole, Ketoconazole, Chloroquine
•• Antihistamines: Terfenadine, Astemizole, Cisapride

9. a. Adenosine  Ref: Nelson’s 20/e p 2282-2287, Ghai 8/e p 396-400


In stable patients with Supraventricular Tachycardia, Adenosine by rapid IV push is the treatment of choice because of its rapid
onset of action and minimal effects on cardiac contractility.
10. c. Paroxysmal supraventricular tachycardia  Ref: Nelson’s 20/e p 2282-2287, Ghai 8/e p 396-400
Chapter 11
Normal Neonate and
Neonatal Resuscitation
High Yield Points  IMPORTANT TERMINOLOGIES M
  M
•• Most common cause of neonatal •• Neonatal period: From birth to under 4 weeks (<28 days) age
mortality in India are prematurity & •• Early neonate period: First week of life (<7 days)
low birth weight •• Late neonate period: 7th–28th day
•• Neonatal infections, comprising •• Live birth: A product of conception irrespective of weight or gestational age, that after separation from
pneumonia, neonatal sepsis and mother, shows any evidence of life, such as breathing, heartbeat, pulsation of umbilical cord or definite
infections of CNS is the next most movement of voluntary muscle
common cause of neonatal mortality •• Fetal death: It is a product of conception that, after separation from mother, does not show any evidence
of life
•• Still birth: A fetal death at a gestational age of 22 weeks or more or weighing > 500 gm
•• Neonatal mortality rate (NMR): Death in 1st 28 days of life per 1000 live births.
L at e s t U p d at e s
 CLASSIFICATION OF NEONATES M
Periviable birth refers to delivery
occurring from 20 0/7 to 25 6/7 weeks, A. Based on gestational age at which they are born (Irrespective of Birth Weight)
at the limit of viability (ACOG 2017)
•• Term: Any neonate born between 37 completed weeks to less than 42 weeks gestation
•• Preterm: Any neonate born before 37 weeks of gestation irrespective of birth weight
•• Post-term: A neonate born at a gestational age of 42 weeks, or beyond.
Question 1
A neonate born at 36 weeks B. Based on birth weight: (Irrespective of gestational age)
gestation has a birth weight •• Low birth Weight (LBW): Any neonate with birth weight < 2500g
of 3.4 kg. What will be this •• Very low birth Weight (VLBW): Any neonate with birth weight < 1500g
neonate referred to as, based on
•• Extremely low birth weight (ELBW): Any neonate with birth weight < 1000g.
the newborn birth weight chart
provided?
C. Based on gestational age and birth weight
•• Appropriate for gestational age (AGA) or Appropriate for date (AFD): Birth weight
between 10th to 90th centile of expected, based on the gestational age
•• Small for gestational age (SGA) or Small for date (SFD): Birth weight < 10th centile of
expected, according to gestational age
•• Large for gestational age (LGA) or Large for date (LFD): Birth weight > 90th centile of
expected, according to gestational age.

 CHARACTERISTICS OF A NORMAL TERM NEONATE AT BIRTH M

•• Heart rate is usually between 120-140/minute (normal range 110-160 beats/minute)


•• Respiratory rate: 40-60 / minute
a. Term neonate
b. Small for date •• Presence of peripheral cyanosis or acrocyanosis is normal
c. Appropriate for gestational age •• Soft ejection systolic murmur may be present (due to increased pulmonary blood flow)
d. Large for date •• Normal length of umbilical cord at term is 55 cm.

 CHARACTERISTIC OF A PRETERM NEONATE M

General Characteristics
•• Subcutaneous fat is reduced
•• There is generalized hypotonia
•• Skin is thin and pinkish
•• Abundant lanugo, but little vernix caseosa
Chapter 11: Normal Neonate and Neonatal Resuscitation 221

Head

Section 2: Neonatology
Question 2 M
•• Head appears relatively large
•• Sutures are widely separated & fontanelles are large The sole of a newborn baby as
Ears: Soft and flat with ear cartilage being deficient. shown below is suggestive of?
Breast buds: < 5 mm size.
Reproductive Organs
•• Males: Undescended testes with poorly formed, smooth scrotum
•• Females: Labia majora appears widely separated, exposing labia minora.
Foot: Absent deep skin creases on sole.
Characteristics of a preterm neonate

a. Neonatal jaundice
b. Large for date baby
c. Term baby
d. Prematurity

Question 3
Which sign is being elicited in the
image below?

EXPANDED NEW BALLARD SCORE


a. Scarf sign b. Arm recoil
High Yield Points   M c. Popliteal angle d. Elbow sign

•• Tool used to assess gestational age of newborn is Expanded new Ballard score (ENBS)
•• ENBS is used to determine gesta­tional age between 20-44 weeks

Neuromuscular Maturity
222 Section 2: Neonatology

Physical Maturity
Review of Pediatrics and Neonatology

 FEEDING OF PRETERM NEONATES


Question 6 M
Gestational age Preferred initial feeding method
The item shown below is used to
feed newborns born at what ges­ <28 wks IV fluids ± Total parenteral nutrition
tational age? 28–31 wks Orogastric or nasogastric tube feeding
32–34 wks Katori Spoon / paladai / cup feeding
>34 wks Breastfeeding

 PRIMITIVE NEONATAL REFLEXES


a. 27 weeks b. 30 weeks
c. 33 weeks d. 37 weeks Important Neonatal Reflexes Present at Birth M

Reflex Onset Fully developed Disappears


Mnemonic M
Palmar grasp 28 wk gestation 32 wk gestation 2-3 mo postnatal
•• Asymmetric tonic neck reflex (ATNR):
present AT birth Rooting 32 wk gestation 36 wk gestation Less prominent after 1 month age
•• So, symmetric tonic neck reflex Moro 28 wk gestation 37 wk gestation 5-6 mo postnatal
appears after birth
Asymmetric tonic 35 wk gestation 1 mo postnatal 6-7 mo postnatal
neck reflex (ATNR)

Question 4 Question 5
When does this reflex completely Which primitive reflex is being
appear? elicited in the given image?

a. Moro’s reflex
b. Swallowing reflex
a. 28 weeks b. 32 weeks c. Palmar grasp reflex
c. 37 weeks d. 40 weeks   d. Rooting reflex
Chapter 11: Normal Neonate and Neonatal Resuscitation 223

Neonatal Reflexes Appearing after Birth

Section 2: Neonatology
Question 7 M

Reflex Appears Disappears Which neonatal reflex is shown in


the picture below?
Parachute 7-8 month after birth Remains throughout life

Symmetric tonic neck 4-6 months after birth 8-12 months

Landau 3 months after birth 24 months

Neck righting 4 months after birth 24 months a. Moro reflex


b. Asymmetric tonic neck reflex
c. Parachute reflex
High Yield Points d. Symmetric tonic neck reflex
  M
•• Voluntary grasp appears when palmar grasp reflex disappears
•• Rooting reflex helps a mother in breastfeeding High Yield Points   M
•• Rooting reflex is the earliest primitive neonatal reflex to disappear
•• Moro’s reflex once disappears, never reappears Clavicle is the most common bone to
fracture in a neonate
•• Parachute reflex once appears, never disappears
•• When asymmetric tonic neck ref­lex disappears, baby learns to roll over (from prone to supine and vice versa)

MORO’S REFLEX M

Components of Complete Moro’s Reflex


•• SYMMETRICQ abduction and extension of upper limbs with
•• Opening of hands followed by flexion and adduction
•• Along with extension of head and trunk
•• Movement of lower limbs and crying
Sequence of appearance:
Opening of hands (28 weeks)Q

Abduction and extension (32 weeks)

Adduction and flexion (37 weeks)
Disappears at: 5-6 monthsQ (latest update from Nelson's 20th ed, 2015).
Persistence of Moro’s reflex indicates Cerebral damageQ.
Exaggerated Moro’s reflex is seen in Stage 1 Hypoxic Ischemic encephalopathy (HIE).
Absent Moro’s reflex is seen in any condition with marked hypotonia like Stage 3 HIE &
Down syndrome

AsymmetricQ Moro’s reflex is seen in: High Yield Points


Fracture clavicleQ Erb’s palsyQ •• Neonatal cardiac arrest is mostly as-
phyxial, so initiation of ventilation is
Shoulder joint dislocation Congenital hemiplegia.
the focus of initial resuscitation
•• Most important indicator of success-
ful PPV is a rising heart rate.
 NEONATAL RESUSCITATION PROTOCOL: 2015 UPDATE •• Umbilical venous catheterQ is the
preferred method of obtaining emer-
gency vascular access in deli-very
High Yield Points room
  M •• PPV can be delivered effectively with
•• 10% of newborns require some assistance to begin breathing at birth. a flow-inflating bag, self-infla­ting
•• < 1% require chest compressions and medications. bag, or T-piece resuscitator
•• Volume expansion is done when
•• 60 seconds (“the Golden Minute”)Q are allotted for completing initial steps, reevaluating and beginning
blood loss is known or suspected
ventilation if required (pale skin, poor perfusion, weak
•• VentilationQ is the most important step for successful resuscitation of a newborn who has not respond- pulse) and baby’s heart rate has not
ed to the initial steps responded adequately to other re-
•• Most sensitive indicator of a successful response to each step is an increase in heart rate.Q suscitative measures
224 Section 2: Neonatology

NEONATAL RESUSCITATION PROTOCOL: 2015 UPDATE


Review of Pediatrics and Neonatology

M
L at e s t U p d at e s
Resuscitation of Baby Born Through
Meconium Stained Liquor (MSL) M
•• Non-vigorous newborns with MSL
do not require routine intubation
and tracheal suctioning;
•• MSL requires presence of a resusci-
tation team with 1 person skilled in
endotracheal intubation
•• For babies born through MSL, rou-
tine intubation for tracheal suction
is no longer suggested because
there is insufficient evidence to con-
tinue this recommendation

High Yield Points


•• Absolute contraindication for bag
and mask ventilation is congenital
diaphragmatic hernia
•• Ratio between chest compressions &
positive pressure ventilation should
be 3:1

Question 8
Identify this instrument used in
neonatal resuscitation:

a. Endotracheal tube
b. Laryngeal mask airway
c. Self-inflating bag and mask
d. Laryngoscope

Question 9
Which statement is true about
the step of neonatal resuscitation Suction of Airways in NRP
shown in the pictures below? •• Not required routinely for all neonates
•• Order of suction should be: mouth followed by nose
•• Size of suction catheter: 12 or 14 F
•• Suction pressure recommended: 80 mm Hg or 100 cm H2O
A B Positive Pressure Ventilation in NRP
a. Method A is preferred over method B •• Done using self inflating bag & mask
b. Method B is preferred over method A •• Recommended rate: 40–60 breathes/minute
c. Both method A & method B are •• E-C clamp technique used to make tight seal between mask & face
equally efficacious •• Recommended pressure for:
d. Method B is preferred when there is –– 1st breathe --> 30-40 cm H2O
1 resuscitator only –– Subsequent breathes > 15-20 cm H2O
Chapter 11: Normal Neonate and Neonatal Resuscitation 225

Section 2: Neonatology
L at e s t U p d at e s Question 10 M

Some important changes in NRP as per latest 2015 guidelines: M Identify this instrument used in
neonatal resuscitation:
Oxygen Use:
•• For ≥ 35 weeks gestation begin resuscitation with 21% oxygen (room air)
•• For < 35 weeks gestation begin with 21%-30% oxygen
•• If a baby is breathing but oxygen saturation (SpO2) is not within target range, start free-flow oxygen at
30%, and titrate to achieve SpO2 target
•• 100% oxygen is to be used whenever chest compressions are provided a. Endotracheal tube
•• Wean the oxygen concentration as soon as heart rate recovers. b. Laryngeal mask airway
Positive pressure ventilation (PPV): M c. Self-inflating bag and mask
d. Laryngoscope
PPV may be considered if baby is breathing, heart rate > 100 beats/min but spO2 cannot be maintained
within target range despite free flow oxygen or CPAP.
Endotracheal Intubation and Laryngeal Masks:
•• Intubation is strongly recommended prior to beginning chest compressions
•• If intubation is not successful or not feasible, a laryngeal mask may be used
•• Newborns > 2 kg and > 34 weeks gestation require a size 3.5mm endotracheal tube
•• Size 4.0 endotracheal tube is no longer used in NRP. Question 11 M

Chest compressions: M What is this instrument reco­


•• Should be administered with 2-thumb technique mmended to be used in neonatal
•• Electronic cardiac monitor (3-lead ECG) is preferred for assessing heart rate during chest compressions resuscitation?
and/or PPV
•• Chest compressions continued for 60 seconds prior to checking a heart rate.
Hypovolemia
•• For treating hypovolemia, 0.9% NaCl (normal saline) or O Rh-negative blood is to be used
•• Ringer’s Lactate solution is no longer recommended for treating hypovolemia.
Indications for withholding Resuscitation: M a. Cardiac monitor b. Pulse oximeter
•• Birth at a confirmed gestational age < 22 weeks c. Bilirubinometer d. Flux meter
•• Congenital malformations (e.g. anencephaly)
•• Chromosomal anomalies (e.g. trisomy 13)
•• For gestation between 22 - 24 weeks decision to be made after discussion with caregivers.

Ways to Assess Correct Placement of Endotracheal Tube M

•• Exhaled CO2 detection (recommended method)Q


•• Bilateral chest movement
•• Presence of equal breath sounds bilaterally
•• Condensation in endotracheal tube.

Use of Epinephrine in Neonatal Resuscitation M

Indication If heart rate remains < 60 beats/min after at least 30 seconds of effective PPV &
another 60 seconds of chest compressions coordinated with PPV using 100% oxygen
Precaution Not indicated before effective ventilation is established
Dose Intravenous: 0.01 to 0.03 mg/kg or 0.1-0.3 ml/kg of 1:10,000 epinephrine
Endotracheal dose: 0.05 to 0.1 mg/kg
Route Intravenous (preferred)Q; endotracheal administration may be done
Concentration 1:10,000Q (0.1 mg/mL).for both IV & intratracheal use High Yield Points   M
•• Umbilical cord normally falls off in
Care Required at Birth (for All Babies) M 7-10 days
•• Regurgitation of feeds is normal
•• Clear the mouth of secretions if required, by wiping with a soft cloth, or gentle suction with as long as baby gains weight and
a bulb syringe or soft catheter, if excessive passes urine 6-8 times a day
•• Weight of baby should be recorded •• Meconium is greenish stool passed
in 1st few days after birth
•• Physical examination to screen for congenital abnormalities
•• White patchy lesions on oral
•• Baby should be wiped dry and wrapped in clean sheets mucosa that are difficult to wipe off
•• Intramuscular injection of 0.5-1 mg of water-soluble vitamin K1 (phytonadione) should be and leave hemorrhagic points, when
given to all infants shortly after birth to prevent hemorrhagic disease of newborn. removed suggest Candidiasis
226 Section 2: Neonatology

Care after Birth and before Discharge from Hospital


Review of Pediatrics and Neonatology

•• Promote exclusive breastfeeding


•• Maintain axillary temperature of baby between 36.5°C and 37.5°C
•• Healthy infants should be placed supine to reduce risk of sudden infant death syndrome (SIDS)
•• Immunization before discharge: BCG, OPV zero dose, Hep B
•• Screening (if available) for diseases like hypothyroidism, cystic fibrosis
•• Screening with pulse oximetry for ductal dependent cyanotic congenital heart disease
•• Screening for hyperbilirubinemia
•• Vit D supplementationQ (400 IU/day) to begin and continue till 1 year age.

L at e s t U p d at e s
Question 12 M Delayed Cord clamping M
Identify the skin lesion shown in •• Should be delayed for at least 30 seconds for vigorous term and preterm newborns
the picture below: •• If placental circulation is not intact e.g. abruptio placenta, cord should be clamped immediately after
birth
•• Delayed cord clamping is associated with less intraventricular hemorr­hage, higher blood pressure
and blood volume, less need for trans­fusion after birth and less necrotizing enterocolitis
•• The only adverse consequence is increased level of bilirubin, associated with more need for phototherapy

IMPORTANT CONDITIONS SEEN IN NEWBORNS THAT DO NOT REQUIRE


a. Erythema toxicum neonatorum ANY SPECIFIC TREATMENT M
b. Mongolian spots Conditions Involving Skin / Mucosa
c. Stork bite
d. Milia •• Erythema toxicum neonatorum: Erythematous papular lesions on trunk and face, commonly
seen on day 2-3 of life. They are eosinophil-filled sterile lesions
•• Mongolian spots: Bluish black pigmented areas mainly on buttocks and lower back
•• Stork bite: Also called Salmon patches or nevus simplex. These are capillary
hemangioma, commonly located at nape of neck, upper eyelids, forehead, and root of nose.
•• Milia: Colourless papules due to plugging of sweat ducts, mainly on scalp
•• Epstein pearls: Epithelial inclusion cysts, mainly on hard palate
•• Acne neonatorum: Occur due to transplacental passage of maternal androgens to the fetus.

Milia Stork bite Erythema toxicum

Other Conditions not Requiring any Specific Treatment


Question 13 M
•• Subconjunctival hemorrhage.
Identify the lesions seen on oral •• Bilateral breast engorgement:
mucosa: –– Also called ‘Mastitis neonatorum’
–– Seen in both male as well as female neonates on 3rd or 4th day of life
–– Due to lack of inactivation of maternal progesterone and estrogens after birth due to
immaturity of neonatal liver
•• Vaginal bleeding
–– Seen only in female neonates, usually after 3-5 days of birth
–– Occurs due to fall in the level of sex hormones after birth when baby is disconnected
a. Koplik spots from the placenta
b. Candidiasis
•• Physiological weight loss
c. Epstein pearls
d. Herpes simplex –– Term babies lose up to 10% of their birth weight in the first week of life which is regained
by day 10 of life
–– Preterm babies lose up to 15% of their birth weight, regained by day 15 of life
•• Hymenal tags: In female neonates
•• Physiological phimosis: In male neonates.
Chapter 11: Normal Neonate and Neonatal Resuscitation 227

Salient differences between Caput Succedaneum & Cephalohematoma

Section 2: Neonatology
M

Characteristic Caput succedaneum Cephalohematoma


Incidence Common Less common
What is it? Edema in the layers of scalp It is subperiosteal hemorrhage, so,
located between skull and its periosteum
Characteristic May cross suture line and midline Does not cross suture lines or midline
Schematic diagram

Mnemonic
Physiological weight loss
•• Term babies lose
•• Ten percent of birth weight,
regained by day Ten of life

Time of presentation Maximum size present at birth Increasing size for 12-24 hours after birth
Disappears in 48-72 hours Takes 4-6 wks
Jaundice Not associated Seen (due to breakdown of Hb in bleed).

Caput succedaneum Cephalhematoma

INTRAUTERINE GROWTH RESTRICTION (IUGR)


High Yield Points   M
Definition: IUGR refers to all babies with clinical features of malnutrition, like: ≥3 Loose
skin folds in buttock region, decreased subcutaneous fat, & peeling of skin •• IUGR is Intrauterine growth
‘restriction’ and not ‘retardation’
•• IUGR is a clinical definition, irrespective of birth weight, while Q
•• Ponderal index (PI) is birth weight
•• SGA is a statistical definition (babies with birth weight < 10th centile of expected). (gm)/length (cm)3 X 100
2 main types of IUGR: •• All SGA babies are also IUGR but all
IUGR babies are not SGA
Symmetric IUGR Asymmetric IUGR
Timing of insult Insult in first and early second Problem in late 2nd or 3rd trimester
trimester
Etiology •• TORCH infection Medical / obstetric problem of mother in
•• Chromosomal anomaly late trimesters like poor maternal nutrition, Question 14
•• Genetic causes maternal hypertension, placental insufficiency In which newborns are the features
Effect on cells Fetal cell number mainly Size of cells more affected than number of shown below seen?
decreased cells
Anthropometric Head circumference, length and Brain growth (head circumference) spared
parameters weight equally affected
Ponderal index (PI) = or > 2 <2
Brain/ Liver size ratio <5 Brain/ Liver size ratio >5
Prognosis Poorer Better outcome a. LFD b. IUGR
c. Preterm d. AFD
High Yield Points
Morphological IUGR refers to neonates with birth weight between 10th–25th centile with features of fetal
undernutrition
228 Section 2: Neonatology
Review of Pediatrics and Neonatology

Answer Keys for Image-Based Questions

Answers Explanations / Identifying features


1. Ans. d. Large for date Birth weight of 3.4 kg at 36 weeks gestation lies above the 95th centile line on this graph; So it is a Large
for date (LFD) neonate
2. Ans. d. Prematurity The sole of this neonate shows absent deep creases, which is an indicator of prematurity
3. Ans. a. Scarf sign It is one of the parameters used to assess gestational age of a neonate, using expanded new ballard score
4. Ans. c. 37 weeks Moro's reflex completely appears by 37 weeks gestation
5. Ans. d. Rooting reflex As the angle of mouth is stimulated, the baby turns his face to that side & opens his mouth
6. Ans. c. 33 weeks The given picture is that of a paladai, that can be used for feeding neonates born between 32-34 weeks
gestation; Initial mode of feeding of a preterm neonate depends on gestational age at which baby is
born
7. Ans. b. Asymmetric tonic neck The baby’s neck is turned to right side, with extended (ipsilateral) right upper & lower limbs & flexed
reflex (contralateral) left upper & lower limb; This is Asymmetric Tonic Neck Reflex (ATNR)
8. Ans. b. Laryngeal mask airway A tube with a cuffed mask at one end, known as laryngeal mask airway
9. Ans. a. Method A is preferred 2 thumb encircling technique is preferred over 2 finger technique
over method B
10. Ans. c. Self-inflating bag and A self-inflating bag with a mask & Oxygen tubing & reservoir attached to it
mask
11. Ans. b. Pulse oximeter A monitor showing waveforms along with numerical display of spO2 & pulse rate, is a pulse oximeter
12. Ans. b. Mongolian spots Bluish-black areas of pigmentation in lower back & buttocks are known as ‘Mongolian spots’
13. Ans. c. Epstein pearls Pearly white papules on hard palate, known as Epstein pearls
14. Ans. b. IUGR The picture shows multiple loose skin folds in the buttock region, suggestive of decreased
subcutaneous fat, seen in IUGR
Chapter 11: Normal Neonate and Neonatal Resuscitation 229

Questions
Questions

 IMPORTANT TERMINOLOGIES a. Galeazzi sign b. Ortolani test


c. Telescopic sign d. Barlow test
1. Ponderal Index is: M (AIIMS Nov 2016) 13. Gynecomastia in neonate is seen due to:
a. Wt/Ht3 × 100 b. Wt/Ht2 × 100  (FMGE Nov 2017)
c. Ht/Wt × 100
2 d. Ht/Wt3 × 100 a. Mother's estrogen b. Mother's progesterone
2. Very low birth weight babies are: M  c. GnRH d. Gonadotropins
a. < 2500 gm of birth weight (Recent Question 2016) 14. True about Cephalhematoma is: (PGI Jan 2017)
b. <1500 gm of birth weight a. Limited by sutures
c. <1000 gm of birth weight. b. Caused due to periosteal injury
d. Between 2500–3000 gm of birth weight c. Due to fluid accumulation below periosteum
3. Neonatal period extends up to: M  d. Regresses in 24 hours
 (Recent Question 2015) e. It is due to birth trauma
a. 21 days of life b. 30 days of life 15. True about term baby is: (PGI Jan 2017)
c. 28 days of life d. 35 days of life a. Elbow crosses xiphisternum on flexion over chest
4. In large-for-date babies all viscera weigh more than b. Weight > 3.8 kg is normal
normal except: M  (MAHA PGM CET 2014)
c. Parachute reflex is not present after the age of 9 months
d. Jaundice is seen within first 24 hours of life
a. Kidney b. Heart
e. Baby can have physiological weight loss up to 10% of
c. Brain d. Liver
birth weight
5. Macrosomia is: M  (Recent Question 2013) 16. Which of the following is true about Cephalhematoma?
a. Large size baby b. Big mouth a. Its size is maximum at birth (Recent Question 2017)
c. Large head d. Large tongue b. It resolves in 7-10 days
6. Definition of low birth weight: M  (JIPMER 2013) c. It is limited by periosteum
a. Wt < 2.5 kg b. Wt < 2 kg d. It is due to edema in the layers of scalp
c. Wt < 1.5 kg d. Wt < 1 kg 17. A hymenal tag in a newborn is best treated by: M
7. “Microsomia” is defined as: (DNB June 2012)  (Recent Question 2016, AIIMS May 2008)
a. Birth weight below 90th percentile a. Steroids b. Surgery
b. Birth weight below 10th percentile c. Leaving it alone d. None of the above
c. Birth weight below 20th percentile
d. Birth weight below 5th percentile 18. Newborn babies are able to breathe and suck at the
8. Macrosomia is seen in all except maternal: same time due to: M  (AIIMS May 2013, Nov 2010)
 (NEET Pattern 2012) a. Wide short tongue b. Short soft palate
c. High larynx d. Short pharynx
a. Diabetes b. Multiparous
c. Hypertension d. Obesity 19. The following skin changes are normal in a newborn
except: (TN PGMEE 2013)
9. Definition of extremely low birth weight:
a. Infantile acropustulosis b. Harlequin color change
a. Wt <2.5 kg b. Wt <2 kg (JIPMER 2009)
c. Mongolian spots d. Pustular melanosis
c. Wt <1.5 kg d. Wt <1 kg
20. About Caput Succedaneum all are true except: M 
10. Small for date baby is: M  (PGI June 2009)  (Recent Question 2012)
a. <10th percentile for the gestational age a. Soft b. Prolonged jaundice
b. <50th percentile for gestational age c. Ill defined d. None are true
c. < 2 kg 21. Early neonatal period is upto how many days of life? M 
d. <2.5 kg
a. 1 day b. 7 days (DNB Dec 2012)
c. 28 days d. 14 days
 NORMAL TERM NEONATE 22. Normal temperature of a newborn: M  (DNB Dec 2012)
a. 34.5–35.5oC b. 35.5–36.5oC
11. The pupil of a newborn is: (FMGE Nov 2017) c. 36.5–37.5oC d. 37.5–38.5oC
a. Dilated b. Mid-dilated 23. Normal HR in a newborn: M  (DNB June 2010)
c. Constricted d. Normal a. 80–100/min b. 100–120/min
12. An orthopedician adducts the flexed hip and gently c. 120–160/min d. 160–200/min
pushes the thigh posteriorly in an effort to dislocate 24. Neonatal MCV is: (PGI Nov 2009)
the femoral head of a neonate. What is this maneuver
a. 90–110 fL b. 110–125 fL
called? (JIPMER 2017) c. 125–135 fL d. 140-150 fL
230 Section 2: Neonatology

25. Which is an abnormal finding in a neonate?


Review of Pediatrics and Neonatology

36. Preterm VLBW infants ingesting unfortified breast milk


 (DNB June 2009) need to be supplemented with what doses of calcium,
a. Glycosuria b. Bacteriuria and vitamin D? (Recent Question 2017)
c. WBCs in urine d. Hyperbilirubinemia a. Calcium (80–120 mg/kg/day) and vitamin D
26. Most common newborn rash which presents at 24–48 (200–400 IU/day)
hours of life is: (DNB Dec 2009) b. Calcium (40–80 mg/kg/day) and vitamin D
a. Erythematous papular pustular lesions (400–1000 IU/day)
b. Milia c. Calcium (500–1000 mg/kg/day) and vitamin D
c. Transient neonatal pustular melanosis (200–400 IU/day)
d. Hemangioma d. Calcium (150–220 mg/kg/day) and vitamin D
27. In a newborn, what is the normal respiratory rate? M  (200–400 IU/day)
 (DNB June 2008) 37. As per the perinatal risk stratification of preterm neo­
a. 10–20 breaths/minutes b. 30–40 breaths/minutes nates and developmental outcomes, which of the
c. 40–60 breaths/minutes d. 60–80 breaths/minutes follow­ing will constitute a moderate risk category?
 (Recent Question 2017)
28. Common sites for Mongolian spot are: (PGI Dec 2008)
a. Gestation 30–32 weeks, birth weight 1000–1500 gm,
a. Face b. Neck
1-3 days of hypoglycemia
c. Lumbosacral area d. Leg
b. Gestation < 28 weeks, birth weight 1251-1500 gm,
e. Thigh
1–4 days of hypoglycemia
29. Skin changes in newborn that disappear spontaneously: c. Gestation < 28 weeks, birth weight <1250 gm, 1-4 days
 (PGI Dec 2008) of hypoglycemia
a. Harlequins skin change b. Mongolian spots d. Gestation 30–32 weeks, birth weight 1251–1500 gm,
c. Erythema toxicum d. Lymphomia more than 5 days of hypoglycemia
e. Milia 38. Which one of the following is a strongly recommended
30. Color of transition stools: M  (TN PGMEE 2006) intervention to reduce mortality & prevent future neuro­
a. Green b. Brown developmental impairment in preterm neonates: 
c. Yellow d. Pale a. IV IG administration (Recent Question 2017)
31. The appropriate approach to a neonate presenting with b. Use of Insulin for hyperglycemia
vaginal bleeding on day 4 of life is: M  (AIPGMEE 2005) c. Avoiding aminoglycoside exposure for > 7 days
d. Use of antenatal steroid
a. Administration of Vitamin K
39. What is the recommended dose of steroids for attaining
b. Investigation for bleeding disorder
fetal lung maturity? M  (AIIMS Nov 2016)
c. No specific therapy
d. Administration of 10 mL/kg of fresh frozen plasma over a. Inj Betamethasone 12 mg for 2 doses 12 hours apart
4 hours b. Inj Betamethasone 12 mg for 2 doses 24 hours apart
c. Inj Dexamethasone 6 mg for 4 doses 24 hours apart
32. Vomiting on the first day of baby’s life may be caused by
d. Inj Dexamethasone 12 mg for 2 doses 12 hours apart
all of the following except:  (MAHA PGM CET 2005)
40. All of the following are features of prematurity in a
a. Pyloric stenosis b. Esophageal atresia neonate, except: M 
c. Aerophagy d. Amniotic gastritis  (Recent Question 2016, AIPGMEE 2006)
33. A full term baby, exclusively breast fed at the end of a. No creases on sole b. Abundant lanugo
1 week was passing golden yellow stools and was c. Thick ear cartilage d. Empty scrotum
found to have adequate hydration with normal systemic 41. Intracranial bleeds in premature infants is seen in the:
examination. The weight of the baby was just same as it  (COMEDK 2016)
was at birth. The pediatrician should now advice:
a. Germinal matrix b. Corpus callosum
a. Give oral solution with breastfeeding (AIIMS May 2002) c. Basal ganglia d. Choroid plexus
b. Start top feeding 42. Anticipated preterm delivery. Dose of dexamethasone
c. Investigate for lactic acidosis given to mother is: M  (AIIMS May 2015)
d. Reassure the mother that nothing is abnormal a. 12 mg 12 hrly 2 doses  b. 12 mg 24 hrly 4 doses 
34. Normal finding in term neonate: (PGI Dec 2002) c. 6 mg 24 hrly 2 doses d. 6 mg 12 hrly 4 doses
a. Erythema toxicum b. Epstein's pearl
c. Bilateral cryptorchidism 43. Key features of kangaroo mother care are all the
d. Subconjunctival hemorrhages following except: M  (APPG 2014)
e. Erythema nodosum a. Skin to skin contact between mother and baby
b. Exclusive breastfeeding
c. Initiated in a facility and continued at home
 PRETERM NEONATE
d. Done for babies with cyanosis
44. Preterm babies have increased chance of: M 
35. Which of the following is a component of kangaroo
mother care? M  (Recent Question 2017) a. Heart disease  (PGI Nov 2014)
b. Respiratory distress syndrome
a. Incubator b. Heat therapy c. Necrotizing enterocolitis
c. Exclusive breastfeeding d. Meconium aspiration syndrome
d. Phototherapy e. Intraventricular hemorrhage
Chapter 11: Normal Neonate and Neonatal Resuscitation 231

45. A 32-week, 1400 g neonate is born to a primigravida. 56. Retinopathy of prematurity is caused by: (AIIMS June 2000)

Questions
The baby did not require resuscitation and showed a. Less gestation age b. Low birth weight
stable vitals. The baby was transferred to the NICU. How c. O2 toxicity d. Carbohydrate excess
will you manage the feeding of the patient?
 (AIIMS May 2014)  PRIMITIVE NEONATAL REFLEXES
a. Start total enteral feeding and IV nutrition not required
b. Start IV nutrition with minimal enteral feeding 57. Asymmetric tonic neck reflex (ATNR) disappears at: M
c. Start IV nutrition and introduce feeds on 2nd day of life  (Recent Question 2017)
d. Start parenteral nutrition and institute oral feeding on 2nd a. 5–6 months b. 7–8 months
day of life c. 9 months d. 10 months
46. Which of the following is not a component of kangaroo
mother care (KMC)? M  (AIIMS May 2014) 58. Asymmetric Moro’s reflex at birth is indicative of:
 (AIIMS May 2014)
a. Skin to skin contact
a. HIE b. Brain damage
b. Supplementary nutrition
c. Erb's palsy d. Kernicterus
c. Exclusive breastfeeding
59. Grasp reflex is fully developed at: (JIPMER 2011)
d. Early discharge and follow-up
a. 24 weeks b. 28 weeks
47. How much time later should ROP screening be done for
c. 32 weeks d. 36 weeks
a 2-week-old baby being treated in NICU for sepsis, who
60. Grasp reflex begins at: (JIPMER 2008)
was born at 28 wk gestation? (AIIMS May 2014)
a. 24 weeks b. 28 weeks
a. 2 weeks b. 4 weeks
c. 32 weeks d. 36 weeks
c. 6 weeks d. 8 weeks
61. Reflex which is not present in a child at birth is?
48. Blood volume in preterm neonate is:  (AIIMS May 2007)
 (Recent Question 2013) a. Moro's reflex b. Symmetric tonic neck reflex
a. 90 mL/kg b. 80 mL/kg c. Crossed extensor reflex d. Asymmetric tonic neck reflex
c. 70 mL/kg d. 60 mL/kg 62. Moro’s reflex disappears at: (AIPGMEE 2007)
49. Preterm neonate target oxygen saturation is: a. 5 months b. 3 months
 (Recent Question 2013) c. 7 months d. 6 months
a. >95% b. 91–95%
c. 85–90% d. <80%  NEONATAL RESUSCITATION
50. A pediatrician calls an ophthalmologist to examine baby if:
63. Which of the following is false regarding neonatal
a. Baby born at 28 weeks gestation (AIIMS Nov 2013)
resuscitation? (JIPMER 2017)
b. Baby has respiratory distress
c. Birth weight of the baby is 2.3 kg a. T-tube delivers free flow oxygen
d. Jaundice b. Self-inflating bag cannot be used without a reservoir
c. Self-inflating bag can deliver free flow oxygen
51. Preterm babies have: (NEET Pattern 2012)
d. Flow inflating bag delivers free flow oxygen
a. Extramedullary hematopoiesis 64. Correct order of suctioning during neonatal resuscitation
b. Greater risk of hypothermia is: (AIIMS May 2018)
c. Greater risk of hypoglycemia a. Trachea→nose→mouth b. Nose→mouth
d. All of above c. Mouth→nose→trachea d. Mouth→nose
52. In a stable 32 weeks neonate, the preferred mode of 65. Resuscitation of a neonate with HR < 60/min includes all
feeding is: (WBPG 2012) of the following except: (NEET pattern Jan 2018)
a. Direct breastfeeding b. Nasogastric feed a. Chest compression b. Inj Adrenaline
c. Katori spoon feed d. Parenteral feed c. Endotracheal intubation d. None of the above
53. A fetus born at 36 weeks, AGA, breast fed, should be 66. A newborn presents in emergency in a state of shock.
adequately monitored for all of the following except: Which is the ideal fluid to be given to the newborn?
 (TN PGMEE 2011)  (Recent Question 2018)
a. Hyperbilirubinemia b. Hypoglycemia a. Normal saline b. Dextrose 5%
c. Hypocalcemia d. Bacterial sepsis c. Ringer lactate d. Dextrose 25%
54. A neonate weighing 1500 gm is delivered at 33 wks. 67. Definite contraindication for bag and mask ventilation
Which of the following would be most appropriate is: M  (JIPMER Nov 2016)
method of nutrition for the baby? (AIPGMEE 2011) a. Cleft lip b. Cleft palate
c. Congenital diaphragmatic hernia
a. Breastfeeding b. Orogastric feeding
d. Umbilical hernia
c. Total parental nutrition d. Katori spoon feeding
68. Delayed clamping of Umbilical Cord at birth is
55. A woman delivered a baby of 2.2 kg weight. Her LMP is
associated with all of the following except:
not known. To know the maturity of baby, which of the
a. Improved Iron status (Recent Question 2017)
following are used: (PGI June 03)
b. Improved clinical anemia at 2–6 month of age
a. Sole crease b. Ear cartilage c. Increase in symptomatic polycythemia among infants
c. Breast nodule d. Anterior fontanel d. Increased chances of neonatal jaundice requiring
e. Weight of the baby phototherapy
232 Section 2: Neonatology

77. Most confirmatory sign of endotracheal intubation is?


Review of Pediatrics and Neonatology

69. Which of the following is the best route of drug adminis­


tration in a newborn baby during neonatal resuscitation?  (MAHA PGM CET 2014)
 (AIIMS Nov 2016) a. Chest rise b. Auscultation
a. Umbilical artery b. Umbilical vein c. Spirometry d. Capnography
c. Intraosseous d. Intratracheal 78. The initiation of the first breath in a newborn is due to:
70. Special newborn care unit should comprise of:  (MAHA PGM CET 2014)
 (WB PGMEE 2016) a. ↑pH; ↑PaO2; ↓PaCO2 b. ↓pH; ↓PaO2; ↑PaCO2
a. 4 bed, 2 rooming in b. 6 bed, 4 rooming in c. ↓pH; ↓PaO2, ↓PaCO2 d. ↓pH; ↑PaO2, ↓PaCO2
c. 12 beds, 4 rooming in d. 2 beds, 6 rooming in 79. A 32 week newborn baby with RR-86/min, grunting present
71. In Neonatal Resuscitation, chest compression to venti­ with no nasal flaring, abdomen behind in movement than
lation ratio is: M  (MAHA PGM CET 2016) chest, minimum interocostals retraction and no xiphisternal
a. 2:1 b. 3:1 retraction. What is the Silverman scoring?
c. 4:1 d. 5:1 a. 1 b. 4 (AIIMS Nov 2010)
72. To provide immediate care to all newborns at birth, a c. 3 d. 6
health facility must have within its delivery room: 80. Severe bradycardia in a newborn is HR: (TN PGMEE 2009
a. Newborn care corner  (MAHA PGM CET 2016) a. <40/min b. <60/min
b. Newborn stabilization unit c. <80/min d. <100/min
c. Special newborn care unit 81. A baby is born with meconium stain liquor. All of the
d. Home based newborn care unit following are taken into account for terming a baby
73. Dose of IV adrenaline in term infant during neonatal Vigorous, except: (AIIMS Nov 2009)
resuscitation: (Recent Question 2016)
a. Tone b. Color
a. 0.1–0.3mL/kg of 1:1000 c. HR d. Respiration
b. 0.3–0.5 mL/kg of 1:1000
82. A non-ventilated preterm baby in incubator is under
c. 0.1–0.3 mL/kg of 1:10,000
d. 0.3–0.5mL/kg of 1:10,000 observation. Which is the best way to monitor the baby’s
breathing and detect apnea? (AIPGMEE 2008)
74. Identify the device shown below:
 (Recent Question 2016) a. Infrared throracic movement study
b. Capnography
c. Nasal digital temperature monitoring
d. Impedance technique
83. Components of neonatal resuscitation: (PGI Dec 2007)
a. Maintenance of temperature
b. Maintenance of respiration
a. Venturi mask b. Ambu bag c. Maintenance of circulation
c. Nasal canula d. Laryngeal mask airway d. Chest compression e. Feeding the baby
75. Normal Apgar Score with Acidosis in a neonate is seen 84. Bag and mask ventilation in newborn resuscitation is
in? (MAHA PGM CET 2015) contraindicated in: (DNB Dec 2007)
a. Spinal cord trauma b. Choanal atresia a. Diaphragmatic hernia
c. Diaphragmatic hernia b. Pulmonary hypoplasia
d. High Fetal Catecholamine levels c. Tracheo-oesophageal fistuala
d. Laryngomalacia
76. In newborn resuscitation, ratio of cardiac compressions 85. A newborn was given a drug in neonatal ICU, but then was
to breaths is: M  (TN PGMEE 2015) found in respiratory distress. The likely drug responsible is:
a. 15:1 b. 30:2 a. Morphine b. Naloxone (AIPGMEE 07)
c. 15:2 d. 3:1 c. Salbutamol d. Soda-bicarb
Chapter 11: Normal Neonate and Neonatal Resuscitation 233

Answers with Explanations


Answers with Explanations

 IMPORTANT TERMINOLOGIES

1. a. Wt/Ht3 × 100  Ref: Clinical Obstetrics by Reece 3/e p 507


Ponderal index (PI) is birth weight (gm)/length (cm)3 × 100; Its value is ≥ 2 in symmetric IUGR & < 2 in asymmetric IUGR
2. b. <1500 gm of birth weight  Ref: Nelson’s 20/e p 789-791, Ghai 8/e p 124-125
Classification of neonates based on birth weight: (irrespective of gestational age).

Category Low birth wt (LBW) Very LBW Extremely LBW


Birth wt <2500 g <1500 g <1000 g

3. c. 28 days of life  Ref: Nelson’s 20/e p 789-791, Ghai 8/e p 124-125


Neonatal period extends from birth to under 4 weeks (<28 days) of age, of which early neonatal period refers to 1st week of life
(<7 days)
4. c. Brain  Ref: Nelson’s 20/e p 789-791, Ghai 8/e p 124-125
Infant of Diabetic mother is an important cause of LFD baby, where all organs increase in size except the brain
5. a. Large size baby  Ref: Nelson’s 20/e p 789-791, Ghai 8/e p 124-125
‘Macro' means large & ‘somia' refers to body; Macrosomia is another name for a large of date baby.
6. a. Wt < 2.5 kg  Ref: Nelson’s 20/e p 789-791, Ghai 8/e p 124-125
7. b. Birth weight below 10th percentile  Ref: Nelson’s 20/e p 789-791, Ghai 8/e p 124-125
Microsomia is another name for a small for date (SFD) or small for gestational age (SGA) baby.
SFD baby is a baby whose birth weight is less than 10th centile of expected according to the gestational age.
8. c. Hypertension  Ref: Nelson’s 20/e p 789-791, Ghai 8/e p 124-125
Maternal hypertension leads to a small for date baby & not large for date.
9. d. Wt <1 kg  Ref: Nelson’s 20/e p 789-791, Ghai 8/e p 124-125
10. a. <10th percentile for the gestational age  Ref: Nelson’s 20/e p 789-791, Ghai 8/e p 124-125

 NORMAL TERM NEONATE


11. c. Constricted  Ref: Nelson's 20/e p 3016
•• Pupils of a newborn are small & difficult to dilate, due to immature iris dilator muscle.
•• Lens of a newborn infant is more spherical than that of an adult.
•• Fundus of a newborn's eye is less pigmented than that of an adult.
12. d. Barlow test  Ref: Nelson's 20/e p 3275
Barlow test:
•• It is a provocative maneuver assesses the potential for dislocation of a non-displaced hip.
•• The examiner adducts the flexed hip and gently pushes the thigh posteriorly in an effort to dislocate the femoral head.
•• In a positive test, the hip is felt to slide out of the acetabulum.
•• As the examiner relaxes the proximal push, the hip can be felt to slip back into the acetabulum.
Ortolani test (reverse of Barlow test):
•• Examiner attempts to reduce a dislocated hip
•• Examiner grasps the child's thigh between the thumb and index finger and, with the 4th and 5th.
13. a. Mother's estrogen  Ref: Nelson 20/e p 2742
•• Transient gynecomastia occurs in 60–90% of male newborns secondary to exposure to estrogens during pregnancy.
•• Breast development may be asymmetrical and galactorrhea is seen in 5%.
•• Most cases resolve within 4–8 wk of birth, but a few can last as long as 12 mo.
14. a. Limited by sutures; b. Caused due to periosteal injury; e. It is due to birth trauma  Ref: Nelson’s 20/e p 834
Cephalohematoma is a subperiosteal hemorrhage, hence always limited to the surface of 1 cranial bone. It may take
2 wk to 3 months to disappear;
234 Section 2: Neonatology

15. e. Baby can have physiological weight loss upto 10% of birth
Review of Pediatrics and Neonatology

weight  Ref: Nelson’s 20/e p 824


About a False: In preterm baby elbow crosses xiphisternum (scarf sign)
on flexion over chest, as shown in Ballard score below;
About b False: In a term neonate, birth weight > 3.8 kg would be > 90th
percentile, making it a large for date baby;
About c False: Parachute reflex appears at around 9 months gestation &
persists throughout life;
About d False: Physiological jaundice never appears in 1st 24 hours of life;
About e True: In term babies, there can be physiological weight loss upto
10% of birth weight, which is regained by day 10 of life
16. c. It is limited by periosteum  Ref: Nelson’s 20/e p 834
17. c. Leaving it alone  Ref: Nelson’s 20/e p 63, Ghai 8/e p 134-136
Hymenal tags around margins of hymen is a normal finding.
18. c. High larynx  Ref: Nelson’s 20/e p, Ghai 8/e p 134-136
Differences between the airway of a young infant, child and adult include:

•• Oral cavity is small in a newborn & totally filled by tongue due to slightly retracted lower jaw
•• The soft palate & epiglottis are in contact at rest, providing an additional valve at the back of the oral cavity
•• Infant larynx has a short funnel shaped lumen, that is disproportionately narrower than that of adult
•• The larynx and hyoid cartilage are higher in the neck and closer to the base of the epiglottis
•• This high position enables an infant to use its nasal airway to breathe while sucking

19. a. Infantile acropustulosis  Ref: Nelson’s 20/e p 794-795, Ghai 8/e p 134-136

a. Infantile acropustulosis It is a recurrent, self-limited, pruritic, vesico-pustular eruption of palms & soles occurring in young children
during first 2-3 years of life; Topical steroids & oral dapsone may be used to treat it.
b. Harlequin color change It is a cutaneous condition seen in 10% of newborns, characterized by momentary red & pale colors of the
two halves of body, sharply demarcated at the body’s midline
c. Mongolian spots Bluish black well-demarcated areas of pigmentation, usually seen over the buttocks & back
d. Pustular melanosis A benign lesion seen predominantly in black neonates, contains neutrophils & is present at birth as a
vesiculopustular eruption around chin, neck, back, extremities, and palms or soles; it lasts 2-3 days

20. b. Prolonged jaundice  Ref: Nelson’s 20/e p 1928-1937, 834, Ghai 8/e p 138-141
Cephalhematoma predisposes to Neonatal Jaundice & not Caput succedaneum.
21. b. 7 days  Ref: Nelson’s 20/e p 1928-1937, Ghai 8/e p 134-136
22. c. 36.5–37.5°C  Ref: Nelson’s 20/e p 832, Ghai 8/e p 134-136
Axillary temperature less than 36.5ºC is called hypothermia in newborns.
23. c. 120–160/min  Ref: Nelson’s 20/e p 63, Ghai 8/e p 134-136
24. a. 90–110 fL  Ref: Nelson’s 20/e p 2309-2312, Ghai 8/e p 330
Normal Red Cell Indices in Infancy and Childhood

Age MCV (fL) (mean) MCH (pg) (mean) MCHC (g/dL) (mean)
Birth (cord blood) 108 34 33
1 month 104 34 33
1–2 years 78 27 33
2–6 years 81 27 34
6 – 12 years 77 29 34

25. b. Bacteriuria  Ref: Ghai 8/e p 134-136


Urine is normally a sterile body fluid, so bacteriuria is an abnormal finding.
26. a. Erythematous papular pustular lesions  Ref: Nelson’s 20/e p 63, Ghai 8/e p 134-136
In many neonates, small, white papules on an erythematous base develop 1-3 days after birth. This benign rash, erythema toxicum,
persists for as long as 1 wk, contains eosinophils, and is usually distributed on the face, trunk, and extremities.
Chapter 11: Normal Neonate and Neonatal Resuscitation 235

27. c. 40–60 breaths/minutes  Ref: Nelson’s 20/e p 794, Ghai 8/e p 134-136

Answers with Explanations


28. c. Lumbosacral area  Ref: Nelson’s 20/e p 63, Ghai 8/e p 679
29. a. Harlequins skin change, b. Mongolian spots, c. Erythema toxicum and e. Milia  Ref: Nelson’s 20/e p 63
30. a. Green  Ref: Nelson’s 20/e p 63, Ghai 8/e p 134-136
Transitional stools are dark greenish yellow and loose, sometimes ‘seedy' in texture.
31. c. No specific therapy  Ref: Nelson’s 20/e p 63, Ghai 8/e p 134-136
Vaginal bleeding is seen only in female neonates, usually seen after 3-5 days of birth; Occurs due to fall in the level of sex hormones
after birth when baby is disconnected from the placenta.
32. a. Pyloric stenosis  Ref: Nelson’s 20/e p 63, Ghai 8/e p 134-136
Congenital hypertrophic pyloric stenosis usually presents with non-bilious projectile vomiting between 6-10 weeks age.
33. d. Reassure the mother that nothing is abnormal  Ref: Nelson’s 20/e p 63
Term babies have physiological weight loss (upto 10% of their birth weight), that requires no treatment.
34. a. Erythema toxicum, b. Epstein’s pearl, and d. Subconjunctival hemorrhages  Ref: Nelson’s 20/e p 835
•• Subconjunctival & retinal hemorrhages are frequently seen in a neonate just after birth
•• Probable cause: sudden increase in intrathoracic pressure during passage of the chest through the birth canal
•• Three lesions resolve rapidly within the 1st 2 wk of life.

 PRETERM NEONATE
35. c. Exclusive breastfeeding  Ref: AIIMS NICU Protocol 2014
36. d. Calcium (150–220 mg/kg/day) and vitamin D (200–400 IU/day)  Ref: AAP recommendations, 2013
As per the latest AAP Recommendations for Enteral Nutrition for VLBW Infants, a daily Calcium intake of 150–220 mg/kg, Phosphate
75–140 mg/kg & Vitamin D 200–400 IU should be ensured
37. a. Gestation 30–32 weeks, birth weight 1000–1500 gm, 1–3 days of hypoglycemia  Ref: NNF Practice Guidelines 2011
NNF Recommendations for risk stratification of preterm neonates for follow-up:

High Risk Moderate Risk Mild Risk


•• Birth wt < 1 kg &/or gest <28 week •• Birth weight 1000-1500 g •• Birth wt 1500 g - 2500 g
•• Chronic lung disease / IVH / PVL •• Gestational age < 33 weeks •• HIE grade I
•• Severe HIE/ Seizures/ Meningitis •• Twins/triplets •• Transient hypoglycemia
•• Surgical conditions, e.g. CDH, TEF •• Moderate HIE •• Suspected sepsis
•• Birth wt <3rd or >97th centile •• Hypoglycemia •• Neonatal jaundice needing
•• Mech ventilation for > 24 hrs •• Neonatal sepsis phototherapy
•• Persistent prolonged hypoglycemia •• Hyperbilirubinemia > 20 mg/dL or need of •• IVH grade 1
•• Shock requiring inotropes exchange transfusion
•• Infants of HIV-positive mothers •• IVH grade 2
•• Bilirubin encephalopathy •• Suboptimal home environment

38. d. Use of antenatal steroid  Ref: Nelson’s 20/e p 852


Antenatal steroids reduce overall mortality, need for and duration of ventilator support and admission to a neonatal ICU, the incidence
of severe IVH, necrotizing enterocolitis, and neurodevelopmental impairment in the neonate.
39. b. Inj Betamethasone 12 mg for 2 doses 24 hours apart  Ref: Nelson’s 20/e p 852
Inj Betamethasone 12 mg IM 2 doses, 24 hrs apart or Inj Dexamethasone 6 mg IM every 12 hrs for 4 doses can be used
40. c. Thick ear cartilage  Ref: Nelson’s 20/e p 789-791, Ghai 8/e p 138-140; Refer pretext for details
The ears in a premature neonate are soft and flat with ear cartilage being deficient and pliant (and not thick)
41. a. Germinal matrix  Ref: Nelson’s 20/e p 835
Intraventricular hemorrhage (IVH)

•• Mainly seen in premature infants, Most commonly in the gelatinous sub-ependymal germinal matrix
•• Immature blood vessels in this highly vascular region with poor tissue support predispose premature infants to hemorrhage
•• Germinal matrix involutes in full-term neonates & vascular integrity improves; so, it is less common in term babies

42. d. 6 mg 12 hrly 4 doses  Ref: Nelson’s 20/e p 852, Ghai 8/e p 170; Refer Ans 39 above
43. d. Done for babies with cyanosis  Ref: AIIMS NICU protocol, Ghai 8/e p p 148; Refer pretext
Kangaroo mother care (KMC) is done for:
•• All stable LBWQ babies are eligible for KMC; it can be started immediately after birth for birth wt > 1800 g
•• Sick & very small babies (<1200 gm), KMC started after baby is hemodynamically stable
236 Section 2: Neonatology

44. a. Heart diseases, b. Respiratory distress syndrome, c. Necrotizing enterocolitis and e. Intraventricular hemorrhage 
Review of Pediatrics and Neonatology

Ref: Nelson’s 20/e p 789-791, Ghai 8/e p 138-140; Refer pretext of this chapter for details
45. a. Start total enteral feeding and IV feeding not required  Ref: Nelson’s 20/e p 789-791, Ghai 8/e p 158-160
46. b. Supplementary nutrition  Ref: Nelson’s 20/e p 789-791, Ghai 8/e158-160; Refer Ans 44 above
47. a. 2 weeks  Ref: Nelson’s 20/e p 3050-3051, Ghai 8/e p 666
Timing of the initial screening exam for ROP is based on the infant's age.
According to AIIMS NICU protocol, 2014
Hardly any ROP is detected before 32 weeks of PMA. So, First screening examination should be carried out at 32 weeks of
post menstrual age (PMA) or 4 weeks of postnatal age, whichever is later
In the given scenario, baby is born at 28 week gestation, so 1st screening examination should be done at 4 weeks postnatal age or
32 weeks post conceptional age; As the baby is already 2 weeks old screening for ROP should be done 2 weeks later.
48. a. 90 mL/kg  Ref: Maternal, Fetal, & Neonatal Physiology by Susan Tucker Blackburn, p 249
Blood volume is 80-100 mL/kg for term neonates & 90-105 mL/kg in preterm neonates (higher plasma volume)
49. b. 91-95%  Ref: Nelson’s 20/e p 854
In preterm neonates, the currently recommended range of oxygen saturation target is 91–95%.
50. a. Baby born at 28 weeks gestation  Ref: Nelson’s 20/e p 3050, Ghai 8/e p 666
Indications of Screening for Retinopathy of prematurity (ROP):
•• Birth weight <1,500 g or gestational age < 32 wk &
•• Selected neonates with a birth weight between 1,500 and 2,000 g or gestational age of >32 wk with an unstable clinical course,
including those requiring cardiorespiratory support.
51. d. All of above  Ref: Nelson’s 20/e p 2309, Ghai 8/e p 138-140
In a premature infant, hematopoiesis is frequently seen in the liver & occasionally in the spleen, lymph nodes or thymus.
52. c. Katori spoon feed  Ref: Ghai 8/e p 158-160, AIIMS NICU Protocol
53. d. Bacterial sepsis  Ref: Nelson’s 20/e p 920-922, Ghai 8/e p 163-165
Sepsis screen is done only when neonatal sepsis is suspected. Pre-emptive monitoring for sepsis is usually not done.
54. d. Katori spoon feeding  Ref: Ghai 8/e p 158-160
55. a. Sole crease, b. Ear cartilage, and c. Breast nodule  Ref: Nelson’s 20/e p 63, Ghai 8/e p 138-140
56. a. Less gestational age > c. O2 toxicity  Ref: Nelson’s 20/e p 3050, Ghai 8/e p 666
Retinopathy of prematurity (ROP) is characterized by abnormal proliferation of small retinal blood vessels in preterm neonate
Risk factors for ROP are Hyperoxia (O2 toxicity), Shock, sepsis, hypoxia, acidosis, blood transfusion, PDA & mechanical ventilation

 PRIMITIVE NEONATAL REFLEXES


57. a. 5–6 months  Ref: Nelson’s 20/e p 2798, Ghai 8/e p 142
Asymmetric tonic neck reflex (ATNR) disappears at 5–6 months postnatal age & only then the baby learns to roll over;
58. c. Erb’s palsy  Ref: Nelson’s 20/e p 2798, Ghai 8/e p 142
59. c. 32 weeks  Ref: Nelson’s 20/e p 2798, Ghai 8/e p 142; Refer pretext of this chapter;
60. b. 28 weeks  Ref: Nelson’s 20/e p 2798, Ghai 8/e p 142; Refer pretext of this chapter;
61. b. Symmetric tonic neck reflex  Ref: Nelson’s 20/e p 2798, Ghai 8/e p 142
Important Neonatal reflexes appearing after birth are parachute reflex & symmetric Tonic Neck reflex.
62. d. 6 months  Ref: Nelson’s 20/e p 2798, Ghai 8/e p 142
This is a controversial question; Previously, Moro's reflex was believed to disappear at 3 months
But as per latest Nelson's 20/e, Moro’s reflex disappears at 5-6 months of life. Hence, 6 months is the best answer to this
question, currently.

 NEONATAL RESUSCITATION
63. b. Self–inflating bag cannot be used without a reservoir  Ref: Ghai 8/e p 126-133
Self-inflating bag can be used with/without a reservoir.
64. d. Mouth → nose  Ref: Ghai 9/e p 129
If suction of mouth & oropharynx is not done prior to doing suction of nose, oropharyngeal secretions may get aspirated, when nasal
suction of a neonate is done.
65. d. None of the above  Ref: Ghai 8/e p 126-133
Chest compression, Endotracheal intubation as well as Inj Adrenaline may be required in a neonate with HR <60/minute.
Chapter 11: Normal Neonate and Neonatal Resuscitation 237

66. a. Normal saline  Ref: Nelson's 20/e p 846

Answers with Explanations


For a newborn in shock, supportive treatment with type O Rh-negative blood or normal saline is indicated for hemorrhage or
hypovolemia, respectively.
67. c. Congenital diaphragmatic hernia  Ref: AAP guidelines 2015, Ghai 8/e p 126-133
Bag & mask ventilation should be avoided in Congenital diaphragmatic hernia, because it enlarges the stomach and small bowel and
thus makes oxygenation more difficult.
68. c. Increase in symptomatic polycythemia among infants  Ref: Nelson’s 20/e p 881; NRP 2015 guidelines
Delayed Cord Clamping
•• It is associated with less intraventricular hemorrhage, higher blood pressure & blood volume, lesser anemia in infants
upto 2-6 months age, lesser need for blood transfusion after birth, and less necrotizing enterocolitis; There is no evidence of
decreased mortality.
•• Late clamping may result in delivery of an extra 20-40 mL of blood and 30-35 mg of iron to the newborn.
•• Polycythemia is a risk with delayed clamping but is often asymptomatic.
•• The only negative consequence appears to be a slightly increased level of bilirubin, associated with more need for
phototherapy.

69. b. Umbilical vein  Ref: NRP 2015 guidelines by AAP


•• Umbilical venous catheter is the preferred method of obtaining emergency vascular access in delivery room;
•• For treating hypovolemia, 0.9% NaCl (normal saline) or O Rh-negative blood is to be used
70. c. 12 beds 4 rooming in  Ref: India Newborn Action plan, 2014
Newborn care facilities at various levels of public health services:

Newborn Care Corner Newborn Stabilization Unit Special Newborn Care Unit
Characteristics (NBCC) (NBSU) (SNCU)
Location All points of childbirth CHC / FRU District/sub-district hospitals
including PHC & subcentre
Function To provide immediate To stabilize sick newborns be- Care for sick newborns < 1800
care at birth, feeding fore referral to higher centres; & gm (all types of care except
support, prompt referral for management of babies > 1800 assisted ventilation and major
gm, jaundice, sepsis surgeries)
Minimum bed 1 bed 4 beds + 2 beds for rooming in 12 beds + 4 beds for rooming in
strength

71. b. 3:1  Ref: AAP guidelines 2015, Ghai 8/e p 126-133


Chest compression: ventilation ratio used in Neonatal resuscitation is 3:1, unless the arrest is known to be of cardiac etiology (rare
in neonates & infants). Then higher ratio should be considered (15:2).
72. a. Newborn care corner  Ref: AAP guidelines 2015, Ghai 8/e p 126-133
73. c. 0.1–0.3 mL/kg in 1:10,000  Ref: AAP guidelines 2015, Ghai 8/e p 126-133
Epinephrine is preferably given by intravenous route at a dose of 0.01 to 0.03 mg/kg or 0.1-0.3 mL/kg of 1:10,000 concentration
Endotracheal administration may be done if venous access not available, at a dose of 0.05 to 0.1 mg/kg
74. b. Ambu bag  Ref: AAP guidelines 2015, Ghai 8/e p 126-133
This is an AMBU bag, that is a self-inflating bag used for ventilating neonates & children.
75. d. High Fetal Catecholamine levels  Ref: Nelson’s 20/e p 798
Important conditions in newborns with

No acidosis but low apgar score Acidosis present with normal apgar score
•• Prematurity; Acute cerebral trauma •• Maternal acidosisQ
•• Analgesics, narcotics, sedatives, Magnesium sulfate •• High fetal catecholamine levelsQ
•• Congenital myopathy or neuropathy or CNS anomaly
•• Lung anomaly (CDH) or Airway obstruction (choanal atresia)

76. d. 3:1  Ref: AAP guidelines 2015, Ghai 8/e p 126-133


77. d. Capnography  Ref: AAP guidelines 2015, Ghai 8/e p 126-133
•• Exhaled CO2 detection or Capnography is the recommended or confirmatory method to assess position of Endotracheal
tube
•• Other methods are: bilateral chest movements, presence of equal breath sounds bilaterally, condensation in endotracheal tube.
238 Section 2: Neonatology

78. b. ↓ pH; ↓ PaO2; ↑ PaCO2  Ref: AAP guidelines 2015, Ghai 8/e p 126-133
Review of Pediatrics and Neonatology

Hypoxia, hypercapnia & acidosis  ⇒  Respiratory center in the brain stimulated  ⇒ Breathing
79. b. 4  Ref: AAP guidelines 2015
Silverman-Anderson Score
•• It is used to assess the degree of respiratory distress in preterms newborns with Respiratory Distress Syndrome.
•• The higher the score, the greater the respiratory distress. Components of Silverman score:

Components 0 1 2 Score in given Q


Upper chest retraction Chest & abdomen rise Lag of upper chest as abdo- Chest & abdomen move as 1
together men rises a “see-saw”
Lower chest retraction Absent Minimal Marked 1
Xiphoid retraction Absent Minimal Marked 0
Nasal flaring Absent Minimal Marked 0
Expiratory Grunt Absent Heard with stethoscope Loudly audible grunt 2
So, total score in given question is 4
80. b. <60/min  Ref: AAP guidelines 2015, Ghai 8/e p 126-133
Heart rate < 60/min is an indication for chest compressions along with positive pressure ventilation in a newborn.
81. b. Color  Ref: AAP guidelines 2015, Ghai 8/e p 126-133; Refer pretext for details;
A ‘vigorous' neonate has Strong respiratory effortsQ, Good muscle toneQ & Heart rate > 100/minuteQ
82. d. Impedance technique  Ref: AAP guidelines 2015
Respiratory monitoring is based on impedance technique, that measures changes in electrical resistance during breathing;
The electrode is fixed on the chest wall to pick up signals which are displayed as respiratory rate.
83. a. Maintenance of temperature, b. Maintenance of respiration, c. Maintenance of circulation, and d. Chest compression
Ref: AAP guidelines 2015, Ghai 8/e p 126-133

High Yield Points


•• The most common indication for neonatal resuscitation is asphyxia followed by prematurity
•• The main components of neonatal resuscitation are maintenance of temperature, respiration & circulation
•• Chest compressions are required if heart rate is < 60/minute.Q

84. a. Diaphragmatic hernia  Ref: AAP guidelines 2015, Ghai 8/e p 126-133
85. a. Morphine  Ref: AAP guidelines 2015
Causes of respiratory depression in neonate

•• Intrapartum asphyxia (most common causeQ •• Prematurity: CNS immaturity, surfactant deficiency
•• Drugs: Morphine anesthetics •• Respiratory: Diaphragmatic hernia, obstructive lesions
•• Sepsis •• CNS abnormalities: Malformation, trauma, myopathy
Chapter 12
Diseases of Newborn
IMPORTANT DISEASES OF NEWBORNS
Diseases more common in: M

Preterm Babies IUGR/SGA babies


•• Neonatal sepsis •• Polycythemia
•• Respiratory: •• Persistent pulmonary hypertension of newborn (PPHN)
–– Hyaline membrane disease •• Meconium aspiration syndrome
–– Apnea of prematurity
–– Pulmonary hemorrhage
–– Broncho pulmonary dysplasia
•• CVS: Patent ductus arteriosus Both preterm and IUGR/SGA
•• CNS: Intraventricular hemorrhage •• Hypothermia
•• Retinopathy of prematurity (ROP) •• Hypoglycemia
•• Gastrointestinal: •• Hypocalcemia
–– Hyperbilirubinemia •• Birth asphyxia or perinatal asphyxia
–– Feeding problems
•• Anemia of prematurity

HEMORRHAGIC DISEASE OF NEWBORN (HDN)


•• Hemorrhagic disease of newborn (HDN) is due to deficiency of vitamin K
•• HDN is also known as ‘vitamin K deficiency bleeding’
•• Both prothrombin time (PT) & activated partial thromboplastin time (APTT) are prolonged
in HDN
•• To prevent HDN, Vitamin K should be given intramuscularly at birth in a dose of 1 mg to all
babies > 1 kg & 0.5 mg to those <1 kg

 NEONATAL SEPSIS M

Definition: Neonatal sepsis refers to systemic bacterial infection in a newborn.


Types:
Early onset sepsis Late-onset sepsis
Salient Infections occurring before 1 week of life Infections occurring after 1 wk of life, High Yield Points   M
features Acquired before or during delivery Organisms acquired in hospital or community;
Most Common Cause of Neonatal
(vertical transmission) Meningitis commonly seenQ
Sepsis throughout the world:
Etiology Gr B Streptococcus,Q E. coli Staph aureus, Klebsiella, E.coli, Acinetobacter •• In hospitals: E. coli
Risk •• Maternal fever •• Lack of breast feedingQ •• Overall: Gr B streptococciQ
factors •• Foul smelling liquor •• LBW/prematurity •• Early onset: Gr B streptococciQ
•• Premature rupture of membrane •• Multiple needle pricks
•• Prolonged labour •• Superficial infections (e.g. Pyoderma, umbilical
•• Multiple PV examinations cord stump infection)
•• Low birth weight / Prematurity

L at e s t U p d at e s
Clinical Features
•• Most common organism responsi-
•• Poor feeding (earliest clinical feature)Q ble for neonatal sepsis in India (or
•• Hypothermia, more common than fever in hospitals in India) is Acineto-
•• Respiratory: Tachypnea, respiratory distress bacter followed by Klebsiella
•• GI: Feed intolerance, abdominal distension, hypoglycemia Source: Lancet Glob Health 2016
•• Septic shock, DIC, sclerema.
240 Section 2: Neonatology

Investigations
Review of Pediatrics and Neonatology

Screening test: Sepsis screenQ


If any 2 out of following 5 present, sepsis screen is said to be positive.
•• Total leukocyte count (TLC) < 5000/mm3
•• Absolute Neutrophil count (ANC) < 1800/mm3
•• Immature to total neutrophil (IT) ratio > 0.2
•• Micro ESR > 15 mm
•• C Reactive protein (CRP) positive
Confirmatory test: Blood cultureQ
Treatment
•• Intravenous broad spectrum antibiotics (e.g. IV Ampicillin and Gentamicin)
•• Add a 3rd generation Cephalosporin (IV Cefotaxime), if meningitis present
•• Supportive care
Duration of antibiotics: Depends on
Sepsis screen Blood culture Meningitis Known as Duration
- - - Clinical sepsis 5-7 days
+ - - Screen +ve sepsis 7-10 days
+/- + - Culture +ve sepsis 14 days
+/- +/- + Meningitis 21 days

Prevention of Neonatal sepsis: Proper handwashing of caregivers


High Yield Points  
It is recommended to handwash for
2 minutes before entering a neonatal
intensive care unit

High Yield Points   M


For recording axillary temperature,
thermometer must be kept in axilla for
at least 3 minutes
 NEONATAL HYPOTHERMIA M

Definition
Question 1 M Axillary temperature of baby < 36.5ºC.
What is the mode of hea­ting in this Classification
equip­ment?
•• Cold stressQ: 36.0 - 36.4ºC
•• Moderate hypothermiaQ: 32 – 35.9ºC
•• Severe hypothermiaQ: < 32ºC.

Why newborns are prone to hypothermia?


•• Preterm and SGA infants have scanty brown fat •• Vulnerability to get exposed
•• Larger surface area •• Shivering is absentQ.

Newborns protect themselves against hypothermia by

a. Conduction b. Convection •• Non shivering thermogenesisQ in brown fat •• Universal flexion


c. Radiation d. None •• Vasoconstriction caused by sympathetic surge •• Increase in heart rate to supply more O2.
Chapter 12: Diseases of Newborn 241

Non Shivering Thermogenesis

Section 2: Neonatology
•• Brown fat is a well vascularized sympathetically innervated lipid collection
•• Areas rich in brown fat are axillae, groin, nape of neck, interscapular area
Cold stress

Release of norepinephrine

Uncoupling of beta-oxidation in fat Question 2 M

Name this device used for
Heat generation preventing hypothermia in
neonates:
Clinical Features of Neonatal Hypothermia
•• Increased metabolism → Hypoglycemia
•• Increased O2 consumption → Hypoxia
•• Anaerobic metabolism → Metabolic acidosis
•• Bradycardia → Increased mortality

High Yield Points   M


•• Temperature of non-asphy­xiated newborns is a strong pre­dictor of mortality at all gestational ages
•• Temperature of newly born non-asphyxiated infants should be maintained between 36.5°C and 37.5°C
•• Nursery temperature should be kept at approximately 22-26°C (72-78°F).Q
a. Radiant warmer
b. Incubator
Management c. Phototherapy unit
Cold stress and moderate hypothermia: d. Room heater
•• Skin to skin contact with mother
•• Supportive measures: Frequent feeding, increase caloric intake, monitor vitals.

Severe Hypothermia
•• Method of warming: Incubator or preheated radiant warmer or thermostatically controlled
heated mattress set at 37–38°C; measure temperature every hour for 3 hours High Yield Points
•• Once baby's temperature reaches 34°C, rewarming process should be slowed down
‘Thermoneutral environment’ refers to
•• Supportive measures: Oxygen, Empirical antibiotics, IV fluids.
the range of environ­ mental tempera-
Prevention ture at which the BMR (Basal Metabolic
Rate) of the baby is at a minimum, oxy-
•• Strategies in preterm infants: plastic wrap with a cap, thermal mattress, warmed humidi- gen consumption is least and the baby
fied gases, increased room temperature
manintains its normal body temperature.
•• In resource-limited settings, use of plastic wraps, skin to-skin contact (Kangaroo Mother
care) and even placing infant after drying in a clean food-grade plastic bag up to neck.

Warm Chain
A set of interlinked procedures to minimize the likelihood of hypothermia, including:
• Warm, draught free room • Early initiation of breastfeeding
•• Drying up • Postpone bathing Question 3 M
•• Cover appropriately • Bedding in
All of the following are the benefits
•• Warm resuscitation • Warm transportation (weakest link)
of this intervention EXCEPT:
•• Skin to skin contact • Training & awareness generation

KANGAROO MOTHER CARE (KMC) M

What is it? It is a method of care of preterm or low birth weight (LBW) infants by placing them in
skin to skin (STS) contact with mother or other caregiver to ensure optimum growth &
development of infant
Done forQ All stable LBWQ babies are eligible for KMC; it can be started immediately after birth
for birth wt > 1800 gm
For sick & very small babies (<1200 gm), KMC started after baby is hemodynamically stable a. Lesser mortality
KMC can be provided while the baby is being fed via orogastric tube or is on oxygen b. Lesser hypothermia
c. Lesser jaundice
Contd... d. Lesser risk of sepsis
242 Section 2: Neonatology

Contd...
Review of Pediatrics and Neonatology

ComponentsQ Kangaroo position; Kangaroo nutrition (Exclusive breastfeeding); Early discharge &
follow upQ
BenefitsQ Improved exclusive breastfeedingQ rates Reduction in hypothermiaQ
Reduction in the risk of mortalityQ Reduction in length of hospital stay
Reduction in nosocomial infection/sepsis

PERINATAL ASPHYXIA / BIRTH ASPHYXIA


High Yield Points   M
Definition
Apgar score 0-3 for more than 5
minutes is seen in severe birth Inability to initiate or sustain breathing. M

asphyxia
Pathophysiology

Hypoxia and ischemia



Anaerobic metabolism

Increased amounts of lactate generated
Excitatory & toxic amino acids, particularly glutamate, accumulate in the damaged tissue
Increased intracellular sodium & calcium result in tissue swelling & cerebral edema.
Increased production of free radicals and nitric oxide in the tissues

Injury to multiple organs esp CNS

Diagnostic Criteria for Severe Birth Asphyxia (All 4 Required)


•• Apgar score 0-3 for > 5 minutes
•• Severe acidosis (cord blood pH < 7.0)
•• Any clinical evidence of CNS dysfunction
•• Any evidence of dysfunction of at least 1 organ other than CNS

Multiorgan Systemic Effects of Asphyxia


•• CNS: HIE, infarction, intracranial hemorrhage, seizures, tone abnormalities;
•• Cardiovascular: Myocardial ischemia, poor contractility, hypotension
•• Pulmonary: Pulmonary hypertension, pulmonary hemorrhage, RDS
•• Renal: Acute tubular or cortical necrosis
•• Adrenal: Adrenal hemorrhage
•• Gastrointestinal: Perforation, ulceration with hemorrhage, necrosis
•• Metabolic: SIADH, hyponatremia, hypo-glycemia, hypocalcemia, myoglobinuria
•• Skin: Subcutaneous fat necrosis
•• Hematology: Disseminated intravascular coagulation.
Staging of HIE (Sarnat and Sarnat)

Signs Stage 1 Stage 2 Stage 3


Question 4
Level of consciousness Hyperalert Lethargic Stuporous, coma
What is this investigation used for Muscle tone Normal Hypotonic Flaccid
diagnosing and prognosticating
Posture Normal Flexion Decerebrate
newborns with severe hypoxic
ische­mic encephalopathy? Tendon reflexes/clonus Hyperactive Hyperactive Absent
Moro reflex StrongQ Weak AbsentQ
Pupils Mydriasis Miosis Mid-dilated
Seizures None Common None
Electroencephalo-graphic Normal Low voltage seizure Burst suppression to
(EEG) findings activity isoelectric
Outcome Good Variable (80% normal) Death, severe deficits 50%
a. aEEG b. bEEG
(99% normal) die; rest 50% have severe
c. cEEG d. dEEG
neurologic sequelae
Chapter 12: Diseases of Newborn 243

Section 2: Neonatology
High Yield Points   M
•• Part of brain most commonly affected in term infants with HIE is parasagittalQ area
•• Part of brain most commonly affected in preterm infants with HIE is periventricularQ area

Diagnosis
Question 5
1. CNS Imaging
–– Diffusion-weighted MRIQ is the preferred imaging modality in neonates with HIE A preterm baby suffered peri-­
–– CT scans are helpful in identifying focal hemorrhagic lesions, diffuse cortical injury and natal asphyxia and was found to
damage to basal ganglia have Periventri­cular leuco­malacia
–– Ultrasonography is initial preferred modality for preterm infant. on CNS imaging, as shown below:
2. Amplitude-integrated electroencephalography (aEEG)
–– Helps to determine which infants are at highest risk for long-term brain injury
–– It is simple to perform and correlates with standard electroencephalogram (EEG)
–– It has good reliability and high negative predictive value
–– It provides information quickly within the window during which intervention is most
likely to be useful
–– aEEG is able to detect seizure activity, which is common in patients.
Which imaging modality was used?
Treatment a. MRI brain
•• Therapeutic hypothermiaQ reduces combined outcome of mortality & neurodevelopmental b. CECT head
impairment c. SPECT brain
d. Transcranial ultrasound
•• Hypothermia decreases rate of apoptosis & suppresses production of mediators known to be
neurotoxic, including glutamate, free radicals, nitric oxide and lactate
•• Complications of hypothermia include thrombocytopenia, subcutaneous fat necrosis
•• Phenobarbital is the drug of choice for neonatal seizuresQ. L at e s t U p d at e s

Prognosis •• Therapeutic hypothermia (33.5ºC–


34.5ºC) is a new, upcoming treat-
Correlates with timing & severity of insult & ranges from complete recovery to death.
ment modality for moderate to
APGAR SCORE M severe HIE

Apgar score has no role in neonatal resuscitation; It just has prognostic value.

Mnemonic M

APGAR score: A-P-G-A-R


Features 0 1 2
Appearance Completely blue or pale Body pink, extremities blue Completely pink
Pulse rate Absent < 100 / min > 100 / min
Grimace* No response Grimaces only Coughs or sneezes
Activity Limp or flaccid Some flexion Active motion
Respiratory Absent Slow, irregular Good, crying
effort
*Response to catheter in nose

 RESPIRATORY DISORDERS OF NEWBORNS

A. MECONIUM ASPIRATION SYNDROME (MAS) M


L at e s t U p d at e s
•• MSL is found in 10–15% births
•• Tracheal suctioning for non vigo­
•• MAS usually occurs in term or post term babies rous babies born through MSL is no
MAS occurs when infants take meconium into their lungs during or before delivery longer recommended M

Pathophysiology
Either in utero or with the first breath, thick, particulate meconium is aspirated into lungs

Small airway obstruction
•• Obstructive emphysema or atelectasis → when aspirated material blocks the airways
•• Pneumonitis and chemical pneumonia → because of irritant property of meconium
•• Defective gas exchange.
244 Section 2: Neonatology

Clinical Manifestations
Review of Pediatrics and Neonatology

High Yield Points   M


•• Respiratory distress within 1st hour with tachypnea, retraction, grunting and cyanosis
•• Vigorous baby has strong spon- •• Over distention of the chest.
taneous respiratory effort, good
muscle tone and heart rate ≥ 100 Chest X-ray Findings
beats/min.
•• Persistent pulmonary hypertension •• Patchy infiltrates and coarse streaking of both lung fields
of newborn (PPHN) and Pneumo-
thorax are complications of MAS
••
••
Increased antero-posterior diameter
Flattening of the diaphragm. }Features of hyperinflation

Management of MAS
Supportive care and standard management for respiratory distress including:
•• Oxygen is given to maintain PaO2
•• Mechanical ventilation
•• Exogenous surfactant and/or iNO to infants with hypoxemic respiratory failure, or pulmonary
hypertension requiring mechanical ventilation, decreases need for ECMO
•• MAS refractory to conventional mechanical ventilation may benefit from High frequency
ventilation (HFV) or Extracorporeal membrane oxygenation (ECMO).
Clinical Course and Prognosis
•• Usually improves within 72 hours, but when its course requires assisted ventilation, it may
be severe with a high risk for mortality
•• Tachypnea may persist for many days or even several weeks
•• Mortality rate of meconium-stained infants is considerably higher than that of non-stained.
Prevention of MAS
•• By rapid identification of fetal distress and initiation of prompt delivery in presence of late
fetal heart rate deceleration or poor beat-to-beat fetal heart rate variability
•• Intrapartum oropharyngeal and naso-pharyngeal suctioning does not prevent MAS.
•• NRP 2010 guidelines mentioned endotracheal intubation and intratracheal suction for non-
vigorous babies with MSL
•• Latest NRP 2015 guidelines do not advocate tracheal suction for even non-vigorous babies.

High Yield Points B. RESPIRATORY DISTRESS SYNDROME (RDS) M


  M
Also known as Hyaline membrane disease (HMD).
Surfactant appears in
Occurs due to Surfactant deficiency (decreased production and secretion).
•• Fetal lung- 20 wk gestationQ
•• Amniotic fluid- 28-32 wkQ Epidemiology
•• Adequate amount of mature •• Occurs primarily in premature infants
surfactant -> 35 wk gestationQ •• Incidence is inversely related to gestational age & birth weight
•• Occurs in 60-80% of infants <28 wk gestation, while in 15–30% born at 32–36 week.
Risk factors: Prematurity, maternal diabetes, multiple births, cesarean delivery, precipitous
delivery, asphyxia, cold stress, maternal history of previously affected infant.

Pathophysiology

Absent pulmonary surfactant



High surface tension of alveoli

Alveolar atelectasis and interstitial edema

Results in perfused but not ventilated alveoli, causing hypoxia and hypercapnia

High Yield Points   M Hypercapnia, hypoxia, and acidosis produce pulmonary arterial vasoconstriction
•• Major constituents of surfactant Increased right-to left shunting through foramen ovale, & within lung itself
are dipalmitoyl phosphatidylcho- ⇓
line (lecithin)Q (65%), phosphati- Hypoxia worsens
dylglycerol, apoproteins (surfac- ⇓
tant proteins SP-A, SP-B, SP-C, and
Progressive injury to epithelial & endothelial cells from atelectasis (atelectrauma), ischemic injury,
SP-D), and cholesterol
oxygen toxicity, volutrauma (caused by mechanical ventilation)
•• Out of the Surfactant proteins, sur-
factant protein BQ is the most im- ⇓
portant Effusion of proteinaceous material into alveolar spaces and hyaline membrane formation
Chapter 12: Diseases of Newborn 245

Clinical Features

Section 2: Neonatology
Silverman scoring: To quantify respiratory distress in a neonate

•• Signs of RDS usually appear within minutes of birth


•• Tachypnea, grunting, intercostal & subcostal retractions, nasal flaring, cyanosis
•• Breath sounds may be normal or diminished with a harsh tubular quality with fine crackles High Yield Points  
•• Respiratory failure may occur in infants with rapid progression of disease
Chest X-ray in HMD:
•• Death can result from severe impairment of gas exchange, alveolar air leaks (interstitial
•• Characteristic fine reticular granu-
emphysema, pneumothorax), pulmonary hemorrhage, or IVH.
larity of parenchyma
Management •• Ground glass haziness of lung
fields
•• Avoid hypothermia to minimize oxygen consumption and start IV fluids and nutrition •• Air bronchograms present
•• Warm humidified oxygen given, to keep arterial oxygen between 50-70 mm Hg (91-95%
saturation), as hyperoxia may also contribute to lung injury in preterm infants
•• If sPO2 cannot be kept >90% at 40-70% inspired O2 concentrations, CPAP (continuous Question 6 M
positive airway pressure)Q via nasal prongs is used
•• If even on CPAP, SpO2 < 90%, assisted ventilation & endotracheal surfactant are indicated A newborn male baby deli­vered at
•• Infants with respiratory failure or persistent apnea require assisted mechanical ventilation 30 weeks gestation presented with
severe respiratory distress within
•• Inhaled nitric oxide (iNO) decreases the need for extracorporeal membrane oxygenation
30 minutes after birth requiring
•• ECMO in term and near-term infants with hypoxic respiratory failure.
intubation. His chest X-ray picture
Prevention is shown below. What is the diag­
nosis?
•• Antenatal corticosteroids significantly reduces incidence and mortality of RDS as well as
overall neonatal mortality
•• CPAP started at birth is as effective as prophylactic / early surfactant & is the approach
of choice for delivery room management of preterm neonate at risk for RDS.

High Yield Points   M


Important Facts About Antenatal Steroids
Indication All women in preterm labor who are likely to deliver a fetus within 1 wk a. Meconium aspiration syndrome
Which steroid? Betamethasone and dexamethasone have both been used antenatally. b. Hyaline membrane disease
Betamethasone may reduce neonatal death to a greater extent as compared c. Pulmonary alveolar proteinosis
to dexamethasone. (Latest 20th ed Nelson’s Pediatrics) d. Transient tachypnea of newborn
Dosage Inj Betamethasone 12 mg IM 2 doses, 24 hrs apartQ
Inj Dexamethasone 6 mg IM every 12 hrs for 4 dosesQ
Contraindication Chorioamnionitis Mnemonic M

Benefits: •• Decreases overall neo­natal mortalityQ •• Betamethasone dose 12 mg × 2


•• Decreases need for and duration of ventilator support doses = 24 mg
•• Decrease risk of RDS •• Dexamethasone dose 6 mg × 4
•• Decreased risk of severe IVH, NEC and neurodevelopmental impairment. doses = 24 mg
246 Section 2: Neonatology

C. PULMONARY ALVEOLAR PROTEINOSIS (PAP)


Review of Pediatrics and Neonatology

L at e s t U p d at e s
What is it?
Pulmonary alveolar proteinosis
includes disorders with surfactant It is a disorder characterized by intra-alveolar accumulation of pulmonary surfactant.
dysfunction, that may be due to:
•• Surfactant protein–B mutation Clinical Forms
•• Surfactant protein–C mutation •• Fatal form: Presents shortly after birth (congenital PAP); usually seen in term neonates
•• ABCA3 mutation
•• Gradually progressive form: Presenting in older infants and children.
•• GM-CSF receptor mutation
Pathophysiology

In congenital pulmonary alveolar proteinosis, there is absence of surfactant protein B



Question 7
In absence of protein B, rapid spread of DPPC does not take place
A 3.5 kg term male baby, born ⇓
of uncomplicated pregnancy, So DPPC cannot form a phospholipid monolayer on alveolar surface
deve­loped, respiratory distress at

birth, X-ray showed ground glass
appearance. He did not respond Failure of expansion of alveoli leading to poor cardiorespiratory adaptation at birth.
to surfactant; His postmortem lung ⇓
bio­psy picture is shown. His­ tory Histological findings are suggestive of disruption of pulmonary surfactant metabolism
of one month female sibling died Similar to RDS
before. What is the diagnosis?
Clinical Manifestation
•• Usually apparent in newborn period & rapidly leads to respiratory failure
•• Family history is usually present
•• Congenital PAP is clinically & obstructive radiographically indistinguishable from
respiratory distress syndrome & total anomalous pulmonary venous return.

a. Respiratory distress syndrome


Diagnosis
b. Congenital diaphragmatic hernia •• Lung biopsy is the gold standard for diagnosis
c. Neonatal pulmonary alveolar •• It shows distal air spaces filled with a granular, eosinophillic material that stain positively
protei­nosis with periodic-acid schiff (PAS) and is diastase resistant.
d. Congenital lobar emphysema
Treatment
•• No successful medical therapy has been developed
•• Lung transplantation is the only therapeutic option but its use is limited by concerns about
disease recurrence.

Prognosis
Pulmonary alveolar proteinosis in newborns is a fatal disease.

D. APNEA OF NEWBORN M

Definition
Cessation of breathing for > 20 secQ or for any duration, if accompanied by bradycardia or
cyanosis.

Epidemiology
High Yield Points
Incidence of apnea of prematurity varies inversely with gestational age, usually seen during 1st
Familial causes of neonatal respiratory 1-2 wk after birth.
distress (not RDS) include pulmonary
alveolar proteinosis, alveolar Important Causes of Apnea
capillary dysplasia, acinar dysplasia,
pulmonary lymphangiectasia, and •• PrematurityQ
mucopolysaccharidosis. •• HypothermiaQ
•• CNS: Intraventricular hemorrhage, drugs, seizures, hypoxic injury
•• Respiratory: Pneumonia, obstructive airway lesions, atelectasis, pneumothorax
•• Infectious: Sepsis, meningitis (bacterial, fungal, viral), pertussis
•• Gastrointestinal: Necrotizing enterocolitis (NEC), intestinal perforation
•• Metabolic: Hypoglycemia, hypocalcemia, hypo or hypernatremia
•• Cardiovascular: Hypotension, hypertension, heart failure, hypovolemia
•• Hematologic: Anemia, polycythemia.
Chapter 12: Diseases of Newborn 247

Pathophysiology

Section 2: Neonatology
•• Direct depression of respiratory center in CNS (hypoglycemia, meningitis, drugs)
•• Disturbances in oxygen delivery (shock, sepsis, anemia)
•• Ventilation defects (obstruction of the airway, pneumonia, muscle weakness).

Types
Central, obstructive, mixed (most common in preterm neonates)Q.

Management
High Yield Points   M
•• Cardiorespiratory monitoring
Drug of choice for apnea of prematu-
•• Gentle tactile stimulation is often adequate therapy for mild and intermittent episodes
rity is IV Caffeine citrate. If not avail-
•• Recurrent apnea of prematurity may be treated with caffeine or theophylline able, aminophylline or theophylline
•• Nasal continuous positive airway pressure (CPAP), 3-5 cm H2O, splints the upper airway and can be used
prevents airway obstruction.

E. TRANSIENT TACHYPNEA OF NEWBORN


MC cause of respiratory distress in a term neonate
•• Basic defect ³→ Delayed clearance of lung fluid
•• Also known as “delayed adaptation”
•• Delivery by caesarean section is a risk factor

Clinical features: A term/post-term neonate, delivered by caesarean section, presents with


mild respiratory distress, soon after birth, which resolves in 48 hrs, with typical CXR findings

CXR Findings

•• Fluid in interlobar fissure


•• Pleural effusion
•• Perihilar streaking (Prominent broncho-
vascular markings)

Treatment
•• Usually TTNB is a mild & self-limiting disorder
•• Distress usually resolves in 48–72 hr spontaneously
•• Supportive care.

F. CONGENITAL DIAPHRAGMATIC HERNIA (CDH): Refer chapter on Pediatric Surgical


disorders

G. BRONCHOPULMONARY DYSPLASIA (BPD)

•• BPD is a result of lung injury in infants requiring mechanical ventilation and supplemental oxygen; also
known as chronic lung disease (CLD)
•• Primarily affects babies with birth weight <1,000 g, born at <28 wk gestation
•• Injury to lungs is caused by: Alveolar collapse (atelectrauma), ventilator-induced overdistention of
lungs (volutrauma) & free radicals due to O2 supplementation.

Also known as chronic lung disease of prematurity.


248 Section 2: Neonatology

Definition of BPD
Review of Pediatrics and Neonatology

•• For infants born at <32 weeks’ gestations who received supplemental oxygen for their
first 28 days, the NIH defined BPD at 36 weeks’ postmenstrual age (PMA) as
–– Mild: No supplemental O2 requirement
–– Moderate: Supplemental O2 requirement <30%
–– Severe: Supplemental O2 requirement ≥ 30% and/or continuous positive airway
pressure (CPAP) or ventilator support
•• For infants born at ≥ 32 weeks, the NIH defined BPD as supplemental O2 requirement
for the first 28 days with severity level based on O2 requirement at 56 days.

 NEONATAL HYPOGLYCEMIA
Definition M

Operational threshold for hypoglycemia has been defined as blood glucose of < 40 mg/dL (plasma
glucose level less than 45 mg/dL).

High Yield Points Screening for Hypoglycemia is Recommended in


  M
Following Neonates M
•• Most common clinical feature
of hypoglycemia in newborns is •• Preterm & SGA babies
jitterness or tremorsQ •• LFD babies including infant of diabetic mothers (IDM)
•• Most common cause of persistent
hypogly­cemia in infancy is Nesi­ •• Any sick neonate such as those with perinatal asphyxia, polycythemia, sepsis, shock
dioblastosisQ or Congenital Hyper­ •• Infants on total parenteral nutrition.
insulinemia of Infancy or PHHI
(Persistent Hyperinsulinemic Hypo­ Schedule of Monitoring for Hypoglycemia
glycemia of Infancy)
Category of infants Time schedule
1. At risk neonates 2, 6, 12, 24, 48, and 72 hrs

2. Sick infants Every 6-8 hrs

3. Stable VLBW infants on parenteral nutrition Initial 72 hr: every 6 to 8 hrs


After 72 hr: once a day

Infants exhibiting signs compatible with hypoglycemia at any time also need to be investigated

Clinical Signs M

•• Asymptomatic
•• Symptomatic: Clinical signs include, jitteriness, tremors, apathy, episodes of cyanosis,
convulsions, intermittent apneic spells or tachypnea, weak and high pitched cry, limpness
and lethargy, difficulty in feeding, and eye rolling. Episodes of sweating, sudden pallor,
hypothermia and cardiac arrest have also been reported.

Management
Management plan of infants with asymptomatic hypoglycemia
Blood glucose Management

20-45 mg/dL Trial of oral feeds (expressed breast milk or formula) and repeat blood test after 1
hour
1. If repeat BGL is >45 mg/dL, two hourly feeds is ensured with 6 hourly monitoring of
BGL for 48 hrs. The target blood glucose value is 50 to 120 mg/dL
2. If repeat blood sugar is <45 mg/dL, IV Dextrose is started and further management
is as for symptomatic hypoglycemia

<20 mg/dL IV Dextrose is started at 6 mg/kg/min of glucose, subsequent management is as for


symptomatic hypoglycemia

For symptomatic babies: IV 10% Dextrose @ 2 mL/kg followed by continuous IV Dextrose, the
rate of which is titrated according to blood glucose values.
Chapter 12: Diseases of Newborn 249

INFANT OF DIABETIC MOTHER (IDM)

Section 2: Neonatology
Question 8
Pathophysiology in Infant of Diabetic Mother A large for date neonate with birth
wt of 4 kg showed the following
finding. What could be the under­
lying abnormality?

a. Congenital hypothyroidism
b. Infant of Diabetic mother
c. Mucopolysaccharidosis
d. Soto’s syndrome

Clinical Features in Infants of Diabetic Mothers


General
High Yield Points   M
•• Incidence of congenital anomalies is increased 3-fold
•• Large for date baby (macrosomia) with puffy, plethoric facies •• Most common congenital abnor­
mality in IDM is Congenital heart
•• Birth trauma is also a common sequela of fetal macrosomia.
diseaseQ
•• Most common congenital heart
Metabolic disease in IDM is VSDQ
•• Hypoglycemia in 25-50% (Pederson's hypothesis)Q explains it •• Most specific congenital heart
•• Hypocalcemia; Hypomagnesemia disease in IDM is TGAQ
•• Increased incidence of hyperbilirubinemia & Polycythemia. •• Most specific abnormality in IDM is
sacral agenesisQ
Respiratory
Higher incidence of respiratory distress syndrome due to delayed maturation of surfactant.

CVS
•• Cardiomegaly is common (30%)
•• Asymmetric septal hypertrophy may occur
•• Ventricular septal defect (most common), ASD, transposition of great arteries (TGA),
truncus arteriosus, double-outlet right ventricle, coarctation of aorta.

Neurologic Mnemonic M

B-O-N-D: Long term problems in


•• Lumbosacral agenesis or ‘caudal regression syndrome' (most specific abnormality) infant of diabetic mother:
•• Neural tube defects (most common neurologic abnormality)Q
•• Blindness
•• Holoprosencephaly. •• Obesity
•• Non ketotic hypoglycemia
Renal •• Diabetes
Increased incidence of renal vein thrombosis; double ureter, hydronephrosis, renal agen-
esis and dysplasia.

Gastrointestinal: Duodenal or anorectal atresia, Situs inversus, Small left colon syn-
drome (transient delay in development of left side of colon).

 NEONATAL HYPERBILIRUBINEMIA / JAUNDICE M

•• Clinical jaundice in newborns appears at bilirubin level > 5 mg/dl


•• Jaundice during 1st week of life is seen in 60% of term infants and 80% of preterm infants.
250 Section 2: Neonatology

Differences between Physiological and Pathological Jaundice of Newborn


Review of Pediatrics and Neonatology

M
Mnemonic M

•• Breastfeeding Jaundice is due to Physiological Jaundice Pathological Jaundice


feeding problem Never appears in 1st 24 hours May appear in 1st 24 hours
•• Breast milk Jaundice is due to Does not stain palms or soles May stain palms or soles
components in breast milk.
Urine does not stain diapers High coloured urine and clay colored stools
(as always unconjugated) may be present
Does not persist beyond 3 weeksQ May persist beyond 3 weeks
High Yield Points   M
•• Most common cause of Jaundice
on day 1 of life is Erythroblastosis
High Yield Points   M
fetalis Why does Physiological jaundice occur in newborns?
•• Physiological jaundice never •• Over production of bilirubin due to increased RBC volume/kg, shorter lifespan of RBCs and more ineffective
appears in 1st 24 hours of life erythropoiesis in newborns
•• Poor hepatic uptake of bilirubin
•• Less conjugation of bilirubin due to immature UDP glucuronyl trans­ferase enzyme
•• Increased enterohepatic circulation

Important Causes of Unconjugated Hyperbilirubinemia M

High Yield Points   M Increased production of bilirubin Decreased conjugation of bilirubin


Crigler Najjar Syndrome Hemolytic anemia: Crigler-Najjar syndrome type I and II
•• Due to deficiency of UDP glycuronyl •• ABO or Rh incompatibility Gilbert syndrome
transferase •• Hereditary Spherocytosis Down syndrome
•• Type I-complete deficiency; AR •• G6Pd deficiency
Hypothyroidism
•• Type II-Partial deficiency; AD Polycythemia
Breast milk jaundice
•• Phenobarbitone useful in type II Delayed cord clamping
Cephalhematoma

High Yield Points High Yield Points   M


  M
Icterus in newborns shows a cephalo­ Breast Feeding Jaundice
caudal progression. It is due to insufficient breast feeding and it usually occurs in 1st week of life.
Breast Milk Jaundice
•• Seen in 2% of breast fed term infants, usually during 2nd – 3rd week.
•• Occurs due to presence of preg­nanediol and free fatty acids in breast milk that interfere with
conjugation

Important causes of Conjugated Hyperbilirubinemia M

Question 9 A. Non-obstructive Causes


A 1 month old newborn presents
with conjugated Hyperbilirubi­ Infections Metabolic (Mnemonic: ACT G)
nemia, intra-hepatic cholestasis •• Viral: CMV, Rubella, Herpes •• Cystic fibrosis
and high alkaline phosphatase. •• Bacterial: Syphilis, Tuberculosis •• Galactosemia
Eosinophilic, PAS positive cyto­ •• Parasitic: Toxoplasmosis •• Alpha 1 antitrypsin deficiency
plasmic granules are seen in liver •• Tyrosinemia
biopsy specimen as shown below. Toxins: Sepsis, UTI, Total parenteral Idiopathic Neonatal hepatitis (Giant cell
Diagnosis is? nutrition hepatitis)

B. Obstructive Causes of Conjugated Hyperbilirubinemia

Intrahepatic causes Extrahepatic causes


•• Caroli disease •• EHBA (Extrahepatic Biliary atresia)
•• Congenital hepatic fibrosis •• Choledochal cyst
a. Alpha 1 AT deficiency •• Alagille syndrome (Paucity of bile ducts) •• Choledocholithiasis
b. Cong hepatic fibrosis •• Choledochal cyst •• Stricture of common bile duct
c. Extrahepatic biliary atresia •• Dubin Johnson syndrome
d. Ductal plate defect •• Rotor syndrome
Chapter 12: Diseases of Newborn 251

Clinical Features of Neonatal Jaundice

Section 2: Neonatology
M
Question 10 M

What is seen in this baby with


cerebral damage due to kernic­
terus?

a. Moro’s reflex
b. Tonic neck reflex
c. Neck rigidity
d. Opisthotonus

High Yield Points   M


Prolonged hyperbilirubinemia is seen
in:
Neurological Features •• Gilbert syndrome
•• Down syndrome
Kernicterus, or bilirubin encephalopathy is a neurologic syndrome resulting from deposition of •• Hypothyroidism
unconjugated (indirect) bilirubin in basal ganglia. •• Breast-feeding Jaundice
•• Crigler-Najjar syndrome
Acute Bilirubin encephalopathy
•• Lethargy, poor feeding and loss of the Moro reflex are common initial signs
•• Infant may appear gravely ill with diminished tendon reflexes and respiratory distress Mnemonic
•• Opisthotonus with a bulging fontanel and a shrill high-pitched cry may follow Features of chronic bilirubin
•• In advanced cases, convulsions, coma and death. encephalopathy
S-A-D M-U-M”
•• Sensorineural hearing loss
Diagnostic Evaluation
•• Athetosis
Done for patients with significant or symptomatic hyperbilirubinemia & includes: •• Dental dysplasia
•• Determination of direct and indirect bilirubin fractions •• Mental retardation
•• Hemoglobin with reticulocyte count and peripheral smear examination •• Upward gaze limitation
•• Blood grouping and Coomb’s test.
252 Section 2: Neonatology

Treatment of Neonatal Jaundice


Review of Pediatrics and Neonatology

M
High Yield Points   M
•• Area of brain most commonly Goal of therapy is to prevent neurotoxicity related to indirect-reacting bilirubin.
involved in neonatal jaundice is 3 main modalities of treatment are:
basal gangliaQ A. Phototherapy
•• Extrapyramidal type of cerebral B.  Exchange transfusion
palsy is seen due to neonatal
jaundice
C. Drugs

A. Phototherapy
Most Effective Wavelength: 450-460 nm (425-475 nm)
High Yield Points
Mechanisms
Indirect hyperbilirubinemia, reti­
culocytosis, and a smear with evidence •• Photo-isomerization: Reversible reaction that converts the toxic native unconjugated
of RBC destruction suggest hemolysis
4Z,15Z-bilirubin into an isomer, 4Z,15E bilirubin, which can then be excreted in bile without
in a newborn with Jaundice
conjugation
•• Structural isomerization: Bilirubin is converted into lumirubin, which is an irreversible
structural isomer, excreted by the kidneys in the unconjugated state
•• Photo-oxidation: a minor method

High Yield Points   M


•• Most important mechanism by which phototherapy acts is-struc­tural isomerisation
•• Effectiveness of phototherapy dose not depend upon the skin pigmentation of the baby

Therapeutic Effect of Phototherapy Depends on


•• Type of lamps used: LED lamps are better than fluorescent lamps
•• Distance between the lights and the infant
•• Surface area of exposed skin & Rate of hemolysis.

Question 11 M High Yield Points   M


What is the most impor­tant mecha­ Precautions to be Taken before Star­ting Phototherapy
nism by which this treat­ment acts? •• Genitalia & eyes of baby must be covered
•• Body temperature should be monitored;
•• Irradiance should be measured directly.

Complications of Phototherapy M

•• Loose stools; dehydration due to increased insensible water loss and diarrhea
a. Photo isomerization •• Hypocalcemia
b. Structural neutralization •• Bronze baby syndrome: Dark, grayish brown skin discoloration mainly seen in infants with elevated
c. Structural isomerization conjugated bilirubin
d. Photo oxidation •• Temparature disturbances: overheating or hypothermia from exposure
•• Erythematous macular or purpuric rash associated with transient porphyrinemia
•• Retinal toxicity; gonadal toxicity or mutations

Contraindication; Phototherapy is contraindicated in the presence of porphyria.

B. Exchange Transfusion
Double-Volume Exchange Transfusion is performed if
•• Bilirubin levels are very high & the risk of kernicterus exceeds the risk of procedure.
•• Intensive phototherapy has failed to reduce bilirubin levels to a safe range
Potential complications: metabolic acidosis, electrolyte abnormalities, hypoglycemia, hypocal-
cemia, thrombocytopenia, volume overload, arrhythmias, NEC, infection, GVHD
Chapter 12: Diseases of Newborn 253

Approximate Cut-off Bilirubin Levels for Phototherapy and

Section 2: Neonatology
High Yield Points   M
Exchange Transfusion
Indications for exchange transfusion
Age Phototherapy cut-off Exchange transfusion cut-off in Rh isoimmunization:
•• Cord blood bilirubin > 5 mg/dl or
Term baby, 24–48 hrs 15 mg/dl 20 mg/dl •• Cord blood PCV < 30 or
•• Cord blood Hb < 10 g/dl
Term baby, 48–72 hrs 18 mg/dl 25 mg/dl

Term baby, > 72 hrs 20 mg/dl 25 mg/dl

For preterm babies 1% of birth wt in grams Phototherapy cut-off + 5 mg/dl

C.  Drugs useful in Treating Neonatal Hyperbilirubinemia Question 12 M

Intravenous Immunoglobulin (IvIg) What is the approximate bilirubin


level of this neonate, whose palms
•• Used as an adjunctive in hyperbilirubinemia caused by isoimmune hemolytic disease are also stained yellow?
•• Recommended when serum bilirubin is approaching exchange levels despite maximal
interventions including phototherapy
•• It reduces the need for exchange transfusion by reducing hemolysis.

 NECROTIZING ENTEROCOLITIS (NEC)


•• NEC is the most common life-threatening emergency of GIT in a neonate.
•• NEC is characterized by mucosal or transmural necrosis of the intestine a. 5-6 mg/dl b. 8-10 mg/dl
•• Exact cause of NEC remains unclear but is most likely multifactorial c. 10-12 mg/dl d. 15-16 mg/dl
•• Triad of intestinal ischemia (injury), enteral nutrition (metabolic substrate) and bacterial
translocation has classically been linked to NEC.

Risk factors for NEC M

•• Prematurity: L at e s t U p d at e s
–– 90% of all cases of NEC occur in preterms but it can occur in full-term neonates
•• Sn Mesoporphyrin (SnMP) is
–– Age of onset is inversely related to gestational age. a promising drug candidate for
•• Aggressive enteral feeding esp use of formula feeds: Neonatal Jaundice that acts by
competitive enzymatic inhibi-
•• Intrauterine hypoxia tion of the rate-limiting conversion
•• Absent or reversed end diastolic flow in umbilical arteries on antenatal USG doppler. of heme-protein to biliverdin by
heme oxygenase
Clinical Features M

Gastrointestinal Systemic

•• Abdominal distention/tenderness •• Lethargy/Apnea/respiratory distress


•• Feed intolerance/Delayed gastric emptying •• Temperature instability/Acidosis
•• Occult/gross blood in stool •• Glucose instability/Poor perfusion/shock
•• Abdominal mass/Erythema of abdominal wall •• Disseminated intravascular coagulopathy
(DIC)

High Yield Points   M


High Yield Points   M

•• Distal part of ileum & proximal colon are most frequently involved in NEC •• Most important risk factor for NEC
is PrematurityQ
•• Coagulation necrosis is the characteristic histologic finding in intestinal specimens in NEC •• Breast milk protects against NEC
•• Pneumatosis intestinalis (air in the bowel wall) is diagnostic for NEC •• Triad of blood findings in severe NEC
•• Portal venous gas is a sign of severe NEC includes severe metabolic acidosis,
•• Mortality in NEC is 10-30%, des­pite best supportive care. refractory thrombocytopenia &
Hypona­tremia
254 Section 2: Neonatology

Modified Bell’s Staging and Treatment of NEC


Review of Pediatrics and Neonatology

M
Question 13 M

Identify the abnormality showed Stage Systemic signs Intestinal signs Radiologic signs Treatment
by the arrowhead on the given IA Temperature instability, Elevated pregavage Normal or mild ileus NPO (Nil per-oral),

Suspected NEC
abdominal X-ray apnea, lethargy residuals, mild IV antibio­tics ×
abdominal 3 days

Stage I:
distension, occult
blood in stool
IB Same as IA same as IA, plus Same as IA Same as IA
gross blood in stool
IIA Same as IA Same as I, plus lleus, pneumatosis NPO, IV antibio­tics
absent bowel intestinalisQ for 7 to 10 days
a. Portal vein gas Definite NEC sounds, abdominal
b. Pneumatosis intestinalis
Stage II:

tenderness
c. Pneumoperitoneum
IIB Same as I, plus mild Same as I, plus absent Portal veinQ gas NPO, IV antibiotics
d. Ascites
metabolic acidosis, bowel sounds, shadows × 14 days
thrombocytopenia definite abdominal
tenderness,
abdominal cellulitis
IIIA Same as IIB, plus Same as I and II, plus Same as IIB, plus NPO, IV antibiotics
hypotension, signs of generalized definite ascites × 14 days,
Advanced NEC

bradycardia, peritonitis, marked fluid boulses,


respiratory & metabolic tenderness and inotropes, mecha­
Stage III:

acidosis, DIC, distension of nical ventilation,


neutropenia abdomen paracentesis
IIIB Same as IIIA Same as IIIA Same as IIB, plus Same as IIIA, plus
pneumoperi- surgery
toneum

NEC Stage IIa: Pneumatosis intestinalis NEC Stage IIb: Portal vein gas

 RETINOPATHY OF PREMATURITY (ROP)

What is it? It is a vaso-proliferative disorder of retina among preterm infants.


Risk Neonates born at < 32 weeks gestation or with birth wt < 1500 gm; Preterm infants
factors born at ≥ 32 weeks, who had a turbulent NICU course or required prolonged O2
High Yield Points therapy
Staging Stage 1 Thin white demarcation line separating vascular from avascular retina
Plus disease in ROP is presence of
dilatation & tortuosity of retinal Stage 2 Demarcation line becomes a ridge
vessels at posterior pole of eye. Stage 3 Extra retinal fibrovascular proliferation extending to vitreous
Stage 4 Partial retinal detachment not involving macula (4A) & involving macula (4B)
Stage 5 Complete retinal detachment
When to First screening examination should be carried out at 32 weeks of post menstrual age
screen? (PMA) or 4 weeks of postnatal age, whichever is laterQ
Treatment Ablation of peripheral normal avascular retina & thereby abolishing hypoxic drive of
retina (mediated by overexpression of VEGF)
Chapter 12: Diseases of Newborn 255

Section 2: Neonatology
Answer Keys for Image-Based Questions

Answers Explanations / Identifying features


1. Ans. b. Convection This covered equipment, with transparent walls is known as an Incubator, which is used for maintaining
temperature of neonates, especially preterm
2. Ans. a. Radiant warmer An open care system for a neonate with a bed along with a overhead heating unit, known as radiant warmer
3. Ans. c. Lesser jaundice The picture shows a mother providing ‘kangaroo mother care’ (KMC)
4. Ans. a. aEEG This is the tracing of an amplitude integrated EEG (aEEG)
5. Ans. d. Transcranial ultra­sound This picture shows Transcranial ultrasound of an infant done through the open anterior fontanelle.
It shows the 2 lateral ventricles & areas of echogenicity near it, suggestive of PVL
6. Ans. b. Hyaline membrane Chest X-ray showing bilateral ground glass opacity of lung fields, in a preterm neonate with respiratory
disease distress soon after birth suggests Hyaline membrane disease
7. Ans. c. Neonatal pulmonary Lung biopsy showing alveoli filled with homogenous, eosinophilic PAS +ve material in a term
alveolar proteinosis newborn with +ve family history suggests Neonatal pulmonary alveolar proteinosis
8. Ans. b. Infant of Diabetic Hairy pinna in an infant, is usually seen in infant of diabetic mother
mother
9. Ans. a. Alpha 1 AT deficiency Liver biopsy showing eosinophilic, PAS +ve granules in an infant with conjugated Hyperbilirubinemia,
suggests α1 antitrypsin deficiency
10. Ans. d. Opisthotonus The arched posture with concavity of the back, is suggestive of opisthotonus
11. Ans. c. Structural isomeri­zation It shows a naked neonate (with eyes & genitalia covered) receiving Phototherapy
12. Ans. d. 15-16 mg/dL Yellow staining of palms and soles signify a bilirubin level of ≥15 mg/dl
13. Ans. c. Pneumoperitoneum Abdominal X-ray showing free gas under the diaphragm, visible around liver & spleen is suggestive of
Pneumoperitoneum
256 Section 2: Neonatology
Review of Pediatrics and Neonatology

Questions
 INTRAUTERINE GROWTH RESTRICTION 12. What is the most common cause of Neonatal mortality
in India? M  (Recent Question 2017)
1. Which of the following statements is NOT true regarding
a. Severe anemia b. Sepsis
IUGR? (APPG 2016)
c. Prematurity d. Hypothermia
a. Elevated Head circumference/ Abdominal circum­ference
13. A 6-day-old neonate weighing 2800 gm (Birth weight
(HC/AC) ratio
b. Ponderal index above 10th percentile 3200 gm), was brought with c/o fever, poor feeding
c. Increased diastolic velocity in Middle cerebral artery and poor activity. No history of vomiting or diarrhea.
(MCA) Doppler Axillary temperature was 39oC, there was depressed
d. Femur length/Abdominal circumference (FL/AC) ratio is >23.5 fontanelle, sunken eyes, decreased urine output and
2. In asymmetrical IUGR which organ is not affected? M decreased skin turgor. Her mother has a history of
 (Recent Question 2016) decreased milk production. What is your diagnosis?
a. Subcutaneous fat b. Muscle  (AIIMS Nov 2016)
c. Liver d. Brain a. Neonatal sepsis b. Galactosemia
c. Dehydration fever d. Acute renal failure
3. Asymmetrical fetal growth restriction is associated with:
 (MAHA PGM CET 2014) 14. Sepsis screen includes all the following except?
a. Chromosomal aberration b. Viral infection a. Total leucocyte count  (JIPMER May 2016)
c. Idiopathic d. Placental insufficiency b. Absolute neutrophil count
4. IUGR is caused by all except: (DNB June 2010) c. Band cell count d. IT ratio
a. Diabetes b. Alcohol 15. Most common cause of neonatal sepsis in India is:
c. Smoking d. Chronic renal failure  (JIPMER May 2016)
5. What is the ponderal index of a neonate with weight a. Staphylococcus b. Escherichia
2 kg and height 50 cm: (AIIMS Nov 2010) c. Streptococcus d. Klebsiella
a. 1.6 b. 3.6 16. Which one of the following statements is TRUE regarding
c. 2.2 d. 2.6 neonatal sepsis? (APPG 2016)
6. Not seen in small for date babies? (APPG 2008) a. Refusal to suck, poor cry and lethargy are common
a. Hypoglycemia b. Polycythemia features
c. Intracranial bleed d. Hypocalcemia b. Neutrophilia is the hallmark of neonatal sepsis
7. Full term, small for date babies are at high risk of: c. Sepsis occurs on the 4th day and is mostly due to
a. Hypoglycemia b. IVH (AIPGMEE 2000) microbes from maternal genitalia
c. BPD d. Hyperthermia d. Antibiotics are withheld till the causative bacterium & drug
sensitivity are known through culture-sensitivity testing
 NEONATAL SEPSIS 17. The most common cause of neonatal sepsis in India is:
 (Recent Question 2016)
8. A forceps delivered premature baby develops fever,
abnormal behavior, bulging fontanelle 5 days after a. Klebsiella b. E. coli
birth. What is the likely causative organism? c. Group B streptococci d. Staph aureus
 (JIPMER May 2018) 18. Which of the following streptococci cause neonatal
a. Neisseria meningitidis b. Listeria monocytogenes meningitis? (MAHA PGM CET 2015)
c. Staph aureus d. Pneumococcus a. S. agalactiae b. S. pyogenes
9. Which is the most common cause of neonatal sepsis in c. S. bovis d. S. mutans
India? (FMGE Nov 2017)
19. The most common fungal infection in the neonates
a. Klebsiella pneumoniae b. Mycoplasma pneumonia
c. Group B Streptococcus d. Listeria monocytogenes transmitted by caregiver’s hands is? (AIIMS May 2014)
10. Most common cause of Neonatal sepsis in India: M  a. Candida albicans b. Candida glabrata
 (FMGE pattern 2017) b. Candida tropicalis d. Candida parapsilosis
a. Acinetobacter b. Klebsiella 20. Neonatal meningitis is caused by all except:
c. Staphylococcus d. Streptococcus
a. N. meningitidis b. E. coli (WBPG 2010)
11. The most common manifestation of late onset neonatal
sepsis is: (Recent Question 2017) c. Listeria d. Group B streptococci
a. Meningitis 21. Most common cause of neonatal sepsis in hospital in
b. Disseminated Intravascular coagulation India is: (AIIMS Nov 07)
c. Pneumonia a. Escherichia coli b. Klebsiella
d. Focal infections of bone and joints
c. Staphylococcus aureus d. Listeria monocytogenes
Chapter 12: Diseases of Newborn 257

22. A 25 years old woman had premature rupture of 33. Hypothermia in neonate is characterized by:

Questions
membranes and delivered a male child who became  (PGI Dec 2002)
lethargic and apenic on the 1st day of birth and went into a. Hyperactivity b. Hypoglycemia
shock. The mother had a previous history of abortion 1 c. Apnea d. Increased urinary output
year back. On vaginal swab culture growth of hemolytic e. Hyperglycemia
colonies on blood agar was found. On staining these
were found to be Gram positive cocci which of the  PERINATAL ASPHYXIA
following is the most likely etiological agent? (AI 2004)
a. Streptococcus pyogenes b. Streptococcus agalactiae 34. If APGAR score is 6, at 5 minutes of life, then what is the
c. Peptostreptococci d. Enterococcus faecium interpretation? (DNB June 2018)
23. True about neonatal sepsis: (PGI June 03) a. Child is normal b. Moderate asphyxia
a. Meningitis commonly occur lately c. Severe birth asphyxia
b. Jaundice predisposes d. It requires immediate resuscitation
c. Fever is a common feature 35. APGAR acronym stands for? (AIIMS Nov 2017)
d. Jaundice is a common feature a. Activity, pulse pressure, grimace, appearance, rate of
e. Leukocytosis seen respiration
b. Appearance, pressure, grimace, MAP, heart rate
 HYPOTHERMIA
c. Appearance, pressure, grimace, appearance, rate of heart
beat
24. A newborn loses maximum heat from:
d. Appearance, pulse, grimace, activity, respiration
 (FMGE Dec 2018)
36. Effects of hypoxemic ischemia in a neonate include?
a. Head b. Abdomen
 (PGI May 2018)
c. Palms & soles d. Neck
25. The range of environmental temperature at which the a. Neurological damage
BMR (Basal Metabolic rate) of the baby is at a minimum, b. Subcutaneous fat necrosis
oxygen consumption is least and the baby maintains its c. Pulmonary hypertension
normal body temperature is called as:  d. Hyperglycemia
a. Thermoneutral environment (Recent Question 2017) e. Hypercalcemia
b. Thermal comfort zone 37. What is the interpretation of Apgar score of 3?
c. Ambient environmental temperature a. Moderately depressed baby (NEET PG Jan 2019)
d. Ideal thermal environment b. Severely depressed baby
26. What is the body temperature range to categorise a neonate c. Normal d. Mild birth asphyxia
as having cold stress? M  (Recent Question 2017) 38. Apgar score < 3 at 5 minutes of life is a predictor of:
a. 35.4 to 36.0 degoC b. 33.4 to 34. degoC a. Poor neurological outcome
c. 34.4 to 35.4 degoC d. 36 to 36.4 degoC b. Increased risk of neonatal mortality
27. What should be the temperature of the delivery room for c. Risk of cerebral palsy
the neonate to be kept in warmer? M  (AIIMS Nov 2016) d. Neonatal depression
a. 37-39°C b. 34-36°C 39. APGAR stands for? M  (AIIMS Nov 2017)
c. 22-26°C d. 30-32°C a. Appearance, Pressure, Grimace, Activity, respiration
28. Brown fat is absent in? (AIIMS May 2015) b. Appearance, Pulse, Grimace, Activity, Respiration
a. Scapula b. Mesentery c. Appearance, Pressure, Grimace, Activity, Respiration
c. Subcutaneous tissue d. Adrenal cortex d. Awareness, Pulse, Grimace, Activity, Respiration
29. Temperature of NICU is: (NEET Pattern 2012) 40. What is the 1 minute APGAR score of a baby who is
a. 20–22°C b. 22–26°C gasping, completely pale, is lying totally limp & showing
c. 26–30°C d. 30–35°C no movements, has heart rate 40/min and grimaces on
30. All of the following can occur in a neonate for heat doing oropharyngeal suction? (Recent Question 2017)
production except: (AIIMS Nov 2011; Nov 2006) a. 0 b. 1
a. Shivering c. 2 d. 3
b. Breakdown of brown fat with adrenaline secretion
c. Universal flexion like a fetus 41. What is the APGAR core for a baby with HR 120/
d. Cutaneous vasoconstriction min, RR-40/min, strong respiratory efforts, peripheral
31. The different manifestations of hypothermia are: cyanosis, grimace while suctioning and having slightly
 (PGI June 2006) flexed posture but no active movements of limbs?
a. Apnea b. Hypoglycemia a. 6 b. 7 (JIPMER Nov 2016)
c. Hyperglycemia d. Tachycardia c. 8 d. 9
e. Hypoxia
42. APGAR score -include all except: M 
32. Which of the following is the principal mode of heat
exchange in an infant incubator? (AIPGMEE 2006)  (Recent Question 2017, 2016)
a. Radiation b. Evaporation a. Heart rate b. Respiratory rate
c. Convection d. Conduction
 c. Muscle tone d. Colour
258 Section 2: Neonatology

43. A 32-week baby is born to a mother with eclampsia, 55. A preterm infant with poor respiration at birth starts
Review of Pediatrics and Neonatology

who was given IV magnesium sulphate. The baby was throwing seizures at 10 hours after birth. Antiepileptic
resuscitated and transferred to the NICU. 12 hours later, of choice shall be: (AIIMS Nov 2012)
the baby showed hypotonia, lethargy, constricted pupils a. Levetiracetam b. Phenytoin
and two episodes of seizures. The staging of HIE is? c. Phenobarbitone d. Lorazepam
a. 1 b. 2 (AIIMS May 2014) 56. Which vitamin deficiency is responsible for neonatal
c. 3 d. There is no HIE seizure: (AIIMS Nov 2009)
44. Grimace has what score in APGAR score: M  a. Pyridoxine b. Vitamin C
a. 0 b. 1 (NEET Pattern 2013) c. Thiamine d. Cobalanum
c. 2 d. 3 57. Most common cause of seizure in newborn is:
45. In Sarnat and Sarnat classification of hypoxic ischemic  (AIIMS May 2008)
encephalopathy, seizures occur in which stage?
a. Hypoxia induced ischemic encephalopathy
 (TN PGMEE 2013)
b. Hypocalcemia
a. Stage 1 b. Stage 2 c. Metabolic abnormality d. Sepsis
c. Stage 3 d. Stage 4
58. Commonest type of seizure in newborn:
46. Perinatal asphyxia leads to all except: (DNB Jun 2011)
 (AIIMS Nov 2007)
a. Hypoxic ischemic encephalopathy
a. Clonic b. Tonic
b. Meconium aspiration syndrome
c. Subtle d. Myoclonic
c. Kernicterus d. Necrotizing enterocolitis
59. Jitteriness can be distinguished from seizures by all of
47. All the following factors affect APGAR score except:
the following except: (AI 2007)
 (COMEDK 2011)
a. Prematurity b. Maternal drugs a. Sensitivity to stimulus b. Frequency of movement
c. Neurological condition of the newborn c. Abnormality of gaze d. Autonomic disturbance
d. Mode of delivery
48. About Hypoxic Ischemic encephalopathy true is:  MECONIUM ASPIRATION SYNDROME
 (AIIMS Nov 10)
a. Lower limbs affected more than upper limbs 60. An infant is born with meconium stained liquor. Which
b. Prox. Muscle > distal muscles of the following is false regarding the management?
c. Seizure d. Trunk involved  (Recent Question 2017)
49. Failure to initiate and maintain spontaneous respiration a. Intrapartum suctioning is not recommended
following birth is clinically known as: (COMEDK 2007) b. Suctioning in vigorous and non-vigorous babies
a. Birth asphyxia c. Risk of cardiac arrhythmias in nasopharyngeal suctioning
b. RDS—Respiratory distress syndrome d. Tracheal suctioning can be done using ET tube
c. Respiratory failure d. Pulmonary edema
50. All of the following therapies may be required in one- 61. Meconium passage in utero leads to which of the
hour-old infant with severe birth asphyxia except: following: (Recent Question 2013, AIIMS MAY 94)
 (APGMEEI 05) a. Listeriosis b. Obstructive emphysema
a. Glucose b. Dexamethasone c. Pathological jaundice d. Meconium ileus
c. Calcium gluconate d. Normal saline 62. A baby is born with meconium stained liquor which
of the following is taken account of in terming a baby
 NEONATAL SEIZURES vigorous except: (AIIMS Nov 2009)
a. Tone b. Colour
51. The type of neonatal seizures with worst prognosis is? c. Heart Rate d. Respiration
 (JIPMER May 2016) 63. Meconium aspiration is done for 3 times but no breathing
a. Subtle b. Focal clonic occurs. Next step in resuscitation would be:
c. Atonic d. Myoclonic  (MAHA PG 08)
52. Commonest type of seizure in newborn is? M  a. Chest compression
 (Bihar PG 2015) b. O2 inhalation
a. Clonic b. Tonic c. Bag and mask ventilation
c. Subtle d. Myoclonic d. Tickling of sole
53. A 2-day-old neonate in the neonatal ICU develops 64. A 3 kg term baby delivered by caesarean section
seizures. Which of the following would be the best develops respiratory distress soon after birth. The liquor
investigation for the child: (AIIMS May 2013) was meconium stained. Breathing rate is 90/minute;
a. Transcranial ultrasound b. Skull X-ray Which of the following are correct statements:
c. CT scan d. MRI a. Transient tachypnea of newborn  (PGI Dec 2004)
54. Best prognosis of neonatal seizures is in? M  b. Meconium aspiration syndrome
 (DNB Pattern 2013) c. Reticulonodular shadows in X-ray chest
a. Myoclonic b. Tonic clonic d. Surfactant production is defective
c. Focal clonic d. Opsoclonus e. Oral feeding should be started immediately
Chapter 12: Diseases of Newborn 259

 RESPIRATORY DISTRESS SYNDROME (RDS) 74. Lecithin/Sphingomyelin ratio is done to assess the

Questions
maturity of: M  (JIPMER 2014)
65. Calculate the Silverman score for a neonate with a. Lung b. Fetal circulation
paradoxical breathing, marked chest retractions, but c. Brain d. Gonad
minimal nasal flare and expiratory grunt audible only 75. Which one of the following medical disorder in the
with stethoscope (video given in exam): mother leads to delayed fetal lung maturity:
 (AIIMS Nov 2018)  (Recent Question 2013)
a. 4 b. 5
a. Heart disease b. Diabetes
c. 7 d. 8
c. Thalassemia minor d. Epilepsy
66. A baby is born prematurely at 29 weeks gestation by
76. A term gestation newborn developed respiratory distress.
caesarean delivery done for fetal distress. Neonate
Which of the following would favour respiratory distress
develops tachypnea, flaring, subcostal and intercostal
retractions immediately after birth. Chest radiography syndrome (HMD)? (AIIMS May 12, Nov 10)
shows bilateral, diffuse, ground glass appearance, air a. History of receiving antenatal corticosteroids
bronchograms and poor lung expansion. What is the b. Air bronchogram on chest X-ray
best treatment plan in the present case?  c. Onset of distress after 6 hours of birth
 (JIPMER May 2017) d. Term birth
a. Oxygen therapy 77. Nile blue sulfatase test is to detect fetal: (WBPG 2012)
b. Surfactant administration and respiratory support a. Brain maturity b. Skin maturity
c. Emergency surgical repair of tracheoesophageal fistula c. Kidney maturity d. Lung maturity
d. Only observation at this point 78. 3.5 kg term male baby, born of uncomplicated
67. A preterm neonate with hyaline membrane disease pregnancy, developed, respiratory distress at birth, not
needs to be treated with: (Recent Question 2018) responded to surfactant, ECHO finding revealed nothing
a. Dexamethasone b. Betamethasone abnormal, X-ray showed ground glass appearance and
c. Hydrocortisone d. Surfactant culture negative. Apgars 4 and 5 at 1 and 5 minutes.
68. A 3 hour old preterm baby develops respiratory grunt History of one month female sibling died before. What
and X-ray shows reticulonodular pattern. Diagnosis is: is the diagnosis?
a. Hyaline membrane disease (Recent Question 2016)  (AIIMS May 11, May 08, Nov 11, Nov 08, Nov 07)
b. Transient tachypnea of newborn a. TAPVC
c. Meconium aspiration syndrome b. Meconium aspiration
d. All of above c. Neonatal pulmonary alveolar proteinosis
69. Lungs do not collapse during expiration because of the d. Difuse herpes simplex infection
presence of? (WB PGMEE 2016) 79. Surfactant is produced by: (JIPMER 2010)
a. Hyaline membrane a. Type 2 pneumocytes b. Type 1 pneumocytes
b. Dipalmitoyl phosphatidyl choline
c. Clara cells d. Endothelium
c. Macrophages
80. Hyaline membrane deposition is seen in: M  (JIPMER 2010)
d. Interstitial fluid 
70. Surfactant appear in amniotic fluid at: M  a. ARDS b. RDS
 (Recent Question 2016) c. Pulmonary hemorrhage d. Interstitial lung disease
a. 20 wk b. 28 wk 81. A 1.5 kg child born at 32 weeks by LSCS presents with
c. 32 wk d. 4 wk moderate respiratory difficulty (RR 70/minutes). Which
71. Which of the following is NOT a feature of acute of the following is the most appropriate management:
respiratory distress syndrome? (COMEDK 2016)  (AIIMS Nov 10, 99)
a. Interstitial inflammatory infiltrates a. CPAP b. Mechanical ventilation
b. Interstitial edema c. Warm oxygen d. Surfactant and ventilation
c. Mucus plugs in the bronchioles 82. Hyaline membrane disease of lungs is characterized by:
d. Hyaline membranes lining the alveoli  (AIIMS Nov 10)
72. A 3.5 kg newborn born by full-term normal vaginal a. FRC is smaller than closing volume
delivery presented with respiratory distress not b. FRC is greater than closing volume
responding to surfactant therapy. There is a history of c. FRC is equal to closing volume
previous sibling’s death at one month of age due to d. FRC is independent of closing volume
respiratory distress. What diagnosis should you suspect? 83. Which of the following will favour the diagnosis of RDS
a. Pulmonary alveolar proteinosis (AIIMS Nov 2015) in newborn: (AIIMS Nov 2010)
b. Hyaline membrane disease
a. Receipt of antenatal steroids
c. Meconium aspiration syndrome
b. Air bronchogram in chest X-ray
d. Persistent pulmonary hypertension of newborn
c. Manifests after 6 hrs
73. Respiratory distress syndrome–true is: (TN PGMEE 2015) d. Occurs after term gestation
a. Not related to premature birth 84. The dose of betamethasone in prenatal to prevent
b. L:S ratio > 2
respiratory distress syndrome is: (DNB June 2008)
c. Easily treatable and has low mortality
d. Amniotic fluid phosphatidyl choline levels is lower than a. 6 mg b. 12 mg every 24 hours
normal c. 6 mg every 12 hours d. 4 mg stat
260 Section 2: Neonatology

85. Loss of pulmonary surfactant in premature infant leads 93. Characteristic radiological feature of transient tachy­
Review of Pediatrics and Neonatology

to: (DNB Dec 2008) pnoea of newborn is: (AIIMS May 05)
a. Pulmonary edema a. Reticulogranular appearance
b. Collapse of alveoli b. Low volume lungs
c. Elastic recoil of lungs c. Prominent horizontal fissure
d. All d. Air bronchogram
86. A newborn weighing 1000 g is born at gestational age of 94. Transient tachypnea of newborn (TTN) is commonly
30 weeks with respiratory distress after 2–3 hrs of birth. seen in which of the following situations?
What are the diagnostic possibilities: (PGI June 2007) a. Term delivery requiring forceps (AIIMS May 2002)
a. Diaphragmatic hernia b. Term requiring ventouse
b. Congenital bronchopulmonary cysts c. Elective cesarean section d. Normal vaginal delivery
c. Bronchopulmonary dysplasia
d. HMD  APNEA OF NEWBORN
e. Pulmonary hemorrhage
95. In a neonate, cessation of breathing for 10 second with
87. Fetal lung maturity is assessed by: M  (DNB June 2006) bradycardia is: M  (Recent Question 2014)
a. L/S ratio a. Apnea b. Dyspnea
b. Bilirubin content of amniotic fluid c. Cheyne Stokes respiration
c. Ultrasound d. X-ray d. None
88. Incidence of respiratory distress syndrome in a 28 weeks 96. Neonatal apnea is seen in all except: (PGI June 08)
baby weighing 1500 g: (TN PGMEE 2005) a. Prematurity b. Hyperglycemia
a. 10% b. 20% c. Hypoglycemia d. Hypocalcemia
c. 40% d. 80% e. Hypothermia
89. With reference to RDS, all of the following statements
 NEONATAL HYPOGLYCEMIA & INFANT OF
are true except: (AIPGMEE 2002)
DIABETIC MOTHER
a. Usually occurs in infants born before 34 weeks of
gestation 97. What is the most probable diagnosis in this neonate
b. Is more common in babies born to diabetic mothers shown below? (NEET PG Jan 2019)
c. Leads to cyanosis
d. Is treated by administering 100% oxygen
90. A neonate delivered at 32 weeks, is put on a ventilator,
X-ray shows “white out lung” and ABG reveals PO2 of
75. Ventilator settings are FiO2 of 70, and rate of 50/
minute. Next step to be taken should be? (AIPGMEE 01)
a. Increase rate to 60 per minute
b. Increase FiO2 to 80 a. Infant of diabetic mother
c. Continue ventilation with the same settings b. Beckwith Wiedemann syndrome
d. Start weaning off ventilator c. Congenital hypothyroidism
d. IUGR baby
 TRANSIENT TACHYPNEA OF NEWBORN 98. After the delivery of an infant of diabetic mother, glucose
of the infant was 60 mg/dL. Which other investigation
91. A 26-year-old third gravida mother delivered a male baby does the sister expects that the physician would ask her
weighing 4.2 kg at 37 weeks of gestation through an to do? (AIIMS May 2018)
emergency caesarean section, for obstructed labour. The a. Serum potassium b. CBC
child developed respiratory distress one hour after birth. c. Serum calcium d. Serum chloride
He was kept nil per orally (NPO) and given intravenous 99. Infant of diabetic mother have the following except: M
fluids. He maintained oxygen saturation on room air. No  (Recent Question 2017)
antibiotics were given. Chest radiograph revealed fluid in a. Macrosomia b. Neural tube defect
interlobar fissure. Respiratory distress settled by 24 hours c. Hyperglycemia d. Hypocalcemia
of life. What is the most likely diagnosis? 100. Which is the most common congenital abnormality in a
baby of diabetic women? M  (AIIMS May 2016)
a. Transient tachypnea of the newborn (DNB Dec 2009)
a. Ventricular Septal defect b. Sacral agenesis
b. Meconium aspiration syndrome
c. Meningomyelocele d. Anencephaly\
c. Persistent fetal circulation
101. Infants of diabetic mothers are at high risk of which of
d. Hyaline membrane disease the following anomalies? (Recent Question 2016)
92. Respiratory rate per minute in a 1 month old baby, to a. Agenesis of corpus callosum
label it tachypnea should be more than: M  b. Hearing loss
a. 40 b. 50 (DNB June 2008) c. Transposition of great arteries
c. 60 d. 70 d. Cataract
Chapter 12: Diseases of Newborn 261

102. Which of the following babies has the least risk of  NEONATAL HYPERBILIRUBINEMIA

Questions
developing hypoglycemia? (AIIMS May 2014)
a. A baby born to mother treated with beta blockers 114. Jaundice at birth or within 24 hours of birth is most
b. Infant of diabetic mothers commonly due to: (FMGE June 2018)
c. Appropriate for gestational age babies a. Erythroblastosis fetalis
d. IUGR babies
b. Congenital hyperbilirubinemia
103. All are features of infant born to diabetic mothers except: c. Biliary atresia
 (MAHA PGM CET 2014)
d. Physiological jaundice of newborn
a. Obesity b. Learning disability 115. MC cause of cholestatic jaundice in newborn is: 
c. Ketotic hypoglycemia d. Future diabetes mellitus  (FMGE June 2018)
104. The cut-off for neonatal hypoglycemia is blood glucose
a. Hypoplasia of biliary tract
less than _____ mg/dl: (WB PGMEE 2010)
b. Neonatal hepatitis
a. 30 b. 35
c. Choledochal cyst d. Physiological
c. 45 d. 40
116. A 4-day-old breastfed, otherwise well, term neonate
105. Infant of diabetic mother with weight 3.8 Kg presented
presented with jaundice, on testing the bilirubin level
with seizures after 16 hours of birth. What is the most
probable cause? (AIPGMEE 11, AIIMS Nov 09)
was found to be 18 mg/dl. Which of the following is the
best step of management? (AIIMS Nov 2017)
a. Hypoglycemia b. Hypocalcemia
c. Birth asphyxia a. Start phototherapy and continue breastfeeding
d. Intraventricular hemorrhage b. Start iv fluids and given phototherapy
106. Infants of diabetic mother manifests which of the c. Initiate exchange transfusion
following? (PGI Dec 07) d. Stop breastfeeding and do phototherapy
a. Hyperglycemia b. Hypoglycemia 117. A 3 days old neonate is brought by the mother to the
c. Hypocalcemia d. Increased fetal defect pediatrician and is found to have a bilirubin of 18 g/dl.
e. Hyperbilirubinemia What is the next step? (AIIMS Nov 2017)
107. All of the following are the complications in the new a. Stop breastfeeding
born of a diabetic mother except: M  (AIIMS May 06) b. Start phototherapy and continue breastfeeding
a. Hyperbilirubinemia b. Hyperglycemia c. Start phototherapy and stop breastfeeding
c. Hypocalcemia d. Hypomagnesemia d. Exchange transfusion
108. Which of the following malformation in a newborn 118. All of the following are true about breast milk jaundice
is specific for maternal insulin dependent diabetes except? (PGI Dec 2016)
mellitus? (AIPGMEE 06) a. Bilirubin is 10-15 mg/dl
a. Transposition of great arteries b. Occurs between day 5-10 of life
b. Caudal regression c. Phototherapy is the treatment of choice
c. Holoprosencephaly d. Meningmyelocele d. Staining of Diaper by urine is present
109. Hypoglycemia in an infant is defined as blood glucose e. Breastfeeding should be stopped to treat it
level less than: M  (DNB June 2006) 119. A newborn presents with jaundice within 1st 24
a. 40 mg/dl b. 60 mg/dl hours. His mother’s blood group is O +ve. Next line of
c. 10 mg/dl d. 20 mg/dl management is? (AIIMS Nov 2016)
110. Administration of glucose solution is prescribed for all of a. Wait for serum bilirubin level before starting treatment
the following situations except: (AIIMS May 06) b. Phototherapy
a. A normal term neonates c. Exchange transfusion d. Stop breastfeeding
b. Infant of a diabetic mother 120. Which of the following is/are true regarding conjugated
c. History of unconsciousness bilirubin in newborn? (PGI May 2016)
d. History of hypoglycemia
a. Direct bilirubin fraction > 2 mg/dL at any time is
111. A 4 kg baby born to a diabetic mother was found to have
pathological
hypoglycemia on routine blood glucose monitoring.
b. Neonatal cholestasis is elevation of conjugated bilirubin
Which of the following is to be done? (PGI Dec 03)
beyond the 1st 14 days of life
a. Reassess the baby again after 2 hours c. Biliary atresia is cause of raised levels of conjugated
b. Give 10% dextrose IV bilirubin
c. Start oral feeding d. Give injection insulin
d. Unconjugated hyperbilirubinemia can be physiologic
e. Give injection glucagon
e. Acholuric stools indicates conjugated hyperbilirubi­nemia
112. Infants of diabetic mother have which of the following:
121. For Double volume exchange transfusion, which of the
 (PGI June 03)
following is to be used? (JIPMER May 2016)
a. Macrosomia b. Neural tube defect
c. Hyperglycemia d. Hypocalcemia a. Plasma alone b. Packed RBCs
e. Hypoglycemia c. RBCs in plasma d. None of these
113. All of the following groups of newborns are at an 122. A 4-day-old newborn baby is having icterus involving
increased risk of hypoglycemia except: M  sole; Approximate bilirubin level (mg/dl) is: M 
 (Recent Question 2016)
a. Birth asphyxia  (AIIMS Nov 02)
b. Respiratory distress syndrome a. 12 b. 15
c. Maternal diabetes d. Postterm infant c. 8 d. 6
262 Section 2: Neonatology

c. DCT positive & History of previous sibling affected


Review of Pediatrics and Neonatology

123. Cause of jaundice in day 1 of a baby are all except:


 (Recent Question 2016) d. Hydrops fetalis
135. Abnormality in Crigler-Najjar syndrome: (JIPMER 2012)
a. Rh incompatibility b. ABO incompatibility
c. Prematurity d. Breast milk jaundice a. Uptake of bilirubin
124. Which mechanism in phototherapy is chiefly responsible b. Defect in hepatic conjugation
c. Canalicular Release d. Reduced bile excretion
for reduction in serum bilirubin: M 
 (Recent Question 2016, AIIMS May 05) 136. If Neonatal Jaundice appears for the first time in the 2nd
week. Not a cause is: (DNB June 2011, DNB June 2010)
a. Photo-oxidation b. Photo-isomerization
a. Galactosemia b. Rh Incompatibility
c. Structural isomerization d. Conjugation
c. Hypothyroidism d. Breast milk Jaundice
125. Phototherapy is used in the treatment of? M 
137. Medical management of kernicterus is: (WBPG 2011)
 (Recent Question 2016)
a. Sulfonamide b. Erythromycin
a. Necrotising enterocolitis b. Polycythemia
c. IVIg d. Phenytoin
c. Neonatal hypocalcemia d. Neonatal jaundice
138. Conjugated hyperbilirubinemia in infancy is seen in:
126. Conjugated hyperbilirubinemia is seen in  (PGI Nov 10, June 09, Dec 06)
 (PGI Nov 2015)
a. Gilbert syndrome b. Crigler-Najjar syndrome
a. Rotor's syndrome b. Dubin Johnson syndrome c. Dubin Johnson syndrome d. Rotor syndrome
c. Gilbert's syndrome d. Breast milk jaundice e. Neonatal hepatitis
e. Crigler-Najjar syndrome 139. Peripheral smear of neonate with ABO incompatibility
127. 1 month old newborn presents with conjugated Hyper­ will show: (AIIMS Nov 10)
bilirubinemia, intrahepatic cholestasis and high alkaline a. Microspherocytes b. Elliptocytes
phosphatase. Eosinophilic, PAS positive cytoplasmic c. Fragmented RBCs d. Polychromasia
granules are seen in liver biopsy specimen. Diagnosis 140. A case of jaundice with 50% direct bilirubin, other LFTs
is? (AIIMS May 2015) normal. Diagnosis is: (AIIMS Nov 09)
a. Alpha1 AT deficiency b. Cong hepatic fibrosis a. Rotor syndrome
c. Extrahepatic biliary atresia b. Gilbert syndrome
d. Ductal plate defect c. Glucuronyl transferase deficiency
128. Which of the following characterizes pathological d. Primary biliary cirrhosis
jaundice? (TN PGMEE 2015) 141. The late features of kernicterus include all except:
a. Jaundice on 3rd day b. Jaundice upto the trunk a. Hypotonia  (DNB June 2009)
c. High colored urine b. Sensorineural hearing loss
d. Jaundice accompanied by yellow stools c. Choreoathetosis
129. Which of the following causes neonatal jaundice? d. Upward gaze palsy
 (PGI May 2015) 142. Bronze baby syndrome is caused by: M  (TN PGMEE 2008)
a. Sickle cell anemia b. β–Thalassemia a. Phototherapy b. Phenobarbitone
c. Meningitis d. G6PD deficiency c. Hemosiderosis d. Chloramphenicol
e. Rh incompatibility 143. Unconjugated hyperbilirubinemia is seen in:
 (PGI Dec 07)
130. Phototherapy is used in: M  (Recent Question 2014)
a. Physiological jaundice b. Breast milk jaundice
a. Neonatal jaundice b. Neonatal meningitis c. Gilbert syndrome d. Biliary atresia
c. Neonatal tetanus d. HIE e. Rotor syndrome
131. All affect effectiveness of phototherapy except: 144. A term neonate with unconjugated hyperbilirubinemia
a. Skin pigmentation  (JIPMER 2014) of 18 mg/dl on 20th day. All are common causes except:
b. Initial level of serum bilirubin a. Breast milk jaundice  (AIIMS May 07)
c. Spectral irradiance b. Congential cholangiopathy
d. Type of lamp used c. G 6PD deficiency d. Hypothyroidism
132. A 32 weeks premature infant, 900 gm weight presents on 145. A 50-hour old full-term breast-fed newborn boy weighing
the third day with jaundice. The serum bilirubin is 15 mg%. 3100 g presents with clinically evident jaundice. Physical
The treatment of choice is: (Recent Question 2013) examination is otherwise normal. The total bilirubin is
11.0 mg/dl with a direct bilirubin of 0.4 mg/dl. What
a. Exchange transfusion b. Phototherapy
would be the correct treatment? (DNB Dec 07)
c. Wait and watch therapy d. Pharmacologic therapy
a. Continue breastfeeds and review after 24 hours
133. One gm of Hb liberates how many mg of bilirubin? M 
b. Stop breastfeeds and review after 24 hours
 (Recent Question 2013)
c. Continue breastfeeds and start blue-light phototherapy
a. 40 b. 34 d. Often become constipated
c. 15 d. 55 146. A newborn presented with jaundice. Most diagnostic
134. In Rh Isoimmunization, exchange transfusion is indicated investigation of choice is:
in all of the following except: (Recent Question 2013)  (DNB Dec 06, AIPGMEE 2000)
a. Cord blood hemoglobin < 10 g/dl a. Total and Direct bilirubin b. Conjugated bilirubin
b. Cord bilirubin is more than 5 mg/dl c. Serum, bilirubin d. Uroporphyrin levels
Chapter 12: Diseases of Newborn 263

147. In unconjugated hyperbilirubinemia, the risk of a. Intusussception b. Necrotising enterocolitis

Questions
kernicterus increases with the use of: (AIPGMEE 05) c. Volvulus d. Hirschsprung disease
a. Ceftriaxone b. Phenobarbitone
c. Ampicillin d. Sulphonamide  MISCELLANEOUS
148. Hyperbilirubinemia in a child can be due to all of the
following except: (MAHA PG 05) 158. Craniopagus is defined as fusion of: (AIIMS May 2018)
a. Breast milk jaundice b. Cystic fibrosis a. Head and spine b. Head only
c. Fanconi's syndrome d. a-1 antitrypsin deficiency c. Thorax and spine d. Thorax only
149. A term baby developed jaundice on 3rd day upto the 159. Twin within 1-3 days of zygote formation:
thigh with normal stool and urine. Mother’s blood group a. Diamniotic dichorionic (FMGE Dec 2018)
is ‘O’ –ve and that of babies ‘A’ +ve. The cause of b. Diamniotic monochorionic
jaundice is (PGI Dec 04, Dec 02) c. Monoamniotic monochorionic
a. Rh incompatibility b. Physiological jaundice d. None of the above
c. Extrahepatic biliary atresia 160. Hyperglycemia in Neonate is blood glucose above? M
d. Sepsis  (Recent Question 2017)
e. Glucose-6 phosphate dehydrogenase deficiency
a. 150 mg/dL b. 125 mg/dL
150. Conjugated hyperbilirubinemia in infancy seen in:
c. 180 mg/dL d. 100 mg/dL
 (PGI Dec 04)
161. Which of the following does not describe a “high risk"
a. Choledochal cyst b. Extrahepatic biliary atresia infant? (Recent Question 2017)
c. Crigler-Najjar disease d. Gilbert disease
a. Third baby b. Birth weight less than 2.5
e. Rotor syndrome
kg
151. True about physiological Jaundice: (PGI June 03)
c. Premature neonate d. Twins
a. Jaundice appear within first 24 hour 162. Which is not a cause of non-immune hydrops fetalis:
b. Jaundice disappear in 3rd week  (Recent Question 2017)
c. Sudden rise of bilirubin
d. Jaundice may be seen on day 5 of life a. α-thalassemia b. Congenital rubella
e. Breastfeeding should be stopped c. Rh incompatibility d. Cystic hygroma
152. A child has total bilirubin of 4 mg/dl. Conjugated bilirubin 163. A 3-week-old child presents with an abdominal mass.
and alkaline phosphatase are normal, bile salts and bile What is the most common cause of the presentation?
in urine are absent. However urobilinogen in urine is  (Recent Question 2017)
raised. What is the likely diagnosis? (AIIMS Nov 01) a. Neuroblastoma b. Wilms tumor
a. Obstructive jaundice b. Rotor's syndrome c. Multicystic dysplastic kidney
c. Biliary cholestasis d. Hemolytic jaundice d. Distended bladder
153. True about physiological jaundice in neonates: 164. Which one of the following does NOT present with a
a. Occurs in first 6 hours of delivery (PGI Dec 2000) large anterior fontanelle? (APPG 2016)
b. Unconjugated hyperbilirubinemia a. Crouzon syndrome b. Hypophosphatasia
c. Neurological sequelae are common c. Achondroplasia
d. Best treated by phototherapy d. Congenital rubella syndrome
e. Starts on 2nd day of life 165. A neonate while suckling milk can respire without
difficulty due to (MAHA PGM CET 2016)
 NECROTIZING ENTEROCOLITIS
a. Stout soft palate b. High larynx
154. Gas in portal veins is seen in c. Small tongue d. Small arytenoides
 (Recent Question 2016, WB PGMEE 2010) 166. Normal level of Hb in a newborn (at 3 days):
 (TN PGMEE 2015)
a. Necrotizing enterocolitis b. Clostridium sepsis
c. Gas gangrene d. Septicemia a. 16.5 g/dL b. 18.5 g/dL
155. Indication for surgery in Necrotizing Enterocolitis is? c. 11 g/dL d. 10 g/dL
 (Recent Question 2015) 167. Monochorionic-Monoamniotic twin occurs if division
a. Pneumatosis intestinalis b. Pneumoperitoneum occurs? (MAHA PGM CET 2015)
c. Portal vein gas d. Peritonitis a. < 24 hrs b. 1-4 days
c. 4-8 days d. > 8 days
156. A very preterm baby on 30 mL/kg of enteral feeding 168. Two substances are needed for a developing fetus to
developed sudden severe abdominal distension with grow male genitalia? (MAHA PGM CET 2015)
visible bowel loops on day 6 of life. The baby also
showed temperature instability and lethargy. X-ray of a. Wolffian factor and Mullerian inhibitory factor
the abdomen showed portal venous gas. The staging of b. Testosterone and Mullerian inhibitory factor
NEC is? (AIIMS May 2014) c. Wolffian factor and testosterone
d. Mullerian factor and testosterone
a. I b. 2a
c. 2b d. 3a 169. Most common type of phagus in twins?
 (Recent Question 2015)
157. A preterm baby presents with feed intolerance,
abdominal distension and bloody stool on day 4 of life. a. Craniophagus b. Pygophagus
Most likely diagnosis is: (WB PGMEE 2010) c. Thorachophagus d. Ischiophagus
264 Section 2: Neonatology

170. Growing fetus derived energy from: (WBPG 2014) 179. Test used to differentiate maternal from fetal blood:
Review of Pediatrics and Neonatology

a. Amino acid b. Carbohydrate  (AIIMS Nov 10)


c. Lipid d. Minerals a. Osmotic fragility test b. Water bulb test
171. A neonate presents with bilious vomiting. What is the c. Coomb's test d. Kleihauer Betke test
first investigation? (AIIMS Nov 2014) 180. A newborn with eyes closed 6 hrs after birth is lustily crying;
a. Chest X-ray b. Baby –gram There are no chest retraction & movements of all four limbs
are present. Neonatal behavioral response grading.
c. CT d. USG
172. Percentage of sucrose solution used for newborn a. State 1 b. State 3 (AIIMS Nov 10)
analgesia is: (JIPMER 2014)
c. State 5 d. State 6
181. Which of the following is not true about late onset
a. 5% b. 10%
Hemorrhagic disease of newborn (HDN)?
c. 24% d. 50%  (MAHA PGM CET 10)
173. When mother complains that baby is more comfortable
a. Begins between 2–7 days of life
and breaths better when held against shoulder. What
b. Intracranial hemorrhage is common
important clinical problem do you suspect? c. Biliary atresia can predispose
 (APPG 2014) d. Warfarin therapy is associated
a. Nose block 182. Hydrops fetalis may be caused by the following, except:
b. Psychological anxiety or insecurity  (COMED 09)
c. Aerophagy with abdominal fullness a. Congenital heart block b. Cystic hygroma
d. Orthopnea c. Congenital varicella syndrome
174. Most common cause of abdominal mass in neonates is? M d. Congenital nephrosis
 (Recent Question 2014) 183. Fetal alcohol syndrome is characterised by all except:
a. Neuroblastoma  (AIIMS Nov 2009)
b. Multicystic dysplastic kidney a. Microcephaly b. Low intelligence
c. Polycystic kidney c. Large proportionate body
d. Umbilical hernia d. Septal defects of heart
175. A neonate at 48 hours of birth with a history of non- 184. A newborn presented with microcephaly, ASD and epi­
passage of meconium. Next step in evaluation will be? canthal folds. Diagnosis: (TN PGMEE 2009)
 (AIIMS Nov 2014) a. Fetal hydantoin syndrome
a. Lower GI study b. Manometry b. Fetal alcohol syndrome
c. Sweat chloride levels d. CFTR mutation analysis c. Warfarin teratogenecity
d. Valproate teratogenecity
176. Non-immune fetal hydrops caused by all except:
 (JIPMER 2013) 185. Breastfeeding for a newborn to be started:
 DNB Dec 2006)
a. Parvovirus B19 infection
b. Rh incompatibility a. As soon as possible b. After 6 hours
c. After 1 day d. After meconium has passed
c. Congenital heart block
d. Twin-twin transfusion syndrome 186. An 8-day-old neonate presents with extensor posture.
177. Which sinus will be absent in a newborn? Most probable diagnosis is? (DNB June 2000)
 (JIPMER 2012) a. Meningitis b. Cerebral palsy
c. Neonatal tetanus
a. Frontal sinus b. Maxillary
d. Hypoxic ischemic encephalopathy
c. Anterior ethmoidal d. Posterior ethmoidal
187. A 3-day-old infant brought with excessive cry, suckling
178. Best screening test to evaluate hearing in a neonate:
difficulty, umbilical sepsis and generalized stiffness.
a. Pure tone audiometry  (AIIMS Nov 2012) Probable diagnosis (DNB May 2000)
b. Stapedial reflex
a. Neonatal sepsis b. Cerebral palsy
c. Oto-acoustic emissions c. Neonatal tetanus
d. Brainstem evoked auditory response d. Hypoxic ischemic encephalopathy
Chapter 12: Diseases of Newborn 265

Answers with Explanations


Answers with Explanations
 INTRAUTERINE GROWTH RESTRICTION
1. b. Ponderal index above 10th percentile  Ref: Nelson’s 20/e p 821-829, Ghai 8/e p 155-157
Discussing about the options one by one,
About option a. True, because, a fetus with asymmetric IUGR has a normal head dimension but a small abdominal circumference (due
to decreased liver size); so HC/AC ratio increased
About option b. False, as Ponderal index should be less than 10th percentile & not more in IUGR
About option c. True, Increased diastolic velocity in Middle cerebral artery (MCA) doppler is seen in IUGR
About option d. True, Femur length/ Abdominal circumference (FL/AC) ratio is >23.5 in IUGR

2. d. Brain  Ref: Nelson’s 20/e p 821-829, Ghai 8/e p 155-157


In Asymmetric IUGR, Brain growth (head circumference) is usually spared & it is usually due to Medical/obstetric problem of mother
in late trimesters like poor nutrition, hypertension, placental insufficiency
3. d. Placental insufficiency  Ref: Nelson’s 20/e p 821-829, Ghai 8/e p 155-157
4. a. Diabetes  Ref: Nelson’s 20/e p 821-829, Ghai 8/e p 155-157;
Maternal Diabetes usually leads to a LFD baby & not IUGR
5. a. 1.6  Ref: Nelson’s 20/e p 821-829, Ghai 8/e p 155-157
Ponderal Index (PI) in Neonate = Weight (gm)/ Length (cm)3 × 100 ⇒ In this case, PI = 2000/(50)3 × 100 = 1.6
6. c. Intracranial bleed  Ref: Nelson’s 20/e p 821-829, Ghai 8/e p 155-157
7. a. Hypoglycemia  Ref: Nelson’s 20/e p 821-829, Ghai 8/e p 155-157; Refer pretext for details;

 NEONATAL SEPSIS
8. b. Listeria monocytogenes  Ref: Nelson 20/e p 1350
Most common cause of Acute Bacterial Meningitis in:
Age group Organism
Neonates Gr B Streptococcus, E. coli, Listeria
Infants and Children S. pneumoniae, N. meningitidis, H. influenzae
Adolescents and Young Adults S. pneumoniae, N. meningitidis

Neonatal Listeriosis
Two clinical presentations are recognized:
Early onset (<5 days) Late onset (≥5 days)
Positive result of maternal listeria culture Negative results of maternal listeria culture
Obstetric complications Uncomplicated pregnancy
Premature delivery Term delivery
Low birthweight Normal birthweight
Neonatal sepsis Neonatal meningitis
Mean age at onset 1.5 days Mean age at onset 14.2 days
Mortality rate >30% Mortality rate <10%
Nosocomial outbreaks

9. a. Klebsiella pneumoniae  Ref: Lancet Glob Health 2016; 4: e752–60


Acinetobactes is not there in options. Klebsiella, the second most common organism is the answer here.
10. a. Acinetobacter  Ref: Lancet Glob Health 2016; 4: e752–60
As per the latest evidence, most common cause of Neonatal sepsis in India is Acinetobacter
11. a. Meningitis  Ref: AIIMS NICU protocol, 2014
12. c. Prematurity  Ref: Ghai 9/e p 125
13. a. Neonatal sepsis  Ref: Nelson’s 20/e p 832, 915
In the given scenario, the baby has excessive weight loss (> 10% of birth weight), features of dehydration (like depressed fontanelle,
sunken eyes, decreased urine output and decreased skin turgor) along with poor feeding & poor activity.
266 Section 2: Neonatology

Failure to feed properly is seen in most sick newborn infants and should lead to a careful search for infection (sepsis), a central or
Review of Pediatrics and Neonatology

peripheral nervous system disorder, intestinal obstruction, and other abnormal conditions.
In ‘Dehydration fever’: Fever usually results from too high environmental temperature because of weather, overheated nurseries or
incubators/radiant warmers, or too many clothes in newborn infants.
14. c. Band cell count  Ref: AIIMS NICU protocol, 2014; For details about sepsis screen, refer pretext
15. d. Klebsiella  Ref: AIIMS NICU protocol, 2014
Among the given options, the best ans is Klebsiella, but as per a recent multicentric study from India (Lancet, 2016), Most
common organism responsible for Neonatal sepsis in India is Acinetobacter
16. a. Refusal to suck, poor cry and lethargy are common features  Ref: Nelson’s 20/e p 918-922
Discussing the options one by one,
a. True; These are the earliest and very common features of neonatal sepsis
b. False, Neutropenia & not neutrophilia is a feature of neonatal sepsis
c. False, Sepsis can occur anytime in the neonatal period
d. False, Empirical antibiotics should be started if neonatal sepsis is suspected, even before culture- sensitivity reports are available

17. a. Klebsiella  Ref: Nelson’s 20/e p 918-922, Ghai 8/e p 163-165


18. a. S. agalactiae  Ref: Nelson’s 20/e p 918-922, Ghai 8/e p 163-165
Group B Streptococcus (S. agalactiae) cause neonatal sepsis & meningitis.
19. d. Candida parapsilosis  Ref: Nelson’s 20/e p 918-922
20. a. N.meningitidis  Ref: Nelson’s 20/e p 918-922, Ghai 8/e p 163-165
21. b. Klebsiella  Ref: Nelson’s 20/e p 918-922, Ghai 8/e p 163-165
22. b. Streptococcus agalactiae  Ref: Nelson’s 20/e p 918-922, Ghai 8/e
23. a. Meningitis commonly occur lately and d. Jaundice is a common feature  Ref: AIIMS NICU protocol, 2014
Hypothermia is more common than fever in neonatal sepsis; Leukopenia is seen in neonatal sepsis & not leukocytosis.

 HYPOTHERMIA
24. a. Head  Ref: AIIMS NICU protocol on Neonatal hypothermia
25. a. Thermoneutral environment  Ref: AIIMS NICU protocol 2014
26. d. 36 to 36.4 deg C  Ref: AIIMS NICU protocol 2014
27. c. 22-26ºC  Ref: Nelson’s 20/e p 800
28. b. Mesentery  Ref: Nelson’s 20/e 830-832; Care of the Newborn by David E. Hertz, pg 43
•• Neonates are incapable of shivering & rely on non-shivering thermogenesis, due to brown fat for heat.
•• Brown fat is located around great vessels, kidneys, adrenals, axillae, nape of neck, & between scapulae.
29. b. 22–26ºC  Ref: Nelson’s 20/e p 800
The nursery temperature should be kept between 22-26oC (72-78°F).
30. a. Shivering  Ref: Nelson’s 20/e p 830-832, Ghai 8/e p 144-146
31. a. Apnea b. Hypoglycemia, e. Hypoxia  Ref: Nelson’s 20/e p 918-922, Ghai 8/e p 144-146
32. c. Convection  Ref: Nelson’s 20/e p 154, Ghai 8/e p 144-146; Refer pretext of this chapter for details
33. b. Hypoglycemia and c. Apnea  Ref: Nelson’s 20/e p 830-832; Ghai 8/e p 144-146

 PERINATAL ASPHYXIA
34. b. Moderate asphyxia  Ref: NNF & WHO guidelines
Apgar score Interpretation
0-3 Severe birth asphyxia
4-7 Moderate birth asphyxia
>7 Normal
35. d. Appearance, pulse, grimace, activity, respiration  Ref: Ghai 8/e p 166-168
36. a. Neurological damage, b. Subcutaneous fat necrosis, c. Pulmonary hypertension  Ref: Nelson’s 20/e p 838-840
Effects of Asphyxia in a neonate:
•• CNS: HIE, infarction, intracranial hemorrhage, seizures, tone abnormalities;
•• Cardiovascular: Myocardial ischemia, poor contractility, hypotension
•• Pulmonary: Pulmonary hypertension, pulmonary hemorrhage, RDS
•• Renal: Acute tubular or cortical necrosis
Chapter 12: Diseases of Newborn 267

•• Adrenal: Adrenal hemorrhage

Answers with Explanations


•• Gastrointestinal: Perforation, ulceration with hemorrhage, necrosis
•• Metabolic: SIADH, hyponatremia, hypoglycemia, hypocalcemia, myoglobinuria
•• Skin: Subcutaneous fat necrosis
•• Hematology: Disseminated intravascular coagulation.
37. b. Severely depressed baby  Ref: NNF guidelines on birth asphyxia. Apgar score of 0-3 indicates a severely depressed neonate.
38. b. Increased risk of neonatal mortality  Ref: Nelson's 20/e p 798
Apgar score is normal in most patients in whom cerebral palsy subsequently develops.
Low Apgar scores (0-3 at 5 min) is a predictor of neonatal death, in both term and preterm neonates
39. b. Appearance, Pulse, Grimace, Activity, Respiration  Ref: Nelson's 20/e p 798
APGAR score is based on 5 parameters: Appearance, Pulse, Grimace, Activity, Respiration
40. d. 3  Ref: Nelson’s 20/e p 798, Ghai 8/e p 166-168
Appearance: completely pale, so score of 0, Pulse rate 40/minute → score 1, Grimaces → 1, Activity → 0 (as no movements),
Respiratory effort → 1 (as gasping); So, the APGAR score for this baby is: 0 + 1 + 1 + 0 + 1 = 3
41. b. 7  Ref: Nelson’s 20/e p 798, Ghai 8/e p 166-168
Appearance: peripheral cyanosis, so score of 1, Pulse rate 120/minute → score 2, Grimaces → 1, Activity → 1 (flexed
posture, but no movements), Respiratory effort → 2 (as strong respiratory efforts present); So, the APGAR score for this
baby is: 1 + 2 + 1 + 1 + 2 = 7
42. b. Respiratory rate  Ref: Nelson’s 20/e p 798, Ghai 8/e p 166-168
Respiratory effort is included in Apgar score & not respiratory rate; For details, refer to pretext of this chapter
43. b. 2  Ref: Nelson’s 20/e p 838-840, Ghai 8/e p 166-168
Seizures are a characteristic of stage 2 HIE; For details about staging of HIE, refer pretext of this chapter
44. b. 1  Ref: Nelson’s 20/e p 838-840, Ghai 8/e p 166-168
45. b. Stage 2  Ref: Nelson’s 20/e p 838-840, Ghai 8/e p 166-168
46. c. Kernicterus  Ref: Nelson’s 20/e p 838-840, Ghai 8/e p 166-168
Discussing the options one by one,
a. HIE CNS changes due to perinatal asphyxia are known as Hypoxic ischemic encephalopathy (HIE)
b. MAS Perinatal asphyxia & fetal distress can lead to passage of meconium in utero giving rise to MAS
c. Kernicterus Perinatal asphyxia does not predispose to neonatal jaundice or kernicterus
d. NEC Perinatal asphyxia & hypoxia can lead to intestinal necrosis & perforation, which manifests as NEC

47. d. Mode of delivery  Ref: Nelson’s 20/e p 798, Ghai 8/e p 166-168
A low Apgar score may be the result of fetal distress, prematurity & drugs given to the mother during labor.
48. c. Seizure > b. Prox. Muscle > distal muscles  Ref: Nelson’s 20/e p 839, Ghai 8/e p 166-168
This is a controversial question: Discussing about each option,
a. In term babies, upper limbs are more affected than lower while in preterms, lower limbs are more affected
b. Proximal muscle weakness is seen in parasagittal injury, commonly seen in term babies
c. Seizures are seen in 20-50% of cases (in both, term & preterm neonates)
d. Limbs are mainly involved; Hence, seizures can be seen in both term & preterm babies with HIE
49. a. Birth asphyxia  Ref: Nelson’s 20/e p 838-840, Ghai 8/e p 166-168
50. b. Dexamethasone  Ref: Nelson’s 20/e p 838-840, Ghai 8/e p 166-168
The management protocol of babies with perinatal asphyxia includes:
•• Oxygen supplementation, thermal control, monitor blood glucose & maintain euglycemia
•• Fluid balance: Give dextrose containing IV fluids at 2/3 maintenance. For shock: Normal saline boluses Inotropes
•• Prevent/control convulsions: PhenobarbitoneQ is the drug of choice of neonatal seizures
•• Treat hypocalcemia, if it occurs calcium gluconate administration.

 NEONATAL SEIZURES
51. d. Myoclonic  Ref: AIIMS NICU Protocol 2014
52. c. Subtle  Ref: Nelson’s 20/e p 2849-2853, Ghai 8/e p 168

High Yield Points


•• Neonatal Seizures •  Type with best prognosis: Focal clonic seizures
•• Most common type is Subtle seizures •  Neonatal seizures with worst prognosis: Myoclonic seizures
•• Least common type is Myoclonic seizures •  CNS imaging of choice: Transcranial ultrasound
• Most common cause is Hypoxia •  Drug of choice: Phenobarbitone
268 Section 2: Neonatology

53. a. Transcranial ultrasound  Ref: Nelson’s 20/e p 2849-2853


Review of Pediatrics and Neonatology

54. c. Focal clonic  Ref: Nelson’s 20/e p 2849-2853


55. c. Phenobarbitone  Ref: Nelson’s 20/e p 2849-2853, Ghai 8/e p 168
56. a. Pyridoxine  Ref: Nelson’s 20/e p 2849-2853
Pyridoxine deficiency is one of the causes of refractory seizures in the neonatal period & infancy.
57. a. Hypoxia induced ischemic encephalopathy  Ref: Nelson’s 20/e p 2849-2853, Ghai 8/e p 168
58. c. Subtle  Ref: Nelson’s 20/e p 2849-2853, Ghai 8/e p 168
59. b. Frequency of movement  Ref: Nelson’s 20/e p 2849-2853
Differences between jitteriness & seizures:

Feature Jitteriness Seizures


Ocular deviation Not seen Ocular deviation seen
Stimulus sensitive Yes, It stops with passive movement of limb It is not stimulus sensitive
Autonomic changes No autonomic changes seen Usually seen

 MECONIUM ASPIRATION SYNDROME


60. b. Suctioning in vigorous and non-vigorous babies  Ref: NRP 2015
As per the latest guidelines, suctioning of trachea is not required for all babies with Meconium stained liquor (refer pretext)
61. b. Obstructive emphysema  Ref: Nelson’s 20/e p 859-860, Ghai 8/e p 170
62. b. Colour  Ref: Nelson’s 20/e p 859-860, Ghai 8/e p 126-133; Refer pretext for details;
63. c. Bag and mask ventilation  Ref: Nelson’s 20/e p 859-860, Ghai 8/e p 126-133
64. a. Transient tachypnea of newborn & b. Meconium aspiration syndrome  Ref: Nelson’s 20/e p859-860,
Meconium stained liquor with respiratory distress soon after birth suggests the diagnosis of meconium aspiration syndrome
Transient tachypnea of newborn is also a possibility as the baby was delivered by caesarean section. In a baby with severe
respiratory distress, oral feeding should be witheld till the respiratory distress improves;

 RESPIRATORY DISTRESS SYNDROME (RDS)


65. d. 8
Ref: Silverman scoring is calculated using the following 5 parameters, each of which can be given a score of 0, 1, 2.

In the given scenario, score for paradoxical breathing (2), marked lower chest & xiphisternal retractions (2+ 2), minimal nasal flare
(1), expiratory grunt audible only with stethoscope (1), give a total score of 8.
Score > 6 indicate impending respiratory failure.
Chapter 12: Diseases of Newborn 269

66. b. Surfactant administration and respiratory support  Ref: Nelson's 20/e p 850–57

Answers with Explanations


The given scenario indicates a diagnosis of moderate – severe respiratory distress syndrome. So, the treatment of choice is: surfactant
administration and respiratory support
67. d. Surfactant  Ref: Nelson's 20/e p 854
Systemic or inhaled corticosteroids have been used, mainly in those infants with RDS, who continue to require respiratory support &
in whom bronchopulmonary dysplasia (BPD) develops.
Routine use of corticosteroids for the prevention or treatment of BPD is not recommended by the American Academy of Pediatrics.
68. a. Hyaline membrane disease  Ref: Nelson’s 20/e p 850-857, Ghai 8/e p 169-170
69. b. Dipalmitoyl phosphatidyl choline  Ref: Ghai 8/e p 169-170
Dipalmitoyl phosphatidyl choline is an important constituent of Surfactant, which prevent alveolar collapse during expiration.
70. b. 28 wk  Ref: Nelson’s 20/e p 850-857, Ghai 8/e p 169-170
Surfactant synthesis begins in fetal lungs at 20 week & appears in amniotic fluid between 28-32 weeks.
71. c. Mucus plugs in the bronchioles  Ref: Nelson’s 20/e p 850-857, Ghai 8/e p 169-170
72. a. Pulmonary alveolar proteinosis  Ref: Nelson’s 20/e p 850-857; For details, refer pretext of this chapter;
73. d. Amniotic fluid phosphatidyl choline levels is lower than normal  Ref: Ghai 8/e p 169-170
Respiratory distress syndrome is due to deficiency of surfactant, of which phosphatidyl choline is an important constituent.
74. a. Lung  Ref: Nelson’s 20/e p 850-857, Ghai 8/e p 169-170
Lecithin/Sphingomyelin ratio of > 2 suggests lung surfactant maturity.
75. b. Diabetes  Ref: Nelson’s 20/e p 850-857, Ghai 8/e p 169-170
76. b. Air bronchogram on chest X-ray  Ref: Nelson’s 20/e p Ghai 8/e p 169-170
Chest X-ray findings in RDS include atelectasis (lung collapse), diffuse reticulogranular or reticulonodular pattern commonly
referred to as “ground glass”, presence of air bronchograms & complete “white out” (severe disease cases)
77. d. Lung maturity  Ref: Nelson’s 20/e p 850-857, Ghai 8/e p 169-170
78. c. Neonatal pulmonary alveolar proteinosis  Ref: Nelson’s 20/e p 850-857
Important clues in this question are: a term neonate with respiratory distress, who also has a family history of similar illness
in a sibling who died, along with Chest X-ray showing ground glass opacity & Surfactant not being effective in treating this
condition. All these point towards a diagnosis of Neonatal pulmonary alveolar proteinosis.
Note: Normal Echocardiogram rules out obstructed TAPVC, which is a close differential diagnosis.
79. a. Type 2 pneumocytes  Ref: Nelson’s 20/e p 850-857, Ghai 8/e p 169-170
80. b. RDS  Ref: Nelson’s 20/e p 850-857, Ghai 8/e p 169-170
81. a. CPAP  Ref: Nelson’s 20/e p 850-857, Ghai 8/e p 169-170
In at-risk neonates for respiratory distress syndrome, CPAP could be started at the earliest sign of respiratory distress;
If one of the options would have been surfactant or surfactant with CPAP, that would have been a better choice... but the option in
this question is surfactant & ventilation, that is why D is not the correct ans..!
High Yield Points about CPAP (Continuous positive airway pressure)
•• CPAP is indicated in spontaneously breathing premature infant with respiratory distress (tachypnea, retractions or grunt) &/or recurrent
apneas not responding to medical management
•• CPAP re-expands collapsed alveoli, splints airway, reduces work of breathing & improves the pattern of respiration
•• Atelecto-trauma, biotrauma & volutrauma, the key determinants of VILI (ventilator induced lung injury) are minimal or absent with CPAP.

82. a. FRC is smaller than closing volume  Ref: Nelson’s 20/e p 850-857
HMD reduces lung compliance & causes a fall in FRC below the normal closing volume of lung, so that the alveoli close.
83. b. Air bronchogram in chest X-ray  Ref: Nelson’s 20/e p 850-857, Ghai 8/e p 169-170
Discussing about the options one by one,
a. Receipt of antenatal steroids No Antenatal steroids protect against RDS
b. Air bronchogram in chest X-ray Yes Air bronchogram on Chest X-ray is a feature of RDS
c. Manifests after 6 hrs No RDS manifests soon after birth & not after 6 hours
d. Occurs after term gestation No It usually occurs in preterm neonates

84. b. 12 mg every 24 hours  Ref: Nelson’s 20/e p 850-857, Ghai 8/e p 169-170
•• Dose of Inj Betamethasone is 12 mg IM 2 doses, 24 hrs apart
•• Antenatal steroids should be given to all mother at risk of preterm delivery between 24 & 34 weeks of gestation.
270 Section 2: Neonatology

85. d. All  Ref: Nelson’s 20/e p 850-857, Ghai 8/e p 169-170


Review of Pediatrics and Neonatology

Surfactant deficiency is the primary cause of RDS; Alveolar atelectasis, hyaline membrane formation & interstitial edema are seen
in RDS.
86. a. Diaphragmatic hernia, b. Congenital Bronchopulmonary cysts, d. HMD and e. Pulmonary hemorrhage  Ref: Nelson’s
20/e p 850-857, Ghai 8/e p 168

Note: Bronchopulmonary dysplasia (BPD) occurs commonly in neonate of HMD who have been treated with O2 & mechanical
ventilation & it usually takes weeks to develop. So, BPD does not present 2–3 hrs after birth.
87. a. L/S ratio  Ref: Nelson’s 20/e p 850-857, Ghai 8/e p 168-170
88. d. 80%  Ref: Nelson’s 20/e p 850-857

Gestation age < 28 wk 32-36 wk > 37 wk


Incidence of RDS 60-80% 15-30% Rare

89. d. Is treated by administering 100% oxygen  Ref: Nelson’s 20/e p 850-857, Ghai 8/e p 168-170
Discussing about the options,
Option a. Yes, RDS is seen mainly in preterm neonates
Option b. Yes, RDS is also common in babies born to diabetic mother due to delayed maturity of surfactant
Option c. Yes, RDS can cause hypoxia & cyanosis
Option d. CPAP ± Surfactant is the treatment of choice; Oxygen may be supplemented to treat hypoxia & titrated according to
O2 saturation; 100% O2 is avoided to prevent Retinopathy of prematurity

90. c. Continue ventilation with the same settings  Ref: Nelson’s 20/e p 850-857, Ghai 8/e p 168-170
Given clinical features & X-ray findings suggest a diagnosis of RDS. With the given ventilator settings, baby is just maintaining
adequate pO2 of 75 mm Hg. So, ventilation should be continued with same settings, till baby improves.

 TRANSIENT TACHYPNEA OF NEWBORN


91. a. Transient tachypnea of the newborn  Ref: Nelson’s 20/e p 858, Ghai 8/e p 171
Respiratory distress, which resolves within 24 hours without any respiratory support and fluid in interlobar fissure on chest
X-ray suggest the diagnosis of Transient tachypnea of newborn.
92. c. 60  Ref: Nelson’s 20/e p 858, Ghai 8/e p 171
93. c. Prominent horizontal fissure  Ref: Nelson’s 20/e p 858, Ghai 8/e p 171
Chest X-ray in transient tachypnea of newborn shows prominent pulmonary vascular markings, fluid in the interlobar fissure &
prominent inter lobar fissure; Occasionally pleural fluid is also seen.
94. c. Elective cesarean section  Ref: Nelson’s 20/e p 858, Ghai 8/e p 171
Delivery by caesarian section is a risk factor for Transient tachypnea of NewbornQ.

 APNEA OF NEWBORN
95. a. Apnea  Ref: Nelson’s 20/e p 849
Apnea in a newborn is defined as: cessation of respiration for 20 seconds with or without bradycardia and cyanosis or cessation
of respiration for less than 20 seconds if it is associated with bradycardia or cyanosis.
96. b. Hyperglycemia  Ref: Nelson’s 20/e p 850-857; For details about neonatal apnea, refer pretext
Chapter 12: Diseases of Newborn 271

 NEONATAL HYPOGLYCEMIA & INFANT OF DIABETIC MOTHER

Answers with Explanations


97. a. Infant of diabetic mother  Ref: Nelson's 20/e p 898-899
Large for date baby with hairy pinna suggests a diagnosis of Infant of diabetic mother.
98. c. Serum calcium  Ref: Nelson's 20/e p 898
•• Hypoglycemia seen in 25-50% of infants of diabetic mothers.
•• The nadir of blood glucose is usually reached at 1-3 hr;
•• Presents with jitteriness, tremulousness & hyperexcitability during the 1st 3 days after birth
•• Early appearance of these signs is more likely to be related to hypoglycemia, and their later appearance to hypocalcemia; these
abnormalities may also occur together.
99. c. Hyperglycemia  Ref: Nelson’s 20/e p 897-898, Ghai 8/e p 181-182
100. a. Ventricular Septal defect  Ref: Nelson’s 20/e p 897-898; For details, refer pretext of this chapter
101. c. Transposition of great arteries  Ref: Nelson’s 20/e p 897-898
102. c. Appropriate for gestational age babies  Ref: Nelson’s 20/e p 897-898, Ghai 8/e p 181-182
Maternal drugs leading to neonatal hypoglycemia include Atenolol, Propranolol, Sulfonylurea drugs
103. b. Learning disability  Ref: Nelson’s 20/e p 897-898, Ghai 8/e p 181-182
104. d. 40  Ref: Nelson’s 20/e p 897-898, Ghai 8/e p 181-182
105. a. Hypoglycemia  Ref: Nelson’s 20/e p 897-898, Ghai 8/e p 181-182
Hypoglycemia is seen in 25–50% of Infants of diabetic mothers & 15–25% of infants of mother with gestational diabetes
Hyperinsulinism is the main cause of hypoglycemia, which manifests as jitteriness, tremulousness & hyperexcitablity during the
1st 3 days after birth, although hypotonia, lethargy, seizures and poor sucking may also occur:
Early appearance of these signs is more likely to be related to hypoglycemia, and their later appearance to hypocalcemia
106. b. Hypoglycemia, c. Hypocalcemia, d. Increased fetal defect, e. Hyperbilirubinemia  Ref: Nelson’s 20/e p 897-898
107. b. Hyperglycemia  Ref: Nelson’s 20/e p 897-898, Ghai 8/e p 181-182
108. b. Caudal regression  Ref: Nelson’s 20/e p 897-898, Ghai 8/e p 181-182
109. a. 40 mg/dl  Ref: Nelson’s 20/e p 897-898, Ghai 8/e p 181-182
110. a. A normal term neonates  Ref: Nelson’s 20/e p 897-898, Ghai 8/e p 181-182
Discussing the options one by one,
a. In a normal term neonate exclusive breastfeeding is usually sufficient to prevent hypoglycemia.
Administration of glucose or dextrose is not routinely required for all neonates
b. These babies are prone to develop hypoglycemia; may require glucose administration, if symptomatic
c. Hypoglycemia can be one of the underlying causes for unconsciousness; so glucose administration may be required
d. Glucose / dextrose administration may be required if there is symptomatic hypoglycemia

111. c. Start oral feeding  Ref: Nelson’s 20/e p 897-898


Feeding is the initial treatment for hypoglycemia in infant of diabetic mother
112. a. Macrosomia, b. Neural tube defect and d. Hypocalcemia, e. Hypoglycemia  Ref: Nelson’s 20/e p 897-898
113. d. Post term infant  Ref: Nelson’s 20/e p 897-898, Ghai 8/e p 181-182
Common causes of Hypoglycemia in newborns:

Inadequate substrate Hyperinsulinism


•• Prematurity •• Respiratory distress syndrome •• Infant of diabetic Mother •• Perinatal asphyxia
•• SGA •• Infant of toxemic mother •• Infant with erythroblastosis fetalis

 NEONATAL HYPERBILIRUBINEMIA
114. a. Erythroblastosis fetalis  Ref: Nelson's 20/e p 873
Jaundice, consisting of either indirect or direct bilirubin, that is present at birth or appears within the 1st 24 hr after birth requires
immediate attention and may be due to erythroblastosis fetalis, concealed hemorrhage, sepsis, or congenital infections, including
syphilis, cytomegalovirus, rubella, and toxoplasmosis.
115. b. Neonatal hepatits  Ref: Nelson's 20/e p 1928
Biliary atresia and neonatal hepatitis are the most common causes of cholestasis in early infancy.
116. a. Start phototherapy and continue breastfeeding  Ref: AIIMS NICU protocols 2014
272 Section 2: Neonatology

AAP nomogram for phototherapy in hospitalized infants of > 35 weeks gestation.


Review of Pediatrics and Neonatology

AAP nomogram for exchange transfusion in hospitalized infants of ≥> 35 weeks gestation

Out of the given options, start phototherapy and continue breastfeeding is the best answer.
117. b. Start phototherapy and continue breastfeeding  Ref: Nelson’s 20/e p 873–74
First line of treatment for pathological neonatal jaundice is phototherapy; breastfeeding should be continued; refer pretext;
118. d. Staining of Diaper by urine is present, e. Breastfeeding should be stopped to treat it  Ref: Nelson’s 20/e p 874
• Breast milk jaundice: It is unconjugated hyperbilirubinemia, so staining of diaper is not present
• Seen in 2% of breastfed term infants after the 7th day, with maximal bilirubin 10-30 mg/dL reached in 2nd-3rd wk.
• Breastfeeding should be continued unless bilirubin level becomes > 20 mg/dL; Phototherapy may be of benefit.
119. b. Phototherapy  Ref: Nelson’s 20/e p 873-874
Any clinical jaundice appearing in the 1st 24 hours of life is always pathological, so the treatment (phototherapy) should start immediately
without waiting for the reports. Exchange transfusion is not the answer here as the bilirubin level is not mentioned & the baby does not
have any clinical evidence of bilirubin encephalopathy (previously called kernicterus), So, Phototherapy is the best answer here.
120. a. Direct bilirubin fraction > 2 mg/dL at any time is pathological, b. Neonatal cholestasis is elevation of conjugated
bilirubin beyond the 1st 14 days of life, c. Biliary atresia is cause of raised levels of conjugated bilirubin, d. Unconjugated
hyperbilirubinemia can be physiologic, e. Acholuric stools indicates conjugated hyperbilirubinemia;  Ref: Nelson’s 20/e
p 873-875; Refer pretext for details;
121. c. RBCs in plasma  Ref: Nelson’s 20/e p 885-886
Fresh packed leukoreduced & irradiated O –ve RBCs reconstituted with AB group fresh-frozen plasma, whose cells are compatible
with both infant’s & mother’s sera, is usually used for double volume exchange transfusion in Erythroblastosis fetalis
122. b. 15  Ref: Nelson’s 20/e p 873-875; Refer pretext for details;
If palms & soles are stained yellow, bilirubin levels are 15-20 mg/dl or even more than that.
123. d. Breast milk jaundice  Ref: Nelson’s 20/e p 874;
124. c. Structural isomerization  Ref: Cloherty’s Manual of Neonatal Care 5/e p 208; Refer pretext for details
Chapter 12: Diseases of Newborn 273

125. d. Neonatal Jaundice  Ref: Nelson’s 20/e p 871-879, Ghai 8/e p 172-176

Answers with Explanations


126. a. Rotors syndrome & b. Dubin Johnson syndrome  Ref: Nelson’s 20/e p 871-879
127. a. Alpha1 AT deficiency  Ref: Nelson’s 20/e p 871-879, Ghai 8/e p 172-176
The given clinical features & investigation findings are suggestive of Alpha1 Antitrypsin deficiency.
Alpha1 AT deficiency
•• It usually presents as neonatal cholestasis or later-onset childhood cirrhosis
•• α1-Antitrypsin, a protease inhibitor synthesized by liver, protects lung alveoli from destruction by neutrophil elastase
•• Normal phenotype is PiMM; In affected patients with PiZZ, polymers of, AT are retained in the ER of hepatocytes
•• In liver biopsy, periodic acid–Schiff (PAS)-positive diastase-resistant globules are seen, primarily in periportal hepatocytes

128. c. High colored urine  Ref: Nelson’s 20/e p 871-879, Ghai 8/e p 172-176; Refer pretext
129. d. G6PD deficiency and e. Rh incompatibility  Ref: Nelson’s 20/e p 871-879, Ghai 8/e p 172-176
Sickle cell anemia & β thalassemia major usually becomes symptomatic beyond 3-6 months age (not in neonatal period)
130. a. Neonatal jaundice  Ref: Nelson’s 20/e p 871-879, Ghai 8/e p 172-176
131. a. Skin pigmentation  Ref: Nelson’s 20/e p 871-879, Ghai 8/e p 172-176
Factors affecting the effectiveness of phototherapy are:
•• Type of lamps used: LED lamps are better than fluorescent •• Surface area of exposed skin
•• Distance between the lights and the infant •• Rate of hemolysis

132. a. Exchange transfusion  Ref: Nelson’s 20/e p 871-879; Refer pretext for details
133. b. 34  Ref: Nelson’s 20/e p 871-879, Ghai 8/e p 172-176
Bilirubin is the end product of catabolism of hemoglobin; Hemoglobin (1 g) → Heme → Biliverdin → Bilirubin (34 mg)
134. d. Hydrops fetalis  Ref: Nelson’s 20/e p 871-879, Ghai 8/e p 172-176
Indications of Exchange transfusion in a neonate with Rh incompatibility are:
• Cord hemoglobin < 10 g/dL or Cord bilirubin >5 mg/dl
• Direct coomb’s test positive & H/o kernicterus or severe erythroblastosis in a sibling.
135. b. Defect in hepatic conjugation  Ref: Nelson’s 20/e p 871-879, Ghai 8/e p 172-176
Crigler-Najjar syndrome is due to glucuronyl transferase deficiency → defective conjugation of bilirubin
136. b. Rh Incompatibility  Ref: Nelson’s 20/e p 871-879, Ghai 8/e p 172-176
In Rh incompatibility, Jaundice usually presents at birth or within 24 hours of life
137. c. IVIg  Ref: Nelson’s 20/e p 871-879, Ghai 8/e p 172-176; Refer pretext of this chapter for details;
138. c. Dubin Johnson syndrome, d. Rotor syndrome & e. Neonatal hepatitis  Ref: Nelson’s 20/e p 871-879
139. a. Microspherocytes  Ref: Nelson’s 20/e p 871-879, Ghai 8/e p 172-176
High Yield Points about ABO incompatibility
•• ABO incompatibility is the most common cause of hemolytic disease of the newborn
•• Usually, the mother is type O and the infant is type A or B; most commonly seen in babies with A, blood group
•• Peripheral smear in these patients show Microspherocytes, Reticulocytosis & Polychromasia

Note: Polychromasia may be seen but is not very prominent in ABO incompatibility, as the hemolysis is usually mild, so the

compensatory erythroid response from the bone marrow too would be mild.
140. a. Rotor syndrome  Ref: Nelson’s 20/e p 871-879, Ghai 8/e p 172-176
• Amongst the given options, Rotor syndrome and Primary biliary cirrhosis cause conjugated hyperbilirubinemia
• In Primary biliary cirrhosis, LFTs are abnormal.
141. a. Hypotonia  Ref: Nelson’s 20/e p 871-879, Ghai 8/e p 172-176
Hypotonia is an early feature of acute bilirubin encephalopathy; For details refer pretext of this chapter;
142. a. Phototherapy  Ref: Nelson’s 20/e p 871-879, Ghai 8/e p 172-176
Bronze baby syndrome: is a dark, grayish brown skin discoloration mainly seen in infants with elevated conjugated bilirubin, as an
adverse effect of phototherapy.
143. a. Physiological jaundice, b. Breast milk jaundice, c. Gilbert syndrome  Ref: Ghai 8/e p 172-176
144. b. Congential cholangiopathy  Ref: Nelson’s 20/e p 871-879 Ghai 8/e p 172-176
All options except ‘b’ are causes of unconjugated hyperbilirubinemia.
145. a. Continue breastfeeds and review after 24 hours  Ref: Nelson’s 20/e p , Ghai 8/e p 172-176
At 48-72 hrs of life, phototherapy cut off for a term neonate is total bilirubin > 18 mg/dl.
146. a. Total and Direct bilirubin  Ref: Nelson’s 20/e p 871-879, Ghai 8/e p 172-176
Measurement of total & direct bilirubin will help to classify jaundice into its respective type and aid in diagnosis.
274 Section 2: Neonatology

147. d. Sulphonamide  Ref: Nelson’s 20/e p 871-879, Ghai 8/e p 172-176


Review of Pediatrics and Neonatology

Sulphonamides displace unconjugated bilirubin from albumin → ↑ level of free unconjugated bilirubin → ↑ risk of kernicterus
148. c. Fanconi’s syndrome  Ref: Nelson’s 20/e p 871-879, Ghai 8/e p 172-176
149. a. Rh incompatibility, b. Physiological jaundice  Ref: Nelson’s 20/e p 871-879, Ghai 8/e p 172-176
The given clinical scenario points towards unconjugated hyperbilirubinemia in the neonate, which is not very severe (upto thigh); so,
both, physiological Jaundice & Rh incompatibility are possible causes.
150. a. Choledochal cyst, b. Extrahepatic biliary atresia, e. Rotor syndrome  Ref: Nelson’s 20/e p 871-879
151. b. Jaundice disappear in 3rd week and d. Jaundice may be seen on day 5 of life  Ref: Nelson’s 20/e p 871- 879
Criteria for physiological jaundice
•• Clinical jaundice appears after 24 hours of age & it is unconjugated hyperbilirubinemia
•• Total bilirubin rises by less than 5 mg/dl per day (no sudden rise)
•• Peak bilirubin occurs at 3–5 days of age, with a total bilirubin of no more than 15 mg/dl; (soles not stained)
•• Clinical jaundice is usually resolved by 1 week in term & 2 weeks in preterm infants; so in 3rd week, there is no clinical jaundice.

152. d. Hemolytic jaundice  Ref: Nelson’s 20/e p 871-879, Ghai 8/e p 172-176
Important clues provided in question are: Increased total bilirubin but normal conjugated bilirubin: So, unconjugated
hyperbilirubinemia present
Amongst the given options, only hemolytic jaundice causes increased unconjugated bilirubin;
Remaining three cause conjugated hyperbilirubinemia.
153. b. Unconjugated hyperbilirubinemia, e. Starts on 2nd day of life  Ref: Nelson’s 20/e p 871-879

 NECROTIZING ENTEROCOLITIS
154. a. Necrotizing enterocolitis  Ref: Nelson’s 20/e p 869-870, Ghai 8/e p 165-166
155. b. Pneumoperitoneum  Ref: Nelson’s 20/e p 869-870, Ghai 8/e p 165-166
156. c. 2b  Ref: Nelson’s 20/e p 869-870, Ghai 8/e p 165-166
Stage of NEC Radiological feature
I Normal/mild intestinal dilatation
II IIa: Pneumatosis intestinalis; IIb: Portal vein gas
III IIIa: Peritonitis / Ascites; IIIb: Pneumoperitoneum

157. b. Necrotising enterocolitis  Ref: Nelson’s 20/e p 869-870, Ghai 8/e p 165-166

 MISCELLANEOUS
158. b. Head only  Ref: Nelson;s 20/e p 820
Conjoined twins (Siamese twins—incidence 1/50,000)
Result from relatively late monovular separation.
Prognosis for conjoined twins depends on the extent to which vital organs are shared.
Site of connections varies:
•• Thoraco-omphalopagus (28% of conjoined twins),
•• Thoracopagus (18%)
•• Omphalopagus (10%)
•• Craniopagus (6%): joined at head
159. a.Diamniotic dichorionic  Ref: Nelson’s 20/e p 171
Time interval between ovulation and Nature of membrances
cleavage of egg
0-72 hr Diamniotic, dichorionic
4-8 days Diamniotic, monochorionic
9-12 days Monoamniotic, monochorionic

160. b. 125 mg/dl  Ref: NeoReviews. November 2010, volume 11 / issue 11


Although the definition of Neonatal hyperglycemia varies, a blood glucose concentration greater than 125 mg/dL (6.9 mmol/L) or a
plasma or serum glucose concentration greater than 150 mg/dL (8.3 mmol/L) is often used.
161. a. Third baby    Ref: Nelson's 20/e p 819
Factors that define an infant as being ‘high risk’:
Chapter 12: Diseases of Newborn 275

Answers with Explanations


Demographic social factors Previous pregnancy Present pregnancy
Maternal age <16 or >40 yr Intrauterine fetal demise Vaginal bleeding (abruptio, placenta previa)
Illicit drug, alcohol, cigarette use Neonatal death Sexually transmitted ds (e.g. syphilis, HIV)
Past medical history Prematurity Multiple gestation
Genetic disorders Intrauterine growth restriction Preeclampsia
Diabetes mellitus Neonate Premature rupture of membranes
Hypertension Birthweight <2,500 or >4,000 g Poly-/oligohydramnios
Asymptomatic bacteriuria Birth <37 or ≥42 wk of gestation Congenital malformation
Rheumatologic illness (SLE) Respiratory dist ress, cyanosis Labor and delivery
Fetal distress
Breech presentation
Meconium-stained fluid

162. c. Rh incompatibility  Ref: Nelson's 20/e p 871-879


Rh incompatibility is a cause of immune hydrops, not non-immune
163. c. Multicystic dysplastic kidney  Ref: Nelson's 20/e p 2555
Multicystic dysplastic kidney is the most common cause of an abdominal mass in the newborn, but the majority are nonpalpable
at birth.
164. a. Crouzon syndrome  Ref: Nelson’s 20/e p 2817
Crouzon syndrome is a cause of craniosynostosis (premature fusion of sutures), so the fontanelle are small & close early.
165. b. High larynx  Ref: Nelson’s 20/e p 794-797
166. a. 16.5 g/dl  Ref: Nelson’s 20/e p 2309, Ghai 8/e p 330
167. d. > 8 days  Ref: Nelson’s 20/e p 171, Hacker & Moore’s Essentials of Obstetrics and Gynecology
The relationship between the timing of cleavage & nature of membranes in twin gestations:
Time interval between ovulation and cleavage of egg Nature of membranes
0-72 hr Diamniotic, dichorionic
4-8 days Diamniotic, monochorionic
9-12 days Monoamniotic, monochorionic

168. b. Testosterone and Mullerian inhibitory factor  Ref: Nelson’s 20/e p 2729-2734
Development of genitalia in males:
•• SRY gene produces SRY protein that directs the development of gonad into testes
•• Male development can only occur when fetal testis secretes Anti-Müllerian hormone (AMH) & testosterone
•• Anti-Müllerian hormone causes the paramesonephric ducts to regress
•• Testosterone converts the mesonephric ducts into male accessory structures, including the epididymis, vas deferens, and seminal
vesicle & also controls the descent of the testes.

169. c. Thorachophagus  Ref: Nelson’s 20/e p 820


Conjoined twins (Also called Siamese twins)
Result from relatively late monovular separation; Types depending on the site of connections are: thoraco-omphalopagus (28% of
conjoined twins), thoracopagus (18%), omphalopagus (10%), craniopagus (6%) & incomplete duplication (10%)

170. b. Carbohydrate  Ref: Nelson’s 20/e p 806


•• Glucose is the main energy source for the fetus & it diffuses easily across to fetal capillaries because the gradient favors it
•• Under non-stressed conditions, fetal glucose is derived entirely from mother through placental transfer

171. b. Baby –gram  Ref: Nelson’s 20/e p 1800-1804, Ghai 8/e


Baby –gram: An X-ray of the entire body of an infant; To look for the underlying cause of obstruction, this is the best initial
investigation, with the radiation exposure much less than a CT scan. It is also called an 'Infantogram'
172. c. 24%  Ref: Ghai 8/e p 181-182
Administration of oral sucrose (in dosages of 0.5–2 ml of 12%–50% solution) approximately two minutes prior to single heel
lance is effective in providing pain relief in both term and preterm infants.
173. d. Orthopnea  Ref: Nelson’s 20/e p 2163-2169
276 Section 2: Neonatology

174. b. Multicystic dysplastic kidney  Ref: Nelson’s 20/e p 2513


Review of Pediatrics and Neonatology

Multicystic dysplastic kidneyQ is the most common cause of abdominal mass in a neonate.
175. a. Lower GI study  Ref: Nelson’s 20/e p 1809-1810
Lower GI study will give clue about underlying etiology like Hirschsprung disease, meconium ileus & other anorectal malformations.
176. b. Rh incompatibility  Ref: Nelson’s 20/e p 883-886, Ghai 8/e p 173-176
Causes of Hydrops fetalis
System involved Underlying cause
Immune Rh incompatibilityQ
Anemia α-thalassemia, G-6PD deficiency
Cardiac Arrhythmias: Supraventricular tachycardia, AF, Congenital heart blockQ
Structural: Hypoplastic left heart, endocardial cushion defect, Cardiomyopathy
Vascular Twin-twin transfusionQ, thrombosis of renal or Umblical vein
Lymphatic Lymphangiectasia, Cystic HygromaQ
CNS Encephalocele, Intracranial hemorrhage
Thoracic Mediastinal teratoma, Diaphragmatic hernia
Tumors Sacrococcygeal teratoma, Neuroblastoma, Hepatoblastoma
Metabolic Gaucher disease, Niemann-Pick disease, Mucopolysaccharidosis
Genetic Trisomy 13, 15, 16, 18, 21, Noonan syndrome
Bone diseases Osteogenesis imperfecta, Skeletal dysplasias
Infections CMV, rubella, Toxoplasmosis, Syphilis, Parvovirus,Q Leptospirosis
Others Congenital nephrosisQ, Myotonic dystrophy, Infant of diabetic mother, Hepatic fibrosis

Note: All the above causes of hydrops, other than Rh incompatibility are non-immune causes.
177. a. Frontal sinus  Ref: Nelson’s 20/e p 2008
High Yielding Points about Development of Paranasal sinuses:
•• Ethmoidal and maxillary sinuses are present at birth, but only the ethmoidal sinuses are pneumatized.
•• Sphenoidal sinuses are present by 5 yr of age
•• Frontal sinuses begin development at age 7-8 yr and are not completely developed until adolescence.

178. d. Brainstem evoked auditory response  Ref: Nelson’s 20/e p 3075-3077


Screening Tests for hearing evaluation in a neonate:

Name of the method Characteristics


Clinical Evaluation Not reliable
Transient Evoked Otoacoustic Emissions Records acoustic feedback from cochlea following a click stimulus
It evaluates the functioning of peripheral auditory system
Unreliable due to false positive results & lack of gestation related norms
Auditory Brainstem Responses It can be used in neonates after 34 weeks of post conceptional maturity
It produces a simple pass or fail result to a fixed 35 dB HL click
It can be conducted even in the presence of background noise of NICU. Manual ABR testing
helps to determine severity as well as nature of hearing loss
From the above discussion, it is clear that Brainstem evoked auditory response is far superior to EOAE in evaluating hearing in
a neonate and hence, is the best screening test.
179. d. Kleihauer Betke test  Ref: Nelson’s 20/e p 883-886
Maternal vs fetal blood: Maternal blood contains adult hemoglobin, while fetal blood contains fetal hemoglobin.
Fetal hemoglobin (HbF) is resistant to alkali denaturation (Apt test) & acid denaturation (Kleihauer betke test).

Apt test Kleihauer Betke test


Source of sample Maternal or neonatal Maternal
Principle Adding 1% NaOH destroys adult HbA but not HbF Adding acid destroys HbA but not fetal HbF
Assessment type Qualitative Quantitative
Results Positive means blood is of fetal origin Reported in estimated ‘ml’s of fetal blood
Chapter 12: Diseases of Newborn 277

180. d. State 6  Ref: Nelson’s 20/e p 63

Answers with Explanations


As the neonate is lustily, (i.e. strongly) crying, his behavioral response grading is state 6 (prechtl's system)

Behavioral State Description


1 Eyes closed, regular respiration, no movements (may have startle) (Quiet sleep)
2 Eyes closed, irregular respiration, may have small isolated/ gross movements (Active sleep)
3 Eyes open, regular respiration, no movement (may have small isolated movements) (Drowsy)
4 Eyes open, gross movements, no cry (Alert)
5 Fussy
6 Strong cry

181. a. Begins between 2–7 days of life  Ref: Nelson’s 20/e p 888, Ghai 8/e p 117

Type of HDN Early onset Classical Late onset


Age of onset 0-24 hr 2-7 days 1-6 months

182. c. Congenital varicella syndrome  Ref: Nelson’s 20/e p 1580-1585


183. c. Large proportionate body  Ref: Nelson’s 20/e p 895
Characteristic of fetal alcohol syndrome include:
•• Prenatal onset with decrease in length, weight and head circumference (microcephaly)
•• Facial abnormalities includes epicanthal folds, maxillary hypoplasia, micrognathia, smooth phiItrum, thin smooth upper lip
•• Cardiac defects, primarily septal defects esp ASD; Delayed development & Intellectual disability

184. b. Fetal alcohol syndrome  Ref: Nelson’s 20/e p 895


185. a. As soon as possible  Ref: Nelson’s 20/e p 1400
186. c. Neonatal tetanus  Ref: Nelson’s 20/e p 1432, Ghai 8/e p 247
187. c. Neonatal tetanus  Ref: Nelson’s 20/e p 1432, Ghai 8/e p 247
Neontal Tetanus
•• Typically manifests within 3-12 days of birth
•• Progressive difficulty in feeding (sucking and swallowing), associated hunger, and crying seen
•• Paralysis or diminished movement, stiffness & rigidity to the touch, and spasms, with or without opisthotonos, are characteristic.
Chapter 13
Pediatric Cardiology
High Yield Points  FETAL CIRCULATION

•• 4 chambered structure of heart Differences between Fetal and Adult Circulation M


develops by 6 weeks of IU life
•• Ductus venosus is a bypass shunt •• In adults gas exchange occurs in the lungs, while in fetus placenta provides exchange
between portal vein & IVC •• In fetus, lungs are collapsed, pulmonary vascular resistance is very high & very little
•• Ductus arteriosus a shunt bet­ween blood goes to lungs
pulmonary trunk and des­cen­ding
aorta •• Ductus arteriosus, ductus venosus & foramen ovale are important structures taking part
in fetal circulation.

High Yield Points


In the fetal circulation:
•• Blood in IVC has more saturation
than blood in SVC
•• Left ventricular blood has more
oxygen saturation than right
•• Brain & coronary circulation
receive blood with higher satu­
ration than the lower half of body
•• Left ventricular output is half of
right ventricular output
•• Aorta and pulmonary trunk are
connected by ductus arteriosus, and
blood flows from pulmonary trunk
to aorta

Mnemonic  M
Umbilical cord contains 1 umbilical vein
& 2 arteries: so, 'Vein is One'
Chapter 13: Pediatric Cardiology 279

Course of Fetal Circulation

Section 3: Systemic Pediatrics


M
High Yield Points
Changes in Circulation after Birth
•• Lungs become the source of
oxygen
•• Pulmonary vascular resistance
decreases & blood flow to lung
increases
•• Systemic vascular resistance
increases
•• Foramen ovale closes
•• Ductus venosus closes by day 7 of
life
•• Ductus arteriosus closes

High Yield Points


Closure of Ductus Arteriosus
•• Functional closure: Immediately
after birth, due to contraction of
smooth muscles
•• Anatomic closure: In 10–21 days
after birth due to intimal thickening
•• Increase in arterial O2 tension
& decrease in concentration of
prostaglandins plays an impor­tant
role in closure of ductus

Remnants of fetal circulation M

Structure Remnant
Ductus arteriosus Ligamentum arteriosum
Umbilical artery Median umbilical ligament
Umbilical vein Ligamentum teres hepatis
Ductus venosus Ligamentum venosum
280 Section 3: Systemic Pediatrics

Pressure in different Chambers of Heart after Birth (in mm Hg)


Review of Pediatrics and Neonatology

M
Mnemonic M

Abnormalities in 22q del or


velocardiofacial syndrome causes:
•• CATCH 22 (conotruncal
anomalies)—cardiac defects,
abnormal facies, thymic hypoplasia,
cleft palate, hypocalcemia.

Note: These pressures in different


High Yield Points chambers of heart usually get established
•• Most congenital heart diseases by 2–3 weeks after birth; but, in presence
are transmitted by multifactorial of heart diseases like VSD or PDA, it
inheritance pattern may take 6–10 weeks.
•• Strongest Familial tendency
for CHD is Holt-Oram syndrome
(AD)—ASD with limb abnormalities  CONGENITAL HEART DISEASE (CHD)
•• High altitude delivery is a risk
factor for development of ASD and •• Occurs in 0.8% of live births
PDA •• Develop between 3rd and 8th weeks of Intrauterine life
•• Transposition of the great arteries and left-sided obstructive lesions are slightly more common
in males

NADA’s Criteria
Used in the past to diagnose presence of congenital heart diseaseQ.
Major Minor
•• Systolic murmur grade 3 or more •• Systolic murmur less than grade 3
•• Any diastolic murmur •• Abnormal S2 
•• Cyanosis  •• Abnormal ECG 
•• Congestive heart failure  •• Abnormal chest X-ray 
•• Abnormal BP

Presence of 1 major or 2 minor criteria indicate very high probability of a CHD.

Important syndromes associated with Congenital Heart Diseases M


Question 1
Syndrome Associated congenital heart diseases
A child with supravalvular aortic
stenosis presents with the follo­ Down's syndrome (Trisomy 21) AV septal defect (Most common)Q, VSD, ASD
wing facies. What is the syn­drome Noonan syndrome Pulmonary stenosis and hypertrophic cardiomyopathy, ASD
this child is suffering from? Apert syndrome Coarctation of aorta and VSD
Holt Oram syndrome ASDQ and VSD
Ellis Van Creveld syndrome Single atrium, ASD
TAR syndrome (Thrombocytopenia Absent Radius) syndrome ASD and TOF
DiGeorge syndrome Cardiac outflow tract defects
Alagille syndrome Pulmonary artery stenosis, Tetralogy of Fallot
William syndrome Supravalvular aortic stenosis
Cri du chat syndrome (5p–) VSD, PDA, ASD

Classification of Congenital Heart Diseases


a. Williams syndrome I. Left to right shunts:
b. Turner syndrome
–– VSD (Ventricular septal defects)
c. Noonan syndrome
d. Di George syndrome –– ASD (Atrial septal defects)
–– PDA (Patent ductus arteriosus).
II. Ductus dependent Pulmonary blood flow: Right sided obstructive lesions like:
–– Critical pulmonary valve stenosis with intact ventricular septum
–– Severe Tetralogy of Fallot
–– Pulmonary atresia
–– Tricuspid atresia
–– Ebstein anomaly
Chapter 13: Pediatric Cardiology 281

III. Ductus dependent Systemic flow: Left sided obstructive lesions like:

Section 3: Systemic Pediatrics


–– Congenital severe aortic stenosis Mnemonic  M
–– Severe Coarctation of the aorta Important syndromes associated
–– Interrupted aortic arch with ASD:
–– Hypoplastic left heart syndrome (HLHS). PeTAR FED HER
These newborns present with severely decreased cardiac output leading to pallor, •• Pierre-Robin syndrome
diminished peripheral pulses, low urine output & cool extremities. •• TAR (Thrombocytopenia absent
IV. Parallel circulation/Ductus dependent oxygenation radius) syndrome
These are babies who are born with CHD that requires a patent ductus for ade­quate mixing of •• Fetal alcohol syndrome
•• Ellis-Van Crevald syndrome
saturated and desaturated blood like Transposition of great vessels (TGV).
•• Down syndrome
V. Ductus independent cyanotic heart diseases •• Holt-Oram syndrome
–– Total anomalous pulmonary venous return (TAPVR) •• Ehler-Danlos syndrome
–– Truncus arteriosus •• Rubinstein-Taybi syndrome

High Yield Points


Common Points for all Left to Right Lesions: (ASD, VSD and PDA)
•• More common in females
•• Congestive cardiac failure mani­fests around 6 weeks, when pulmo­nary vascular pressures fall
•• Due to increased pulmonary blood flow, they have recurrent lower respiratory tract infections
•• Infective endocarditis is an important complication:
–– Most common in VSDQ
–– Least likely: Ostium secundum ASDQ
•• Most dangerous complication: pulmonary hypertensionQ (PHT), which if left untreated will lead on to
reversal of shunt (Eissenmenger’s syndrome)
•• PHT develops earliest in PDA,Q followed by VSD and comparatively late in ASD.

ATRIAL SEPTAL DEFECT (ASD) M


Question 2
Types: 4 Types Which congenital heart dis­ease is
•• Ostium primum associated with this child’s con­
•• Ostium secondum (most common)Q dition?
•• Sinus venosus type, (in upper part of atrial septum, where SVC enters)
•• Coronary sinus type
Note: All the features discussed below are of Ostium secundum ASD (unless mentioned otherwise).

Hemodynamics

a. ASD b. VSD
c. PFO d. PDA

Assessment of severity
Larger the shunt→louder
the pulmonary & tricuspid
murmurs & more cardio­
megaly.

Clinical Presentation
Usually asymptomatic.
Mnemonic  M
Chambers Enlarged
•• R with S → Right axis deviation in
Right atrium & right ventricle. secundum ASD
•• L with P → Left axis deviation in
Chest X-ray Finding primum ASD
RAH, RVH, pulmonary artery dilated.
282 Section 3: Systemic Pediatrics

ECG Finding
Review of Pediatrics and Neonatology

•• Right axis deviation in ostium secundum ASD (R-S)


•• Left axis deviation in ostium primum ASD (due to RV conduction defect).
Treatment
•• In asymptomatic cases: Treatment is not required
•• Indications for Surgery: All symptomatic patients and also for asymptomatic patients with a
Qp: Qs (pulmonary blood: systemic blood) ratio of at least 2:1
•• Timing for elective closure is usually after the 1st year and before entry into school
•• Procedure of choice is percutaneous catheter device closure using an atrial septal occlusion
device, implanted transvenously in the cardiac catheterization laboratory.
Patent Foramen Ovale (PFO)
•• Embryological connection between left & right atrium which usually closes after birth.
•• Can be a channel for paradoxical embolization (right to left).

Ostium Primum ASD


Defect is situated in the lower portion of the atrial septum and overlies the mitral and tricuspid
High Yield Points valves.

Katz watchel phenomenon—common AV Septal Defect/AV Canal Defect/Endocardial Cushion Defect  M

ventricular hypertrophy in moderate Consists of contiguous atrial & ventricular septal defects with markedly abnormal AV valves. Common
VSD is associated with equiphasic QRS in children with Down’s syndrome.
complexes in all precordial leads
VENTRICULAR SEPTAL DEFECT (VSD) M

High Yield Points High Yield Points


•• Restrictive VSD—size of the lesion •• Most common congenital heart disease is VSD
< 0.5 sq. cm •• Most common congenital heart disease affected by infective endo­carditis is VSD.
•• Nonrestrictive VSD—size usually >
1.0 sq. cm, right and left ventricular Types: Membranous (more common)Q & Muscular.
pressure equalized
•• Qp:Qs ratio > 2:1 Hemodynamics
•• Indicates a large VSD with
pulmo­ nary blood flow more than
double of systemic blood flow
•• Swiss Cheese septum—is a special
type of muscular VSD with multiple
defects—difficult to close surgically

Assessment of
Question 3 severity

A child with VSD develops the Larger the shunt:


following findings. There is no shorter the murmur
history of fever. What is the and presence of Mid
probable cause of these findings? diastolic murmur in
mitral area.
•• Chambers Enlarged: Left atrium & left ventricle.
•• ECG: Left Axis Deviation, Left atrial hypertrophy & left ventricular hypertrophy.
•• CXR: Cardiomegaly with left ventricular apex and pulmonary plethora.

Management
Medical: Digoxin and Frusemide for treatment of heart faliure.
a. Infective endocarditis
Indications for surgery:
b. Eissenmenger syndrome
•• Any age with large defects in whom symptoms cannot be controlled medically
c. Cyanotic spell
d. Patent ductus arteriosus •• 6 to 12 months age with large defects associated with pulmonary hypertension;
•• Patients older than 24 months with a Qp:Qs ratio greater than 2:1
•• Supracristal VSD of any size (high risk of aortic regurgitation.
•• Contraindication to closure of VSD: Severe­pulmonary vascular disease non­-responsive to
pulmonary vasodi­lators.
Chapter 13: Pediatric Cardiology 283

Complications

Section 3: Systemic Pediatrics


High Yield Points
Eissenmenger syndrome & infective endocarditis are 2 important complications of VSD.
•• Drug used to keep ductus open
in duct dependent conditions is
PATENT DUCTUS ARTERIOSUS (PDA) M Alprostadil (PGE1 analogue)Q
•• Differential cyanosisQ is caused
Ductus arteriosus is a connection between aorta & pulmonary artery. by PDA with pulmonary artery
hypertension or reversal of shunt
Hemodynamics and severe coarctation of aorta

Assessment of severity
Larger the shunt—more
cardio­megaly, MDM in
mitral area and wider the
pulse pressure.
High Yield Points
Clinical Features: Similar to VSD •• Continuous murmur is heard
Signs of PDA in a preterm baby: in—PDA, pulmonary and systemic
AV fistulas, ruptured sinus of
•• Apnea for unexplained reasons in an infant recovering from RDS
Valsalva, aortopulmonary defect,
•• Hyperdynamic precordium with bounding peripheral pulses
venous hum and persistent truncus
•• Wide pulse pressure arteriosus
•• Continuous or systolic murmur •• To and fro murmur (mimicks conti­
•• CO2 retention ±±Increasing oxygen dependence nuous murmur)—VSD with AR,
•• Hepatomegaly repaired TOF, combined AR and MR

ECG
Left atrial & left ventricular hypertrophy & left axis deviation.

CXR
Similar to VSD.

Management of PDA
High Yield Points
Medical M
•• In preterm infants, IndomethacinQ, a potent inhibitor of prostaglandin synthesis can be used Indomethacin or Ibuprofen are not
for medical closure of PDA; alternatively, Ibuprofen may be used effective in full term infants or children
•• Management of heart failure (Furosemide, Digoxin). with PDA
Surgical
•• If indomethacin is not successful and the ductus remains hemodynamically significant,
surgical ligation or coil embolisation (by cardiac catheterisation) should be performed
•• Surgical closure is done through left thoracotomy approach. Preferably closed before 1
year of age.

EISSENMENGER SYNDROME

Also called Reversal of shunt


Definition Reversal of shunt at the level of atria / ventricles / PDA, due to irreversible
pulmonary vascular obstructive changes
Clinical features Cyanosis & clubbing
Contd...
284 Section 3: Systemic Pediatrics

Contd...
Review of Pediatrics and Neonatology

Sequence of changesQ Left to right shunt (VSD/PDA/ASD)



Increased flow through the pulmonary vasculature (Hyperkinetic PAH)

Irreversible/obstructive pulmonary hypertension

Increased pressure in the right side of the heart & right ventricular
hypertrophy

Right to left shunt (Reversal of shunt)
Clinical features Cyanosis & clubbing

BICUSPID AORTIC VALVE


•• It is the most common heart disease in Turner syndromeQ
Question 4
•• Treatment of choice is balloon valvuloplasty for moderate to severe AS.
A 6-year-old child presents with
hypertension. A chest X-ray was COARCTATION OF AORTA
done, which showed the follo­wing •• Most common site: 90% occur just below the origin of the left subclavian artery at the
picture. What is the congenital origin of the ductus arteriosus (juxaductal coarctaton)
heart disease this child is suffering
•• More common in males: Associated with Turner syndrome & bicuspid aortic valve.
from?
Clinical Features
•• Severe CoA presents in newborn period with CCF with weak femoral pulses
•• Milder forms can present in older children with weakness and pain in legs after exercise
and incidentally detected hypertension
•• Disparity in pulsation and blood pressure in the arms and legs
•• The femoral, popliteal, posterior tibial, and dorsalis pedis pulses are weak (or absent in up
to 40% of patients), in contrast to the bounding pulses of the arms and carotid vessels.

Chest X-ray M

•• 'Figure of 3 appearance' due to pre-stenotic & post-stenotic dilatation


•• 'E sign' seen on contrast esophagogram, due to pressure on anterior wall of esophagus
•• Notching of the inferior border of 3rd-9th ribs from pressure erosion by enlarged collateral
a. Atrial septal defect
vessels, usually seen beyond the age of 3 years.
b. Bicuspid aortic valve
c. Patent ductus arteriosus Treatment
d. Coarctation of aorta
•• Severe COA is ductus dependent and using PGE1 can be life saving
•• Surgery is done after stabilisation—resection of the coarctation with end to end anastamosis
is the treatment of choice
High Yield Points •• For recoarctation, balloon angioplasty is the treatment of choice.

•• Acyanotic/Pink tetralogy of  CYANOTIC CONGENITAL HEART DISEASES (CCHD)


Fallot: When obstruction to right
ventricular outflow is mild-mode­
rate, the patient is not cyanotic
•• Heart Failure is uncommon in TOF
except in anemia, Infective endocar­
ditis, hypertension, myocarditis

High Yield Points


•• Trilogy of Fallot = ASD + RVH + PS
•• Pentalogy of Fallot = TOF + ASD
Chapter 13: Pediatric Cardiology 285

TETRALOGY OF FALLOT (TOF)

Section 3: Systemic Pediatrics


Four Components
 Obstruction to RV outflow tract (mostly infundibular pulmonary stenosis)
 VSD—large unrestricted outlet type of VSD
 Dextroposition of aorta (over-riding of Aorta)
 Right ventricular hypertrophy (RVH).

Hemodynamics Question 5
In which cyanotic heart disease the
following chest X-ray findings are
seen?

a. TOF b. TGA
c. VSD d. TAPVC

Clinical Features High Yield Points


•• Infants — May present as a ductal dependent lesion
Severity of TOF is directly proportional
•• Older children —Cyanosis, clubbing, dyspnea on exertion & cyanotic spells. to severity of cyanosis, while it is
inversely proportional to duration &
Examination Findings intensity of murmur
•• Prominent left anterior hemithorax in older children (due to long standing RVH
•• A systolic thrill/systolic murmur in left sternal 2nd-3rd parasternal space caused by turbulence
through right ventricular outflow tract
•• Single 2nd heart sound or soft pulmonary component. High Yield Points

High Yield Points Murmur of TOF becomes less


prominent with severe obstruction and
Chest X-ray in TOF shows hypercyanotic spell
“Cor en sabot” (Boot shaped heart) due to cardiac apex elevation (due to right ventricular hypertrophy),
pulmonary oligemia.

Why Cyanotic Spell Occurs?


High Yield Points
Complications of TOF
•• Cerebral thromboses
±± Common in children < 2 years
±± Involves cerebral veins or
dural sinuses & occasionally
cerebral arteries.
±± Iron deficiency anemia,
polycythemia & dehydration
aggravate thromboses.
•• Brain abscess
±± Common after 2 years age
±± Insidious onset of fever or
gradual behavior change are
the common presentations.
•• Bacterial endocarditis may occur
286 Section 3: Systemic Pediatrics

Treatment of TOF
Review of Pediatrics and Neonatology

Mnemonic  M
i. Treatment of Cyanotic Spell M
“BaS WahA Pahucha Do”
Name of shunt Pulmonary artery to Treatment modality How it helps?
Blalock Taussig Subclavian artery
Squatting or knee chest positionQ Increases systemic vascular resistance &
Waterston’s Ascending aorta
Decreases venous return
Pott’s Descending aorta
100% oxygen Combats hypoxia

IV/SC morphineQ 0.1–0.2 mg/Kg Depresses respiratory centre

Soda bi carbonateQ Neutralizes acidosis

b blocker (IV propranolol)Q Causes systemic vasoconstriction


Question 6
Methoxamine or phenylephrine a agonist → Causes systemic vasoconstriction
A neonate presenting with heart
failure on day 7 of life has this
chest X-ray. What is the most
probable diagnosis? ii. Supportive Therapy
Correction of iron-deficiency anemia; prevention of spell (propranolol).

iii. Surgical Treatment of TOF


Corrective—
•• Resection of RVOT obstruction and patch closure of VSD
•• Electively between 4–6 months of age in case of less severe cyanosis without spells
•• Immediately in infants with severe cyanosis (marked right ventricular outflow obstruction)
a. TOF b. VSD
c. TAPVC d. TGA Palliative—shunt surgeries (see high Yield box).

Shunt surgeries for TOF:


•• Blalock Taussig (BT) Shunt—Subclavian artery to pulmonary artery anastamosis
•• Pott’s Shunt—descending aorta to pulmonary artery
•• Waterston’s Shunt—ascending aorta to right pulmonary artery

TRANSPOSITION OF GREAT ARTERIES (TGA)


Transposition of Great vessels is a condition characterized by aorta arising from the right ventricle
High Yield Points and pulmonary artery from the left ventricle.

X-ray Features of TGA High Yield Points


•• Cardiomegaly with “Egg on Side”
appearanceQ •• TGA is the most common CCHD in a neonate and 2nd most common CCHD in infancy
•• Plethoric lung fields (Increased •• TGA is associated with uncon­trolled Phenylketonuria in mother.Q
pulmonary vascularity) •• TGA is the most specific heart disease in infant of diabetic mother
•• Absent thymic shadow •• TGA is more common in males

Hemodynamics:
•• In TGA, there is complete separation of systemic and pulmonary circulation
•• Since systemic and pulmonary circulations are separate, survival depends on mixing of
oxygenated pulmonary blood and deoxygenated systemic blood which can occur:
At Atrial level : Through ASD
At Ventricle level : Through VSD
At Great vessel level : Through Patent ductus arteriosus
•• Oxygenation is normal in fetus but after birth as ductus arteriosus closes, severe hypoxia
occurs & in absence of VSD, or ASD, child presents with cyanosis & tachypnea at birth
Chapter 13: Pediatric Cardiology 287

Section 3: Systemic Pediatrics


High Yield Points
•• CHD which present with cyanosis
at birth are: TGA, TAPVC, Tricuspid
atresia, pulmonary atresia,
Ebstein anomaly
•• In TOF cyanosis usually presents
after neonatal period

•• In d TGA, aorta lies to the right & anterior of pulmonary artery


•• In Corrected TGA, there is atrioventricular and ventriculoarterial discordance.

Clinical Features of TGA


•• Time of presentation depends on whether the ventricular septum is intact or VSD is present
•• Patients of TGA with intact septum are cyanotic at birth, with CHF occuring in first week
because adequate mixing does not occur
•• Patients of TGA with VSD presents with mild cyanosis and CHF occurs in 6–8 weeks.
•• Important manifestations are:
– Cyanosis –  CHF –  Ejection systolic murmur High Yield Points
–– Single S2 –  Normal S1
•• Most common type of TAPVC is
supracardiac
Treatment •• Infracardiac TAPVC is always
•• Administration of PGE1 will keep the ductus arteriosus open temporarily & allow mixing of obstructive
blood of pulmonary & systemic circulation & cause transient improvement
•• Balloon atrial septostomy is a an emergency procedure to save the life of this neonate.
•• Definitive surgery: Arterial switch operation or 'Jatene repair'.
Question 7
TAPVC (TOTAL ANOMALOUS PULMONARY VENOUS CONNECTION) The following X-ray picture is seen
in which CCHD?
Basic defect:
Pulmonary veins instead of draining into Left atrium drain into right atrium directly or indirectly

3 Types:
Supracardiac: Common pulmonary vein drains into Superior vena cava or left innominate vein
Cardiac: Pulmonary veins drain into right atrium or coronary sinus
Infracardiac: Pulmonary veins drain into hepatic veins or portal vein or inferior vena cava
a. TGA
Chest X-ray findings:  M b. TAPVC
Supracardiac TAPVC: Snowman or 'figure of 8' appearance c. TOF
Obstructive TAPVC: Ground glass appearance of lung fields (d/d hyaline membrane disease) d. Tricuspid atresia
288 Section 3: Systemic Pediatrics

HYPOPLASTIC LEFT HEART SYNDROME (HLHS)


Review of Pediatrics and Neonatology

High Yield Points


HLHS is a group of anomalies that include underdevelopment of the left side of the heart &
hypoplasia of ascending aorta.
HLHS is the most common congenital
heart disease causing mortality in the Clinical Features: Cyanosis & heart failure seen within first few days of life.
1st week of life
TRICUSPID ATRESIA
•• Congenital absence of tricuspid valve is called tricuspid atresia
•• There is Hypoplastic right ventricle with absence of inflow portion of RV
•• So, blood cannot flow directly from right atrium to right ventricle and following associated
defect, are necessary for survival:
–– For flow of blood from RA to LA: Patent foramen ovale or ASD.
–– For flow of blood from LV to RV: VSD
•• Left ventricular hypertrophy with left axis deviation is seen in ECG
EBSTEIN ANOMALY
•• Ebstein anomaly consists of downward displacement of an abnormal tricuspid valve into
the right ventricle, leading to atrialization of right ventricle.
Question 8
Hemodynamics
A 2-month-old baby with central •• Right atrium is enlarged as a result of tricuspid valve regurgitation
cyanosis presented with the
•• Effective output from the right side of the heart is decreased because of poorly functioning
following findings on chest X-ray
small right ventricle, tricuspid valve regurgitation & obstruction of the right ventricular outflow
& ECG. What is the most probable
tract produced by large, sail-like, anterior tricuspid valve leaflet
underlying diagnosis?
•• Right ventricle is unable to generate enough force to open the pulmonary valve in systole,
thus producing “functional” pulmonary atresia
•• The increased volume of right atrial blood shunts through the foramen ovale (or through an
associated atrial septal defect) to the left atrium and produces cyanosis.
Auscultation: A gallop rhythm with multiple clicks at lower left sternal border.
Chest X-ray: Box shaped heartQ.
ECG: Himalayan P waves
High Yield Points
Anomalous Origin of Left Coro­nary Artery from Pulmonary Artery (Alcapa)
a. Tetralogy of Fallot
•• Blood supply to the left ven­tricular myocardium is severely compromised
b. Hypoplastic left heart syndrome
•• Soon after birth, as pulmonary arterial pressure falls, perfusion pressure to left coronary artery becomes
c. Coarctation of aorta
inadequate; myocardial ischemia, infarction, & fibrosis result
d. Ebstein anomaly
•• ECG resembles lateral wall myo­cardial infarction.

 RHEUMATIC FEVER / RHEUMATIC HEART DISEASE (RHD)

High Yield Points RHD is the most common cause of heart disease in children in developing countries.Q

Valvular Involvement in RF Epidemiology


•• 100% of pts. with RF have patho­ •• Incidence of RF following streptococcal sore throat in general population is 0.3%
logical mitral valve involve­ment •• Most common age group involved is 5–15 years
•• 95% have clinical MR •• Mitral valve disease and chorea are more common in females
•• 5–8% presents with pure AR •• Aortic valve involvement is more common in males.
•• 20–25% have both AR and MR •• Most common major manifestation of RHD in children is Carditis, seen in 50–60%.
•• Clinical evidence of pulmonary valve •• Second most common manifestation in children—arthritis—30–50% a
involvement in acute RF never seen
Diagnostic Criteria
Modified Duckett Jones criteria. M

Mnemonic  M Major Criteria Minor Criteria Essential Criteria


Major criteria for RF diagnosis: •• Rheumatic nodules •• Fever •• Increased ASO titer
‘R-A-C-E-S’ •• Arthritis •• Arthralgia •• Positive throat culture
Rheumatic Nodules •• Carditis •• Acute phase reactants •• Recent scarlet fever
Arthritis •• Erythema marginatum increased—ESR and CRP
Carditis •• Sydenham’s chorea •• ECG—prolonged PR interval
Erythema maginatum For a diagnosis of RF—2 major or 1 major and 2 minor are needed with essential criteria;
Sydenham’s chorea It is only for the first diagnosis of RF and not for recurrences.
Chapter 13: Pediatric Cardiology 289

Section 3: Systemic Pediatrics


High Yield Points Question 9
Circumstances in which strict RF criteria not needed: The skin lesions shown in this
•• ChoreaQ picture are seen in which heart
•• Indolent carditis disease?
•• Recurrence of RF

Treatment of RF
•• For all patients with carditis bed rest is recommended
•• Antibiotic Therapy—(to eradicate streptococci in throat) single dose benzathine pencillin, or
10 days oral penicillin V or erythromycin
•• Anti-inflammatory Therapy: a. Eissenmenger syndrome
b. Rheumatic heart disease
Aspirin Steroid c. Infective endocarditis
•• Given in Arthritis or mild carditis. •• Given in carditis with CCF & cardiomegaly d. Heart failure
•• Dosage is 100 mg/kg/day for 2–3 weeks. •• Prednisolone is the drug of choice
•• Then dose is tapered and given for a total period •• Dosage: 2 mg/kg/day for 2–3 weeks (or till ESR
of 12 weeks normalizes) and then tapered
•• Total duration of treatment 12 weeks.

Prophylaxis in RF
•• Prevention of recurrent episodes of GAS infection is the most effective method to prevent
the development of severe rheumatic heart disease
•• For secondary prophylaxis is— Benzathine penicillin–1.2 million units or 6 lakh units
(if < 30 kg) every 3 weeks in developing countries.

High Yield Points


Prophylaxis for RF should be given within 9th day of symptomes of acute streptococcal pharyngitis for
being effective.

 HEART FAILURE (HF)

Definition
Heart failure occurs when the heart cannot deliver adequate cardiac output to meet the
metabolic needs of the body. HF can be due to systolic dysfunction (decreased EF) or diastolic
dysfunction (normal EF).

High Yield Points


•• The leading cause of HF in infants and children is CHD
•• Most common cause of heart failure in older children is rheumatic heart disease.
•• Heart failure in children is mostly due to systolic dysfunction

L at e s t U p d at e s
Timing of Heart Failure Newer Drugs for Heart Failure
Birth 1 week 1-4 weeks 1-2 months 2-6 months •• Nesritide: It is a recombinant BNP.
It produces vasodilatation and
•• Ductus dependent systemic circu­­­ •• PDA in pre terms •• PDA in term •• PDA diuresis when given IV.
lation •• VSD with •• VSD •• VSD •• Levosimendan—is a calcium
� HLHS coarctation •• ALCAPA •• ALCAPA sensi­tizer that prolongs the effects
� critical AS •• Truncus arteriosus •• Non-obstructive •• Coarctation of of calcium on the myocardium.
� severe coarctation •• TGA with VSD or TAPVC aorta It has positive chronotropic,
� Interrupted Aortic arch PDA •• E n d o c a r d i a l inotropic & vasodilatory effects
cushion defect without increa­sing myocar­dial
•• Obstructed TAPVC
oxygen demand.
•• D-TGA with Intact ventr septum •• Ivabradine—Ivabradine inhibits
the If channel in the sinus node and
•• Cong. severe MR or TR
has the specific effect of slowing
•• Ebstein anomaly sinus rate.
290 Section 3: Systemic Pediatrics

Clinical Features
Review of Pediatrics and Neonatology

•• Symptoms of CCF in infant: Feeding difficulty, forehead sweating, fast breathing and signs
of tachycardia, tachypnea, hepatomegaly and cardiomegaly
•• Most common symptoms of HF in older children is—fatigue and shortness of breath.
•• Right heart failure: Elevated JVP, facial puffiness, tender hepatomegaly, edema of feet
•• Left heart failure: Tachypnea, tachycardia cough, wheezing, rales in chest, dyspnea on
exertion orthopnea, PND (Paroxysmal nocturnal dyspnoea)
•• Features of both right and left heart failure—cardiac enlargement, gallop rhythm, peripheral
cyanosis, small volume pulse, absence of weight gain.

Chest X-ray Finding


•• Cardiomegaly is a very important sign on CXR in almost all types of Heart failure.
•• A cardiothoracic ratio of > 60% in neonates and > 55% in older children is suggestive
of cardiomegaly.

Treatment of CCF
•• Diuretics (Furosemide): usual 1st line treatment
•• Next line: ACE inhibitors and Angiotensin Receptor Blockers
•• Digoxin helps by improving cardiac contractility
•• Chronic treatment with beta blockers like carvedilol, metaprolol —is used in patients with
cardiomyopathy with chronic failure
•• Milrinone—is useful in treating patients with low cardiac output who are refractory to standard
therapy esp. after open heart surgery.

 SYSTEMIC HYPERTENSION
Definition
High Yield Points Hypertension in children is defined as average systolic blood pressure (SBP) and/or diastolic blood
pressure (DBP) that is ≥ 95th percentile for age, sex, and height on ≥ three occasions.
•• In children most common cause of
hypertension is secondary hyper­
tension Prehypertension
•• Secondary hypertension in children
is mostly due to renal abnormalities Prehypertension is defined as average SBP or DBP that are ≥ 90th percentile but< 95th percentile.
(90%)
Etiology of Hypertension in Children
Renal Endocrine
•• Chronic pyelonephritis •• Hyperthyroidism
•• Chronic glomerulonephritis •• Hyperparathyroidism
•• Congenital dysplastic kidney •• Congenital adrenal hyperplasia (11β-hydroxylase
•• Multicystic kidney and 17-hydroxylase defect)
•• Vesicoureteral reflux nephropathy •• Cushing syndrome
•• Primary aldosteronism
Vascular •• Apparent mineralcorticoid excess
•• Coarctation of aorta •• Glucocorticoid remedial aldosteronism (familial
•• Renal artery stenosis/ Renal vein thrombosis aldosteronism type 1)
High Yield Points •• Umbilical artery catheterization with thrombus •• Pheochromocytoma
formation •• Neuroblastoma
•• Renal parenchymal disease is the •• Neurofibromatosis (intrinsic or extrinsic
most common cause followed by Central nervous system
narrowing of vascular lumen)
renovascular hypertension. •• Vasculitis •• Intracranial mass/Hemorrhage
•• Chronic glomerulonephritis
accounts for most common cause of Treatment
HT in children.
•• Most common cause of hyper­tension •• For asymptomatic mild hypertension without evidence of target organ damage: lifestyle
in the newborn period—asso­ciated modification with dietary changes and regular exercise
with umbilical artery cathe­terization •• Indications for pharmacologic therapy: symptomatic hypertension, secondary hyperten­
and renal artery thrombosis sion, hypertensive target organ damage, diabetes (types 1 and 2), and persistent hyperten­
•• Most common congenital cardio­ sion despite non-pharmacologic measures
vascular cause of hypertension— •• Drugs used: ACE inhibitors, angiotensin receptor blockers, β-blockers, calcium channel
coarctation of aorta.
blockers, and diuretics.
Chapter 13: Pediatric Cardiology 291

 INFECTIVE ENDOCARDITIS (IE)

Section 3: Systemic Pediatrics


High Yield Points
Clinical Features •• Staphylococcal endocarditis is
Prolonged low grade fever with new or changing murmurs is the most common clinical more common in patients with no
underlying heart disease;
manifestation. •• Viridans group streptococcal
infection is more common after
High Yield Points dental procedures;
•• Fungal organisms are encountered
•• Most common symptom of infec­tive endocarditis is fever
after open heart surgery.
•• Other manifestations due to vasculitis produced by circulating antigen-antibody complexes include:
•• Coagulase-negative staphylococci
±± Osler nodes (tender, pea-sized nodules in pads of fingers).
are common in the presence of an
±± Janeway lesions (painless erythematous lesions on palms & soles) indwelling central venous cathe­ter
±± Splinter hemorrhages (linear lesions beneath the nails).
•• Investigation of choice is blood culture (3 sets at least 1 hour apart).

Diagnosis
Duke criteria help in the diagnosis of endocarditis:
Major criteria include: High Yield Points
(1) positive blood cultures (2 separate cultures for a usual pathogen, 2 or more for less typi­ High-risk cardiac lesions for which
cal pathogens), and (2) evidence of endocarditis on echocardiography (intracardiac mass on endocarditis prophylaxis is advised
a valve or other site, regurgitant flow near a prosthesis, abscess, partial dehiscence of prosthetic before dental procedures are:
valves, or new valve regurgitant flow). •• Prosthetic heart valves
Minor criteria include: predisposing conditions, fever, embolic-vascular signs, immune •• Prior endocarditis
complex phenomena (glomerulonephritis, arthritis, rheumatoid factor, Osler nodes, Roth spots), a •• Congenital heart disease
single positive blood culture or serologic evidence of infection, and echocardiographic signs not
meeting the major criteria.
2 major criteria or 1 major and 3 minor, or five minor criteria suggest definite endocarditis.

Treatment
•• Empirical therapy before the identifiable agent is recovered initiated with vancomycin plus High Yield Points
ceftriaxone to cover S. aureus, enterococcus or viridans streptococci Most common cause of myocarditis
•• 6 weeks treatment is recommended. in Infant = Coxsackie virus

 CARDIOMYOPATHY
Definition
Cardiomyopathy refers to a group of diseases of heart muscle that is not secondary to structural
heart disease, hypertension or pulmonary vascular disease.

Classification
Three main categories—dilated, hypertrophic & restrictive. High Yield Points
•• Dilated cardiomyopathy is
Etiology
the most common form of
•• Primary cardiomyopathy–etiology unknown cardiomyopathy in children.
•• Secondary cardiomyopathy— •• Vitamin deficiency associated
–– Infection–Viral, bacterial, parasitic, rickettsial with cardiomyopathy: Thiamine
deficiency (beri beri)
–– Collagen vascular diseases–SLE, dermatomyositis
•• Mineral deficiency associated
–– Metabolic–Beriberi, Pompe disease, Mucopolysaccharidosis with cardiomyopathy: selenium
–– Medications–Adriamycin, phenothiazine, lead deficiency (Keshan disease)
–– Neoplastic–Rhabdomyoma, myxoma, Leukemia, Lymphoma •• Dilated cardiomyopathy is the
–– Neurological–Friedrich ataxia, muscular dystrophies most common cause for cardiac
–– Hematological–Sickle cell anemia, TTP transplantation in children.
–– Miscellaneous–Cystic fibrosis.
292 Section 3: Systemic Pediatrics
Review of Pediatrics and Neonatology

Answer Keys for Image-Based Questions

Answers Explanations / Identifying features


1. Ans a. Williams syndrome This child has round face, long philtrum, upturned nostrils (elfin facies) with supravalvular aortic
stenosis, as seen in William syndrome
2. Ans a. ASD This child with micrognathia & retrognathia has Pierre Robin syndrome, which is associated with ASD
3. Ans b. Eissenmenger Clubbing & cyanosis in a child with VSD, with no history of fever, has Eissenmenger syndrome
syndrome
4. Ans d. Coarctation of aorta Chest X-ray showing notching of inferior border of ribs, in a child with hypertension suggests Coarctation
of aorta
5. Ans a. TOF Chest X-ray showing boot shaped heart is seen in Tetralogy of Fallot (TOF)
6. Ans d. TGA Chest X-ray showing ‘egg on side’ appearance in a neonate presenting with heart failure on day 7 of life
has TGA (Transposition of great arteries)
7. Ans b. TAPVC Chest X-ray showing ‘snowman’ or ‘figure of 8’ appearance is seen in supracardiac TAPVC
8. Ans d. Ebstein anomaly ECG showing tall (Himalayan) P waves & chest X-ray showing ‘box shaped heart’ are seen in Ebstein’s
anomaly
9. Ans b. Rheumatic heart Skin lesions suggestive of erythema marginatum are seen in Rheumatic heart disease
disease
Chapter 13: Pediatric Cardiology 293

Questions
Questions
� FETAL CIRCULATION M 12. Which of the following are true about ASD?
(PGI Jan 2017)
1. Oxygenated blood from fetus is carried by:
a. Ostium primum is the most common defect
 (Recent Question 2015, WBPG 2010)
b. Children are always growth retarded
a. Umbilical artery b. Umbilical vein
c. Surgery is electively done at 1-3 year age for large defects
c. Ductus venosus d. Hypogastric artery
d. Commonly affected by Infective endocarditis
2. Umbilical cord has: M  (DNB Dec 2010) e. It is the commonest congenital heart disease
a. 1 vein and 2 arteries b. 2 vein and 2 arteries 13. Which of the following is NOT a major NADA’s criteria
c. 1 vein and 1 artery d. 2 veins and 1 artery for clinical diagnosis of congenital heart disease:
3. Oxygenated blood to the fetus is carried by: M  (MAHA PGM CET 2016)
 (PGI June 07)
a. Diastolic murmur b. Cyanosis
a. Umbilical artery b. Umbilical vein c. Congestive cardiac failure
c. Superior vena cava d. Pulmonary artery d. Abnormal X-ray
 Acyanotic Congenital Heart Diseases 14. Drug used to keep PDA open: M  (Recent Question 2016)
a PGE1 b. PGI2
4. Most common cardiac abnormality in Noonan syndrome c. PGE2 d. PGH2
is? (FMGE Dec 2018) 15. The heart lesion not found in congenital rubella
a. Ventricular septal defect b. Atrial septal defect infection is: (Recent Question 2016, AIPGMEE 95)
c. Pulmonary stenosis d. Tetralogy of Fallot a. ASD b. VSD
5. Small cuff size will lead to? (FMGE Dec 2018) c. PDA d. PS
a. Falsely increased BP b. Falsely low BP 16. Infective endocarditis is most commonly seen in? M 
c. No effect on BP d. Fluctuating BP a. ASD b. VSD (Bihar PG 2015)
6. Indomethacin causes: (FMGE Dec 2018) c. PDA d. Pulmonary Stenosis
a. Closure of ductus in premature neonate 17. Inferior Rib notching is seen in? M  (Bihar PG 2015)
b. Patent ductus arteriosus a. Coarctation of aorta b. Marfan's syndrome
c. Closure of ductus in term & premature neonates c. Rickets d. SLE
d. Closure of ductus in older children also 18. Coarctation of aorta is most commonly associated with?
7. Which of the following is a stimulus for closure of  (MAHA PGM CET 2015)
ductus arteriosus? (NEET PG Jan 2019) a. ASD b. PDA
a. Increased O2 tension b. Hypoxia c. Bicuspid aortic valve d. VSD
c. Prematurity d. Hypercapnia 19. Not seen in William syndrome? (Recent Question 2015)
8. The innocent murmur is best heard in children at: a. Elfin facies b. Subvalvular aortic stenosis
 (FMGE Nov 2017) c. Hypercalcemia d. Hypertension
a. Pulmonic area b. Aortic area 20. Which type of ASD is commonest?
c. Left lower sternal border d. Apex  (Recent Question 2015)
9. Most common cardiac defect in Turner syndrome: a. Patent foramen ovale b. Ostium primum
 (DNB June 2018) c. Ostium secundum d. Sinus venosus
a. Coarctation of aorta b. VSD 21. True about ostium primum defect: (PGI May 2015)
c. ASD d. TOF a. It is found in ASD
10. Eisenmenger complex is: (PGI May 2017) b. More commonly associated with ASD than VSD
a. ASD with PDA c. May be associated with Down syndrome
b. VSD with pulmonary hypertension d. Ostium primum ASD is more common than ostium
c. Pulmonary vascular obstructive disease with VSD secundum ASD
d. Pulmonary hypertension with ASD e. Right axis deviation is seen on ECG
e. Pulmonary stenosis with PDA 22. Large PDA leads to: (Recent Question 2015)
11. Surgical or device closure Is advised in all symptomatic a. Endocardial valvulitis b. Essenmenger syndrome
and also for asymptomatic children having Atrial septal c. Heart failure d. All of the above
defect when the Qp: Qs ratio is at least: 23. In Ductus dependent heart diseases, drug used is
 (Recent Question 2017)  (Recent Question 2015)
a. 1 : 5 : 1 b. 2 : 1 a. PGE1 b. PGI2
c. 2.5: 1 d. 1 : 1 c. PGE2 d. PGH2
294 Section 3: Systemic Pediatrics

38. In ostium secundum ASD, maximally affected is:


Review of Pediatrics and Neonatology

24. Commonest type of congenital heart disease is: M 


 (WBPG 2010)
 (Recent Question 2015)
a. Right atrium → right ventricle → left atrium
a. ASD b. VSD
b. Right ventricle → right atrium → left atrium
c. TOF d. PDA
c. Left atrium → right atrium → right ventricle
25. Holt-Oram syndrome is characterized by? d. Left ventricle → left atrium → right atrium
 (Recent Question 2014) 39. Patient with VSD develops pulmonary hypertension.
a. ASD b. VSD Which of the following will be present?  (JIPMER 2010)
c. TGA d. TAPVC a. Cyanosis
26. In septum primum type of ASD overburden occurs in b. Initial resolution of symptoms
which chamber?  (WBPG 2014) c. Clubbing
a. RA + RV b. LA + LV d. Ejection systolic murmur in pulmonary area
c. RA + LA d. LA 40. A ten-year-old boy presents to the pediatric emergency
27. Pick up the correct one: Total fetoplacental blood unit with seizures. Blood pressure in the upper extremity
volume at term is estimated to be: measured as 200/140 mm Hg. Femoral pulses were
 (Recent Question 2014; AIPGMEE 05) not palpable. The most likely diagnosis amongst the
a. 150 ml/kg body wt b. 250 ml/kg body wt following is: (AIPGMEE 2010)
c. 350 ml/kg body wt d. 450 ml/kg body wt a. Takayasu aortoarteritis
28. Most common cardiac anomaly in Turner’s syndrome is? b. Renal parenchymal disease
 (DNB Pattern 2013) c. Grand mal seizures d. Coarctation of aorta
a. Coarctation of aorta b. Bicuspid aortic valve 41. Child with PDA will not have:  (AIIMS Nov 2010)
c. Ventricular septal defect d. Atrial septal defect a. Bounding pulses b. Pulmonary hemorrhage
29. Coarctation of aorta is associated with? c. Necrotising enterocolitis d. CO2 washout
 (Recent Question 2013, AIPGMEE 2011, 2008) 42. Which of the following statements is/are FALSE about
a. VSD b. PDA ostium secundum ASD?  (PGI June 09)
c. Bicuspid aortic valve d. None of above a. Fixed splitting of 2nd heart sound
30. Down syndrome is most commonly associated with? b. Narrow splitting of 2nd heart sound
a. ASD with ostium secundum  (Recent Question 2013)
c. Lt axis deviation in ECG
b. ASD with ostium primum d. Shunt murmur prominent
e. Rt axis deviation
c. VSD d. TOF
43. The clinical features associated with coarctation of aorta
31. Most common syndrome associated with AV canal
in older children are the following except:
defect? M  (Recent Question 2013)
a. Upper body hypertension  (UPSC-II 08)
a. Down syndrome b. Klinefelter syndrome
b. Prominent pulsation in neck
c. Turner syndrome d. Marfan syndrome
c. Fatiguability and tiredness in legs
32. Wide fixed S2 is seen in: M  (Recent Question 2013)
d. Absence of flow murmurs over scapular region
a. ASD b. VSD 44. Ductus arteriosus closes in response to: (UPSC-II 08)
c. PDA d. All of above
a. Decrease in peripheral oxygen saturation
33. ASD is associated with all except: b. Indomethacin therapy
 (Recent Question 2013) c. Prostaglandin E1
a. Infective endocarditis b. Stroke d. Increase in pulmonary vascular resistance
c. Arrhythmia d. Pulmonary hypertension 45. A child after 4 weeks of birth is acyanotic, but a systolic
34. In patent ductus arteriosus, connection is between: murmur is detected. Causes are/is: (PGI June 08)
a. Aorta and coronary artery  (Recent Question 2013) a. VSD b. PDA
b. Aorta and pulmonary artery c. TOF d. Coarctation of aorta
c. Aorta and subclavian artery e. Tricuspid stenosis
d. Pulmonary artery and subclavian artery 46. In ostium secundum ASD, which of the following is
35. In a preterm baby with patent ductus arteriosus all are maximally affected: (WBPG 2007)
true except?  (AIIMS Nov 2012) a. Right ventricle b. Left ventricle
a. Narrow pulse pressure b. Necrotising enterocolitis c. Left atrium d. Right atrium
c. Continuous murmur d. Congestive heart failure 47. Most common type of atrial septal defect is: M 
36. The most common congenital heart disease is:  (UPPG 2007, 2006)
a. ASD b. VSD  (TN PGMEE 2011) a. Ostium primum b. Ostium secondum
c. PDA d. TOF c. Endocardial cushion defect
37. A child is having perimembranous VSD, CCF develops d. Endocardial hypertrophy
due to: (TN PGMEE 2011) 48. Uncommon finding in congestive cardiac failure in
a. Aortic regurgitation b. Spontaneous closure newborn:  (AIPGMEE 2001, 2000)
c. Changes in pulmonary vasculature a. Tachycardia b. Tachypnea
d. Infective endocarditis c. Hepatomegaly d. Pedal edema
Chapter 13: Pediatric Cardiology 295

49. A 1-month-old boy is referred for failure to thrive. On 58. A neonate has recurrent attacks of restlessness, irritability

Questions
examination, he shows features of congestive failure. and diaphoresis on feeding. Cardiac auscul­tation reveals
The femoral pulses are feeble as compared to brachial a nonspecific murmur. He is believed to be at risk for
pulses. The most likely clinical diagnosis is: M.I. Likely diagnosis here is:  (AIPGMEE 01)
a. Congenital aortic stenosis  (AIPGMEE 06) a. ASD b. VSD 
b. Coarctation of aorta c. TOF d. Anomalous coronary artery
c. Patent ductus arteriosus 59. All of the following are true about ASD except:
d. Congenital aortoiliac disease  (AIPGMEE 01)
50. In which of the following conditions left atrium is not a. Right atrial hypertrophy b. Left atrial hypertrophy
enlarged? (AIPGMEE 06) c. Right ventricular hypertrophy
a. Ventricular septal defect b. Atrial septal defect d. Pulmonary hypertension
c. Aortopulmonary window d. Patent ductus arteriosus 60. All of the following cause death in coarctation of aorta
51. A premature infant is born with a patent ductus except: (PGI June 2000)
arteriosus. Its closure can be stimulated by adminis­ a. Infective endocarditis b. Heart failure
tration of: (AIIMS May 06) c. Intracranial hemorrhage d. Anterior MI
a. Prostaglandin analogue b. Estrogen e. Hypotension
c. Anti-estrogen compounds d. Prostaglandin inhibitors 61. A 29-day-old child presents with features of congestive
52. Which of the following syndromes is best associated cardiac failure and left ventricular hypertrophy. Auscul­
with congenital heart disease? (AIIMS May 05) tation shows a short systolic murmur. Most likely
diagnosis is: (AIIMS Nov 2000)
a. Lesch-Nyhan syndrome b. Rasmussen syndrome
c. Holt-Oram syndrome d. Leopard syndrome a. Rheumatic fever b. Tetralogy of Fallot
53. A young female presents with history of dyspnea on c. Transposition of great arteries
exertion. On examination, she has wide, fixed split d. Ventricular septal defect
S2 with ejection systolic murmur (III/VI) in left second 62. The following statements are true about patent ductus
intercostal space. Her ECG shows left axis deviation. arteriosus (PDA) except: (Kerala 2000)
The most probable diagnosis is: (AIIMS May 2003) a. Spontaneous closure occurs in some term infants
a. Total anomalous pulmonary venous drainage b. Pulmonary hypertension develops
b. Tricuspid atresia c. Bacterial endocarditis is more frequent with small PDA
c. Ostium primum atrial septal defect d. Recurrent chest infection and congestive failure may
d. Ventricular septal defect with pulmonary arterial hyper­ develop
tension e. Anatomic existence of PDA is an indication for surgery
54. Which one of the following does not produce cyanosis 63. True statement about ductus arteriosus is:
in the first year of life? M  (AIIMS May 03)
 (AIPGMEE 2000)

a. Atrial septal defect a. It undergoes anatomic closure within 24 hours of birth


b. Hypoplastic left heart syndrome b. Forms the ligamentum venosum in later life
c. Truncus arteriosus c. It is induced to close by high levels of prostaglandins
d. Double outlet right ventricle d. May cause a machinery murmur by its patency.
55. A 4-year-old girl always had to wear warm socks even in 64. Coarctation of the aorta is common in which syndrome?
 (AIPGMEE 95)
summer season. On physical examination, it was noticed
that she had high blood pressure and her femoral pulse a. Down's b. Turner's
was weak as compared to radial and carotid pulse, a c. Klinefelter's d. Noonan's
chest radiography showed remarkable notching of ribs
along with their lower borders. This was due to: � CYANOTIC CONGENITAL HEART DISEASES
a. Femoral artery thrombosis (AIIMS Nov 02)
65. All of the following statements are true regarding TOF,
b. Coarctation of aorta
except: (FMGE Nov 2017)
c. Raynaud’s disease
d. Takayasu arteritis a. Right ventricular hypertrophy
56. A child with large perimembranous VSD has congestive b. Atrial septal defect
heart failure. What may be the cause of improvement of c. Overriding of aorta d. Subpulmonic stenosis
cardiac failure in the patient? (AIIMS Nov 01) 66. Single second heart sound (S2) is heard in:
a. Aortic regurgitation a. Transposition of great vessels (FMGE June 2018)
b. Vascular changes in pulmonary circulation b. Ebstein's anomaly
c. Infective endocarditis c. Tetralogy of Fallot d. Eisenmenger's syndrome
d. Closure of VSD spontaneously 67. True about Pott shunt: (JIPMER May 2017)
57. ASD is seen in all except:  (PGI June 01) a. Ascending aorta and right pulmonary artery
a. Turner’s syndrome b. Ellis-Van Creveld syndrome b. Left pulmonary artery with descending aorta
c. Down’s syndrome d. Holt-Oram syndrome c. Subclavian artery and pulmonary artery
e. TAR syndrome d. Superior vena cava and right pulmonary artery
296 Section 3: Systemic Pediatrics

78. WPW syndrome is associated with:


Review of Pediatrics and Neonatology

68. What is the treatment for sudden worsening of TOF? 


 (Recent Question 2013)
(Recent Question 2017)
a. Ebstein anomaly b. TOF
a. Morphine b. O2 therapy c. VSD d. TAPVC
c. Propranolol d. All of the above 79. A 2 weeks baby with central cyanosis has grade II systolic
69. Most common type of TAPVC: M  (Recent Question 2017) murmur, normal S1, single S2, plethoric lung. What is the
a. Supracardiac b. Cardiac diagnosis? (Recent Question 2013)
c. Infracardiac d. Mixed a. TGA b. TAPVC
70. Patent truncus arteriosus leads to: c. TOF d. Pulmonary atresia
a. Both ventricular hypertrophy (Recent Question 2017) 80. Pulmonary plethora is not seen in: M (JIPMER 2012)
b. Pulmonary oligemia a. VSD b. TOF
c. Ductus dependent flow d. CCF at birth c. Truncus arteriosus d. TAPVC
71. In a child with tetralogy of Fallot with fever and 81. Reversal of shunt is not possible in natural history of? M
diarrhoea, which of the following is the surest sign of a 
cyanotic spell? (AIIMS May 2016) a. ASD b. VSD (AIIMS Nov 2012)
a. Arterial oxygen saturation of less than 75% c. TOF d. PDA
b. S3 gallop rhythm 82. Most common cyanotic heart disease: M 
 (Recent Question 2012)
c. Absence of murmur d. Hepatomegaly
72. All are seen in tetralogy of Fallot except? M  a. TOF b. VSD
c. TAPVC d. Ebstein anomaly
 (Recent Question 2015, 2013)
83. Ebstein anomaly is due to maternal intake of:
a. VSD b. RVH  (Recent Question 2012)
c. ASD d. PS
a. Lithium b. Copper
73. Which of the following represents the additional c. Cobalt d. Nickel
component in pentalogy of Fallot? (Bihar PG 2015)
84. Double arch of aorta is seen in: (Recent Question 2012)
a. Atrial Septal Defect (ASD) a. CATCH22 b. Di George syndrome
b. Ventricular Septal Defect (VSD) c. Velo cardiofacial syndrome
c. Right ventricular Hypertrophy (RVH) d. All of the above
d. Pulmonic Stenosis 85. Cyanotic congenital heart disease with LA enlargement
74. A child comes with cyanotic spells and chest X-ray was and left axis deviation? (Recent Question 2012)
as shown below; What is the most probable diagnosis? a. Tricuspid atresia b. TOF
 (Recent Question 2015, 2013) c. TAPVC d. Ebstein anomaly
86. Ductus arteriosus dependent blood flow is mandatory in
all of the following congenital heart diseases except:
 (WBPG 2011, AIIMS May 2011, AIIMS May 05, 02)
a. Truncus arteriosus
b. Hypoplastic left heart syndrome
c. TGA with Intact ventricular septum
d. Obliterated aortic arch
87. A child is admitted on 7th days of life with severe
respiratory distress and shock. He was discharged 2 days
a. Tetralogy of Fallot b. Tricuspid atresia back healthy. What could be the probable diagnosis?
c. Pulmonary atresia with intact ventricular septum a. VSD large (AIIMS May 2011)
d. TAPVC b. Hypoplastic left heart syndrome
75. All are characteristic of Tetralogy of Fallot except: M  c. Ebstein anomaly d. AP window defect
 (Recent Question 2015, DPG 2009) 88. Which of the following is a component of pentalogy of
a. Infundibular stenosis Fallot: (AIPGMEE 2011, 07; AIIMS May 10)
b. VSD a. Atrial Septal Defect (ASD)
c. Overriding of aorta b. Patent Ductus Arteriosus (PDA)
d. Left ventricular hypertrophy c. Coarctation of Aorta (COA)
d. Left Ventricular Hypertrophy (LVH)
76. All of the following are acyanotic congenital heart 89. A child with tetralogy of Fallot uses which of the
diseases except: (Recent Question 2014) following positions? M  (DNB Jun 2010)
a. VSD b. PDA a. Supine b. Prone
c. ASD d. Tetralogy of Fallot c. Squatting d. Leaning forwards
77. Differential cyanosis occurs in which disease? 90. Which one of the following is a cyanotic congenital
a. Severe coarctation  (Recent Question 2013) heart disease? (UPSC-II 10)
b. Interrupted aortic arch a. Patent ductus arteriosus b. Atrial septal defect
c. PPHN c. Ventricular septal defect
d. All of the above d. Tetralogy of Fallot
Chapter 13: Pediatric Cardiology 297

91. True about TOF: (PGI Nov 09) sound. Liver was not enlarged. What would be the likely

Questions
a. Recurrent chest infection b. Clubbing of hands and feet diagnosis? (UPSC 07)
c. Squatting d. Cyanotic spells a. Congenital methemoglobinemia
e. CHF not seen b. Eisenmenger syndrome
92. A newborn baby develops cyanosis on day three c. Aortic stenosis
of life. On auscultation, there is a systolic murmur. d. Tetralogy of Fallot
Echocardiography reveals a cyanotic heart disease in 101. True about TGA: (PGI Dec 07)
the baby. Which one of the following drugs can be a. Cyanosis at birth b. CHF
administered to prolong the life of the baby pending
c. VSD d. AS
intervention? (UPSC-I 09)
e. None
a. Indomethacin b. Ibuprofen
c. Prostaglandin E1 d. Propanolol 102. A 7-day-old baby presented in emergency department
with unconsciousness, cyanosis and 85% oxygen
93. A two-month-old infant is brought to the hospital emergency
saturation. The diagnosis:  (PGI Dec 07)
with marked respiratory distress. On examination, the
infant has cyanosis and bilateral crepitations. Heart rate a. Tetralogy of Fallot b. TGA
is 180/min, respiratory rate 56/min and the liver span c. TAPVC d. PDA
7.5 cm. The child has had repeated episodes of fever, e. Ebstein anomaly
cough and respiratory distress since the time of birth. 103. The following features are true for tetralogy of Fallot,
Cardiovascular examination reveals a grade III ejection except: (AIPGMEE 06)
systolic murmur in left parasternal area and the chest X-ray a. Ventricular septal defect
reveals cardiomegaly with a narrow base and plethoric
b. Right ventricular hypertrophy
lung fields. What is the most likely diagnosis?
c. Atrial septal defect
a. Congenital methemoglobinemia  (UPSC-1 09)
d. Pulmonary stenosis
b. Transposition of great arteries
104. Blalock-Taussig shunt is done between: (AIPGMEE 06)
c. Cystic fibrosis
d. Tetralogy of Fallot a. Aorta to pulmonary artery
94. Right aortic arch is most commonly associated with: b. Aorta to pulmonary vein
 (AIPGMEE 08) c. Subclavian artery to pulmonary artery
a. Tetralogy of Fallot b. Truncus arteriosus d. Subclavian vein to artery
c. Corrected TGA 105. All of the following are true regarding tetralogy of Fallot
d. Total anomalous pulmonary venous connection except: (AIIMS May 05)
95. Systolic murmur in TOF is due to? M (APPG 08) a. Ejection systolic murmur in second intercostal space
a. VSD b. Pulmonary stenosis b. Single second heart sound
c. ASD d. None c. Predominantly left to right shunt
96. “Egg on side appearance” is seen in: (UP PG 08) d. Normal jugular venous pressure
a. Fallot’s tetralogy 106. Not a feature of Fallot’s tetralogy: M (MAHA 05)
b. Transposition of great vessels a. Left ventricular hypertrophy
c. Ebstein’s anomaly d. Ventricular septal defect
b. Boot shaped heart
97. In which of the following differential cyanosis is
c. VSD
found?
d. Overriding of arch of aorta
a. VSD with reversal of shunt  (UPSC-I 08)
107. Recurrent respiratory tract infections may occur in all of
b. PDA with reversal of shunt
the following except:  (AIIMS Nov 05)
c. ASD with reversal of shunt
d. Tetralogy of Fallot a. Ventricular septal defect
98. All of the following about PDA are true except: b. Tetralogy of Fallot
a. More common in males (AIPGMEE 2008) c. Transposition of great arteries
b. Common heart lesion in rubella d. Total anomalous venous return
c. Treatment is closure of defect by ligation and division of 108. Obstruction in pulmonary stenosis may occur at the
ductus following sites: (MAHA 05)
d. Hypoxia and immaturity are important in maintaining the a. Supravalvular b. Valvular
patency c. Subvalvular d. All of the above
99. Essential criteria for TOF includes all except: M 109. The most common type of total anomalous pulmonary
 (AIIMS Nov 2007) venous connection is:  (AIPGMEE 05)
a. Valvular stenosis b. Infundibular stenosis a. Supracardiac b. Infracardiac
c. Over riding of aorta d. RVH c. Mixed d. Cardiac
100. A two-year-old boy presented with episodes of becoming 110. The following cardiac defects are characterized by ductus
dusky. On examination, there was central cyanosis and dependent blood flow except: (AIIMS May 05, 02)
clubbing. There was no pallor, oedema or respiratory
distress. The heart was normal sized with a parasternal a. Transposition of great arteries with intact septum
heave. A systolic thrill was palpable over the left middle b. Interrupted aortic arch
sternal border. First heart sound was normal and only c. Truncus arteriosus
the aortic component was audible in the second heart d. Hypoplastic left heart syndrome
298 Section 3: Systemic Pediatrics

111. All of the following are acyanotic congenital heart a. Tricuspid atresia b. Eisenmenger syndrome
Review of Pediatrics and Neonatology

diseases except: M  (SGPGI 05) c. Tetrology of Fallot


a. VSD b. PDA d. Total anomalous pulmonary venous drainage
c. ASD d. Tetralogy of Fallot 122. Pulmonary plethora is seen in all except: M 
112. In which of the following a ‘Coeur en Sabot’ shape of the a. VSD b. ASD (AIPMEE 2000)
heart is seen: (AIPGMEE 04) c. Fallot’s tetralogy d. PDA
a. Tricuspid atresia b. Ventricular septal defect 123. A child with VSD presents with development of cyanosis
c. Transposition of great arteries because of Eisenmenger physiology. What is the correct
d. Tetralogy of Fallot sequence of events which leads to this change?
113. A 6-month-old child with Tetralogy of Fallot develops  (AIIMS Nov 2000)
cyanotic spell initiated by crying. Which one of the a. Left to right shunt, pulmonary hypertension, right ventri­
following drugs you would like to avoid?
cular hypertrophy, right to left shunt.
 (AIIMS Nov 04)
b. Left to right shunt, right ventricular hypertrophy, pulmo­
a. Sodium bicarbonate b. Propranolol nary hypertension, right to left shunt.
c. Phenylephrine d. Isoprenaline
c. Pulmonary hypertension, right to left shunt, right ventri­
114. Which of the following are true about Tetralogy of
cular hypertrophy, left to right shunt.
Fallot? (PGI June 04)
d. Left to right shunt, right ventricular hypertrophy, right to
a. VSD b. Lt. Ventricular hypertrophy left shunt, pulmonary hypertension.
c. Lt. Axis deviation
124. Definitive treatment of TGV:  (AIPGME 1999)
d. Taussig-Blalock shunt is between pulmonary and subcla­
vian artery a. Arterial switch b. Atrial switch
e. Morphine is given for cyanotic spell c. Atrial septostomy d. Medical management
115. A five-day-old, full term male infant was severely
cyanotic at birth. Prostaglandin E1 was administered � RHEUMATIC HEART DISEASE
initially and later balloon atrial septosomy was done
which showed improvement in oxygenation. The most
125. Modified Jones’s criteria includes all the following as
likely diagnosis of this infant is: (AIPGMEE 04)
major criteria except: M  (Recent Question 2017)
a. Tetralogy of Fallot
a. Subcutaneous nodules b. Arthritis
b. Transposition of great vessels
c. Fever d. Carditis
c. Truncus arteriosus d. Tricuspid atresia
126. All of the following are true regarding the jones criteria
116. Cyanosis is seen in:  (PGI Dec 01)
as revised in 2015 by the American heart association,
a. Patent ductus arteriosus b. Tricuspid atresia except: (Recent Question 2017)
c. Ostium primum ASD d. Eisenmenger complex a. Describes 5 Major criteria, 4 minor criteria and a require­
e. Tetralogy of Fallot ment of evidence of recent group A streptococcal
117. Pott’s shunt is:  (AIPGMEE 01) infection
a. Rt subclavian artery to right pulmonary artery b. Diagnosis of recurrent of acute rheumatic fever in
b. Descending aorta to left pulmonary artery Moderate/high risk population requires 2 minor criteria
c. Left subclavian to left pulmonary artery and evidence of preceding GAS infection
d. Ascending aorta to right pulmonary artery c. Has separate criteria for low risk and moderate/high risk
118. A patient with cyanosis presents with LVH and pulmonary population
complications. ECG, shows left axis deviation. Most likely d. Acute rheumatic fever is diagnosed in the presence of 2
diagnosis is: (AIPGMEE 01) major or 1 major and 2 minor criteria with evidence of
a. TOF b. Tricuspid atresia preceding GAS infection
c. TAPVC d. VSD 127. Which of the following is a major criteria in the diag­
119. A neonate has central cyanosis and short systolic murmur nosis of rheumatic fever? M  (Recent Question 2015)
on the 2nd day of birth. The diagnosis is: a. Erythema nodosum b. Erythema multiforme
a. Tetralogy of Fallot  (AIIMS May 01)
c. Erythema toxicum d. Erythema marginatum
b. Transposition of great vessels 128. A girl child comes with complaints of involuntary move­
c. Atrial septal defect d. Ventricular septal defect ments and is diagnosed as a case of Sydenham chorea,

120. A newborn presents with cyanosis. The X-ray chest suggestive of a diagnosis of acute rheumatic fever. There
reveals oligemic lung fields and normal sized heart. is no complaint of carditis or arthritis. Throat culture is
Most likely diagnosis is: (AIIMS Nov 01) negative. Which of the following will most likely suggest
a. Ebstein’s anomaly b. Pulmonary atresia recent streptococcal infection? (AIIMS Nov 2014)
c. Transposition of great arteries a. Antistreptolysin S b. ASLO 
d. Tetralogy of Fallot c. PCR for M protein d. Antihyaluronidase

121. A five-year-old child presents with left ventricular 129. To prevent acute rheumatic fever, acute pharyngitis
hypertrophy and central cyanosis; What is the most due to group A streptococci should be treated with
probable diagnosis:  (AIIMS Nov 2000) antibiotics before: M  (JIPMER 2014)
Chapter 13: Pediatric Cardiology 299

a. 7 days of illness b. 8 days of illness � HEART FAILURE

Questions
c. 9 days of illness d. 10 days of illness
130. All are minor Jones criteria except: M  143. In infants, CHF (congestion heart failure) is diagnosed
by: (Recent Question 2012)
 (Recent Question 2013)
a. Raised JVP b. Pedal edema
a. Arthralgia b. Fever c. Tender hepatomegaly d. Basal crepts
c. Prolonged PR interval d. Chorea 144. Max limit of cardiothoracic ratio in children:
131. Most common cause of acquired heart disease in a. 0.40 b. 0.45  (JIPMER 2010)
children: M  (TN PGMEE 2013) c. 0.50 d. 0.55
a. Kawasaki disease b. Ischaemic heart disease 145. Causes of heart failure in newborn are all except:
c. Viral myocarditis d. Acute rheumatic fever  (WBPG 2008)
132. Major criteria for rheumatic fever are all except: M a. ASD b. TGA with large VSD
 (AIPGMEE 2011, 2009, 2007) c. Hypoplastic left ventricle d. Baby of diabetic mother
146. Which of the following symptoms of rheumatic fever
a. Carditis b. Erythema marginatum
may be the only isolated manifestation? (JIPMER 2008)
c. Migratory polyarthrits d. Increased CRP
a. Carditis b. Arthritis
133. Which is not a major criteria of Jones in rheumatic
c. Chorea d. Subcutaneous nodules
fever? M  (AIIMS Nov 10)
147. Commonest cause of heart failure in infancy is: M 
a. Pancarditis b. Arthritis  (COMED 08)
c. Chorea d. Elevated ESR a. Myocarditis b. Rheumatic fever
134. A 7-year-old child with rheumatic heart disease presents c. Cardiomyopathy d. Congenital heart disease
with pallor, fever and a palpable spleen. The following
investigations would be needed to arrive at a diagnosis � OTHER HEART DISEASES
except: (UPSC-I 10)
a. Electrocardiogram b. Echocardiogram 148. Causes of wide fixed splitting of second heart sound
c. Blood culture d. Urine examination include all except: (Recent Question 2017)
135. Not a minor criteria for acute rheumatic fever: M  a. Atrial septal defect b. Ebstein anomaly
 (WBPG 2009, WBPG 2007)
c. Total anomalous pulmonary venous return
d. Right bundle branch block
a. Arthralgia b. Increased ESR
149. What is the minimal internal diameter of Giant coronary
c. Increased PR interval d. ASO titer aneurysm in children with Kawasaki disease which
136. Least common cause of pericarditis/pericardial effusion pose highest risk for ruptures, thrombosis. Stenosis and
in children: (WBPG 2008) myocardial infarction? (Recent Question 2017)
a. Rheumatic fever b. Sarcoidosis a. 8 mm b. 10 mm
c. Rheumatoid arthritis d. SLE c. 12 mm d. 15 mm
137. Major criteria for Rheumatic fever: M  (TN PGMEE 2008) 150. The V wave of jugular venous pulses is produced due
a. Fever b. Erythema marginatum to: (Recent Question 2017)
c. ECG changes d. Positive throat culture a. Atrial systole preceding tricuspid valve closure
b. Descent of the tricuspid valve ring
138. Which of the following is a minor criteria for diagnosis
c. Closure of tricuspid valve
of rheumatic fever (RF) according to modified Jones d. Passive filing of atrium during ventricular systole
criteria: (AIPGMEE 07)
151. What is the approximate Total fetal cardiac output?
a. ASO titer  (Recent Question 2017)
b. Past history of rheumatic fever a. 300 ml/kg/min b. 450 ml/kg/min
c. Fever d. Subcutaneous nodules c. 600 ml/kg/min d. 150 ml/kg/min
139. Drug of choice for rheumatic fever prophylaxis in 152. Which of the following drugs is not given in dilated
penicillin allergic patient:  (AIIMS May 2007) cardiomyopathy? (Recent Question 2017)
a. Erythromycin b. Clindamycin a. Beta blocker b. Calcium channel blocker
c. Vancomycin d. Gentamycin c. Spironolactone d. ACE inhibitors
140. In a patient of rheumatic carditis, full dose of steroid is 153. Congenital heart disease associated with pre-excitation
given for: (Kerala 04) is: (Recent Question 2017)
a. 3 weeks b. 6 weeks a. Atrial septal defect b. Bicuspid aorta valve
c. 9 weeks d. 12 weeks c. Ebstein anomaly d. Patent ductus arteriosus
141. True statement about rheumatic fever in children: 154. A very large ventricular septal defect should be operated:
a. In school going age  (Recent Question 2017)
a. Polyarthritis (PGI Dec 03)
b. Only if CHF is uncontrolled
b. Caused by alpha hemolytic streptococci c. During infancy d. Soon after birth
c. Erythema marginatum is most common manifestation
155. The most common congenital cardiac defect associated
d. MC valve involvement is Mitral with sudden death in children is: (Recent Question 2017)
e. Erythema marginatum is common in face
a. Valvular aortic stenosis
142. Commonest cause of enlarged cardiac shadow in X-ray b. Origin of the left coronary artery from the
of a child is: M  (Karn. 2000) right sinus of Valsalva
a. PDA b. Coarctation of aorta c. Hypertrophic cardiomyopathy
c. Pericarditis d. Rheumatic carditis d. Congenital mitral regurgitation
300 Section 3: Systemic Pediatrics

163. Eisenmenger’s syndrome—True are all except:


Review of Pediatrics and Neonatology

156. A 3-month-old child has a typical mewing cry and a


 (AIIMS May 2010, AIPGMEE 2005)
congenital heart disease. The most probable chromo­
somal abnormality is: (Recent Question 2016) a. Pulmonary veins are not distended
b. RV and LV walls come back to normal size
a. 5q- b. 5p-
c. Dilatation of central pulmonary artery
c. 4p- d. 9p- d. Peripheral pruning of pulmonary arteries
157. Choose the TRUE statement regarding the disease 164. All are signs of impending Eisenmenger syndrome
depicted in the images below:  (APPG 2016) except: (AIIMS May 2010)
a. Increased flow murmur across tricuspid and pulmonary
valve
b. Single S2
c. Loud P2 d. Graham-Steel murmur
165. Common cardiac tumor in childhood is/are:
 (PGI Dec 07)

a. Rhabdomyoma b. Lymphoma
a. IV immunoglobulin is used in treatment c. Atrial myxoma d. Sarcoma
b. Bilateral purulent conjunctivitis e. Fibroma
c. Lymphadenopathy is rare 166. About Carey Coombs murmur, which is false:
d. Aspirin is contraindicated a. Delayed diastolic murmur  (AIIMS Nov 06)
158. Superior rib notching is seen in all except? b. Seen in rheumatic fever
c. Associated with AR d. Low pitched murmur
a. Osteogenesis imperfecta  (WB PGMEE 2016)
167. An 8 years old child presented with altered sensorium
b. Neurofibromatosis and seizures. On examination, BP was 180/120. Correct
c. Marfan’s syndrome statements are:  (PGI Dec 04)
d. Coarctation of aorta a. Sodium nitroprusside drip
159. Match List I with List II and select correct answer using b. Cause of hypertension is essential hypertension
code given below the lists: (UPSC CMS 2015) c. IV labetolol, hydralazine, and diazoxide can be used
List I List II d. Nifedipine is used
A. Wide fixed split of S2 1. Pulm hypertension e. Pheochromocytoma mimics the condition
168. A 2-year-old child presented with sudden onset of
B. Continuous machinery murmur 2. ASD
altered sensorium. On examination, BP was 200/100;
C. Muffled heart sounds 3. PDA Which of the following can be the underlying cause?
D. Wide variable split of S2 4. Pericard. effusion  (PGI Dec 04)
Code: A B C D a. Renal artery stenosis b. Coarctation of aorta
a. 2 4 3 1 b. 2 3 4 1 c. Glomerulonephritis d. Essential hypertension
e. Pheochromocytoma
c. 1 4 3 2 d. 1 3 4 2
169. Children born to mothers with systemic lupus erythema­
160. Cardiomyopathy is not a feature of?
tosus are likely to have one of the following anomalies:
 (Bihar PG 2015, AIPMEE 2007)  (Karnataka 03)
a. Duchenne muscular dystrophy a. Atrial septal defect b. Tetralogy of Fallot
b. Friedreich's ataxia c. Transposition of great vessels
c. Pompe disease d. Lowe's syndrome d. Complete heart block
161. Infective endocarditis is rarely seen in: M  170. Pulsatile varicose veins in lower limbs is seen in:
a. VSD b. ASD (WBPG 2015) a. Klippel-Trenaunay syndrome  (AIIMS 2001)
c. PDA d. RHD b. TR
c. RV failure
162. An infant presents with cardiac failure. Examination d. Carcinoid stenosis of tricuspid
reveals a weaker femoral pulse when compared to the 171. Lutembacher’s syndrome comprises of:
radial pulse. What is the probable diagnosis? a. Ventricular septal defect with aortic stenosis
 (JIPMER 2013) b. Ruptured sinus of valsalva aneurysm
a. PDA b. Coarctation of aorta c. Atrial septal defect with mitral stenosis
c. Aortoilliac vasculitis d. VSD d. Complete common atrioventricular canal
Chapter 13: Pediatric Cardiology 301

Answers with Explanations


Answers with Explanations
 FETAL CIRCULATION
1. b. Umbilical vein  Ref: Nelson’s 20/e p 2161-2162, Ghai 8/e p 402
Blood oxygenated in placenta returns via umbilical vein, which enter the fetus at the umbilicus and joins portal vein
2. a. 1 vein and 2 arteries  Ref: Nelson’s 20/e p 2161, Ghai 8/e p 402
Normally Umbilical cord has 1 vein 2 arteries; A single umbilical artery increases risk for occult renal anomaly;
3. b. Umbilical vein  Ref: Nelson’s 20/e p 2161, Ghai 8/e p 402

 ACYANOTIC CONGENITAL HEART DISEASES


4. c. Pulmonary stenosis  Ref: Nelson’s 20/e p 622
Most common cardiac abnormality in Noonan syndrome is Pulmonary stenosis.
5. a. Falsely increased BP  Ref: Nelson’s 20/e p 2164
In older children, a mercury sphygmomanometer with a cuff that covers approximately two-thirds of the upper part of the arm or leg
may be used for blood pressure measurement. A cuff that is too small results in falsely high readings, whereas a cuff that is too large
records slightly decreased pressure.
6. a. Closure of ductus in premature neonate  Ref: Nelson’s 20/e p 2197
Indomethacin, a Prostaglandin inhibitor is used in medical closure of Ductus arteriosus in preterm neonates with PDA.
7. a. Increased O2 tension  Ref: Nelson’s 20/e p 2162
In the course of several days after birth, the high arterial Po2 constricts and eventually closes the ductus arteriosus, which eventually
becomes the ligamentum arteriosum
8. c. Left lower sternal border   Ref: Nelson 20/e p 2170
•• MC innocent murmur is a medium-pitched, vibratory or “musical,” short systolic ejection murmur, best heard along left lower &
mid-sternal border & has no significant radiation to the apex, base, or back.
•• It is heard most frequently in children between 3 and 7 years of age.
•• Intensity of the murmur often changes with respiration and position and may be attenuated in the sitting or prone position.
9. a. Coarctation of aorta  Ref: Nelson’s 20/e p 2189-2191
Most common cardiac defect in Turner syndrome is bicuspid aortic valve, followed by coarctation of aorta.
10. b. VSD with pulmonary hypertension & c. Pulmonary vascular obstructive disease with VSD  (Ref: Stedman's Medical
Eponyms-Page 212)
Eisenmenger complex: The combination of ventricular septal defect with pulmonary hypertension and consequent right-to-left shunt
through the defect
11. b. 2:1  Ref: Nelson’s 20/e p 2190-2191
12. c. Surgery is electively done at 1-3 year age for large defects  Ref: Nelson’s 20/e p 2190-2191
The timing for elective closure of ASD is usually after the 1st yr and before entry into school.
13. d. Abnormal X-ray  Ref: Nelson’s 20/e p, Ghai 8/e p 406; Refer pretext for details of Nada’s criteria
14. a PGE1  Ref: Nelson’s 20/e p 2189-2191, Ghai 8/e p 413-414
PGE1 or Alprostadil can be used to keep the ductus arteriosus open in Ductus dependent heart diseases.
15. a. ASD  Ref: Nelson’s 20/e p 2189, Ghai 8/e p 413
Cardiac anomalies in congenital rubella are PDA (most common) > PS > VSD > ASD (least associated).
16. b. VSD  Ref: Nelson’s 20/e p 2191-2194, Ghai 8/e p 415-416
Most common congenital heart disease is & Most common congenital heart disease affected by infective endocarditis is are VSD
17. a. Coarctation of aorta  Ref: Nelson’s 20/e p 2205
Notching of the inferior border of 3rd-9th ribs from pressure erosion by enlarged collateral vessels, is usually seen beyond the
age of 3 years, in a child with coarctation of aorta.
18. c. Bicuspid aortic valve  Ref: Nelson’s 20/e p 2205, Ghai 8/e p 429-431
Coarctation of aorta is associated with a bicuspid aortic valve in more than 70% of patients
19. b. Subvalvular aortic stenosis  Ref: Nelson’s 20/e p 2203, Ghai 8/e p 431-433
Supravalvular aortic stenosis is usually seen in William syndrome and not subvalvular.
20. c. Ostium secundum  Ref: Nelson’s 20/e p 2189-2191, Ghai 8/e p 413-414; Refer pretext for details
21. a. It is found in ASD, b. More commonly associated with ASD than VSD & c. May be associated with Down syndrome 
Ref: Nelson’s 20/e p 2189-2192
Left axis deviation is seen in ostium primum ASD due to RV conduction defect.
302 Section 3: Systemic Pediatrics

Ostium primum type ASD


Review of Pediatrics and Neonatology

•• It is situated in the lower portion of atrial septum and overlies mitral and tricuspid valves.
•• In most instances, a cleft in anterior leaflet of mitral valve is also noted; Ventricular septum is intact.
•• Tricuspid valve is usually functionally normal, but anatomic abnormality of septal leaflet is present.
22. d. All of above  Ref: Nelson’s 20/e p 2189-2191, Ghai 8/e p 413-414
Complications of large PDA are Cardiac failure; Infective endarteritis, Pulmonary or systemic emboli, Pulmonary hypertension
(Eisenmenger syndrome).
23. a. PGE1  Ref: Nelson’s 20/e p 2189-2191, Ghai 8/e p 413-414
24. b. VSD  Ref: Nelson’s 20/e p 2189-2191, Ghai 8/e p 413-414
25. a. ASD  Ref: Nelson’s 20/e p 2189-2191, Ghai 8/e p 413-414
Holt-Oram syndrome is characterized by skeletal abnormalities of hands and arms (upper limbs) and heart problems like ASD;
Autosomal dominant inheritance seen.
26. a. RA + RV  Ref: Nelson’s 20/e p 2189-2191, Ghai 8/e p 413-414
In ASD (ostium primum as well as secundum), overload of Right atrium and ventricle occurs.
27. c. 350 ml/kg body wt  Ref: Nelson’s 20/e p 2189-2191, Ghai 8/e p 413-414
•• Total fetoplacental blood volume at term will be equivalent to cardiac output of newborn
•• Cardiac output of a term newborn and the blood going through systemic circulation is 350 ml/kg/minute.
28. b. Bicuspid aortic valve   Ref: Nelson’s 20/e p 2189-2191, Ghai 8/e p 413-414
Most common heart defects in Turner syndrome are bicuspid aortic valves, coarctation of aorta and aortic stenosis.
29. c. Bicuspid aortic valve  Ref: Nelson’s 20/e p 2189-2191, Ghai 8/e p 413-414; Ref. Nelson’s 20/621
30. b. ASD with ostium primum  Ref: Nelson’s 20/e p 2189-2191; Down Syndrome by Mark Selikowitz p 87
It is a controversial question; There is no doubt about the most common cardiac lesion in Down syndrome, confusion is about
the 2nd most common disease… this is what most references say (but obviously, not very clearly)..
Most common heart disease in Down syndrome is: Atrioventricular septal defect > Ostium primum ASD (partial AVSD)
> VSD > Ostium Secundum ASD
31. a. Down syndrome  Ref: Nelson’s 20/e p 2189-2191, Ghai 8/e p 413-414
32. a. ASD  Ref: Nelson’s 20/e p 2189-2191, Ghai 8/e p 413-414
Auscultatory findings in ASD are Wide fixed splitting of S2, Shunt remains silent, Ejection systolic murmur in pulmonary area,
Mid-diastolic murmur in tricuspid area.
33. a. Infective endocarditis  Ref: Nelson’s 20/e p 2189-2191, Ghai 8/e p 413-414
Infective endocarditis is usually not seen in ASD, as the gradient between right and left atria is very small in ASD.
34. b. Aorta and pulmonary artery  Ref: Nelson’s 20/e p 2189, Ghai 8/e p 413
Ductus arteriosus is a connection between the aorta and pulmonary artery.
35. a. Narrow pulse pressure  Ref: Nelson’s 20/e p 2197-2199, Ghai 8/e p 417-420
In PDA, there is shunting of blood from systemic circulation to pulmonary circulation, which leads to:
congestive cardiac failure, “wide pulse pressure”, “bounding pulses”
Overloading of pulmonary vasculature pulmonary edema/hemorrhage and CO2 retention
Blood flow to kidney & GIT is compromised → acute renal failure; necrotizing enterocolitis & metabolic acidosis.
36. b. VSD  Ref: Nelson’s 20/e p 2191, Ghai 8/e p 414-417
37. d. Infective endocarditis  Ref: Nelson’s 20/e p 2191, Ghai 8/e p 414-417;
38. a. right atrium → right ventricle → left atrium  Ref: Nelson’s 20/e p 2191, Ghai 8/e p 414-417
39. b. Initial resolution of symptoms  Ref: Nelson’s 20/e p 2191, Ghai 8/e p 414-417
•• Initial resolution of symptoms can be seen in a child with VSD, when pulmonary hypertension develops;
•• This is so because pulmonary hypertension poses resistance to blood flowing to lungs → lesser blood goes to lungs;
•• Ultimately, when Eissenmenger syndrome develops, cyanosis and clubbing appears.
40. d. Coarctation of aorta  Ref: Nelson’s 20/e p 2205, Ghai 8/e p 431-433
Isolated upper extremity hypertension (200/140 mmHg), together with absent/diminished femoral pulses is a characteristic
feature of coarctation of aorta.
41. d. CO2 washout  Ref: Nelson’s 20/e p 2205, Ghai 8/e p 431; Refer Ans 29 above;
42. b. Narrow splitting of 2nd heart sound, c. Lt axis deviation in ECG, d. Shunt murmur prominent  Ref: Nelson’s 20/e p 2189,
Ghai 8/e p 413
•• In ASD S2 is fixed and wide split (not narrow split); Ostium secundum ASD is associated with Right axis deviation
•• Pressure difference between 2 atria is small, blood passes at a narrow pressure difference → So, no shunt murmur.
43. d. Absence of flow murmurs over scapular region  Ref: Nelson’s 20/e p 2189, Ghai 8/e p 413
44. b. Indomethacin therapy  Ref: Nelson’s 20/e p 2197, Ghai 8/e p 417
Indomethacin, a Prostaglandin inhibitor is used in medical closure of Ductus arteriosus.
45. a. VSD, b. PDA, d. Coarctation of aorta, e. Tricuspid stenosis  Ref: Nelson’s 20/e p 2211-2217
A systolic murmur can be heard in all the above heart diseases; PDA usually produces a continuous murmur, but may sometimes
present with a systolic murmur in the initial period of life. Out of the above diseases, TOF is a cyanotic heart disease.
46. d. Right atrium  Ref: Nelson’s 20/e p 221, Ghai 8/e p 420
Chapter 13: Pediatric Cardiology 303

47. b. Ostium secondum  Ref: Nelson’s 20/e p 2189, Ghai 8/e p 413

Answers with Explanations


48. d. Pedal edema  Ref: Nelson’s 20/e p 2282-2287, Ghai 8/e p 396-400; Refer pretext for details;
49. b. Coarctation of aorta  Ref: Nelson’s 20/e p 2205, Ghai 8/e p 413-433
•• Feeble femoral pulsations compared to strong brachial pulsations suggests of coarctation of aorta.
50. b. Atrial septal defect  Ref: Nelson’s 20/e p 2189, Ghai 8/e p 413
•• ASD is associated with an enlarged right atrium and right ventricle to accommodate large volume of blood
•• Left atrium is not enlarged in ASD, because it decompresses itself by shunting its blood to right atrium.
51. d. Prostaglandin inhibitors  Ref: Nelson’s 20/e p 2189, Ghai 8/e p 431
52. c. Holt-Oram syndrome  Ref: Nelson’s 20/e p 746, Ghai 8/e p 413
Discussing about the options one by one,
Name of the syndrome Clinical features
a. Lesch-Nyhan syn- Hyperuricemia, intellectual disability, dystonia, choreoathetosis, spasticity, dysarthric speech and compulsive
 drome self-biting, usually beginning with eruption of teeth
b. Rasumussen syn- It is an immune mediated disease associated with intractable unilateral seizures, progressive hemiparesis and
 drome intellectual dysfunction
c. Holt-Oram syndrome ASD & bony abnormalities, with a strong familial tendency (autosomal dominant inheritance)
d. LEOPARD syndrome Lentigines (multiple), Electrocardiographic conduction abnormalities, Ocular hypertelorism, Pulmonary
stenosis, Abnormalities of genitalia, Retardation of growth, Deafness
53. c. Ostium primum atrial septal defect  Ref: Nelson’s 20/e p 2189, Ghai 8/e p 413
•• Wide fixed splitting of S2 in association with ejection systolic murmur suggests the diagnosis of atrial septal defect
•• The presence of left axis deviation in A.S.D., suggests ostium primum type of defect.
54. a. Atrial septal defect  Ref: Nelson’s 20/e p 2189, Ghai 8/e p 413
Apart from ASD, rest of the 3 heart diseases are cyanotic heart diseases.
55. b. Coarctation of aorta  Ref: Nelson’s 20/e p 2205, Ghai 8/e p 431
Hypertension, Weak femoral pulses and notching of inferior margin of ribs point towards Coarctation of aorta.
56. b. Vascular changes in pulmonary circulation  Ref: Nelson’s 20/e p 2191-2194, Ghai 8/e p 414-417
Spontaneous closure of a large membranous VSD is unlikely; Vascular changes in pulmonary circulation & pulmonary hypertension
can lead to improvement of cardiac failure, but may subsequently lead to cyanosis & clubbing.
57. a. Turner’s syndrome  Ref: Nelson’s 20/e p 2189; For Mnemonic on syndromes associated with ASD, refer pretext
58. d. Anomalous coronary artery  Ref: Nelson’s 20/e p 2227-2238, Ghai 8/e p 426-427
Heart failure in a neonate with recurrent attacks of discomfort, restlessness, irritability, sweating, dyspnea and pallor and
increased risk for Myocardial infarction is seen in Anomalous origin of Left Coronary Artery from Pulmonary Artery (ALCAPA); Refer
pretext for details;
59. b. Left atrial hypertrophy  Ref: Nelson’s 20/e p 2192-2192
60. e. Hypotension  Ref: Nelson’s 20/e p 2205-2206
The causes of mortality in coarctation of aorta are:
•• Hypertension •• Premature atherosclerosis •• Left ventricular failure
•• Aortic dissection & rupture •• Cerebral hemorrhage •• Infective endocarditis

61. d. Ventricular septal defect  Ref: Nelson’s 20/e p 2191-2194, Ghai 8/e p 414
Rheumatic fever usually presents between 5-15 years; TOF and TGA are cyanotic heart diseases.
VSD patients usually become symptomatic around 6 to 10 weeks age, but a large VSD may manifest at an earlier age.
62. e. Anatomic existence of PDA is an indication for surgery  Ref: Nelson’s 20/e p 2197, Ghai 8/e p 417
Discussing about the options one by one
a.  True, Spontaneous closure can occur in term as well as preterm babies with PDA
b. True, as pulmonary blood flow increased in PDA, pulmonary hypertension & EIssenmenger syndrome can develop
c. True, smaller the PDA, more forceful will be the jet of blood flow, causing more turbulence and erosion of endocardium, predisposing to
infective endocarditis
d. True, because of increased pulmonary blood flow, recurrent chest infection and congestive failure may develop
e. False, because, even though ,Irrespective of age, patients with PDA require closure.(which may be surgical or by cardiac catheterization); So,
closure of PDA can be done using coil embolization by cardiac catheterization, without surgery also
63. d. May cause a machinery murmur by its patency.  Ref: Nelson’s 20/e p 497, Ghai 8/e p 413
Closure of ductus arteriosus
•• Physiological closure: occurs almost immediately after birth by contraction of muscular wall, mediated by bradykinin
•• Anatomic closure: occurs by proliferation of intima in 10-21 days; After obliteration, it forms ligamentum arteriosum.
Note: Murmur due to PDA is classically referred to as a continuous machinery murmur.
64. b. Turner’s.  Ref: Nelson’s 20/e p 2189, Ghai 8/e p 413
304 Section 3: Systemic Pediatrics

 CYANOTIC CONGENITAL HEART DISEASES


Review of Pediatrics and Neonatology

65. b. Atrial septal defect  Ref: Nelson’s 20/e p 2211-2217


Components of Tetralogy of fallot include ventricular septal defect, infundibular pulmonic stenosis, right ventricular hypertrophy &
overriding of aorta.
66. c. Tetralogy of Fallot  Ref: Nelson’s 20/e p 2211-2217
In Tetralogy of Fallot, P2 is soft & inaudible, giving rise to single second heart sound.
67. b. Left pulmonary artery with descending aorta  Ref: Nelson 20/e p 2211-2217
68. d. All of the above  Ref: Nelson 20/e p 2211-2217
69. a. Supracardiac
70. a. Both ventricular hypertrophy  Ref: Nelson 20/e p 2228-2229
•• In truncus arteriosus, both ventricles are at systemic pressure & both eject blood into the truncus.
•• When pulmonary vascular resistance is relatively high immediately after birth, pulmonary blood flow may be normal;
•• As pulmonary resistance drops in 1st month of life, blood flow to the lungs is greatly increased → heart failure
•• Cardiac enlargement develops over 1st several weeks of life, as a result of prominence of both ventricles.
71. c. Absence of murmur  Ref: Nelson 20th ed/ p 2212; Refer pretext for details
During cyanotic spell, temporary disappearance or a decrease in intensity of systolic murmur is usual as flow across the right ventricular
outflow tract diminishes.
72. c. ASD  Ref: Nelson’s 20/e p 2211-2217, Ghai 8/e p 420-423
4 components of TOF are: Large VSD, infundibular pulmonary stenosis, overriding of Aorta, Right ventricular hypertrophy.
73. a. Atrial Septal Defect (ASD)  Ref: Nelson’s 20/e p 2211-2217, Ghai 8/e p 420-423
Pentalogy of Fallot = Tetralogy of Fallot + Atrial Septal defect.
74. a. Tetralogy of Fallot  Ref: Nelson’s 20/e p 2211-2217, Ghai 8/e p 420-423
The given history along with chest X-ray finding of Boot shaped heart suggests a diagnosis of Tetralogy of Fallot.
75. d. Left ventricular hypertrophy  Ref: Nelson’s 20/e p 2211-2217, Ghai 8/e p 420-423
76. d. Tetralogy of Fallot  Ref: Nelson’s 20/e p 2211-2217, Ghai 8/e p 420-423
77. d. All of the above  Ref: Nelson’s 20/e p 2211-2217, Ghai 8/e p 420-423
In severe coarctation and interrupted aortic arch, lesser amount of blood goes to lower limbs, as compared to upper limbs. In
Persistent pulmonary hypertension of newborn (PPHN), deoxygenated blood flows from pulmonary artery to descending aorta.
So in all these 3 conditions, cyanosis may be seen in lower limbs, but not in upper limbs (differential cyanosis), as upper limbs get
adequate amount of oxygenated blood.
78. a. Ebstein anomaly  Ref: Nelson’s 20/e p 2211-2217, Ghai 8/e p 420-423
Wolff-Parkinson White syndrome may be present in patients with Ebstein anomaly and these patients may have episodes of
supraventricular tachycardia.
79. a. TGA  Ref: Nelson’s 20/e p 2211-2217, Ghai 8/e p 420-423
•• The given clinical picture suggests a diagnosis of TGA (Transposition of great arteries)
•• In TOF and Pulmonary atresia, pulmonary vascularity is decreased
•• In TAPVC, S2 is not single, rather there is a wide fixed split of the 2nd heart sound.
80. b. TOF  Ref: Nelson’s 20/e p 2211-2217, Ghai 8/e p 420-423
In TOF, pulmonary oligemia i.e. decreased pulmonary vascularity is seen and not pulmonary plethora.
81. c. TOF  Ref: Nelson’s 20/e p 2211-2217, Ghai 8/e p 420-423
•• Reversal of shunt or Eisenmenger syndrome is seen in– VSD, ASD, AV canal defect, PDA.
82. a. TOF  Ref: Nelson’s 20/e p 2211-2217, Ghai 8/e p 420-423
•• Overall the most common cyanotic heart disease is TOF.
83. a. Lithium  Ref: Nelson’s 20/e p 2211-2217, Ghai 8/e p 420-423
Maternal drug Congenital heart disease
Phenytoin VSD, ASD, coarctation of aorta, PDA
Valproate Coarctation of aorta, aortic stenosis, pulmonary atresia, VSD
Lithium Ebstein anomaly, Tricuspid atresia
Alcohol ASD, VSD

84. d. All of above  Ref: Nelson’s 20/e p 2211-2217, Ghai 8/e p 420-423
Double aortic arch
•• It is associated with a chromosome 22q11 deletion in approximately 20% of patients
•• 22q11 deletion is responsible for DiGeorge, velocardiofacial, and conotruncal anomaly face syndromes, which are often
referred to using the unified terms CATCH-22 syndrome
•• It is also associated with VACTERL (vertebral, anal, cardiac, tracheal, esophageal, renal, and limb abnormalities) or
CHARGE (posterior coloboma, heart defect, choanal atresia, retardation, genital, and ear) associations.
Chapter 13: Pediatric Cardiology 305

Previously classified as: (now considered part of same spectrum).

Answers with Explanations


•• Di-George syndrome (thymic hypoplasia, diminished T-cell immunity, parathyroid hypoplasia with hypocalcemia) and
•• VelocardiofacialQ syndrome (congenital heart disease, dysmorphism, developmental delay).
85. a. Tricuspid atresia  Ref: Nelson’s 20/e p 2211-2217, Ghai 8/e p 420-423
For all practical purposes, Cyanotic heart disease with left axis deviation is Tricuspid atresia.
86. a. Truncus arteriosus  Ref: Nelson’s 20/e p 2211-2217, Ghai 8/e p 420-423
In Truncus arteriosus, both the aorta and pulmonary artery arise from a common large blood vessel.
So there is already a complete mixing of blood of systemic and pulmonary circulation, with no obstructive lesion in the pulmonary
or systemic circulatiosssn; So ductus dependent blood flow is not mandatory.
87. b. Hypoplastic left heart syndrome  Ref: Nelson’s 20/e p 2211-2217, Ghai 8/e p 420-423

High yield points about hypoplastic left heart syndrome (HLHS)  


•• It usually presents in the first few days of life with Cyanosis and heart failure
•• It is the most common congenital heart disease causing mortality in 1st week of life

88. a. Atrial Septal Defect (ASD)  Ref: Nelson’s 20/e p 2211-2217, Ghai 8/e p 420-423
89. c. Squatting  Ref: Nelson’s 20/e p 2211-2217, Ghai 8/e p 420-423
•• The patients of TOF assume a sitting posture (squatting) as soon as they get dyspneic
•• TOF is the commonest congenital lesion in which squatting is noted.
Squatting helps by:
•• Increasing systemic vascular resistance, so that more blood goes to lungs for oxygenation
•• Trapping deoxygenated venous blood in lower limbs.
90. d. Tetralogy of Fallot  Ref: Nelson’s 20/e p 2211-2217, Ghai 8/e p 420-423
91. b. Clubbing of hands and feet, c. Squatting, d. Cyanotic spells, e. CHF not seen  Ref: Nelson’s 20/e p 2211- 2217
•• Clubbing, cyanosis and squatting are seen in TOF; Heart failure does not occur in TOF
•• Recurrent chest infection is not common in TOF because of decreased pulmonary vascularity.
92. c. Prostaglandin E1  Ref: Nelson’s 20/e p 2223-2225, Ghai 8/e p 424-426
Most common cyanotic heart disease presenting in the 1st week of life is TGA.
In TGA, which is a ductus dependent heart disease, PGE1 may be administered or balloon atrial septostomy may be done for
initial stabilization of the patient. Definite surgery for TGA is arterial switch operation or Jatene repair.
93. b. Transposition of great arteries  Ref: Nelson’s 20/e p 2223-2225, Ghai 8/e p 424-426
In the given case scenario, an infant has features of heart failure, cyanosis with repeated episodes of pneumonia suggesting increased
pulmonary blood flow. These clinical features along with the chest X ray findings are suggestive of Transposition of great arteries.
94. b. Truncus arteriosus  Ref: Nelson’s 20/e p 2223-2225, Ghai 8/e p 424-426
In Right aortic arch, aorta curves to right, instead of left; Right aortic arch is associated with 30% cases of Truncus arteriosus &
20% cases of TOF.
95. b. Pulmonary stenosis  Ref: Nelson’s 20/e p 2223-2225, Ghai 8/e p 424-426
Ejection systolic murmur in TOF is due to pulmonary stenosis.
The intensity and duration of this systolic murmur is inversely proportional to the severity of TOF.
96. b. Transposition of great vessels  Ref: Nelson’s 20/e p 2223-2225, Ghai 8/e p 424-426

Congenital heart diseases Important chest X-ray finding


Transposition of great vessels ‘Egg on side’ appearance
Tetralogy of Fallot Boot shaped heart or ‘Cor en sabot’ appearance
Supracardiac TAPVC ‘Figure of 8’ or Snowman appearance
Ebstein anomaly Box shaped heart

97. b. PDA with reversal of shunt  Ref: Nelson’s 20/e p 2223-2225, Ghai 8/e p 424-426
•• Differential cyanosis is characterized by bluish discoloration of lower limbs with normal upper limbs
•• Differential cyanosis can be caused by PDA with Eisenmenger syndrome or reversal of shunt
98. a. More common in males  Ref: Nelson’s 20/e p 2223-2225, Ghai 8/e p 424-426

Mnemonic: In Boys—‘Baaya’ Meaning Left; So, Left Sided Obstructive Lesions


•• Congenital heart diseases more common in girls are ASD, PDA, VSD, Pulmonic stenosis
•• Congenital heart diseases more common in boys are TGA, Left sided obstructive lesions like coarctation of aorta
306 Section 3: Systemic Pediatrics

99. a. Valvular stenosis  Ref: Nelson’s 20/e p 2211, Ghai 8/e p 420
Review of Pediatrics and Neonatology

•• In TOF, obstruction to pulmonary arterial blood flow is usually both at the right ventricular infundibulum (subpulmonic area)
and the pulmonary valve
•• Pulmonary valve stenosis (alone) rarely contributes to the right ventricular outflow obstruction in TOF.
100. d. Tetralogy of Fallot  Ref: Nelson’s 20/e p 2211, Ghai 8/e p 420
In the given case scenario, the child has cyanosis and clubbing with a normal sized heart, no evidence of heart failure and a single
audible second heart sound. All these features are compatible with a diagnosis of TOF.
101. a. Cyanosis at birth b. CHF  Ref: Nelson’s 20/e p 2223, Ghai 8/e p 424
Clinical Features of TGA
•• Time of presentation of TGA depends on whether ventricular septum is intact or VSD is present
•• Patients of TGA with intact septum are cyanotic at birth, with heart failure occuring in 1st week of life because adequate
mixing of blood of pulmonary and systemic circulation does not occur
•• Patients of TGA with VSD presents with mild cyanosis and CHF occurs in 6–8 weeks
102. b. TGA, c. TAPVC, e. Ebstein anomaly  Ref: Nelson’s 20/e p 2223, Ghai 8/e p 424
•• Congenital heart diseases which present with cyanosis at birth are: TGA, TAPVC, Tricuspid atresia, pulmonary atresia,
Ebstein anomaly; In TOF cyanosis usually presents after the neonatal period (after 28 days).
103. c. Atrial septal defect  Ref: Nelson’s 20/e p 2189, Ghai 8/e p 413
104. c. Subclavian artery to pulmonary artery  Ref: Nelson’s 20/e p 2211-2217, Ghai 8/e p 420-423

Mnemonic for Shunt Surgeries for TOF: “Bas Waha Pahucha Do”
Name of surgery Mnemonic Connects
Blalock Taussig (BT) Shunt B with S in BaS Subclavian artery to pulmonary artery
Waterston’s Shunt W with A in WahA Ascending aorta to right pulmonary artery
Pott’s Shunt P with D in Pahucha Do Descending aorta to pulmonary artery

105. c. Predominantly left to right shunt  Ref: Nelson’s 20/e p 2211-2217, Ghai 8/e p 420-423
Right to left shunt is seen in ToF i.e. blood going from the pulmonary circulation to the systemic circulation Physical
examination in a case of TOF shows: Cyanosis and clubbing, Normal jugular venous pulse (as there is no heart failure), a
Single second heart sound (S2) and Ejection systolic murmur in pulmonary area.
106. a. Left ventricular hypertrophy  Ref: Nelson’s 20/e p 2211-2217, Ghai 8/e p 420-423
107. b. Tetralogy of Fallot  Ref: Nelson’s 20/e p 2211-2217, Ghai 8/e p 420-423
In ToF lesser blood goes to the pulmonary circulation, so it does not predispose to recurrent respiratory tract infections.
108. d. All of the above  Ref: Nelson’s 20/e p 2211-2217, Ghai 8/e p 420-423
Pulmonary stenosis can occur due to obstruction at supravalvular, valvular or subvalvular level.
109. a. Supracardiac  Ref: Nelson’s 20/e p 2211-2217, Ghai 8/e p 420-423
Remember, Most common type of TAPVC is Supracardiac and Infracardiac type of TAPVC is always obstructive
110. c. Truncus arteriosus  Ref: Nelson’s 20/e p 2211-2217; Refer Ans 73 above;
111. d. Tetralogy of Fallot  Ref: Nelson’s 20/e p 2211-2217, Ghai 8/e p 420-423
112. d. Tetralogy of Fallot  Ref: Nelson’s 20/e p 2211-2217, Ghai 8/e p 420-423
113. d. Isoprenaline  Ref: Nelson’s 20/e p 2211-2217, Ghai 8/e p 420-423; Refer pretext for details;
During cyanotic spell, any drug that causes peripheral pooling of blood should be avoided because this will reduce return of blood
to heart; Therefore isoprenaline which has marked agonistic action should be avoided as it will cause vasodilatation, which leads
to peripheral pooling of blood.
114. a. VSD, d. Taussig-Blalock shunt is between pulmonary and subclavian artery, e. Morphine is given for cyanotic spell  Ref:
Nelson’s 20/e p 2211, Ghai 8/e p 420-423
Treatment of cyanotic spell involves use of knee chest position (or squatting), oxygen, morphine, sodium bicarbonate, beta blockers
(propranolol) and alpha agonists (phenylephrine)
115. b. Transposition of great vessels  Ref: Nelson’s 20/e p 2223-2225, Ghai 8/e p 424-426
Balloon atrial septostomy is helpful in a baby with TGA with intact ventricular septum for initial stabilization, as by creating a
communication between the left and right atria, it helps in mixing blood of systemic and pulmonary circulations.
116. b. Tricuspid atresia, d. Eisenmenger complex, e. Tetralogy of Fallot  Ref: Nelson’s 20/e p 2223-2225
117. b. Descending aorta to left pulmonary artery  Ref: Nelson’s 20/e p 2223-2225; Refer Ans 91 above;
118. b. Tricuspid atresia  Ref: Nelson’s 20/e p 2223-2225, Ghai 8/e p 424-426
119. b. Transposition of great vessels  Ref: Nelson’s 20/e p 2223-2225, Ghai 8/e p 424-426
ASD and VSD are acyanotic heart diseases and TOF usually does not present with cyanosis in neonatal period.
120. b. Pulmonary atresia  Ref: Nelson’s 20/e p 2216-2217
In Pulmonary atresia, child is cyanotic at birth and there is pulmonary oligemia.
Chapter 13: Pediatric Cardiology 307

121. a. Tricuspid atresia  Ref: Nelson’s 20/e p 2218, Ghai 8/e p 420-423

Answers with Explanations


The combination of cyanosis and left axis deviation is highly suggestive of tricuspid atresia.
122. c. Fallot’s tetralogy  Ref: Nelson’s 20/e p 2211, Ghai 8/e p 420
123. a. Left to right shunt, pulmonary hypertension, right ventricular hypertrophy, right to left shunt.  Ref: Nelson’s 20/e p 2241-
2243
Sequence of changes in Eisenmenger syndrome is: Left to right shunt (VSD or PDA) → Increased blood flow to lungs → Pulmonary
hypertension → Right ventricular hypertrophy → Right to left shunt → cyanosis and clubbing.
124. a. Arterial switch  Ref: Nelson’s 20/e p 2223, Ghai 8/e p 424
Transposition of Great vessels or arteries (TGV or TGA) is a condition characterized by aorta arising from right ventricle and pulmonary
artery from left ventricle; Definite treatment for TGA is Arterial switch operation or Jatene’s repair;

 RHEUMATIC HEART DISEASE


125. c. Fever  Ref: Nelson’s 20/e p 1332-1334; For Mnemonic, refer pretext of this chapter
126. b. Diagnosis of recurrent of acute rheumatic fever in Moderate/high risk population requires 2 minor criteria and evidence
of preceding GAS infection  Ref: Nelson’s 20/e p 1332-1334
Changes in 2015 AHA revision of modified Jone’s criteria include:
•• Separate criteria for Low-Risk populations (defined as those with incidence ≤ 2 per 100,000 school-age children per year or
all-age rheumatic heart disease prevalence of ≤1 per 1000 population) and Moderate/High-Risk populations (defined as those
with higher incidence or prevalence rates).
•• Diagnosis of recurrent acute rheumatic fever can also be made only in the Moderate/High Risk population by presence of 3
minor criteria with evidence of preceding GAS infection.
•• Definition of the major criterion–carditis is now expanded to include subclinical evidence (i.e., in the absence of a murmur,
echocardiographic evidence of mitral regurgitation)
127. d. Erythema marginatum  Ref: Nelson’s 20/e p 2269-2271, Ghai 8/e p 433-443; Refer pretext of this chapter;
128. b. ASLO  Ref: Nelson’s 20/e p 2269-2271, Ghai 8/e p 433-443
129. c. 9 days of illness  Ref: Nelson’s 20/e p 2269-2271;
Antibiotic therapy for patients with GAS pharyngitis can prevent acute rheumatic fever, shorten the clinical course of the
illness, reduce transmission of the infection to others, and prevent suppurative complications.
When therapy is instituted within 9 days of onset of symptoms and continued for the full course, acute rheumatic fever is almost
always prevented.
130. d. Chorea  Ref: Nelson’s 20/e p 2269-2271, Ghai 8/e p 433-443
131. d. Acute rheumatic fever  Ref: Nelson’s 20/e p 2269-2271, Ghai 8/e p 433-443
132. d. Increased CRP  Ref: Nelson’s 20/e p 2269-2271, Ghai 8/e p 433-443
133. d. Elevated ESR  Ref: Nelson’s 20/e p 2269-2271, Ghai 8/e p 433-443
134. a. Electrocardiogram  Ref: Nelson’s 20/e p 2269-2271, Ghai 8/e p 433-443
Infective endocarditis should be suspected in presence of pallor, fever and splenomegaly in a known case of RHD
•• Urine examination shows albuminuria and microscopic hematuria
•• Blood culture should be done for diagnosis of the causative agent
•• Echocardiography is done to detect the cardiac changes like vegetations.
135. d. ASO titre  Ref: Nelson’s 20/e p 2269-2271, Ghai 8/e p 433-443
136. b. Sarcoidosis  Ref: Nelson’s 20/e p 2269-2271, Ghai 8/e p 433-443
Important causes of pericarditis/pericardial effusion in children:
Infectious
•• Viral (Coxsackie virus B, EBV, Influenza, Adenovirus); Fungal (Histoplasmosis, Actinomycosis)
•• Bacterial (TB, Streptococcus, Pneumococcus, Staphylococcus, Meningococcus, Mycoplasma, Tularemia, Listeria, Leptospirosis)
•• Immune complex (Meningococcus, Hemophilus influenzae)
•• Parasitic (Toxoplasmosis, Echinococcosis)

Connective tissue diseases Metabolic-endocrine


Rheumatoid arthritis Systemic sclerosis Uremia
Rheumatic fever Sarcoidosis Hypothyroidism
Systemic lupus erythematosus Wegener granulomatosis Chylopericardium
Out of the causes given in the options, Sarcoidosis is the least common.
137. b. Erythema marginatum  Ref: Nelson’s 20/e p 2269-2271, Ghai 8/e p 433-443
138. c. Fever  Ref: Nelson’s 20/e p 2269-2271, Ghai 8/e p 433-443
139. a. Erythromycin  Ref: Nelson’s 20/e p 2269-2271, Ghai 8/e p 433-443
For a patient who is allergic to both penicillin and sulphonamides, a macrolide (erythromycin or clarithromycin or azithromycin)
may be used.
308 Section 3: Systemic Pediatrics

Recommendations of American heart association for the duration of secondary prophylaxis


Review of Pediatrics and Neonatology

Category: Rheumatic fever Duration of secondary prophylaxis


Without carditis 5 years or until 21 years of age, whichever is longer
With carditis but without residual heart disease 10 years or until 21 years, whichever is longer
With carditis and residual heart disease 10 years or until 40 years age, whichever is longer, sometimes lifelong

140. a. 3 weeks  Ref: Nelson’s 20/e p 2269-2271, Ghai 8/e p 433-443

High Yield Points about corticosteroids in rheumatic carditis  


•• Indication: Patients with carditis and more than minimal cardiomegaly and/or congestive heart failure
•• Usual dose: Prednisone 2 mg/kg/day in 4 divided doses for 2-3 wk followed by half the dose for 2-3 wk and then tapering of the dose by
5 mg/24 hr every 2-3 days.
•• When prednisone is being tapered, aspirin should be started at 50 mg/kg/day in 4 divided doses for 6 wk
141. a. Polyarthritis, d. MC valve involvement is Mitral  Ref: Nelson’s 20/e p 2269-2271, Ghai 8/e p 433-443
•• Most commonly involved valve in case of rheumatic carditis is the mitral valve and the most common valvular deformity
produced is mitral regurgitation; Most common manifestation of rheumatic fever is arthritis
•• Erythema marginatum spares the face.
142. d. Rheumatic carditis  Ref: Nelson’s 20/e p 2269-2271, Ghai 8/e p 433-443
•• Cardiac failure causes cardiomegaly
•• MC cause of cardiac failure in children (older than infants) is rheumatic fever and rheumatic heart disease.

 Heart Failure
143. c. Tender hepatomegaly  Ref: Nelson’s 20/e p 2282-2287, Ghai 8/e p 396-400
Clinical features of heart failure in infants:
•• Tachypnea, feeding difficulties, poor weight gain, excessive perspiration, irritability, labored respiration
•• Tachycardia, gallop rhythm, wheezing are frequently seen
•• Hepatomegaly and cardiomegaly seen; palpation of an enlarged liver is a more reliable sign
•• Assessment of jugular venous pressure may be difficult because of shortness of the neck in infants
•• Edema may be generalized and usually involves eyelids as well as sacrum and less often the legs and feet

144. c. 0.50  Ref: Nelson’s 20/e p 2282-2287, Ghai 8/e p 396-400


When maximal cardiac width is > half the maximal chest width (cardiothoracic ratio >50%), the heart is usually enlarged.
145. a. ASD  Ref: Nelson’s 20/e p 2282-2287, Ghai 8/e p 396-400; Discussing the options one by one:
a.  ASD •• A child with an ostium secundum ASD is most often asymptomatic;
•• Even an extremely large secundum ASD rarely produces clinically evident heart failure in childhood.
TGA with large VSD
b.  When the VSD is large and not restrictive to ventricular ejection, significant mixing of oxygenated and
deoxygenated blood usually occurs and clinical manifestations of cardiac failure are seen
Hypoplastic left ventricle
c.  The condition is diagnosed in most infants in 1st few hr or days of life, once the ductus arteriosus begins
to close; Signs of poor systemic perfusion and shock predominate and hypoxia may lead to heart failure
Baby of diabetic mother
d.  Cardiomegaly is common (30%), and heart failure occurs in 5-10% of infants of diabetic mothers
146. c. Chorea  Ref: Nelson’s 20/e p 2282-2287, Ghai 8/e p 396-400
3 circumstances in which strict application of diagnostic criteria of Rheumatic fever is not needed are:
Chorea, Indolent carditis and Recurrence of Rheumatic fever.
147. d. Congenital heart disease  Ref: Nelson’s 20/e p 2282-2287, Ghai 8/e p 396-400
•• The most common cause of Congestive heart failure (CHF) in infants— Congenital heart disease
•• The most common cause of CHF in the older children—Rheumatic fever and rheumatic heart disease.

 OTHER HEART DISEASES


148. b. Ebstein anomaly  Ref: Nelson’s 20/e p 2190
Wide fixed splitting of second heart sound is seen in Atrial septal defect, Total anomalous pulmonary venous return (TAPVC) and
Right bundle branch block (RBBB)
149. a. 8 mm   Ref: Nelson’s 20/e p 1211-1212
Giant coronary artery aneurysms (classic definition of > 8 mm internal diameter) pose the greatest risk for rupture, thrombosis or
stenosis, and myocardial infarction
Chapter 13: Pediatric Cardiology 309

150. d. Passive filling of atrium during ventricular systole  Ref: Nelson’s 20/e p 2164-2169

Answers with Explanations


Normal waves in jugular venous pulse

Name of Underlying cause


wave
a wave Atrial contraction
c wave Transmission of carotid arterial impulse through
jugular veins or by bulging of tricuspid valve cusps
into right atrium in early systole.
v wave Filling up of Right atrium in late systole and early
diastole.
x wave Atrium relaxes and tricuspid valve moves downward
y wave Filling of ventricle after tricuspid opening

151. b. 450 ml/kg/min  Ref: Nelson’s 20/e p 2161


Total fetal cardiac output, i.e. combined output of both left and right ventricles—is ≈450 mL/kg/min.
65% of descending aortic blood flow returns to the placenta and remaining 35% perfuses the fetal organs and tissues.
152. b. Calcium channel blocker  Ref: Nelson’s 20/e p 2275
Treatment of Dilated Cardiomyopathy:
•• Drugs used include diuretics, ACE inhibitors, angiotensin receptor blockers, digitalis, β-blockers (carvedilol or metoprolol)
•• In patients presenting with extreme degrees of heart failure or circulatory collapse, intensive care measures are often required,
including IV inotropes, diuretics, mechanical ventilatory support ± circulatory support, which may include ventricular assist
devices, extracorporeal membrane oxygenation (ECMO), and ultimately cardiac transplantation.
•• In patients with DCM and atrial or ventricular arrhythmias, specific antiarrhythmic therapy should be instituted.
153. c. Ebstein’s anomaly   Ref: Nelson’s 20/e p 2222; Diagnosis of Congenital Heart Disease By M Satpathy, p 245
Pre-excitation refers to early activation of ventricles due to impulses bypassing the AV node via an accessory pathway;
Accessory pathways, also known as bypass tracts, are abnormal conduction pathways formed during cardiac development and are
seen in conditions like WPW syndrome;

ECG findings of Ebstein


anomaly:
•• Normal or tall and broad
(‘himalayan’) P waves, and
a normal or prolonged P-R
interval
•• Right bundle-branch block
without increased right
precordial voltage.
•• Wolff-Parkinson-White
syndrome (pre-excitation)
and supra­ventricular
tachycardia may be present

Note: Himalayan P-waves (amplitude > 5 mm) are classically associated with congenital heart diseases with right to left shunt like
tricuspid atresia, Ebstein anomaly, combined tricuspid and pulmonic stenosis, ischemic heart disease and restrictive cardiomyopathy,
where they indicate a dilated right atrium
154. c. During infancy  Ref: Nelson’s 20/e p 2196, 2249
Indications for surgical closure of a VSD include:
•• Patients at any age with large defects in whom clinical symptoms and failure to thrive cannot be controlled medically;
•• Infants 6 – 12 months age with large defects & pulmonary hypertension, even if symptoms are controlled by medication;
•• Patients older than 24 months with a Qp : Qs ratio greater than 2 : 1.
•• Patients with a supracristal VSD of any size (because of the high risk for aortic valve regurgitation)
310 Section 3: Systemic Pediatrics

155. a. Valvular aortic stenosis  Ref: Nelson’s 20/e p.2205, 2261


Review of Pediatrics and Neonatology

High Yield Points  


•• Valvular aortic stenosis is the congenital defect most commonly associated with sudden death in children.
•• A history of syncope, chest pain, severe obstruction & left ventricular hypertrophy are risk factors for sudden death in AS
•• Most common Coronary artery anomaly associated with sudden death is the origin of left main coronary artery from right sinus of
Valsalva.
•• Hypertrophic cardiomyopathy (HCM) is the most common cause of sudden death in the athletic adolescent.

156. b. 5p-  Ref: Nelson’s 20/e p 2165; Refer pretext for list of important syndromes associated with CHD
Cat cry with congenital heart disease in an infant suggests Cri du chat syndrome
157. a. IV immunoglobulin is used in treatment  Ref: Nelson’s 20/e p 2292, Ghai 8/e p 631
The given clinical picture shows erythematous rash and strawberry tongue and the angiography image shows Coronary artery
aneurysm. These findings point towards a diagnosis of Kawasaki disease.
IV Immunoglobulin is used as treatment of choice for Kawasaki disease.
For details about Kawasaki disease refer chapter 8, Diseases of Immune system.
158. d. Coarctation of aorta  Ref: Nelson’s 20/e p 2205; Coarctation of aorta causes inferior rib notching.
159. b. 2 3 4 1  Ref: Nelson’s 20/e p 2189-2196
The correctly matched list is as follows:
A. Wide fixed split of S2 2. ASD
B. Continuous machinery murmur 3. PDA
C. Muffled heart sounds 4. Pericard. effusion
D. Wide variable split of S2 1. Pulm hypertension

160. d. Lowe’s syndrome  Ref: Nelson’s 20/e p 2271-2276; Refer pretext for important causes of Cardiomyopathy
161. b. ASD  Ref: Nelson’s 20/e p 2263-2268, Ghai 8/e p 443
Risk of infective endocarditis in various lesions:
High risk Moderate risk Low risk
•• Prosthetic heart valve •• AS •• Mitral valve prolapse •• PS •• Mitral valve prolapse
•• Tetralogy of Fallot •• VSD with MR •• MS •• ASD
•• PDA •• MR •• TS
•• AR •• Coarctation of aorta •• TR

162. b. Coarctation of aorta  Ref: Nelson’s 20/e p 2205, Ghai 8/e p 431
163. b. RV and LV walls come back to normal size  Ref: Nelson’s 20/e p 2242, Ghai 8/e p 428
In Eisenmenger syndrome, irreversible pulmonary vascular obstructive changes occur;
•• There is mainly pulmonary arterial hypertension, so pulmonary veins are not distended
•• Because of pulmonary hypertension, right ventricular hypertrophy occurs
•• Chest X Ray shows that pulmonary vessels are enlarged in the hilar areas and taper rapidly in caliber in the peripheral branches
(peripheral pruning).
164. a. Increased flow murmur across tricuspid and pulmonary valve  Ref: Nelson’s 20/e p 2242, Ghai 8/e p 428
In Eisenmenger syndrome, because of the resistance offered by pulmonary hypertension, flow across the pulmonary and tricuspid
valve actually decreases, so the flow murmur decreases and not increases;
Clinical examination findings in Eisenmenger syndrome:
•• Right ventricular heave; Narrowly split 2nd heart sound (may appear as single S2) with a loud P2
•• Palpable pulmonary artery pulsation may be present at the left upper sternal border
•• A holosystolic murmur of tricuspid regurgitation may be audible along the left sternal border
•• An early decrescendo diastolic murmur of pulmonary insufficiency may also be heard along left sternal border, called “Graham steel
murmur”, due to dilatation of pulmonic valve ring as a result of pulmonary hypertension

165. a. Rhabdomyoma, c. Atrial myxoma, e. Fibroma  Ref: Nelson’s 20/e p 2281

High Yielding Points about cardiac tumors  


•• Primary tumors of the heart are rare in infancy and childhood and are most often benign
•• Most common benign cardiac tumors in children are rhabdomyomas, fibromas, and myxomas
•• Myxomas occur most frequently in the left atrium
Chapter 13: Pediatric Cardiology 311

166. c. Associated with AR  Ref: Nelson’s 20/e p 2269, Ghai 8/e p 433-443

Answers with Explanations


Carey-Coombs Murmur is a soft, mid-diastolic murmur, heard in patients with acute rheumatic fever
It occurs due to inflammation of mitral valve cusps or excessive left atrial blood flow as a consequence of MR.
167. a. Sodium nitroprusside drip, c. IV labetolol, hydralazine, and diazoxide can be used, d. Nifedipine is used,
e. Pheochromocytoma mimics the condition  Ref: Nelson’s 20/e p 2294-2303, Ghai 8/e p 451-456
Treatment of hypertensive crisis
• Because too rapid reduction in BP may interfere with adequate organ perfusion, a stepwise reduction should be planned.
–– Pressure should be reduced by 1/3 of the total planned reduction during first 6 hours
–– The remaining amount is reduced over the following 48-72 hours
• Aggressive parenteral therapy is indicated:
–– Drug of choice are IV labetalol or sodium nitroprusside or sublingual nifedipine
–– If hypertension is less severe, hydralazine or diazoxide can be used; Furosemide is given IV to initiate diuresis
–– Because fluid balance must be controlled carefully, intake is limited to urine output plus insensible loss
Note: In children with pheochromocytoma, hypertension is more often sustained rather than paroxysmal, unlike what is
seen in adults.
168. a. Renal artery stenosis, b. Coarctation of aorta, c. Glomerulonephritis, e. Pheochromocytoma  Ref: Nelson’s 20/e p 2294,
Ghai 8/e p 451-456; Refer pretext for list of important causes of hypertension in children;
169. d. Complete heart block  Ref: Nelson’s 20/e p 2171-2174, Ghai 8/e p 457-461
Congenital complete AV block
•• It is presumed to be caused by autoimmune injury of fetal conduction system by maternally derived anti-SSA/ Ro, anti-SSB/
La antibodies in a mother with systemic lupus erythematosus (SLE) or Sjögren syndrome
•• Autoimmune disease accounts for 60-70% of all cases of congenital complete heart block
170. a. Klippel-trenaunay syndrome  Ref: Nelson’s 20/e p 2292
Klippel-Trenaunay syndrome (KT syndrome)
•• Triad of port-wine stain, varicose veins and bony and soft tissue hypertrophy involving an extremity seen
•• When there is associated Arterio venous malformation, it is known as Klippel-Trenaunay-Weber syndrome
•• Varicose veins may be pulsatile because of AV malformation

171. c. Atrial septal defect with mitral stenosis  Ref: Nelson’s 20/e p 2189-2191, Ghai 8/e p 413-414
Lutembacher syndrome is defined as a combination of mitral stenosis and a left-to-right shunt at the atrial level. Typically, the
left-to-right shunt is an atrial septal defect (ASD) of the ostium secundum variety.
Chapter 14
Pediatric Hematology
High Yield Points  HEMATOPOIESIS IN HUMAN EMBRYO AND FETUS
  M
•• Normal life span of RBC is 120 days Type Description
in older children, 90 days in term
neonates & 40-60 days in preterm Mesoblastic hemato­poiesis Begins in yolk sac, by 3rd Wk & ceases by 10–12 weeks
neonates Hepatic hematopoiesis Starts at 6–8 wks of gestation ceases by 2nd trimester
Myeloid hematopoiesis From 2nd trimester myeloid hematopoiesis predominates

NORMAL HUMAN HAEMOGLOBINS


Haemoglobin Structural formula
High Yield Points   Adult Hb-A α2 β2 97%
Liver remains the main erythro­poietic Hb-A2 α2 δ2 1.5–3.2%
organ through 20–24 weeks gestation
Fetal Hb-F α2 γ2 0.5–1%
Embryonic Hb-Gower 1 ζ2 ε2
Hb-Gower 2 α2 ε2
Hb-Portland ζ2 γ2

 ANEMIA
High Yield Points  
WHO—Definition of Anemia M
In term neonates, cord blood Hb is
16.8 g/dL (14-20 g/dL); Hb level in VLBW Children 6 months –5 years < 11 g/dL
neonates is 1-2g/dl less than that; Children 6–14 years < 12 g/dL

Etiologic Classification of Anemia


Category Diseases

Impaired RBC 1. Abnormal bone marrow 1.1 Aplastic anemia 1.2 Myelophthisis: Myelofibrosis,
production Leukemia, Cancer metastasis
2. Nutritional deficiency 2.1 Deficiency anemia: Fe, Vit. B12, Folic acid
3. Stimulation factor deficiency 3.1 Anemia in chronic disease 3.2 Anemia in
hypopituitarism 3.3 Anemia in hypothyroidism

Excessive 1. Intracorpuscular defect 1.1 Membrane: Hereditary spherocytosis, Hereditary


destruction ovalocytosis, etc. 1.2 Enzyme: G-6PD deficiency, PK def., etc. 1.3 Hemoglobin:
(Hemolytic Thalassemia, Hemoglobinopathies
anemia) 2. Extracorpuscular defect 2.1 Mechanical: March hemolytic anemia, MAHA
(Microangiopathic HA) 2.2 Chemical/Physical 2.3 Infection: Clostridium tetani, 2.4
Antibodies: HTR, SLE, 2.5 Hypersplenism

Blood loss •• Acute blood loss: Accident, GI bleeding


•• Chronic blood loss: Hypermenorrhea, Parasitic infestation

Clinical Features
•• Pallor, sleepiness, irritability, decreased exercise tolerance & a flow murmur
•• Severe anemia leads to weakness, dyspnea, tachycardia, & high-output heart failure.
Chapter 14: Pediatric Hematology 313

Section 3: Systemic Pediatrics


M
High Yield Points  
•• Iron deficiency is the most common
cause of anemia worldwide
•• Iron transport across membrane is
facilitated by DMT–1, Nramp 2 or
DCT 1

IRON DEFICIENCY ANEMIA


Question 1
High Yield Points   M A 6-year-old child presents with
anemia. His peripheral smear
•• Iron deficiency anemia is the most common nutritional disorder in the world
shows the following picture. All
•• Typical age of manifestation in term babies is 9–24 months
of the following are differential
•• Amount of elemental iron required daily per 1000 calories is 6 mg
diagnoses for his condition except?

Etiology M

Decreased iron intake Inadequate presentation to erythroid


•• Inadequate diet precursors
•• Impaired absorption: Celiac disease •• Atransferrinemia
Increased iron loss •• Antitransferrin receptor antibodies
•• Gastrointestinal bleeding, varices Abnormal iron balance
•• Hookworm, Schistosomiasis, Trichuriasis, Gastritis •• Aceruloplasminemia a. Anemia of chronic disease
•• Hiatal hernia •• Autosomal dominant hemochromatosis b. Lead poisoning
•• Inflammatory bowel disease, Meckel’s due to Mutations in ferroportin c. Aplastic anemia
•• Milk-induced enteropathy (infants) Increased requirements d. Iron deficiency anemia
•• Infancy

Clinical Features
•• Smooth tongue, glossitis, cheilosis
•• Brittle nails & koilonychia
•• Pica
•• Dysphagia due to esophageal webs (Kelly-Paterson or Plummer-Vinson syndrome)
•• Impairment of immunity

Diagnosis of Iron Deficiency Anemia M

Peripheral smear: Anisocytosis with microcytic hypochromic RBCs, Target cells, Pencil RBCs

Differential Diagnosis of Microcytic Hypochromic Anemias


Iron Deficiency β- Thalassemia Anemia of Chronic
Feature (Normal value) Anemia M Minor Disease
RDW (11.5 to 14.5) IncreasedQ Normal Normal to increased
FEP (30-50 microg/dL) IncreasedQ Normal Increased
S. Iron (50–150 microg/dL) Decreased Normal to high Decreased
S. Ferritin (50-200 microg/L) Decreased Normal to high IncreasedQ
TIBC (300–360 microgl/dL) Increased Normal Decreased
Transferrin saturation (30–50%) Decreased (18%) Normal Decreased
Hb electrophoresis Normal Elevated HbA2 Normal
Mentzer’s index > 14 < 13 Not applicable
Contd...
314 Section 3: Systemic Pediatrics

Contd...
Review of Pediatrics and Neonatology

Iron Deficiency β- Thalassemia Anemia of Chronic


Feature (Normal value) Anemia M Minor Disease
RBC count Proportionate to the Normal to increased Normal to decreased
Hb level (decreased)
Soluble Transferrin receptor Increased (> 9 mg/L) Normal Normal to decreased
(sTfR)
STfR: Serum ferritin ratio Increased (> 0.8) Normal Decreased
Bone marrow iron Absent marrow iron Normal to increased Iron trapped in RE cell

High Yield Points Treatment 3–6 mg/kg of No treatment If severe anemia,


  M elemental iron/day needed. Genetic transfusion and erythro­
In iron deficiency anemia, bone marrow counseling. poietin therapy maybe
iron decreases earlier than serum iron. needed

Treatment
Question 2
•• A daily total dose of 3-6 mg/kg of elemental iron in 3 divided doses is given
In which condition is the following
•• Maximum dose would be 150-200 mg of elemental iron daily
finding in tongue common seen?
•• Iron is continued for 8 weeks after blood values normalize

For IV correction—Formula to calcu­late the amount of iron needed:


•• Body weight (kg) × (15-patients Hb) × 2.3 + 500 or 1000 mg (for stores)

Responses to Iron Therapy in Iron Deficiency Anemia M

Time after Iron administration Response


12–24 hours Decreased irritability, increased appetite
26–48 hours Erythroid hyperplasia
a. Sickle cell anemia
48–72 hours Reticulocytosis, peaking at 5–7 days
b. AML
c. Iron deficiency anemia 4–30 days Increase in Hb level (best measure)
d. MDS
1–3 months Repletion of stores

MEGALOBLASTIC ANEMIA

Definition Impairment of DNA synthesis that leads to ineffective hematopoiesis & distinctive
morphologic changes, including abnormally large erythroid precursors in bone
marrow & RBCs in peripheral smear

Question 3 Etiology Cobalamin (Vit B12) Deficiency: Folic acid deficiency


•• Dietary deficiency •• Dietary deficiency
A 10-year-old child presents with •• Malabsorption: Ileal resection, Fish •• Impaired absorption
Anemia. His peri­ pheral smear tapeworm infection, bacterial over­ –– Celiac SprueQ
growth, Sprue, regional enteritis –– Small bowel resection/disease
finding is shown below. What is
•• Intrinsic factor Deficiency: Pernicious •• Increased requirements
the most probable type of anemia,
anemia, –– Infancy, Hemolytic anemias
this child is suffe­ring from?
Drug-induced suppression of DNA –– Myeloproliferative disorders
synthesis
Inborn errors
•• Defective folate metabolism
•• Defective vitamin B12 metabolism
•• Hereditary orotic aciduriaQ
•• Lesch-Nyhan syndromeQ

Peripheral Anisopoikilocytosis, Macro-ovalocytosis  are highly characteristic.Q


smear M Macrocytes appear “hyperchromic,” but the MCHC is not elevated 
Nuclear hypersegmentationQ (>=5 lobes in >5% neutrophils) Earliest abnor­-
a. Hemolytic anemia
mality to appearQ
b. Megaloblastic anemia
c. Aplastic anemia Bone marrow M Hallmark: nuclear cytoplasmic maturation asynchronyQ
d. Iron deficiency anemia Megaloblast with sieve like chromatinQ
Chapter 14: Pediatric Hematology 315

ANEMIA OF CHRONIC DISEASE (ACD)

Section 3: Systemic Pediatrics


•• Found in conditions where there is ongoing immune activationQ
•• It occurs in a infections, malig­nancies, autoimmunity, and graft-versus-host disease
•• Low serum iron and elevated serum ferritin is classical of ACD
•• Transferrin saturation is low and TIBC is high
•• Treatment: Transfusions and EPO (or long acting darbepoetin alfa), treatment of the cause.

RBC MORPHOLOGY IN THE DIAGNOSIS OF HEMOLYTIC ANEMIAS


Morphology Images Causes Syndromes
Target cells Increased RBC surface Hb disorders—thalassemias,
area: volume Hb S, Hb C, liver disease
Question 4
A 4-year-old child presents with
Schistocytes Traumatic disruption of Microangiopathy (HUS, TTP, pica & constipation; He is anemic
membrane DIC), intravascular prosthesis
& his peripheral smear shows
following. Diagnosis?
Sickled cells Polymerization of Hb Sickle cell syndromes

Agglutinated Presence of IgM Ab Cold agglutinin disease


cells

Heinz body Precipitated Hb Unstable Hb, G6PD


deficiency, oxidant stress

a. G6PD deficiency
b. Nutritional anemia
c. Thalassemia
Bite or blister Exposed to oxidant G6PD deficiency
d. Lead poisoning
cells stress

HEREDITARY SPHEROCYTOSIS

High Yield Points


•• Most common mutation in Here­di­tary Spherocytosis is AnkyrinQ> band 3> Spectrin and band 4.2Q
•• Peripheral Smear shows Micro-spherocytosis of uniform size in Hereditary spherocytosis; (variable
size in immune hemolytic anemia)
•• In Spectrin mutations, the most common RBC abnormality is Here­ditary Elliptocytosis > sphero­cy­
tosisQ
Question 5
Definition Inherited intrinsic defect in the red cell membrane leading to spherocytesQ
A 2-year-old child pre­sents with
Inheritance Autosomal Dominant in 75%Q cases anemia & sple­ nome­galy. There is
Proteins forming •• Spectrin (α/β)-major protein forming tetramersQ a family history of anemia, sple­
RBC membrane •• Actin-binds spectrin tetramersQ nec­tomy & cholecys­tec­tomy in
cytoskeleton •• Protein 4.1- binds spectrin to inner surface of RBC membrane father. The child’s condition is
•• Band 3- RBC ‘s transmembrane ion transporterQ most commonly due to mutation in
•• Ankyrin-bridge between Spectrin and Band 3 helped by Pallidin (band 4.2)Q which gene?
Pathogenesis Spectrin/Ankyrin/Band-3 defect

Loss of membrane surface area-leads to spherocytic shape

Decreased RBC deformability/plasticity

RBC entrapment in splenic sinusoids

Extravascular destruction of RBCs in splenic Reticulo-Endothelial cellsQ
↓ a. Ankyrin b. Spectrin
Jaundice, Splenomegaly, Spherocytes on smearQ c. Band 3 d. Band 4.2

Contd...
316 Section 3: Systemic Pediatrics

Contd...
Review of Pediatrics and Neonatology

Clinical feature •• Triad of Jaundice + hemolytic anemia + splenomegaly (with a family history of
gall stones); Pigment typeQ of gall stones are common
Diagnosis M •• Peripheral Smear: Micro-spherocytosis of uniform size
•• Increased fragility on Osmotic fragility testing (OFT): shift of curve to rightQ
Crisis episodes •• Aplastic crisis (due to Parvovirus B infection)Q
•• Hemolytic crisis (infectious mononucleosis)Q
Treatment •• Phototherapy and exchange transfusion after birth to decrease bilirubin levels.
•• Splenectomy eliminates most of the hemolysis

Question 6  G6PD DEFICIENCY


A 7-year-old child from West Pathophysiology •• G6PD is X-linked disorder
Bengal presented with high •• 2 clinical syndromes—episodic hemolytic anemia and chronic non
grade fever with chills and rigor. spherocytic hemolytic anemia
Peripheral smear examination •• G6PD is important for synthesis of NADPH which maintains glutathione in
revealed Plas­ modium vivax. He reduced functional state, which protects RBC from oxidant stress
was given treat­ment for the same. 2 normal variants G6PD B+: most common normal variant; in middle east and Asian
But he again presented with 3 Abnormal variants •• G6PD A Variant—most common clinical variant-seen in African American
jaundice, and a repeat peripheral population—Enzyme activity 5–15% of normal.
smear showed following. What is •• G6PD B Variant—also called mediterranean variant;
your diag­nosis? Most severe variant. Enzyme activity < 5% of normal
•• G6PD canton variant—occurs in Chinese. Enz activity about 5%
Triggered by: Fava beans, infections, and drugs
Diagnosis: •• Poikilocytes: bite cells or blister cells; HEINZ Bodies
•• LDH & unconjugated bilirubin are high
•• Confirmation of diagnosis is by estimation of RBC G6PD levels
Treatment •• Rx is mainly prevention and supportive

Drugs that carry risk of clinical hemolysis in persons with G6PD deficiency
Definite Risk Possible Risk Doubtful Risk
a. Viral Hepatitis Antimalarials •• Primaquine •• Chloroquine •• Quinine
b. Sickle cell anemia •• Dapsone/chlorproguanil
c. Hemolytic uremic syndrome
d. Glucose 6 Phosphate dehydroge- Sulphonamides/ •• Sulfamethoxazole •• Sulfasalazine •• Sulfisoxazole
nase deficiency sulphones •• Dapsone •• Sulfadimidine •• Sulfadiazine
Antibacterial/ •• Cotrimoxazole •• Ciprofloxacin
antibiotics •• Nalidixic acid •• Norfloxacin
•• Nitrofurantoin
Antipyretic/ •• Acetanilide Acetylsalicylic acid high Acetylsalicylic acid (< 3 g/d)
analgesics •• Phenazopyridine dose (> 3 g/d) Acetaminophen
Others •• Naphthalene •• Vitamin K analogues •• Doxorubicin
•• Methylene blue •• Ascorbic acid > 1 g •• Probenecid
•• Rasburicase

 THALASSEMIA
A. ALPHA-THALASSEMIA
•• People who do not produce enough alpha globin chain have alpha-thalassemia
•• Alpha chain is made by 4 genes, each gene contributes to 25% of the α-globin chains.
Severity of α-thalassemia varies, depending on the number of a-globin genes affected.
Classification of Alpha Thalassemia Syndromes M

No. of α-globin
Syndrome Clinical features Hb Pattern genes affected
Silent carrier No anemia, normal red cells 1–2% Hb Bart (r4) at birth 1 (aa/a-)
Alpha Mild anemia, hypochromic, 5-10% Hb barts at birth 2 (aa/.. or a-/a-)
thalassemia trait microcytic red cells

Contd...
Chapter 14: Pediatric Hematology 317

Contd...

Section 3: Systemic Pediatrics


High Yield Points   M
No. of α-globin
Syndrome Clinical features Hb Pattern genes affected •• Mentzer index is MCV/RBC count
Hemoglobin H Moderate hypochromic, 5-30% HbH (4 beta) 3 (--/-a) •• Normal Hb electrophoresis
microcytic anemia shows-HbA1-95%, HbA2-1.2-3.4% &
Hydrops fetalis Death in utero caused by Mainly Hb barts (g4), small 4 (--/--) HbF = <2%
severe anemia amount of HbH

B. BETA THALASSEMIA
•• In β Thalassemia, there is decreased production of β chains
•• Most common cause of β(+) thalassemia is Splicing mutation.

Classification of Beta Thalassemia Syndromes


β-globin genes
Syndrome Clinical features Hb pattern deleted
Silent carrier Asymptomatic, no anemia Normal, diagnosed by Heterozygous state
chain synthesis (b/b+)

Thalassemia trait Mild anemia, hypochromic Elevated HbA2 > 3.4 Heterozygous state
High Yield Points   M
microcytic red cells (b/b0) 24-hr incubated OFT is the most
sensitive test to diagnose HSQ
Thalassemia Moderate anemia, may HbF elevated Homozygous state
intermedia require some transfusions (b/b+)

Thalassemia major Dependent on blood HbF markedly elevated Homozygous state


transfusion for survival, (b0/b0)
require regular transfusions
*β0—beta chain absent; β+—partial production of beta chain; β—normal production of beta chains.

Treatment
•• Blood transfusions to maintain pretransfusion hemoglobin level between 9.5–10.5 g/dl
•• The only curative treatment for thalassemia major is HSCT-Hematopoietic stem cell trans­
plantation
•• Drug of Choice for transfusional hemosiderosis in thalassemia—Subcutaneous Desferrio­
xamine + oral Deferiprone/Deferasirox.

SICKLE CELL ANEMIA

High Yield Points  


Sickle cell anemia:
•• Most common structural hemoglobinopathy is sickle cell disease
•• Most common clinical feature acute painful crises
•• Increased incidence of osteomyelitis due to salmonella
•• Heterozygous sickle trait protects against falciparum malaria. Question 7
Identify the abnormality in given
Defect M Point mutation in 6th codon of β-globinQ → replacement of glutamate with valine peripheral smear?
Production of HbS with abnormal physiochemical properties that promotes the
polymerization of deoxygenated hemoglobin

Pathogenesis
HbS on deoxygenation forms long polymers

Needle like polymers cause RBC membrane damage

Initially, reversible sickling → Vaso-occlusive spasms &
microvascular obstruction → ischemic tissue damage

Irreversible sickling a. Megaloblastic anemia
↓ b. Thalassemia
Hemolysis in reticulo-endothelial cells (extravascular hemolysis) c. Sickle cell anemia
d. Iron deficiency anemia
Contd...
318 Section 3: Systemic Pediatrics

Contd...
Review of Pediatrics and Neonatology

Question 8 Clinical •• Hand-foot syndrome or dactylitisQ: of the bones of the hands or feet, or both
features •• Priapism in 45% of affected males and erectile dysfunctionQ
The following X-ray findings in
•• Stroke and retinopathyQ leading to loss of visual acuity and even blindness
skull can be a manifestation of?
•• AutosplenectomyQ: multiple infarcts in splenic artery causes spleen to be reduced to
fibrous tissue (splenic atrophy → spleen may become non-palpable)Q
•• Chronic hemolysis: Generalized impairment of growth and development
•• Renal involvement: Papillary necrosis & hyposthenuriaQ (inability to concentrate urine)
•• Cardiomegaly is a feature
•• Infections: mainly by encapsulated bacteria

Diagnosis •• Peripheral smear finding:


a. AML –– Anisopoikilocytosis, polychromasia, Increased Retic %
b. Hereditary spherocytosis –– Irreversibly sickle RBCs and target cells (increased after autosplenectomy)
c. G-6PD deficiency –– Howell-Jolly bodies due to aspleniaQ
d. Thalassemia •• Sickling test: Mixing a blood sample with an oxygen consuming reagent, such as
metabisulfite or dithionate induces sickling of RBCs, if HbS is presentQ
•• Bone marrow: hyperplastic with compensatory erythroid hyperplasia
•• Spleen biopsy: Gamma gandy bodiesQ
•• X-ray: “Crew-cut” appearance of skull, fish-mouth vertebraQ
•• Hemoglobin electrophoresis to detect HbS
•• Hb-HPLC (High Performance Liquid Chromatography): investigation of choiceQ
High Yield Points   •• Prenatal diagnosis: by analysis of fetal DNA obtained by amniocentesis or chorionic
biopsy
•• Quantitative liver iron by MRI is the
best indicator of total-body iron Treatment •• Avoiding infections, and prompt correction of any inflammation
stores. •• Pneumococcal and Haemophilus influenza vaccination early in life
•• Quantitative cardiac iron should be •• Acute painful crisis: vigorous hydration, and aggressive analgesia
obtained after 7 years of transfusion •• Nonsteroidal anti-inflammatory agents for sickle cell arthropathy
therapy. •• Hydroxyurea as a mainstay of therapy for patients with severe symptoms.: HbF levels
increase in most patients within a few months
•• 5-azacytidine was the first agent found to elevate HbF
•• Bone marrow transplantation; Gene therapy
High Yield Points   M 
In patients with sickle cell disease

Primary High Yield Points


Most common   M 
sites of
pathogen (s) Crises in Sickle Cell Disease
infection
Septicemia S. pneumoniae Vaso-occlusive crises/ Severe pain in the affected region. Most common sites are bonesQ,
Acute painful crises lungs, liver, brain, spleen & penis
Meningitis S. pneumoniae
Acute chest syndromeQ Fever, cough, chest pain & pulmonary infiltrates
Osteomyelitis, Salmonella spp, S.
Sequestration crises Splenic enlargement, hypovolemia, and sometimes shock
septic arthritis pneumoniae
Aplastic crises Due to infection of red cell progenitors by parvovirus B19Q
Pneumonia Mycoplasma
pneumonia,
Respiratory viruses
 PANCYTOPENIA
•• Pancytopenia refers to a reduction below normal values of all 3 peripheral blood lineages:
leukocytes, platelets, and erythrocytes.
•• 3 categories of pancytopenia depending on marrow findings.

Hypocellular marrow Cellular marrow Marrow infiltration


•• Inherited/constitutional Primary bone marrow disease: e.g. •• Metastatic solid tumors
marrow failure syndromes, Acute leukemia, MDS •• Myelofibrosis,
•• Acquired aplastic anemia of •• Hemophagocytic
varied etiologies Secondary to systemic disease, e.g. lymphohistiocytosis
•• Hypoplastic variant of myelo­ SLE, vitamin B12 or folate deficiency, •• Osteopetrosis
dysplastic syndrome (MDS) overwhelming sepsis, sarcoidosis,
•• Some cases of paroxysmal hypersplenism
noctur­nal hemoglobinuria
Chapter 14: Pediatric Hematology 319

Inherited Pancytopenia Syndromes

Section 3: Systemic Pediatrics


•• Fanconi anemia: Thumb and radial malformations, café-au-lait spots, short stature, renal
Question 9
malformations, hypogonadism Which of the following condition
•• Shwachman-Diamond syndrome: Short stature, abnormal thorax, pancreatic insufficiency can the given figures be associated
•• Dyskeratosis congenita: Lacy reticular pigmentation of skin, dystrophic fingernails and with?
toenails, premature graying of hair, short stature
•• Congenital amegakaryocytic thrombocytopenia: Petechial rash, bleeding in 1st year of life
and developmental delay
•• Reticular dysgenesis: A variant of SCID in which cellular and humoral immunity are absent
and severe lymphopenia and neutropenia are also seen.

FEATURES OF FANCONI’S ANEMIA


Part of the body Manifestations
Skin change Darkened areas of the skin, cafe-au-lait spots, vitiligo a. Parvo B infection
Upper limbs Absent or hypoplastic thumbQ, hypoplastic radii, dysplastic ulnae, clinodactyly b. MDS
Gonads Abnormal or atrophic testis c. Down's syndrome
d. Fanconi anemia
Head and face Microcephaly,Q sprengels deformity, spina bifida
Kidneys Ectopic or pelvic, horse shoe, hypoplastic or dysplastic
Hematological Pancytopenia + thrombocytopenia and neutropenia these are the most
abnormalities common and most serious symptoms of Fanconi’s anemia
Complications Bone marrow failure, Myelodysplastic syndromes High Yield Points  
 LYMPHOID NEOPLASMS Chromosomal breakage analysis
is used for the diagnosis of Fanconi
•• Lymphoid neoplasms are tumors of: B-cells, T-cells and natural killer cells
anemia.
•• Precursor B-cell and T-cell neoplasm is Acute lymphoblastic leukemia (ALL).

High Yield Points  


•• Incidence of ALL is 3-4 cases/1,00,000 children below 15 years of age

ACUTE LYMPHOBLASTIC LEUKEMIA (ALL)


High Yield Points   M
•• Leukemias are the most common malignant neoplasms in child­hood
•• B cell ALL is more common in children than T-cell
•• Superior mediastinal syndrome is more common in adolescent boys with T-ALL.
ALL accounts for about 77% of cases of childhood leukemia.
Factors Predisposing to Childhood Leukemia
Genetic Conditions Environmental Factors
•• Down syndrome, Fanconi anemia, Bloom syndrome, •• Alkylating agents
Diamond – Blackfan anemia •• Nitrosourea
•• Shwachman – Diamond syndrome, Kostmann syndrome •• Benzene exposure
•• Neurofibromatosis type I, Li-fraumeni syndrome •• Epipodophyllotoxin
•• Ataxia–telangiectasia, SCID, PNH •• Ionizing radiation

Cellular Classification of ALL


French-American-British (FAB) system divides ALL into three morphological subtypes.
Cytologic features L1 L2 L3
Cell size Small cells, Homogenous Large, Heterogenous Large, Homogenous High Yield Points  
in size
Classification Categories
Amount of cytoplasm Scant Variable Moderately abundant
Nucleoli Small & inconspicuous One or more, often large One or more, often prominent 1. Morphology FAB L1, L2, L3
criteria categories
Nuclear chromatin Homogenous Heterogenous Finely stippled and homogenous
Nuclear shape Regular Irregular clefting Regular 2. Immunophenotype Precursor B-cell,
B-cell or T-cells
Basophilia of Variable Variable Intensly basophilic
cytoplasm
Cytoplasmic Variable Variable Prominent
vacuolation
320 Section 3: Systemic Pediatrics

Diagnosis
Review of Pediatrics and Neonatology

High Yield Points  


•• Peripheral blood picture indicates bone marrow failure, anemia, thrombocytopenia,
•• Two most important prognostic elevated WBC count with/without Leukemic Blast Cells (atypical Lymphocytes)
factors of ALL include age at •• ALL is diagnosed by a bone marrow evaluation that demonstrates > 25% Lymphoblasts.
diagnosis and initial TLC
•• Patients with minimal residual
disease MRD <0.01% on day 29 are
Prognostic Factors in ALL
at low risk of relapse Factor Good prognosis Bad prognosis
Race White Black
Age 2–8 years < 1 year, > 10 yearsQ
Sex Female MaleQ
Types L1 L2, L3
Cytogenetics Hyperploidy; t (9;12) Hypoploidy; t(4;11) t(9;22)-BCR-ABL;
t(12;21), Trisomy 4, 10, 17 t(8;14); t(1;19)
TLC < 20 × 109/L > 50 x 109/L
Meningeal involvement (–) (+)
High Yield Points   Mediastinal mass (–) (+)
Immunophenotype Early pre - B-cell; CD10+ Pre - B-cell; Mature B-cell, T-cell ALL
Average duration of treatment in ALL
ranges between 2–2 1/2 years Testicular involvement Absent Present
Time for remission < 14 days > 28 days
Minimal residual disease < 10–4 < 10–3

Management
Treatment of ALL is divided into 4 Stages
Induction Lasts 4–6 weeks. Current induction regime includes vincristine, prednisolone,
therapy L-asparaginase and anthracycline.
CNS prophylaxis Essential to eradicate Leukemic cells which have passed the blood brain barrier.
It comprises cranial irradiation with intrathecal methotrexate. Other alternative
Question 10 regimens include use of Methotrexate, Hydrocortisone, Cytarabine.
Intensification To tackle problem of drug resistance. Commonly used agents include
Which of the following hae­ ma­
therapy methotrexate, L-asparaginase, cyclophosphamide, cytarabine and
tological malignan­cies can lead to
Epipodophyllotoxin.
following?
Maintenance To prevent relapse. It is continued for as additional 2–2.5 years after induction
therapy remission. The main agents used include 6-mercaptopurine daily and
methotrexate once a week given orally.

 MYELOID NEOPLASMS
•• Myeloid neoplasms are relatively less commonly seen in children
•• They can be classified as: Acute myeloid leukemia (AML), Myelodysplastic syndrome and
a. MDS myeloproliferative disorders (CML, JMML).
b. Childhood-CML
c. ALL ACUTE MYELOID LEUKEMIA (AML)
d. AML
High Yield Points  
•• AML accounts for 15–20% of leukemia in children
•• Congenital leukemia (occurring du­ring 1st 4 weeks of life) is mostly AML
•• Down syndrome is the most common genetic predisposing factor associated with risk of developing AML
in the first 3 years of life
•• Early bone marrow relapse before completing maintenance therapy of all has worst prognosis.

Classification of AML
High Yield Points   Genetic
AML subtype Associated features
•• Subcutaneous nodules or blue­ abnormality
berry muffin lesions, infiltra­tion M0—Minimally differentiated Inv 3q, t (3: 3) MPO–ve; Auer rods –ve
of gingiva especially in M4 and M5 M1—AML without maturation Auer rods –ve
subtypes
•• Chloromas (granulocytic sarcomas) M2—AML with maturation t (8;21), t (6;9) •• Chloromas;
typically are associated with M2 AML •• Aurer rods are present; blasts are MPO +ve
with t(8;21) translocation Contd...
Chapter 14: Pediatric Hematology 321

Contd...

Section 3: Systemic Pediatrics


Question 11
Genetic
AML subtype Associated features A 5-year-old female presented with
abnormality
M3—Promyelocytic leukemia t (15;17) •• DIC,Q Low leukocyte count fever, bleeding gums and fatigue
•• Most curable form of AML for 4 days. On examination she
•• Auer rods are most common had splenomegaly. CBC shows Hb
•• Blasts are called faggot cells of 6 gm%, TLC 74,000/uL, Platelet
•• Promyelocytes are MPO and NSE +ve 35,000/uL Bone marrow aspiration
M4—Myelomonoblastic 11q abnormalities, inv •• CNS, skin & gum involvement seen showed these cells as com­ prising
leukemia 3, t(3;3), t(6;9) •• MPO +ve & auer rod +ve 50% of marrow. What is your
•• Non specific esterase +ve (monoblastic) diagnosis?
M5—Monoblastic leukemia 11q abnormalities, •• CNS, skin & gum involvement
t(9;11) t (8;16) •• Auer rods absent but MPO –ve & NSE +ve
M6—Erythroleukemia •• Dysplastic erythroid precursors
M7—Megakaryoblastic L t(1;22) •• Blasts of megakaryocytic lineage

Clinical Features
•• Pallor, bleeding, fever or fatigue
•• Infants have more organomegaly, high WBC counts & CNS disease at diagnosis
•• Disseminated intravascular coagulation is common in AML– M3.

High Yield Points   a. Infectious mononucleosis


b. Hemophagocytic syndrome
•• Myeloblasts are positive for mye­loperoxidase (MPO), Sudan black B and monoblasts are positive c. Acute leukemia
for nonspecific esterase (NSE) d. Myelofibrosis
•• AML-M7 is most common leukemia associated with Down syndrome

Treatment
•• Induction: Cytosine arabinoside and an anthracycline (doxorubicin or daunorubicin)
•• Consolidation (post-remission) therapy: Cytosine arabinoside, etoposide
•• Patients with AML M3 are treated with all transretinoic acid (ATRA) and systemic
chemotherapy; Arsenic may also be used for treatment.
JUVENILE MYELOMONOCYTIC LEUKEMIA/JUVENILE CML
•• It is associated with NF1, monosomy 7, del 7q & PTPN II mutation
•• Philadelphia chromosome or bcr/abl fusion gene is absent
•• Most common age group affected is less than 2 years
•• Presents with hepatosplenomegaly, lymphadenopathy & café au lait spots
Question 12
•• Anemia, thrombocytopenia, leucocytosis, absolute monocytosis and high Hb F A 10-year-old child with bilateral
•• The treatment of choice is hematopoietic stem cell transplantation (HSCT). cervical lymphadenopathy. Lymph
node biopsy was performed, which
showed cells as given in the figure.
 HODGKIN'S LYMPHOMA

New WHO classification of Hodgkin's disease


Nodular lymphocyte predominance
Classical Hodgkin lymphoma
•• Lymphocyte rich
•• Mixed cellularity
•• Nodular sclerosis
•• Lymphocyte depletion

Ann Arbor Staging Classification for Hodgkin Lymphoma What is the diagnosis?
a. Burkitt Lymphoma
Stage Definition b. Hodgkin disease
I Involvement of a single lymph node (I) or of a single extralymphatic organ or site (IE) c. Acute Lymphoblastic Leukemia
d. Chloroma
II Involvement of 2 or more lymph node regions &/or extralymphatic sites on the same side of
the diaphragm
III Involvement of lymph node regions on both sides of the diaphragm (III), which may be accom- High Yield Points  
panied by involvement of the spleen (IIIS)
•• Patients of Noonan syndrome &
IV Diffuse or disseminated involvement of 1 or more extralymphatic organs or tissues with or neurofibromatosis-1 have predi­
without associated lymph node involvement lection for JMML.
322 Section 3: Systemic Pediatrics
Review of Pediatrics and Neonatology

High Yield Points  


Treatment regimen used for treatment of Hodgkin lymphoma in children is ABVD, that comprises of Doxo-
rubicin (Adriamycin), bleomycin, vinblastine, dacarbazine

LANGERHANS CELL HISTIOCYTOSIS

Etiology Reactive proliferation of the dendritic cells.

Types •• Eosinophilic granuloma: localized disease characterized by not only proliferation of


histiocytes but also other cells and predominantly eosinophils
High Yield Points  
•• Hand-Schuller-Christian disease: triad of lytic bone defects, diabetes insipidus
and exophthalmos
Three classes of histiocytosis are: •• Letterer-Siwe disease: involvement at multiple foci and organs
•• Class I: Langerhans cell
histiocytosis, Clinical •• Incidence: adults = children, i.e. 50% in both
•• Class II: Hemophagocytic Presentation •• Localized disease: osteolytic bony lesions is most common presentation
lymphohistiocytosis and •• Most common bone: skull & mastoid involvement (chronic ear discharge)
•• Class III: Acute monocytic leukemia, •• Seborrheic dermatitis of the scalp and diaper area (diaper rash) is seen
true malignant histiocytosis. •• Bone marrow dysfunction can cause pancytopenia and compen­satory extramedullary
hematopoiesis can lead to hepatomegaly and splenomegaly
•• Cystic lesions in the lung (honeycomb lung) ± pneumothorax
•• Pituitary involvement can lead to diabetes insipidus & GH deficiency

Diagnosis •• Biopsy of lesions & demonstration of histiocytes (antigen presenting cells)


•• These cells are CD1a, S-100 positive and MHC II (HLADR) positive and have Bierbeck
granules of tennis racket shape

Treatment •• Localized lesions intralesional corticosteroids, radiation & curettage


•• Multifocal diseases vinblastine or etoposide with prednisolone
•• Refractory cases can be treated with stem cell transplantation & drugs like
cladribine, interferon & etanercept

Question 13
Given below is skull X-ray lateral view and electron microscopy picture of biopsy done from
lesion of 1/m presenting with bone pain, What is your diagnosis?

a. Multiple myeloma b. LCH


c. Thalassemia d. AML

 THROMBOCYTOPENIA

Differential Diagnosis of Thrombocytopenia in Children and Adolescents

DESTRUCTIVE THROMBOCYTOPENIAS
Primary Platelet Consumption Syndromes Combined Platelet & Fibrinogen Consumption
•• Immune thrombocytopenias: Acute & chronic ITP Syndromes
•• Evans syndrome; SLE; HIV •• Disseminated intravascular coagulation (DIC)
•• Kasabach-Merritt syndrome
•• Antiphospholipid antibody syndrome
•• Virus-associated hemophagocytic syndrome
•• Neonatal immune thrombocytopenia
•• Drug-induced thrombocytopenia (e.g. Heparin)
•• Post-transfusion purpura
•• Type 2B VWD or platelet-type VWD
Contd...
Chapter 14: Pediatric Hematology 323

Contd...

Section 3: Systemic Pediatrics


IMPAIRED PLATELET PRODUCTION
Aplastic anemia Osteopetrosis
Myelodysplastic syndrome Nutritional deficiency (iron, folate, vitamin B12)
Marrow infiltrative process Drug- or radiation-induced thrombocytopenia
SEQUESTRATION
•• Hypersplenism •• Hypothermia •• Burns

IMMUNE THROMBOCYTOPENIA (ITP)


Incidence •• Peak age group 1–4 years; in childhood age—male = female incidence High Yield Points  
Etiology •• EBV, HIV H.pylori, rarely following vaccines, SLE & Hepatitis C Most common cause of acute onset
•• Adolescent with chronic ITP—SLE should be ruled out of thrombocytopenia in an otherwise
Presentation A previously healthy child has sudden onset of generalized petechiae & purpura well child is ITP
There is a history of a preceding viral infection 1–4 weeks before the onset Physical
examination is normal, except for petechiae and purpura
Pathogenesis Autoantibody directed against the gpIb/IX & gp IIb/IIIa platelet surface
Usually more often in late winter after the peak season of viral respiratory illness L at e s t U p d at e s
Lab •• Severe thrombocytopenia (platelet count < 20,000/mm3)
is common •• Two effective agents that act to
findings M •• Coagulation profile is normal stimulate thrombopoiesis, Romi­
•• Peripheral smear: Platelet size increased or Normal, BM examination shows normal or plastin & Eltrombopag, are
increased number of megakaryocytes other useful investigations: HIV testing & coomb’s test. approved by the FDA to treat adults
•• Bone marrow indications: Abnormal WBC count, unexplained anemia with chronic ITP & have shown
encouraging results in children;
Treatment M •• Platelets transfusion is usually avoided (unless life threatening bleed is present)
•• Intravenous immunoglobulin (IVIG) induces a rapid rise in platelet count by
downregulating Fc mediated phagocytosis of antibody coated platelets
•• Intravenous anti D therapy: used in Rh positive patients; RBC—antibody complexes
bind to macrophage Fc receptors and interfere with platelet destruction
•• Corticosteroids: Prednisolone in dose of 1–4 mg/kg/day is used for 2–3 weeks or until a
rise of platelet count to >20,000/mm3 has been achieved
•• Splenectomy is reserved for cases with severe ITP which have lasted > 1 year and whose Question 14
symptoms are not easily controlled with medical therapy. It can also be considered A previously healthy child has
when life threatening hemorrhage complicates acute ITP sudden onset of red spots on body.
•• Rituximab induces remission in 30-40% children & has been used as an alternative to
There is a history of a preceding
splenectomy
viral infection 1–4 weeks before
the onset.
The differences between acute and chronic ITP are:
Features Acute Chronic
Duration 4–6 weeks > 6 months/years
Onset Acute Insidious
Peak age incidence 2–6 years All ages
Sex predilection None F:M = 3:1
Antecedent infection Viral in 50% cases Unusual a. Dengue fever
Spontaneous remission 80% Uncommon b. Hemophilia A
c. Thrombotic thrombocytopenic
purpura
d. Idiopathic thrombocytopenic
High Yield Points   purpura
Defects of Platelet Function
Disease Abnormality
Bernard Soulier syndrome gp Ib / IX defect; Giant platelets seen
Glanzmann thrombasthenia gp IIb / IIIa defects; Normal sized platelets
Dense body deficiency Absence of the granules that contain ADP, ATP, Ca2+, and serotonin.
Gray platelet syndrome Caused by the absence of platelet α granules, resulting in large platelets
that are large and appear gray on Wright stain of peripheral blood
324 Section 3: Systemic Pediatrics

 HEMORRHAGIC DISEASE OF NEWBORN (HDN)


Review of Pediatrics and Neonatology

It is characterized by bleeding from various sites due to a transient deficiency of vitamin K dependent
clotting factors (II, VII, IX and X). It is now called ‘Vitamin K deficiency bleeding.

Classification
Early Onset Classic Disease Late Onset
Age 0–24 hours 2–7 days 1–6 months
Site of Cephalohematoma Gastrointestinal Intracranial/Thoracic
hemorrhage Subgaleal/Intracranial Ear-nose-throat-mucosal Gastrointestinal
High Yield Points   GI/Umbilicus Intracranial Cutaneous
abdominal Circumcision/Injection Ear-nose-throat-mucosal
•• Breast milk is a poor source of
vitamin K and hemorrhagic mani­ Cutaneous Injection sites
festation are more common in Etiology/risks Maternal drugs Vitamin K deficiency Cholestasis-malabsorption of
breast-fed infants (Phenobarbital, Breastfeeding vitamin K (biliary atresia, cystic
•• Vitamin K given at birth helps in phenytoin, warfarin, fibrosis, hepatitis)
preventing HDN rifampin, isoniazid) that Abetalipoproteinemia
interfere with vitamin K Warfarin ingestion

High Yield Points  


 BLEEDING DUE TO SECONDARY HEMOSTATIC PATHWAYS ABNORMALITIES

•• Parahemophilia is due to factor V


deficiency Factor Synonym Disorder
•• Hemophilia A is due to factor VIII I Fibrinogen Congenital deficiency (afibrinogenemia) or dysfunction
deficiency (dysfibrinogenemia)
•• Hemophilia B is due to factor IX
deficiency II Prothrombin Congenital deficiency or dysfunction
•• Hemophilia C is due to factor XI
deficiency V Labile factor, proaccelerin Congenital deficiency (parahemophilia)

VII Stable factor or proconvertin Congenital deficiency

Question 15 VIII Antihemophilic factor Congenital deficiency is hemophilia A (classic hemophilia)


Given below is picture of a 5-year- IX Christmas factor Congenital deficiency is hemophilia B (Christmas disease)
old boy presenting to OPD with
recurrent joint pain & pallor requi­ X Stuart-Power factor Congenital deficiency
ring transfusion since last 4 years.
What is the pro­bable cause? XI Plasma thromboplastin antecedent Congenital deficiency (Hemophilia C)

XII Hageman factor Deficiency is not associated with clinical symptoms

XIII Fibrin-stabilizing factor Bleeding from umbilical cord stump seen


HEMOPHILIA A & B M

Inheritance X-linked Recessive (so males are more commonly affected)

Incidence 1:5000 with 85% having hemophilia A and 15% having hemophilia B
a. ITP b. Hemophilia
c. Osteosarcoma d. Thalassemia Severity •• Severe Hemophilia (levels < 1%)—spontaneous bleeding
•• Moderate hemophilia (1–5% levels)—bleed with mild trauma and
•• Mild hemophilia (levels > 5%) bleed only with significant trauma

Hallmark Of hemophilic bleeding is hemarthrosis


Earliest joint hemorrhages appear most commonly in the ankle

Clinical findings Hemophilia A and hemophilia B are virtually identical

Laboratory findings Increased PTT, Normal PT, Normal BT, and Normal platelet counts

Treatment Factor replacement

Long-term Chronic arthropathy, the development of an inhibitor to either factor VIII or factor
complications IX, and the risk of transfusion-transmitted infectious diseases
Chapter 14: Pediatric Hematology 325

VON WILLEBRAND DISEASE (VWD)

Section 3: Systemic Pediatrics


High Yield Points  
Types •• Type I is AD and caused due to reduced amounts, type II is AD & is because of •• Most common hereditary blee­ding
structurally defective VWF & type III is AR & is due to absent VWF disorder is VWD
Function •• VWF serves to tether platelets to injured subendothelium via GpIb/IX •• Patients with blood group O have
•• Serves as a carrier for factor VIII, protecting it from degradation physiologically low levels of VWF
•• VWF is stored in endothelial cells and in platelet Weibel-Palade bodies and circulates levels compared with other blood
as a large multimeric glycoprotein group

Features •• VWF is an acute phase reactant and hence patients may not bleed with procedures
that incur major stress, such as appendectomy and childbirth but may bleed
excessive at time of cosmetic or mucosal surgery. VWF levels can double or triple
during pregnancy.

Platelet count •• Normal except in type 2B and platelet type VWD in which it is decreased

Acquired VWD •• Seen in patients with underlying lymphoproliferative disorders, including mono-
clonal gammopathies of undetermined significance (MGUS), multiple myeloma,
and Waldenström’s macroglobulinemia

Diagnosis Bleeding time and aPTT are increased, Ristocetin cofactor assay is reduced

Treatment Desmopressin is effective in type 1 and some type 2 cases; vWF concentrates & anti­
fibrinolytics may be used

VWD Variants
Types Characteristics
Type I VWD AD, Most common type, VWF levels decreased but structurally normal

Type 2A Structurally abnormal VWF, prone for proteolysis by ADAMTS 13

Type 2B Increased binding of VWF, with platelets → thrombocytopenia

Type 2M Reduction of platelet binding to VWF

Type 2N Reduction of factor 8 binding to VWF (Autosomal Hemophilia)

Platelet type Increased binding of GpIb receptor on platelets with VWF, associated with
thrombocytopenia

Type III Least common and most severe type; Undetectable levels of VWF
326 Section 3: Systemic Pediatrics
Review of Pediatrics and Neonatology

Answer Keys for Image-Based Questions

Answers Explanations / Identifying features


1. Ans. c. Aplastic anemia The given peripheral smear shows microcytic hypochromic anemia; But in Aplastic Anemia, normo-
cytic normochromic anemia is seen
2. Ans. c. Iron deficiency Smooth & shiny tongue is seen in Iron deficiency anemia
anemia

3. Ans. b. Megaloblastic Peripheral smear shows macro-ovalocytes & hypersegmented neutrophils, which are seen in
anemia Megaloblastic anemia
4. Ans. d. Lead poisoning Basophilic stippling is seen in the given peripheral smear that suggests a diagnosis of Lead poisoning.

5. Ans. a. Ankyrin Anemia with splenomegaly in a child with a family history of anemia, splenectomy & gall stones
suggest hereditary spherocytosis, which is most commonly due to ankyrin gene mutation
6. Ans. d. Glucose 6 Phosphate Peripheral smear showing ‘bite cells’ in a child with malaria, who developed jaundice after treatment
dehydrogenase deficiency with antimalarials, suggest G6PD deficiency
7. Ans. c. Sickle cell anemia Sickle shaped RBCs are seen in peripheral smear suggestive of sickle cell anemia
8. Ans. d. Thalassemia X-ray skull showing ‘crew cut appearance’ is suggestive of expansion of bone marrow as seen in
Thalassemia
9. Ans. d. Fanconi anemia A → A child with short stature, dislocated hips, microcephaly, broad nasal base, epicanthal folds, micrognathia
& thumb defects
B → Abnormal chromosomal fragility on exposure to die-poxy­butane or mitomycin C
10. Ans. d. AML The given picture shows gum hypertrophy, which seen in AML (esp M5 & M4 variety)
11. Ans. c. Acute leukemia Excessive blast cells making up 50% of bone marrow aspirate is suggestive of Acute Leukemia
12. Ans. b. Hodgkin disease The given lymph node biopsy shows Reed Sternberg cell, which is seen in Hodgkin disease
13. Ans. b. LCH A → Lytic lesion involving skull
B → Birbeck granules (tennis racket shape)
Both these are features of Lan­gerhans cell Histiocy-tosis (LCH)
14. Ans. d. Idiopathic Acute onset purpura and thrombocytopenia in a previously healthy child, with preceding history of
thrombocytopenic purpura viral infection is suggestive of ITP
15. Ans. b. Hemophilia A child presenting with recurrent joint swelling (as seen), joint pain & pallor requiring transfusions, suggests
hemarthrosis due to an underlying coagulation disorder like Hemophilia
Chapter 14: Pediatric Hematology 327

Questions
Questions
 NORMAL HEMATOPOIESIS  NUTRITIONAL ANEMIA

1. RBC Lifespan in preterm babies: (JIPMER May 2017) 12. True about iron deficiency anemia is/are: (PGI Nov 2015)
a. < 40 days b. 40-60 days a. Normochromic, normocytic hypoproliferative anemia
c. 75-90 days d. 100-120 days b. Normochromic, normocytic hyperproliferative anemia
2. The two embryonic hemoglobins are: c. Plasma transferrin saturation decreases
 (Recent Question 2017) d. Marked poikilocytosis
a. Fetal hemoglobin and Gower hemoglobin e. Blood transfusion is indicated at Hb < 4.0g/dL
b. Fetal hemoglobin and Portland hemoglobin 13. Transfer of Iron from enterocyte to plasma is inhibited
c. Portland hemoglobin and HbA2 by: (Recent Question 2016)
d. Gower hemoglobin and Portland hemoglobin a. Hepcidin b. DMT-1
3. Which of the following is the major site of erythropoietin c. Ferroportin d. Hepestin
production during the fetal stage: M  14. Iron stores are best indicated by: M 
 (Recent Question 2016)  (Recent Question 2016)
a. Liver b. Yolk a. S. ferritin b. S. Iron
c. Bone d. Spleen c. S. Transferrin d. TIBC
4. First cell of RBC development: (Recent Question 2016) 15. Release of ferroportin store is controlled by:
a. Proerythroblast b. Intermediate normoblast  (Recent Question 2016)
c. Reticulocyte d. Basophilic erythroblast a. Hepcidin b. Transferrin
5. Which of the following is not associated with a high c. Ferritin d. Hepoxin
Reticulocyte count? (Recent Question 2016) 16. Best test for assessment of iron status is: M 
a. Acute bleed b. Hemolytic anemia  (Recent Question 2016)
c. Megaloblastic anemia a. Transferrin b. Ferritin
d. Response to treatment in ‘nutrition-deficiency' anemia c. Serum iron d. Hemoglobin
6. What happens when normal erythrocytes (blood – group
17. True about Hereditary Hemochromatosis are all except:
matched) are transfused into a patient with anemia
secon­dary to an intracorpuscular defect? a. Mutations in the HFE gene (Recent Question 2016)
b. Autosomal recessive disorder
a. Donor cells are destroyed (Recent Question 2016)
c. Excess iron affects organ function
b. Donor cells have normal survival
d. Diagnosed by serum ferritin
c. Depends on the severity of anemia
d. Depends on whether the donor cells are fresh or stored 18. All of the following cause microcytic hypochromic
(older than a week) anemia, except: (Bihar PG 2015)
a. Lead poisoning b. Thalassemia
7. MCV (fl) in infant of 1 month omf age is: (PGI Nov 09) c. Iron deficiency d. Fanconi's anemia
a. 76–80 b. 80–100 19. Iron metabolism and regulation are important for RBC
c. 90–100 d. 101–125 precursor cell. Which of the following helps in regulation
e. 125–135 of iron metabolism but is not specific for iron?
8. Which organ is primary site of hematopoiesis in the fetus  (AIIMS Nov 2015)
before mid pregnancy?  (AIIMS May 06) a. Hepcidin b. DMT-1
a. Bone b. Liver c. Ferroportin d. Ferritin
c. Spleen d. Lung 20. Iron absorption is increased by: M 
9. Hemoglobin with zeta 2 and gamma 2 chains are seen in  (Recent Question 2015)
which of the following?  (APPG 06) a. Phytates b. Tannates
a. Gower 1 b. Gower 2 c. Plant food d. Ascorbic acid
c. Portland Hb d. Fetal Hb 21. Microcytic anemia is not seen in which of the following
10. When does switchover from fetal to adult hemoglobin condition? (Recent Question 2015)
synthesis begin? (AIIMS Nov 05, May 05, Nov 04) a. Osteomyelitis b. Leukopenia
a. 14 weeks gestation b. 30 weeks gestation c. Papillary necrosis d. Rheumatoid arthritis
c. 36 weeks gestation d. 7–10 days postnatal 22. True statement about anemia of chronic disease is:
11. Adult hemoglobin consists of which of the following a. Increased serum ferritin (Recent Question 2015)
tetramer of chains? M b. Increased serum iron
a. 2α+2γ b. 2α+2b c. Increased TIBC
c. 2α+2e d. 2α+2d d. Increased transferrin saturation
328 Section 3: Systemic Pediatrics

23. Most important inflammatory mediator, involved in 33. A nine-month-old boy of Sindhi parents presented to
Review of Pediatrics and Neonatology

anemia of chronic disease: (Recent Question 2015) you with complaints of progressive lethargy, irritability
a. IL-1 b. IL-6 and pallor since 6 months of age. Examination revealed
c. TNF alpha d. IFN-Y severe pallor. Investigation showed Hb-3.8 g%; MCV-
58 fL, MCH-19.4 pg/cell. Osmotic fragility is normal;
24. To diagnose anemia in age group 2 years to 6 years,
Blood film shows target cells and normoblasts. X-ray
level of hemoglobin should be: (Recent Question 2014)
skull shows expansion of erythroid marrow. Which of
a. Hb 14g/dL b. Hb 11g/dL the following is the most likely diagnosis? (AIPGMEE 04)
c. Hb 12g/dL d. Hb 13g/dL
a. Iron deficiency anemia
25. First change of improvement noted after iron therapy is b. Acute lymphoblastic anemia
initiated: M  (Recent Question 2013) c. Hemoglobin D disease
a. Decreased irritability d. Hereditary spherocytosis
b. Reticulocytosis 34. Most sensitive and specific test for diagnosis of iron
c. Increase in serum iron levels deficiency is: (AIPGMEE 03)
d. Replenishment of iron stores a. Serum iron level b. Serum ferritin levels
26. True about iron deficiency anemia in children: M  c. Serum transferrin receptor populations
a. Iron absorption from terminal ileum (PGI June 09) d. Transferrin saturation
b. Cow milk contains less iron than breast milk 35. A 6 months old infant with Hb of 6 gm% has peripheral
c. Serum ferritin depletes first blood showing target cells. Family history of similar
d. Decreased alertness complaints is Positive. Diagnosis can be best done with
e. Decreased Red cell distribution width (RDW) which of the following? (AIIMS Nov 01; AIPGMEE 2001)
27. Which of the following findings is diagnostic of iron a. Hb electrophoresis b. ELISA
deficiency anemia?  (AIPGMEE 07) c. Coomb's Test d. Osmotic fragility
a. Increased TIBC, decreased serum ferritin 36. Which is not seen in iron deficiency anemia?
b. Decreased TIBC, decreased serum ferritin a. Hypersegmented neutrophils (AIPGMEE 2000)
c. Increased TIBC, increased serum ferritin b. Microcytosis precedes hypochromia
d. Decreased TIBC, increased serum ferritin c. MCHC < 50%
28. Megaloblastic anemia due to folic acid deficiency is d. Commonest cause of anemia in India
commonly due to:  (AIPGMEE 06) 37. A 6-month-old baby with severe pallor and hepatosple­
a. Inadequate dietary intake nomegaly. Similar history is present in a sibling.
b. Defective intestinal absorption Investigation of choice:  (AIPGMEE 2002)
c. Absence of folic acid binding protein in serum a. Bone marrow biopsy b. Hb electrophoresis
d. Absence of glutamic acid in the intestine c. Hb estimation d. Platelet count
29. The earliest indicator of response after starting iron in a 38. Iron requirement is determined from the equation:
6-year-old girl with iron deficiency is: (AIPGMEE 06) a. 3 × wt(kg) × Hb deficit (gm/dL) (AMU 05)
a. Increased reticulocyte count b. 3.3 × wt(kg) × Hb deficit (gm/dL)
b. Increased hemoglobin c. 4 × wt(kg) × Hb deficit (gm/dL)
c. Increased ferritin d. Increased serum iron d. 4.3 × wt(kg) × Hb deficit (gm/dL)
30. True about iron deficiency anemia: (PGI June 05)
a. Microcytic hypochromic anemia  HEMOLYTIC ANEMIA
b. Decreased TIBC c. Increased ferritin
d. Bone marrow iron decreased earlier than serum iron 39. A child with Thalassemia major has a history of
e. Decreased transferrin saturation treatment for cardiac arrhythmia due to iron overload.
31. Which of the following is true about oral therapy for iron Now the child came for regular transfusion and while
deficiency anemia?  (PGI Dec. 05)
transfusion the child became anxious and developed
complaints of backpain. What is the next management?
a. In 300 mg elemental iron given, 100 mg gets absorbed
a. ECG (AIIMS Nov 2018)
b. Reticulocytosis appears in one to 2 weeks and then peaks
b. Stop transfusion and look for clerical errors
in 3–4 weeks
c. Continue transfusion while monitoring for vitals
c. Response to treatment seen in 4 weeks
d. Check for reddish discolouration of urine
d. Decrease in absorption occurs with improvement of
symptoms 40. Which of the following can result in dactylitis?
e. Stop the treatment after normalising the Hb  (NEET pattern Jan 2018)
32. A 5-year-old girl came with history of progressively a. Hemophilia b. Von Willebrand disease 1
increasing pallor since birth and hepatosplenomegaly. c. Thalassemia d. Sickle cell anemia
Which of the following is the most relevant test for 41. A child aged 2 years presents with non-specific symp­
achieving diagnosis?  (AIPGMEE 04) toms suggestive of anemia. On examination, hepato­
a. Hb electrophoresis spleno­megaly is present. Peripheral smear shows target
b. Peripheral smear examination cells. He has microcytic hypochromic picture & Hb of 6
c. Osmotic fragility test gm%. There is positive family history. Next investigation
is: (FMGE June 2018)
d. Bone marrow examination
Chapter 14: Pediatric Hematology 329

55. Hemolysis in G6PD (glucose 6 phosphate dehydro­

Questions
a. NESTROFT test b. HbA1c estimation
genase) enzyme deficiency may occur with all of the
c. Hb electrophoresis d. Osmotic fragility test
following drugs except: (UPSC CMS 2015)
42. Which of the following is ABSENT in hemolytic
anemia? M  a. Primaquine b. Phenacetin
c. Probenecid d. Penicillin
a. Increased indirect bilirubin (PGI Jan 2017)
56. Screening test for thalassemia: M  (Recent Question 2015)
b. Increased direct bilirubin
c. Increased reticulocyte count a. Alkali denaturation test b. Kleihauer test
d. Jaundice c. Hb electrophoresis d. NESTROFT
e. Increased Hematocrit 57. Bart hemoglobin is tetramer of: (Recent Question 2015)
43. Hair on end appearance is seen in: M  a. α chain b. β chain
 (Recent Question 2016)
c. γ chain d. δ chain
58. Hemoglobin H disease is caused by deletion:
a. Dermoid cyst b. Thalassemia
 (Recent Question 2015)
c. Kwashiorkar d. Tinea capitis
a. Single α globin chain b. Two α globin chains
44. Best treatment for Sickle cell anemia is:
c. Three α globin chains d. All α globin chain
 (Recent Question 2016)
59. All the following features precipitate sickling of HbS
a. Hydroxyurea b. Sulphonamide except: (Recent Question 2015)
c. Iron Injection d. Blood transfusion a. Hypoxia b. Dehydration
45. Beta globin missense gene mutation seen in: c. Infections d. Alkalosis
 (Recent Question 2016) 60. Schistocytes are characteristic feature of: M 
a. Thalassemia b. Sickle cell anemia a. Hereditary spherocytosis (Recent Question 2015)
c. BART d. HbH b. Microangiopathic hemolytic anemia
46. Mutation in Sickle cell anemia is: (Recent Question 2016) c. G6PD deficiency
a. Frameshift b. Non sense mutation d. Thalassemias
c. Non conservative missense 61. In G6PD deficiency, which cells are more prone for
d. Complete hemolysis? (Recent Question 2015)
47. Autosplenectomy occurs in: M  (MAHA PGM CET 2016) a. Older red cells b. Young red cells
a. Thalassemia b. Sickle cell anemia c. Reticulocytes d. All are susceptible
c. HbH disease d. Hydrops fetalis 62. Which of the following conditions does not cause
48. In beta thalassemia, the most common gene mutation in pancytopenia? (UPSC CMS 2015)
India is: (Recent Question 2016) a. Hypersplenism b. Aplastic anemia
a. IVS1 G C b. Intron 22 c. Cancer infiltrating the bone-marrow
c. 619 bp deletion d. 3.7 bp deletion d. Hemolysis from G6PD enzyme deficiency
49. Fanconi anemia can lead to: (Recent Question 2015) 63. A 5 year old child presented with microcytic hypo­
chromic anaemia. Serum ferritin is low. Which of the
a. B12 deficiency b. Folate deficiency following is least likely? (UPSC CMS 2015)
c. Iron deficiency d. Aplastic anemia
a. Celiac disease b. Thalassemia major
50. Most common mutation in hereditary spherocytosis:
c. Nutritional anemia
 (Recent Question 2015)
d. Paroxysmal nocturnal hemoglobinuria
a. Spectrin b. Ankyrin 64. Best treatment of atypical HUS is:
c. Glycophorin A d. Band 3  (Recent Question 2014)
51. Most common mutation in hereditary elliptocytosis:
a. Plasmapheresis b. Antibiotics
 (Recent Question 2015)
c. IvIg d. Dialysis
a. Spectrin b. Ankyrin 65. In sickle cell anemia, all are true except:
c. Glycophorin A d. Band 4.2  (Recent Question 2014)
52. Drug that is safe in G6PD deficiency: a. Sickle cells b. Target cells
 (Recent Question 2015) c. Howell Jolly bodies d. Ringed sideroblast
a. Primaquine b. Acetanilide 66. Person having heterozygous sickle cell trait is protected
c. Quinidine d. Dapsone from infection of: (Recent Question 2014)
53. Direct Coomb's test is positive in hemolytic anemia due a. Plasmodium falciparum b. P. vivax
to: (Recent Question 2015) c. Pneumococcus d. Salmonella
a. Paraxysmal cold hemoglobinuria 67. All are features of hemolytic uremic syndrome, except:
b. Paroxysmal nocturnal hemoglobinuria  (Recent Question 2014)
c. Idiopathic thrombocytopenic purpura a. Hyperkalemia b. Anemia
d. Hemolytic uremic syndrome c. Renal microthrombi
54. Kleihaeur Bethke test is done for: (APPGMEE 2015) d. Neuro-psychiatric disturbances
a. Cephalopelvic disproportion 68. Thalassemia gives protection against: M 
b. Fetomaternal hemorrhage  (Recent Question 2014)
c. Determining karyotype of normal fetus a. Filaria b. Kala-azar
d. Diagnosing fetal infections c. Malaria d. Leptospirosis
330 Section 3: Systemic Pediatrics

69. Patients with sickle cell anemia commonly have infection 81. What is the definitive finding of G6PD? (Manipal 06)
Review of Pediatrics and Neonatology

with all except: (NEET Pattern 2013) a. Bite cells b. Intravascular hemolysis
a. Streptococcus pneumoniae c. Splenomegaly d. Hemoglobinuria
b. H. influenza-B 82. Regarding G6PD deficiency true are: (PGI June 06)
c. Myco tuberculosis d. All of the above a. Autosomal dominant
70. Salmonellosis is most common in: b. Bite cells seen
 (NBE based NEET 2013–12; PGI June 98) c. Protects against kala azar
a. Sickle cell anemia b. Thalassemia d. Enzyme level directly proportional to age of RBC
c. Hemophilia d. Cystic fibrosis e. Sex preponderance
71. Which of the following hemoglobin (Hb) estimation will 83. All of the following are true of thalassemia major,
be diagnostically helpful in a case of beta thalassemia except: (AIIMS May 06)
trait? (NBE based NEET 2013-12; AIIMS May 06, UP 08) a. Splenomegaly
a. Hb-F b. Hb1-C b. Target cells on peripheral smear
c. Hb-A2 d. Hb-H c. Microcytic hypochromic anemia
72. In β-thalassemia there is:  (NBE pattern 2013) d. Increased osmotic fragility
a. Excess α-chain b. Reduced α-chain 84. Which of the following hemoglobin (Hb) estimation will
c. Excess β-chain d. Reduced β-chain be diagnostically helpful in a case of beta thalassemia
73. True about sickle cell anemia are all except: trait? M  (AIIMS May 06)
 (NEET Pattern 2012) a. Hb-F b. HbA
a. Sequestration is common in older children c. HbA2 d. Hb-H
b. Infection is major cause of mortality 85. A 10-year-old boy presents with mucosal bleeding of 1
c. Glutamic acid is replaced by valine week duration. The investigation of choice that will be
d. Common infections are by encapsulated bacteria most useful in him is:  (COMED 06)
74. HbA2 is increased in: M  (NEET Pattern 2012) a. Prothrombin time b. Clotting time
c. Partial thromboplastin time
a. Beta thalassemia trait b. α-thalassemia
d. Platelet count
c. alpha-thalassemia trait d. All of above
86. Diagnosis of beta thalassemia is established by:
75. A 6 years old child belonging to Punjabi family with
past history of blood transfusions presented with a. NESTROFT Test  (AIPGMEE 05)
hemoglobin–3.5 gm/dL, MCV–30 fL. Peripheral smear b. HbA1 c estimation
findings of microcytic hypochromic anemia with c. Hb electrophoresis
target cell and reduced osmotic fragility. The probable d. Target cells in peripheral smear
diagnosis of patient: (AIIMS Nov 2012) 87. Bart's hydrops fetalis is lethal because: (AIPGMEE 05)
a. Alpha thalassemia b. Beta thalassemia a. Hb Bart's cannot bind oxygen
c. Sickle cell anemia d. G6Pd deficiency b. The excess α-globin form insoluble precipitates
76. Drug which may lead to hemolysis in a child with G6PD c. Hb Bart's cannot release oxygen to fetal tissues
deficiency is? (COMEDK 2011) d. Microcytic red cells become trapped in the placental
a. Penicillin b. Primaquine 88. True about β-thalassemia trait is: (PGI June 04)
c. Ceftriaxone d. Erythromycin a. Increased HbF b. Increased HbA2
77. NESTROFT test is a screening test for: M  (AIIMS Nov 11) c. Microcytosis d. Severe anemia
a. β-thalassemia e. Hydrops fetalis seen
b. Hereditary spherocytosis 89. All of the following statements are true about sickle cell
c. Autoimmune hemolytic anemia disease except: (AIPGMEE 04)
d. Megaloblastic anemia a. Patient may require frequent blood transfusions
78. A neonate has ABO incompatibility. Peripheral smear b. Acute infection is the most common cause of mortality
will show: (AIIMS Nov 2010) before 3 years of age
a. Microspherocytes b. Elliptocytes c. There is positive correlation between concentration of.
c. Fragmented RBCs d. Polychromophilia HbS and polymerisation of HbS
79. The following are the features of thalassemia major d. Patient present early in life before 6 months of life
except: (UPSC-I 09) 90. The primary defect which leads to sickle cell anemia is:
a. Bone marrow hyperplasia  (AIPGMEE 03)
b. Hair-on-end appearance a. An abnormality in porphyrin part of hemoglobin
c. Splenomegaly b. Replacement of glutamate by valine in β-chain of HbA
d. Increased osmotic fragility c. A nonsense mutation in the β-chain of HbA
80. Which of the following is not associated with a high d. Substitution of valine by glutamate in the β-chain of HbA.
reticulocyte count? (UPSC 07) 91. A child died soon after birth. On examination there was
a. Acute bleed hepatosplenomegaly and edema all over body. Most
b. Hemolytic anemia probable diagnosis in:  (AIIMS May 02)
c. Megaloblastic anemia a. α-thalassemia b. β-thalassemia
d. Response to treatment in nutrition deficiency anemia c. Hereditary spherocytosis d. ABO incompatibility
Chapter 14: Pediatric Hematology 331

92. Mutation leading to sickle cell anemia: (PGI June 01) 103. Prominent reticulocytosis is a feature of: (WBPG 2015)

Questions
a. Crossover mutation b. Frame shift a. Aplastic Anemia b. Hemolytic Anemia
c. Deletion d. Non dysjunction c. Nutritional Anemia d. Anemia of chronic disease
e. Point mutation 104. Aplastic anemia can progress to all except: (DPG10)
93. Which of the following is not seen on hemoglobin a. AML
electrophoresis in sickle cell anemia? (AIPGMEE 01) b. Myelodysplastic anemia
a. Hb A b. Hb A2 c. Pure red cell aplasia
c. Hb F d. Hb S d. Paroxysmal nocturnal hemoglobinuria
94. A child aged 2 years presents with non-specific symptoms 105. Pure red cell aplasia is associated with all except:
suggestive of anemia. On peripheral blood smear, target  (AIIMS Nov 2013)
cells are seen. He has hypochromic microcytic picture
a. ABO incompatibility after renal transplant
and Hb of 6 g%. He also has a positive family history.
b. 5q- syndrome
Next investigation of choice is: (AIPGMEE 01)
c. Drugs
a. Hb electrophoresis b. Coomb's test d. Large granular lymphocytic leukemia
c. Liver function tests d. Osmatic fragility test 106. Constitutional pancytopenia can be seen in following
95. Sickle cell trait patient do not have manifestations as except: (Maharashtra 10)
that of sickle cell disease because: (AIIMS Nov. 01) a. Fanconi anemia
a. 50% HbS is required for occurrence of sickling b. Diamond-Blackfan syndrome
b. HbA prevents sickling c. Dyskeratosis congenita
c. 50% sickles d. Schwachman Diamond syndrome
d. HbA prevents polymerisation of HbS 107. Kostmann's syndrome—treatment is: (AIIMS Nov 2009)
96. In beta thalassemia, there is: M  (AIIMS May 01) a. Antithymocyte globulin + cyclosporin
a. Increase in beta chain, decrease in alpha chain b. Antithymocyte globulin + cyclosporine + GM-CSF
b. Decrease in beta chain, increase in alpha chain c. G-CSF
c. Decrease in beta chain, decrease in alpha chain d. GM-CSF
d. Increase in beta chain, increase in alpha chain
97. Thalassemia occurs due to which mutations?  NON-NEOPLASTIC DISORDERS OF WBC
 (PGI Dec 2000)
a. Missense b. Splicing 108. A 20 months old female child is brought for routine
c. Transition d. Frame-shift check-up. Complete blood count (CBC) shows moderate
e. Truncation neutropenia. Child looks healthy & eats well. Family
history is unremarkable. CBC after 1 and 2 weeks shows
 APLASTIC ANEMIA same results. Bone marrow examination is normal. Next
step: (JIPMER May 2016)
98. Which of the following drugs is not associated with pure
a. Corticosteroid administration
red cell aplasia? (Recent Question 2015)
b. Multivitamin administration
a. Phenytoin b. Isoniazid c. Watch and wait strategy
c. Erythropoietin d. Chloramphenicol d. Antibiotic to prevent infection
99. Pancytopenia with hypocellular bone marrow seen in: 109. Serious infections can occur when absolute neutrophil
 (Recent Question 2015) count decreases to? (Recent Question 2016)
a. Fanconi's anemia
a. Less than 500/uL b. Less than 800/uL
b. Paroxysmal nocturnal hemoglobinuria
c. Less than 1000/uL d. Less than 1500/uL
c. Hairy cell leukemia
110. Eosinophilia is found in infection with:
d. Myelopthisis
 (Recent Question 2016)
100. 2-year-old child presents with short stature and café-au
lait spots. Bone marrow aspiration yields a little material a. Cryptococcus b. HPV
and mostly containing fat. What is your diagnosis? c. Strongyloides d. Typhoid
 (Recent Question 2015) 111. Pan T cell marker is (Recent Question 2015)
a. Fanconi anemia b. Dyskeratosis congenita a. CD3 b. CD8
c. Tuberous sclerosis d. Osteogenesis imperfecta c. CD19 d. CD20
101. Dry tap is a feature of: (Recent Question 2015) 112. Dohle bodies are seen in: (Recent Question 2015)
a. Anemia of chronic disease a. Neutrophils b. Macrophages
b. Megaloblastic anemia c. Plasma cells d. Histiocytes
c. Aplastic anemia 113. All are true regarding Hypereosinophilic syndrome in
d. Sickle cell anemia children except: (JIPMER 2008)
102. Pancytopenia with cellular bone marrow is seen in all a. Eosinophilia of > 1500 cells/uL
except: (WBPG 2015) b. Persistent eosinophilia for 6 months
a. Megaloblastic Anemia b. MDS c. Absence of any specific organ involvement
c. PNH d. G6PD deficiency d. Corticosteroids are first line therapy
332 Section 3: Systemic Pediatrics

 LYMPHOID NEOPLASMS 126. Most common type of ALL in children:


Review of Pediatrics and Neonatology

 (Recent Question 2015)


114. A 6-year-old boy with lymphoreticular malignancy a. Pre-B cell ALL b. Mature B cell ALL
has come for a cycle of chemotherapy. Which of the c. Pre-T cell ALL d. Mature T cell ALL
following investigations need to be done for next 4 hours 127. A 10-year-child presented with bilateral cervical
to diagnose tumour lysis syndrome? (JIPMER 2017) lymphadenopathy. Lymph node biopsy was performed,
a. Urea, creatinine, chloride, Ca2+ and K+ which showed cells as given in the figure. Which of the
b. Urea, creatinine, phosphate, Ca2+ and K+ following is true regarding this condition?
c. Urea, creatinine, magnesium, Ca2+ and K+  (AIIMS Nov 2015)
d. Urea, creatinine, sodium and phosphate a. Hodgkin lymphoma; EBV and embryo cell
115. MC malignancy in children is: (FMGE June 2018) b. Non Hodgkin lymphoma; HIV and Giant B cell
a. Retinoblastoma b. Leukemia c. TB, Mycobacteria and tiny granuloma
c. Wilms tumor d. Neuroblastoma d. Hodgkin lymphoma: EBV and Reed Sternberg cell
128. A 10-year-old boy presents with mass in the abdomen.
116. Good prognosis factors of ALL are all except:
On imaging, the paraaortic LN is enlarged. On biopsy,
 (JIPMER Nov 2016) starry sky appearance is seen. What is the underlying
a. Hyperdiploidy b. WBC count < 5000/µL abnormality? (AIIMS May 2015)
c. Age > 10 years d. t(12;21) translocation a. p53 gene mutation
117. Reed Sternberg cells are found in: M  b. RB gene mutation
 (WBPG 2016; Recent Question 2014) c. Translocation involving BCR-ABL genes
a. Hodgkin's disease b. Sickle cell anemia d. Translocation involving MYC gene
c. Thalassemia d. CML 129. In a 6-year-child with ALL relapse, most common site:
118. Which of the following statements is true regarding  (NEET Pattern 2012)
juvenile chronic myeloid leukemia? (UPSC CMS 2015) a. Bone marrow b. Liver
a. Philadelphia chromosome is negative c. Testes d. Lung
b. Thrombocytopaenia is uncommon 130. Common childhood tumors are:
c. The prognosis is better than the adult form of chronic  (PGI Dec 08, AIPGMEE 07, 99)
myeloid leukemia a. Acute lymphoblastic leukemia
d. Single agent chemotherapy with busulfan or hydroxyurea b. Chronic lymphoblastic leukemia
can achieve remission c. Chronic myeloid leukemia
119. Which one is the best prognostic factor for ALL? d. Brain tumor e. Wilms' tumor
 (AIIMS Nov 2015) 131. All of the following are good prognostic factors for
a. Hyperploidy b. Orgnomegaly childhood ALL except:  (AIPGMEE 07)
c. TLC more than 50,000/uL d. Response to treatment a. Hyperdiploidy b. Female sex
120. True about ALL: (Recent Question 2015) c. Pre B cell ALL d. t(12:21) translocation
a. tdT positive b. Gamma globulins 132. ALL L3 morphology is a malignancy arising from which
c. t(8,14) d. Insidious onset cell lineage? (AIPGMEE 07)
121. True about Hodgkin's lymphoma is/are: (PGI May 2015) a. Mature B-cell b. Precursor B-cell
a. Often localized to single axial group of lymph nodes c. Immature T-cell d. Mixed B-cell and T-cell
b. Hepatomegaly is always present 133. All of the following are associated with good prognosis
c. Contiguous spread of lymph nodes in childhood leukemia except: (AIPGMEE 07)
d. Can be cured by Chemotherapy and Radiotherapy a. Hyperdiploidy b. Female sex
e. Commonly presents with painless lymphadenopathy c. Precursor B-cell ALL d. t(12; 21) translocation
122. Bimodal distribution is seen in: (Recent Question 2015) 134. A 5-year-old child presents with history of fever off- and-
a. ALL b. DLBCL on for past 2 weeks and petechial spots all over the body
c. Hodgkin's Lymphoma d. CML and increasing pallor for past 1 month. Examination
123. In infant, Bone Marrow biopsy is done from? M  reveals splenomegaly of 2 cm below costal margin. The
a. Tibia (Recent Question 2015) most likely diagnosis is: (AIPGMEE 04)
b. Sternum a. Acute leukemia
c. Posterior superior Iliac spine b. Idiopathic thrombocytopenic purpura
d. Iliac crest c. Hypersplenism d. Aplastic anemia
124. The following parameter in ALL indicates poor prognosis: 135. Treatment of ALL includes use of: (PGI Dec 02)
a. Age >10 years (Recent Question 2015) a. Hydroxyurea b. All transretinoic acid
b. WBC count <50000/mm3 at diagnosis c. Prednisolone d. L-asparginase
c. Hyperdiploidy e. Vincristine
d. Early pre-B phenotype 136. Poor prognostic indicator of ALL is: (AIIMS May 02)
125. Drug that is not used in the treatment of ALL: a. Female sex
 (Recent Question 2015) b. Leukocyte count < 50,000/µL
a. Rituximab b. Methotrexate c. Age greater than 1 year
c. Vincristine d. Daunorubicin d. Hypoploidy
Chapter 14: Pediatric Hematology 333

 MYELOID NEOPLASMS 148. A 15-year-old boy presented with 1 day history of blee­

Questions
ding gums, subconjunctival bleed and purpuric rash.
137. In AML best prognosis is seen with: Inves­ti­­gations revealed: Hb 6.4 gm/dL; TLC-26,500/mm3;
 (Recent Question 2016) prothrombin time-20 sec with a control of 13 sec; partial
a. Acute megakaryocytic leukemia thromboplastin time-50 sec; and fibrinogen 10 mg/dL.
b. Acute monocytic leukemia Peripheral smear was suggestive of acute myeloblastic
c. Acute promyelocytic lukemia (M3) leukemia. Which of the following is the most likely?
e. Erythroleukemia  (AIIMS May 06)
138. Most common translocation in acute promyelocytic a. Myeloblastic leukemia without maturation
leukemia: (PGI Nov 2017) b. Myeloblastic leukemia with maturation
a. t(8:14) b. t(15:17) c. Promyelocytic leukemia
c. t(9:22) d. t(8:11) d. Myelomonocytic leukemia
e. t(11:14) 149. Genetic risk factors for leukemia are all except: M 
 (Kerala 04)
139. Auer rods are specific for: (Recent Question 2015)
a. Down's syndrome b. Bloom syndrome
a. Acute myeloid leukemia
c. Ataxia telangiectasia d. Turner's syndrome
b. Acute lymphocytic leukemia
c. Chronic lymphocytic leukemia 150. Transient myeloproliferative disorder of the newborn is
d. Hodgkin's lymphoma seen in association with:  (AIPGMEE 03)
140. Most common type of AML in Down's syndrome: a. Turner syndrome b. Down syndrome
 (Recent Question 2015) c. Neurofibromatosis d. Ataxia telangiectasia
a. M2 b. M3 151. A 2-year-old boy is suffering from leukemia; following
c. M6 d. M7 X-ray findings are seen:  (PGI June 03)
a. Osteolytic lesion in flat bones
141. Juvenile CML is characterized by all except:
b. Metaphyseal osteoporosis
a. Increased HbF  (NEET Pattern 2012) c. Periosteal new bone formation
b. Philadelphia chromosome d. Osteosclerosis of long bones
c. Immature granulocyte e. Transverse line of dark band below the growth plate
d. Nucleated RBC 152. X-ray features of leukemia in a 2-year-old child is/
142. Increased fetal hemoglobin is seen in: (DPG 10) are:
a. Juvenile CML a. Osteolytic lesions in flat bones  (PGI Dec 2000)
b. Congenital red cell aplasia b. Subperiosteal erosions
c. Hereditary spherocytosis c. Osteoporosis
d. AML d. Thick luscent line just below growth plate
143. A child with acute myeloid leukemia presents with hyper­ e. Metaphyseal lucent bands
leukocytosis. Treatment includes all of the following 153. Non-specific esterase is present in: (PGI Dec 97)
except: (AIPGMEE 09)
a. Megakaryocytic leukemia
a. IV fluid b. Allopurinol b. Lymphocytic leukemia
c. Alkalinization c. Erythroleukemia d. AML M4
d. Immediately start induction chemotherapy e. AML M5
144. Non-specific esterase is positive in all the categories of 154. All of the following are features of juvenile CML except:
AML except: (AIPGMEE 07)  (AIPGMEE 94)
a. M3 b. M4 a. Thrombocytopenia
c. M5 d. M6 b. Fetal Hb is increased
145. AML with worst prognosis:  (AIIMS May 07) c. Philadelphia chromosome is positive
a. 8/21 translocation b. Inversion 16 d. Lymphadenopathy
c. Normal cytogenetics d. Monosomy 7
146. Arsenic is used in the management of: (AIIMS May 07)  LANGERHANS CELL HISTIOCYTOSIS
a. Acute promyelocytic leukemia 155. In Langerhans Cell Histiocytosis, the characteristic
b. ALL abnormality seen is: (AIIMS May 2015)
c. CML
a. Birbecks granules b. Macrophages
d. Transient myeloproliferative disorder
c. Plasma cell d. Giant cell
147. A 17-year-old boy presented with TLC of 138 x 109/L
with 80% blasts on the peripheral smear. Chest X-ray 156. Langerhans cell shows which of these?
demonstrated a large mediastinal mass. Immunopheno­  (Recent Question 2015)
typing of this patient's blasts would most likely demons­ a. Badminton racquet appearance
trate: (AIIMS May 2006) b. CD100a
a. No surface antigen (Null phenotype) c. MPO + d. Birbeck's granules
b. An immature T-cell phenotype (Tdt/CD34/CD3) 157. CD marker of histiocytosisis: M  (Recent Question 2014)
c. Myeloid markers, such as CD13, CD33 and CD15 a. CD 1a b. CD 1b
d. B-cell markers such as CD19, CD20 and CD22 c. CD 1c d. CD 1d
334 Section 3: Systemic Pediatrics

158. Localised Langerhans cells histiocytosis affecting head 167. True about Kasabach-Merritt syndrome, are all except:
Review of Pediatrics and Neonatology

and neck is (Recent Question 2014)  (PGI Nov. 09)


a. Letterer-Siwe disease a. Platelet sequestration b. Infantile hemangioma
b. Pulmonary Langerhans cell histiocytosis c. Consumption coagulopathy
c. Hand-Schuller-Christian disease d. Portwine hemangioma
d. Eosinophilic granuloma e. Thrombocytosis
159. A 2-year-old child presents with scattered lytic lesions 168. A 5 years old boy comes with appearance of petechial
in the skull. Biopsy revealed Langerhans giant cells. The spots overnight. 2 weeks back he had history of abdo­
most commonly associated marker with this condition minal pain and no hepatosplenomegaly. Diagnosis is:
will be: (AIPGMEE 12, 10, AIIMS Nov 07)  (AIIMS May 2007)
a. CD la b. CD57 a. Acute lymphatic leukemia
c. CD3 d. CD68 b. Aplastic anemia
160. About Langerhans cell histiocytosis, true are all except: c. Idiopathic thrombocytopenic purpura
 (AIIMS May 09) d. Acute viral infection
169. True about acute ITP: (PGI June 05)
a. Peak incidence less than 3 years of age
b. Radiosensitive a. More common in females
c. Diffuse form is known as Letterer-Siwe-disease b. Specific anti platelet antibodies detected
d. Gonadal involvement occurs c. Viral infection predisposes, as seen after vaccination
d. 80% cases transform to chronic
161. Which of the following drugs is used for the treatment of
e. More common in children
refractory histiocytosis?  (AIIMS Nov 08)
170. Quinine induced thrombocytopenia is:(JIPMER 04)
a. High dose methotrexate
a. Antibody mediated
b. High dose cytarabine
b. Dose related toxicity
c. Cladribine d. Fludrabine
c. Idiosyncratic reaction
162. A 2-year-old child comes with ear discharge, seborrheic
d. Inhibits production of platelets
dermatitis, polyuria and hepatosplenomegaly. Which of
171. Which of the following is generally not seen in idiopathic
the following is the most likely diagnosis?
thrombocytopenic purpura (ITP)? (AIIMS Nov 04)
 (AIIMS May 04, AIPGMEE 01, 94)
a. More common in females
a. Leukemia b. Lymphoma
b. Petechiae, ecchymosis and bleeding
c. Langerhan's cell histiocytosis
c. Palpable splenomegaly
d. Germ cell tumor
d. Increased megakaryocytes in bone marrow
163. A one-year-old boy presented with hepatosplenomegaly
172. Platelet function defect is seen in: (PGI June 03)
and delayed milestones. The liver biopsy and bone
marrow biopsy revealed presence of histiocytes with PAS a. Glanzmann syndrome b. Bernard-Soulier syndrome
posi­tive cytoplasm. Electron microscopic examination of c. Wiskott-Aldrich syndrome
these histiocytes is most likely to reveal the presence of: d. von Willebrand disease
e. Weber Christian disease
a. Birbeck granules in the cytoplasm (AIIMS 03)
173. Platelet function is assessed by: (PGI Dec. 02)
b. Myelin figures in the cytoplasm
c. Parallel rays of tubular structures in lysosomes a. Platelet adhesion b. BT
c. CT d. PTT
d. Electron dense deposit in the mitochondria
e. PFA-100
164. True about Langerhan's histocytosis ‘X': (PGI June 01)
174. The following laboratory determinant is abnormally
a. Can be associated with diabetes insipidus prolonged in ITP: M
b. X-ray shows pathognomonic osteosclerotic lesions
a. APTT b. Prothrombin time
c. Birbeck's granules in Langerhan's cell
c. Bleeding time d. Clotting time
d. Proliferation of antigen presenting cells
e. Associated with HLA DR  COAGULATION DISORDERS
165. A child presented in the OPD with multiple permeating
lesions involving all the bones of the body which of the 175. True about hematological disorder: (PGI Nov 2017)
following is the most probable diagnosis?
a. In Hemophilia B cryoprecipitate is useful
a. Neuroblastoma  (AIIMS Nov 2000)
b. Both PT and aPTT are increased in DIC
b. Metastasis from osteosarcoma
c. IVIg is useful in ITP
c. Histiocytosis X
d. Metastasis from Wilms' tumor d. Desmopressin has no role in vWD
e. Hemorrhagic disease of newborn is due to vit C deficiency

 BLEEDING DISORDERS 176. Low molecular weight heparins are used very frequently
to prevent propagation of thrombus in children with
166. In Alloimmune thrombocytopenia antigen against which stroke. What is the laboratory test used in blood to
antibody is found? (JIPMER Nov 2016) monitor the LMW Heparins? (Recent Question 2017)
a. HPA 1a b. HPA 1b a. Anti factor X a b. Factor Xa
c. HPA 2a d. HPA 2b c. Thrombin d. Antithrombin
Chapter 14: Pediatric Hematology 335

177. Leiden mutation involves: M  (Recent Question 2016) 191. Which plasma protein is necessary for adhesion of

Questions
a. Factor V b. Factor VI platelets to subendothelial fibres? (APPG 2014)
c. Protein C d. Protein S a. Glycoprotein IIb b. Von Willebrand factor
178. Which of the following has autosomal dominant c. Platelet factor 3 d. Factor X
inheritance? M  (Recent Question 2016) 192. Select the TRUE statement among the following:
a. Hereditary spherocytosis b. Thalassemia  (APPG 2014)
c. Sickle cell disease d. G6PD deficiency a. Anti-D is used in Immune Thrombocytopenic purpura
179. In Glanzman's thrombasthenia, there is defect of: b. DDAVP is used in von Willebrand disease type 3
 (Recent Question 2016) c. DDAVP is used in severe form of Hemophilia A
a. Gp IIb/IIa b. Gp IIb/IIIa d. EACA is used in Factor XI deficiency for minor bleeds
c. Gp Ib/IX d. Gp IIb/IX 193. Spontaneous bleeding occurs when platelet count falls
180. Affinity of fetal Hemoglobin to oxygen is more than below: (MAHA PGM CET 2014)
adult hemoglobin due to: (Recent Question 2016) a. 50,000/µL b. 40,000/µL
a. 2, 3 BPG in blood b. Decreased pH c. 30,000/µL d. 20,000/µL
c. Increased pCO2 d. Right shift 194. Bernard-Soulier syndrome is a defect in platelet:
181. Cryoprecipitate contains all except: M   (MAHA PGM CET 2014)
 (MAHA PGM CET 2016) a. Aggregation b. Adhesion
a. Factor VIII b. Factor IX c. Release reaction d. Morphology
c. Fibrinogen d. Von- Willebrand factor 195. Chloroma is due to: M  (MAHA PGM CET 2014)
182. Immune destruction of platelet is seen in: a. AML b. CLL
 (WB PGMEE 2016)
c. ALL d. Non Hodgkin's lymphoma
a. SLE b. DIC 196. Microangiopathic hemolytic anemia with thrombocy-
c. HUS d. TTP topenia and renal insufficiency is commonly seen in
183. Characteristic cell in Hodgkin lymphoma is: young children with: (MAHA PGM CET 2014)
 (WB PGMEE 2016)
a. Sickle cell anemia
a. Reed Sternberg b. Gaucher cell b. Hemolytic uremic syndrome
c. Erythrophagocytes d. Langhans cell c. Malaria
184. Best test for assessment of Iron store is? M  d. SLE (systemic lupus erythematosus)
 (MAHA PGM CET 2015)
197. Thalassemia is common within: M  (WBPG 2014)
a. Hemoglobin b. Serum Iron
c. Ferritin d. Transferrin a. Tribal b. Marathi
c. Sindhis d. Konkan
185. The gold standard Lab test for diagnosis of PNH
(Paroxysmal Nocturnal Hemoglobinuria) is: 198. A child with prolonged aPTT underwent laparotomy
 (MAHA PGM CET 2015) and still did not develop any bleeding complications.
Which of the following clotting factors is most likely to be
a. Ham test b. Haptoglobin
deficient? (JIPMER 2012)
c. Flow cytometry d. Sucrose Lysis test
186. Children with Thalassemia and Iron overload are at an a. Factor VII b. Factor XII
increased risk for infection with: c. Factor X d. Factor XIII
 (MAHA PGM CET 2015) 199. Laboratory diagnosis of thalassemia major are all
a. Yersinia Enterocolitica b. Campylobacter jejuni except: (WBPG 2012)
c. Escherichia coli d. Vibrio cholera a. Normal ferritin level
187. vWF protects: b. Normal TIBC
c. Microcytic hypochromic anemia
a. Factor V b. Factor VI
c. Factor VII d. Factor VIII d. High transferrin saturation percentage
188. Deficiency of factor VIII leads to: M  200. A child presented with fever, anemia, lymphadenopathy
 (Recent Question 2015) and hepatosplenomegaly. All of the following may be a
possible cause except: M  (WBPG 2010)
a. Hemophilia b. Christmas disease
c. Von Willebrand disease d. Bernard Soulier syndrome a. ALL b. TB
189. Which of the following is a qualitative defect of platelets? c. Leishmaniasis d. Aplastic anemia
 (Recent Question 2015) 201. A newborn baby presented with profuse bleeding from
a. Immune thrombocytopenic purpura umbilical stump after birth. Probable diagnosis is:
b. Bernard Soulier syndrome  (AIIMS May 07, Nov 06)
c. Systemic Lupus erythematosus a. Factor XIII deficiency
d. Von Willebrand disease b. VWF deficiency
190. Factor Xa is necessary for conversion of prothrombin to c. Factor XII deficiency
thrombin in which pathway? (APPG 2014) d. Glanzmann thrombasthenia
a. Only in the extrinsic pathway 202. In a case of acute hemarthrosis in hemophilia, treatment
b. Only in the intrinsic pathway with factor VIII along with fresh frozen plasma is given for:
c. As part of both extrinsic and intrinsic pathways a. 3 days b. 7 days (WBPG 2007)
d. Only if the normal blood clotting cascade is inhibited c. 1 day d. Till joint effusion subsides
336 Section 3: Systemic Pediatrics

203. Late onset hemorrhagic disease of newborn is characte­ 206. The coagulation profile in a 13 year old girl with
Review of Pediatrics and Neonatology

rized by all of the following features except: Menorrhagia having von Willebrands disease is:
a. Usually occurs in cow milk fed babies (AIPGMEE 06) a. Isolated prolonged PTT with a normal PT (AIPGMEE 05)
b. Onset occurs at 4–12 weeks of age b. Isolated prolonged PT with a normal PTT
c. Intracranial hemorrhage can occur c. Prolongation of both PT and PTT
d. Intramuscular vitamin K prophylaxis at birth has a d. Prolongation of thrombin time
protective role 207. A child underwent a tonsillectomy at 6 years of age
204. A 15-year-old female presented to the emergency with no complications. He underwent a preoperative
department with history of recurrent epistaxis, hematuria screening for bleeding at the age of 12 years before an
and hematochezia. There was a history of profuse bleeding elective laparotomy, and was found to have a prolonged
from the umbilicus stump at birth. Previous investigations partial thromboplastin time but normal prothrombin
revealed normal prothrombin time, activated partial time. There was no family history of bleeding. The
thromboplastin time, thrombin time and fibrinogen levels. patient is likely to have: (AIIMS Nov 04)
Her platelet counts as well as platelet function tests were a. Acquired Vit K deficiency b. Acquired liver disease
normal but urea clot lysis test was positive. Which one of c. Factor XII deficiency d. Mild hemophilia A
the following clotting factor is most likely to be deficient? 208. Fanconi's anemia is a:  (JIPMER 03)
a. Factor X b. Factor XI (AIIMS May 06) a. Constitutional anemia b. Hemolytic anemia
c. Factor XII d. Factor XIII c. Iron deficiency anemia d. Auto-immune anemia
205. Characteristic lab findings of hemophilia A are: 209. The commonest mode of inheritance of von Willebrand's
 (PGI June 06) disease: M  (AIIMS Nov 2000)
a. Increased PT b. Increased aPTT a. Codominant b. Autosomal recessive
c. X-linked recessive d. Increased bleeding time c. Autosomal dominant d. X-linked recessive
e. Normal aPTT
Chapter 14: Pediatric Hematology 337

Answers with Explanations


Answers with Explanations
 NORMAL HEMATOPOIESIS

1. b. 40-60 days  Ref: Nelson 20/e p2319


Premature infant's RBCs have a short life span of only 40-60 days
2. d. Gower hemoglobin and Portland hemoglobin  Ref: Nelson's 20/e p 2307
The embryonic hemoglobins are Gower-1 (ζ2ε2), Gower-2 (a2e2) and Hb Portland (ζ2γ2)
3. a. Liver  Ref: Nelson's 20/e p 2306
Erythropoietin (EPO) is produced in fetal liver during 1st & 2nd trimesters, by cells of monocyte/macrophage origin.
After birth, the anatomic site of EPO production shifts to the kidney.
4. a. Proerythroblast  Ref: Nelson's 20/e p 2304-2308, Ghai 8/e p 330
Stage of RBC development are: Proerythroblast → Basophillic → polychromatic → Orthochromatic erythroblast → Nucleated
RBC → Reticulocyte → Mature RBC.
5. c. Megaloblastic anemia  Ref: Nelson's 20/e p 2319-2321
Megaloblastic anemia has a low normal Reticulocyte count, as it is a nutritional anemia.
6. b. Donor cells have normal survival  Ref: Nelson's 20/e p 2327-2334
This is purely a common sense question; in intracorpuscular defect, the defect lies in the patient's RBCs itself and not in spleen or
vessels and so if any NORMAL erythrocytes (blood – group matched) are transfused into a patient will not be destroyed. Hence the
answer is donor cells have normal survival.
7. d. 101-125  Ref: Nelson's 20/e p 2304-2308, Ghai 8/e p 330
Normal Red Cell Indices in Infancy and Childhood

Age MCV (fL) MCH (pg) MCHC (g/dL) (mean)


Birth (cord blood) 108 34 33

1 month 104 34 33

1–2 years 78 27 33

2–6 years 81 27 34

6–12 years 77 29 34

8. b. Liver  Ref: Nelson's 20/e p 2304-2308, Ghai 8/e p 330

Timing Site of hematopoiesis


2–6 weeks Yolk sac

6–24 weeks Liver

After 24 weeks Bone marrow

9. c. Portland Hb  Ref: Nelson's 20/e p 2304-2308, Ghai 8/e p 330

Hemoglobin type Chains Hemoglobin type Chains


Hb A (adult Hb) α2β2 Hb Portland ζ2γ2

Hb A2 α2δ2 Gower-1 Hb ζ2ε2

Hb F (Fetal Hb) α2γ2 Gower-2 Hb α2ε2

10. c. 36 weeks gestation  Ref: Nelson's 20/e p 2304-2308, Ghai 8/e p 330
•• During the initial fetal life, the major hemoglobin synthesized in the body is fetal hemoglobin (α2γ2), because in fetus, the
chain synthesized predominantly is the γ-chain; β-chain is present in only trace amount in early embryos
•• At 36 weeks of gestation, rate of synthesis of β-chain exceeds that of γ-chain. This is called switch over.
11. b. 2α+2b  Ref: Nelson's 20/e p 2304-2308, Ghai 8/e p 330
338 Section 3: Systemic Pediatrics

 NUTRITIONAL ANEMIA
Review of Pediatrics and Neonatology

12. c. Plasma transferrin saturation decreases and e. Blood transfusion is indicated at Hb < 4.0g/dL  Ref: Nelson's 20/e p 2324
In Iron deficiency Anemia, peripheral blood smear reveals hypochromic, microcytic red cells with anisocytosis. Elliptocytic or
cigar-shaped red cells are often seen

Parameter MCV and RBC RDW Reticulocyte Hb Serum Ferritin TIBC Transferrin Transferrin FEP
count conc saturation Receptor
In Iron def. Decreased Increased Decreased Decreased Increased Decreased Increased Increased
Anemia

13. a. Hepcidin  Ref: Nelson's 20/e p 2323-2325, Ghai 8/e p 334-335


Hepcidin: principal iron regulatory hormone that negatively regulates ferroportin on basolateral surface of enterocyte.
14. a. S. ferritin  Ref: Nelson's 20/e p 2323-2325, Ghai 8/e p 334-335
Serum ferritin reflects the storage of Iron which is decreased even in the pre-latent stage of Iron deficiency Anemia and is the
most sensitive marker.
15. a. Hepcidin  Ref: Nelson's 20/e p 2323-2325, Ghai 8/e p 334-335
Ferroportin which regulates the Iron release from tissue store house is regulated by hepcidin hormone released by liver.
16. b. Ferritin  Ref: Nelson's 20/e p 2323-2325, Ghai 8/e p 334-335
Serum ferritin level indicates the amount of iron present in both liver and reticuloendothelial system and hence is the best assessment
of iron status.
17. d. Diagnosed by serum ferritin  Ref: Wintrobe's Hematology 13/e p 860, Nelson's 20/e p 2323-2325;

Hereditary hemochromatosis (Autosomal recessive) is caused due to mutations of HFE gene on chr 6p21.3 and diagnosed on the basis of
tissue ferritin; Staining for Iron is done on buccal scraping smear.

18. d. Fanconi's anemia  Ref: Nelson's 20/e p 2323-2325, Ghai 8/e p 334-335
Fanconi's anemia presents as pancytopenia and normocytic normochromic anemia.
19. b. DMT-1  Ref: Nelson's 20/e p 2323-2325, Ghai 8/e p 334-335
DMT1 has been shown to transport a variety of divalent metal ions, including Mn2+, Co2+, Cu2+, Zn2+, Cd2+, and Pb2+.
(Hence non-specific).
20. d. Ascorbic acid  Ref: Nelson's 20/e p 2323-2325, Ghai 8/e p 334-335

Iron Absorption
•• Site of iron absorption: duodenum and upper jejunum; Ferrous form (Fe2+) of Iron is absorbedQ
•• Transport of Fe2+ into enterocyte occurs via DMT1Q
Increased by Decreased by
•• Acids e.g. ascorbic acid (Vitamin C)Q •• Alkalies •• Tetracycline
•• Amino acid containing SH-group •• Phosphates and PhytatesQ •• Presence of other food in stomach

21. b. Leukopenia  Ref: Nelson's 20/e p 2323-2325, Ghai 8/e p 334-335


Differential diagnosis of microcytic hypochromic RBCs include
•• Iron Deficiency Anemia • Thalassemia
• Anemia of Inflammation/chronic disease • Sideroblastic Anemia
22. a. Increased serum ferritin  Ref: Nelson's 20/e p 2323-2325; Refer pretext of this chapter for details;
23. b. IL-6  Ref: Wintrobes 13/1224 Nelson's 20/e p 2323-2325, Ghai 8/e p 334-335
Cytokines most often implicated in pathogenesis of anemia of chronic disease are TNF, IL-1, IL-6 and interferons. However, IL-6 is
also a potent inducer of hepcidin, proposed as a major mediator of iron abnormalities of anemia of chronic ds.
24. b. Hb 11g/dL  Ref: Nelson's 20/e p 2323-2325, Ghai 8/e p 334-335; Refer pretext for details;
25. a. Decreased irritability  Ref: Robbins 9th/pg 649-652; Wintrobe's 12th/pg 829; Refer pretext for details;
Rapid subjective improvement, with decrease in fatigue & irritability occurs before improvement in anemia
26. c. Serum ferritin depletes first and d. Decreased alertness  Ref: Nelson's 20/e p 2323-2325, Ghai 8/e p 334-335
Discussing about the options one by one,
a. Iron is absorbed from proximal small intestine mainly from duodenum
b. Cow milk has more iron than breast milk. But, bioavailability of iron of breast milk is much more than cow milk.
c. S. ferritin depletes first: Next, serum iron level decreases, iron binding capacity of serum (serum transferrin)increases, and
percent saturation (transferrin saturation) is reduced
Chapter 14: Pediatric Hematology 339

d. Iron deficiency affects attention span, alertness and learning in both infant and adolescents.

Answers with Explanations


e. RDW is increased in iron deficiency anemia.
27. a. Increased TIBC, decreased serum ferritin  Ref: Nelson's 20/e p 2323-2325, Ghai 8/e p 334-335
28. a. Inadequate dietary intake  Ref: Nelson's 20/e p 2323-2325, Ghai 8/e p 334-335
•• Folic acid deficiency is primarily due to insufficient intake and increased demand
•• Vitamin B12 deficiency is primarily due to malabsorption.
29. a. Increased reticulocyte count  Ref: Nelson's 20/e p 2323-2325, Ghai 8/e p 334-335; Refer Ans. 21 above
30. a. Microcytic hypochromic anemia, d. Bone marrow iron decreased earlier than serum iron, e. Decreased transferrin
saturation  Ref: Nelson's 20/e p 2323-2325, Ghai 8/e p 334-335; Refer pretext for details;
31. d. Decrease in absorption occurs with improvement of symptoms  Ref: Nelson's 20/e p 2323-2325
Discussing about the options one by one,
a. Only 10% of the iron given by oral route gets absorbed
b. Reticulocytosis appears in 48-72 hours, peaking at 5–7 days
c. Clinical response to treatment appears within 24 hours
d. The normalisation of body iron store relatively decreases the iron absorption
32. a. Hb electrophoresis  Ref: Nelson's 20/e p 2323-2325, Ghai 8/e p 334-335
•• Progressively increasing pallor and hepatosplenomegaly suggest the diagnosis of thalassemia
•• Hb electrophoresis is the diagnostic test of choice for thalassemia.
33. c. HbD disease  Ref: Nelson's 20/e p 2322-2326

a. Iron deficiency anemia So severe anemia and expansion of erythroid marrow are usually not seen

b. Acute lymphoid Leukemia Low MCV, no obvious involvement of other cell lines, and expansion of erythroid marrow are
incompatible with diagnosis of ALL

c. Hemoglobin D disease Low MCV and MCH with target cells suggest abnormality of hemoglobin like HbD disease

d. Hereditary spherocytosis Normal osmotic fragility goes against hereditary spherocytosis

34. b. Serum ferritin levels  Ref: Nelson's 20/e p 2323-2325, Ghai 8/e p 334-335
Low serum ferritin level invariably indicates iron deficiency and hence it is the most specific and sensitive test for iron deficiency
anemia.
35. a. Hb electrophoresis  Ref: Nelson's 20/e p 2323-2325, Ghai 8/e p 334-335
•• Presence of: 1. Anemia (Hb < 6%), 2. Target cells, 3. Increased reticulocytes suggest the diagnosis of thalassemia
•• Out of the investigations given in the options, Hb electrophoresis will help the most in diagnosing Thalassemia.
36. a. Hypersegmented neutrophils  Ref: Nelson's 20/e p 2323-2325, Ghai 8/e p 334-335
•• Hypersegmented neutrophils are seen in megaloblastic anemia.
37. b. Hb electrophoresis  Ref: Nelson's 20/e p 2323-2325, Ghai 8/e p 334-335
•• Presence of pallor and hepatosplenomegaly at 6 months of age with a positive family history indicates towards thalassemia major,
which can be diagnosed by Hb electrophoresis.
38. c. 4 × wt (kg) × Hb deficit (gM/dL)  Ref: Nelson's 20/e p 2323-2325, Ghai 8/e p 334-335
•• Blood volume averages 75 to 80 mL/kg. Each gram of hemoglobin contains 3.4 mg of iron. 50% more iron should be given to
replenish iron stores. Therefore iron requirements are determined from the equation.
Iron requirement (mg) = wt (kg) × Hb deficit (g/dL) × 80/100 × 3.4 × 1.5. = wt (kg) × Hb deficit (g/dL) × 4.

 HEMOLYTIC ANEMIA

39. b. Stop transfusion and look for clerical errors  Ref: https://www.rch.org.au
Transfusion reactions:

Category 1: Mild reactions


Signs Symptoms Possible cause Immediate management
Urticaria/ Pruritis (itching) Allergic •• Stop transfusion
rash •• Assess patient
•• An antihistamine may be required
•• Transfusion may be restarted if no other signs/symptoms are present
•• If signs/symptoms worsen treat as Category 2.
340 Section 3: Systemic Pediatrics
Review of Pediatrics and Neonatology

Category 2: Moderately severe reactions


Signs Symptoms Possible cause Immediate management
Flushing Anxiety Allergic (moderately-severe) •• Stop transfusion and maintain IV line with NSaline
Urticaria Pruritis Febrile non-hemolytic transfusion •• Contact Medical Officer
Rigors Palpitations reaction:-antibodies to white cells or •• Patient may require antihistamine and/or paracetamol
platelets •• Further investigation and management according to
Fever Mild dyspnea
•• Antibodies to proteins including IgA clinical features
Restlessness Headache •• If investigation required: complete Transfusion Reaction
•• Possible contamination with pyro­gens
Tachycardia Form and send blood pack, form and samples to blood
and /or bacteria
bank

Category 3: Life-threatening reactions


Signs Symptoms Possible cause Immediate management
Rigors Anxiety Acute intravascular hemolysis (wrong •• Stop transfusion and maintain IV line with N.Saline
Fever Chest pain blood) •• Contact Medical Officer
Restlessness Pain at infusion Bacterial contamination and septic shock •• Manage immediate needs:
site Fluid overload –– Fluid for hypotension
Hypotension
–– Oxygen
Tachycardia Respiratory Anaphylaxis
–– Adrenaline for anaphylaxis
Dark Urine distress Transfusion related acute lung injury –– Diuretic for fluid overload
Unexplained Loin/back pain (TRALI) •• Complete Transfusion Reaction Form and send blood
bleeding (DIC) Headache pack, form and samples to blood bank
Dyspnea •• Further management according to likely cause

40. d. Sickle cell anemia  Ref: Nelson 20/e p 2338


Dactylitis, referred to as hand-foot syndrome, is often the first manifestation of pain in infants and young children with sickle cell
anemia
Occurring in 50% of children with sickle cell disease by their 2nd yr of life
Dactylitis often manifests with symmetric or unilateral swelling of the hands and/or feet.
41. c. Hb electrophoresis  Ref: Nelson 20/d p 2351
Microcytic hypochromia anemia, in a child with hepatosplenomegaly & positive family history, suggest a diagnosis of Thalassemia.
Hb electrophoresis or Hb HPLC can be done for diagnosing Thalassemia.
NESTROFT test (Naked Eye Single Tube Red Cell Osmotic Fragility Test)
•• Normally, red cells put in saline solution begin to lyse at a saline concentration of 0.4-0.5% and lysis is complete at 0.32%.
•• However, in microcytic anemias (Thalassemia/Iron deficiency anemia), due to alteration in osmotic resistance of the affected
RBCs, lysis begins at a saline concentration between 0.4-0.35% and it may not be completed even at 0.1% solution.
•• NESTROFT is done at a saline concentration of 0.36%.
42. b. Increased direct bilirubin and e. Increased Hematocrit  Ref: Nelson's 20/e p 2327-2329
43. b. Thalassemia  Ref: Nelson's 20/e p 2327-2332; Refer pretext for details;
44. a. Hydroxyurea  Ref: Nelson's 20/e p 2236-2345, Ghai 8/e p 344-345
Hydroxyurea increases HbF levels and so is used for treatment of sickle cell anemia.
45. b. Sickle cell anemia  Ref: Nelson's 20/e p 2336, Ghai 8/e p 344
Point mutation (missense) in 6th codon of β-globin gene that leads to replacement of glutamate by valine residue.
46. c. Non conservative missense  Ref: Nelson's 20/e p 2236, Ghai 8/e p 334
47. b. Sickle cell anemia  Ref: Nelson's 20/e p 2336, Ghai 8/e p 345
Patients of sickle cell anemia may undergo auto-splenectomyQ due to multiple infarcts in splenic artery, that causes spleen to
be reduced to fibrous tissue (splenic atrophy → spleen may become non-palpable).
48. a. IVS1 G → C  Ref: Nelson's 20/e p 2349, Ghai 8/e p 341
Most common mutations in β thalassemia in India are IVS-1, position 5 (G → C) followed by 619-bp deletion
49. d. Aplastic anemia  Ref: Nelson's 20/e p 2363-2366
Fanconi anemia can lead to Pancytopenia with Hypocellular Bone Marrow.
50. b. Ankyrin  Ref: Nelson's 20/e p 2330, Ghai 8/e p 338
Most common mutation in Hereditary Spherocytosis are AnkyrinQ > band 3> Spectrin and band 4.2.
51. a. Spectrin  Ref: Nelson's 20/e p 2330, Ghai 8/e p 338
In Spectrin mutations, the most common RBC abnormality is Hereditary Elliptocytosis > spherocytosis.
52. c. Quinidine  Ref: Nelson's 20/e p 2335, Ghai 8/e p 339
Quinidine does not carry risk of hemolysis in persons with G-6-PD deficiency.
Chapter 14: Pediatric Hematology 341

53. a. Paraxysmal cold hemoglobinuria  Ref: Nelson's 20/e p 2335 340

Answers with Explanations


Direct globulin test is positive in Paroxysmal cold hemoglobinuria, as anti P antibody is formed.
54. b. Fetomaternal hemorrhage  Ref: Nelson's 20/e p 2335, Ghai 8/e p 340
A common approach to evaluating Fetomaternal Hemorrhage (FMH) is the rosette test to screen for the presence of fetal cells
followed by the Kleihauer-Betke acid elution method to quantify the magnitude of feto-maternal bleed.
55. d. Penicillin  Ref: Nelson's 20/e p 2335; Refer pretext of this chapter;
56. d. NESTROFT  Ref: Nelson's 20/e p 2349, Ghai 8/e p 841
Screening test for Thalassemia trait: NESTROFT (Naked Eye Single Tube Red cell Osmotic Fragility Test)Q
Assesses osmotic fragility of red cells at a single concentration of buffered saline (0.36% in single tube).
57. c. γ chain  Ref: Nelson's 20/e p 2349, Ghai 8/e p 341
58. c. Three α globin chains  Ref: Nelson's 20/e p 2349, Ghai 8/e p 341
59. d. Alkalosis  Ref: Nelson's 20/e p 2349; Acidosis rather than alkalosis precipitate sickling of HbS.
60. b. Microangiopathic hemolytic anemia  Ref: Nelson's 20/e p 2349, Ghai 8/e p 341
Schistocytes are fragmented RBCs, also called ‘helmet' cells, found in microangiopathic hemolytic anemia.
61. a. Older red cells  Ref: Nelson's 20/e p 2335, Ghai 8/e p 339
Because mature red cells do not synthesize new proteins, G6PD enzyme activity falls quickly to levels inadequate to protect
against oxidant stress as red cells age. Thus, older red cells are much more prone to hemolysis than younger ones.
62. d. Hemolysis from G6PD enzyme deficiency  Ref: Nelson's 20/e p 2335
63. b. Thalassemia major  Ref: Nelson's 20/e p 2324; S. ferritin is normal or increased in Thalassemia major
64. a. Plasmapheresis  Ref: Nelson's 20/e p 2508, Ghai 8/e p 492
In HUS, Fresh frozen plasma can induce remission by replenishing the missing Complement Factor H or I or removing the
antibodies.
65. d. Ringed sideroblast  Ref: Nelson's 20/e p 2336, Ghai 8/e p 345
Ringed sideroblast is seen in cases of Iron overload and not in sickle cell anemia.
66. a. Plasmodium falciparum  Ref: Nelson's 20/e p 2346, Ghai 8/e p 344; Refer pretext for details;
67. d. Neuro-psychiatric disturbances  Ref: Nelson's 20/e p 2507-2509
Neuro-psychiatric disturbances are not a feature of hemolytic uremic syndrome, but may be seen in TTP.
68. c. Malaria  Ref: Nelson's 20/e p 2349, Ghai 8/e p 341
Thalassemia gives a reduced parasite invasion/growth and an increased susceptibility to phagocytosis of the infected RBC
as a malaria-protective effect.
69. c. Myco tuberculosis  Ref: Nelson's 20/e p 2336, Ghai 8/e p 344
Regardless of age, all patients with sickle cell anemia are at increased risk of bacterial infection, particularly with encapsulated
organisms like Streptococcus pneumoniae, Haemophilus influenzae type b and Neisseria meningitidis.
70. a. Sickle cell anemia  Ref: Nelson's 20/e p1386 and 2339, Ghai 8/e p
•• Children with sickle cell disease are at increased risk for Salmonella septicemia and osteomyelitis.
71. c. Hb-A2  Ref: Nelson's 20/e p 2349, Ghai 8/e p 341
HbA2 accounts for 1.5% to 3.5% of the normal adult hemoglobin and it gets elevated in β thalassemia trait cases
72. d. Reduced β-chain  Ref: Nelsosn's 20/e p 2349, Ghai 8/e p 341
In beta-thalassemia there is reduced β chain and relatively excess α chain.
73. a. Sequestration is common in older children  Ref: Nelson's 20/e p2338, Ghai 8/e p 345
Acute splenic sequestration is a life-threatening complication occurring primarily in infants and young children with sickle cell
anemia. Sequestration can occur as early as 5 wk of age, but most often occurs in children between 6 mo and 2 yr.
74. a. Beta thalassemia trait  Ref: Nelson's 20/e p 2349; HbA2 is increased (>3.5%) in beta thalassemia trait.
75. b. Beta thalassemia  Ref: Nelson's 20/e p 2349, Ghai 8/e p 341
Peripheral smear findings of microcytic hypochromic anemia with target cell and reduced osmotic fragility with severe transfusion
requiring anemia is suggestive of Beta thalassemia.
76. b. Primaquine  Ref: Nelson's 20/e p 2335, Ghai 8/e p 339; Refer pretext of this chapter;
77. a. β-thalassemia  Ref: Nelson's 20/e p 2349, Ghai 8/e p 341
NESTROFT: Naked Eye Single Tube Red cell Osmotic Fragility TestQ
Assesses osmotic fragility of RBCs at a single concentration of buffered saline (0.36% in single tube).Q
78. a. Microspherocytes  Ref: Nelson's 20/e p 2340, Ghai 8/e p 340
ABO incompatibility causes immune hemolytic anemia which is characterized by microspherocytes on peripheral smear.
79. d. Increased osmotic fragility  Ref: Nelson's 20/e p 2330, Ghai 8/e p 338
342 Section 3: Systemic Pediatrics

80. c. Megaloblastic anemia  Ref: Nelson's 20/e p 2319 - 2321


Review of Pediatrics and Neonatology

Megaloblastic anemia causes a low reticulocyte count because it's a nutritional anemia, others being hemolytic anemia have a high
reticulocyte count.
81. a. Bite cells  Ref: Nelson's 20/e p 2335-2356, Ghai 8/e p 339
Bite cells are characteristic finding of G6PD deficiency while others can be seen in any intravascular hemolysis.
82. b. Bite cells seen and e. Sex preponderance  Ref: Nelson's 20/e p 2335, Ghai 8/e p 339
G6PD deficiency is an X-linked inherited disorder which is seen more frequently in boys;
83. d. Increased osmotic fragility  Ref: Nelson's 20/e p 2330, Ghai 8/e p 339
Cells that are more spherical, have a limited capacity to expand in hypotonic solutons lyse at a higher concentration of NaCl
than do normal biconcave cells → increased osmotic fragility, as in hereditary spherocytosis
Cells that are flatter have a greater capacity to expand in hypotonic solutions, lyse at lower concentration, are said to have decreased
osmotic fragility e.g. iron deficiency, thalassemia, liver disease
84. c. HbA2  Ref: Nelson's 20/e p 2349, Ghai 8/e p 341
Hb electrophoresis findings in Thalassemia:
Type of Hb β thalassemia major β-Thalassemia trait
HbA Decreased 90–96%
HbA2 5–7% 3.5–7%
HbF 10–100% 1–5%

85. d. Platelet count  Ref: Nelson's 20/e p 2349-2352, Ghai 8/e p 341
Mucosal bleeding can be a result of thrombocytopenia or any platelet abnormality. So platelet count needs to be done .
86. c. Hb electrophoresis  Ref: Nelson's 20/e p 2349-2352, Ghai 8/e p 341
87. c. Hb Bart's cannot release oxygen to fetal tissues  Ref: Nelson's 20/e p 2349-2352, Ghai 8/e p 341
As Hb Bart's consists of 4y chains, which have very high oxygen affinity, it cannot release oxygen to fetal tissues.
88. b. Increased HbA2, c. Microcytosis  Ref: Nelson's 20/e p 2349-2352, Ghai 8/e p 341
89. d. Patient present early in life before 6 months of life  Ref: Nelson's 20/e p 2349-2352, Ghai 8/e p 341
•• Patients rarely present before 6 months due to abundance of HbF which inhibits polymerisation of Hb S.
90. b. Replacement of glutamate by valine in β-chain of HbA  Ref: Nelson's 20/e p 2349-2352, Ghai 8/e p 341
91. a. α-thalassemia  Ref: Nelson's 20/e p 2349-2352, Ghai 8/e p 341
This case history suggests NON IMMUNE HYDROPS which is seen in severe variety of alpha thalassemia (HbH or Hb Barts). ABO
incompatibility usually leads to mild hemolysis, which is not severe enough to lead to hydrops fetalis.
92. e. Point mutation  Ref: Nelson's 20/e p 2349-2352, Ghai 8/e p 341
•• Sickle cell anemia is caused by point mutation of globin gene, which leads to change of glutamate to valine.
93. a. Hb A  Ref: Nelson's 20/e p 2349-2352, Ghai 8/e p 341
•• Hb electrophoresis shows 80–90% of HbS and the rest is HbF and HbA2, however HbA is absent.
94. a. Hb electrophoresis  Ref: Nelson's 20/e p 2349-2352, Ghai 8/e p 341
•• A low Hb of 6 g%, microcytic and hypochromic RBCs and target cell points towards thalassemia major
•• Thalassemia major is diagnosed by elevated HbF in Hb electrophoresis.
95. d. HbA prevents polymerisation of HbS  Ref: Nelson's 20/e p 2349-2352, Ghai 8/e p 341
•• Sickling depends on multiple factors like pH, ionic strength, PaO2 level and not only on HbS%
•• Increased HbF and HbA (Hb F > Hb A) concentration prevents polymerisation of HbS.
96. b. Decrease in beta chain, increase in alpha chain  Ref: Nelson's 20/e p 2349-2352, Ghai 8/e p 341
97. b. Splicing and d. Frame-shift  Ref: Nelson's 20/e p 2349-2352, Ghai 8/e p 341
Molecular defects in pathogenesis of thalassemia:
β-thalassemia:
•• Most common type of genetic abnormality in β-thalassemia is point mutation, i.e. nonsense
•• Some may also occur due to deletion or insertion i.e. frameshift mutations
Mutations leading to aberrant splicing are the most common cause of β-thalassemia
α-thalassemia
•• The most common cause of reduced α-chain synthesis is deletion of a-globin genes (rarely non sense mutation)

 APLASTIC ANEMIA
98. c. Erythropoietin  Ref: Nelson's 20/e p 2362-2371, Ghai 8/e p 345-347
Chapter 14: Pediatric Hematology 343

Answers with Explanations


Classification of pure red cell aplasia Acquired
•• Self-limited: Transient erythroblastopenia of childhood •• Thymoma
•• Hereditary: Congenital pure red cell aplasia (Diamond- •• Lymphomas: Hodgkin's, or Non Hodgkin's, large granular Lymphoma
Blackfan syndrome)Q •• Paraneoplastic to solid tumors
•• Connective tissue disorders: SLE, JRA, RA
•• Virus: Parvovirus B19Q, hepatitis, HTLV, EBV
•• Drugs: phenytoin, azathioprine, chloramphenicol, isoniazid
•• Post ABO incompatible transplantQ

99. a. Fanconi's anemia  Ref: Nelson's 20/e p 2362-2371, Ghai 8/e p 345-347
Fanconi's anemia shows pancytopenia with hypocellular bone marrow while others in the options cause hypercellular bone
marrow.
100. a. Fanconi anemia  Ref: Nelson's 20/e p 2362-2371, Ghai 8/e p 345-347
Common physical findings in Fanconi Anemia (FA):

•• Skin pigment changes including Café-au-lait spots (55% cases) •• Thumb and radial malformations
•• Short stature (in 50% cases) •• Renal and cardiac malformations
•• Microcephaly •• Hypogonadism

The most common anomaly in FA is hyperpigmentation of the trunk, neck, and intertriginous areas, as well as café-au-lait spots
and vitiligo, alone or in combination.
101. c. Aplastic anemia  Ref: Nelson's 20/e p 2362-2371, Ghai 8/e p 345-347
102. d. G6PD deficiency  Ref: Nelson's 20/e p 2362-2371, Ghai 8/e p 345-347
G6PD deficiency causes hemolytic anemia and not pancytopenia.
103. b. Hemolytic anemia  Ref: Nelson's 20/e p 2362-2371, Ghai 8/e p 345-347
104. c. Pure red cell aplasia  Ref: Nelson's 20/e p 2362-2371, Ghai 8/e p 345-347
Aplastic anemia can progress to AML, PNH and Myelodysplastic syndrome.
105. b. 5q- syndrome  Ref: Nelson's 20/e p 2362-2371
106. b. Diamond-Blackfan syndrome  Ref: Nelson's 20/e p 2362-2371, Ghai 8/e p 345-347
Diamond-Blackfan syndrome causes PRCA and not aplastic anemia.
107. c. G-CSF  Ref: Nelson's 20/e p 2362-2371, Ghai 8/e p 345-347
Kostman Syndrome (Severe congenital neutropenia)
•• Arrest in myeloid maturation at promyelocyte stage in bone marrow resulting in consistent ANCs < 200/µL
•• This disorder occurs sporadically or with autosomal dominant (ELA2 mutation) or recessive (HAXI) inheritance
•• Monocytosis, eosinophilia, recurrent, severe pyogenic infections, and anemia of chronic inflammation are usually present
•• 20% of patients develop AML or myelodysplasia associated with monosomy 7
Treatment: Usually respond to administration of G-CSF; bone marrow transplantation may be needed.

 NON-NEOPLASTIC DISORDERS OF WBC

108. c. Watch and wait strategy  Ref: Nelson 20/e p 1051


Cyclic neutropenia
•• It is a rare autosomal dominant disorder where neutrophil elastase gene, ELANE is affected)
•• Regular, periodic oscillations, with the ANC ranging from normal to <200/μL, mirrored by reciprocal cycling of monocytes seen.
•• Mean oscillatory period of the cycle is 21 days (± 4 days).
•• Affected patients with severe neutropenia may be treated with filgrastim (G-CSF)
109. a. Less than 500/mL  Ref: Nelson's 20/e p 1048, Ghai 8/e p 357
Serious infections are most likely when the neutrophil count falls below 500 per mm3.
110. c. Strongyloides  Ref: Nelson's 20/e p 1048, Ghai 8/e p 357
Parasitic infestations: Ascariasis, Hookworm, Strongyloides, Filariasis, Trichinosis can cause eosinophillia.
111. a. CD3  Ref: Wintrobes 12th/e p 2522
CD3 is a marker present on all T cells and so is Pan T cell marker.
344 Section 3: Systemic Pediatrics
Review of Pediatrics and Neonatology

Hematological cells Immunophenotyping markers (on Flow cytometry)


Blasts CD34,
Megakaryocytic marker CD41,Q CD61
WBC's CD 45Q (Leukocyte Common Antigen)Q

Myeloid cells Anti- MPO,Q CD13, CD33, CD14, CD117

B –cells CD19,Q CD20, CD22, FMC7, CD23, CD79 a, CD79 b, S lg, IgM

T-cells CD3,Q CD2, CD5, CD7, CD8, TCR-α/β, TCR-g/d


NK cells CD16,Q CD56,Q CD57
Plasma cells CD38,Q CD138,Q kappa and Lambda light chains

112. a. Neutrophils  Ref: Nelson's 20/e p 1048, Ghai 8/e p 357


In sepsis or severe inflammatory disorders, morphologic changes can be seen in neutrophils, like cytoplasmic vacuoles,
Toxic granules (coarser and darker than normal neutrophilic granules) and Döhle bodies, which are patches of dilated endoplasmic
reticulum that appear as sky-blue cytoplasmic “puddles.”
113. c. Absence of any specific organ involvement  Ref: Nelson's 20/e p 1039

•• Hypereosinophilic syndrome is characterized by sustained overproduction of eosinophils;


•• Diagnostic criteria include: eosinophilia >1,500 cells/L persisting for >6 months, signs and symptoms of organ involvement and
absence of another diagnosis to explain the eosinophilia;
•• Most serious & life-threatening complication is cardiac disease due to endomyocardial thrombosis & fibrosis.
•• Therapy is initiated with corticosteroids; Imatinib mesylate, Hydroxyurea or Interferon may be beneficial.

 LYMPHOID NEOPLASMS

114. b. Urea, creatinine, phosphate, Ca2+ and K+  Ref: Nelson 20/e p 2432
Tumor lysis syndrome (TLS)
•• Uric acid, phosphates & potassium are released in circulation in large quantities from death of tumor cells.
•• Hyperuricemia can lead to impairment of renal function, which further exacerbates the metabolic abnormalities.
•• Hypocalcemia is a consequence of acute hyperphosphatemia with subsequent precipitation of calcium phosphate in soft tissues.
•• TLS can occur before therapy in patients with a large tumor burden (e.g., Burkitt lymphoma, Hyperleukocytosis)
•• It is usually seen within 12-48 hr of initiating chemotherapy.
115. b. Leukemia  Ref: Nelson 20/e p 2416
Most frequent childhood cancers are: (in order of decreasing frequency)
•• Leukemia (Most common is ALL)
•• Brain Tumors
•• Neuroblastoma
•• Wilms tumor
116. c. Age > 10 years  Ref: Nelson's 20/e p 2437, Ghai 8/e p 599
117. a. Hodgkin's disease  Ref: Nelson's 20/e p 2445-2448, Ghai 8/e p 608-610
Diagnostic Hallmark of Hodgkin Lymphoma are Reed-Sternberg cells, which are binucleate cell or having a single nucleus with
multiple nuclear lobes.
118. a. Philadelphia chromosome is negative;  Ref: Nelson's 20/e p 2444
119. d. Response to treatment  Ref: Nelson's 20/e p 2437, Ghai 8/e p 599
Discussing options one by one:
a. Hyperploidy Intermediate prognosis; can be considered good
b. Organomegaly L. Node, liver, spleen enlargement, Testicular enlargement → poor prognosis
c. TLC > 50,000/uL Poor prognosis; TLC <10,000/uL has good prognosis
d. Response to treatment Early response to treatment is a good prognostic factor for ALL

120. a. tdT positive  Ref: Nelson's 20/e p 2437; tdt is a marker of lymphoblasts and so seen in ALL.
Chapter 14: Pediatric Hematology 345

121. a. Often localized to single axial group of lymph nodes, c. Contiguous spread of lymph nodes, d. Can be cured by

Answers with Explanations


Chemotherapy and Radiotherapy, e. Commonly presents with painless lymphadenopathy  Ref: Nelson's 20/e p 2445-2447
Harrison's 19/e p 708-709
HL appears to arise in lymphoid tissue and spread to adjacent lymph node areas in a relatively orderly fashion.
Patients commonly present with painless, non-tender, firm, rubbery, cervical or supraclavicular lymphadenopathy and usually some
degree of mediastinal involvement. Chemotherapy & radiation therapy are both effective in treatment of HL.
122. c. Hodgkin's Lymphoma  Ref: Nelson's 20/e p 2445
123. a. Tibia  Ref: Nelson's 20/e p 2425, Ghai 8/e p 736

Age group Site of bone marrow sampling


Younger children Anterior medial tibialQ area, below tibial tuberosity
Adult Anterior or posterior iliac crest area

124. a. Age >10 years  Ref: Nelson's 20/e p 2425, Ghai 8/e p 736;
125. a. Rituximab  Ref: Nelson's 20/e p 2437, Ghai 8/e p 599;
126. a. Pre-B cell ALL  Ref: Nelson's 20/e p 2437, Ghai 8/e p 599;
127. d. Hodgkin lymphoma: EBV and Reed Sternberg cell  Ref: Nelson's 20/e p 2445, Ghai 8/e p 608
The given cell shows Reed Sternberg cell seen in Hodgkin lymphoma and caused by EBV.
128. d. Translocation involving MYC gene  Ref: Nelson's 20/e p, Ghai 8/e p 600
•• Starry sky appearance is a feature of Burkitt lymphoma
•• The genetic hallmark of Burkitt is translocation involving MYC gene on chr 8.
129. a. Bone marrow  Ref: Nelson's 20/e p 2439-2441, Ghai 8/e p 599
•• Infants with ALL, and patients with t(4;11), have a higher risk of relapse despite intensive therapy
•• Higher levels of MRD (>0.01%) present at end of induction suggest a poorer prognosis
•• Relapse occurs in the bone marrow in 15-20% of patients with ALL and carries the most serious implications

130. a. Acute lymphoblastic leukemia, d. Brain tumor, e. Wilms' tumor  Ref: Nelson's 20/e p 2437, Ghai 8/e p 599 Common
Childhood Tumors are: Leukemia (ALL) > Brain tumors > Lymphoma > Neuroblastoma > Wilm's tumor.

High Yield Points about childhood tumors  


•• Most common childhood leukemia is ALL •  Most common tumor in infancy is Neuroblastoma
• Most common solid tumor of childhood is Brain tumor •  Most common abdominal tumor in child is Neuroblastoma

131. c. Pre B ceII ALL  Ref: Nelson's 20/e p 2437, Ghai 8/e p 599
132. a. Mature B-cell  Ref: Nelson's 20/e p 2437, Ghai 8/e p 599
•• ALL L3 is Burkitt Lymphoma with spill over, so it arises from mature B cell
•• ALL L1 and L2 are Acute Leukemia, so it arises from immature B cell.
133. c. Precursor B-cell ALL  Ref: Nelson's 20/e p 2437, Ghai 8/e p 599
Precursor B-cell ALL, B-cell ALL and T-cell ALL has poor prognosis.
134. a. Acute leukemia  Ref: Nelson's 20/e p 2437, Ghai 8/e p 599
Pallor (anemia), petechial spot (thrombocytopenia), fever (infection due to neutropenia) with splenomegaly suggest the
diagnosis of acute leukemia. Splenomegaly eliminates aplastic anemia; presence of anemia and recurrent infections, go against
ITP; Hypersplenism presents with massive splenomegaly.
135. c. Prednisolone, d. L-asparginase, e. Vincristine  Ref: Nelson's 20/e p 2437, Ghai 8/e p 599
Anticancer therapy with prednisolone, vincristine, daunorubic, L-asparginase, 6-mercaptopurine and methotrexate are used for
induction and consolidation in ALL.
136. d. Hypoploidy  Ref: Nelson's 20/e p 2437, Ghai 8/e p 599; Refer Ans. 115

 MYELOID NEOPLASMS
137. c. Acute promyelocytic lukemia (M3)  Ref: Nelson's 20/e p 2442-2443
Acute Promyelocytic Leukemia (APML), characterized by a gene rearrangement involving retinoic acid receptor [t(15;17);
PML-RARA], is very responsive to all-transretinoic acid (ATRA, tretinoin) combined with anthracyclines and cytarabine.
The success of this therapy makes marrow transplantation in 1st remission unnecessary for patients with this disease.
Arsenic trioxide is also an effective non-cytotoxic therapy for APML
346 Section 3: Systemic Pediatrics

138. b. t(15:17)
Review of Pediatrics and Neonatology

139. a. Acute myeloid leukemia  Ref: Nelson's 20/e p 2442-2443, Ghai 8/e p 605-607
•• Auer rods are Needle-like azurophilic fusiform inclusions in cytoplasm of myeloblasts
•• Auer rods are most reliable morphological feature of AML; Myeloblasts Stain +ve with MPO and Sudan Black B
140. d. M7  Ref: Nelson's 20/e p 2442-2443, Ghai 8/e p 605-607
AML with megakaryocytic maturation is the most common Acute Leukemia in Down syndrome.
141. b. Philadelphia chromosome  Ref: Nelson's 20/e p 2442-2443, Ghai 8/e p 605-607
JMML is a childhood mixed MDS/MPD that includes childhood leukemias previously classified as CMML, juvenile CML, and
infantile monosomy 7 syndrome. Remember: presence of bcr/abl fusion (Philadelphia chromosome) gene is not compatible
with a diagnosis of juvenile myelomonocytic leukemia (JMML).
142. a. Juvenile CML  Ref: Nelson's 20/e p 2442-2443, Ghai 8/e p 605-607
Increased hemoglobin F for age is a feature of JMML or Juvenile CML.
143. d. Immediately start induction chemotherapy  Ref: Nelson's 20/e p 2442-2443, Ghai 8/e p 605-607
Hyperleukocytosis
•• Hyperleukocytosis refers to leukocyte count usually greater than 100,000/mL
•• It is a medical emergency most commonly seen in with AML or CML in blast crisis
•• There is sludging of blasts in microcirculation → obstruction of small vessels esp in b ra i n & l u n g s
•• Treatment includes: IV fluids (hydration), Alkalinisation, Allopurinol (to prevent tumor lysis syndrome) and Hydroxyurea (for leukoreduction)
•• Induction chemotherapy is initiated only after leukoreduction, as it carries a high-risk of acute tumor lysis syndrome

144. d. M6  Ref: Nelson's 20/e p 2442-2443, Ghai 8/e p 605-607


FAB classification of AML is based on degree of maturation (M0 to M3) & lineage of leukemic blast (M4 to M7).
•• Nonspecific esterase is positive in monoblasts (AML M4 and M5) and Promyelocytes (AML M3).
145. d. Monosomy 7  Ref: Nelson's 20/e p 2442-2443, Ghai 8/e p 605-607
•• Good prognostic factors in AML are t(8;21), inv16, t(15;17), CEBPa mutation, NPM mutation, Down syndrome
•• Bad prognostic factors in AML are monosomy 5 or 7, 5q-, FLT3 gene duplication and abnormal 11q
146. a. Acute promyelocytic leukemia  Ref: Nelson's 20/e p 2442-2443, Ghai 8/e p 605-607
•• Arsenic trioxide can be used for treatment of resistant cases of Acute promyelocytic leukemia (AML M3).
147. b. An immature T-cell phenotype (Tdt/CD34/CD3)  Ref: Nelson's 20/e p 2442-2443, Ghai 8/e p 605-607
•• Anterior mediastinal mass can be seen due to thymic infiltration in case of T-cell ALL
•• Tdt and CD34 are markers of Blasts while CD3 is a marker of T cell.
148. c. Promyelocytic leukemia  Ref: Nelson's 20/e p 2442-2443, Ghai 8/e p 605-607
•• This patient has complaints of bleeding along with thrombocytopenia, decreased fibrinogen & increased PT & APTT. This
is consistent with DIC; DIC is most commonly seen with M3 AML
149. d. Turner's syndrome  Ref: Nelson's 20/e p 2442-2444

Genetic conditions predisposing to leukemia


•• Down syndrome •• Klinefelter syndrome •• Bloom syndrome
•• Diamond-Blackfan anemia •• Neurofibromatosis type 1 •• Schwachman syndrome
•• Turner syndrome (least associated) •• Ataxia - telangiectasia •• Paroxysmal nocturnal hemoglobinuria
•• Fanconi syndrome •• Severe combined immune deficiency. •• Li-Fraumani syndrome

Note: All the given conditions are risk factors for leukemia. Out of them Turner syndrome is least associated with leukemia.
150. b. Down syndrome  Ref: Nelson's 20/e p 2442-2443, Ghai 8/e p 605-607
‘Neonates and infants with Down Syndrome may have a condition known as ‘transient myeloproliferative syndrome which
mimics congenital leukemia'.
151. a. Osteolytic lesion in flat bones, b. Metaphyseal osteoporosis, c. Periosteal new bone formation, e. Transverse line of
dark band below the growth plate  Ref: Nelson's 20/e p 2442-2443, Ghai 8/e p 605-607
Features seen on X-ray in a child with Acute Leukemia:
•• Reduced bone density •• Collapsed vertebra
•• Metaphyseal lucent band •• Widening of sutures
•• Metaphyseal cortical bone erosions •• Periosteal reactions & lytic bone lesions

152. a. Osteolytic lesions in flat bones, b. Subperiosteal erosions, c. Osteoporosis, d. Thick luscent line just below growth plate
and e. Metaphyseal lucent bands  Ref: Nelson's 20/e p 2442-2443, Ghai 8/e p 605-607
Chapter 14: Pediatric Hematology 347

153. d. AML M4, e. AML M5  Ref: Nelson's 20/e p 2442-2443, Ghai 8/e p 605-607

Answers with Explanations


Monocytic stain is non-specific esterase (NSE) is positive in monocytic leukemia like AML M4 and M5.
154. c. Philadelphia chromosome is positive  Ref: Nelson's 20/e p 2442-2443, Ghai 8/e p 605-607; Refer Ans. 127

 LANGERHANS CELL HISTIOCYTOSIS


155. a. Birbecks granules  Ref: Nelson's 20/e p 2484-2489, Ghai 8/e p 620-623
X-bodies called Birbeck granules are characteristically seen in LCH.
156. d. Birbeck's granules  Ref: Nelson's 20/e p 2484-2489, Ghai 8/e p 620-623
Tennis racquet and not badminton racquet appearance is seen in EM of LCH; LCH is positive for CD1a and S100.
157. a. CD 1a  Ref: Nelson's 20/e p 2484-2489, Ghai 8/e p 620-623
Immunohistochemistry shows: CD1a, S-100 or Langerin (CD 207) in Histiocytosis.
158. d. Eosinophilic granuloma  Ref: Nelson's 20/e p 2484-2489, Ghai 8/e p 620-623; Refer pretext of this chapter;

•• Eosinophilic granuloma: Bone lesions, with no visceral involvement; Localised; affects head and neck

159. a. CD la  Ref: Nelson's 20/e p 2484-2489, Ghai 8/e p 620-623


160. d. Gonadal involvement occurs  Ref: Nelson's 20/e p 2484-2489, Ghai 8/e p 620-623
a. Peak incidence less than 3 years of age: True; More common and more severe in younger children
b. Radiosensitive: True, as Bone lesions and Pituitary lesions, causing DI, respond well to radiotherapy
c. Diffuse form is known as Letterer-Siwe disease: True, Refer Ans. 154 above
d. Gonadal involvement occurs: FALSE; Gonadal involvement is usually not seen in LCH.
161. c. Cladribine  Ref: Nelson's 20/e p 2484-2489, Ghai 8/e p 620-623
Treatment of LCH
•• Single system disease: spontaneous remission seen; curettage or local radiotherapy;
•• Multisystem disease: Multiagent chemotherapy, which may include vinblastine, etoposide, prednisolone
•• For unresponsive disease: Cyclosporine/antithymocyte globulin, Imatinib, Cladribine or Stem cell transplantation.
162. c. Langerhan's cell histiocytosis  Ref: Nelson's 20/e p 2484-2489, Ghai 8/e p 620-623
Seborrheic dermatitis, polyuria, hepatosplenomegaly and ear discharge suggests diagnosis of LCH.
163. c. Parallel rays of tubular structures in lysosomes  Ref: Nelson's 20/e p 2484-2489, Ghai 8/e p 620-623
Given scenario suggests Gaucher disease, in which glucocerebroside in lysosomes seen.
164. a. Can be associated with diabetes insipidus, c. Birbeck's granules in langerhan's cell, d. Proliferation of antigen presenting
cells, e. Associated with specific HLA DR  Ref: Nelson's 20/e p 2484-2489, Ghai 8/e p 620-623
165. c. Histiocytosis X  Ref: Nelson's 20/e p 2484-2489, Ghai 8/e p 620-623
Osteolytic lesions in a child points towards the diagnosis of Histiocytosis X; Most common is skull.

 BLEEDING DISORDERS
166. a. HPA 1a  Ref: Nelson's 20/e p 2406
•• Neonatal alloimmune thrombocytopenic purpura (NATP) is caused by development of maternal antibodies against antigens present
on fetal platelets that are shared with father and recognized as foreign by maternal immune system.
•• The most common cause is incompatibility for the platelet alloantigen HPA-1a.
167. d. Portwine hemangioma and e. Thrombocytosis  Ref: Nelson's 20/e p 2400-2407, Ghai 8/e p 349-352
Kasabach Meritt syndrome:
•• Association of giant hemangioma with localized intravascular coagulation causing thrombocytopenia and hypo­fibrinogenemia is
Kasabach-Merritt syndrome; Heart failure can be seen due to AV malformations
•• Treatment: Drugs (corticosteroids, alpha interferon and vincristine), laser photocoagulation and radiation
•• Surgery is usually not indicated due to risk of hemorrhage

168. c. Idiopathic thrombocytopenic purpura  Ref: Nelson's 20/e p 2400-2407, Ghai 8/e p 349-352
169. c. Viral infection predisposes and e. More common in children  Ref: Nelson's 20/e p 2400- 2407
a. False; In childhood, males and females are equally affected
b. False; Platelet antibody testing is seldom useful in acute ITP.
c. True; A recent history of viral illness is described in 50-65% of cases of childhood ITP
d. False; 20% of children (and 50% of adolescents) who present with acute ITP go on to have chronic ITP.
e. True; The peak age is 1-4 yr, although the age ranges from early in infancy to the elderly.
348 Section 3: Systemic Pediatrics

170. a. Antibody mediated  Ref: Nelson's 20/e p 2400-2407, Ghai 8/e p 349-352
Review of Pediatrics and Neonatology

Quinine causes immune mediated thrombocytopenia.


171. c. Palpable splenomegaly  Ref: Nelson's 20/e p 2400-2407, Ghai 8/e p 349-352
a. Chronic ITP occurs most commonly in adult women, younger than age 40 years (Female: Male = 4:1)
b. Cutaneous bleeding occurs in the form of pinpoint hemorrhages, petechiae or ecchymoses. Often there will be history of easy
bruising, nose bleeds, bleeding from gums and hemorrhage in soft tissues from relatively minor trauma.
c. Despite the increased destruction of platelets in spleen, spleen size usually remains normal in ITP.
d. Bone marrow reveals a modestly increased number of megakaryocytes.
172. a. Glanzmann syndrome, b. Bernard-Soulier syndrome, c. Wiskott-Aldrich syndrome  Ref: Nelson's 20/e p 2400-2407
Platelet function defect can be seen in:
•• Glanzmann thrombasthenia •• Hermansky-Pudlak syndrome •• Wiskott-Aldrich syndrome
•• Bernard-Soulier syndrome •• Chediak-Higashi syndrome •• Grey platelet syndrome.

173. a. Platelet adhesion, b. BT, e. PFA-100  Ref: Nelson's 20/e p 2400-2407, Ghai 8/e p 349-352
Platelet function assessment:
•• PFA-100—This is a test to measure the platelet adhesion-aggregation in response to agonist combination like collagen-
epinephrine or collagen-ADP.
•• Bleeding time—The usual bleeding time is 4–8 minutes. Bleeding for a longer time suggests platelet defects.
174. c. Bleeding time  Ref: Nelson's 20/e p 2400-2407, Ghai 8/e p 349-352
•• ITP is characterized by thrombocytopenia which prolongs the bleeding time.

 COAGULATION DISORDERS
175. b. Both PT and aPTT are increased in DIC & (c) IV Ig is useful in ITP
Cryoprecipitate is not useful for treating hemophilia B, as cryoprecipitate contains a high concentration of coagulation factor VIII,
coagulation factor XIII, and fibrinogen (but not factor IX)
176. a. Anti factor X a  Ref: Nelson's 20/e p 2396
Low Molecular Weight Heparin (LMWH) use in children:
•• Because of ease of dosing and need for less monitoring, LMWH is being used more frequently in pediatric patients.
•• Unlike unfractionated Heparin, which is monitored using the aPTT, LMWH is monitored via anti-Xa activity.
•• LMWH formulation that has been used most often in pediatric patients is enoxaparin
•• Recommended standard starting dose of enoxaparin for infants <2 mo is 1.5 mg/kg/dose subcutaneously every 12 hr; for patients >2 mo it
is 1 mg/kg subcutaneously every 12 hr.
•• Peak levels are achieved 2-6 hr following an injection.

177. a. Factor V  Ref: Nelson's 20/e p 2384-2389, Ghai 8/e p 350-352


Factor V Leiden: glutamine → arginine substitution at amino acid residue 506 that renders factor V resistant to cleavage and
inactivation by protein C (also called Activated Protein C resistance)
178. a. Hereditary spherocytosis  Ref: Nelson's 20/e p 2330, Ghai 8/e p 338
179. b. Gp IIb/IIIa  Ref: Nelson's 20/e p 2384, Ghai 8/e p 350

Mnemonic
Mnemonic “B comes before G and Ib comes before IIb”
Disease Deficient Gp Qualitative defect in
Bernard Soulier syndrome Ib/IX Platelet adhesion (Bernard-defective adhesion)
Glanzmann thrombasthenia IIb/IIIa Platelet aggregation

180. a. 2, 3 BPG in blood  Ref: Nelson's 20/e p 2336, Ghai 8/e p 345
•• Greater affinity of fetal Hb for oxygen is explained by lack of fetal Hb's interaction with 2,3-bisphosphoglycerate (2,3-BPG
or 2,3-DPG); In adult red blood cells, 2,3 DPG decreases the affinity of hemoglobin for oxygen
•• 2,3-BPG interacts less efficiently with fetal hemoglobin than adult hemoglobin, due to a change in a single amino acid (residue
143) found in the 2,3-BPG ‘binding pocket': from histidine to serine, leading to greater oxygen affinity.
181. b. Factor IX  Ref: Nelson's 20/e p 2376-2377
Cryoprecipitate contains, Factor VIII, Fibrinogen, and Von-Willebrand factor.
Chapter 14: Pediatric Hematology 349

182. a. SLE  Ref: Nelson's 20/e p 1176-1180

Answers with Explanations


183. a. Reed Sternberg  Ref: Nelson's 20/e p 2445, Ghai 8/e p 608
184. c. Ferritin  Ref: Nelson's 20/e p 2336, Ghai 8/e p 345
185. c. Flow cytometry  Ref: Nelson's 20/e p 2334-2336
Flow cytometry of RBC showing absence of CD55 and CD59 is gold standard diagnosis of PNH.
186. a. Yersinia enterocolitica  Ref: Nelson's 20/e p 2325
Iron therapy may increase the virulence of malaria and certain, gram-negative bacteria, particularly in developing countries.
Iron overload is associated with Yersinia infection.
187. d. Factor VIII  Ref: Nelson's 20/e p 2390, Ghai 8/e p 349
2 major functions of vWF are: Platelet adhesion via Gp Ib/IX and stabilization of factor VIII in circulation.
188. a. Hemophilia  Ref: Nelson's 20/e p 2390, Ghai 8/e p 349
189. b. Bernard Soulier syndrome  Ref: Nelson's 20/e p 2400, Ghai 8/e p 349
190. c. As part of both extrinsic and intrinsic pathways  Ref: Nelson's 20/e p, Ghai 8/e p
Factor Xa is necessary for conversion of prothrombin to thrombin in the common pathway, that means as as part of both extrinsic
and intrinsic pathways.
191. b. Von Willebrand factor  Ref: Nelson's 20/e p 2386-2390, Ghai 8/e p 349
Von Willebrand factor present in plasma and expressed on endothelium after tissue injury is necessary for platelet adhesion
with the help of Gp Ib/IX on platelets.
192. a. Anti-D is used in Immune Thrombocytopenic purpura > d. EACA…  Ref: Nelson's 20/e p 2386-2390
a. True; Anti-D can be used in treatment of Immune Thrombocytopenic purpura
b. False; DDAVP (Desmopressin) is used in Type 1 VWD and some cases of type 2 VWD (and not in vWD type 3)
c. False; With mild factor VIII hemophilia, the patient's endogenously produced factor VIII can be released by the administration
of DDAVP; But, in patients with moderate or severe factor VIII deficiency, the stored levels of factor VIII in the body are
inadequate, and desmopressin treatment is ineffective.
d. True (partly); Factor XI deficiency or Hemophilia C is associated with mild to moderate bleeding; Patients undergoing dental
extractions may benefit from treatment with fibrinolytic inhibitors like aminocaproic acid, with plasma replacement therapy used
only if hemorrhage occurs. So, if bleeding occurs, FFP is to be given;
193. d. 20,000/µL  Ref: Wintrobes 13/689
In general, the risk of significant spontaneous hemorrhage increases gradually as the platelet count drops to <50 × 109/L and is high
at counts <5 × 109/L. So among the options the best answer is 20,000/uL.
194. b. Adhesion  Ref: Nelson's 20/e p 2390, Ghai 8/e p 349;
195. a. AML  Ref: Nelson's 20/e p 2442-2444
196. b. Hemolytic uremic syndrome  Ref: Nelson's 20/e p 2508, Ghai 8/e p 492
•• Neurologic findings (due to microthrombi formed in vessels of CNS) are present in TTP in addition to the triad seen in
HUS (Microangiopathic hemolytic anemia, Thrombocytopenia and Renal failure).
197. c. Sindhis  Ref: Nelson's 20/e p 2349, Ghai 8/e p 341
198. b. Factor XII  Ref: Nelson's 20/e p 2384, Ghai 8/e p 352

a. Factor VII Causes increase in PT not aPTT


b. Factor XII Increased aPTT; Usually asymptomatic or can result in thrombosis rather than bleeding
c. Factor X Deficiency results in increase of PT and aPTT
d. Factor XIII Both PT and aPTT is normal

199. a. Normal ferritin level  Ref: Nelson's 20/e p 2351


In children with Thalassemia major, even if the child does not receive transfusions, iron eventually accumulates with elevated serum
ferritin and transferrin saturation.
200. d. Aplastic anemia  Ref: Nelson's 20/e p 2384, Ghai 8/e p 352
Lymphadenopathy and hepatosplenomegaly are very rare in aplastic anemia.
201. a. Factor XIII deficiency  Ref: Nelson's 20/e p 2384, Ghai 8/e p 352
Factor XIII deficiency (Fibrin stabilizing factor or transglutaminase deficiency):
•• Clinical symptoms include bleeding from umbilical stump
•• Because factor XIII is responsible for the cross linkage of fibrin to stabilize the fibrin clot, symptoms of delayed hemorrhage are secondary to the
clot instability
•• Results of the usual screening tests for hemostasis like PT and aPTT are normal in these patients
•• The normal clot remains insoluble in the presence of urea, whereas in a patient with factor XIII deficiency, the clot dissolves due to failure of
cross linking
350 Section 3: Systemic Pediatrics

202. d. Till joint effusion subsides  Ref: Nelson's 20/e p 2384, Ghai 8/e p 352
Review of Pediatrics and Neonatology

Treatment of Hemarthrosis (in Hemophilia): 50-60 IU/kg factor VIII concentrate on day 1; then 20-30 IU/kg on days 2, 3, 5 until
joint function is normal or back to baseline. Consider additional treatment every other day for 7-10 days.
203. a. Usually occurs in cow milk fed babies  Ref: Nelson's 20/e p 2384; Refer pretext for details;
“Hemorrhagic disease of newborn is more common in breast fed infants because breast milk is poor in vitamin K”.
204. d. Factor XIII  Ref: Nelson's 20/e p 2384, Ghai 8/e p 352; Refer Ans 203 above;
Individuals with a positive bleeding history, particularly with delayed bleeding, umbilical stump bleeding or miscarriages and
in whom the initial panel of screening test is negative, should be tested for Factor XIII deficiency.
205. b. Increased aPTT, c. X-linked recessive  Ref: Nelson's 20/e p 2384, Ghai 8/e p 352
206. a. Isolated prolonged PTT with a normal PT  Ref: Nelson's 20/e p 2384, Ghai 8/e p 352
Von Willebrand's factor acts as a plasma carrier of factor VIII and circulates in blood as factor VIII-VWF complex. Its deficiency
therefore impairs intrinsic pathway of coagulation and prolongs aPTT; As the extrinsic pathway of coagulation remains unimpaired,
PT is not altered.

Interpretation of important tests of blood coagulation

Bleeding time (Normal: •• It is a not a test for coagulation rather it tests the ability of platelets to form a hemostatic plug
2-9 minutes) •• Prolongation generally indicates the defect in platelet number or function
•• This assay tests the extrinsic and common coagulation pathway
Prothrombin time (PT)
•• So, a prolonged PT can result from deficiency of factor V, VII, X, prothrombin or fibrinogen
Partial thromboplastin •• This assay tests the intrinsic and common coagulation pathways
time (PTT) •• Prolonged PTT results from deficiency of factor V, VIII, IX, X, XI, XII, prothrombin or fibrinogen
•• It is the time taken for clotting to occur when thrombin is added to the plasma
Thrombin time
•• It tests the conversion of fibrinogen to fibrin and depends on adequate fibrinogen level

207. c. Factor XII deficiency  Ref: Nelson's 20/e p 2388


Remember: Deficiency of the “contact factors” (factor XII, prekallikrein and high molecular-weight kininogen) causes prolonged
aPTT but no bleeding symptoms.
Child is question underwent tonsillectomy without hemorrhage and is having prologed PTT, so the diagnosis is factor XII deficiency,
among the given options.
208. a. Constitutional anemia  Ref: Nelson's 20/e p 2390-2392, Ghai 8/e p 349-351
Constitutional pancytopenia is defined as decreased marrow production of the 3 major hematopoietic lineages on an inherited basis,
resulting in anemia, neutropenia and thrombocytopenia. Important causes are:
•• Fanconi anemia •• Down syndrome
•• Amegakaryocytic thrombocytopenia •• Dyskeratosis congenita
•• Shwachman-Diamond syndrome •• Noonan syndrome

209. c. Autosomal dominant  Ref: Nelson's 20/e p 2384-2385


Most common type of vWD is type I, which has autosomal dominant inheritance
Chapter 15
Tumors of Infancy
and Childhood
 RETINOBLASTOMA High Yield Points   M
Features M
The most frequent childhood cancers
•• Retinoblastoma is the most common primary intraocular tumorQ in children are: (in order of decreasing frequency)
•• 60% are acquired while remaining 40% are hereditary •• Leukemia (Most common is ALL)
•• Neuroblastoma
•• Caused by inactivation of both alleles of tumor suppressor gene (RB1) on chr 13q 14.
•• Wilms tumor
•• Hepatoblastoma
Clinical Presentation •• Retinoblastoma
•• Most common presentation is leukocoriaQ earliest presentation is strabismus
•• Arises from the inner layer of retina and its overgrowth can lead to vitreous seeding and
retinal detachment, necrosis and calcification.
High Yield Points
Diagnosis
•• Risk of transmitting Retinoblas­
•• Most of the cases are diagnosed before 3 years of age tomas to offspring is 50%
•• Ophthalmoscopy under general anaesthesia is the mainstay of diagnosis •• Bilateral disease can be seen in 25%
•• CT scan demonstrates the presence of calcification cases
•• MRI is preferred for diagnosis to rule out trilateral retinoblastoma syndrome

Treatment
Question 1
•• Chemotherapy (carboplatin, vincristine, etoposide) for reduction of tumor size followed by
A 2-year-old child pre­sented with
laser photocoagulation for small tumor or cryotherapy for large tumors
the following abnormality. There
•• Enucleation is done only for tumors with no possibility of vision restoration is a history of similar illness in
•• Radiotherapy is reserved for treatment resistant cases. his father. What might be the
underlying condition responsible?
 NEUROBLASTOMA
•• MC abdominal tumor of childhoodQ 98% cases are sporadic, 2% cases are familialQ
•• Most frequently diagnosed tumor of infancy (<1yr of age)Q
•• 90% of neuroblastomas produce catecholamines: vanillyl-mandelic acid (VMA) &
homovanillic acid.

Sites of Tumor M
a. Retinitis pigmentosa
•• Adrenal medullaQ (most common site), along the sympathetic chain > pelvisQ, neck,
b. Acute Leukemia
and brain (cerebral neuroblastomas). c. Retinoblastoma
d. Coloboma
Morphology
•• Foci of punctate intra-tumoral calcificationQ
•• Homer-Wright pseudorosettesQ; Stains +ve with Neuron-specific enolase (NSE) High Yield Points
•• Electron microscopy: central dense cores (containing catecholamines) surrounded by a
peripheral halo. •• Trilateral retinoblastoma = b/l
retinoblastoma syndrome + tumor of
Clinical Features the pineal gland
•• Most common secondary tumor
•• Depend on tumor site and extent of disease with retinoblastoma: Osteosar­coma,
•• Metastatic ds: Fever, irritability, failure to thrive, bone pain, bluish subcutaneous nodules, soft tissue sarcoma and malignant
orbital proptosis, periorbital ecchymoses melanoma
•• Localized disease: Asymptomatic mass or cognitive dysfunction. •• Most common route of spread:
•• Catecholamine producing Tumors: Hypertension, profound secretory diarrhea. Direct spread through optic nerve
•• Extensive tumors: Tumor lysis syndrome & disseminated intravascular coagulation. occurs
352 Section 3: Systemic Pediatrics

Staging
Review of Pediatrics and Neonatology

High Yield Points


•• HemangiomasQ are the most International Neuroblastoma Staging System
common tumors of infancy Stage Definition 5 year survival
•• Characteristic translocation, t(12;15)
Q, has been described in infantile 1 Localized tumor with complete gross excision with ipsilateral >90%
fibrosarcomas (ETV6-NTRK3 fusion lymph nodes negative
transcript)
•• Sacrococcygeal teratomas are the 2A Localized tumor with incomplete gross excision; with lymph 70-80%
most common teratomas of child­ nodes negative
hoodQ 2B Localized tumor with ipsilateral nonadherent lymph nodes 70-80%
•• Neuroblastoma: Most common positive for tumor
extra­cranial solid tumor of
childhoodQ 3 Unresectable unilateral tumor infiltrating across the midline, 40-70%
with or without regional lymph node involvement

4 Any primary tumor with dissemination to distant lymph nodes; 85-90% (<18 mo)
bone; bone marrow, liver skin, and other organs (except as 30-40% (>18 mo)
defined for stage 45)

4S Localized primary tumor (as defined for stage 1, 2A, or 2B), with
dissemination limited to skin, liver, and bone marrow, (limited
to infants <1 year of age)
•• Most common sites of metastasis in Neuroblastoma are regional or distant lymph nodes, long
Question 2 bones, skull, bone marrow, liver & skin. Lung and brain metastases are rare.
The following appearance is
usually seen in which disease? Prognosis
Prognostic Factors in NeuroblastomaQ
Variable Favorable Unfavorable
Stage Stage 1, 2A, 2B, 4S Stage 3, 4
a. Neuroblastoma
Age <18 months >18 months
b. Retinoblastoma
c. Acute leukemia Schwannian stroma & gangliocytic Present Absent
d. Osteosarcoma differentiation on histology
 Mitosis-karyorrhexis index <200/5000 cells >200/5000 cells
DNA ploidy Hyperdiploid Near-diploid
Mnemonic N-MYC Not amplified Amplified
Expression of: Chromosome 17q gain Absent Present
TRKA - 'Achaa' - Good prognosis Chromosome 1p loss Absent Present
TRKB - 'Buraa' - Poor prognosis
Chromosome 11q loss Absent Present
TRKA expression Present Absent
TRKB expression Absent Present
Telomerase expression Low or absent Highly expressed

Treatment
•• Low-risk neuroblastoma: surgery for stages 1 and 2 and observation for stage 4S with
cure rates generally >90% without further therapy
•• Intermediate-risk neuroblastoma: surgery, chemotherapy ± radiotherapy
–– The chemotherapy usually includes moderate doses of cisplatin or carboplatin,
cyclophosphamide, etoposide, and doxorubicin given for several months
High Yield Points •• High-risk neuroblastoma (survival rate ~25–35%):
–– High-dose chemotherapy with autologous stem cell rescue
•• Stage 4S neuroblastomas have a
very favorable prognosis, and –– Surgery, radiation, and 13-cis-retinoic acid (isotretion)
many regress spontaneously with­ –– Induction chemotherapy: Cyclophosphamide, topotecan, doxorubicin, vincristine,
out therapy. cisplatin, and etoposide.
Chapter 15: Tumors of Infancy and Childhood 353

 WILMS' TUMOR

Section 3: Systemic Pediatrics


•• Most common primary renal tumor of childhoodQ
•• Peak incidence : between 2 and 5 yearsQ.

High Yield Points


•• Most common initial clinical presentation for WT is incidental discovery of an asymptomatic abdominal
mass by parents while bathing or clothing or by a physician during a routine examination.

Morphology High Yield Points


Triphasic combination: Blastemal, Stromal, epithelial cell; Wilms' Tumor Can present as
•• Synchronous: Both kidneys involved
Staging of Wilms' Tumor
simultaneouslyQ
Stage I Tumor confined to the kidney and completely resected. Renal capsule, vessels or •• Metachronous: Kidneys affected
lymph nodes not involved one after the otherQ

Stage II Tumor extend beyond the kidney but is incompletely resected with negative margins
and lymph nodes. At least 1 of the following has occurred: (a) penetration of renal
capsule, (b) invasion of renal sinus vessels

Stage III Residual tumor present following surgery confined to abdomen, including gross or
microsopic tumor, spilllage of tumor preoperatively or intraoperatively, biopsy prior
to nephrectomy, regional lymph node metastases; tumor implants on peritoneal surface;
extension of tumor thrombus into inferior vena cave and heart

Stage IV Hematogenous metastases (lung, liver, bone, brain, etc.) or lymph node metastases
outside the abdominopelvic region

Stage V Bilateral renal involvement by tumor

Congenital Malformations with Increased Risk of Wilms' tumor


WAGR syndrome Denys-Drash syndrome Beckwith-Wiedemann
(33%risk)Q (90% risk, Maximum risk)Q syndrome (BWS) High Yield Points
WT1 gene: Chr 11p13Q Gonadal dysgenesisQ (male Organomegaly: macroglossiaQ, Anaplasia in Wilm's tumor
Wilms tumor, Aniridia, Genital pseudohermaphroditism) hemihypertrophyQ, •• Large, hyperchromatic, pleomor­
anomalies, and mental Early-onset nephropathy (diffuse omphalocele, and abnormal phic nuclei & abnormal mitoses.
Retardation mesangial sclerosis)Q large cells in the adrenal cortex •• Signifies: presence of TP53
increased risk of (adrenal cytomegaly) mutationsQ & resistanceQ to
gonadoblastomasQ Genomic imprintingQ is the chemotherapy
causative mechanism •• Indicates poor prognosis

Treatment
•• Nephrectomy alone may be sufficient for patients younger than 2 yr of age with stage I
disease and a tumor weighing <550 g
•• Patients with stages I and II disease receive chemotherapy with 2 drugs, vincristine and
actinomycin D (also called dactinomycin), every 1-3 wk for a total of 18 wk (regimen EE4A) Question 3
•• Patients with stage III or IV disease receive chemotherapy with 3 drugs (vincristine,
Identify this syndrome which
doxorubicin, and actinomycin D) every 1-3 wk for a total of 24 wk (regimen DD4A) and carries increased risk of Wilms’
radiation therapy. Patients with regional lymph node metastases, residual disease after surgery, tumor
or tumor rupture receive radiation therapy to the flank or abdomen, and those with lung
metastases receive radiation therapy.
Prognosis
•• Presence of loss of heterozygosity at 1p and 16q confers an adverse prognosis
•• Patients with diffuse anaplasia have a poor outcome.
•• Secondary Tumors following Wilms’ tumor are: bone and soft-tissue sarcomas, leukemia,
lymphomas, and breast cancers. a. WAGR syndrome
b. Sturge-Weber syndrome
HEMATOLOGICAL MALIGNANCIES c. Denys-Drash syndrome
Refer chapter on 'Pediatric Hematology' of this book. d. Beckwith-Wiedemann syndrome
354 Section 3: Systemic Pediatrics

 SACRO-COCCYGEAL TERATOMA (SCT)


Review of Pediatrics and Neonatology

Question 4
What is the condition this newborn •• Sacro-coccygeal teratoma (SCT) refers to a teratoma arising in the sacro-coccygeal region
is suffering from? •• They are thought to arise from totipotent cells from the node of Hensen at the anterior aspect
of the coccyx by about the 2nd to 3rd weeks of gestation
•• There are mostly mixed solid/cystic, although purely cystic types occur in ~15%.
Classification
•• Benign (mature): much more common, comprising ≈60-70%
•• Malignant (immature).

High Yield Points


a. Osteosarcoma •• The coccyx is almost always involved in sacrococcygeal tera­toma
b. Sacrococcygeal teratoma •• It is the commonest congenital tumour in the fetus & neonate
c. Meningomyelocele •• Sacrococcygeal region is the commonest location for non CNS teratomas
d. Neuroblastoma •• The tumour is composed of all 3 germ cells (i.e. ectoderm, mesoderm & endoderm)
•• Most common teratoma of childhood is sacrococcygeal teratoma.

Complications
•• High output cardiac failure from AV shunting: which in turn can cause hydrops fetalis
•• Ureteric obstruction/Gastrointestinal tract obstruction
High Yield Points •• Compression of underlying nerves: giving urinary/faecal incontinence
•• Anemia, Dystocia, Tumour rupture.
•• Myelomeningocoele & vertebral
anomalies are associated with SCT
•• Can have elevated levels of: alpha
Treatment and Prognosis
fetoprotein (AFP) & beta HCG in SCT •• A SCT can be benign or malignant depending on whether mature or immature
•• Those presenting in older infants tend to have a higher malignant potential while those
presenting in utero have poor prognosis due to complications
•• Malignant change may be also commoner in males
•• Treatment is with surgical excision inclusive of coccygectomy with additional chemotherapy
for malignant tumours.
High Yield Points   M  RHABDOMYOSARCOMA (RMS)
•• Most common pediatric soft tissue
sarcoma is RhabdomyosarcomaQ •• Rhabdomyosarcoma is the most common pediatric soft tissue sarcoma
•• Most commonly involves head and neck (25%) followed by genitourinary tract (24%),
extremities (19%), orbit (9%), & retroperitoneum.

Classification
Three recognized histologic subtypes:
•• Embryonal typeQ accounts for 60% of all cases and has an intermediate prognosis
Question 5
•• Botryoid type, a variant of the embryonal form in which tumor cells & an edematous stroma project
A 3-year-old child presented with into a body cavity like a bunch of grapes,Q is found most often in the vagina, uterus, bladder,
the lesions shown in the picture. nasopharynx, and middle ear
What is the most probable
•• Alveolar type (25-40% of cases):
diagnosis?
–– characterized by the presence of PAX/FOXO1 fusion transcript
–– The tumor cells tend to grow in nests that often have cleft-like spaces resembling alveoli
–– Occur most often in trunk & extremities and carry the poorest prognosis
•• Pleomorphic type (adult form) is rare in childhood, accounting for <1% of cases

Clinical Manifestations
•• Most common presenting feature of RMS is a massQ that may or may not be painful
•• Symptoms are caused by displacement or obstruction of normal structures
•• Origin in the nasopharynx may be associated with nasal congestion, mouth breathing,
a. Wilms tumor
epistaxis, and difficulty with swallowing and chewing
b. Sacrococcygeal teratoma
c. Rhabdomyosarcoma •• Regional extension into the cranium can produce cranial nerve paralysis, blindness, and
d. Lipoma signs of increased intracranial pressure with headache and vomiting
Chapter 15: Tumors of Infancy and Childhood 355

•• When the tumor develops in the face or cheek, there may be swelling, pain, trismus, and,

Section 3: Systemic Pediatrics


as extension occurs, paralysis of cranial nerves Mnemonic
•• Tumors in the neck can produce progressive swelling with neurologic symptoms.
Small round blue cell tumors
Microscopy “Low NEW MRP”
•• Lymphoma
•• It a small, round, blue cell tumorQ like Neuroblastoma, lymphoma, and Ewing sarcoma
•• Neuroblastoma
•• Definitive diagnosis is established by biopsy, microscopic appearance & results of •• Ewing Sarcoma
immunohistochemical stains and analysis of PAX/FOXO1 expression •• Wilms Tumor
•• CT or MRI is necessary for evaluation of the primary tumor site. •• Medulloblastoma
•• Retinoblastoma
Treatment •• Primitive Neuroectodermal Tumor
•• It is based on risk classification of tumor, which is determined by stage of tumor, histology, &
amount of tumor surgically resected prior to chemotherapy
•• For low-risk: vincristine and D-actinomycin
•• For high-risk disease: vincristine, actinomycin, cyclophosphamide & radiation.

 BRAIN TUMORS
•• They are the second most common malignancy in childhood and adolescence
•• Brain tumors are the most common solid tumor of childhood
•• Histologically Gliomas account for majority of tumors including malignant tumors
•• Hereditary syndromes & cranial exposure to ionizing radiation are associated with a higher
incidence of brain tumors.

Syndromes Associated with Brain Tumors M

Syndrome Tumors associated Chromosome Gene


Neurofibromatosis type 1 Optic pathway gliomas, astrocytoma, malignant peripheral nerve sheath 17q11 NF1
(autosomal dominant) tumors, neurofibromas
Neurofibromatosis type 2 Vestibular schwannomas,meningiomas, spinal cord ependymoma, 22q12 NF2
(autosomal dominant) spinal cord astrocytoma, hamartomas
Von Hippel–Lindau Hemangioblastoma 3p25-26 VHL
(autosomal dominant)
Tuberous sclerosis Subependymal giant cell astrocytoma, cortical tubers 9q34 TSC1
(autosomal dominant) 16q13 TSC2
Li-Fraumeni Astrocytoma, primitive neuroectodermal tumor 17q13 TP53
(autosomal dominant)
Cowden Dysplastic gangliocytoma of the cerebellum (Lhemitte-Duclos disease) 10q23 PTEN
(autosomal dominant)

High Yield Points


•• During 1st year of life, supratentorial tumors predominate & include choroid plexus complex tumors
and teratomas.
•• In children 1–10 years, infratentorial tumors predominate mostly Juvenile pilocytic astrocytoma and
medulloblastoma.
•• After 10 years of age, supratentorial tumors again predominate, with diffuse astrocytomas most
common.
•• Most aggressive tumor in children—medulloblastoma
•• Most common tumor in children—pilocytic astrocytoma.

Clinical Features
Depends on the tumor location, type and age of child.
Supratentorial (cortical) lesions:
•• Subtle changes in personality, mentation, and speech
•• Lateralized deficits, such as focal motor weakness, focal sensory changes, language disorders,
focal seizures, and reflex asymmetry.
356 Section 3: Systemic Pediatrics

•• Infants with supratentorial tumors may present with premature hand preference
Review of Pediatrics and Neonatology

High Yield Points


•• Features of increased intracranial pressure such as vomiting, lethargy, irritability &
•• Diencephalic syndrome: Failure to macrocephaly (in infants)
thrive, emaciation despite normal Midline or infratentorial tumors:
caloric intake, and inappropriately •• Disorders of equilibrium, gait, and coordination can occur;
normal or happy affect, occurs in •• Classic triad of headache, nausea, and vomiting as well as papilledema is seen.
infants and young children with
•• Blurred vision, diplopia, and nystagmus can be seen.
suprasellar region tumors.
•• Parinaud syndrome is seen Tumors of brainstem: Gaze palsy, multiple cranial nerve palsies, and upper motor neuron deficits
with pineal region tumors and is (e.g. hemiparesis, hyperreflexia, clonus).
manifested by paresis of upward
gaze, pseudo-Argyll Robertson Optic pathway tumors: Manifest as visual and/or afferent oculomotor disturbances, such as
pupil, nystagmus to convergence or decreased visual acuity, Marcus Gunn pupil (afferent pupillary defect), nystagmus, and/or visual
retraction, and eyelid retraction. field defects.
Suprasellar region and 3rd ventricular region tumors: May manifest initially as neuroendocrine
deficits, such as subacute development of obesity, abnormal linear growth velocity, diabetes
insipidus, galactorrhea, precocious puberty, delayed puberty, hypothyroidism, Diencephalic
syndrome.

Posterior Fossa Tumors of Childhood

Relative
Tumor incidence Presentation Diagnosis Prognosis

Medulloblastoma 35–40% 2–3 months of headache, Enhancing 4th ventricular 65-85% survival dependent on stage/type;
(midline vomiting, truncal ataxia mass, Homer Wright rosettes poorer (20–70%) in infants.
cerebellar tumor) characteristic

Cerebellar 35–40% 3–6 months of limb Cerebellar hemisphere mass 90–100% survival in totally resected
astrocytoma ataxia; secondary usually with cystic and solid pilocytic type.
headaches, vomiting components. Malignant astrocytomas
Rosenthal fibres characteristic in with over expression of P53 is a bad
pilocytic type prognostic factor

Brainstem glioma 10–15% 1–4 months of double Diffusely expanded, partially > 90% mortality in diffuse tumors; better in
vision unsteadiness, enhancing mass in 80%; more focal localized
cranial nerve deficits lesion in 20%

Ependymoma 10–15% 2–5 months of Usually enhancing 4th ventricular > 75% survival in totally resected lesions
unsteadiness, headaches, mass with cerebellopontine
double vision, facial predilection
asymmetry

Atypical teratoid/ > 5 (10–15% As in medulloblastoma, As in medullo- blastoma, but often 10–20% survival in infants
rhabdoid of infantile but primarily in infants; more laterally extended
malignant often associated facial
tumors) weakness and strabismus

SOME IMPORTANT CNS TUMORS


PILOCYTIC ASTROCYTOMA

High Yield Points •• It is a localized astrocytomas, WHO grade I tumor


•• Occurs in children; indolent clinical course
Medulloblastoma with WNT type has •• Located in cerebellum but may also appear in 3rd ventricle or optic nerves.Q
best prognosis, while Group 3 type
•• Biphasic tumors; Rosenthal fibers & eosinophilic granular bodies seen.
with MYC amplification & i17q have
worst prognosis
MEDULLOBLASTOMA
•• WHO grade IV tumor; it is an embryonal tumor or PNET
•• Occurs predominantly in children and exclusively in the cerebellum.
Chapter 15: Tumors of Infancy and Childhood 357

Histopathology

Section 3: Systemic Pediatrics


High Yield Points
•• Largely undifferentiated tumor, appears as small round blue cell tumorQ
•• Most common genetic abnormality
•• Abundant mitoses & markers of cellular proliferation, such as Ki-67 in medulloblastoma is chr 17p
•• Homer-Wright rosettes seen. Positive for synaptophysin deletion
•• Medulloblastoma is the childhood
Metastasis
brain tumor that most commonly
Dissemination through the CSF is a common complication—giving rise to nodular masses at some metastasizes extraneuronally
distance from the primary tumor called as “drop metastases.”Q

Treatment
Exquisitely radiosensitive.Q

Prognosis
Chang staging system used;
Age < 4 years, disseminated disease at presentation & those with residual disease after surgery have
poorer outcome

CRANIOPHARYNGIOMA

•• Adamantinomatous variant predominates in childhood


•• Often seen endocrinologic abnormalities like growth failure, delayed sexual maturation
•• Visual changes can occur
•• Surgery is the primary treatment; there is no role for chemotherapy

Answer Keys for Image-Based Questions

Answers Explanations / Identifying features


1. Ans. c. Retinoblastoma Leucocoria or white eye reflex is present in left eye; Presence of leukocoria in a young child with a
positive family history suggests a diagnosis of Retinoblastoma

2. Ans. a. Neuroblastoma The given picture shows ‘Raccoon eyes’, which is characteristically seen in Neuroblastoma, because of
proptosis & periorbital ecchymosis

3. Ans. d. Beckwith-Wiedemann This baby has hemihypertrophy of left side of body (compare left side with right side) & omphalocele;
syndrome These are seen in Beckwith-Wiedemann syndrome, in which increased risk of Wilms’ tumor is seen

4. Ans. b. Sacrococcygeal Large soft tissue tumor in an infant, involving the sacrococcygeal region
teratoma

5. Ans. c. Rhabdomyosarcoma ‘Bunch of grapes’ appearance of a tumor (due to tumor cells & edematous stroma), protruding out from
the vagina of a child, is suggestive of Sarcoma Botryoides, a type of Rhabdomyosarcoma
358 Section 3: Systemic Pediatrics
Review of Pediatrics and Neonatology

Questions
 RETINOBLASTOMA 11. All of the following are true about neuroblastoma
except: (Recent Question 2017)
1. All are true about Retinoblastoma gene except: M 
a. Diarrhea b. Bone pain
 (JIPMER Nov 2017)
c. Cafe-au-lait spots d. Opsomyoclonus
a. Autosomal dominant inheritance 12. All are good prognostic factors in neuroblastoma
b. Needs both mutation of both copies of gene–two hit except: M  (Recent Question 2015)
c. Chr 13p14
a. Stage 4S b. Trk A expression
d. Tumor suppressor gene
c. Trk B expression d. Age <18 months
2. Retinoblastoma spreads to CNS through: M 
13. Most common site of metastases in neuroblastoma:
 (AIIMS May 2015)
 (Recent Question 2015)
a. Direct b. Lymphatic
a. Lung b Skull
c. Arterial d. Vitreous
c. Liver d. Vertebrae
3. Flexner Wintersteiner rosettes are seen in:
 (APPG 2014)
14. True regarding stage IV-S of neuroblastoma are all
except: (Recent Question 2015)
a. Hepatoblastoma b. Neuroblastoma
a. Limited to infants < 1 year
c. Nephroblastoma d. Retinoblastoma
b. Primary localized tumor
4. Children with germline retinoblastoma are more likely
to develop other primary malignancies in their later c. Dissemination to bone
lifetime course. Which of the following can occur in d. Good prognosis
such patients? (AIIMS Nov 2013) 15. Which SOLID Tumor causes bilateral proptosis?
a. Osteosarcoma of lower limbs and soft tissue sarcoma  (AIIMS Nov 2013)
b. Thyroid carcinoma a. Rhadbomyosarcoma b. Retinoblastoma
c. Seminoma c. Neuroblastoma d. PNET
d. Squamous cell carcinoma 16. A 4-year-old child having palpable abdominal mass and
5. Highest cure rate is of: (NEET Pattern 2012) hypertension with sweating and diarrhea is due to:
a. Wilms‘ tumor b. Retinoblastoma a. Neuroblastoma (Recent Question 2013)
c. Rhabdomyosarcoma d. ALL b. Nephroblastoma
6. True about retinoblastoma are all except: (WBPG 2012) c. PCKD (Polycystic kidney disease)
a. AD d. All of the above
b. Most common intraocular tumor of childhood 17. Opsoclonus is associated with: M 
c. Females are more affected than males  (Recent Question 2013)
d. Bilateral involvement not rare a. Neuroblastoma b. Nephroblastoma
7. Most common inherited childhood tumor is: M  c. Retinoblastoma d. Hypernephroma
 (AIIMS May 2008, AIIMS May 05) 18. Catecholamines are secreted by: M  (TN PGMEE 2010)
a. Leukemia b. Neuroblastoma a. Retinoblastoma b. Nephroblastoma
c. Retinoblastoma d. Wilms' tumor c. Neuroblastoma d. Glioblastoma
8. Childhood malignancy producing proptosis are: 19. Which of the following statements about neuroblastoma
 (PGI June 05) is not true? (AIPGMEE 09)
a. Neuroblastoma b. Hepatoma a. Most common extracranial solid tumor in childhood
c. Retinoblastoma d. Germ cell tumor b. > 50% present with metastasis at the time of diagnosis
e. Nephroblastoma c. Lung metastasis are common
9. Retinoblastoma gene is located on: M  (TNPGMEE 2000) d. Often encase aorta and its branches at the time of
a. Chromosome 5 b. Chromosome 8 diagnosis
c. Chromosome 13 d. Chromosome 16 20. Neuroblastoma originates from:  (PGI June 2008)
a. Adrenals b. Mediastinum
 NEUROBLASTOMA c. Chest wall d. Peripheral nerves
e. Neck
10. A malignant tumor of childhood that frequently 21. A malignant tumor of childhood, that metastasizes to
metastasizes to bone marrow most often is: bones most often, is: (AIPGMEE 2006, 2002)
 (FMGE June 2018) a. Wilms' tumor
a. Wilms' tumor b. Neuroblastoma b. Neuroblastoma
c. Adrenal gland tumors c. Adrenal gland tumors
d. Granulosa cell tumor of ovary d. Granulosa cell tumor of ovary
Chapter 15: Tumors of Infancy and Childhood 359

22. The most common malignant neoplasm of infancy is: 33. Most common intra-abdominal solid tumor in children:

Questions
 (AIPGMEE 2005)  (Recent Question 2014)
a. Malignant teratoma b. Neuroblastoma a. Wilms' tumor (Nephroblastoma)
c. Wilms' tumor d. Hepatoblastoma b. Neuroblastoma
23. To which of the following events is ‘good' outcome in c. Rhabdomyosarcoma
neuroblastoma associated? (AIPGMEE 2004) d. Hodgkin lymphoma
a. Diploidy b. N-myc amplification 34. A 2-year-old male child presents with a lump in the right
c. Chromosome I p deletion side of the abdomen. Ultrasound revealed it to be a solid
d. Trk A expression mass. On examination, his right arm and leg were found
to be longer. The most likely diagnosis is: 
24. Neuroblastoma differs from Wilms' tumor radiologically
 (AIIMS May 2013)
by all except: (AIIMS June 01)
a. Wilms' tumor b. Neuroblastoma
a. Calcification
c. Angiomyolipoma d. Nephroblastoma
b. Aorta and IVC are not eroded but pushed aside
35. All are true about Wilms' tumor except:
c. Location
 (Recent Question 2013)
d. Intraspinal extension of tumor
a. Most commonly presents as abdominal mass in children
25. Neuroblastoma—good prognostic factor is:
b. Locally invasive
 (PGI June 2000)
c. Chloramphenicol is used
a. N-myc amplification b. RAS oncogene
d. All are true
c. Hyperdiploidy d. Translocation
36. Wilms' tumor is associated with the following except:
e. Hypodiploidy
 (UPSC-I 10)
26. A 1-year-old child presented with abdominal mass with
calcification, possibilities are: (PGI Dec 2000) a. Aniridia b. Horse-shoe kidney
c. Hemihypertrophy d. Opsoclonus
a. Neuroblastoma b. Wilms' tumor
c. Nephronophthisis d. Pheochromocytoma 37. Which of the following is post-chemotherapy based
e. Hydronephrosis staging system in Wilms' tumor? (AIPGMEE 2009)
a. National Wilms' tumor staging system (NWTS)
 WILMS' TUMOR b. International society of pediatric oncology (ISPO)
c. AJCC TNM
27. Which of the following are true about Wilms’ tumor? d. Chadwick
 (PGI May 2017) 38. Wilms' tumor most commonly presents as: (DPGEE 08)
a. In stage 2 tumor extend into inferior vena cava a. Fever b. Hematuria
b. Most common presentation is asymptomatic abdominal c. Abdominal mass d. Pain in abdomen
mass 39. Which statement(s) is/are true about neuroblastoma with
c. Hypertension is seen in >80% patients respect to Wilms' tumor? (PGI Dec 08)
d. Patient with stage 1 requires radiation therapy after
a. Neuroblastoma causes displacement of kidney infero­
chemotherapy
laterally without distortion of collecting system
e. Prognosis depends upon histology and staging
b. Stippled calcification is present in Wilms' tumor
28. The most common presentation of a child with Wilms'
c. Aortic and IVC invasion by neuroblastoma
tumor is: M  (Recent Question 2016)
d. Neuroblastoma crosses midline
a. As asymptomatic abdominal mass e. Intraspinal extension by Wilms' tumor
b. Hematuria
40. Which of the following are associated with Wilms'
c. Hypertension
tumor? (PGI Dec 08)
d. Hemoptysis due to pulmonary secondary
29. Most common tumor of kidney in children is: a. Aniridia
 (PGI Jan 2017) b. Genitourinary abnormalities
a. Wilms’ tumor b. Neuroblastoma c. Cataract
c. Polycystic kidney disease d. Hemihypertrophy
d. Renal cell carcinoma e. Lymphoma e. Macroglossia
30. Wilm's tumor is associated with all the following 41. The ideal timing of radiotherapy for Wilms' tumor after
except: (Recent Question 2016) surgery is: (AIPGMEE 06)
a. Aniridia b. Beckwith syndrome a. Within 10 days b. Within 2 weeks
c. Polycystic kidney d. Hemi-hypertrophy c. Within 2 months d. Any time after surgery
31. Most common abdominal mass in children: M 42. The most important determinant of prognosis in Wilms'
 (Recent Question 2015) tumor: M  (AIPGMEE 06)
a. Hydronephrosis b. Wilms’ tumor a. Stage of disease
c. Neuroblastoma d. Rhabdomyosarcoma b. Loss of heterozygosity of chromsome I p
32. All are true about Wilms' tumor except: c. Histology
a. Triphasic morphology (Recent Question 2015) d. Age less than one year at presentation
b. MC renal malignancy in children 43. Deletion of chromosome 11 leads to: (DNB 2001)
c. Associated with hemihypertrophy a. Wilms' tumor b. Neuroblastoma
d. Does not respond to chemotherapy and radiotherapy c. Retinoblastoma d. Osteosarcoma
360 Section 3: Systemic Pediatrics

44. The most common presentation of a child with Wilms' 54. A 6-year-old boy has been complaining of headache,
Review of Pediatrics and Neonatology

tumor is: (AIPGMEE 05, AIIMS Dec 95) ignoring to see objects on the sides for four months. On
a. As asymptomatic abdominal mass examination, he is not mentally retarded, his grades at
b. Hematuria school are good, and visual acuity is diminished in both
c. Hypertension eyes. Visual charting showed significant field defect. CT
d. Hemoptysis due to pulmonary secondary scan of head showed suprasellar mass with calcification.
Which of the following is the most probable diagnosis?
 (Maha PG 2014, AIIMS May 04)
 CNS TUMORS
a. Astrocytoma b. Craniopharyngioma
45. Medulloblastoma is: (FMGE Nov 2017) c. Pituitary adenoma d. Meningioma
a. Malignant tumor in adults 55. Medulloblastoma exclusively occurs in the: M 
b. Malignant tumor in children  (MAHA PGM CET 2016)
c. Benign brain tumor a. Medulla b. Cerebellum
d. Malignant cerebellar tumor in adults c. Cerebral hemispheres d. Spinal cord
46. Syndromes related to child brain tumor include:
56. A 10-year-old child presented with headache, vomiting,
 (PGI Nov 2017)
gait instability and diplopia. On examination he
a. Tuberous sclerosis b. Neurofibromatosis
had papilledema and gait ataxia. The most probable
c. Cockayne syndrome d. Cowden syndrome
diagnosis is:  (AIIMS Nov 02)
e. Pierre Robin syndrome
a. Hydrocephalus b. Brainstem tumor
47. All of the following are true about Medulloblastoma
c. Suprasellar tumor d. Midline posterior fossa tumor
except: M  (FMGE pattern 2017)
a. It arises from Medulla b. It is an embryonal tumor 57. True statement regarding brain tumor in children is:
c. It is a small round blue cell tumor a. Mostly is infratentorial (AIPGMEE 2000)
d. Chang staging system used b. Papilledema is rare
48. A 1.5-year-old female is brought to the clinic with c. Is the most common tumor in children
complaints of excessive enlargement of head, intolerance d. Hydrocephalus is rare
to feeds and severe malnourishment. MRI imaging was
suggestive of a medulloblastoma causing obstructive  OTHER CHILDHOOD TUMORS
hydro­cephalus. Which of the following is an example of
irrational management of the patient? (AIIMS May 2014) 58. A child with pigment dilution of skin and eyes,
a. Craniotomy and sub-total excision of the tumor. Surgeon hemorrhagic diathesis, ceroid deposition within the
leaves the layer of the tumor adherent with colliculus reticuloendothelial system has: (JIPMER May 2018)
b. First ventriculoperitoneal shunt was done a. Bloom syndrome
c. CCNU and vincristine were given as chemotherapy b. Hermansky Pudlak syndrome
d. Radiotherapy 35–40 Gy was given to the whole cranio­ c. Alezzandrini syndrome d. Vogt-Koyanagi syndrome
spinal axis 59. All are associated with malignancy except:
49. MC optic nerve tumor in children causing blindness:  (Recent Question 2016, MP 2000)
 (Recent Question 2013) a. Down's syndrome b. Fragile X-syndrome
a. Glioma b. Meningioma c. Bloom syndrome d. Fanconi's anemia
c. Craniopharyngioma d. Astrocytoma
60. Most common tumor in infant: M 
50. Which of the following childhood tumors show
 (Recent Question 2015)
extracranial metastasis? (AIIMS May 2010)
a. Hemangioma b. Brain tumor
a. Glioblastoma multiforme
c. Leukemia d. Ewing sarcoma
b. Medulloblastoma
61. Rhabdomyosarcoma is treated by:
c. Choroid plexus carcinoma
 (MAHA PGM CET 2015)
d. Ependymoblastoma
a. Chemotherapy b. Radiation
51. Chang staging is used for: M  (AIIMS May 2010)
c. Surgery d. All of the above
a. Retinoblastoma b. Medulloblastoma
62. Most common cancer in children less than 10 years: M
c. Ewing's sarcoma d. Rhabdomyosarcoma
 (Recent Question 2015)
52. About Pilocytic astrocytoma, false is:  (AIIMS May 09)
a. Leukemia b. Neuroblastoma
a. Spreads to posterior fossa c. Brain tumor d. Wilm's tumor
b. Seen at eighty years of age 63. The following tumor is not common in the first decade:
c. Best prognosis among all intracranial neoplasms.  (Recent Question 2015)
d. Surgery and radiation therapy used for treatment
a. Ameloblastoma b. Retinoblastoma
53. Which of the following statements about cerebellar c. Neuroblastoma d. Rhabdomyosarcoma
astrocytomas in pediatric age group is False?
64. All of the following are small round blue cell tumors
a. These are usually low grade tumors (AIPGMEE 2008) except: (Recent Question 2015)
b. These tumors have a good prognosis
a. Lymphomas b. Osteosarcoma
c. These are more commonly seen in 1st & 2nd decades
c. Neuroblastoma d. Rhabdomyosarcoma
d. These tumors are more common in females
Chapter 15: Tumors of Infancy and Childhood 361

65. A grape like, polypoid, bulky mass protruding through 72. Germ cell tumor(s) of pediatric age group includes all

Questions
vagina in a 4-year-old girl is characteristic of: except: (PGI Nov 09)
 (MAHA PGM CET 2014) a. Pure yolk sac tumor b. Leydig cell tumor
a. Fibrosarcoma b. Sarcoma botryoides c. Choriocarcinoma d. Embryonal cell carcinoma
c. Leiomyosarcoma d. Inflammatory polyp e Endodermal sinus tumor
66. The most common malignancy in childhood resulting to 73. True about childhood tumor are all except: (PGI June 08)
death is: (TNPGMEE 2014) a. Wilms' tumor b. Neuroblastoma
a. Leukemia b. Wilms’ tumor c. Retinoblastoma d. Embryonal rhabdomyoma
c. Retinoblastoma d. Neuroblastoma e. Pleomorphic rhabdomyosarcoma
67. Uncommon in children: (NEET Pattern 2013) 74. In PML, all of the following are seen except:
a. Ewing's sarcoma b. Neuroblastoma a. Retinoic acid is used in treatment (AIPGMEE 2007)
c. Osteosarcoma d. None of above b. 15/17 translocation may be seen
68. Which of the following is the most common malignant c. CD 15/34 both seen in same cell
tumor of infancy? (DNB June 2012) d. Associated with disseminated intravascular coagulation
a. Neuroblastoma b. Wilm's tumor 75. Most common benign tumors during infancy is: 
c. Polycystic kidney disease d. Renal cell carcinoma  (UPPG 2007)
69. Which of the following does not require a lumbar a. Lymphangioma b. Hemangioma
puncture in children?  (AIIMS Nov 2012) c. Cystic hygroma d. Lipoma
a. ALL b. HL 76. Most common site of histiocytosis is: (AIIMS May 07)
c. NHL d. AML a. Bone b. Skin
70. A child presents with seborrheic dermatitis, sinusitis and c. Lung d. Liver
chronically draining ears. On examination child has failure 77. A 7-year-old boy with left renal mass had bone pain and
to thrive, with hepatosplenomegaly and exophthalmos, was detected to have bone metastatic deposits. The most
probable diagnosis is: (AIIMS Nov 2012) likely renal tumor is:  (AIIMS Nov 04)
a. Histiocytosis X a. Wilms' tumor b. Renal cell carcinoma
b. Wegener's granulomatosis c. Clear cell sarcoma d. Rhabdoid tumor
c. Retinoblastoma 78. Which of the following can present as mass in posterior
d. Chediak-Higashi syndrome mediastinum in children? (PGI June 2001)
71. Which is not a tumor of the first decade of life? a. Rhabdosarcoma
 (DPG 10, AIIMS 92) b. Duplication cyst of esophagus
a Rhabdomyosarcoma b. Neuroblastoma c. Lymphoma
c. Ameloblastoma d. Retinoblastoma d. Neuroblastoma
e. Thymoma
362 Section 3: Systemic Pediatrics
Review of Pediatrics and Neonatology

Answers with Explanations


 RETINOBLASTOMA
1. c. Chr 13p14
Autosomal dominant inheritance is seen in familial retinoblastoma
Rb gene is a tumor suppressor gene on Chr 13q14.
2. a. Direct  Ref: Nelson's 20/e p 2476-2478, Ghai 8/e 614-615
Spread of Retinoblastoma:
•• By direct extension to choroid; Along the optic nerve beyond lamina cribrosa to the CNS, or
•• By hematogenous or lymphatic spread to distant sites, including bones, bone marrow, and lungs.

3. d. Retinoblastoma  Ref: Nelson's 20/e p 2476-2478, Ghai 8/e 614-615


Rosette type Associated tumors
Homer Wright rosetteQ Neuroblastoma, medulloblastoma, primitive neuroectodermal tumor, pineoblastoma
Flexner-Wintersteiner rosetteQ Retinoblastoma, pineoblastoma, medulloepithelioma
Perivascular pseudorosette Ependymoma, medulloblastoma, primitive neuroectodermal tomor, central neurocytoma, glioblas-
toma, monomorphous pilomyxoid astrocytomas

4. a. Osteosarcoma of lower limbs and soft tissue sarcoma  Ref: Nelson's 20/e p 2476-2478
Most common secondary tumor in a child with Retinoblastoma are Osteosarcoma, soft tissue sarcoma and malignant
melanoma
5. d. ALL  Ref: Nelson's 20/e p 2441, 2467, 2470, 2476-2478;
Discussing about the options one by one,
a. Wilms’ Tumor: Overall survival of children with WT approaches 90%, with some prognostic factors (low stage, favorable
histology, young age, low tumor weight) conferring even better outcomes.
b. Retinoblastoma: 95% of children in US with retinoblastoma are cured with modern treatment. Although survival in developed
countries is high, retinoblastoma progresses to metastatic disease & death in > 50% of children worldwide;
c. Rhabdomyosarcoma: Among patients with resectable tumor & favorable histology, 80-90% have prolonged disease-
free survival
d. ALL: Overall 5 yr survival in ALL is 90% due to improvements in therapy & risk stratification.
6. c. Females are more affected than males  Ref: Nelson's 20/e p 2476-2478, Ghai 8/e 614-615
Discussing options one by one:
a. AD: true; because of the loss of heterozygosity it behaves as autosomal dominant
Let's try to understand it, in the hereditary form of retinoblastoma, the first mutation in RB gene is inherited through germinal cells
and a 2nd mutation occurs subsequently in somatic retinal cells. 2nd mutations that lead to retinoblastoma often result in loss of
the normal allele & concomitant loss of heterozygosity.
b. True; Retinoblastoma is an embryonal malignancy of the retina and the most common intraocular tumor in children.
c. False, as males & females are equally affected in retinoblastoma
d. True; Overall, about 66-75% of children with retinoblastoma have unilateral tumors, with the remainder having bilateral
retinoblastoma (that is 25-34%), so not rare
7. c. Retinoblastoma  Ref: Nelson's 20/e p 2476-2478, Ghai 8/e 614-615
8. a. Neuroblastoma, c. Retinoblastoma  Ref: Nelson's 20/e p 2476-2478, Ghai 8/e 614-615
Proptosis
Definition Protrusion of the eye; it is a common indicator of orbital disease
Pathophysiology It may be caused by: shallowness of the orbits, as in many craniofacial malformations, or by increased
tissue mass within the orbit, as with neoplastic, vascular, and inflammatory disorders
Tumors of Benign tumors: Most common are vascular lesions (principally hemangiomas) & dermoids
orbit causing Malignant neoplasms: Rhabdomyosarcoma, Lymphosarcoma & Metastatic Neuroblastoma are the most
Proptosis frequent
Optic nerve gliomas are most commonly seen in patients with neurofibromatosis and may present with poor
vision or proptosis
Retinoblastoma may extend into the orbit if it is discovered late or goes untreated
Teratomas are rare tumors that typically grow rapidly after birth and exhibit explosive proptosis
Ocular Exposure keratopathy, ocular motor disturbances, and optic atrophy with loss of vision
complications
Chapter 15: Tumors of Infancy and Childhood 363

9. c. Chromosome 13  Ref: Nelson's 20/e p 2476-2478, Ghai 8/e 614-615

Answers with Explanations


Retinoblastoma (RB) gene

•• It is located on chr 13q14; its mutation or deletion is responsible for hereditary form of Retinoblastoma
•• It encodes the retinoblastoma protein, a tumor suppressor protein that controls cell-cycle phase transition and has roles in
apoptosis and cell differentiation

 NEUROBLASTOMA
10. b. Neuroblastoma  Ref: Indian J Hematol Blood Transfus. 2011;27(2):93-95.
The malignancies of prostate, breast, lungs, kidney & thyroid in adults and neuroblastoma and rhabdomyosarcoma in children are
the primary tumors which frequently involve bone marrow.
11. c. Cafe-au-lait spots  Ref: Nelson's 20/e p 2461-2463
Cafe-au-lait spots are associated with neurofibromatosis.
12. c. Trk B expression  Ref: Nelson's 20/e p 2461-2463, Ghai 8/e 616-617; Refer pretext for details;
13. b. Skull  Ref: Nelson's 20/e p 2461-2463, Ghai 8/e 616-617

High Yield Points  


Metastasis in Neuroblastoma
•• Metastatic spread is more common in children >1 yr of age at diagnosis
•• Most common organs involved are lymph nodes, long bones, skull, bone marrow, liver, & skin
•• Lung & brain metastases are rare, occurring in less than 3% of cases

14. c. Dissemination to bone  Ref: Nelson's 20/e p 2461-2463, Ghai 8/e 616-617
International Neuroblastoma Staging System (INSS) stage 4S refers to neuroblastoma in children <1 yr age with dissemination to
liver, skin, and/or bone marrow without bone involvement and with a primary tumor that would otherwise be staged as INSS
stage 1 or 2.
15. c. Neuroblastoma  Ref: Nelson's 20/e p 2461-2463, Ghai 8/e 616-617
•• Retinoblastoma is an intraocular malignancy; Rhabdomyosarcoma presents as a unilateral proptosis.
•• In Neuroblastoma, orbital metastases may be bilateral & present with abrupt onset proptosis & lid ecchymosis (Raccoon eyes).
16. a. Neuroblastoma  Ref: Nelson's 20/e p 2461-2463, Ghai 8/e 616-617
Sometimes neuroblastoma produce catecholamines that can cause increased sweating and hypertension, and some release
vasoactive intestinal peptide, causing a profound secretory diarrhea.
17. a. Neuroblastoma  Ref: Nelson's 20/e p 2461-2463, Ghai 8/e 616-617
‘Opsoclonus–myoclonus–ataxia syndrome'

•• Paraneoplastic syndrome of autoimmune origin associated with Neuroblastoma


•• Rapid, uncontrollable jerking eye & body movements, poor coordination, cognitive dysfunction scan
18. c. Neuroblastoma  Ref: Nelson's 20/e p 2461-2463, Ghai 8/e 616-617
19. c. Lung metastasis are common  Ref: Nelson's 20/e p 2461-2463, Ghai 8/e 616-617
20. a. Adrenals, b. Mediastinum and e. Neck  Ref: Nelson's 20/e p 2461-2463, Ghai 8/e 616-617
Origin of neuroblastoma:

•• Adrenal medullaQ (Most common site) f/b Along the sympathetic chain in the paravertebral region of abdomen (25%)
•• Posterior mediastinumQ (15%); Other sites: pelvis, neck, and brain (cerebral neuroblastomas)
21. b. Neuroblastoma  Ref: Nelson's 20/e p 2461-2463, Ghai 8/e 616-617
22. b. Neuroblastoma  Ref: Nelson's 20/e p 2461-2463, Ghai 8/e 616-617
23. d. Trk A expression  Ref: Nelson's 20/e p 2461-2463, Ghai 8/e 616-617; (Trk'A' = ‘A'chaa)
24. a. Calcification  Ref: Nelson's 20/e p 2461-2463, Ghai 8/e 616-617; Refer Ans. 36 below;
25. c. Hyperdiploidy  Ref: Nelson's 20/e p 2461-2463, Ghai 8/e 616-617; Refer pretext of this chapter;
26. a. Neuroblastoma, b. Wilms' tumor  Ref: Nelson's 20/e p 2461-2463, Ghai 8/e 616-617; Refer Ans. 36;
Both neuroblastoma & Wilms' tumor can present as abdominal mass with calcification.

 WILMS' Tumor
27. b. Most common presentation is asymptomatic abdominal mass, e. Prognosis depends upon histology and staging
Ref: Nelson 20/e p 2465-2467
Extension to Inferior vena cava occurs in stage 3 of Wilms’ tumor;
Hypertension is present in ~25% of tumors at presentation & has been attributed to increased renin activity.
364 Section 3: Systemic Pediatrics

28. a. A
s asymptomatic abdominal mass  Ref: Nelson's 20/e p 2466
Review of Pediatrics and Neonatology

•• Most common initial clinical presentation for WT is incidental discovery of an asymptomatic abdominal mass by parents while bathing or clothing
an affected child or by a physician during a routine physical examination;
•• Hypertension is present in 25% of tumors at presentation and has been attributed to increased renin activity.
•• Abdominal pain, gross painless hematuria, and fever are other frequent findings at diagnosis.

29. a. Wilms' tumor  Ref: Nelson's 20/e p 2464-2465


•• Wilms' tumor (WT), also known as nephroblastoma, is the most common primary malignant renal tumor of childhood;
•• It is the second most common malignant abdominal tumor in childhood.
•• The most common sites of metastases from Wilms tumor are the lungs, regional lymph nodes, and liver.
30. c. Polycystic kidney  Ref: Nelson's 20/e p 2464-2466, Ghai 8/e 617-618; Refer pretext for details:
Wilms' tumor is associated with WAGR syndrome, Denys Drash Syndrome & Beckwith Wiedemann Syndrome (hemi­hypertrophy).
31. c. Neuroblastoma  Ref: Nelson's 20/e p 2464-2466, Ghai 8/e 617-618
•• Neuroblastoma is the most common extracranial solid tumor in children
•• Localized Neuroblastoma can manifest as an asymptomatic mass or can cause symptoms because of the mass itself, including
spinal cord compression, bowel obstruction & superior vena cava syndrome.
Note: Both Neuroblastoma & Wilms' tumor can present as abdominal mass in children but Neuroblastoma is more common than
Wilms' tumor, so it is the better answer to this question
32. d. Does not respond to chemotherapy and radiotherapy  Ref: Nelson's 20/e p 2464-2466; Refer pretext
Patients with favorable histologic findings on Wilms’ tumor biopsy, have a good outcome, as it is sensitive to chemotherapy &
radiotherapy.
33. b. Neuroblastoma  Ref: Nelson's 20/e p 2461-2466, Ghai 8/e 617-618
•• Most common intra-abdominal solid tumor in children is Neuroblastoma, followed by Wilms' tumor.
34. a. Wilms' tumor  Ref: Nelson's 20/e p 2461-2466, Ghai 8/e 617-618
•• The finding of right arm and leg to be longer suggests hemihypertrophy.
•• This is a feature of Beckwith Wiedemann syndrome associated with Wilms’ tumor.
35. c. Chloramphenicol is used  Ref: Nelson's 20/e p 2464-2466, Ghai 8/e 617-618
•• Chloramphenicol is not used for treatment of Wilms’ tumor.
36. d. Opsoclonus  Ref: Nelson's 20/e p 2464-2466, Ghai 8/e 617-618
Opsoclonus is seen in Neuroblastoma & not Wilms' tumor
37. a. National Wilms' tumor staging system (NWTS)  Ref: Nelson's 20/e p 2464-2466, Ghai 8/e 617-618
•• The two major groups which have tremendous contributions in the management of Wilms' tumor are National Wilms' Tumor
Study (NWTS) and the Societe Internationale D'oncologie Pediatrique (SIOP).
•• In 2001, NWTS merged with several other pediatric oncology cooperative groups to create the Children's Oncology Group
(COG). However, the NWTS is still active in name today completing follow-up of the late effects of treatment for patients
previously enrolled in its trials.
38. c. Abdominal mass  Ref: Nelson's 20/e p 2464-2466, Ghai 8/e 617-618
39. a. Neuroblastoma causes displacement of kidney inferolaterally without distortion of collecting system, b. Stippled
calcification is present in Wilms' tumor and d. Neuroblastoma crosses midline  Ref: Nelson's 20/e p 2464-2466
Comparison between Wilms' tumor & Neuroblastoma:

Wilms' Tumor Neuroblastoma


Abdominal mass Yes Yes
Primary origin IntrarenalQ Extrarenal; Arises from adrenal gland or paravertebral
sympathetic ganglia
Physical examination Displacing mass, mainly confined to the flank Non-mobile mass. more likely to cross the midlineQ
Pattern of tumor Direct expansion with displacement of Encasement of vessels and aortic elevation, but does not
spread adjacent structures invade them
Intraspinal extension Rare Intraspinal extradural extension is another typical finding
Effect on kidneys Intrinsically displaces urinary collecting Externally displaces kidneyQ; Neural extension; Often
systems calcified
Calcification Stippled calcificationQ in the abdominal Calcification very common: 90%
mass seen (less common)
Chapter 15: Tumors of Infancy and Childhood 365

40. a. Aniridia, b. Genitourinary abnormalities, d. Hemihypertrophy and e. Macroglossia  Ref: Nelson's 20/e p 2464-2466

Answers with Explanations


Aniridia & Genitourinary abnormalities are seen in WAGR syndrome; Hemihypertrophy & macroglossia are features of
Beckwith Wiedemann syndrome; Both these syndromes are associated with Wilms' tumor.
41. b. Within 2 weeks  Ref: Nelson's 20/e p 2464-2466, Ghai 8/e 617-618
Ref: Kalapurakal et al. Int J Radiat Oncol Biol Phys. 2003.57:495-9.
Radiation therapy for Wilms' tumor:
•• Dose for favorable histology Wilms' tumor is 1080 cGy for the abdomen & 1200 cGy for the lung.
•• Postoperative radiotherapy is started within 14 days of nephrectomy.
42. c. Histology  Ref: Nelson's 20/e p 2464-2466, Ghai 8/e 617-618

•• Histology plays a major role in risk stratification of Wilms' tumor; Absence of anaplasia is considered favorable.
•• Other prognostic factors are: age, stage, tumor weight, & loss of heterozygosity at chromosomes 1p & 16q.

43. a. Wilms' tumor  Ref: Nelson's 20/e p 2464-2466, Ghai 8/e 617-618
44. a. As asymptomatic abdominal mass  Ref: Nelson's 20/e p 2464-2466, Ghai 8/e 617-618

 CNS TUMORS
45. b. Malignant tumor in children  Ref: Nelson’s 20/e p 2458
Medulloblastoma
•• WHO grade IV tumor;
•• It is an embryonal tumor or PNET
•• Occurs predominantly in children & exclusively in Cerebellum
•• Dissemination through the CSF is a common complication—giving rise to nodular masses at some distance from the primary
tumor called as “drop metastases.”
•• Treatment: Exquisitely radiosensitive.
46. a. Tuberous sclerosis, b. Neurofibromatosis & d. Cowden syndrome  Ref: Nelson 20/e p3159
Cockayne syndrome is a rare autosomal recessive disorder, characterized by facial erythema in a butterfly distribution after sun
exposure. Patients with Cockayne syndrome do not have increased risk of cancers.
47. a. It arises from Medulla  Ref: Nelson 20/e p 2458-9
Medulloblastoma is a cerebellar tumor occurring predominantly in males & at a median age of 5–7 years.
48. d. Radiotherapy 35–40 Gy was given to the whole craniospinal axis  Ref: Nelson's 20/e p 2458, Ghai 8/e 612-613
In children < 3 years age, use of radiation therapy is not preferred, as it results in severe late neurologic sequelae,
including microcephaly, learning disabilities, cognitive impairment, neuroendocrine dysfunction (growth failure, hypothyroidism,
hypogonadism, and absence/delay of puberty) &/or second malignancies.
Treatment of Medulloblastoma:
•• A multimodal treatment approach is pursued in medulloblastoma, with surgery as the starting point of treatment
•• Medulloblastoma is sensitive to both chemotherapy and radiation therapy.

Category Treatment
Patients < 3 yr age High-dose chemotherapy with peripheral stem cell reinfusion;
Radiation therapy is not used
Standard-risk patients > 3 yr age with surgical total Lower-dose craniospinal radiation (24 Gy) with chemotherapy
resection & no metastasis during & after radiation therapy
High-risk patients > 3 yr age with metastasis and/or High-dose cranial–spinal radiation (36 Gy) with chemotherapy
bulky residual disease after surgery during & after radiation therapy

49. a. Glioma  Ref: Nelson's 20/e p 3059, Ghai 8/e 612-613

High Yield Points about Optic Glioma  


•• Optic nerve glioma is the most frequent tumor of optic nerve in childhood; it is a benign hamartoma;
•• Principal clinical manifestations are unilateral loss of vision, proptosis, deviation of the eye & optic atrophy

50. b. Medulloblastoma  Ref: Nelson's 20/e p 2453-2460, Ghai 8/e 612-613


Among the given options, Medulloblastoma is the most malignant brain tumor seen in children. Up to 30% of patients with
medulloblastoma present with neuroimaging evidence of leptomeningeal spread.
366 Section 3: Systemic Pediatrics

51. b. Medulloblastoma  Ref: Nelson's 20/e p 2453-2460, Ghai 8/e 612-613


Review of Pediatrics and Neonatology

Chang Staging System: was published in 1969 and was the most widespread classification system for the staging of medullo­
blastomas. However, it is no longer used regularly in clinical practice.
52. b. Seen at eighty years of age  Ref: Nelson's 20/e p 2453-2460, Ghai 8/e 612-613
Pilocytic Astrocytoma usually occurs in children & young adults.
53. d. These tumors are more common in females  Ref: Nelson's 20/e p 2453-2460, Ghai 8/e 612-613
54. b. Craniopharyngioma  Ref: Nelson's 20/e p 2453-2460, Ghai 8/e 612-613
Craniopharyngioma

•• It is a solid tumor with cysts and calcification located in the suprasellar region.
•• It presents with (short stature), visual field defects & hydrocephalus.

55. b. Cerebellum  Ref: Nelson's 20/e p 2458, Ghai 8/e 612-613


•• Medulloblastoma is a cerebellar tumor occurring predominantly in males and at a median age of 5-7 yr.
•• CT & MRI show a solid, homogeneous, contrast enhancing 4th ventricular obstruction & hydrocephalus.
56. d. Midline posterior fossa tumor  Ref: Nelson's 20/e p 2453-2460, Ghai 8/e 612-613
57. a. Mostly is infratentorial  Ref: Nelson's 20/e p 2453-2460, Ghai 8/e 612-613
Remember, the most common tumor in children is acute leukemia.

 OTHER CHILDHOOD TUMORS


58. b. Hermansky pudlak syndrome  Ref: Nelson’s 20/e P 3140-3159
Bloom syndrome
•• AR mutation in BLM/RECQL3 gene, encoding a DNA helicase.
•• Patients are sensitive to UV radiation with increased rates of chromosomal breaks & sister chromatid exchanges.
•• Erythema & telangiectasia on face after exposure to sunlight.
•• Café-au-lait spots & hypopigmented macules may be present.
•• Immunodeficiency, GI malabsorption & hypogonadism are common.
•• Increased tendency to develop solid tumors & lymphoreticular malignancies.
•• Sister chromatid exchange analysis used for diagnosis.
•• The only effective measures to reduce skin disease are sun protection and avoidance.

Hermansky-Pudlak syndrome Alezzandrini syndrome Vogt-Koyanagi-Harada syndrome


•• Autosomal recessive •• Tapetoretinal degeneration •• Vitiligo associated with uveitis
•• Oculocutaneous albinism •• Deafness •• Depigmentation of eyebrows & eyelashes
•• Ceroid accumulation in lysosomes •• Vitiligo •• Dysacusia
•• Prolonged bleeding time •• Poliosis (hypopigmented eyebrows & •• Meningoencephalitis.
eyelashes)

59. b. Fragile X-syndrome  Ref: Nelson's 20/e p 2420-2421


Genetic diseases associated with cancer

Disorder Tumor/Cancer
Down syndrome AML M7, ALL, Transient leukemoid reaction
Fanconi anemia Leukemia, Myelodysplastic syndrome, Gastrointestinal and genitourinary cancers
Bloom syndrome Leukemia, Lymphoma, Solid tumors
Neurofibromatosis type 1 Neurofibroma, Optic glioma, Acoustic neuroma, Astrocytoma, Meningioma, Pheochromocytoma
Neurofibromatosis type 2 Bilateral acoustic neuromas, Meningiomas
Tuberous sclerosis Fibroangiomatous nevi, Myocardial rhabdomyoma

60 a. Hemangioma  Ref: Nelson's 20/e p 3124, Ghai 8/e 138


Hemangiomas are the most common tumors of infancy
61. d. All of the above  Ref: Nelson's 20/e p 2468-2470, Ghai 8/e 618-619
Treatment of Rhabdomyosarcoma:
Chapter 15: Tumors of Infancy and Childhood 367

•• It is based on risk classification of the tumor, which is determined by the stage of tumor, the tumor histology & the amount of

Answers with Explanations


tumor that was surgically resected prior to chemotherapy:
– For low-risk: vincristine & actinomycin
– For higher-risk disease: vincristine, actinomycin & cyclophosphamide along with radiation.
62. a. Leukemia  Ref: Nelson's 20/e p 2437, Ghai 8/e 599-607
Most frequent childhood cancers are: (in order of decreasing frequency) Leukemia (Most common is ALL)Q >
Neuroblastoma > Wilms' tumor > Hepatoblastoma > Retinoblastoma
63. a. Ameloblastoma  Ref: Nelson's 20/e p 2471-2475
64. b. Osteosarcoma  Ref: Nelson's 20/e p 2471-2474, Ghai 8/e 619-620
Small round blue cell tumors are tumors with similar histology of small round cell.
65. b. Sarcoma botryoides  Ref: Nelson's 20/e p 2468, Ghai 8/e 618
Sarcoma Botryoides
•• It is a variant of the embryonal form of RMS
•• Tumor cells & an edematous stroma project into a body cavity like a bunch of grapes.
66. d. Neuroblastoma  Ref: Nelson's 20/e p 2417, Ghai 8/e 616-617

Tumor Leukemia Wilms tumor Retinoblastoma Neuroblastoma


5 year survival 80-90% 90% 97% 78%
This shows the most common cause of death (malignancy associated) is Neuroblastoma
67. d. None of above   Ref: Nelson's 20/e p 2471, Ghai 8/e 619-620
•• Osteosarcoma is most common primary malignant bone tumor in children followed by Ewing sarcoma.
•• In children younger than 10 yrs of age, Ewing sarcoma is more common than osteosarcoma.
•• Both tumor types are most likely to occur in the second decade of life, i.e. in adolescent age group
68. a. Neuroblastoma  Ref: Nelson's 20/e p 2417, Ghai 8/e 616
Both Neuroblastoma & Wilms' tumor can present as a solid abdominal mass with calcifications.
As the child in the given question is an infant, Neuroblastoma is the best answer.
69. b. HL  Ref: Ghai 8/e 616-617
Discussing the options one by one,
a. ALL CNS infiltration is common in ALL, so diagnostic lumbar puncture & CSF cytopathology to look for blasts is done; LP is also
done to administer intrathecal methotrexate
b. HL Usually Lumbar puncture is not required
c. NHL Hematogenous spread may cause CNS spread, so LP done
d. AML CNS spread though rare can be seen, which will require lumbar puncture for diagnosis as well as treatment

70. a. Histiocytosis X  Ref: Nelson's 20/e p 2484-2489, Ghai 8/e 620-621; Refer pretext of chapter 12;
The given clinical picture prints to a diagnosis Langerhans cell; Histiocytosis of previously called Histiocytosis X.

71. c. Ameloblastoma  Ref: Nelson's 20/e p 2471-2475, Ghai 8/e 618
72. b. Leydig cell tumor  Ref: Nelson's 20/e p 2471-2478, Ghai 8/e 618
Testicular Tumors

Germ cell tumors (95%) Non-germinal tumors (5%)


a. Seminomatous b. Non-seminomatous •• Sertoli cell tumor
•• Seminoma •• Embryonal carcinoma •• Leydig cell tumor
•• Spermatocytic seminoma •• Choriocarcinoma
•• Yolk sac tumor
•• Endodermal sinus tumor
•• Teratoma

•• Most malignant tumors of the gonads in children are Germ cell tumors
•• Testicular GCTs occur predominantly before age 4 yr and after puberty.
73. e. Pleomorphic rhabdomyosarcoma  Ref: Nelson's 20/e p 2471
Pleomorphic rhabdomyosarcoma arises in the deep soft tissue of adults; It is a rare tumor with poor prognosis.
74. c. CD 15/34 both seen in same cell  Ref: Nelson's 20/e p 2442-2443
Acute Promyelocytic Leukemia is negative for CD34
75. b. Hemangioma  Ref: Nelson's 20/e p 3124, Ghai 8/e 618
368 Section 3: Systemic Pediatrics

76. a. Bone  Ref: Nelson's 20/e p 3124, Ghai 8/e 618


Review of Pediatrics and Neonatology

77. a. Wilms' tumor  Ref: Ghai 8/e 616


78. b. Duplication cyst of esophagus, c. Lymphoma & d. Neuroblastoma  Ref: Robbins 9th/pg 721; 8th/pg 731
Important causes of Mediastinal Mass

Superior mediastinum Anterior mediastinum Posterior mediastinum Middle mediastinum


•• Lymphoma •• Thymoma (MC) •• Neurogenic tumors (MC) •• Bronchogenic cyst
•• Thymoma •• Teratoma (schwannoma, neurofibroma) •• Pericardial cyst
•• Thyroid lesions •• Lymphoma •• Lymphoma •• Lymphoma
•• Metastatic carcinoma •• Thyroid lesions •• Gastroenteric hernia
•• Parathyroid tumors •• Parathyroid tumors •• Duplication cyst of esophagus
Chapter 16
Pediatric Gastrointestinology
 A. ESOPHAGEAL DISORDERS

1. TRACHEOESOPHAGEAL FISTULA (TEF): Refer pretext of chapter on 'Pediatric


Surgical disorders'

2. GASTROESOPHAGEAL REFLUX DISEASE (GERD)


•• Gastroesophageal reflux (GER) signifies retrograde movement of gastric contents across the
lower esophageal sphincter (LES) into the esophagus, which occurs physiologically.
•• GER in Infancy resolves in up to 88% by 12 mo & in nearly all by 24 mo.
•• Pathologic GERD refers to clinical manifestations because of frequent or persistent GER,
producing esophagitis-related symptoms, respiratory symptoms or nutritional effects.

Clinical Features of GERD M

In infants In older children High Yield Points


•• Esophagitis (irritability, arching, choking) •• Food refusal •• GERD is the most common esopha-
•• Failure to thrive •• Sandifer syndrome with neck contortions geal disorder in children
•• Obstructive apnea or as stridor (arching, turning of head) •• Presence of Respiratory symptoms
point towards pathological GERD
•• Otitis media, sinusitis •• Abdominal and chest pain
•• Stricture esophagus & Upper GI
•• Lymphoid hyperplasia, hoarseness •• Airway manifestations related to asthma bleeding are complications due to
severe GERD.
Diagnosis of GERD
•• Contrast (usually barium) esophagogram: poor sensitivity & specificity for GERD, but gives
clue about achalasia, esophageal strictures, hiatal hernia
•• Distal esophageal pH monitoring: Sensitive documentation of acidic reflux episodes
•• Endoscopy allows diagnosis of erosive esophagitis & complications such as strictures
•• Radionucleotide scintigraphy using technetium (GER scan) can demonstrate GERD
•• Multichannel intraluminal impedance, done for understanding esophageal function

Management of GERD
•• Dietary measures for infants include thickening of feeds
•• Positioning measures: prone position & upright position can be used to minimize reflux.
•• Pharmacotherapy includes Antacids, Histamine-2 receptor antagonists (Famotidine,
nizatidine, and ranitidine) & PPIs (omeprazole, lansoprazole, rabeprazole)
•• Surgery, (fundoplication), for intractable GERD with refractory esophagitis.

 B. DISORDERS OF STOMACH
HYPERTROPHIC PYLORIC STENOSIS: Refer chapter on 'Pediatric surgical disorders'

 C. DISORDERS OF INTESTINE
1. HIRSCHSPRUNG DISEASE
Refer chapter on 'Pediatric surgical disorders'
370 Section 3: Systemic Pediatrics

2. LACTASE DEFICIENCY
Review of Pediatrics and Neonatology

High Yield Points


•• Most common enzyme deficiency Types •• Congenital: Caused by mutation of lactase gene in chromosome 2
in humans is lactase deficiency •• Acquired: Due to downregulation of lactase after weaning from breast milk
•• Secondary lactase deficiency is •• Secondary: Due to viral intestinal infection, inflammatory disease, radiation & drugs
much more common than primary
lactase deficiency Presentation •• Diarrhea, bloating, abdominal pain & vomiting after ingestion of milk/milk products
•• Perianal excoriation because of acidic stools

Diagnosis •• Reducing substances and organic acids are seen in stool; lactase breath test
•• Confirmed by improvement of symptoms on exclusion of milk products
•• Demonstration of decreased lactose activity in small intestine (biopsy)

Treatment •• Avoid milk & milk products like ice cream; but yoghurt may be given

Mnemonic  M
3. CELIAC DISEASE

Important conditions asso­ciated with (Non-tropical sprue, Gluten sensitive enteropathy) M

celiac disease: “Doctor Don’t WAIT •• T-cell mediated autoimmune disorder with intolerance to wheat, rye, barley, containing
today” gluten.
•• Dermatitis herpetiformis
•• Down syndrome Etiology
•• William’s syndrome
•• Addison’s disease •• Environmental factor: Gliadin, a component of gluten, present in wheat, barley
•• IgA deficiency •• Immunological factors:
•• Turner syndrome –– Antiendomysial antibody (most sensitive and most specific)Q
•• Type I Diabetes mellitus –– Anti-tissue transglutaminase antibodies (Anti-ttG)
–– Antideaminated gliadin peptide antibody (Anti-DGP).
•• Genetic factors: associated with HLADQ2 &/or HLADQ8 haplotypesQ.

Clinical Features
System Manifestation (Possible) Cause
GIT Diarrhea, Abdominal distension, Vomiting, Atrophy of small bowel mucosa,
Anorexia, Failure to thrive, Aphthous stomatitis malabsorption

Hematologic Anemia Iron malabsorption

Skeletal Rickets, Osteoporosis, enamel hypoplasia of teeth Calcium/Vit D malabsorption

Question 1 Muscular Atrophy Malnutrition


A 2-year-old child presen­ted with Neurologic Peripheral neuropathy, epilepsy, irritability Thiamine/Vit B12 deficiency
recurrent pain abdomen, failure to
thrive and iron deficiency anemia Endocrine Short stature, secondary hyperparathyroidism Malnutrition, calcium/Vit D dif.
refractory to Iron therapy. His anti Dermatologic Dermatitis herpetiformis, alopecia areata, Autoimmunity
TTG anti-body titres were 300 U/L. erythema nodosum
An upper GI endoscopy was done
and duodenal biopsy was taken
which showed the following. What Diagnosis
is the diagnosis?
According to European society for pediatric Gastroenterology, hepatology & Nutrition
(ESPGHAN) current criteria: 2 requirements mandatory for diagnosis of celiac disease are:
•• Finding of Villous atrophy with hyperplasia of the crypts and abnormal surface epithelium,
while the patient is eating adequate amounts of gluten
•• A full clinical remission after gluten withdrawal from diet
a. Crohn disease
Gluten challenge is considered in situations where there is doubt about the initial diagnosis, e.g.
b. Hirschsprung disease
c. Celiac disease when the biopsy specimen was inadequate or atypical of celiac disease.
d. Congenital lactase deficiency
Treatment
•• Lifelong gluten free diet, (i.e. Wheat, rye, barley, oat free diet)
•• Rice, Soyabean, Potato and Corn can be given.
Chapter 16: Pediatric Gastrointestinology 371

Complications of Celiac Disease

Section 3: Systemic Pediatrics


M o s t C o mm o n
• Intestinal T-cell Lymphoma •  Myopathy/Neuropathy •• Most common extraintestinal mani­
festation of celiac disease is iron
•• Intestinal ulceration •  HyposplenismQ
deficiency anemia, unresponsive to
•• Refractory sprue •  Osteoporosis iron therapy
4. INFLAMMATORY BOWEL DISEASE (IBD)
Includes disorders of chronic intestinal inflammation: Crohn disease & ulcerative colitis.
Bimodal distribution: Early onset at 10–20 yrs & 2nd peak at 50–80 years.

ULCERATIVE COLITIS M

Definition: Chronic, recurrent disease characterized by diffuse mucosal inflammation involving


the rectum and large intestine.

Involvement: Upper gastrointestinal tract is spared.


•• 40–50% patients: Most commonly rectum and recto–sigmoid involved;
•• 30–40%: Involve colon also, while 20% have total colitis.

Clinical Features: Depends on the site and severity (remission & relapses occur)
•• Rectum: Tenesmus, fecal urgency and passage of fresh blood or blood stained mucus
•• Sigmoid Colon: Slowness of proximal transit leading to constipation
•• Colon: Bloody diarrhea + Pus along with abdominal pain and tenderness

Diagnosis
High Yield Points
Hematology/biochemistry Serology
•• Extraintestinal manifestations are
•• Anemia & Leukocytosis •• P-ANCA (6–7%)
more common in Ulcerative colitis
•• Raised ESR, CRP •• Anti–Saccharomyces cerevisiae antibody (10–15%)
•• Include pyoderma gangrenosum,
•• Increased platelet count •• Anti–goblet cell antibody (40%)
sclerosing cholangitis, chronic active
•• Low serum albumin •• Anti–colon & Pancreatic autoantibody hepatitis, and ankylosing spondylitis
•• X–ray abdomen – Loss of haustral markings or marked dilation with toxic megacolon
•• Barium enema
–– Earliest change is fine mucosal granularityQ
–– Deep ulceration appear as collar button; ulcers or polyps may be seen
–– Loss of haustration in long standing disease; colon becomes shortened and narrowed
•• Ultrasound – Mild mural thickening, increased perirectal & presacral fat.
•• Sigmoidoscopy & Colonoscopy to determine extent of disease & assess disease activity.

Complications of Ulcerative Colitis


•• Massive hemorrhage
•• Toxic megacolon with perforation
•• Colonic epithelial dysplasia and carcinoma (risk is increased with duration and extent of
disease)

Treatment
Mild Moderate-Severe Fulminant Maintenance
Sulphasalazine, 5-ASA, Glucocorticoid, 5ASA, 6-MP, Azathioprine
Olsalazine Glucocorticoids Cyclosporin

Indications of Surgery in Ulce­rative Colitis

•• Perforation •  Toxic megacolon


•• Massive hemorrhage •  Failure to respond to medical treatment

CROHN'S DISEASE M

Definition: Recurrent chronic disease with transmural involvement anywhere from mouth
to anus.
372 Section 3: Systemic Pediatrics

Clinical Features (Depends on Site of Involvement)


Review of Pediatrics and Neonatology

Question 2
A 5-year-old boy presented with Ileocolitis Jejunoileitis
abdominal distension, fever, bil- •• Recurrent right lower quadrant colicky pain & •• Malabsorption, steatorrhea
ious vomiting and constipation. diarrhea with usually a palpable mass •• Nutritional deficiency, weight loss
He also has a history of recurrent •• Diarrhea followed by chronic bowel obstruction •• Low grade fever
episodes of severe pain abdomen, Colitis and perianal disease Gastroduodenal disease
tenesmus and blood in stools. Ex- •• Fever, hematochezia, diarrhea •• Nausea
ploratory laparotomy was done •• Stricture, Bacterial overgrowth •• Vomiting
and a portion of intestine had to •• Fistula into stomach or duodenum •• Epigastric pain
be resected, which is seen below. •• Rectovaginal fistula, peri anal fistula •• Chronic gastric outlet obstruction
What is the probable diagnosis?
Diagnosis M

Laboratory investigations Endoscopic features Radiological features


•• Anemia, Leucocytosis •• Rectal sparingQ •• Thickened folds
•• Increased ESR, CRP •• Aphthous ulceration •• Cobble stoning of small
•• ASCA antibodies (60– •• Fistula intestine
70%)Q •• Skip lesionsQ •• Stricture, fistula, String sign
a. Crohn disease •• Anti-colon & anti goblet Ab •• CT findings – mural thickening,
b. Ulcerative colitis •• P-ANCA (5–10%) adenopathy
c. Celiac disease
d. Intestinal lymphangiectasia
Treatment: For Active disease: 5-Amino-salicylic Acid (5-ASA), Glucocorticoids, A tathioprime,
6- Mercapto­purine, Cyclosporine, Infliximab.
High Yield Points
•• Rectum is often spared in Crohn's High Yield Points
diseases
•• Most common site of inflammation •• Rectum is often spared in Crohn's diseases
is terminal ileum •• Most common site of inflammation is terminal ileum
•• Most people have involvement of •• Most people have involvement of both small and large intestine (40–55%)
both small and large intestine (40– •• Non-caseating granuloma in all layers of intestinal wall from mucosa to serosa
55%)
•• Non-caseating granuloma in all Complications of Crohn's disease
layers of intestinal wall from mucosa •• Fistula formation •  Malabsorption
to serosa •• Perforation peritonitis •  Carcinoma especially of colon
•• Intestinal obstruction
High Yield Points
Difference between Ulcerative Colitis and Crohn’s Disease M
Indications for Surgery in Crohn’s
Disease Features Ulcerative colitis Crohn’s disease
•• Stricture, Obstruction Systemic symptoms Rare Present
•• Refractory fistula Perianal disease & fistula Absent Present
•• Abscess.
Rectal sparing Rare Present
Continuous disease Present Rare (Skip lesions seen)
Question 3
Cobble stone appearance Absent Present
A 10-year-child presents with re- Granuloma & stricture Absent Present
current episodes of blood in stool.
On examination his lips shows the ANCA positivity 60–70% 5–10%
following. He may be suffering Radiology sign Lead pipe appearance String sign of Kantor
with? Crypt abscesses Common Less common

5. INTUSSUSCEPTION: Refer chapter on 'Pediatric Surgical disorders'

6. PEUTZ JEGHERS SYNDROME (PJS)


•• Autosomal dominant disorder with mutations in LKB1/STK11; (chr 19p13)
•• Mucocutaneous pigmentation & extensive GI hamartomatous polyposis seen
•• Polyps are primarily found in small intestine (jejunum > ileum > duodenum), but may also
a. Intussusception be colonic or gastric, leading to bleeding & abdominal cramping.
b. Meckel’s diverticulum •• Diagnostic criteria: Histologically proven hamartomatous polyps with 2 of 3 conditions:
c. Peutz Jegher syndrome (i) positive family history with an autosomal dominant inheritance pattern, (ii) mucocutaneous
d. Crohn disease hyperpigmentation & iii) small bowel polyposis.
Chapter 16: Pediatric Gastrointestinology 373

•• Patients with PJS have increased risk of Colorectal, breast, and reproductive tumors.

Section 3: Systemic Pediatrics


•• Lifetime cancer risk has been reported from 47-93%.
•• GI surveillance should begin in childhood (by age 8 yr or when symptoms occur) with upper
& lower endoscopy.

 DIARRHEA
Definition
•• Passage of 3 or more liquid or watery stools in a day
•• Recent change in consistency of stools is more important rather than the number of stools
•• Also defined as an increase in daily stool weight > 200 gm (older children) or > 10 gm/kg/
day (up to 3 year), along with abnormal increase in stool liquidity & frequency.

Infectious Diarrhea: Pathophysiologic Mechanism and Causes


Mechanism Location Illness Stool examination Examples
Non-inflammatory Proximal Watery No fecal leukocytes V. cholera, E. coli, C. perfringers,
(enterotoxin or small diarrhea B. cereus, S. aureus. Giardia,
adherence) bowel Rotavirus, Norwalk virus,
Microsporidia, Cyclospora
Inflammatory Colon Dysentry Fecal leukocytes present Shigella, Salmonella
(invasion, E. coli, C. jejuni, E. histolytica
cytotoxin) V. parahemolyticus

Assessment of Severity of Dehydration M

Some High Yield Points


Parameters No dehydration dehydration Severe dehydration
•• Important consequences of diar-
Look at Sensorium Well alert “Restless, irritable” “Iethargic”, floppy rhea in children are malnutrition &
Eyes Normal Sunken Very sunken and dry dehydration
•• Intestinal infections are the most
Tears Present Absent Absent common cause of acute diarrhea
Mouth & Tongue Moist Dry Very dry •• Rotavirus is the leading cause of
severe diarrhea worldwide
Thirst Drinks normally, “Thirsty, drinks “Drinks poorly/not able to
•• In India, Rotavirus & Enterotoxi-
not thirsty eagerly” drink”
genic E. coli cause majority of diar­
Feel Skin pinch Goes back quickly “Goes back slowly” “Goes back very slowly” rhea in children
•• Rice watery stools are seen in
Decide No signs of dehydration If the patient has 2 or more signs, including at least
Cholera
1 “sign”

Treatment
Composition of oral rehydration solutions M

Components of ReSoMal
Composition Reduced osmolarity ORS* Standard WHO-ORS
•• Glucose 125 mmol/L
Glucose (mmol/L) 75 111 •• Sodium 45 mmol/L
Sodium (mmol/L) 75 90 •• Potassium 40 mmol/L
Potassium (mmol/L) 20 20 •• Chloride 70 mmol/L
Chloride (mmol/L) 65 80 •• Magnesium 3 mmol/L
Citrate (mmol/L) 10 10 •• Zinc 0.3 mmol/L
•• Copper 0.045 mmol/L
Osmolarity (mmol/L) 245 311
•• Citrate 7 mmol/L

Plan A: Patients without Physical signs of Dehydration–Oral Rehydration M

Age Amount of ORS fluids to give after each loose stool


< 24 months 50–100 mL

2–10 years 100–200 mL

> 10 years As much as wants


374 Section 3: Systemic Pediatrics

Plan B: Patients with Physical Signs of Dehydration—3 components:


Review of Pediatrics and Neonatology

i. Deficit replacement/Rehydration therapy: Give 75 mL/kg of ORS in the first 4 hrs


ii. Replacement of ongoing losses: ORS 10–20 mL/kg/stool
iii. Provision of normal daily fluid requirements: Offer plain water in between loose stools
& Breastfeed infants.

Treatment Plan C: Children with Severe Dehydration M

Age First 30 mL/kg Next 70 mL/kg Total 100 mL/kg


< 1 year over 1 hr over 5 hrs over 6 hrs

1–5 year over ½ hr over 2½ hrs over 3 hrs

High Yield Points


•• Best IV fluid for treatment of dehy­dration due to diarrhea is Ringer’s lactate > Normal saline (0.9. % NaCl)
•• Dextrose is not effective

Recommendations on Dietary Management M

•• Breastfeeding should be continued uninterrupted


•• In non-breastfed infant, Cow or Buffalo milk can be given undiluted
•• Routine lactose free feeding is not required during acute diarrhea. It maybe required in infants
in whom diarrhea persists >8–10 days with ≥1% reducing substance in stools.
High Yield Points
Indications of using Antibiotics in
Zinc in acute diarrhea
Diarrhea •• WHO and IAP (Indian Academy of Pediatrics) recommend Zinc supple­mentation at a dose of
•• In malnourished or premature
20 mg/day for Infants > 6 months and 10 mg/day for infants < 6 months for 10–14 days
infants presuming sepsis
•• In well nourished infants, antibiotics
should be considered only if High Yield Points
±± Fever or hypothermia
±± Abdominal distension •• Antimotility agents should not be used in children with diarrhea
±± Fast breathing •• There is no large scale evidence about efficacy & safety of anti-secretory agents like racecadotril
±± Lethargy •• No consensus about utility of probiotics in children with diarrhea
•• Suspected infection with Shigella, V.
cholerae, Giardia, E. histolytica
PERSISTENT DIARRHEA
Definition: Diarrhea that starts as an acute episode, and lasts for at least 14 days.
Causes
•• Persistent infection with one or more enteric pathogens
•• Malabsorption especially carbohydrates and fat; infrequently, protein intolerance.

Management
•• Assess the child for dehydration and give fluids according to treatment plan A, B, or C
•• The mainstay of therapy of persistent diarrhea is dietary management.
High Yield Points
Dietary Algorithm for Treatment of Persistent Diarrhea
Antibiotics effective for cholera
are single dose azithromycin & Initial Diet ‘A’: Reduced lactose diet, e.g. milk rice gruel, mil sooji gruel, rice with curd, dalia
Doxycycline
Second Diet ‘B’: (Lactose free diet with reduced starch)
•• About 65–70% of children improve on the initial diet ‘A’; Diet B for remaining
•• It is milk (lactose) free & provides carbohydrates in the form of mixtures of cereals & glucose, because some
children may have impaired digestion of starch, and disaccharides other than lactose

Third Diet ‘C’: (Monosaccharide – based diet)


•• 80–85% of patients recover on initial diet A or B, Rest started on diet C
•• It contains only glucose & a protein source as egg white or chicken or protein hydrolysates
Chapter 16: Pediatric Gastrointestinology 375

CHRONIC DIARRHEA

Section 3: Systemic Pediatrics


Diarrhea episode which has an insidious onset and is mostly a manifestation of malabsorption.

1. Due to Impaired Intraluminal Digestion


Impaired digestion Conditions
All nutrients Cystic fibrosis, other pancreatic exocrine deficiencies
Fat Isolated lipase deficiency, bile duct atresia, interrupted enterohepatic circulation
(e.g. Ileal resection, Crohn’s disease)
Protein Congenital trypsinogen deficiency, congenital enterokinase deficiency

2. Due to Intestinal Malabsorption


Mucosal changes Condition
Total villious atrophy Celiac disease
Partial villious atrophy Food protein sensitivity, Giardia infestation; Immunodeficiency, malnutrition
Fat filled enterocytes Abetalipoproteinemia
Villi distorted Lymphangiectasia (ectatic lymphatics distort villi)

Answer Keys for Image-Based Questions

Answers Explanations / Identifying features


1. Ans. c. Celiac disease Intestinal biopsy showing villous atrophy, crypt hyperplasia, intraepithelial lymphocytic infiltration
in the given case scenario, suggest a diagnosis of Celiac disease

2. Ans. a. Crohn disease Multiple skip lesions with areas of intervening normal mucosa, along with history of abdominal
distension, fever, bilious vomiting & blood in stools, suggest a diagnosis of Crohn ds
3. Ans c. Peutz Jegher syndrome Macular pigmented lesions around lips and oral mucosa, with a history of GI bleeds suggests Peutz
Jegher syndrome
376 Section 3: Systemic Pediatrics
Review of Pediatrics and Neonatology

Questions
 ESOPHAGEAL DISORDERS 10. A newborn has dribbling after feeds. He has respiratory
distress and froths at the mouth. Diagnosis is:
1. The Gold standard investigation for Gastro-esopha­geal a. Tracheoesophageal fistula (AIPGMEE 2001)
reflux disease is: (MAHA PGM CET 2016) b. Tetralogy of Fallot
a. Endoscopy c. Respiratory distress syndrome
b. 24 hour pH recording d. None of the above
c. Oesophageal manometry
d. Measurement of length of lower oesophageal sphincter  DISORDERS OF INTESTINE: DIARRHEA
2. Which one of the following life-threatening conge­nital
anomalies in the newborn presents with polyhydra­mnios, 11. Low osmolarity ORS has (as compared to WHO-ORS):
aspiration pneumonia, excessive salivation and difficulty a. Low glucose and high sodium (FMGE June 2018)
in passing a nasogastric tube? M  (APPG 2016) b. Low glucose and low sodium
a. Gastroschisis b. Diaphragmatic hernia c. Low glucose and low potassium
c. Tracheo-esophageal fistula c. Low glucose and high potassium
d. Choanal atresia 12. Baby with diarrhea presented with restlessness but was
3. A coin is loaded in the esophagus of an asymptomatic 3 able to drink water. Skin turgor goes back in 2 sec. Best
years female child, Treatment is: (JIPMER 2014) management is: (NEET pattern Jan 2018)
a. Endoscopic removal within 24 hours a. Plan A b. Plan B
b. Immediate endoscopic removal c. Plan C d. Plan D
c. Wait for 48 hours
13. A 5-year-old child comes to hospital with history of
d. Dislodge the coin to the stomach by inserting Ryle's tube
loose stools but no history of fever or blood in stools.
4. The commonest type of Congenital atresia is? Mother says he is irritable and drinks water if given. On
 (APPG 2014)
examination eyes sunken and on Skin pinch test—the
a. Proximal end blind, distal end communicating with
skin retracted within two seconds but not immediately.
trachea
What will be the appropriate management?
b. Distal end blind, proximal end communicating with
 (AIIMS May 2017)
trachea
c. Proximal and distal ends open and communicating with a. Administer the first dose of IV antibiotic and immediately
trachea refer to higher centre
d. Both ends blind b. Give oral antibiotics and ORS and ask the mother to
5. Gasless abdomen on X-ray is seen in what type of continue the same and visit again next day
trachea-esophageal fistula  (TN PGMEE 2013) c. Consider severe dehydration start IV fluids, IV antibiotics
a. Isolated TEF b. EA with proximal TEF and refer to higher centre
c. EA with distal TEF d. EA with double TEF d. ORS with zinc for 14 days
6. A one-day-old baby presents with excessive frothing 14. A 10-year-old child presents with fever since 24 hours.
from mouth and baby was cyanosed: (AIIMS Nov 2013) History reveals 3 episodes of chest infection and passage
a. Esophageal atresia b. Diaphragmatic hernia of bulky, foul smelling stools. The most probable
c. Congenital lung cyst d. Lung hypoplasia diagnosis is? (Recent Question 2017)
7. Which of the following is the earliest indicator of a. Cystic fibrosis
patholo­gical gastroesophageal reflux in infants (GERD)? b. Maple syrup urine disease
 (AIPGMEE 2011) c. Bilirubin conjugation defect
a. Respiratory symptoms b. Postprandial regurgitation d. Crigler-Najjar syndrome
c. Upper GI bleed d. Stricture esophagus 15. The most common finding in children with severe viral
8. Esophageal atresia may occur as a part of VACTER group enteritis: (Recent Question 2017)
of anomalies. What does ‘TE' stand for?
a. Isotonic dehydration with Acidosis
a. Tetralogy of Fallot (DNB Dec 2010) b. Isotonic dehydration with Alkalosis
b. Thoracic empyema
c. Hypotonic dehydration with Acidosis
c. Tracheo-esophageal fistula
d. Hypotonic dehydration with Alkalosis
d. Talipes equinovarus
9. A newborn baby had normal APGAR score at birth and 16. A 6-year-old drowsy child came in emergency with
developed excessive frothing and choking on attempted history of vomiting, loose motion for 3 days. On
feeds. The investigation of choice is: M  (AIIMS May 03) examination he had sunken eyes, hypothermia, skin on
pinching was taking 5 seconds to revert. What is your
a. Esophagoscopy b. Bronchoscopy
diagnosis? (Recent Question 2016)
c. MRI chest
d. X-ray chest and abdomen with the red rubber catheter a. No dehydration b. Mild dehydration
passed per orally into esophagus c. Some dehydration d. Severe dehydration
Chapter 16: Pediatric Gastrointestinology 377

30. The most common bacterial cause for diarrhea in

Questions
17. Most common cause of Neonatal diarrhea worldwide is?
M (AIIMS Nov 2015) children in India is:  (AIPGMEE 2012)
a. Norwalk virus b. Rotavirus a. Enterotoxigenic E. coli (ETEC)
c. Adenovirus d. Cholera b. Enteropathogenic E. coli (EPEC)
c. Enterohemorrhagic E. coli (EHEC)
18. A child presents with malnutrition and persistent
d. V. cholerae
diarrhea. Apart from antibiotics what would you like to
add in your prescription? (WB PGMEE 2015) 31. A 2-year-old child presents with loose stools. On
examination she has signs of severe dehydration. If her
a. Zinc, Vit A, Iron b. Zinc, Iron
weight is 10 kg, what would be the volume of fluid to be
c. Zinc, Vit A d. B complex, Zinc
given in the first 4 hours?  (JIPMER 2012)
19. Fluid of choice in an infant with diarrhea is: M 
a. 1000 mL b. 100 mL
 (AIIMS May 2015)
c. 200 mL d. 500 mL
a. ORS b. Dextrose
32. Most common bacteria causing diarrhea in children in
c. Salt water d. Sugar water
India is: (COMEDK 2011)
20. Fasting provides relief in? (Recent Question 2015)
a. Enterotoxigenic E. coli
a. Osmotic diarrhea b. Secretory diarrhea b. Enteroinvasive E. coli
c. Infective diarrhea d. Dysentery c. Enterohemorrhagie E. coli
21. An infant presents with 15-20 watery stools since 9 d. Enteropathogenic E. coli
days. Along with Zinc, what else should be given to the 33. Amount of ORS given in a 4 kg child with diarrhea with
patient? M  (AIIMS Nov 2015) some dehydration:  (WBPG 2011)
a. ORS alone b. ORS and low lactose feed a. 200 mL b. 300 mL
c. ORS with antibiotics d. ORS with probiotics c. 400 mL d. 800 mL
22. Low osmolar ORS composition? (WBPG 2014) 34. Concentration of K+ in ORS is ____ mEq/L: M 
a. Na 90 + 311 mOsmol b. Na 75 + 245 mOsmol a. 20 b. 40 (WBPG 2010)
c. Na 60 + 245 mOsmol d. Na 60 + 240 mOsmol c. 90 d. 10
23. Molar ratio of Sodium and Glucose in WHO ORS is: 35. A two-year-old child presents with persistent diarrhea,
 (JIPMER 2014) acidic stools and presence of one percent of reducing
a. 1 : 1 b. 1 : 2 substance in the fresh stools. What is the most probable
c. 1 : 4 d. 2 : 1 diagnosis? (UPSC 07)
24. Chronic constipation in children is seen in all except: a. Cystic fibrosis
 (Recent Question 2014) b. Lactose intolerance
a. Hirschsprung disease b. Jejunal polyp c. Rotavirus induced diarrhea
c. Hypothyroidism d. Stricture d. Intestinal tuberculosis
25. Enzyme deficiency that occurs after viral gastroente­ritis: 36. All are true regarding osmotic diarrhea except:
 (TN PGMEE 2013)  (JIPMER 2007)
a. Lipase b. Amylase a. Due to highly osmotic substances in lumen
c. Lactase d. Enterokinase b. Fasting may improve symptoms
26. Profuse watery diarrhea in an immuno-compromised c. Stool reducing substances are present
child is due to:  (Recent Question 2013) d. Can be mediated by bacterial toxins
a. Cryptosporidium b. Ameba 37. One of the intestinal enzymes that is generally deficient
c. Giardia d. Lactose intolerance in children following an attack of severe infectious
27. A 4-month-old child has 10 episodes of vomiting and enteritis is: (AIPGMEE 05)
2–3 episodes of loose stools and crying since the last 24 a. Lactase b. Trypsin
hours. Best line of management will be: c. Lipase d. Amylase
 (Recent Question 2013) 38. A 6-month-old infant presents to the diarrhea clinic
a. Intravenous fluids b. ORS unit with some dehydration. The most likely organism
c. Breastfeeding only d. Antibiotics causing diarrhea is: (AIIMS Nov 2003)
28. A child presents with acute diarrhea with signs of a. E. histolytica b. Rotavirus
dehydration; On investigation, Na+ 125 mEq/L, K+ 3 c. Giardia lamblia d. Shigella
mEq/L, HCO3- 16 mEq/L and pH is 7.23. IV Fluid to be 39. True about diarrhea: (PGI Dec 2003)
given is?  (TN PGMEE 2013) a. Defined as passage of 2–3 formed stools/day
a. Normal saline b. 3% saline b. Blood mixed with mucus stool is defined as dysentery
c. N/3 saline + 10% dextrose c. Rotavirus is the most common organism in children
d. N/3 saline + 5% dextrose d. Persistent diarrhea is defined if duration is more than 21 days
29. In a 12-month-old child with Diarrhea, the dose of zinc e. Enterotoxigenic E. coli is the most common bacteria
is: M  (Recent Question 2012) causing diarrhea in children
a. 1 mg/day for 10–14 days 40. Intractable diarrhea in children is caused by all except:
b. 10 mg/day for 10–14 days  (PGI June 2000)
c. 15 mg/day for 10–14 days a. Cystic fibrosis b. Giardiasis
d. 20 mg/day for 10–14 days c. Secreting tumors d. Milk allergy
378 Section 3: Systemic Pediatrics

 CELIAC DISEASE 51. An eight-year-old boy had abdominal pain, fever with
Review of Pediatrics and Neonatology

bloody diarrhea for 18 months. His height is 100 cm and


41. The following tests can be used for diagnosing celiac weight is 14.5 kg. Stool culture was negative for known
disease except: (UPSC CMS 2015) enteropathogens. The sigmoidoscopy was normal. During
a. Anti-gliadin antibodies the same period child had an episode of renal colic and
b. Anti-nuclear antibodies passed urinary gravel. The Mantoux test was 5 x 5 cm. The
c. Anti-endomysial antibodies most probable diagnosis is: (AIPGMEE 2003)
d. Anti-tissue transglutaminase antibodies a. Ulcerative colitis b. Crohn's disease
42. The IgA anti TTG (Tissue Transglutaminase) is a sensi­tive c. Intestinal tuberculosis d. Strongyloidiasis
marker for diagnosis of: M  (MAHA PGM CET 2014)
 OTHER DISORDERS OF INTESTINE
a. Celiac Disease
b. Eosinophilic Gastroenteritis 52. A child presents to the emergency with a history of
c. Primary Immunodeficiency ingestion of a button battery, 6 hrs ago. On X-ray it was
d. Crohn's Disease found in the stomach or duodenum. What is the next
43. The following cereals should be avoided in patient with step? (AIIMS May 2018)
celiac disease, except: M  (AIIMS Nov 2003) a. Endoscopic removal of battery
a. Wheat b. Barley b. Wait and watch
c. Maize d. Rye c. Repeat X-ray after 5 days
44. Gluten sensitive enteropathy is most strongly associated d. Immediate laparotomy
with: (AIPGMEE 2003) 53. Exocrine pancreatic insufficiency is seen in: 
a. HLA-DQ2 b. HLA-DR4 a. Shwachman-Diamond syndrome (JIPMER Nov 2017)
c. HLA-DQ3 d. Blood group ‘B' b. Diamond-Blackfan syndrome
45. The histological features of celiac disease include all of c. Rubinstein-Taybi syndrome
the following except: (AIPGMEE 02) d. Seckel syndrome
a. Crypt hyperplasia 54. False statement on FAP: (JIPMER Nov 2017)
b. Increase in thickness of the mucosa a. Most common genetic polyposis syndrome
c. Increase in intraepithelial lymphocytes b. >100 polyps may be present in colon
d. Increase in inflammatory cells in lamina propria c. Risk of developing colon cancer is 90–100%
46. A 2-yr-old boy presented with abdominal distension, d. Colon cancer usually develops around 20 years of age
chronic diarrhea, severe anemia and failure to thrive. 55. All of the following are true about Peutz-Jeghers
Which of the following is the investigation of choice? M syndrome except: (FMGE pattern 2017)
a. Anti milk protein antibody (Recent Question 2015) a. It is the most common genetic polyposis syndrome
b. Anti endomysial antibody b. Oral mucocutaneous pigmentation seen
c. Antinuclear antibody c. Polyps most commonly involve small intestine
d. Intestinal biopsy d. Autosomal dominant inheritance
56. A mother comes with her 3 month child asking the
 INFLAMMATORY BOWEL DISEASE physician, if she can give cereals to her child. What
problem can this lead to her child? (AIIMS May 2016)
47. True about Crohn disease is: (PGI Nov 2017) a. Allergy due to the food content
a. Loss of haustration b. Growth failure b. Retarded oralmotor development
c. Cobblestone colon d. String sign of Kantor c. Contaminated food leading to reflux
e. Perianal fistula formation d. Risk of gastrointestinal infection
48. Ulcerative Colitis and Crohn's diseases are associated 57. Carbohydrates malabsorption is detected by:
with all except? (COMEDK 2016)  (Recent Question 2016)
a. Wiskott Aldrich Syndrome a. Breath hydrogen test b. Nitrogen breath test
b. Turner's syndrome c. Fecal nitrogen d. Rapid urease test
c. Glycogen storage diseases Type III 58. All the following are True regarding Familial Adenoma-
d. Hermansky Pudlak Syndrome tous Polyposis except? (APPG 2015)
49. In Crohn's disease all are true except: a. > 100 colorectal polyps are present
 (UPSC CMS 2015) b. Congenital Hypertrophy of Retinal Pigment Epithelium is
a. Can occur anywhere in GIT seen in up to 50% patients
b. Fistula formation is common c. It is due to a mutation of APC gene on chromosome 15
c. Crypt abscess are common d. It is an Autosomal Dominant disorder
d. Full thickness bowel involvement 59. A preterm baby on 30 mL/kg of enteral feeding developed
sudden severe abdominal distension with visible bowel
50. Which of the following are inflammatory bowel disease loops on day 6 of life. He also showed temperature
in children? (PGI June 05) instability and lethargy. X-ray of the abdomen showed
a. Celiac disease b. Tropical sprue portal venous gas. The staging of NEC is:
c. Regional ileitis (Crohn's disease)
a. Ib b. IIa (AIIMS May 2014)
d. Cystic fibrosis
c. IIb d. IIIa
e. Ulcerative colitis
Chapter 16: Pediatric Gastrointestinology 379

60. A neonate at 48 hours of birth presents with a history of 67. The commonest cause of vomiting in a one-month old-

Questions
non-passage of meconium. Next step in evaluation will infant is: M  (Karanataka PGMEE 2006)
be? (AIIMS Nov 2014) a. Pyloric stenosis b. Cardiac chalasia
a. Lower GI study b. Manometry c. Aerophagy d. Gastro-esophageal reflux
c. Sweat Chloride levels d. CFTR mutation analysis 68. A newborn with recurrent vomiting, cyanosis after each
61. True about Peutz-Jegher's syndrome: (PGI May 2013) feed is likely to be suffering from: M (APPG 05)
a. Pigmentary changes in skin and mucous membrane a. Tracheo-esophageal fistula
around mouth b. Tetralogy of Fallot
b. Adenomatous polyp in intestine c. Congenital hypertrophic pyloric stenosis
c. Most common of pattern inheritance is autosomal recessive d. ARDS
d. 20-30% premalignant 69. Acquired megacolon in children most commonly occurs
e. May presents as anemia in children due to: (SGPGI 05)
62. A neonate is suspected to be suffering from necrotizing a. Psychological problems
enterocolitis (NEC). On further examination and investi­ b. Bad bowel habit
gation, he is diagnosed to be Bell's stage I NEC. The c. Chagas disease
management of choice would be: (AIIMS May 2012) d. Hirschsprung's disease
a. Laparotomy and proceed 70. A 12-year-old girl has history of recurrent bulky stools
b. Insertion of bilateral pelvic drains and abdominal pain since 3 years of age. She has
c. Conservative management with IV fluids and antibiotics moderate pallor and her weight and height are below
d. Initial conservative management and laparotomy after the 3rd percentile. Which of the following is the most
24 hours appropriate investigation to make a specific diagnosis?
63. A 9-month-old girl is brought with PR bleed, vomiting,  (AIIMS Nov 2004)
mass in right lumbar region with masked liver dullness. a. Small intestinal biopsy
She is in shock like condition. Management should b. Barium studies
include: (PGI Nov 2010) c. 24 hours fecal fat estimation
a. Barium enema b. Saline reduction d. Urinary d-xylose test
c. Give O2 d. Nasogastric tube 71. Double bubble sign in children is seen in all except:
e. IV fluid  (PGI Dec 04)
64. Organic causes of constipation in infant are all except: M a. Ladds band b. Annular pancreas
 (PGI May 2010) c. Pancreatic pseudocyst d. Diaphragmatic hernia
a. High fibre diet b. Cystic fibrosis e. Duodenal atresia
c. Hypothyroidism d. Hyperthyroidism 72. Which one of the following is most suggestive of
65.
A newborn suffering from perforated necrotizing neonatal small bowel obstruction: (AIPGMEE 03)
enterocolitis is having very poor general condition. a. Generalized abdominal distension
He is currently stabilized on ventilator. Which of the b. Failure to pass meconium in the first 24 hours
following should be done in the management of this c. Bilious vomiting
patient? (AIPGMEE 2008) d. Refusal of feeds
a. Conservative treatment 73. A 6-month-old baby presents with history of bloody
b. Resection and anastomosis diarrhea of 2 days duration with abdominal distension
c. Stabilization with membrane oxygenator and defer and on examination the baby screams, diagnosis is: M 
surgery a. Intussusception b. HUS (PGI Dec 2003)
d. Peritoneal drainage by putting drains in the flanks c. Appendicitis d. Ac. enterocolitis
66. True about Peutz-Jegher syndrome (PGI Nov 09) 74. 14-year-old girl with history of postprandial abdominal
a. Most common site is small intestine pain (periumbilical), fever, weight loss since ten months.
b. Polypectomy and Colonoscopy is treatment of choice She also has episodes of passing blood in stools. What
c. Hamartomatous polyp may be the likely diagnosis?  (DNB June 2002)
d. Mucosal involvement present a. Chronic appendicitis b. Chronic pancreatitis
e. Have no malignant potential c. Crohn's disease d. Bulimia
380 Section 3: Systemic Pediatrics
Review of Pediatrics and Neonatology

Answers with Explanations


 ESOPHAGEAL DISORDERS
1. b. 24 hour pH recording  Ref: Nelson's 20/e p1787-1788, Ghai 8/e p 280
Extended esophageal pH monitoring of distal esophagus, provides a quantitative sensitive documentation of acidic reflux
episodes, the most important type of reflux episodes for pathologic reflux
2. c. Tracheo-esophageal fistula  Ref: Nelson's 20/e p 1783, Ghai 8/e p 176
•• Presence of polyhydramnios, aspiration pneumonia, excessive salivation and difficulty in passing a nasogastric tube all
point towards a diagnosis of tracheoesophageal fistula, which is usually accompanied by esophageal atresia.
Esophageal atresia (EA) and TEF (Tracheoesophageal fistula):

Important •• EA is the most frequent congenital anomaly of esophagus; 90% of EA babies have TEF
features •• Most common type is upper esophagus ends in a blind pouch and TEF connected to distal esophagus

Typical •• Maternal polyhydramnios is seen; New born presents with respiratory distress and excess
presentation oral frothing;
•• Newborn regurgitates all of its feeds; Saliva pours almost continuously from the mouth of the infant
•• Attacks of coughing and cyanosis occurs on feeding; Feeding tube cannot be passed more than 10 cm

Diagnosis •• Chest X-ray with a red rubber catheter passed per orally shows coiled rubber catheter in a lucent
proximal esophageal pouch
•• X-ray abdomen: Gas in the stomach is seen if there is communication between the lower part of the
esophagus and trachea, which occurs in the commonest variety of tracheo-esophageal fistula

3. a. Endoscopic removal within 24 hours  Ref: Nelson's 20/e p 1793-1794; Refer pretext of this chapter for details;
Asymptomatic blunt objects and coins can be observed for up to 24 hr anticipating passage into stomach; If they still remain
lodged in the esophagus, endoscopic removal is warranted.
4. a. Proximal end blind, distal end communicating with trachea  Ref: Nelson's 20/e p 783, Ghai 8/e p 176
5. b. EA with proximal TEF  Ref: Nelson's 20/e p 1783, Ghai 8/e p 176
•• In EA with proximal TEF, as there is no connection of the distal esophagus with the trachea or proximal esophagus, gasless
abdomen will be seen on X-ray.
6. a. Esophageal atresia  Ref: Nelson's 20/e p 1783, Ghai 8/e p 176

Congenital disease Mode of presentation


Cong diaphragmatic hernia New born with cyanosis, respiratory distress, shift of mediastinum and scaphoid abdomen
Bilateral choanal atresia New born with feeding dificulty, cyanosis on closing mouth, which disappears on crying
Pure esophageal atresia Neonate with frothing and bubbling at mouth and nose, with coughing, cyanosis and respiratory
distress; X-ray shows coiled feeding tube in esophageal pouch and absent gastric air shadow

7. a. Respiratory symptoms  Ref: Nelson's 20/e p 1787, Ghai 8/e p 280


•• Presence of Respiratory symptoms point towards pathological GERD
•• Stricture esophagus and Upper GI bleeding are complications due to severe GERD.
8. c. Tracheo-esophageal fistula  Ref: Nelson's 20/e p 1783, Ghai 8/e p 173
•• VACTERL association (also called VATER association) refers to the non-random co-occurrence of birth defects including
Vertebral anomalies, Anal atresia, Cardiac defects, Tracheoesophageal fistula and/or Esophageal atresia, Renal and Radial
anomalies and Limb defects.
9. d. X-ray chest and abdomen with the red rubber catheter passed per orally into esophagus  Ref: Nelson's 20/e p 1783
10. a. Tracheoesophageal fistula  Ref: Nelson's 20/e p 1783, Ghai 8/e p 176

 DISORDERS OF INTESTINE
DIARRHEA
11. b. Low glucose and low sodium  Ref: Nelson 20/e p 1869-1871
Chapter 16: Pediatric Gastrointestinology 381

Answers with Explanations


Composition of oral rehydration solutions
Composition Reduced-osmolarity ORS Standard WHO-ORS
Glucose (mmol/L) 75 111
Sodium (mmol/L) 75 90
Potassium (mmol/L) 20 20
Chloride (mmol/L) 65 80
Citrate (mmol/L) 10 10
Osmolarity (mmol/L) 245 311

12. b. Plan B  Ref: Nelson 20/e p 1869-1871


The given scenario indicates a child with acute gastroenteritis with some dehydration. So treatment plan B is to be used.
13. d. ORS with zinc for 14 days
This child has acute diarrhea with some dehydration. So, the treatment of choice is ORS with zinc for 14 days.
14. a. Cystic fibrosis
Recurrent chest infections & bulky, foul smelling stools point to a diagnosis of cystic fibrosis
15. a. Isotonic dehydration with Acidosis  Ref: Nelson's 20/e p 1617
Isotonic dehydration with acidosis is the most common finding in children with severe viral enteritis.
16. d. Severe dehydration  Ref: Nelson's 20/e p 1854-1874; Refer pretext of this chapter for details;
In this case, sunken eyes and skin “goes back very slowly” (in 5 seconds) in a “lethargic” (drowsy) child, so it is a case of severe
dehydration.
17. b. Rotavirus  Ref: Nelson's 20/e p 1854-1874
18. c. Zinc, Vit A  Ref: Ghai 8/e pg 291-300
Management of Persistent diarrhea involves:
•• Correction of dehydration, electrolytes and hypoglycemia; Evaluation for infections and their management
•• Nutritional therapy, including vitamin A and Zinc supplementation.
19. a. ORS  Ref: Nelson's 20/e p 1854-1874, Ghai 8/e p 291-300; Refer pretext for detailed composition of ORS;
ORS is now the global standard of care and more effective than home fluids, including decarbonated soda beverages, fruit
juices, and tea for fluid replacement in diarrhea.
20. a. Osmotic diarrhea  Ref: Nelson's 20/e p 1854-1874, Ghai 8/e p 291-300
21. a. ORS alone  Ref: Nelson's 20/e p 1854-1874, Ghai 8/e p 291-300
This child is suffering from acute gastroenteritis (as duration is less than 14 days); treatment of Acute Gastroenteritis in children:
includes oral rehydration therapy (ORS), zinc supplementation, normal diet.
22. b. Na 75 + 245 mOsmol  Ref: Nelson's 20/e p 1854-1874, Ghai 8/e p 291-300
23. a. 1:1  Ref: Approach to Practical Pediatrics 2nd ed by Narang Manish/ pg 24
Molarity of sodium and glucose in ORS should be 1:1, which helps in the efficient transport of sodium and water.
24. b. Jejunal polyp  Ref: Nelson's 20/e p 1854-1874, Ghai 8/e p 291-300
Important Causes of constipation in children:
Nonorganic (Functional)—Retentive Psychiatric—Anorexia nervosa
Anatomic Abnormal Musculature
•• Anal stenosis, atresia with fistula •• Prune-Belly syndrome
•• Imperforate anus and other malformations •• Gastroschisis
•• Intestinal stricture •• Down syndrome
Intestinal Nerve or Muscle Abnormalities Drugs
•• Hirschsprung disease •• Anticholinergics, Narcotics, Lead
•• Spinal cord defects including Tethered cord •• Chemotherapeutic agents (vincristine)
•• Spinal cord trauma •• Pancreatic enzymes (fibrosing colonopathy)
Metabolic Disorders Intestinal Disorders
•• Hypokalemia •• Celiac disease
•• Hypercalcemia •• Cow's milk protein intolerance
•• Hypothyroidism •• Cystic fibrosis (meconium ileus equivalent)
•• Diabetes mellitus, diabetes insipidus •• Inflammatory bowel disease (stricture)

25. c. Lactase  Ref: Nelson's 20/e p 1854-1874, Ghai 8/e p 291-300


Post infectious diarrhea
382 Section 3: Systemic Pediatrics
Review of Pediatrics and Neonatology

Definition Chronic diarrhea following infectious enteritis, regardless of the nature of the pathogen
Epidemiology Usually seen in infants and very young toddlers
Pathogenesis May be related to secondary lactase deficiency, food protein allergy, antibiotic associated colitis (including pseudo-
membranous colitis caused by Clostridium difficile toxin), or a combination of these.
Treatment Supportive; may include a lactose free diet in the presence of secondary lactase deficiency

26. a. Cryptosporidium  Ref: Nelson's 20/e p 1854-1874, Ghai 8/e p 291-300


Opportunistic microorganisms induce diarrhea exclusively, more severely or for more prolonged periods, in specific
populations, such as immunocompromised children. Cryptosporidium induces severe and protracted diarrhea in AIDS patients.
27. a. Intravenous fluids  Ref: Nelson's 20/e p 1854-1874, Ghai 8/e p 291-300
Risks associated with severe dehydration that might necessitate intravenous resuscitation include: age <6 mo; prematurity;
chronic illness; high fever; bloody diarrhea; persistent emesis; poor urine output; sunken eyes and a depressed level of
consciousness. In this case, as the child is < 6 months old and is having persistent emesis, intravenous fluid is indicated.
28. a. Normal saline  Ref: Nelson's 20/e p 1854-1874, Ghai 8/e p 291-300
•• Hyponatremic dehydration is treated by correction of intravascular volume with isotonic fluid (NS or LR).
•• An overly rapid (>12 mEq/L over the first 24 hr) or overcorrection of serum sodium concentration (>135 mEq/L) is associated with
an increased risk of central pontine myelinolysis.
29. d. 20 mg/day for 10–14 days  Ref: Nelson's 20/e p 1854-1874, Ghai 8/e p 291-300
Dose of Zinc for children older than 6 months with diarrhea is 20 mg/day for 10–14 days.
30. a. Enterotoxigenic E. coli (ETEC)  Ref: Nelson's 20/e p 1854-1874, Ghai 8/e p 291-300

High Yield Points  


•• E. coli is the most common bacteria causing diarrhea in children in developing countries
•• Rotavirus is the most common viral agent causing diarrhea
•• Shigella, ETEC and EHEC can cause dysentery; EHEC is a cause of hemolytic uremic syndrome.

31. a. 1000 mL  Ref: Nelson's 20/e p 1854-1874, Ghai 8/e p 291-300; Refer pretext for details;
For treatment of severe dehydration in a 2 yr child (10 kg wt), 100 ml/kg is to be given over 3 hrs, i.e. 100 x 10 = 1000 ml
32. a. Enterotoxigenic E. coli  Ref: Nelson's 20/e p 1854-1874, Ghai 8/e p 291-300
33. b. 300 mL  Ref: Nelson's 20/e p 1854-1874, Ghai 8/e p 291-300
For some dehydration, amount of ORS given is 75 ml/kg = 75 × 4 = 300 ml, over 4 hours
34. a. 20  Ref: Nelson's 20/e p 1854-1874, Ghai 8/e p 291-300
35. b. Lactose intolerance  Ref: Nelson's 20/e p 1854-1874, Ghai 8/e p 291-300
Presence of acidic stools with reducing substance positive, suggests a diagnosis of lactose intolerance.
36. d. None ≥ Can be mediated by bacterial toxins  Ref: Nelson's 20/e p 1854-1874, Ghai 8/e p 291-300
Osmotic diarrhea is caused by non-absorbed nutrients in intestinal lumen as a result of:
•• Intestinal damage (e.g., enteric infection) or Reduced absorptive surface area (e.g., active celiac disease)
•• Defective digestive enzyme or nutrient carrier (e.g., lactase deficiency)
•• Decreased intestinal transit time (e.g., functional diarrhea); and
•• Nutrient overload, exceeding the digestive capacity (e.g., overfeeding, sorbitol in fruit juice).
Whatever the mechanism, the osmotic force generated by non-absorbed solutes drives water into the intestinal lumen.
37. a. Lactase  Ref: Nelson's 20/e p 1854-1874, Ghai 8/e p 291-300
Because Lactase activity in mucosa is marginal, it is likely to be depleted secondary to diffuse mucosal disease
Causes of Acquired Lactase deficiency

•• Acute Infective enteritis: Viral or Bacterial gastroenteritis •• Cow's milk protein intolerance
•• Cystic fibrosis •• Prolonged use of drugs like Neomycin
•• Protein energy malnutrition

38. b. Rotavirus  Ref: Nelson's 20/e p 1854-1874, Ghai 8/e p 291-300;


39. b. Blood mixed with mucus stool is defined as dysentery, c. Rotavirus is the most common organism in children,
e. Enterotoxigenic E. coli is the most common bacteria causing diarrhea in children  Ref: Nelson's 20/e p 1854-1874
40. b. Giardiasis  Ref: Nelson's 20/e p 1854-1874, Ghai 8/e p 291-300
Intractable diarrhea
Chapter 16: Pediatric Gastrointestinology 383

Answers with Explanations


Definition Severe, protracted diarrhea for which the common pharmacologic and dietetic treatment had been unsuccessful

Pathophysiology It is often the result of a permanent defect in the structure or function of the enterocyte, leading to progressive,
potentially irreversible intestinal failure

Etiology •• Structural enterocyte defects: e.g. Microvillus inclusion disease, Tufting enteropathy
•• Disorders of intestinal motility: e.g. Hirschsprung disease, intestinal pseudoobstruction
•• Immune-based: e.g. IPEX (immune-dysregulation, polyendocrinopathy and enteropathy) syndrome
•• Short gut syndrome: It is the single most frequent etiology of chronic diarrhea
•• Disorders without demonstrable abnormalities: e.g. Multiple food protein hypersensitivity
•• Congenital diarrheal disorders: e.g. Cystic fibrosis
•• Neoplastic diseases: Neuroendocrine tumors

 CELIAC DISEASE
41. b. Anti-nuclear antibodies  Ref: Nelson's 20/e p 1835-1837
42. a. Celiac Disease  Ref: Nelson's 20/e p 1835-1837, Ghai 8/e p 301-302
Celiac disease
•• It is an immune mediated disorder elicited by wheat, rye and barley in genetically susceptible individuals
•• It is characterized by presence of–specific antibodies, HLADQ2 or DQ8 haplotypes, and enteropathy
•• Celiac disease–specific antibodies comprise autoantibodies against TG2 (Tissue Transglutaminase), Endomysial antibodies (EMA), and
antibodies against deamidated forms of gliadin peptides

43. c. Maize  Ref: Nelson's 20/e p 1835-1837, Ghai 8/e p 301-302


44. a. HLA-DQ2  Ref: Nelson's 20/e p 1835-1837, Ghai 8/e p 301-302
45. b. Increase in thickness of the mucosa  Ref: Nelson's 20/e p 1835-1837, Ghai 8/e p 301-302
•• In celiac disease proximal intestine is commonly involved while in tropical sprue whole of intestine is involved. Biopsy of
intestine in celiac sprue demonstrates (overall mucosal thickness remains same)
–– Villous atrophy, loss of microvilli brush border, with crypt hyperplasia
–– Plasma cells, lymphocytes, macrophages, eosinophils, mast cells are present in lamina propria
46. b. Anti endomysial antibody  Ref: Nelson's 20/e p 1835-1837
The given clinical features suggest a diagnosis of Celiac disease
Anti Endomysial antibody (anti EMA) & Anti tissue transglutaminase (anti TTG) antibody help in diagnosis of Celiac disease.

 INFLAMMATORY BOWEL DISEASE


47. b. Growth failure, c. Cobblestone colon, d. String sign of Kantor & e. Perianal fistula formation  Ref: Nelson 20/e p 1824-7
Discussing about the options one by one,
a. Loss of haustrations is a feature of ulcerative colitis
b. Growth failure can even be the only manifestation of Crohn disease.
c. Linear ulcers can give a cobblestone appearance to the mucosal surface in Crohn disease
d. 'String sign of Kantor' refers to the string-like appearance of a contrast-filled bowel loop caused by severe narrowing of a bowel
loop. Can be seen in Crohn disease, hypertrophic pyloric stenosis, gastrointestinal TB, carcinoid tumor and colon cancer.
e. Perianal disease is common (tag, fistula, deep fissure, abscess) in Crohn disease.
48. c. Glycogen storage diseases Type III  Ref: Nelson's 20/e p 1819-11830, Ghai 8/e p 304-305
Genetic disorders that have been associated with IBD include Turner syndrome, the Hermansky-Pudlak syndrome, glycogen
storage disease type Ib, Wiskott Aldrich syndrome, SCID and Agammaglobulinemia.
49. c. Crypt abscess are common  Ref: Nelson's 20/e p 1820
Crypt abscesses are common in Ulcerative colitis, but less common in Crohn's disease
50. c. Regional ileitis (Crohn's disease) and e. Ulcerative colitis  Ref: Nelson's 20/e p 1819-1825
51. b. Crohn's disease  Ref: Nelson's 20/e p 1819-11830, Ghai 8/e p 304-305
•• The boy in question is obviously malnourished (Expected weight 20-25 kg, height 125-130 cm)
•• A normal sigmoidoscopy rules out ulcerative colitis as it extends in a continuous fashion proximally from the rectum
•• A negative mantoux test goes against active tuberculosis and a negative stool culture also rules out strongyloidosis. Hence,
the obvious choice left is option ‘b', i.e. Crohn's disease.
384 Section 3: Systemic Pediatrics

 OTHER DISORDERS OF INTESTINES


Review of Pediatrics and Neonatology

52. b. Wait and watch  Ref: Nelson 20e/ p 1815


•• Ingestion of batteries → symptoms from leakage of alkali/Hg
•• If symptoms such as vomiting/abdominal pain, if a large battery (> 20 mm) remains in stomach for > 48 hr, or if a lithium battery
is ingested, the battery should be removed
•• In children < 6 yr age, batteries > 15 mm, are not likely to pass spontaneously & should be removed endoscopically
•• If patient develops peritoneal signs surgical removal
•• Batteries beyond duodenum pass per rectum in 85% within 72 hr
•• Lithium batteries → more severe injury than a button alkali battery
•• Button batteries in symptomatic child or if there are multiple batteries, they should be removed.
53. a. Shwachman-Diamond syndrome  Ref: Nelson 20/e p 1911-2
Disorders associated with pancreatic insufficiency include cystic fibrosis, Shwachman-Diamond syndrome, Pearson syndrome,
Johanson-Blizzard syndrome, isolated enzyme deficiencies, enterokinase deficiency, chronic pancreatitis & protein-calorie
malnutrition.
Shwachman-Diamond syndrome is an autosomal recessive syndrome with pancreatic insufficiency, neutropenia, neutrophil
chemotaxis defects, metaphyseal dysostosis, failure to thrive & short stature.
54. d. Colon cancer usually develops around 20 years of age  Ref: Nelson 20/e p 1902
Familial adenomatous polyposis (FAP)
•• It is the most common genetic polyposis syndrome
•• Characterized by numerous adenomatous polyps (100s to 1000s) throughout colon; stomach & duodenum may be involved
•• Extraintestinal manifestations include hypertrophy of retinal pigment epithelium, desmoid tumors, epidermoid cysts, osteomas,
fibromas, and lipomas.
•• Caused by mutations in APC gene, a tumor suppressor gene (chr 5q21).
•• Polyps generally develop in adolescence (mean age of presentation is 16 year).
•• These are precancerous lesions & without intervention, the risk of developing colon cancer is 100% by 5th decade of life
(average age of cancer diagnosis is 40 years).
•• Risk of periampullary or duodenal carcinoma & hepatoblastoma are also increased.
55. a. It is the most common genetic polyposis syndrome
Familial adenomatous polyposis is the most common genetic polyposis syndrome.
56. a. Allergy due to the food content  Ref: Nelson's 20/e p 1139
Nelson latest Ed says “Exclusive breastfeeding for the first 4-6 months of life may reduce allergic disorders in first few years of life.
Potentially allergenic foods (eggs, milk, wheat, soy, peanut and fish) should be introduced after this period of exclusive breastfeeding
to decrease chances of food allergy;” Hence, Increased food allergy is the best answer.
57. a. Breath hydrogen test  Ref: Nelson's 20/e p 1833-1834
Breath hydrogen test
•• It is used to identify the specific carbohydrate (lactose, sucrose, fructose, or glucose) that is malabsorbed.
•• After an overnight fast, the suspected sugar is administered as an oral solution (carbohydrate 1-2 g/kg, maximum 50 g).
•• In malabsorption the sugar is not digested in the small bowel, passes on to colon & is metabolized by colonic bacteria flora
•• Note: child should not be on antibiotics at the time of test, because colonic flora is essential for fermenting the sugar
•• One of the products of this process is hydrogen gas, which is absorbed through the colon mucosa & excreted in breath.
•• Increased hydrogen concentration in breath samples (20 ppm above baseline) suggests carbohydrate malabsorption.

58. c. It is due to a mutation of APC gene in chromosome 15  Ref: Nelson's 20/e p 1900-1902
General Features of Inherited Colorectal Cancer Syndromes

Polyp Polyp Age of Risk of Genetic


Syndrome histology distribution onset colon cancer lesion Clinical manifestations
Familial Adenoma Large intestine 16 years 100% 5q (APC gene) Rectal bleeding, abdominal pain,
adenomatous (often >100) (range 8–34 years) bowel obstruction
polyposis
Gardner Adenoma Large and small 16 year (range 100% 5q (APC gene) Rectal bleeding, abdominal pain,
syndrome intestine 8–34 year) bowel obstruction
Peutz-Jeghers Hamartoma Large and small 1st decade Increased 19p (STK11) Possible rectal bleeding,
syndrome intestine abdominal pain, intussusception
Juvenile Hamartoma Large and small 1st decade ≈10–50% PTEN, SMAD4, Possible rectal bleeding,
polyposis intestine BMPR1 abdominal pain, intussusception
Hereditary non- Adenoma Large intestine 40 year (range 30% Mismatch repair Rectal bleeding, abdominal pain,
polyposis colon 18–65 year) genes bowel obstruction
cancer
Chapter 16: Pediatric Gastrointestinology 385

Familial adenomatous polyposis (FAP):

Answers with Explanations


•• It is the most common genetic polyposis syndrome, due to mutation in APC gene on chr 5q21
•• Extraintestinal manifestations are hypertrophy of retinal pigment epithelium, desmoid tumors, epidermoid cysts, osteomas,
fibromas, and lipomas.
59. c. IIb  Ref: Nelson's 20/e p 869-870, Ghai 8/e p 165

Stage of NEC Radiological feature


I Normal/mild intestinal dilatation
IIa Pneumatosis intestinalis
IIb Portal vein gas
IIIa Peritonitis/Ascites
IIIb Pneumoperitoneum

60. a. Lower GI study  Ref: Nelson's 20/e p 1810-1812


This is a controversial question; Lower GI series is the best initial investigation in this case as it will help in evaluating for hirschsprung
disease as well as other diseases that manifest as lower bowel obstruction in a neonate.
61. a. Pigmentary changes in skin and mucous membrane around mouth; e May present as anemia in children Ref: Nelson's
20/e p 1902, 3134
Peutz Jegher Syndrome
What is it? Characterized by mucocutaneous pigmentation and extensive GI hamartomatous polyposis
Epidemiology Median age of 11 years; GI cancer in 2-3% of patients
Genetics Autosomal dominant; Heterozygous loss-of-function of STK11Q
Site Jejunum is the most common site
Symptoms •• Intussusception; Hematochezia can be present in 14% cases – can present as anemia
•• Mucocutaneous pigmentation (lips, nostrils, buccal mucosa,Q palms, genitalia and perianal region)
Malignant potential Increased risk of sex cord tumors (testes), small intestinal, pancreatic, breast, lung & ovarian cancers.
Treatment Polypectomy when discovered on colonoscopy

62. c. Conservative management with IV fluids and antibiotics  Ref: Nelson's 20/e p 1894, Ghai 8/e p 291
63. c. Give O2, d. Nasogastric tube, e. IV fluid  Ref: Nelson's 20/e p 1854, Ghai 8/e p 291
•• In patients with prolonged intussusception with signs of shock, peritoneal irritation, intestinal perforation or pneumatosis
intestinalis, reduction (with barium, saline or air) should not be attempted
•• In such cases, urgent laprotomy should be planned
•• But the first priority is urgent resuscitation ie A, B, C (airway, breathing, circulation).
Hence in case of shock, and as the child in question is also vomiting, she should be made NPO and Nasogastric tube insertion should
be done.
64. a. High fibre diet  Ref: Nelson's 20/e p 869, Ghai 8/e p 165
65. d. Peritoneal drainage by putting drains in the flanks  Ref: Nelson's 20/e p 536, Ghai 8/e p 165
Management of necrotizing enterocolitis (NEC):
Medical treatment of NEC include Surgical interventions in NEC
•• Cessation of feeding and Nasogastric decompression. •• Evidence of perforation is an indication for laparotomy
•• Administration of IV fluids. and resection of necrotic bowel.
•• Systemic broad spectrum antibiotics •• Peritoneal drainage in unstable patients with stage III

From this discussion, we choose the answer to be peritoneal drainage because; Patients is on ventilator (and unstable).
66. a. i.e. Most common site is small intestine; b. Polypectomy and Colonoscopy is treatment of choice; c. Hamartomatous
polyp; d. Mucosal involvement present  Ref: Nelson's 20/e p 1902
67. a. Pyloric stenosis  Ref: Nelson's 20/e p 1797-1799, Ghai 8/e p 279-280
68. a. Tracheo-esophageal fistula  Ref: Nelson's 20/e p 1783, Ghai 8/e p 176
69. Ans. b. Bad bowel habits  Ref: Nelson's 20/e p 1807-1808
Megacolon
•• Megacolon is an abnormal dilatation of colon that is not caused by mechanical obstruction
•• Two main types of megacolon are:
386 Section 3: Systemic Pediatrics

–– Congenital megacolon->Hirschsprung disease


Review of Pediatrics and Neonatology

–– Acquired megacolon: Poor bowel habit, Infections (Chagas disease), Idiopathic, antipsychotic drugs.
70. a. Small intestinal biopsy  Ref: Nelson's 20/e p, Ghai 8/e p 1831-1838
•• The given child has malabsorption syndrome associated with chronic diarrhea
•• Intestinal biopsy is necessary to differentiate chronic diarrhea due to different pathophysiologic reasons
•• Work-up of a child suspected to have chronic diarrhea and malabsorption includes the following investigations- Repeated stool
examination (especially for giardia), fecal fat excretion studies, D-xylose test, intestinal biopsy, specific tests, e.g. sweat
chloride for cystic fibrosis, serology for celiac disease, exocrine pancreatic function.
71. d. Diaphragmatic hernia  Ref: Nelson's 20/e p 1801
•• Obstruction of 2nd part of duodenum cause ‘double bubble' sign on plain abdominal radiograph
•• It may be due to- Annular pancreas, Pancreatic pseudocyst, Ladds band a Tumor in head of pancreas.
72. c. Bilious vomiting  Ref: Nelson's 20/e p 1802
Bilious vomiting is the most common symptom of Intestinal obstruction in newborn;
Presentation depends on the site of obstruction:
•• Proximal obstruction
–– A history of polyhydramnios is common; Abdominal distension and constipation will be less prominent
–– The higher the obstruction in the intestine the earlier the infant will develop vomiting which may be bile stained
•• Distal obstruction, Abdominal distension and constipation are more common, while vomiting is less common. Note: In
neonates proximal intestinal obstruction are much more common than distal. So, bilious vomiting is the most common
presentation of neonatal intestinal obstruction.
73. a. Intussusception  Ref: Nelson's 20/e p 1812, Ghai 8/e p 287
Discussing the options one by one:
a. Intussusception The typical age group with short history of bloody diarrhea and extreme tenderness on examination point
towards intussusception
b. HUS In HUS, the typical clinical picture is of sudden onset pallor, oliguria, edema, petechiae and lethargy;
Usually there is history of severe gastroenteritis with blood stained stools but no severe pain abdomen;
c. Appendicitis Typically presents with right lower quadrant pain with/ without gastroenteritis
It is most common in older children, with peak incidence between the age of 12 and 18 year
d. Ac. enterocolitis There is usually a history of fever, vomiting and colicky pain abdomen, usually associated with tenesmus

74. c. Crohn's disease  Ref: Nelson's 20/e p 1819, Ghai 8/e p 304
Blood in stools, abdominal pain and systemic features like fever and weight loss for months is suggestive of Inflammatory bowel
disease.
Chapter 17
Pediatric
Hepatology
 NEONATAL CHOLESTASIS High Yield Points M

Definition
Enzymes elevated in cholestasis are:
Prolonged elevation of the serum levels of conjugated bilirubin to >2 mg% or >20% of total •• Alkaline phosphate
bilirubin beyond the 1st 14 days of life. •• 5’ nucleotidase
•• Gamma glutamyl transpeptidase
Etiology of Neonatal Cholestasis
Infections Metabolic Bile duct anomalies
Viral: HBV, HCV, rubella, adenovirus, •• Galactosemia, glycogen storage Extrahepatic biliary atresia
coxsackie-virus, HHV-6, Varicella disease, tyrosinemia, neonatal Choledochal cyst
zoster, HSV, HIV parvovirus hemochromatosis, hereditary Alagille syndrome
Bacterial: Sepsis, UTI, syphilis, TB, fructose intolerance, cystic fibrosis High Yield Points M
Caroli’s disease 
Listerosis •• Dubin Johnson syndrome, Rotor
syndrome, Niemann Pick disease, Choledocholithiasis
Parasitic: Toxoplasmosis, •• Screening test of choice for biliary
α-1 antitryp in deficiency Neonatal sclerosing
Malaria atresia is HIDA scan (Hepatic
cholangitis
scintigraphy)
Idiopathic neonatal
Endocrinopathies Chromosomal disorders hepatitis •• Surgery done for biliary atresia is
Kasai’s portoenterostomy
Hypothyroidism Turner syndrome, trisomy 18, Trisomy
•• Success rate for establishing good
Hypopituitarism 21, trisomy 13
bile flow (in EHBA) after the Kasai
 BILIARY ATRESIA operation is about 90% if performed
before 8 weeks of life
Definition
Idiopathic inflammatory process involving the bile ducts resulting in biliary tract obstruction,
chronic cholestasis, and progressive fibrosis & eventually to biliary cirrhosis.
Anatomically of 3 Types
•• Type I: Involving common bile duct and a patent proximal biliary system (5% cases)
•• Type II: Involving the hepatic duct with patent proximal duct (3%) Question 1
•• Type III: Involving right and left hepatic duct at the porta hepatis (90%). A 20-day-old baby presented with
conjugated hyperbilirubinemia
Differences Between Neonatal Hepatitis and Extrahepatic and clay coloured stools. What is
Biliary Atresia (EHBA) the following investigation done to
Features Neonatal hepatitis EHBA know whether the baby has biliary
atresia?
Onset Anytime during neonatal period End of first week
Jaundice Mild-moderate Moderate to severe
Stools Variable in color Clay colored
Hepatosplenomegaly Early Late
Alkaline phosphatase Not increased Increased
SGOT/SGPT Severe derangement Mild-moderate derangement
Abdominal Identifies choledocholithiasis or other Triangular cord (IC) sign (cone shaped
ultrasound structural abnormalities of biliary fibrotic mass cranial to bifurcation of
tree, e.g. choledochal cyst portal vein) maybe seen
HIDA scan Radioactivity seen in intestine No radioactivity seen in intestine
Liver biopsy (most Diffuse hepatocellular disease with Bile ductular proliferation, portal or
valuable procedure) distortion of lobular architecture, perilobular edema and fibrosis hepatic
a. Ultrasound abdomen
inflammatory cell infiltration + giant lobular architecture intact
b. Hepatic scintigraphy
cells
c. Barium meal
Operative Normal Usually determines the presence and
d. GER scan
cholangiogram site of obstruction
388 Section 3: Systemic Pediatrics

Management
Review of Pediatrics and Neonatology

High Yield Points


•• Exploratory laparotomy & direct cholangiography show site of obstruction
•• GGT elevation in serum is less specific
•• Direct drainage can be accomplished in the few patients with a correctable lesion.
than alkaline phosphatase and 5’
nucleotidase •• When no correctable lesion is found: hepatoporto-enterostomy (Kasai procedure).
•• GGT is elevated to values 10 times
normal in biliary atresia while in  PORTAL HYPERTENSION
hepatic causes it is raised to about
•• Normal portal venous pressure is 7 mm Hg M
three times normal.
•• Normal value for GGT is 5–40 U/L •• Portal hypertension is defined as an elevation of portal pressure >10-12 mm Hg. M

A. Etiology of Portal Hypertension


Site of defect Causes
Presinusoidal (Prehepatic) •• It is the most common cause of portal hypertension in children
•• It is due to portal vein obstruction from any cause
Sinusoidal (Hepatic) •• It is due to intrahepatic obstruction
•• Most common cause is cirrhosis of liver from any cause (viral or
High Yield Points  M
autoimmune hepatitis, Wilson’s disease).

Biliary atresia is the most common Post sinusoidal (Post hepatic) •• It is due to obstruction in hepatic vein or IVC, e.g. Budd-Chiari syndrome
indication for liver transplantation in
children. B. Differentiation between Causes of Portal Hypertension
•• Post sinusoidal and hepatic causes of portal hypertension will have
–– Evidence of liver parenchymal damage → deranged LFT (jaundice)
–– Dilated portal veins on ultrasonography
•• Extrahepatic presinusoidal obstruction will not have any effect on the liver and hence there
will be no evidence of hepatocellular damage
–– No jaundice & no deranged LFT
–– Non visualization of portal vein is a characteristic of portal vein thrombosis, an
extrahepatic cause.

C. Budd-Chiari Syndrome M

High Yield Points What is it? Portal hypertension caused by obstruction to hepatic veins anywhere between the
efferent hepatic veins & the entry of the inferior vena cava into the right atrium
Etiology •• Hypercoagulable states e.g. antithrombin III deficiency, protein C or S deficiency, factor
Esophageal varices and spleno­
V Leiden or prothrombin mutations, paroxysmal nocturnal hemoglobinemia
megaly are common manifestations of
portal hypertension irrespective of the •• Hepatic or metastatic neoplasms, Collagen vascular disease, Infection, Trauma
underlying cause
D. Extrahepatic Portal Vein Obstruction (EHPVO)
The obstruction can occur at any level of the portal vein; usual causes of EHPVO are:

In neonates In older children


•• Omphalitis with/without a history of •• Intra-abdominal infections, e.g. acute appendicitis,
catheterization of umbilical vein peritonitis, biliary tract infections
•• Neonatal dehydration and sepsis •• Inflammatory bowel disease
•• Agenesis/atresia/stenosis of portal vein •• Primary sclerosing cholangitis.
•• Obstruction by a web or diaphragm •• Hypercoagulable states, e.g. factor V Leiden, protein C,
or protein S deficiency

 ACUTE VIRAL HEPATITIS

Clinical and epidemiologic features of viral hepatitis


Feature Hepatitis A Hepatitis B Hepatitis C Hepatitis D Hepatitis E
Incubation (days) 15–45, mean 30 30–180, 15–160, 30–180, 14–60,
mean 60–90 mean 50 mean 60–90 mean 40
Onset Acute Insidious or acute Insidious Insidious or acute Acute
Contd...
Chapter 17: Pediatric Hepatology 389

Contd...

Section 3: Systemic Pediatrics


Feature Hepatitis A Hepatitis B Hepatitis C Hepatitis D Hepatitis E

Age perference Children, young Young adults, babies, any age, more Any age (similar to HBV) Epidemic cases: young
adults toddlers common in adults adults (20–40 years)

Transmission Feco-oral Percutaneous/Perinatal Percutaneous Percutaneous Feco-oral


sexual

Fulminant 0.1% 0.1–1% 0.1% 5–20% 1–29%

Progression to None Occasional (1–10%) (90% of Common (85%) Common None


chronicity neonates)

Carrier None 0.1–30% 1.5–3.2% Variable None

Prognosis Excellent Worse with age Moderate Acute: good Chronic: poor Good

Clinical Course of Hepatitis B Infection

Outcome of Acute Hepatitis B in Children

Treatment of Viral Hepatitis in Children


•• Acute viral hepatitis: Supportive treatment
•• Chronic Hep B: Interferon-α2b, Lamivudine, Adefovir, Entecavir or Tenofovir.
•• Chronic Hep C: Peginterferon, IFN-α2b & ribavirin are FDA approved for children > 3 yr
390 Section 3: Systemic Pediatrics

 REYE’S SYNDROME (JAMSHEDPUR FEVER)


Review of Pediatrics and Neonatology

High Yield Points


•• Infection by RSV (Resp. syncitial What is it? An acute metabolic disorder resulting in generalised mitochondrial
virus) does not precipitate Reye’s dysfunction due to inhibition of fatty acid oxidation
syndrome
•• Even though hepatic involvement is Characterised by Fatty liver & Encephalopathy (due to liver damage & cerebral edema)
there, jaundice is usually absent in Precipitating factors •• Drugs, toxins, virus, IEM can precipitate it
Reye syndrome
•• Viruses that precipitate it are Influenza A & B, Varicella, Adeno, Cox sackie A
•• It is not caused by RSV (Respiratory syncitial virus)

Clinical features •• History of precipitating factors like Aspirin intake


•• Liver dysfunction: Prolonged PT, hypoglycemia seen
•• Seizures are commonly seen

Treatment Supportive

Prognosis Poor (25-70% mortality)

Answer Keys for Image-Based Questions

Answer Explanation / Identifying feature


1. Ans. b. Hepatic scintigraphy This investigation, that shows concentration of radioisotope in the liver & its excretion in intestine, is
Hepatic scintigraphy or HIDA scan
Chapter 17: Pediatric Hepatology 391

Questions
Questions
1. Most common indication of liver transplant in children 12. Which of the following viral markers signifies the
is: (NEET PG Jan 2019) ongoing viral replication in the case of Hepatitis-B
a. Biliary atresia b. Von Gierke disease infection? (UPSC CMS 2015)
c. Tyrosinemia d. Chronic viral hepatitis a. Anti-HBs b. Anti-HBc
2. Gene encoding which protein is defective in Dubin c. HBe Ag d. HBs Ag
Johnson syndrome? (NEET PG Jan 2019) 13. Which of the following if seen on Day 10 of life, is
a. JAG1 b. MRP2 worrisome? (AIIMS Nov 2014)
c. ATP7A d. ATP 7B a. Conjugated hyperbilirubinemia
3. A 2-year-old child presented with hepatomegaly. Cause b. Dolls Eye Reflex
may be: (PGI May 2017) c. No weight gain
a. Biliary atresia b. Hepatoblastoma d. Unconjugated hyperbilirubinemia
c. Cirrhosis d. Glycogen storage disease 14. Most common cause of portal hypertension in children
e. Gauchers is: M (Recent Question 2013)
4. Test to differentiate between neonatal hepatitis and a. Extrahepatic portal vein obstruction
biliary atresia in infant with jaundice: b. Budd-Chiari syndrome
 (Recent Question 2017) c. Veno-occlusive disease d. Postnecrotic
a. Gamma glutamyl transpeptidase 15. Most common cause of cholestatic jaundice in new born
b. Alanine transaminase is:
c. Aspartate transaminase d. Ultrasound a. Hypoplasia of biliary tract (Recent Question 2013)
5. Gilbert syndrome True is: (Recent Question 2017) b. Neonatal hepatitis
a. Conjugated hyperbilirubinemia c. Choledochal cyst d. Physiological
b. Starvation improves the condition 16. True about Wilson's disease is: (Recent Question 2013)
c. Phenobarbitone improves the condition a. Increased serum ceruloplasmin
d. Life-threatening b. Decreased liver copper
6. Most common indication for liver transplant in children c. Increased urinary copper excretion
is: (Recent Question 2016) d. Decreased urine copper excretion
a. Viral hepatitis with fulminant hepatic failure 17. False about kernicterus is: (Recent Question 2013)
b. Biliary atresia a. No long-term effect
c. Metabolic disease d. Hepatic tumor b. Occurs with bilirubin more than 25 mg%
7. The obstruction of two or more major hepatic veins is c. Deposition in basal ganglion
seen in: M  (MAHA PGM CET 2016) d. Opisthotonus
a. Budd-Chiari syndrome b. Reye's syndrome 18. In neonatal cholestasis direct bilirubin is:
c. Rotor syndrome d. Crigler-Najjar syndrome  (Recent Question 2012)
8. In Wilson's disease, hepatic copper content usually a. >10% b. >15%
exceeds .............. ug per gram dry weight. c. >20% d. >25%
 (MAHA PGM CET 2016) 19. Wilson disease true all except: (Recent Question 2012)
a. 150 b. 250 a. AR b. KF ring
c. 350 d. 450 c. Raised copper level d. Raised ceruloplasmin level
9. Investigation of choice for biliary atresia in a 2 month 20. Aspirin is associated with: M  (Recent Question 2012)
old infant is: (AIIMS May 2015)
a. Reye's syndrome b. Sjogren syndrome
a. Hepatic scintigraphy b. ERCP
c. Reiter synderome d. None of above
c. USG d. CECT
21. A 12-year-old boy presents with hematemesis, melaena
10. Post-hepatic Portal Hypertension is caused by: and mild splenomegaly. There is no obvious jaundice or
a. Portal vein thrombosis (MAHA PGM CET 2015) ascites. The most likely diagnosis is: (AIPGMEE 11)
b. Banti syndrome a. EHPVO b. NCPF
c. Budd-Chiari syndrome
c. Cirrhosis d. Malaria with DIC
d. Congenital hepatic fibrosis
22. In a child with acute liver failure, the most important
11. Which one of the following inherited conditions causes abnormal serum biochemical test that indicates poor
direct hyperbilirubinemia? (APPG 2015)
prognosis:  (COMEDK 09)
a. Gilbert syndrome a. Increasing transaminases
b. Type II Crigler-Najjar syndrome b. Increasing bilirubin
c. Rotor syndrome c. Increasing prothrombin time
d. Type I Crigler-Najjar syndrome d. Reversal of serum albumin-globulin ratio
392 Section 3: Systemic Pediatrics

23. Treatment of choice in 11 years old children with 31. Ramu, an 8-year-old boy presents with upper GI bleeding.
Review of Pediatrics and Neonatology

hepatitis C infection:  (UP PGMEE 08) On examination, he is found to have spleno­megaly; there
a. Vaccine b. Interferon are no signs of ascites, or hepato­ megaly; esophageal
c. Gamma-globulin d. Corticosteroids varices are found on UGIE. Most likely diagnosis is:
24. Regarding Gilberts syndrome all the following are true a. Budd Chiari syndrome (AIPGMEE 2001)
except:  (WBPG 2007) b. Non cirrhotic portal fibrosis
a. More common in females c. Cirrhosis d. Veno-occlusive disease
b. Autosomal dominant 32. Hepatitis B incubation period is: M  (FMGE Pattern 2017)
c. Phenobarbitone inproves condition a. < 7 days b. 20-30 days
d. Fasting increases the bilirubin condition c. 60-90 days d. 10-15 days
25. In a child presenting with obstructive Jaundice all are 33. Chronic hepatitis is seen in infection due to which of the
elevated except:  (AIIMS Nov 06) following viruses? (WB PGMEE 2016)
a. Gamma glutamyl transpeptidase a. Hep A b. Hep B
b. Alkaline phosphatase c. Hep E d. Hep D
c. Glutamate dehydrogenase 34. A neonate born to infected hepatitis-B mother, should
d. 5' Nucleotidase be treated with: M  (AIPGMEE 09, 99)
26. True about Extrahepatic biliary atresia: (PGI June 03)
a. Isolation b. Immunoglobulins
a. Acholic stool c. Hepatitis-B-vaccine
b. Unconjugated hyperbilirubinemia d. Immunoglobulins and hepatitis-B-vaccine
c. Conjugated hyperbilirubinemia 35. A 30-year-old lady delivered a healthy baby at 37 week
d. Absence of radionuclide in duodenum in HIDA scan of gestation. She was known case of chronic hepatitis B
e. Jaundice is presenting feature infection and positive for HbsAg but negative for HbeAg.
27. Portal hypertension in children in India is commonly Which of the following is the most appropriate treatment
due to:  (AIIMS Nov 2003) for the baby? M  (AIIMS Nov 05)
a. Indian childhood cirrhosis a. Both active and passive immunization soon after birth
b. Extrahepatic portal venous obstruction b. Passive immunization soon after birth and active
c. Idiopathic portal hypertension immunization at 1 year of age
d. Hepatic outflow tract obstruction c. Only passive immunization soon after birth
28. A 7-year-old girl from Bihar presented with three d. Only active immunization soon after birth
episodes of massive hematemesis and melena. There is 36. A 5-year-old boy is detected to be HBsAg positive on
no history of jaundice. On examination, she had a large two separate occasions during a screening program for
spleen, non-palpable liver and mild ascites. Portal vein hepatitis B. He is otherwise asymptomatic. Child was given
was not visualized on ultrasonography. Liver function three doses of recombinant hepatitis B vaccine at the age
tests were normal and endoscopy revealed esophageal of 1 year. His mother was treated for chronic hepatitis B
varices. The most likely diagnosis is: (AIPGMEE 2003) infection around the same time. The next relevant step for
a. Kala azar with portal hypertension further investigating the child would be to:
b. Portal hypertension of unknown etiology a. Obtain HbeAg and anti-Hbe levels (AIPGMEE 03)
c. Chronic liver disease with portal hypertension b. Obtain anti-Hbs levels
d. Portal hypertension due to extrahepatic obstruction c. Repeat HbsAg
29. In neonatal cholestasis, if the serum gamma glutamyl- d. Repeat another course of Hepatitis B vaccine
transpeptidase (GGT) is more than 600 IU/L the most 37. Which one of the following hepatitis viruses have
likely diagnosis is:  (AI 04, AIIMS Nov 02) significant perinatal transmission: M  (AIPGMEE 03)
a. Neonatal hepatitis a. Hepatitis E virus b. Hepatitis C virus
b. Choledochal cyst c. Hepatitis B virus d. Hepatitis A virus
c. Sclerosing cholangitis 38. A 45-day-old infant developed icterus and two days
d. Biliary atresia later symptoms and signs of acute liver failure appeared.
30. A neonate presents with jaundice and clay colored Child was found to be positive for HbsAg. The mother
stools. Liver biopsy shows giant cells diagnosis is: was also HBsAg carrier. The mother’s hepatitis B
 (AIIMS Nov 01, AI 01) serological profile is likely to be: (AIPGMEE 03)
a. Neonatal hepatitis with physiological jaundice a. HbsAg positive
b. Neonatal hepatitis with extrahepatic biliary atresia b. HbsAg and HbeAg positivity
c. Physiological jaundice c. HbsAg and anti-Hbe antibody positivity
d. Physiological jaundice with extrahepatic biliary atresia d. Mother infected with mutant HBV
Chapter 17: Pediatric Hepatology 393

Answers with Explanations


Answers with Explanations
1. a. Biliary atresia  Ref: Nelson's 20/e p 1975
Biliary atresia is the most common indication for liver transplantation in children, followed by metabolic and inborn disorders,
autoimmune and familial cholestatic disorders, and acute hepatic necrosis.
2. b. MRP2  Ref: Nelson's 20/e p 1931, 1938
In Dubin Johnson syndrome, ABCC2 gene is affected, that codes for MRP2 protein. Absent function of multiple drug-resistant protein
2 (MRP2), an adenosine triphosphate–dependent canalicular transporter, is the responsible defect, that leads to defective conjugated
bilirubin excretion.
3. a. Biliary atresia, b. Hepatoblastoma, d. Glycogen storage disease, e. Gauchers  Ref: Nelson 20/e p 709
In cirrhosis, the liver is usually shrunken, leading to non-palpable liver.
4. d. Ultrasound  Ref: Nelson's 20/e p 1930
All the 3 hepatic enzymes are elevated in both neonatal hepatitis and biliary atresia. But USG may show triangular cord sign in
biliary atresia.
5. c. Phenobarbitone improves the condition
Phenobarbitone, being an enzyme inducer, improves the condition by increasing the activity of UDP glucuronyl transferase enzyme.
6. b. Biliary atresia  Ref: Nelson's 20/e p 1975
Biliary atresia is the most common indication for liver transplantation in children, followed by metabolic and inborn disorders,
autoimmune and familial cholestatic disorders, and acute hepatic necrosis.
7. a. Budd-Chiari syndrome  Ref: Nelson's 20/e p 1973, Ghai 8/e p 320; Refer pretext for details;
8. b. 250  Ref: Nelson's 20/e p 940, Ghai 8/e p 320-321;
9. a. Hepatic scintigraphy  Ref: Nelson's 20/e p 1933-1934
Investigations in Biliary atresia

Investigation Features seen


Hepatobiliary Can differentiate intrahepatic cholestasis from extrahepatic obstruction in neonates;
scintigraphy (HIDA Hepatic uptake of the agent is normal in biliary atresia, but excretion into the intestine is absent;
scan) Phenobarbital (5 mg/kg/day) for 5 days before scan enhances biliary excretion of isotope
It is a sensitive but not specific test for biliary atresia
It fails to identify other structural abnormalities of the biliary tree or vascular anomalies
USG Shows small or absent gallbladder; non-visualization of common duct & presence of triangular cord sign
Percutaneous liver Most valuable procedure in the evaluation of neonatal hepatobiliary diseases
biopsy Biliary atresia is characterized by bile ductular proliferation, the presence of bile plugs, and portal or perilobular
edema and fibrosis, with the basic hepatic lobular architecture intact
Endoscopic It is an alternative method of examining the bile ducts in older children
imaging (ERCP) The papilla of Vater is cannulated under direct vision and contrast material is injected into the biliary and pancreatic
ducts to outline the anatomy
Direct Exploratory laparotomy and Transhepatic cholangiography to determine presence and site of obstruction
cholangiography

10. c. Budd-Chiari Syndrome  Ref: Nelson's 20/e p 1973, Ghai 8/e p 320
Postsinusoidal causes of Portal hypertension include: Budd-Chiari syndrome and Veno-occlusive disease.
11. c. Rotor syndrome  Ref: Nelson's 20/e p 1937-1939
Disorders of bilirubin metabolism
Type of bilirubin elevated Diseases
Unconjugated Hyperbilirubinemia Crigler-Najjar syndrome Types I and II (UDP GT gene mutation)
Gilbert disease (UPD GT polymorphism)
Conjugated Hyperbilirubinemia Dubin-Johnson syndrome (multiple drug-resistant protein 2 mutation)
Rotor syndrome (deficiency in organic anion uptake)

Unlike Dubin-Johnson syndrome, in Rotor syndrome:


•• Total urinary coproporphyrin excretion is elevated, with a relative increase in coproporphyrin I isomer
•• Gallbladder is normal by roentgenography and Liver cells contain no black pigment.
394 Section 3: Systemic Pediatrics

12. c. HBe Ag  Ref: Nelson's 20/e p 1947; Refer pretext for details
Review of Pediatrics and Neonatology

13. a. Conjugated hyperbilirubinemia  Ref: Nelson's 20/e p 1937


Discussing about the options one by one,
a. Presence of conjugated hyperbilirubinemia is abnormal at anytime of life including Neonatal period
b. A Neonate may fixate on face or light in line of vision and “Doll's-eye” movement of eyes is seen
c. Physiological weight loss is seen in neonates and birth weight is regained by day 10 of life in term neonates, after which weight
gain begins
d. Jaundice is seen in 60% of term infants and 80% of preterm infants; Physiological jaundice is always unconjugated.
14. a. Extrahepatic portal vein obstruction  Ref: Nelson's 20/e p 1973
•• Extrahepatic portal vein obstruction is an important cause of portal hypertension in childhood
•• The obstruction can occur at any level of the portal vein
•• Umbilical infection (omphalitis) with or without a history of catheterization of the umbilical vein may be causal in neonates.
Three most common causes of portal hypertension in children in India: are Extrahepatic portal vein obstruction (most
common), Cirrhosis, Non-cirrhotic portal fibrosis.
15. b. Neonatal hepatitis  Ref: Nelson's 20/e p 1928-1936, Ghai 8/e p 311-312
Most Common causes of Neonatal cholestasis is Idiopathic Neonatal hepatitis: (25-50% cases).
16. c. Increased urinary copper excretion  Ref: Nelson's 20/e p 1939-1940; Refer pretext of chapter 18 for details;

High Yield Points  


•• In Wilson disease serum ceruloplasmin is decreased (<20 mg/dL), “free” copper level may be elevated in early ds (>1.6 µmol/L)
•• Urinary copper excretion is increased to >100-1000 µg/day & hepatic copper content is > 250 µg/g dry weight in Wilson disease

17. a. No long-term effect  Ref: Nelson's 20/e p 876-880, Ghai 8/e p 327-329
Kernicterus is due to deposition of conjugated bilirubin in basal ganglia.
Mnemonic for features of Chronic Bilirubin Encephalopathy (Kernicterius): “S-A-D M-U-M” (Refer pretext of ch 3, Neonatology)
18. c. >20%  Ref: Nelson's 20/e p 1928-1930, Ghai 8/e p 311-312
Neonatal cholestasis is defined biochemically as prolonged elevation of the serum levels of conjugated bilirubin (> 2 mg/dL or > 20%
of total bilirubin level) beyond the 1st 14 days of life.
19. d. Raised ceruloplasmin level  Ref: Nelson's 20/e p 1940, Ghai 8/e p 320
•• Wilson disease is an autosomal recessive disorder caused by a mutation of ATP7B gene on chromosome 13.
20. a. Reye's syndrome  Ref: Nelson's 20/e p 2902-2904; Refer pretext of this chapter for details of Reye syndrome
21. a. EHPVO  Ref: Nelson's 20/e p 1972
•• The given history is suggestive of portal hypertension
•• Extrahepatic portal vein obstruction is an important cause of portal hypertension in childhood.
•• The obstruction can occur at any level of the portal vein.
22. c. Increasing prothrombin time  Ref: Nelson's 20/e p 1967
•• Hepatic synthetic function is reflected in serum albumin and protein levels and in the PT or INR
•• Unresponsiveness of PT to vitamin K suggests severe hepatic disease
•• Persistently low levels of factor VII are evidence of a poor prognosis in fulminant liver disease
•• Serum aminotransferase activities do not correlate well with the severity of the illness.
23. b. Interferon  Ref: Nelson's 20/e p 1947-1950
•• Peginterferon, IFN- 2b, and ribavirin are approved by the FDA for use in children older than 3 yr of age with HCV hepatitis
•• Treatment of Chronic hepatitis B in children involves use of Interferon- α2b, Lamivudine, Adefovir, Entecavir or Tenofovir.
24. a. More common in females  Ref: Nelson's 20/e p 1937, Ghai 8/e p 312
Gilbert Syndrome

Feature Gilbert's Syndrome


Inheritance (all autosomal) Promoter mutation: recessive; Missense mutations: 7 of 8 dominant; 1 reportedly recessive

Total serum bilirubin < 4 mg/dL in absence of fasting or hemolysis (mild unconjugated hyperbilirubinemia);
Unconjugated Hyperbilirubinemia increased in stress, fatigue, reduced caloric intake and intercurrent illness

Routine liver tests Normal


Hepatic histology Usually normal; increased lipofuscin pigment in some

Prognosis No increased risk of cirrhosis


Chapter 17: Pediatric Hepatology 395

25. c. Glutamate dehydrogenase  Ref: Nelson's 20/e p 1926

Answers with Explanations


•• Elevations in serum Alkaline phosphatase, 5′ nucleotidase, and γ-glutamyl transpeptidase levels are sensitive indicators of
obstruction or inflammation of the biliary tract.
26. a. Acholic stool, c. Conjugated hyperbilirubinemia, d. Absence of radionuclide in duodenum in HIDA scan, e. Jaundice
is presenting features  Ref: Nelson's 20/e p 1972
27. b. Extrahepatic portal venous obstruction  Ref: Nelson's 20/e p 1972; Refer pretext for details
28. d. Portal hypertension due to extrahepatic obstruction  Ref: Nelson's 20/e p 1928
Splenomegaly and esophageal varices point to the diagnosis of portal hypertension while absence of jaundice and
deranged liver function test indicate an extrahepatic etiology.
29. Ans. d. Biliary atresia  Ref: Nelson's 20/e p 1933
3 Enzymes reflect cholestasis: Alkaline phosphatase, 5'nucleotidase & Gamma glutamyl transpeptidase.
About GGT in Cholestasis: (Normal value of GGT is 5-40 IU/L)
“GGT levels are commonly elevated to values more than 10 times normal in biliary atresia while in hepatic causes it is raised to about three
times normal”

Thus a value of 600 IU/L in the patient in question reflects an elevation of more than ten times the normal value. The patient is thus
likely suffering from ‘biliary atresia'.
30. b. Neonatal hepatitis with extrahepatic biliary atresia  Ref: Nelson's 20/e p 1936
•• Jaundice with clay colored stools suggests cholestasis
•• Both neonatal hepatitis and extrahepatic biliary atresia cause cholestasic jaundice
•• “Giant cells” are characteristics of both neonatal hepatitis and extrahepatic biliary atresia
•• Therefore the answer is neonatal hepatitis with extrahepatic biliary atresia.
Idiopathic neonatal hepatitis (Also caused “giant cell hepatitis”) is cholestatic jaundice of unknown cause that has a typical liver
biopsy appearance.
31. b. Non cirrhotic portal fibrosis  Ref: Nelson's 20/e p 1936
Clues in this question are: Upper GI bleeding from esophageal varices, Splenomegaly, No ascites or hepatomegaly, No jaundice; All
these indicate non-cirrhotic portal fibrosis.
About other options:
•• Budd-Chiari syndrome (Hepatic vein thrombosis) and venocclusive disease: Hepatomegaly, jaundice and ascites seen.
•• Cirrhosis: jaundice and ascites will be seen.
32. c. 60–90 days  Ref: Nelson’s 20/e p 1942-1952
33. b. Hep B  Ref: Nelson’s 20/e p 1942-1952, Ghai 8/e p 220-224
Hepatitis B & Hepatitis C can present as chronic hepatitis.
34. d. Immunoglobulins and hepatitis-B-vaccine  Ref: Nelson’s 20/e p 1942-1952
“Infants born to HbsAg positive mothers should receive both Hep B Ig and Hep B vaccine at separate sites within 12 hours of
birth, followed by 2nd and 3rd dose of vaccine at 1 and 6 months.
35. a. Both active and passive immunization soon after birth  Ref: Nelson’s 20/e p 1942-1952
36. b. Obtain anti Hbs levels  Ref: Nelson’s 20/e p 1942-1952, Ghai 8/e p 220-224
The child in question has received complete vaccination for Hepatitis B, is currently asymptomatic but has tested positive for HBsAg.
In order to know whether the infection is acute or chronic, as well as for knowing the immune status of child, anti HBsAg levels
should be done next.
37. b. Hepatitis C virus  Ref: Nelson’s 20/e p 1942-1952, Ghai 8/e p 220-224
38. b. HbsAg and HbeAg positivity  Ref: Nelson’s 20/e p 1942-1952, Ghai 8/e p 220-224
•• Most important risk factor that determines perinatal transmission of Hepatitis B is HBeAg
•• HBsAg carrier mothers who are HBe Ag positive almost invariably (> 90%) transfer infection to their offsprings.
Chapter 18
Pediatric Respiratory
Disorders
E m b ryo lo g i c a l basis
High Yield Points
•• Lung develops from outpouching
of foregut during 4th week of Differences between larynx of an infant/child & an adult:
gestation •• Lumen is short, funnel shaped & narrower than of adult.
•• Stages in lung development are: •• At rest, the upper border of infant‘s epiglottis is at the level of 2nd-3rd cervical vertebra (higher),
Embryonic stage which enables an infant to use its nasal airway to breathe while sucking.
↓ •• Subglottic area is the narrowest portion of the upper airway in children < 10 year of age.
Pseudoglandular or canalicular stage

Saccular stage  CONGENITAL MALFORMATIONS OF LUNGS

Alveolar stage
1. PULMONARY HYPOPLASIA

Defective development of one or both lungs.

2. PULMONARY SEQUESTRATION
•• Discrete area of lung tissue that lacks any connectionQ to the airway system
Question 1 •• Abnormal blood supply arising from aortaQ
Identify the congenital lung abnor-
mality seen in this chest X-ray of a Extralobar sequestration Intralobar sequestration
2 year child
•• External to lungs •• Occur within the lung more commonQ

•• With pleural cover •• Without pleural cover

•• Causing mass effect •• Localized infection or bronchiectasisQ


•• Venous return to right side of heart through IVC •• Venous drainage is through pulmonary veins

High Yield Points


a. Congenital cystic adenomatoid Intralobar sequestration is asso­ ciated with diaphragmatic hernia, colonic duplication, verte­
bral
malformation abnormalities & pulmo­nary hypoplasia.
b. Congenital lobar emphysema
c. Congenital diaphragmatic hernia
d. Pulmonary sequestration 3. CONGENITAL LOBAR EMPHYSEMA (CLE)

Definition Over-distension of 1 or more lobes of lung

Etiology Congenital deficiency of bronchial cartilage, compression by aberrant vessels,


bronchial stenosis, or kinking of bronchus can lead to bronchial obstruction & CLE

Characteristics •• Left upper lobe is the most common site


High Yield Points •• Affected lobe is essentially nonfunctional because of over-distention & atelectasis
Foregut Cysts of ipsilateral normal lung can ensue
•• Abnormal detachments of primitive Clinical •• Mild tachypnea & wheeze to severe dyspnea & cyanosis in neonatal period
foregut manifestations
•• Most often located in the hilumQ or
middle mediastinum Diagnosis A radiolucent lobe with mediastinal shift is seen on Chest X-ray
•• 3 types: Bronchogenic (most
common),Q Esophageal or Enteric Treatment Immediate surgery & excision of lobe may be lifesaving in severe respiratory distress
Chapter 18: Pediatric Respiratory Disorders 397

4. CONGENITAL CYSTIC ADENOMATOID MALFORMATION (CCAM)

Section 3: Systemic Pediatrics


Question 2
Now called Congenital pulmonary airway malformation (CPAM) A 4 week baby presents with noisy
Definition Hamartomatous or dysplastic lung tissue, generally confined to 1 lobe breathing, which worsens on
crying & feeding, while improves
Basic defect Injury before 35th day of gestation, with maldevelopment of terminal bronchioles
when the baby is made to lie in
X-ray Cystic mass, sometimes with mediastinal shift prone position. Laryngoscopy
Clinical Patients can present in the newborn period or early infancy with respiratory distress, shows the following picture. What
features recurrent respiratory infection, and pneumothorax is the diagnosis?

Treatment Antenatal intervention or Postnatal, surgery for symptomatic patients

 CAUSES OF STRIDOR

Laryngeal obstruction Extrinsic masses


•• Laryngomalacia •• Mediastinal masses incl. Thyroid enlargement
•• Papillomas, other tumors •• Vascular ring
•• Cysts and laryngoceles •• Lobar emphysema
•• Laryngeal webs •• Bronchogenic cysts
•• Bilateral abductor paralysis of the cords •• Esophageal foreign body
•• Foreign body •• Tracheoesophageal fistula a. Laryngomalacia
b. Choanal atresia
Tracheobronchial disease Others c. Laryngocele
•• Tracheomalacia •• Gastroesophageal reflux d. Vocal cord papilloma
•• Subglottic tracheal webs •• Pierre Robin syndrome
•• Endobronchial, endotracheal tumors •• Hypocalcemia
•• Subglottic tracheal stenosis •• Vocal cord paralysis

LARYNGOMALACIA M
High Yield Points
•• Choanal atresia is the MC con­genital
High Yield Points
anomaly of nose
•• MC congenital laryngeal anomaly is laryngomalacia •• In choanal atresia, respiratory
distress improves while crying
•• Most common cause of stridor in infants children is laryngomalacia

Clinical Features
•• Stridor is inspiratory, low pitched & exacerbated by crying, agitation, or feeding
•• Results from the collapse of supraglottic structures inwards during inspiration
•• Symptoms appear within first 2 weeks of life increase in severity for up to 6 months.
•• Stridor improves when the baby sleeps in prone position. Question 3
What is the most common bacteria
Diagnosis: Confirmed by flexible laryngoscopy.
responsible for the following con-
Treatment: Expectant observation; Most symptoms resolve spontaneously dition seen in a child who presents
with high grade fever and pain in
the throat?
 INFECTIONS OF RESPIRATORY TRACT

ACUTE PHARYNGITIS
Definition Inflammation of pharynx (erythema, edema, exudates, ulcers)

Clinical Streptococcal Pharyngitis is suspected when a child presents with acute pharyngitis
features with fever, tonsillar exudates, palatal petechiae, cervical adenopathy

Diagnosis Throat swab RADT (rapid antigen detection test) and culture

Treatment Penicillin/amoxicillin for 10 days a. Group A streptococcus


b. Group B streptococcus
Complications • Rheumatic fever c. Corynebacterium diphtheria
•• Acute Glomerulonephritis, •  Peritonsillar abscess d. Staphylococcus aureus
398 Section 3: Systemic Pediatrics

LARYNGOTRACHEOBRONCHITIS (CROUP)
Review of Pediatrics and Neonatology

Most Common

Most common What is it?


Condition
cause •• Laryngotracheitis refers to viral infection of glottic & subglottic regions
Common cold RhinovirusQ •• Laryngotracheobronchitis is a more severe form, that is considered an extension of
Pharyngitis Adeno V, Strepto laryngotracheitis associated with bacterial superinfection that occurs 5-7 days later.
pyogenes
Etiology: Viruses typically cause croup esp parainfluenza virus. M
Croup Parainfluenza virusQ
Whooping B. pertussis Symptoms M
cough
•• Other family members might have mild respiratory illnesses with laryngitis
Bronchiolitis Respiratory syncytial
virusQ Examination Findings
•• Hoarse voice, coryza, mild tachypnea ± inflammation of pharynx
•• Hypoxia is seen only when complete airway obstruction is imminent.

Classification of Severity of Croup

Mild Moderate Severe


General appearance Happy, feeds well, interested in Irritable but can be comforted by Restless or agitated or altered sensorium
surrounding parents
Stridor Stridor on coughing Stridor at rest & gets worse when Stridor at rest & worsens on agitation
No stridor at rest agitated

Respiratory distress No distress Tachypnea & chest retractions Marked tachypnea with chest retractions

Oxygen saturation > 92% in room air > 92% in room air < 92% in room air

Investigations
Question 4
A 20-month-old child presents •• Croup is a clinical diagnosis & does not require a radiograph of neck
at 2 am with running nose, low- •• X-ray neck can show the typical subglottic narrowing, or 'steeple sign'.
grade fever for 3 days, along with
“barking” cough, hoarseness & High Yield Points
stridor, only on crying. What is the
Indications for Hospitalisation in Croup:
following sign seen on the imaging
of his upper airways? Progressive stridor, severe stridor at rest, respiratory distress, hypoxia, cyanosis, depressed mental status,
poor oral intake, or the need for reliable observation

Treatment M

(i) Airway management and treatment of hypoxia


(ii) Nebulized racemic (1:1 mixture of d- & l-isomers) epinephrine
•• Used in moderate or severe croup; duration of action is <2 hrs
•• Acts by constriction of precapillary arterioles through a-adrenergic receptors, causing
fluid resorption from interstitial space decrease in laryngeal mucosal edema.
(iii) Corticosteroids
•• Decreases edema in laryngeal mucosa through their anti-inflammatory action.
•• Oral dexamethasone used as a single dose of 0.15-0.6 mg/kg, reduces the need for
a. Thumb sign & duration of hospitalization & need for epinephrine administration.
b. Steeple sign (iv) A helium-oxygen mixture (heliox) may be considered in severe croup.
c. Signet ring sign
d. Omega sign High Yield Points
•• Croup is the most common form of acute upper respiratory obstruction.
•• Oral Dexamethasone is beneficial, even in mild croup
•• Antibiotics are not indicated in croup.
•• Epiglottitis usually has more acute onset and rapid course than croup.
Chapter 18: Pediatric Respiratory Disorders 399

EPIGLOTTITIS

Section 3: Systemic Pediatrics


M
Question 5
Etiology: H influenzae type B, Pneumococcus, β-hemolytic Streptococcus and Staphylococcus. A 5-year-old child presents with
high fever, sore throat, dyspnea &
L at e s t U p d at e s stridor. The lateral X-ray of neck &
finding seen on laryngoscopy are
•• In the past, H. influenzae type b was the most common cause of acute epiglottitis. given below. Identify the condition
•• With the widespread use of H. influenzae type b vaccine, invasive disease caused by H. Influenzae type b in this child is suffering from?
children has reduced drastically
•• Currently, Streptococcus pyogenes, Streptococcus pneumoniae, non-typeable H. Influenzae & Staphylococcus
aureus are commoner causes of epiglottitis in vaccinated children.

Clinical Features
•• Severe condition characterized by acute rapidly progressive course of high fever, sore
throat, dyspnea, and rapidly progressing respiratory obstruction
•• Drooling, neck is hyperextended in an attempt to maintain in airway
•• Child may assume the tripod position, sitting upright and leaning forward with the chin up.

Treatment
•• Medical emergency and warrants immediate artificial airway
a. Croup
•• IV Ceftriaxone, Cefotaxime, or Meropenam should be given, pending culture reports b. Acute bronchiolitis
•• Endotracheal intubation or tracheotomy is required for most patients with epiglottitis. c. Acute tracheitis
d. Acute epiglottitis

High Yield Points


In epiglottitis
•• Laryngoscopy shows ‘cherry red’ swollen epiglottis
•• X-ray neck lateral view shows ‘thumb sign’
•• Epinephrine and steroids are not effective

BACTERIAL TRACHEITIS
•• Etiology: Staph aureus, Moraxella catarrhalis, non typeble H influenzae, anaerobes L at e s t U p d at e s
•• Mean age affected is between 5 and 7 years L-epinephrine has now been found
•• Clinical features: Child has a brassy cough, high fever toxicity, respiratory distress to have similar efficacy as racemic
•• Treatment: Appropriate antibiotic therapy (esp including anti staph. agents) should be epinephrine with no additional side
initiated. Intubation or tracheotomy may be necessary. effects

ACUTE BRONCHIOLITIS
Epidemiology •• A Common cause of wheezing in infants
•• Usually occurs in winter and spring
•• Usually affects children between 1 and 6 months, but can affect up to 2 years
Etiology Respiratory syncytial virus (most common); Others: Parainfluenza virus,
adenovirus, influenza virus, adenovirus, influenza viruses and M pneumoniae
Clinical Features •• Prodrome of URI; Later: breathing becomes fast & respiratory distress develops
•• Majority recover in 3–7 days; X-ray chest shows hyperinflation & infiltrates
Treatment •• Symptomatic: Antibiotics have no role
•• Moist oxygen inhalation remains the mainstay of treatment
•• b2 adrenergic drugs & ipratropium are not recommended for infants < 6 months
•• Ribavirin, an antiviral agent, shortens the cause of illness in infants with congenital
heart disease, chronic lung disease and immunodeficiency
•• RSV Ig & palivizumab (monoclonal antibody to the RSV F protein), should be
considered for infants with chronic lung disease, prematurity or CHD
Most Common
•• Most common cause of common
L at e s t U p d at e s
cold is Rhinovirus
Emerging pathogens responsible for acute bronchiolitis include human metapneumovirus and human •• Most common complication of a
bocavirus cold is acute otitis media
400 Section 3: Systemic Pediatrics

Clinical Clues about Cough


Review of Pediatrics and Neonatology

M
Mnemonic  M
General danger signs include: Characteristic Think of
C-L-I-V-U-S Staccato, paroxysmal Pertussis, cystic fibrosis, foreign body, Chlamydia spp., Mycoplasma spp
•• Convulsions Followed by “whoop” Pertussis
•• Lethargy
•• Inability to drink or breast feed Barking, brassy Croup, tracheomalacia, tracheitis, epiglottitis
•• Vomiting Hoarseness Laryngeal involvement (croup, recurrent laryngeal nerve involvement)
•• Unconsciousness Abrupt onset Foreign body, pulmonary embolism
•• Sleepiness
Follows exercise Reactive airway disease
With eating, drinking Aspiration, gastroesophageal reflux, tracheoesophageal fistula
Throat clearing Postnasal drip
Seasonal Allergic rhinitis, reactive airways disease

Common organisms responsible for cough in:


•• Immunosuppressed patient: P. carinii, M. tuberculosis, M. avium–intracellulare, CMV
Most Common •• Animal exposure: Chlamydia psittaci (birds), Yersinia pestis (rodents),Q fever (sheep,
cattle), histoplasmosis (pigeons).
Category Organism
Bacterial Streptococcus  PNEUMONIA M
pneumonia in pneumoniae
children Etiology: See table of high yield points.
2nd MC cause Hemophilus
of bacterial influenzae type b
Chest X-ray Findings
pneumonia in •• Pneumococcal pneumonia: LobarQ consolidation > patchy or interstitial
children •• Staphylococcal pneumonia: Pneumatoceles,Q Empyema
Viral pneumonia Respiratory •• Streptococcal pneumonia: Interstitial pneumonia, segmental involvement, diffuse peribron-
in children Syncytial Virus chial densities or an effusion.
(RSV)
Treatment
Pneumonia in Group B strepto­
neonates coccus, Escherichia •• Antibiotics: Penicillin G or amoxycillin or ampicillin
coli •• In penicillin allergy, alternatives: Chloramphenicol or ceftriaxone or cefotaxime
Pneumonia in Pneumocystis •• Additional antibiotics to cover Staph: Cloxacillin, Vancomycin or teicoplanin
infants with HIV jirovecii •• Empyema is drained.

Differences between Typical & Atypical Pneumonia


Typical Pneumonia Atypical Pneumonia
Prodrome Short (<2 days) Long (>3 days) (headache, malaise, body aches)
Fever High (>102°F [ 38.9°C]) Low (<102°F [38.9°C])
Chest radiograph One distinct lobe involved Diffuse or multilober involvement
Etiology Streptococcus pneumoniae Mycoplasma, Chlamydia spp.
Antibiotic Ceftriaxone, broad Macrolides (e.g., azithromycin), doxycycline,
spectrum or certain fluoroquinolones (e.g., levofloxacin,
moxifloxacin)

•• In atypical pneumonia, chest examination maybe relatively benign, with few crackles and
rhonchi, while CXR shows significant pneumonia (b/l reticular infiltrates)
•• Macrolide/Tetracycline antibiotics are the recommended treatment of choice.

 ACUTE RESPIRATORY INFECTION


Cut-off for Fast Breathing M

Age (in months) Resp. Rate per minute


<2 ≥ 60Q
2–12 ≥ 50Q
12–60 ≥ 40
Chapter 18: Pediatric Respiratory Disorders 401

Previous Categories of ARI Programme M

Section 3: Systemic Pediatrics


Question 6
A 10-year-old child presented with
Diagnosis Clinical features New WHO ARI classification (2014) M
high grade fever, cough & respira-
Pneumonia Fast breathing Diagnosis Clinical features tory distress. On examination, bi-
lateral crackles were present. CXR
Severe Pneumonia Fast breathing + Chest indrawing Pneumonia Fast breathing
picture is shown below. What is
and / or chest
Very severe disease Fast breathing + general danger signs indrawing the most probable etiological agent
  responsible?
Severe
pneumonia or General danger
Very severe signs present
disease

Latest updates: IMNCI protocol, 2018

Signs Classify as Identify Treatment


•• Any general danger Severe pneumonia •• Give first dose of ampicillin/oral amoxycillin and a. Streptococcus pneumoniae
sign or or very severe intramuscular gentamicin. b. Mycoplasma pneumoniae
•• Stridor in calm child disease •• Refer URGENTLY to hospital. c. Staphylococcus aureus
d. Chlamydia pneumoniae
•• Chest indrawing or Pneumonia •• Give Amoxycillin for 5 days.
•• Fast breathing •• If wheezing (or disappeared after rapidly acting
bronchodilator) give an inhaled bronchodilator for
5 days
•• Soothe the throat and relieve the cough with a safe
L at e s t U p d at e s
remedy if child is 6 months or older.
•• If coughing more than 14 days or recurrent wheeze, Use of pulse oximeter & bronchodilators
refer for possible TB or asthma assessment are new additions in the latest IMNCI
guidelines for a child with cough or
•• Advise mother when to return immediately.
difficulty in breathing
•• Follow-up in 2 days.
•• If Oxygen saturation <90% by Pulse Oximeter
refer urgently

No signs of No pneumonia: •• If wheezing (or disappeared after rapidly acting


pneumonia or very cough or cold bronchodilator) give an inhaled bronchodilator for
severe disease. 5 days
•• Soothe the throat and relieve the cough with a safe
home remedy if child is 6 months or older.
•• If coughing more than 14 days or recurrent wheeze,
refer for possible TB or asthma assessment
•• Advise mother when to return immediately.
•• Follow-up in 5 days if not improving.

LOEFFLER’S SYNDROME

New name Transient pulmonary infiltrates with eosinophilia syndrome

Epidemiology Most often seen in children; Caused by larvae of nematodes like Ascaris

Pathogenesis •• Larvae pass through portal circulation, liver, hepatic vein & IVC into heart & lungs.
•• In the lungs, larvae penetrate the capillaries, enter the alveoli, plug the bronchi
with mucus and eosinophilic material due to allergic reaction
•• Cause fleeting patchy pulmonary infiltrates.
•• May also be due to drug reaction to aspirin, penicillin, sulphonamide.

Clinical Features •• Cough, low grade fever & scattered crepitations along with eosinophilia
•• CXR may resemble miliary tuberculosis
402 Section 3: Systemic Pediatrics

 ASTHMA
Review of Pediatrics and Neonatology

High Yield Points


•• FEV1/FVC < 0.8 is a sensitive Definition
indicator of airway obstruction
•• Diurnal variation of > 20% in PEFR, Increased responsiveness of the airways to various stimuli.
80% of predicted improvement of 20%
after bronchodilator are suggestive of
asthma Classification
•• PFTs play an important role in
monitoring response to treatment •• Extrinsic (IgE mediated, triggered by allergens) &
•• Intrinsic (non-IgE mediated, triggered by infection).

Clinical Features
Paroxysmal dyspnea, wheezing or cough.

Diagnosis M

•• 3 distinct episodes of wheezing or prolonged cough with response to anti-asthmatics


•• Bronchial reversibility to antispasmodics
•• Diagnosis is clinical in most cases; chest X-ray may be normal
•• Eosinophilia indicates allergic component; but skin test & RAST have limited usefulness.

Step-wise Treatment of Asthma

Day time Night time


symptoms symptoms PEFR Long term prevention

Step 1 < 1 time a week, < 2 times a month > 80% Inhaled short acting β agonist
Inter­ asymptomatic pre­dicted, SOS
mittent and normal PEFR variability < If they are needed more than 3
between attack 20% times a week, move to step 2.

Step 2 Mild > 1 times a week > 2 times a month > 80% variability Inhaled short acting β agonist
persistent but <1 time a day 20–30% SOS + low dose inhaled
steroid + sustained release +
leukotriene modifiers

Step 3 Daily use β2 > 1 times a week 60–80% Inhaled short acting β agonist
Moderate agonist, attacks pre­dicted; vari­ SOS + Medium dose inhaled
persistent affect activity ability >30% steroids + long acting β 2
agonist + sustained release
theophylline

Step 4 Continuous, Frequent < 60% Inhaled short acting β agonist


L at e s t U p d at e s
Severe limited physical predicted; SOS + inhaled corticosteroids
•• Omalizumab (anti IgE), Mepoli­ persistent activity variability < (High dose) + long acting
zumab (anti–IL-5 Ab) & Dupilumab
30% bronchodilator
(anti–IL-4 receptor Ab) are newer
drugs effective in Br Asthma ± oral steroids low dose
Alternate day

High Yield Points


•• Montelukast can be used in moderate or severe persistent asthma as an add on treatment in children >
1 year age.
•• Long acting β agonists e.g. salmeterol, Formoterol are used in children > 4 years of age.
Chapter 18: Pediatric Respiratory Disorders 403

 MANAGEMENT OF ACUTE SEVERE ASTHMA

Section 3: Systemic Pediatrics


Question 7
Identify the following device pre-
scribed for adminis­ tration of beta
agonist to a 5-year child with mod-
erate persistent asthma:

a. Rotahaler
b. Metered dose inhaler with spacer
c. AMBU
d. T-piece

 CYSTIC FIBROSIS High Yield Points


Autosomal recessive disease caused by an abnormal gene on the long arm of chromosome 7. •• Initially non-mucoid strains of
pseudomonas are found in
Defective Channels in Cystic Fibrosis M
airways of CF patient, which are
subsequently replaced by resistant
Primary defect: Abnormal function of an epithelial chloride channel encoded by Cystic mucoid strains of pseudomonas
Fibrosis Transmembrane conductance Regulator (CFTR) gene on chr7q31.2. •• Burkholderia cepacia is an un­
Other defects lie in: Chloride channelsQ, potassium channels (Kir 6.1)Q, Epithelial sodium common organism that uni­ quely
infects patients with CF
channel (ENaC)Q.

Effect of CFTR Gene Mutation on ENaC


Normal
Site In cystic fibrosis Effect seen in CF
Function
Apical exocrine glands Na+ uptake from Increased Na+ & H2O uptake Thick, viscid secretionsQ
of airway & GIT luminal fluid from lumen

Sweat ducts Absorb luminal Na+ Decreased reabsorption of Salty sweat (high Na+
Na+ & Cl– & Cl–Q (Hallmark
of CF)
404 Section 3: Systemic Pediatrics

Systemic Involvement
Review of Pediatrics and Neonatology

High Yield Points


Respiratory Tract M
•• Conditions associated with False
positive sweat chloride test: •• Upper respiratory tract disease is almost universal in patients with CFQ
•• Eczema –– Haemophilus influenzaeQ and S. aureusQ are the first organisms recovered from
•• Ectodermal dysplasia lung secretions; antibiotic-resistant mucoid P. aeruginosaQ colonizes next
•• Malnutrition –– Burkholderia cepacia is pathognomonic organism
•• Congenital adrenal hyperplasia •• Chronic coughQ and sputumQ production
•• Adrenal insufficiency •• BronchiectasisQ, atelectasisQ, infiltrates, hyperinflation, Nasal polypsQ.
Genitourinary System
•• Azoospermia and infertilityQ
•• Congenital bilateral absence or obliterationQ of vas deferensQ, may be seen.
Gastrointestinal and Nutritional Abnormalities
•• Intestinal: meconium ileusQ, distal intestinal obstructionQ, rectal prolapse
•• Pancreatic: pancreatic exocrine insufficiencyQ, recurrent acute / chronic pancreatitisQ
•• Hepatic: focal biliary cirrhosis, or multi lobular cirrhosis, prolonged neonatal jaundice
•• Nutritional: failure to thrive, hypoproteinemia, edema, fat-soluble vitamin deficiency
•• Salt-loss syndromes: acute salt depletion, chronic metabolic alkalosisQ.
Diagnostic Criteria for Cystic Fibrosis
Any of the following: Plus any one of:
•• ≥ 1 characteristic phenotypic features •• Increased sweat chloride (> 70 mEq/L on ≥ 2
•• History of cystic fibrosis in a sibling occasions)
•• +ve newborn screening test (Serum •• 2 cystic fibrosis mutations
immunoreactive trypsinogen) •• Abnormal nasal transepithelial potential difference

Treatment
•• Pulmonary: Bronchodilators, steroids, hypertonic saline, aerosolized antibiotics, ventilator
assistance may be required in cases of respiratory failure
High Yield Points •• Nutritional: High caloric duct, Pancreatic enzyme replacement, fat soluble vitamin (A,D,E,K)
•• Kartagener syndrome = supplementation, Mineral supplements
bronchiectasis, sinusitis, and situs •• Treatment of Bowel complications: like Meconium ileus, intestinal obstruction, chronic
inversus;Q appendicitis, pancreatitis, hyperglycemia, Biliary cirrhosis.
•• Kartagener syndrome is seen in
50% patients with primary ciliary  PRIMARY CILIARY DYSKINESIA M
dyskinesia
•• It is an inherited disorder characterized by impaired ciliary function
•• Diverse clinical manifestations, including chronic sinopulmonary disease, persistent middle
ear effusions, laterality defects (situs inversus or dextrocardia) & infertility.
Question 8
Identify the lung abnorma­ lity in  BRONCHIECTASIS
this 10-year-old girl with chronic
cough with purulent sputum Definition Destruction of smooth muscle and elastic tissue by chronic necrotizing infection,
that leads to permanentQ dilation of bronchi and bronchiolesQ
Etiology •• Idiopathic
•• Congenital/hereditary: Eg cystic fibrosis,Q intralobar sequestration,Q primary ciliary
dyskinesia & Kartagener syndromeQ
•• Infections-Bacterial (TuberculosisQ, Staph aureus), viral (Influenza) and fungal (Aspergillus)
•• Bronchial obstruction-tumor, mucus plug, Foreign bodyQ
•• Others-Rheumatoid ArthritisQ, SLE, IBD, GVHDQ
Gross •• Involves lower  lobes  bilaterallyQ
Morphology •• Airways are dilated, sometimes up to four times normal sizeQ
Clinical •• Cough with production of copious purulent sputum
features •• Hemoptysis, clubbing & fever (with exacerbations) may be seen
a. Lung abscess b. Sequestration •• Crackles localized to the affected area, but wheezing may also occur
c. Foreign body d. Bronchiectasis Diagnosis M •• Chest X-ray may show cystic spaces, occasionally with air–fluid levels & honeycombing
•• Thin-section HRCT scanning is the gold standard & has excellent sensitivity & specificity
•• CT chest shows cylindrical (“tram lines,” “signet ring appearance”), varicose (bronchi
with “beaded contour”), cystic (cysts in “strings & clusters”), or mixed lesions
Treatment Chest physiotherapy (postural drainage), antibiotics & bronchodilators are used
Chapter 18: Pediatric Respiratory Disorders 405

PULMONARY TUBERCULOSIS Refer chapter on infectious diseases.

Section 3: Systemic Pediatrics


 FOREIGN BODY (FB) ASPIRATION

Epidemiology M

•• Most victims of foreign-body aspiration are older infants and toddlers


•• Most common objects that children choke on are food, coins, balloons, and toys.
•• One-third of aspirated objects are nuts, particularly peanuts.
•• Most serious complication of foreign-body aspiration is complete obstruction of airway.

Clinical Manifestations
3 stages of symptoms may result from aspiration of an object into the airway: Question 9
1. Initial event: Violent coughing, choking, gagging immediately when FB is aspirated. A 4-year-old child developed sudden
2. Asymptomatic interval: FB becomes lodged, reflexes fatigue & irritating symptoms subside. bouts of cough & respiratory distress
3. Complications: Obstruction, erosion, or infection develops to direct attention again to after eating peanuts. His C×R shows
presence of FB. Complications include fever, cough, hemoptysis, pneumonia & atelectasis. the following; Diagnosis?

Diagnosis M

Choking or coughing episodes accompanied by new onset wheezing are highly suggestive.
The patient is asymptomatic and the radiograph is normal in 15-30% of cases (CT may
help).
Opaque foreign bodies occur in only 10-25% of cases.

Treatment
Treatment of choice is prompt removal with rigid instruments.

High Yield Points


a. Obstructive emphysema right side
•• Complete airway obstruction by foreign body is recognized in the conscious child as sudden respi­ratory b. Obstructive emphysema left side
distress followed by inability to speak or cough c. Pneumonia
•• Most airway foreign bodies lodge in a bronchus (right bronchus more common). d. Bronchiectasis

Answer Keys for Image-Based Questions

Answers Explanations / Identifying features


1. Ans a. Congenital cystic Cystic areas in right lung of a child suggestive of Congenital cystic adenomatoid malformation (CCAM)
adenomatoid malformation
2. Ans a. Laryngomalacia Noisy breathing in a 4 week baby which worsens on crying & feeding, with Laryngoscopy showing
omega shaped epiglottis is suggestive of Laryngomalacia
3. Ans a. Group A streptococcus Enlarged & congested tonsils with palatal petechiae, in a symptomatic child, are suggestive of acute
pharyngotonsillitis due to Streptococcus viridans
4. Ans b. Steeple sign Steeple sign is seen in acute laryngotracheobronchitis (Croup)
5. Ans d. Acute epiglottitis Thumb sign on lateral X-ray neck & presence of red, swollen epiglottis (‘cherry red epiglottis’) on
laryngoscopy in a child with high fever, suggests a diagnosis of acute epiglottis;
6. Ans c. Staphylococcus aureus Chest X-ray showing pneumonia with pneumatoceles suggests pneumonia due to Staphylococcus
aureus
7. Ans b. Metered dose inhaler with This is a metered dose inhaler with spacer, used for bronchial asthma
spacer
8. Ans d. Bronchiectasis This CT chest shows large cystic areas, suggestive of Bronchiectasis
9. Ans a. Obstructive emphysema This chest X-ray shows increased radiolucency of right hemithorax suggestive of hyperinflated chest on
right side right side;
406 Section 3: Systemic Pediatrics
Review of Pediatrics and Neonatology

Questions
 CONGENITAL MALFORMATIONS  INFECTION OF AIRWAYS
1. Congenital lobar emphysema is commonly seen in 11. A 10-month-old child presents with an abnormal
which lobe? (JIPMER May 2017) inspiratory sound; mother complaints that the sound
a. Left lower b. Right upper increases with cry, and decreases when the child
c. Left upper d. Right lower sleeps in prone position. What would be the probable
2. Which one of the following life-threatening congenital diagnosis? (JIPMER May 2016)
anomaly in newborn presents with polyhydramnios, a. Croup b. Acute epiglottitis
aspiration pneumonia, excessive salivation and difficulty c. Laryngitis d. Laryngomalacia
in passing a nasogastric tube? (APPG 2016) 12. A 5-year-old unimmunised child presents with pseu-
a. Gastroschisis b. Diaphragmatic hernia domembrane in throat, fever and sore throat, the organ-
c. Tracheo-esophageal fistula ism responsible is: (Recent Question 2016)
d. Choanal atresia a. Comma shaped gram negative bacilli
3. Kartagener syndrome includes all of the following b. Pleomorphic gram positive bacilli
except: (WB PGMEE 2016) c. Spore forming gram positive bacilli
a. Dysphagia b. Bronchiectasis d. Comma shaped gram positive bacilli
c. Sinusitis d. Dextrocardia 13. 2 week old, otherwise well baby is presented with history
4. The most common cause of stridor in infant and children of stridor in supine position-what is the treatment?
is: M  (MAHA PGM CET 2015)  (Recent Question 2016)
a. Congenital subglottic stenosis a. Oral calcium b. Wait and watch
b. Laryngomalacia c. Nebulization d. Antibiotic
c. Vocal Cord Paralysis d. Foreign body in airway 14. Steeple sign is seen in: (Recent Question 2016)
5. Respiratory system develops from: a. Acute epiglottitis
 (Recent Question 2015) b. Laryngotracheobronchitis (croup)
a. Ventral wall of foregut b. Dorsal wall of foregut c. In both cases d. None of above
c. Ventral wall of midgut d. Dorsal wall of midgut 15. All are common causes of stridor except: (APPG 2015)
6. Most common type of foregut cysts are? a. Foreign body in larynx and trachea
 (Recent Question 2014) b. Laryngomalacia
a. Bronchogenic b. Esophageal c. Atrophic laryngitis
c. Enteric d. Mixed type d. Multiple papillomas of larynx
7. A neonate presents with respiratory distress with 16. A child with fever and barky cough presented to emer-
enlargement of left upper lobe of lung and mediastinal gency at 3 am. His respiratory rate is 36/min, stridor
shift towards the right. What is the most likely diagnosis? only on coughing, fever 39 degrees celsius. No other
 (AIIMS Nov 2014) abnormality, What is the next logical step? M 
a. Cong lobar emphysema  (AIIMS Nov 2015)
b. Alpha 1 antitrypsin deficiency a. Dexamethasone b. Racemized epinephrine
c. Pneumonia d. Cystic fibrosis c. Do nothing, just reassure d. Give surfactant
8. Blood supply of bronchogenic sequestration is: 17. An 18 months child came with LRTI with Respiratory
 (Recent Question 2014) rate of 50/min. Child was not lethargic but irritable and
a. Aorta b. Pulmonary artery has stridor. Examination reveals bilateral rhonchi. SpO2
c. Pulmonary vein d. Bronchogenic artery is 88%. What is next best management? (JIPMER 2014)
9. Kartagener syndrome comprises of: (WBPG 2009) a. Give O2 by mask with child on the lap of the mother,
a. Chronic sinusitis, situs inversus and bronchiectasis along with beta agonist and adrenaline nebulization
b. Chronic sinusitis, situs inversus and bronchitis b. Examine throat, give O2 by mask along with beta agonist
c. Chronic sinusitis, situs inversus and chronic ottitis media nebulization with one dose of steroid
d. Chronic sinusitis, dextrocardia and bronchitis c. Immediate intubation and ventilate the child and establish
10. A 2-month-old infant has had inspiratory stridor since the IV/IO access
1st month of life, but has been otherwise well. Physical d. Urgent referral to tertiary care
examination is unremarkable except for moderate 18. A child with cyanosis and choking on feeding at birth
inspiratory stridor and retractions which are worse when was diagnosed as TEF and had underwent a corrective
the infant is supine or agitated and better when he is surgery. He comes to you with complaints of barking
prone and quiet. The most likely cause of these findings cough and expiratory wheeze. What is the most likely
is: (MAHA 2005) diagnosis? (AIIMS Nov 2014)
a. Reactive airway disease b. Laryngomalacia a. Bronchial asthma b. Croup
c. Viral croup d. An aspirated foreign body c. Tracheomalacia d. Subglottic stenosis
Chapter 18: Pediatric Respiratory Disorders 407

19. A 4-year-old child presents to the emergency department c. Give racemic epinephnne

Questions
with respiratory difficulty and noisy breathing. On d. Order for a chest X-ray e. Give some decongestant
examination, X-ray shows thumb sign. The most 28. A girl child with fever, cough, dyspnea with X-ray
probable diagnosis is: (AIIMS May 2013) showing right lower lobe patchy consolidation, for
a. Recurrent papillomatosis b. Croup which treatment was given. After 8 weeks symptom
c. Epiglottitis d. Tonsillitis improved but X-ray showed more dense consolidation
20. A 2-year-old child was brought to the emergency involving the whole of the right lower lobe. What is the
department at 3 AM. The child presented with fever and next best line of investigation? (AIIMS Nov 2007)
cough. On examination, respiratory rate was 36/min, a. Bronchoscopy b. Culture from nasopharynx
temperature was 39°°C and saturation on pulse oximetry c. Barium esophagogram d. Allergic skin test
was 96%. The child had barking cough and stridor only 29. The most common etiological agent for acute
on crying. Otherwise, the child was hydrated, able to bronchiolitis in infancy is: M  (AI 2006)
drink and consolable. The next step in the management a. Influenza virus b. Para influenza virus
would be: (AIIMS May 2013) c. Rhino virus d. Respiratory syncytial virus
a. Single dose dexamethasone 30. Bronchitis in children is caused by: (PGI June 05)
b. Nebulized racemic epinephrine solution
a. H. influenzae b. RSV
c. Complete blood count and culture
c. Mycoplasma d. EBV
d. Nasal washings for influenza virus and RSV
e. Influenza virus
21. Most common cause of Bronchiolitis is: M  31. Which of the following is the etiological agent most
 (DNB Pattern 2013)
often associated with epiglottitis in children? 
a. RSV b. Adenovirus  (AIIMS May 2005, Nov 2004)
c. Hospitalize and treat d. Mycoplasma a. Steptococcus pneumoniae
22. Treatment of bronchiolitis includes all except: M  b. H. influenza type b
 (NEET Pattern 2013) c. Neisseria spp d. Moraxella catarrhalis
a. Macrolides b. Humid oxygen 32. A 6-month-old baby is brought with H/O increasing
c. Bronchodilator d. All of above difficulty in breathing of 2 days duration. On
23. Two months old child presented with fever, respiratory examination, the baby is afebrile but B/L wheeze is
distress, recurrent apnea followed by bradycardia and audible; CXR shows B/L hyperinflation of the lungs with
cyanosis. What is the probable diagnosis? normal WBC count, the diagnosis is: (PGI Dec 2003)
 (JIPMER 2012) a. Bronchiolitis b. Asthma
a. Congenital heart block b. Whooping cough c. Ch. Bronchitis d. Pneumonia
c. Tracheobronchiomalacia d. Epiglottitis e. Foreign body bronchus
24. A 3-month-old child has moderate fever and non­ 33. In bronchiolitis, following is/are seen: (PGI Dec 2002)
productive cough and mild dyspnea. After a course of a. Seen in children 5 months–3 years of age
antibiotic, the condition of the child improved transiently b. Caused by Strep. pneumoniae
but he again develops high fever, productive cough and c. CXR shows bilateral hyperinflation
increased respiratory distress. CXR shows hyperlucency d. Symptomatic treatment is given
and PFT shows obstructive pattern. Most probable e. Antibiotics should be started
diagnosis is: (AIPGMEE 2011; AIIMS May 2007) 34. Features of bronchiolitis are: (PGI June 2001)
a. Alveolar microlithiasis b. Postviral syndrome a. Caused by RSV b. Wheeze present
c. Follicular bronchitis d. Bronchiolitis obliterans c. Pulmonary edema common
25. What is the most frequent causative agent for acute d. Seen in immunocompromised hosts
epiglottitis in children under 5 years of age? M  e. Common age group is from 5–15 years age
a. Haemophilus influenzae type B (UPSC-I 10) 35. Which of the following is/are true about bronchiolitis in
b. Influenza virus type A children: (PGI June 2001)
c. Para influenza virus type 1 a. Caused by RSV
d. Staphylococcus aureus b. Hyperinflation of the chest
26. A child with 3 days history of upper respiratory tract c. Pleural effusion
infection presents with stridor, which decreases on lying d. May lead to bronchial asthma later in life
down position. What is the most probable diagnosis? e. Lymphopenia is seen
 (AI 2007) 36. A 9-month-old HIV positive child following URTI
a. Acute epiglottitis b. Laryngotracheobronchitis developed sudden onset of breathlessness. The chest
c. Foreign body aspiration d. Retropharyngeal abscess X-ray shows hyperinflation. The O2 saturation was greater
27. A 3 weeks male child born to a Primi mother, is than 90%; the treatment of choice is: (AIIMS May 2001)
presenting with noisy breathing sound. Child is afebrile, a. Cotrimoxazole b. Ribavarin
sleeping and feeding well and on clinical examination, c. IV ganciclovir d. Nebulized acyclovir
he is normal. Management protocol include: 37. An child with recent onset of URTI after 2 days presents
 (PGI Dec 08) with acute onset of breathlessness, cough and fever. All
a. Begin IV antibiotics of the following can be given except: M 
b. Reassure and give saline nasal drop  (AIIMS May 2001)
408 Section 3: Systemic Pediatrics

a. Antipyretics b. Morphine
Review of Pediatrics and Neonatology

46. A mother in a village has brought her 1 month baby with


c. Antibiotics d. O2 inhalation a complaint of fever to the PHC. You find that the baby's
38. An 11-month-old child presents with complaints of respiratory rate is 69/minute. The baby is able to feed,
respiratory distress. On examination there is bilateral does not have chest indrawing or stridor. There is no
crepitation and wheezing. Which of the following is the history of convulsions. According to the RCH program,
most likely cause?  (AIIMS Nov 2000) the baby's illness is classified as: (APPG 2015)
a. Pneumonia b. Adenovirus a. Severe pneumonia b. Very severe disease
c. RSV d. Rhinovirus c. No pneumonia: cough or cold
39. A 4-year-old child has seal barking like croupy cough. d. Pneumonia
Management includes all except: 47. When mother complains that baby is more comfortable
 (PGI June 2000) and breathes better when held against shoulder, what
a. O2 inhalation b. Antibiotic important clinical problem do you suspect? M 
c. Hydration d. Morphine a. Nose block (APPG 2014)
e. Salbutamol b. Psychological anxiety or insecurity
c. Aerophagy with abdominal fullness
 PULMONARY INFECTIONS d. Orthopnea
48. Under IMNCI programme in outpatient health facility,
40. Which of the following is abnormal?
all of the following are danger signs except: M 
 (NEET PG Jan 2019)
 (MAHA PGM CET 2014)
a. Respiratory rate > 40/min in a child between 2-12 months
a. Convulsions b. Difficult breathing
age
c. Vomiting d. Lethargy
b. Respiratory rate > 50/min in a child between 2-12 months
49. A 10-month-child weighing 5 kg and with 65 cm length,
age
presents with cough and cold. He was found to have
c. Respiratory rate > 30/min in a child between 1-5 years
a respiratory rate of 48 per minute with no retractions,
age
grunting, cyanosis. There is no history of convulsions.
d. Respiratory rate > 50/min in a child between < 2 months
Which is true? (AIIMS Nov 2014)
age
41. Which of the following clinical presentation is correct a. No pneumonia, only cough and cold
for a child with pneumonia? (FMGE Nov 2017) b. Child has pneumonia
c. Severe pneumonia d. Very severe disease
a. <50/min RR, with age of 2 months
50. Most dangerous sign in LRTI in children is: (JIPMER 2014)
b. >40/min RR, for 2-12 months
c. >40/min RR, for 12-60 months a. Abdominal breathing b. Chest retraction
d. <60/min RR, for 0-2 months c. Grunting d. Tachypnea
42. An 18-month-old child comes to subcenter with history 51. A 6-week baby presents with cough and cold for the
of fever and cough. On examination weight was past 3 days. Respiratory rate is 48/min. On examination,
11.5 kg, respiratory rate was 46/min, and there was patient is febrile with wheezing but no chest indrawing.
no chest indrawing. Which is the most appropriate Which of the following is not true?
management? M  (AIIMS May 2017)
 (AIIMS May 2014, AIIMS Nov 2013)

a. IV antibiotics at PHC and observe a. Child has pneumonia b. Antibiotics are not required
b. Give first dose of IV antibiotics & refer to higher center c. Wheezing should be treated
c. Oral cotrimoxazole for 5 days and send home d. Fever should be treated
d. Home remedies 52. Lecithin–Sphingomyelin ratio is used for: M 
43. Fast breathing in a child less than 2 months is defined as  (NEET Pattern 2013)
respiratory rate more than: M  (Recent Question 2017) a. Lung maturity b. Age of infant
a. 40 breaths per minute b. 50 breaths per minute c. Kidney maturity b. Brain maturity
c. 60 breaths per minute d. 70 breaths per minute 53. WHO criteria for hospital admission in pneumonia:
44. A 4-year-old malnourished child is brought to subcentre  (PGI Dec 2008)
with breathing rate of 55 per minute, excessive crying, a. High fever b. Nasal flaring
irritability, fever and not taking feeds. The ANM assesses c. Difficulty in breathing d. Difficulty in feeding
the child and categorizes under the IMNCI guidelines e. Chest indrawing
for management of ARI as? (COMEDK 2016) 54. Which of the following is the leading cause of mortality
a. No pneumonia b. Very severe disease in under 5 children in developing countries?
c. Pneumonia a. Malaria (AIIMS May 2005)
d. Upper respiratory tract infection b. Acute lower respiratory tract infections (LRTI)
45. Pneumatoceles in chest X-ray are characteristically seen c. Hepatitis d. Prematurity
in pneumonia due to: (APPG 2015) 55. An infant develops cough and fever. Chest X-ray is
a. Streptococcus pyogenes suggestive of broncho-pneumonia. All of the following
b. Haemophilus influenzae viruses can be the causative agents except: (AIPG 2004)
c. Streptococcus pneumoniae a. Parainfluenza viruses b. Influenza virus A
d. Staphylococcus aureus c. Respiratory syncytial virus
d. Mumps virus
Chapter 18: Pediatric Respiratory Disorders 409

56. A child with pyoderma becomes toxic and presents 66. A 3-year-old boy is brought to the casualty by his mother

Questions
with respiratory distress. His CXR shows patchy areas with progressive shortness of breath for 1 day. The child
of consolidation and multiple bilateral thin walled air has history of bronchial asthma. On examination, the
containing cysts. The most likely etiological agent in this child is blue, gasping and unresponsive. What will you
case is: (AIIMS Nov 2003) like to do first: (AIIMS Nov 2002)
a. Mycobacterium TB a. Intubate
b. Staph aureus b. Administer 100% Oxygen by mask
c. Mycobacterium avium intracellulare (MAC) c. Ventilate with bag and mask with oxygen
d. Pneumocystis jiroveci d. Administer nebulized Salbutamol
57. Pneumothorax could be a complication of:
 (UPSC 2000)  CYSTIC FIBROSIS
a. Staphylococcal pneumonia
67. Which of the following is seen in cystic fibrosis?
b. Pneumococcal pneumonia
c. Klebsiella pneumonia d. Viral pneumonia a. Low sweat chloride levels (NEET PG Jan 2019)
b. Elevated sweat chloride levels
c. Low sweat sodium levels
 ASTHMA d. Elevated sweat potassium levels
68. Which of the following exocrine glandular ducts are not
58. Humanized monoclonal antibody that binds IgE to obstructed in cystic fibrosis? (Recent Question 2016)
prevent its binding to the high affinity IgE receptor and a. Pancreas b. Lung
blocking IgE mediated allergic response and inflam- c. Sweat gland d. All of above
mation in Bronchial Asthma is:(Recent Question 2017)
69. A child is brought to the pediatric OPD with fever of
a. Palivizumab b. Natalizumab 24 hours duration. History reveals 3 episodes of chest
c. Omalizumab d. Etilizumab infection and passage of foul smelling stools. The most
59. Which one of the following values is not a feature of probable diagnosis is: (AI 2012)
Acute Severe Asthma? (APPG 2015) a. Cystic fibrosis b. Maple syrup urine disease
a. Heart rate more than 110/min c. Bilirubin conjugation defect
b. PEF of 60 to 70% of expected d. Criggler Najar syndrome
c. Pulsus paradoxus d. PaO2 of less than 8 kPa 70. Most common organism associated with cystic fibrosis:
60. A child of less than one year should use the following  (AIIMS May 2011)
device for asthma treatment: (Recent question 2015) a. Pseudomonas aeruginosa (non mucoid)
a. MDI with Spacer b. MDI with Mask b. Burkholderia cepacia
c. MDI with Spacer with Mask c. Pleisomonas d. Aeromonas
d. MPI with Mask 71. Which of the following statements about cystic fibrosis
61. Which of the following is NOT recommended in (CF) is not true: (AIIMS May 2011)
immediate treatment of acute severe asthma? a. Autosomal recessive, disorder
a. Oxygen supplementation (UPSC CMS 2015) b. Abnormality in CFTR which leads to defective calcium
b. High doses of inhaled β2 –adrenoreceptor agonists transport
c. Systemic corticosteroids c. Mutation in cystic fibrosis transport regulator
d. Intravenous aminophylline d. CFTR protein has 1,480 amino acids
72. A child presents with respiratory distress and failure
62. In a child with exercise induced asthma, which is done: to thrive. His sweat chloride levels were estimated 35
a. Prophylaxis with steroids (Recent question 2014) mEq/L and 41 mEq/L. What is next best test to do cystic
b. Prophylaxis with beta agonist fibrosis for diagnosis: (AIIMS May 2011, Nov 2007)
c. Prophylaxis with Montelukast a. Nasal transmembrane potential difference
d. Breathing exercise b. DNA analysis ∆F 508 mutation
63. A 4-year-child with bronchial asthma presents history c. CT chest d. 72 hours fecal fat estimation
of 3 or more episodes during daytime and 2 wheezing 73. Cystic fibrosis is associated with: M  (JIPMER 2008)
episodes during night in a week. How will you grade this a. Meconium ileus b. Short bowel syndrome
asthma? (JIPMER 2013) c. Hirschsprung disease d. Congenital pyloric stenosis
a. Mild persistent b. Moderate persistent 74. Infant with cystic fibrosis (CF) are likely to develop: M 
c. Severe persistent d. Mild intermittent  (MAHA PG 07)
64. Most common mode of treatment of a 1 year child with a. Meconium ileus b. Loose motions
asthma is: (AIPG 2007) c. Vomiting d. Constipation
a. Inhaled short acting β2 agonist
b. Oral short acting theophylline  MISCELLANEOUS
c. Oral ketotifen d. Leukotriene agonist
75. When can one diagnose acute respiratory distress in a child?
65. Wheeze in children is caused by: (PGI June 06, 05)
a. Within 7 days of known insult (NEET pattern Jan 2018)
a. Foreign body
b. Respiratory failure not fully explained by cardiac failure
b. Gastroesophageal reflux disease or fluid overload
c. Bronchial asthma d. Epiglottitis c. No left ventricular dysfunction
e. Laryngomalacia d. All of the above
410 Section 3: Systemic Pediatrics
Review of Pediatrics and Neonatology

76. Kartagener's syndrome is characterized by all of the 79. Most common cause of ARDS in children is:
following except: M  (Recent Question 2017) a. Aspiration (Recent Question 2016)
a. Bronchiectasis b. Differential pulse b. Injury
c. Sinusitis d. Situs inversus c. Severe pneumonia with sepsis
77. A 4-year-old child is admitted in the ward with d. DIC
pneumonia. He develops sudden respiratory distress.
80. Pediatric airway differs from adult in having: M 
The following chest X-ray was recorded. What will you
 (Recent Question 2016)
do next? (AIIMS May 2016)
a. Large tongue
b. Short epiglottis
c. Narrowest part at glottis
d. Larynx in lower position
81. Level of trachea bifurcation in pediatric patient is: M 
 (Recent Question 2016)
a. T2 b. T3
c. T4 d. T5

a. Increase mechanical ventilation 82. “Loeffler's syndrome” is characterized by:


b. Decrease mechanical ventilation  (Recent Question 2016)
c. Immediate needle thoracostomy a. Transient, migratory pulmonary infiltrations
d. Intercostal chest drainage tube insertion b. Fibrosis in the pulmonary apices
78. A 16-year-male presents with rapidly developing bilateral c. Fibrosis in the base of one or both lungs
pleural effusion. The following chest radiograph was d. Miliary mottling
taken in the emergency. All are correct interpretation
about this radiograph except: (AIIMS May 2016) 83. In mechanical ventilation of a newborn with ARDS, the
end tidal volume is kept at? (Recent question 2014)
a. 5 mL/kg b. 7 mL/kg
c. 10 mL/kg d. 15 mL/kg
84. Apnea in infant is cessation of breathing for: M 
 (Recent question 2013)
a. > 10 sec b. > 20 sec
c. > 30 sec d. > 40 sec
85. In a child with foreign body in bronchus, next step is: M
a. Pneumothorax is present  (Recent question 2012)
b. Bilateral ICD tubes are present
a. Rigid bronchoscopy b. Chest X-ray
c. Mediastinum is shifted
d. Left sided pleural effusion is present c. Flexible endoscopy d. Direct laryngoscopy
Chapter 18: Pediatric Respiratory Disorders 411

Answers with Explanations


Answers with Explanations
 CONGENITAL MALFORMATIONS

1. c. Left upper  Ref: Nelson's 20/e p 2113-2116


2. c. Tracheo-esophageal fistula  Ref: Nelson's 20/e p 2457-2060
The given clinical picture with polyhydramnios, aspiration pneumonia, excessive salivation and difficulty in passing a
nasogastric tube point towards a diagnosis of trachea esophageal fistula.
3. a. Dysphagia  Ref: Nelson's 20/e p 2113-2116
Kartagener syndrome includes situs inversus, chronic sinusitis and otitis, and airway disease leading to bronchiectasis.
4. b. Laryngomalacia  Ref: Nelson's 20/e p 2036; Ghai 8/e p 369
Laryngomalacia is the most common cause of stridor in infants and children.
5. a. Ventral wall of foregut  Ref: Nelson's 20/e p 2113-2116
Respiratory system develops from: ventral wall of foregut.Q
6. a. Bronchogenic  Ref: Nelson's 20/e p 2113-2116
•• Foregut cysts are abnormal detachments of primitive foregut most often located in hilum or middle mediastinum.
•• 3 types: Bronchogenic (most common), Esophageal or Enteric.
7. a. Cong lobar emphysema  Ref: Nelson's 20/e p 2113-2116; Refer pretext of this chapter for details;
Congenital Lobar Emphysema (CLE)
Definition Over-distension of 1 or more lobes of lung
Characteristics Left upper lobe is the most common site; It is over-distended with mediastinal shift to opposite side;

8. a. Aorta  Ref: Nelson's 20/e p 2113-2116


•• Pulmonary sequestration refers to a discrete area of lung tissue that lacks any connectionQ to the airway system
•• It has abnormal blood supply arising from aorta.
9. a. Chronic sinusitis, situs inversus and bronchiectasis  Ref: Nelson's 20/e p 2113
10. b. Laryngomalacia  Ref: Nelson's 20/e p 2036, Ghai 8/e p 369
The given clinical picture of stridor in an otherwise well infant which improves in prone position and when the baby is quiet, suggests
a diagnosis of laryngomalacia.

 INFECTION OF AIRWAYS
11. d. Laryngomalacia  Ref: Nelson's 20/e p 2036; Refer pretext of this chapter for details
12. b. Pleomorphic gram positive bacilli  Ref: Nelson's 20/e p 1345
The child is most probably suffering from Diphtheria, caused by Corynebacterium diphtheria
Corynebacteria are aerobic, nonencapsulated, non-spore-forming, mostly nonmotile, pleomorphic, Gram-positive bacilli.
13. b. Wait and watch  Ref:  Ref: Nelson's 20/e p 2036
It is most probably a case of Laryngomalacia, that resolves spontaneously
14. b. Laryngotracheobronchitis (croup)  Ref: Nelson 20/e p 2032-2034
‘Steeple sign' due to subglottic narrowing is characteristically seen in Croup
15. c. Atrophic laryngitis  Ref: Nelson's 20/e p 2113-2116
Atrophic Laryngitis usually manifests with hoarseness & irritable cough; Rest 3 conditions can manifest as stridor.
16. a. Dexamethasone  Ref: Ghai Pediatrics 8/e p 376, 398; Nelson’s 20/e p 2032-2034
This child is suffering from Croup (Laryngotracheobronchitis): mild variety.
Oral Dexamethasone is beneficial, even in mild croup; Antibiotics are not indicated in croup
17. b. Examine throat, give O2 by mask along with beta agonist nebulization with one dose of steroid  Ref: Ghai Pediatrics 8/e
p 376, 398; Nelson’s 20/e p 2032-2034
This child has croup along with hypoxia (severe croup); So O2 supplementation along with nebulization with Adrenaline (beta
agonist) and single dose Dexamethasone (steroid) are indicated.
18. c. Tracheomalacia  Ref: Ghai Pediatrics 8/e 376, 398; Nelson’s 20/e p 2032-2034
•• Tracheomalacia is the rule following correction of tracheoesophageal fistula
•• Bronchomalacia is common following lung transplantation.
Clinical features of Tracheomalacia:
412 Section 3: Systemic Pediatrics
Review of Pediatrics and Neonatology

•• Stridor is inspiratory, low pitched and exacerbated by any exertion; crying, agitation, or feeding.
•• Results from the collapse of supraglottic structures inwards during inspiration.
•• Symptoms appear within the first 2 weeks of life and increase in severity for up to 6 months

19. c. Epiglottitis  Ref: Ghai Pediatrics 8/e p 376, 398; Nelson’s 20/e p 2032-2034
The given clinical picture along with the X-ray finding suggests a diagnosis of Acute Epiglottitis.
20. a. Single dose dexamethasone  Ref: Ghai Pediatrics 8/e p 376, 398; Nelson’s 20/e p 2032-2034
21. a. RSV  Ref: Nelson's 20/e p 2044-2047, Ghai 8/e p 381-382
Acute bronchiolitis
•• It is predominantly a viral disease and RSV is responsible for more than 50% of cases
•• Other agents include parainfluenza, adenovirus, rhinovirus, Mycoplasma, human metapneumovirus and bocavirus.
22. a. Macrolides  Ref: Nelson's 20/e p 2044-2047, Ghai 8/e p 381-382
Treatment of bronchiolitis:
•• Symptomatic; Moist oxygen inhalation remains the mainstay of treatment; Antibiotics have no role
•• Beta-2-adrenergic drugs and ipratropium are not recommended for infants < 6 months
•• Ribavirin, shortens the course of illness in infants with congenital heart disease, chronic lung disease & immunodeficiency
•• RSV Ig and palivizumab, (monoclonal antibody to the RSV F protein) should be considered for infants < 2 years of age with chronic lung
disease, a history of prematurity and congenital heart disease

23. b. Whooping cough  Ref: Nelson's 20/e p 2044-2047, Ghai 8/e p 381-382
Whooping cough in Infants:
•• Infants < 3 months of age do not display the classic stages of Whooping cough; Catarrhal phase is usually unnoticed,
•• In Paroxysmal stage:
–– A well-appearing young infant begins to choke, gasp, gag and flail the extremities, with face reddened
–– Cough may not be prominent, especially in the early phase and Whoop infrequently occurs
–– Apnea & cyanosis can follow a coughing paroxysm; Apnea may be the only symptom
•• Paradoxically, in infants, cough and whooping may become louder and more classic in convalescence.
24. d. Bronchiolitis obliterans  Ref: Nelson's 20/e p 2044-2047, Ghai 8/e p 381-382
Bronchiolitis obliterans (BO)
A chronic obstructive lung disease of bronchioles and smaller airways, that results from an insult to the lower
What is it?
respiratory tract leading to fibrosis of small airways
•• After the initial insult, inflammation affecting terminal bronchioles, respiratory bronchioles and alveolar ducts
can result in the obliteration of the airway lumen
Pathogenesis •• Epithelial damage resulting in abnormal repair is characteristic of BO
•• Complete or partial obstruction of the airway lumen can result in air trapping or atelectasis
•• Cough, fever, cyanosis, dyspnea, chest pain and respiratory distress followed by initial improvement
Clinical features •• It is easily confused with pneumonia, bronchitis, or bronchiolitis
•• Progression of disease with increasing dyspnea, chronic cough, sputum production and wheezing seen
•• Chest X-ray may be normal or can demonstrate hyperlucency and patchy infiltrates
Diagnosis •• Occasionally, a Swyer-James syndrome (unilateral hyperlucent lung) develops
•• PFT shows obstructive pattern

25. a. Haemophilus influenzae type B  Ref: Nelson's 20/e p 2032-2033; Refer to pretext for details
26. b. Laryngotracheobronchitis  Ref: Nelson's 20/e p 2032-2034, Ghai 8/e p 397-398
The given clinical features are suggestive of a diagnosis of Croup or Laryngotracheobronchitis.
27. b. Reassure and give saline nasal drop  Ref: Nelson's 20/e p 2048-2050
In this case, the baby appears to have no evidence of respiratory distress like tachypnea or retractions. The baby does not have any
pneumonia, but only some coryza. So the appropriate management here is Reassure and give saline nasal drops.
28. a. Bronchoscopy  Ref: Nelson's 20/e p 1193-1196, Ghai 8/e p 373
Persistence of symptoms in the form of x-ray showing more dense consolidation involving the whole of the right lower lobe after
treatment indicates some localized pathology in that area like foreign body.
This will need bronchoscopy for further management and evaluation.
29. d. Respiratory syncytial virus  Ref: Nelson's 20/e p 2044-2047, Ghai 8/e p 381-382
30. e. Influenza virus  Ref: Nelson's 20/e p 2047-2048, Ghai 8/e p 381-382
•• Nonspecific bronchial inflammation is termed bronchitis; It is usually viral in origin, with cough as a prominent feature;
•• It is most commonly caused by Influenza virus.
31. b. H. influenza type β  Ref: Nelson's 20/e p 2044-2047, Ghai 8/e p 381-382
Chapter 18: Pediatric Respiratory Disorders 413

32. a. Bronchiolitis  Ref: Nelson's 20/e p 2044-2047, Ghai 8/e p 381-382

Answers with Explanations


The given clinical picture with presence of dyspnea, wheeze, hyperinflation of lungs on chest X-ray and absence of fever and
leukocytosis, suggest a diagnosis of Bronchiolitis.
33. a. Seen in children 5 months–3 years of age, c. CXR shows bilateral hyperinflation, d. Symptomatic treatment is given
Ref: Nelson's 20/e p 2043-2047, Ghai 8/e p 381-382
•• Most common cause of acute bronchiolitis is Respiratory Syncitial Virus (RSV)
•• Treatment of Bronchiolitis is symptomatic and antibiotics are not indicated.
34. a. Caused by RSV, b. Wheeze present  Ref: Nelson's 20/e p 2044-2047, Ghai 8/e p 381-382
35. a. Caused by RSV, b. Hyperinflation of the chest, d. May lead to bronchial asthma later in life  Ref: Nelson's 20/e p 2044-
2047
•• X-ray chest in Bronchiolitis shows hyperinflation and infiltrates
•• Relationship of acute bronchiolitis to bronchial asthma observed in about 1/4th those with a family history of allergy.
36. b. Ribavarin  Ref: Nelson's 20/e p 2044-2047, Ghai 8/e p 381-382
This HIV positive child has clinical features of Acute Bronchiolitis
In immunocompromised child with Bronchiolitis, Ribavirin, an antiviral agent, shortens the course of illness.
37. b. Morphine  Ref: Nelson's 20/e p 2044-2047, Ghai 8/e p 381-382
Morphine or any other sedative is to be avoided in a child with respiratory distress.
38. c. RSV  Ref: Nelson's 20/e p 2044-2047, Ghai 8/e p 381-382
The clinical features are suggestive of Acute Bronchiolitis, for which the most common causative agent is RSV.
39. b. Antibiotic, d. Morphine, e. Salbutamol  Ref: Nelson's 20/e p 2044-2047, Ghai 8/e p 381-382
This child probably has Croup; Antibiotics, Morphine and Salbutamol are not indicated in the management of croup.

 PULMONARY INFECTIONS

40. b. Respiratory rate > 50/min in a child between 2-12 months age  Ref: Nelson's 20/e p 2088-2093, Ghai 8/e p 379-381
Cut-off for fast breathing:
Age <2 months 2–12 months 12–60 months
Respiratory Rate ≥ 60/min ≥ 50/min ≥ 40/min

41. c. >40/min RR, for 12-60 months  Ref: Nelson's 20/e p 2088-2093, Ghai 8/e p 379-381
42. c. Oral cotrimoxazole for 5 days and send home
Child has fast breathing with no danger signs; So he is suffering from Pneumonia. So, oral antibiotics need to be started & child can
stay at home
43. c. 60 breaths per minute
44. b. Very severe disease  Ref: Nelson's 20/e p 2088-2093, Ghai 8/e p 379-381
New ARI classification (2014) by WHO:

Diagnosis Clinical features


Pneumonia Fast breathing and/or chest indrawing
Severe pneumonia or Very severe disease General danger signs present

As the child is not able to feed, he has very severe disease; Refer pretext of this chapter for details;
45. d. Staphylococcus aureus  Ref: Nelson's 20/e p 2088-2093, Ghai 8/e p 379-381
Pneumatoceles in chest X-ray are characteristically seen in pneumonia due to Staphylococcus aureus.
46. d. Pneumonia  Ref: Nelson's 20/e p 2088-2093, Ghai 8/e p 379-381; Refer Ans. 39 above
47. d. Orthopnea  Ref: Nelson's 20/e p 2088-2093, Ghai 8/e p 379-381
Baby being comfortable in a vertical position indicates orthopnea.
48. b. Difficult breathing  Ref: Nelson's 20/e p 2088-2093, Ghai 8/e p 379-381; Refer pretext for details;
49. a. No pneumonia, only cough and cold  Ref: Nelson's 20/e p 2088-2093, Ghai 8/e p 379-381
Cut-off for fast breathing:

Age <2 months 2 – 12 months 12 – 60 months


Respiratory Rate ≥ 60/min ≥ 50/min ≥ 40/min
414 Section 3: Systemic Pediatrics

So this baby does not have fast breathing; So this child has No pneumonia, only cough and cold, even though the child is malnourished
Review of Pediatrics and Neonatology

(wt = 5 kg at 10 months against expected of 9 kg).


50. c. Grunting  Ref: Nelson's 20/e p 2088-2093, Ghai 8/e p 379-381
Of the above signs, grunting is most dangerous, as it indicates widespread lung involvement.
About Grunting:
•• It is produced by expiration against a partially closed glottis
•• It is attempt to maintain positive airway pressure during expiration for as long as possible.
Interpreting the Clinical Signs of Respiratory Disease
Extrathoracic airway Intrathoracic extrapulmonary Intrapulmonary airway Parenchymal
Sign obstruction airway obstruction obstruction pathology
Tachypnea + + ++ ++++
Retractions ++++ ++ ++ +++
Stridor ++++ ++ – –
Wheezing ± +++ ++++ ±
Grunting ± ± ++ ++++

Hence, grunting is most suggestive of lung Parenchymal pathology, so most severe disease.
Also note, tachypnea is a very non-specific sign that can be seen in many non-respiratory conditions also.
51. a. Child has pneumonia  Ref: Nelson's 20/e p 2088-2093, Ghai 8/e p 379-381
This child does not have fast breathing; So there is no pneumonia; Hence antibiotics are not required;
52. a. Lung maturity  Ref: Nelson's 20/e p 2088-2093, Ghai 8/e p 379-381
By 35 wk of gestation, the lecithin: sphingomyelin (L: S) ratio averages about 2:1, indicative of lung maturity.
53. c. Difficulty in breathing, d. Difficulty in feeding  Ref: Nelson's 20/e p 2088-2093, Ghai 8/e p 379-381
Indications of hospitalization in a child with pneumonia

•• Age <6 months/Immunocompromised state •• Complicated pneumonia


•• Sickle cell anemia with acute chest syndrome •• Dehydration
•• Multiple lobe involvement •• Vomiting or inability to tolerate oral fluids
•• Toxic appearance •• No response to appropriate oral antibiotic therapy
•• Moderate to severe respiratory distress •• Social factors (e.g., inability of caregivers to administer medications
•• Requirement for supplemental oxygen at home or follow-up appropriately)

54. d. Prematurity  Ref: Nelson's 20/e p 2088-2093, Ghai 8/e p 379-381

Causes of under 5 child deaths: The area to the right of the dotted line indicates neonatal conditions
55. d. Mumps virus  Ref: Nelson's 20/e p 2088-2093, Ghai 8/e p 379-381
Viral causes of pneumonia in children:
•• Of the respiratory viruses, respiratory syncytial virus (RSV) and rhinoviruses are the most common;
•• Others include influenza virus, parainfluenza, adenoviruses, enteroviruses and human metapneumovirus.
56. b. Staph aureus  Ref: Nelson's 20/e p 2088-2093, Ghai 8/e p 379-381
The given chest X-ray finding is suggestive of pneumonia with pneumatoceles, seen in infection due to Staph aureus.
Chapter 18: Pediatric Respiratory Disorders 415

57. a. Staphylococcal pneumonia  Ref: Nelson's 20/e p 2088-2093, Ghai 8/e p 379-381

Answers with Explanations


Pneumothorax could be a complication of Staphylococcal pneumonia, due to rupture of pneumatoceles.

 ASTHMA
58. c. Omalizumab  Ref: Nelson's 20/e p 1107
An anti-IgE preparation, Omalizumab (Xolair), is approved by the FDA for use as an add-on therapy in children ≥12 yr who have
moderate to severe allergic asthma that is difficult to control.
59. b. PEF of 60 to 70% of expected  Ref: Nelson's 20/e p 1095-1115, Ghai 8/e p 382-392
Note: 1 kPa = 7.5 mm Hg; So 8 kPa = 60 mm Hg
Grading of severity of Asthma exacerbation

Characteristics Mild Moderate Severe


Heart rate <100 100-120 >120
PEF (Peak expiratory flow) ≥70% or response <40% 40-69% <40%
Pulsus paradoxus Absent May be present Often present
PaO2 Normal ≥60 mm Hg (8 KPa) <60 mm Hg (8 KPa)

60. c. MDI with Spacer with Mask  Ref: Nelson's 20/e p 1095-1115, Ghai 8/e p 382-392
61. d. Intravenous aminophylline  Ref: GINA guidelines 2016; Refer pretext for details;
Immediate treatment of acute severe asthma includes oxygen supplementation, high doses of inhaled β2 –adrenoreceptor
agonists, systemic corticosteroids and intravenous aminophylline
62. c. Prophylaxis with Montelukast  Ref: Nelson's 20/e p 1095-1115, Ghai 8/e p 382-392
Leukotriene receptor antagonists like Montelukast and Zafirlukast, have bronchodilator and targeted anti-inflammatory properties
and reduce exercise-, aspirin-, and allergen-induced bronchoconstriction.
63. b. Moderate persistent  Ref: Nelson's 20/e p 1095-1115, Ghai 8/e p 382-392; Refer pretext for details;
64. a. Inhaled short acting β2 agonist  Ref: Nelson's 20/e p 1095-1115, Ghai 8/e p 382-392
Most commonly beta agonists (bronchodilators) are used in the treatment of Asthma in all age groups.
65. a. Foreign body, b. Gastroesophageal reflux disease, c. Bronchinal asthma and e. Laryngomalacia  Ref: Nelson's 20/e p
1095-1115, Ghai 8/e p 382-392
Wheeze is a musical and continuous sound that originates from oscillations in narrowed airways.
Important causes of Wheezing in Infancy:

Category Causes
Infection Viral/Tuberculosis

Asthma Very common cause

Anatomic abnormalities •• Malacia of larynx, trachea, and/or bronchi •• Mediastinal lymphadenopathy/mass/tumor


•• Tracheoesophageal fistula •• Lung malformations
•• Laryngeal cleft •• Congenital heart diseases with L → R shunt
•• Vascular ring or sling •• Foreign body

Immunodeficiency states •• IgA deficiency •• Primary ciliary dyskinesia; Bronchiectasis


•• B-cell deficiencies •• AIDS

Others •• Cystic fibrosis •• Interstitial lung disease


•• Gastroesophageal reflux disease •• Heart failure
•• Bronchopulmonary dysplasia •• Anaphylaxis

66. c. Ventilate with bag and mask with oxygen  Ref: Nelson's 20/e p 1095-1115, Ghai 8/e p 382-392
As the child is cyanotic, gasping and unresponsive, there is respiratory failure, only oxygen supplementation will not help; So he must
be ventilated. So bag and mask ventilation with oxygen is to be used.

 CYSTIC FIBROSIS
67. b. Elevated sweat chloride levels  Ref: Nelson’s 20/e p 2100
Because the function of sweat gland duct cells is to absorb rather than secrete chloride, salt is not retrieved from the isotonic primary
sweat as it is transported to the skin surface; chloride and sodium levels are consequently elevated. So sweat chloride is usually ≥
60 mEq/L in cystic fibrosis.
68. c. Sweat gland  Ref: Nelson's 20/e p 2104
Excessive loss of salt in the sweat occurs in Cystic fibrosis
416 Section 3: Systemic Pediatrics

69. a. Cystic fibrosis  Ref: Nelson's 20/e p 2098-2112, Ghai 8/e p 393
Review of Pediatrics and Neonatology

High grade fever and chest infection along with passage of foul smelling stools suggests systemic disease.
In cystic fibrosis because of pancreatic exocrine insufficiency, foul smelling stool is common.
70. a. Pseudomonas aeruginosa (non mucoid)  Ref: Nelson's 20/e p 2098-2112, Ghai 8/e p 393
In cystic fibrosis, there is high prevalence of airway colonization with Staphylococcus aureus, Pseudomonas aeruginosa and
Burkholderia cepacia complex. Commonest organism to cause pulmonary infection in cystic fibrosis is Peudomonas aeruginosa,
which is initially non-mucoid, later mucoid colonies form.
71. b. Abnormality in CFTR which leads to defective calcium transport  Ref: Nelson's 20/e p 2098-2112, Ghai 8/e p 393
The cyclic adenosine monophosphate (cAMP)–mediated apical membrane conductance of Cl associated with the CF transmembrane
regulator (CFTR) does not function in CF epithelia.
So, there is defective transport of Chloride and not calcium in cystic fibrosis.
72. a. Nasal transmembrane potential difference  Ref: Nelson's 20/e p 2098-2112; Refer pretext for details;
73. a. Meconium ileus  Ref: Nelson's 20/e p 2098-2112, Ghai 8/e p 393
Intestinal manifestation of CF include meconium ileusQ, distal intestinal obstruction syndrome (DIOS), rectal prolapse.
74. a. Meconium ileus  Ref: Nelson's 20/e p 2098-2112, Ghai 8/e p 393

 MISCELLANEOUS
75. d. All of the above  Ref: Nelson 20/e p 531
Diagnostic criteria of ARDS in children:

Berlin definition criteria


Timing Within 1 wk of a known clinical insult or new or worsening respiratory symptoms
Chest X-ray or Bilateral opacitires not fully explained by effusions, lobar/lumg collapse, or nodules. (Illustrative clinical cases and
tomography scan chest X-ray have been provided)
Origin of edema Respiratory failure not fully explained by cardiac failure or fluid overload. Need objective assessment (e.g.,
echocardiography) to exclude bhydrostatic edema, if no ARDS risk facrors are present
Oxygenation
Mild Moderate Severe 200 mm Hg < PaO2/FIO2 ≤ 300 mm Hg with PEEP or CPAP ≥ 5 cm H2O
100 mm Hg < PaO2/FIO2 ≤ 200 mm Hg with PEEP ≥ 5 cm H2O PaO2/FIO2 < 100 mm Hg with PEEP
≥ 5 cm H2O

76. b. Differential pulse; Refer pretext for details


77. b. Decrease mechanical ventilation  Ref: Ghai essential pediatrics 8/e p393
In the given chest X-ray, we can see hyperinflated left lung (overinflation of lungs can give rise to volutrauma). Hence, decreasing
mechanical ventilator settings is the management of choice.
78. c. Mediastinum is shifted;  Ref: No reference needed;
Mediastinum does not appear shifted in the given chest X-ray
79. c. Severe pneumonia with sepsis  Ref: Neonatal and Pediatric Respiratory Care 4/e By Brian K. Walsh p 564
Pneumonia, sepsis and aspiration are the most common cause of ARDS and ALI in children
80. a. Large tongue  Ref: Core Topics in Airway Management 2/e edited by Ian Calder, Adrian Pearce; p 193
Differences between pediatric and adult airway are:
•• Infants and small children have a disproportionately large head and short neck.
•• Their tongues are relatively large with respect to the mandible
•• Their larynx is more anterior and cephalad in comparison to the adult.
•• They are obligate nasal breathers & find it more difficult to switch to mouth breathing when necessary.
•• The epiglottis is relatively long ‘floppy' & U-shaped.
•• The narrowest part of the airway is at the level of the cricoid cartilage
•• The trachea is approximately 5 cm long in the newborn, and this increase to about 8 cm in the first year of life.
•• The carina initially is at the level of T2 (T4 in adults) and is also wider than in the adult.
•• The angle between the left and right main bronchi is initially equal.

81. a. T2  Ref: See Ans 80 above;


82. a. Transient, migratory pulmonary infiltrations  Ref: Nelson 20/e p 2080; Refer pretext;
Chapter 18: Pediatric Respiratory Disorders 417

Loeffler's syndrome is now called “transient pulmonary infiltrates with eosinophilia syndrome” and is most commonly caused

Answers with Explanations


by Ascaris lumbricoides or roundworm.
83. a. 5 mL/kg  Ref: Nelson's 20/e p 542
Diseases associated with decreased time constants (decreased static compliance, e.g., ARDS, pneumonia, pulmonary edema) are
best treated with small tidal volume of 5-6 mL/kg.
84. b. > 20 sec  Ref: Nelson's 20/e p 849-850, Ghai 8/e p 171
In infants, apnea is defined as cessation of breathing for >20 sec, or any duration if accompanied by cyanosis or bradycardia.
85. a. Rigid bronchoscopy  Ref: Nelson's 20/e p 1193, Ghai 8/e p 373
Treatment of choice in a child with foreign body in bronchus is Rigid bronchoscopy.
Chapter 19
Disorders of Kidney
and Urinary Tract
High Yield Points  NORMAL RENAL PHYSIOLOGY
  M
•• Minimum urine osmolality (urine Glomerular Filtration Rate (GFR) M Maximum Urinary Osmolality M
diluting ability) is well developed,
even in preterm babies & is same as Age GFR (ml/min/1.73m2) U osmolality (mOsm/
adults (40-50 mOsm/kg) Age kg)
Preterm neonate (1st wk) 10
Preterm neonate 500
Term Neonate (1st wk) 20-40
Term Neonate 500-700
1 month age 50
1 year (adult value) 1200-1400
12 months 100

2 yr (adult value) 120

 EMBRYONIC DEVELOPMENT
•• During 5th week of gestation, the ureteral bud arises from mesonephric (Wolffian) duct
which penetrates the metanephric blastoma
•• Ureteral bud undergoes divisions by 20th week of gestation and forms the collecting system:
ureter, renal pelvis, calyces, papillary ducts, and collecting tubules.

Question 1  CONGENITAL ANOMALIES


Identify the underlying kidney abnor­
mality in the fetus shown below:
Renal Agenesis
•• It refers to the absence of single or both kidneys
•• Occurs due to defect of Wolffian duct, ureteric bud, or metanephric blastoma
•• Differs from aplasia, in which nonfunctioning renal tissue is seen

High Yield Points


Syndromes associated with renal age­nesis are:
a. Polycystic kidney disease
b. Multicystic dysplastic kidney •• “Mayer-Rokitansky-Küster Hauser syndrome”: Unilateral renal agenesis or ectopia, ipsilateral
c. Renal agenesis mullerian defects & vaginal agenesis
d. Nephronophthisis •• Potter Syndrome: Bilateral renal age­nesis; characte­ristic potter facies seen

Potter Syndrome
•• “Potter facies”: Widely separated eyes, low set ears, flat nose, receding chin.
•• MC cause of death in Potter syndrome is pulmonary hypoplasia.
Mnemonic •• Causes of neonatal renal failure with potter phenotype include bilateral renal agenesis, cystic
“POTTER” for Potter syndrome: renal dysplasia, obstructive uropathy, renal hypoplasia, polycystic kidney disease and
•• P-Pulmonary hypoplasia medullary dysplasia.
•• O-Oligohydramnios
•• T-Twisted skin (wrinkled)
Renal Dysgenesis
•• T-Twisted face (retrognathia)
•• E-Extremity deformities •• Maldevelopment of kidney that affects its size, shape or structure
•• R-Renal agenesis •• 3 principal types are: Dysplastic, Hypoplastic and Cystic.
Chapter 19: Disorders of Kidney and Urinary Tract 419

Multicystic Dysplastic Kidney (MCDK)

Section 3: Systemic Pediatrics


M

•• Entire kidney is dysplastic, non-functional and is replaced by cysts


•• It is usually unilateral & not inherited; B/L is incompatible with life
•• Increases risk of associated hypertension, Wilms tumor
•• Multicystic dysplastic kidney is the most common cause of an abdominal mass in the newborn.

Renal Hypoplasia
•• Refers to a small nondysplastic kidney that has fewer than the normal number of calyces
•• Unilateral renal hypoplasia remains asymptomatic and is incidentally diagnosed
•• “Ask-Upmark” kidney (segmental hypoplasia): refers to small kidneys with deep grooves
& parenchyma consists of tubules resembling thyroid gland.

Nephronophthisis (NPH)
Affects structural & functional integrity of DCT & collecting ducts, resulting in their cystic
dilatation & progression to ESRD. Autosomal recessive; 2 forms:
Juvenile NPH (more common) Infantile NPH
•• Mutations in NPHP1 or NPH3 to NPH9 •• Secondary to mutations in NPHP2 gene
•• Presents with impairment of urinary concen­tration, •• It presents with advanced renal function
polyuria & poly­dipsia at 3–5 years of age impairment developing before 5 years
•• Anemia, rickets & growth retardation seen of age
•• End stage renal disease by 8–13 yr age

Polycystic Kidney Disease M

Autosomal recessive polycystic Autosomal dominant polycystic kidney


kidney disease (ARPKD) disease (ADPKD)
Features •• Renal collecting duct dilatation and •• Multisystem disorder involving liver, brain &
biliary ectasia GI system
Responsible •• PKHD1 (chr 6p12), encodes the •• Mutations in PKD1 (chr 16p13) or PKD2 (chr
gene protein polyductin/fibrocystin 4q21)
Clinical •• Presents in infancy with enlarged •• Episodic hematuria, hypertension, pal­ pable
features kidneys, hypertension & renal failure kidneys & hepatic cysts in 3rd decade of life
•• Older children: hepatomegaly and •• Presence of >2 renal cysts with a posi­tive
portal hypertension family history is diagnostic
Question 2
Horseshoe Kidney Identify the congenital renal
abnormality shown:
•• Lower poles of kidneys fuse
•• Most commonly Lt kidney crosses & fuses with lower pole of Rt kidney
•• Seen with Turner syndrome
•• Increased incidence of Wilms tumor nephrolithiasis & hydronephrosis.

Medullary Cystic Kidney Disease (MCKD)


•• Autosomal dominant condition that presents in 3rd decade
•• Usually no extrarenal features
a. Renal hypoplasia
Alport Syndrome M b. Horseshoe kidney
c. Nephronophthisis
Genetic forms X-linked (80%), autosomal recessive (15%) and dominant (5%) d. Multicystic dysplastic kidney

Onset < 10 years of age


Clinical features •• Proteinuria along with persistent microscopic hematuria or recurrent episodes of
gross hematuria that follow upper respiratory tract infection
•• High frequency sensorineural hearing impairment
•• Ocular defects including anterior lenticonus, retinopathy, cataract
Renal histology Characteristic abnormalities of glomerular capillary basement membrane
Prognosis •• Males with X-linked disease are at high risk of progression to ESRD
420 Section 3: Systemic Pediatrics
Review of Pediatrics and Neonatology

Question 3 High Yield Points


What is this infant suffe­ring from? PRUNE-BELLY SYNDROME
•• Also called 'triad syndrome' or 'Eagle Barrett syndrome'
•• Deficient abdominal muscles, undescended tests & urinary tract abnormalities, e.g. large bladder,
patent urachus & dysplastic kidneys
•• Prognosis depends on degree of pulmonary hypoplasia & renal dysplasia

 NEPHROTIC SYNDROME M

Nephrotic syndrome is characterized by massive proteinuria, hypoalbuminemia, edema, and


a. Ehlers-Danlos syndrome hyperlipidemia.
b. Hydronephrosis
c. Vesicoureteric reflex Etiology of Nephrotic syndrome in childhood
d. Prune belly syndrome
Idiopathic Nephrotic Syndrome
•• Minimal change disease •• Membranoproliferative glomerulonephritis
•• Focal segmental glomerulosclerosis •• Crescentic glomerulonephritis
•• Membranous nephropathy •• Immunoglobulin A nephropathy

Secondary Causes
Mnemonic Infections Drugs Immunologic Malignancy
NPHS1 → Nephrin (starting with N •• Endocarditis •• Penicillamine •• Vasculitis syndromes •• Lymphoma
i.e. 1st alphabet) •• Hep B, C, HIV, EBV •• NSAIDs •• Bee sting •• Leukemia
•• Malaria; Syphilis •• Pamidronate •• Serum sickness •• Solid tumors
NPHS2 → Podocin (starting with P
i.e. 2nd alphabet)
Congenital Nephrotic Syndrome (Steroid Resistant)
Gene Name Location Inheritance Renal disease
NPHS1 NephrinQ 19q13.1 RecessiveQ Finnish-typeQ
NPHS2 PodocinQ 1q25 Recessive FSGS
FSGS1 α-actinin-4Q 19q13 Dominant FSGS
FSGS2 Unknown 11q21–22 Dominant FSGS
WT1 Wilms' tumorQ- 11p13 Dominant •• Denys-Drash syndrome
suppressor gene •• Frasier's syndrome with FSGS
LMX1B LIM protein 9q34 Dominant Nail-patella syndrome
SMARCAL1 SW1/SNF2-related 2q35 Recessive Schimke osseous dysplasia

High Yield Points   M Clinical Features


•• Nephrotic range proteinuria is •• Generalized edema is the most common presenting symptom
defined as protein excretion of > 40 •• Usually associated with decreased urine output
mg/m2/hr or 1 gm/m2/24 hrs
•• Increased susceptibility to infections such as cellulitis, spontaneous bacterial peritonitis, and
bacteremia.

High Yield Points


Indication of Renal biopsy in Nephrotic syndrome are:
Gross hematuria, hypertension, renal insufficiency, age < 1 year or > 8 years, initial steroid resistance
High Yield Points   M
•• Most common cause of idiopathic Important Definitions Related to Treatment of Nephrotic Syndrome: M

nephrotic syndrome in children is


minimal change disease •• Remission: Protein free urine (Urine protein negative/trace for 3 consecutive days
•• Commonest form of congenital •• Relapse: Up:Uc ≥ 2 or urine pr ≥ 3+ by dipstick for 3 consecutive days
Nephrotic syndrome is the auto­ •• Frequent relapse: 2 or more relapses within 6 months of initial response, or 4 or more
somal recessive (Finnish neph­rotic relapses in any 12-month period
syndrome) NPHS1 mutation (en­
codes for nephrin) •• Steroid resistant Nephrotic syn­drome (SRNS): Absence of remission despite 8 weeks of
•• Most common mutation in: steroid treatment
Steroid Resistant Nephrotic syndr → •• Steroid dependent Nephrotic syndrome (SDNS): 2 consecutive relapses during steroid
NPHS2 treatment or within 14 days of ceasing therapy.
Chapter 19: Disorders of Kidney and Urinary Tract 421

Treatment of Nephrotic Syndrome in Children

Section 3: Systemic Pediatrics


M
Question 4
A 4 years old child pre­sented with
generalized edema and ascites.
There was no hypertension or
hematuria. Renal function tests
were normal. Urina­ lysis revealed
massive proteinuria. Most pro­bable
underlying cause is:

a. Membranous glomerulo­nephritis
b. Minimal change disease
c. Post streptococcal glomerulone­
phritis
d. IgA nephropathy

High Yield Points


•• In frequent relapses and ste­roid dependent nephrotics: Levamisole is the first line drug used
•• Drug of choice in SRNS are Calcineurin inhibitors (Cyclospo­rine A/Tacrolimus)
•• Pneumococcus is the most frequent cause of peritonitis in Nephrotic syndrome

Complications of Nephrotic Syn­drome


•• Increased Susceptibility to Infections such as cellulitis, spontaneous bacterial peritonitis,
and bacteremia. M
•• Hypercoagulability: Thrombosis (venous, arterial) of major vessels due to vascular
stasis, increased platelet number & aggregability, increase in hepatic production of fibrinogen,
urinary losses of antithrombin III & protein S
•• Hyperlipidemia: elevated LDL, VLDL, ILDL and reduced HDL due to upregulation of HMG-
CoA reductase and down-regulation of lipoprotein lipase

 ACUTE GLOMERULONEPHRITIS/NEPHRITIC SYNDROME M

Characterized by Hematuria, edema, hypertension, oliguria along with diminished GFR.

Etiology
Postinfectious •• Streptococci, Staphylococci, Pneumococci, Meningococci, T. Pallidum, P. Malariae,
P. Falciparum, Hepatitis B & C, CMV, Parvovirus, Ebstein-Barr virus
•• Infections of shunts, prosthesis, endocarditis
Systemic Vasculitis •• Henoch-Schönlein purpura •• Wegener's granulomatosis
•• Microscopic PAN •• SLE
Other Primary Renal •• MPGN (membranoproliferative GN)
Diseases •• IgA nephropathy

POST STREPTOCOCCAL ACUTE GLOMERULONEPHRITIS (PSGN) M

Etiology
Group A b-hemolytic streptococcal infection of the throat or skin, by nephritogenic strains
including severe protein M types 12, 4, 25 and 49.
422 Section 3: Systemic Pediatrics

Clinical Features
Review of Pediatrics and Neonatology

Question 5
A 5 years old child pre­sents with •• Most common age: 5–12 years
red coloured urine as shown in the •• Presents 1–2 weeks after an streptococcal pharyngitis or pyoderma
given picture. On exami­ nation, •• Asymptomatic microscopic hematuria or gross hematuria with acute renal failure
BP was normal and there were no •• Hypertensive encephalopathy, acute LVF, nephrotic syndrome may be seen.
rashes. There is a history of sore
throat 2 days before this. There is a Lab Investigations
history of simi­lar episode 4 months •• Urine: Dysmorphic RBCs, RBC casts, proteinuria & polymorphonuclear leukocytes
back.
•• Serum C3 level is significantly reduced and returns to normal after 6–8 weeks
•• Elevated antistreptolysin O (ASLO) titer
•• In pyoderma : antihyaluronidase (AH) and anti DNAase B are raised
•• Indication of Renal biopsy in PSGN: presence of acute renal failure, nephrotic syndrome,
normal comple­ment levels or when hematuria & proteinuria diminished renal function, &/low
C3 level persist > 2 months after onset.

Kidney Biopsy Findings


•• Light microscopy: Hypercellularity due to proliferation of endothelial & mesangial cells &
early neutrophilic infiltration with obliteration of capillary lumen
What is the most probable under­ •• Immunofluorescence Deposition of IgG & C3 along the capillary wall labeled as ‘starry
lying abnormality?
sky' appearance
a. Post streptococcal glomerulone- •• Electron microscopy: Electron dense subepithelial deposits or “humps”Q.
phritis
b. Polycystic kidney disease Management
c. Nephrotic syndrome
d. IgA nephropathy •• Treat ARF and hypertension
•• 10 days course of penicillin is to limit the spread of nephritogenic organisms.

IgA NEPHROPATHY (BERGER DISEASE) M

High Yield Points •• Most commonQ chronic glomerular disease (primary glomerulonephritis)
•• Recurrent gross hematuria 2-3 days after throat infectionQ
Mesangial IgA deposition is seen in:
•• Rarely, heavy proteinuria and renal function impairment may be present
H.S. purpura, SLE, psoriasis, dermatitis
herpetiformis, celiac disease and non-
•• The condition is diagnosed by demonstration of IgA in the mesangium (usually on IF)
Hodgkin lymphoma •• Treatment of IgA nephropathy: Patients with heavy proteinuria or impaired renal function
should be treated with steroid along with azathioprine or mycophenolate mofetil
•• Fish oil, (omega-3-PUFA) decreases rate of progression in adults.

High Yield Points   M


Kidney diseases with recurrent gross hematuria
•• Hereditary nephritis
•• Thin GBM disease
•• Primary glomerulonephritis
•• IgA nephropathy (Berger ds)

 OBSTRUCTIVE NEPHROPATHY
•• Obstructive nephropathy is a common cause of ESRD in children
•• Early detection and appropriate management may prevent or retard kidney damage.
•• Common causes: posterior urethral valves in boys, PUJ obstruction, neuropathic bladder,
ureterocele, megaureter.

POSTERIOR URETHRAL VALVES M

•• Most common cause of severe obstructive uropathy in children


•• Dilatation of posterior urethra, hypertrophy of bladder, vesicoureteric reflux leading to
bilateral hydroureteronephrosis.

Clinical Features
•• Dribbling of urine, weak stream, recurrent UTI and straining during micturition
•• Undetected cases often present later with chronic renal failure.
Chapter 19: Disorders of Kidney and Urinary Tract 423

Diagnosis

Section 3: Systemic Pediatrics


Question 6
Confirmed by micturating cystourethrography; may be seen on antenatal USG.
Identify the abnormality shown in
the Voiding Cystourethrogram of
Management of PUV this 4-year-old boy with history of
recurrent urinary tract infection:
•• Continuous Antibiotic prophylaxis: High-risk groups include < 1 year of age, dilating
VUR (grades 3-5), a history of febrile UTI, those with bladder & bowel dysfunction
•• Surgery: Open neoureterocystostomy or ureteral reimplantation is the “gold standard”.

PELVIURETERIC JUNCTION (PUJ) OBSTRUCTION M

It is the most common obstructive lesion in childhood.

Manifests as a. Vesicoureteric reflex


b. Posterior urethral valve
•• On antenatal USG: fetal hydronephrosis c. Ureterocele
•• Newborn or infant: palpable renal mass, pain; febrile UTI; or hematuria. d. Bladder diverticulum

Diagnosis
•• After birth, USG after 3rd day & at 1 month age if no dilation is seen on USG
•• Diuretic renogram with MAG-3 is performed at 4–6 weeks of age.

Treatment
Pyeloplasty may be required.

UROLITHIASIS / RENAL CALCULI


Composed of High Yield Points   M
Calcium salts (70%), uric acid, magnesium, ammonium phosphate or cystine. •• Most common cause of severe
obstructive uropathy in children are
posterior urethral valves
Predisposing Factors •• 30% of PUV patients experience
Hypercalciuria, cystinuria, hyperuricosuria, & hyperoxaluria. end stage renal disease or chronic
renal insufficiency
•• Vesicoureteral reflux occurs in 50%
Clinical Features of patients with PUV
Recurrent flank pain, renal colic, recurrent UTI and gross or microscopic hematuria, obstructive uropathy.

Diagnosis
•• Ultrasonography for kidney and proximal ureter (less sensitive for stones < 3 mm)
•• Helical CT without contrast: most sensitive for diagnosing renal calculi.

Treatment
Decision whether to remove the stone depends on its location, size, and composition (if known),
and whether obstruction or infection, or both, is present.
•• Stones < 5–7 mm in size: may pass spontaneously
•• Extracorporeal shock wave lithotripsy (ESWL) may suffice for small stones
•• Percutaneous nephrolithotomy with relative contraindication for ESWL or larger stones.
•• Ureteroscopy is useful for distal and mid ureteric calculi
•• Open surgery for stones > 3 cm in size or those with associated PUJ obstruction
•• UTI should be treated and a large fluid intake ensured
•• Patients with idiopathic hypercalciuria may benefit from a low salt intake
•• Persistent hypercalciuria is treated with oral potassium citrate
•• Thiazide diuretics reduce urine calcium excretion
•• Prolonged alkali supplementation is necessary in patients with distal RTA.
424 Section 3: Systemic Pediatrics

 IDIOPATHIC HYPERCALCIURIA
Review of Pediatrics and Neonatology

Characterized by Normal concentration of S. calcium but excess urinary calcium


Genetic basis It may be a familial, autosomal dominant condition
Clinical features •• It is the most common cause of renal calculi in children
•• Recurrent gross hematuria or microscopic hematuria may be seen
Differential diagnosis •• Primary hyperparathyroidism, prolonged immobilization, hyperthyroidism,
Cushing syndrome, hypervitaminosis D, distal renal tubular acidosis.
Diagnosis •• Spot urinary calcium/creatinine ratio of > 0.2 (mg/mg) or 24 urinary calcium
exertion > 4 mg/kg. Other causes of Hypercalciuria should be excluded
Management •• High fluid intake; Dietary calcium is not restricted
•• Moderate salt restriction is advised as it decreases urinary calcium excretion
•• A high protein diet (e.g. meat) should be avoided.
•• Thiazide increase calcium reabsorption in the distal convoluted tubule,
thereby reducing its excretion.

 URETEROCELE

What is it? It is a cystic dilatation of the terminal ureter, and is obstructive because of a
pinpoint ureteral orifice,
Epidemiology More Common In Females; Usually associated with ureteral duplication
Diagnosis IVP shows a round filling defect in the bladder corresponding to the ureterocele &
characteristic finding of duplication of the collecting system
Treatment Transurethral incision of ureterocele: effectively relieves the obstruction but it
may result in VUR necessitating ureteral reimplantation later, or Open excision of
ureterocele and reimplantation ‘as primary management

Question 7  VESICOURETERIC REFLUX (VUR)


Identify the abnormality shown in Definition
the Micturating cystourethrogram
of a 3-year-old boy who had Retrograde passage of urine into the upper urinary tract.
urinary tract infection 1 month
ago: High Yield Points
•• VUR is present in 5–15% of infants with antenatal hydronephrosis
•• VUR is seen in 50% of boys with posterior urethral valves
•• Among children diagnosed with VUR during evaluation for a UTI, 80% are females & average age at
diagnosis is 2–3 yrs.

Types of VUR
•• Primary: Congenital incompetence of the valvular mechanism of the vesico-ureteral junction
•• Secondary: e.g.
– Neuropathic bladder –  Foreign bodies
–– Vesical calculi –  Severe cystitis
–– Bladder outlet obstruction –  Surgery involving ureterovesical junction

a. Vesicoureteric reflux Grading & Diagnosis M


b. Posterior urethral valve
c. Ureterocele VUR is diagnosed and graded on a radiocontrast MCU.
d. Pelviureteric junction obstruction
Grade Description
I Reflux into the nondilated distal ureter
II Reflux into the upper collecting system in nondilated ureter
III Reflux into dilated ureterQ
IV Reflux into grossly dilated ureter
V Massive reflux with ureteral dilation, tortuous, effaced calyces
Chapter 19: Disorders of Kidney and Urinary Tract 425

Section 3: Systemic Pediatrics


Question 8
What does the Radio-nuclide
cystogram in a 3 years old child
with recurrent urinary tract
infection show?

Grade I II III IV V

Resolution of VUR
•• Primary VUR tends to resolve by the age of 6 to 10 years
•• Severity of VUR is the most important factor that determines its resolution
•• Grade I–III are more likely to disappear than grade IV–V
•• Higher the grade, the longer it takes for VUR to resolve.

Management of VUR a. Ureterocele


The goals of treatment are to prevent pyelonephritis & renal injury b. Unilateral VUR
•• Long-term chemoprophylaxis is instituted for most patients (except grades I, II may not be put) c. Bilateral VUR
d. Posterior urethral valves
till 5 years or earlier if VUR disappears
•• The drugs used are: nitrofurantoin or cotrimoxazole; Cephalexin for age <3 months
•• Surgery for VUR: Ureteric reimplantation is considered for Bilateral grade IV–V VUR that
persists beyond infancy.

L at e s t U p d at e s
Antibiotic prophylaxis is recommended in:
•• Children at greatest risk for VUR-related renal injury i.e., < 1 year age
•• Children with bladder and bowel dysfunction and VUR
•• Children with VUR who are being managed by surveillance, if a febrile UTI occurs, prophylaxis is
recommended.

 URINARY TRACT INFECTION (UTI) M

Definition
Invasion of urinary tract by pathogens, which may involve the upper or lower tract depending upon
the infection in the kidney, bladder or urethra.
Etiology
•• 90% 1st symptomatic UTI and 70% of recurrent infection are due to E. coli
•• Proteus and Pseudomonas are associated with recurrent UTI, instrumentation and
nosocomial infection
•• Candida albicans in preterms immunocompromised & following prolonged antibiotics therapy.

Incidence of UTI
•• During the 1st yr of life, the male : female ratio is 3.5 : 1 High Yield Points   M
•• Beyond 1-2 yr, there is a female preponderance, with a male : female ratio of 1 : 10.
Best method to collect urine sample
Risk Factors for UTI for culture:
•• Neonates and infants: Best tech-
•• Female gender •• Bacteria with P fimbriae •• Obstructive uropathy nique is supra pubic aspiration.
•• Wiping from back to front in girls •• Anatomic abnormality •• Urethral instrumentation •• Older children: Clean catch mid-
•• Uncircumcised male •• Neuropathic bladder •• Voiding dysfunction stream specimen.
•• Pin worm infestation •• Vesicoureteral reflux •• Constipation •• Children with indwelling catheters:
urine aspirated from the catheter
Clinical Features using a sterile needle and syringe.

•• Infants and small children – Fever, diarrhea, vomiting, foul smelling urine
Older children – Dysuria, hypogastric or flank pain, fever, toxicity, leukocytosis.

Diagnosis
Urine Microscopy
•• > 10 WBCs/mm3 in an uncentrifuged specimen and
•• > 5 WBCs/high power field in a centrifuged sample is suggestive of UTI
•• WBC casts indicate renal parenchymal involvement.
426 Section 3: Systemic Pediatrics

Urine Culture
Review of Pediatrics and Neonatology

Question 9
Colony count of >105/mL organism of a single species is considered confirmatory. However,
What does this DMSA image indi­
presence of any number of organisms in suprapubic sample is abnormal.
cate?
Dipstick Tests
Based on nitrite reduction and detection of leukocyte esterase is highly suggestive of UTI.

ECTOPIC URETER
•• Definition: Ureter that drains outside the bladder
•• Girls > Boys; UTI is common because of urinary stasis
a. Bilateral renal scarring
•• Ureter usually drains the upper pole of a duplex collecting system (two ureters)
b. Posterior urethral vlave •• In girls: They mostly enter urethra at the bladder neck, or urethrovaginal septum
c. Vesico urethral reflux •• In boys: Ectopic ureters most commonly enter the posterior urethra.
d. Nephrolithiasis
 KIDNEY FAILURE
ACUTE KIDNEY INJURY (AKI)
Definition M

Absolute increase of serum creatinine of > or = 0.3 mg/dl or percentage increase of 50% or
more or oliguria of < 0.5 ml/kg/hr for > 6 hours

AKI Staging: RIFLE Criteria M

Criteria Estimated CCL Urine output


Risk eCCI decrease by 25% < 0.05 mL/kg/hr for 8 hr
Injury eCCI decrease by 50% < 0.5 mL/kg/hr for 16 hr
Failure eCCI decrease by 75% or < 0.3 mL/kg/hr for 24 hrQ
eCCI <35 mL/min/1.73 m2 or anuric for 12 hrQ
Loss Persistent failure >4 weeks
End-stage Persistent failure >3 mo

CCI, creatinine clearance; eCCI, estimated creatinine clearance; pRIFLE, pediatric risk, injury, failure, loss,
and end-stage renal disease.

Common Causes of AKI M

Prerenal Intrinsic renal Postrenal


•• Dehydration •• Postinfectious/poststreptococcal GN •• Posterior urethral valves
•• Hemorrhage •• SLE, H S purpura •• Ureteropelvic junction
High Yield Points •• Sepsis •• Membranoproliferative obstruction
•• Hypoalbuminemia •• Anti-glomerular basement membrane •• Ureterovesical junction
•• Fractional excretion of sodium •• Cardiac failure •• Hemolytic-uremic syndrome obstruction
UNa × PCr •• Acute tubular/cortical necrosis •• Ureterocele
FENa (%) = × 100
UCr × PNa •• Renal vein thrombosis •• Tumor
•• Rhabdomyolysis •• Urolithiasis
FeNa is <1% in pre-renal AKI •• Acute interstitial nephritis •• Hemorrhagic cystitis
•• Tumor lysis syndrome •• Neurogenic bladder

Urinalysis in Acute Renal Failure


Hypo­-volemia Acute tubular necrosis Acute Interstitial nephritis Glomerulone-phritis
Sediment Bland Broad, brownish granular WBCs, eosinophils, cellular casts Red blood cells, red blood cell
casts casts
Protein None or low None or low Minimal but may be increased Increased, >100 mg/dL
with NSAIDs
Urine sodium, mEq/L <20 >30 >30 <20
Urine osmolality, mOsm/kg >400 <350 <350 >400
Fractional excretion of Na% <1 >1 Varies <1
Chapter 19: Disorders of Kidney and Urinary Tract 427

Section 3: Systemic Pediatrics


High Yield Points
•• Schwartz formula is used for assessment of GFR in a child
•• According to this, estimated GFR = k X Ht / Serum Cr, where k is a constant
•• Usual value of k in Indian children is 0.42

Supportive Care for AKI


•• Fluids: Amount given equals insensible losses plus urine volume and other losses
•• Hyperkalemia: Calcium gluconate,Q SalbutamolQ nebulization, Sodium bicarbonateQ or
calcium resonium if required
•• Hyponatremia: Fluid restriction, if altered sensorium or seizures: 3% saline infusion
•• Metabolic acidosis: Sodium bicarbonate correction.
Indications for dialysis in AKI:
•• Anuria/oliguriaQ High Yield Points   M
•• Volume overload with refractory pulmonary edema Life-threatening Complications in AKI
•• Persistent hyperkalemiaQ •• Hyperkalemia → Arrhythmia
•• Severe metabolic acidosis unresponsive to medical management •• Pulmonary edema ± heart failure
•• Uremia (encephalopathy, pericarditis, neuropathy) •• Severe hypertension with
•• Uncontrolled hypocalcemic tetany encephalopathy
•• Hyponatremia
 HEMOLYTIC UREMIC SYNDROME (HUS) •• Severe anemia; profound acidosis

It is defined as thrombotic microangiopathy (TMA), wherein vascular endothelial injury causes


acute haemolytic anaemia, thrombocytopenia and kidney dysfunction M

L at e s t U p d at e s
Atypical HUS (aHUS)
•• Caused by inappropriate activation or insufficient inhibition of alternative complement pathway
•• Homozygous deficiency of CFHR1 predisposes to generation of anti-Factor H Ab
•• There is absence of infection by Shigatoxin producing bacteria or pneumococci
•• Characterized by onset at 5–15 yr age, severe illness & marked extrarenal features
•• Increased risk of progression to end-stage renal disease present
•• Plasma exchange is preferred therapy as it reduces levels of antiFH antibodies;
•• Eculizumab blocks terminal complement pathway & is effective in refractory cases Question 10
•• It commonly recurs after renal transplantation A 4-year-old child presented with
decreased urine output for last 20
 CHRONIC KIDNEY DISEASE (CKD) hours & petechial spots over the
body. There was a history of dia­
Definition M
rrhea 2 weeks prior to this. Blood
•• Any structural or functional kidney impairment that persists for 3 months investigations revealed a Hb level
of 7 g/dl, TLC 11,800/mm3, Platelet
•• Irreversible loss of renal function that eventually requires renal replacement therapy.
count of 35,000/mm3. His peripheral
smear findings are shown below
Stage of CKD M

Stage Description GFR (mL/min/1.73 m2)


1 Kidney damage with normal or increased GFR > 90
2 Kidney damage with mild decrease in GFR 60–89
3 Moderate decrease in GFR 30–59
4 Severe decrease in GFR 5–29
5 Kidney failure < 15 or on dialysis

Causes of CKD What is the diagnosis?

CKD may be the result of congenital, acquired, inherited, or metabolic renal disease. a. Malaria
b. Idiopathic thrombocytopenic
Age < 5 yr Age > 5 yr Any age purpura
Congenital abnormalities (renal Acquired diseases Metabolic disorders c. Acute tubular necrosis
hypoplasia, dysplasia, ARPKD) •• Glomerulonephritis (various forms) •• Cystinosis d. Hemolytic uremic syndrome
Obstructive uropathy •• Lupus nephritis •• Hyperoxaluria
Congenital nephrotic syndrome Inherited disorders Inherited disorders
FSGS •• Familial juvenile nephronophthisis •• ADPKD
Renal vein thrombosis, HUS •• Alport syndrome •• ARPKD
428 Section 3: Systemic Pediatrics

Pathophysiology of CKD
Review of Pediatrics and Neonatology

L at e s t U p d at e s
Patient has CKD if either of the
Manifestation Mechanisms
following criteria are present: Acidosis Decreased acid excretion, Impaired bicarbonate reabsorption
Urinary concentrating defect Solute diuresis, Tubular damage
1. Kidney damage for ≥3 mo, as
defined by structural or functional Hyperkalemia Decrease in GFR, Metabolic acidosis, Hyporeninemic hypoaldosteronism
abnormalities, with or without Renal osteodystrophy Decreased 1,25-dihydroxy vit D3, Hyperphosphatemia, Hypocalcemia,
decreased GFR, manifested by 1 Secondary hyperparathyroidism
or more of following: Growth retardation •• Inadequate caloric intake, Renal osteodystrophy
•• Abnormalities in blood or urine •• Metabolic acidosis, Growth hormone resistance
•• Abnormalities in imaging tests Anemia Decreased erythropoietin production, Decreased erythrocyte survival
•• Abnormalities on kidney biopsy Bleeding tendency Defective platelet function
2. GFR < 60 mL/min/1.73 m2 for ≥
3 mo, with or without the other  RENAL REPLACEMENT THERAPY
signs of kidney damage
Types
•• Peritoneal dialysis
•• Hemodialysis
Question 11 •• Renal transplant
What is this machine used for?
Indications
•• Estimated GFR of less than 8 mL/minute/1.73 m2.
•• At GFR of 9–14 mL/minute/1.73 m2 if complications like fluid overload, dyselectrolytemia,
uremia or refractory acidosis are present.

Choice of Dialysis Modality


The choice of dialysis modality in children depend on patient and family choice, patient size,
medical comorbidities, vascular access and family support.

Comparison of Peritoneal & Hemodialysis


Peritoneal dialysis (PD) Hemodialysis (HD)
a. Peritoneal dialysis Technically easier as compared to HD Requires higher technical expertise
b. Blood transfusion No Vascular access required Needs central vascular access or arteriovenous fistula
c. Mechanical ventilation Can be used for neonates & infants Used in older children
d. Hemodialysis
More convenient with flexibility in scheduling Done at Hemodialysis centres/ hospitals at fixed time
of exchanges. slots available
Allows children to participate in age Limitation in activities may be due to presence of
appropriate extracurricular activities hemodialysis catheter
Superior in achieving goals of nutrition & Inferior to PD in optimising growth & nutrition
enabling satisfactory somatic growth
Question 12 Can be done in children with cardiovascular Difficult in children who are hemodynamically
instability. unstable
An 8-year-old child pre­sented with
Precise control of ultrafiltration difficult Precise control of ultrafiltration possible
a history of respiratory distress,
alte­red sensorium & not passing Not preferred in children with abdominal wall Preferred in children who had abdominal surgeries
urine for the last 15 hours. What abnormalities or recent major abdominal
is the procedure shown below that surgery;
can be done as a part of emergency
management of this child?  TUBULAR DISORDERS
Renal tubular disorders include the following:
Disease Gene Defect in:
Disorders Involving the Proximal Tubule
Proximal renal tubular acidosisQ SLC4A4 Sodium bicarbonate cotransporter
Fanconi-Bickel syndromeQ SLC2A2 Glucose transporter, GLUT2
Isolated renal glycosuria SLC5A2 Sodium glucose cotransporter
a. Hemodialysis
CystinuriaQ SLC3A1, SLC7A9 Cystine, dibasic & neutral amino acid transporter
b. Peritoneal dialysis
c. Kidney biopsy Dent's diseaseQ CLCN5 Chloride channel, ClC-5
d. Hydration therapy Contd…
Chapter 19: Disorders of Kidney and Urinary Tract 429

Contd…

Section 3: Systemic Pediatrics


Disorders Involving the Loop of Henle
Bartter's (Classical) SLC12A1 Na, K, Cl cotransporter
Type 2 KCNJ1 Potassium channel, ROMK
Type 3 CLCNKB Chloride channel, ClC-Kb
Familial hypocalciuric CASR Calcium-sensing receptor
hypercalcemia
Disorders Involving the Distal Tubule and Collecting Duct
Gitelman's syndromeQ SLC12A3 Sodium chloride co-transporter
Pseudohypoaldosteronism SCNN1B, 1G Epithelial sodium channel (ENaC)
(Liddle's syndrome)
Nephrogenic diabetes insipidusQ AQP2 Water channel, aquaporin-2
Distal renal tubular acidosis SLC4A1 Anion exchanger-1

RENAL TUBULAR ACIDOSIS (RTA) M

Definition
Renal Tubular Acidosis (RTA) is a syndrome due to either a defect in proximal tubule bicarbonate
reabsorption, or a defect in distal tubule hydrogen ion secretion, or both.

Salient Features
Hyperchloremic metabolic acidosis Normal to moderately decreased GFR. Anion gap is
normal.

Characteristic Type 1 (Distal) Type 2 (Proximal) Type 4


Hyperchloremic acidosis YesQ Yes Yes

Minimum Urine pH > 5.5 < 5.5 < 5.5

Plasma potassium Low-normalQ Low-normal HighQ

Renal stones Yes No No

Defect Reduced H+excretionQ Impaired HCO3 Impaired cation ex-


in distal tubule reabsorptionQ in proximal change in distal tubule
tubule

BARTTER SYNDROME & GITELMAN SYNDROME

Syndromes Bartter syndrome Gitelman syndrome High Yield Points   M


Characterized by Hypokalemic metabolic alkalosis Hypokalemic metabolic
•• All types of Bartter syndrome have
with hyper calciuria & salt wasting & alkalosis, with hypocalciuriaQ and autosomal recessive inheritance,
hyponatremia hypomagnesemia.Q except type V, which has autosomal
inheritance
Symptoms •• Polyuria, polydipsia, vomiting, consti­ Recurrent muscle cramps and
•• Sensorineural hearing loss is seen in
pation, failure to thrive. BP: Low/normal spasms type IV Bartter syndrome
Molecular basis Mutations in genes that encode Na+/K+/2Cl− Mutations in sodium chloride
transporter (NKCC2), luminal K channel cotransporter NCCT, present in the
(ROMK), CL channel (CLC-Ka, CLC-Kb). distal convoluted tubule.

Treatment K supplementationQ & Indomethacin K & Mg Supplementation

High Yield Points


•• Antenatal Bartter syndrome (types I, II and IV; also called hyperprostaglandin E syndrome) typically
manifests in infancy and has a more-severe phenotype than classic Bartter syndrome (type III)
•• Nephrocalcinosia is found in most subtypes of Bartter syndrome (except type IV)
430 Section 3: Systemic Pediatrics

DENT DISEASE
Review of Pediatrics and Neonatology

Question 13
Slit lamp examination of a child What is it? It is an X-linked proximal tubulopathy with characteristic abnormalities
with clinical features of Rickets,
who presented with polyuria, Pathogenesis: •• It occurs due to loss of function mutation of the CLCN5 gene (located on XP11.22)
polydipsia, recurrent episodes of which encodes a renal CL–/ H+ antiporter (CIC-5)
dehydration & metabolic acidosis, Clinical •• LMW–proteinuria •• Amino aciduria •• Nephrocalcinosis
showed the following. What could features •• Hypercalciuria •• Phosphaturia •• Renal failure
be the underlying cause? •• Hypophosphatemic rickets •• Glucosuria

CYSTINOSIS

What is it? It is a systemic disease caused by a defect in metabolism of cysteine that results in
accumulation of cysteine crystals in kidney, liver, eye & brain.
Genetic basis •• Mutations in CTNS gene, which encodes cystinosin, a cystine transporter
Clinical features •• Fanconi syndrome (polyuria, polydipsia, growth failure & rickets)
•• Fever, caused by dehydration or diminished sweat production, is common.
Diagnosis •• Suggested by the detection of cysteine crystals in cornea & confirmed by
measurement of increased leukocyte cystine content
a. Hypercalcemia
b. Alport syndrome Treatment •• Specific therapy is available with cysteamine, which binds to cystine and converts
c. Cystinosis it to cysteine. This facilitates lysosomal transport and decreases tissue cysteine.
d. Dent disease •• Kidney transplantation is a viable option

Answer Keys for Image-Based Questions

Answers Explanations/Identifying features


1. Ans c. Renal agenesis A → Washerman's hand with wrinkled skin
B → Widely separated eyes with epicanthic folds, low set ears, flat nose & receding chin, s/o “Potter
facies”
2. Ans b. Horseshoe kidney Lower poles of the kidneys fuse in midline, to give a horseshoe appearance
3. Ans d. Prune belly syndrome Distended lax abdominal wall is seen in this infant suggestive of ‘Prune belly syndrome'

4. Ans b. Minimal change disease Generalized edema in a child with massive proteinuria, with no hypertension or hematuria & nor-
mal renal function suggests a diagnosis of Nephrotic syndrome due to minimal change disease;
5. Ans d. IgA nephropathy Recurrent gross Hematuria in a child, with history of sore throat 2 days ago, suggest a diagnosis
of IgA nephropathy
6. Ans b. Posterior urethral valve Small arrow shows constriction of urethra, at the level of posterior urethral valve;
Large arrow shows dilated posterior urethra;
7. Ans a. Vesicoureteric reflux Bilateral dilatation of ureter along with bilateral hydronephrosis suggests Vesicoureteric reflex
8. Ans c. Bilateral VUR This Radio-nuclide cystogram showing contrast going to dilated bilateral ureters & renal pelvis, from the
urinary bladder, in a child with recurrent UTI, is consistent with a diagnosis of Bilateral VUR
9. Ans a. Bilateral renal scarring This DMSA image shows heterogenous areas of low uptake in both kidneys s/o scars
10. Ans d. Hemolytic uremic Peripheral smear showing schistocytes in a child with oliguria, thrombocytopenia & a history of recent
syndrome diarrhea suggest a diagnosis of Hemolytic uremic syndrome
11. Ans. d. Hemodialysis This machine with a blood pump, dialyser and tubings carrying blood is a hemodialysis machine
12. Ans. b. Peritoneal dialysis This child presented with a history suggestive of acute kidney injury; The given picture shows a stiff
catheter inserted into the abdomen for peritoneal dialysis;
13. Ans. c. Cystinosis Accumulation of cystine crystals in cornea, in this child with clinical features showing renal tubular
involvement, suggest a diagnosis of Cystinosis
Chapter 19: Disorders of Kidney and Urinary Tract 431

Questions
Questions
 CONGENITAL ANOMALIES OF KIDNEY 10. Most common anomaly of upper urogenital tract is:
 (Recent Question 2013)
1. Basic Pathophysiology in Potter syndrome: a. Uretero pelvic junction stenosis
 (JIPMER 2017)
b. Ectopic urethral opening
a. Maternal diabetes b. Pulmonary hypoplasia c. Ureterocele
c. Bilateral renal agenesis d. Oligohydramnios d. Ectopic ureter
2. In Potters syndrome – primary pathology is: 11. Malformations of the following organ system of the fetus
 (JIPMER Nov 2017) are found to be most commonly associated with single
a. Oligohydraminos b. Renal agenesis umbilical artery: (DNB June 2011)
c. Dysmorphism d. Limb defects a. Central nervous system b. Cardiovascular
3. Features of autosomal recessive PCKD include? c. Genitourinary d. Skeletal
 (PGI Nov 2017) 12. Baby born at 30 weeks to 18 years old primigravida of
a. Can be diagnosed intrauterine weight 2 kg, died after 48 hours. Apgar scores were 5
b. May proceed to renal failure by 10 yr age and 8 at 1 and 5 minutes. On autopsy, bilateral enlarged
c. Kidneys cannot be palpated abdominally kidney with multiple radially arranged cysts were seen.
d. Mutation in PKD1 Which of the following finding is expected to be associ-
e. Cysts are seen mainly in renal cortex ated with it? (AIIMS June 2008)
4. Which of the following developmental defects of a. Imperforate anus b. Hepatic cyst and fibrosis
the Urogenital sinuses NEVER occurs in the female? c. Absence of ureter d. Holoprosencephaly
 (APPG 2016) 13. True about infantile polycystic kidney disease are all the
a. Epispadias b. All these defects can occur following except: (AIPGMEE 2008)
c. Hypospadias d. Ectopia vesicae a. Autosomal dominant b. Caroli disease associated
5. Hepatic fibrosis is found in: (Recent Question 2016) c. Renal cysts present at birth d. Periportal fibrosis
a. Medullary cystic kidney b. ADPCKD 14. The most common congenital anomaly of the urinary
c. ARPCKD d. Nephronophthosis tract: (TN PGMEE 2008)
6. Chromosome a/w ADPKD: M  (Recent Question 2016) a. Epispadias b. Hypospadias
a. 14 and 16 b. 14 and 13 c. Exstrophy of bladder d. Posterior urethral valve
c. 16 and 4 d. 12 and 16 15. Which of the following is the most common renal cystic
7. The gross section of kidney shown below depicts? disease in infants is?  (AIPGMEE 2005)
 (APPGMEE 2015) a. Polycystic kidney b. Simple renal cyst
c. Unilateral renal dysplasia
d. Calyceal cyst
16. “Potter's syndrome” is associated with: (MAHE 05)
a. Renal anomalies b. Severe oligohydramnios
c. Flattened nose d. All the above
17. Which one of the following is the most common cause
of abdominal mass in neonates? (AIPGMEE 2003)
a. Neuroblastoma
a. Renal cell carcinoma b. Hydatid cyst b. Wilm's tumor
c. Polycystic kidney d. Medullary sponge kidney c. Distended bladder
8. The following are true regarding kidney development d. Multicystic dysplastic kidneys
except? (APPG 2015) 18. A 12 years old boy is referred for evaluation of nocturnal
enuresis and short stature. The blood pressure is normal.
a. Human kidney develops mainly from metanephros The blood urea is 112 mg/dL, Creatinine 6 mg/dL, Sodium
b. Collecting parts of kidney develop from the ureteric bud 119 meq/L, Potassium 4 meq/L, Calcium 7 mg/dL, Phos-
c. Mesonephric duct disappears phate 6 mg/dL and alkaline phosphatase 400 U/L. Urina­lysis
d. Kidney starts developing in the sacral region and ascends shows trace proteinuria with hyaline casts; no red and white
to the lumbar position cells are seen. Ultrasound shows bilateral small kidneys and
9. Unilateral renal agenesis is associated with: M  the micturating cystourethrogram is normal. The most likely
 (Recent Question 2014, 2012; AIIMS May 1995) diagnosis is: (AIIMS Nov 2003; AIIMS May 2003)

a. Polycystic disease of pancreas a. Alport's syndrome


b. Hiatus Hernia b. Medullary sponge kidney
c. Chronic glomerulo nephritis
c. Single umbilical artery d. Hypogonadism
d. Nephronophthisis
432 Section 3: Systemic Pediatrics

19. An 8 years old child suffering from recurrent attacks


Review of Pediatrics and Neonatology

28. A 10 year old child presents with edema and decreased


of polyuria since childhood presents to the pediatrics urine output. On evaluation, serum albumin is 2.5 g/dL,
OPD. On examination, the child has short stature. Vitals S.Creatinine is 0.5 mg/dL, urine protein is 3+ with no
and B.P. are normal. S.Creatinine – 6 mg/dL, HCO3 – RBC casts. Pathological change expected is?
16 meq/L, S Na+ – 134 meq/L. On USG, bilateral small  (AIIMS May 2015)
kidneys are seen. Diagnosis is: (AIIMS May 2001) a. Minimal change disease b. Interstitial nephritis
a. Reflux nephropathy b. Nephronophthisis c. IgA nephropathy d. FSGS.
c. Polycystic kidney disease 29. Nephrotic range proteinuria is defined as protein
d. Medullary cystic kidney disease excretion of more than? M 
 (WBPGMEE 2015/MAHA PGM CET 2015)
 NEPHROTIC SYNDROME a. 10 mg/m2/hr b. 30 mg/m2/hr
c. 20 mg/m2/hr d. 40 mg/m2/hr
20. Best method of estimation of amount of proteinuria in a
30. True about minimal change disease: (PGI May 2015)
2 year child with nephrotic syndrome is
 (Recent Question 2017) a. Peak incidence is between 2 and 4 years of age
b. Renal failure occur in adolescent
a. Dipstick testing
c. Steroid not very effect
b. 24 hr urine protein
d. Selective proteinuria
c. Spot urine sample for protein/creatinine ratio
d. Microalbuminuria 31. Most common cause of nephrotic syndrome in children
21. Which of the following is not a feature of Minimal M (Recent Question 2015, 2014)
change disease? M  (Recent Question 2017) a. Membranous GN b. Minimal change disease
a. Nephrotic syndrome c. PSGN d. RPGN
b. Hypertension 32. A child with nephrotic syndrome following an episode
c. Affects children of diarrhea presented with acute kidney injury with a
d. No abnormality detected on light microscopy creatinine of 4.5 mg/dl. All of the following are possible
22. In minimal change disease, which of the following is reasons except? (AIIMS Nov 2014)
seen? (WB PGMEE 2016) a. Renal vein thrombosis
a. Immunodeposits in glomerulus b. Diarrhoea induced water depletion
b. Immunodeposits in mesangium  c. Frusemide d. Steroid induced diabetes
c. Immunodeposits in blood vessels 33. Which is seen in nephrotic syndrome: M 
d. No immunodeposits  (Recent Question 2014)
23. Which of following is not seen in nephritic syndrome: M a. Low serum calcium b. Raised AT-III
 (Recent Question 2015) c. Low lipid d. Platelet activation
a. Edema b. Hematuria 34. Most common cause of nephrotic syndrome in children:
c. Hypertension d. Hypocholesterolemia  (Recent Question 2014)
24. A child with Malignancy is most likely to have which of a. Minimal change disease b. IgA nephropathy
the following: (AIIMS May 2015; Nov 2014)
c. Mesangial GN d. FSGN
a. Minimal change disease 35. Patients with minimal change disease are at risk of?
b. Membranous nephropathy
a. Spontaneous bacterial peritonitis (PGI May 2014)
c. Rapidly progressive glomerulonephritis
d. Focal segmental Glomerulosclerosis b. Sepsis c. DVT
d. Atherosclerosis e. Renal Carcinoma
25. A 10–years old child presents with edema, oliguria
and frothy urine. He has no past history of similar 36. Mutation in NPHS1 gene causes: (APPGMEE 14)
complaints. On examination, his urine was positive for a. Alport syndrome
3+ proteinuria, no RBCs/WBCs and no casts. His serum b. Congenital Finnish type nephrotic syndrome
albumin was 2.5 g/dL and serum creatinine was 0.5 mg/ c. Focal segmental glomerulosclerosis
dL. The most likely diagnosis is: d. Nail patella syndrome
 (AIIMS May 2015, 2014) 37. In which one of the primary Glomerulonephritides the
a. IgA nephropathy b. Minimal change disease glomeruli are normal by light microscopy but shows loss
c. Acute interstitial nephritis d. Membranous nephropathy of foot processes of the visceral epithelial cells and no
26. True statement about minimal change disease deposits by electron microscopy (APPGMEE 14)
 (Recent Question 2015) a. Poststreptococcal glomerulonephritis
a. Effacement of foot process on light microscopy b. Membranoproliferative glomerulonephritis type
b. Severe hypoalbuminemia c. IgA nephropathy d. Minimal change disease
c. Non selective proteinuria 38. A 7-year-old child with steroid dependent nephrotic
d. Cyclosporine is the first line of treatment syndrome has developed corticosteroid toxicity and
27. Most common nephropathy in world is? M  posterior subcapsular cataracts. Which of the following
 (Recent Question 2015) is the best alternative for the treatment of the patient:
a. IgA nephropathy b. FSGS  (AIIMS May 2013)
c. Minimal Change ds d. Adult PSGN a. Levamisole b. Cyclophosphamide
c. Mycophenolate mofetil d. Rituximab
Chapter 19: Disorders of Kidney and Urinary Tract 433

39. Which of the following statements is true about Conge­ 48. A 7 years old girl is brought with complaints of generalized

Questions
nital nephrotic syndrome caused by Nephrin protein swelling of the body. Urinary examination reveals grade
mutation? (PGI May 2013) 3 proteinuria and presence of hyaline and fatty casts.
a. Cause steroid resistant nephrotic syndrome She has no history of hematuria. Which of the following
b. Nephrin is a key component of the slit diaphragm statements about her condition is true?  (AIPGMEE 2009)
c. Coded by NPHS-1 gene a. No IgG deposits or C3 deposition on renal biopsy
d. Symptom occur only after 1st month of age b. C3 level will be low
e. Autosomal dominant pattern c. IgA nephropathy is the likely diagnosis
40. Edema in nephrotic syndrome is mainly due to: M  d. Alport's syndrome is the likely diagnosis.
 (AIIMS May 2012; Nov 2010) 49. The most common gene defect in idiopathic steroid
resistant nephrotic syndrome:
a. Sodium and water retention
 (DNB Dec 09) (AIIMS Nov 11, May 07, Nov 06)
b. Increased venous pressure
c. Hypoalbuminemia a. ACE b. NPHS 2
d. Hyperlipidemia c. HOX II d. PAX
41. A child comes with steroid resistant nephrotic syn­drome 50. Nephrotic syndrome in children is caused by:
secondary to FSGS, not responding to methy­lprednisolone. a. Minimal change disease b. RPGN (PGI Dec 2008)
What should be given next? (AIIMS May 2011) c. MPGN d. Membranous nephritis
a. Oral cyclophosphamide e. Hemolytic uremic syndrome
b. Oral cyclosporine 51. Most common infection in a child with nephrotic
c. Oral mycophenolate syndrome: M  (MAHA PG 2007)
d. IV Cyclophosphamide a. Spontaneous bacterial peritonitis
42. A 7-year-old girl is brought with complains of generalized b. Pneumonia
swelling of the body. Urinary examination reveals grade c. UTI d. Cellulitis
3 proteinuria and the presence of hyaline and fatty casts. 52. The most common gene defect in idiopathic steroid resistant
She has no history of hematuria. Which of the following nephrotic syndrome: M  (AIIMS Nov 2006, May 2007)
statements about her condition is true? (AIIMS May11) a. ACE b. NPHS2
a. No IgG deposits or C3 deposition on renal biopsy c. HOX11 d. PAX
b. Her C3 level will be low 53. The finnish type of congenital nephrotic syndrome
c. IgA nephropathy is the likely diagnosis occurs due to gene mutations affecting the following
d. Alport's syndrome is the likely diagnosis protein: (AI 2006)
43. Idiopathic nephrotic syndrome is associated with the a. Podocin b. Alpha – actinin
following except: (MH 11) c. Nephrin d. CD2 activated protein
a. Focal segmental glomerulosclerosis 54. A 9 years old boy has steroid dependent nephrotic
b. Minimal change disease syndrome for the last 5 years. The patient is markedly
c. Membranoproliferative glomerulonephritis cushingoid with blood pressure of 120/86 mmHg and
d. Mesangioproliferative glomerulonephritis small subcapsular cataracts. The most appropriate
therapy of choice is: (AIIMS Nov 2004, May 2003)
44. A child presented with decreased albumin, creatinine
0.6 mg/dl, microscopic hematuria, proteinuria 3+, a. Long term frusemide with enalapril
edema, hypercholesterolamia, C3 level normal. Most b. Cyclophosphamide
likely diagnosis is (WBPG 2010) c. Intravenous immunoglobulin
d. Intravenous pulse corticosteroids
a. Minimal change disease
55. Which of the following is included in definition of
b. Post streptococcal glomerulonephritis
nephrotic syndrome:  (PGI June 2004)
c. Focal segmental glomerulonephritis
d. MPGN a. Micro albuminuria b. Massive proteinuria
45. Spontaneous Bacterial Peritonitis in Nephrotic Syndrome c. Microscopic hematuria d. Edema
is most commonly caused by: M (WBPG 2010)
e. Hyperlipidemia
56. True about nephrotic syndrome in a child:
a. E. coli b. Pseudomonas
 (PGI June 2003)
c. Salmonella d. Streptococcus pneumoniae
a. Minimal change disease is commonest cause
46. Hypercoagulation in nephrotic syndrome is caused by
b. Proteinuria of 4 gm/m2/hr is seen
 (AI 10)
c. Cyclosporine and azathioprine is mainstay of therapy
a. Loss of antithrombin III d. Pretreatment biopsy is done in all cases
b. Decreased fibrinogen e. Spontaneous bacterial peritonitis is associated with it
c. Decreased metabolism of Vitamin K 57. A 5 years old child suffering from nephrotic syndrome is
d. Increase in protein C responding well to steroid therapy. What would be the
47. Edema in nephrotic syndrome is due to: (AIIMS Nov 10) most likely finding on light microscopy? M  (AI 2001)
a. Sodium and water retention a. No finding
b. Increased venous pressure b. Basement membrane thickening
c. Hypoalbuminemia c. Hypercellular glomeruli d. Fusion of foot process
d. Hyperlipidemia
434 Section 3: Systemic Pediatrics

 NEPHRITIC SYNDROME 64. Following all are syndromes associated with renal
Review of Pediatrics and Neonatology

disease except (MAHA PGM CET 2016)


58. A 2 year old child presents with hematuria. There is also a. Lowe Syndrome
history of cough and cold 2-3 days back. Which of the
b. Denys-Drash Syndrome
following is the most likely electron microscopy picture?
c. Alport Syndrome
 (Recent Question 2017)
d. Sinding-Larsen-Johansson Syndrome
a. Irregular focal thickening of the glomerular basement 65. A 5-year-old boy presents with hematuria 2 days after
membranes sore throat. Most probable cause is? M 
b. Absence of IgG deposits and/or complement on  (Recent Question 2016)
immunofluorescence.
a. Post-streptococcal glomerulonephritis
c. Subepithelial humps
b. Berger disease
d. Segmental sclerosis
c. Paroxysmal nocturnal Hemoglobinuria
59. A 10 years boy presented with Cola coloured urine,
d. Microscopic polyangitis
oliguria for 3 days, facial puffiness, edema and
hypertension. Urine albumin is positive and C3 levels 66. Post-streptococcal glomerulonephritis results in M 
are reduced. BP is 130/80. He had skin infection two  (Recent Question 2015)
weeks back. What is the next appropriated management? a. Nephrotic syndrome b. Nephritic syndrome
 (JIPMER May 2017) c. Interstitial nephritis d. Acute tubular necrosis
a. Renal biopsy b. Conservative therapy 67. Which is seen in Electron microscopy in PSGN?
c. Cyclosporine d. Steroids a. Epithelial humps (Recent Question 2015)
60. A 10 year old boy presents with gross hematuria, loin b. Spike and dome appearance
pain, and diarrhea; urine examination is normal. Serum c. Mesangial deposits
C3 levels are normal; Probable diagnosis is? d. Subendothelial deposits
 (Recent Question 2016)
68. Type I RPGN is seen in (Recent Question 2015)
a. Berger's disease
a. SLE
b. PSGN
b. IgA nephropathy
c. Microangiopathic hemolytic anemia
c. Henoch Schonlein purpura
d. None of the above
d. Good pasture syndrome
61. A 9-year-old boy is brought with history of decreased
69. False regarding nephritic syndrome M 
urine output, cola colored urine and swelling of the face
 (Recent Question 2015)
and hands, of 2 days duration. He is hypertensive, has
a puffy face and pitting edema of the lower limbs. He a. Edema b. Proteinuria <3.5g/day
has history of skin lesions 4 weeks earlier. A diagnosis c. Hypoalbuminemia d. Hypertension
of post streptococcal glomerulonephritis is made. ASLO 70. True regarding IgA Nephropathy  (Recent Question 2015)
titers are likely to be? (COMEDK 2016) a. Usually in children < 10 years
a. Elevated b. Microscopic hematuria is the most common presentation
b. Not elevated c. Recurrent gross hematuria following respiratory infection
c. Increased progressively over 2 weeks d. Decreased serum IgA
d. Initially elevated, rapidly fall in 3-4 days
71. Henoch Schonlein purpura commonly involves which
62. A 9-year-old boy is brought with history of decreased age group? (Recent Question 2014)
urine output, cola colored urine and swelling of the face
and hands of 2 days duration. He is hypertensive, has a a. 4–8 years b. 10–15 years
puffy face and pitting edema of the lower limbs. He has c. 15–20 years d. > 25 years
history of skin lesions 4 weeks earlier. A diagnosis of 72. In Post-streptococcal glomerulonephritis: (WBPG 2014)
post streptococcal glomerulonephritis is made. C3 levels a. C3 decreases, ASO increases
are likely to return to normal in? (COMEDK 2016) b. C3 increases, ASO increases
a. 1-2 years b. 1-2 weeks c. C3 decreases, ASO decreases
c. 6-8 weeks d. 3-4 months d. C3 increases, ASO decreases
73. In shigella dysentery associated hemolytic uremic
63. A 9-year-old boy is brought with history of decreased syndrome, the false statement is:
urine output, cola colored urine and swelling of the face  (Recent Question 2014; AIPGMEE 95)
and hands of 2 days duration. He is hypertensive, has a. Leucocytosis
a puffy face and pitting edema of the lower limbs. He b. Neurological abnormalities
has history of skin lesions 4 weeks earlier. A diagnosis c. Hepatic failure
of post streptococcal glomerulonephritis is made. If d. Thrombotic angiopathy
an antibiotic is to be used to limit the spread of the
74. A 8 years old child has throat infection followed by
nephritogenic organisms, the drug of choice is?
hematuria; Diagnosis? (Recent Question 2013)
(COMEDK 2016)
a. IgA nephropathy
a. Cefixime b. Penicillin b. Post streptococcal glomerulonephritis
c. Meropenem d. Amoxycillin c. Nephrotic syndrome
d. Can be (a) or (b)
Chapter 19: Disorders of Kidney and Urinary Tract 435

75. Most common cause of hemolytic uremic syndrome is: 85. A six years old male baby presents to a hospital with

Questions
 (Recent Question 2013) recurrent gross hematuria for 2 years. There is no
a. E. coli b. Shigella history of burning micturition or pyuria. Urine routine
c. Salmonella d. Pseudomonas examination demonstrated no pus cells and urine
culture was sterile. Serum C3 were normal. What is the
76. In Henoch-Schönlein purpura renal involvement is not
most probable diagnosis? (AIPGMEE 2008)
seen generally if no involvement till?
 (Recent Question 2013) a. Wilm's tumor b. IgA nephropathy
c. Post streptococcal glomerulonephritis
a. 1 month after onset b. 2 months after onset
d. Urinary tract infection
c. 3 months after onset d. 4 months after onset
86. Which of the following is/are not the features of Henoch-
77. Not seen in HSP:  (Recent Question 2012) Schonlein purpura (HSP)? (PGI Dec 2008)
a. Thrombocytopenia b. Arthritis a. Abdominal pain b. Splinter hemorrhage
c. Abdominal pain d. Renal involvement c. Thrombocytopenia d. Epistaxis
78. A 5 years old child presents with perivascular IgA depo- e. Arthritis
sition and neutrophilic collection. There is erythema- 87. A 8 years old male had non blanching rashes over the
tous rash on the lower limb and non blanching purpura. skin and swelling of knee joint with hematuria +++ and
Probable diagnosis is: (AIIMS Nov 2011) protein + in urine. Microscopic analysis of his renal
a. Henoch-Schonlein purpura biopsy specimen is most likely to show:
b. Wegener's granulomatosis  (AIIMS Nov 2007)
c. Vasculitis a. Tubular necrosis b. Visceral podocyte fusion
d. Kawasaki diseases c. Mesangial deposits of IgA
79. Post-streptococcal reactive arthritis is differentiated d. Basement membrane thickening
from Acute Rheumatic Fever by all except: 88. Most common cause of renal artery stenosis in children
 (WB PG 2011) in India is: (AIIMS May 2007)
a. Small joint involvement and often symmetric a. Takayasu aortoarteritis b. Fibromedial hypertrophy
b. Caused by non-group A hemolytic streptococci c. Fibrointimal hyperplasia d. Polyarteritis nodosa
c. Non-responsiveness to salicylate 89. A four-year old child presents with mild fever, malaise,
d. Shorter incubation period purpura, arthritis, abdominal pain and microscopic
80. Which of the following is not seen in Henoch-Schonlein hematuria. What would be the most likely diagnosis?
purpura?  (DNB June 2011)
 (UPSC 07)
a. Thrombasthenia
a. Purpura b. Abdominal pain
b. Idiopathic thrombocytopenic purpura
c. Seizure d. Arthralgia
c. Systemic lupus erythematosus
81. In case of PSGN complications commonly seen are all d. Henoch-Schonlein purpura
except: (WBPG 2011)
90. A 3 years old child presents with four days history of
a. LVF puffi­ness of face, fever and tea colored urine. During the
b. Hypertensive encephalopathy course of his disease, he can have any of the compli­
c. Hyperkalemia cations except: (UPSC 06)
d. Bleeding diathesis a. Hypokalemia
82. A child presented with cola coloured urine, proteinuria b. Hypertensive encephalopathy
2+ and h/o rash 2 weeks ago. Probable diagnosis is: c. Acute renal failure d. Acidosis
 (PGI May 2010) 91. The most common leukocytoclastic vasculitis affecting
a. IgA nephropathy children is:  (AIPGMEE 2005)
b. HSP c. HUS a. Takayasu disease
d. Wegener's granulomatosis b. Mucocutaneous lymph node syndrome (Kawasaki
e. Paroxysmal nocturnal hemoglobinuria disease)
83. A 8 years old child with BP 180/100 mm Hg, Blood Urea c. Henoch-Schonlein purpura
90 mg/dl, creatinine 5.3 mg/dl, urinalysis shows 15–20 d. Polyarteritis nodosa
pus cells, 1–2 RBC, protein 1+; Most likely diagnosis is: 92. Henoch-Schonlein purpura is characterized by the
 (AIIMS Nov 2010) deposition of the following immunoglobulin around the
a. Post Infective glomerulonephritis vessels: M  (AIIMS Nov 2005)
b. Accelerated hypertension with ARF a. IgM b. IgG
c. Idiopathic RPGN c. IgA d. IgE
d. Chronic interstitial nephritis with VUR 93. The renal biopsy of a 6 years old boy with recurrent
84. A child develops non-blanching macules and papules on gross hematuria shows IgA nephropathy. The urinary
lower extremities, mild abdominal pain and skin biopsy protein excretion is 130 mg/day. Which of the following
is the most appropriate next step in the management:
showed IgA deposition. Most appropriate diagnosis is: 
 (AIIMS May 2004)
a. Drug induced vasculitis (AIIMS May 2009)
a. Administer corticosteroids
b. Henoch Schonlein purpura
c. Wegener's granulomatosis b. Give Azathioprine
d. Kawasaki disease c. Start cyclosporine d. Urinary bag sample
436 Section 3: Systemic Pediatrics

94. A 12 years old boy Shyam presented with gross hematuria


Review of Pediatrics and Neonatology

103. Vesico-ureteric reflex is commonly diagnosed by:


with 80% dysmorphic RBCs 2 days after an attack of a. Micturating cystography (PGI May 2015)
upper respiratory tract infection. Diagnosis is: b. Radioisotope renography
 (AIIMS Nov 2001) c. IVU
a. Microangiopathic thrombotic anemia d. CT Scan
b. IgA nephropathy e. Abdominal X ray
c. PSGN d. H.S. Purpura
104. A male child presents with repeated urinary infections
95. A picture resembling microthrombi in renal vasculature
and failure to gain weight. A MCU was carried out as
occurs in: (PGI Dec 2001)
shown in plate, most probable diagnosis is?
a. Pre eclampsia  (Recent Question 2013)
b. Henoch-schonlein purpura a. Posterior urethral valve b. Meatal stenosis
c. Benign hypertension c. Bladder diverticula d. Bladder polyp
d. Hemolytic uremic syndrome 105. Which is not a feature of posterior urethral valve:
e. Malignant hypertension  (Recent Question 2012)
96. Characteristic of acute Glomerulonephritis: M  a. Palpable bladder b. Painful stress incontinence
 (PGI June 2000) c. Recurrent UTI d. Hydronephrosis
a. RBC cast b. Hemoglobinuria 106. True statement(s) about posterior urethral valves in
c. Proteinuria d. Broad cast neonate: (PGI May 2010)
e. Hyaline cast a. Bilateral hydronephrosis may be present
97. Child with B.P. 190/110, pedal edema, facial edema, b. Urea and creatinine levels remain normal
absent ascites, gross hematuria. Diagnosis is: c. Large and wide prostatic urethra
 (PGI June 2000) d. More common in girls
a. Acute GN b. Nephrotic syndrome e. Proteinuria occurs
c. Renal thrombosis d. Renal amyloidosis 107. A child presented with intermittent episodes of left
e. Renal tubular acidosis side flank pain. Ultrasonography reveals large hydro­
nephrosis with dilated renal pelvis and cortical thinning
98. A child was diagnosed as a case of pauci-immune
with a normal ureter. Kidney differential function was
crescentic glomerulonephritis. The treatment to be given
observed to be 19%, which of the following is the best
in this child is:
management: (AI 2010)
a. Prednisolone + Cyclophosphamide
a. Nephrectomy b. Pyeloplasty
b. Methyl prednisolone
c. External drainage d. Endopylostomy
c. Cyclophosphamide d. Immunoglobulins
108. A 3 years old boy presents with fever, dysuria and gross
hematuria. Physical examination shows a prominent
 OBSTRUCTIVE UROPATHY suprapubic area which is dull on percussion. Urinalysis
reveals red blood cells but no proteinuria. Which of the
99. A 25-year-old primigravida with history of 20 weeks following is the most likely diagnosis: (AIIMS May 2006)
gestation comes for routine antenatal examination. The a. Acute glomerulonephritis
fetal ultrasonogram reveals unilateral hydronephrosis. b. Urinary tract infection
The obstruction is likely to be at the level of: c. Posterior urethral valves
 (JIPMER May 2016) d. Teratoma
a. Vesicourethral junction b. Ureters 109. The most common underlying anomaly in a child with
c. Ureteropelvic junction d. Urethra recurrent urinary tract infection is: M 
100. Most common cause of hydronephrosis in children M  (AI 2005, AIIMS Nov 2003)
a. PUJ obstruction (Recent Question 2017) a. Posterior urethral valves b. Vesicoureteric reflux
b. Posterior urethral valve c. Neurogenic bladder d. Renal calculi
c. Ureterocele 110. Investigation of choice in VUR M  (TN PGMEE 2005)
d. Ectopic ureter a. Ultrasound b. MRI
101. The most common cause of renal scarring in a 3 years c. Voiding cystourethrogram
old child is: M  (Recent Question 2016) d. Cystoscopy
111. Vesicoureteric reflux is more common in:
a. Trauma b. Tuberculosis
 (AIIMS May 2004)
c. Vesicoureteral reflux induced pyelonephritis
d. Interstitial nephritis a. Newborn females b. Older girls
102. The treatment of choice for primary grade V VUR c. Older boys d. Only during pregnancy
involving both kidneys in a 6 month old boy is: 112. A one year old male child presented with poor urinary
 (Recent Question 2015) stream since birth. The investigation of choice for
a. Antibiotic prophylaxis evaluation is:  (AIIMS May 2003)
b. Ureteric implantation a. Voiding cystourethrography (VCUG)
c. Cystoscopy followed by subureteric injection of Teflon b. USG bladder
d. Bilateral ureterostomies c. Intravenous urography d. Uroflowmetry
Chapter 19: Disorders of Kidney and Urinary Tract 437

113. A 6 month old girl is having recurrent UTI. Ultrasound 122. A boy suffering from acute pyelonephritis, most specific

Questions
abdomen shows bilateral hydronephrosis. MCU shows urinary finding will be: (AIIMS Nov 2006)
B/L grade IV VUR. The treatment that should be done a. Leukocyte esterase test b. WBC casts
immediately is: (AIPGMEE 2002) c. Nitrite test d. Bacteria in gram stain
a. Endoscopic injection of polyteflon at ureteric orifices 123. Which of the following is the most appropriate method
b. Ureteric reimplantation for obtaining a urine specimen for culture in an 8 month
c. Bilateral ureterostomy d. Prophylactic antibiotics old girl? (AIIMS May 2004)
114. Most common cause of urinary obstruction in a male a. Suprapubic aspiration b. Indwelling catheter sample
infant is: (AIPGMEE 2001) c. Clean catch void d. Urinary bag sample
a. Anterior urethral valves b. Posterior urethral valves 124. An 8 years old boy during a routine check up is found to
c. Stone d. Stricture have E. coli 1,00,000 cc/ml on a urine culture. The urine
115. A 6-year-old girl presents with recurrent E. coli infection specimen was obtained by mid stream clean catch void.
in urine. Ultrasound of abdomen shows hydroureter and The child is asymptomatic. Which is the most appropriate
hydronephrosis. Micturating cystourethrogam shows next step in the management?  (AIIMS May 2004)
defect in urinary bladder. The likely diagnosis is: a. Treat as an acute episode of urinary tract infection
a. Sacrococcygeal teratoma (AIPGMEE 2000) b. No therapy
b. Vesicoureteric reflux – grade II c. Prophylactic antibiotics for 6 months
c. Duplication of ureter d. Ureterocele d. Administer long term alkalinizer
116. Most common cause of acute retention of urine in a 125. True about UTI: (PGI Dec 2003)
child of 2 years age: (PGI Dec 2000)
a. Common in female infants
a. Posterior urethral valve b. Duplication of renal pelvis b. If two episodes of UTI in female of 7 years occur, then
c. Meatal ulceration with scabbing cystometric evaluation needed.
d. Urethral stones c. If two episodes of UTI in male of 5 years occur, then
117. A 4 months old female child presents with grade IV cystometric evaluation needed.
VUR, without dilation of urinary bladder. The treatment d. Urine should be obtained by suprapubic aspiration for
of choice is: (AIPGMEE 2000) culture in infants
a. Septran + follow up b. Re-implantation of ureter e. 70% of neonates with UTI have VUR
c. Injection of collagen at ureteric orifices 126. Which one of the following statement is FALSE with
d. Bilateral ureterostomy regard to pyuria in children: (AIPGMEE 2003)
a. Presence of more than 5 WBC/hpf for girls and more than
� URINARY TRACT INFECTIONS
3 WBC/hpf for boys
118. A 4 months old infant with UTI was treated for 14 b. Infection can occur without pyuria
days with IV cefotaxime. What is the next step in c. Pyuria maybe present without UTI
management? (JIPMER May 2017) d. Isolated pyuria is neither confirmatory nor diagnostic for
a. USG only b. USG + MCU urinary tract infection
c. USG + DMSA 127. Which one of the following statements is false with regard
d. USG + MCU + DMSA to xanthogranulomatous pyelonephritis in children:
 (AI 2003)
119. A 5 years old male child presents with complaints of
fever and abdominal distension. He is having vomiting a. Often affects those younger than 8 years of age
for the last five days. On examination there are 6–8 pus b. It affects the kidney focally more frequently than diffusely
cells/hpf in urine. WBC count shows 78% neutrophils. c. Boys are affected more frequently
What is the best line of management: d. Clinical presentation in children is same as in adults
 (Recent Question 2014) 128. A 6 years old girl presents with recurrent E.coli infection
a. Send urine for culture and sensitivity and wait for results in urine. Ultrasound of abdomen shows hydroureter and
hydronephrosis. Micturating cysto-urethrogram shows
b. Send urine for culture and sensitivity and start IV anti­
filling defect in urinary bladder. The likely diagnosis is:
biotics immediately
c. Send urine for culture, do an USG and start chloroquine a. Sacro coccygeal teratoma (AI 2000)
b. Vesicouretric reflex – grade II
d. Radionuclide studies
c. Duplication of ureter
120. Least likely cause of renal papillary necrosis?
d. Ureterocele
 (Recent Question 2014; APPG 2014)
a. Sickle cell disease b. Analgesic nephropathy  ACUTE KIDNEY INJURY
c. Posterior urethral valves d. Diabetes with UTI
121. Straining and dribbling of urine in a male infant with 129. Which of the following is true for a child with acute
recu­rrent urinary infection should lead to the suspicion kidney injury? (Recent Question 2017)
of: (UPSC 07)
a. Urine Specific gravity is higher in pre-renal failure
a. Vesico-ureteric reflux b. Posterior urethral valve b. Schwartz formula uses only serum Cr to calculate GFR
c. Pelvic ureteric junction obstruction c. Greater loss of sodium through urine in pre-renal failure
d. Phimosis d. S. Creatinine value may remain normal
438 Section 3: Systemic Pediatrics
Review of Pediatrics and Neonatology

130. A 4-year-old male child presents with fever, anemia and 139. An infant with failure to thrive, hypertension, metabolic
azotemia after an episode of dysentery 9 days earlier. alkalosis and hyperkalemia presents to a clinician. Most
The commonest organism responsible for this condition probable cause is (Recent Question 2018)
is? (COMEDK 2016) a. Liddle's syndrome b. Bartter's syndrome
a. Meningococcus b. E.coli c. Gittelman's syndrome d. Gordon syndrome
c. E. histolytica d. Staphylococcus 140. All are features of Bartter syndrome except
131. In children, renal failure in terms of urine output is  (MAHA PGM CET 2016)
defined as? M  (Recent Question 2013) a. Hypokalaemia b. Metabolic Alkalosis
a. Less than 0.3 ml/kg/hr b. Less than 0.5 ml/kg/hr c. Hypocalciuria d. Salt wasting
c. Less than 0.8 ml/kg/hr d. Less than 1 ml/kg/hr 141. A 6 month infant presented with recurrent episodes of
132. An 8 day old breast fed baby presents with vomiting, poor polyuria and dehydration. Lab tests revealed serum Na
feeding and loose stools. On examination, the heart rate of 130 meq/L, K of 3 meq/L, Cl– of 92 meq/L, bicarbonate
is 190/min, blood pressure 50/30 mm Hg, respiratory of 30 meq/L. pH was 7.48. Serum creatinine was 0.4
rate 72 breaths/min and CFT~4sec. Investigations show mg/dl. The patient had hypercalciuria and ultrasound
hemoglobin level of 15 gm/dL. Na+ 120 meq/L, K+ 6.8 showed medullary nephrocalcinosis. Which of the
meq/L, Cl– 81 meq/L, bicarbonate 15 meq/L, Urea 30 mg/ following is the most likely diagnosis?  (APPG 2015)
dL, and creatinine 0.6 mg/dL, the most likely diagnosis
is: (DNB June 2012) a. Liddle syndrome
b. Gitelman syndrome
a. Congenital adrenal hyperplasia
b. Acute tubular necrosis c. Bartter syndrome
c. Congenital hypertrophic pyloric stenosis d. Distal renal tubular acidosis
d. Galactosemia
142. A male child with Fanconi syndrome with nephro­
133. A child has diarrhea since 8 days. He is dehydrated
and urine output is reduced. Which of the following is calcinosis has a variant of dent disease. All are true
not correct regarding the renal failure in this patient? except: (AIIMS May 2011)
 (AIPGMEE 2010) a. Hypercalciuria
a. Urinary sodium > 40 mEq/L b. Proteinuria
b. Urinary osmolality > 500 mosm/L c. Similar presentation in father
c. FENa < 1% d. BUN/Creatinine > 20 d. Rickets
134. An infant with severe dehydration secondary to diarrhea 143. The maximum urinary concentration capacity in full
suddenly presents with flank mass and blood in urine. term neonates is:  (DNB June 2011)
The most probable diagnosis is? (DNB Dec 2010)
a. > 1000 mOsm/litre b. 350–450 mOsm/litre
a. Renal vein thrombosis b. Pyelonephritis c. 900–1000 mOsm/litre d. 600–700 mOsm/litre
c. Acute glomerulonephritis d. Lower nephrosis
144. A 9-years-old female child presented with polyuria,
135. Renal causes of acute renal failure in children
polydipsia and metabolic acidosis. On slit lamp exami­
include:
nation crystal deposits are seen in cornea. What is the
a. Minimal changes disease (PGI Dec 2001)
diagnosis?  (TN PGMEE 2010)
b. Renal amyloidosis c. Pre-eclampsia
d. Malignant hypertension a. Cystinuria b. Cystinosis
e. Hemolytic uremic syndrome c. Cystothioninuria d. Homocysteinuria
145. A 3 month old infant presents with bilateral medullary
� CHRONIC KIDNEY DISEASE nephrocalcinosis. All of the following can cause
medullary nephrocalcinosis except: (AIIMS May 2009)
136. In Schwartz formula for calculation of creatinine clea­
rance in a child, the constant depends on the following a. Hyperoxaluria
except: (AIIMS Nov 2006) b. Bartter's syndrome
a. Age c. Prolonged use of furosemide
b. Method of estimation of creatinine d. ARPKD
c. Mass 146. The neonatal kidney achieves concentrating ability equi­­
d. Severity of renal failure valent to adult's kidney by:  (AIPGMEE 2004)
137. In a child, non-functioning kidney is best diagnosed by: a. One year of age b. Eighteen months of age
 (AIPGMEE 2005) c. Three to six months of age
a. Ultrasonography b. IVU d. Just before puberty
c. DTPA renogram d. Creatinine clearance 147. A 2 months old girl has failure to thrive, polyuria and
medullary nephrocalcinosis affecting both kidneys.
 RENAL TUBULAR DISORDERS Investigations show blood pH 7.48, bicarbonate 25
meq/L, potassium 2 meq/L, sodium 126 meq/L and
138. A 10-year-old girl present with polyuria and polydipsia chloride 88 meq/L. The most likely diagnosis is:
with hypokalemia, hypercalciuria and metabolic
alkalosis. What is the probable diagnosis? a. Distal renal tubular acidosis (AIIMS Nov 2004)
 (JIPMER May 2018) b. Primary hyperaldo steronism
a. Gitelman syndrome b. Liddle syndrome c. Bartter syndrome
c. Bartter syndrome d. Alport's syndrome d. Pseudohypoaldosteronism
Chapter 19: Disorders of Kidney and Urinary Tract 439

148. A 10 months old boy, weighing 3 kg has polyuria, a. Ureterolithotomy b. Endoscopic removal

Questions
polydipsia and delayed motor milestones Investigations c. ESWL d. Observation
show blood levels of creatinine 0.5 mg/dL, potassium 3 152. A child presents with abdominal colic and hematuria.
meq/L, sodium 125 meq/L, chloride 88 meq/L, calcium On ultrasonography a stone 2.5 cm in diameter is seen
8.8 mg/dL, pH – 7.46, and S. bicarbonate 26 meq/L. in the renal pelvis. The next step in management of this
Ultrasonography shows medullary nephrocalcinosis. case is:  (AIIMS Nov 2000, AI 2001)
The most likely diagnosis:  (AIIMS Nov 2003, May 03) a. Pyelolithotomy b. Nephroureterostomy
a. Renal tubular acidosis b. Diabetes insipidus c. Conservative d. ESWL
c. Bartter syndrome d. Pseudo-hypoaldosteronism
149. A 3 years old boy is detected to have bilateral renal
calculi. Metabolic evaluation confirms the presence of � RENAL PHYSIOLOGY
marked hypercalciuria with normal levels of calcium,
153. Regarding FeNa, which of the following is true?
magnesium, phosphate, uric acid and creatinine. A
 (JIPMER 2017)
diagnosis of idiopathic hypercalciuria is made. The
dietary management includes all, except: a. FeNa is lower in neonates when compared to children
 (AIIMS May 2003, Nov 2004) b. FeNa is higher in intrinsic renal failure than prerenal
a. Increased water intake failure
b. Low sodium diet c. Measurement of FeNa is not affected by use of diuretic
c. Reduced calcium intake d. FeNa is similar in both preterm and term neonate
d. Avoid meat proteins 154. A 2-year-old child with length 85 cm and weight of 11
150. A 10 years old boy is having polyuria, polydipsia. kg was found to have serum urea–49 mg/dL, & serum
Laboratory data showed (in meq/L): (AIIMS May 2002) creatinine 2 mg/dL. What is the estimated GFR of this
child, as per Schwartz formula? (JIPMER 2017)
Na+ – 154
K+ – 4.5 a. 90 b. 48
HCO3– 22 c. 18 d. 9
S. Osmolality – 295 155. Following parameters are used in Schwartz formula for
B. Urea – 50 estimating glomerular filtration rate:
Uric specific gravity – 1.005  (JIPMER Nov 2017)
What is the diagnosis? a. 24 hour fluid intake & Urine output
a. Diabetes insipidus b. Renal tubular acidosis b. Serum creatinine & urine output
c. Bartter's syndrome d. Recurrent UTI c. Height & serum creatinine
151. A 5 years old child presents with a calculus of size 2 d. Height & muscle mass
cm in the upper ureter. He also complains of hematuria. 156. Maximum urine osmolality in preterm neonate: M 
USG shows no further obstruction in the urinary tract.  (JIPMER May 2017)
Treatment of choice for this patient would be: a. 200 b. 400
 (AI 2001)
c. 500 d. 800
440 Section 3: Systemic Pediatrics
Review of Pediatrics and Neonatology

Answers with Explanations


 CONGENITAL ANOMALIES OF KIDNEY
1. c. Bilateral renal agenesis  Ref: Nelson 20/ed p 2554
Potter syndrome
•• Bilateral renal agenesis incompatible with extrauterine life produces Potter syndrome.
•• Death occurs shortly after birth from pulmonary hypoplasia
•• Potter facies: Eyes are widely separated with epicanthic folds, ears are low set, nose is broad & compressed flat, chin is receding,
and there are limb anomalies.
Neonates with bilateral renal agenesis die of pulmonary insufficiency from pulmonary hypoplasia rather than renal failure.
2. b. Renal agenesis  Ref: Nelson 20/e 2513
Components of oligohydramnios complex (Potter syndrome), including low-set ears, micrognathia, flattened nose, limb positioning
defects, intrauterine growth restriction & death may result from pulmonary hypoplasia.
Bilateral renal agenesis is incompatible with extrauterine life & produces the Potter syndrome.
3. a. Can be diagnosed intrauterine and b. May proceed to renal failure by 10 yr age Ref: Nelson 20/e 2513
a. ARPKD can be diagnosed on antenatal USG as bilateral, symmetrical, enlarged, and hyper-echogenic fetal kidneys with loss of
cortico-medullary differentiation
b. 50% of patients with a neonatal-perinatal presentation develop end-stage renal disease (ESRD) by age 10 yr.
c. Children with ARPKD usually present with bilateral flank masses during the neonatal period or in early infancy.
d. The gene for ARPKD is PKHD1 (polycystic kidney and hepatic disease) that encodes fibrocystin
e. In ARPKD, both kidneys are markedly enlarged & grossly show innumerable cysts throughout cortex & medulla
4. c. Hypospadias  Ref: Nelson 20/e p 2576-2578
•• Girls have epispadias, with separation of the 2 halves of the clitoris and wide separation of the labia
•• Ectopia vesicae also occurs in both sexes; But hypospadias is seen only in males.
5. c. ARPCKD  Ref: Robbins 9th/pg 945-949; 8th/pg 956-960, Ghai 8/e p 507-509
Inherited Cystic Kidney Diseases

Disease Gene Protein Renal Abnormally Extrarenal Abnormally


AD polycystic kidney AD PKD1 (Chr 16) Polycystin-1 Cortical and medullary Cerebral aneurysms;Q liver and
diseaseQ PKD2 (Chr 4) Polycystin-2 cysts spleen cysts
Infantile polycystic AR PKHD1 (Chr 6) Fibrocystin Distal tubule and Hepatic fibrosis;Q Caroli's
kidney diseaseQ (polyductin) collecting duct cysts diseaseQ
Medullary cystic kidney AD MCKD1/2 Uromodulin Small fibrotic kidneys; Hyperuricemia and goutQ
disease medullary cysts

6. c. 16 and 4  Ref: Nelson's 20/e p, Ghai 8/e p 507-509


7. c. Polycystic kidney  Ref: Nelson's 20/e p, Ghai 8/e p 507-509
8. c. Mesonephric duct disappears  Ref: Ghai 8/e p 464-466
Summary of Kidney development:
•• The kidney is derived from the ureteral bud and the metanephric blastema
•• Ureteral bud arises from mesonephric duct and forms entire collecting system including ureter, renal pelvis, collecting tubules
•• During renal development, the kidneys normally ascend from the pelvis into their normal position behind the ribs.
9. c. Single umbilical artery  Ref: Nelson's 20/e p 2554
Malformations associated with Unilateral Renal Agenesis are: single umbilical artery, contralateral vesicoureteric reflux, absent
ureter, absent ipsilateral vas deferens.
10. a. Uretero pelvic junction stenosis  Ref: Nelson 20/e p 2570
Ureteropelvic junction (UPJ) obstruction is the most common obstructive lesion in childhood and usually is caused by
intrinsic stenosis.
11. c. Genitourinary  Ref: Nelson's 20/e p 2554
12. b. Hepatic cyst and fibrosis  Ref: Nelson's 20/e p, Ghai 8/e p 507-509
Chapter 19: Disorders of Kidney and Urinary Tract 441

In the given question, neonatal death along with the presence of bilaterally enlarged kidneys with multiple cysts indicate

Answers with Explanations


ARPKD (autosomal recessive polycystic kidney disease).
ARPKD is associated with hepatic fibrosis and portal hypertension which is the correct answer.
13. a. Autosomal dominant  Ref: Nelson's 20/e p, Ghai 8/e p 507-509
Infantile polycystic kidney disease is autosomal recessive.
14. b. Hypospadias  Ref: Nelson 20/e p 2573-2578
Hypospadias is a urethral opening on the ventral surface of the penile shaft affecting 1 in 250 male newborns.
Exstrophy of urinary bladder:
•• It occurs in 1 in 35,000-40,000 births; Male: female ratio is 2 : 1
•• Severity ranges from simple epispadias (in boys) to cloacal exstrophy. Posterior urethral valve:
The most common cause of severe obstructive uropathy in children is posterior urethral valves, affecting 1 in 8,000 boys.
15. c. Unilateral renal dysplasia  Ref: Nelson's 20/e p 2555
Multicystic dysplastic kidney
•• It is the most frequent form of renal cystic disease in childhood; It is usually unilateral,
•• DTPA renography shows no function and USG or CT scanning shows replacement of kidney with cysts of varying sizes that do not
communicate with each other

Simple, renal solitary cysts: Mostly observed in adults, are uncommon in children.
16. d. All the above  Ref: Nelson's 20/e p 2554-2555
Potter Syndrome:
•• Bilateral renal agenesis and Pulmonary hypoplasia are seen
•• Potter facies: widely separated eyes with epicanthic folds, low set ears, broad compressed flat nose, receding chin
•• This condition is incompatible with life, death occurs shortly after birth from pulmonary hypoplasia
•• Maternal USG demonstrates, Oligohydramnios, Non-visualization of bladder, Absent fetal kidney

17. d. Multicystic dysplastic kidneys  Ref: Nelson's 20/e p 2555, Ghai 8/e p 509
a. Neuroblastoma is the MC extra cranial solid tumor in children & most commonly diagnosed malignancy in infants
b. Wilm's tumor or Nephroblastoma is the most common primary malignant renal tumor of childhood and the second most
common malignant abdominal tumor in childhood
c. Distended bladder: examination after voiding rules out swelling due to distended bladder
d. Multi cystic dysplastic kidney (MCDK) is the most common cause of an abdominal mass in the newborn, but the vast
majority are non palpable at birth.
18. d. Nephronophthisis  Ref: Nelson's 20/e p, Ghai 8/e p 509
In the given case scenario, a 12 years old boy presents with:
•• Clinical features of chronic kidney disease (CKD) like short stature, nocturnal enuresis, and
•• Biochemical evidence of CKD like high serum creatinine, low sodium, borderline calcium and raised ALP
•• Ultrasound shows bilateral small kidneys – again an evidence of chronic kidney disease.
About juvenile nephronophthisis (JN)–medullary cystic kidney disease complex (MCKD):
•• It is a group of inherited, genetically determined cystic renal diseases that share the common histologic finding of chronic
tubulointerstitial nephritis; Usually inherited as an autosomal recessive disease
•• Presents with polyuria, growth failure, unexplained anemia and chronic renal failure in late childhood or adolescence
•• Often associated with extrarenal features such as retinal degeneration, hepatobiliary disease, cerebellar vermis hypoplasia, laterality defects,
intellectual disability, and shortening of bones

19. b. Nephronophthisis  Ref: Nelson's 20/e p 2537


In this case bilateral small kidneys with clinical features of renal failure & polyuria are suggestive of
Nephronophthisis.

 NEPHROTIC SYNDROME
20. c. Spot urine sample for protein/creatinine ratio  Ref: Nelson 20/e 2518
Though 24 hr urine for protein and creatinine, is more accurate, but due to difficulty in collecting 24 hour urinary sample in children,
Spot urine sample for protein/creatinine ratio is preferred.
21. b. Hypertension  Ref: Nelson 20/e 2524-2525
Important features of minimal change idiopathic nephrotic syndrome are the absence of hypertension and gross hematuria
22. d. No immunodeposits  Ref: Nelson's 20/e p 2521-2528, Ghai 8/e p 477-482
In minimal change disease:
•• Light microscopy: Glomeruli appear normal: No Ig/complement deposits, GBM: normal
442 Section 3: Systemic Pediatrics

•• Immunofluorescence microscopy: No electron-dense deposits


Review of Pediatrics and Neonatology

•• Electron microscopy: Diffuse effacement of foot processes of podocytes (“podocytopathy”).


23. d. Hypocholesterolemia  Ref: Nelson's 20/e p 2521-2528, Ghai 8/e p 477-482
Features of nephritic syndrome are: Hematuria and RBC casts, Nephritic range Proteinuria (<3.5g/day), hence edema is present,
Hypertension, Uremia and Oliguria.
24. a. Minimal change disease  Ref: Nelson's 20/e p 2521-2528, Ghai 8/e p 477-482
Most likely kidney biopsy finding associated with malignancy in children is Minimal change disease
Most likely kidney biopsy finding associated with malignancy in Adults is Membranous Glomerulonephropathy.
25. b. Minimal change disease  Ref: Nelson's 20/e p 2521-2528, Ghai 8/e p 477-482
In the present case, massive proteinuria (3+) along with hypoalbuminemia is suggestive of nephrotic syndrome. Out of the
options, minimal change disease is the most common cause in children.
26. b. Severe hypoalbuminemia  Ref: Nelson's 20/e p 2521-2528, Ghai 8/e p 477-482
27. a. IgA nephropathy  Ref: Nelson's 20/e p 2496

High Yield Points  


•• IgA nephropathy is the most common type of glomerulonephritis worldwide
•• IgA nephropathy is the most common cause of gross hematuria

28. a. Minimal change disease  Ref: Nelson's 20/e p 2521-2528, Ghai 8/e p 477-482
29. d. 40 mg/m2/hr  Ref: Nelson's 20/e p 2521-2528

High Yield Points  


•• Microalbuminuria: Excretion of 30-300 mg/dayQ of albumin in urine or 30-300 mg albumin/gm of creatinine in urineQ
•• Macroalbuminuria: Excretion of 300-3500 mg/day of albumin in urineQ
•• Nephrotic range proteinuria: Excretion of > 3.5 gm/dayQ of albumin in urine or 40 mg/m2/hr

30. a. Peak incidence is between 2 and 4 years of age, d. Selective proteinuria  Ref: Nelson's 20/e p 2521-2528
31. b. Minimal change disease  Ref: Nelson's 20/e p 2521-2528, Ghai 8/e p 477-482
32. d. Steroid-induced diabetes  Ref: Nelson's 20/e p 2521-2528, Ghai 8/e p 477-482
Diarrhoea induced water depletion (dehydration) is a common cause of Acute Renal Failure in children. Renal vein thrombosis can
also predispose to AKI in a child with diarrhea and dehydration
33. a. Low serum calcium  Ref: Nelson's 20/e p 2521-2528, Ghai 8/e p 477-482
In nephrotic syndrome there is hypocalcemia and secondary hyperparathyroidism due to loss of cholecalciferol binding
protein; Secondary hyperparathyroidism causes hypophosphatemia by increasing renal phosphate excretion.
34. a. Minimal change disease  Ref: Nelson's 20/e p 2521-2528, Ghai 8/e p 477-482
35. a. Spontaneous bacterial peritonitis, b. Sepsis, c. DVT, d. Atherosclerosis  Ref: Nelson's 20/e p 2521-2528
36. b. Congenital Finnish type nephrotic syndrome  Ref: Nelson's 20/e p 2521-2528, Ghai 8/e p 477-482
37. d. Minimal change disease  Ref: Nelson's 20/e p 2521-2528, Ghai 8/e p 477-482
38. a. Levamisole  Ref: Pediatric Nephrology 5th ed, page 205-206
Levamisole has been successfully used in patients with Frequently relapsing Nephrotic syndrome (FRNS) & Steroid Dependent
Nephrotic syndrome (SDNS); If relapses continue despite Levimasole therapy, Cyclophosphamide is recommended.
39. a. Cause steroid resistant Nephrotic syndrome, b. Nephrin is a key component of slit diaphragm and c. Coded by NPHS-1 gene
Ref: Nelson 20/e p 2521-2528
Congenital nephrotic syndrome
•• Defined as nephrotic syndrome manifesting at birth or within 1st 3 months of life
•• The Finnish type of congenital nephrotic syndrome is caused by autosomal recessive mutations in NPHS1 or NPHS2 gene,
which encodes nephrin and podocin, critical components of the slit diaphragm
•• Affected infants most commonly present at birth with edema caused by massive proteinuria.
40. c. Hypoalbuminemia  Ref: Nelson 20/e p 2521 and Harrison 19th ed/pg 253
Edema is the most common presenting symptom of children with nephrotic syndrome. Causes of Edema:
In Nephrotic Syndrome: (primarily hypoalbuminemia)
•• Primary alteration in the nephrotic syndrome is a diminished colloid oncotic pressure due to losses of large
•• quantities ( 3.5 g/day) of protein into the urine
•• With severe hypoalbuminemia and reduced colloid osmotic pressure, the sodium and water that are retained cannot be restrained within
the vascular compartment, 50 total and effective arterial blood volumes decline
•• This process initiates the edema-forming sequence of events described above, including activation of the RAAS
Chapter 19: Disorders of Kidney and Urinary Tract 443

In Nephritic Syndrome:

Answers with Explanations


•• Edema that occurs associated with hematuria, proteinuria, and hypertension
•• Although some evidence supports the view that the fluid retention is due to increased capillary permeability, in most instances, the edema
results from primary retention of sodium and water by the kidneys owing to renal insufficiency

41. b. Oral cyclosporine  Ref: Nelson's 20/e p 2521-2528, Ghai 8/e p 477-482
FSGS:

•• Renal biopsies show FSGS in about 30–35% children with non minimal nephrotic syndrome
•• It is the most common cause of acquired kidney failureQ necessitating renal replacement therapies in children
•• Most patients with FSGS have initial or late corticosteroid resistance; Cyclophosphamide is generally ineffective;
•• The best results in FSGS have been obtained with the use of calcineurin inhibitors like cyclosporine

42. a. No IgG deposits or C3 deposition on renal biopsy  Ref: Nelson's 20/e p 2521-2528, Ghai 8/e p 477-482
This is a classic presentation of Minimal change nephrotic syndrome (MCNS); In MCNS, light microscopy is normal and
glomerular deposits are not seen on immunofluorescence examination. About the other options.

Disease Characteristic features


IgA nephropathy Recurrent gross hematuria is the typical manifestation
On light microscopy, focal segmental proliferation of mesangial matrix is seen
Immunofluorescence shows mesangial deposits of IgA
Alport syndrome X–linked recessive; progressive nephropathy, sensorineural hearing loss and ocular abnormalities
Electron microscopy shows characteristic abnormalities of the glomerular capillary basement membrane

43. None  Ref: Nelson's 20/e p 2521-2528, Ghai 8/e p 477-482


Idiopathic Nephrotic syndrome is associated with:
•• Minimal change disease •• Membranoproliferative glomerulonephritis
•• Focal segmental glomerulosclerosis •• Crescentic glomerulonephritis
•• Membranous nephropathy •• IgA nephropathy (Mesangioproliferative glomerulonephritis)

44. a. Minimal change disease  Ref: Nelson's 20/e p 2521-2528, Ghai 8/e p 477-482
45. d. Streptococcus pneumoniae  Ref: Nelson Textbook of Pediatrics 20th pg 2522
Infections in children with Nephrotic syndrome
•• Especially susceptible to cellulitis, spontaneous bacterial peritonitis (SBP) and bacteremia
•• Due to urinary losses of IgG and complement factors, impaired opsonization of microorganisms and increased risk of infection with
encapsulated bacteria, especially pneumococcus
•• SBP presents with fever, abdominal pain and peritoneal signs; Pneumococcus is the most frequent cause of SBP
•• Peritoneal leukocyte counts >250/mm3 are highly suggestive of spontaneous bacterial peritonitis

46. a. Loss of antithrombin III  Ref: Nelson's 20/e p 2521-2528, Ghai 8/e p 477-482
Hypercoagulability: Thrombosis (venous, arterial) of major vessels is seen in Nephrotic syndrome.
Reason: urinary losses of antithrombotic factors such as antithrombin III (most important) and protein S due to vascular stasis,
increased platelet number and aggregability, increase in hepatic production of fibrinogen.
47. c. Hypoalbuminemia  Ref: Nelson's 20/e p 2521-2528, Ghai 8/e p 477-482; Refer Ans. 35
48. a. No IgG deposits or C3 deposition on renal biopsy  Ref: Nelson's 20/e p 254, Ghai 8/e p 477
49. b. NPHS 2  Ref: Nelson's 20/e p 2521-2528, Ghai 8/e p 477-482; Refer pretext for details;
50. a. Minimal change disease, b. RPGN c. MPGN; and d. Membranous nephritis  Ref: Nelson's 20/e p 2506, 2521-2528
51. a. Spontaneous bacterial peritonitis  Ref: Nelson's 20/e p 2521-2528, Ghai 8/e p 477-482
52. b. NPHS2  Ref: Nelson's 20/e p 2521-2528, Ghai 8/e p 477-482, Pediatric Nephrology by Ellis D. Avner 6/e pg 645
Most common gene defect in idiopathic steroid resistant nephrotic syndrome is NPHS2.
53. c. Nephrin  Ref: Nelson's 20/e p 2521-2528, Ghai 8/e p 477-482
Congenital Nephrotic syndrome more commonly occurs due to mutation of NPHS1gene that encodes nephrin
54. b. Cyclophosphamide  Ref: Nelson's 20/e p 2521-2528, Ghai 8/e p 477-482
444 Section 3: Systemic Pediatrics

•• This child with SDNS has evidence of steroid toxicity (cushingoid appearance, hypertension, cataracts);
Review of Pediatrics and Neonatology

•• Cyclophosphamide prolongs the duration of remission in SDNS/FRNS patients


•• Levamisole is usually the first line treatment for SDNS; In Levamisole failure, Cyclophosphamide is used.
55. b. Massive proteinuria, d. Edema, e. Hyperlipidemia  Ref: Nelson's 20/e p 2521-2528, Ghai 8/e p 477-482
56. a. Minimal change disease is commonest cause, e. Spontaneous bacterial peritonitis is associated with it
Ref: Nelson's 20/e p, 2521-2528
57. a. No finding  Ref: Nelson's 20/e p 2521-2528
This is a case of typical MCNS (Minimal Change Nephrotic Syndrome) with a good response to steroid therapy.

 NEPHRITIC SYNDROME
58. a. Irregular focal thickening of the glomerular basement membranes  Ref: Nelson 20/e 2496-2497
The given history of hematuria 2-3 days after an upper respiratory infection suggests a diagnosis of IgA Nephropathy. Electron
microscopy in IgA nephropathy reveals electron microscopy reveals diffuse thickening, thinning, splitting, and layering of the
glomerular and tubular basement membranes
59. b. Conservative therapy  Ref: Nelson 20/e 2500-1
The given clinical picture suggests a diagnosis of nephritic syndrome, most probably post-streptococcal glomerulonephritis (PSGN).
Management of PSGN involves:
•• Treating the acute effects of renal insufficiency and hypertension.
•• A 10 day course of systemic antibiotic therapy is recommended to limit the spread of the nephritogenic organisms, antibiotic
therapy does not affect the natural history of APSGN.
•• Na restriction, diuresis, Ca channel antagonists, vasodilators, or ACE inhibitors, are standard therapies used to treat hypertension.
60. a. Berger's disease  Ref: Nelson 20/e p 2496-2497
In Berger's disease (IgA Nephropathy), gross hematuria often occurs within 1-2 days of onset of an upper respiratory or
gastrointestinal infection, in contrast to the longer latency period observed in acute post-infectious glomerulonephritis, and may
be associated with loin pain.
61. b. Not elevated  Ref: Nelson 20/e p 2500-2501, Ghai 8/e p 474-477
Lab Diagnosis of PSGN;
•• Serum C3 level is reduced in > 90% of patients in acute phase of PSGN, and returns to normal in 6-8 wk
•• Confirmation of the diagnosis requires evidence of a prior streptococcal infection eg positive throat culture or rising antibody titer to
streptococcal antigen(s)
•• Antistreptolysin O titer is commonly elevated after a pharyngeal infection but rarely increases after streptococcal skin infections.Q (as in this
case)
•• Best single antibody to document cutaneous streptococcal infection is anti deoxyribonuclease (DNase) B level

62. c. 6-8 weeks  Ref: Nelson 20/e p 2500-2501, Ghai 8/e p 474-477;
63. b. Penicillin  Ref: Nelson 20/e p 2500-2501, Ghai 8/e p 474-477
Treatment of PSGN
•• Management is directed at treating the acute effects of renal insufficiency and hypertension
•• Although a 10 day course of systemic antibiotic therapy with penicillin is recommended to limit the spread of the
nephritogenic organisms, antibiotic therapy does not affect the natural history of PSGN
•• Treatment of hypertension: Calcium channel antagonists, vasodilators, or angiotensin-converting enzyme inhibitors.
64. d. Sinding-Larsen-Johansson Syndrome  Ref: Nelson's 20/e p 2497-2498, Ghai 8/e p 473, 500
Discussing about the options one by one,

Options Clinical manifestations


a. Lowe Syndrome Oculocerebrorenal syndrome

b. Denys-Drash Syndrome Wilms tumor in kidney

c. Alport Syndrome Hematuria, eye defects and sensorineural hearing loss

d. Sinding-Larsen – Johansson Syndrome Point tenderness at the inferior pole of the patella associated with focal swelling

65. b. Berger disease  Ref: Nelson's 20/e p 2496-2497, Ghai 8/e p 476
IgA Nephropathy or Berger disease presents as Gross hematuria 1-2 daysQ after a respiratory, urinary tract or GI infection.
66. b. Nephritic syndrome  Ref: Nelson 20/e p 2500-2501, Ghai 8/e p 474-477
67. a. Epithelial humps  Ref: Nelson 20/e p 2500-2501, Ghai 8/e p 474-477
68. d. Good pasture syndrome  Ref: Nelson's 20/e p 2506, Ghai 8/e p 474-477
Chapter 19: Disorders of Kidney and Urinary Tract 445

Types of RPGN:

Answers with Explanations


Entity Type I (20%) Type II (25%) Type III (55%)
Mechanism Anti-GBM Antibody Immune Complex Pauci-Immune
Etiology Good pasture •• Post-infectious: Post streptococcal GN ANCA-associated
syndromeQ •• Noninfectious: SLE, Henoch-Schönlein purpura, Mixed •• Wegener granulomatosisQ
cryoglobulinemia •• Microscopic polyangiitisQ
•• Primary Renal Disease: IgA nephropathyQ •• Hypersensitivity vasculitisQ
•• Membranoproliferative glomerulonephritisQ

69. c. Hypoalbuminemia  Ref: Nelson's 20/e p 2498-2500, Ghai 8/e p 474-477


70. c. Recurrent gross hematuria following respiratory infection  Ref: Nelson's 20/e p 2496-2497, Ghai 8/e p 476
71. a. 4–8 years  Ref: Nelson 20/e p 1216, Ghai 8/e p 632-633
•• Henoch-Schönlein purpura (HSP) is the most common small vessel vasculitis in childhood
•• 90% of HSP cases occur in children, usually between 3-10 yr age; Presents with purpura, arthritis and abdominal pain.
72. a. C3 decreases, ASO increases  Ref: Nelson 20/e p 2500-2501, Ghai 8/e p 474-477
73. c. Hepatic failure  Ref: Nelson's 20/e p 2508-2510, Ghai 8/e p 474-477, 492-493
Hepatic failure does not occur in HUS. Laboratory findings of HUS include:

•• Thrombocytopenia, Leukocytosis •• Peripheral smear: Helmet cells, burr cells, fragmental RBCs (Schistocytes)
•• ARF: Increased BUN and creatinine •• Hemoglobinuria with hemosiderinuria
•• Microscopic hematuria and Proteinuria •• Normal PT and APTT

Option ‘b' requires some explanation: Though neurological manifestations are not common in HUS, but they may occur.
74. d. Can be (a) or (b)  Ref: Nelson's 20/e p 2498-2500, Ghai 8/e p 474-477
As the time interval between sore throat and hematuria is not mentioned, it can either be IgA Nephropathy or PSGN.
75. a. E. coli  Ref: Nelson's 20/e p 2508-2510, Ghai 8/e p 492-493
Etiology of HUS: HUS usually follows an episode of acute gastroenteritis (less commonly, upper respiratory infection).
The organisms responsible for HUS are:
•• E. coli (most common) esp EHEC (shiga toxin producing) •• Bartonella; Campylobacter
•• Salmonella; Shigella dysenteriae •• Viruses (Coxsackie virus, echovirus, influenza, varicella, HIV, EBV)
•• Streptococcus pneumoniae

76. d. 4 months after onset  Ref: Nelson 20/e p 2505, Ghai 8/e p 632
Screening in a patient of HSP:
•• Most patients who develop nephritis have urinary abnormalities by 1 month, nearly all by 3 months after onset of HSP
•• Therefore, a urinalysis should be performed weekly in patients with HSP during the period of active clinical disease
•• Thereafter, a urinalysis should be performed once a month for up to 6 months; after that nephritis is unlikely

So among the given options 4 months is the best answer, as it is closest to 6 months.
77. a. Thrombocytopenia  Ref: Nelson's 20/e p 1216, Ghai 8/e p 632-633
78. a. Henoch-Schonlein purpura  Ref: Nelson's 20/e p 1216, Ghai 8/e p 632-633
•• The typical non blanching purpura with presence of IgA and neutrophils around vessels confirms the diagnosis of H.S.P
•• Although palpable purpura may be present in Wegener's granulomatosis, biopsy characteristically shows necrotizing
granulomatous vasculitis.
79. b. Caused by non-group A hemolytic streptococci  Ref: Nelson's 20/e p 2498-2500, Ghai 8/e p 474-477
80. c. Seizure  Ref: Nelson's 20/e p 1216, Ghai 8/e p 632-633
81. d. Bleeding diathesis  Ref: Nelson's 20/e p 2498-2500, Ghai 8/e p 474-477
82. b. HSP  Ref: Nelson's 20/e p 1612, 2335; Ghai 8/e p 632
•• Here, a child is presenting with acute nephritic syndrome with history of skin rash manifestations 2 weeks back
•• Skin manifestations are typically absent in IgA nephropathy and Wegener's granulomatosis. About PNH: Hemoglobinuria is
rarely seen in children as compared to adults with PNH. Chronic hemolysis, thrombocytopenia and leukopenia are often
characteristic.
83. d. Chronic interstitial nephritis with VUR  Ref: Nelson's 20/e p 2537, Ghai 8/e p 483
In the given clinical scenario, child has hypertension, renal failure (increased urea and creatinine), pyuria, absence of hematuria
(presence of 5 RBCs/HPF is considered significant) and mild proteinuria, Out of the options given, Chronic Interstitial Nephritis with
VUR seems most appropriate diagnosis.
Chronic Interstitial disease
446 Section 3: Systemic Pediatrics

•• In children, it commonly occurs as a result of underlying obstructive uropathy or VUR


Review of Pediatrics and Neonatology

•• Presence of urinary tract infection due to VUR explains the presence of pus cell on urinanalysis
•• Hypertension is common in chronic interstitial nephritis; Proteinuria is mild < 1.5 g/day.
About other options:
a. Post-infective glomerulonephritis: Absence of hematuria and presence of mild proteinuria rules out post infective
glomerulonephritis. It is always accompanied by significant proteinuria and hematuria
b. Accelerated hypertension with acute renal failure: This cannot explain the presence of pus cells on urinalysis
c. Idiopathic RPGN: This also does not explain the presence of pus cells on urinalysis.
84. b. Henoch Schonlein purpura  Ref: Nelson's 20/e p 1612, Ghai 8/e p 632
Classification criteria for H.S. purpura according to Pediatric Rheumatology European Society:
Palpable purpura (in absence of coagulopathy or thrombocytopenia) and 1 or more of the following criteria:

•• Diffuse abdominal pain •• Biopsy of affected tissue demonstrating predominant IgA deposition
•• Arthritis or arthralgia •• Renal involvement (proteinuria, hematuria or red cell casts)

Hence, it is a typical case presentation of Henoch–Schonlein purpura.


85. b. IgA nephropathy  Ref: Nelson's 20/e p 2496, Ghai 8/e p 632

a. Wilms' tumor Usually presents with abdominal mass. Fever, abdominal pain and hematuria are occasionally seen;

b. IgA nephropathy History of recurrent gross hematuria for 2 years indicates IgA nephropathy

c. PSGN Serum C3 is decreased in 90% and returns to normal in 6-8 wks. Recurrent episodes are rare

d. Urinary tract infection Clinical manifestations of UTI are absent; urine R/M has no pus cells and C/S is sterile

86. c. Thrombocytopenia  Ref: Nelson's 20/e p 1612, Ghai 8/e p 632


•• HSP is characterized by a tetrad of palpable purpura, arthritis, glomerulonephritis and abdominal pain
•• Less common skin lesions are: macules, vesicles, bullous lesions, splinter hemorrhage and ulcerations; Rarely, epistaxis may
occur.
87. c. Mesangial deposits of IgA  Ref: Nelson's 20/e p 1612, Ghai 8/e p 632
The given case scenario (palpable purpura, arthritis, hematuria and proteinuria) suggests a diagnosis of Henoch Schonlein purpura
In H.S. purpura and IgA nephropathy, IgA deposits are present in mesangium (by immunofluorescence)
88. a. Takayasu aortoarteritis  Ref: M. Kanitkar, Indian Pediatrics 2005; 42:47-54; Refer pretext chapter 8;
Renovascular hypertension and Renal artery stenosis:
•• The commonest cause for renovascular hypertension in children reported in western literature is fibromuscular dysplasia (FMD) and the
midaortic syndrome.
•• However, in India, Takayasu disease (idiopathic aortoarteritis) is the most important cause, accounting for upto 87% of children with
renovascular hypertension.
•• Renal artery stenosis may also occur in association with neurofibromatosis, Williams syndrome, Marfan syndrome, rubella syndrome,
Klippel Trenaunay Weber syndrome, Kawasaki disease and Crohn disease.
•• Bilateral renal artery disease is more common than unilateral; ACE inhibitors are C/I in bilateral renal artery stenosis

89. d. Henoch-Schonlein purpura  Ref: Nelson's 20/e p, Ghai 8/e p 632


90. a. Hypokalemia  Ref: Nelson 20/e p 2500-2501, Ghai 8/e p 474-477, 1612
It is a case of acute nephritic syndrome which can result in Hypertensive encephalopathy, Acute renal failure and metabolic acidosis.
91. c. Henoch-Schonlein purpura  Ref: Nelson's 20/e p 1612, Ghai 8/e p 632
Henoch-Schonlein purpura (HSP) is the most common vasculitis of childhood and is characterized by leukocytoclastic vasculitis and
IgA deposition in the small vessels in skin, joints, GIT and kidney.
92. c. IgA  Ref: Nelson's 20/e p 2496, Ghai 8/e p 476
93. a. Administer corticosteroids  Ref: Nelson's 20/e p 2497, Ghai 8/e p 476
Treatment of IgA nephropathy: The primary t/t is proper BP control; Other drugs found useful are:
•• Fish oil (omega-3 fatty acids): An anti-inflammatory agent and it prevents the rate of renal progression
•• Immunosuppressive therapy: Done with corticosteroids with or without cytotoxic therapy
•• ACE inhibitors and Angiotensin II receptor antagonists: role in reducing proteinuria
•• Prophylactic antibiotics and tonsillectomy: may reduce frequency of gross hematuria but no effect on progression of renal ds.
94. b. IgA nephropathy  Ref: Nelson's 20/e p 2496, Ghai 8/e p 476
Chapter 19: Disorders of Kidney and Urinary Tract 447

•• This is a classic case presentation of acute glomerulonephritis (gross hematuria, 80% dysmorphic RBCs)

Answers with Explanations


•• Hence, 3 possibilities are: IgA nephropathy, PSGN, HSP
•• Since it has occurred 2 days after URI, ‘PSGN' option is not the correct option as minimum incubation period for clinical
manifestations in PSGN is 1–2 weeks after an antecedent streptococcal pharyngitis
•• Furthermore, with absence of GIT symptoms, arthritis, skin rash, possibility of H.S. purpura is also low.
95. d.Hemolytic uremic syndrome, e. Malignant hypertension  Ref: Nelson's 20/e p 2508, Ghai 8/e p 492
•• A picture resembling microthrombi in renal vasculature is called “Thrombotic microangiopathy” (TMA).
•• TMA is the histologic result of microangiopathic hemolytic anemia which consumes platelets and RBCs and is characterized
by thrombocytopenia and presence of schistocytes. Renal failure may be seen along with it.
Diseases that present with Thrombotic microangiopathy:
•• Thrombotic thrombocytopenia (TTP) •• Scleroderma renal crisis
•• Hemolytic uremic syndrome (HUS) •• Antiphospholipid syndrome
•• Malignant hypertension •• HIV infection

96. a. RBC cast, c. Proteinuria  Ref: Ghai 8/e p 469

a. RBC cast Produced due to bleeding from within the glomerulus seen in acute glomerulonephritis (GN)

b. Hemoglobinuria Hemoglobinuria without hematuria usually occurs in the presence of hemolysis.

c. Proteinuria cola colored urine, proteinuria > 100 mg/dL, RBC casts and dysmorphic RBCs are all seen in acute GN

d. Broad cast Broad casts are seen in chronic kidney disease.

e. Hyaline cast Seen in Nephrotic syndrome and not nephritic

97. a. Acute GN, c. Renal thrombosis  Ref: Nelson's 20/e p 2500-2501, Ghai 8/e p 474, 477
a. This is a classic case presentation of Acute glomerulonephritis with gross hematuria, hypertension, edema.
b. Nephrotic syndrome: Massive proteinuria, hypoalbuminemia and edema seen but Gross hematuria is usually absent.
c. Renal vein thrombosis:
–– It usually presents with hematuria and flank masses in presence of predisposing factors; Hypertension usually seen
–– Doppler flow studies of the inferior vena cava and renal vein confirm the diagnosis.
d. Renal amyloidosis
–– It presents with proteinuria which ranges from sub-nephrotic to nephrotic range proteinuria
–– Diagnosis is confirmed on renal biopsy, where amyloid takes up apple green color on congo red staining.
e. Renal tubular acidosis: Presents with polyuria, polydipsia and metabolic acidosis.
98. a. Prednisolone + Cyclophosphamide  Ref: Nelson's 20/e p 2500-2501, Ghai 8/e p 474-477
Pauci-immune Crescentic GN

Definition Crescentic GN with few or no immune deposits on immunofluorescence

Causes •• Microscopic polyangitis •• Churg Strauss syndrome


•• Wegener's granulomatosis •• Medications, e.g. Pencilllamine, hydralazine, propylthiouracil
•• Idiopathic Crescentic GN

Treatment Induction therapy with steroids and cyclophosphamide is used initially followed by maintenance with Azathioprine or MMF.

 OBSTRUCTIVE UROPATHY

99. c. Ureteropelvic junction  Ref: Protocols in Pediatric Nephrology by Dr Bagga pg 19


100. a. PUJ obstruction  Ref: Nelson 20/e 2540-2548, Practical Algorithms in Pediatric Nephrology edited by Israel Zelikovic, Israel
Eisenstein p 49
Most common cause of hydronephrosis in children is PUJ obstruction;
However, a neonate with a history of hydronephrosis on prenatal ultrasound and a palpable bladder most likely has posterior urethral
valves.
101. c. Vesicoureteral reflux induced pyelonephritis  Ref: Nelson's 20/e p 2562-2567, Ghai 8/e p 485-486
Most common cause of renal scarring in a child is VUR-induced pyelonephritis.
102. a. Antibiotic prophylaxis  Ref: Nelson's 20/e p 2562-2566, Latest VUR treatment guidelines by AUA & EUA, 2015;
There is a lack of consensus regarding management of VUR in children,
Latest treatment guidelines by the American Urological Association (AUA) and European Urological Association (EUA):
448 Section 3: Systemic Pediatrics
Review of Pediatrics and Neonatology

L at e s t U p d at e s

I. Continuous Antibiotic prophylaxis (CAP):


According to AUA, High-risk groups who would benefit from CAP included those 1 year of age, dilating VUR [grades 3-5] and/or a history
of febrile UTI, those with bladder and bowel dysfunction; Otherwise, CAP is considered optional for those >1 year of age.
According to EAU:
•• Regardless of the grade of reflux or presence of renal scars, all patients diagnosed at age < 1 year can be treated initially with CAP as
during early childhood, the kidneys are at higher risk of developing new scars
•• Definitive surgical or endoscopic correction is preferred for those with frequent breakthrough infections
•• In all children presenting at age 1-5 years, CAP is the preferred, initial option
•• However, those with lower grades of reflux and no symptoms are considered potential candidates for surveillance without CAP.
New findings from the RIVUR trial [Randomized Intervention for Children with Vesico-Ureteral Reflux] show that:
•• CAP is associated with a substantially reduced risk of UTI recurrence among children with VUR but not of renal scarring.
•• Benefit of CAP was most pronounced in those with bladder dysfunction as well as those who initially presented with a febrile UTI.
II. Surgery for VUR:
•• Open neoureterocystotomy or ureteral reimplantation is the “gold standard” for surgical correction of VUR
•• The success rate of open ureteral reimplantation is 95% regardless of VUR grade or presence of bladder bowel dysfunction
•• Patients receiving CAP with a febrile breakthrough-UTI should be considered for surgical correction with curative intent.

103. a. Micturating cystography and b. Radioisotope renography  Ref: Nelson's 20/e p 2562-2567, Ghai 8/e p 485-486
VUR is diagnosed and graded on a radio contrast MCU.
104. a. Posterior urethral valve  Ref: Nelson's 20/e p 2562-2567, Ghai 8/e p 485-486
Voiding cystourethrogram in an infant with posterior urethral valves showing the dilation of the prostatic urethra and the transverse
linear filling defect corresponding to the valves.
105. b. Painful stress incontinence  Ref: Nelson's 20/e p 2562-2567, 2573, Ghai 8/e p 485-486

High Yield Points  


•• Posterior urethral valves are the commonest cause of lower urinary tract obstruction in boys.
•• In PUV, Prostatic urethra dilates, & bladder muscle undergoes hypertrophy; Hydronephrosis or renal dysplasia may also develop;
•• Vesicoureteral reflux occurs in 50% and 30% of patients experience end stage renal disease or chronic renal insufficiency.

106. a. Bilateral hydronephrosis may be present, b. Urea and creatinine levels remain normal, c. Large and wide prostatic
urethra  Ref: Nelson's 20/e p 2562-2567, Ghai 8/e p 485-486
107. b. Pyeloplasty  Ref: Nelson's 20/e p 2562-2567, Ghai 8/e p 485-486
This is a case of pelviureteric junction (PUJ) obstruction as there is normal sized ureter with pelvi-calyceal dilatation.

High Yield Points  


•• Most patients presenting with prenatally diagnosed PUJ obstruction are asymptomatic.
•• Children with PUJ obstruction may present with recurrent flank pain or UTI; USG and DTPA renogram establish the diagnosis.
•• Children with kidney differential function < 40% and AP diameter > 2 cm, should undergo pyeloplasty.

108. c. PUV > b. Urinary tract infection  Ref: Nelson's 20/e p 2562-2567, Ghai 8/e p 485-486
The clinical features in this case include: Fever, Dysuria, Gross hematuria, no proteinuria, a palpable suprapubic area, most probably
urinary bladder. These are typical features of a UTI.
However, in upper tract UTI, ie pyelonephritis, there is usually fever and flank pain associated with gross hematuria, but
bladder is not usually palpable
•• The only condition leading to urinary stasis, palpable bladder (due to stasis, bladder wall hypertrophy) and recurrent UTI is
posterior urethral valves
•• Acute glomerulonephritis is characterized by relatively abrupt onset of hematuria, edema, oliguria, and hypertension.
109. b. Vesicoureteric reflux  Ref: Nelson's 20/e p 2562-2567, Ghai 8/e p 485-486

High Yield Points  


•• 30–40% children and 40–50% of neonates with UTI have VUR; Reflux is the most common cause of pyelonephritis
•• Upto 20-40% children with UTI andVUR develop renal scarring; 5–10% progress to end stage renal disease.
•• Females are twice as likely as males to have VUR; However, in infancy, VUR is more prevalent in boys

110. c. Voiding cystourethrogram  Ref: Nelson's 20/e p 2562-2567, Ghai 8/e p 485-486
Chapter 19: Disorders of Kidney and Urinary Tract 449

Answers with Explanations


•• The most useful process for conclusive diagnosis of VUR is micturating cystourethrogram (MCU)
•• MCU demonstrates the grade of reflux as well as urethral anatomy.
•• Radionuclide imaging is sensitive at detecting reflux but does not demonstrate anatomic details

111. b. Older girls  Ref: Nelson's 20/e p 2562-2567, Ghai 8/e p 485-486
112. b. USG bladder  Ref: Nelson's 20/e p 2562-2567, Ghai 8/e p 485-486.
Underlying PUV may be responsible for poor urinary stream that can be diagnosed on USG abdomen.
113. d. Prophylactic antibiotics  Ref: The Diagnosis and Treatment of Vesicoureteral Reflux The Open Urology and Nephrology
Journal, 2015, Volume 8:97;
114. b. Posterior urethral valves  Ref: Nelson's 20/e p 2562-2567, Ghai 8/e p 485-486
Most common cause of severe obstructive uropathy in children is posterior urethral valves, affecting 1 in 8,000 boys.
115. d. Ureterocele  Ref: Nelson's 20/e p 2562-2567, Ghai 8/e p 485-486
In Ureterocele, IVP shows a round filling defect in the bladder; Refer pretext of this chapter for details;
116. c. Meatal ulceration with scabbing  Ref: Nelson's 20/e p 2562-2567, Ghai 8/e p 485-486
Meatal ulceration with scabbing can cause acute retention of urine.
Dont get confused by option ‘a' – Though posterior urethral valve is the most common cause of obstructive uropathy in children,
it does not cause acute retention.
Posterior urethral valve usually presents with chronic obstructive symptoms like weak urinary stream, distended bladder, uremia or
sepsis caused by infection in the obstructed urinary tract.
Urethral stone may cause obstruction, but the most common presentation is hematuria with or without pain.
117. a. Septran + follow up  Ref: Nelson's 20/e p 2562-2567, Ghai 8/e p 485

 URINARY TRACT INFECTIONS


118. a. USG only  Ref: Nelson 20/e 2560-2561, EUA guidelines 2014
AAP recommends that in a typical first-episode of UTI, initial imaging should be ultrasonography of the kidneys, ureters, and bladder.
VCUG is indicated if the ultrasound study is abnormal, the patient has atypical features, or after a recurrent febrile UTI.
As per EUA (European Association of Urology), Renal and bladder USG is advised in all children with febrile UTI to exclude
dilatation or anomalies of the upper & lower urinary tract, if no improvement is seen within 24 h
119. b. Send urine for culture and sensitivity and start IV antibiotics immediately  Ref: Nelson's 20/e p 2557-2562
The given case scenario indicates an upper UTI (pyelonephritis) as the presence of fever in the presence of pus cells in urine
indicates renal parenchymal involvement
•• In acute febrile infections suggesting pyelonephritis, a 10–14 day course of broad spectrum antibiotics is preferable.
•• Parenteral Ceftriaxone/cefotaxime, or ampicillin with an aminoglycoside is preferable.
120. c. Posterior urethral valves  Ref: Nelson's 20/e p 2562-2567, Ghai 8/e p 485-486
Sickle cell disease, use of analgesics and Diabetes are important causes of Renal papillary necrosis.
121. b. Posterior urethral valve  Ref: Nelson's 20/e p 2562-2567, Ghai 8/e p 485-486
Straining and Dribbling suggest the diagnosis of obstructive uropathy, for which posterior urethral valve is the commonest cause.
122. d. Bacteria in gram stain  Ref: Nelson's 20/e p 2557, Ghai 8/e p 482
•• UTI may be suspected based on symptoms or findings on urinalysis, or both.
•• A urine culture is necessary for confirmation and appropriate therapy.
Hence, although WBC casts in urine indicate renal parenchymal involvement and both leukocyte esterase and Nitrite test indicate
UTI, identification of bacteria on gram staining while doing urine culture is the most specific urinary finding.
123. a. Suprapubic aspiration  Ref: Nelson's 20/e p 2557, Ghai 8/e p 482
O.P. Ghai: “Urine is obtained by suprapubic bladder aspiration or urethral catheterization in children below 2 years.” Pediatric
Nephrology protocols, AIIMS says: “In neonates and infants, the best technique for obtaining an uncontaminated urine specimen
is supra pubic aspiration from the bladder.”
124. b. No therapy  Ref: Nelson 20/e p 2558; Revised Statement on Management of UTI. Indian Pediatrics, Vol 48, 2011
Criteria for diagnosis of UTI on midstream urine sample: Colony count > 105/ml
Asymptomatic Bacteriuria:
•• It is a form of chronic pyelonephritis characterized by destruction of renal parenchyma and the presence of granulomas, abscesses and
collection of lipid laden foamy macrophages (foam cells)
•• Rare in children; more common in adults; Focal form is more common in children;
•• Those who are affected diffusely, present with non-specific symptoms like Weight loss, Fever, Lethargy, Failure to thrive
•• Proteus is the most common causative organism.
450 Section 3: Systemic Pediatrics

125. b. If two episodes of UTI in female of 7 years occur, then cystometric evaluation needed, c. If two episodes of UTI in male
Review of Pediatrics and Neonatology

of 5 years occur, then cystometric evaluation needed, d. Urine should be obtained by suprapubic aspiration for culture in
infants;  Ref: Nelson's 20/e p 2557, Ghai 8/e p 482

High Yield Points  


•• During 1st year of life, the male : female ratio is around 4 : 1; Beyond 1–2 year, it is 1 : 10
•• In females, 75 to 90% of all infections are caused by E. coli followed by Klebsiella and Proteus

Indication of voiding cystourethrogram (VCUG):


•• All children younger than 5 years with a UTI •• School aged girls who have had two or more UTIs
•• Any child with a febrile UTI •• Any male with UTI.

Most common finding in UTI is vesicoureteral reflux, which is identified in 40% of patients.
126. a. Presence of more than 5 WBC/hpf for girls and more than 3 WBC/hpf for boys  Ref: Nelson's 20/e p 2557

High Yield Points  


•• Pyuria suggests infection, but infection can occur in the absence of pyuria; Conversely, pyuria can be present without UTI;
•• Sterile pyuria (positive leukocytes, negative cultures) occurs in partially treated bacterial UTI, viral infections, renal tuberculosis,
renal abscess, interstitial nephritis, inflammation near ureter or bladder

In an uncentrifuged urine specimen > 5 WBCs/hpf is abnormal and is called pyuria.


127. a. Often affects those younger than 8 years of age  Ref: Nelson's 20/e p 2558, Ghai 8/e p 482
Xanthogranulomatous pyelonephritis
•• It is a form of chronic pyelonephritis characterized by destruction of renal parenchyma and the presence of granulomas, abscesses and
collection of lipid laden foamy macrophages (foam cells)
•• Rare in children; more common in adults; Focal form is more common in children;
•• Those who are affected diffusely, present with non-specific symptoms like Weight loss, Fever, Lethargy, Failure to thrive
•• Proteus is the most common causative organism.

128. d. Ureterocele  Ref: Nelson's 20/e p 2558, Ghai 8/e p 482

 ACUTE KIDNEY INJURY


129. a. Urine Specific gravity is higher in pre-renal failure  Ref: Nelson 20/e 2540-2541
130. b. E. coli  Ref: Nelson's 20/e p 2540-2543, Ghai 8/e p 487-491
The given condition is most likely HUS for which the most common organism is E. coli, among the given options
131. a. Less than 0.3 mL/kg/hr  Ref: Nelson's 20/e p 2540-2543, Ghai 8/e p 487-491
Acute kidney injury (AKI) in children is defined as eCCl (estimated creatinine clearance) decrease by 75% or eCCl <35 mL/
min/1.73 m2 or Urine output <0.3 mL/kg/hr for 24 hr or anuric for 12 hr
132. a. Congenital adrenal hyperplasia  Ref: Nelson's 20/e p 2540-2543, Ghai 8/e p 487-491
Dehydration, salt wasting, hyponatremia and hyperkalemia suggest Congenital adrenal hyperplasia
133. a. Urinary sodium > 40 mEq/L  Ref: Nelson's 20/e p 2540-2543, Ghai 8/e p 487-491
The child in question has pre-renal azotemia, urinary sodium should be less than 20 meq/L.

Classic laboratory findings in AKI


Type UOsm UNa FeNa BUN/
Cr
Prerenal >500 <10 <1% >20

Intrinsic <350 >20 >2% <15

Postrenal <350 >40 >4% >15

134. a. Renal vein thrombosis  Ref: Nelson's 20/e p2511, Ghai 8/e p 487-491
Renal vein thrombosis (RVT) occurs in 2 distinct clinical settings:
•• In newborns and infants: with asphyxia, dehydration, shock, sepsis, hypercoagulable states, and maternal diabetes; and
•• In older children, RVT is seen in patients with nephrotic syndrome, cyanotic heart disease, inherited hypercoagulable states,
sepsis, following kidney transplantation, and following exposure to angiographic contrast agents
Chapter 19: Disorders of Kidney and Urinary Tract 451

Diagnosis:

Answers with Explanations


•• Suggested by hematuria and flank masses in patients seen in the high-risk situations
•• USG shows marked renal enlargement; Radionuclide studies reveal little or no renal function in the affected kidney(s).
•• Doppler flow studies of the inferior vena cava and renal vein confirm the diagnosis.
135. d. Malignant hypertension, e. Hemolytic uremic syndrome  Ref: Nelson's 20/e p 2540-2543, Ghai 8/e p 487-491
a. Minimal change disease usually does not cause renal failure
b. Renal amyloidosis usually presents with nephrotic syndrome or CKD, but is exceedingly rare in children.
c. Pre-eclampsia refers to high BP with proteinuria in pregnant females, so this option is not applicable to children.
d. Malignant hypertension or as Hypertensive emergency is hypertension with acute impairment 1 or of more organ systems esp
CNS, CVS or renal system. Most common causes in children are renal and renovascular.
e. HUS is an important “renal” cause of AKI.

 CHRONIC KIDNEY DISEASE

136. d. Severity of renal failure  Ref: Nelson's 20/e p 2544-2546, Ghai 8/e p 493-496
Schwartz formula (for creatine clearance in child) is Creatinine clearance = K × height/creatinine where K = constant that
depends upon age, muscle mass, method of creatinine estimation.
137. c. DTPA renogram
In order to assess function of an apparent non-functioning kidney, the best modality would be DTPA renogram. DTPA
renogram is a relatively non invasive radionuclide study which enables evaluation of function of individual kidney.
99Tc-DMSA scan is used for providing high quality images of renal morphology (especially useful in follow up of UTIs).
About other options:
a. Ultrasonography is the initial modality for imaging of the kidney and urinary tract. The procedure is operator dependent and
good for anatomic details and not functional details of kidneys ureter and bladder.
b. IVP (Intravenous pyelography) gives valuable information about renal anatomy and function. But due to the risk of anaphylaxis
with the contrast material use of IVP has declined following the availability of radionuclide imaging.
d. 24 hrs creatinine clearance can be calculated by collecting a timed urinary specimen and taking a blood sample around the
mid-collection period. In children, in addition to problem with urine collection, this overestimates the actual GFR and can't be
used to assess isolated kidney function.

 RENAL TUBULAR DISORDERS


138. c. Bartter syndrome  Ref: Nelson's 20/e p 358-359

Syndromes Bartter syndrome Gitelman syndrome


Characterized by Hypokalemic metabolic alkalosis with hyper calciuria & Hypokalemic metabolic alkalosis, with hypocalciuriaQ
salt wasting & hyponatremia and hypomagnesemia.Q
Symptoms Polyuria, polydipsia, vomiting, consti­pation, failure to Recurrent muscle cramps and spasms
thrive. BP: Low/normal
Molecular basis Mutations in genes that encode Na+/K+/2Cl− transporter Mutations in sodium chloride cotransporter NCCT,
(NKCC2), luminal K channel (ROMK), CL channel present in the distal convoluted tubule.
(CLC-Ka, CLC-Kb).
Treatment K supplementationQ & Indomethacin K & Mg Supplementation

139. d. Gordon syndrome  Ref: Nelson 20/e p 2535


Out of the given causes, hyperkalemia is seen only in Gordon syndrome
Gordon syndrome or familial hyperkalemic hypertension, is due to gain-of-function mutations in WNK1 & loss-of-function
mutations in WNK4 in distal convoluted tubule, leading to excessive NCCT-mediated salt reabsorption with the clinical picture
of pseudohypoaldosteronism type 2
Liddle syndrome is due to gain-of-function mutations of the gene that encodes the epithelial sodium channel in the collecting
duct, leading to hypertension, hypokalemia & suppressed aldosterone
140. c. Hypocalciuria  Ref: Nelson's 20/e p 358-359, 2534, Ghai 8/e p 501
In Bartter syndrome, Metabolic alkalosis, hypokalemia, hyponatremia and increased urinary chloride loss are seen.
141. c. Bartter syndrome  Ref: Nelson's 20/e p 358-359, 2534 Ghai 8/e p 501
•• The infant in question presents with failure to thrive, polyuria, bilateral nephrocalcinosis.
•• Lab features are Metabolic alkalosis (S. Bicarb – 25 meq/L, pH – 7.48) and Hypercalciuria, Hypokalemia (S.K+ – 2 meq/L),
452 Section 3: Systemic Pediatrics

Hyponatremia (S. Na+ 126 meq/L), Hypochloremia (S.CL– 88 meq/L), This is clearly the picture of ‘Bartter syndrome'.
Review of Pediatrics and Neonatology

About other options:


a. Pseudo hypoaldosteronism – Feature of aldosterone deficiency like decreased Na, increased K, metabolic acidosis are usually
present.
b. In Gitelman syndrome, hyprocalciuria and hypomagnesemia are seen
d. Distal renal tubular acidosis – can be easily ruled out as it causes acidosis not alkalosis.
142. c. Similar presentation in father  Ref: Nelson's 20/e p 2535; Refer pretext for details of Dent Disease
As Dent disease is an X-linked disorder, it cannot be transmitted from males to males i.e. from father to son
143. d. 600–700 mOsm/litre  Ref: Nelson's 20/e p 2490-2492, Ghai 8/e p 465-467
A full term infant can concentrate his urine to a maximum of 700–800 mOsm/kg, as compared to older child
(1200–1400 mOsm/kg); A newborn can dilute his urine to a minimum of 50 mOsm/kg like an older child
144. b. Cystinosis  Ref: Nelson 20/e p 2529-2530; Refer pretext of this chapter for details;
Features of renal tubular dysfunction like polyuria, polydipsia and metabolic acidosis, along with presence of crystal deposits in
cornea on slit lamp examination of eye suggest a diagnosis of Cystinosis
145. d. ARPKD  Ref: Nelson's 20/e p 2534-2537;
Causes of Nephrocalcinosis

Common causes of Nephrocalcinosis Less common causes


•• Idiopathic hypercalciuria Primary hyperparathyroidism Prolonged furosemide treatment
•• Distal renal tubular acidosis Prematurity Dent's disease
•• Hyperoxaluria Bartter syndrome Idiopathic hypercalcemia Cushing
Hypomagnesemia with hypercalciuria syndrome

146. a. One year of age  Ref: Nelson's 20/e p 2490, Ghai 8/e p 465-467 The healthy newborn infant, subject to water deprivation
for 10–14 hours can achieve an osmolality of only 500–700 mOsm/kg. This value increases gradually during the first few
months and approximates adult value by about one year of age.
147. c. Bartter syndrome  Ref: Nelson's 20/e p 258, Ghai 8/e p 501
Here, an infant presenting with polyuria, polydipsia, has metabolic alkalosis with hyponatremia, hypokalemia & hypochloremia
along with medullary nephrocalcinosis on USG. This is a typical case of Bartter's syndrome.
148. c. Bartter syndrome  Ref: Nelson's 20/e p 258, 2534, Ghai 8/e p 501
149. c. Reduced calcium intake  Ref: Nelson's 20/e p 2522, Ghai 8/e p 502; Refer pretext of this chapter for details;
In Idiopathic hypercalciuria, dietary calcium is not restricted as low calcium diet are accompanied by higher urinary oxalate
excretion, since intestinal oxalate is no longer bound to calcium.
150. a. Diabetes insipidus  Ref: Nelson's 20/e p 348-349, Ghai 8/e p 501-519
The given child in question has presented with clinical features: Polyuria, polydipsia. Lab data: Hypernatremia (S. Na+ – 154 meq/L),
Raised S. Osmolality (S. Osmolality – 295), Raised B. Urea (B. urea – 50), Normal K+, HCO – (4.5, 22 respectively). The given
clinical and laboratory features are compatible with a diagnosis of diabetes insipidus
151. c. ESWL  Ref: Nelson's 20/e p 2511-2512, Ghai 8/e p 502; Refer Ans. 138 below;
A calculus of size upto 2 cm in the upper ureter, the treatment of choice is ESWL (extra corporeal shock wave lithotripsy).
152. a. Pyelolithotomy  Ref: Nelson's 20/e p 2511, Ghai 8/e p 502
Primary surgical treatment options for renal stones based on stone size and location.

Stones Shock wave lithotripsy Ureteroscopy Percutaneous nephrolithotomy


Renal    <1 cm Most common Optional Optional
     1–2 cm Most common Optional Optional
     >2 cm Optional Rare Most common
Ureteral   Proximal Most common Optional Occasional
      Distal Optional Most common Rare

153. b. FeNa is higher in intrinsic renal failure than prerenal failure  Ref: Nelson 20/e p 2541
Fractional excretion of sodium (FeNa)
•• FeNa is the urine:plasma (U:P) ratio of sodium divided by U:P of creatinine x 100.
•• Neonates have higher FeNa because of tubular immaturity,
Chapter 19: Disorders of Kidney and Urinary Tract 453

•• FeNa is higher in preterm neonates due to immaturity of sodium reabsorptive capacity.

Answers with Explanations


•• FeNa may be affected by diuretic use. But fractional excretion of urea (FEurea) is dependent on passive forces and is less
influenced by diuretic therapy.

Classic laboratory findings in AKI


Type UOsm UNa FeNa BUN/
Cr
Prerenal >500 <10 <1% >20

Intrinsic <350 >20 >2% <15

Postrenal <350 >40 >4% >15

154. c. 18  Ref: Hari P, Biswas B, Pandey R, Kalaivani M, Kumar R, Bagga A. Clin Exp Nephrol 2012;16:697-705.
Schwartz formula for creatinine clearance calculation = k x length/serum creatinine
For Indian children, k = 0.42
eGFR = 0.42 x 85/2 = 17.8
155. c. Height & serum creatinine  Ref: Nelson 20/e 2492
Schwartz formula is the most widely used pediatric formula for estimating GFR, and is based on serum creatinine, patient height, and
an empirical constant. According to Schwartz formula,
GFR= k x height (cm)/serum Creatinine (mg/dl), where K is 0.33 in preterm infants, 0.45 in term infants & 0.55 for children
156. c. 500  Ref: Nelson 20/e 349; Pediatric Urology By John G. Gearhart pg 15-20
Premature infants have an even more limited capacity to concentrate their urine than those born at term, with a maximum urine
osmolality of about 500 mOsm/kg H2O.
Chapter 20
Pediatric
Endo­crinology
High Yield Points  A. DISORDERS OF PITUITARY GLAND
  M
•• Most commonly diagnosed Hypopituitarism Hyperpituitarism
adenoma during childhood is
prolactinomaQ > corticotropinoma MPHD: Multiple pituitary hormone deficiency Primary hyperpituitarism (Adenoma): rare in
>somatotropinoma IGHD: Isolated growth hormone deficiency children
•• Most common tumor Diabetes insipidus Secondary hyperpituitarism: occurs in response
causing hypopituitarism is to target hormone deficiencies like hypogonadism,
craniopharyngioma hypoadrenalism, or hypothyroidism
•• Trophic hormone failure with pitui­
tary compression or destruc­tion:
GHQ > FSH > LH > TSH > ACTH Genetic Basis of Hypopituitarism
•• Most common genetic cause for MPHD—Mutations of PROP1
•• The only transcription factor mutation causing MPHD which is X-linked is SOX 3 mutation
•• Other genes which may be involved are POU1F1, HESX1, LHX3, LHX4, and PTX2.

Causes of Acquired Hypopituitarism

Brain Damage Pituitary tumors Infarction


•• Traumatic brain injury •• Adenomas •• Apoplexy
•• Subarachoid hemorrhage •• Others •• Sheehan syndrome
•• Neurosurgery Non-pituitary tumors Autoimmune disorder
•• Irradiation •• Craniopharyngiomas Lymphocytic hypophysitis
•• Stroke •• Meningiomas Others
Infection •• Gliomas •• Hemochromatosis
•• Abscess •• Chordomas •• Granulomatous diseases
•• Hypophysitis •• Ependymomas Histiocytosis
•• Meningitis •• Metastases •• Empty sella
•• Encephalitis •• Perinatal insults

High Yield Points DIABETES INSIPIDUS (DI)


•• Primary brain tumors associated with Characterized by pathologic polyuria or polydipsia (exceeding 2 L/m2/24 hr).
DI are Germinomas & pinealomas
Types Central DI Nephrogenic DI
Congenital causes AD inheritance: Mutations in •• X-linked: inactivating mutations of the
vasopressin gene (AVP-NPII gene) vasopressin V2 receptor
•• AR and AD: defects in aquaporin
2 gene
Acquired causes Trauma, neoplasms, autoimmune, •• Long standing hypercalcemia or
infiltrative, infectious diseases & drugs hypokalemia, drugs and kidney
diseases
Mnemonic •• Acquired are more common in children
For features of Wolfram syndrome Treatment of choice DesmopressinQ intra nasal spray Thiazide diureticsQ
“DIDMOAD” Other treatment Central DI of acute onset following Low sodium diet, amiloride, indomethacin
Diabetes insipidus (central), diabetes modalities neurosurgery is best managed with
mellitus, optic atrophy and neural continuous administration of synthetic
deafness, WF 1 gene, AR aqueous vasopressin
Chapter 20: Pediatric Endo­crinology 455

Differences between SIADH, Cerebral Salt Wasting, and Central DI

Section 3: Systemic Pediatrics


M
High Yield Points   M
Clinical Parameters SIADH Cerebral Salt Wasting Central DI
•• Investigation to differentiate DI from
Serum sodium Low Low High other causes of polyuria and confirm
Urine output Normal or low High High diagnosis—Water deprivation test.
Urine sodium High Very high Low •• Diagnosis of DI is established if the
serum osmolality is >300 mOsm/
Intravascular volume state Normal or high Low Low
kg and the urine osmolality is 300
Vasopressin level High Low Low mOsm/kg.

 SHORT STATURE

GROWTH HORMONE (GH) DEFICIENCY


Etiology
•• Can be caused by abnormalities of GH genes or GH-releasing hormone receptor
•• Acquired GH deficiency is due to radiotherapy meningitis, histiocytosis & trauma.

Clinical Features of Growth Hormone Deficiency


Craniofacies Growth & development Other features
•• Frontal bossing •• Birth weight & length: normal •• Hypoglycemia
•• Normal head size •• Postnatal severe growth failure •• Motor milestones: delayed
•• Delayed dentition •• Bone age: delayed •• Skin: thin, prematurely aged
•• Voice: high pitched •• Micropenis; Puberty: delayed •• Osteopenia

Diagnosis
•• Measurements of GH, IGF-1, & IGF-1–binding protein levels
•• Determination of peak GH levels after stimulation test with insulin, arginine, clonidine, or
glucagon.
Treatment
•• Recombinant hGH given by subcutaneous injection
•• Important adverse effects of GH therapy include pseudotumor cerebri, slipped capital
femoral epiphysis, gynecomastia & worsening of scoliosis.

High Yield Points High Yield Points   M


The cut off level to diagnose GH deficiency in children after GH stimulation test is < 10 ng/mL Other conditions for which GH is used
Side Effects of GH Therapy: for treating short stature are:
•• Pseudotumor cerebri •• Chronic renal failure
•• Slipped capital femoral epiphysis •• Turner syndrome
•• Gynecomastia •• Prader Willi syndrome
•• Worsening of scoliosis •• Babies born SGA
•• Impaired glucose tolerance. •• Idiopathic short stature

 B. THYROID DISORDERS IN CHILDREN


CONGENITAL HYPOTHYROIDISM
•• Congenital Hypothyroidism is seen in 1 in 3000 infants worldwide
•• It is twice more common in girls.

Causes of Congenital Hypothyroidism M

Primary hypothyroidism Central (hypopituitary) hypothyroidism


•• Thyroid dysgenesis •• TSH deficiency: mutation in TSH β-subunit
•• Thyroid dyshormonogenesis gene
•• Iodine deficiency (endemic goiter) •• TRH deficiency: mutation in TRH gene
•• TSH unresponsiveness •• TRH unresponsiveness: mutation in TRH
•• Defect in thyroid hormone transport: MCT8 gene receptor gene
•• Resistance to thyroid hormone •• Multiple congenital pituitary hormone
•• Maternal Thyrotropin receptor–blocking Ab deficiencies.
•• Maternal medications. e.g. Iodides, amiodarone,
Propylthiouracil, methimazole, Radioiodine
456 Section 3: Systemic Pediatrics

Gene mutations involved in congenital hypothyroidism


Review of Pediatrics and Neonatology

Question 1 M
Defective Gene Abnormalities in addition to congenital hypothyroidism
A 3-month-old baby presented
NKX2.1 (TTF-1) Respiratory distress syndrome, ataxia, choreoathetosis
with jaundice and a history
of constipation. His picture is NKX2.5 Congenital heart defects
provided below. What is the TTF-2 Cleft palate, spiky hair
probable diagnosis?
PAX-8 Genitourinary anomalies, including renal agenesis.

Clinical Features M

•• Thyroid dysgenesis presents at birth; ectopic thyroid usually presents late


•• 10–15% of cases due to inborn errors of thyroid hormone synthesis and often present after infancy
•• One of the earliest signs of congenital hypothyroidism is patent posterior fontanele and
wide open cranial sutures.
a. Growth hormone deficiency •• Umbilical hernia, constipation, hypothermia, feeding difficulty, breathing difficulty, pro-
b. Congenital hypothyroidism
longed physiological jaundice and hypotonia.
c. Congenital adrenal hyperplasia
d. Congenital adrenal insufficiency •• Muscles are usually flaccid and hypotonic except in one variant, Kocher-Debre-Semelaigne
syndrome, where muscles (esp calf muscles) are hypertrophied; returns to normal with
treatments
•• Dentition is delayed and DTR slow relaxation; refractory anemia is common.

Diagnosis M

•• Thyroid profile: Low T3, T4, Raised TSH (TSH may be low in hypopitutary hypothyroidism)
•• ECG shows: low-voltage P and T waves with diminished amplitude of QRS complexes and
suggest poor left ventricular function and pericardial effusion
•• Echocardiography (ECG) can show pericardial effusion
•• Electroencephalogram (EEG) often shows low voltage
•• In children >2 year of age, the serum cholesterol level is usually elevated.

High Yield Points High Yield Points   M


•• Most common cause of congenital X-ray Changes in Hypothyroidism
hypothyroidism: thyroid dysgenesis •• Absence of femoral epiphyses at birth
(80-85%) > dyshormonogenesis (15%) •• Punctuate epiphyseal dysgenesis
•• T3 and T4 synthesis begins at 12 •• Beaking of 12th thoracic, 1st and 2nd lumbar vertebra.
weeks of gestationQ •• Skull X-rays shows large fontanel, wide sutures and wormian bones
•• Transcription factors needed for
thyroid hormone synthesis: TTF-1/
NKX2.1, TTF-2 [FOXE1] and PAX8
Treatment
•• Diagnosis done ideally on day 2-day Oral Thyroxine is used in a dose of:
4 of life, can be done using dried 10-15 µg/kg/day in newborns and 4–8 µg/kg/day in older children
blood spot
Neonatal Thyroid Screening: Different Strategies used:
1. Primary TSH, backup T4: T4 is measured only if TSH > 20 mU/L
Limitations: can miss central Hypothyroidism.
2. Primarily T4 and backup TSH: TSH is measured only if T4 is low
will miss subclinical Hypothyroidism.
3. Concomitant T4 and TSH: Most sensitiveQ

CRETINISM
•• Endemic cretinism is the most serious consequence of iodine deficiency.
•• Cretinism is a condition of severely stunted physical and mental growth due to untreated
High Yield Points   M congenital hypothyroidism or from prolonged nutritional deficiency of iodine
•• Cretinism is due to nutritional deficiency of iodine is endemic cretinism
•• Iodine deficiency remains the
•• When cretinism is due to congenital hypothyroidism it is called sporadic cretinism.
most common cause of acquired
hypothyroidism worldwide.
Endemic cretinism includes 2 different overlapping syndromes: Neurologic & Myxedematous.
•• In areas of iodine sufficiency, Neurological Myxedematous
autoimmune disease (Hashimoto’s
Thyroid gland size Enlarged (goiter) Normal in size
thyroiditis) and iatrogenic causes
(treatment of hyperthyroidism) are Hormonal status Euthyroid Hypothyroid
most common. Mental Retardation Present, severe Present but less severe

Contd…
Chapter 20: Pediatric Endo­crinology 457

Contd…

Section 3: Systemic Pediatrics


Neurological Myxedematous
Deaf Mutism & squint Usually present Absent
Cerebral Diplegia Often present Absent
Stature Usually normal Severe growth retardation
General features No physical sign of Physical signs present; e.g. Coarse dry skin,
hypothyroidism husky voice
Reflexes Brisk Delayed relaxation
ECG & X-ray Normal Low voltage QRS & Epiphyseal dysgenesis
Effects of treatment No effect Improvement

ACQUIRED HYPOTHYROIDISM
•• Autoimmune thyroid disease may be part of polyglandular syndromes.
•• Risk factors for autoimmune thyroid disease
–– Children with Down, Turner, and Klinefelter syndromes and celiac disease or diabetes
are at higher risk for associated autoimmune thyroid disease High Yield Points   M
–– HLA-DR polymorphisms: especially HLA-DR3, -DR4, and -DR5 in Caucasians.
Most common type of CAH is 21
hydroxylase deficiency (90%)> 11 beta
 C. DISORDERS OF ADRENAL GLAND hydroxylase deficiency (5%)
Pathways of Adrenal Steroidogenesis M

1. CONGENITAL ADRENAL HYPERPLASIA (CAH)


•• Autosomal recessive disorders of cortisol biosynthesis
•• Cortisol deficiency increases secretion of corticotropin (ACTH) adrenocortical hyperplasia
and overproduction of intermediate metabolites.

A. CAH due to 21–Hydroxylase Deficiency M

Pathophysiology   M
•• In the absence of 21 hydroxylase,
progesterone and metabolites are
diverted to production of androgens
•• Deficiency of mineralocorticoid →
salt wasting, hypotension, hypo­
natremia, hyperkalemia & aci­dosis
•• Excess androgens → virilisation
of females & precocious puberty in
males
458 Section 3: Systemic Pediatrics

Types of 21 Hydroxylase Deficiency


Review of Pediatrics and Neonatology

Phenotype Classic salt wasting Simple virilizing Non classic


%21 Hydroxylase activity 0 1–5 20–50
Incidence 1 in 20,000 1 in 60,000 1 in 1000 (most common)
Presentation Ambiguous genitalia in females & precocious Normal to virilized
puberty in males
FTT, recurrent episodes No salt wasting Acne, Hirsutism
of adrenal crisis PCOS like syndrome; Infertility

Diagnosis
Question 2 M
•• Electrolyte abnormalities: Hyper­kalemia, Hyponatremia & Acidosis
A 6-week infant with ambi­guous •• Elevated 17 hydroxy proges­terone is diagnostic
genitalia (as shown below) presents •• Genetic tests can also be done (gene on chr 6).
with an episode of dehydration &
shock requiring hospitalization. Treatment
What is the electrolyte abnormality
that you expect in this baby? i. Glucocorticoid Replacement
•• Hydrocortisone is used, 15–20 mg/m2/24 hr in 3 divided doses, continued lifelong
•• Used to treat Cortisol deficiency & suppress excessive production of androgens by the
adrenal cortex & thus minimizes excessive growth, skeletal maturation and virilization
•• Double or triple doses are indicated during periods of stress, such as infection or surgery.
ii. Mineralocorticoid Replacement
•• Fludrocortisone used for patients with salt-wasting disease
•• Daily dose required is higher in Infants (0.1–0.3 mg) than older children (upto 0.1 mg)
•• Infants often require additional sodium supplementation (NaCl 8 mmol/kg).
a. Hypokalemia b. Hyperkalemia
c. Hypocalcemia d. Hypercalcemia iii. Surgical Management of Ambiguous Genitals
•• Significantly virilized females usually undergo surgery between 2-6 months of age
•• Risks and benefits of surgery should be fully discussed with parents of affected females
•• Sex assignment of infants with CAH is usually based on expected sexual functioning & fertility in
adulthood with early surgical correction of the external genitals to confirm with the sex assignment.
iv. Prenatal Treatment
•• Genetic counselling & Prenatal diagnosis using Chorionic villus sampling.
•• Mothers with pregnancies at risk (previous affected child) are given.
Dexamethasone 20 µg/kg as soon as pregnancy is diagnosed

This suppresses secretion of steroids by the fetal adrenal, including secretion of adrenal
androgens and ameliorates virilization of external genitals in affected females

Chorionic villus biopsy is then performed to determine the sex and genotype of the fetus
therapy is continued only if the fetus is an affected female
B. CAH due to 11-β Hydroxylase Deficiency

•• Excess 11- DOC (Deoxycorticosterone) → mineralocorticoid → Hypertension


•• Excess androgens → Virilization of female, precocious puberty in males
•• Elevated 11 deoxycortisol, deoxycorticosterone.
Chapter 20: Pediatric Endo­crinology 459

C. CAH due to 3-β Hydroxyl Steroid Dehydrogenase Deficiency (3β HSD)

Section 3: Systemic Pediatrics


•• Decreased aldosterone → salt wasting.
•• Decreased cortisol → Shock, Hypoglycemia
•• Increased androgens (but DHEA is a weak androgen)
–– In males: Incomplete virilisation (hypospadias, bifid scrotum)
–– In females: hirsutism acne, irregular menses.

D. CAH Due to 17-hydroxylase Deficiency Question 3 M


•• Increased aldosterone: Hypertension, hypokalemia A 2-year-old child presents with
•• No symptoms of glucocorticoid deficiency as Corticosterone is active glucocorticoid obesity, short stature & hyper­
•• Absent sex steroids leading to male pseudohermaphroditism & failure of sexual development tension. His photograph is shown
at puberty in females below. What is the most probable
diagnosis?
Enzymes Deficient in CAH M

Female virilization Male incomplete virilization


With hypertension 11 b hydroxylaseQ 17 a hydroxylaseQ
With salt crisis 21 hydroxylaseQ 3 b hydroxysteroid dehydrogenase

2. CUSHING’S SYNDROME M
a. Addison’s disease
Definition b. Congenital hypothyroidism
c. Cushing's syndrome
Group of clinical features that result from chronic exposure to excess gluco­corticoids of any etiology.
d. Congenital adrenal hyperplasia
Etiology
•• Prolonged exogenous administration of glucocorticoid
•• Bilateral adrenal hyperplasia/Adenoma
•• Hypersecretion of corticotropin (Cushing disease)
•• Ectopic secretion of corticotropin/Exogenous corticotropin
•• Pigmented nodular adrenocortical disease (Carney complex) High Yield Points
•• McCune-Albright syndrome.
•• Most common clinical symptoms of
High Yield Points cushing syndrome is obesity
•• Normally, cortisol levels are maxi­
•• Most common cause of Cushing syndrome is prolonged exogenous administration of glucocorticoids
mum at 8 AM & lowest at midnight
•• Endogenous Cushing syndrome is most often by a functioning adreno cortical tumor in children < 7 year
•• Nelson syndrome is a complication
•• Most common cause of endogenous cushing syndrome in children > 7 years is ACTH producing
following bilateral adrenalectomy
pituitary adenoma (Cushing’s disease)
& is characterized by enlargement
•• Cushing’s disease is most commonly caused by pituitary microadenomas
of sella and hyperpigmentation.

Clinical Features
In infants
•• Rounded face with prominent flushed cheeks (moon facies) with generalized obesity
High Yield Points
•• In children with adrenal tumors, signs of abnormal masculinization (hirsutism, pubic hair, •• Loss of diurnal variation is the
acne, deepening of voice & enlargement of clitoris in girls) earliest biochemical marker of
•• Growth is impaired, with length falling below the 3rd percentile hypercortisolism
•• Hypertension is common and may occasionally lead to heart failure •• Confirmatory Test for Cushing's
Disease: Bilateral inferior petrosal
•• An increased susceptibility to infection may also lead to sepsis.
sinus sampling, baseline and post
In Older Children ACTH—both will be high in pituitary
tumor secreting excess ACTH
•• Obesity and short stature are common presenting features •• Treatment of choice for cushing
•• Purplish striae on the hips, abdomen, and thighs are common disease: Transphenoidal pituitary
•• Pubertal development may be delayed, or amenorrhea may occur in girls past menarche Microsurgery
•• Weakness, headache, hypertension, hyperglycemia & osteoporosis seen.

Investigations for Cushing's Syndrome


Screening tests for Cushing's syndrome
•• 24 hours urine free cortisol levels elevated (Salivary cortisol is a better alternative)
•• Overnight dexamethasone suppression test—single dose 1 mg dexamethasone at 11 PM
suppresses serum cortisol in normal persons but not in Cushing syndrome.
460 Section 3: Systemic Pediatrics

Confirmatory Test for Cushing's Syndrome


Review of Pediatrics and Neonatology

Low dose dexamethasone suppression test—0.5 mg 6 hourly dexamethasone for 2 days does
not suppress cortisol in Cushing syndrome.
Tests for differentiating Cushing's disease from other causes of Cushing's Syndrome
•• MRI can localize the pituitary tumor in some patients
•• ACTH levels are elevated in Cushing's disease and is normal in adrenal tumors
•• High-dose dexamethasone suppression test—(2 mg of dexa 6 hourly for 2 days) suppresses
cortisol only in pituitary microadenomas not in ectopic ACTH or adrenal tumors
•• CRH test—CRH infusion increases ACTH and cortisol in Cushing's diseases, not in ectopic
ACTH or adrenal tumors.

Treatment
High Yield Points •• Pituitary irradiation or bilateral adrenalectomy is recommended in children with failed
pituitary surgery or recurrent disease
Autoimmune Polyglandular •• Adrenal tumors causing Cushing syndrome are treated by surgical resection.
Syndrome 1 (APS-1)
•• Also called APECED (autoimmune 3. ADRENAL INSUFFICIENCY
polyendocrinopathy candidiasis-
ectodermal dystrophy), Primary Adrenal Insufficiency Secondary Adrenal Insufficiency
•• Mutation in AIRE gene; AR mode of •• Congenital or acquired lesions of adrenal •• Dysfunction of hypothalamus or
inheritance
•• Chronic mucocutaneous candidiasis cortex prevent production of cortisol ± anterior pituitary gland
is the 1st manifestation of APS 1, aldosterone •• Deficiency of corticotropin (ACTH)
followed by hypoparathyroidism •• Acquired primary adrenal insufficiency is •• Hypofunction of adrenal cortex
•• Gonadal failure, type I DM, termed Addison disease
hypothyroidism can also develop
Causes of Primary Adrenal Insufficiency
(i)  Congenital Adrenal Hyperplasia (Discussed Above)
(ii)  Other Genetic Disorders
Mnemonic •• Adrenoleukodystrophy (ABCD1 mutations): Weakness, spasticity, dementia, blindness,
APSI, show HAM: quadriparesis
•• Hypoparathyroidism •• Triple A or Allgrove syndrome (AAAS mutations): Achalasia, alacrima, cognitive
•• Addison disease & deficits, hyperkeratosis
•• Mucocutaneous candidiasis
•• Smith-Lemli-Opitz syndrome (DHCR7 mutations): Craniofacial malformations,
developmental delay, growth failure, cholesterol deficiency
•• Wolman disease (LIPA mutations): Bilateral adrenal calcification, hepatosplenomegaly.
(iii)  Autoimmune: Autoimmune Polyglandular Syndrome (APS) types 1, 2, 4.
High Yield Points
(iv)  Infectious
APS-2 (Schmidt Syndrome) •• Tuberculosis, AIDS, Meningococcemia
•• Polygenic inheritance associated •• Fungal: Histoplasmosis, cryptococcosis, coccidioidomycosis.
with HLADR3, (v)  Other acquired causes: Bilateral adrenal hemorrhage/metastases/infiltration/adrenalec­tomy.
•• Associated with thyroid autoimmune
disease & viti­ligo
Clinical Features
•• Due to loss of glucocorticoids: fatigue, myalgia, hypoglycemia, hyponatremia, hypotension
•• Due to mineralocorticoid deficiency: hyperkalemia, hyponatremia, hypotension
•• Hyperpigmentation (over genitalia, nipples, recent scars, palmar crease, buccal mucosa) is a
feature of primary adrenal insufficiency due to elevated ACTH.
High Yield Points   M
Laboratory Diagnosis of Adrenal Insufficiency
•• Most common cause of adrenal
•• Most definitive test for adrenal insufficiency is ACTH stimulation test (Cosyntropin test)—
insufficiency is sudden withdrawal
of steroids in a patient on long term measurement of cortisol levels before and after administration of ACTH
steroid therapy •• In primary adrenal insufficiency, elevated renin & low aldosterone levels confirm the
•• Most common cause of primary presence of mineralocorticoid deficiency.
adrenal insufficiency in children- •• Gold standard test for diagnosing secondary adrenal insufficiency is insulin induced hypoglycemia
CAH •• MC test to diagnose secondary adrenal insufficiency is low dose ACTH stimulation test
•• Second most common cause of
primary adrenal insufficiency in Treatment of Acute Adrenal Insufficiency
children is autoimmune
•• Most common infectious etiology for •• IV 5% glucose in 0.9% saline (DNS) should be given to correct hypoglycemia, hypovolemia
adrenal insufficiency in children is & hyponatremia
meningococcemia •• IV hydrocortisone is the preferred glucocorticoid for supplementation
Chapter 20: Pediatric Endo­crinology 461

•• Mineralocorticoid supplemention is done if aldosterone deficiency is also present.

Section 3: Systemic Pediatrics


•• If both adrenal insufficiency & hypothyroidism are present, gluco­corticoid replacement is done High Yield Points
before treating hypothyroidism, otherwise, adrenal crisis may be precipitated, because thyroxine can •• In secondary adrenal insuffi­ciency,
increase cortisol clearance. only glucocorticoid defi­
ciency is
present
4. FAMILIAL GLUCOCORTICOID DEFICIENCY
Clinical Features of Familial Glucocorticoid Deficiency:
•• Most common initial presenting sign is deep hyperpigmentation of skin, mucous membrane or both
as a result of the action of ACTH on cutaneous melanocyte stimulating hormone (MSH) receptors
•• Many patients present with recurrent hypoglycemia or severe infections
Neonates usually present with feeding problems, failure to thrive, regurgitation & hypogly­cemia
manifesting as seizures.

5. MINERALOCORTICOID EXCESS
•• Primary aldosteronism: excessive aldosterone secretion independent of the renin-angiotensin
system
•• Glucocorticoid Remediable Aldosteronism (GRA): is caused by a chimeric gene resulting
from cross-over of promoter sequences between CYP11B1 & CYP11B2 genes involved in
glucocorticoid & mineralocorticoid synthesis, respectively
•• Characterized by: hypertension, hypokalemia, & suppression of renin-angiotensin system
•• Screening test: Ratio of plasma aldosterone to renin activity is always high
•• Confirmatory test: saline infusion test—which should suppress aldosterone normally, but
does not in patients with mineralocorticoid excess.

 D. DISORDERS OF INSULIN (PANCREAS)


1. DIABETES MELLITUS High Yield Points   M
Types of DM •• The most common type of diabetes
in children is type 1 DM.
•• Type 1 DM—auto immune destruction of pancreatic beta cells
•• In Indian children another cause of
•• Type 2 DM—due to insulin resistance DM is—calcific chronic pancreatitis
•• Genetic defects of beta cell function (MODY) or insulin action •• The ‘hygiene hypothesis’ states
•• Miscellaneous—drug/toxin induced, other endocrinopathies, etc. that lack of exposure to childhood
infections may increase an indi­vidual’s
Characteristics of Type 1, Type 2 and MODY in Children chances of developing autoimmune
Characteristic Type 1 Type 2 MODY diseases, including T1DM
Age at onset 5–7 years and 2nd peak at Pubertal; Incidence increases 9–25 years, AD
puberty with age.
Onset Polyuria, polydipsia and Insidious, ketosis less common Gradual.
polyphagia. DKA common
Pathogenesis Autoimmune destruction Insulin resistance Decreased beta
of beta cells, decreased cell function due to
insulin secretion monogenetic defects High Yield Points
Genetics Polygenic, major Polygenic. Most consistently Monogenetic
susceptibility loci-HLA DR3, identified are variants of TCF7L2 defects MODY 2 is •• T1DM is the most common
DR4, DQ2, DQ8. HLA DR15/ (transcription factor 7—like 2) due to mutation in endocrine-metabolic disorder of
DQ6 offers protection gene glucokinase childhood and adolescence
Treatment Insulin is the main line First line drug—metformin; MODY l & •• In type 1 Diabetes mellitus, Insulin
therapy. Pramlintide 2nd line drug—sulfonylureas 4-sulfonylureas; associated antibodies (IAA) are the
(amylin analogue) used in MODY 5— only first to appear
adolescents with insulin insulin therapy •• Most common subtype of MODY is
MODY 3
Symptoms of Diabetes include polyuria, polydipsia, and unexplained weight loss with glucosuria
and ketonuria

Diagnostic Criteria M

Impaired glucose tolerance Diabetes mellitus


Fasting glucose 100-125 mg/dL Symptoms of diabetes mellitus plus random or casual plasma
L at e s t U p d at e s
or glucose >200 mg/dL (11.1 mmol/L) or
2-hr plasma glucose during the OGTT Fasting (8 hr) plasma glucose > 126 mg/dL (7.0 mmol/L) or HbA1c should be maintained
>140 mg/dL, but <200 mg/dL 2 hr plasma glucose during the OGTT >200 mg/dL or at <7.5% for children of all ages
Hemoglobin A1c> 6.5% according to latest ISPAD guidelines.
462 Section 3: Systemic Pediatrics

Treatment of Diabetes in Children


Review of Pediatrics and Neonatology

L at e s t U p d at e s
Newer Advances in Treatment of •• Drugs of choice in type 1 DM is insulin
Diabetes •• Drugs approved for use in children are insulin, metformin and sulfonyl ureas.
•• Insulin Pump Therapy Insulin preparations
•• Continuous subcutaneous insulin
Rapidly acting Insulin lispro, aspart, glulisine
infusion
•• Use of Continuous Glucose Short acting Regular
Monitoring Systems Intermediate acting NPH insulin (Neutral Protamine Hagedorn), Lente insulin
•• Pramlintide acetate, a synthetic Long acting Glargine, detemir
analog of amylin, decreases
postprandial hyperglycemia, insulin
dose, gastric emptying & HbA1c Dose of Insulin depends on the Age and Weight of the Child:
levels.
Starting Doses of Insulin (units/kg/day)
No diabetic ketoacidosis Diabetic ketoacidosis
Prepubertal 0.25-0.50 0.75-1.0
Pubertal 0.0-0.75 1.0-1.2
High Yield Points Postpubertal 0.25-0.50 0.8-1.0

•• Dawn phenomenon is early mor­ Different Insulin regimens can be used like Basal Bolus Regime or Mixed Split Regime.
ning hyperglycemia, caused by
overnight growth hormone secretion Complications of DM M
and increased insulin clearance.
•• Somogyi phenomenon is due to Acute Complications of Diabetes
an exaggerated counterregulatory •• Diabetic ketoacidosis (DKA)
response to late-night or early- •• Cerebral edema, an extremely serious complication of DKA is common in children.
morning hypoglycemia, •• HHS (Hyperglycemic Hyperosmolar State) or nonketotic hyperosmolar coma (uncommon in children)
•• World diabetes day—November
14th—birthday of Banting who Chronic Complications of Diabetes
discovered insulin along with Best in Microvascular Macrovascular Others
1921. •• Eye disease-Retinopathy, •• Coronary heart disease •• Gastrointestinal
•• Most common pattern of macular edema. •• Peripheral arterial disease (gastroparesis , diarrhea)
dyslipidemia in DM is hypertrigly­ •• Neuropathy-Sensory •• Cerebrovascular disease •• Infections
ceridemia & reduced HDL choles­ and motor (mono- and •• Cataracts Glaucoma
terol levels. polyneuropathy), autonomic. •• Hearing loss
•• Nephropathy •• Periodontal disease

Screening Guidelines for Type 1 DM


When to commence Preferred method of
Question 4 M
screening (from diagnosis) Frequency screening
Identify this instrument used to Retinopathy Annually Fundal photography
treat a common endocri­ nologic After 5 years duration in
Nephropathy Annually 24 hr urine excretion of
problem in children: prepubertal children, after 2 years
albumin
in pubertal children
Neuropathy Annually Physical examination
Macrovascular After age 2 years Every 5 year Lipid profile
disease
Thyroid disease At diagnosis Every 2–3 year TSH
Celiac disease At diagnosis Every 2–3 year Anti-Tissue trasglutaminase or
a. Tuberculin syringe anti-endomysial antibody
b. Insulin syringe
c. Glucometer 2. NESIDIOBLASTOSIS
d. Infusion pump
What is it? It is a severe recurrent hypoglycemia associated with an inappropriate elevation of
serum insulin & C-peptide
Also called Persistent hyperinsulinemic hypoglycemia of infancy (PHHI) or congenital hyperinsulinism
Epidemiology It is the most common cause of hyperinsulinemic hypoglycemia in neonates & infancyQ
Presentation Most patients present shortly after birth with symptoms of hypoglycemia (e.g. hunger,
jitteriness, lethargy, apnea, seizures)
Treatment Immediate treatment of hypoglycemia: IV glucose; Glucagon may also be administered;
Other drugs used are diazoxide, octreotide & nifedipine
Chapter 20: Pediatric Endo­crinology 463

 E. DISORDERS OF PUBERTY

Section 3: Systemic Pediatrics


High Yield Points   M
•• Assessed by Tanner’s stages also known as sexual maturity rating, SMR
•• In boys: penis and testis development, as well as pubic hair growth are assessed •• In girls, the first sign of puberty is
•• In girls: breast development and pubic hair growth are assessed thelarche
•• Menarche usually occurs about 2 years after breast development begins •• In boys, the first sign of puberty is
•• In boys, testicular enlargement (>3 ml) marks the onset of puberty and is followed by penile testicular enlargement (>3 m1)
enlargement & appearance of pubic hair •• Tanner stage 1 is prepubertal, where
•• Pubertal growth spurt occurs later & lasts longer in boys. as Tanner stage 5 is adult maturity

PRECOCIOUS PUBERTY M

Definition
Precocious puberty is defined by the onset of secondary sexual characteristics before the age of 8
year in girls and 9 year in boys.

Classification
Based on the primary source of the hormonal production
•• Central (also known as gonadotropin dependent, or true) or
•• Peripheral (also known as gonadotropin independent or precocious pseudopuberty). High Yield Points
Important Features of: •• In India, CNS infections (TB
meningitis) are important cause of
Central precocious puberty Peripheral Precocious Puberty central precocious puberty

•• Most common form of precocious puberty. •• Can be either isosexual (developing puberty
•• More often in girls than in boys and is usually features of same sex) or heterosexual
sporadic. (developing features of other sex.
•• Most common cause: idiopathic (girls); CNS •• More common in boys.
pathology (boys) •• Most common cause: CAH (boys); ovarian
•• Treatment: therapy with GnRH agonists causes (girls).

High Yield Points High Yield Points   M


Hypothalamic Hamartomas McCune Albright syndrome: fibrous
•• Most common brain lesion caus­ing central precocious puberty bony dysplasia, skin pigmentation
•• Congenital malformation consists of ectopically located neural tissue and precocious puberty and predomi­
•• Glial cells within the hamartoma produce TGF which activates GnRH pulse generator nantly affects girls
•• MRI: small pedunculated mass attached to tuber cinereum or floor of 3rd ventricle
•• May be associated with gelastic seizures (laughing spells).

DELAYED PUBERTY
•• In boys no increase in volume of testes by 14 years and no breast bud by 13 years in Question 5 M

girls is called delayed puberty A 6-year-old girl presents with


•• Absence of menarche by age 16 or within 5 years of pubertal onset is also included in diagnosis precocious puberty, some bony
•• Delayed puberty is more common in boys than in girls. lesions & hyperpigmented skin
lesions as shown below. What is
Etiology of Delayed Puberty the most probable diagnosis?

•• Chronic systemic diseases: Gluten enteropathy, anorexia nervosa


•• Constitutional delay in growth and pubertal development—most common cause
•• Hypogonadotropic hypogonadism (Low FSH, LH)
–– CNS causes: Congenital anomalies, intracranial tumors, inflammatory & traumatic lesions
of CNS
–– Primary gonadotropin deficiency: Hypopituitarism
–– Syndromes with gonadotropin deficiency: Kallmann, Prader-Willi; Laurence-Moon-Bied-
Bardet, Froehlich syndrome
–– Miscellaneous: Hypothyroidism, hyperprolactinemia a. Prader Willi syndrome
•• Hypergonadotropic hypogonadism (high FSH, LH, Gonadal disorders) b. Laurence Moon syndrome
–– Turner syndrome (45 XO), Noonan syndrome, Klinefelter syndrome (47 XXY) c. Cushing syndrome
–– Anticancer therapy for neoplasms of gonads d. McCune-Albright syndrome
464 Section 3: Systemic Pediatrics

–– Orchitis; vanishing testes syndrome


Review of Pediatrics and Neonatology

–– Testosterone biosynthetic defects


–– Infiltration and autoimmune disease of gonads.
Treatment: Treatment of the underlying cause.

ANDROGEN INSENSITIVITY SYNDROMES (AIS)


High Yield Points
Definition Mutations in the androgen receptor (AR) cause resistance to androgen
In Androgen insensitivity syn­drome, (testosterone, DHT) action
genotype is that of a male (i.e. XY),
while phenotype is of a female Epidemiology It is the most common forms of male DSDQ

Genetic basis X-linked recessiveQ inheritance; caused by mutations in the androgen receptor
gene, located on Xq11

Clinical features •• Spectrum varies from phenotypic females to males with ambiguous genitalia &
undervirilization; Most patients have inguinal hernia (gonads)
•• In complete AIS, genetic males appear female at birth
•• The vagina ends blindly in a pouch
•• At puberty, there is normal development of breasts and the habitus is female
•• Menstruation does not occur and sexual hair is absent.
•• Adult heights of these women are commensurate with those of normal
males

Investigations Karyotype: 46,XY


USG: testes present; uterus is absent as a result of normal production of AMH
Normal or elevated testosterone and LH levels

Treatment •• Gonadectomy as there is a risk of malignancy; estrogen replacement.


•• Alternatively, the gonads can be left in situ until breast development is complete.
•• Use of graded dilators in adolescence is usually sufficient to dilate the vagina
and permit sexual intercourse.

 F. DISORDERS OF CALCIUM METABOLISM

IDIOPATHIC HYPERCALCIURIA: Refer Chapter on 'Disorders of kidney & urinary tract'


Hypercalciuria is diagnosed by a 24 hours urinary calcium excretion exceeding 4 mg/kg.

FAMILIAL HYPERCALCEMIC HYPOCALCIURIA


•• It is an autosomal dominant disorder caused by mutation in the calcium sensing receptor
•• Primary defect is abnormal sensing of blood Ca by parathyroid gland & renal tubule
•• As a result of false perception that Ca level is low in body, parathyroid gland starts secreting
more PTH & renal tubule starts reabsorbing more Ca in tubule.

 G. OBESITY
Question 6 Definition
What is the instrument used to Excess fat deposition in the body to the extent that health may be impaired.
measure the anthropometric para­
meter shown below? Etiology
Excessive caloric intake + decreased energy expenditure.
•• Endocrine causes - Hypothyroidism, Cushing's syndrome, GH deficiency
•• Hypothalamic obesity – head injury, infection, brain tumour, radiation, post neurosurgery
•• Drugs: like Corticosteroids and Valproate.

Methods to Detect Obesity


Anthropometric methods: Other ways to measure body fat:
•• BMI = wt (in kg)/ht2 (in m) •• Density-based methods
a. Harpenden caliper
•• Skinfold thickness •• Scanning methods
b. Orchidometer
c. Measuring tape •• Waist circumference •• Bioelectrical impedance methods
d. Infantometer •• Waist-hip ratio
Chapter 20: Pediatric Endo­crinology 465

Classification of Children According to BMI

Section 3: Systemic Pediatrics


Question 7
Category Percentile A 6-year-old child has neurofibro-
Underweight <5th percentile mas, cafe au lait spots and follow-
ing findings on oral examination.
Healthy weight 5th percentile-85th percentile
Which gene mutation does this
Overweight 85th to <95 th percentile child probably have?
Obesity ≥95th percentileQ

Management of Obesity
•• Lifestyle modification; Psychological evaluation and counselling
•• Pharmacotherapy for obesity not of much significance; Drugs which may be used are:
–– Drugs acting peripherally: Orlistat, Metformin a. RET b. TSC1
–– Centrally acting drugs: Sibutramine; SSRIs- fluoxetine, sertraline c. NF-2 d. VHL
–– Octreotide/Caffeine/ephedrine
•• Bariatric Surgery: An emerging modality for morbid obesity.
Bariatric surgery include:
•• Roux-en-gastric bypassQ
•• Gastric banding
•• Sleeve gastrectomy
•• Intragastric balloon.

 H. MULTIPLE ENDOCRINE NEOPLASIA (MEN) SYNDROMES Mnemonic


•• 3 “P”s for MEN-1
MEN 1 (Wermer syndrome) MEN 2A (Sipple syndrome) MEN 2B
•• “HAPPY” for MEN 2A
MEN1 (Chr 11q), encodes Menin RET on Chr 10q •• 3 "M"s for MEN 2B
Parathyroid Hyperplasia/ Hirschsprung disease Pheochromocytoma
Adenoma Amyloidosis Pheochromocytoma Medullary thyroid carcinoma
Pituitary: Hyperplasia/Adenoma Parathyroid hyperplasia/ Mucosal & gastrointestinal
Pancreas: Hyperplasia/Adenoma/ Adenoma thYroid Carcinoma neuromas
Neuroendocrine Tumor (Medullary) Marfanoid features

Answer Keys for Image-Based Questions

Answers Explanations / Identifying features


1. Ans b. Congenital This picture showing coarse facies, abdominal distension with umbilical hernia, in an infant with
hypothyroidism jaundice & constipation, suggests a diagnosis of ‘Congenital hypothyroidism’

2. Ans b. Hyperkalemia Ambiguous genitalia in an infant with dehydration & shock, suggests the diagnosis of CAH due to 21
hydroxylase deficiency;

3. Ans c. Cushing’s syndrome Rounded face with prominent flushed cheeks (moon facies) with obesity in a child with short stature &
hypertension, suggests a diagnosis of Cushing syndrome

4. Ans b. Insulin syringe A 1 ml syringe with markings in units, is a insulin syringe

5. Ans d. McCune-Albright Precocious puberty, polyostotic fibrous dysplasia & café au lait spots are features of McCune
syndrome Albright syndrome

6. Ans a. Harpenden caliper This instrument used for measuring skin fold thickness is called ‘Harpenden Callipers’

7. Ans. a. RET Multiple neuromas in the tongue are seen in MEN2B, also called the mucosal neuroma syndrome
466 Section 3: Systemic Pediatrics
Review of Pediatrics & Neonatology

Questions
 DISORDERS OF PITUITARY AND HYPOTHALAMUS 10. Thyroxine secretion begins in fetus at how many weeks
of gestation? M  (Recent Question 2016)
1. A 3-year male presents with recurrent episodes of a. 6 weeks b. 12 weeks
hypoglycemia. He also has short stature & micropenis. c. 20 weeks d. 28 weeks
He might be suffering from: M  (JIPMER Nov 2017) 11. A 2-year-old male child presents with congenital
a. Hypothyroidism b. Hypopituitarism hypothyroidism with generalized muscular pseudo-
c. Adrenal insufficiency d. Diabetes mellitus hypertrophy. What is the diagnosis?
2. Acromegaly is due to: (Recent Question 2016)  (Recent Question 2017)
a. Increased secretion of cortisol a. Lyonized Duchenne muscular dystrophy
b. Decreased secretion of cortisol b. Limb girdle dystrophy with congenital hypothyroidism
c. Increased secretion of growth hormone c. Kocher-Debre-Semelaigne syndrome
d. Decreased secretion of growth hormone d. Isolated Pseudohypertrophy with congenital
3. Lesions of which of the hypothalamic nuclei cause hypothyroidism
diabetes insipidus: (COMEDK 2016) 12. The most common cause of Permanent Congenital
a. Dorsomedial nuclei Hypothyroidism is: (MAHA PGM CET 2016)
b. Supraoptic and paraventricular nuclei a. TSH Receptor Mutations
c. Median preoptic nuclei b. Thyroid Dysgenesis
d. Ventromedial nuclei c. Defective Iodide Transport
4. All are involved in MEN IIA except: (WB PGMEE 2016) d. PIT – 1 Mutations
a. Pituitary b. Parathyroid 13. Clinical manifestations of hoarse cry, umbilical hernia,
c. Thyroid d. Adrenal hypotonia, mottling of skin, lethargy, prolonged
jaundice is seen in: M  (MAHA PGM CET 2016)
5. A 9-year-old boy presents with growth retardation and
a. Gaucher's disease
propensity to hypoglycemia. Physical examination
b. Mucopolysaccharidosis
reveals short stature, micropenis, increased fat and high-
c. Growth hormone deficiency
pitched voice. The skeletal survey reveals bone age of
d. Congenital hypothyroidism
5 years. Which of the following is most appropriate
14. Newborn with prolonged jaundice and umbilical
diagnosis? (AIIMS Nov 04)
hernia—the likely diagnosis is: M 
a. Malabsorption b. Growth hormone deficiency
a. Congenital hypothyroidism (Recent Question 2016)
c. Adrenal tumor d. Thyroxine deficiency
b. Down's syndrome
6. Which is false in congenital hypopituitarism? c. Turner's syndrome
 (AIPMEE 2003) d. Noonan syndrome
a. Growth hormone levels 7 ng/mL
b. Hypoglycemia 15. All are true about Hypothyroidism except: M 
c. Baby small at birth  (Recent Question 2013)
d. Delayed puberty a. Delayed dentition b. Widened fontanelle
7. Growth hormone deficiency is associated with all c. Distended abdomen d. All are true
except: (AIPGMEE 2002) 16. Most common cause of congenital hypothyroidism: M 
a. Micropenis b. Doll like facies  (JIPMER 2013)
c. IUGR d. Obesity a. Thyroid dysgenesis b. Dyshormonogenesis
c. Antithyroid antibodies d. Maternal Hypothyroidism
17. A 1-year-old child is brought to the hospital with thyroid
 THYROID DISORDERS swelling. The child has abnormal weight gain and has
poor activities. His TSH is found raised and T4 is very
8. Neonatal hypothyroidism when is the best time for low. Which of the following is most likely cause of his
screening? M  (JIPMER Nov 2016) disease? M  (AIPGMEE 2011)
a. 2–4 hours after birth a. Hypothalamic disease
b. 6–12 hours after birth b. Thyroid dysgenesis
c. 1–2 days after birth c. Dyshormonogenesis
d. 2–4 days after birth
d. End organ receptor insensitivity
9. Most common cause of congenital Hypothyroidism
18. The features of neonatal hypothyroidism include all
is? M  (JIPMER Nov 2016)
except: (DPG 09)
a. Iodine deficiency
a. Triangular facies with craniosyntosis
b. Thyroid agenesis
b. Congestive cardiac failure
c. Thyroid dysgenesis
c. Delayed osseous maturation
d. Thyroid dyshormonogenesis
d. Goitre is rare
Chapter 20: Pediatric Endo­crinology 467

19. Common presentations of Juvenile Hypothyroidism: 30. Which of the following is/are true in cretinism?

Questions
 (PGI June 06) a. Goiter present at birth (PGI Dec 2001)
a. Growth retardation b. Can be diagnosed by serum T4 levels
b. Mental retardation within 2 years c. Prolonged physiological jaundice present
c. Delayed puberty d. Common in iodine deficiency endemic areas
d. Umbilical Hernia e. Delayed skeletal development
e. Moist skin
20. Features of hypothyroidism in infancy include the
 ADRENAL DISORDERS
following except: (UPSC 06)
a. Premature closure of posterior fontanelle
31. Adrenal cortex of fetus mainly releases which hormone
b. Coarse facies during early part of pregnancy? (AIIMS Nov 2018)
c. Umbilical hernia a. Cortisol b. Corticosterone
d. Constipation c. Progesterone d. DHEAS
21. Clinical features of “hypothyroidism” in a newborn are 32. Congenital adrenal hyperplasia, most commonly
all except: (DPG 09, MAHE 05) presents as: (Recent Question 2018)
a. Sluggishness +++ b. Large tongue
a. Male pseudohermaphroditism
c. Large posterior fontanel d. Mental retardation
b. Female pseudohermaphroditism
22. Blood specimen for neonatal thyroid screening is c. True hermaphroditism d. 46,XY intersex
obtained on: (AIPGMEE 2005)
33. MC enzyme defect seen in a patient of congenital
a. Cord blood b. 24 hours after birth
adrenal hyperplasia is: M  (Recent Question 2018)
c. 48 hours after birth d. 72 hours after birth
a. 21-hydroxylase b. 11-hydroxylase
23. True about pendred syndrome: (PGI Dec 04)
c. 3-hydroxylase d. 17-hydroxylase
a. Diffuse colloid goiter b. Nodular goiter
c. Mental retardation d. B/L sensory neural deafness 34. Congenital adrenal hyperplasia with hypotension due to
e. Normal cochlea deficiency of enzymes: (PGI May 2017)
24. Congenital hypothyroidism is characterized by all a. Aromatase b. 5 a reductase
except: M  (AIPG 2003) c. 17 b hydroxylase d. 21 hydroxylase
a. Open posterior fontanel b. Large tongue e. 11 hydroxylase
c. Large abdomen d. Hyperthermia 35. Congenital adrenal hyperplasia with hypertension is
25. Pendred syndrome is: (PGI June 02) due to deficiency of: (Recent Question 2017)
a. Consistently associated with deafness a. 21-hydroxylase b. 3-b HSD
b. Hypothyroidism is seen c. 17-a hydroxylase d. All of the above
c. Mutation in connexin coding gene 36. In congenital adrenal hyperplasia, deficient enzyme is
d. Mutation in chromosone 21 causing receptor defect most commonly: (Recent Question 2016)
26. A 6-month-old infant is brought with a history of a. 11β hydroxylase deficiency
constipation and excessive sleepiness. On examination, b. 21α hydroxylase deficiency
he is lethargic, has periorbital puffiness, large tongue c. 3 hydroxylase deficiency d. 17 hydroxylase deficiency
and umbilical hernia. The investigation which will help 37. Commonest cause of female pseudohermaphrodism:
to diagnose this condition is: (UPSC 2002)  (Recent Question 2016)
a. T4, TSH assay b. Karyoptyping a. Congenital Adrenal hyperplasia
c. Rectal mucosal biopsy d. Knee X-ray b. Maternal ovarian tumor
27. A 8-year-old-child presents with lethargy, multiple c. Maternal drug intake
epiphyseal breaks, wormian bones with growth d. Chromosomal abnormalities
retardation and mental retardation. Diagnosis is: M  38. Drug used in the treatment of CAH in a child is: M 
 (AIIMS Nov 01)  (Recent Question 2016)
a. Rickets b. Hypothyroidism a. Dexamethasone b. Betamethasone
c. Scurvy d. Hypoparathyroidism c. Prednisolone d. Hydrocortisone
28. Manifestations of endemic cretinism include: (AI 01) 39. What percentage of pheochromocytomas are malignant?
a. Deafness and facial nerve involvement a. 5 b. 10 (COMEDK 2016)
b. Blindness and hypothyroidism
c. 20 d. 15
c. Strabismus and spastic diplegias
40. In Pheochromocytoma, which of the following is
d. Multinodular goitre and mental retardation
increased in urine? (Recent Question 2016)
29. Which of the following is true regarding cretinism?
a. VMA b. Aldosterone
 (AI 01)
c. Cortisol d. 17 hydroxyprogesterone
a. Short limbs compared to trunk
b. Proportionate shortening 41. False about Addison's disease is: (Recent Question 2015)
c. Short limbs and short stature a. Hyperpigmentation b. Hypoglycemia
d. Short limbs and long stature c. Hypotension d. Hypertension
468 Section 3: Systemic Pediatrics

53. Most common enzyme deficiency leading to childhood


Review of Pediatrics & Neonatology

42. Pheochromocytoma is the disease of:


 (MAHA PGM CET 2015) hypertension is: (DNB Pattern 2013)
a. Pituitary gland b. Adrenal medulla a. 17-Alpha hydroxylase
c. Pancreas d. Adrenal cortex b. 21-Beta Hydroxylase
43. Aldosterone acts by promoting: (MAHA PGM CET 2015) c. 11-Beta Hydroxylase
a. Reabsorption of sodium and excretion of potassium d. 3-Beta Hydroxy steroid dehydrogenase
b. Reabsorption of potassium and excretion of sodium 54. Pseudohermaphroditism in a female child is most
c. Excretion of both sodium and potassium commonly due to: (Recent Question 2013-12)
d. Reabsorption of both sodium and potassium a. 21-hydroxylase deficiency
44. Which of the following effects is NOT SEEN in primary b. 17-hydroxylase deficiency
hyperaldosteronism (Conn's syndrome)? c. 11-hydroxylase deficiency
 (MAHA PGM CET 2015) d. 3-hydroxylase deficiency
a. Hypertension 55. All of the following can cause precocious puberty in
b. Metabolic Alkalosis males except:
c. Hyperkalemia  (Recent Question 2013-12; AIIMS Dec 1995)
d. Expansion of Extracellular and Plasma volume a. 17 alpha hydroxylase deficiency
45. All are features of Autoimmune Addison Disease except: b. 21 alpha hydroxylase deficiency
 (MAHA PGM CET 2015) c. 11 hydroxylase deficiency
a. Hypoglycemia b. Ketosis d. None of the above
c. Hyponatremia d. Hypokalemia 56. A 2-year-old child presented with diarrhea and failure to
46. Two substances are needed for a developing fetus to thrive, Blood examination shows: Na = 122, K = 6. He is
grow male genitalia: (MAHA PGM CET 2015) most probably suffering from:
a. Wolffian factor and Mullerian inhibitory factor  (Recent Question 2013-12; NEET/DNB Pattern)
b. Testosterone and Mullerian inhibitory factor a. Bartter syndrome
c. Wolffian factor and testosterone b. 21 Hydroxylase deficiency
d. Mullerian factor and testosterone c. 11-β Hydroxylase deficiency
47. The most common cause of Cushing's syndrome is: M d. 17-Hydroxylase deficiency
 (Bihar PG 2015) 57. About Cushing syndrome all are true except:
a. Pituitary adenoma b. Adrenal adenoma  (Recent Question 2012)
c. Ectopic ACTH d. Iatrogenic steroids a. Purple striae b. Plethora
48. A 3-week-female infant is brought for ambiguous c. Hypoglycemia d. Obesity
genitalia and hyperpigmentation of skin. She has 58. A 7-year-old child presents to the pediatric clinic
hyponatremia and hyperkalemia. Which one of the with ambiguous genitalia increasing with age. On
following is the most likely diagnosis? (APPG 2015) examination her height, weight and blood pressure were
a. 21 hydroxylase deficiency recorded within normal limits. Labia appeared bifid
b. 17 alpha hydroxylase deficiency with 2 separate perineal opening, phallic length was 2.5
c. 17, 20 lyase deficiency cm and no palpable gonads were noted in the inguinal
d. 11 beta hydroxylase deficiency region. USG shows presence of mullerian structure. The
49. Addison's disease is characterized by all, except: most probable diagnosis is: (AIPG 12)
 (Bihar PG 2015) a. Classic Salt-Wasting 21 Hydroxylase deficiency
a. Hyperglycemia b. Hypotension b. Simple virilizing congenital adrenal hyperplasia
c. Hyperkalemia d. Hyponatremia c. Complete Androgen Insensitivity Syndrome
50. Accepted screening test for Hyperaldosteronism is: d. 5-Alpha Reductase Deficiency
 (MAHA PGM CET 2014) 59. A 2-month-old infant is presented with failure to
a. Measurement of sodium levels thrive, recurrent emesis, hepatosplenomegaly, and
b. Measurement of aldosterone levels adrenal insufficiency. Adrenal calcification is noted
c. Calculation of aldosterone renin ratio radiologically. The most likely diagnosis is:
d. Measurement of potassium levels  (AIIMS May 2011, 2010, 2009)
51. Congenital Adrenal Hyperplasia is due to deficiency of: a. Adrenal hemorrhage b. Wolman's disease
 (Recent Question 2014) c. Pheochromocytoma d. Addison's disease
a. 21 beta hydroxylase b. 17 alpha hydroxylase 60. A 4-week-old female child with normal genitalia presents
c. 5 alpha reductase d. 17 beta reductase to the emergency department with severe dehydration,
52. Nelson syndrome is seen in: (DNB Pattern 2014) hyperkalemia and hyponatremia. Measurement blood
a. Adrenalectomy levels of which of the following will be helpful?
b. Hypopitutarism  (AIIMS May 11)
c. Deficiency of beta cells a. 17 hydroxy progesterone b. Renin
d. Deficiency of growth hormone c. Cortisol d. Aldosterone
Chapter 20: Pediatric Endo­crinology 469

61. A 5-year-old boy presents with pubic hair development. 68. Most common type of congenital adrenal hyperplasia

Questions
He is tall and has increased pigmentation of his genitalia (CAH): M  (MAHA PG 2010)
and phallic enlargement. Blood pressure is 130/90 mm a. 21-hydroxylase deficiency
Hg. Measurement of which of the following hormones b. 11-beta hydroxylase deficiency
would be most likely to be diagnostic? c. 3-hydroxylase deficiency
 (AIIMS May 2011) d. 17-alpha hydroxylase deficiency
a. Increase 17-hydroxyprogesterone 69. A newborn presents with shock, hyperkalemia and
b. Increase cortisol hypoglycemia. What is the most likely diagnosis? M 
c. Increase aldosterone  (UPSC 09)
d. Increase 11 deoxycortisol a. Septicemia
62. Neonatal hypoglycemia is caused by: (WBPG 2011) b. Inborn error of metabolism
a. IV glucocorticoid administration c. Diabetes mellitus
b. Epinephrine deficiency d. Congenital adrenal hyperplasia
c. Mesothelioma 70. A 3-month-old male child with normal genitalia presents
d. Pancreatic atresia to the emergency department with severe dehydration of
63. Which of the following is the least common cause of hyperkalemia and hyponatremia. The measurement of
blood levels of which of the following will be helpful?
ambiguous genitalia in a female child? (AIPGMEE 11)
 (AIIMS May 2009)
a. Fetal placental steroid sulfatase deficiency
a. 17-hydroxy progesterone b. Renin
b. Fetal placental aromatase deficiency
c. Cortisol d. Aldosterone
c. Congenital adrenal hyperplasia
d. WNT-4 gene mutation 71. A five-year-old boy has precocious puberty along with
BP 130/80 mm Hg. Estimation of which of the following
64. An 8-day-old male infant is brought to the emergency
will help in diagnosis? (AI 09)
department with vomiting, lethargy, dehydration
a. 17-Hydroxyprogesterone b. 11-Deoxycortisol
and features of shock. Clinical examination reveals
c. Aldosterone d. DOCA
hyperpigmentation of the genital skin and normal
72. All of the following statements about androgen
external genitalia. Blood investigations reveal Sodium
insensitivity syndrome are true except: (AIPGMEE 08)
124 mEq/L, potassium of 7 mEq/L and hypoglycemia.
a. It is an X linked disorder
Which of the following is the most likely diagnosis?
b. XY genotype is present
 (AIPGMEE 2011)
c. Affected individuals have female phenotype
a. Congenital Adrenal Hyperplasia (CAH)
d. Abundant pubic hairs are present
b. Adrenal Hemorrhage
73. Which of the following statements about 21 alpha
c. Acute Gastorenteritis with dehydration
hydroxylase deficiency is false? (AIPGMEE 08)
d. Hyperaldosteronism
a. Most common cause of congenital adrenal hyperplasia in
65. A 7-year-old boy underwent neurosurgery for cranio- children
pharyngioma following which pituitary functions were b. Affected females present with ambiguous genitalia
lost. Which of the following hormone should be re- c. Affected males present with precocious puberty
placed first? (AIPGMEE 11) d. Hypokalemic alkalosis is seen
a. Hydrocortisone b. Thyroxine
74. A 5-year-old girl presents with hypertension and
c. Growth hormone d. Prolactin virilization. There is also finding of hypokalemia. What
66. A 10-month-old baby previously normal, suddenly is the diagnosis? (AIIMS May 07)
becomes distressed in his crib. The external appearance a. 21-hydroxylase deficiency
of genitalia was normal, except hyperpigmentation. b. 3-β hydroxy steroid deficiency
Blood glucose showed a level of 30 mg%. What is the c. 11-β hydroxylase deficiency
most probable diagnosis? d. Conn's disease
 (AIPGMEE 2011 AIIMS May 2010, 2009) 75. Male pseudohermaphroditism is seen in: (PGI Dec 07)
a. 21 hydroxylase deficiency a. 5α reductase deficiency
b. Hyperinsulinism b. 21 hydroxylase deficiency
c. Familial glucocorticoid deficiency c. 17 hydroxylase deficiency
d. Cushing's syndrome d. Gonadal dysgenesis
67. A 1-week-old male newborn suddenly presents with e. 11 beta hydroxylase deficiency
lethargy, poor oral intake and shock. Investigations 76. A 5-year-old girl presents with hypertension and
reveal hyperkalemia, hyponatremia and hypoglycemia. virilization. There is also finding of hypokalemia. What
What is the most likely diagnosis? (UPSC-I 10) is the diagnosis? (AIIMS May 07)
a. Syndrome of inappropriate ADH secretion a. 21-hydroxylase deficiency
b. Gram negative sepsis b. 3-β hydroxy steroid deficiency
c. Congenital adrenal hyperplasia c. 11-β hydroxylase deficiency
d. Phenylketonuria d. Conn's disease
470 Section 3: Systemic Pediatrics

77. Which one of the following drugs is used for fetal therapy
Review of Pediatrics & Neonatology

87. All are features of type I Diabetes mellitus except: M


of congenital adrenal hyperplasia? (AIPGMEE 2005)
 (FMGE Pattern 2017)
a. Hydrocortisone b. Prednisolone a. Polyuria b. Polydipsia
c. Fludrocortisone d. Dexamethasone c. Polyphagia d. Diarrhea
78. Which one of the following is the earliest manifestation 88. Definition of Diabetes mellitus includes:
of Cushing's syndrome: (AIIMS May 05)
 (Recent Question 2015)
a. Loss of diurnal variation
a. Fasting blood glucose > 126 mg/dL
b. Increased ACTH
b. Random blood glucose > 140 mg/dL
c. Increased plasma cortisol
c. HbA1c < 7%
d. Increased urinary metabolites of cortisol
d. Post prandial blood glucose > 180 mg/dL
79. ‘Weak giants' are produced by: (AIIMS May 04)
89. Which of the following is a rapid acting insulin? 
a. Thyroid adenomas b. Thyroid carcinomas
 (Recent Question 2015)
c. Parathyroid adenomas d. Pituitary adenomas
a. Glargine b. Detemir
80. Adrenal hyperplasia due to 21 hydroxylase deficiency is c. Lente d. Glulisine
treated with low dose: (Kerala PG 2004)
90. Monogenic transmission of diabetes mellitus occurs in:
a. Androgen b. Estrogen
 (MAHA PGM CET 2014)
c. Cortisone d. Anti-androgen
a. Insulin Dependent Diabetes Mellitus (IDDM)
81. An 8-day-old breastfed baby presents with vomiting, b. Non-insulin Dependent Diabetes Mellitus (NIDDM)
poor feeding and loose stools. On examination, the c. Latent Autoimmune Diabetes in Adults (LADA)
heart rate is 190/minute, blood pressure 50/30 mm Hg,
d. Maturity Onset Diabetes of the Young (MODY)
respiratory rate 72 breaths/minute and capillary refill
time of 4 seconds. Investigations show hemoglobin level 91. Percentage of dose given as basal insulin in basal-bolus
of 15 g/dL, Na-124 mEq/L, K-6.8 mEq/L, Cl-81 mEq/L, regimen in children is: (DNB Pattern 2013)
bicarbonate 15 mEq/L, Urea 30 mg/dL, creatinine 0.6 a. 0–25% b. 25–50%
mg/dL. The most likely diagnosis is: (AIIMS Nov 03) c. 50–75% d. None of these
a. Congenital adrenal hyperplasia 92. In a 2–5 year-old child with DM, target HbA1C is:
b. Acute tubular necrosis  (Recent Question 2013)
c. Congenital hypertrophic pyloric stenosis a. < 8% b. < 7.5%
d. Galactosemia c. < 7% d. < 6.5%
82. The most common cause of ambiguous genitalia in a 93. The following is true about Nesidioblastosis except:
newborn is: (AIPGMEE 2002)  (AIPGMEE 2011)
a. 21 hydroxylase deficiency a. Presents with hypoglycemic attacks
b. 11-β hydroxylase deficiency b. More common in adults than in children
c. 17-β hydroxyalse deficiency c. Histopathology shows hyperplasia of islet cells
d. 3-β hydroxysteroid dehydrogenase deficiency
d. Diazoxide is used for treatment
83. Salt losing hydroxylase deficiency is characterized by:
94. Oral glucose tolerance test in children is done with:
 (PGI June 2001, PGI Dec 2000)
 (PGI Dec 07)
a. Hyponatremia b. Hyperkalemia
a. 1.5 gm/kg glucose b. 1.75 gm/kg glucose
c. Hypoglycemia d. Hypocalcemia
e. Hypercalcemia c. 2 gm/kg glucose d. 2.5 gm/kg glucose
e. 75 gm as an adult
84. A female has previous child with congenital adrenal
hyperplasia. In the present pregnancy, steroid therapy 95. In children with type I Diabetes mellitus, when is
should be started: (AIPGMEE 2000) ophthalmologic evaluation indicated: (PGI June 06)
a. After karyotyping and determination of sex of the baby a. At the time of diagnosis
b. At the time of delivery b. After 1 year
c. Before conception c. After 2 years
d. As soon as pregnancy is diagnosed d. After 5 years
85. A 10-day-old male pseudohermaphrodite child with 46 e. After 10 years
XY karyotype presents with BP of 110/80 mm Hg. Most 96. Injection of Glucagon is effective for management of
likely enzyme deficiency is: (AIPGMEE 2001) persistent hypoglycemia in all except: (AIIMS May 04)
a. 21 hydroxylase b. 17 hydroxylase a. Large for date baby
c. 11 hydroxylase d. 3-beta hydroxylase b. Galactosemia
c. Infant of diabetic mother
d. Nesidioblastosis
 DIABETES MELLITUS
97. In India the commonest cause of Juvenile Onset of
86. Screening for nephropathy in prepubertal children with Diabetes mellitus: (AIIMS May 01)
type 1 DM should be initiated after how many years of a. IDDM
disease onset? (FMGE Pattern 2017) b. Fibrocalcific pancreaticopathy
a. 2 years b. 3 years c. MODY
c. 4 years d. 5 years d. Gallstones
Chapter 20: Pediatric Endo­crinology 471

 DISORDERS OF PUBERTY

Questions
108. Causes of precocious puberty in girls are all except: M 
a. Hypothalamic hamartoma (Recent Question 2014)
98. Most common cause of central precocious puberty in b. Hypothyroidism
girls: (Recent Question 2017) c. McCune Albright syndrome
a. Exogenous estrogen b. Idiopathic d. Prader Willi syndrome
c. CNS tumor d. Hypothyroidism 109. Gonads to testes differentiation: (Recent Question 2013)
99. Precocious puberty in girls is defined by the onset of a. SRY gene b. WNT-4 gene
secondary sexual characteristics before the age of: M  c. DAX1 gene d. None
 (Recent Question 2016)
110. Male pseudohermaphroditism: (Recent Question 2013)
a. 5 years b. 7 years
a. XX genotype, male external genitalia
c. 8 years d. 9 years
b. XY genotype, female external genitalia
100. Drug of choice for precocious puberty: c. Testis and ovary both present
 (AIIMS Nov 2016) d. Male external genitalia and ovary present
a. Cyproterone acetate b. Danazol 111. For precocious puberty in girls, age limit is? M 
c. Medroxyprogesterone d. GnRH agonists  (Recent Question 2012)
101. Most common brain lesion causing central precocious a. 8 year b. 10 year
puberty is: (Recent Question 2016) c. 9 year d. 11 year
a. Tuberculous meningitis b. Tuberous sclerosis 112. A child with decreased levels of LH, FSH and Testosterone
c. Astrocytoma d. Hypothalamic hamartoma presents with delayed puberty. Which of the following is
102. Select the FALSE combination of chromosomal pattern the most likely diagnosis: (AIPGMEE 12)
and the syndrome: (APPG 2016) a. Klinfelter's syndrome
a. Mayer Rokitansky-46XY b. Kallmann's syndrome
b. Swyer's syndrome-46XY c. Androgen Insensitivity syndrome
c. Turner's syndrome-45XO d. Testicular Infection
d. Klinefelter's syndrome-47XXY 113. Features of Laurence-Moon-Biedl syndrome include:
103. Androgen insensitivity syndrome – true is?  (Manipal 09)
 (WB PGMEE 2016) a. Hypogonadism b. Obesity
a. 46XX b. Vagina present c. Polydactyly d. All of the above
c. Androgen receptors working 114. Most common cause of delayed puberty in males is:
d. Amenorrhea with normal secondary sexual characters  (AIPGMEE 2008)
104. Delayed puberty is when primary amenorrhea is seen a. Kallmann's syndrome b. Klinefelter syndrome
without development of secondary sexual characters c. Constitutional d. Prader-Willi syndrome
beyond the age of? (WB PGMEE 2016) 115. Association of sexual precocity, multiple cystic bone
a. 12 years b. 14 years lesions and endocrinopathies are seen in: M 
c. 16 years d. 18 years  (COMED 08)
105. In hypogonadotropic hypogonadism? a. McCune-Albright's syndrome
 (Recent Question 2016) b. Granulosa cell tumor
a. Both LH and FSH decreased c. Androblastoma
b. Both LH and FSH increased d. Hepatoblastoma
c. LH increased and FSH decreased 116. A 16-years-old girl with primary amenorrhea attends
d. LH decreased and FSH increased OPD. She has normal sexual development and
106. Central precocious puberty in a girl is defined as breast normal breast but with absent pubic and axillary hair.
development before the age of? Examination shows B/L inguinal hernias. USG shows
 (MAHA PGM CET 2015) absent uterus and blind vagina. Diagnosis will be:
a. 6 years b. 8 years  (AIIMS May 07)
c. 12 years d. 10 years a. Turner syndrome
107. A 16-year-old woman athlete presents with complaints b. Mullerian agenesis
of primary amenorrhea. Her height is 153 cm, weight is c. Star syndrome
51 kg. She has well developed breasts. She has no pubic d. Androgen insensitivity syndrome
or axillary hair and no hirsutism. She has been given 117. Delayed puberty seen in: (PGI Dec 06)
6 months of progesterone and estrogen but no benefit. a. Chronic disease b. Hypothyroidism
Which of the following is the most probable diagnosis? c. Turner's syndrome d. Malabsorption syndrome
 (AIIMS May 2015) e. McCune Albright syndrome
a. Turner syndrome 118. The karyotype of a patient with androgen Insensitivity
b. Stein-Leventhal syndrome Syndrome is: (AI 05)
c. Premature ovarian failure a. 46 XX b. 46, XY
d. Complete androgen insensitivity syndrome c. 47 XXY d. 45 XO
472 Section 3: Systemic Pediatrics

119. A 6-month-old boy weighing 3.2 kg presents with


Review of Pediatrics & Neonatology

125. Obesity in a child of age 2 years is defined as:


recurrent vomiting and polyuria. Investigations show  (COMEDK 2016)
blood urea 60 mg/dL, creatinine 0.7 mg/dL, calcium a. Weight for height more than double of expected
12.8 mg/dL, phosphate 3 mg/dL, pH 7.45, bicarbonate b. Weight for age more than double of expected
25 mEq/L and PTH 140 pg/mL (normal <60 pg/mL). c. BMI > 30
Daily urinary calcium excretion is reduced. Ultrasound d. BMI > 95th percentile
abdomen shows bilateral nephrocalcinosis. The most 126. Which protein secreted by adipocytes prevents obesity?
likely diagnosis is: (AIIMS Nov 05) a. Cathepsin b. Leptin (COMEDK 2016)
a. Bartter syndrome c. Neuropeptide Y d. Galanin
b. Mutation of the calcium sensing receptor
c. Pseudo-pseudohypoparathyroidism
d. Parathyroid adenoma  DISORDERS OF CALCIUM METABOLISM
120. True statement about testicular feminising syndrome;
127. Which of the following is/are true about neonatal
are all except: (PGI June 04)
hypoparathyroidism? (PGI Nov 2015)
a. Absent uterus
a. Familial isolated hypoparathyroidism is where a genetic
b. Absent vagina
mutation like SOX3 is the underlying cause
c. Chromosome pattern 46 XY
b. Calcium sensing receptor (CaSR) mutation is another
d. Absent ovary disorder of genetic origin
121. A baby girl presents with bilateral inguinal masses, c. DiGeorge syndrome (22q11 deletion syndrome) is a
thought to be hernias but are found to be testes in the condition which includes hypoparathyroidism at birth
inguinal canals. Which karyotype would you expect to d. Hypercalcemia is a salient feature
find in the child? (AIIMS Nov 04) e. Seizures are not seen
a. 46,XX b. 46,XY 128. Osteitis fibrosa cystica is a feature seen in:
c. 46,XXY d. 47,XXY  (APPG 2016)
122. Precocious puberty is seen in: (PGI Dec 02) a. Paget's disease b. Hyperparathyroidism
a. Hypothyroidism c. Multiple myeloma d. Hypocalcemia
b. CNS irradation 129. Pseudohypoparathyroidism is characterized by:
c. McCune-Albright syndrome  (MAHA PGM CET 2016)
d. Turner's syndrome a. Normal serum Ca++ and decreased serum PTH
e. Congenital adrenal hyperplasia b. Decreased serum Ca++ and decreased serum PTH
123. True of testicular feminization syndrome is: c. Decreased serum Ca++ and Increased serum PTH
 (PGI June 02) d. Normal serum Ca++ and Increased serum PTH
a. Testes are present b. Female habitus 130. Familial hypocalciuric hypercalcemia is an autosomal
c. XY genotype d. Secondary amenorrhea dominant disorder characterized by mild elevation of
e. Uterus present calcium and PTH levels. It occurs secondary to what
mutation? (COMEDK 2016)
a. Missense mutation of mitochondrial calcium receptor
 OBESITY b. Missense mutation of ribosomal calcium surface protein
c. Missense mutation of Golgi complex calcium receptor
124. Which disease is the most likely diagnosis as depicted
d. Mutations of the calcium-sensing receptor (CASR)
by these pictures of a child, his hands and retina?
 (APPG 2016) 131. A 3-year-old boy is detected to have bilateral renal
calculi. Metabolic evaluation confirms the presence
of marked hypercalciuria with normal blood levels
of calcium, magnesium, phosphate, Uric acid and
creatinine. A diagnosis of idiopathic hypercalciuria is
made. The dietary management includes all, except:
 (AIIMS May 03)
a. Increased water intake b. Low sodium diet
c. Reduced calcium intake d. Avoid meat proteins
132. A baby presents with tetany. First thing to be done is
administration of: M (AIIMS June 2000)
a. Carpenter syndrome b. Alstrom syndrome a. Diazepam b. Vitamin D
c. Bardet-Biedl syndrome d. Prader-Willi syndrome c. Calcium gluconate d. Calcitonin
Chapter 20: Pediatric Endo­crinology 473

Answers with Explanations


Answers with Explanations
 DISORDERS OF PITUITARY and HYPOTHALAMUS

1. b. Hypopituitarism  Ref: Nelson's 20/e p 2642


2. c. Increased secretion of growth hormone  Ref: Nelson's 20/e p 2653
In young persons with open epiphyses, overproduction of GH results in gigantism; in persons with closed epiphyses, the result is
acromegaly.
3. b. Supraoptic and paraventricular nuclei  Ref: Nelson's 20/e p 348-349, Ghai 8/e p 511
Vasopressin, a 9-amino-acid peptide, has both antidiuretic and vascular pressor activity and is synthesized in the paraventricular and
supraoptic nuclei of the hypothalamus.
Deficiency of Vasopressin is responsible for Diabetes insipidus.
4. a. Pituitary  Ref: Nelson's 20/e p 2694-2481, Ghai 8/e p 523
In Multiple Endocrine Neoplasia (MEN) IIA, features seen are: “HAPPY”, i.e. Hirschsprung disease, Amyloidosis, Pheochromocytoma,
Parathyroid hyperplasia/Adenoma and thyroid Carcinoma (Medullary).
5. b. Growth hormone deficiency  Ref: Ghai 8/e p 511-516
The given clinical features like growth retardation, short stature, hypoglycemia, micropenis, increased fat, high-pitched voice
and bone age less than chronological age, all point towards a diagnosis of growth hormone deficiency.
6. c. Baby small at birth  Ref: Nelson's 20/e p 2642, Ghai 8/e p 511-516
The child with hypopituitarism usually has normal size & weight at birth, although those with MPHD (Multiple pituitary
hormone deficiency) & genetic defects of the GH1 or GHR gene have birth lengths <1 SD below the mean.
Clinical Features of Growth Hormone deficiency:
Craniofacies Growth and development Other features
Frontal bossing Birth weight and length: near-normal Hypoglycemia: in infants and children;
Normal head circumference; Postnatal severe growth failure Motor milestones: delayed
Delayed dentition Bone age: delayed (but advanced relative Skin: thin, prematurely aged Osteopenia
Voice: high pitched to height) Micropenis; Puberty: delayed
but fertility: normal

7. c. IUGR  Ref: Ghai 8/e p 511

 THYROID DISORDERS

8. d. 2-4 days after birth


9. c. Thyroid dysgenesis  Ref: Nelson's 20/e p 2669-2773, Ghai 8/e p 516-521
Most cases of congenital hypothyroidism are not hereditary and result from thyroid dysgenesis.
10. b. 12 weeks  Ref: Nelson's 20/e p 2663
11. c. Kocher-Debre-Semelaigne syndrome  Ref: Nelson's 20/e p 2669; Refer pretext for details
12. b. Thyroid Dysgenesis  Ref: Nelson's 20/e p 2669-2773, Ghai 8/e p 516-521
13. d. Congenital hypothyroidism  Ref: Nelson's 20/e p 2669-2773, Ghai 8/e p 516-521; Refer pretext
The given clinical features, i.e. hoarse cry, umbilical hernia, hypotonia, mottling of skin, lethargy, prolonged jaundice suggest a
diagnosis of congenital hypothyroidism
14. a. Congenital Hypothyroidism  Ref: Nelson's 20/e p 2669-2773, Ghai 8/e p 516-521
15. d. All are true  Ref: Nelson's 20/e p 2669-2773, Ghai 8/e p 516-521
If left untreated, a child with Hypothyroidism develops the following features:
•• Growth will be stunted, the extremities are short
•• Face: Depressed bridge of nose, swollen eyelids, thick, broad, protruding tongue
•• Delayed Dentition, hoarse voice; Broad hands with short fingers; Dry and scaly skin with little perspiration

16. a. Thyroid dysgenesis  Ref: Nelson's 20/e p 2669-2773, Ghai 8/e p 516-521
17. c. Dyshormonogenesis  Ref: Nelson's 20/e p 2669-2773, Ghai 8/e p 516-521
Most common cause of congenital hypothyroidism in a child with goiter is Thyroid dyshormonogenesis.
474 Section 3: Systemic Pediatrics

18. a. Triangular facies with craniosyntosis  Ref: Nelson's 20/e p 2669-2773, Ghai 8/e p 516-521
Review of Pediatrics & Neonatology

•• Craniosynostosis refers to premature fusion of sutures and closure of fontanelle


•• But in congenital hypothyroidism, there is delayed closure of fontanelle.
19. a. Growth retardation, c. Delayed puberty, d. Umbilical Hernia  Ref: Nelson's 20/e p 2669-2673
Clinical features of acquired hypothyroidism:

•• Deceleration of growth is usually the first clinical manifestation
•• Goiter associated with Hashimoto thyroiditis, is non tender, firm, with a rubbery consistency and a pebbly surface
•• Weight gain is mostly fluid retention (myxedema); myxedematous changes of skin, constipation, cold intolerance
•• Bradycardia, muscle weakness or cramps, nerve entrapment, and ataxia
•• Osseous maturation is delayed and adolescents typically have delayed puberty
••

Surprisingly, schoolwork and grades usually do not suffer, even in severely hypothyroid children

20. a. Premature closure of posterior fontanelle  Ref: Nelson's 20/e p 2669-2773, Ghai 8/e p 516-521
21. d. Mental retardation  Ref: Nelson's 20/e p 2669-2773, Ghai 8/e p 516-521
Mental retardation (Intellectual disability) develops by 3-6 months of age (not in neonatal period).
22. d. 72 hours after birth  Ref: Nelson's 20/e p 2669-2773, Ghai 8/e p 516-521
This is a controversial question
•• Blood for neonatal thyroid screening is ideally obtained by heel prick between 2 and 5 days of life is placed on a filter
paper card and sent to a central screening laboratory
•• Cord blood may also be used, especially in regions where the neonates may not be brought for follow-up after 48 hrs life; But
the best answer, amongst the given options is 72 hours after birth.
23. a. Diffuse colloid goiter, d. B/L sensory neural deafness  Ref: Nelson's 20/e p 2669-2773, Ghai 8/e p 516-521
Pendred Syndrome is an autosomal recessive disorder caused by mutations in the PDS gene on chr 7, which codes for pendrin
(SLC26A4), a transporter protein found in the cochlea, thyroid and kidney; Congenital b/l SNHL and goitre (after puberty) with
hypothyroidism seen.
24. d. Hyperthermia  Ref: Nelson's 20/e p 2669-2773, Ghai 8/e p 516-521
25. a. Consistently associated with deafness, b. Hypothyroidism is seen  Ref: Nelson's 20/e p 2669-2773
26. a. T4, TSH assay  Ref: Nelson's 20/e p 2669-2773, Ghai 8/e p 516-521
It is a typical presentation of hypothyroidism, so thyroid hormone assay is required.
27. b. Hypothyroidism  Ref: Nelson's 20/e p 2669-2773, Ghai 8/e p 516-521
28. c. Strabismus and spastic diplegias  Ref: Nelson's 20/e p 2669-2773, Ghai 8/e p 516-521
The term endemic cretinism includes two different overlapping syndromes: Neurologic and Myxedematous.
In Neurological Cretinism, there is goiter, but the child is euthyroid; Deaf mutism, squint and cerebral diplegia are usually present
29. c. Short limbs and short stature  Ref: Nelson's 20/e p 2669-2773, Ghai 8/e p 516-521
Hypothyroidism is a cause of disproportionate short stature with short limbs.
30. a. Goiter present at birth, b. Can be diagnosed by serum T4 levels, c. Prolonged physiological jaundice present,
d. Common in iodine-deficiency endemic areas, e. Delayed skeletal development  Ref: Nelson's 20/e p 2669-2773

 ADRENAL DISORDERS
31. a. Cortisol  Ref: Nelson's 20/e p 2700
Shortly afer the fetal adrenal gland forms (wk 8-10), it efficiently secretes cortisol, which is able to negatively feedback on fetal
pituitary and hypothalamus to suppress ACTH secretion.
  After wk 12, 3b hydroxy steroid dehydrogenase activity in the fetal adrenal gland decreases and steroid sulfokinase activity
increases. Thus, the major steroid products of the midgestation fetal adrenal gland are DHEA and DHEA sulfate (DHEAS).
  Cortisol activity is low during the 2nd trimester, which might serve to prevent premature secretion of surfactant by the developing
fetal lungs; surfactant levels can affect the timing of parturition.
32. b. Female pseudohermaphroditism  Ref: Nelson 20/e p 2714-2718
Note: Male pseudohermaphrodite is now called 46,XY DSD, while female → 46 XX DSD;
So female pseudohermaphrodite is genotypically female (46,XX), whose external genitalia looks like that of a male;
In CAH due to 21-hydroxylase deficiency, females have ambiguous genitalia, while males have normal genitalia
33. a. 21-hydroxylase  Ref: Nelson's 20/e p 2714
More than 90% of CAH cases are caused by 21-hydroxylase deficiency.
Chapter 20: Pediatric Endo­crinology 475

34. d. 21 hydroxylase  Ref: Nelson's 20/e p 2714

Answers with Explanations


35. c. 17-a hydroxylase  Ref: Nelson's 20/e p 2714
36. b. 21 α hydroxylase deficiency  Ref: Nelson's 20/e p 2714
37. a. Congenital Adrenal hyperplasia  Ref: Nelson's 20/e p 2751
38. d. Hydrocortisone  Ref: Nelson's 20/e p 2719; Refer pretext of this chapter
Treatment of CAH (21 hydroxylase deficiency) involves use of Hydrocortisone 15-20 mg/m2/24 hr, administered orally in
3 divided doses. Double or triple doses are indicated during periods of stress, such as infection or surgery.
39. b. 10  Ref: Nelson's 20/e p 2227-2729, Ghai 8/e p 525

Mnemonic
‘Rule of ten' for Pheochromocytoma: (“BG MEN”): 10% of Pheochromocytoma are:
•• Bilateral •• Extra-adrenal: paragangliomas
•• Malignant •• Normotensive (No hypertension)

Note: 10% of sporadic Pheochromocytoma are bilateral; but 50% of familial tumors are bilateral.
40. a. VMA  Ref: Nelson's 20/e p 2227-2729, Ghai 8/e p 525
Biochemical tests for Pheochromocytoma:
•• Pheochromocytomas synthesize and store catecholamine, so elevated plasma and urinary levels of catecholamines methylated
metabolites (VMA or vanillyl mandelic acid), metanephrines, are the cornerstone for the diagnosis
•• Among these tests, the fractionated metanephrines and catecholamines are the most sensitive.Q
•• In contrast to adults with pheochromocytoma in whom both norepinephrine and epinephrine are elevated, children with
pheochromocytoma predominantly excrete norepinephrine in the urine.
41. d. Hypertension  Ref: Nelson's 20/e p 2227-2729, Ghai 8/e p 525
Hypotension is seen in Addison's disease due to glucocorticoid deficiency and not hypertension.
42. b. Adrenal medulla  Ref: Nelson's 20/e p 2227-2729, Ghai 8/e p 525
Pheochromocytoma is a neoplasms of chromaffin cells in medulla that release catecholamines.
43. a. Reabsorption of sodium and excretion of potassium  Ref: Nelson's 20/e p 2227-2729, Ghai 8/e p 525
44. c. Hyperkalemia  Ref: Nelson's 20/e p 2227-2729, Ghai 8/e p 525
Primary aldosteronism encompasses disorders caused by excessive aldosterone secretion independent of the reninangiotensin
system. These disorders are characterized by hypertension and hypokalemia.
45. d. Hypokalemia  Ref: Nelson's 20/e p 2227-2729, Ghai 8/e p 525
Hyperkalemia is a feature of adrenal insufficiency due to mineralocorticoid insufficiency and not hypokalemia.
46. b. Testosterone and Mullerian inhibitory factor  Ref: Nelson's 20/e p 2227-2729, Ghai 8/e p 525
Testosterone and Mullerian inhibitory factor help in the development of male genitalia in a fetus.
47. d. Iatrogenic steroids  Ref: Nelson's 20/e p 2227-2729, Ghai 8/e p 525
The most common cause of Cushing's syndrome is prolonged exogenous administration of glucocorticoid hormones,
especially at the high doses used to treat lymphoproliferative disorders.
48. a. 21 hydroxylase deficiency  Ref: Nelson's 20/e p 2227-2729, Ghai 8/e p 525
The given clinical scenario of ambiguous genitalia, hyperpigmentation and electrolyte disturbances suggest a diagnosis of
CAH due to 21 hydroxylase deficiency.
Features Seen in Adrenal Enzyme Deficiency
17-α-hydroxylase 11-β hydroxylase 21 hydroxylase and 3β-HSD
Salt retention and hypertension Salt retention and hypertension Salt losing
Feminization of male Virilization of female Virilization of female
Precocious puberty in male Precocious puberty in male

49. a. Hyperglycemia  Ref: Nelson's 20/e p 2227-2729, Ghai 8/e p 525
Hypoglycemia is a feature of glucocorticoid deficiency in Addison's disease.
50. c. Calculation of aldosterone renin ratio  Ref: Nelson's 20/e p 2227-2729, Ghai 8/e p 525
The ratio of plasma aldosterone concentration to renin activity is always high, and this represents a cost-effective screening
test for primary aldosteronism (hyperaldosteronism).
51. a. 21 beta hydroxylase  Ref: Nelson's 20/e p 2714-2723, Ghai 8/e p 526-527
Most common cause of Congenital Adrenal Hyperplasia is 21 beta hydroxylase deficiency.

476 Section 3: Systemic Pediatrics

52. a. Adrenalectomy  Ref: Nelson's 20/e p 2714-2723, Ghai 8/e p 526-527


Review of Pediatrics & Neonatology

Adrenalectomy, in a case of pituitary adenoma, may lead to increased ACTH secretion by an unresected pituitary adenoma,
evidenced mainly by marked hyperpigmentation; this condition is termed Nelson syndrome.
53. c. 11-Beta Hydroxylase  Ref: Nelson's 20/e p 2714-2723, Ghai 8/e p 526-527
All the above enzyme deficiencies lead to CAH. Out of these, hypertension is seen in 11-β hydroxylase deficiency and 17-hydroxylase
deficiency. But 11-β hydroxylase deficiency is more common of the two.
54. a. 21-hydroxylase deficiency  Ref: Nelson's 20/e p 2714-2723, Ghai 8/e p 526-527
Classic 21-hydroxylase deficiency is the most common cause of ambiguous genitalia in a female newborn. Virilization is
usually apparent at birth in females, i.e. Pseudohermaphroditism of a female child.
55. a. 17 alpha hydroxylase deficiency  Ref: Nelson's 20/e p 2714-2723, Ghai 8/e p 526-527; Refer pretext for details
56. b. 21 Hydroxylase deficiency  Ref: Nelson's 20/e p 2714-2723, Ghai 8/e p 526-527
Presence of failure to thrive along with salt wasting leading to hyponatremia and hyperkalemia are compatible with a diagnosis
of Congenital adrenal hyperplasia due to 21 hydroxylase deficiency.
57. c. Hypoglycemia  Ref: Nelson's 20/e p 2714-2723, Ghai 8/e p 526-527
Hyperglycemia is a feature of Cushing's syndrome and not hypoglycemia.
58. b. Simple virilizing congenital adrenal hyperplasia  Ref: Nelson's 20/e p 2714-2723, Ghai 8/e p 526-527
As this child with ambiguous genitalia (with presence of Mullerian structures) never had features of shock and dehydration, she is
most probably suffering from 'simple virilizing' type of CAH due to 21 hydroxylase deficiency; Refer pretext for details.
59. b. Wolman's disease  Ref: Nelson's 20/e p 2714-2723, Ghai 8/e p 526-527
Wolman's disease

What is it? A rare lipid disorder caused by a deficiency of lysosomal acid lipase
Clinical features Presents in 1st few wks of life with failure to thrive, vomiting, abdominal distention, steatorrhea &
hepatosplenomegaly
Important finding Extensive bilateral calcifications of adrenal glands; Hyperlipidemia

60. d. Aldosterone  Ref: Nelson's 20/e p 2714-2723, Ghai 8/e p 526-527
The serum electrolyte abnormalities (hyperkalemia and hyponatremia) along with normal genitalia suggests aldosterone
deficiency or altered end-organ sensitivity to mineralocorticoids (Pseudohypoaldosteronism).
•• Both plasma renin and aldosterone are markedly elevated in Pseudohypoaldosteronism, but aldosterone level will be low in
hypoaldosteronism; so aldosterone level will help in diagnosing the illness
•• Levels of cortisol and ACTH are normal; If hypovolemia is severe, patients may develop prerenal azotemia.
61. d. Increase 11 deoxycortisol  Ref: Nelson's 20/e p 2714-2723, Ghai 8/e p 526-527
Presence of precocious puberty in a male child and hypertension (BP of 130/80 in a 5 year old child) suggests a diagnosis of
congenital adrenal Hyperplasia due to 11-Hydroxylase deficiency.
In 11 hydroxylase deficiency:
•• Excess DOC (Deoxycorticosterone) → excess mineralocorticoid activity → Hypertension
•• Excess androgens → Virilization of female, precocious puberty in males
•• Elevated precursors → Excess concentration of 11 deoxycortisol, deoxycorticosterone.
62. b. Epinephrine deficiency  Ref: Nelson's 20/e p 2714-2723, Ghai 8/e p 526-527
Insulin and counter-regulatory hormones in Hypoglycemia:
•• Hypoglycemic effects of insulin are opposed by the actions of counter-regulatory hormones like glucagon, growth hormone,
cortisol, and epinephrine, whose concentration in plasma increases as blood glucose falls; They act by:
– Activating glycogenolytic enzymes (glucagon, epinephrine)
– Inducing gluconeogenic enzymes (glucagon, cortisol)
– Inhibiting glucose uptake by muscle (epinephrine, growth hormone, cortisol)
– Mobilizing amino acids from muscle for gluconeogenesis (cortisol)
– Activating lipolysis, providing glycerol for gluconeogenesis & fatty acids for ketogenesis (epinephrine, cortisol, growth
hormone, glucagon); and
– Inhibiting insulin release and promoting growth hormone and glucagon secretion (epinephrine)
•• Congenital/acquired deficiency of any of these hormones will result in hypoglycemia, as the endogenous glucose production
cannot be mobilized to meet energy needs in the post-absorptive state, that is, 8-12 hr after meals or during fasting

Chapter 20: Pediatric Endo­crinology 477

63. a. Fetal placental steroid sulfatase deficiency  Ref: Nelson's 20/e p 2714-2723, Ghai 8/e p 526-527

Answers with Explanations


Discussing the options one by one,
a. Fetal placental steroid sulfatase deficiency has X-linked inheritance
Affected individuals with this disorder are mostly males; so, it is uncommon in females
b. Fetal placental aromatase deficiency
•• Aromatase catalyzes conversion of androgens into estrogens; So in aromatase deficiency, estrogen synthesis cannot occur;
•• So, DHEA-S produced by fetal adrenal glands cannot be converted to estrogen by the placenta, so is converted to testosterone
peripherally and results in virilization of both fetus and mother
•• Virilization manifests as pseudohermaphroditism in female infants, with hirsutism and acne in the mother
c. Congenital adrenal hyperplasia: It is a common cause of ambiguous genitalia in females
d. WNT4 gene mutation: It causes Müllerian aplasia and hyperandrogenism
•• Girls and women with this condition typically have an underdeveloped or absent uterus and do not menstruate
•• They may also have abnormally high levels of androgens, which cause acne and excessive facial hair

64. a. Congenital Adrenal Hyperplasia (CAH)  Ref: Nelson's 20/e p 2714-2723, Ghai 8/e p 526-527
The given clinical scenario of dehydration, hyperpigmentation of genitalia and laboratory features of hypoglycemia,
hyponatremia and hyperkalemia suggest a diagnosis of CAH due to 21 hydroxylase deficiency.
65. a. Hydrocortisone  Ref: Nelson's 20/e p 2714-2723, Ghai 8/e p 526-527
Hydrocortisone should be supplemented first if all pituitary function is lost.
66. c. Familial glucocorticoid deficiency  Ref: Nelson's 20/e p 2706-2707; Refer pretext of this chapter for details
Hypoglycemia with hyperpigmentation suggests a diagnosis of glucocorticoid deficiency.
67. c. Congenital adrenal hyperplasia  Ref: Nelson's 20/e p 2714-2723, Ghai 8/e p 526-527
The given laboratory findings along with dehydration in a male newborn with normal genitalia suggest a diagnosis of CAH,
which is most commonly due to 21 hydroxylase deficiency.
68. a. 21-hydroxylase deficiency  Ref: Nelson's 20/e p 2714-2723, Ghai 8/e p 526-527
69. d. Congenital adrenal hyperplasia  Ref: Nelson's 20/e p 2714-2723, Ghai 8/e p 526-527
70. a. 17-hydroxy progesterone  Ref: Nelson's 20/e p 2717-2726, Ghai 8/e p 526-527
71. b. 11-Deoxycortisol  Ref: Nelson's 20/e p 2714-2723, Ghai 8/e p 526-527
Hypertension and precocious puberty in males can be seen in 11 beta hydroxylase deficiency, which can be diagnosed by increased
11 deoxycortisol levels.
72. d. Abundant pubic hairs are present  Ref: Nelson's 20/e p2757; Refer pretext of this chapter for details
Sexual hairs are absent in Androgen insensitivity syndrome.
73. d. Hypokalemic alkalosis is seen  Ref: Nelson's 20/e p 2714-2723, Ghai 8/e p526-527
In congenital adrenal hyperplasia due to 21 alpha hydroxylase deficiency, there is hyperkalemia (not hypokalemia).
74. c. 11-β hydroxylase deficiency  Ref: Nelson's 20/e p 2714-2723, Ghai 8/e p 526-527
Hypertension and Virilization in females is seen in 11-β hydroxylase deficiency
75. a. 5α reductase deficiency, c. 17 Hydroxylase deficiency and d. Gonadal dysgenesis  Ref: Nelson's 20/e p 2714-2723
True Hermaphroditism: Both ovarian and testicular tissues are present either in the same (ovotestis) or opposite gonads. Male
Pseudohermaphroditism: Genotype is XY, but the external genitalia is female.
Causes of Male Pseudohermaphroditism
A. Defect in testicular differentiation B. Defect in testicular hormone synthesis
•• Deletion of short arm Y chromosome •• Leydig cell aplasia
•• XY pure/mixed gonadal dysgenesis •• CAH due to17 hydroxylase deficiency or 17–20 lyase deficiency or
3 beta-hydroxy steroid dehydrogenase deficiency
C. Defect in mullerian inhibiting hormone action D. Defect in androgen action
5-alpha reductase deficiency and Testicular feminization syndrome

76. c. 11-β hydroxylase deficiency  Ref: Nelson's 20/e p 2714-2723, Ghai 8/e p 526-527
Hypertension, hypokalemia and virilization suggest the diagnosis of 11-β hydroxylase deficiency.
77. d. Dexamethasone  Ref: Nelson's 20/e p 2714-2723, Ghai 8/e p 526-527
Recommendations for pregnancy at risk consists of administration of dexamethasome a steroid that readily crosses the placenta,
starting as soon as pregnancy (with a fetus suspected of having CAH) is diagnosed.
78. a. Loss of diurnal variation  Ref: Nelson's 20/e p 2724, Ghai 8/e p 526-527
In patients with Cushing's syndrome loss of circadian rhythm of corticosteroids is the earliest finding.
478 Section 3: Systemic Pediatrics

79. d. Pituitary adenomas  Ref: Nelson's 20/e p 2714-2724


Review of Pediatrics & Neonatology

Hyperpituitarism causes tall stature (Giants), with weak muscles; Most common cause of Hyperpituitarism is Pituitary
adenoma.
80. c. Cortisone  Ref: Nelson's 20/e p 2714-2724, Ghai 8/e p 526-527
Oral hydrocortisone with or without mineralocorticoid supplementation is the treatment of choice for CAH due to 21
hydroxylase deficiency.
81. a. Congenital adrenal hyperplasia  Ref: Nelson's 20/e p 2714-2724, Ghai 8/e p 526-527
This is a case with hypotension, hyponatremia and hyperkalemia, consistent with a diagnosis of Congenital adrenal hyperplasia.
82. a. 21 hydroxylase deficiency  Ref: Nelson's 20/e p 2714-2724, Ghai 8/e p 527
83. a. Hyponatremia, b. Hyperkalemia, c. Hypoglycemia  Ref: Nelson's 20/e p 2714-2724, Ghai 8/e p 526-527
84. d. As soon as pregnancy is diagnosed  Ref: Nelson's 20/e p 2714-2724, Ghai 8/e p 526-527
85. b. 17 hydroxylase  Ref: Nelson's 20/e p 2714-2423, Ghai 8/e p 526-527; Refer pretext of this chapter for details
In 17-hydroxylase deficiency: All pregnonolone is converted to mineralocorticoid. So, there will be:
•• Excess of mineralocorticoids → Salt retention and hypertension.
•• No androgens → Feminization of external male genitalia, while genitalia development will be normal in females
Coming back to this question, this neonate has: Hypertension (normal BP in a neonate = 75/50) and male pseudohermaphroditism,
both of which are features of 17-alpha hydroxylase deficiency.

 DIABETES MELLITUS
86. d. 5 years  Ref: Nelson 20/e p 2781; ADA recommendations 2017
Annual screening for albuminuria with a random spot urine sample for albumin-to-creatinine ratio should be considered once the
child has had type 1 diabetes for 5 years.
87. d. Diarrhea
88. a. Fasting blood glucose >126 mg/dL  Ref: Nelson's 20/e p 2760-2780, Ghai 8/e p 541-548
Fasting blood glucose > 12.6 mg/dL or 2 hr post prandia or rangdom glucose 200 mg/dL or HbA1c >6.5% in presence of symptoms
like polyuria and polydiapsia are diagnostic of diabetes mellitus.
89. d. Glulisine  Ref: Nelson's 20/e p 2760-2780, Ghai 8/e p 541-548; Refer pretext for details
Ultrashort acting Insulins include: Insulin Lispro, Glulisine and Aspart.
90. d. Maturity Onset Diabetes of the Young (MODY)  Ref: Nelson's 20/e p 2760-2780, Ghai 8/e p 541-548
MODY refers to forms of diabetes are associated with monogenic defects in pancreatic β-cell function.
91. b. 25–50%  Ref: Nelson's 20/e p 2760-2780, Ghai 8/e p 541-548
Basal-bolus regimen of Insulin:
•• Basal insulin glargine should be 25–30% of the total dose in toddlers and 40-50% in older children
•• Remaining portion of the total daily dose is provided as bolus insulin that is directed by both the carbohydrate content of the
meal as well as the preprandial glucose value
•• Glargine (basal insulin) may be given either as a single dose or every 12 hr in young children.
92. b. < 7.5%  Ref: Latest ISPAD guidelines 2015
As per the latest ISPAD guidelines 2015, Target HbA1c <7.5% for children of all ages.
93. b. More common in adults than in children  Ref: Nelson's 20/e p 2760-2780; Refer pretext for details
Nesidioblastosis or congenital hyperinsulinism is the most common cause of hyperinsulinemic hypoglycemia in neonates and
infancy;
Drugs used for treatment include: IV Glucose; Glucagon, Diazoxide, Octreotide and Nifedipine.
94. b. 1.75 gm/kg glucose  Ref: Nelson's 20/e p 2760-2780, Ghai 8/e p 541-548
Dose of glucose used in Oral Glucose Tolerance Test (OGTT) in adults → 75 gm and in children → 1.75 gm/kg of ideal weight, up
to a maximum of 75 gm.
95. c. After 2 years and d. After 5 years  Ref: Nelson's 20/e p 2760-2780, Ghai 8/e p 541-548
•• In IDDM, initial retinal examination should be done 5 years after the onset of disease in prepubertal children and after 2 yrs in
pubertal children; As the sight threatening retinopathy usually does not appear immediately after diagnosis
•• Thereafter 1–2 yearly examination is required.
96. b. Galactosemia  Ref: Nelson's 20/e p 2760-2780, Ghai 8/e p 541-548
Glucagon will not be able to release Glucose from an already damaged liver, as seen in Galactosemia.
97. b. Fibrocalcific pancreaticopathy  Ref: Nelson's 20/e p 2760-2780, Ghai 8/e p 541-548
“In tropical countries 50% of young onset diabetes is constituted by protein deficient diabetes mellitus; next in frequency is
Fibrocalculus Pancreatic diabetes”.
Chapter 20: Pediatric Endo­crinology 479

 DISORDERS OF PUBERTY

Answers with Explanations


98. b. Idiopathic  Ref: Nelson's 20/e p 2656
Although approximately 90% of girls have an idiopathic form, a structural central nervous system (CNS) abnormality can be
demonstrated in up to 75% of boys with central precocious puberty.
99. c. 8 years  Ref: Nelson's 20/e p 2656
Precocious puberty is defined by the onset of secondary sexual characteristics before the age of 8 yr in girls and 9 yr in boys.
100. d. GnRH agonists  Ref: Nelson's 20/e p 2658
Nelson latest ed says: “Long-acting formulations of GnRH agonists, which maintain fairly constant serum concentrations of the drug
for weeks or months, constitute the preparations of choice for treatment of central precocious puberty.”
101. d. Hypothalamic hamartoma  Ref: Nelson's 20/e p 2658-2659
Hypothalamic hamartomas are the most common brain lesion causing central precocious puberty.
102. a. Mayer Rokitansky-46XY  Ref: Nelson's 20/e p 2628-2631
Mayer-Rokitansky-Küster-Hauser syndrome (also called' Mullerian Agenesis') is characterized by the failure of development
of uterus and vagina, in women with normal ovarian function and normal external genitalia.
103. b. Vagina present  Ref: Nelson's 20/e p 2628, Ghai 8/e p 534
Discussing about the options one by one,
a. 46XX False Karyotype in Androgen insensitivity syndrome is 46, XY
b. Vagina present True A blind vagina is present
c. Androgen receptors working False Resistance or insensitivity of androgen receptors present
d. Amenorrhea with normal secondary sexual characters
False Pubic hair, which is a secondary sexual character is absent

104. c. 16 years  Ref: Nelson's 20/e p 2673, Ghai 8/e p 534


Delayed puberty is defined as absence of secondary sexual characters by 14 yrs age or absence of menarche by 16 years.

105. a. Both LH and FSH decreased  Ref: Nelson's 20/e p 2757, Ghai 8/e p 534
LH and FSH are increased in hypergonadotrophic hypogonadism.
106. b. 8 years  Ref: Nelson's 20/e p2656, Ghai 8/e p 534
Central precocious puberty is defined by onset of breast development before the age of 8 yrs in girls and by onset of testicular
development (vol 4 mL) before the age of 9 yrs in boys, as a result of early activation of hypothalamic-pituitary-gonadal axis.
107. d. Complete androgen insensitivity syndrome  Ref: Nelson's 20/e p 2757-2759, Ghai 8/e p 534
In the given scenario, a phenotypic female with tall stature, primary amenorrhea has well-developed breasts and absent pubic
hairs. All these point towards a diagnosis of Complete Androgen insensitivity syndrome.
108. d. Prader Willi syndrome  Ref: Nelson's 20/e p2673, Ghai 8/e p 534
Discussing about the options one by one
a. Hypothalamic hamartomas that secrete excess gonadotropin-releasing hormone are responsible for a significant proportion of
cases of precocious puberty
c. McCune-Albright syndrome is a cause of peripheral precocious puberty, where, cutaneous pigmentation and fibrous
dysplasia of the skeletal system are also seen
d. In Prader Willi syndrome, there is hypogonadism; So, sexual development is delayed
b. In hypothyroidism, though puberty is usually delayed in adolescents, pseudoprecocious puberty may be seen in younger
children, as a result of abnormally high TSH concentrations binding to FSH receptor with subsequent stimulation
109. a. SRY gene  Ref: Nelson's 20/e p 2514, Ghai 8/e p 532
110. b. XY genotype, female external genitalia  Ref: Nelson's 20/e p 2750-2759, Ghai 8/e p 532
Male pseudohermaphrodites are genetic males (45,XY), with testes, but their external genitalia & appearance are that of a female.
111. a. 8 year  Ref: Nelson's 20/e p 2514
112. b. Kallmann's syndrome  Ref: Nelson's 20/e p 2739, Ghai 8/e p 532
Among the given options, only Kallmann syndrome is a cause of hypogonadotrophic hypogonadism, where LH, FSH, as well as
testosterone levels will be low.
113. d. All of the above  Ref: Nelson's 20/e p 2514, Ghai 8/e p 532
Laurence-Moon Syndrome: It is an autosomal recessive disorder of hypogonadotrophic hypogonadism, which is characterized
by mental retardation, obesity, polydactyly, retinitis pigmentosa and hypogonadism.
114. c. Constitutional  Ref: Nelson's 20/e p 2657, Ghai 8/e p 532
Most common cause of delayed puberty is constitutional.
480 Section 3: Systemic Pediatrics

115. a. McCune-Albright's syndrome  Ref: Nelson's 20/e p 2660, Ghai 8/e p 532
Review of Pediatrics & Neonatology

McCune-Albright syndrome is characterized by:


• Cafe-au-lait spots • Polycystic fibrous dysplasia of skeleton
• Precocious puberty • Endocrinopathy-Hypothyroidism
116. d. Androgen insensitivity syndrome  Ref: Nelson's 20/e p 2727-2759, Ghai 8/e p 538
B/L Inguinal masses are probably gonads (testis). This along with absent uterus and phenotypic female without axillary and pubic
hairs suggests 'Androgen insensitivity syndrome'.
117. a. Chronic disease, b. Hypothyroidism, c. Turner's syndrome, d. Malabsorption syndrome  Ref: Nelson's 20/e p 2657
Refer pretext of this chapter for important causes of delayed puberty; In McCune-Albright syndrome, precocious puberty is seen and
not delayed puberty.
118. b. 46,XY  Ref: Nelson's 20/e p 2757, Ghai 8/e p 534
119. b. Mutation of the calcium-sensing receptor  Ref: Nelson's 20/e p 2757-2759; Refer pretext for details
This patient has increased serum calcium and parathormone levels, but urinary calcium excretion is reduced.
It is a close diagnosis between hyperparathyroidism & familial hypercalcemic hypocalciuria (mutation of calcium sensing receptor)
The clues that help in making a diagnosis here are: age of the patient and decrease in urinary calcium excretion.
120. b. Absent vagina  Ref: Nelson's 20/e p 2757-2759, Ghai 8/e p 534
Blind vagina is present in testicular feminizing syndrome (androgen insensitivity syndrome) and genotype is 46, XY with presence
of testis and absence of ovary & uterus.
121. b. 46,XY  Ref: Nelson's 20/e p 2757-2759, Ghai 8/e p 534
This baby girl with bilateral inguinal masses (testis) is probably suffering from Androgen insensitivity syndrome, where
karyotype is 46,XY.
122. a. Hypothyroidism c. McCune-Albright syndrome and e. Congenital adrenal hyperplasia  Ref: Nelson's 20/e p 2669, 2673
•• In children with hypothyroidism, sexual maturation may also be delayed or might not take place at all.
•• Adolescents with acquired hypothyroidism also typically have delayed puberty;
•• Younger children with hypothyroidism might present with galactorrhea or pseudoprecocious puberty.
•• Galactorrhea is a result of increased TRH stimulating prolactin secretion and precocious puberty (breast development and vaginal
bleeding in girls and macro-orchidism in boys), is thought to be the result of abnormally high TSH concentrations binding to the
FSH receptor with subsequent stimulation.

123. a. Testes are present, b. Female habitus, c. XY genotype  Ref: Nelson's 20/e p 2757-2759, Ghai 8/e p 534

 OBESITY
124. c. Bardet-Biedl syndrome  Ref: Nelson's 20/e p 307-315, Ghai 8/e p 528-529
The given pictures show obesity, hypogonadism, post axial polydactyly and pigmentary retinal changes, which are all suggestive of
Laurence-Moon-Bardet Biedl syndrome.
125. d. BMI > 95th percentile  Ref: Nelson's 20/e p 307-315, Ghai 8/e p 528-529
Obesity in children is defined as BMI > 95th percentile; Body mass index (BMI) is equal to weight/height2 (in kg/m2).
126. b. Leptin  Ref: Nelson's 20/e p 307-315, Ghai 8/e p 528-529
•• Leptin is a peptide produced by the ob gene; Its name derived from the Greek root leptos, meaning thin
•• Leptin is secreted by adipose cells and acts primarily through the hypothalamus
•• High leptin levels decrease food intake and increase energy expenditure, thereby preventing obesity

 DISORDERS OF CALCIUM METABOLISM

127. a. Familial isolated hypoparathyroidism is where a genetic mutation like SOX3 is the underlying cause, b. Calcium sensing
receptor (CaSR) mutation is another disorder of genetic origin, c. DiGeorge syndrome (22q11 deletion syndrome) is a
condition which includes hypoparathyroidism at birth  Ref: Nelson's 20/e p 2690-2692
Discussing about the options one by one,
a. True;
Causes of Congenital hypoparathyroidism include:
••Familial isolated hypoparathyroidism: Autosomal recessive (GCMB, PTH), dominant (CaSR), X-linked (SOX3)
••DiGeorge syndrome (TBX1)
••Kenny-Caffey syndrome 1 (short stature, medullary stenosis) (TBCE)
••Mitochondrial fatty acid disorders (Kearns-Sayre, Pearson, MELAS)
Chapter 20: Pediatric Endo­crinology 481

b. True; Patients with autosomal dominant hypoparathyroidism have an activating (gain-of-function) mutation of Ca2+ sensing

Answers with Explanations


receptor, forcing the receptor to an “on” state with subsequent depression of PTH secretion even during hypocalcemia. These
patients have hypercalciuria and hypocalcemia.
c. True; Aplasia or hypoplasia of the parathyroid glands is often associated with the DiGeorge/velocardiofacial (22q11 deletion)
syndrome
d. False; Hypocalcemia is a salient feature of neonatal hypoparathyroidism;
e. False; Seizures are usually seen in neonatal hypocalcemia.
128. b. Hyperparathyroidism  Ref: Nelson's 20/e p 2695-2697, Ghai 8/e p 521-524
Brown tumor (also known as osteitis fibrosa cystica) is one of the manifestations of hyperparathyroidism
Osteitis fibrosa may cause bone pain or tenderness. There may be fractures in the arms, legs, or spine.
129. c. Decreased serum Ca++ and Increased serum PTH  Ref: Nelson's 20/e p 2695-2697, Ghai 8/e p 521-524
Pseudohypoparathyroidism (PHP)
•• Parathyroid glands are normal or hyperplastic and they can synthesize and secrete parathyroid hormone (PTH)
•• Serum levels of immunoreactive PTH are elevated even when the patient is hypocalcemic normocalcemic
•• Neither endogenous nor administered PTH raises the serum levels of calcium or lowers the levels of phosphorus.
130. d. Mutations of the calcium-sensing receptor (CASR)  Ref: Nelson's 20/e p 331-336; Refer pretext for details
131. c. Reduced calcium intake  Ref: Nelson's 20/e p 331-336; Refer pretext of this chapter for details
Dietary calcium restriction is not recommended in Idiopathic Hypercalciuria, because of obligate requirement for growth.
132. c. Calcium gluconate  Ref: Nelson's 20/e p 331-336, Ghai 8/e p 113-115
The baby presenting with tetany is hypocalcemic, so must be treated immediately with calcium gluconate IV.
Chapter 21
Pediatric Neurology
 NEURAL TUBE DEFECTS (NTD)
Question 1
What does the following demon- •• Failure of proper closure of neural tube & covering mesoderm and ectoderm.
strate in a full term neonate? •• Neural tube defects are the most common congenital anomaly of CNS.
•• Primary Neural tube defects have multifactorial inheritance
•• Maternal risk factors include alcohol, radiation exposure, IDDM, valproate, carbamazepine,
zinc and folate deficiency

Different Types of NTDs


Spina bifida occulta, meningocele, encephalocele, myelomeningocele, anencephaly,
caudal regression syndrome, dermal sinus, tethered cord, syringomyelia, diastematomyelia, lipoma
involving the conus medullaris and/or filum terminale, iniencephaly.
M
a. Normal tone
b. Hypotonia
c. Hypertonia
d. Head control

Spina Bifida Occulta M

•• It is a common anomaly consisting of a midline defect of the vertebral bodies without


protrusion of the spinal cord or meninges
•• Most patients are asymptomatic & condition is usually of no consequence.

Craniorachischisis
Question 2 M •• A rare malformation characterized by skull & spinal bone defects which leaves the brain &
What is the name of the congenital nerves in spine exposed; often other defects such as imperforate anus present.
anomaly this child is suffering
from? Meningocele M

•• It is formed when the meninges herniate through a defect in the posterior vertebral arches or
the anterior sacrum; Spinal cord is usually normal
•• A fluctuant midline mass that might transilluminate
•• If well covered with skin, it does not pose immediate threat
•• In asymptomatic children with normal neurologic findings and full-thickness skin covering the
meningocele, surgery may be delayed or sometimes not performed.

Myelomeningocele M

•• Most commonly involves lumbosacral region (75% of cases)


a. Encephalocele •• Appearance: a sac-like cystic structure with remnants of neural tissue visible beneath the
b. Anencephaly membrane, which occasionally ruptures and leaks CSF or an exposed flat neural placode
c. Hydranencephaly •• Hydrocephalus with a type II Chiari malformation develops in 80% of patients with MMC
d. Diastematomyelia
•• The lower the deformity in the neuraxis, the less likely is the risk of hydrocephalus.
Chapter 21: Pediatric Neurology 483

Anencephaly

Section 3: Systemic Pediatrics


M
Question 3 M
•• A large defect of the calvarium, meninges & scalp, associated with a rudimentary brain, which Identify the congenital defect seen
results from failure of closure of the rostral neuropore in this baby:

Iniencephaly
•• Accentuated retroflexion of head, varying degree of defect in occiput and upper spine,
and nearly absent neck
•• The defect of occiput can vary from acrania to minimal defect.
•• Tools for prenatal diagnosis of NTDs include detailed ultrasound examinations and measurements a. Craniorachisis
of MSAFP, amniotic fluid α-fetoprotein, and amniotic fluid acetylcholinesterase (AChE) b. Meningocele
c. Myelomeningocele
d. Spina bifida occulta
Prevention of Neural Tube Defects M

•• To prevent NTDs, it is recommended that all women of childbearing age and who are
capable of becoming pregnant take 0.4 mg (400 mg) of folic acid daily Question 4
•• If a pregnancy is planned in high risk women (previously affected child), supplement 4 mg of Identify the abnormality in this
folic acid daily, beginning 1 month before the time of planned conception. baby:
•• Risk of recurrence after 1 affected child is 3-4% & with 2 prior affected children, it is 10%.

Treatment
•• Surgery of the defect and a Ventriculoperitoneal shunt (if associated with hydrocephalus)
•• Open lesions draining CSF should be closed within 24 hours
•• If the lesion is infected, IV antibiotics are given and surgery is done only when three 24 hours
a. Anencephaly
CSF cultures are sterile.
b. Craniorachischisis
c. Spina bifida occulta
 FOREBRAIN ANOMALIES d. Iniencephaly

•• Lissencephaly: reduction in the number of gyri


•• Agyria: no gyral pattern
•• Polymicrogyria: small, unusually numerous, irregularly formed cerebral convolutions
•• Neuronal heterotopias: presence of collections of neurons in inappropriate locations
•• Holoprosencephaly: incomplete midline separation of cerebral hemispheres
•• Arhinencephaly: absence of the olfactory cranial nerves and related structures
•• Agenesis of the corpus callosum: absence of the white matter bundles that carry cortical
projections from one hemisphere to the other

Holoprosencephaly
•• Failure of the brain to divide into right and left hemispheres
•• Severity of facial defects corresponds to severity of brain defect.
•• Signs and symptoms include intellectual disability, seizures, pituitary dysfunction, short stature  Question 5 M
•• Associated with trisomy 13 (MC), trisomy 18, CHARGE syndrome, Smith-Lemli-Opitz
A newborn baby has severe facial
syndrome, Rubinstein–Taybi syndrome
abnormalities, as shown below.
3 main types (from most to least severe): What could be the underlying CNS
abnormality in this baby?
Alobar Semi-lobar Lobar
Complete failure of the brain Left side of brain is fused to Cerebral hemispheres are fused in
to divide into right and left the right side in frontal & the frontal cortex
hemispheres parietal lobe areas

a. Lissencephaly
b. Dandy walker malformation
c. Holoprosencephaly
d. Iniencephaly
484 Section 3: Systemic Pediatrics

 HYDROCEPHALUS
Review of Pediatrics and Neonatology

Question 6
Identify the abnormality seen in
Caused by
the MRI brain of a 12 year old child
presenting with headache. Impaired circulation &/or absorption of CSF, rarely by increased production

Types of Hydrocephalus
•• Communicating: No block in the ventricular system
•• Non-communicating (obstructive): Block present at any level in the ventricular system.

Clinical Manifestations of Hydrocephalus


a. Dandy Walker malformation
b. Arnold-Chiari malformation •• Depend on age at onset, the nature of the lesion causing obstruction, and the duration and rate
c. Vein of Galen malformation of increase of intracranial pressure (ICP)
d. Encephalocele •• In an infant, an accelerated rate of enlargement of the head, wide open and bulging
anterior fontanel and dilated scalp veins are seen
•• Macewen sign is cracked pot sound produced on Percussion of skull in raised intracranial
tension
•• Setting-sun eye sign due to impingement of dilated suprapineal recess on brainstem tectum
•• Brisk tendon reflexes, spasticity, clonus, Babinski sign and abducens nerve palsies are
Mnemonic M common owing to stretching and disruption of the corticospinal fibers originating from the leg
region of the motor cortex
Causes of Hydrocephalus
•• Irritability, lethargy, poor appetite, vomiting (headache & Papilledema in older patients).
Communicating “C-A-M-P”
•• Choroid plexus papilloma
•• AchondroplasiaQ Treatment
•• Meningeal malignancy •• Ventriculoperitoneal (VP) shunt is usually done for treating hydrocephalus
•• Post-hemorrhagic
Non-Communicating “M-A-A-A-D H-I-V” •• Medical management usually acetazolamide is given if surgery is not indicated.
•• Mass lesions
•• Aqueductal stenosisQ Arnold-Chiari Malformation
•• Arnold Chiari malformationQ
•• Abscess Type I Type II
•• Dandy-Walker malformationQ
•• Hematoma •• Displacement of cerebellar tonsils into the •• Elongation of 4th ventricle
•• Infections (Toxoplasmosis, Mumps, spinal canal •• Displacement of inferior vermis, pons and
Neurocysticercosis) •• Usually not associated with hydrocephalus medulla into the cervical canal
•• Vein of Galen malformation •• Presents usually during adolescence with •• Usually associated with hydrocephalus and
recurrent headache, neck pain, lower extremity myelo meningocele
spasticity, urinary frequency

Question 7 Dandy Walker Malformation


What is your diagnosis in this •• Cystic expansion of 4th ventricle and midline cerebellar hypoplasia.
infant who was found to have •• 90% patients have hydrocephalus
hydrocephalus? •• Papilledema, pyramidal tract signs & cranial nerve palsies may occur along with symptoms
of raised ICP.

Vein of Galen Malformation


•• Neonates with vein of Galen malformations may present with heart failure or progressive
macrocephaly
•• A vein of Galen malformation can obstruct the flow of CSF
•• A cranial bruit may be audible

 PSEUDOTUMOR CEREBRI
a. Dandy walker malformation •• Also known as Idiopathic intracranial hypertension
b. Chiari malformation •• Cerebrospinal fluid (CSF) cell count and protein content is normal
c. Vein of Galen malformation •• Normal to slightly decreased ventricular size and normal ventricular position
d. Holoprosencephaly
•• Papilledema is universally present in children old enough to have a closed fontane.
Chapter 21: Pediatric Neurology 485

Etiology of Childhood Pseudotumor Cerebri

Section 3: Systemic Pediatrics


High Yield Points
Hematologic Nutritional Inheritance of Neurofibromatosis is
•• Wiskott-Aldrich syndrome •• Hypovitaminosis A/Vit A intoxication autosomal dominant. 2 types:
•• Iron-deficiency anemia •• Vitamin D-dependent Rickets •• Type NF1 (Von Recklinghausen
•• Aplastic anemia/Fanconi anemia •• Obesity disease): mutation on chr 17
•• Sickle cell disease •• Type NF2 (Central neurofibromatosis):
Infections Connective Tissue Disorders mutation on chr 22

•• Acute sinusitis/Mastoiditis/Otitis media •• Antiphospholipid antibody syndrome


•• Tonsillitis •• Systemic lupus erythematosus
•• Measles/Roseola/Varicella •• Behcet disease
Drugs Endocrine
•• Tetracyclines •• Hypothyroidism
•• Sulfonamides •• Hypo/hyperparathyroidism
•• Nalidixic acid •• CAH; Addison disease Question 8 M

•• Fluoroquinolones •• Growth hormone therapy A child has multiple hyperpig­


•• Corticosteroid therapy/withdrawal mented spots all over the body &
Others
•• Nitrofurantoin some abnormal lesions involving
•• Cyclosporine •• Dural sinus thrombosis the eyes as shown below. What is
•• Phenytoin •• Head trauma the disease this child is suffering
•• Turner syndrome from?

 NEUROCUTANEOUS SYNDROMES

A heterogeneous group of disorders characterized by abnormalities of skin & CNS.

Neurofibromatosis
NF1: 2 or more of the following should be present:
•• 6 or more café-au-lait spots, each >5 mm, before puberty or >15 mm in older persons
•• 2 or more neurofibromas or one plexiform neuroma
•• Freckling in axillary or inguinal regions
•• Optic glioma
a. Tuberous sclerosis
•• 2 or more lisch nodules, dysplasia of the sphenoid bone or thinning of the cortex of long
b. Neurofibromatosis
bones with or without pseudarthrosis c. Sturge-Weber syndrome
•• A first degree relative with NF1 d. Von Hippel-Lindau disease
NF2: Presence of bilateral auditory neuroma, unilateral auditory neuroma along with a first degree
relative with meningioma, Schwannoma or Juvenile posterior subcapsular lenticular opacity.

Tuberous Sclerosis (TSC)


•• It is inherited as an autosomal dominant trait with 2 identified genetic loci:
–– TSC1 gene, located on chromosome 9q34 (encoding protein hamartin),
–– TSC2 gene, located on chromosome 16q13 (encoding protein tuberin)
•• Cardinal features are skin lesions, convulsions and mental retardation
•• Skin lesions – adenoma sebaceum, shagreen patches, subungual fibromas, hypomelanotic
(ash leaf) macules, Confetti skin lesions Question 9
•• Retinal hamartomas, Cardiac rhabdomyoma, Renal angiomyolipoma may be present What is the name of this skin lesion
•• It may present during infancy with infantile spasms & hypsarrhythmic EEG pattern seen in Tuberous sclerosis?
•• VigabatrinQ is the drugs of choice for Infantile spasms in Tuberous sclerosis.

Sturge-Weber Syndrome (SWS)


•• Angiomas involving the leptomeninges & the skin of the face, typically in the ophthalmic
(V1) and maxillary (V2) distributions of the trigeminal nerve
•• The hallmark of SWS is a facial cutaneous venous dilation, also referred to as a nevus a. Adenoma sebaceum
flammeus or port-wine stain b. Ash leaf macules
•• Contralateral focal seizures, calcification of the cerebral cortex and glaucoma on the c. Confetti skin lesions
d. Shagreen patches
same side as the skin lesions are seen.
486 Section 3: Systemic Pediatrics

Von Hippel-Lindau (VHL) Disease


Review of Pediatrics and Neonatology

High Yield Points


•• Retinal and cerebellar hamangioblastoma seen
All neurocutaneous disorders except for
Sturge-Weber syndrome. •• Spinal cord angiomas and cystic tumors of pancreas, kidneys and epididymis
•• Patients may show nystagmus, ataxia and signs of increased Intracranial pressure.

Question 10 Ataxia Telangiectasia


What is the underlying abnor­mality •• It is an autosomal recessive inherited disease (chr 11q)
in this child with seizures? •• Progressive cerebellar ataxia, oculo cutaneous telangiectasias, choreoathetosis, pulmonary
and sinus infections, immuno deficiency and lymphoreticular malignancies
• a-Fetoprotein is almost universally elevated
•• Decreased serum IgA is found in around 75% of patients.

 IMPORTANT IMAGING FINDINGS IN PEDIATRIC CNS DISORDERS

a. Tuberous sclerosis
b. Sturge-Weber syndrome
c. McCune Albright syndrome
d. Neurofibromatosis

“Molar tooth sign” “Eye of tiger sign”


Joubert syndrome Hallervorden-Spatz syndrome or pantothenate
kinase-associated neurodegeneration (PKAN)

High Yield Points


Preferred Imaging Procedures in Neu-
rologic Diseases
•• Intraparenchymal hemorrhage: CT
if <24 hr; MRI if >24 hr
“Tram-track sign” “Puff of smoke appearance”
•• Hypoxic–ischemic brain injury:
Sturge-Weber syndrome Moyamoya disease
Ultrasound in infants; If –ve, consider
MRI
•• Metabolic disorders: MRI, parti-
cularly T2-weighted & FLAIR images  FEBRILE SEIZURES M
•• Head trauma: CT without contrast
initially •• Febrile seizures are seizures that occur between the age of 6 and 60 months with a
•• Epilepsy brain tumor: MRI with and temperature of 38°C or higher, that are not the result of CNS infection or any metabolic
without gadolinium imbalance, and that occur in the absence of a history of prior afebrile seizures

Types of Febrile Seizures

High Yield Points Simple Complex

•• Febrile seizures occur in 2% to 5% of Generalized seizures Focal


all children Lasting <15 minutes Prolonged
•• Most common cause of seizure in
children < 5 yrs Febrile seizures.Q No recurrence within 24 hrs Recurrent
Chapter 21: Pediatric Neurology 487

Risk Factors for

Section 3: Systemic Pediatrics


High Yield Points
Recurrence of Febrile Seizures & Occurence of Subsequent Epilepsy
•• Febrile seizures recur in 30% of
Major Occurrence of Epilepsy Risk those experiencing a first episode &
in 50% after 2 or more episodes
•• Age < 1 year Simple febrile seizures 1%Q •• Febrile seizures recur in 50% of
•• Duration of fever < 24 hr infants <1 yr old at febrile seizure
•• Fever 38–39°C Neurodevelopmental abnormalities 33% onset.
•• 15% of children with epilepsy have
Focal complex febrile seizure 29%
had febrile seizures
Minor Family history of epilepsy 18% •• 2-7% of children with febrile seizures
develop epilepsy later in life
•• Family history of febrile seizures Fever <1hr before febrile seizure 11%
•• Family history of epilepsy
•• Complex febrile seizure •  Low Na Complex febrile seizure (any type) 6%
• Day care •  Male gender High Yield Points
Recurrent febrile seizure 4%
•• Recurrent simple febrile seizures
Investigations do not damage the brain
•• Epilepsy syndromes that start with
•• Lumbar puncture is recommended in children < 12 months of age after first febrile seizure febrile seizures are generalized
to rule out meningitis epilepsy with febrile seizures plus
•• EEG and neuroimaging is indicated in children with complex febrile seizures and those (GEFS+) & severe myoclonic epilepsy
with febrile status epilepticus. of infancy (Dravet syndrome)

Treatment and Prophylaxis of Febrile Seizures M

•• Home management: for seizure > 2 min


–– Per Rectal diazepam or buccal/nasal midazolam
•• In hospital:
–– Ongoing seizure: IV midazolam or lorazepam
–– No response: full status epilepticus protocol
•• Intermittent prophylaxis: Oral Diazepam or Clobazam:Q 1 mg/Kg/day in 2–3 divided
doses for 1st 3 days of fever
•• Antipyretics: Do not reduce the risk of recurrent seizures.

 EPILEPSY
Epilepsy is a disorder characterized by occurrence of at least 1 unprovoked epileptic seizure
with either a second such seizure or enough EEG and clinical information suggesting predisposition
to develop recurrences.

Types of Epilepsy
•• Symptomatic epilepsy refers to an epilepsy syndrome caused by an underlying brain
disorder e.g., secondary to tuberous sclerosis
•• Structural/metabolic epilepsy: Seizures caused by a distinct structural or metabolic entity
that increases the risk for seizures
•• Cryptogenic epilepsy or presumed symptomatic epilepsy refer to an epilepsy syndrome
in which there is a presumed underlying brain disorder causing the epilepsy and affecting
neurologic function, but the underlying disorder is not known.

Important Epilepsy Syndromes with Good Prognosis


Epilepsy syndrome Age of onset
Benign infantile seizures Infant High Yield Points
Benign neonatal seizures Neonate
•• West syndrome is a triad of Infantile
Benign familial neonatal convulsions Early infancy spasms, mental retardation (ID) &
Benign childhood epilepsy with centrotemporal spikes 3–13 yrs Hypsarrhythmia
•• Treatment of choice for West
Childhood absence epilepsy 5–6 yrs syndrome is ACTH (Adrenocortico
Juvenile myoclonic epilepsy 12–18 yrs trophic hormone)
488 Section 3: Systemic Pediatrics

Important Epilepsy Syndromes with Poor Prognosis


Review of Pediatrics and Neonatology

High Yield Points


•• 30% of patients who have a first Epilepsy syndrome Usual age of onset
afebrile seizure have later epilepsy Ohtahara syndrome Infancy
•• Generalized epilepsy with febrile
seizures plus is a disorder of sodium Rasmussen syndrome 6–12 yrs
channel Dravet syndrome Infancy
•• Benign familial neonatal seizures
is a disorder of Potassium voltage- Lennox-Gastaut syndrome 3–10 yrs
gated channel
•• Myoclonus is a “shock-like” Important Terminologies:
contraction of a muscle of <50 msec
that is often repeated •• Acute symptomatic seizures occur secondary to an acute problem affecting brain excitability
such as electrolyte imbalance.
•• An unprovoked seizure is one that is not an acute symptomatic seizure.
•• Remote symptomatic seizure is one that is considered to be secondary to a distant brain
injury, such as an old stroke.
•• Reflex seizures are usually precipitated by a sensory stimulus such as flashing lights

Treatment of Status Epilepticus in Children


Time Drug treatment
0 minute IV Access available: Inj Lorazepam-0.1 mg/kg/IV (max 4 mg) OR Inj
Midazolam 0.15–0.2 mg/kg/IV (Max 5 mg)
IV access not available: Buccal Midazolam 0.3 mg/kg (Max 5 mg) OR PR
Diazepam 0.5 mg/kg (Max 10 mg)
5 minutes Inj Lorazepam-0.1 mg/kg/IV (max 4 mg) OR Inj Midzolam-0.5–0.2 mg/
kg/IV (Max 5 mg)
10 minutes IV Phenytoin* 20 mg/kg (Max: 100 mg) in NS @ 1 mg/kg/min (Max 50
mg per min), OR Inj Fosphenytoin 20 mg PE/kg, Rate: 3 mg PE/kg, if no
response to initial dose
Refractory SE-even after IV Valproate 20–30 mg/kg-IV @ max 6 mg/kg/minute OR
10 min of phenytoin/ IV Phenobarbitone 20 mg/kg in NS @ 1.5 mg/kg/min; Repeat 10 mg/kg
fosphenytoin administration if no response to initial dose OR
IV Levetiracetam (If Liver disease/Metabolic disease/coagulopathy/on
chemotherapy)- 20–30 mg/kg @ 5 min infusier

If the Child is Already on Anti Epileptic Drugs (AEDs)

•• Phenytoin ≤6 mg/kg/day •• OR If the child has missed few doses of AED


•• Phenobarbitone ≤4 mg/kg/day •• OR child has vomiting/diarrhea
•• Valproate ≤30 mg/kg/day

Give half the loading dose of the same antiepileptic drug.

Question 11 M
 CEREBRAL PALSY (CP) M
Identify the disease this child is
suffering from: A group of disorders of movement or posture, causing activity limitations, due to non-progressive
disturbances that occurred in the developing fetal or infant brain.

Clinicopathological Correlation of Cerebral Palsy


CP Subtype Pathology
Spastic Diplegia Periventricular Leukomalacia (PVL)
Periventricular Hemorrhagic infract (PVHI)
Spastic Quadriplegia Multicystic encephalopathy
Parasagittal brain injury
a. Spastic Diplegia Spastic Hemiplegia Middle cerebral artery (MCA) territory infarction
b. Spastic Quadriplegia
c. Spastic Hemiplegia Dyskinetic cerebral palsy Basal ganglionQ (involved in kernicterus)
d. Dyskinetic Cerebral palsy Hypotonic cerebral palsy Cerebellar lesion
Chapter 21: Pediatric Neurology 489

Management: Holistic Approach

Section 3: Systemic Pediatrics


•• Physiotherapy, Occupational therapy, Control of seizures
•• Control of spasticity – Baclofen, Botulinum toxin, Tizanidine
•• Management of Contractures: Surgical procedures and orthopedic support.

 CNS INFECTIONS
ACUTE BACTERIAL MENINGITIS M

•• Acute bacterial meningitis is commoner in neonates and infants than in older children.

Pathogenesis of Acute Bacterial Meningitis


•• Infection spreads hematogenously to meninges from bacterial sepsis or distant foci
•• Recurrent meningitis maybe associated with pilonidal sinus, CSF rhinorrhea, traumatic
lesions of cribriform plate/ethmoidal sinus or immune deficiency disorders
•• Pneumococcal meningitis usually follows otitis media, sinusitis, pneumonia or head injury
•• Neonatal staphylococcal meningitis is often associated with umbilical sepsis, pyoderma or
septicemia
•• Subdural effusion should be suspected in infants in whom focal neurological signs and
fever persist after the CSF cleans biochemically and microbiologically.

Clinical Features High Yield Points


•• Seizures (focal or generalized) due to cerebritis, infarction, or electrolyte disturbances
Most Common Cause of Acute
•• Focal neurological signs usually are due to vascular occlusion Bacterial Meningitis in world
•• Cranial neuropathies e.g. ocular, facial may also be due to focal inflammation. Age group Organism
Neonates Gr B Strepto,
Diagnosis: See section on 'lumbar puncture & CSF study' E. coli, Listeria
Infants & S. pneumoniae,
Treatment Children N. meningitidis,
H. influenzae
Empirical therapy: 3rd generation cephalosporin IV × 10–14 days.
Adolescents & S. pneumonia,
Corticosteroids reduce residual neurological complications in H. influenzae meningitis. Young Adults N. meningitidis

Specific Therapy
•• Meningococcal meningitis: Penicillin or Ceftriaxone
•• Pneumococcal meningitis – Ceftriaxone or penicillin, (Vancomycin, If resistant)
•• H. influenzae: Ceftriaxone or chloramphenicol
•• Listeria monocytogenes – Ampicillin or trimethoprim Sulfamethoxazole
•• Gram negative rods e.g. E. coli – Third generation cephalosporins
•• Pseudomonas – Ceftazidime and aminoglycoside. Alternatives: meropenem/cefepime
•• Staphylococcal meningitis – Vancomycin is the drug of choice. Rifampicin may be added.

Complications
Subdural effusion (10-30%), SIADH, DIC (e.g. purpura fulminans in meningococcal meningitis).

 LUMBAR PUNCTURE AND CSF STUDY IN CHILDREN

Normal CSF Findings


Opening pressures are:
•• 90-120 mm H2O in newborns
•• 60-180 mm H2O in young children &
•• 12-120 mm H2O in older children
•• CSF Cell Count: up to 5–15 WBCs/mm3 in newborns; No RBCs
•• CSF Protein is 10-40 mg/dL and upto 120 mg/dL in a neonate
•• CSF protein falls to the normal childhood range by 3 month of age
•• CSF Glucose is 60% of blood glucose

Important CSF Abnormalities


•• Elevation of CSF IgG is seen in subacute sclerosing panencephalitis (SSPE), postinfectious
encephalomyelitis & multiple sclerosis
490 Section 3: Systemic Pediatrics

•• Hypoglycorrhachia is seen in bacterial, tubercular and fungal meningitis, neoplastic


Review of Pediatrics and Neonatology

involvement of meninges, subarachnoid hemorrhage & disorders involving GLUT1


•• Most common cause of an elevated CSF opening pressure is an agitated patient
•• Polymorphonuclear cells are always abnormal in a child, but 1-2/mm3 may be present in a
normal neonate
•• Xanthochromia may be absent in bleeds <12 hr old
•• Xanthochromia can occur in subarachnoid hemorrhage, hyperbilirubinemia & carotenemia

High Yield Points CSF Findings in CNS Infections


If disc edema or focal findings suggest Bacterial Tuberculous Viral
Parameters Normal values
a mass lesion, a head CT should be Meningitis Meningitis Meningitis
obtained before proceeding with Pressure 50–180 mm water Raised Raised Raised
lumbar puncture to prevent uncal or
Gross appearance Clear and colorless TurbidQ Clear (may clot)Q ClearQ
cerebellar herniation
Protein 20–50 mg/dL High (> 100) Very HighQ Slightly high
Glucose 40–70 mg/dL Very lowQ Low Normal
Chloride 110–125 mEq/L Low Very lowQ Normal
Cells < 5/microliter Neutrophils 100–1000 10–100
1000-100000/uL mononuclear/uL mononuclear/uL

In partially treated meningitis, CSF maybe clear with predominant lymphocytes, culture is
usually sterile.

Contraindications to performing a lumbar puncture


•• Suspected mass lesion of brain
•• Suspected mass lesion of spinal cord
•• Symptoms and signs of impending cerebral herniation
•• Critical illness
•• Skin infection at site of LP
•• Thrombocytopenia with a platelet count <20 ×109/L

 ENCEPHALITIS
It is defined as an inflammatory process of the CNS with dysfunction of the brain.

Etiology
Viral Non-viral
RNA viruses: Mumps, Measles, Rubella, Enteroviruses Rickettsial
DNA viruses: Herpes simplex, CMV, Epstein Barr, Fungal (Cryptococcus)
Arthropod borne: Japanese B, West Nile virus Protozoa (T. Gondi)
Others: HIV, Rabies, Dengue Bacteria (Tuberculous, Listeria)
Slow viral infection, e.g. prion

High Yield Points Clinical Features


•• Subdural effusion is a usual •• Sudden onset illness with high fever, confusion, vomiting, loss of consciousness, often
complication of meningitis due to associated with seizures
Pneumococcus or H. influenzae
•• Usually there are features of raised intracranial pressure with evidence of brainstem
dysfunction.

Diagnosis: clinical, supported by


•• CSF – Moderate cellularity, mostly mononuclear cells with moderately elevated protein and
generally normal sugar except in Mumps in which case it may be reduced
•• EEG – Diffuse slow wave changes usually without focal changes except in herpes encephalitis
•• CT or MRI may show brain parenchymal swelling.

HERPES SIMPLEX ENCEPHALITIS


•• HSV 1 is the usual causative organism
•• In addition to seizures, headache, confusion, focal neurological deficits are common
Chapter 21: Pediatric Neurology 491

•• Localizing signs (focal seizures/deficits focal EEG changes), presence of RBCs in the CSF,

Section 3: Systemic Pediatrics


focal involvement of temporal lobeQ on CT/MRI are important diagnostic clues
High Yield Points
•• Diagnosis can be established by CSF culture or PCR Enteroviruses are the most common
•• Drug of choice is Acyclovir (30 mg/kg/d) in 3 divided doses for 10 days cause of viral meningoencephalitis in
•• Prognosis is variable about half of patients recover after timely therapy. the world

 TUBERCULOUS MENINGITIS (TBM)


•• TBM is a serious complication of childhood tuberculosis Question 12
•• TBM may occur at any age, but is most common between 6 months and 4 years of age. A 5-year child presented with fever
for 15 days with early morning
Pathogenesis headaches vomiting. His NCCT &
CECT head showed the following
•• There is usually a focus of primary infection, that reaches meninges by hematogenous
picture. What is the diagnosis?
route
•• Gelatinous exudate infiltrates the corticomeningeal blood vessels, producing inflammation,
obstruction and subsequent infarction of cerebral cortex
•• Exudate also interferes with normal flow of CSF at the level of the basilar cisterns, leading
to a communicating hydrocephalus
•• Combination of vasculitis, infarction, cerebral edema, and hydrocephalus results in
severe CNS damage.

Clinical Course of TBM is Described in Three Stages a. Herpes encephalitis


b. Neurocysticercosis
•• Prodromal stage: Non-specific symptoms like low grade fever, loss of appetite, irritability c. Tubercular meningitis
headache, photophobia d. Tuberous sclerosis
•• Stage of meningitis: Neck rigidity, seizures, neurological deficits like hemiplegia
•• Stage of coma: Loss of consciousness, episodic decerebration with features of herniation e.g.
unequal dilated pupils, altered respiratory pattern.

Diagnosis
•• CSF study: Cells between 10–500/mm3, neutrophils may predominate in early stages,
protein usually between 100–3000 mg/dL, and sugar usually < 50% of blood sugar
•• Demonstration of acid fast bacilli, Bactec culture and PCR are useful tests on CSF
•• CT Scan: Basal exudates, inflammatory granulomas, infarcts and hydrocephalus (both
communicating and less commonly obstructive type) may be seen. High Yield Points

Treatment •• Brainstem is often the site of


greatest involvement in CNS TB
•• Antitubercular drugs given for at least 12 months •• TBM complicates 0.3% of untreated
•• Corticosteroids decrease mortality rates & long-term neurologic sequelae by reducing tuberculosis infections in children
vasculitis, inflammation, and intracranial pressure •• Tuberculoma is a tumor-like
•• Ventriculoperitoneal shunt may be required in cases with increasing hydrocephalus. mass resulting from aggregation of
caseous tubercles
WILSON DISEASE: Refer chapter on 'Inborn errors of metabolism'
BRAIN TUMORS: Refer chapter on 'Tumors of childhood'

 TRAUMATIC BRAIN INJURY (TBI) & RAISED INTRACRANIAL PRESSURE (ICP)


Pathogenesis
•• Primary injury from the impact produces irreversible tissue disruption.
•• Secondary injury can occur due to edema, apoptosis, secondary axotomy or due to
disruption of autoregulation of cerebral blood flow.

Clinical Manifestations
•• The hallmark of severe TBI is coma (GCS score 3-8).
•• Increased ICP with impending herniation may be heralded by new onset/worsening headache,
depressed consciousness, vital sign changes (hypertension, bradycardia, irregular respiration)
& signs of 6th (lateral rectus palsy) or 3rd (anisocoria/ptosis) cranial nerve compression.

Laboratory Findings
•• Cranial CT should be done immediately after resuscitation.
492 Section 3: Systemic Pediatrics

Treatment
Review of Pediatrics and Neonatology

High Yield Points


•• Children with severe or moderate TBI (GCS score 3–8 or 9–12) should receive ICU care.
Target cerebral perfusion pressure •• Initial stabilization with severe TBI include rapid sequence tracheal intubation with spine
(CPP) ≈50 mm Hg for children 2–6 yr
precautions along with maintenance of normal extracerebral hemodynamics & blood gas
of age; 55 mm Hg for 7–10 yr age; &
•• Euvolemia is the target & hypotonic fluids should be avoided; NS is the fluid of choice.
65 mm Hg for those 11–16 yr of age.
•• ICP should be maintained <20 mm Hg;
•• 1st tier therapy for raised ICT includes elevation of head of bed, ensuring midline
positioning of head, controlled mechanical ventilation, sedation & analgesia (i.e.,
High Yield Points benzodiazepines/narcotics), osmolar agents like mannitol (0.25–1.0 g/kg IV over 20 min),
given in response to ICP spikes >20 mm Hg & hypertonic (3%) saline @ 0.1–1.0 mL/kg/hr.
•• Most TBIs in children are from closed- •• If ICP remains refractory to treatment, rule out hypercarbia, hypoxemia, fever, hypotension,
head injuries
hypoglycemia, pain & seizures. Consider repeat imaging to rule out a surgical lesion.
•• A child with epidural hematoma, may
be alert on presentation, but may •• Treatment for refractory raised ICP include decompressive craniectomy, pentobarbital infusion,
deteriorate after few hours mild hypothermia (32–34˚C), hyperventilation (PaCO2 = 25–30 mm Hg), lumbar CSF drainage
•• Peak ICP generally is seen at 48–72 hr
after trauma Prognosis
•• Mortality rates for children with severe TBI who reach pediatric ICU is 10–30%.

 INVOLVEMENT OF EYE IN NEUROLOGICAL DISEASES


Question 13
A 6-year-old child presented with PAPILLEDEMA M

severe headache and vomiting for •• It is the swelling of optic disc secondary to increased ICP
2 days along with fever. What is
•• It rarely occurs in infancy because skull sutures can separate to accommodate expanding brain
the finding is seen on eye exami-
nation? •• In older children, papilledema may be graded according to the Frisen scale.

NYSTAGMUS
Definition
It is an involuntary, rapid movement of the eye.

Types
a. Optic atrophy
b. Retinal hemorrhage •• Pendular nystagmus: The 2 phases have equal amplitude and velocity
c. Nystagmus •• Jerk nystagmus: In this, there is a fast and slow phase.
d. Papilledema
Clinical Significance
•• A few beats of end-gaze nystagmus with extreme lateral gaze has no significance
High Yield Points •• Pathologic horizontal nystagmus is most often congenital, drug-induced (e.g. anticonvul-
Horner Syndrome sants), or a result of vestibular system dysfunction
•• Characterized by ipsilateral ptosis, •• Vertical nystagmus is often seen in structural abnormalities of brainstem & cerebellum.
miosis and anhidrosis of the face •• Ocular bobbing is characterized by a downward jerk followed by a slow drift back to primary
•• It may be congenital or may
position and is associated with pontine lesions
be caused by a lesion of the
sympathetic pathway in the hypo­ •• Opsoclonus describes involuntary, chaotic oscillations of the eyes, which are often seen in
thalamus, brainstem, cervical spinal the setting of neuroblastoma or viral infection.
cord, or sympathetic plexus
 BRAIN DEATH
•• Brain death is the irreversible cessation of all functions of the entire brain, including the brainstem.
•• In children, brain death most commonly develops following Traumatic brain injury or asphyxia
injury.
•• 3 key components of clinical brain death diagnosis are demonstrations of coexisting
irreversible coma with a known cause, absence of brainstem reflexes, and apnea.
•• To establish diagnosis of brain death, findings must remain consistent for 2 examinations
separated by an observation period.
Recommended observation periods are:
–– 24 hr for neonates
–– 12 hr for infants & older children
•• An observation period of 24-48 hr prior to initiation of brain death assessment is recommended
following CPR or severe acute brain injury
Chapter 21: Pediatric Neurology 493

Complete documentation should include statements of the following:

Section 3: Systemic Pediatrics


L at e s t U p d at e s
•• Etiology and irreversibility of the coma.
•• Absence of confounding factors: hypothermia, hypotension, hypoxia, significant metabolic Full Outline of Unresponsiveness
(FOUR) score is a score to assess
derangement, significant drug levels.
sensorium, like the Glassgow coma
•• Absence of motor response to noxious stimulation.
score & includes Eye Response, Motor
•• Absence of brainstem reflexes: pupillary light reflex, corneal reflex, oculocephalic/ Response, Brainstem Reflexes &
oculovestibular reflex, cough & gag reflex. Respiration
•• Absence of respiratory effort in response to an adequate stimulus.

Features occasionally seen, compatible with diagnosis of brain death:


•• Respiratory-like movements (shoulder elevation and adduction, back arching, intercostal
expansion without significant tidal volumes)
•• Sweating, flushing, tachycardia
•• Normal BP without pharmacologic support or sudden increase in BP
•• Absence of diabetes insipidus
•• Deep tendon reflexes; superficial abdominal reflexes; triple flexion response, Babinski reflex

Features incompatible with brain death:


•• Decerebrate or decorticate posturing
•• True extensor or flexor motor responses to painful stimuli (at either a cranial nerve or somatic
nerve site)
•• Seizures

Answer Keys for Image-Based Questions

Answers Explanations / Identifying features


1. Ans. a. Normal tone To test tone of a newborn, vertical suspension is done, holding axillae without gripping thorax. Here, this
baby is remaining suspended with less in flexion, signifying normal tone

2. Ans. b. Anencephaly Here the skull bones are absent & the brain parenchyma is exposed; this is anencephaly

3. Ans. c. Myelomeningocele A sac-like cystic structure with remnants of neural tissue is suggestive of a Myelomeningocele

4. Ans. d. Iniencephaly Extreme retroflexion of head with absence of neck is s/o Iniencephaly

5. Ans c. Holoprosencephaly Single eye (cyclopia) with a tubular-shaped nose (proboscis), are usually seen in a child with
Holoprosencephaly

6. Ans. b. Arnold-Chiari The given MRI shows cerebellar tonsillar herniation into foramen magnum suggestive of Arnold-Chiari
malformation syndrome

7. Ans. a. Dandy walker The given MRI brain image shows dilated 4th ventricle with cerebellar vermis hypoplasia suggestive of
malformation Dandy walker malformation

8. Ans. b. Neurofibromatosis A → Café au lait spots on skin


B → Lisch nodules on iris } Both these point towards Neurofibromatosis

9. Ans. d. Shagreen patches Areas of thick leathery, pebbly skin, with orange peel appearance is called ‘shagreen patches’

10. Ans. b. Sturge-Weber Port wine stain of half of the face involving ophthalmic & maxillary divisions of trigeminal nerve suggest
syndrome Sturge-Weber syndrome

11. Ans. a. Spastic Diplegia Scissoring of lower limbs, as seen in this child suggests ‘Spastic Diplegia’

12. Ans. c. Tubercular Early morning headache & vomiting, along with papilledema, s/o raised intracranial tension, along with
meningitis fever & hydrocephalus with basal exudates on CT scan are seen in TB meningitis

13. Ans. d. Papilledema Fundus of the given child shows papilledema, which is also suggestive of raised intracranial tension
494 Section 3: Systemic Pediatrics
Review of Pediatrics and Neonatology

Questions
 NEURAL TUBE DEFECTS 12. Features of sacral meningomyelocele are all except:
 (TN PGMEE 2007)
1. Which is the best marker for NTD? (FMGE Nov 2017) a. Hydrocephalus b. Neuropathic bladder
a. aFP b. hCG c. Spastic limbs d. Areflexia
c. Pseudocholinesterase d. Inhibin-A
13. All of following are neural tube defects except:
2. Content of meningocele: M  (Recent Question 2017) a. Myelomeningocele b. Anencephaly  (AI 2004)
a. Dura mater b. Spinal cord c. Encephalocele d. Holoprosencephaly
c. Brain matter d. Cauda equina
14. Porencephaly is due to: (MP 2K)
3. Tuft of hair over the lumbosacral region in a newborn is a. Dandy Walker syndrome b. Cerebral infarction
suggestive of: (Recent Question 2017) c. Fetal alcohol syndrome d. Trisomy 13
a. Spina bifida occulta b. Sinus tract
c. Tumor d. Any of the above  HYDROCEPHALUS
4. Myelomeningocele most commonly involves which 15. A baby is born with fused eyes, single nasal chamber
region of spine: M  (Recent Question 2016) and undeveloped callosum. What is diagnosis?
a. Cervico-dorsal b. Dorsolumbar  (JIPMER 2017)
c. Lumbosacral d. Sacro-coccygeal a. Holoprosencephaly b. Schizencephaly
5. Marker of neural tube defects is: M  (WB PGMEE 2016)  c. Plagiocephaly d. Brachycephaly 
a. PAPPA b. hCG 16. Identify the condition shown in the image below:
c. Estriol d. Acetylcholinesterase   (NEET Pattern Jan 2018)
6. In a lady with history of birth of a previous child with
neural tube defect, amount of folic acid to be given
in microgram as secondary prophylaxis before next
conception is: (AIIMS May 2015)
a. 40 b. 400
c. 4000 d. 500
7. Folic acid requirement in pregnancy for primary
prevention of neural tube defects is: (TN PGMEE 2015)
a. 200 mg b. 400 mg
c. 600 mg d. 800 mg
8. When should folic acid supplementation start, in order
a. Dandy–Walker malformation
to be effective in preventing neural tube defects? M  b. Vein of Galen malformation
 (Recent Question 2015)
c. Mega cistern magna d. Arnold-Chiari malformation
a. As soon as pregnancy is diagnosed
17. Arnold Chiari malformation is characterized by all
b. Before the beginning of 2nd trimester
except: (Recent Question 2017)
c. At least 1 month before conception
a. Hypoplasia of cerebellar vermis
d. At least 1 week before conception
b. Herniation of cerebellum
9. A case of meningomyelocele was posted for surgery. Till c. Flattened base of skull
the patient is waiting for surgery, the covering of the sac d. Adulthood syndrome not associated with hydrocephalus
will be protected by a gauze soaked in: 18. Which is the most common cause of ventriculomegaly
 (AIIMS May 2013, AI 2012) in newborn? (Recent Question 2017)
a. Normal saline b. Tincture iodine a. Dandy-Walker syndrome b. Arnold-Chiari malformation
c. Methylene blue d. Mercurochrome c. Aqueductal stenosis d. Arachnoid villi malfunction
10. Neural tube defects have which one of the following 19. Commonly used shunt in hydrocephalus management:
inheritance patterns? (UPSC-II 09)  (AIIMS May 2015)
a. Autosomal dominant b. Autosomal recessive a. Ventriculoatrial b. Ventriculoperitoneal
c. X-linked recessive d. Multifactorial c. Ventriculopericardial d. Ventriculopleural
11. Which of the following is a marker for neural tube defects: 20. Which of the following is not a cause of communicating
a. Phosphatidyl cholinesterase (AI 2009) hydrocephalus? (Recent Question 2015)
b. Pseudo cholinesterase a. Choroid plexus papilloma
c. Acetyl cholinesterase b. Vein of Galen malformation
d. Butyrylcholinesterase c. Achondroplasia d. Meningeal metastasis
Chapter 21: Pediatric Neurology 495

32. Arnold chiari malformation is associated with:

Questions
21. Investigation of choice for hydrocephalus in infants: M
 (Recent Question 2013) a. Meningomyelocele (JIPMER 2006)
a. Cranial USG b. CT Scan b. Leptomeningeal cyst
c. MRI d. X-ray skull c. Agenesis of corpus callosum
22. Most common cause of fetal ventriculomegaly is: d. Dandy-Walker anomaly
a. Arnold-Chiari Malformation – I (DNB Pattern 2013) 33. Which one of the following is a common cause of
b. Arnold-Chiari malformation – II congenital hydrocephalus? M  (AI 2005)
c. Aqueductal stenosis a. Craniosynostosis b. Intrauterine meningitis
d. Dandy-Walker Malformation c. Aqueductal stenosis
23. Common deformity in Chiari II malformation is: d. Malformations of great vein of Galen
 (Recent Question 2013) 34. The following are recognized signs and symptoms of
a. Syringomyelia b. Meningomyelocele raised intracranial tension in a 9-month-old infant,
c. Hydrocephalus d. All of above except: (SGPGI 05)
24. About Dandy-Walker syndrome, true is: a. Bulging fontanel b. Setting-sun sign
a. Mostly have hydrocephalus  (Recent Question 2013) c. Papilledema d. Increase in head size
b. Cystic expansion of 4th ventricle 35. Most common cause of hydrocephalus in children is:
c. Mid cerebellar hypoplasia a. Post inflammatory obstruction (SGPGI 04)
d. All are true b. Budd-Chiari syndrome
25. Congenital hydrocephalus treatment: c. Brain tumor d. Perinatal injury
 (Recent Question 2012) 36. MacEwen sign is seen in: M (2004)
a. V–P shunt b. Acetazolamide a. Hydrocephalus b. Encephalitis
c. Glycerine d. Furosemide c. Meningitis d. Microcephaly
26. Which of the following agents is likely to cause cerebral 37. Acquired extracranial infection that causes Aqueductal
calcification and hydrocephalus in a newborn whose stenosis is: (UPSC 04)
mother has history of taking spiramycin but was not a. Bacterial endocarditis b. Mumps
compliant with therapy? (AI 09) c. Measles d. Staphylococcal septicemia
a. Rubella b. Toxoplasmosis
c. CMV d. Herpes  NEUROCUTANEOUS SYNDROMES
27. Bulging anterior fontanelle is/are seen in:
 (PGI Dec 2008) 38. Drug of choice for infantile spasms in a child with
a. Rickets b. CMV infection Tuberous sclerosis is: (Recent Question 2016)
c. Scurvy d. Hypothyroidism a. Diazepam b. ACTH
e. Tetracycline therapy c. Levetiracetam d. Vigabatrin
28. Macrocephaly is seen in which of the following 39. Lisch nodules are seen in which of the following
syndromes? (AIIMS Nov, May 2008) conditions? M  (UPSC CMS 2015)
a. Metachromatic leukodystrophy a. von Recklinghausen's disease
b. Adrenoleukodystrophy b. Louis-Bar syndrome
c. Canavan disease d. Krabbe's disease c. Tuberous sclerosis
29. In Arnold-Chiari malformation type-II there is: d. von Hippel-Lindau syndrome
a. Herniation of medulla oblongata (WBPG 2008) 40. Lisch nodule is seen in? (Recent Question 2015)
b. Herniation of medulla with cerebellum a. Tuberous sclerosis b. Sturge-Weber syndrome
c. Meningomyelocele c. Neurofibromatosis d. Wilson disease
d. Aqueductal stenosis 41. A 7-year-old boy presents with a right-sided hemangioma
30. Commonest cause of obstructive hydrocephalus in and left-sided focal seizures. The most likely diagnosis
children: (UP 07, 05) is: (AIIMS May 2014)
a. Aqueductal stenosis a. Neurofibromatosis b. Incontinentia pigmenti
b. Aqueductal gliosis c. Hypermelanosis of Ito d. Sturge-Weber syndrome
c. Subarachnoid hemorrhage 42. Which is not seen in tuberous sclerosis?
d. Tubercular meningitis  (AIIMS Nov 2014)
31. Which type of shift in intracranial content is common in a. Astrocytoma b. Ependymoma
children with progressive hydrocephalus? c. White matter lesions d. Subependymal nodules
a. Transforaminal herniation (Karnataka PGMEE 06) 43. Which is not seen in tuberous sclerosis?
b. Upward cerebellar herniation  (Recent Question 2014)
c. Unilateral transtentorial herniation a. Ash leaf macules b. Lisch nodules
d. Central transtentorial herniation c. Adenoma sebaceum d. Subependymal nodules
496 Section 3: Systemic Pediatrics

44. A child presented to the casualty with seizures. On


Review of Pediatrics and Neonatology

51. A previously well 2-year-old boy presents with a brief


exami­­nation an oval hypo­ pigmented macules were generalized seizure. There is no past or family history
noted on the trunk, along with sub-normal IQ. Probable of seizures. On examination, child is alert, active,
diag­nosis of the child is: (AIIMS Nov 2012) febrile with axillary temperature of 39.4°C. except
a. Neurofibromatosis for bilateral tonsillar enlargement and congestion,
b. Sturge-Weber syndrome systemic examination is normal. Management at this
c. Tuberous Sclerosis time may include: (UPSC CMS 2015)
d. Incontinentia Pigmenti 1. Immediate reduction of body temperature
45. A 10-year-old male child was presented to the pediatrician 2. Intravenous diazepam and phenytoin, followed by a
for evaluation of a seizure disorder. On examination a lumbar puncture
vascular plaque was found along the ophthalmic and 3. Institution of phenytoin for maintenance therapy for at
maxillary divisions of the trigeminal nerve. The mother least one year
informed the pediatrician that the lesion was present 4. Rectal diazepam every 8 hourly for 2-3 days to reduce
since birth and there was no change in morphology. The the risk of recurrence
most likely possibility is: (AIIMS May 2012) Select the correct answer using the code given below:
a. Sturge-Weber syndrome a. 1 and 2 only b. 1 and 4 only
b. Infantile hemangioma c. 3 and 4 d. 1, 2 and 4
c. Congenital hemangioma 52. Continuous prophylactic anticonvulsant therapy is not
d. Proteus syndrome needed in a child with febrile convulsions with? M 
46. Which among the following is the most common tumor  (APPG 2014)
associated with neurofibromatosis in a child? (AI 11) a. Developmental delay b. Family history of Epilepsy
a. Juvenile myelomonocytic leukemia c. Typical simple febrile fits
b. Acute lymphoblastic leukemia d. Persistent neurological deficit
c. Acute monocytic leukemia 53. % of children with simple febrile seizure developing
d. Acute myeloid leukemia epilepsy is: (Recent Question 2013)
47. A 2-year-old child is brought by parents with history a. 1–2% b. 2–5%
of seizures and developmental delay. He has multiple c. 5–10% d. 10–15%
hypopigmented macules over the back. What is the most
54. Treatment of simple febrile convulsion is: M 
probable diagnosis: (AI 2010)
 (Recent Question 2012)
a. Neurofibromatosis type I
a. Control of fever b. Rectal diazepam
b. Tuberous sclerosis
c. CSF finding d. Blood reports
c. Sturge Weber syndrome
55. What is correct about febrile seizures:
d. Linear sebacous nevus syndrome
 (Recent Question 2012)
48. Which one of the following is the most common tumor
a. Causes developmental delay
associated with type I neurofibromatosis?
b. Focal deficits
 (AIIMS May 2003)
c. Repeated seizures in 24 hours
a. Optic nerve glioma b. Meningioma
d. Does not need long term antiepileptics
c. Acoustic Schwannoma d. Low grade astrocytoma
56. Which of the following is not associated with increase in
the risk of epilepsy in a child with febrile seizures:
 FEBRILE SEIZURES a. Development delay (AIPGMEE 2010)
b. Late age of onset
49. True about febrile seizures: (PGI Nov 2017) c. Complex partial seizures
a. Most common cause of seizure in childhood d. Family history positive for epilepsy
b. May persist beyond 5 years if associated with atypical
57. Management of typical febrile seizures include all
feature
except: (DPG 10)
c. Family h/o febrile seizure is a risk factor a. Sponging b. Paracetamol or ibuprofen
d. Febrile seizure is always associated with neurologically c. Intermittent diazepam
abnormal child d. Prophylactic phenobarbitone
e. Anticonvulsants are the treatment of choice
58. True regarding febrile convulsions is: (PGI Dec 2000)
50. Midazolam given after how much minutes after a. Carbamazepine is good drug to treat it
continuous febrile seizures? M  (JIPMER Nov 2017) b. Patient with family h/o Febrile seizures have increased
a. 10 minutes of febrile seizures (midazolam nasal spray) incidence of recurrence
b. 5 minutes c. Long-term neurological deficits are common
c. 3 minutes d. Usually last for short while
d. 1 minute e. Continuous antiepileptic therapy is required
Chapter 21: Pediatric Neurology 497

59. A 4-year-old male had febrile seizures, best prophylaxis: 70. In a premature baby with convulsion after 2 days of

Questions
 (PGI June 2000) birth, first investigation to be done is: (AIIMS Nov 2013)
a. Paracetamol 6 hourly b. Paracetamol & diazepam a. Transcranial USG b. MRI
c. Diazepam d. Phenobarbitone c. Skull radiography d. CT skull
60. The most common cause of convulsions in a 9 month 71. Drug of choice for juvenile myoclonic epilepsy is:
old infant with fever: M  (AIPGMEE 2000)  (DNB Pattern 2013)
a. Septicemia b. Febrile seizures a. Phenytoin b. Lamotrigine
c. Brain tumor d. Phenylketonuria c. Valproate d. Zonisamide
72. Most common cause of convulsion on the first day of life
 EPILEPSY/SEIZURES in a newborn is: (Recent Question 2013-12)
a. Hypoxia b. Head injury
61. An adolescent school girl complaints of dropping c. Hypoglycemia d. Hypocalcemia
objects from hands, it gets precipitated during morning 73. Initial drug of choice in a child with status epilepticus:
and during exams. There is no history of loss of  (Recent Question 2013)
consciousness and her cousin sister has been diagnosed a. Lorazepam b. Phenobarbitone
with epilepsy. EEG was done and was suggestive of c. Valproate d. Phenytoin
epileptic spikes. What is the diagnosis?
74. A 7-year-old girl is easily distracted in class and exhibits
a. Juvenile myoclonic epilepsy (AIIMS May 2018)
poor scholastic performance. Seizures are precipitated
b. Atypical absence
by hyperventilation, what is the diagnosis?
c. Choreo–athetoid epilepsy
d. Centrotemporal spikes  (AIIMS Nov 2012)
a. Myoclonic seizures b. Absence seizures
62. Benign neonatal seizures is: (PGI May 2017)
c. Atonic seizures d. Myoclonia
a. Common in late preterms & terms
b. Followed by febrile seizures 75. A school going boy was noted with vacant stare several
c. Followed by infantile spasms times a day. There was no history of fever, seizures and
d. Prolonged seizures neurological deterioration. What is the diagnosis?
e. Also known as ‘fifth day fits'  (AI 2010)
a. Atonic seizures b. Absence seizures
63. Which of the following is not useful in the management
of status epilepticus? (Recent Question 2016)
c. Myoclonic seizures d. School phobia
a. Lorazepam b. Phenytoin 76. Cause of status epilepticus in a child: (PGI Dec 2008)
c. Phenobarbitone d. Carbamazepine a. Hypernatremia b. Hyponatremia
64. Lennox-Gastaut Syndrome is characterized by: c. Hyperkalemia d. Hypokalemia
a. Single seizure types (MAHA PGM CET 2016) e. Hypocalcemia
b. Good prognosis and adequate controlled epilepsy 77. Not a cause of neonatal seizures: (APPG 08)
c. Impaired cognitive function in all cases a. Pyridoxine deficiency b. Hypokalemia
d. EEG showing slow (< 3Hz) spike and wave discharge c. Hypoxia d. None
65. Which of the following is True regarding absence 78. Commonest cause of convulsions in a child with fever is:
seizures? (APPG 2015)  (AIIMS 84, UPSC 08)
a. Attack lasts for 60 to 90 seconds a. Febrile convulsions b. Meningitis
b. Hyperventilation often precipitates an attack c. Epilepsy d. Hypothyroidism
c. Onset of disease in the first year of life 79. True about Juvenile myoclonic epilepsy: (PGI Dec 2007)
d. Attack followed by postictal confusion a. Drug of choice is sodium valproate
66. Absence seizures are seen in? (Bihar PG 2015) b. Mental retardation
a. Grand mal epilepsy b. Myoclonic epilepsy c. Seizure can develop
c. Petit mal epilepsy d. Hyperkinetic child d. Neurological examination abnormal
67. A 15-year-old boy presented with day dreaming and e. Life long treatment needed
decline in school performance. The likely possibility is? 80. Which one of the following is the characteristic feature
 (MAHA PGM CET 2015) of juvenile myoclonic epilepsy? (AIIMS May 2006)
a. Atonic seizure b. Myoclonic seizure a. Myoclonic seizures frequently occur in morning
c. Typical absence seizure d. Atypical absence seizure b. Complete remission is common
c. Response to anticonvulsants is poor
68. Which of the following drugs is not used in Juvenile d. Associated absence seizures are present in majority of
Myoclonic Epilepsy? (Bihar PG 2015, AIPG 2010) patients
a. Topiramate b. Zonisamide 81. True about juvenile myoclonic epilepsy: (PGI June 2005)
c. Carbamazepine d. Valproate a. Focal seizure b. Generalized seizure
69. Treatment of rolandic epilepsy is: (DNB Pattern 2014) c. Myoclonus
a. Phenytoin b. Lamotrigine d. Response to sodium valproate
c. Carbamazepine d. ACTH e. Spike and waves in EEG
498 Section 3: Systemic Pediatrics

82. A child in status epilepticus should not be given:


Review of Pediatrics and Neonatology

93. Subdural empyema is a complication of all the following


 (JIPMER 04) conditions except? (MAHA PGM CET 2015)
a. Clonazepam b. Phosphenytoin a. Frontal Sinusitis b. Skull vault osteomyelitis
c. Lamotrigine d. Diazepam c. Middle ear disease d. Boil over face
83. All are feature of absence seizures except: (AI 2004) 94. A CSF/serum glucose ratio of < 0.4 is highly suggestive
a. Usually seen in childhood of: (MAHA PGM CET 2015)
b. 3 Hz spike wave in EEG a. Bacterial Meningitis b. Viral Meningitis
c. Postictal confusion c. Carcinomatous Meningitis
d. Precipitation by hyperventilation d. None of the above
95. What is the most common lobe involved in Herpes
 CEREBRAL PALSY Simplex virus encephalitis? (Recent Question 2015)
84. Diplegia means: (JIPMER 2012) a. Frontal lobe b. Temporal lobe
a. Weakness of upper limbs b. Weakness of lower limbs c. Occipital lobe d. Parietal lobe
c. Weakness of unilateral upper and lower limbs 96. The appearance of cobweb formation in CSF indicates?
d. Weakness of the all four limbs  (MAHA PGM CET 2014)
85. Most common sequelae due to periventricular leuko­ a. Pyogenic meningitis b. Viral meningitis
malacia: (AIIMS Nov 2009) c. Tuberculous meningitis d. Fungal meningitis
a. Spastic diplegia b. Spastic quadriplegia 97. India ink preparation of CSF is done to identify:
c. Mental retardation d. Seizures a. Streptococcus agalactiae (MAHA PGM CET 2014)
86. The most common type of cerebral palsy is? b. Naegleria fowleri
 (TN PGMEE 2007) c. Cryptococcus neoformans
a. Spastic b. Atonic d. Coxsackie B
c. Extrapyramidal d. Mixed 98. In a small child diagnosed with H. influenza meningitis.
87. A child presented with mental retardation and spastic What investigation must to be done before discharging
limbs with h/o perinatal distress. Diagnosis: him from the hospital? (DNB Pattern 2014)
a. Cerebral palsy b. HIE (TN PGMEE 2007) a. BERA b. MRI
c. Down's syndrome d. Edward syndrome c. CT scan d. X-ray skull
99. Commonest cause of meningoencephalitis in children
 CNS INFECTIONS
is: M  (APPG 2014)
a. HSV b. Measles
88. How long should a child be isolated after being
c. Arbovirus d. Enterovirus
diagnosed with bacterial meningitis to prevent further
transmission?  (AIIMS May 2018)
100. Residual auditory defect is a common complication in
a. Till 24 hours after starting antibiotics meningitis caused by: (Karn 11)
b. Till cultures become negative a. Staphylococcus b. Meningococcus
c. Till antibiotics course in complete c. Escherichia coli d. Haemophilus influenzae
d. Till 12 hours after admission 101. Which of the following is the most common cause of
89. Most common cause of meningitis in post-neonatal meningoencephalitis in children? (AIPGMEE 2010)
period: a. Mumps b. Arbovirus
a. Klebsiella pneumoniae (FMGE June 2018) c. HSV d. Enterovirus
b. Streptococcus pneumoniae 102. Neonatal meningitis is caused by all except?
c. Staphylococcus aureus
a. N. meningitis b. E. coli (WBPG 2010)
d. Mycobacterium tuberculosis
c. Listeria d. Group B streptococci
90. What is the commonest organism causing cerebrospinal
fluid shunt related infections? (Recent Question 2017) 103. Meningitis in a 4-year-old child; microscopy of gram
a. Staph aureus stained CSF sample will reveal: (WBPG 2008)
b. Gram Negative Bacilli a. Gram pos cocci b. Gram pos bacilli
c. Coagulase negative Staphylococcus c. Gram neg diplococci d. Gram neg bacilli
d. Enterococci 104. A child presents with generalized petechiae. CSF shows
91. Which of the following streptococci cause neonatal gram negative diplsococci. Treatment: (PGI June 2007)
meningitis? (MAHA PGM CET 2015) a. IV Ceftriaxone b. IV Penicillin G
a. S. agalactiae b. S. pyogenes c. IV Cefotaxime d. IV Amoxicillin
c. S. bovis d. S. mutans e. IV Ofloxacin
92. Which of the following is the antihelminthic of choice 105. The CSF findings in TB meningitis include: M 
for the treatment of neurocysticercosis ? a. High sugar + low protein (AIPGMEE 2007)
 (MAHA PGM CET 2015) b. Low sugar + high protein and lymphocytosis
a. Mebendazole b. Albendazole c. High sugar + high chloride
c. Thiabendazole d. Ivermectin d. Low sugar + high protein and lymphopenia
Chapter 21: Pediatric Neurology 499

106. Most common organism causing meningitis in a 1 year  MISCELLANEOUS

Questions
old child: (AIPGMEE 2007)
a. Streptococcus pneumoniae 116. Most common cause of syncope in a child:
b. H. influenzae  (DNB June 2018)
c. Listeria a. Breath holding spell b. Tantrum spell
d. Neisseria meningitidis c. Hypoglycemia d. Neurocardiogenic
117. You have been called to declare a brain dead 12-year-
107. A 7-day-old newborn presented with meningitis.
old child in PICU. All of the given are signs of brain
Common causes are: (PGI Dec 2007)
death except:  (AIIMS May 2018)
a. E. coli b. Strep. pneumoniae a. Normal BP without pharmacological support
c. N. meningitis d. Strep. agalactiae b. Positive spinal reflexes on stimulation
e. Staph aureus c. Sweating and tachycardia
108. Neonatal meningitis is caused by: (PGI June 2005) d. Decorticate and decerebrate posturing
a. Group A Streptococcus b. Group B streptococcus 118. A child with head injury is admitted in ICU. Management
c. E. coli d. H. influenzae of raised ICP includes all except: (JIPMER Nov 2017)
e. Klebsiella a. Mannitol when ICP>20
109. The following bacteria are most often associated with b. Minimal use of sedation and analgesia
acute neonatal meningitis except: (AI 2005) c. Controlled mechanical ventilation
a. E. coli b. Step. agalactiae d. Hypertonic saline may be used
c. N. meningitides d. L. monocytogenes 119. A neonate with left to Right shunt & high cardiac output
110. A 10-month-old child presents with 2 weeks history heart failure. CT Angiography of the neonate is shown.
of fever, vomiting and alteration of sensorium. Cranial What is the diagnosis? (Recent Question 2018)
CT scan reveals basal exudates and hydrocephalus, the
most likely etiological agent is: (AI 2004)
a. Mycobacterium tuberculosis
b. Cryptococcus neoformans
c. Listeria monocytogenes
d. Streptococcus pneumoniae
111. A neonate develops signs of meningitis at 7 days of birth.
The presence of which of the following infections agent
in the maternal genital tract can be the causative agent a. Sagittal venous thrombosis
of this disease: (AIIMS May 2004) b. Vein of Galen malformation
a. Neisseria gonorrhoeae c. Intracranial hemorrhage
b. Chlamydia trachomatis d. Arnold-Chiari malformation
c. Strep agalactiae 120. Which of the following is not seen in fetal alcohol
d. H. ducreyi syndrome? (AIIMS May 2017)
112. Bacterial meningitis in children (2 months–12 years) is a. Microcephaly b. Macrocephaly
usually due to the following organisms except: c. Holoprosencephaly
a. Streptococcus pneumoniae  (AI 2004) d. Thinning of corpus callosum
b. Neisseria meningitides 121. Age by which myelination of child completed is:
 (Recent Question 2017)
c. H. influenzae type B
a. By birth b. 6 months
d. L. monocytogenes
c. 6 years d. Adulthood
113. The most common agent associated with neonatal 122. True about Migraine in children are all except:
bacterial meningitis is: (AI 2004) a. Preceding aura (JIPMER May 2016)
a. H. influenzae type b b. N. meningitides b. Recurrent headache lasting 2–72 hours per episode
c. Strep. pneumoniae d. Strep. agalactiae c. Bilateral headache
114. A neonate develops encephalitis without any skin d. Waking up with headache in the morning
lesions. Most probable causative organism is:
 (AIIMS May 2002) 123. The clinical features of eye opening, best verbal
response and best motor response in Glasgow coma
a. HSV I b. HSV II
scale of 7 will best fit as: (Recent Question 2017)
c. Meningococci d. Streptococci
a. Eye opening to pain, making incomprehensive sounds
115. After 5 days of birth, baby developed poor feeding, and has flexion as best motor response
convulsions, fever with low protein low sugar and high b. Spontaneous eye opening, confused and localizes pain
chloride in CSF, is most likely due to: (AIIMS June 2000) c. Eye opening to pain, confused and localizes pain
a. L. monocytogenes b. M. pneumoniae d. No response, incomprehensive sounds and extension
c. TB d. Leptospira as best motor response
500 Section 3: Systemic Pediatrics

134. True about ischemic stroke in children: 


Review of Pediatrics and Neonatology

124. The genetic mutation most commonly linked with con­


genital central hypoventilation syndrome is:  (Recent Question 2014)
 (Recent Question 2017) a. Middle cerebral artery territory is
a. Mutation in sodium channel alpha 1 subunit most commonly involved
b. Mutation in paired like homeobox 2B gene b. Anterior cerebral artery territory is
c. Mutation in outer dynein arm most commonly involved
d. All of the above c. Initial CT scan is diagnostic in > 80% cases
125. Neuromyelitis Optica (Devic's disease) is a demyelinating d. MRI brain is not required
disorder characterized by monophasic or polyphastic 135. A 2-year-old unresponsive child came to casualty with
optic neuritis and transverse myelitis. The pathogenesis history of fall from a height. On examination, he is
Includes demonstration of IgG antibodies against responsive to verbal stimuli intermittently, respiratory
astrocyte foot processes within the periventricular rate is 30 per min, pulse 130 per min, spO2 is 94% and
region, brainstem, optic nerves and spinal cord. What BP is 104/60 mm Hg. What should be the next step of
is the name of these antibodies? (Recent Question 2017) management? (JIPMER 2014)
a. Antineuronal antibody a. Observe the child carefully and shift if necessary
b. Anticholinesterase antibody b. Transfer immediately to a tertiary center for CT brain and
c. Antiglial antibody further management
d. Anti Aquaporin-4 water channel antibody c. Start oxygen by face mask, immobilize cervical spine and
transfer to a tertiary center accompanied by doctor
126. Normal intracranial pressure in a child is:
d. Start oxygen by face mask and give mannitol
 (Recent Question 2016)
a. 30-70 mm of H2O b. 50-80 mm of H2O 136. A 7-days-old neonate present with recurrent seizures.
c. 100-150 mm of H2O d. 50-150 mm of H2O On examination, tachycardia and S3 gallop, bruit in
anterior fontanelle. Blood investigations are normal.
127. Following feature differentiate hydranencephaly from
Neuro­sonogram shows a hypoechoic lesion. What is the
hydrocephalus: (Recent Question 2016)
diagnosis? (JIPMER 2013)
a. Hydranencephaly is static while hydrocephalus continue
a. Arachoid cyst
to increase in size
b. Vein of Galen malformation
b. Hydranencephaly does not cause increase in head size
c. Dilated ventricles
c. Cerebral cortex is deficient/hypoplastic in
d. Intraventricular hemorrhage
hydranencephaly
137. Unilateral or bilateral clefts in cerebral hemisphere is
d. All of the above
called: (TN PGMEE 2013)
128. Drooping eyelids in evening in boy can be seen in:
a. Prosencephaly
 (Recent Question 2016)
b. Schizencephaly
a. Myasthenia gravis b. Oculomotor nerve palsy
c. Lissencephaly
c. Botulism toxin d. All of above
d. Hydranencephaly
129. Cerebral perfusion pressure in children is:
138. True about status marmoratus is: (DNB Pattern 2013)
 (Recent Question 2016)
a. Present in basal ganglia
a. 11-15 mm Hg b. 16-20 mm Hg b. Associated with asphyxia
c. 20-40 mm Hg d. 40-60 mm Hg c. Have a marbled appearance
130. In CSF examination, albuminocytologic dissociation d. All of the above
occurs in cases of: (MAHA PGM CET 2016)
139. Treatment of Guillain-Barre Syndrome (GBS) in a child
a. TB meningitis b. Motor neuron disease includes: (Recent Question 2012)
c. Guillain-Barre syndrome d. Demyelinating disorder a. IV Ig
131. Puff of smoke appearance on cerebral angiography is b. Ventilation
seen in which of the following condition: c. Plasmapheresis
a. Takayasu's disease (MAHA PGM CET 2016) d. All of above
b. Arteriovenous malformation 140. Decorticate child – False statement is:
c. Moyamoya disease a. Acute brain injury (Recent Question 2012)
d. Aortoarteritis b. Subthalamic and frontal lobe lesion
132. Wide open anterior fontanelle is found in following c. More dangerous than decerebrate lesion
diseases except: M  (Recent Question 2015) d. Flexion of arm and extension of lower limb
a. Rickets b. Cretinism 141. A child presented with open anterior fontanelle and
c. Osteogenesis imperfecta d. Craniosynostosis bruit. Midline lesion in brain was seen on USG. What is
133. Pseudotumor cerebri is caused by all except: your diagnosis: (Recent Question 2012)
 (Recent Question 2014) a. Malformation of vein of galen
a. OCP b. Acetazolamide b. Congenital hydrocephalus
c. Vit A excess d. Ciprofloxacin c. Down syndrome d. Rickets
Chapter 21: Pediatric Neurology 501

142. A newborn presents with congestive heart failure, on 144. Regarding polio which is true? (AIIMS Nov 2007)

Questions
exami­­nation has bulging anterior fontanelle with a bruit a. Most of the cases are symptomatic
on auscultation. Transfontanelle USG shows a hypoechoic b. Spastic paralysis is seen
mid­ line mass with dilated lateral ventricles. Most likely c. IM injections and increased muscular activity increases
diagnosis is: M  (AIIMS Nov 2011, Nov 2006, AI 2007) the risk of paralytic polio
a. Medulloblastoma b. Encephalocele d. Pulse polio immunization is indicated in all children less
c. Vein of Galen malformation than 3 years of age
d. Arachnoid cyst 145. Banana sign seen in the fetal brain suggests:
143. A 10-year-old boy, unconscious with 2 days h/o fever,  (COMEDK 05)
comes to pediatric ICU with RR 46/min, BP 110/80 a. Renal agenesis b. Encephalocele
and E1 V1 M3 on Glasgow coma scale, next step of c. Spina bifida d. Porencephaly
management includes: (PGI Nov 2009) 146. Which one of the following drugs usually does not cause
a. Intubate and ventilate pseudo­tumor cerebri? (UPSC 04)
b. Give 0.9% NaCl a. Salicylic acid
c. Start dopamine at the rate of 1–2 g/min/kg b. Nalidixic acid
d. Dopamine at the rate of 1–2 g/min/kg and furosemide c. Tetracycline
e. Start antibiotic and order for CT scan d. Nitrofurantoin
502 Section 3: Systemic Pediatrics
Review of Pediatrics and Neonatology

Answers with Explanations


 NEURAL TUBE DEFECTS
1. a. FP  Ref: Nelson's 20/e p 2802
• Prenatal screening of maternal serum for AFP in the 16th-18th wk of gestation is an effective method for identifying pregnancies
at risk for fetuses with neural tube defects in utero.
2. a. Dura mater  Ref: Nelson's 20/e p 2806
A meningocele consists of a CSF-filled meningeal sac only,
3. d. Any of the above  Ref: Nelson's 20/e p 2580, 2816
90% of patients with occult spinal dysraphism, have a cutaneous abnormality overlying the lower spine, including a small dimple,
tuft of hair, dermal vascular malformation, or subcutaneous lipoma
Occult spinal dysraphism includes spina bifida occulta, lipomeningocele, intradural lipoma, diastematomyelia, tight filum terminale,
dermoid cyst-sinus, aberrant nerve roots, anterior sacral meningocele, and cauda equina tumor.
4. c. Lumbosacral  Ref: Nelson’s 20/e p 2802-2803; Refer pretext for details
5. d. Acetylcholinesterase  Ref: Nelson’s 20/e p 2802-2803, Ghai 8/e p 575-577
Diagnosis Tools for prenatal diagnosis of Neural tube Defects include detailed ultrasound examinations and measurements of
maternal serum α-fetoprotein MSAFP, amniotic fluid α-fetoprotein, and amniotic fluid acetylcholinesterase (AChE).
6. c. 4000  Ref: Nelson’s 20/e p 2802-2803, Ghai 8/e p 575-577
•• To prevent NTDs, it is recommended that all women of childbearing age, who are capable of becoming pregnant take
•• 0.4 mg (400 g) of folic acid daily.
•• If a pregnancy is planned in high risk women (previously affected child), supplementation should be started with 4 mg (4000
µmg) of folic acid daily, beginning 1 month before the time of the planned conception.
7. b. 400 g  Ref: Nelson’s 20/e p 2802-2803, Ghai 8/e p 575-577
8. c. At least 1 month before conception  Ref: Nelson’s 20/e p 2802-2803
9. a. Normal saline  Ref: Nelson’s 20/e p 2802-2803, Ghai 8/e p 575-577
Meningomyelocele is covered with sterile saline-soaked gauze to prevent it from drying, trauma and infection
10. d. Multifactorial  Ref: Nelson’s 20/e p 2802-2803, Ghai 8/e p 575-577
Most Neural tube defects show multifactorial inheritance.
11. c. Acetyl cholinesterase  Ref: Nelson’s 20/e p 2802-2803, Ghai 8/e p 575-577
12. c. Spastic limbs  Ref: Nelson’s 20/e p 2802-2803, Ghai 8/e p 575-577
Meningomyelocele in lower sacral region lead to: Bowel and bladder incontinence, Anesthesia in perineal area seen; but no
impairment of motor function; So, spastic limbs are not seen.
13. d. Holoprosencephaly  Ref: Nelson’s 20/e p 2802-2803, Ghai 8/e p 575-577
Different types of neural tube defects are: Spina bifida occulta, meningocele, encephalocele, myelomeningocele, anencephaly,
caudal regression syndrome, dermal sinus, tethered cord, syringomyelia, diastematomyelia and iniencephaly Holoprosencephaly
is incomplete separation of the cerebral hemispheres across the midline.
14. b. Cerebral infarction  Ref: Nelson’s 20/e p 2802-2803, Ghai 8/e p 575-577
Porencephaly

•• Porencephaly is the presence of cysts or cavities within the brain that result from developmental defects or acquired lesions,
including infarction of tissue. They represent developmental abnormalities of cell migration.
•• True porencephalic cysts are most commonly located in the region of the sylvian fissure and typically communicate with
the subarachnoid space, the ventricular system, or both.
•• Often associated with microcephaly, abnormal patterns of adjacent gyri, and encephalocele.

 HYDROCEPHALUS
15. a. Holoprosencephaly  Ref: Nelson 20/e p 901
Holoprosencephaly
• An abnormality of brain development in which the brain does not properly divide into the right and left hemispheres.
• 3 main types (from most to least severe): Alobar, semi-lobar & lobar
• Severity of facial defects corresponds to severity of brain defect.
• Other signs and symptoms often include intellectual disability, seizures, pituitary dysfunction, short stature 
Chapter 21: Pediatric Neurology 503

• Associated with trisomy 13 (MC), trisomy 18, CHARGE syndrome, Smith-Lemli-Opitz syndrome, Rubinstein–Taybi syndrome

Answers with Explanations


   In alobar holoprosencephaly, there is loss of midline structures of the brain and face as well as fusion of lateral & 3rd ventricles
of the brain. Facial findings may include a single eye (cyclopia) or very closely spaced eyes  (ethmocephaly) or absent eyes
(anophthalmia), or very small eye (microphthalmia) with a tubular-shaped nose (proboscis);
16. a. Dandy-Walker malformation  Ref: Nelson's 20/e p 2811
Dandy-Walker malformation
Consists of:
• Cystic dilation of 4th ventricle, Hypoplasia of the cerebellar vermis
• Hydrocephalus and
• Enlarged posterior fossa
Variable degrees of neurologic impairment seen.
17 a. Hypoplasia of cerebellar vermis  Ref: Nelson's 20/e p 2811-2815
Chiari malformation
••It is the most common malformation of the posterior fossa and hindbrain.
••It consists of herniation of the cerebellar tonsils though the foramen magnum.
••Often, there is associated abnormality of the bones of the skull base leading to a small posterior fossa.
••Type I typically produces symptoms during adolescence or adult life and is usually not associated with hydrocephalus.
••Type II Chiari malformation is characterized by progressive hydrocephalus with a myelomeningocele.
18. c. Aqueductal stenosis  Ref: Nelson's 20/e p 2814-2815; Volpe’s Neurology of the Newborn 6/e p62
Ventriculomegaly is seen in neonates with hydrocephalus;
Obstructive or noncommunicating hydrocephalus develops most commonly in children because of an abnormality of the
aqueduct of Sylvius or a lesion in the fourth ventricle.
Nonobstructive or communicating hydrocephalus most commonly follows a subarachnoid hemorrhage, which is usually a
result of intraventricular hemorrhage in a premature infant.
19. b. Ventriculoperitoneal  Ref: Nelson's 20/e p 2814
Therapy for hydrocephalus:
•• Medical management, including use of acetazolamide and furosemide, can provide temporary relief.
•• Most cases of hydrocephalus require extracranial shunts, particularly a ventriculoperitoneal shunt.
•• The major complications of shunting are occlusion (characterized by headache, papilledema, emesis, mental status changes)
and bacterial infection (fever, headache, meningismus), usually due to Staph epidermidis.
•• Endoscopic third ventriculostomy (ETV) has evolved as a viable approach and criteria have been developed for its use, but
the procedure might need to be repeated to be effective.
20. b. Vein of Galen malformation  Ref: Refer pretext of this chapter for details
Vein of Galen is one of the causes of Obstructive or Non-communicating hydrocephalus.
21. a. Cranial USG  Ref: Nelson’s 20/e p 2814-2817, Ghai 8/e p 574-575
As the infants have an open anterior fontanelle, transcranial ultrasound is the investigation of choice, as it also avoids the risk
of radiation to the baby and gives an idea about the structural details
22. c. Aqueductal stenosis  Ref: Nelson’s 20/e p 2814, Ghai 8/e p 574
Obstructive or non-communicating hydrocephalus develops most commonly in children because of an abnormality of the
aqueduct of Sylvius or a lesion in the fourth ventricle.
23. b. Meningomyelocele, c. Hydrocephalus  Ref: Nelson’s 20/e p 2150; Refer pretext of this chapter for details
Explanation: Chiari malformation type I is usually associated with syringomyelia, while type II is usually associated with hydrocephalus
& meningomyelocele
24. d. All are true  Ref: Nelson’s 20/e p 2815-2816 Dandy Walker malformation
•• Cystic expansion of 4th ventricle and midline cerebellar hypoplasia seen; 90% patients have hydrocephalus
•• Papilledema, pyramidal tract signs and cranial nerve palsies may occur along with symptoms of raised ICP.
•• It is managed by shunting the cystic cavity in the presence of hydrocephalus.

25. a. V–P shunt  Ref: Nelson’s 20/e p 2814, Ghai 8/e p 574
26. b. Toxoplasmosis  Ref: Nelson’s 20/e p 2814, Ghai 8/e p 574
Cerebral calcification with hydrocephalus in seen in congenital toxoplasmosis.
27. a. Rickets, d. Hypothyroidism, e. Tetracycline therapy  Ref: Nelson’s 20/e p 2814, Ghai 8/e p 574
Rickets, tetracycline and Hypothyroidism are the causes of pseudotumor cerebri, which leads to manifestations of raised intracranial
tension like bulging fontanelle; Refer pretext for details about Pseudotumor cerebri.
28. c. Canavan disease  Ref: Nelson’s 20/e p 677
504 Section 3: Systemic Pediatrics

Canavan disease

Review of Pediatrics and Neonatology

•• Due to deficiency of the enzyme aspartoacylase, that leads to accumulation of N-acetylaspartic acid in brain
•• Usually normal at birth, but develop progressive macrocephaly, hypotonia, persistent head lag and delayed milestones
•• As the disease progresses, there is spasticity, joint stiffness, contractures, optic atrophy and seizures
••
Feeding difficulties, poor weight gain, and gastroesophageal reflux may occur in the 1st year

29. c. Meningomyelocele  Ref: Nelson’s 20/e p 2814, Ghai 8/e p 574


30. a. Aqueductal stenosis  Ref: Nelson’s 20/e p 2814, Ghai 8/e p 574
31. d. Central transtentorial herniation  Ref: Harrison’s 19/e p 1772
Herniation of Brain:
•• It refers to displacement of brain tissue by an overlying or adjacent mass into a contiguous compartment that it normally
does not occupy.
•• In the most common form of herniation, brain tissue is displaced from the supratentorial to the infratentorial compartment
through the tentorial opening; this is referred to as transtentorial herniation.
•• Increases in ICP due to a progressive hydrocephalus can ultimately displace the brain downward into the foramen magnum—a
process called central transtentorial herniation.
32. a. Meningomyelocele  Ref: Nelson’s 20/e p 2814, Ghai 8/e p 574
33. c. Aqueductal stenosis  Ref: Nelson’s 20/e p 2814, Ghai 8/e p 574
34. c. Papilledema  Ref: Nelson’s 20/e p 2814, Ghai 8/e p 574
Papilledema is usually not seen in infancy, as the fontanelle are open
35. a. Postinflammatory obstruction  Ref: Nelson’s 20/e p 2814, Ghai 8/e p 574
36. a. Hydrocephalus  Ref: Nelson’s 20/e p 2814, Ghai 8/e p 574
In Hydrocephalus, percussion of the skull might produce a cracked pot sound or MacEwen sign, indicating separation of the
sutures.
37. b. Mumps  Ref: Nelson’s 20/e p 1552-1554, Ghai 8/e p 217
Mumps virus is neurotropic and is thought to enter the CNS via the Choroid plexus and infect the choroidal epithelium and
ependymal cells, both of which can be found in CSF along with mononuclear Leukocytes.
Less-common CNS complications of mumps include transverse myelitis, aqueductal stenosis, and facial palsy.

 NEUROCUTANEOUS SYNDROMES

38. d. Vigabatrin  Ref: Nelson’s 20/e p 2878


Drug of choice for infantile spasms in a child with Tuberous sclerosis is Vigabatrin
39. a. von Recklinghausen’s disease  Ref: Nelson’s 20/e p 2874, Ghai 8/e p 586; Refer pretext
Von Recklinghausen's disease is the other name for Neurofibromatosis type I.
40. c. Neurofibromatosis  Ref: Nelson’s 20/e p 2874, Ghai 8/e p 586
Important clinical features of Neurofibromatosis (NF1) are cafe-au-lait spots, neurofibromas, axillary freckling, Lisch nodules optic
glioma and positive family history
41. d. Sturge-Weber syndrome  Ref: Nelson’s 20/e p 2879, Ghai 8/e p 586
Sturge-Weber Syndrome (SWS)
•• Angiomas involving leptomeninges & skin of face, typically in ophthalmic & maxillary divisions distributions of trigeminal nerve.
•• The hallmark of SWS is a facial cutaneous venous dilation, also referred to as a nevus flammeus or port-wine stain.
•• Contralateral focal seizures, calcification of cerebral cortex and glaucoma on same side as skin lesions are seen.
42. b. Ependymoma  Ref: Nelson’s 20/e p 2877-2878, Ghai p 586
Diagnostic criteria of Tuberous sclerosis:

Major features (Mnemonic: “PUSH CAR FAST”) Minor features
P: Pulmonary lymphangioleiomyomatosis F: Facial Angiofibroma Cerebral white matter migration lines
U: Ungual or periungual fibroma A: Astrocytoma (giant cell) Multiple dental pits
S: Shagreen patch, S: Subependymal nodule Gingival fibromas
H: Hypomelanotic macules (>3) T: Tuber (Cortical) Bone cysts/Multiple renal cysts
C: Cardiac rhabdomyoma Retinal achromatic patch
A: Angiomyolipoma of kidney Confetti skin lesions
R: Retinal hamartomas Hamartomatous rectal polyps
Non-renal hamartomas
Definite Tuberous sclerosis is diagnosed when at least 2 major or one major plus 2 minor features are present.
Chapter 21: Pediatric Neurology 505

43. b. Lisch nodules  Ref: Nelson’s 20/e p 2877-2878, Ghai p 586

Answers with Explanations


44. c. Tuberous Sclerosis  Ref: Nelson’s 20/e p 2877-2878, Ghai p 586
Hypopigmented macules (ash leaf macules), sub-normal IQ and seizures point towards a diagnosis of Tuberous sclerosis.
45. a. Sturge-Weber syndrome  Ref: Nelson’s 20/e p 2877-2878, Ghai p 586
46. a. Juvenile myelomonocytic leukemia  Ref: Nelson’s 20/e p 2877-2878, Ghai p 586
Patients with neurofibromatosis type 1 and Noonan syndrome have a predilection for JMML, since they have germline mutations
involved in RAS signaling.
47. b. Tuberous sclerosis  Ref: Nelson’s 20/e p 2877-2878, Ghai p 586
History of seizures and developmental delay along with multiple hypopigmented macules over the back suggests Tuberous
sclerosis.
48. a. Optic nerve glioma  Ref: Nelson’s 20/e p 2877-2878, Ghai p 586
Optic gliomas are present in 15% of patients with NF-1 and represent mostly low-grade astrocytomas. They are the main central
nervous system tumor with a marked increased frequency in NF-1.

 FEBRILE SEIZURES
49. a. Most common cause of seizure in childhood, b. May persist beyond 5 years if associated with atypical feature,
c. Family h/o febrile seizure is a risk factor  Ref: Nelson's 20/e p 2825-2831
Febrile seizures are the most common cause of seizure in under 5 children. Long-term antiepileptic therapy is not required for simple
febrile seizures.
50. b. 5 minutes  Ref: Nelson's 20/e p 2831
If the seizure lasts for longer than 5 min, acute treatment with diazepam, lorazepam, or midazolam is needed.
51. b. 1 and 4 only  Ref: Nelson’s 20/e p 2825
Immediate reduction of body temperature and Rectal diazepam or Oral Clobazam for 2-3 days is used in the management of
simple febrile seizures.
52. c. Typical simple febrile fits  Ref: Nelson’s 20/e p 2825, Ghai 8/e p 556
Continuous antiepileptic therapy is not required for simple febrile seizures, but intermittent prophylaxis may be given.
53. a. 1–2%  Ref: Nelson’s 20/e p 2825, Ghai 8/e p 556; Refer pretext of this chapter for details
54. b. Rectal diazepam  Ref: Nelson's 20/e p 2825, Ghai 8/e p 556
Rectal diazepam is prescribed for aborting the episode of febrile convulsions.
55. d. Does not need long-term antiepileptics  Ref: Nelson's 20/e p 2825, Ghai 8/e p 556
Long-term antiepileptic therapy is not required for simple febrile seizures.
56. b. Late age of onset  Ref: Nelson’s 20/e p 2825, Ghai 8/e p 556
57. d. Prophylactic phenobarbitone  Ref: Nelson’s 20/e p 2825, Ghai 8/e p 556
Measures to lower temperature (in a child with fever) like paracetamol and sponging and intermittent prophylaxis with Diazepam or
Clobazam are used in the treatment of febrile seizures
58. b. Patient with family h/o Febrile seizures have increased incidence of recurrence, d. Usually last for a short while
Ref: Nelson’s 20/e p 2825, Ghai 8/e p 556
Simple Febrile seizures last for < 15 minutes
59. b. Paracetamol and Diazepam, c. Diazepam  Ref: Nelson’s 20/e p 2825, Ghai 8/e p 556
Paracetamol does not reduce the risk of recurrent febrile seizures; Intermittent Diazepam or Clobazam are used for the first 3 days of
fever, for the same.
60. b. Febrile seizures  Ref: Nelson’s 20/e p 2825, Ghai 8/e p 556

 EPILEPSY/SEIZURES
61. a. Juvenile myoclonic epilepsy  Ref: Nelson's 20/e p 2836
Juvenile myoclonic epilepsy (Janz syndrome)
• Most common generalized epilepsy in young adults
• Starts in early adolescence with 1 or more of:
– Myoclonic jerks in morning, causing patient to drop things;
– Generalized tonic–clonic seizures upon awakening;
– Juvenile absences.
• Sleep deprivation & photic stimulation can act as precipitants.
• EEG shows generalized 4-5 Hz polyspike& slow- wave discharges.
506 Section 3: Systemic Pediatrics

Atypical absence seizures


Review of Pediatrics and Neonatology

• Less abrupt onset and offset of loss of awareness


• Associated myoclonic components & tone changes of head (head drop) & body
• Usually more difficult to treat.
• Precipitated by drowsiness
• Accompanied by 1-2 Hz spike–and–slow-wave discharges.
Benign childhood epilepsy syndrome with centrotemporal spikes
• Starts during childhood (3-10 yr) & is outgrown in adolescence.
• Child wakes up at night owing to a focal (simple partial) seizure causing buccal & throat tingling & tonic or clonic contractions
of 1 side of face, with drooling & inability to speak but with preserved consciousness & comprehension.
• EEG shows typical broad-based centrotemporal spikes that are markedly increased in frequency during drowsiness & sleep; MRI
is normal.
• Patients respond very well to antiepileptic drugs (AEDs) such as carbamazepine.
62. a. Common in late preterms & terms, e. Also known as ‘fifth day fits'  Ref: Swaiman's Pediatric Neurology, 6/ e p 1318
Benign neonatal seizures
This is a benign electroclinical syndrome that is seen in neonates. It is also referred to as fifth day fits or benign idiopathic neonatal
convulsions.
These seizures occur in term and later preterm infants born after an uneventful pregnancy, labor and delivery. Brief, but repetitive
seizures lasting for 1–2 minutes seen.
63. d. Carbamazepine  Ref: Nelson’s 20/e p 2831-2856; Refer pretext for details about status epilepticus treatment;
64. d. EEG showing slow (< 3 Hz) spike and wave discharge  Ref: Nelson’s 20/e p 2831-2856, Ghai 8/e p 557-561
Lennox-Gastaut syndrome (LGS): High yield facts
•• It typically starts between the ages of 2 to 10 yr.
•• It consists of a triad of developmental delay, multipleQ seizure types and typical EEG changes.
•• Patients commonly have myoclonic, atypical absences atonic, astatic, and tonic seizures that are difficult to control, and
most are left with long-term cognitive impairment and intractable seizures despite multiple therapies.
•• EEG findings are 1-2 Hz spike–and–slow waves, polyspike bursts in sleep, and a slow background in wakefulness.
•• Some, patients start with Ohtahara syndrome, develop West syndrome and then progress to LGS.

65. b. Hyperventilation often precipitates an attack  Ref: Nelson’s 20/e p 2831-2856, Ghai 8/e p 557-561
Typical absence seizures
•• Usually start at 5–8 yr of age; They do not have an aura, usually last for only a few seconds and are accompanied by
eyelid flutter or upward rolling of eyes and simple automatisms like lip-smacking or picking at clothing
•• Absence seizures do not have a postictal periodQ and there is immediate resumption of what patient was doing
•• Hyperventilation for 3–5 min can precipitate seizures & 3 Hz spike–and–slow-wave discharges
•• Early onset absence seizures (before the age of 4 yr) should trigger evaluation for glucose transporter defect.

66. c. Petit mal epilepsy  Ref: Nelson’s 20/e p 2831-2856, Ghai 8/e p 557-561
Absence seizures are sometimes referred to as petit mal seizures (from the French for “little illness”).
67. c. Typical absence seizure  Ref: Nelson’s 20/e p 2831-2856, Ghai 8/e p 557-561
Typical absence seizures may present in adolescents with day dreaming and decline in school performance.
68. c. Carbamazepine  Ref: Nelson’s 20/e p 2831-2856, Ghai 8/e p 557-561
Drugs used for treatment of JME are Valproate (drug of choice), Levetiracetam; Lamotrigine, Topiramate, Zonisamide.
69. c. Carbamazepine  Ref: Nelson’s 20/e p 2831-2856, Ghai 8/e p 557-561
Partial idiopathic epilepsy with rolandic spikes
•• Also called benign epilepsy with centrotemporal spikes
•• They are seizures with falling asleep or on awakening; focal sharp waves with centrotemporal location on EEG.
•• Carbamazepine is the most frequently used 1st line drug, but valproate, phenytoin, gabapentin and levetiracetam have been
found effective as well.
70. a. Transcranial USG  Ref: Nelson's 20/e p 2831-2856, Ghai 8/e p 557-561
Intracranial hemorrhage can be a common cause for seizures in a preterm neonate, which can be picked up easily on a transcranial
USG through the open anterior fontanelle.
71. c. Valproate  Ref: Nelson's 20/e p 2831-2856, Ghai 8/e p 557-561
72. a. Hypoxia  Ref: Nelson’s 20/e p 2831-2856, Ghai 8/e p 557-561
Hypoxic–Ischemic Encephalopathy is the most common cause of neonatal seizures, accounting for 50-60% of patients. Seizures
secondary to this encephalopathy usually occur within 12 hr of birth.
Chapter 21: Pediatric Neurology 507

73. a. Lorazepam  Ref: Nelson’s 20/e p 2831-2856, Ghai 8/e p 557-561

Answers with Explanations


Initial drug given in Status epilepticus in a child is a short acting Benzodiazepine like Lorazepam or Midazolam, to abort the seizures
74. b. Absence seizures  Ref: Nelson’s 20/e p 2831-2856, Ghai 8/e p 557-561
This is a typical description of Absence seizures.
75. b. Absence seizures  Ref: Nelson’s 20/e p 2831-2856, Ghai 8/e p 557-561
76. b. Hyponatremia and e. Hypocalcemia  Ref: Nelson’s 20/e p 2831-2856, Ghai 8/e p 557-561
Etiology of Status epilepticus in children:

•• Metabolic disturbances: hypoglycemia; hypocalcemia; hyponatremia; hypomagnesemia;
•• Drug intoxication (e.g., tricyclic antidepressants) in children and drug and alcohol abuse in adolescents;
•• Drug withdrawal or overdose in patients on antiepileptic drugs;
•• Acute head trauma; Infections: Encephalitis; meningitis;
•• Autoimmune encephalitis (such as anti-NMDA receptor and anti–voltage-gated K+ channel antibody syndromes);
•• Ischemic (arterial or venous) stroke; Intracranial hemorrhage; Hypoxic–ischemic injury
•• Inborn errors of metabolism such as nonketotic hyperglycinemia in neonates and MELAS in infants and children;
•• Ion channel–related epilepsies (e.g., sodium and potassium channel mutations);
•• Systemic conditions (such as hypertensive encephalopathy, hepatic encephalopathy);
•• Brain tumors; brain malformations, neurodegenerative disorders, storage diseases.
77. b. Hypokalemia  Ref: Nelson’s 20/e p 2831-2856, Ghai 8/e p 557-561
Pyridoxine deficiency and pyridoxal-5-phosphate deficiency, Hypoxia, Hypocalcemia, Hypoglycemia, Hypomagnesemia, Hypo­
natremia and hypernatremia may precipitate neonatal seizure. Hypokalemia usually does not cause neonatal seizure.
78. a. Febrile convulsions  Ref: Nelson’s 20/e p 2831-2856, Ghai 8/e p 557-561
Febrile seizures are the most common cause of convulsions in a child with fever.
79. a. Drug of choice is sodium valproate, c. Seizure can develop, d. Neurological examination abnormal, e. Life long
treatment needed  Ref: Nelson’s 20/e p 2831-2856, Ghai 8/e p 557-561
Juvenile myoclonic epilepsy (JANZ syndrome):
•• It is the most common generalized epilepsy in young adults, accounting for 5% of all epilepsies
•• It has been linked to mutations in many genes including CACNB4; CLNC2; EJM2, 3, 4, 5, 6, 7, 9; GABRA
•• Typically, it starts in early adolescence between ages of 12–16 years
•• Myoclonic jerks in the morning, often causing the patient to drop things
•• Generalized tonic-clonic or clonic seizures upon awakening; and Juvenile absences seen
•• Sleep deprivation and photic stimulation can act as precipitants
•• EEG usually shows generalized 4-5 Hz polyspike–and–slow-wave pattern, enhanced by photic stimulation
•• Drug of choice are valproate and lamotrigine; complete remission is relatively uncommon

80. a. Myoclonic seizures frequently occur in morning  Ref: Nelson’s 20/e p 2831-2856, Ghai 8/e p 557-561
81. b. Generalized seizure, c. Myoclonus, d. Response to sodium valproate, e. Spike and waves in EEG  Ref: Nelson’s 20/e p
2831-2856
82. a. Clonazepam  Ref: Nelson’s 20/e p 2831-2856, Ghai 8/e p 557-561
Shorter acting Benzodiazepines are preferred for Status epilepticus treatment
So a longer acting drug, Clonazepam, with a duration of action if 6–12 hours is not used.
83. c. Postictal confusion  Ref: Nelson’s 20/e p 2831-2856, Ghai 8/e p 557-561; Refer Ans. 56 above

 CEREBRAL PALSY
84. b. Weakness of lower limbs  Ref: Nelson’s 20/e p 2896-2899, Ghai 8/e p 581-583
85. a. Spastic diplegia  Ref: Nelson’s 20/e p 2896-2899, Ghai 8/e p 581-583
Most common sequelae due to periventricular leukomalacia seen in preterm neonates is Spastic diplegia.
86. a. Spastic  Ref: Nelson’s 20/e p 2896-2899, Ghai 8/e p 581-583
•• Most common type of cerebral palsy is spastic diplegia in 35% cases, followed by Hemiplegia in 25% cases
•• Spastic quadriplegia is the most severe form of CP because of marked motor impairment of all extremities and the high
association with intellectual disability and seizures.
87. a. Cerebral palsy  Ref: Nelson’s 20/e p 2896-2899, Ghai 8/e p 581-583
The CNS consequences and Sequelae due to perinatal asphyxia is referred as cerebral plasy.
508 Section 3: Systemic Pediatrics

 CNS INFECTIONS
Review of Pediatrics and Neonatology

88. a. Till 24 hours in after starting antibiotics  Ref: Nelson 20/e p 2941
The CSF becomes sterile within 24-48 hours of initiation of appropriate antibiotic therapy.
89. b. Streptococcus pneumoniae  Ref: Nelson 20/e p 2936-2948
90. c. Coagulase negative Staphylococcus  Ref: Nelson’s 20/e p 2938
CSF shunt infections increase the risk of meningitis caused by staphylococci (especially coagulase-negative species) and other
low-virulence bacteria that typically colonize the skin.
91. a. S. agalactiae  Ref: Nelson’s 20/e p 2936-2948, Ghai 8/e p 563-565
92. b. Albendazole  Ref: Nelson’s 20/e p 2936-2948, Ghai 8/e p 563-565
Albendazole is the antithelminthic of choice for the treatment of neurocysticercosis.
93. d. Boil over face  Ref: Nelson’s 20/e p 2936-2948, Ghai 8/e p 563-565
Infections of the sinuses, ear and skull bones can lead to Subdural empyema.
94. a. Bacterial Meningitis  Ref: Nelson’s 20/e p 2936-2948, Ghai 8/e p 563-565; Refer pretext of this chapter;
Hypoglycorrhachia (Low CSF: serum glucose ratio) is commonly seen in Bacterial meningitis.
95. b. Temporal lobe  Ref: Nelson’s 20/e p 2936-2948
96. c. Tuberculous meningitis  Ref: Nelson’s 20/e p 2936-2948, Ghai 8/e p 563-565
Cob web coagulum formation in CSF is a feature of Tuberculous meningitis.
97. c. Cryptococcus neoformans  Ref: Nelson’s 20/e p 2936-2948, Ghai 8/e p 563-565
India ink preparation of CSF is a negative staining technique done to identify Cryptococcus neoformans.
98. a. BERA  Ref: Nelson’s 20/e p 2936-2948, Ghai 8/e p 563-565
Sensorineural hearing loss is the most common sequelae of bacterial meningitis; So a hearing evaluation should be
done before discharge of a child with meningitis, from hospital.
99. d. Enterovirus  Ref: Nelson’s 20/e p 2936-2948, Ghai 8/e p 563-565
Enteroviruses are the most common cause of viral meningoencephalitis in children.
100. d. Haemophilus influenzae  Ref: Nelson’s 20/e p 2936-2948, Ghai 8/e p 563-565
Sensorineural hearing loss as a complication of bacterial meningitis, is a result of cochlear infection.
It occurs in as many as 30% of patients with pneumococcal meningitis, 10% with meningococcal, and 5-20% of those with
H. influenzae type b meningitis. So, the best answer among the given options is H. influenzae.
101. d. Enterovirus  Ref: Nelson’s 20/e p 2936-2948, Ghai 8/e p 563-565
102. a. N. meningitidis  Ref: Nelson’s 20/e p 2936-2948, Ghai 8/e p 563-565
Common causes of Neonatal meningitis include E.coli, Listeria and Gr B Streptococci
103. a. Gram pos cocci  Ref: Nelson’s 20/e p 2936-2948, Ghai 8/e p 563-565
Meningitis in a 4 yr child is mostly caused by Streptococcus pneumoniae, which is a gram positive cocci.
104. a. IV Ceftriaxone, b. IV Penicillin G, c. IV Cefotaxime  Ref: Nelson’s 20/e p 2936-2948, Ghai 8/e p 563-565
The given clinical and laboratory findings are suggestive of Meningococcal infection
Most strains of N. meningitidis are sensitive to penicillin and cephalosporins.
105. b. Low sugar + high protein and lymphocytosis  Ref: Nelson’s 20/e p 2936-2948, Ghai 8/e p 563-565
Lymphocytosis is seen in Tuberculous meningitis, though in early stages, CSF neutrophilia may be seen
106. a. Streptococcus pneumoniae  Ref: Nelson’s 20/e p 2936-2948, Ghai 8/e p 563-565
As most common organism for meningitis in India, is not mentioned in the question, the best ans is S. pneumoniae, which is the most

common organism responsible for meningitis throughout the world. Refer pretext for details.
107. a. E. coli, d. Strep. agalactiae  Ref: Nelson’s 20/e p 2936-2948, Ghai 8/e p 563-565; common causes of Acute bacterial meningitis
in neonates are Gr. B Strepto, E. coli & Listeria
108. b. Group B Streptococcus, c. E. coli  Ref: Nelson’s 20/e p 2936-2948, Ghai 8/e p 563-565
109. c. N. meningitides  Ref: Nelson’s 20/e p 2936-2948, Ghai 8/e p 563-565
110. a. Mycobacterium tuberculosis  Ref: Nelson’s 20/e p 2936-2948, Ghai 8/e p 563-565
Fever for 2 weeks along with neurological features with CNS imaging showing basal exudates and hydrocephalus are compatible
with a diagnosis of Tuberculous meningitis.
111. c. Strep agalactiae  Ref: Nelson’s 20/e p 2936-2948, Ghai 8/e p 563-565
Chapter 21: Pediatric Neurology 509

112. d. L. monocytogenes  Ref: Nelson’s 20/e p 2936-2948, Ghai 8/e p 563-565

Answers with Explanations


Listeria is an important cause of meningitis in the neonatal period, but not in older children.
113. d. Strep. agalactiae;  Ref: Nelson’s 20/e p 2936-2948, Ghai 8/e p 563-565
114. b. HSV II  Ref: Nelson's 20/e p 2936-2948, Ghai 8/e p 563-56

High Yield Points


CNS infection by Herpes Simplex:
•• In the neonatal period, HSV II (acquired from maternal genital lesions) is an important cause of encephalitis
•• Beyond the neonatal period, HSV encephalitis is almost always caused by HSV I
•• HSV encephalitis usually involves the (most common) & temporal frontal cortex and the limbic system
•• HSV is a cause of aseptic meningitis and is the most common cause of recurrent aseptic meningitis (Mollaret meningitis).
•• Fever is relatively uncommon and skin vesicles occur in only 60% of cases of HSV encephalitis

115. a. L. monocytogenes  Ref: Nelson's 20/e p 2936-2948, Ghai 8/e p 563-565
Out of the given options, most common cause of neonatal meningitis is: Listeria monocytogenes.

 MISCELLANEOUS

116. d. Neurocardiogenic  Ref: Nelson's 20/e p 514


Syncope is sudden transient loss of consciousness with inability to maintain postural tone. Most common cause of syncope in
the normal pediatric population is neurocardiogenic syncope, also known as vasovagal syncope. Syncope is caused by a lack of
adequate cerebral blood flow.
117. d. Decorticate and decerebrate posturing  Ref: Nelson's 20/e p 512-513
•• Brain death is the irreversible cessation of all functions of the entire brain, including the brainstem.
•• 3 key components of clinical brain death diagnosis are demonstrations of coexisting irreversible coma with a known cause,
absence of brainstem reflexes, and apnea.
•• To establish diagnosis of brain death, findings must remain consistent for 2 examinations separated by an observation period of
24 hr for neonates & 12 hr for infants & older children
Complete documentation should include statements of the following:
•• Etiology and irreversibility of the coma.
•• Absence of confounding factors: hypothermia, hypotension, hypoxia, significant metabolic derangement.
•• Absence of motor response to noxious stimulation.
•• Absence of brainstem reflexes: pupillary light reflex, corneal reflex, oculocephalic/oculovestibular reflex, cough & gag reflex.
•• Absence of respiratory effort in response to an adequate stimulus.
Features occasionally seen, compatible with diagnosis of brain death:
•• Respiratory-like movements (shoulder elevation and adduction, back arching, intercostal expansion without significant tidal
volumes)
•• Sweating, flushing, tachycardia
•• Normal BP without pharmacologic support or sudden increase in BP
•• Absence of diabetes insipidus
•• Deep tendon reflexes; superficial abdominal reflexes.
Features incompatible with Brain death:
•• Decerebrate or decorticate posturing
•• True extensor or flexor motor responses to painful stimuli
•• Seizures.
118. b. Minimal use of sedation and analgesia  Ref: Nelson's 20/e p 510-512
ICP should be maintained <20 mm Hg; First-tier therapy for raised ICP includes elevation of head of bed, ensuring midline
positioning of head, controlled mechanical ventilation, and sedation and analgesia (i.e., benzodiazepines and narcotics) &
osmolar agents like mannitol and hypertonic saline.
119. b. Vein of Galen malformation
CT angiography shows enhancing rounded lesion at tentorial apex, extending posteriorly & draining into dilated superior sagittal
sinus, through dilated straight sinus, suggestive of vein of Galen malformation.
The anomaly is actually due to a cerebral arteriovenous fistula of the median prosencephalic vein (a precursor of the vein of
Galen) occurring at 6-11 weeks gestation.
510 Section 3: Systemic Pediatrics

120. b. Macrocephaly  Ref: Nelson's 20/e p 2813


Review of Pediatrics and Neonatology

Microcephaly, Growth retardation, ptosis, absent philtrum, congenital heart disease (ASD), feeding problems, neuroglial
heterotopia & disorganization of neurons are features of fetal alcohol syndrome.
121. d. Adulthood  Ref: Infant and Early Childhood Neuropsychology by Glen P. Aylward
There are several general rules regarding myelination: (1) Proximal pathways myelinate before distal pathways, (2) sensory pathways
myelinate before motor pathways, and (3) projection pathways myelinate before association pathways. Myelination progresses from
the central sulcus outward toward the occipital, frontal, and temporal poles. For example, pyramidal tract efferents (transmitting
outwards) from the motor cortex are myelinated by the end of the first year.
The medical longitudinal fasciculus acquires myelin at 24 weeks' gestation and is fully myelinated within 2 weeks. The
corticospinal tract begins myelinating at approximately 38 weeks and is not complete until 2 years of age. The corpus callosum
begins myelination at approximately 4 months postnatally and is not complete until late adolescence. The last tract to complete
myelination is the ipsilateral association bundle that interconnects the anterior frontal and the temporal lobes, which is not
complete until 32 years of age.
122. d. Waking up with headache in the morning  Ref: Nelson's 20/e p 2866-2867
Childhood Migraine
••It is characterized by moderate to severe, episodic headache, focal in location, having throbbing.
••Quality & may be associated with nausea, vomiting, light/sound sensitivity.
••Compared to adults, pediatric migraine is shorter in duration & often has bilateral location.
••Migraine can also be associated with an aura that may be typical (visual, sensory, or dysphasic) or atypical (i.e., hemiplegic,
“Alice in Wonderland” syndrome).
••When the headaches associated with vomiting episodes are sporadic and not worsening, it is more likely that the diagnosis is
migraine.
Note: In secondary headache, particularly related to increased intracranial pressure, there is daily or near daily early morning
vomiting, or headaches waking the child up from sleep.
123. a. Eye opening to pain, making incomprehensive sounds and has flexion as best motor response  Ref: Swaiman Vol 2, p 864
Glassgow coma scale (GCS): It is the most common scoring system used to describe the level of consciousness in a person esp
following a traumatic brain injury.
Modified GCS for infants and children: (Max score is 15 and minimum is 3)
Child Infant Score
Eye opening Spontaneous Spontaneous 4
To speech To speech 3
To pain only To pain only 2
No response No response 1
Best verbal response Oriented, appropriate Coos and babbles 5
Confused Irritable cries 4
Inappropriate words Cries to pain 3
Incomprehensible sounds Moans to pain 2
No response No response 1
Best motor response Obeys commands Moves spontaneously and purposefully 6
(most important in localizes painful stimulus Withdraws to touch 5
intubated or unconscious Withdraws in response to pain Withdraws to response in pain 4
patient) Flexion in response to pain Abnormal flexion posture to pain 3
Extension in response to pain Abnormal extension posture to pain 2
No response No response 1

Discussing about the options one by one,
a. GCS = E2 + V2 + M3 = 7 b. GCS = E4 + V4 + M5 = 13
c. GCS = E2 + V4 + M5 = 11 d. GCS = E1 + V2 + M2 = 5
124. b. Mutation in paired like homeobox 2B gene  Ref: Nelson’s 20/e p 2148
Congenital central hypoventilation syndrome (CCHS) or Ondine curse is most commonly related to a PHOX2B (paired-like
homeobox 2B) mutation, which presents in the neonatal period and also associates with autonomic problems (Hirschsprung
disease) and neural crest tumors
In the classic case of CCHS, symptoms of alveolar hypoventilation are manifest in the newborn period and during sleep only—with
diminished tidal volume and a typically monotonous respiratory rate with cyanosis and hypercarbia. In more severe cases of CCHS,
the hypoventilation is manifest during wakefulness and sleep.
Chapter 21: Pediatric Neurology 511

125. d. Anti Aquaporin-4 water channel antibody  Ref: Nelson’s 20/e p 2922-2923

Answers with Explanations


Neuromyelitis Optica (NMO) or Devic’s disease:
What is it? It is a demyelinating disorder characterized by monophasic or polyphasic episodes of optic neuritis and/or
transverse myelitis.
Pathogenensis NMO is associated with IgG antibodies against aquaporin-4 water channel, which is most abundant on
astrocyte foot processes within periventricular regions, brainstem, optic nerves and spinal cord

Antibody binding to aquaporin-4 activates classical complement pathway with C5b-C9 components lead-
ing to leukocytes attraction and degranulation, causing astrocyte death

Chemokines from dying astrocytes and activated leukocytes attract macrophages, leading to death of
oligodendrocytes and neurons with subsequent necrosis and cavitation in affected tissues.
Diagnostic criteria Optic neuritis and transverse myelitis plus at least 2 of 3 supporting criteria:
(1) brain MRI not diagnostic of MS, (2) seropositivity for anti–aquaporin-4 antibody, or
(3) spine MRI with longitudinally extensive transverse myelitis involving at least 3 spinal segments

126. d. 50-150 mm of H2O  Ref: Nelson’s 20/e p 2799


Normal opening pressures are 90-120 mm H2O in newborns, 60-180 mm H2O in young children and 12-120 mm H2O in older
children and adults.
127. c. Cerebral cortex is deficient/hypoplastic in hydranencephaly  Ref: Nelson’s 20/e p 2817
Hydranencephaly
•• Cerebral hemispheres are absent or represented by membranous sacs with remnants of cortex dispersed over the membrane.
•• Midbrain and brainstem are relatively intact; Transillumination shows an absence of cerebral hemispheres.
•• Cause of hydranencephaly is thought to be bilateral occlusion of internal carotid arteries during early fetal development.
•• Affected infants can have a normal or enlarged head circumference at birth that grows at an excessive rate postnatally.
••
The child is irritable, feeds poorly, develops seizures & spastic quadriparesis, & has no cognitive development.

128 d. All of above  Ref: Nelson’s 20/e p 2794, 2992-2995


Discussing about the options one by one,
a. Myasthenia gravis is a chronic autoimmune disease of neuromuscular blockade, characterized clinically by rapid fatigability of
striated muscle, particularly extraocular & palpebral muscles and those of swallowing. Patients are more symptomatic late in the
day or when tired.
b. Oculomotor nerve palsy:
•• Oculomotor nerve innervates superior, inferior & medial recti, inferior oblique & levator palpebrae superioris muscles.
•• Complete paralysis causes ptosis, dilation of pupil, displacement of eye outward & downward & impairment of adduction &
elevation.
c. Botulism:
Etiology Botulism results from ingestion of food containing the toxin of Clostridium botulinum, a Gram-positive, spore-
bearing, anaerobic bacillus; Honey is a common source of contamination
Pathophysiology Botulinum toxin cleaves structural glycoproteins of the wall (i.e. membrane) of synaptic vesicles including
synaptobrevin & synaptotagmin, but synaptophysin is resistant.
Clinical features Incubation period is short, only a few hours, & symptoms begin with nausea, vomiting & diarrhea.
Cranial nerve involvement soon follows, with diplopia, dysphagia, weak suck, facial weakness & absent gag
reflex. Generalized hypotonia & weakness then develop & can progress to respiratory failure.
Diagnosis Neuromuscular blockade is documented by EMG with repetitive nerve stimulation.
Treatment Respiratory support may be required until toxin is cleared from body. No specific antitoxin is available;
Guanidine may be effective for extraocular & limb muscle weakness but not for respiratory muscle.

129. d. 40-60 mm Hg  Ref: Nelson’s 20/e p 507


Cerebral perfusion pressure (CPP) is calculated as the difference between the systemic mean arterial pressure (MAP) and the
intracranial pressure (ICP); So, CPP = MAP – ICP
The normal range of CPP is thought to be approximately 50 to 150 mm Hg in healthy adults, and 40 to 60 mm Hg, in children.
130. c. Guillain-Barre Syndrome  Ref: Nelson’s 20/e p 3010-3013, Ghai 8/e p 590
Albuminocytologic dissociation occurs in cases of Guillain-Barre SyndromeQ
131. c. Moyamoya disease  Ref: Nelson’s 20/e p 2927
Puff of smoke appearance on cerebral angiography is seen in Moyamoya disease.
132. d. Craniosynostosis  Ref: Nelson’s 20/e p 2817-2819, Ghai 8/e p 41
In Craniosynostosis there is premature fusion of cranial sutures and so fontanelles close early and are small;
While, Rickets, Cretinism (Hypothyroidism), Ostoeogenesis imperfecta & Achondroplasia are important causes of large anterior fontanelle.
512 Section 3: Systemic Pediatrics

133. b. Acetazolamide  Ref: Nelson’s 20/e p 2951, Ghai 8/e p 575, 108; Refer pretext for details
Review of Pediatrics and Neonatology

134. a. Middle cerebral artery territory is most commonly involved  Ref: Nelson’s 20/e p 2925-2930
135. c. Start oxygen by face mask, immobilize cervical spine and transfer to a tertiary centre accompanied by doctor
Ref: Nelson’s 20/e p 545-552
The best management in this case is to supplement O2 as there is hypoxia, to prevent further CNS injury; Cervical spine should be
immobilized and the patient should be transferred to a tertiary centre accompanied by doctor.
136. b. Vein of Galen malformation  Ref: Nelson’s 20/e p 1193
Newborn presenting with features of heart failure and cranial bruit with a hypoechoic lesion seen on cranial ultrasound suggests
Vein of Galen malformation
Vein of Galen malformation is an arteriovenous connection between primitive choroidal vessels & median prosencephalic
vein of Markowski, that develops between 6th & 11th weeks of gestation, after development of circle of Willis.
137. b. Schizencephaly  Ref: Nelson’s 20/e p, Ghai 8/e p 2808
Unilateral or bilateral clefts in cerebral hemisphere is called Schizencephaly.
138. d. All of the above  Ref: Nelson’s 20/e p 1126
Status Marmoratus
•• It is a congenital condition due to maldevelopment of corpus striatum, associated with choreoathetosis, in which the striate
nuclei have a marble-like appearance caused by altered myelination in the putamen, caudate and thalamus.
•• It results from acute total asphyxia in basal ganglia of full-term infants.
139. d. All of above  Ref: Nelson’s 20/e p 3010, Ghai 8/e p 590
•• The drug of choice for Guillain-Barre syndrome in a child is IVIg; Plasmapheresis is also used in refractory cases
•• In severe case, mechanical ventilation is also required to provide resp support.
140. c. More dangerous than decrebrate lesion  Ref: Nelson’s 20/e p 1453
Discussing about the options one by one,
a. True Decorticate posturing indicates damage to areas of brain including cerebral hemispheres, internal capsule, thalamus,
midbrain and red nucleaus
b. True
c. False While decorticate posturing is an ominous sign of severe brain damage, decerebrate posturing is usually indicative of
more severe damage at the rubrospinal tract, indicating a lesion lower in the brainstem.
d. True Patients with decorticate posturing present with the arms flexed, or bent inward on the chest, the hands are clenched
into fists, and the legs extended and feet turned inward.

Decorticate posturing, with elbows, wrists and fingers flexed, Decerebrate rigidity or abnormal extensor
and legs extended and rotated inward posturing
141. a. Malformation of vein of Galen  Ref: Nelson’s 20/e p 2293
142. c. Vein of Galen malformation  Ref: Nelson’s 20/e p 2293
143. a. Intubate and ventilate, b. Give 0.9% NaCl, e. Start antibiotic and order for CT scan  Ref: Nelson’s 20/e p 2951, Ghai 8/e
p 575
GCS less than 8 (in this case 1 + 1 + 3 = 5), is an indication of intubation and mechanical ventilation
IV fluid (Normal saline) should be started; Inotropes are not required in this case as blood pressure is maintained
As fever is there, antibiotics should be started; A CT scan should be done to look for underlying intracranial pathology.
144. c. IM injections and increased muscular activity increases the risk of paralytic polio  Ref: Nelson’s 20/e p 1554-1559
a. False Most of the cases of Polio are asymptomatic
b. False Acute flaccid paralysis is seen in Polio
c. True IM injections and increased muscular activity increases the risk of paralytic polio
d. False Pulse polio immunization is indicated in all children < 5 years age

145. c. Spina bifida  Ref: Nelson’s 20/e p 2802-2804


‘Banana sign' is seen in fetuses with spina bifida.
146. a. Salicylic acid  Ref: Nelson’s 20/e p 2951, Ghai 8/e p 575; Refer pretext of this chapter for details
Chapter 22
Behavioral Disorders
in Children
 DISORDERS OF HABIT
Question 1 M
•• Action or pattern of behavior that is repeated often. All of the following are true about
•• Habits are common in childhood and range from benign and transient (thumb sucking, nail the activity shown in picture except?
biting) to more problematic (trichotillomania, bruxism).

1. THUMB SUCKING M

•• It is a self-soothingQ behavior, common in infancy & is seen in 25% of children aged 2 yr


•• Thumb sucking beyond 5 yr may be associated with sequelaeQ (paronychia, anterior open bite)
•• Basic behavioral management is often effective treatment.

2. BRUXISM a. It is self soothing


b. It is common in infancy
•• Bruxism or teeth grinding is common (5–30% of children) & can begin in the 1st 5 yr of life
c. It may lead to dental malocclusion
•• It may be associated with daytime anxietyQ d. Early pharmacotherapy is effective
•• Persistent bruxism can manifest as muscular or temporomandibular joint painQ
•• Untreated bruxism can cause problems with dental occlusionQ.
3. TICS & STEREOTYPIC MOVEMENT DISORDER High Yield Points
•• Tics are sudden, rapid, recurrent, non-rhythmic motor movements or vocalizations •• Tourette disorder is characterized
•• Stereotypies are stereotyped, rhythmic, repetitive movements or patterns of speech, with by involuntary, rapid, repetitive,
lack of variation over time. single or multiple motor and/or
•• Tics can be suppressed when deeply engaged in a focused task or activity, or during sleep. vocal/phonic tics that wax and wane
•• Tics are worsened by anxiety, excitement, or exhaustion. in frequency but have persisted for
•• Decision to treat tics is made with the child and family based upon level of impairment and > 1 year since first tic onset
distress caused by the tics.
•• If tics are mild in severity, there may be no need for intervention
•• Referral to a behavioral treatment specialist should be considered when tics are distressing or
Question 2 M

functionally impairing. What abnormality does this child


•• The behavioural intervention with the strongest empirical support is habit reversal therapy (HRT) have?

4. PICA M

•• Definition: It is the persistent eating of non-nutritive, non-food substances over a period


of at least 1 month
•• It is more common in those with intellectual disability & autism spectrum disorders
•• Combined behavioral, social, and medical approaches are generally indicated.
5. BREATH HOLDING SPELLS
Results from Immaturity of autonomic nervous system
2 forms Pallid → Caused by reflex vagal bradycardia & asystole
Cyanotic → Result from prolonged expiration, apnea & intrapulmonary shunting
a. Bruxism
Triggers Injury, anger, frustration b. Thumb sucking
Clinical features Usually begins around 6-18 monthsQ c. Pica
Episodes usually start with a cry & progress to apnea d. Breath holding spell
Syncope, tonic posturing & seizures may occur
Treatment Reassurance-Not to provide secondary gain, otherwise, episodes are reinforced
Rx of coexisting Iron deficiency Anemia (IDA) is needed, if present, as spells are made
worse by iron deficiency anemia
High Yield Points
Atropine or anti epileptics many be needed for coexisting anoxic seizures that are •• Temper tantrums and breath-
recurrent & do not respond to other measures holding spells are common during
Prognosis Self limitingQ condition; patients outgrow this in a few years the 1st years of life and are age-typical
No increased risk of seizure disorder expressions of frustration or anger
514 Section 3: Systemic Pediatrics

6. NOCTURNAL ENURESIS (NE)


Review of Pediatrics and Neonatology

L at e s t U p d at e s
Intranasal desmopressin use is no Definition Occurrence of involuntary voiding at night after 5 yr age M
longer advocated for Nocturnal Types Primary (more common)Q & secondary
enuresis, due to the risk of
hyponatremia & convulsions with this Epidemiology •• 60% of children with NE are boysQ
formu­lation; Oral Desmopressin is •• Family History is positive in 50% cases
now pre­ferred •• If 1 parent has NE → each child has 44% risk of NE
•• If both parents had NE → each child has 77% risk of NE
Treatment Reassure parents-self limited
•• Fluid intake restricted after 6 pm
High Yield Points •• Caffeine & sugar avoided after 4 pm
•• Child to void at bedtime
•• Treatment of choice for Noc­turnal
enuresis is Alarm therapy Simplest initial measure → motivational therapy (‘star chart’)
•• Drug of choice for Nocturnal
Conditioning therapy- Alarm therapy (success 30-60%): Lower relapse rate than drugs
enuresis is Oral Desmopressin
Pharmacologic Rx – 2nd line, not curative
•• Oral Desmopressin tablet at bed time (effective in 40%)
•• Rx resistant/ overactive bladder-oxybutynin (anticholinergic)
High Yield Points •• 3rd line-Imipramine
Combination of alarm & desmopressin more effective than alone
•• Hallmark of Autism is aberrant
development of social Prognosis NE ceases spontaneously in 15% of involved children every year (Adults → < 1%)
communication & impaired ability
to engage in reciprocal interaction  BEHAVIORAL DISORDERS
•• In Rett syndrome, brain is the only
organ that is decreased in size
1. AUTISTIC SPECTRUM DISORDER
compared to height
Definition Persistent impairment in reciprocal social communicationQ & interaction and
restricted, repetitive patternsQ of behavior or interest
Includes Asperger disorder, childhood disintegrative disorder, Pervasive developmental
disorder, Not otherwise specified (PDD-NOS)
Risk factors •• Closer spacing of pregnancies; instead of: Advanced maternal or paternal age
•• Extremely preterm birth (<26 weeks)
•• Family members with learning/psychological problems
•• Antenatal exposure to thalidomide, valproate, organophosphate, Rubella
DSM-5 diagnostic •• Persistent deficit in social communication & interaction
criteria •• Restricted, repetitive patterns of behavior/interest or activities
•• Symptoms must be present in early developmental period
•• Symptoms cause clinically significant social impairment
•• Not better explained by ID or GDD
Screening tool M-CHAT (Modified Checklist for autism in toddlers) for 16–30 months age
Treatment Cognitive behavior therapy
Drugs used to target comorbid conditions like Atomoxetine for hyperactivity
Intranasal oxytocin: an upcoming novel therapy for Autism
Question 3
What is the disease in this 4-year-
old girl with stereotypic hand
2. RETT SYNDROME
wringing movements, microcephaly Genetics X linked dominantQ inheritance; primarily affects females M
& inability to walk?
Mutations involving MECP2 (Chr Xq28), CDKL5 (Chr Xp22), Netrin G1 & Fox G1 genes
Hallmark A period of normal development followed by regression of speech &
development of stereotypic hand movements
Criteria for •• Period of normal development (0-6m)
diagnosis •• Deceleration of head growthQ (3m-4yr)
•• Loss of purposeful hand movements (9m-2y)
•• Classic stereotypic hand movement (1-3 yr)
•• Gait or posture dyspraxia (2-4 yr)
a. Autism
b. Rett syndrome Other •• Seizures (in 30-80%), Breathing irregularities, poor weight gain
c. Asperger syndrome abnormalities •• Bruxism, sleep disturbance, night laughter
d. Attention deficit hyperactivity •• Scoliosis, dystonia, Arrhythmias, swallowing dysfunction, constipation, GERD
syndrome MRI brain Generalised atrophy of cerebral hemispheres
Chapter 22: Behavioral Disorders in Children 515

3. ADHD (ATTENTION DEFICIT HYPERACTIVITY DISORDER)

Section 3: Systemic Pediatrics


High Yield Points
Etiology/Risk •• Maternal smoking, alcohol, lead or mercury exposure
factor •• Genetic component-DAT1 & DRD4 genes •• ADHD is the most common neurobe-
•• Abnormal brain structure; Severe traumatic brain injury havioral disorder of childhood
•• Psychosocial family stresses •• Treatment of choice for ADHD is
• ↑ed incidence in epilepsy, tuberous sclerosis, neurofibromatosis Methylphenidate
Pathogenesis •• Disturbances in dopamine system may be related to onset of ADHD
DSM5 criteria for •• Persistent Inattention &/or Hyperactivity-Impulsivity that interferes with
diagnosis functioning or development
•• Present for at least 6 months, in 2 or more settings
•• Must begin before 12 yr age; Must not be secondary to another disorder
Subtypes: •• ADHD, predominantly inattentive type: includes cognitive impairment and is more
common in females.
•• ADHD, predominantly hyperactive-impulsive type, and
•• ADHD, combined type, are more commonly diagnosed in males.
Treatment 1st choice – presynaptic dopaminergic agonists-Methylphenidate or Amphetamine M
If psychostimulants unsuccessful, Atomoxetine, a noradrenergic reuptake inhibitor
Prognosis 60-80% children continue to experience it in adolescence
60% of adolescents have it in adulthood

 MENTAL RETARDATION INTELLECTUAL DISABILITY (ID)


•• Definition: Sub-average general intelligence, manifesting during early developmental period.

Grades of Mental Handicap


High Yield Points
Degree IQ level
Intelligence quotient (IQ)Q is mental
Mild ID 51–70 age divided by chromological age
Moderate ID 36–50 multiplied by 100

Severe ID 21–35
Profound ID 0–20

Causes
•• Prenatal: IEM, developmental defects, iodine deficiency chromosomal disorders, neuro­ High Yield Points
ectodermal dysplasia.
•• IQ of idiot: < 30
•• Maternal: Intrauterine infections, placental insufficiency, teratogen/radiation exposure
•• IQ of imbecile: 30-50
•• Natal: Birth injuries, HIE, intracerebral hemorrhage •• IQ of moron: 50-70
•• Post natal or acquired factors:
–– Infections of CNS
–– Head injuries, thrombosis
–– Hypoglycemia, Kernicterus
–– Hypoxia, Hypothyroidism
–– Malnutrition, child abuse.

Answer Keys for Image-Based Questions

Answers Explanations / Identifying features


1. Ans. d. Early pharmacotherapy This picture shows a child sucking her thumb
is effective

2. Ans. c. Pica This picture shows a child eating tar (charcoal); this is suggestive of Pica

3. Ans. b. Rett syndrome Stereotypic hand wringing movements (as shown), microcephaly & inability to walk in a 4 year old
girl, suggests a diagnosis of Rett syndrome
516 Section 3: Systemic Pediatrics
Review of Pediatrics and Neonatology

Questions
 NOCTURNAL ENURESIS 11. Pervasive disorder is associated with all except:
 (AIIMS Nov 2015)
1. A 6-year old-child was brought with complaints of
a. Repetitive behavior b. Impaired communication
bedwetting at night and not during day time. His urine
c. Impaired language d. Impaired cognition
specific gravity was 1.020 and other tests were normal.
What will you advice? (AIIMS May 2016)
12. True about Autistic disorder: (PGI Nov 2014)
a. Quality decrease in social interaction
a. Reassure
b. All affected children have subnormal intelligence
b. Consult a child Psychologist
c. Treatment should be targeted toward speech development
c. USG abdomen
d. Seen only after 3 yr of age
d. CT pelvis
e. Stereotyped patterns of behavior
2. Amongst various treatment modalities for nocturnal
13. True about Asperger syndrome: (PGI Nov 2014)
enuresis, the relapse rates have been observed to be
a. More common in girl
lowest for: (Bihar PG 2015)
b. Repetitive activity pattern
a. Desmopressin b. Imipramine
c. Subnormal intelligence is consistent feature
c. Bell alarm systems d. Oxybutinin
d. Severe language impairment is characteristic
3. Bedwetting is normal up to what age in girls and boys
14. True about autism: (NBE Based AI 2013-14 Pattern)
respectively? M  (JIPMER 2014)
a. Occurs in high economic strata
a. 3 years and 4 years b. 4 years and 5 years
b. Normal communication
c. 5 years and 5 years d. 5 years and 6 years
c. Starts before 2-3 years of age
4. A 5-year-old child presents with bedwetting. Rx of d. More common in girls
choice is: (NBE based 2013-14 Pattern)
15. Which of the following is not true about the autistic
a. No treatment b. Imipramine spectrum disorder? (AIPGMEE 2010)
c. Desmopressin d. Motivational therapy a. Impaired communication
5. Nocturnal enuresis is abnormal after: M  (JIPMER 2013) b. Impaired imagination
a. 2 years b. 3 years c. Language developmental delay
c. 4 years d. 5 years d. Vision problems
6. Nocturnal enuresis best t/t is: (Recent Question 2013) 16. Autism is: (TN PGMEE 2010)
a. Positive reinforcement b. Punishment a. Pervasive developmental disorder
c. Bed alarm d. Desmopressin b. Attention deficit hyperactive disorder
7. Lowest recurrence in nocturnal enuresis is seen with: c. Psychotic disorder
 (AIPGMEE 2008) d. Neurotic disorder
a. Bed alarms b. Desmopressin 17. A 3-year-old boy with normal developmental milestones
c. Imipramine d. Oxybutynin with delayed speech, has difficulty in communication
8. If behavioral therapy fails in the management of enuresis, and concentration. He is not making friends. Most
the pharmacological drug of choice for this case is: probable diagnosis is: (AIIMS May 2007)
 (DNB Jun 2007) a. Autism
a. Phenytoin b. Diazepam b. ADHD
c. Imipramine d. Alprax c. Mental retardation d. Specific learning disability
9. Which of the following nasal spray is very effective in 18. The following are characteristics of autism except: M 
control of enuresis: (COMEDK 2006) a. Onset after 6 years of age (AIPGMEE 06)
a. Pitressin b. Repetitive behavior
b. Desmopressin c. Delayed language development
c. Lypressin d. None of the above d. Severe deficit in social interaction
19. A 14-year-old boy has difficulty in expressing himself
 AUTISTIC SPECTRUM DISORDER in writing and makes frequent spelling mistakes, does
not follow instruction and cannot wait for his turn while
10. True about Asperger’s syndrome is: playing a game. He is likely to be suffering from:
a. Cognitively delayed (MAHA PGM CET 2015) a. Mental retardation (AIIMS Nov 2005)
b. Normal social interaction b. Lack of interest in studies
c. Language skills relatively intact c. Specific learning disability
d. Normal occupational functioning d. Examination anxiety
Chapter 22: Pediatric Respiratory Disorders 517

20. Infantile autism is characterized by: (PGI Dec 2004) 28. All of the following are essential features of ADHD

Questions
a. Impaired vision except: M  (Recent Question 2014)
b. Impaired Neurobehavioral development a. Lack of concentration b. Impulsivity
c. Impaired folate level c. Hyperactivity d. Mental retardation
d. A socioeconomic hazard e. Parenting 29. Consider the following behavioral conditions in a child
21. A 6-year-old child has history of birth asphyxia, does not of school age: (DNB Dec 2010)
communicate well, has slow mental and physical growth, 1. Fidgets with hands or squirms in seat
does not mix with people, has limited interests, and gets 2. Easily distracted by extraneous stimuli
widely agitated if disturbed, diagnosis is: (AIIMS Nov 2001) 3. Often has difficulty awaiting turn
a. Hyperkinetic child b. Autistic disorder 4. Does not seem to listen when spoken to directly
c. Attention deficit disorder d. Schizophrenia Which of the above diagnostic criteria are suggestive of
inattention (attention deficit) in this child?
 RETT SYNDROME a. 1 and 2 only b. 1 and 3 only
c. 2 and 4 only d. 1,2, 3 and 4
22. A female child who initially had normal development
30. A 10-year-old child is always restless, inattentive to
followed by regression of developmental milestones,
study and always wants to play outside. Parents are
presented with slow, hand wringing movements; she
extremely distressed. What would you advise?
also had microcephaly. What is the probable diagnosis?
a. It is a normal behavior (AIIMS Nov 2008)
a. Autistic spectrum disorder (Recent Question 2016)
b. Behavior therapy
b. Lysosomal storage disorder
c. It is a serious illness requires medical treatment
c. Rett syndrome d. Congenital infection
d. Needs change in environment
23. All of the following are characteristics of Rett syndrome
31. Drugs used in ADHD are: (PGI Dec 2008)
except: (AIIMS May 2013)
a. Atomoxetine b. Methylphenidate
a. Increased incidence of mental retardation
c. Dextro-amphetamine d. Quetiapine
b. Seizures
c. Abnormal dendritic morphology in cortical pyramidal e. Benzodiazepine
cells 32. A 9-year-old child disturbs other people, is destructive,
d. Macrocephaly interferes when two people are talking, does not follow
24. A 2-year-old girl child is brought to the outpatient instructions and cannot wait for his turn while playing a
department with features of hand wringing stereotype game. He is likely to be suffering from:
movements, impaired language & communication, a. Emotional disorders (AIIMS Nov 2005)
breath holding spells, poor social skills and deceleration b. Behavioral problems
of head growth after 6 months of age. The most likely c. No disorder
diagnosis is: M (AIIMS Nov 2003) d. Attention deficit hyperactive disorder
a. Asperger's syndrome 33. All of the following are essential features of attention
b. Rett syndrome deficit hyperactive disease (ADHD) except:
c. Fragile X-syndrome d. Calorad syndrome  (AIPGMEE 2004)
a. Lack of concentration b. Impulsivity
 ADHD c. Mental retardation d. Hyperactivity
34. A 9-year-old child is restless. He is hyperactive and
25. All are true about attention deficit hyperactivity disorder his teacher complaints that he does not listen to the
except: (PGI Nov 2017) teachings. Disturbs other students, he also shows less
a. In two third cases symptoms may continue in adulthood interest in playing. The likely diagnosis is:
b. For making the diagnosis, the behavioral changes must a. Cerebral palsy (AIIMS May 2002)
begin before 5 years of age b. Attention deficit hyperactive child
c. May be associated with mood disorder c. Delirium
d. Amphetamine may be used for treatment
d. Mania
e. More prevalent in boys
35. About Fragile X Syndrome, true are all except: M 
26. Drug not given in ADHD is: (JIPMER Nov 2016)
 (WB PGMEE 2009)
a. Clonidine b. Atomoxetine
a. Large testis b. Large nose
c. Methylphenidate d. Barbiturate
c. Large ears d. High arched palate
27. Which is true concerning attention deficit hyperactivity
disorder? (APPG 2014)
a. Impulsive behavior is a feature  OTHER BEHAVIORAL DISORDERS
b. Higher incidence in tic disorders
c. May respond to treatment with stimulants such as 36. All are habit disorders except: (NEET pattern Jan 2018)
a. Nail biting b. Thumb sucking
amphetamine
c. Temper tantrum d. Tics
d. All of the above
518 Section 3: Systemic Pediatrics

46. Early strict toilet training can result in: (Manipal 09)
Review of Pediatrics and Neonatology

37. Treatment of breath holding spells is: M 


a. Pyridoxine b. Zinc (DNB June 2013) a. Nocturnal enuresis b. Encopresis
c. Iron d. Molybdenum c. Night terror d. Temper tantrums
38. Pica refers to: M  (Recent Question 2013)
a. IU sucking b. Thumb sucking  MENTAL RETARDATION
c. Foreign object being put in the mouth
d. None of above 47. What is the cut off level of imbecile?
 (FMGE Nov 2017)
39. Which of the following childhood disorder improves
a. 0-30 b. 30-49
with increase in age? (DNB Dec 2013, MP 2008)
c. 50-69 d. >70
a. Conduct disorder b. Emotional problems
48. Formula to calculate intelligence quotient (IQ):
c. Temper tantrum d. Sleep disorder
a. Chronological age/Mental age ××100 (JIPMER 2017)
40. The following is not true regarding bruxism: b. Mental age–Chronological Age × 100
a. Associated with day time anxiety (TN PGMEE 2013) c. Mental age/Chronological age × 100
b. If untreated, can result in dental malocclusion d. Mental age + Chronological × 100
c. Condition improves if proper deworming is done 49. Range of IQ in mild mental retardation is:
d. Chronic bruxism leads to temporomandibular joint pain
 (Recent Question 2015)
41. Which of the following is not true regarding “thumb a. < 50 b. 50-70
sucking”? (DNB Dec 2007, JIPMER 2005) c. 70-100 d. >100
a. Is a sign of insecurity
50. Preventable cause of mental retardation is:
b. Pleasurable sensation
 (Recent Question 2013)
c. Leads to dental problems
a. Downs b. Phenylketonuria
d. Seen only in child < 4 years of age
c. Cretinism d. Cerebral palsy
42. Waking up at night, screaming with fear, at 2.5 years 51. Which is most common genetic cause of mental retar­
age, is generally a manifestation of: (PGI June 2003) dation? (Recent Question 2012)
a. Organic illness a. Tuberous sclerosis
b. Normal development pattern b. Cri-du-chat syndrome
c. Castration anxiety c. Fragile-X-syndrome
d. Separation anxiety d. Angel's syndrome
43. True regarding breath holding spells is all except: M  52. A 5-year-old child is assessed to have developmental
a. Antiepileptic treatment is necessary (AIIMS May 2002) age of one year. His developmental quotient would be:
b. Atropine is sometimes used a. 100 b. 80 (DPG 10)
c. Attacks of cyanosis can occur c. 60 d. 20
d. Occurs between 6 months to 5 years
53. Conditions associated with mental retardation are:
44. Treatment for breath holding spells in a child is:  (PGI June 2008)
a. Give extra care and love to the child (PGI Dec 2001) a. Trisomy 21 b. Fragile X
b. Inflicting painful stimulus at the beginning of the attack c. Homocystinuria d. Phenylketonuria
c. Do not give attention to the child e. Tuberous sclerosis
d. Fulfill all the wishes of the child to prevent the attack 54. Mild MR features are: (PGI June 2006)
e. Low dose barbiturates a. Present in 5–10% population
45. A 6-year-old boy presents with complaints of hyper­acti­ b. Higher incidence in low socioeconomic group
vity, excessive shoulder shrugging, throat clearing and c. Presents in 2 years
eye blinking. During examination, he is not co­operative, d. Genetic background present
uses obscene words and runs around in the consultation e. Specific cause found in 75% cases
room. What is the most probable diagnosis? 55. A male child of 15 years with a mental age of 9 years has
a. Attention deficit hyperactivity disorder (JIPMER 2009) an IQ of: (AI 2003)
b. Tic disorder a. 50 b. 60
c. Psychomotor seizures c. 70 d. 80
d. Childhood psychosis
Chapter 22: Behavioral Disorders in Children 519

Answers with Explanations


Answers with Explanations
 NOCTURNAL ENURESIS
1. b. Consult a child Psychologist;  Ref: Nelson’s 20/e p 2585
In this case, the child has nocturnal enuresis, with normal findings on urine examination (normal urine specific gravity is
1.016-1.022)
Nocturnal enuresis
••
It refers to the occurrence of involuntary voiding at night after 5 yr.
••
If no daytime symptoms and physical examination and urinalysis are normal, further evaluation for urinary tract pathology is
not warranted.
•• An ultrasonogram is done in an older child (usually ≥ 7 yrs) with enuresis or in those who do not respond to therapy.
•• Active treatment should be avoided in children younger than 6 yr of age.
•• Psychologic therapy includes motivational therapy (star charts), Conditioning therapy (auditory or vibratory alarm
attached to a moisture sensor in underwear) and self-hypnosis.
•• It helps the child deal with enuresis psychologically and motivate the child to void at night if he or she awakens with a full bladder.
•• Pharmacologic therapy is intended to treat the symptom of enuresis and thus is regarded as second line and is not curative.

2. c. Bell alarm systems  Ref: Nelson’s 20/e p 2585-2586, Ghai 8/e p 468
Bell alarm system has a reported success of 30-60%, although the relapse rate is significant.
Direct comparisons of the bell alarm system with pharmacologic therapy favor the former because of lower relapse rates,
although initial response rates are equivalent.
3. c. 5 years and 5 years  Ref: Nelson’s 20/e p 2585-2586, Ghai 8/e p 468
Girls typically acquire bladder control before boys and bowel control is usually achieved before bladder control.
Nocturnal enuresis refers to the occurrence of involuntary voiding at night after 5 yr, in both, boys and girls.
4. d. Motivational therapy  Ref: Nelson’s 20/e p 2585-2586, Ghai 8/e p 468
5. d. 5 years  Ref: Nelson’s 20/e p 2585-2586, Ghai 8/e p 468
6. c. Bed alarm  Ref: Nelson’s 20/e p 2585-2586, Ghai 8/e p 468
7. a. Bed alarms  Ref: Nelson’s 20/e p 2585-2586, Ghai 8/e p 468
Lowest recurrence rate is with combination therapy (bed alarm + pharmacotherapy), followed by bed alarm.
8. c. Imipramine  Ref: Nelson’s 20/e p 2585-2586, Ghai 8/e p 468
9. b. Desmopressin  Ref: Nelson’s 20/e p 2585-2586, Ghai 8/e p 468
•• Desmopressin ia a synthetic analog of antidiuretic hormone (ADH) that reduces urine production overnight
•• This medication is FDA-approved and is available as a tablet, with a dosage of 0.2-0.6 mg at bedtime
•• In the past a nasal spray was used, but some children experienced hyponatremia and convulsions with this formulation and
the nasal spray is no longer recommended for nocturnal enuresis.

 AUTISTIC SPECTRUM DISORDER


10. c. Language skills relatively intact  Ref: Nelson’s 20/e p 177-182, Ghai 8/e p 61-70
11. d. Impaired cognition  Ref: Nelson’s 20/e p 210, 2067,  Ref: Ghai Pediatrics 8/e p 61
Autistic spectrum disorders (ASD)
•• It includes childhood autism, atypical autism, Asperger disorder, childhood disintegrative disorder and pervasive
developmental disorder not otherwise specified
•• Essential features are persistent impairment in reciprocal social communication and restricted, repetitive patterns of
behavior or interests
•• Language impairment can range from nonverbal speech to 'phrase speech' to fluent speech
•• Intelligence can vary from intellectual impairment to superior intellectual functioning in select areas
•• Motor deficits, including odd gait, clumsiness, dyspraxia, and walking on tiptoes; Self-injury may occur;
••
Epilepsy is a common comorbidity, and any type of seizure may be observed in ASD.
12. a. Quality decrease in social interaction and e. Stereotyped patterns of behavior  Ref: Nelson’s 20/e p 172- 182
ASD symptoms are typically recognized during the 2nd yr of life but can been seen earlier than 12 months, if it is severe.
520 Section 3: Systemic Pediatrics

13. b. Repetitive activity pattern  Ref: Nelson’s 20/e p 172-182, Ghai 8/e p 61
Review of Pediatrics and Neonatology

Discussing about the options one by one


a. More common in girl Autistic spectrum disorder is more common in boys
b. Repetitive activity pattern True
c. Subnormal intelligence is consistent feature Intelligence may or may not be normal
d. Severe language impairments is characteristic Language impairment may or may not be there

14. c. Starts before 2-3 years of age  Ref: Nelson’s 20/e p 172-182, Ghai 8/e p 61
Autism is more common in lower socioeconomic strata, more common in boys; Communication is impaired in Autism.
15. d. Vision problems  Ref: Nelson’s 20/e p 172-182, Ghai 8/e p 61
16. a. Pervasive developmental disorder  Ref: Nelson’s 20/e p 172-182, Ghai 8/e p 61
17. a. Autism  Ref: Nelson’s 20/e p 172-182, Ghai 8/e p 61
The given scenario where the child is having delayed speech and difficulty in communication and concentration and is not
making friends, suggests a diagnosis of Autistic spectrum disorder.
18. a. Onset after 6 years of age  Ref: Nelson’s 20/e p 172-182, Ghai 8/e p 61
19. c. Specific learning disability  Ref: Nelson’s 20/e p 172-182, Ghai 8/e p 61
Difficulty in expressing himself in writing and making frequent spelling mistakes, point towards the diagnosis of Dyslexia, a specific
learning disability.
20. b. Impaired Neurobehavioral development and d. A socioeconomic hazard  Ref: Nelson’s 20/e p 172-182, Ghai 8/e p 61
Infantile autism is a socioeconomic hazard; Impaired Neurobehavioral development is seen.
21. b. Autistic disorder  Ref: Nelson’s 20/e p 172-182, Ghai 8/e p 61

 RETT SYNDROME
22. Ans c. Rett syndrome   Ref: Nelson’s 20/e p 2916-2917
23. Ans d. Macrocephaly  Ref: Nelson’s 20/e p 2916-2917
Deceleration of head growth leading to acquired microcephaly is a feature of Rett syndrome.
24. b. Rett’s syndrome  Ref: Nelson’s 20/e p 2916-2917
The given clinical picture of a girl with hand wringing stereotype movements, impaired language and communication, breath
holding spells, poor social skills and deceleration of head growth is consistent with the diagnosis of Rett syndrome.

 ADHD
25. b. For making the diagnosis, the behavioral changes must begin before 5 years of age  Ref: Nelson’s 20/e p 200-203
Attention deficit hyperactivity disorder (ADHD)
It is Persistent in attention and/or Hyperactivity-Impulsivity, that interferes with functioning or development, present for at least 6
months, in 2 or more settings, beginning before 12 yr age.
Must not be secondary to another disorder.
3 subtypes:
•• ADHD, predominantly inattentive type: includes cognitive impairment & is more common in females.
•• ADHD, predominantly hyperactive-impulsive type, and
•• ADHD, combined type, are more commonly diagnosed in males.
About 60-80% of children continue to experience symptoms in adolescence & up to 60% of adolescents exhibit ADHD symptoms
into adulthood.
26. d. Barbiturate  Ref: Nelson’s 20/e p 204
Drugs useful in the treatment of ADHD include methylphenidate, dexmethylphenidate, dextroamphetamine, atomoxetine,
tricyclic antidepressants like imipramine and agonists like clonidine and guanfacine
27. d. All of the above  Ref: Nelson’s 20/e p 200, Ghai 8/e p 59-60
Attention-deficit/hyperactivity disorder (ADHD) is characterized by inattention, including increased distractibility, difficulty
sustaining attention; poor impulse control decreased self-inhibitory capacity; motor overactivity and restlessness. Diagnoses
associated with ADHD behaviors are Fragile X syndrome, Fetal alcohol syndrome, Pervasive developmental disorders, Obsessive-
compulsive disorder and Gilles de la Tourette syndrome (tic disorder).
28. d. Mental retardation  Ref: Nelson’s 20/e p 2067, Ghai 8/e p 59-60
29. c. 2 and 4 only  Ref: Nelson’s 20/e p 2067, Ghai 8/e p 59-60
Easily distracted by extraneous stimuli and does not seem to listen when spoken to directly are suggestive of inattention.
Chapter 22: Behavioral Disorders in Children 521

30. c. It is a serious illness requires medical treatment  Ref: Nelson’s 20/e p 200, Ghai 8/e p 59-60

Answers with Explanations


The given history suggests ADHD; For ADHD, Stimulants have been more effective than behavioral treatments used alone.
31. a. Atomoxetine, b. Methylphenidate, c. Dextro-amphetamine  Ref: Nelson’s 20/e p 2067, Ghai 8/e p 59-60
32. d. Attention deficit hyperactive disorder  Ref: Nelson’s 20/e p 2067, Ghai 8/e p 59-60
33. c. Mental retardation  Ref: Nelson’s 20/e p 200, Ghai 8/e p 59-60
Affected children with ADHD commonly experience academic underachievement, problems with interpersonal relationships with
family members and peers, and low self-esteem. But as such, mental retardation is not an essential feature of ADHD.
34. b. Attention deficit hyperactive child  Ref: Nelson’s 20/e p 2067, Ghai 8/e p 59-60
35. b. Large nose  Ref: Nelson’s 20/e p 2067, Ghai 8/e p 59-60
Elongated face with large ears, high arched palate and large testis (macro-orchidism) are seen in fragile X-syndrome.

 OTHER BEHAVIORAL DISORDERS


36. c. Temper tantrum  Ref: Nelson’s 20/e p 144-175
•• Habits involve an action or pattern of behavior that is repeated often.
•• Habits are common in childhood and range from benign and transient (thumb sucking, nail biting) to more problematic
(trichotillomania, bruxism).
•• Temper tantrums and breath-holding spells are common during the 1st years of life and are age-typical expressions of
frustration or anger.
Tics:
•• Sudden, rapid, recurrent, non-rhythmic motor movements or vocalizations.
•• Tics can be suppressed when deeply engaged in a focused task or activity, or during sleep.
•• Tics are worsened by anxiety, excitement, or exhaustion.
•• Decision to treat tics is made with the child and family based upon level of impairment and distress caused by the tics.
•• If tics are mild in severity, there may be no need for intervention.
•• Referral to a behavioral treatment specialist should be considered when tics are distressing or functionally impairing.
•• The behavioral intervention with the strongest empirical support is habit reversal therapy.
37. c. Iron  Ref: Nelson’s 20/e p 139, Ghai 8/e p 58
Iron deficiency with or without anemia may be present and some children with breath-holding spells respond to iron therapy.
38. c. Foreign object being put in the mouth  Ref: Nelson’s 20/e p 139, Ghai 8/e p 58
39. c. Temper tantrum  Ref: Nelson’s 20/e p 175, Ghai 8/e p 58
Temper tantrums and breath-holding spells are common during the first years of life and are age-typical expressions of frustration
or anger; They improve as the child grows up.
40. c. Condition improves if proper deworming is done  Ref: Nelson’s 20/e p 175, Ghai 8/e p 58
Bruxism or teeth grinding can begin in the first 5 yr of life, and may be associated with daytime anxiety. Persistent bruxism can
manifest as muscular or temporomandibular joint pain; Untreated bruxism can cause problems with dental occlusion.
41. d. Seen only in child < 4 years of age  Ref: Nelson’s 20/e p 144, Ghai 8/e p 59
Thumb Sucking
•• Thumb sucking is a self-soothing behavior, common in infancy and is seen in 15% of children aged 5 yr
•• Thumb sucking beyond 5 yr may be associated with sequelae (paronychia, anterior open bite).
•• Management includes encouraging parents to ignore thumb sucking and focus on praising the child for substitute behaviors.

42. d. Separation anxiety  Ref: Nelson’s 20/e p 144, Ghai 8/e p 59
•• Anxiety disorders are the most common psychiatric disorders of childhood
•• Separation anxiety disorder (SAD) is one of the most common childhood anxiety disorders
•• Separation anxiety is developmentally normal when it begins about 10 months of age and tapers off by 18 mo
•• By 3 yr of age, most children can accept the temporary absence of their mother or primary caregiver.
43. a. Antiepileptic treatment is necessary  Ref: Nelson’s 20/e p 175; Refer pretext of this chapter for details;
44. c. Do not give attention to the child  Ref: Nelson’s 20/e p 175, Ghai 8/e p 59
For management of breath-holding spells, parents should be advised to intercede early in defiant behavior by calmly placing the
child in timeout for a period of time approximating 1 min for each year of age.
45. b. Tic disorder  Ref: Nelson’s 20/e p 140-142, Ghai 8/e p 59
Tourette disorder (TD), persistent motor or vocal tic (PTD), and are characterized by involuntary, rapid, repetitive, single or
multiple motor and/or vocal/phonic tics that wax and wane in frequency but have persisted for > 1 year since first tic onset.
46. b. Encopresis  Ref: Nelson’s 20/e p 1807
Encopresis is involuntary passage of feces into inappropriate places at least once a month for 3 consecutive months, after a
chronologic or developmental age of 4 yr is reached. Early strict toilet training can cause it.
522 Section 3: Systemic Pediatrics

 MENTAL RETARDATION
Review of Pediatrics and Neonatology

47. b. 30-49
An Idiot has IQ < 30, an imbecile has IQ of 30-50, a moron has an IQ of 50-70
48. c. Mental age/chronological age×××100  Ref: Nelson's 20/e p 216-222
Intelligence Quotient (IQ) = Mental age/Chronological age × 100
49. b. 50-70  Ref: Nelson's 20/e p 216-222, Ghai 8/e p 584-585
Categories of intellectual disability: mild (IQ 50-70), which is more associated with environmental influences & severe (IQ <50),
which is more frequently linked to biologic and genetic causes.
50. c. Cretinism  Ref: Nelson's 20/e p 216-222, Ghai 8/e p 584-585
Hypothyroidism is the most important preventable or treatable cause of intellectual disability in children.
51. c. Fragile-X-syndrome  Ref: Nelson's 20/e p 216-222, Ghai 8/e p 584-585
52. d. 20  Ref: Nelson's 20/e p 216-222, Ghai 8/e p 584-585
DQ = Developmental age x 100/Actual age ⇒ So, in this case, DQ = 1 x 100/5 = 20
53. a. Trisomy 21, b. Fragile-X, c. Homocystinuria, d. Phenylketonuria, e. Tuberous sclerosis  Ref: Nelson’s 20/e p 216-222
54. b. Higher incidence in low socioeconomic group, d. Genetic background present  Ref: Nelson’s 20/e p 216-222;
Mild intellectual disability (Note: New name for Mental retardation is Intellectual disability).
••
It is more likely in offspring of women who have not completed high school than those who have graduated.
••
This is presumably a consequence of both genetic and socioeconomic (poverty, malnutrition) factors.
••
Specific causes are identifiable in <50% of affected; Familial clustering is common.
••
Common causes include syndromes (velocardiofacial, William, Noonan syndrome), intrauterine growth restriction,
prematurity, perinatal insults and intrauterine exposure to drugs of abuse (e.g. alcohol)
55. b. 60  Ref: Nelson’s 20/e p 216-222, Ghai 8/e p 584-585
IQ = Mental age x 100/Chronological age ⇒ So, in this case, IQ = 9 x 100/15 = 60
Chapter 23
Musculoskeletal
System
 DISORDERS INVOLVING MUSCLES
Question 1 M

The finding shown in this picture


A. MUSCULAR DYSTROPHIES
is commonly seen in which of the
Inherited disorders of skeletal muscle with progressive muscle damageQ following disorders?
1. DUCHENNE MUSCULAR DYSTROPHY (DMD) M

Epidemiology It is the most common hereditary neuromuscular disease affecting all races

Genetics X-linked recessive: Xp21 (Dystrophin gene), one of the largest genes is affected
Clinical Progressive weakness, developmental delay & intellectual impairment seen;
features Enlargement of calves (pseudohypertrophy) & wasting of thigh muscles seen;
After calves, next most common site of muscular hypertrophy is tongue;
Gowers' sign is often evident by age 3 years but is fully expressed by 5-6 years
A Trendelenburg gait, or hip waddle, appears at this time. a. Myotonic dystrophy
b. Becker’s muscular dystrophy
Diagnosis Serum creatine kinase (CK) level is consistently greatly elevated in DMD, (even in c. Facioscapulohumeral dystrophy
presymptomatic stages), to 15,000–35,000 IU/L (normal <160 IU/L); d. Congenital muscular dystrophy
PCR for dystrophin gene mutation confirms the diagnosis;
If PCR is normal, but clinical suspicion is high, muscle biopsy with dystrophin
immunocytochemistry done
Biopsy detects the 30% of cases that do not show a PCR abnormality.
L at e s t U p d at e s
Complications Contractures, cardiomyopathy, malignant hyperthermia after anesthesia A potential treatment still under
Treatment Treatment of cardiac decompensation and pulmonary infections; investigation for Duchenne muscular
dystrophy is injection of antisense
Nutritional management; Physiotherapy
oligonucleotide drugs (drisapersen
Glucocorticoids decreases rate of apoptosis & decelerate myofiber necrosis and eteplirsen), that bind RNA & skip
Prognosis Death occurs usually at about 18–20 yr of age. Causes of death are respiratory (bridge) over the defective exon, thus
failure during sleep, intractable heart failure, pneumonia, aspiration producing a shorter but potentially
functional dystrophin protein

Gowers’ Sign M

•• It is a medical sign that indicates weakness of proximal muscles, esp of lower limb.
•• The sign describes a patient that has to use their hands & arms to “walk” up their own body Question 2
from a squatting position due to lack of hip and thigh muscle strength.
This sign is seen in all the following
•• It can be seen in Duchenne or Becker muscular dystrophy, centronuclear myopathy, disorders EXCEPT?
myotonic dystrophy & various other conditions associated with proximal muscle weakness

2. BECKER MUSCULAR DYSTROPHY (BMD)


•• Similar to DMD, but clinically it follows a milder & more protracted course.
•• In BMD, onset of weakness is later & boys remain ambulatory until late adolescence.
•• Calf pseudohypertrophy, cardiomyopathy & elevated serum CK are seen;
•• Learning disabilities are less common; Death often occurs in mid to late 20s.
3. FACIOSCAPULOHUMERAL MUSCULAR DYSTROPHY (LANDOUZY-DEJERINE DISEASE)
•• Autosomal dominant with genetic anticipation. a. Duchenne muscular dystrophy
•• Earliest & most severe weakness in facial & shoulder muscles (scapular winging). b. Becker’s muscular dystrophy
•• Hearing loss and retinal vasculopathy are associated features. c. Landouzy-Dejerine disease
•• Flattening or concavity of deltoid contour; with wasted biceps & triceps. d. Spinal muscular atrophy
524 Section 3: Systemic Pediatrics

•• Muscles of hip & thighs also eventually atrophy & Gowers' sign seen.
Review of Pediatrics and Neonatology

High Yield Points


•• Calf hypertrophy is not a feature; foot drop may be seen;
•• Unlike most other muscular dys­ •• Lumbar lordosis & kyphoscoliosis are common complications.
trophies, asymmetry of weakness is
common in Facioscapulohumeral Cardiac Involvement in Muscular Dystrophies
dystrophy
Dilated cardiomyopathy Arrhythmias/ conduction abnormalities
•• Duchenne muscular dystrophy •• Myotonic dystrophy
•• Becker muscular dystrophy •• Emery-Dreifuss muscular dystrophy
•• Sarcoglycanopathies •• Sarcoglycanopathies
•• Limb-girdle muscular dystrophy •• Limb-girdle muscular dystrophy

High Yield Points 4. EMERY-DREIFUSS MUSCULAR DYSTROPHY


•• Also called scapuloperoneal or scapulohumeral muscular dystrophy
•• Facial weakness in myotonic •• X-linked recessive inheritance; contractures of elbows & ankles develop early
dystrophy leads to an inverted
V-shaped upper lip, while, the 5. MYOTONIC MUSCULAR DYSTROPHY
mouth in facioscapulohumeral •• It is the second most common muscular dystrophy
dystrophy is rounded and appears •• Autosomal dominant trait; caused by CTG trinucleotide expansion on chr 19q13
puckered because the upper and •• Dysfunction in multiple organ systems like GIT, uterus, endocrinopathies, immunologic
lower lips protrude.
deficiencies, cataracts, & increased risk for malignancies.
•• Myotonia, i.e. a very slow relaxation of muscles after contraction seen (> 5 yrs age).
6. MYOTONIA CONGENITA (THOMSEN DISEASE)
Question 3 •• It is a chloride channelopathy, characterized by weakness and generalized muscular
Identify the disease this child is hypertrophy so that affected children resemble ‘bodybuilders'.
suffering from?
7. CONGENITAL MUSCULAR DYSTROPHIES
•• It includes several distinct diseases, with autosomal recessive inheritance that have
severe involvement at birth but often follow a more benign clinical course;
•• There is a high association with brain malformations like lissencephaly/pachygyria and
polymicrogyria, often complicated by severe epilepsy.

B. CONGENITAL MYOPATHY
a. Duchenne muscular dystrophy
Congenital myopathies are a heterogeneous group of non-progressive congenital neuro­
b. Myotonic dystrophy
c. Facioscapulohumeral dystrophy muscular disorders which have subcellular abnormalities that can be demonstrated only by
d. Spinal muscular atrophy muscle biopsy.

Important congenital myopathies and their clinical features:


Neonatal Severe with Extraocular
High Yield Points Hypotonia & Neonatal Facial muscular
Myopathy weakness death weakness Ptosis weakness
•• Paramyotonia is a temperature-
related myotonia that is aggra­ Central core disease + - ± - -
vated by cold and alleviated by
warm external temperatures. Nemaline myopathy + + + - -

Myotubular (centro- + + + + +
nuclear) myopathy

Congenital fiber-type + ± ± - +
disproportion

C. SPINAL MUSCULAR ATROPHY (SMA)

What is it?
SMA is degenerative disease of motor neurons that begin in fetal life and continue to be
progressive in infancy and childhood.

Genetic Basis
Autosomal recessive mendelian trait due to defective SMN gene on chr 5q
The function of survivor motor neuron (SMN) gene is to arrest apoptosis of motor neuroblasts.
Chapter 23: Musculoskeletal System 525

Classification

Section 3: Systemic Pediatrics


High Yield Points
•• SMA type 0: A severe fetal form that is usually fatal in the perinatal period
•• Tongue fasciculation is seen in
•• SMA type 1 or Werdnig-Hoffmann disease: a severe infantile form
spinal muscular atrophy (SMA)
•• SMA type 2 or late infantile and more slowly progressive form
•• SMA type 3 or Kugelberg-Welander disease: chronic or juvenile form

Clinical Features
•• Hypotonia, generalized weakness; thin muscle mass.
•• Absent tendon stretch reflexes; Diaphragmatic involvement is late. High Yield Points
•• Involvement of tongue, face & jaw muscles, with sparing of extraocular muscles & sphincters.
•• In Klippel-Feil Syndrome, a triad
•• Fasciculations are a specific sign of denervation of muscle, best seen in tongue.
of short neck, low hairline and
restriction of neck motion in a
Diagnosis
patient with multiple coalitions of
•• Serum creatine kinase level is usually normal. cervical vertebrae is seen.
•• Definitive diagnostic test is detection of SMN gen defect; Prenatal diagnosis possible; •• 30–40% of these patients have
urinary tract abnormalities of like
•• Muscle biopsy is used in patients showing equivocal or negative genetic findings.
double collecting systems, renal
aplasia and horseshoe kidney
 IMPORTANT SKELETAL DISORDERS IN CHILDREN
A. RICKETS M

What is it?
It is a disease of growing bones that is due to defective mineralization of the bone matrix at the
growth plate and occurs in children only before the fusion of epiphyses. Question 4 M

Identify the disease this child has?


Important Causes of Rickets
Vitamin D disorders Renal losses
•• Nutritional vitamin D deficiency •• Hypophosphatemic rickets
•• Congenital vitamin D deficiency •• Overproduction of phosphatonin by Tumors
•• Secondary vitamin D deficiency •• Tumor-induced rickets
•• Malabsorption •• McCune-Albright syndrome
•• Increased degradation •• Epidermal nevus syndrome
•• Decreased 1,25 hydroxylase •• Neurofibromatosis
•• Vitamin D – dependent rickets type 1 •• Fanconi syndrome
•• Vitamin D – dependent rickets type 2 •• Dent disease
•• Chronic renal failure •• Distal renal tubular acidosis
a. Rickets b. Scurvy
Calcium deficiency Phosphorus deficiency c. Marasmus d. Kwashiorkor
•• Low intake •• Inadequate intake
•• Premature infants •• Premature infants (rickets of prematurity)
•• Malabsorption •• Aluminium containing antacids
•• Dietary inhibitors of calcium absorption

Question 5 M

The following is the wrist X-ray of a


Clinical Features of Rickets child with bowing of legs & failure
to thrive. What is the diagnosis?
•• Failure to thrive, Listlessness •• Tetany
•• Protruding abdomen •• Seizures
General Hypocalcemic
•• Muscle weakness •• Stridor due to laryngeal spasm
•• Fractures

•• Craniotabes •• Wrist widening


•• Frontal bossing •• Double malleoli
Head •• Large anterior fontanelle Extremities •• Valgus or varus deformities
•• Delayed fontanel closure •• Windswept deformity
•• Delayed dentition •• Anterior bowing of tibia and femur
a. Rickets
•• Rachitic rosary •• Scoliosis b. Scurvy
Chest •• Harrison’s sulcus Spine •• Kyphosis c. Osteopetrosis
•• Respiratory infections •• Lordosis d. Osteogenesis imperfecta
526 Section 3: Systemic Pediatrics

Radiology
Review of Pediatrics and Neonatology

Question 6
What is the name of this clinical •• Rachitic changes are most easily visualized on AP radiograph of the wrist
finding that can be seen in Rickets? •• At metaphysis – fraying, cupping, widening or splaying is seen.
•• Other features – Generalized rarefaction, coarse trabeculation of diaphysis, fractures, etc.

Biochemical Findings in Nutritional Rickets


•• Hypocalcemia (variable finding due to raised PTH)
•• Hypophosphatemia (PTH induced renal loss and decrease in absorption)
•• 25 hydroxy Vit D levels: low, normal or high, depending on the underlying etiology
•• Metabolic acidosis secondary to PTH-induced renal bicarbonate wasting may be seen

Laboratory Findings in Rickets due to Important Causes


a. Cubitus varus
Disorder Ca P PTH 25-OHDs 1,25-(OH)2Ds Alk phos
b. Double malleoli
c. Windswept deformity Vitamin D deficiency N/↓ ↓ ↑ ↓ ↓/N/↑ ↑
d. Plagiocephaly
VDDR, type 1 N/↓ ↓ ↑ N ↓ ↑

VDDR, type 2 N/↓ ↓ ↑ N ↑ ­↑ ↑

Chronic renal failure N/↓ ↑ ↑ N ↓ ↑

Dietary phosphate deficiency N ↓ N/↓ N ↑ ↑

Hypophosphatemic rickets N ↓ N N Relatively ↓ ↑


High Yield Points   M
Dietary Ca deficiency N/↓ ↓ ↑ N ↑ ↑
•• Rickets is a disease of growing bones;
so, in severe malnutrition, signs of
Rickets are usually absent. Treatment of Nutritional Rickets
•• Cholecalciferol (vit D3) is used in
•• Stoss therapy: 300,000 – 600,000 I.U. of Vit D (orally or i/m) single dose
treatment of nutritional rickets
•• Alternative therapy: 2,000 – 5,000 IU Vit D /day X 4-6 wks
•• Either strategy should be followed by daily Vit D intake of 400 IU/day
•• Rarely, orthopedic intervention required for leg deformities, causing functional problems

X-LINKED HYPOPHOSPHATEMIC RICKETS

Importance It is the most common cause of inherited hypophosphatemic rickets

Genetic basis It has X-linked dominant inheritance; the defective gene is PHEX (PHosphate-
regulating gene with homology to Endopeptidases on X chromosome)

Pathophysiology Mutations in PHEX gene → increased levels of FGF-23


Function of FGF-23 is inhibition of phosphate reabsorption in proximal tubule
and inhibition of renal 1α-hydroxylase, leading to decreased production of
1,25-dihydroxy vit D3
High Yield Points
Clinical features Rickets with abnormalities of lower extremities, poor growth, delayed dentition
•• Patients with vit D deficiency rickets and tooth abscesses; Girls usually have less severe disease;
show evidence of radio­logical
healing within 4 weeks of therapy. Lab findings High renal excretion of phosphate, hypophosphatemia and increased
•• If no healing can be demonstrated serum alkaline phosphatase; PTH and serum calcium levels are normal; low or
with 2 mega doses of vitamin D, inappropriately normal levels of 1,25-vit D
patients should be evaluated for Treatment Combination of oral phosphorus and 1,25-D3 (calcitriol) used;
refractory rickets
Daily 1–3 g of elemental phosphorus is given, divided into 4–5 doses;

Prognosis The response to therapy is usually good;

FANCONI SYNDROME
•• Fanconi syndrome is secondary to generalized dysfunction of renal proximal tubule
•• There are renal losses of phosphate, amino acids, bicarbonate, glucose & urate
•• There is rickets as a result of hypophosphatemia, proximal renal tubular acidosis (RTA)
caused by bicarbonate losses & failure to thrive due to both rickets & RTA
Chapter 23: Musculoskeletal System 527

B. SCURVY (BARLOW’S DISEASE)

Section 3: Systemic Pediatrics


Question 7 M
Basic defect: Deficiency of Vitamin C
These X-ray features are seen
in which diseases of bones in
Who are at Risk?
children?
Children fed predominantly heat-treated cow's milk are at significant risk for symptomatic disease.

Pathogenesis

a. Rickets
b. Scurvy
c. Osteopetrosis
d. Osteogenesis imperfecta

High Yield Points


•• Thanatophoric dysplasia is the most
Clinical Features lethal chondrodysplasia

•• Anorexia, pallor, irritability, increased susceptibility to infections


•• Subperiosteal hemorrhages and long bone tenderness (pseudoparalysis of the lower
extremities) can occur
•• Swelling of costochondral junction – angulation of scorbutic beads is sharper &
tender as compared to rachitic rosary.

Radiological Signs of Scurvy


•• Ground-glass appearance of long bones due to trabecular atrophy with pencil thin cortex
•• White line of Frãnkel: Seen at metaphysis, represents well-calcified cartilage
•• Wimberger sign: Sclerotic ring around epiphyseal centers of ossification
•• Trumerfeld zone: A zone of rarefaction under white line at metaphysis; more specific;
•• Pelkan spur: Lateral prolongation of the white line and maybe present at cortical ends
•• Subperiosteal hemorrhages; dumbbell shape of affected bone.

Treatment
Vitamin C supplements of 100–200 mg/day orally or parenterally ensure rapid and complete cure.

C. OSTEOPETROSIS (Marble bone disease) Question 8


A 9-month-old boy pre­ sents with
Pathophysiology
anemia and hepa­ tosplenomegaly.
It is a disorder involving defective bone resorption due to mutation of CLCN7. His X-ray lower limbs done as a
part of skeletal survey showed the
following pic­ ture. What is your
diag­nosis?

a. Scurvy
Clinical Features b. Achondroplasia
Macrocephaly, hepatosplenomegaly, deafness, blindness, anemia, failure to thrive, develop­ c. Osteopetrosis
d. Osteogenesis imperfecta
mental delay, dental problems, osteomyelitis of mandible, pathologic fractures.
528 Section 3: Systemic Pediatrics

Radiographs
Review of Pediatrics and Neonatology

Diffuse bone sclerosis, ‘bone-within-bone' appearance, clubbing of metaphyses, alternating


lucent & dense bands produce a ‘sandwich’ appearance of vertebral bodies.

Lab Investigations
Low serum calcium & phosphorus levels with elevated parathyroid hormone & normal vit D levels.

Treatment
Hematopoietic stem cell transplantation, RANKL replacement therapy and symp­tomatic treatment.

High Yield Points


•• Osteopetrotic bones are stiff and brittle and can absorb little energy before breaking; thus despite
greatly increased bone mineral density, fracture risk is increased.

D. OSTEOGENESIS IMPERFECTA (OI)


Question 10
A 4-year-old boy presents with Also known as ‘brittle bone disease’
recurrent fractures, hyperexten­
Basic defect It is due to structural or quantitative defects in type I collagen
sible joints and hearing problems.
It is a generalized disorder of connective tissue, with variable presentation (lethal in
His X-ray of lower limbs show the
perinatal period to a mild form which may remain asymptomatic)
follo­wing. What is your diagnosis?
Inheritance There are 12 types, of which types I-V have autosomal dominant inheritance.

Clinical Triad of fragile bones, blue sclerae and deafness seen; other features include
features dentinogenesis imperfecta, hyperextensible joints, easy bruising, thin skin, joint laxity,
scoliosis, wormian bones, hernia and short stature
Fractures result from mild to moderate trauma but decrease after puberty.

Diagnosis X rays; DNA sequencing for mutation identification;


a. Achondroplasia Complications Cardiopulmonary, recurrent pneumonias, basilar invagination, brainstem
b. Osteogenesis imperfecta
compression, hydrocephalus and syringohydromyelia.
c. Osteopetrosis
d. Ehler-Danlos syndrome Treatment Bisphosphonates (IV pamidronate or oral olpadronate or risedronate), active physical
rehabilitation, fracture management & correction of orthopedic deformity.

Question 9
What is the diagnosis in this infant who presents with recurrent fractures & multiple bony
defor­mities?

a. Achondroplasia b. Ehler-Danlos syndrome


c. Rickets d. Osteogenesis imperfecta

High Yield Points


•• Osteogenesis imperfecta is the most common genetic cause of osteoporosis
•• Infants with OI type II may be stillborn or die in the 1st year of life.
•• Severe OI can be detected prenatally by level II USG, except types IV and I
Chapter 23: Musculoskeletal System 529

E. ACHONDROPLASIA

Section 3: Systemic Pediatrics


Mnemonic
It is inherited as an autosomal dominant trait due to mutation in FGFR3 gene, but most cases
Important features of Achondroplasia
arise from a new mutation to normal parents.
‘aCHONDROPLaSIa’
•• C: Champagne glass pelvis
Clinical Features •• H: Hand abnormality (Trident hand)
•• Manifests at birth with short limbs, a long narrow trunk and a large head with midfacial •• O: Bowing of legs
hypoplasia and prominent forehead. •• N: Neurological complications
•• Limb shortening is greatest in proximal segments (rhizomelic) & trident fingers seen. •• D: Delayed motor milestones
•• R: Recognized at birth
•• Delayed motor milestones, due to hypotonia and mechanical difficulty balancing the large
•• O: Obesity
head on a normal-sized trunk and short extremities. •• P: Proximal limb shortening
•• Intelligence is normal unless CNS complications develop. •• L: Large head
•• Bowing of legs, dental crowding, articulation difficulties, obesity, frequent episodes of •• S: Short stature
otitis media (leading to hearing loss) seen. •• I: Interpedicular distance decreased

Radiological Features
•• Interpedicular distance, which normally increases from L1 to L5, decreases in achondroplasia. Question 11
•• Iliac bones are short & round, & acetabular roofs are flat (‘Champagne glass pelvis’). A 5-year-old short child presented
•• Tubular bones are short with mildly irregular and flared metaphyses. with normal intelligence. His hands
showed a typical 3-pronged appea­
Limb Defects rance. What is your diagnosis?
Based on which part of limb is shortened:
•• Micromelia: Entire limb shortened
•• Rhizomelia: Proximal segment shortened
•• Phocomelia: Proximal & intermediate segment shortened
•• Mesomelia: Intermediate segment shortened
•• Acromelia: Distal segment shortened

 JUVENILE IDIOPATHIC ARTHRITIS (JIA)


Previously called ‘Juvenile Rheumatoid Arthritis'
a. Achondroplasia
b. Osteogenesis imperfecta
Definition of JIA c. Hypothyroidism
•• Age at onset: <16 yr •• Duration of disease: At least ≥ 6 weeks d. Osteopetrosis
•• Arthritis in at least 1 joint •• Exclusion of other forms of juvenile arthritis

Pathophysiology Question 12
A neonate had these severe limb
•• JIA is an autoimmune disease associated with alterations in both humoral & cell-mediated
malformations visible at birth.
immunity. T lymphocytes have a central role, releasing proinflammatory cytokines.
There is a history of exposure
Classification of fetus to Thalidomide in the
antenatal period. What is the name
•• Polyarthritis: Rheumatoid factor (RF) negative & RF positive •• Psoriatic arthritis of this defect?
•• Oligoarthritis (≤ 4 inflamed joints) •• Enthesitis-related arthritis
•• Systemic-onset disease

Diagnostic criteria of systemic onset JIA includes:


Arthritis in ≥ 1 joint with, or preceded by, fever of at least 2 wk in duration that is documented to be
daily (“quotidian”) for at least 3 days and accompanied by ≥1 of the following:
1. Evanescent (nonfixed) erythematous rash 3.  Hepatomegaly or splenomegaly or both
2.  Generalized lymph node enlargement 4.  Serositis

Complications
Macrophage activation syndrome or hemophagocytic lymphohistiocytosis (HLH)

High Yield Points


a. Achondroplasia
•• Juvenile idiopathic arthritis (JIA) is the most common rheumatic disease in children b. Osteogenesis imperfecta
•• Rheumatiod factor is absent in systemic onset JIA c. Phocomelia
•• Non-erosive arthritis is seen in SLE. d. Acromelia
530 Section 3: Systemic Pediatrics
Review of Pediatrics and Neonatology

Answer Keys for Image-Based Questions

Answers Explanations / Identifying features


1. Ans. b. Becker’s muscular The given picture shows pseudohypertrophy of calf muscles, characteristically seen in Duchenne &
dystrophy Becker muscular dystrophy

2. Ans. d. Spinal muscular atrophy The given picture shows a child using his upper limb to help him get up from sitting position; This is
called ‘Gower sign’

3. Ans. b. Myotonic dystrophy Facial weakness, inverted V–shaped upper lip, and loss of muscle mass in temporal fossae are
characteristic of myotonic muscular dystrophy

4. Ans. a. Rickets Rachitic rosary, frontal bossing, wrist widening & pot belly are s/o Rickets

5. Ans a. Rickets The given wrist X rays shows cupping, fraying & splaying, suggestive of Rickets

6. Ans. c. Windswept deformity Genu Valgum of 1 leg & genu varum of another leg, are together called ‘windswept deformity’: as if
the legs are sweeped off to one side because of the speedy wind;

7. Ans. b. Scurvy Ground glass appearance of bones with pencil thin cortex, sclerotic ring around epiphysis
(Wimberger sign) & subperiosteal haemorrhage are X ray features of Scurvy

8. Ans. c. Osteopetrosis X-ray showing markedly dense bones, with no visible marrow cavity is s/o osteopetrosis

9. Ans. d. Osteogenesis imperfecta Recurrent fractures, multiple bony deformities and blue sclera suggest a diagnosis of Osteogenesis
imperfecta

10. Ans. b. Osteogenesis imperfecta Osteoporosis with metaphyseal flaring & “popcorn” formation at growth plates are features of Osteogenesis
Imperfecta

11. Ans. a. Achondroplasia Trident hand, as shown is seen in Achondroplasia

12. Ans. c. Phocomelia History of exposure of fetus to Thalidomide in the antenatal period & the limbs of this child appearing
like the flippers of a seal, suggest 'Phocomelia'.
Chapter 23: Musculoskeletal System 531

Questions
Questions
 DISORDERS OF MUSCLES  RICKETS

1. The following finding is found in: M (AIIMS Nov 2016) 7. A 4-year-old child presented with painless genu varum.
X-ray bilateral knee was done and is shown below:
 (AIIMS May 2018)

a. Muscular dystrophy b. Neuropathy


c. GBS d. Polymyositis
2. A 5-year-old male comes with complains of pain in neck
and right shoulder with restricted range of motion. What
is the likely diagnosis? (AIIMS May 2016) Which of the following is most likely diagnosis?
a. Rickets b. Scurvy
c. Congenital anomaly d. Trauma to epiphysis
8. Which of the following conditions typically has rickets
with normal or low calcium, elevated phosphorus,
elevated parathormone, and elevated alkaline phos­
phatase? (APPG 2016)
a. Vitamin D deficiency b. Fanconi syndrome
c. Dent's disease d. Chronic kidney disease
9. Causes of Rickets include all EXCEPT:
a. Facioscapulohumeral dystrophy
a. Renal Tubular Acidosis (MAHA PGM CET 2016)
b. Long thoracic nerve palsy
b. Chronic Kidney Disease
c. Klippel Feil syndrome
c. Benign Mesenchymal Tumors
d. Ankylosing spondylitis
d. Severe Malnutrition
3. Gower's maneuver is classically seen in: M  (APPG 2016)
10. Widening of epiphysis is seen in: M 
a. Cerebral palsy b. Friedreich's ataxia
 (Recent Question 2015)
c. Duchenne muscular dystrophy
d. Parkinsonism a. Scurvy b. Rickets
4. In Duchenne muscular dystrophy, the defect is in the c. Achondroplasia d. Osteopetrosis
gene producing the following protein:  M 11. An infant presents with lethargy, frontal bossing and
(TN PGMEE 2011) bowing of legs; X-ray shows widening of epiphysis.
a. Dystrophin b. Alpha-actinin What is the diagnosis? M  (JIPMER 2014)
c. Nebulin d. Desmin a. Scurvy b. Rickets
5. About Duchenne muscular dystrophy-false statement is: M c. Osteomalacia d. Osteopetrosis
 (TN PGMEE 2010) 12. Most potent form of vitamin D is:
a. Becker's dystrophy more severe a. Ergocalciferol (Vit D2) (Recent Question 2013)
b. X-linked recessive b. 7 dehydrocholecalciferol
c. Cardiomyopathy c. 25 hydroxycholecalciferol
d. Mental retardation common d. 1,25 dihydroxycholecalciferol
6. All are congenital myopathies EXCEPT: (WBPG 2008) 13. Abnormality in RTA with rickets is: (DNB June 2012)
a. Nemaline b. Central core a. Loss of HCO3 b. Loss of Ca2+
c. Centronuclear/myotubular d. Mitochondrial c. Loss of K+ d. All of above
532 Section 3: Systemic Pediatrics

14. The basic defect in rickets is: (DNB June 2012)


Review of Pediatrics & Neonatology

� SCURVY
a. Excessive ossification b. Excessive mineralization
c. Defective resorption d. Defective mineralization 25. Subperiosteal hemorrhage is due to deficiency of: M 
15. Which of the following is not a sign of active rickets?  (WB PGMEE 2016, JIPMER 2006)
 (COMEDK 2009) a. Vit A b. Vit K
a. Prominent fontanelle b. Hot cross bun sign c. Vit C d. Vit D
c. Saddle nose d. Caries teeth 26. All are radiologic signs in scurvy EXCEPT: (JIPMER 2013)
16. Rickets in infant present as all EXCEPT: (AIIMS May 2007) a. White line in metaphysis b. Cupping and fraying
a. Craniotabes b. Widened fontanel c. Ground glass appearance d. Zone of rarefaction
c. Rachitic Rosary d. Bow legs 27. A 5-month-old baby (exclusively cow feed) presented
to pediatric OPD with history of crying on touch. Cause
17. Alkaline phosphatase level are not elevated in: 
may be: (WB PGMEE 2012)
 (JIPMER 2006)
a. Scurvy b. Rickets
a. Renal rickets b. Nutritional rickets
c. JRA d. Caffey's disease
c. Hypophosphatasia d. Hypophosphatemic rickets
28. Primary metabolic bone disorder in scurvy is:
18. In Rickets which of the following is/are seen:
a. Decreased mineralization (AIPGMEE 2010)
a. Increased alkaline phosphatase (PGI Dec 2006)
b. Decreased osteoid matrix formation
b. Decreased alkaline phosphatase
c. Increased bone resorption
c. Hypophosphatemia in blood
d. Decreased bone mass with normal mineralization and
d. Hypocalcemia
osteoid formation
e. Alkalosis
29. Child with frog like position and resistance to move the
19. A 2-year-old boy has clinical features of rickets. His investi­
limbs, is probably suffering from: (DNB June 2010)
gations revealed serum Calcium-9 mg/dL, Phosphate-2.4
a. Scurvy b. Rickets
mg/dL, alkaline phosphatase – 1041 IU, normal intact
c. Trauma d. Congenital dislocation
parathyroid hormone and bicarbonate 22 mEq/L. Which of
30. Wimberger ring sign is present in: M  (PGI Dec 2008)
the following is the most probable diagnosis:
a. Rickets b. Scurvy
 (AIIMS Nov 2004, AIIMS May 2002, AIIMS May 2000)
c. Secondary syphilis d. Tuberculosis
a. Distal renal tubular acidosis
31. Pseudoparalysis in an infant is suggestive of:
b. Hypophosphatemic rickets
 (PGI Dec 2006)
c. Vitamin D dependent rickets
a. Acute rheumatic fever b. Vit B6 deficiency
d. Hypoparathyroidism
c. Vit E deficiency d. Vit C deficiency
20. A 10-year-old boy has a fracture of femur. Biochemical
32. About scurvy, true are all EXCEPT: (PGI June 2005)
evaluation revealed Hb 11.5 gm/dL and ESR 18 mm 1st
a. Subperiosteal hematoma with tenderness
hour. S. calcium 12.8 mg/dL, S. phosphorus 2.3 mg/
b. Zone of rarefaction
dL alkaline phosphate 28 KA units and blood urea 32
c. Increased serum alkaline phosphatase
mg/dL. Which of the following is the most probable
d. Gingival bleeding
diagnosis in his case: (AIPGMEE 2004)
e. Pelkan spurs
a. Nutritional rickets b. Renal rickets
c. Hyperparathyroidism d. Skeletal dysplasia
 JUVENILE IDIOPATHIC ARTHRITIS
21. True about Vit D deficiency rickets: (PGI Dec 2004)
a. Vit D3 given at a dose of 50–100 mg/day 33. Characteristic of systemic juvenile idiopathic arthritis
b. X-ray knee joint is diagnostic a. Begins after 16 years of age (Recent Question 2017)
c. Rachitic rosary is tender b. Uveitis is a feature
d. Increased chances of respiratory tract infections c. RA factor is negative
e. Hyponatremia d. NSAIDs are contraindicated
22. True about nutritional rickets: (PGI Dec 2003)
34. Non-erosive arthritis is seen in: (Recent Question 2016)
a. Craniotabes b. Multiple fractures
a. Systemic lupus erythematosus
c. Wrist widening d. Increased serum phosphate
b. Ankylosing spondylitis
e. Growth retardation
c. Rheumatic fever
23. All of the following are seen in Rickets EXCEPT: M 
d. Juvenile idiopathic arthritis
 (AIIMS May 2003)
a. Bow legs b. Gum bleeding 35. A 5-year-old girl has 3 month history of joint pains and
c. Pot belly d. Craniotabes morning stiffness with swelling of knees and ankles.
24. Windswept deformity is seen in: She also has eye pain and photophobia. What is the
 (NEET, DNB Pattern; AIIMS 98) diagnosis? (Recent Question 2014)
a. Scurvy b. Rickets a. Rheumatic fever b. Juvenile idiopathic arthritis
c. Achondroplasia d. Osteoporosis c. Behcet syndrome d. Chikungunya fever
Chapter 23: Musculoskeletal System 533

36. Juvenile idiopathic arthritis includes all the following � MISCELLANEOUS

Questions
categories EXCEPT: (JIPMER 2009)
a. Psoriatic arthritis 38. Phocomelia is a problem affecting: (JIPMER Nov 2017)
a. Long bones b. Skull bones
b. Enthesitis related arthritis
c. Carpals d. Metacarpals
c. Systemic arthritis
39. Osteogenesis imperfecta with normal dentition and
d. Reactive arthritis
sclera: (JIPMER May 2017)
37. In systemic form of JIA, all are true EXCEPT: a. Type 1A b. Type 2
 (WBPG 2008) c. Type 3 d. Type 4B
a. RA factor positive 40. A baby was vigorously shaken by parents. What do you
b. High fever with rash expect in the baby? (JIPMER Nov 2016)
c. Hepatosplenomegaly a. Rib fractures b. Pelvic fracture
d. Elevated ESR c. Ruptured spleen d. Sub-dural hematoma
534 Section 3: Systemic Pediatrics
Review of Pediatrics & Neonatology

Answers with Explanations


 DISORDERS OF MUSCLES
1. a. Muscular dystrophy  Ref: Nelson's 20/e p 2976-2977; Refer pretext of this chapter for details;
The given picture shows pseudohypertrophy of calf muscles, characteristically seen in Duchenne muscular dystrophy;
Enlargement is caused by hypertrophy of some muscle fibers, infiltration of muscle by fat & proliferation of collagen.
2. a. Facioscapulohumeral dystrophy  Ref: Nelson's 20/e p 2984; Refer pretext of this chapter for details;
The picture is showing Winging of Left scapula. It can be seen in two of the mentioned conditions, Facioscapulohumeral dystrophy
or Long thoracic nerve palsy. But the latter is usually due to sports trauma or breast surgeries, and hence unlikely in a child. So FSHD
is a better answer. It is one of the muscular dystrophies which can present asymmetrically and unilaterally.
Note: Klippel-Feil Syndrome is a triad of short neck, low hairline and restriction of neck motion in a patient with multiple
coalitions of cervical vertebrae.
3. c. Duchenne muscular dystrophy  Ref: Nelson's 20/e p 2915-2985, Ghai 8/e p 595-597
Gowers' sign indicates weakness of the proximal muscles, especially of the lower limb. It can be seen in Duchenne or Becker
Muscular dystrophy, centronuclear myopathy and myotonic dystrophy.
4. a. Dystrophin  Ref: Nelson's 20/e p 2915-2985, Ghai 8/e p 595-597
Duchenne & Becker Muscular dystrophy are X linked Recessive disorders due to defect in Dystrophin gene on Chr X.
5. a. Becker's dystrophy more severe  Ref: Nelson's 20/e p 2915-2985, Ghai 8/e p 595-597; Refer pretext for details;
Becker's dystrophy is a less severe variant of Duchenne muscular dystrophy.
6. d. Mitochondrial  Ref: Nelson's 20/e p 2915-2985, Ghai 8/e p 595-597
Congenital myopathies are a heterogeneous group of non-progressive congenital neuromuscular disorders which have
subcellular abnormalities that can be demonstrated only by muscle biopsy; They include Central core disease, Nemaline myopathy,
Myotubular (centronuclear) myopathy and Congenital fiber- type disproportion.

 RICKETS
7. a. Rickets  Ref: Nelson 20/e p 331-335
The given X-ray shows cupping, fraying and splaying of ends of long bones suggestive of Rickets.
8. d. Chronic kidney disease  Ref: Nelson's 20/e p 336-340, Ghai 8/e p 112-115
All of the above conditions are causes of Rickets, so elevated alkaline phosphatase is seen in all the above.
For all practical purposes, Chronic kidney disease is the only condition out of the above, where Phosphate is elevated.
9. d. Severe Malnutrition  Ref: Nelson's 20/e p 331-336, Ghai 8/e p 113-115
Rickets is a disease of growing bones and is not seen in severe malnutrition.
10. b. Rickets  Ref: Nelson's 20/e p 331-336, Ghai 8/e p 113-115
Wrist widening is an important clinical feature of Rickets.
11. b. Rickets  Ref: Nelson's 20/e p 331-336, Ghai 8/e p 113-115
12. d. 1,25 dihydroxycholecalciferol  Ref: Nelson's 20/e p 336-340, Ghai 8/e p 112-115
13. d. All of above  Ref: Nelson's 20/e p 336-340, Ghai 8/e p 112-115
Rickets is mainly seen in Distal Renal tubular acidosis, where,
•• Acidosis is due to loss of Bicarbonate
•• There is also hypokalemia due to Potassium loss, along with hypocalcemia.
14. d. Defective mineralization  Ref: Nelson's 20/e p 336-340, Ghai 8/e p 112-115
15. c. Saddle nose  Ref: Nelson's 20/e p 336-340, Ghai 8/e p 112-115
Saddle nose is not seen in rickets, but in Congenital Syphilis.
16. d. Bow legs  Ref: Nelson's 20/e p 336-340, Ghai 8/e p 112-115
Bowing of legs appears only after weight bearing; Hence, Bow legs is a feature of rickets in toddlers and not infants.
17. c. Hypophosphatasia  Ref: Nelson's 20/e p 331-336, Ghai 8/e p 113-115
18. a. Increased alkaline phosphatase, c. Hypophosphatemia in blood, d. Hypocalcemia  Ref: Nelson's 20/e p 336-340
19. b. Hypophosphatemic rickets  Ref: Nelson's 20/e p 336-340, Ghai 8/e p 112-115
Clues in this question are – Normal calcium, Low phosphate, Normal parathormone, & Elevated alkaline phosphatase All these
suggest the diagnosis of hypophosphatemic rickets.
Chapter 23: Musculoskeletal System 535

20. c. Hyperparathyroidism  Ref: Nelson's 20/e p 2694-2697, Ghai 8/e p 521-522

Answers with Explanations


The given case describes a 10-year-old boy with fracture femur.
Blood investigations reveal normal ESR, Hypercalcemia, hypophosphatemia and raised ALP levels (normal = 3–13 kA units) with
normal B. urea; Out of the options available, only hyperparathyroidism can give rise to this picture.
About other options,
•• In renal rickets, S phosphate is usually high and S. calcium low/normal with deranged kidney functions
•• In nutritional rickets, S phosphate/calcium can be normal/low but never high calcium
•• In Skeletal dysplasia, abnormal bone & cartilage are seen, but blood levels of calcium & phosphorus are usually normal.
21. b. X-ray knee joint is diagnostic, d. Increased chances of respiratory tract infections  Ref: Nelson's 20/e p 336
Discussing about the options one by one,
a. False Daily dose of Vit D for treatment of Rickets is 2,000–5,000 IU/day X 4-6 wks
b. True X-ray wrist or knee are usually done to diagnose Rickets
c. False Rachitic rosary is nontender (in contrast to scorbutic rosary which is tender & more angular or sharp)
d. True Risk of pneumonia appears to be elevated in children with rickets
e. False
Not seen in Rickets; Hypocalcemia is usually seen

22. a. Craniotabes, c. Wrist widening, e. Growth retardation  Ref: Nelson's 20/e p 336
23. b. Gum bleeding  Ref: Nelson's 20/e p 336, Ghai 8/e p 115
Gum bleeding is a feature of Scurvy, and not Rickets.
24. b. Rickets  Ref: Nelson's 20/e p 336, Ghai 8/e p 115
Windswept deformity
Definition A valgus deformity of one in association with varus deformity of other knee.
Causes •• Rickets •• Physeal osteochondromatosis
•• Rheumatoid Arthritis •• Hereditary dysplasia (epiphyseal dysplasia) of bone.

 SCURVY
25. c. Vit C  Ref: Nelson's 20/e p 329-331, Ghai 8/e p 120-121
26. b. Cupping and fraying  Ref: Nelson's 20/e p 329-331, Ghai 8/e p 120-121; Refer pretext for details;
Radiological Signs of Scurvy include Ground-glass appearance of long bones due to trabecular atrophy with Pencil
thin cortex, White line of Frãnkel, Wimberger sign (Sclerotic ring around epiphysis), Trumerfeld zone, Pelkan spur and
Subperiosteal hemorrhages;
27. a. Scurvy  Ref: Nelson's 20/e p 329-331, Ghai 8/e p 120-121
In scurvy, there is painful pseudoparalysis of limbs, due to subperiosteal hemorrhage; so the child cries on touching.
28. b. Decreased osteoid matrix formation  Ref: Nelson's 20/e p 329-331, Ghai 8/e p 120-121
Pathogenesis of scurvy:
•• There is defective formation of connective tissues & collagen in skin, cartilage, bone, & blood vessels → ↑ fragility
•• In the long bones, osteoid is not deposited by osteoblasts, cortex is thin, trabeculae become brittle & fracture easily.
29. a. Scurvy  Ref: Nelson's 20/e p 329-331, Ghai 8/e p 120-121
It is seen in pseudoparalysis, caused by painful subperiosteal hemorrhage in Scurvy.
30. b. Scurvy  Ref: Nelson's 20/e p 329-331, Ghai 8/e p 120-121
31. d. Vit C deficiency  Ref: Nelson's 20/e p 329-331, Ghai 8/e p 120-121
Pseudoparalysis
Definition A voluntary restriction of motion, mainly because of pain, but not due to actual muscular paralysis

Causes Scurvy, Osteomyelitis, surgery, fractures, myositis, metabolic myopathy, Congenital syphilis, septic arthritis

32. c. Increased serum alkaline phosphatase  Ref: Nelson's 20/e p 336-340, Ghai 8/e p 112-115
Note: Alkaline Phosphatase level is elevated in Rickets and not in Scurvy.

 JUVENILE IDIOPATHIC ARTHRITIS


33. c. RA factor is negative
JIA begins at < 16 year age; Uveitis is usually seen in oligoarticulat JIA; NSAIDs are used for symptomatic relief
34. a. Systemic lupus erythematosus  Ref: Nelson's 20/e p 1177-1179, 1821
Non-erosive arthritis includes those without significant radiographic changes, as in systemic lupus erythematosus (SLE);
536 Section 3: Systemic Pediatrics

35. b. Juvenile idiopathic arthritis  Ref: Nelson's 20/e p 1160-1170, Ghai 8/e p 624-627
Review of Pediatrics & Neonatology

Criteria for Classification of Juvenile Rheumatoid Arthritis/juvenile Idiopathic arthritis.


•• Age at onset: <16 yr •• Duration of disease: at least ≥ 6 weeks
•• Arthritis in at least 1 joint •• Exclusion of other forms of juvenile arthritis

36. d. Reactive arthritis  Ref: Nelson's 20/e p 1162
Juvenile Idiopathic Arthritis includes the following categories:
•• Polyarthritis: RF negative and RF positive •• Psoriatic arthritis
•• Oligoarthritis (≤ 4 inflammed joints) •• Enthesitis related arthritis
•• Systemic-onset disease

37 a. RA factor positive  Ref: Nelson's 20/e p 1162
RA factor may be positive in polyarthritis, but not in systemic onset JIA.

 MISCELLANEOUS
38. a. Long bones  Ref: Nelson’s 20/e p 3423
Phocomelia is a birth defect in which the hands and feet are attached to abbreviated arms & legs. The term comes from phoco
(meaning ‘seal') & melia (meaning ‘limb'), to indicate that a limb is like a seal's flipper; One of the important cause is exposure of fetus
to thalidomide.
39. a. Type 1A  Ref: Nelson’s 20/e p 3381
Osteogenesis imperfecta (OI) is the most common genetic cause of osteoporosis; OI Type I is a mild disease.
Both types I & IV are divided into A and B subtypes, depending on the absence (A) or presence (B) of dentinogenesis imperfecta.
40. d. Sub-dural hematoma  Ref: Nelson’s 20/e p 239-243, 3067
Child abuse/Shaken baby syndrome/Inflicted childhood neurotrauma
•• It occurs secondary to violent, non-accidental, repetitive, unrestrained acceleration-deceleration head & neck movements, with
or without blunt head trauma in children typically < 3 year of age.
•• Inflicted childhood neurotrauma accounts for 10% of all cases of child abuse and carries a mortality rate of up to 25%.
•• Retinal hemorrhage (multiple, involving > 1 layer) is the most common ophthalmic finding.
•• Fractures that strongly suggest abuse include classic metaphyseal corner lesions, posterior/1st rib fracture, scapular/sternal
fracture, spinous process fracture. These fractures require more force than would be expected from a minor fall or routine
handling and activities of a child.
•• Abusive head trauma (AHT) results in the most significant morbidity and mortality & may be caused by direct impact, asphyxia,
or shaking.
•• Subdural hematomas, retinal hemorrhages & diffuse axonal injury strongly suggest AHT, especially when they co-occur.
Chapter 24
Pediatric Surgical
Disorders
 A. GASTROINTESTINAL DISORDERS
Question 1 M
1. TRACHEOESOPHAGEAL FISTULA (TEF) ± ESOPHAGEAL ATRESIA A term newborn presents with
frothing, excessive drooling and
Classification coughing with feeds. There was a
•• 5 types based on the presence of esophageal atresia and location of the fistula. history of polyhydramnios in the
Type Type A Type B Type C Type D Type E antenatal period. A chest X-ray was
done which showed the follo­wing
Description Esophageal Proximal TEF Proximal eso­ Both proximal and Isolated TEF
finding. What is the diagnosis?
atresia without pha­geal atresia distal fistula (H or N type
TEF with distal TEF fistula)

a. Achalasia cardia
b. Esophageal atresia only
c. Esophageal atresia with tracheo­
esophageal fistula
d. Hypertrophic pyloric stenosis

Features •• 2nd most •• No gas in •• Most •• Least •• Distal gas


common abdomen common type common type present
type •• Gas in •• Gas in
•• No gas in abdomen abdomen
abdomen present present

Clinical Presentation M

•• Associated with polyhydramnios due to inability of fetus to swallow amniotic fluid


•• After birth neonates present with frothing, excessive drooling and coughing with feeds →
followed by cyanosis and respiratory distress
•• H or N type fistulas usually present later, with recurrent pneumonia due to aspiration
•• TEF is commonly associated with CVS abnormalities, VACTERL, hypo­s­­padias, undescended
testis, duo­denal atresia, aqueductal stenosis, Goldenhar syndrome & CHARGE syndrome.
Diagnosis M

•• Nasogastric tube is inserted and inability to pass it confirms the diagnosis of TEF
•• X-ray shows a coiled tube in the atretic end and gas in the abdomen type C, D and E TEF High Yield Points
•• Prenatal USG: polyhydramnios, microgastria and absent stomach bubble.   M
•• VACTERL association: Comprises
Treatment of Vertebral anomalies, Anal atresia,
•• If the distance between both ends < 3 cm: Surgical repair of the fistula and end to end Cardiac defects, Tracheo-esophageal
anastomosis of esophagus fistula and/or Esophageal atresia,
•• If the gap is more: gastrostomy is done and esophageal repair by conduits (stomach, jejunum Renal & Radial anomalies & Limb
or colon) is delayed by 2–3 months. defects
•• CHARGE stands for Coloboma,
Complication of Surgery Heart defect, Atresia choanae
•• Anastomotic leak: earliest complicationQ (choanal atresia), Retarded growth &
•• Stricture formationQ: most common complication, at the site of anastomosis development, Genital abnormality &
•• Other complications: GERD, Barret esophagus and esophageal cancer. Ear abnormality
538 Section 3: Systemic Pediatrics

2. FOREIGN BODY ESOPHAGUS


Review of Pediatrics and Neonatology

Question 2
Epidemiology Majority of foreign-body ingestions occur in children, between 6 months & 3 yrs
A 4 year child was brought with the
following finding on the chest X Coins & small toy items are the most commonly ingested foreign bodies
ray. What is the probable location Diagnosis History of foreign body ingestion
of the foreign body visible on the Plain anteroposterior radiographs of neck, chest & abdomen, along with lateral views
skiagram?
Treatment •• Endoscopic visualization & removal using foreign-body retrieving instrument
•• Sharp objects, disk button batteries, or respiratory symptoms mandate urgent removal
•• Asymptomatic blunt objects & coins can be observed for up to 24 hr anticipating
passage into stomach
•• An alternative technique for removing esophageal coins impacted for <24 hr is passage
of a Foley catheter beyond the coin at fluoroscopy, inflating the balloon & pulling the
catheter & coin back simultaneously with patient in a prone oblique position

a. Trachea
b. Esophagus 3. HYPERTROPHIC PYLORIC STENOSIS
c. Vallecula
d. Right main bronchus
Etiology
Decreased NO synthase production, enteric denervation & loss of interstitial cells of Cajal.

Clinical Presentation M

•• Infant typically presents with forceful, projectile, postprandial non-bilious vomiting


between 2–6 weeks of age
•• After vomiting child is hungry and has a voracious appetite
•• Long-term vomiting may lead to dehydration, weight loss and constipation.
•• On examination a visible peristalsis from left to right can be seen and a mobile olive
shaped pyloric mass can be palpated in 80–90% of cases
•• Feeding is useful for diagnosis as it increases persistalsis & palpation of mass is easier.

Diagnosis
•• Ultrasonography is the most specific & sensitive diagnostic method
•• A pyloric thickness of > 4 mm and a length of >14 mm is diagnostic
•• Upper gastrointestinal contrast (definitive method) which shows string sign,Q double tractQ
sign and shoulder sign
Question 3 Treatment
A 5-day-old term male neo­ nate
•• Ramstedt pyloromyotomyQ is the surgical treatment of choice
presents with delayed passage of
•• Electrolyte abnormality should be corrected before surgery.
meconium, abdo­ minal distension
and bilious vomiting. Barium enema
and intestinal biopsy findings are High Yield Points   M
shown below. What is the diag­
nosis? In hypertrophic pyloric stenosis
•• Males (first born) are more commonly affected than females
•• Hypokalemic metabolic alkalosis with paradoxical aciduria is seen
•• Imaging of choice is Ultrasonography
•• Hypertrophic pyloric stenosis is the most common surgical cause of non-bilious vomiting in infants

4. HIRSCHSPRUNG DISEASE (CONGENITAL AGANGLIONIC MEGACOLONQ)

Caused by M

Defective migration of neural crest cells into the mesodermal layers of gut

Absence of ganglions in the submucosal and myenteric plexus of intestine

Decreased peristalsis and contraction of the involved segment

Clinical Presentation M

a. Congenital aganglionic megacolon •• Newborns characteristically present with delayed passage of meconium followed by bilious
b. Hypertrophic pyloric stenosis vomiting and abdominal distension.
c. Cystic fibrosis
•• Enterocolitis (fever, bloody stools) is usually seen between 2-4 weeks;
d. Intestinal malrotation
•• Complications: Intestinal perforation, constipation and rectal bleeding.
Chapter 24: Pediatric Surgical Disorders 539

Diagnosis

Section 3: Systemic Pediatrics


M

•• Definitive diagnosis: Intestinal biopsyQ which shows absence of ganglionsQ, nerve trunk
hypertrophy & increased acetylcholinesterase activity in involved segment
•• Barium enema shows: Contracted involved segment with dilation of normal proximal
segment
•• Rectal manometry:
–– Shows failure of the internal anal sphincter to relax after dilation of the rectum
–– An abnormal test is not absolutely diagnostic but a normal test excludes the disorder. Question 4
Treatment A 3-year-old child presen­ted with
sudden onset severe paroxysmal
•• Diversion colostomy is done initially to gain time if patient is not fit for surgery colicky pain abdomen and red
•• Surgical resection of aganglionic segment & anastomosis of normal segments is the current jelly stool. His barium
treatment of choice enema picture is shown below.
•• Different surgical options used: Swenson, Duhamel, Soave and Boley procedures. What is the diagnosis?

5. INTUSSUSCEPTION M

•• Occurs when a portion of the alimentary tract is telescoped into an adjacent segment
•• Upper portion of bowel (intussusceptum), invaginates into lower (intussuscipiens).

Clinical Features
Sudden onset in a previous well child of severe paroxysmal colicky pain or episodes of a. Intussusception
b. Ulcerative colitis
excessive crying; Vomiting occurs in most cases.
c. Crohn disease
d. Hirschsprung disease
Diagnosis
•• Abdominal palpation
–– Usually (about 70% cases) reveals a slightly tender sausage shaped mass
–– Presence of bloody mucus on rectal examination supports the diagnosis
•• Ultrasound: Sensitivity 98–100%, Specificity 88% High Yield Points
•• Tubular mass in longitudinal views Classic Triad in Intussusception
–– Doughnut or target appearance in transverse images Pain, a palpable sausage shaped
–– Barium enema – 'Coiled spring' sign or the 'claw sign'. abdo­minal mass, and bloody or currant
jelly stool
Management
•• Immediate reduction of acute intussusception
•• In prolonged intussusception and signs of shock, peritoneal irritation, intestinal perforation or
pneumatosis intestinalis, urgent laparotomy should be planned. High Yield Points
Acute appendicitis is the most common
High Yield Points acute surgical condition in children
•• Most common cause of intestinal obstruction between 3 months and 6 years of age is intussusception
•• Most common abdominal emer­gency in children <2 year is intus­susception
•• 90% of cases of Intussusception are idiopathic
•• Swollen Peyer’s patches, in response to GIT infection or intro­duction of new food proteins may lead to
mucosal prolapse causing an intussusception
•• Intussusceptions are most often ileocolic Question 5
•• Spontaneous reduction of intus­susception occurs in 4-10% patients.
A neonate presents with bilious
6. ACUTE APPENDICITIS M vomiting. His abdominal X-ray is
shown below. Diagnosis?
•• Peak incidence between ages of 12–18 yr; rare in children < 5 yr age
•• Localized abdominal tenderness is the single most reliable finding in the diagnosis of acute appendicitis.
•• Ultrasound has >90% sensitivity & specificity for the diagnosis of acute appendicitis
•• Standard treatment for acute appendicitis is appendectomy.

7. BOWEL ATRESIA
•• Duodenal atresia accounts for 25-40% of all intestinal atresias (Most common).
•• Hallmark of duodenal obstruction is bilious vomiting without abdominal distention, which a. Duodenal atresia
is usually noted on 1st day of life. b. Imperforate anus
•• History of polyhydramnios is present in 50% c. Hirschsprung disease
d. Meckel diverticulum
•• Diagnosis is suggested by the presence of a “double-bubble” sign on a plain abdominal X-ray.
540 Section 3: Systemic Pediatrics

8. MECKEL DIVERTICULUM
Review of Pediatrics and Neonatology

M
Mnemonic M

‘Rule of 2’ in Meckel Diverticulum: •• Meckel diverticulum is the most common congenital anomaly of GI tract.
•• It is a 2 inch outpouching of ileum, •• Meckel diverticulum accounts for 50% of all lower GI bleeds in children younger than 2 years of age.
2 feet from ileocecal valve •• Caused by incomplete obliteration of omphalomesenteric duct during 7th week of gestation.
•• Found in 2% of general popu­lation •• Usually symptomatic in 1st or 2nd year of life, but can occur in 1st decade.
•• Can contain 2 types of ectopic •• Lined by an ectopic acid-secreting mucosa, that causes intermittent painless rectal bleeding by
tissue (pancreatic or gastric) ulceration of adjacent normal ileal mucosa.
•• Generally present at <2 years age •• The stool is typically described as brick colored or currant jelly colored.
•• 2 times more common in females •• May present as bowel obstruction at younger age of onset.
•• The most sensitive study is a Meckel radionuclide scan, which is performed after intra­venous infusion
of technetium-99m pertechnetate.

Question 6 M  DISORDERS OF ABDOMINAL WALL


What is your diagnosis in this
neonate with visible bowel loops
1. GASTROSCHISIS
at birth? What is it?
Gastroschisis is a congenital defect of the abdominal wall that results in a protrusion of viscera
through a paraumbilical defect. The anomaly occurs lateral and generally to the right of the
umbilicus. Peritoneal membranes covering the exposed viscera are not present.

Associated Anomalies
Associated anomalies occur in 10 to 30% of cases and most commonly include intestinal atresia and
a. Gastroschisis stenosis. Other associated anomalies are congenital heart disease, hydrocephalus, polyhydramnios,
b. Omphalocele
oligohydramnios, prune belly, and genitoruinary tract malformations.
c. Congenital Diaphragmatic
d. Bladder exstrophy
Identifying Feature
In this anomaly, the intestines & other contents are not covered by a membrane.

Treatment
•• Delivery of a fetus with gastroschisis should be performed in a center with immediate pediatric
surgical expertise available. Because closure depends on bowel volume, it is important to
decompress the stomach.
•• Fluid and heat loss are rapid, and the neonate should be either wrapped in plastic wrap or
placed into a plastic bag to minimize both.

2. OMPHALOCELE
An omphalocele is a herniation or protrusion of abdominal contents into base of umbilical
cord. In contrast to the more common umbilical hernia, the sac is covered with peritoneum
without overlying skin.

Comparison of Gastroschisis & Omphalocele

Characteristic Gastroschisis Omphalocele


Defect size (diameter) 2–3 cm 2–15 cm
Question 7 M Sac Never present Always, may be ruptured
Identify this congenital abnorma­ Gestational age Prematurity Term
lity: Umbilical cord Adjacent (left side) to defect Attached to sac
Bowel character Inflammatory, edematous Normal
Enteral nutrition Delayed Normal
Associated anomalies Less common Common (50%)

3. CONGENITAL DIAPHRAGMATIC HERNIA (CDH)


What is it?
a. Sacrococcygeal teratoma
b. Malrotation •• It is a defect in diaphragm through which abdominal contents may herniate into thorax
c. Gastroschisis •• Pulmonary hypoplasia and malrotation of intestine, in varying degrees are part of the
d. Omphalocele lesion, not associated anomalies.
Chapter 24: Pediatric Surgical Disorders 541

Associated Abnormalities

Section 3: Systemic Pediatrics


High Yield Points
•• Include esophageal atresia, omphalocele & cardiovascular lesions
•• Most comon type of CDH is
•• CDH is associated with trisomy 21, trisomy 13, trisomy 18 and Turner syndrome.
posterolateral or BochdalekQ
•• CDH is more common in females
Clinical Presentation
•• CHH is more common on left side
•• Presents in the neonatal period with scaphoid abdomen, bowel sounds present in chest, •• Most cases of CDH are sporadic
respiratory distress.

Diagnosis
•• Antenatally by ultrasound (between 16 and 24 weeks of gestation)
•• Postnatally by chest X-ray High Yield Point   M
•• Echocardiography is important to guide therapeutic decisions by measuring pulmonary and
•• Congenital Diaphragmatic hernia
systemic vascular pressures and defining the presence of cardiac dysfunction.
is an absolute contraindication for
bag & mask ventilation
Management
•• Aggressive respiratory support is often needed in children with CDH
•• Use of bag mask ventilation enlarges the stomach and small bowel and thus makes
oxygenation more difficult and must be avoided.
Question 8 M

What is the abnormality seen in


this Chest X-ray?

a. Bronchopulmonary dysplasia
b. Meconium aspiration syndrome
c. Pulmonary alveolar proteinosis
d. Congenital diaphragmatic hernia

Prognosis
Predictors of poor prognosis in CDH include:
•• Severe pulmonary hypoplasia
•• Associated major anomaly
•• Symptoms before 24 hr of age
•• Herniation to the contralateral lung
•• Need for ECMO.

 ABNORMALITIES OF GENITOURINARY SYSTEM


1. BLADDER EXSTROPHY
Basic Defect
It results when the mesoderm fails to invade the cephalad extension of cloacal membrane.

Spectrum
Severity ranges from simple epispadias (in boys) to complete exstrophy of cloaca involving
exposure of entire hindgut & bladder (termed cloacal exstrophy).
542 Section 3: Systemic Pediatrics

Clinical Features
Review of Pediatrics and Neonatology

•• Male: Female ratio is 2 : 1.


•• In classic bladder exstrophy, bladder protrudes from abdominal wall & its mucosa is exposed.
Question 9 Umbilicus is displaced downward, pubic rami are widely separated in the midline, rectus
A neonate was born with a reddish muscles are separated & anus is displaced anteriorly
lesion on the lower abdomen, as •• In boys, there is complete epispadias with dorsal chordee & penile length is half that of
shown in the image below. What is unaffected boys. Undescended testes & inguinal hernias are common.
the probable diagnosis? •• Girls also have epispadias, with separation of 2 halves of clitoris & wide separation of labia.

Treatment
•• Management should start at birth.
•• Bladder should be covered with plastic wrap to keep bladder mucosa moist.
•• Application of gauze or petroleum-gauze to the bladder mucosa should be avoided, because
significant inflammation will result. Latex allergy is also common in these babies.
•• Infant should be transferred promptly to a center with pediatric urologic & anesthetic support
for the treatment of such anomalies
a. Umbilical granuloma •• 2 surgical approaches: staged reconstruction & total single-stage reconstruction.
b. Rectal prolapse •• Most babies also undergo bilateral iliac osteotomy, which allows the pubic symphysis to be
c. Meckel’s diverticulum approximated, which supports the bladder closure.
d. Bladder exstrophy •• In a staged reconstruction, the initial stage is bladder closure, the 2nd stage (in boys) is
epispadias repair, and the final stage is bladder neck reconstruction.
•• When single stage reconstruction is performed in neonatal period, there is an increased risk of
intraoperative penile injury & postoperative hydronephrosis.

Prognosis
•• In untreated cases, there is total urinary incontinence & increased incidence of bladder cancer,
usually adenocarcinoma.

2. HYPOSPADIAS
•• Basic defect: Urethral opening is on the ventral surface of penile shaft
•• Epidemiology: Affects 1 in 250 male newborns; its incidence is increased in disorders of sex
Question 10 differentiation, anorectal malformation & congenital heart disease.
What is the name of this conge­nital
defect? Clinical Features
•• 65% of cases are distal, 25% are subcoronal or midpenile, and 10% are proximal.
•• Usually there is incomplete development of the prepuce, called a dorsal hood
•• Those with proximal hypospadias, have chordee, in which there is ventral penile curvature
during erection
•• 10% of boys with hypospadias have an undescended testis; inguinal hernias also are common

Management
•• Circumcision should be avoided, because the foreskin is used in repair in most cases.
a. Epispadias •• Ideal age for repair in a healthy infant is 6-12 months
b. Hypospadias
•• The goal of hypospadias surgery is to correct the functional and cosmetic deformities.
c. Posterior urethral valve
•• Whereas hypospadias repair is recommended for all boys with midpenile and proximal
d. Ureterocele
hypospadias, some boys with distal hypospadias have no functional abnormality and do not
need any surgical correction.

Complications
Untreated hypospadias can lead to deformity of urinary stream, typically ventral deflection or
severe splaying; sexual dysfunction secondary to penile curvature; infertility if the urethral meatus
is proximal; meatal stenosis (congenital) & cosmetic problems.
Chapter 24: Pediatric Surgical Disorders 543

Section 3: Systemic Pediatrics


Answer Keys for Image-Based Questions

Answers Explanations / Identifying features


1. Ans. c. Esophageal atresia with Chest X-ray showing a feeding tube coiled in upper esophagus with presence of gases in bowel loops, in
tracheoesophageal a newborn with frothing, excessive drooling & coughing with feeds, with history of antenatal polyhydra­
fistula mnios suggests a diagnosis of Esophageal atresia with tracheoesophageal fistula (EA with TEF)

2. Ans. b. Esophagus The coin shadow being visible fully in AP view and the thickness being visible in lateral view, suggest its
position in esophagus

3. Ans. a. Congenital aganglionic •• Ba enema shows narrow distal segment with dilated proximal segment of intestine
megacolon •• Intestinal biopsy shows hypertrophied nerve bundles between muscular layer & submucosa

4. Ans. a. Intussusception Ba enema showing ‘coiled spring sign’ or ‘claw sign’, in a child with pain abdomen & red currant jelly
stool, points to a diagnosis of intussusception

5. Ans. a. Duodenal atresia The given radiograph shows “double-bubble” sign, caused by distended stomach & proximal duodenum

6. Ans. a. Gastroschisis This congenital defect of abdominal wall through which bowel loops, which are not covered by any other
external membrane protrude is gastroschisis

7. Ans. d. Omphalocele Abdominal contents protruding through the defect are covered by a sac

8. Ans. d. Congenital Chest X-ray showing gas filled bowel loops in left hemithorax, with mediastinal shift to right side,
diaphragmatic hernia suggestive of Congenital diaphragmatic hernia

9. Ans. d. Bladder exstrophy The bladder mucosa being exposed to outside through an abdominal wall defect, along with presence of
epispadias & anteriorly placed anal opening suggest this diagnosis

10. Ans. b. Hypospadias Urethral opening is seen on the ventral surface of penis; dorsal hood of foreskin is also seen
544 Section 3: Systemic Pediatrics
Review of Pediatrics & Neonatology

Questions
 ESOPHAGEAL DISORDERS  DISORDERS OF STOMACH

1. H type TEF presents with: (PGI May 2017) 10. The abdominal mass in congenital hypertrophic pyloric
a. Dysphagia b. Aspiration Stenosis is a new born child can be best palpated:
c. Hematemesis d. Hemoptysis  (NEET Pattern Jan 2018)
e. Recurrent pneumonia a. In the midepigastric area b. In the right iliac quadrant
2. Which one of the following life-threatening conge­nital c. In the umbilical area d. During feeding
anomalies in the newborn presents with polyhydra­ 11. Investigation of choice for congenital hypertrophic
mnios, aspiration pneumonia, excessive salivation and pyloric stenosis is? M  (Recent Question 2015)
difficulty in passing a nasogastric tube? M  (APPG 2016) a. Barium meal b. Ultrasound
a. Gastroschisis b. Diaphragmatic hernia c. CT scan d. MRI
c. Tracheo-esophageal fistula
12. Consider the following:
d. Choanal atresia
1. Visible gastric peristalsis
3. A coin is loaded in the esophagus of an asymptomatic 3 2. Bilious vomiting
years female child, Treatment is: (JIPMER 2014) 3. Palpable mass
a. Endoscopic removal within 24 hours 4. Melena
b. Immediate endoscopic removal
Which of the above is/are the feature/features of infantile
c. Wait for 48 hours
hypertrophic pyloric stenosis? (UPSC CMS 2015)
d. Dislodge the coin to the stomach by inserting Ryle's tube
a. 1, 2 and 3 b. 1 and 3 only
4. The commonest type of Congenital atresia is:
c. 2 and 4 d. 4 only
 (APPG 2014)
13. In Hypertrophic pyloric stenosis all the following
a. Proximal end blind, distal end communicating with trachea
electrolytes need to be replenished except:
b. Distal end blind, proximal end communicating with trachea
 (Recent Question 2015)
c. Proximal and distal ends open and communicating with
a. Na+ b. K+
trachea
c. Cl– d. HCO3–
d. Both ends blind
5. Gasless abdomen on X-ray is seen in what type of 14. In a dehydrated child with hypertropic pyloric stenosis,
trachea-esophageal fistula: (TN PGMEE 2013) the fluid of choice: (WBPG 2010)
a. Isolated TEF b. EA with proximal TEF a. Normal saline b. 5% dextrose
c. EA with distal TEF d. EA with double TEF c. Ringer lactate d. Darrows solution
6. A one day old baby presents with excessive frothing from 15. A 3-month infant presents with palpable abdominal mass
mouth and baby was cyanosed: M  (AIIMS Nov 2013) and non bilious vomiting. Most likely diagnosis is?
a. Esophageal atresia b. Diaphragmatic hernia a. Hypertrophic biliary stenosis  (Recent Question 2012)
c. Congenital lung cyst d. Lung hypoplasia b. Hypertrophic pyloric stenosis
7. Esophageal atresia may occur as a part of VACTER group c. Tracheoesophageal fistula
of anomalies. What does ‘TE’ stand for? d. Duodenal atresia
a. Tetralogy of Fallot  (DNB Dec 2010) 16. The metabolic derangement in congenital pyloric
b. Thoracic empyema stenosis is: (AIIMS Nov 06, Nov 02)
c. Tracheo-esophageal fistula a. Hypochloremic alkalosis
d. Talipes equinovarus b. Hyperchloremic alkalosis
8. A newborn baby had normal APGAR score at birth and c. Hyperchloremic acidosis
developed excessive frothing and choking on attempted d. Hypochloremic acidosis
feeds. The investigation of choice is: (AIIMS May 03) 17. Congenital hypertrophic pyloric stenosis usually
a. Esophagoscopy b. Bronchoscopy presents: (AIIMS May 04)
c. MRI chest a. Within 2 days after birth
d. X-ray chest and abdomen with the red rubber catheter b. Around 1 week after birth
passed per orally into esophagus c. Around 2 weeks after birth
9. A newborn has dribbling after feeds. He has respiratory d. Around 2 months after birth
distress and froths at the mouth. Diagnosis is: 18. Projectile vomiting is a feature of: (WBPG 2015)
a. Tracheoesophageal fistula  (AIPGMEE 2001) a. Esophageal atresia
b. Tetralogy of Fallot b. Hypertrophic pyloric stenosis
c. Respiratory distress syndrome c. Cholera
d. None of the above d. Intussusception
Chapter 24: Pediatric Surgical Disorders 545

 DISORDERS OF INTESTINE 31. X-ray detect congenital anorectal malformation at:

Questions
 (UP PGMEE 2007)
19. Congenital mega colon is confirmed by: a. Immediately after birth b. 24–48 hours
 (WB PGMEE 2016) c. 48–72 hours d. After 72 hours
a. Rectal biopsy b. Sigmoidoscopy 32. A male infant presented with distension of abdomen
c. Barium enema d. Invertogram shortly after birth with passing of less meconium. Sub­
20 Meckel Diverticulum is a remnant of: M  se­quently a full-thickness biopsy of the rectum was
 (Recent Question 2015) performed. The rectal biopsy is likely to show:
a. Vitelloumbilical duct b. Vitellointestinal duct a. Fibrosis of submucosa  (AIIMS Nov 04)
c. Vitelloportal duct d. Vitellodiaphragmatic duct b. Lack of ganglion cells
21. Meckel’s diverticulum is a derivative of? c. Thickened muscularis propria
 (Bihar PG 2015) d. Hyalinization of the muscular coat
a. Allantoic diverticulum b. Vitellointestinal duct
33. Failure to pass meconium within 48 hours of birth in a
c. Ventral mesogastrium d. Ductus arteriosus
newborn with no obvious external abnormality should
22. A child complains of fluid coming out of umbilicus on lead to the suspicion of: (AIIMS Nov 2002)
straining. What is the diagnosis? (AIIMS Nov 2014) a. Anal atresia b. Congenital pouch colon
a. Patent vitellointestinal duct c. Congenital aganglionosis d. Meconium ileus
b. Urachal fistula
34. True statement regarding Hirschsprung’s disease:
c. Umbilical hernia d. Gastroschisis
a. Giant ganglia are present  (AIIMS June 99)
23. The absence of ganglion cells within the affected seg­
b. Mucosa is involved and show foldings
ment of bowel is a feature of: (MAHA PGM CET 2014)
c. Manometry excludes the disease
a. Tropical sprue b. Hirschsprung's disease
d. Rectal biopsy is contraindicated in infants
c. Celiac disease d. Crohn's disease
24. FALSE statement regarding Hirschsprung disease is: 35. Recurrent obstruction, mass per rectum and diarrhea are
 (APPG 2014) seen in a child diagnosis: (Recent Question 2016)
a. Aganglionosis always involves distal rectum a. Intussuception b. Rectal prolapse
b. Non passage of meconium in 1st 24 hours is a cardinal c. Internal hernia d. Hemorrhoids
feature 36. A 6-month-old child woke up in night, crying with
c. Diagnosis is established by Suction rectal biopsy abdominal pain, which got relieved on passing red
d. No passage of stools after per rectal examination stools. What is the diagnosis? (AIIMS Nov 2014)
25. Most common cause of Acute Intestinal Obstruction in a. Meckel’s diverticulum b. Intusussception
neonates is: (Recent Question 2014) c. Malrotation d. Intestinal obstruction
a. Jejunal atresia b. Malrotation 37. Intussusception is caused by: (JIPMER 2014)
c. Duodenal atresia d. Acute Intussusception a. Submucous lipoma b. Subfascial lipoma
26. Commonest cause of intestinal obstruction in children c. Subserous lipoma d. Intramural lipoma
is: M (Recent Question 2014) 38. A previously healthy infant presents with recurrent
a. Intussusception b. Volvulus episode of abdominal pain. The mother says that the
c. Hernia d. Adhesions child has been passing stool after episodes of pain, but
27. In Hirschsprung disease, marker used for early diagnosis gives no history of vomiting or bleeding per rectum.
is: (Recent Question 2013) Which of the following is the most likely diagnosis?
a. Acetyl cholinesterase b. Adrenaline  (Recent Question 2013)
c. VIP d. None of above a. Rectal polyps b. Intussusception
28. Hirschsprung disease is confirmed by: c. Meckel's diverticulum d. Necrotizing enterocolitis
 (Recent Question 2012) 39. A 10-month-old infant presents with acute intestinal
a. Rectal biopsy b. Per Rectal examination obstruction. Contrast enema X-ray shows the
c. Rectal manometry d. X-ray abdomen intussusception, likely cause is: (AIPGMEE 2002)
29. A neonate presented with fever, lethargy, abdominal a. Peyer's patch hypertrophy
distension, vomiting and constipation. Clinically he was b. Meckel's diverticulum
diagnosed as volvulus neonatorum with suspected perfo­ c. Mucosal polyp d. Duplication cyst
ration. Best investigation would be: (AIPGMEE 2010)
40. Infants with blood in stools and mass in abdomen,
a. Plain X-ray b. Barium enema
diagnosis is: (PGI June 2001)
c. Upper GI endoscopy
a. Intussusception b. Volvulus
d. Barium meal followthrough
c. Idiopathic abdominal epilepsy
30. True about Hirschsprung’s disease: (PGI Dec 08)
d. Hirschsprung's disease
a. Aganglionic segment is contracted not dilated
b. Descending colon is most common site 41. Best diagnostic investigation for acute appendicitis in
c. Barium enema is diagnostic children: (AIIMS May 2015)
d. Barium enema shows calcification a. USG b. MRI 
e. Rectal biopsy is the confirmatory investigation c. CECT d. X-ray 
546 Section 3: Systemic Pediatrics

42. A 9-month-old girl is brought with PR bleed, vomiting,


Review of Pediatrics & Neonatology

52. Diaphragmatic hernias can occur through the:


mass in right lumbar region with masked liver dullness.  (COMEDK 2016)
She is in shock like condition. Management should a. Foramen of Morgagni b. Foramen of Magendie
include: (PGI Nov 2010) c. Foramen of Winslow d. Esophageal hiatus
a. Barium enema b. Saline reduction 53. Which of the following is associated with good prognosis
c. Give O2 d. Nasogastric tube in congenital diaphragmatic hernia?
e. IV fluid a. Lung head ratio < 1 (Recent Question 2015)
43. A newborn with recurrent vomiting, cyanosis after each b. Detection of CDH in first trimester scan
feed is likely to be suffering from: (APPG 05) c. Symptoms develop after 48 hours
a. Tracheo-esophageal fistula b. Tetralogy of Fallot d. Liver herniated into thorax
c. Congenital hypertrophic pyloric stenosis
54. Congenital diaphragmatic hernia through foramen of
d. ARDS Bochdalek is situated: (MAHA PGM CET 2014)
44. Double bubble sign in children is seen in all except: a. Anteriorly b. Posterolaterally
(PGI Dec 04) c. Diaphragmatic hiatus d. Antero laterally
a. Ladds band b. Annular pancreas 55. Most important prognostic factor in congenital diaphrag­
c. Pancreatic pseudocyst d. Diaphragmatic hernia matic hernia: (AIPGMEE 11, AIIMS Nov 09)
e. Duodenal atresia a. Pulmonary hypertension b. Size of hernia
45. Which one of the following is most suggestive of c. Timing of surgery d. Gestational age
neonatal small bowel obstruction: (AIPGMEE 03) 56. A neonate having congenital diaphragmatic hernia deve­
a. Generalized abdominal distension loped respiratory distress. Breath sounds were decreased
b. Failure to pass meconium in the first 24 hours on the left side. After bag and mask ventilation, ET tube
c. Bilious vomiting d. Refusal of feeds was put and the maximal cardiac impulse shifted to the
46. A 6-month-old baby presents with history of bloody right side. What should be the next step in management:
diarrhea of 2 days duration with abdominal distension  (AIPGMEE 08)
and on examination the baby screams, diagnosis is: a. Confirm the position of endotracheal tube by X-ray chest
a. Intussusception b. HUS  (PGI Dec 2003) b. Remove tube and reattempt intubation
c. Appendicitis d. Ac. enterocolitis c. Naso gastric tube insertion
47. A 14-year-old girl with history of abdominal pain (peri­ d. Chest X-ray
umbilical), postprandial, passing blood in stools, fever, 57. A child presented with respiratory distress was brought
weight loss since ten months. She also has episodes of to emergency with bag and mask ventilation. Now child
passing blood in stools. What may be the likely diag­ is intubated. Chest X-ray shows right-sided deviation of
nosis? (DNB June 2002)
mediastinum with scaphoid abdomen. His Pulse rate is
increased. What is the next step? (AIIMS Nov 07)
a. Chronic appendicitis b. Chronic pancreatitis
a. Endotracheal intubation b. Put a nasogastric tube
c. Crohn's disease d. Bulimia
c. Surgery
 DISORDERS OF ABDOMINAL WALL d. End tidal CO2 to confirm intubation
58. 2 days after birth, a baby developed respiratory distress
48. Chromosomal anomalies more than 20% is associated and had scaphoid abdomen. Breath sounds were
with: (AIIMS Nov 2009) decreased on the left side. After bag and mask venti­
a. Gastroschisis b. Omphalocele lation, ET tube was put and the maximal cardiac impulse
c. Spina bifida d. Cleft palate shifted to the right side. What should be the next step in
management? (AIIMS Nov 2007)
 CONGENITAL DIAPHRAGMATIC HERNIA a. Confirm the position of endotracheal tube
b. Emergency surgery
49. A term newborn baby born by vaginal delivery had res­ c. Naso gastric tube insertion
piratory distress, grunt, with scaphoid abdomen. Most d. Chest X-ray to confirm position of the endotracheal tube
probable diagnosis is: M  (Recent Question 2016) 59. A newborn baby has been referred to the casualty as
a. Hyaline membrane disease a case of congenital diaphragmatic hernia. The first
b. Diaphragmatic hernia clinical intervention is to: (AIIMS May 03)
c. Pneumothorax a. Insert a central venous pressure line
d. Meconium aspiration syndrome b. Bag and mask ventilation
50. Diaphragmatic hernias can be diagnosed by: c. Insert a nasogastric tube
a. Ultrasound of the chest (COMEDK 2016) d. Ventilate with high frequency ventilator
b. Barium meal follow through 60. Newborn with APGAR score of 2 at 1 min. and 6 at 5
c. None of the above d. Both the above min, has respiratory distress and mediastinal shift. What
can be the diagnosis? (PGI Dec 2000)
51. Congenital eventration of diaphragm requires demons­
a. Congenital adenomatoid lung disease
tration of: (COMEDK 2016)
b. Pneumothorax
a. An intact diaphragm b. Diaphragmatic tear
c. Diaphragmatic hernia
c. A diaphragmatic defect
d. Transient tachypnea of newborn
d. Normal diaphragmatic excursions e. HMD
Chapter 24: Pediatric Surgical Disorders 547

Answers with Explanations


Answers with Explanations
 ESOPHAGEAL DISORDERS
1. b. Aspiration, e. Recurrent pneumonia  Ref: Nelson's 20/e p 1783
The infant with an isolated TEF, in the absence of EA (“H-type” fistula) might come to medical attention later in life with chronic
respiratory problems, including refractory bronchospasm and recurrent pneumonias.
2. c. Tracheo-esophageal fistula  Ref: Nelson’s 20/e p 1783, Ghai 8/e p 176
•• Presence of polyhydramnios, aspiration pneumonia, excessive salivation and difficulty in passing a nasogastric tube all
point towards a diagnosis of tracheoesophageal fistula, which is usually accompanied by esophageal atresia.
Esophageal atresia (EA) and TEF (Tracheoesophageal fistula):
Important •• EA is the most frequent congenital anomaly of esophagus; 90% of EA babies have TEF
features •• Most common type is upper esophagus ends in a blind pouch and TEF connected to distal esophagus
Typical •• Maternal polyhydramnios is seen; New born presents with respiratory distress and excess oral frothing;
presentation •• Newborn regurgitates all of its feeds; Saliva pours almost continuously from the mouth of the infant
•• Attacks of coughing and cyanosis occurs on feeding; Feeding tube cannot be passed more than 10 cm
Diagnosis •• Chest X-ray with a red rubber catheter passed per orally shows coiled rubber catheter in a lucent proximal
esophageal pouch
•• X-ray abdomen: Gas in the stomach is seen if there is communication between the lower part of the esophagus
and trachea, which occurs in the commonest variety of tracheo-esophageal fistula

3. a. Endoscopic removal within 24 hours  Ref: Nelson’s 20/e p1793-1794; Refer pretext of this chapter for details;
Asymptomatic blunt objects and coins can be observed for up to 24 hr anticipating passage into stomach; If they still remain
lodged in the esophagus, endoscopic removal is warranted.
4. a. Proximal end blind, distal end communicating with trachea  Ref: Nelson’s 20/e p 783, Ghai 8/e p 176
5. b. EA with proximal TEF  Ref: Nelson’s 20/e p 1783, Ghai 8/e p 176
•• In EA with proximal TEF, as there is no connection of the distal esophagus with the trachea or proximal esophagus, gasless
abdomen will be seen on X-ray.
6. a. Esophageal atresia  Ref: Nelson’s 20/e p 1783, Ghai 8/e p 176

Congenital disease Mode of presentation


Cong diaphragmatic hernia New born with cyanosis, respiratory distress, shift of mediastinum and scaphoid abdomen
Bilateral choanal atresia New born with feeding dificulty, cyanosis on closing mouth, which disappears on crying
Pure esophageal atresia Neonate with frothing and bubbling at mouth and nose, with coughing, cyanosis and respiratory
distress; X-ray shows coiled feeding tube in esophageal pouch and absent gastric air shadow

7. c. Tracheo-oesophageal fistula  Ref: Nelson’s 20/e p 1783, Ghai 8/e p 173
•• VACTERL association (also called VATER association) refers to the non-random co-occurrence of birth defects including Vertebral
anomalies, Anal atresia, Cardiac defects, Tracheoesophageal fistula and/or Esophageal atresia, Renal and Radial anomalies and
Limb defects.
8. d. X-ray chest and abdomen with the red rubber catheter passed per orally into esophagus  Ref: Nelson’s 20/e p 1783
9. a. Tracheoesophageal fistula  Ref: Nelson’s 20/e p 1783, Ghai 8/e p 176

 DISORDERS OF STOMACH
10. a. In the midepigastric area  Ref: Nelson 20/e p 1797-1799
•• Non-bilious vomiting is the initial symptom of pyloric stenosis, occurring immediately after a feeding.
•• Vomiting usually starts after 3 wk of age, but symptoms can develop as early as the 1st wk of life
•• As vomiting continues, a progressive loss of fluid, hydrogen ion, and chloride leads to hypochloremic metabolic alkalosis.
•• Diagnosis established by palpating the pyloric mass.
•• The mass is firm, movable, 2 cm in length, olive shaped, hard, best palpated from the left side & located above & to the right of
umbilicus in midepigastrium beneath liver's edge.
•• The olive is easiest palpated after an episode of vomiting.
•• After feeding, there may be a visible gastric peristaltic wave that progresses across the abdomen.
548 Section 3: Systemic Pediatrics

11. b. Ultrasound  Ref: Nelson’s 20/e p 1797-1799, Ghai 8/e p 279-280


Review of Pediatrics & Neonatology

USG confirms the diagnosis of congenital hypertrophic pyloric stenosis


12. b. 1 and 3 only  Ref: Nelson’s 20/e p 1797-1799; Refer pretext of this chapter for details;
13. d. HCO –  Ref: Nelson’s 20/e p 1797-1799, Ghai 8/e p 279-280
3
Electrolyte abnormalities in congenital hypertrophic pyloric stenosis:

•• These infants can develop hypochloremic, hypokalemic, metabolic alkalosis


•• Hydrogen ion and Na+ re-absorption occurs with an accompanied potassium ion excretion
•• The resulting hypokalemia leads to the excretion of hydrogen ions in the face of alkalosis, a paradoxic aciduria
••
Treatment includes replacement of volume deficit with isotonic saline and potassium once adequate urine output is ensured

14. a. Normal saline  Ref: Nelson’s 20/e p 1797-1799, Ghai 8/e p 279-280
15. b. Hypertrophic pyloric stenosis  Ref: Nelson’s 20/e p 1797-1799, Ghai 8/e p 279-280
16. a. Hypochloremic alkalosis  Ref: Nelson’s 20/e p 1797-1799, Ghai 8/e p 279-280

High Yield Points  


Congenital Hypertrophic Pyloric Stenosis
• It is the commonest surgical disorder of the stomach during infancy
• More common in boys and in infants of blood group ‘B’ and ‘O’; Usually does not present at birth, but after 3 weeks age
• Non-billious vomiting is the initial symptoms of pyloric stenosis; leads to Hypochloremic metabolic alkalosis

17. c. Around 2 weeks after birth  Ref: Nelson’s 20/e p 1797-1799, Ghai 8/e p 279-280
Usual age of presentation of congenital pyloric stenosis is 2-6 weeks;
18. b. Hypertrophic pyloric stenosis  Ref: Nelson’s 20/e p 1797-1799, Ghai 8/e p 279-280
Discussing the options one by one
a. Esophageal atresia Frothing & bubbling at mouth & nose & coughing, cyanosis, respiratory distress seen
b. Hypertrophic pyloric Progressive non-bilious vomiting is the initial symptom of pyloric stenosis
stenosis
c. Cholera Profuse watery diarrhea, with profuse rice water stools with a fishy smell is seen
d. Intussusception Sudden onset of severe paroxysmal colicky pain that is accompanied by straining efforts with legs
and knees flexed. Vomiting occurs in most cases, but is not projectile.

 DISORDERS OF INTESTINE
19. a. Rectal biopsy  Ref: Nelson’s 20/e p 1894-1897, Ghai 8/e p 285-286
•• Congenital Aganglionic Megacolon is the other name for Hirschsprung disease
•• Diagnosis of Hirschsprung disease is confirmed by rectal biopsy.
20. b. Vitellointestinal duct  Ref: Nelson’s 20/e p 1894-1897, Ghai 8/e p 285-286

High Yield Points  


• Meckel diverticulum is the most common congenital anomaly of the GI tract
• It is caused by the incomplete obliteration of the omphalomesenteric or vitellointestinal duct during the 7th wk of gestation

21. b. Vitellointestinal duct  Ref: Nelson’s 20/e p 1894-1897, Ghai 8/e p 285-286
22. b. Urachal fistula  Ref: Nelson’s 20/e p 1894-1897, Ghai 8/e p 285-286
•• A persistent urachus is due to failure of closure of allantoic duct and is associated with bladder outlet obstruction
•• Patency should be suspected if a clear, light yellow, urine-like fluid is being discharged from the umbilicus.
23. b. Hirschsprung’s disease  Ref: Nelson’s 20/e p 1894-1897, Ghai 8/e p 285-286
24. d. No passage of stools after per rectal examination  Ref: Nelson’s 20/e p 1894-1897, Ghai 8/e p 285-286
Clinical features of Hirschsprung disease:
•• It usually presents in the neonatal period with a distended abdomen, failure to pass meconium, delayed passage of stool and/or
bilious emesis or aspirates with feeding intolerance. May present later with constipation
•• Rectal examination: Rectum is usually empty, but when finger is removed, there may be explosive discharge of foul smelling
feces and gas.

Chapter 24: Pediatric Surgical Disorders 549

25. c. Duodenal atresia  Ref: Nelson’s 20/e p 1894-1897, Ghai 8/e p 285-286

Answers with Explanations


High Yield Points  
• Most common cause of neonatal intestinal obstruction — Meconium plug syndrome > Duodenal atresia
• Other important causes of intestinal obstruction in newborn are: Jejunoileal atresia, Malrotation of gut, Meconium ileus
• Intussusception is the most common cause of intestinal obstruction between 3 months and 6 years of age (not in neonate)

Meconium plug syndrome—when meconium plug causes obstruction of colon. It is seen in Hirschsprung disease, maternal DM,
maternal preeclampsia, prematurity, sepsis, hypothyroidism.
Meconium ileus—when meconium plug obstruct ileum. It is seen in cystic fibrosis.
26. a. Intussusception  Ref: Nelson’s 20/e p 1894-1897, Ghai 8/e p 285-286
Intussusception occurs when a portion of the alimentary tract is telescoped into an adjacent segment.
27. a. Acetyl cholinesterase  Ref: Nelson’s 20/e p 1894-1897, Ghai 8/e p 285-286
28. a. Rectal biopsy  Ref: Nelson’s 20/e p 1894-1897, Ghai 8/e p 285-286; Refer Ans. 55
29. a. Plain X-ray  Ref: Nelson’s 20/e p 1894-1897, Ghai 8/e p 285-286
Volvulus neonatorum

What is it? Intestinal obstruction in a newborn due to twisting of bowel caused by malrotation or non-fixation of colon

Investigations Barium meal followthrough is the investigation of choice as X-ray is not reliable in early stages of volvulus;
However, if perforation is suspected, barium contrast should not be used because of risk of perotinitis

30. a. Aganglionic segment is contracted not dilated, e. Rectal biopsy is the confirmatory investigation  Ref: Nelson’s 20/e p
1894-1897, Ghai 8/e p 285-286
Diagnosis of Hirschsprung’s disease

Name of investigation Features seen


Rectal suction biopsy •• It shows Hypertrophied nerve bundles and absence of ganglion cells
(gold standard) •• Acetylcholinesterase or Calretinin stains used
Anorectal manometry •• Measures the pressure of internal anal sphincter while a balloon is distended in the rectum
•• In Hirschsprung’s disease, there is paradoxical rise in pressure with rectal distention
•• Normal anorectal manometry precludes the diagnosis of Hirschsprung disease
Barium enema •• Useful in children > 1 mo of age because the proximal ganglionic segment might not be significantly
dilated in the 1st few wk of life
•• It demonstrates the presence of transition zone between normal dilated proximal colon and a smaller
caliber obstructed distal colon caused by non-relaxation of a ganglionic bowel

31. b. 24–48 hours  Ref: Nelson’s 20/e p 1894-1897, Ghai 8/e p 285-286
The invertogram or upside-down X-ray is of little value to detect congenital anorectal malformation, but a prone cross-table lateral
plain X-ray at 24 hours of life (to allow time for bowel distention from swallowed air) with a radiopaque marker on the perineum
is helpful in diagnosis.
32. b. Lack of ganglion cells  Ref: Nelson’s 20/e p 1894-1897, Ghai 8/e p 285-286
“Whenever neonate presents with delayed passage of stool, Hirschsprung’s disease should be suspected”.
33. c. Congenital aganglionosis  Ref: Nelson’s 20/e p 1894-1897, Ghai 8/e p 285-286
Read what Nelson says about Hirschsprung's disease (Congenital aganglinosis):
•• “Ninety nine percent of full term infants normally pass meconium within 48 hrs of birth. Hirschsprung disease
should be suspected in any full term infant with delayed passage of stool”.
34. c. Manometry excludes the disease  Ref: Nelson’s 20/e p 1894-1897, Ghai 8/e p 285-286; Refer Ans. 59
35. a. Intussusception  Ref: Nelson’s 20/e p 1812-1814, Ghai 8/e p 287
Discussing about the options one by one,
Option a. Recurrent intussusception is noted in 5-8%
Advancing intestine can prolapse through the anus in intussusception and is distinguished from rectal prolapse by
separation between the protruding intestine and the rectal wall, which does not exist in rectal proplase
Option b. Rectal prolapse usually occurs during defecation and is usually painless or produces mild discomfort
Option c. There is no diarrhea in rectal prolapse, hernia and hemorrhoids
Option d. External hemorrhoids are associated with extreme pain & itching
Internal hemorrhoids (those located above the dentate line) manifest primarily with bleeding, prolapse and
occasional incarceration

550 Section 3: Systemic Pediatrics

36. b. Intussusception  Ref: Nelson’s 20/e p 1812-1814, Ghai 8/e p 287


Review of Pediatrics & Neonatology

37. a. Submucous lipoma  Ref: Nelson’s 20/e p 1812-1814, Ghai 8/e p 287; Refer pretext for details;
Intussusception
Predisposing Peyer’s patches or prominent mounds of lymph tissue lead to mucosal prolapse of ileum into colon;
conditions Other lead points: Meckel diverticulum, intestinal polyp, neurofibroma, intestinal duplication cysts, hamartomas,
ectopic pancreatic tissue, hemangioma, lipoma
Clinical features Sudden onset of severe paroxysmal colicky pain in a previously well child, that recurs frequently
60% infants pass a stool containing red blood and mucus, the currant jelly stool
Vomiting occurs in most cases and is usually more frequently in the early phase.

38. b. Intussusception  Ref: Nelson’s 20/e p 1812-1814, Ghai 8/e p 287
In the given question, out of the given options, Intussusception is most appropriate one because
•• Usual presentation of rectal polyps and Meckel’s diverticulum is painless rectal bleeding
•• Necrotizing Enterocolitis is usually seen in premature infants with abdominal distension, bloody stools, lethargy and shock.
39. a. Peyer’s patch hypertrophy  Ref: Nelson’s 20/e p 1812-1814, Ghai 8/e p 287
40. a. Intussusception  Ref: Nelson’s 20/e p 1812-1814, Ghai 8/e p 287
About rest of the options,
In volvulus: Features of obstruction predominate, i.e. Vomiting, abdominal distension but there is no abdominal mass palpable.
In Hirschsprung’s disease: history of constipation and abdominal distension may be present.
In abdominal epilepsy: There is abdominal pain with/without a seizure prodrome.
41. a. USG  Ref: Nelson's 19/e p 1351-1352; Gore & Levine Textbook of Gastrointestinal Radiology, 3/e, p 2282
•• Investigation of choice for acute appendicitis in a child is USG, as a child has thin abdominal wall
•• So, even though USG does not have much penetration power, appendix can be visualized
•• Best investigation would be CT abdomen, but that would expose the child to radiation, hence it is not the investigation of choice.
42. c. Give O2, d. Nasogastric tube, e. IV fluid  Ref: Nelson’s 20/e p 1854, Ghai 8/e p 291
•• In patients with prolonged intussusception with signs of shock, peritoneal irritation, intestinal perforation or pneumatosis
intestinalis, reduction (with barium, saline or air) should not be attempted
•• In such cases, urgent laparotomy should be planned
•• But the first priority is urgent resuscitation i.e. A, B, C (airway, breathing, circulation).
Hence in case of shock, and as the child in question is also vomiting, she should be made NPO and Nasogastric tube insertion should
be done.
43. a. Tracheo-esophageal fistula  Ref: Nelson’s 20/e p 1783, Ghai 8/e p 176
44. d. Diaphragmatic hernia  Ref: Nelson’s 20/e p 1801
•• Obstruction of 2nd part of duodenum cause ‘double bubble’ sign on plain abdominal radiograph
•• It may be due to- Annular pancreas, Pancreatic pseudocyst, Ladds band a Tumor in head of pancreas.
45. c. Bilious vomiting  Ref: Nelson’s 20/e p 1802
Bilious vomiting is the most common symptom of Intestinal obstruction in newborn;
Presentation depends on the site of obstruction:
•• Proximal obstruction
–– A history of polyhydramnios is common; Abdominal distension and constipation will be less prominent
–– The higher the obstruction in the intestine the earlier the infant will develop vomiting which may be bile stained
•• Distal obstruction, Abdominal distension and constipation are more common, while vomiting is less common. Note: In
neonates proximal intestinal obstruction are much more common than distal. So, bilious vomiting is the most common
presentation of neonatal intestinal obstruction.
46. a. Intussusception  Ref: Nelson’s 20/e p 1812, Ghai 8/e p 287
Discussing the options one by one:
a. Intussusception The typical age group with short history of bloody diarrhea and extreme tenderness on examination point
towards intussusception
b. HUS In HUS, the typical clinical picture is of sudden onset pallor, oliguria, edema, petechiae and lethargy;
Usually there is history of severe gastroenteritis with blood stained stools but no severe pain abdomen;
c. Appendicitis Typically presents with right lower quadrant pain with/ without gastroenteritis
It is most common in older children, with peak incidence between the age of 12 and 18 year
d. Ac. enterocolitis There is usually a history of fever, vomiting and colicky pain abdomen, usually associated with tenesmus

47. c. Crohn’s disease  Ref: Nelson’s 20/e p 1819, Ghai 8/e p 304
Blood in stools, abdominal pain and systemic features like fever and weight loss for months is suggestive of Inflammatory bowel
disease.
Chapter 24: Pediatric Surgical Disorders 551

 DISORDERS OF ABDOMINAL WALL

Answers with Explanations


48. Ans. b. Omphalocele Ref: Nelson’s 20/e p 797
Omphalocele

•• It is a midline defect of anterior abdominal wall, with herniation of abdominal viscera into base of umbilical cord.
•• The gut is covered with peritoneum & amnion
•• Anomalies associated with omphalocele are: Chromosomal abnormalities (in ~43% cases) esp. Turner, Trisomy 13, 18, 21 syndrome
•• Cardiac abnormalities e.g.: TGA, VSD; GIT anomalies – Diaphragmatic hernia & Genitourinary anomalies
•• Skeletal malformation – kyphoscoliosis; CNS: holoprosenecephaly, encephalocele, cerebellar hypoplasia

 CONGENITAL DIAPHRAGMATIC HERNIA


49. b. Diaphragmatic hernia  Ref: Nelson’s 20/e p 862, Ghai 8/e p 178
50. d. Both the above  Ref: Nelson’s 20/e p 862, Ghai 8/e p 178
Chest X-ray, Ultrasound of chest or barium meal followthrough can be used to diagnose Diaphragmatic hernia.
51. a. An intact diaphragm  Ref: Nelson’s 20/e p 862
Eventration of the diaphragm is an abnormal elevation of the dome of diaphragm. There is no defect (hole) in diaphragm.
52. a. Foramen of Morgagni  Ref: Nelson’s 20/e p 862, Ghai 8/e p 178
In diaphragmatic hernia, the defect may be in the posterolateral (Bochdalek) portion of the diaphragm (most common), retrosternal
(Morgagni) is the 2nd most common, at the esophageal hiatus (hiatal) or paraesophageal (adjacent to the hiatus).
53. c. Symptoms develop after 48 hours  Ref: Nelson’s 20/e p 862, Ghai 8/e p 178
Predictors of a poor prognosis in CDH are:
•• Lung: Head ratio < 1 •• Severe pulmonary hypoplasia
•• An associated major anomaly •• Liver herniation into thorax
•• Symptoms before 24 hr of age •• Detection of CDH in 1st trimester scan

•• Herniation to the contralateral lung •• Need for ECMO
54. b. Post erolaterally  Ref: Nelson’s 20/e p 862, Ghai 8/e p 178
55. a. Pulmonary hypertension  Ref: Nelson’s 20/e p 862, Ghai 8/e p 178
The two main factors that affect morbidity and mortality in CDH are pulmonary hypoplasia and pulmonary hypertension.
56. b. Remove tube and reattempt intubation  Ref: Nelson’s 20/e p 862, Ghai 8/e p 178
Best answer would have been, confirm the position of endotracheal tube (but, not by X-ray!). Amongst the given options, best course
of action is to remove the tube and reattempt intubation.
57. b. Put a nasogastric tube  Ref: Nelson’s 20/e p 862, Ghai 8/e p 178
Mediastinal deviation, scaphoid abdomen with respiratory distress suggest a diagnosis of CDH. The resuscitation of CDH patient
consists of: Stabilization by mechanical ventilation with 100% O2 & Nasogastric suction, In this case, the child is already intubated.
Right sided deviation of mediastinum seen in this case could be due to herniated bowel loops, which would have been further
distended by bag & mask ventilation. So, the best next step is to put a NG tube to decompress the bowel loops.
58. a. Confirm the position of endotracheal tube  Ref: Nelson’s 20/e p 862, Ghai 8/e p 178
The given clinical features point to a diagnosis of left sided CDH; In this case, baby has been intubated but shift of cardiac impulse
to the right side after intubation indicates that most probably wrong intubation in the esophagus has been done, which has led to
gaseous distension of intestines in left hemithorax & shift of cardiac impulse to the right. Hence, the most appropriate strategy would
be checking the endotracheal tube position (but not by chest X-ray).
59. c. Insert a nasogastric tube  Ref: Nelson’s 20/e p 862, Ghai 8/e p 178
If a child with congenital diaphragmatic hernia is otherwise stable & does not have severe respiratory distress, a nasogastric tube is
inserted to decompress the stomach & bowel loops.
60. a. Congenital adenomatoid lung disease, b. Pneumothorax & c. Diaphragmatic hernia Ref: Nelson’s 20/e p 862
Note

You might also like

pFad - Phonifier reborn

Pfad - The Proxy pFad of © 2024 Garber Painting. All rights reserved.

Note: This service is not intended for secure transactions such as banking, social media, email, or purchasing. Use at your own risk. We assume no liability whatsoever for broken pages.


Alternative Proxies:

Alternative Proxy

pFad Proxy

pFad v3 Proxy

pFad v4 Proxy